Vous êtes sur la page 1sur 637

SAUNDERS Review of

DENTAL
HYGIENE
This page intentionally left blank
SAUNDERS Review of
DENTAL
HYGIENE
Second Edition

Margaret J. Fehrenbach, RDH, MS


Dental Hygiene Educational Consultant
Dental Science Technical Writer
Seattle, Washington

Jane Weiner, RDH, BS Candidate


Owner, Jane Weiner, RDH, Board Reviews, Inc.
Faculty and Staff
Nova Southeastern University College of Dental Medicine
Fort Lauderdale, Florida
11830 Westline Industrial Drive
St. Louis, Missouri 63146

SAUNDERS REVIEW OF DENTAL HYGIENE, SECOND EDITION ISBN: 978-1-4160-6255-4


Copyright © 2009, 2000 by Saunders, an imprint of Elsevier Inc.

All rights reserved. No part of this publication may be reproduced or transmitted in any form or by any means,
electronic or mechanical, including photocopying, recording, or any information storage and retrieval system,
without permission in writing from the publisher. Permissions may be sought directly from Elsevier’s Rights
Department: phone: (+1) 215 239 3804 (US) or (+44) 1865 843830 (UK); fax: (+44) 1865 853333; e-mail:
healthpermissions@elsevier.com. You may also complete your request on-line via the Elsevier website at  
http://www.elsevier.com/permissions.

Notice

Knowledge and best practice in this field are constantly changing. As new research and experience broaden
our knowledge, changes in practice, treatment and drug therapy may become necessary or appropriate.
Readers are advised to check the most current information provided (i) on procedures featured or (ii) by the
manufacturer of each product to be administered, to verify the recommended dose or formula, the method
and duration of administration, and contraindications. It is the responsibility of the practitioner, relying on
their own experience and knowledge of the patient, to make diagnoses, to determine dosages and the best
treatment for each individual patient, and to take all appropriate safety precautions. To the fullest extent of
the law, neither the Publisher nor the Authors assume any liability for any injury and/or damage to persons
or property arising out of or related to any use of the material contained in this book.

The Publisher

Library of Congress Cataloging-in-Publication Data


Fehrenbach, Margaret J.
Saunders review of dental hygiene / Margaret J. Fehrenbach, Jane Weiner. -- 2nd ed.
p. ; cm.
Rev. ed. of : Saunders review of dental hygiene / Debralee McKelvey Nelson. 1st ed. c2000.
Includes bibliographical references and index.
ISBN 978-1-4160-6255-4 (pbk. : alk. paper) 1. Dental hygiene. 2. Dental hygiene--Case studies.  
3. Dental hygiene--Examinations, questions, etc. I. Weiner, Jane, 1944- II. Nelson, Debralee McKelvey.  
Saunders review of dental hygiene. III. Title. IV. Title: Review of dental hygiene.
[DNLM: 1. Dental Prophylaxis--Case Reports. 2. Dental Prophylaxis--Examination Questions.  
WU 18.2 F296w 2009]
RK60.7.N44 2009
617.6'01--dc22
2008034346

Vice President and Publisher: Linda Duncan


Senior Acquisitions Editor: John Dolan
Developmental Editor: Joslyn Dumas
Publishing Services Manager: Patricia Tannian
Design Direction: Jessica Williams

Printed in United States of America

Last digit is the print number: 9  8  7  6  5  4  3  2  1


To the student dental hygienists we have known who have gone on to become potent
professionals and those we hope to meet, as well as their hardworking and
devoted instructors, without whom learning would not be possible
This page intentionally left blank
Preface

ABOUT THE TEXT   complete the exam as you will when you actually take the
Since its introduction 8 years ago, Saunders Review of NBDHE. At the end of the exam, the CD will score you
Dental Hygiene has solidified its place in the market as and give you a breakdown of what answers you got cor-
one of the leading supplemental dental hygiene review rect and what answers were incorrect.
books. Students often look for a supplemental text, in In addition to the computerized simulated exam, there
addition to their required texts, to help them prepare for are crossword puzzles and word searches to serve as
exams. Saunders Review of Dental Hygiene has been study tools. These are available for each chapter and will
that text for many students. The success of the first edi- aid you in memorizing terminology and recognizing ana-
tion, as well as new guidelines in the areas of infection tomical structures. Key terms (in bold), references, we-
control and pharmacology, has made a second edition a blinks, and other related articles are available for each
­necessity. chapter and provide additional resources for information
Examinations are designed to test and improve your to complement your school studies.
intellect. They take both mental and physical stamina,
and you should be well prepared for them as you enter OTHER FEATURES  
your professional career. This text is a tool to help you Updated case study format. The case studies on the 
prepare for the National Board of Dental Hygiene Exam CD-ROM have been created within the updated guide-
­(NBDHE), as well as to aid you in recognizing your lines published by the ADA. This format allows you to
strengths and weaknesses. The text will make obvious the consider other external factors that can have an effect on
areas you may need to review, strengthen your knowl- dental and general health. It also eliminates the use of
edge, and improve your test-taking abilities. names, removing any insinuation of ethnicity or national-
This new edition maintains many of the successful key ity. The cases are designed to test your ability to make
features of the first edition, such as more than 60 clinical sound objective judgments when faced not only with den-
studies and over 1500 questions, while offering several tal but also with pharmacological and medical concerns.
exciting updates, including a CD-ROM. These elements They are followed by test questions that focus on provid-
will help you to better review the theories, skills, and con- ing quality care to the patient. Within the computerized
clusions you will be expected to know for the National simulated exam on the CD-ROM, each case is presented
Board Examination. with a patient history, dental charts, radiographs, and clin-
ical photographs. In addition, throughout the text are clin-
CD-ROM   ical, community health, and professional practice studies
The CD-ROM is an integral part of this text and offers in a “testlet” mode that combines a scenario with the case
another vehicle for reviewing. It includes a full-length study format to further test your judgmental abilities.
computerized simulated exam that is divided into two Text written in an easy to read summary mode. The
sections, Component A and Component B. Component authors have remodeled the text into an easy to read sum-
A consists of 200 randomly ordered questions with an- mary format. This format provides a straightforward out-
swers and rationales available. Component B includes  line that makes vital material easier to retain. The format
15 case studies with accompanying answers and ration­ is consistent throughout the book, which gives the text a
ales that test your ability to make case-based decisions. cohesive look and feel.
These cases are in the new format introduced by the Fully modeled after specific NBDHE guidelines. The
American Dental Association. purpose of Saunders Review of Dental Hygiene is to pro-
You will be able to take the test in two formats. The vide you with a review text that will help you prepare for
first format will not be timed, and you will be able to the NBDHE. Therefore it is essential that the NBDH for-
move through the exam at your leisure. If an incorrect mat be followed as closely as possible. This allows you to
answer is chosen, you will be provided with the rationale get used to the testing conditions you will experience when
for why that answer is incorrect. The second format will you take the board exam. The full-length, computerized
be timed, and you will have the same amount of time to simulated exam on the CD-ROM is modeled directly after

vii
viii Preface

the ­NBDHE. The formats of each section, the amount of abilities. The benefit of this text is invaluable and will
time you have, and the case studies are all representative of greatly reduce the anxiety associated with test-taking.
the NBDHE. Now you can take advantage of this invalu- All the elements of the text and CD package combine to
able resource to make the review process much easier. ­deliver a review book that will carry you through the re-
The second edition of Saunders Review of Dental view process and aid in your success on the NBDHE.
­Hygiene will be a vital tool in helping you prepare for
the NBDHE by both familiarizing you with the format Margaret J. Fehrenbach
you will see on the exam and improving your test-taking  Jane Weiner
Acknowledgments

We would like to thank Debralee McKelvey Nelson, Amy M. Nieves, RDH


RDH, MS, and her contributors from the first edition of Webmaster, amyrdh.com and www.amysrdhlist.com.
this textbook. In addition, we would like to thank the  Preparing for the NBDHE
editorial staff in Dentistry, Dental Assisting, Dental
Hygiene at Elsevier, Inc., including John Dolan, Senior Michael A. Siegel, DDS, MS, FRCSEd
Editor, and Joslyn Dumas, Developmental Editor, as well Professor and Chair, Department of Diagnostic Sciences
as Patrica Tannian, Publishing Services Manager. We Nova Southeastern University College of Dental
would also like to thank the following reviewers of this Medicine
second edition: Fort Lauderdale, Florida
House Staff, Department of Surgery, Division of Oral
Jorge Arenas, DMD and Maxillofacial Surgery
Department of Radiology Broward General Medical Center, Fort Lauderdale,
Nova Southeastern University, College of Dental Florida
Medicine Dental and Medical Emergencies; Anatomy,
Fort Lauderdale, Florida Biochemistry, and Physiology
Radiology
Sharon Crane Siegel, DDS, MS
Marge Bell, BS, RDH Chair and Associate Professor, Department of
Dental Hygiene Program Coordinator Prosthodontics
Ozarks Technical Community College Nova Southeastern University, College of Dental
Springfield, Missouri Medicine
Community Oral Health Fort Lauderdale, Florida
Clinical Associate Professor, Department of Restorative
Elizabeth L. Fehrenbach, BA Dentistry
Student Nurse Practitioner, John Hopkins University Baltimore College of Dental Surgery, University of
Baltimore, Maryland Maryland
Anatomy, Biochemistry, and Physiology; Microbiology Baltimore, Maryland
and Immunology Dental Biomaterials

Barbara G. Hammaker, RDH, BASDH Laura J. Webb, CDA, RDH, MS


Adjunct Instructor Independent Consultant
St. Petersburg College, St. Petersburg, Florida Fallon, Nevada
Broward Community College, Davie, Florida Nutrition; Community Oral Health
Pharmacology

Juli Kagan, RDH, MEd


Clinical Instructor and Lecturer, Department of
Periodontology
Nova Southeastern University
College of Dental Medicine, Fort Lauderdale, Florida
Periodontology and Component B

ix
This page intentionally left blank
Contents

Chapter 1 Preparing for the National Board Dental Hygiene Examination, 1

Chapter 2 Embryology and Histology, 7

Chapter 3 Anatomy, Biochemistry, and Physiology, 33

Chapter 4 Head, Neck, and Dental Anatomy, 80

Chapter 5 Radiology, 135

Chapter 6 General and Oral Pathology, 168

Chapter 7 Nutrition, 209

Chapter 8 Microbiology and Immunology, 233

Chapter 9 Pharmacology, 278

Chapter 10 Medical and Dental Emergencies, 320

Chapter 11 Clinical Treatment, 342

Chapter 12 Instrumentation, 398

Chapter 13 Periodontology, 425

Chapter 14 Pain Management, 470

Chapter 15 Dental Biomaterials, 502

Chapter 16 Special Needs Patient Care, 530

Chapter 17 Community Oral Health, 549

Chapter 18 Ethics and Jurisprudence, 575

xi
This page intentionally left blank
SAUNDERS Review of
DENTAL
HYGIENE
This page intentionally left blank
CHAPTER  1

Preparing for the National Board Dental


Hygiene Examination

NATIONAL BOARD DENTAL HYGIENE Component B of the examination contains 12 to 15


EXAMINATION   case-based problems with comprehensive scenarios
The National Board Dental Hygiene Examination ­(discussed later) that describe a specific situation and re-
(NBDHE) is developed and administered by the Joint quire you to demonstrate skill in the following areas:
Commission on National Dental Examinations of the • Assessing patient characteristics
American Dental Association. This examination fulfills • Obtaining and interpreting radiographs
or partially fulfills the examination requirement in all 50 • Planning and managing dental hygiene care
states of the United States and the District of ­Columbia, • Performing periodontal procedures
Puerto Rico, and the Virgin Islands. Individual states de- • Using preventive agents
termine the acceptance of the examination as a require- • Providing supportive treatment service
ment, the period before expiration of results, and the • Professional responsibility
percentage score accepted as passing. (See the Web­Links The case-based scenarios may include descriptions,
on the CD-ROM that accompanies the textbook for con- photographs, illustrations, radiographs, dental and peri-
tact sites and candidate guidelines.) odontal data, or other information necessary to present
The purpose of the NBDHE is to determine the profes- the case of both adult and child patients. Each case is fol-
sional competency of applicants for licensure. The exam- lowed by multiple-choice test items related to the scenario.
ination assesses the candidate’s ability to recall, analyze, Each examination includes at least one case regarding the
and apply basic information and concepts typically taught following types of patients who may be encountered in
in the dental hygiene curriculum. dental practice: Geriatric, Adult-periodontal, Pediatric,
The NBDHE is in English and consists overall of 350 Special Needs, and Medically Compromised; 80% of
multiple-choice questions. Of this total, approximately cases concern adult patients, and 20% children. Just as in
200 questions are traditional (stand-alone) ­ multiple- a real ­dental setting, many distractions will be included!
choice questions and the remaining 150 questions
are based on approximately 12 to 15 dental hygiene USING SAUNDERS REVIEW OF DENTAL
­treatment cases. The examination is comprehensive HYGIENE  
in that it covers topics taught to enable competency in The updated new second edition of this text has been
­performing dental hygiene care as determined by del- prepared by well-respected dental hygiene educators
egated functions of dental hygienists in the majority of who have researched the topics and concepts needed to
the nation. ­succeed on the NBDHE. One author has been a mem-
More specifically, Component A of the examination, ber of the board examination basic science committee in
the stand-alone multiple-choice section, has 176 indepen- the past and has textbooks on various dental science top-
dent items in the areas of basis for dental hygiene practice ics that are used in many dental hygiene programs in the
(~30%) and provision of clinical dental hygiene services United States and abroad. The other author owns her own
(~60%) and 24 testlet-dependent items on community NDHBE review company and conducts review courses
health or research principles (~10%). for the board exam in venues across the United States.
Multiple-choice questions contain a question stem and Both authors have worked hard to ensure that the text is
four or five answer choices (see later discussion of ques- to the point and easy to read and use as a study guide.
tion types, test-taking strategies, and test anxiety). Each The material contained in the NBDHE is primarily ap-
question has only one CORRECT answer. It is important plicable to dental hygiene practice. Success on the national
to note that some of the questions are only for trial run board examination means that the test taker has an adequate
to see if they will be included in future tests. Passage
�����������
of understanding of the concepts and applications of the
the NBDHE requires a score of 75% or greater.� dental hygiene knowledge base. This updated ­ edition of


 Saunders Review of Dental Hygiene

the textbook consists of 18 chapters. Its newly produced science, provision of clinical dental hygiene services, and
CD-ROM has a simulated NBDHE as well as other community health activities. Rationales for all questions are
IMPORTANT and fun study features. The chapters pro- included.
vide a comprehensive review of basic knowledge of dental Component B consists of 15 case-based studies de-
hygiene scientific subjects that are MORE likely to appear veloped to help you assess, plan, integrate, evaluate, and
on the NBDHE, as well as complete discussion of each of analyze clinical information. The case studies, selected
the possible types of patients who may appear in the cases to imitate situations a dental hygienist might encounter
on the examination. The material provided is so much in practice, require you to carefully assess the informa-
MORE than flash cards or dental study facts! tion provided, including medical and dental data, photo-
Each chapter in the new edition includes a short-form graphs, and radiographs (enlarged slightly for learning
clinical study and/or testlet. It is a short-form case study, purposes) to answer 150 case-specific questions. A thor-
since it is without the photographs, illustrations, radio- ough study of this section of the simulated examination
graphs, and dental and periodontal data normally pres- will help you develop the knowledge and skills neces-
ent for a complete clinical study. However, these are sary to successfully manage the cases presented on the
used to illustrate IMPORTANT application of content in NBDHE and in clinical practice. Rationales for all ques-
the chapter and contain a scenario filled with common tions are included.
­problems followed by a series of related open-ended ques- Additional study resources on the CD-ROM include
tions with answers and rationale. Presenting clinical stud- the following:
ies in this updated form FIRST is helpful in ­ improving • Chapter Review Schedule
analytical and application skills. Even though testlets are • Learning Strategies
used mainly with community health activity items on the • Mnemonic Study Tips
­NBDHE, a careful study of these short-form clinical stud- • Chapter Terms and Puzzles
ies and/or testlets with their discussions will help prepare • Study Programs
you for more complex cases with closed-ended questions • Guidelines and Graphics
that appear on the NBDHE and for situations that arise in • Chapter WebLinks
clinical practice. • Textbook References
Along with each clinical study is a comprehensive re-
view of the subject with relevant material updated for this PREPARING FOR THE NDHBE  
new edition. We recommend that you read each clinical There is NO guaranteed method for a student dental hy-
study both before and after reviewing the chapter content gienist to prepare for the NBDHE, but we would like
related to it. After you study the entire chapter’s content to share some concepts with you as you prepare for the
review and clinical studies, answer the review questions at ­examination.
the end of the chapter and compare the answers with those • Standardized testing attempts to assess knowledge and
in the rationale section that follows. Questions included comprehension of specific subject matter, as well as the
were specifically developed to be SIMILAR in format, ability to analyze information and apply the knowledge
content, and depth to those that appear on the ­NBDHE. in practical ways. Examination questions therefore run
The rationale with answer key is particularly useful be- the gamut from easy (rote memory or recall) to difficult
cause it contains discussion of why certain answers are (analysis or case study). While memorization is ade-
correct or incorrect. Thus it can help you BETTER under- quate to answer simple questions, you MUST have a
stand why one answer choice is better than another choice. deeper understanding of concepts and theory to answer
In addition, the index in the text is a useful resource to questions that assess higher level learning. However,
look up information about a specific term or topic that you should NOT forget to include the fun hints and
comes up during your study for the examination. mnemonic devices that your instructors share with you
(see Mnemonic Study Tips on the CD-ROM with the
CD-ROM with the Textbook textbook).
The CD-ROM accompanying the textbook has many sec- • The BEST preparation for the NDHBE is thorough study
tions useful in studying for the NDHBE. throughout the course of the professional program. Also,
The Computer-Simulated National Board Dental Hy- attending a review course with your study group is in-
giene Examination is comparable in length, format, and valuable and memorable, too! Review for the NBDHE
difficulty to the NBDHE. The computer-simulated ex- and other professional examinations can refresh the
amination can be taken as a timed examination or can be memory and help clarify information NOT understood
browsed through during initial studies. well previously.
The computer-simulated examination is broken down into • Proper preparation is essential for successful test taking.
two parts. Component A contains 200 randomly ordered, The MORE time you initially spend learning the material,
multiple-choice questions covering basic dental hygiene the LESS time you will spend when reviewing ­material
Preparing for the National Board Dental Hygiene Examination 

for competency testing. Conversely, those who have answer. While group study is MORE difficult to orga-
NOT mastered a subject will need to spend MORE time nize and somewhat MORE time consuming, it allows
reviewing for a competency examination, since they must group members to learn from one another, is LESS bor-
“relearn” much of the material. (See Learning Strategies ing, and sparks the discussion of topics. A study group
on the CD-ROM.) should be limited to six or fewer members, have a set
• Becoming familiar with the NDHBE examination is time and place for meeting, and require members to
vitally IMPORTANT in test taking. Take time to ex- prepare for meetings.
amine the testing materials and sample booklets that ▷ All members should be informed before the study
you can access through your program or online to gain period of what the topic of review is and how they
a general knowledge of which topics are covered and should prepare for it. This can help bring members to
in what depth, the type or style of questions on the ex- a similar level of knowledge and will usually lead to
amination and their depth and number, the sections or a more meaningful discussion. Keeping members on
parts of the examination, the amount of time allowed the task of reviewing is the MOST difficult part of the
to answer each question or section on the exam, com- group study. Keep discussion of children and pets to
mon directions or instructions used, materials that can a ­minimum!
be used during the examination, and how the examina- ▷ Set a specific start and end time for each review
tion is scored. (See the WebLinks on the CD-ROM for ­session. This helps motivate you to stay on task. Plan-
candidate guidelines.) ning the study period to end when a favorite televi-
• Schedule study time. Begin studying for national sion program or event begins can be a good incentive
board examinations approximately 3 to 6 months in and reward. Using enjoyable means of learning is also
advance. Schedule study time into your weekly calen- helpful. (See Chapter Puzzles and Chapter WebLinks
dar, ­preferably setting aside the same period each week on the CD-ROM.)
(for example, reserving Thursday mornings from 8 to ▷ After every review session, check off the subject matter
10 am for review). In this way the review is organized, studied on the Review Schedule (on the CD-ROM) and
manageable, and NOT likely to be forgotten. Set up a plan the next review topic. Charting your progress on a
schedule of study topics (use the Review Schedule on form such as the Review Schedule provides the BEST
the CD-ROM). If inadequate time remains for a thor- sense of accomplishment and further incentive to con-
ough review of all subject matter, select a few (three tinue reviewing.
to five) of your weakest subjects or the subjects with ▷ Studying individually also has its benefits and det-
greater coverage on the examination and learn them riments. Individual study sessions are easier to or-
well. Trying to review the entire curriculum in too short ganize, require less time, and allow the student to
a time will only frustrate you and seldom leads to much review at his or her own pace. This type of studying
learning. is BEST for the self-directed learner who is able to
• Plan study time in 2- to 5-hour segments. Be sure to take set a study schedule and follow through. Like study
scheduled, frequent breaks. The breaks should occur group sessions, individual study sessions should be
every 30 to 60 minutes and last less than 10 minutes. held at specific times in specific locations.
Set a timer to ring whenever a break is due. This tech- • Getting enough rest is essential before a big examina-
nique is helpful for training you to concentrate on the tion. This is much easier to accomplish when you feel
task rather than on the clock. When a break arrives, use comfortably prepared for the examination. Not surpris-
the time to visit the restroom, refill beverage contain- ingly, the ability to sleep well before a stressful event is
ers, take a brisk walk, or exercise before sitting down much greater when you feel up to the task. It is MOST
to study again. Do NOT turn on the television, pick up important that you not study the evening before a big
a magazine, or do anything that might interfere with examination. You should have completed all your prep-
the resolve to study. aration by noon the day before. Instead, do something
• Make it a rule to study the MOST difficult subject first, relaxing to relieve anxiety—take a long walk, watch a
saving easier subjects for later in the study period. In favorite movie, prepare a nice meal, and go to bed at the
this way you will avoid procrastination and achieve an usual time.
immense sense of accomplishment. Use the Chapter • Making sure to eat the CORRECT type of meals and
Terms on the CD-ROM for clarification of each sub- knowing proper nutrition are essential while you are
ject. If you are having difficulty with a particularly reviewing for the examination and before you take the
challenging ­subject, however, take on an easier one for exam. Your diet should be well balanced and provide
a day and then go back to the harder one. enough calories for sustenance, as you have learned
• Studying in groups or individually is a personal pref- in your classes. Eating properly the day of the exam
erence. For those who find it difficult to set a study has been shown to affect exam results. Do NOT eat a
schedule and follow through, group study may be the heavy carbohydrate breakfast or lunch. Rather, eat a
 Saunders Review of Dental Hygiene

full breakfast that is high in protein and a light, high- C. antibodies deficient in the Fc region.
protein lunch. This will help you stay alert throughout D. molecules that stimulate phagocytosis.*
the exam day. See Chapter 7, Nutrition, for a good re- Question-type questions are the simplest to read and
view of this information. understand. They ask a specific question for which there
• After thorough preparation, attitude is probably ���� the is one BEST answer.
MOST important factor in successful test taking. Which one of the following collimator types can most
­Taking the exam with the attitude that you are pre- ­effectively reduce scatter radiation to the patient?
pared and capable of success is extremely important. A. Long round
Many people will attest that the pressure they put on B. Pointed plastic
themselves before an examination can adversely affect C. Rectangular*
their performance. Think positively. Having a clear D. Short round
and relaxed mind is very important. A little anxiety is Negative-type (exception) questions ask the reader to
normal and can be beneficial during an examination determine which answer does NOT pertain to informa-
(see later discussion of test-taking anxiety). After all, tion in the question stem. They usually include the words
you have taken the CORRECT courses of study and “NOT,” “LEAST LIKELY,” or “EXCEPT.”
have done well; now you are to be tested on knowl- Candida albicans is known or suspected to cause all of
edge that you already have in your head and for the the following denture-related problems, EXCEPT one.
most part will use in your new career! Which one is the EXCEPTION?
• A thing you might find challenging on the exam day is A. Denture stomatitis
time expediency. Do NOT panic if you think that you B. Thrush
are taking too long to complete the test. That is why it is C. Papillary hyperplasia*
a timed exam. Some people will NOT require as much D. Angular cheilitis
time as others, and that is OK. The important thing Paired true-false questions contain two statements
is NOT to change your answers and to answer every about a related topic. You will be asked to determine if
question, even questions on which you might need to either one or both of the statements are TRUE. As with
“guesstimate.” Once you answer, leave the answer as is. multiple-choice questions having four answer choices,
Usually when people randomly change an answer, that you have a 25% chance of selecting the correct answer by
is when they get it wrong. guessing.
• Remember that this is what you want to do with your Gingival inflammation always precedes periodontal
career, so be confident that the exam is one of the disease. Periodontal disease always follows gingival
FIRST steps to reaching that first day in your profes- ­inflammation.
sion. Smile and have fun with the dear friends you have A. Both statements are true.
made in your dental hygiene program; they will prob- B. Both statements are false.
ably be your personal and professional friends for life! C. The first statement is true, the second is false.*
D. The first statement is false, the second is true.
Question Types on the NBDHE Cause-and-effect questions are similar in format to
Seven basic question formats are used on the NBDHE. paired true-false questions. They consist of a single state-
These include the completion, question, negative, paired ment that is divided into cause-and-effect portions. The
true-false, cause-and-effect, testlet, and case study ques- two portions are separated by the word “because.” You
tion formats. Each question stem is followed by four or will be asked to determine if the statement and reason are
five answer choices. CORRECT and related. As with multiple-choice questions
Remember that you CANNOT learn everything and having four or five answer choices, you have a 25% or
you CANNOT remember everything, but you can be fa- 20% chance of selecting the correct answer by ­guessing.
miliar enough with the information that you will be able Obesity is often associated with Type 1 diabetes mel-
to make an educated decision and use critical thinking to litus because obesity affects insulin resistance.
identify the CORRECT answer out of four or five mul- A. Both the statement and reason are correct and
tiple-choice answers. Following are a brief description ­related.*
and example of each type of question (correct answers are B. Both the statement and reason are correct but NOT
marked with an asterisk). related.
Completion-type questions are simple and easy to C. The statement is correct, but the reason is NOT.
read and understand. The question stem consists of a par- D. The statement is NOT correct, but the reason is
tial statement that the BEST answer completes. ­correct.
Opsonins are E. NEITHER the statement NOR the reason is correct.
A. cytokines produced by B cells. Testlet-type questions include a brief description of
B. antigens that directly neutralize viruses. a situation or case study (usually one to two paragraphs
Preparing for the National Board Dental Hygiene Examination 

in length) and a series of five or more situation-related 4. Remember that it does NOT matter if you are first or
­questions. This type of question is commonly used on last to finish…take all the time you need and try NOT
the NBDHE to test Community Health Activity items. to be distracted as people leave the testing facility.
(Refer to the Community Health Testlet on the simu-
lated ­ NBDHE on the CD-ROM that accompanies the TEST-TAKING ANXIETY  
­textbook.) However, we expanded the use of this type of Test anxiety has affected nearly everyone at some time. It
question format throughout the text in all areas of study, is often described as an extreme anxiety that causes shaky
so you can become comfortable with its easy situational legs, sick stomach, jittery hands, chaotic thinking, and
format and since it prepares you for the involved case- outright panic. If unmanaged, it can result in wasted time
based-type queations. and ineffective test taking. When you are prepared for an
Case-based-type questions contain patient informa- examination and have a positive attitude (confidence in
tion (medical and dental history, significant oral findings, your abilities) and a strategy to deal with the jitters, this
radiographs, periodontal probe readings, photographs, allows you to take control of the anxiety and make test
etc.) followed by a series of 10 to 12 case-based ques- taking a success.
tions. This type of question requires you to study the case Mild anxiety is normal and is actually helpful because
information carefully, paying particular attention to its it increases alertness. When you are faced with the pan-
focus. You must identify and analyze information useful icky feeling of high anxiety, however, your ability to con-
for answering the case questions. (Refer to the simulated centrate is reduced. Following are several effective ways
NBDHE on the CD-ROM, discussed earlier.) to prevent and control test anxiety (also see Learning
Strategies on the CD-ROM).
Test-Taking Strategies • Careful preparation for an examination is essential in pre-
for a Multiple-Choice-Style Test venting test anxiety. Knowing the material on which you
Multiple-choice-style questions found on the NDHBE are being tested provides you with confidence to pass the
tend to be MORE time consuming than other ques- exam. Procrastination and disorganized time management
tions. They require careful reading and analysis of the are usually the reasons for less than helpful exam prepara-
question stem for content and meaning. Following are tion. Use the Review Schedule on the CD-ROM to plan
some standard guides for taking multiple-choice-style study times and stick with them.
tests. • Self-confidence is key to success. A positive attitude,
1. You MUST read each question carefully for meaning. along with careful exam preparation, is essential. The
a. AVOID choices with the words “never,” “always,” type of careful exam preparation that increases self-
and “all,” since they are so restrictive that they sel- ­confidence includes a thorough review of exam subjects,
dom appear in the correct answer; nothing is ever a adequate study time, familiarization with the exam for-
definite in science or health. mat and instructions, and understanding of test scoring
b. When the question has the words “except,” “not,” (e.g., whether wrong answers are penalized or not).
or “but,” the focus of the question changes and you • Take the night off before the exam. Relax, eat properly,
MUST change your focus too. and get plenty of sleep. Be confident in your prepara-
c. Look for answers that are different from the rest. tion; studying the evening before the exam increases
If three answers say MAINLY the same thing but anxiety.
in different words, choose the answer that is more • Plan to arrive at the exam site early with needed mate-
radically different than the others. rials (identification, wristwatch, etc.) so that you have
2. At the examination site, quickly preview each ­question time to relax. Some of the hurdles you might find when
in the examination before beginning to answer each of taking the exam are extraneous noises (bring earplugs)
them. or radiographs (remember your prescription or reading
a. Try to answer each question before looking at the glasses).
possible answer choices. Find an answer choice • Relaxation techniques can help relieve anxiety. Prac-
that parallels yours. If the correct answer cannot be tice deep breathing (breathe in deeply, hold breath a
determined easily, begin first by eliminating answer few seconds, then release) for at least 10 minutes when-
choices with the MOST obviously wrong answer. ever you feel muscle tension—when studying or dur-
Once choices are limited to two or three, attempt to ing the examination. Visualization is another technique
clarify the question by redefining or restating it or that can relieve tension. To visualize, close your eyes
drawing on other knowledge. and sit comfortably. Use as many senses (hearing, vi-
b. When unsure, choose the answer that was instinc- sion, smell, taste, feel) as possible to relive a positive
tively the FIRST choice. experience. Imagine floating on water in warm sun-
3. Answer ALL questions while going through the ex- light or hearing the sounds of nature on a hike in the
amination and record the answers as indicated. woods.
This page intentionally left blank
C H A P T E R  2

Embryology and Histology

Embryology  b. Bilaminar disc (also known as disk); two-­layered,


Embryology is the study of prenatal development (gesta- flattened, essentially circular plate of cells with
tion period), which begins with fertilization and continues superior epiblast layer (high columnar cells) and
until birth. Divided into three periods: ­ preimplantation, inferior hypoblast layer (small cuboidal cells).
embryonic, fetal. c. Disc is suspended in uterus’s endometrium be-
• See CD-ROM for Chapter Terms and WebLinks. tween two fluid-filled cavities: amniotic cavity
• See Chapters 9, Pharmacology: drugs and placental faces epiblast layer, and yolk sac faces hypo-
barrier; 11, Clinical Treatment: pregnant patient. blast layer; BOTH serve as initial nourishment
A. Preimplantation period (FIRST week): for embryonic disc.
1. Union of ovum and sperm undergoes fertilization d. Placenta: prenatal organ that joins pregnant
to form egg (zygote) where final stages of meiosis woman and developing embryo; permits selec-
occur in ovum: tive exchange of soluble bloodborne substances
a. To allow formation of new individual, sperm (including drugs and nutrients) through the pla-
and ovum when joined have CORRECT num- cental barrier.
ber of chromosomes (diploid number of 46). 2. Third week:
b. Ovum and sperm have to reduce by one half a. Within the disc, primitive streak forms, which
the normal number of chromosomes (to haploid is a furrowed, rod-shaped thickening in the
number of 23). middle of the disc:
c. Thus zygote received half its chromosomes from (1) Cells from epiblast layer migrate between
female and half from male, with resultant genetic the two layers, becoming mesenchyme,
material a reflection of BOTH ­biological parents. embryonic connective tissue.
d. Photographic analysis of person’s ­chromosomes (2) Mesenchyme layer has potential to prolif-
is done by orderly arrangement of pairs in a erate and differentiate into IMPORTANT
karyotype. types of connective tissue-forming cells
2. After fertilization, zygote undergoes mitosis (in- (e.g., fibroblasts, chondroblasts, osteoblasts).
dividual cell division); solid ball of cells is a b. Differentiation of new embryonic layer within
­morula; review of the stages and terms of mitosis disc from mesenchyme, called mesoderm:
is ­recommended (see CD-ROM WebLinks). (1) Trilaminar disc: three-layered disc with
3. Because of ongoing process of mitosis and secre- epiblast layer now ectoderm and hypoblast
tion of fluid by the cells within morula, ­ zygote layer now endoderm.
becomes a blastocyst (blastula), which undergoes (a) Ectoderm: forms epidermis of skin,
successive cell divisions by mitosis: nervous system, other structures.
a. By end of first week, blastocyst stops traveling, (b) Mesoderm: forms muscle coats, con-
undergoes implantation, and becomes embed- nective tissues, vessels supplying tis-
ded in prepared endometrium, which is the in- sues and organs, other tissues.
nermost layer of uterine lining. (c) Endoderm: forms epithelial linings of
b. After 1 week of cleavage, blastocyst consists of respiratory passages and digestive tract,
trophoblast layer (layer of peripheral cells) and including some glandular organ cells.
embryoblast layer (small inner mass of embry- c. Disc now has a cephalic end (head) where oro-
onic cells). pharyngeal membrane (buccopharyngeal mem-
B. Embryonic period (weeks 2 through 8): implanted brane, future mouth) forms and caudal end (tail)
blastocyst becomes embryo after first week. where cloacal membrane forms (­future anus).
1. Second week: 3. Fourth week:
a. Increased number of cells creates embryonic a. Central nervous system (CNS) begins to de-
cell layers (germ layers) and forms disc. velop in embryo:


 Saunders Review of Dental Hygiene

(1) Neuroectoderm differentiates from ectod­erm, 3. Performed in older age groups: one or both par-
localized to neural plate. ents have chromosomal abnormality or neural tube
(2) Neural plate is a band of cells that extends defect, previous child was affected, or parents
length of the embryo, from cephalic to are carriers of inborn errors of metabolism, e.g.,
­caudal end. X-linked disorders such as hemophilia.
(3) Plate undergoes further growth and thick-
ening, which cause it to deepen and invagi- DEVELOPMENT OF FACE AND ORAL CAVITY  
nate centrally, forming the neural groove. Face and structures of the oral cavity begin development
(4) Neural groove deepens further and be- early in embryonic period. ALL three embryonic layers
comes surrounded by neural folds, which are involved in facial development (layers are discussed
meet superior to neural groove, forming a earlier). Depends on five facial processes (prominences)
neural tube. that form during fourth week and surround stomo-
(5) Neural tube undergoes fusion at its most deum (primitive mouth): (1) single frontonasal process;
­superior portion, future spinal cord. (2 and 3) paired maxillary processes; and (4 and 5) paired
b. Neural crest cells develop from neuroectoderm, mandibular processes (Figure 2-1). In the future, the sto-
then migrate from neural folds and disperse modeum will form the oral cavity.
within mesenchyme; they are IMPORTANT in A. Facial processes are centers of growth for the face:
development of head and neck structures such 1. If an adult’s face is divided into thirds, these
as branchial arches. portions roughly correspond to centers of facial
4. Embryo also undergoes folding, placing three growth:
types of tissues in proper positions; recognizably a. Upper face: from frontonasal process.
human at end of embryonic period (eighth week). b. Middle face (midface): from maxillary processes.
C. Fetal period (weeks 9 through birth): encompasses c. Lower face: from mandibular processes.
beginning of ninth week (third month) to ninth month; B. Overall growth of the face is in inferior and anterior
involves maturation of existing structures as embryo direction in relation to cranial base:
enlarges to become a fetus. 1. Growth of upper face is MORE rapid, in keeping
D. Congenital malformations (birth defects): with association with developing brain; forehead
1. Developmental problems evident at birth (or ceases to grow much after age 12.
before because of prenatal diagnostic tests): 2. In contrast, middle and lower portions of the face
­occur during BOTH preimplantation and embry- grow MORE slowly over prolonged period of time:
onic periods, thus in first trimester of pregnancy. a. Finally cease to grow late in puberty.
­Examples: b. Eruption of permanent third molars at around
a. Ectopic pregnancy: implantation that occurs 17 to 21 years marks end of major growth of
outside the uterus. lower two thirds of face.
b. Down syndrome (trisomy 21): extra chromo- C. Underlying facial bones also developing at this time
some present after meiotic division. depend on centers of bone formation by intramembra-
c. Ectodermal dysplasia: abnormal development nous ossification.
of one or more ectodermal structures.
d. Spina bifida: neural tube defect. Stomodeum and Oral Cavity Formation
2. Malformations: caused by genetic factors such as Before fourth week, stomodeum initially appears as a
chromosome abnormalities, environmental agents, shallow depression in embryonic surface ectoderm at ce-
teratogens. phalic end, limited in depth by oropharyngeal membrane
a. Teratogens: infections (e.g., rubella virus), (Figure 2-1).
drugs (e.g., fetal alcohol syndrome [FAS] A. FIRST event during fourth week is disintegration of
­causing serious physical and mental disorders, oropharyngeal membrane:
systemic tetracycline antibiotic therapy causing 1. Stomodeum is increased in depth, enlarging it to
intrinsic dental staining), radiation. become oral cavity.
b. Women of reproductive age should AVOID 2. Now there is access by way of stomodeum from
­teratogens at time of first missed menstrual pe- internal primitive pharynx to outside fluids of
riod and thereafter, to protect developing child. ­amniotic cavity surrounding embryo.
E. Amniocentesis: sampling of amniotic fluid. B. Stomodeum will form oral cavity:
1. MOST common invasive prenatal diagnostic 1. Lined by oral epithelium, which is derived from
­procedure. ectoderm, as a result of embryonic folding.
2. Fluid sampled during fourteenth to sixteenth weeks 2. Oral epithelium and underlying tissues will form
after last missed menstrual period. teeth and associated tissues.
Embryology and Histology 

4 5 6
3
8

10
2

11
1 7

6 Stomodeum
disintegrating
11 Mandibular arch 5 Oropharyngeal membrane

10 Maxillary process 4 Frontonasal process


9 Lens placode 3 Oropharyngeal membrane

8 Frontonasal process 2 Maxillary processes


7 Mandibular symphysis 1 Mandibular processes

Figure 2-1  Embyro at third to fourth weeks of development. (From Bath-Balogh M, Fehrenbach MJ:
­Illustrated dental embryology, histology, and anatomy, ed 2, St. Louis, 2006, Saunders/Elsevier.)

Mandibular Arch and Lower Face Formation


During fourth week, two bulges of tissue appear infe- frontonasal­ process will form upper face, which includes
rior to stomodeum; these paired mandibular processes ­forehead, bridge of nose, primary palate, nasal septum, and
are formed in part by neural crest cells that migrated to all structures related to medial nasal processes (Figure 2-1).
the facial region, covered externally by ectoderm and in- A. Placodes become submerged, forming depression in
ternally by endoderm (Figure 2-1). center of each:
A. Paired mandibular processes BOTH fuse at midline 1. Form nasal (olfactory) pits, which develop into
to form the mandibular arch (mandibular symphysis nasal cavities.
is indication): 2. Deepening of nasal pits produces a nasal sac
1. Mandibular arch and related tissues (also known as that grows internally toward developing brain;
first branchial arch) are first portions of the face to separated from the stomodeum by oronasal mem-
form after creation of stomodeum. brane.
2. Directly forms lower face, including lower lip. B. On outer portion of nasal pits are two crescent-shaped
3. Will also form mandible, mandibular teeth, associ- swellings, lateral nasal processes that form the alae
ated tissues, as well as tongue. (sides of the nose).
B. Meckel’s cartilage forms within each side of mandibu- 1. Middle portion of the tissue growing around nasal
lar arch, IMPORTANT in alveolar bone development. placodes appears as medial nasal processes.
1. Makes contribution to mandible, and portion of 2. BOTH fuse together externally to form the middle
cartilage participates in formation of middle ear portion of the nose from root to apex, center por-
bones. tion of upper lip, philtrum region.
2. Part of perichondrium surrounding Meckel’s carti- C. BOTH paired medial nasal processes also fuse inter-
lage becomes ligaments of the jaws and middle ear. nally and grow inferiorly on inside of stomodeum:
C. Mesoderm of mandibular arch forms muscles of mas- 1. Form intermaxillary (premaxillary) segment,
tication, as well as some palatal muscles and suprahy- MOST anterior portion of the tissues.
oid muscles. Because these muscles are derived from 2. Involved in formation of maxillary incisor teeth and
mandibular arch, they are innervated by nerve of the associated tissues, primary palate, nasal ­septum.
first arch, fifth (V) cranial nerve (trigeminal).
Maxillary Process and Midface Formation
Frontonasal Process and Upper Face Formation During fourth week, adjacent swellings, the maxillary
During fourth week, frontonasal process is a bulge of ­tissue processes, form from increased growth of mandibular
in upper facial area, at most cephalic end of embryo, which arch. Maxillary processes grow superiorly and anteriorly
is also the cranial boundary of the stomodeum. In the future, on each side of stomodeum (Figure 2-1).
10 Saunders Review of Dental Hygiene

A. Maxillary process will form the midface: ­fetus (Figure 2-2). Formed from two separate embryonic
1. Includes sides of upper lip, cheeks, secondary palate. structures, primary palate and secondary palate. Palate is
2. Also posterior portion of maxilla, with maxillary developed in three consecutive stages: formation of pri-
canines and posteriors, associated tissues. mary palate, formation of secondary palate, completion
3. Also zygomatic bones and portions of temporal bones. of final (definitive) palate.
B. Each maxillary process fuses with each medial nasal A. Intermaxillary segment forms primary (primitive)
process: palate, a triangular mass:
1. Contribute to sides of upper lip, and the two medial
nasal processes contribute to middle of upper lip.
2. If the maxillary process on each side does NOT
fuse with medial nasal process, cleft lip can result A 4
(discussed later).
1 5
Branchial Apparatus Formation
6
Branchial apparatus consists of the branchial arches,
2
branchial grooves and membranes, pharyngeal pouches. 7
A. Arches are six pairs of U-shaped stacked bilateral 3
swellings of tissue appearing inferior to the stomo- 8
deum and include the mandibular arch (FIRST bran-
chial arch), which will form the lower face; each arch
B
will form into different structures.
1. Each arch contains cartilaginous core, aortic arch,
definite cranial nerve; each cranial nerve will sup- 9
ply the structures that develop from the mesen-
10 12
chyme of the arch.
2. Second (hyoid) arch enlarges and grows so that by 13
11
the sixth week it will overlap the third, fourth, and 14
sixth arches and cover them.
3. Tracts of epithelial cells can remain after develop-
ment of the second pharyngeal arch; on occasion, C 18
a fistula opens to both the external surface and the
pharynx, connecting these structures and becom- 15 19
ing a complete branchial fistula.
20
4. However, a fistula can open externally on the an- 16
terolateral surface of the neck or internally to the 17
pharynx by way of a ruptured pharyngeal mem- 21
brane and become an incomplete branchial fistula.
B. Between neighboring branchial arches, branchial
(pharyngeal) grooves are noted on each side of the
11 Developing tongue

embryo; FIRST groove forms the external auditory 21 Developing mandible 10 Palatal shelf

meatus and mesenchyme of first and second arches, 20 Fusing palate Maxillary process 9
which are located on either side of this pharyngeal
groove, and will also give rise to the external ear.
19 Nasal septum Developing mandible 8

1. Four well-defined pairs of pharyngeal pouches 18 Developing nasal cavity Palatal shelf 7

develop as endodermal evaginations form the lat- 17 Tongue Stomodeum 6


eral walls lining the pharynx and form different
­structures.
16 Oral cavity Maxillary process 5

2. Space between the second arch and the other three 15 Maxillary process Developing nasal septum 4

arches is called the cervical sinus and is lined by 14 Developing mandible Developing tongue 3
ectoderm; sinus can undergo enlargement at a later
time and form cervical cysts.
13 Stomodeum Palatal shelf 2

Palatal Development 12 Palatal shelf Maxillary process 1

Formation of the palate starts in the embryo during Figure 2-2  Development of palate. (From Bath-Balogh M,
fifth week, takes place during several weeks of prenatal Fehrenbach MJ: Illustrated dental embryology, histology, and
­development, and is completed in twelfth week in the a­ natomy, ed 2, St. Louis, 2006, Saunders/Elsevier.)
Embryology and Histology 11

1. In the future, primary palate will form premaxil- C. Surgical intervention:
lary portion of the maxilla, anterior one third of 1. Plastic surgery: lip closure (adhesion) is ­performed
final (definitive) palate. before third month of life; palatal closure is per-
2. Consists of small portion of the hard palate, ante- formed at 1 to 5 years of age.
rior to incisive foramen; contains maxillary incisor 2. Bone grafting, orthodontics, and speech and hearing
teeth. evaluation and treatment occur at 6 to 15 years of age.
B. During sixth week of prenatal development (within 3. Final plastic surgical procedures and replace-
embryonic period), bilateral maxillary processes form ment of teeth occur after age of 15, when physical
two palatal shelves (lateral palatine processes), BOTH growth is complete.
fusing together to form secondary palate (fusing from 4. Medical anomalies such as heart, ear, skeletal, and
anterior to posterior): genitourinary tract deformities may also occur.
1. Secondary palate will form MOST posterior two D. Oral signs: increased risk of oral infection (including
thirds of hard palate, contains maxillary canines periodontal disease, dental caries) from malposition-
and posteriors, posterior to incisive foramen as well ing of the teeth, wearing of a full or partial denture to
as soft palate and its uvula (fusion line ­indicated replace teeth or use of an obturator (dental appliance
by median palatine raphe). covering clefted area), mouth breathing, associated
2. If the palatal shelves do NOT fuse with the pri- oral deformities; all complications also make oral hy-
mary palate (or with each other), cleft palate can giene procedures MORE difficult.
result, with varying degrees of disability (see next E. Risk factors for oral health: upper respiratory and
discussion). middle ear infections and inadequate nourishment
(before completion of surgical procedures), since
Patient with Cleft Palate and/or Cleft Lip cleft complicates nursing and/or feeding.
Cleft palate and/or cleft lip are craniofacial deformities F. Barriers to care:
that occur during fourth to twelfth weeks of prenatal de- 1. Economic issues; multiple oral and facial surgeries
velopment. Etiology is unknown; may involve genetics and care by professionals from different disciplines are
and exposure to environmental factors such as drugs or required to correct defect and associated conditions.
toxins (tobacco use, retinoic acid analogues, alcohol, 2. Difficult communication because of inadequate
anticonvulsants) during early prenatal development. speech production, hearing loss related to defect,
MOST common class of congenital malformations in self-consciousness.
United States. G. Professional and homecare:
A. Cleft lip: failure of fusion of maxillary processes 1. Frequent oral prophylaxis (every 3 or 4 months) to
with medial nasal process: reduce risk of infection.
1. Located at side of upper lip; unilateral or bilateral; 2. When premaxilla is unfixed or immediately after
may range from LEAST severe case, notch in ver- surgical procedures, fulcruming during instrumen-
milion border of upper lip (incomplete cleft), to tation at the site SHOULD be avoided or limited.
MORE severe cases (complete cleft) that extend 3. Fluoride and calcium products to reduce incidence
into floor of the nostril and through alveolar process of caries.
of maxilla. H. Patient or caregiver education:
2. Cleft lip, with or without cleft palate, occurs in 1. Supervision and/or performance of oral hygiene
about 1 in 1000 live births. procedures, depending on age and abilities.
3. Cleft lip is MORE common and MORE severe in 2. Care of dental appliance or prosthesis similar to care
males and is MORE frequently unilateral, occur- of denture; removal after meals to cleanse thor-
ring on left side. oughly and reduce halitosis (see Chapter 15, Dental
B. Cleft palate: failure of fusion of palatal shelves with Biomaterials).
primary palate (or with each other): 3. Risk for infection and need for excellent daily
1. Cleft uvula (bifid uvula) is the LEAST severe form. homecare, frequent professional oral prophylaxis
2. Occurs, with or without cleft lip, in 1 in 2500 live and examination; care to reduce periodontal dis-
births. ease or caries.
3. Isolated forms are LESS common than cleft lip I. Commissural lip pits (dimple like) can occur at one or
and are MORE common in females. both commissures and are hereditary.
12 Saunders Review of Dental Hygiene

CLINICAL STUDY  

Age 21 YRS Scenario

Sex ☒  Male   ☐  Female Upon extraoral examination during an initial appoint-


ment, the dental hygienist notes that there is a notch
Height 5’�����
8”� in the upper lip on the left side of the patient. When
asked about it, the patient says it has been there since
Weight 150 LBS
he was born. Nothing else is noted.
BP 115/75

Chief ­Complaint None

Medical ­History Hay fever

Current Medications OTC allergy medications prn

Social History College student in engineering

1. Why is there a notch noted the patient’s upper lip? A. Develops from independent swellings located inter-
2. How did this occur in the patient? nally on floor of the primitive pharynx, formed by
3. How will this affect the patient’s treatment? first four paired branchial arches:
1. Body: develops from first paired branchial arches.
2. Base: originates later from second, third, fourth
1. Notch in the vermilion border of upper lip may be an paired branchial arches.
incomplete cleft of the lip. 3. Body and base fuse together; fusion demarcated by
2. Each maxillary process fuses with each medial nasal pro- sulcus terminalis.
cess to contribute to the sides of upper lip, and two medial B. Ankyloglossia (“tongue tied”): short lingual frenum
nasal processes contribute to middle of upper lip. Failure of that extends to tongue apex:
fusion of maxillary processes with medial nasal process can 1. Restricts movement of tongue to varying degrees.
result in cleft lip. Clefts of the lip are located at one or both 2. Frenum usually stretches with time or myofunc-
sides of upper lip; they are unilateral or bilateral and may tional exercises; thus surgical correction may NOT
range from notch in vermilion border of upper lip (incom- be necessary.
plete cleft) to more severe cases (complete cleft) that extend
into floor of nostril and through alveolar process of maxilla; TOOTH DEVELOPMENT 
sometimes associated with cleft palate. Cleft lip, with or Child’s primary (deciduous) dentition develops during
without cleft palate, occurs in about 1 in 1000 live births. prenatal period and consists of 20 teeth, which erupt and
3. Since it is only notch in the vermilion border of the lip, are later shed (lost). As primary teeth are shed and jaws
this will not affect treatment. However, the dental hy- grow and mature, permanent dentition, consisting of as
gienist should ask if the patient has any surgical repair many as 32 teeth, gradually erupts and replaces primary
history or if any complications have resulted from the dentition (Figure 2-3). Primary dentition begins on aver-
notch. Consideration should be given to the way the age with eruption of the mandibular central incisor at 6 to
question is asked, since the patient may be sensitive 10 months and is completed with eruption of the maxillary
about this facial difference. The dental hygienist may second molar at 25 to 33 months. Permanent dentition be-
also ask if she can answer any questions about the dis- gins on average with the eruption of the mandibular first
order and note the presence of the notch in the chart. molar or central incisor at 6 to 7 years and is completed
Cleft lip is more common and more severe in males, is with the eruption of the third molars at 17 to 21 years. Dis-
more frequently unilateral, and occurs mostly on left cussion of tooth development will center first on the pri-
side. If severe, it can complicate nursing and/or feed- mary dentition and then include the permanent dentition.
ing, speech development, appearance, and oronasal in- • See Chapters 4, Head, Neck, and Dental Anatomy: den-
fection levels. Treatment includes plastic surgery with titions; 6, General and Oral Pathology: developmental
dental intervention and speech and hearing therapy. dental disorders.
A. Initiation stage:
Tongue Development 1. FIRST stage for primary dentition begins between
Tongue develops during fourth to eighth weeks of prena- the sixth and seventh weeks (during embryonic
tal development. ­period).
Saunders/Elsevier.)
Figure 2-3  Tooth development. (From Fehrenbach MJ, ed: Dental anatomy coloring book, St. Louis, 2008,
24
10
11 16
20 15
19 9
23 22
Maturation stage
Initiation and bud stages 20
1
9
Oral epithelium 8 19
2 Dental lamina 18
3 Ectomesenchyme 21
17
4 Tooth bud 16
1
Cap stage Apposition stage
5 Tooth germ
15
6 Enamel organ 14
13
7 Dental papilla
11
12 10
8 Dental sac 9
Bell stage 7
9 Stellate reticulum
10 Stratum intermedium
6
11 Inner enamel epithelium
12 Basement membrane Bell stage
13 Outer cells of the dental papilla
14 Central cells of the dental papilla 8
15 Outer enamel epithelium 7 5
Apposition stage 6
16 Ameloblasts
17 Enamel matrix Cap stage
18 Dentin matrix
3
19 Odontoblasts
4
20 Pulp 2
21 Dentinoenamel junction
Maturation stage 1
22 Enamel Initiation and bud stages
23 Dentin
24 Hertwig’s epithelial root sheath 3
2
1
13 Embryology and Histology
14 Saunders Review of Dental Hygiene

2. Stomodeum (primitive mouth) is lined by ecto- E. Apposition stage:


derm, with outer portion of ectoderm forming oral 1. Enamel, dentin, cementum are secreted in succes-
epithelium. sive layers.
3. Deep to this is ectomesenchyme, influenced by a. Initially secreted as a matrix, extracellular sub-
neural crest cells; basement membrane separates stance that is partially calcified yet serves as a
the two tissues, future site of dentinoenamel junc- framework for later calcification:
tion (DEJ). (1) Preameloblasts repolarize and differenti-
4. During seventh week, oral epithelium grows ate into ameloblasts and then induce den-
deeper into the ectomesenchyme and is induced to tal papilla cells to repolarize and become
produce the dental lamina for the formation of the odontoblasts.
succedaneous teeth (all teeth EXCEPT permanent (2) Thus odontoblasts produce predentin before
molars). ameloblasts produce enamel from Tomes’
B. Bud stage: process.
1. Second stage occurs at eighth week (during em- b. With enamel matrix in contact with preden-
bryonic period). tin, mineralization of disintegrating basement
2. Growth of dental lamina into buds penetrating membrane now occurs, forming DEJ.
into ectomesenchyme; each will develop into a 2. Structure responsible for root development is the
tooth germ. cervical loop, most cervical portion of enamel organ:
C. Cap stage: a. Bilayer rim consisting of ONLY inner and outer
1. Third stage occurs between ninth and tenth weeks enamel epithelium.
(during fetal period). b. Grows deeper into the dental sac to become
2. Differentiation of the tooth bud’s dental lamina into Hertwig’s epithelial root sheath (HERS):
the enamel organ, which will produce enamel, an (1) Function of HERS is to shape the root(s).
ectodermal product, in the future. (2) Also induces dentin formation in root area
a. Enamel organ covers dental papilla from the so that it is continuous with coronal dentin,
ectomesenchyme. as well as cementum on roots overlying the
b. Dental papilla will produce future dentin and newly formed dentin.
pulp tissue, which have mesenchymal origin. c. After the disintegration of HERS, its cells may
c. Remaining ectomesenchyme surrounding become epithelial rests of Malassez, which
ena­mel organ (outside of the cap) forms into are located in the mature PDL and may become
dental sac (follicle), which will produce peri- cystic in the future (periapical cyst formation).
odontium, cementum, periodontal ligament 3. Ectomesenchyme from dental sac (follicle) begins
(PDL), alveolar bone; all of mesenchymal to form PDL, adjacent to newly formed ­cementum.
origin. 4. After enamel apposition ceases in crown area, am-
3. Together, enamel organ, dental papilla, and dental eloblasts place an acellular dental cuticle on new
sac form the tooth germ (tooth primordium), giv- enamel surface and the enamel organ is ­compressed,
ing the tooth bud its overall shape. forming reduced enamel epithelium (REE).
4. Extension of dental lamina into ectomesenchyme 5. The REE fuses with oral epithelium lining the oral
lingual to developing primary tooth germs forms cavity to allow eruption process for both dentitions.
the successional dental lamina for the suc- a. Primary teeth act as guides for the developing
cedaneous permanent teeth; for nonsuccedane- permanent teeth; premature loss of primary
ous ­permanent molars, which have NO primary teeth can have a serious effect on eruption of
­predecessors, these teeth will form from a poste- permanent teeth and their position in the dental
rior extension of the dental lamina. arch.
D. Bell stage: b. To allow removal of the primary for replace-
1. Fourth stage occurs between eleventh and twelfth ment by the permanent tooth, odontoclast
weeks (during fetal period). resorption of primary teeth (MAINLY root,
2. Stage marked by extensive proliferation, with possibly crown) occurs.
differentiation of the enamel organ into four c. Succedaneous permanent tooth erupts into oral
­layers: cavity in position lingual to roots of shed pri-
a. Outer and inner enamel epithelium surrounds mary tooth, just as it will develop, EXCEPT
inner cells; inner cells become preameloblasts. permanent maxillary incisors, which move into
b. Stellate reticulum and stratum intermedium are a MORE facial position for eruption.
located between the two outer and inner layers; d. After a permanent tooth erupts in the oral cavity,
provide support for ameloblasts. its root begins to form in the cervical area after
Embryology and Histology 15

1
9
2
3 10

4
11

5
12

13
6

8
14

reticulum
14 Smooth endoplasmic
Centriole 7
reticulum with ribosomes
13 Rough endoplasmic Microfilaments 6

12 Nucleus Peroxisome 5

11 Nucleolus Plasma membrane 4

10 Microtubules Exocytotic vesicle 3

Mitochondrion 9 Golgi complex (apparatus) 2

Lysosome 8 Vesicle 1

Figure 2-4  Cell structure.

crown formation. Thus prevention of ­traumatic Cell Structure


injury to the permanent teeth before they are A cell is the smallest unit of organization in the body. Cell
fully anchored into the jaws is very important structure includes cell components, such as organelles,
(see Chapter 11, Clinical Treatment). plasma, and cytoskeletal elements. All are IMPORTANT
e. Nasmyth’s membrane: residue of the eruption in cell function (Figure 2-4).
process that may form on newly erupted teeth • See Chapter 3, Anatomy, Biochemistry, and Physiol-
of BOTH dentitions; leaves teeth with extrinsic ogy: cell physiology.
green stain that is MAINLY removed by first A. Plasma membrane: surrounds each cell; selectively al-
brushing. lows movement of water and other substances into and
F. Maturation: reached when the dental tissues mineral- out of cells and is involved in cell-to-cell recognition:
ize into their mature hard tissues. 1. Fluid mosaic model is the current working model
of a membrane.
GENERAL HISTOLOGY  2. Contains proteins that consist of channels, pumps,
Histology is the study of the structure, composition, and and transport systems; these regulate composition
function of tissues that form organs and then into systems. of the intracellular fluid.
Tissues consist of a group of similar cells that combine in 3. Components: phospholipids, which are polar and
a characteristic pattern and perform a specific function. In have charged heads; lipids, which are ­hydrophobic;
addition to cells, intercellular substance and tissue fluid cholesterol, which affects membrane fluidity;
tissues are found in tissues. ­proteins, such as channels, enzymes, receptors,
• See CD-ROM for Chapter Terms and WebLinks. carbohydrates.
16 Saunders Review of Dental Hygiene

B. Endoplasmic reticulum (ER): fluid-filled membrane 2. Microfilaments are composed of helically inter-
organelle found within cells; has two types: twined chains of actin and myosin found in mus-
1. Rough ER: contains large amounts of ribosomes, cles (contraction) and microvilli.
involved in synthesis of proteins for cell struc- 3. Intermediate filaments and rods are made of pro-
ture and function, found in large amounts in tein and form scaffolding (skeleton) of cell (cyto-
growing cells and in cells that make digestive skeleton).
enzymes. K. Extracellular fluid: interstitial fluid and plasma lo-
2. Smooth ER: production of steroid hormones, de- cated outside cell; high in Na+, Cl–, Ca+2; low in K+;
toxifies harmful substances, produces lysosomes, pH approximately 7.4.
uses calcium in muscle cells for contraction and L. Cytoplasm (intracellular fluid): located within the
relaxation. cell membrane, low in Na+, Cl–, Ca+2; high in K+; pH
C. Golgi apparatus (complex): made up of layers of approximately 7.0.
membranes and enclosed sacs, involved in processing
substances made in the ER, directs finished products Tissue Components and Types
to intracellular or extracellular locations. Tissue components include cells, cell products (known
D. Vesicles: membrane-surrounded buds, formed from as intercellular substance), and fluids derived from blood
the Golgi apparatus: plasma (tissue fluid). Tissue cells vary considerably in
1. Coated vesicles contain chemicals for intracel- size, shape, structure, and function. Intercellular sub-
lular use; secretory vesicles are used for export stance consists of fibrous elements, such as collagen,
(exocytosis). and ground substance, known as mucopolysaccharides.
2. Also involved in trafficking substances into the Tissue fluid is a component of blood plasma that serves
cell from the environment (e.g., endocytosis, im- to carry nutrients to the cells of the tissue. Tissues may
port; pinocytosis, absorption of fluids; phagocyto- be grouped into four MAIN types: epithelial, connective,
sis, engulfing of solids). nerve, muscle. ALL types of tissues can be found in the
E. Lysosomes: membrane-enclosed organelles derived oral cavity.
from the Golgi apparatus, contain enzymes to digest • See Chapters 3, Anatomy, Biochemistry, and Physi-
unwanted debris and foreign matter, involved in apop- ology: muscle tissue; 14, Pain Management: nerve
tosis (tissue ­regression). tissue.
F. Peroxisomes: membrane-enclosed organelles that A. Epithelial tissues:
­contain oxidative enzymes (e.g., catalase) for detoxi- 1. Surface coverings (e.g., skin) lining tissues (e.g.,
fying chemicals. mucous membranes, including oral mucosa in oral
G. Mitochondria: organelles with a folded inner mem- cavity) or specialized tissues (e.g., salivary glands
brane (containing cristae with respiratory ­ enzymes) and tooth enamel).
and outer membrane: 2. Avascular (NO blood supply), rely on neighbor-
1. Produce energy for cell in form of adenosine tri- ing connective tissue for nutrients and removal
phosphate (ATP), produce heat (e.g., in brown fat). of cell waste (via blood vessels in connective
2. Found in greater numbers within metabolically ac- tissue).
tive cells such as cardiac and skeletal muscles. 3. Classified according to type and arrangement of cells.
H. Ribosomes: granules of RNA and proteins that are a. Simple epithelium: single layer of squamous,
found on ER or are free in cytoplasm; involved in cuboidal, or columnar cells.
protein production. b. Pseudostratified epithelium: falsely appears
I. Nucleus: contains DNA molecules that act as the ge- stratified; only one layer present.
netic “blueprint” for the cell’s protein production: c. Stratified epithelium: consists of two or more
1. Has inner and outer bilayers that make up nuclear layers of cells (discussed later); includes skin
envelope; pores for transporting substances across and oral mucosa of oral cavity.
membranes. B. Connective tissues:
2. Has nucleolus that is involved in the assembly of 1. Contain fibers and cells: fibroblasts, macrophages,
ribosomes. mast cells.
3. Organelle MOST sensitive to radiation. 2. Provide functions: transporting material (vascu-
J. Cytoskeleton: three-dimensional system of support lar tissue), providing structural support (cartilage
within the cell that includes microtubules, which and bone), forming ligaments, supporting and
are long, slender, hollow tubes involved in cell surrounding other tissues (located inferior to base-
­movement: ment membrane of skin and mucous membranes,
1. Transport of vesicles within the cell, formation of including oral cavity as the lamina propria and
mitotic spindles in cellular reproduction. PDL).
Embryology and Histology 17

1 8
2 9

10
4
5

10 Lamina propria Papillary layer 5


Basement membrane 9 Basal layer 4

Oral epithelium 8 Prickle layer 3

Submucosa 7 Granular layer 2

Dense fibrous layer 6 Keratin layer 1

Figure 2-5  Oral mucosa. (From Fehrenbach MJ, ed: Dental anatomy coloring book, St. Louis, 2008, ­Saunders/
Elsevier.)

3. Classified as loose, dense, cartilage; bone, or fluid • See Chapter 6, General and Oral Pathology: oral
(blood and lymph). ­mucosal hyperkeratinization.
C. Muscle tissues: A. Lining mucosa: buccal mucosa, labial mucosa, al-
1. Specialized fibers that allow contraction. veolar mucosa, floor of the mouth, ventral surface of
2. Classified as skeletal, smooth, or cardiac; consid- the tongue, soft palate.
ered involuntary (e.g., heart muscle, contracts in 1. Nonkeratinized squamous epithelium:
response to body controls) or voluntary (e.g., arm, a. Basal layer (stratum basale): innermost single
leg, and trunk muscles). layer of cuboidal epithelial cells, overlying base-
D. Nerve tissue: ment membrane, superior to lamina propria.
1. Part of central nervous system (CNS: brain and (1)  Produces basal lamina of the basement
spinal cord) and peripheral nervous system (PNS: membrane.
associated with other body organs). (2)  Mitosis (cell division) of epithelial cells oc-
curs within, creating new cells for the tissue.
ORAL MUCOSA  b. Intermediate layer (stratum intermedium): su-
Oral mucosa almost continuously lines the oral cavity. perficial to the basal layer; composed of larger,
Composed of stratified squamous epithelium overlying stacked, polyhedral-shaped cells.
lamina propria (connective tissue proper) (Figure 2-5). c. S uperficial layer (stratum superficiale):
Three MAIN types of oral mucosa are found in the oral outermost layer; larger, similarly stacked,
cavity: lining, masticatory, specialized mucosa; for regional polyhedral epithelial cells; the MOST su-
differences associated with each type, see Table 2-1. perficial flatten into squames, showing
18 Saunders Review of Dental Hygiene

Table 2-1  Regional differences in oral mucosa

Region/appearance Epithelium Lamina propria Submucosa

Lining Mucosa

Labial mucosa and buccal Thick Irregular and blunt CT Present with adipose and ­minor
mucosa: opaque pink, ­nonkeratinized ­papillae, some elastic salivary glands, with firm
shiny, moist; with areas of fibers, extensive vascular ­attachment to muscle
melanin pigmentation and supply
Fordyce’s spots possible

Alveolar mucosa: ­reddish Thin CT papillae sometimes ab- Present with minor salivary
pink, shiny, moist, ­nonkeratinized sent, many elastic fibers, glands and many elastic fibers,
­extremely mobile with extensive vascular with loose attachment to
supply muscle or bone

Floor of the mouth and Extremely thin Extensive vascular supply Present
ventral tongue surface: ­nonkeratinized Floor: broad CT papillae Floor: adipose with subman-
reddish pink, moist, shiny, dibular and sublingual glands,
­compressible, with vascular loosely attached to bone/
blue areas; mobility varies muscles
Ventral tongue: numerous Ventral tongue: extremely thin
CT papillae, some elastic and firmly attached to muscle
fibers, minor salivary
glands

Soft palate: deep pink with Thin ­nonkeratinized Thick lamina propria with Extremely thin with adipose
a yellow hue and moist numerous CT papillae and ­tissue and minor salivary
surface; compressible and distinct elastic layer glands, with a firm attachment
extremely elastic to underlying muscle
Masticatory Mucosa

Attached gingiva: opaque Thick keratinized ­ Tall, narrow CT ­papillae, Not present
pink, dull, firm, ­immobile; (mainly extensive vascular
with ­areas of melanin ­parakeratinized, ­supply, and serves as a
pigmentation­ possible and some ­mucoperiosteum to bone
varying amounts of slipping ­orthokeratinized)

Hard palate: pink, immobile, Thick Medial portion, rugae Present only in lateral portions,
and firm medial portion, ­orthokeratinized and raphe serve as a with anterior part having
with rugae and raphe; ­mucoperiosteum to bone ­adipose and posterior part
­cushioned lateral portions having minor salivary glands.
Absent in medial portion.

CT, Connective tissue.

shedding or loss as they age and die during two types of keratinization: parakeratinized
turnover of tissue. (retained nuclei in cytoplasm) and orthokera-
B. Masticatory mucosa: attached gingiva, hard palate, tinized (NO nuclei); note that healthy skin does
dorsal surface of the tongue. NOT show parakeratinization, which is nor-
1. Keratinized squamous epithelium: mally found ONLY in oral cavity.
a. Deeper layers: same nonkeratinized squamous 2. Lamina propria: connective tissue deep to base-
epithelium described above is present over ment membrane for BOTH types of epithelium;
basement membrane. has two layers:
b. Prickle layer (stratum spinosum): superficial to a. Papillary layer: MORE superficial; consists of
basal layer; cells look spiky when fixed. loose connective tissue within connective tissue
c. Granular layer (stratum granulosum): superficial to papillae.
prickle layer; cells in this layer are flat and stacked b. Dense layer: deepest dense connective tissue;
and have keratohyaline granules in cytoplasm. large amount of fibers.
d. Keratin layer (stratum corneum): MOST super- 3. Submucosa may or may NOT be present deep to
ficial, consists of squames filled with keratin; lamina propria.
Embryology and Histology 19

Table 2-2  Comparison of lingual papillae

Comparison Filiform Fungiform Foliate Circumvallate

Clinical Most common on Lesser numbers on 4 to 11 vertical ridges 7 to 15 large raised


­appearance body; fine-pointed body; ­mushroom- on the lateral mushroom-
cones give the shaped small red dots ­surface of the shaped structures
tongue a velvety ­posterior tongue anterior to the
texture sulcus terminalis

Microscopic Pointed structure Mushroom-shaped Leaf-shaped ­structure Mushroom-shaped


­appearance with a thick layer structure with a thin of keratinized ­structure with
of ­keratinized layer of keratinized epithelium ­overlying similar histology
­epithelium, epithelium ­overlying a core of lamina to ­fungiform, that
­overlying a core of a core of lamina propria, with taste is, sunken deep
lamina propria; no ­propria, with taste buds superficial to the tongue
taste buds. buds in the most surface, taste
superficial portion buds in base, and
­surrounded by a
trough, with von
Ebner’s minor
salivary glands in
the submucosa

Function Possibly mechanical Taste Taste Taste

C. Specialized mucosa: lingual papillae on dorsal and a. Tip and lateral borders are marginal tissue,
lateral surface of the tongue; consist of epithelium ­continuous with the free gingiva.
and lamina propria; see Table 2-2 for comparison of b. Center of the papilla consists of attached gin-
lingual papillae. giva; its shape is determined by amount of in-
terproximal space created by contact point of
Gingival Tissues adjacent teeth.
Gingival tissues surround the teeth in the alveoli and c. In healthy tissue is pointed in the anterior, then
cover the alveolar processes. flatter and wider in the posterior.
• See Chapters 6, General and Oral Pathology: gingival 2. Col: depression between lingual and facial ­papillae.
hyperplasia; 11, Clinical Treatment: periodontal eval- a. Shape conforms to interproximal space; con-
uation and charting; 12, Instrumentation: probing; 13, sists of nonkeratinized tissue.
Periodontology: gingival conditions or diseases. b. MORE susceptible to inflammation because of
A. Marginal (free) gingiva: located at gingival margin fragility; FIRST site of periodontal inflammation.
of each tooth: 3. Mucogingival junction: scalloped line that marks
1. Width is 0.5 to 1.5 mm (approximates level of base connection between alveolar mucosa and attached
of gingival sulcus). gingiva.
2. Continuous with attached gingiva; has SIM­ C. Alveolar mucosa: movable tissue loosely attached to
ILAR features, with mainly parakeratinized epi- underlying bone; attached by frena (singular, frenum),
thelium. band(s) of tissue that attaches MORE fixed tissue to
3. Gingival margin (free gingival crest): MOST su- MORE movable tissue; may pull (displace) tissue
perficial portion of marginal gingiva. when attached gingival margin is narrow or missing.
4. Free gingival groove: shallow linear groove that de- D. Dentogingival junction tissues: ALL tissues that
marcates marginal gingiva from attached ­gingiva. face the tooth; nonkeratinized epithelium, UNLIKE
B. Attached gingiva: composed of stratified, keratin- ­marginal and attached gingiva, with smooth ­interface
ized epithelial tissue firmly bound to underlying between tissues and underlying lamina propria
cementum and alveolar bone (masticatory mucosa); (Figure 2-6):
measured from free gingival margin to mucogingival 1. Sulcular (crevicular) epithelium (SE): stands
junction; stippling can be present in health. Tissues away from the tooth, creating gingival sulcus,
can show orthokeratinization or (MAINLY) parakera- filled with crevicular gingival fluid (CGF) from
tinization (retention of nuclei in superficial layers). blood vessels of lamina propria:
1. Interdental papilla: occupies interdental space a. Depth of healthy gingival sulcus varies from
between two adjacent teeth. 0.5 to 3 mm, with average of 1.8 mm.
20 Saunders Review of Dental Hygiene

6 7

1 8

9
10
2
13
11
12

14

3 15

4 16
5

16 Cementum Sulcular epithelium 8


15 Periodontal ligament Tooth surface 7
14 Gingival fiber group Gingival sulcus 6

13 Junctional epithelium Alveolar bone 5


12 External basal lamina Alveolar mucosa 4
11 Cementum Attached gingiva 3
10 Internal basal lamina Free gingival groove 2

Enamel 9 Marginal gingiva 1

Figure 2-6  Dentogingival junction tissues. (From Fehrenbach MJ, ed: Dental anatomy coloring book, St.
Louis, 2008, Saunders/Elsevier.)

b. Normal gingival fluid flow rate of CGF is regular; s­ uprabasal epithelial cells provide hemides-
however, amount in healthy state is minimal. mosomes and internal basal lamina that
c. CGF seeps out between epithelial cells into the create EA (cell to noncellular intercellular
sulcus; includes white blood cells (WBCs); MAIN junction type).
cells are polymorphonuclear leukocytes (PMNs, (2) Permeable tissue that allows emigration
neutrophils) as well as IgG, IgM, serum IgA. of large numbers of mobile WBCs from
2. Junctional epithelium (JE): deeper extension of lamina propria’s blood vessels into JE and
SE into sulcus: then into GCF, even in healthy state to pro-
a. Created during tooth eruption from fused tis- tect JE.
sues (REE) along tooth surface. c. Basal layer of JE undergoes constant and rapid
b. Continuous with SE of the gingival sulcus, lines cell division:
floor of the gingival sulcus, attached to the tooth (1) Cells do NOT mature UNLIKE other gingi-
surface in a cufflike manner by way of epithe- val tissues, but still migrate to surface and are
lial attachment (EA): lost in sulcus.
(1) Thin tissue (15 to 30 cells thick), wid- (2) Highest turnover time in entire oral cavity:
est at its junction with the SE; superficial 4 to 6 days.
Embryology and Histology 21

3. Clinical probing depth of gingival sulcus is mea- b. Clinical ramifications of enamel rod structure:
sured by calibrated periodontal probe; probe is gen- (1) Acid etch is briefly used to remove some
tly inserted, slides by SE, and is stopped by EA in a of organic portions of enamel crystals
healthy state, with NO bleeding on probing (BoP) and thus enable an enamel sealant or
noted; boundaries on probing gingival sulcus: tooth-colored restoration to flow into
(a) inner: tooth surface (enamel and/or cementum); newly created gaps; offers MORE sur-
(b) outer: SE; (c) base: JE lining sulcus, contacting face area for better adherence (mechani-
EA. cal bonding).
(2) Oxygen radicals from peroxide in the whit-
DENTAL TISSUES  ening (bleaching) agents contact stains in
Tooth consists of enamel, dentin, cementum, and pulpal interprismatic spaces within enamel layer;
tissues. Hard tissues (enamel, dentin, cementum, as well when this occurs, the stains are removed
as the surrounding bone) consist MAINLY of calcium hy- (bleached).
droxyapatite with the chemical formula Ca10(PO4)6(OH)2. 5. Hunter-Schreger bands: light or dark bands ori-
Soft tissues (pulp) are mesodermally derived connective ented perpendicular to DEJ, caused by curvature
tissues. See earlier discussion of the development of of rods.
tissues. 6. Lines of Retzius: dark incremental lines; in longi-
• See Chapters 6, General and Oral Pathology: dental tudinal sections appear as traverse rods; on trans-
pathology; 11, Clinical Treatment: pulpal evaluation, verse sections appear as concentric rings:
dentinal hypersensitivity, enamel sealants; 15, Dental a. Perikymata: raised imbrication lines (grooves)
Biomaterials: endodontic therapy, whitening proce- noted on nonmasticatory surfaces of some teeth
dure. in the oral cavity; wear away with attrition.
A. Enamel: ~96% inorganic crystal (hardest calcified/ b. Neonatal line: MOST accentuated incremental
mineralized tissue in the body): line; marks stress and/or trauma to ameloblasts
1. Covers dentin in crown; NOT found in the root; during birth.
SAME calcium hydroxyapatite makeup. 7. Enamel spindles: short dentinal tubules near the
2. UNLIKE dentin, cementum, and bone, enamel DEJ; produced by odontoblasts that cross the base-
does not contain collagen; instead, it is formed ment membrane before it is mineralized into DEJ.
from two proteins, amelogenins and enamelins. 8. Enamel tufts: small dark brushes with bases near
3. Amelogenesis: process of enamel matrix forma- DEJ.
tion during apposition stage: 9. Enamel lamellae: partially calcified vertical sheets
a. Ectodermal product from enamel organ, which of enamel matrix that extend from the DEJ near the
is derived from the ectoderm. cervix to the outer occlusal surface.
  b. First formed in incisal or occlusal portion of B. Dentin: ~70% inorganic crystal (LESS hard than
future crown, near forming dentinoenamel enamel; second hardest of dental tissues):
junction (DEJ); then moves cervically to ce- 1. Located in BOTH crown and root; covered by
mentoenamel junction (CEJ). enamel in crown and cementum in root; encloses
4. Enamel rod (prism): crystalline structural unit of innermost pulp tissue.
enamel. 2. Makes up the bulk of tooth and protects the pulp.
a. Tomes’ process: end portion of each ameloblast 3. Dentinogenesis: process of dentin matrix forma-
nearest the DEJ that dictates the specific shape tion during the apposition stage:
of each rod; takes four ameloblasts to produce a. Predentin: initial material laid down by odon-
one enamel rod. toblasts; mesenchymal product; mantle dentin
(1) Each rod is cylindrical in longitudinal sec- is first predentin.
tion; 4 micrometers (µm) in diameter. (1) Globular: areas where BOTH primary and
(2) Rods are stacked in rows; surrounding secondary mineralization has occurred
outer portion is interprismatic region (in- with complete crystalline fusion; appear as
terrod enamel); each rod is oriented perpen- lighter rounded areas.
dicular to DEJ and outer surface (at angles (2) Interglobular: areas where ONLY primary
from <90° to 60°). mineralization occurred and globules did
(3) MOST rods extend the width of the enamel NOT fuse completely; appear as darker
from DEJ to outer surface; thus each rod ­arclike areas.
varies in length because the width of b. Imbrication lines of von Ebner: incremental
enamel varies in different locations of lines (bands) that stain darkly, show daily pro-
crown area. duction of 4-micrometer (µm) increment.
22 Saunders Review of Dental Hygiene

c. Contour lines of Owen: adjoining parallel im- substance (calcium salts) similar to
brication lines; demonstrate disturbance in body peritubular.
metabolism that affects odontoblasts; neonatal (b) Noted with arrested caries; appears
line is MOST pronounced and occurs during brown, smooth, shiny.
trauma of birth. C. Cementum: ~ 65% inorganic (LESS hard than BOTH
4. Tomes’ granular layer: peripheral portion of den- enamel and dentin; MORE similar to bone):
tin beneath root’s cementum, adjacent to dentino- 1. Covers the entire root, overlying Tomes’ granular
cemental junction (DCJ). layer in dentin.
5. Dentinal tubules: long tubes in dentin that extend 2. Thickest at apex and in interradicular areas of mul-
from DEJ in crown area or DCJ in root area to tirooted teeth; thinnest at CEJ at cervix (neck).
outer wall of pulp: a. Can form throughout life of the tooth; forms ar-
a. Dentinal fluid: tissue fluid in tubule surround- rest lines (smooth lines formed by layered ap-
ing the cell membrane of the odontoblast, con- position).
tinuous from the cell body in the pulp; changes b. Repair of traumatic resorption involves apposi-
in flow are related to dental pain from the pulp tion of cementum by cementoblasts in adjacent
(may cause dental hypersensitivity; see Chapter PDL, resulting in reversal lines (scalloped lay-
11, Clinical Treatment). ers formed by absorption).
b. Odontoblastic process: long cellular extension 3. Part of periodontium that attaches the teeth to the
located within dentinal tubule, still attached to alveolar bone by anchoring PDL:
cell body of odontoblast within pulp. a. Sharpey’s fibers: collagen fibers from PDL.
(1) Avascular (NO blood supply); nutrition for b. “Brush ends” that are each partially inserted
odontoblasts comes from tissue fluid by into outer cementum at 90°.
way of tubule from blood vessels located in 4. Formed from dental sac (follicle), formed on root
adjacent pulp. after disintegration of HERS:
(2) Tubules become exposed as a result of car- a. Cementoblasts then disperse to cover the root
ies, cavity preparation, recession, or attri- dentin and undergo cementogenesis, laying
tion; open-ended dentinal tubules may be down cementoid; many become entrapped, be-
painful, causing dentinal hypersensitivity coming cementocytes.
(see earlier discussion). b. Each cementocyte lies in lacuna (plural,
6. Types of dentin: ­lacunae), similar to pattern in bone, with cana-
a. Peritubular: creates wall of tubule; MOST min- liculi.
eralized; with age, diameter of tubule becomes c. All the canals are oriented toward PDL and
narrower because of peritubular deposition. contain cementocytic processes.
b. Intertubular: between tubules; LESS mineral- 5. No innervation; avascular (NO blood supply),
ized than peritubular. receiving nutrition by way of own cells from
c. Circumpulpal: layer around outer pulpal wall. the surrounding PDL (reason for orientation of
d. Primary: formed in a tooth before completion of canals toward PDL).
apical foramen of the root, with regular pattern 6. CEJ exhibits varying patterns overall in each in-
of tubules. dividual oral cavity and even on one tooth: (a)
e. Secondary: formed after the completion of the cementum overlaps enamel (MOST common
apical foramen and continues to form: at 60%; makes it hard to discern from calcu-
(1) Formed MORE slowly than primary and lus); (b) cementum and enamel meet end to end;
LESS mineralized; fills in along the outer (c) gap between cementum and enamel, leaving
pulpal wall. exposed dentin with increased risk of dentinal
(2) Made by the odontoblastic layer that lines ­hypersensitivity.
the dentin-pulp interface, with a regular D. Pulp: soft tissue; innermost tissue of tooth:
pattern of tubules. 1. Formed from the central cells of the dental pa-
f. Tertiary (reparative, reactive): quickest type to pilla (same derivation as dentin); mesenchymal
form in localized regions in response to a local- product.
ized trauma to exposed dentin: a. Forms during odontogenesis, when dentin
(1) MORE irregular course than in secondary. forms around dental papilla.
(2) Sclerotic: type of tertiary associated with b. Innermost tissue is considered pulp.
chronic injury of caries and aging. 2. Anatomy:
(a) Has dead odontoblastic processes that a. Pulp chamber: mass of pulp is contained within
leave tubules vacant but that become tooth.
retrofilled, occluded by mineralized b. Coronal pulp: located in crown of tooth.
Embryology and Histology 23

c. Pulp horns: smaller extensions of coronal pulp bodies of the odontoblasts remains along outer
into the cusps of posteriors; MOST prominent pulpal wall:
under buccal cusp of premolar teeth and MB a. Fibroblasts: MOST common cells in the pulp;
cusp of molar teeth and MOST primary teeth odontoblasts are second MOST common.
(care taken during restoration). b. Fibers: mainly collagen fibers and some reticu-
d. Radicular (root) pulp (“root canal”) located in lar fibers; NO elastic fibers.
root: c. Pulp has extensive vascular supply and rudimentary
(1) Apical foramen: opening at the apex; lymphatics; ALL sensation is transmitted as pain.
­surrounded by cementum; allows arteries, d. Pulp stones (denticles) are sometimes present;
veins, lymphatics, nerves to enter and exit becomes MORE fibrotic with increased age and
from PDL. may make endodontic therapy MORE difficult.
(2) Accessory (lateral) canals: extra openings 4. With endodontic therapy, pulp is removed:
from pulp to PDL; NOT just laterally lo- a. Tooth is NO longer vital; tooth may darken, be-
cated. come brittle, and fracture during mastication.
3. Involved in support, maintenance, and continued b. Full restorative coverage is recommended to
formation of dentin because inner layer of the cell protect tooth from fracture.

CLINICAL STUDY 

Age 60 YRS SCENARIO

Sex ☐  Male   ☒  Female A patient reports for new patient examination.


­Radiographs reveal endodontic therapy of tooth
Height 5’4” #30. The intraoral examination reveals that the
­distobuccal cusp of tooth #30 has fractured off.
Weight 185 LBS

BP 119/78

Chief Complaint “My tooth broke off, but there is no


pain in any of my teeth.”

Medical History Carpal tunnel surgery 5 years ago

Current Medications Scarlet fever and tonsillectomy as a child


None except OTC vitamin E qd

Social History Grandmother and church organist

1. Why is tooth #30 not painful? PERIODONTIUM 


2. Which cells are no longer active in tooth #30? Periodontium includes alveolar bone and PDL, as well
3. Discuss possible causes of the distobuccal cusp frac- as gingival tissues and cementum (Figure 2-7). See also
ture on the tooth. earlier discussion about development of the jaws and dis-
cussion of gingival tissues and cementum.
1. Although tooth exhibits a fractured cusp, patient feels • See Chapters 2, Embryology and Histology: bone tis-
no pain because nerve tissues within pulp were re- sue; 11, Clinical Treatment: periodontal evaluation and
moved during endodontic therapy. charting.
2. Cells associated with pulp tissue (odontoblasts, fi- A. Alveolar bone: ~60% inorganic (MORE similar to
broblasts, histiocytes, undifferentiated mesenchymal cementum than other dental tissues):
cells) are not active, since they are no longer present 1. Part of the jaws that supports and protects teeth,
after endodontic therapy; removed during therapy to attached to cementum of the tooth by way of
control pulpal infection. PDL.
3. Endodontic therapy of tooth #30 has increased the 2. Formed from dental sac (similar to PDL); mesen-
likelihood that tooth will fracture in the future. Pos- chymal product.
teriors are often restored with full coverage such as a 3. Anatomy:
crown to reduce the chance of a tooth’s fracturing af- a. Basal bone is apical to roots, forms body of jaws.
ter endodontic therapy. Age-related changes may also b. Alveolar bone is the portion that contains the
cause a tooth to become more brittle. roots of the teeth.
24 Saunders Review of Dental Hygiene

5
6

2 9

3 10

Mantle dentin 5
10 Cementocytes in cementum
Enamel 4
Cementoblasts in pulp 9
Periodontal ligament 3
Tomes’ granular layer in dentin 8
bone proper
Dentinocemental junction 7 Alveolar crest of alveolar 2

Cementoenamel junction 6 1 Sharpey’s fibers

Figure 2-7  Periodontium. (From Fehrenbach MJ, ed: Dental anatomy coloring book, St. Louis, 2008,
­Saunders/Elsevier.)

(1) Alveolar bone proper (ABP) (cribriform (2) Trabecular bone: located between ABP
plate, tooth socket): lining of alveolus (plu- and cortical plates; consists of cancellous
ral, alveoli). (spongy) bone.
(a) Has Volkmann’s canals, passing from d. Interdental septum: alveolar bone between
alveolar bone into PDL, and Sharpey’s two neighboring teeth; consists of BOTH
fibers, portion of the fibers of PDL in- compact bone of ABP and cancellous bone of
serted here at 90°. ­trabecular bone.
(b) Consists of compact bone, noted on ra- e. Interradicular septum: found ONLY on mul-
diographs as lamina dura, a solid ra- tirooted teeth; alveolar bone between the roots
diopacity surrounding radiolucency of of the same tooth; consists of BOTH ABP and
PDL space. trabecular bone.
(c) Lamina propria of the attached gingiva 4. Ramifications of alveolar bone:
serves as its mucoperiosteum. a. Easily remodeled in comparison with cemen-
(d) Alveolar crest: MOST cervical rim tum, thus allowing orthodontic tooth move-
of the ABP, apical to CEJ by approxi- ment; shows arrest lines (smooth) and reversal
mately 1 to 2 mm. (scalloped) lines:
c. Supporting alveolar bone: cortical plates and (1) On one side of tooth, compression zone in
trabecular bone: PDL leads to bone resorption.
(1) Cortical plates (bone): plates of compact (2) At the same time, on opposite side of tooth
bone on facial and lingual surfaces of al- and bone, tension zone develops in PDL
veolar bone. and causes deposition of new bone.
Embryology and Histology 25

b. Density of alveolar bone in an area also deter- 7. Has blood vessels that provide nutrition for its cells
mines route that dental infection takes with ab- and surrounding cells of cementum and ABP.
scess formation. 8. Components:
c. With the loss of teeth, patient becomes edentu- a. Fibroblast is MOST common cell, with ce-
lous, either partially or completely, and alveolar mentoblasts along cemental surface; osteo-
bone undergoes resorption (loss); underlying blasts are also present at the periphery of the
basal bone and both the body of maxilla and ABP.
mandible remain less affected. b. Osteoclasts, odontoclasts; also undifferentiated
B. Periodontal ligament (PDL): soft tissue that pro- mesenchymal cells, which can differentiate into
vides attachment of teeth to surrounding ABP by any of these cells if their cell populations are
way of cementum; serves periosteum for these tis- injured or lost.
sues: c. Epithelial rests of Malassez become loc­ated in
1. Derived from the dental sac (similar to alveolar mature PDL after disintegration of HERS dur-
bone and cementum); mesenchymal product. ing formation of root; may become cystic.
2. Organized fibrous connective tissue that maintains d. Principal fiber groups of PDL (see Table 2-3
gingiva in proper relationship to the teeth. and Figure 2-8):
3. Appears on radiographs as radiolucent periodon- (1) Alveolodental group: largest of the five fiber
tal ligament space, bordered by radiopaque lam- groups of PDL; oblique fibers are MOST
ina dura of ABP and then tooth root. numerous.
4. Transmits occlusal forces from the teeth to bone, (2) Approximately 5 mm in diameter; resist the
allowing small amount of movement; nerve supply forces exerted on tooth such as rotational,
provides MOST efficient proprioceptive mecha- tilting, extrusive, or intrusive; some fibers
nism (as evidenced when biting on metal foil in resist a combination of forces.
candy wrappers—ewww!). (a) Sharpey’s fibers: “brush ends” of the
5. UNLIKE the pulp, transmits touch, pressure, and principal fibers are within ABP and
temperature sensations as well as pain. ­cementum.

6. Cells in tissue participate in formation and resorp- (b) Partially inserted into the hard tissues of
tion of hard tissues of periodontium. periodontium (bone or cementum) at 90°.

Table 2-3  Fiber groups of the alveolodental ligament

Fiber group Location Function

Alveolar crest group Originates in alveolar crest of ABP and fans To resist tilting, intrusive, extrusive,
out to insert into cervical cementum at and rotational forces
various angles
Horizontal group Originates in ABP apical to its alveolar crest To resist tilting forces and rotational
and inserts into cementum in horizontal forces
manner
Oblique group Originates in ABP and extends apically to To resist intrusive forces and
insert more apically into cementum in ­rotational forces
oblique manner
Apical group Radiates from apical region of cementum to To resist extrusive forces and
insert into surrounding ABP ­rotational forces.
Interradicular group (only on Inserted on the cementum of one root to the To resist intrusive, extrusive, tilting,
­multirooted teeth) cementum of the other root(s) ­superficial and rotational forces
to the interradicular septum

ABP, Alveolar bone proper.


26 Saunders Review of Dental Hygiene

3 9

4
1

10
2

11

7 12

12 Interradicular group Interdental bone 6

11 Apical group Interradicular septum 5


Oblique group 10 Alveolar bone 4

Horizontal group 9 Alveolar crest 3

Alveolar crest group 8 Sharpey's fibers within cementum 2

Cementum 7 Sharpey's fibers within alveolar bone 1

Figure 2-8  Fiber groups of the periodontal ligament. (From Fehrenbach MJ, ed: Dental anatomy
coloring book, St. Louis, 2008, Saunders/Elsevier.)


Review Questions
3 Which structure becomes the dentinoenamel junction of the
 ������
1 The midface and all structures of the oral cavity EXCEPT
 ���������� fully formed tooth?
the posterior part of the tongue develop from the A. Outer enamel epithelium
A. frontal process and first branchial arch. B. Stellate reticulum
B. frontal process and second branchial arch. C. Basement membrane
C. stomodeum and buccopharyngeal membrane. D. Dental papilla
D. forebrain and tuberculum impar. 4 Tooth development begins when a localized growth of cells
 ������
2 The formation of an ameloblast from cells of the inner
 ������ on the jaw ridge produces a strand of epithelium. What is
enamel epithelium is an example of this strand called?
A. histodifferentiation. A. Tooth germ
B. morphodifferentiation. B. Tooth bud
C. odontogenesis. C. Dental lamina
D. dentinogenesis. D. Enamel organ
Embryology and Histology 27

  5 Which of the following develops into the posterior one third 15 One of the following is NOT true as the pulp ages. Which
of the tongue? one is the EXCEPTION?
A. Meckel’s cartilage A. Vascularity decreases.
B. Copula B. Pulp chamber size decreases.
C. Tuberculum impar C. Mineralization decreases.
D. Lingual swellings D. Sensitivity increases.
  6 Which of the following determines the shape of the tooth 16 At which location does cementum have the greatest
root? ­thickness?
A. Rests of Malassez A. At the apex or furcation areas
B. Stellate reticulum B. Closest to the cementoenamel junction
C. Hertwig’s sheath C. At Sharpey’s attachment sites
D. Dental lamina D. At mesial root surfaces
 7 Enamel contains which percentage of inorganic substance? 17 Which cells are MOST active on the mesial side of a tooth
A. 4% being translated mesially during orthodontic treatment?
B. 38% A. Fibroblasts
C. 74% B. Osteoblasts
D. 96% C. Cementoblasts
 8 Which of the following is created with the changes in direc- D. Osteoclasts
tion of developing enamel rods? 18 Which is the MOST common type of epithelium found in
A. Lines of Retzius the oral cavity?
B. Hunter-Schreger bands A. Stratified squamous
C. Perikymata lines B. Cuboidal
D. Neonatal lines C. Transitional
 9 Of what are perikymata an external manifestation? D. Simple squamous
A. Lines of Retzius 19 One of the following lingual papillae is NOT conside­red a pri-
B. Hunter-Schreger bands mary location for taste buds. Which one is the ­EXCEPTION?
C. Lines of Ebner and Owen A. Circumvallate
D. Enamel spindles B. Fungiform
10 An enamel pearl is C. Filiform
A. located on the occlusal surface, causing discomfort dur- D. Foliate
ing chewing. 20 Epithelial rests of Malassez are associated with which of the
B. confused with calculus because of its location. following?
C. present as an opacity ring on the facial surface of ante- A. Dental sac
rior teeth. B. Dental papilla
D. located within the pulp chamber, obliterating the blood C. Enamel organ
supply. D. Lamina dura
11 Where are odontoblast nuclei located in a tooth? 21 Which of the following lingual papillae is MOST keratin-
A. Predentin ized?
B. Dentinal tubules A. Circumvallate
C. Pulp tissue B. Filiform
D. Periodontal ligament C. Foliate
12 Where is cellular cementum MOST likely to be found? D. Fungiform
A. At the apical portion of the tooth 22 Which of the following types of bone tissue increases in
B. At the cementoenamel junction amount because of an increase in occlusal activity?
C. Equally throughout the root surface A. Cortical
D. Closest to the dentin B. Trabecular
13 A chronic inflammation of a tooth that causes a localized C. Circumferential
thickening of cementum is referred to as D. Lamellar
A. a cementicle. 23 The histological pocket depth is
B. hypercementosis. A. less than the clinical probing depth.
C. cementogenesis. B. equal to the clinical probing depth.
D. a cementoid. C. greater than the clinical probing depth.
14 Rank the following tissues from MOST to LEAST resistant D. lessened as periodontal disease increases.
to abrasion.
A. Dentin, enamel, cementum
B. Enamel, cementum, dentin
C. Enamel, dentin, cementum
D. Cementum, enamel, dentin
28 Saunders Review of Dental Hygiene

24 One of the following is NOT true of sclerotic dentin. Which 33 The bell stage occurs between the eleventh and twelfth
one is the EXCEPTION? weeks of prenatal development. There is differentiation of
A. More rapid progression of caries than in normal dentin the enamel organ into four layers during the bell stage.
B. Located in MOST older teeth A. Both statements are true.
C. Located where odontoblasts have degenerated B. Both statements are false.
D. Formed from calcium salts that have filled dentinal tu- C. The first statement is true, the second is false.
bules D. The first statement is false, the second is true.
25 Corkscrew-shaped structures that lie among dentin are 34 Dentin is a crystalline material that is less hard than enamel
called because it is 70% inorganic.
A. Tomes’ fibers. A. Both the statement and reason are correct and related.
B. Korff’s fibers. B. Both the statement and reason are correct but NOT
C. enamel spindles. ­related.
D. dentinal tubules. C. The statement is NOT correct, but the reason is correct.
26 What is a commissural lip pit? D. NEITHER the statement NOR the reason is correct.
A. Cancerous lesion on the lips 35 The lines of Retzius appear as incremental lines in dentin
B. Sunburned lips that are chapped that stain brown. In transverse sections the lines of Retzius
C. Dimplelike invaginations of the corner of the lips appear as concentric rings.
D. Disturbance of lip development in utero A. Both statements are true.
27 What is a bifid uvula? B. Both statements are false.
A. Disruption in palatal fusion C. The first statement is true, the second is false.
B. Minor cleft of the posterior soft palate D. The first statement is false, the second is true.
C. Disturbance of lip development in utero 36 The hard tissues of the tooth consist mainly of cal-
D. Dimplelike invaginations of the corner of the lips cium carbonate because it has the chemical formula
28 How often does cleft lip occur? Ca10(PO4)6(OH)2.
A. 1 in 2500 A. Both the statement and reason are correct and ­related.
B. 1 in 1000 B. Both the statement and reason are correct but NOT
C. 1 in 100 ­related.
D. 1 in 50 C. The statement is NOT correct, but the reason is correct.
29 Ankyloglossia is a short lingual frenum that extends to the D. NEITHER the statement NOR the reason is correct.
tongue apex. Ankyloglossia restricts the movement of the 37 With endodontic therapy the tooth may lighten because the
tongue to varying degrees. tooth is no longer in occlusion.
A. Both statements are true. A. Both the statement and reason are correct and related.
B. Both statements are false. B. Both the statement and reason are correct but NOT
C. The first statement is true, the second is false. ­related.
D. The first statement is false, the second is true. C. The statement is NOT correct, but the reason is correct.
30 The secondary palate will form the posterior two thirds of D. NEITHER the statement NOR the reason is correct.
the hard palate. The secondary palate contains the maxillary 38 Enamel sealants must be placed in any of the pronounced
central and lateral incisors. occlusal fissures because the pulp horns are especially
A. Both statements are true. prominent under the mesiobuccal cusp of molar teeth.
B. Both statements are false. A. Both the statement and reason are correct and related.
C. The first statement is true, the second is false. B. Both the statement and reason are correct but NOT
D. The first statement is false, the second is true. ­related.
31 The branchial arches are six pairs of stacked bilateral swell- C. The statement is NOT correct, but the reason is correct.
ings of tissue that appear inferior to the stomodeum. The D. NEITHER the statement NOR the reason is correct.
branchial arches include the maxillary arch. 39 The alveolar crest is the MOST apical rim of the alveolar
A. Both statements are true. bone proper. The alveolar crest is apical to the cemento­
B. Both statements are false. enamel junction by approximately 1 to 2 mm.
C. The first statement is true, the second is false. A. Both statements are true.
D. The first statement is false, the second is true. B. Both statements are false.
32 The tongue develops during the second to third weeks of C. The first statement is true, the second is false.
prenatal development. The tongue develops from indepen- D. The first statement is false, the second is true.
dent swellings located internally on the floor of the primi- 40 The periodontal ligament provides a MOST efficient pro-
tive pharynx. prioceptive mechanism because the fibroblast is the MOST
A. Both statements are true. common cell in the ligament.
B. Both statements are false. A. Both the statement and reason are correct and related.
C. The first statement is true, the second is false. B. Both the statement and reason are correct but NOT
D. The first statement is false, the second is true. ­related.
C. The statement is NOT correct, but the reason is correct.
D. NEITHER the statement NOR the reason is correct.
Embryology and Histology 29

41 The functions of the periodontium include all of the follow-


ing, EXCEPT one. Which one is the EXCEPTION? Answer Key and Rationales
A. Attaching the tooth to the bony socket
B. Resisting forces generated by mastication and speech
C. Defending against external noxious stimuli
1 (A) ������������������������������������������������
Frontal process and first branchial arch are re-
D. Protecting the tooth against cariogenic bacteria
sponsible for formation of the face and oral cavity,
42 Oral epithelium is classified as
A. simple squamous epithelium. NOT including base of the tongue. Branchial arches
B. stratified squamous epithelium. II, III, and IV merge to form base of tongue. Stomo-
C. pseudostratified columnar epithelium. deum and buccopharyngeal (oropharyngeal) mem-
D. simple cuboidal epithelium. branes are IMPORTANT in formation of primitive
43 Sulcular or crevicular epithelium is mouth. Tuberculum impar is IMPORTANT in forma-
A. keratinized. tion of anterior portion of tongue.
B. parakeratinized. 2 (A) ���������������������������������������������
Formation of an ameloblast from cells of the
C. pseudokeratinized. inner enamel epithelium is an example of histodif-
D. nonkeratinized. ferentiation, the formation of a MORE specialized
44 Attached gingiva is found between the
cell from a primitive cell. Primitive cells are the cells
A. mucogingival junction and the free gingival groove.
of the inner enamel epithelium, and specialized cells
B. alveolar mucosa and the marginal gingiva.
C. marginal gingiva and the free gingival groove. are the ameloblasts. Morphodifferentiation is the de-
D. teeth of the dentition. velopment of a different form, such as occurs when
45 The connective tissue of the gingival tissues is called the cells of Hertwig’s epithelial root sheath (HERS) align
A. lamina dura. themselves to dictate the shape of the root. Odonto-
B. lamina propria. genesis refers to formation of teeth; dentinogenesis
C. lamina lucida. refers to formation of dentin.
D. lamina densa. 3 (C) �������������������������������������������
Basement membrane marks the junction where
46 One of the following descriptors does NOT pertain to the dentin and enamel will meet. Outer enamel epithe-
alveolar mucosa. Which is the EXCEPTION? lium and stellate reticulum are cell layers of the
A. Nonkeratinized
enamel organ. Dental papilla forms the pulp.
B. Found beyond the mucogingival junction
4 (C) ���������������������������������������������������
Dental lamina is a strand of epithelial cells that
C. Flexible
D. Component of gingiva marks the beginning of tooth formation and develops
47 Brush ends of the fibers of the PDL that are embedded in along jaw ridge. Primary dental lamina forms pri-
both the cementum and alveolar bone are referred to as mary teeth, and secondary dental lamina is responsi-
A. oblique fibers. ble for formation of permanent teeth. Tooth germ and
B. Sharpey’s fibers. tooth bud refer to later stages of tooth development.
C. principal fibers. Enamel organ and dental papilla refer to tissues asso-
D. transseptal fibers. ciated with tooth development after initial localized
48 All of the following are terms given to the compact bone growth of cells.
that surrounds the roots of the tooth and forms the lining 5 (B) �������������������������������������������������
Copula is posterior swelling that forms the base
of the tooth socket, EXCEPT one. Which is the EXCEP-
of the tongue. Lingual swellings and tuberculum im-
TION?
par form the anterior two thirds of the tongue or body.
A. Alveolar bone proper
B. Cribriform plate First branchial arch is responsible for formation of
C. Lamina dura the maxilla, mandible, and middle portion of the face.
D. Lamina propria Meckel’s cartilage is ­ IMPORTANT in formation of
49 The supporting alveolar bone is made of alveolar bone.
A. compact and cancellous bone. 6 (C) ������������������������������������������������
Shape of developing root is determined by HERS,
B. cortical and spongy bone. which forms when outer and inner epithelial tissues
C. cancellous and spongy bone. join. Rests of Malassez are epithelial remnants of the
D. cortical, cancellous, and spongy bone. sheath and are trapped in tissues of the periodontal
50 Which of the following tissues lines the oral cavity? ligament (PDL). Dental lamina and stellate reticulum
A. Simple squamous epithelium for diffusion of fluids
are structures associated with the developing tooth
B. Simple columnar epithelium for absorption
and are NOT responsible for the shape of the root.
C. Pseudostratified columnar epithelium with goblet cells
for production of mucus 7 (D) �������������������������������������������������
Enamel is the hardest tissue in the body and con-
D. Stratified squamous epithelium for protection from tains about 96% inorganic (mineralized) material.
abrasion Dentin contains about 70% inorganic material, and
cementum, LIKE bone, contains about 65% inorganic
material.When considering the inorganic percentage
30 Saunders Review of Dental Hygiene

with each portion of the tooth (or alveolar bone), 17 (D) �����������������������������������������


Cells would be MOST active on the mesial
choose the closest one listed because the percentage side of a tooth being moved mesially and would be
can vary according to source. cells responsible for bone removal and osteoclasts.
8 (B) ��������������������������������������������
Light and dark bands of Hunter-Schreger are ­Osteoblasts would become active on the distal side of
oriented perpendicular to the dentinoenamel junction mesially moved tooth. Fibroblasts and cementoblasts
(DEJ) and are caused by curvature of the develop- would NOT have a great change in activity.
ing enamel rods. Neonatal lines, lines of Retzius, and 18 (A) �������������������������������������������
Stratified squamous epithelium is the MOST
perikymata are also lines found in enamel. common type of soft tissue of the oral cavity. Cuboi-
9 (A) �������������������������������������������������
Perikymata are fine horizontal lines that may be dal and simple squamous epithelial and transitional
visible on the enamel surface as a manifestation (or cells are NOT considered primary cell layers of oral
sign) of the lines of Retzius, seen microscopically as cavity.
narrow lines that extend from dentinoenamel junc- 19 (C) ������������������������������������������������
Taste buds are NOT associated with filiform lin-
tion (DEJ) to cusp tips. Hunter-Schreger bands and gual papillae; they are associated with fungiform, fo-
enamel spindles are features of enamel that would liate, and circumvallate lingual papillae.
NOT be visible on external enamel surface. 20 (C) �������������������������������������������������
Epithelial rests are trapped epithelial cells as-
10 (B) ������������������������������������������������
Enamel pearl is often confused with calculus be- sociated with the inner and outer enamel epithelium
cause of its location and shape. Enamel pearls often that is associated with enamel organ. Dental sac and
form near cementoenamel junction (CEJ) as rounded dental papilla are mesenchymal components of the
structures that protrude from the tooth surface. They tooth bud (germ) and would NOT be associated with
are never found on the occlusal surface of teeth and trapped epithelial cells.
are never seen as opacity rings on facial surfaces 21 (B) �����������������������������������������������
Filiform lingual papillae are MORE keratinized
of anteriors. Denticles (pulp stones) are commonly than fungiform, foliate, and circumvallate lingual
found within pulp chamber, UNLIKE enamel pearls. ­papillae.
11 (C) ����������������������������������������������
Nuclei of odontoblasts, cells responsible for 22 (B) ���������������������������������������������
When tooth function increases, the amount of
dentin formation, are found in pulp. Dentinal tubules trabeculae also increases. Cortical, lamellar, and cir-
may contain cytoplasmic extensions of odontoblast. cumferential bone do NOT show significant changes
Newly formed dentin (predentin) would NOT contain as the result of increased occlusal forces.
odontoblast nuclei. 23 (A) ����������������������
Histological depth is less than clinical prob-
12 (A) ������������������������������������������������
Apical and furcation areas are sites associated ing depth. The probe will slightly penetrate junctional
with cellular cementum. Cementocytes are NOT lo- epithelium (JE), leading to greater clinical probing
cated equally throughout the root surface. Thin cer- depth. As tissue health decreases as with periodontal
vical cementum has few or no cementocytes. The disease, the probe penetrates MORE easily because
region closest to dentin contains few cementocytes. of ulceration of the JE, so there is even greater clini-
13 (B) ���������������������������������������������
Thickening of apical cementum is referred to cal probing depth.
as hypercementosis; it may be a result of persistent 24 (A)  Presence of sclerotic dentin may decrease the
inflammation. Cementicle is a calcification found in rate at which caries spreads. Sclerotic dentin is a site
PDL. Cementogenesis refers to normal formation of where odontoblasts have degenerated and may be
cementum. Cementoid is cementum MOST recently noted in older teeth with arrested dentin. Dentinal tu-
formed by cementoblasts. bules contain calcium salts.
14 (C) ����������������������������������������������
Enamel is the hardest and therefore MOST abra- 25 (B) ������������������������������������������������
Korff’s fibers are corkscrewlike collagen struc-
sion-resistant tissue, followed by dentin and cemen- tures that lie among cytoplastic extensions of odon-
tum. Strength of enamel as the covering of the tooth toblasts. Enamel spindles are extensions of the
is IMPORTANT in longevity of tooth structure. odontoblastic process into enamel. Dentinal tubules
15 (C) ����������������������������������������������
Mineralization of pulp increases as the tooth contain the odontoblastic process. Tomes’ granu-
ages and has clinical significance when endodontic lar layer is an area of unmineralized regions in the
therapy is indicated. Both vascularity and pulp cham- peripheral portion of dentin beneath the root’s ce-
ber size decrease as the result of aging. Increased oc- mentum, adjacent to the dentinocemental junction
clusal function may cause pulp to fill completely with (DCJ).
reparative dentin. As pulp shrinks, patients may note 26 (C) ���������������������������������������������
Commissural lip pits are dimplelike invagina-
a decrease in tooth sensitivity. tions of the corner (commissure) of lips. They may be
16 (A) �����������������������������������������������
Apex and furcation areas are sites where cemen- unilateral or bilateral but generally occur on vermil-
tum is thicker (>0.05 mm), while the cervical third ion portion. Cleft lip is a disturbance of lip develop-
is thinner (0.02 to 0.05 mm). Cementum is approxi- ment in utero. Basal cell or squamous cell carcinoma
mately 0.05 mm thick on remaining portions of the is usually a cancerous lesion on the lips that presents
root surface. as an ulcer.
Embryology and Histology 31

27 (B) ��������������������������������������������������
Bifid uvula (cleft uvula) is a minor cleft of the whether vital or not. Oxygen radicals from peroxide
posterior soft palate. Cleft palate is a disruption in in the whitening agents contact stains in interpris-
palatal fusion. Cleft lip is a disturbance of lip devel- matic spaces within the enamel layer; when this oc-
opment in utero. Commissural lip pits are dimplelike curs, stains will be bleached (whitened).
invaginations of the corner of lips. 38 (B) ��������������������������������������������������
Both the statement and reason are correct but not
28 (B) ���������������������������������������������������
Cleft lip, with or without cleft palate, occurs in related. Pulp horns are MOST prominent under the
about 1 in 1000 live births. Cleft palate, with or without MB cusp of molar teeth (and MOST primary teeth).
cleft lip, occurs 1 in 2500 live births. Cleft palate is a Thus extra care MUST be taken when restorative
disruption in palatal fusion. Cleft lip is a disturbance of treatment is performed. Enamel (pit and fissure) seal-
lip development in utero. ants are not invasive; however, they MUST be placed
29 (A) ���������������������������������������������
Both statements are true. Ankyloglossia is a in any of pronounced occlusal fissures on molar teeth,
short lingual frenum that extends to the tongue apex, even in adults, to prevent occlusal ­caries.
restricting the movement of the tongue to varying 39 (D) ����������������������������������������������
First statement is false, the second is true.
degrees. Surgery may or may not be needed; if the ­Alveolar crest is the MOST cervical, not apical, rim
condition is severe, therapy may help with speech of alveolar bone proper, apical to the CEJ by approxi-
problems. mately 1 to 2 mm.
30 (C) ��������������������������������������������������
First statement is true, the second is false. The 40 (B) ����������������������������������������������
Both the statement and reason are correct but
secondary palate will form posterior two thirds of the not related. The PDL does provides MOST efficient
hard palate. However, the secondary palate contains proprioceptive mechanism because of its innervation.
maxillary canines and posteriors and NOT central However, fibroblast is MOST common cell in PDL
and lateral incisors, which would be contained in and produces fibers for ligament.
the primary palate, the anterior one third of the hard 41 (D) �����������������������������������������������
Protection against cariogenic bacteria has NOT
palate. been shown to be a function of periodontium. Func-
31 (C) ���������������������������������������������������
First statement is true, the second is false. Bran- tions of the periodontium include attaching tooth to
chial arches are six pairs of stacked bilateral swell- its bony housing; providing resistance to forces of
ings of tissue that appear inferior to the stomodeum. mastication, speech, and deglutition; maintaining
However, branchial arches include the mandibular body surface integrity by separating external and in-
arch and NOT the maxillary arch. ternal environments; defending against external nox-
32 (D) ��������������������������������������������������
First statement is false, the second is true. The ious stimuli.
tongue develops from independent swellings located 42 (B) �����������������������������������������������
Oral epithelium consists of keratinized (ortho­
internally on floor of the primitive pharynx. However, keratinized or parakeratinized) stratified squamous
the tongue develops during fourth to eighth weeks of epithelium. Simple epithelia, pseudostratified colum-
prenatal development. nar epithelia, and simple cuboidal epithelia are found
33 (A) ������������������������������������������������
Both statements are true. The bell stage occurs in other areas of the body but NOT in the oral mucosa
between the eleventh and twelfth weeks of prenatal lining the oral cavity.
development; there is differentiation of the enamel 43 (D) ��������������������������������������������
Sulcular and crevicular epithelium is nonke-
organ into four layers. ratinized. ONLY masticatory mucosa, such as the
34 (A) ��������������������������������������������������
Both the statement and the reason are correct and attached gingiva and hard palate, is keratinized (or-
related. Dentin is a crystalline material that is LESS thokeratinized or parakeratinized). No areas of the
hard than enamel because it is 70% inorganic. mouth are considered pseudokeratinized.
35 (D) ����������������������������������������������������
First statement is false, the second is true. Lines 44 (A) �������������������������������������������������
Attached gingiva is portion that is firmly bound
of Retzius appear as incremental lines in enamel that to underlying bone and is located between free gingi-
stain brown and in transverse sections appear as con- val groove and mucogingival junction. Free gingival
centric rings. Imbrication lines of von Ebner are in- groove demarcates part of the tissues that are mar-
cremental lines or bands that stain darkly in dentin. ginal (free) and NOT attached to the tooth. Alveolar
36 (C) ������������������������������������������������
Statement is not correct, but the reason is cor- mucosa is loose and movable. Connective tissue of
rect. Hard tissues of the tooth consist mainly of PDL is ONLY tissue that attaches to tooth.
calcium hydroxyapatite with the chemical formula 45 (B) �����������������������������������������������
Lamina propria is the connective tissue of the
Ca10(PO4)6(OH)2; they include enamel, dentin, and gingival tissues that contains collagen fibers, fibro-
cementum. blasts, undifferentiated cells, macrophages. Lamina
37 (D)  Neither the statement nor reason is correct. With dura is the radiographic term for alveolar bone proper
endodontic therapy, tooth may darken because it is (ABP), which surrounds tooth root. Lamina lucida
no longer vital. That is because the pulp that provided and lamina densa are BOTH components of base-
nutrients for the dentin has been removed. Teeth ment membrane between lamina propria and the at-
will lighten with the whitening (bleaching) process, tached gingiva.
32 Saunders Review of Dental Hygiene

46 (D) ������������������������������������������������
Alveolar mucosa is nonkeratinized, flexible, lo- 49 (C) �������������������������������������������
Supporting alveolar bone contains BOTH com-
cated beyond mucogingival junction. However, it is a pact and cancellous bone. Facial and lingual cortical
separate part of oral mucosa and is NOT classified as plates are compact, and spongy bone between these
gingiva. plates is cancellous.
47 (B) �����������������������������������������������
Brush ends of principal fibers of the PDL that 50 (C) ���������������������������������������������
Diffusion and absorption do NOT occur across
are embedded in both cementum and alveolar bone tissue that lines the oral cavity. Mucus in oral cav-
are referred to as Sharpey’s fibers. Transseptal and ity is a component of the saliva produced by salivary
the rest of the principal fibers span across entire PDL glands. Lining of oral cavity is subject to abrasion
space; do NOT refer to just embedded portions of the by food during mastication; therefore the lining con-
fibers. sists of many layers of cells, primarily stratified squa-
48 (C) ����������������������������������������������
Lamina propria is the oral connective tissue. mous epithelium, to provide protection from abrasive
Alveolar bone proper (ABP) is compact bone that ­action.
forms housing of the root and tooth socket. Referred
to histologically as cribriform plate, anatomically as
bundle bone, and radiographically as lamina dura.
CHAPTER 3

Anatomy, Biochemistry, and Physiology

GENERAL ANATOMY    5. Stratum basale: innermost layer and closest to


Anatomy is the study of structure; physiology is the blood supply; active mitotic layer; contains mela­
study of function; gross anatomy is the study of the nocytes (produce dark pigment, melanin) ���� and
body’s structure that is visible with the naked eye; and immune cells (Langerhans’ cells)������������������
; tissue color de-
microanatomy (histology) examines the cellular compo- pends on melanocytic activity, amount of carotene
sition of tissues. Structures and associated functions are (yellow pigment), dermal blood supply.
organized in interacting hierarchy from simple to com- B. Basement membrane: where epidermis and dermis
plex (i.e., chemicals, cells, tissues, organs, body systems, interface; derives components from both layers.
total organism). C. Dermis (stratum corium): superior papillary layer of
Organs of the body include integumentary, skeletal, fibrous connective tissue; contains vessels, nerves,
muscular, nervous (including special sense organs), en- embedded accessory structures; lower reticular layer
docrine, cardiovascular, lymphatic, respiratory, digestive, is meshwork of collagen and elastic fibers.
urinary, and reproductive. Knowing these systems can D. Hypodermis (subcutaneous tissue): deep to dermis
help you to understand a medical term. A term related to and serves to anchor skin to underlying organs; com-
each of these systems might be augmented with roots, pre- posed of loose connective tissue and adipose tissues;
fixes, and suffixes (see CD-ROM for commonly encoun- acts as cushion and insulates body from heat and
tered ones), filling out the other two thirds of the term. If cold.
you know those, plus the name of the body system, you E. Accessory structures (e.g., glands, hair, nails): derived
are well on your way to understanding another third or by mitosis in stratum basale of epidermis:
more of a term. In addition, anatomical nomenclature 1. Sweat glands: merocrine (MORE numerous,
includes directional terms that describe relative position widely distributed) and apocrine (larger, LESS nu-
of body parts in anatomical position (see CD-ROM). merous); sebaceous glands are oil glands associ-
• See CD-ROM for Chapter Terms and WebLinks. ated with hair follicles.
• See Chapter 2, Embryology and Histology: cell structure. 2. Hair: divided into visible shaft and root, embedded
in skin and surrounded by a follicle; central core:
Integumentary System medulla, surrounded by cortex, cuticle, arrector
I�������������������
ntegumentary system consists of skin (cutaneous mem- pili muscles (contract in response to cold or fear to
brane), including glands, hair, and nails (adnexa). Skin make hair stand on end).
has outermost epidermis and innermost dermis anchored 3. Nails: thin plates of hardened stratum corneum;
to underlying tissues by hypodermis or subcutaneous each has free edge, nail body, root, eponychium,
­tissue; it is the largest external organ. nail bed, nail matrix, lunula.
A. Epidermis: stratified squamous epithelium, five lay-
ers in thicker areas or four layers in thinner areas Skeletal System
(Figure 3-1): Skeletal system (206 bones) includes axial skeleton (80)��,�
1. Stratum corneum: outermost layer of dead cells, forming axis of the body, and appendicular skeleton (126),
continually sloughed off, replaced by cells from consisting of appendages and attachments. System func-
deeper layers; contains keratin. tions to provide support, protect soft body parts, produce
2. Stratum lucidum: thin, clear layer just beneath movement with muscles, store minerals (calcium), form
stratum corneum; not in thinner areas. blood cells.
3. Stratum granulosum: appears granular because • See Chapter 4, Head, Neck, and Dental Anatomy: skull,
organelles are shrinking, with intracellular keratin hyoid bone.
(keratin granules) being deposited. A. Osseous tissue: osteons (Haversian systems) (Figure
4. Stratum spinosum: located next to stratum basale; 3-2):
undergoes limited mitosis and adds strength to the 1. Compact bone: MORE dense; osteons are packed
skin owing to intracellular bridging. closely together.

33
34 Saunders Review of Dental Hygiene

5 6 b. Facial: 14 bones (2 maxillae, 2 zygomatic,


1
2 nasal, 2 inferior nasal conchae, 2 lacrimal,
2 palatine, 1 mandible, 1 vomer).
c. Auditory ossicles (malleus, incus, stapes in
each ear): 6 bones.
2. Hyoid bone: midline body and two pairs of pro-
jections, greater and lesser cornu:
2 7
a. Forms base of the tongue and larynx; suspended
in the neck, mobile, LACKS other bony articu-
lations but is attached to many muscles.
8 b. Superior and anterior to thyroid cartilage of the
3 larynx (voice box).
3. Vertebral column (26): separated by intervertebral
discs (7 cervical, 12 thoracic, 5 lumbar, 1 sacrum,
4 1 coccyx):
a. Atlas (C1): allows back-and-forth movement of
skull.
b. Axis (C2): allows side-to-side movement of
skull (contains dens, pivoting fulcrum for atlas).
4. Thoracic cage (25): 1 sternum and 24 ribs
(12 pairs).
Dense connective tissue 8 E. Appendicular skeleton: upper and lower extremi-
ties, pectoral and pelvic girdles, attaching ­extremities
to axial skeleton (Figure 3-3):
Loose connective tissue 7

Connective tissue papillae 6 1. Upper extremities (60): each extremity consists


Rete ridges 5 of 30 bones (1 humerus in arm, 1 lateral radius
and 1 medial ulna in forearm, 8 carpals in wrist,
5 metacarpals in hand, and 14 phalanges in
Hypodermis 4

Dermis (connective tissue proper) 3 fingers).


Epidermis (epithelium) 2 2. Pectoral girdle (4): clavicle (collar bone) and scap-
ula (shoulder blade) on each side.
3. Lower extremities (60): 30 bones in each extremity
Basement membrane 1

Figure 3-1  Structure of the skin, with epidermis and (1 femur in the thigh, 1 medial tibia and 1 lateral
dermis. (From Bath-Balogh M, Fehrenbach MJ: Illustrated den- fibula in the leg, 7 tarsals in the ankle and instep,
tal embryology, histology, and anatomy, ed 2, St. Louis, 2006, 5 metatarsals in the foot, 14 phalanges in the toes,
­Saunders/Elsevier.) and 1 patella [kneecap]).
4. Pelvic girdle (2): os coxae (innominate bones);
each is formed from ilium, ischium, pubis, fused
2. Spongy bone: LESS dense; consists of bony plates together; acetabulum, a large depression, provides
called trabeculae around irregular spaces that con- socket for head of femur.
tain red bone marrow. F. Articulations (joints): bones are held together by
B. Long bones, such as femur, humerus, phalanges: fibrous joint capsule lined with synovial mem-
1. Diaphysis: located around medullary cavity, with brane; secretes synovial fluid for lubrication and
epiphysis at each end. protection:
2. Epiphysis: covered with articular cartilage; 1. Synarthrosis: immovable fibrous joint, e.g., suture
­remainder of the bone is covered with perios- in skull.
teum. 2. Amphiarthrosis: slightly movable joint; bones are
C. Surface markings (elevations and depression) on connected by hyaline cartilage or fibrocartilage;
bones: allow articulation, muscle attachment; open- e.g., symphysis pubis, intervertebral discs.
ings: allowed for vessels. 3. Diarthrosis (synovial joint): freely movable joint;
D. Axial skeleton: skull, hyoid, vertebral column, tho- motion limited ONLY by surrounding tendons, liga­
racic cage: ments, bones; joint movement includes gliding,
1. Skull: 28 bones: condyloid, hinge, saddle, pivot, ball-and-socket;
a. Cranium: 8 bones (frontal, occipital, 2 parietal, e.g., temporomandibular, hip, knee, shoulder,
2 temporal, ethmoid, sphenoid). ­elbow, wrist.
Anatomy, Biochemistry, and Physiology 35

3 2

10

10 Volkmann's canal Osteon 5


Haversian canal 9 Canaliculi 4
Trabeculae of spongy bone 8 Lacunae containing osteocytes 3

Osteon of compact bone 7 Haversian canal 2

Periosteum 6 Lamellae 1

Figure 3-2  Structure of bone tissue. (From Fehrenbach MJ, ed: Dental anatomy coloring book, St. Louis,
2008, Saunders/Elsevier.)

Muscular System B. Attachments and actions: muscles are attached to


Muscular system includes skeletal muscle tissue that bones by tendons:
can be controlled voluntarily. Functions include mov- 1. Origin is the LESS movable attachment; inser-­
ing the skeleton, maintaining posture, supporting tion is the MORE movable attachment.
soft tissues, producing heat to maintain body temp­ 2. In function, insertion moves toward origin.
erature. C. Major axial muscles (according to site of action):
• See Chapter 4, Head, Neck, and Dental Anatomy: 1. Head and neck: facial expression, mastication,
muscles. cervical.
A. Structure of skeletal muscle (Figure 3-4): 2. Spine: erector spinae, forming a mass on either
1. Muscles and muscle fibers are surrounded by con- side of vertebral column.
nective tissue coverings, extend beyond muscle to 3. Trunk: thoracic wall (external and internal inter-
form tendons; endomysium surrounds individual costals), abdominal wall (external and internal
muscle fibers; perimysium surrounds each fascicu- oblique, transversus abdominis, rectus abdominis),
lus (functional muscle fiber group); epimysium diaphragm.
covers entire muscle. 4. Pelvic floor: superficial (bulbospongiosus, ischio-
2. Each skeletal muscle fiber represents single mus- cavernosus, transversus perinei) and deep (levator
cle cell: sarcolemma, sarcoplasm, sarcoplasmic ani, coccygeus).
reticulum, mitochondria, and myofibrils composed D. Major appendicular (Figure 3-5):
of myofilaments actin and myosin, organized in 1. Shoulder (trapezius, serratus anterior); arm
­repeating sarcomeres; arrangement is responsible (pectoralis major, latissimus dorsi, deltoid, ­rotator
for striations in skeletal muscle fibers. cuff).
36 Saunders Review of Dental Hygiene

15
2
6 16
3 7
4

17
5 8

9 18
10 19

11
12

13
14

Cervical vertebrae
15 Verterbal column/ 10 Carpals Os coxae 5

19 Coccyx 14 Fibula Ulna 9 Ribs 4


18 Sacrum 13 Tibia Radius 8 Sternum 3

17 Lumbar vertebrae 12 Patella Humerus 7 Clavicle 2


16 Thoracic vertebrae 11 Femur Scapula 6 Skull 1

Figure 3-3  Bones of the skeleton. (From Fehrenbach MJ, ed: Dental anatomy coloring book, St. Louis, 2008,
Saunders/Elsevier.)

2. Forearm (triceps brachii, biceps brachii, brachia- hamstrings in posterior compartment (semimem-
lis, brachioradialis); hand (extensors on posterior branosus, semitendinosus, biceps femoris).
surface of forearm, flexors on anterior surface, 5. Ankle and foot: dorsiflexion (tibialis anterior in
pronator, supinator). anterior compartment), plantar flexion (gastrocne-
3. Thigh: abduct (“move away”) and rotate (gluteus mius and soleus in posterior compartment), ever-
maximus, gluteus medius, gluteus minimus, tensor sion (peroneus in lateral compartment).
fasciae latae), adduct (“move toward”) (adductor
longus, adductor brevis, adductor magnus, graci- Nervous System
lis), flex (iliopsoas). Nervous system includes all neural tissue (neurons and
4. Leg: extensors are the quadriceps femoris in ante- neuroglia). The synapse is in the region of communica-
rior compartment (vastus lateralis, vastus medialis, tion between neurons; communication occurs through
vastus intermedius, rectus femoris); flexors are the effects of neurotransmitters. Sensory nerves
Anatomy, Biochemistry, and Physiology 37

1 (1) Outer surface is cerebral cortex, composed


of gray matter and marked by gyri (raised
folds) and sulci (grooves, furrows).
2 (2) ����������������������������������������������
 ���������������������������������������������
Each hemisphere has frontal, parietal, oc-
cipital, temporal lobe, insula; primary so-
matosensory area is in parietal, primary
3
somatomotor area is in frontal, visual cor-
tex is in occipital, auditory and olfactory
areas are in temporal, taste is in parietal.
b. Diencephalon: surrounded by cerebrum; in-
cludes thalamus, hypothalamus, pituitary
4
gland:
(1) Thalamus: serves as relay station for sen-
sory impulses that travel to cerebral cortex;
5
hypothalamus plays MAIN role in main-
taining homeostasis (body’s thermostat).
Myofilaments 5 (2) Brainstem: between diencephalon and
Myofibril 4 spinal cord; consists of midbrain, pons,
medulla oblongata; midbrain: contains vol-
untary motor tracts and visual and auditory
Myofiber 3
Muscle fascicle 2 reflex centers; pons: contains the pneumo­-
Muscle 1 t­axic and apneustic areas that help regulate
breathing movements; medulla oblongata:
Figure 3-4  Structure of muscles. (From Fehrenbach MJ, ed:
Dental anatomy coloring book, St. Louis, 2008, Saunders/Elsevier.)
contains ascending and descending tracts
and vital cardiac, vasomotor, and respira-
tory centers.
c. Cerebellum: second largest portion; motor area
(dorsal root) transmit information to the brain and spi- that coordinates skeletal muscle activity; main-
nal cord. Motor nerves (ventral root) transmit infor- tains muscle tone, posture, balance.
mation from the brain and spinal cord to effectors, such 3. Spinal cord: continuation of medulla oblongata
as muscles and glands. Integrative functions interpret and extends from foramen magnum to first lumbar
the sensory ­ information and act by stimulating motor vertebra; acts as conduction pathway and reflex
pathways. Divided into central nervous system (CNS) center:
and peripheral nervous system (PNS), which func- a. Ascending tracts: conduct sensory impulses to
tion together to integrate and coordinate body activi- brain.
ties, assimilate experiences, and assist in memory and b. Descending tracts: carry motor impulses from
learning. brain to effectors.
• See Chapters 6, General and Oral Pathology: nervous B. PNS includes all cranial and spinal nerves:
system pathology; 9, Pharmacology: drugs and nervous 1. Consists of afferent (sensory) division and effer-
system; 14, Pain Management: nerve anatomy and ent (motor) division; efferent division includes
physiology. somatic nervous system (SNS), which enables
A. CNS: composed of the brain and spinal cord (Figures voluntary control over skeletal muscle contrac-
3-6 and 3-7): tion, and autonomic nervous system (ANS), which
1. Within the brain, covered by meninges: outermost regulates smooth and cardiac muscle contraction
layer (dura mater), middle layer (arachnoid), and and glandular activity.
innermost layer (pia mater); subarachnoid space 2. Components: bundles of nerve fibers (axons and
between arachnoid and pia mater contains blood dendrites), 12 pairs of cranial nerves that emerge from
vessels and cerebrospinal fluid. inferior surface (caudal) of brain (see next section).
2. Components of the brain: 3. Spinal nerves contain both sensory and motor fi-
a. Cerebrum: largest portion, divided into two bers; 31 pairs emerge from spinal cord and are
hemispheres by longitudinal fissure; connected grouped according to origin; in all but thoracic re-
by band of white fibers, corpus callosum, re- gion, MAIN portions of nerves form complex net-
sponsible for higher level thought processes works called plexuses (cervical, brachial, lumbar,
(i.e., what we need to channel during the case sacral); nerves emerge from plexuses to supply
study portions of NDHBE): specific regions.
MJ, ed: Dental anatomy coloring book, St. Louis, 2008, Saunders/Elsevier.)
Figure 3-5  General view of body musculature. A, Anterior view. B, Posterior view. (From Fehrenbach
1 Deltoid 13 Infraspinatus 1 Deltoid 12 Brachialis

2 Rhomboideus major 14 Teres minor 2 Biceps brachii 13 Linea alba
3 Trapezius 15 Teres major 3 Serratus anterior 14 External abdominal
oblique
4 Latissimus dorsi 16 Triceps brachii 4 Brachioradialis
15 Iliopsoas
5 Gluteus medius 17 Extensor digitorum 5 Tensor fasciae latae
16 Adductor longus
6 Gluteus maximus 18 Tensor fasciae latae 6 Sartorius
17 Adductor magnus
7 Adductor magnus 19 Semitendinosus 7 Gastrocnemius
18 Gracilis
8 Gracilis 20 Semimembranosus 8 Tibialis anterior
19 Vastus medialis
9 Soleus 9 Soleus
21 Biceps femoris 20 Rectus femoris
10 Calcaneal tendon 10 Trapezius
22 Gastrocnemius 21 Vastus lateralis
11 Cut edge of trapezius 11 Pectoralis major
23 Peroneus longus
12 Supraspinatus
B A
23
10
9 9
8
22 7
21
21
20
8 20
19
19 6
7 18
17
18 6 16
15 5
5
17
14
4
13
16
3
4 12
15 2
11
14 3
1
13 2
12 10
11 1
Saunders Review of Dental Hygiene 38
Anatomy, Biochemistry, and Physiology 39

is MOST active when body is in emergencies


or stressful conditions; MOST postganglionic
neurons release norepinephrine.
1 b. Parasympathetic division (craniosacral divi-
sion) (PANS) is energy-conserving system that
is MOST active when the body is in normal re-
2 laxed condition; MOST postganglionic neurons
release acetylcholine.
3 Cranial Nerves
Cranial nerves are part of the PNS; designated by both
number and Roman numerals I through XII, as well as
specific name (Figure 3-8). Include 12 pairs that are con-
4 nected to the brain at its base and pass through the skull
by way of fissures and foramina; may be either afferent or
efferent or have both types.
• See Chapters 4, Head, Neck, and Dental Anatomy: fifth
(trigeminal) and seventh (facial); 6, General and Oral
Cerebellum 4

Brainstem 3 Pathology, and 16, Special Needs Patient Care: neuro-


Diencephalon 2 logical disorders.
A. First (I) (olfactory): afferent for smell from the nasal
mucosa to the brain, enters skull through perforations
Cerebral hemisphere 1

Figure 3-6  Brain. (From Fehrenbach MJ, ed: Dental anatomy in cribriform plate of ethmoid to join olfactory bulb in
coloring book, St. Louis, 2008, Saunders/Elsevier.) brain.
B. Second (II) (optic): afferent for vision from retina to
brain, enters skull through optic canal of sphenoid,
where right and left optic nerves join at the optic
chasm and where many fibers cross to the opposite
1 side before continuing into the brain as optic tracts.
2 Glaucoma results from damage to this nerve.
3 C. Third (III) (oculomotor): efferent to eye muscles, car-
4 ries preganglionic parasympathetic fibers to ciliary
5
7 ganglion near the eyeball and postganglionic fibers
6 innervate muscles inside the eyeball; located in lateral
8 wall of cavernous sinus and then exits skull through
superior orbital fissure on its way to the orbit; stra-
bismus (lack of eye coordination, “cross-eyed”) can
Spinal cord 8

Cerebellum 7 occur with damage.


D. Fourth (IV) (trochlear): efferent to one eye muscle
Hypothalamus 3
Medulla 6
Thalamus 2 (superior oblique) for proprioception; runs in lateral
wall of cavernous sinus and exits skull through the
Pons 5
Diencephalon
Midbrain 4
hemisphere superior orbital fissure on its way to the orbit.
Brainstem Cerebral 1 E. Fifth (V) (trigeminal): efferent for muscles of mas-
Figure 3-7  Brain and spinal cord. (From Fehrenbach MJ, ed: tication and for other cranial muscles; afferent for
Dental anatomy coloring book, St. Louis, 2008, Saunders/Elsevier.) ALL of oral cavity, including teeth and portions of
tongue, and for MOST of facial skin; trigeminal neu-
ralgia is a neural lesion involving afferent nerves to
the face.
4. Autonomic nervous system (ANS) is a visceral 1. Trigeminal ganglion (semilunar, gasserian) is on
efferent system part of PNS; innervates smooth anterior surface of petrous portion of temporal
muscle, cardiac muscle, and glands; has two bone; largest cranial nerve; has two roots: sensory
­divisions, with MOST organs being innervated by and motor:
BOTH to maintain homeostasis: a. Sensory root has three divisions: ophthalmic,
a. Sympathetic division (thoracolumbar divi- maxillary, mandibular; each enters skull in dif-
sion) (SANS) is energy-expending system that ferent location in sphenoid:
40 Saunders Review of Dental Hygiene

4 1

2
3
6

5
7

9
8

12

10 11

12 Hypoglossal nerve (XII) Abducens nerve (VI) 6


11 Accessory nerve (XI) Trigeminal nerve (V) 5
10 Vagus nerve (X) Trochlear nerve (IV) 4
Glossopharyngeal nerve (IX) 9 Oculomotor nerve (III) 3
Vestibulocochlear nerve (VIII) 8 Optic nerve (II) 2
Facial nerve (VII) 7 Olfactory nerve (I) 1

Figure 3-8  Cranial nerves. (From Fehrenbach MJ, ed: Dental anatomy coloring book, St. Louis, 2008,
Saunders/Elsevier.)

(1) Ophthalmic division: provides sensation cavernous sinus; often is FIRST nerve affected by si-
for upper face and scalp; enters through su- nus infection.
perior orbital fissure (SOF). G . S eventh (VII) (facial): efferent for muscles of
(2) Maxillary division: provides sensation for facial expression, for preganglionic parasympa-
middle face; enters by way of foramen ro- thetic innervation of lacrimal, and for subman-
tundum. dibular and sublingual; afferent for skin behind
(3) Mandibular division: provides sensation the ear, for taste sensation, and for the body of the
for lower face and sensory for anterior one tongue:
third of tongue; enters by way of foramen 1. Leaves cranial cavity by passing through internal
ovale. acoustic meatus; leads to facial canal inside tem-
b. Motor root accompanies mandibular division poral bone; exits the skull by way of stylomastoid
of the sensory root and also exits skull through foramen.
foramen ovale. 2. Nerve lesions include permanent or transient fa-
F. Sixth (VI) (abducens): efferent to one of the eyeball cial paralysis, loss of movement of muscles of fa-
muscles (lateral rectus); exits skull through superior cial expression caused by injury to nerve or Bell’s
orbital fissure on its way to the orbit; runs through palsy (BOTH motor and sensory deficit).
Anatomy, Biochemistry, and Physiology 41

H. Eighth (VIII) (vestibulocochlear): afferent for hear- 4. Pain (nociceptors): free nerve endings that are
ing and balance; enters cranial cavity through internal stimulated by tissue damage.
acoustic meatus. B. Gustatory (taste) and olfactory (smell) are localized
I. Ninth (IX) (glossopharyngeal): efferent for pharyn- special senses, closely related, complementary:
geal muscle (stylopharyngeal) and preganglionic 1. Taste buds are located on the lingual papillae on
parasympathetic innervation for parotid; afferent the dorsal surface of tongue; salty, sweet, sour,
for pharynx (swallowing), taste, and general sen- bitter are four taste sensations (possibly savory is
sation from base of tongue (posterior one third of another) (see Chapter 4, Head, Neck, and Dental
tongue). Anatomy).
1. Passes through skull by way of jugular foramen 2. Seventh cranial nerve (facial): transmits taste im-
and tympanic branch; has sensory fibers for ­middle pulses from anterior two thirds of tongue’s body,
ear and preganglionic parasympathetic fibers for and ninth cranial nerve (glossopharyngeal) transmits
parotid; arises within the gland and reenters the impulses from posterior one third of tongue’s root;
skull. impulses are interpreted in parietal lobe of brain.
2. After supplying ear, parasympathetic fibers leave 3. Receptors for smell: located in olfactory epithelium
skull through foramen ovale as lesser petrosal of nasal cavity; first cranial nerve (olfactory) trans-
nerve; these preganglionic fibers then end in otic mits impulses from olfactory epithelium; impulses
ganglion, located near mandibular division of tri- are perceived as odors in temporal lobe of brain.
geminal. C. Visual sense is a localized special sense:
J. Tenth (X) (vagus): longest nerve in body; efferent for 1. Protective features of eye: eyebrows, eyelids, and
muscles of soft palate, pharynx, and larynx and for para- eyelashes (cilia) protect from foreign particles and
sympathetic fibers to many major body organs, includ- irritants; lacrimal glands produce antibacterial
ing thymus, heart, stomach; afferent for skin around ear tears that moisten, cleanse, protect.
and for taste sensation of the epiglottis; passes through 2. Structure of eyeball: sclera (white) and cor-
skull by way of the jugular foramen; originates in brain- nea (clear): outermost fibrous layer; iris: middle
stem and terminates in transverse colon. ­vascular layer that regulates pupil size; retina: in-
K. Eleventh (XI) (accessory): efferent for trapezius and nermost neural layer and contains visual receptors
sternocleidomastoid, as well as muscles of soft palate (Figure 3-9).
and pharynx; ONLY partly a cranial nerve; consists of 3. Refraction (bending of light rays): necessary to
two roots: one arising from brain and one from spinal focus light rays on retina; in normal relaxed eye,
cord; exits skull through jugular foramen. refractive media are sufficient to focus light rays
L. Twelfth (XII) (hypoglossal): efferent for intrinsic and from 20 or more feet on retina; closer objects re-
extrinsic muscles of tongue; exits skull through hypo- quire accommodation to focus, which involves
glossal canal; tongue is ONLY organ in the body with change in shape of lens, convergence of eyes, con-
own cranial nerve; damage, such as by cerebrovascu- striction of pupil.
lar accident (CVA, stroke), can result in deviation of 4. Photoreceptors are rods for vision in dim light and
tongue to one side. cones for color and acuity in bright light; cones are
concentrated in fovea centralis and rods are located
Sense Organs peripherally in retina; visual pathway begins with
Sense organs include the widely distributed general impulses that are triggered by chemical reactions
senses and localized special senses of taste, smell, vision, in rods and cones that are transmitted to second
hearing, equilibrium. cranial nerve (optic).
• See Chapter 4, Head, Neck, and Dental Anatomy: lacri- 5. There is 25% crossover of visual fields between
mal gland. right (o.d.) and left eye (o.s.).
A. General senses: widely distributed in the body; in- D. Auditory sense is a localized special sense (Figure
clude touch, pressure, proprioception, thermorecep- 3-10):
tion, pain: 1. Regions of ear include outer ear (auricle and
1. Touch and pressure: free nerve endings, Meissner’s external auditory meatus), ending at tympanic
corpuscles, pacinian corpuscles. membrane; middle ear, containing auditory os-
2. Proprioception (sense of position or orientation): sicles (malleus, incus, stapes); inner ear, con-
Golgi tendon organs and muscle spindles. taining vestibule, semicircular canals, cochlea.
3. Thermoreception (temperature changes): thermo- 2. Receptors for hearing: located in organ of Corti
receptors are free nerve endings; some are sensi- in cochlea.
tive to heat, others to cold; extremes in temperature 3. Sound waves cause tympanic membrane vi-
stimulate pain receptors. brations, which are transmitted to inner ear by
42 Saunders Review of Dental Hygiene

7
8
9

1 10
1
11 3
2 12 4
3 13
5
4
5 14
6 6
15
7

Suspensory ligament 8
15 Conjunctiva Tragus 4
Ciliary body 7
14 Vitreous humor Lobule 7 External acoustic meatus 3
Sclera 6
13 Iris Antitragus 6 Helix 2
Choroid 5
with aqueous humor Intertragic notch 5 Auricle 1
12 Anterior cavityfilled Retina 4
11 Pupil Optic disk 3 Figure 3-10  External ear. (From Fehrenbach MJ, ed: Dental
anatomy coloring book, St. Louis, 2008, Saunders/Elsevier.)
10 Lens Optic nerve 2

E. Sense of equilibrium is a localized special sense:


Cornea 9 Fovea centralis 1

Figure 3-9  Anatomy of the eye. (From Fehrenbach MJ, ed: 1. Receptors are located in vestibule and semicir-
Dental anatomy coloring book, St. Louis, 2008, Saunders/Elsevier.) cular canals of inner ear.
2. As fluids in inner ear move relative to gravity,
ossicles; organ of Corti moves and hairs bend, hairs bend, triggering impulses transmitted to
triggering impulses transmitted by eighth cra- eighth cranial nerve (vestibulocochlear); brain
nial nerve (vestibulocochlear) to brain, where interprets impulses in terms of acceleration and
impulses are interpreted as sound. position.

clinical study  

Age 68 YRS SCENARIO

Sex ☐  Male   ☒  Female The dental hygienist calls to remind the


patient that she is due for her 3-month
Height 5’5” recall appointment. The patient says
Weight 145 LBS that she has just been diagnosed with
an eye disease that has permanently
BP 118/74 narrowed her field of vision. She does
not like the drops prescribed and won-
Chief Complaint “How am I going to get to the ­appointment?” ders why she should take them.
Medical History Osteoarthritis in both hands and neck
Removal of gallstones
Basal cell carcinoma of the lip
Past history of smoking

Current Medications Ibuprofen p.r.n.

Social History Retired grade school teacher


Anatomy, Biochemistry, and Physiology 43

1. What has happened to the patient’s eyesight? What is


the reason for it?
2. How does this affect her eyesight? To what drug is she 1 6
referring, and should she take it?
3. What type of test did she have? How often should per-
sons at risk have the test? 2 7
4. What factors would reduce the effectiveness of oral
care in the dental office for this patient?
3

1. The patient probably has glaucoma, a group of eye


diseases of second (II) cranial nerve (optic), which
8
involves loss of retinal ganglion cells in character-
4 9
istic pattern of optic neuropathy. Raised intraocu-
lar pressure (ocular hypertension) is significant risk 5
factor, as well as female gender, age, family history,
race (African-American or Asian), diabetes mellitus
(DM). Unstable ocular blood flow could be involved
in pathogenesis; may be related to hypertension (high
blood pressure). This information should be added to
patient’s health history.
2. Involves loss of visual field that often occurs gradually Ovary 9
and may not be recognized until advanced. Once lost, Adrenal gland 8
the visual field can never be recovered. Glaucoma is
second leading cause of blindness in United States. If
(on posterior surface of thyroid gland)
Parathyroid gland 7
increased intraocular pressure (above 20 mm Hg) is
present, can be lowered with drugs, usually eye drops,
Pineal gland 6

or surgery if severe. Major reason for vision loss is Testis 5


poor compliance with drugs and follow-up visits. One Pancreas 4
type of drop medication is pilocarpine, a nonselective
muscarinic receptor agonist in the parasympathetic
Thymus 3

nervous system, which relieves the symptoms by de- Thyroid gland 2


creasing intraocular pressure (also available in weekly Pituitary gland 1
time-released inserts). This drug is also given in intra-
oral form (Salagen) for non-drug-induced xerostomia. Figure 3-11  Endocrine glands. (From Fehrenbach MJ, ed:
Any new medications should be added to patient’s Dental anatomy coloring book, St. Louis, 2008, Saunders/Elsevier.)
drug history.
3. The patient had a dilated eye examination to check
nerve. Includes measurements of intraocular pressure
via tonometry, changes in size or shape of eye, and
retinal examination of nerve to look for visible dam- called a hormone, and effects are MORE generalized and
age; if damage is suspected, formal visual field test is of MORE duration.
performed. Those at risk are advised to have the test at Endocrine glands are ductless; secretions flow directly
least once a year. into blood, which carries the hormones to appropriate
4. For visually impaired, not being able to see objects region (exocrine glands release product to the body sur-
in path or to view instructions or procedures being face). Exert effects on specific target organs by attaching
done. to receptor sites, causing some type of reaction in organ.
A. Chemical classes of hormones:
Endocrine System 1. Proteins and polypeptides: long chains of amino
Endocrine system consists of endocrine glands (release acids; class includes MOST of hormones NOT
product into bloodstream) and hormones produced (Fig- listed below.
ure 3-11). Acts with nervous system to regulate body 2. Amines: derivatives of amino acids; class includes
activities to maintain homeostasis. Nervous system acts epinephrine, norepinephrine, thyroxine.
through electrical impulses and uses a neurotransmitter, 3. Steroids: derivatives of cholesterol, lipid; class
and effects are MORE localized and of LESS duration; includes sex hormones and adrenal cortex hor-
endocrine system acts through a chemical messenger mones.
44 Saunders Review of Dental Hygiene

B. Mechanism of hormone action: react with specific provide defense against disease, mediate
receptor sites on or in cells; cells with appropriate re- ­inflammatory reactions. See Table 3-1 for more
ceptors make up the target tissues. information.
1. Protein hormones react with receptors on the cell (1) Lymphocytes: second MOST common leu-
membrane; amines and steroids react with recep- kocytes; three types: B cells, T cells, NK
tors in the cytoplasm or nucleus. (natural killer) cells.
2. MOST action is regulated by negative feedback, (a) B cells: mature in bone marrow and
with some positive feedback; stimulus may be gut-associated lymphoid tissue such as
other hormones, substances in the blood, or direct lymph nodes.
neural stimulation. (b) T cells: mature in thymus.
C. Endocrine glands and related hormones: (c) NK cells: mature in bone marrow, large
1. Anterior pituitary (adenohypophysis): growth hor- cells involved in first line of defense
mone (GH), adrenocorticotropic hormone (ACTH), against tumor cells or virally infected
thyroid-stimulating hormone (TSH), follicle- cells by killing them; thus are NOT
­stimulating hormone (FSH), luteinizing hormone considered part of immune response.
(LH), prolactin. (2) Cytokines: chemical mediators of immune
2. Intermediate pituitary (pars intermedius): melano- response; produced by B and T cells.
cyte-stimulating hormone (MSH). (3) B cells divide during immune response to
3. Posterior pituitary (neurohypophysis): antidiuretic form plasma cells:
hormone (ADH) and oxytocin. (a) Plasma cell: round cell with a single
4. Thyroid gland: thyroxine, triiodothyronine (iodin- round nucleus (cartwheel appearance).
ated amino acids), calcitonin; parathyroid gland: (b) Once mature, plasma cells produce an-­
parathyroid (PTH) (see Chapter 4, Head, Neck, tibody (immunoglobulin [Ig]).
and Dental Anatomy). (c) Five distinct classes of antibodies (Igs):
5. Adrenal (suprarenal) cortex: mineralocorticoids IgA, IgE, IgD, IgG, IgM (Table 3-2).
(aldosterone), glucocorticoids (cortisol), sex ste- (d) Two types of IgA: serum and secretory.
roids; adrenal (suprarenal) medulla: epinephrine (e) Thrombocytes (platelets), small frag-
and norepinephrine. ments of cells, aid in blood clotting.
6. Pancreas (islets of Langerhans): insulin and glucagon. 4. Blood types (ABO and Rh):
7. Testes: androgens (testosterone); ovaries: estrogen a. ABO blood types are based on agglutinogens
and progesterone. (antigens) and agglutinins (antibodies).
b. In transfusion reactions, recipient’s agglutinins
Cardiovascular System react with donor’s agglutinogens; type AB is
Cardiovascular system (CVS) includes blood, heart, universal recipient; type O is universal ­donor.
and blood vessels. c. Example: people with Rh+ blood have Rh ag-
• See Chapters 8, Microbiology and Immunology: immu- glutinogens; those with Rh− blood do NOT;
nology of the blood; 9, Pharmacology: drugs that affect neither type has anti-Rh agglutinins; expo-
CVS. sure to Rh+ blood causes an Rh− individual
A. Blood (liquid connective tissue): to develop anti-Rh agglutinins, and subse-
1. Composition: MOSTLY plasma at ~55% of blood quent exposures may result in transfusion
volume; formed elements compose ~45% of ­reaction.
­volume. B. Heart: muscle that pumps the blood, with two thirds
2. Plasma: MAINLY water; also contains plasma on the left of the sternum; enclosed in fibrous pericar-
proteins, nutrients, gases, electrolytes, nitrogenous dial sac lined with serous membrane (Figure 3-12):
waste compounds. 1. Heart wall: outermost layer of epicardium; thickest
3. Formed elements in blood: middle layer of myocardium composed of cardiac
a. Erythrocytes (red blood cells [RBCs]): bicon- muscle; innermost layer of endocardium.
cave disks without nuclei that contain hemoglo- 2. Four chambers: atria are thin-walled receiving
bin, transport oxygen (O2) and carbon dioxide chambers (superior); ventricles are thick-walled
(CO2). pumping chambers (inferior).
b. Leukocytes (white blood cells [WBCs] ): have 3. Right atrium: receives O2-poor blood from sys-
nuclei and do NOT have hemoglobin; five temic circulation through inferior and superior ve-
types include neutrophils (polymorphonuclear nae cavae; left atrium receives O2-rich blood from
leukocytes [PMNs], MOST numerous), mono- lungs through pulmonary veins; the ONLY veins
cytes, lymphocytes, eosinophils, basophils; that carry oxygenated blood.
Anatomy, Biochemistry, and Physiology 45

Table 3-1  Blood cells and related tissue cells

Cells Microscopic appearance Description Functions

Neutrophil or Multilobulated nucleus with Inflammatory response:


polymorphonuclear granules phagocytosis
leukocyte (PMN)

Lymphocyte Round eccentric nucleus with- B and T types of cells:


out granules, B, T, and NK ­immune response: ­humoral
types and cell‑mediated; also
NK cell: defense against
tumor cells and virally
infected cells
Plasma cell Round nucleus derived from Humoral immune
B-cell lymphocytes ­response: produces
­immunoglobulins

Monocyte (blood) or Bean-shaped nucleus with Inflammatory and immune


macrophage (tissue) poorly staining granules response: phagocytosis
as well as process and
­present immunogens

Eosinophil Double‑lobed nucleus with red Immune response


granules

Basophil Irregularly shaped double- Immune response


lobulated nucleus with blue
granules

Mast cell (tissue Irregularly shaped double- Immune response


eosinophil) ­lobulated nucleus with
red granules

From Bath-Balogh M, Fehrenbach MJ: Illustrated dental embryology, histology, and anatomy, ed 2, St. Louis, 2006, Saunders/Elsevier.

4. Right ventricle: pumps blood to lungs through pul- 6. Purkinje fibers: in inner ventricular walls; con-
monary trunk; left ventricle pumps blood to sys- duct electrical impulse that enables heart to con-
temic circulation through aorta. tract in coordinated fashion; work with sinoatrial
5. Four heart valves: maintain unidirectional flow of node (SA node, has inherent rhythmicity) and
blood by opening and closing depending on the ­atrioventricular node (AV node, bundle of His) to
difference in pressure on each side. control heart rate.
a. Atrioventricular valves: between atria and ven- C. Blood vessels:
tricles; tricuspid valve is on right and bicuspid 1. Arteries: carry blood away from heart; thicker
(mitral) valve is on left. muscular walls than veins:
b. Semilunar valves: at exits from heart; pulmonary a. Radial artery, located on thumb side of wrist,
semilunar valve regulates flow from right ventri- palpated by two fingers of clinician to obtain
cle into pulmonary trunk; aortic semilunar valve radial pulse so as to measure heart rate in con-
is between left ventricle and ascending aorta. scious adults; in unconscious adults, common
46 Saunders Review of Dental Hygiene

Table 3-2  Known immunoglobulins (antibodies) 2. Systemic circulation: carries O2-rich blood from
left ventricle through systemic arteries to capil-
Immunoglobulin Description laries in the tissues of the body for O2 and CO2
Has two subgroups: serous in exchange; systemic veins transport O2-poor
blood and secretory in saliva blood containing CO2 from capillaries to supe-
IgA and other secretions; aids in rior and inferior venae cavae and then into right
defense against proliferation of atrium.
microorganisms in body fluids 3. Hepatic portal circulation: carries venous blood
Involved in hypersensitivity from digestive system to liver, filters through sinu-
­reactions, since can bind to soids, and then enters hepatic veins to inferior vena
IgE mast cells and basophils and
bring about release of bioactive cava (IVC).
substances such as ­histamine
Lymphatic System
Functions in activation of B-cell
IgD Lymphatic system is composed of lymph, lym-
­lymphocytes.
Major antibody in blood serum; phatic organs, lymphatic vessels. Functions to return
can pass placental barrier and excess interstitial fluid to the blood, absorb fats and
IgG
forms first passive immunity for fat-soluble vitamins from digestive system, provide
newborn defense against invading microorganisms and dis-
Involved in early immune res­ ease. Lymph is interstitial fluid that has entered lymph
IgM
ponses owing to involvement vessels.
with IgD in activation of B-cell • See Chapters 4, Head, Neck, and Dental Anatomy:
lymphocytes
nodes, tonsils; 6, General and Oral Pathology: lymph-
adenopathy.
carotid artery in neck palpated on ONLY one A. Lymphatic organs include lymph nodes, tonsils,
side by emergency ­personnel. spleen, thymus (Figure 3-14):
b. Arterioles: small arteries that branch off larger 1. Lymph nodes: dense mass of lymphocytes sepa-
artery; connect with capillary beds. rated by spaces called sinuses; filters and cleanses
c. Capillary beds: form connection between arter- lymph before enters blood; periphery of node
ies and veins; site for exchange of materials be- contains B cells, and germinal center contains
tween blood and cells. T cells:
2. Veins: carry blood toward heart; walls are thin- a. Nodes cluster in three areas: inguinal nodes
ner and have valves to prevent backflow of blood cluster in groin, axillary nodes in armpit, and
­(EXCEPT for emissary veins of face): cervical nodes in neck; NO nodes exist in CNS;
a. Veins are BEST point of access to ­bloodstream, must be enlarged (lymphadenopathy) to be pal-
permitting withdrawal of blood specimens pated.
­(venipuncture) for testing purposes and b. Tonsil: aggregate of lymphatic tissue that pro-
­intravenous (IV) delivery of fluid, electrolytes, vides protection against pathogens that enter
nutrition, drugs by needle or catheter. through nose, mouth, and throat; pharyngeal
b. Venules: smaller veins; drain the capillaries of a (adenoids) are located in nasopharynx; palatine
tissue area and then join larger veins. are located in oropharynx; lingual are located at
c. Venous sinuses: blood-filled spaces between posterior portion of tongue.
two layers of tissue. 2. Spleen: large masses of lymphocytes and macro-
3. Anastomosis: communication (joining) of one phages supported by reticular fibrous framework;
blood vessel with another by connecting ­channel. SIMILAR to lymph node but is larger and filters
4. Plexus: network of blood vessels (usually veins). blood instead of lymph, also acts as reservoir for
5. Pressure of circulating blood decreases as blood blood; contains red and white pulp.
moves through arteries, arterioles, capillaries, 3. Thymus: processes and matures T-cell lympho-
­venules, veins; blood pressure refers to arterial cytes, involved in defense against pathogens; also
pressure in larger arteries. produces hormone thymosin; involutes (shrinks)
D. Circulatory pathways (Figure 3-13): with aging.
1. Pulmonary circulation: transports O2-poor blood B. Lymphatic vessels carry fluid from interstitial spaces
from right ventricle through pulmonary arteries and return it to subclavian veins, where it becomes
to lungs, where blood picks up new O2 supply; part of blood plasma:
then pulmonary veins return O2-rich blood to left 1. Vessels have thin walls and valves; small lym-
atrium. phatic vessels merge to form larger ones until there
Anatomy, Biochemistry, and Physiology 47

11
12
1
13
2
14
3

4 15

16
5
17
6
18
7
19

8 20

9
21

10 22

22 Interventricular septum
21 Left ventricle 14 Cut edge of pericardium Right ventricle 7

20 Papillary muscle 13 Pulmonary artery Tricuspid (AV) valve 6


19 Chordae tendineae 12 Pulmonary trunk Right atrium 5

18 Mitral (AV) valve 11 Aorta (arch) Pulmonary veins 4


17 Aortic valve 10 Aorta (thoracic) Pulmonic valve 3

Left atrium 16 Trabeculae carneae 9 Pulmonary arteries 2


15 Pulmonary veins Inferior vena cava 8 Superior vena cava 1

Figure 3-12  Internal view of the heart. (From Fehrenbach MJ, ed: Dental anatomy coloring book, St. Louis,
2008, Saunders/Elsevier.)

are two MAIN ducts that empty into subclavian 3. Lymphatic ducts: larger lymphatic vessels; drain
veins. smaller vessels, then empty into venous system;
2. Afferent lymph flows into node; efferent lymph right lymphatic duct drains upper right quadrant of
flows out (area of hilum). body, and thoracic duct drains remainder of body.
48 Saunders Review of Dental Hygiene

2 10

9
3

4 8

5 7

10 Venules
9 Pulmonary veins

8 Aorta

7 Arteries and arterioles

6 Systemic capillaries
5 Veins and venules

4 Venae cavae

3 Pulmonary artery

2 Arterioles
1 Lung capillaries

Figure 3-13  Circulatory pathways. (From Applegate EJ: The anatomy and physiology learning system, ed 3, Philadelphia, 2006, Saun-
ders/Elsevier.)
­Saunders/Elsevier.)
Figure 3-14  Lymph organs. (From Fehrenbach MJ, ed: Dental anatomy coloring book, St. Louis, 2008,
1 Right lymphatic duct draining 7 Thoracic duct draining
into right subclavian vein into left subclavian vein
2 Axillary nodes 8 Thymus
3 Cisterna chyli 9 Thoracic duct
4 Inguinal nodes 10 Spleen
5 Palatine tonsils 11 Area drained by right lymphatic duct
6 Cervical nodes 12 Area drained by thoracic duct
10
4
12
3
11
9
2
8
7 1
6
5

49 Anatomy, Biochemistry, and Physiology
50 Saunders Review of Dental Hygiene

clinical study  

Age 23 YRS SCENARIO

Sex ☒  Male   ☐  Female During an extraoral examination, a hard fixed mass,


­approximately 6 mm wide by 36 mm long by 6 mm
Height 6’2” deep, is noted on the right side of the neck; enlarge-
ment is clearly visible but no discomfort is noted when
Weight 185 LBS
it is ­palpated. Other enlarged areas have been detected
BP 112/69 by the patient in his armpit and groin areas on same
side. He states that he feels fine, although he tires easily.
Chief Complaint “There is a swelling on my neck Intraoral exam shows poor oral hygiene and moderate
that has been there for 6 months gingivitis but no evidence of caries, only decalcification.
and is getting larger.”

Medical History Broken kneecap at age 8

Current Medications None

Social History Real estate lawyer


Drinks diet colas to keep awake

1. What could the enlargement along the right side of the A. Upper respiratory tract:
patient’s neck possibly be? 1. Nose (nasal cavity): warms, moistens, and filters
2. What would cause such a severe enlargement on his air; opens to outside through external nares and in
neck? to pharynx through internal nares; separated from
3. Identify the treatment planning procedures that should oral cavity by palate.
be completed. 2. Pharynx (throat): passageway for air; divided into
nasopharynx, oropharynx, laryngopharynx; open-
ing from oral cavity into oropharynx is the fauces.
1. Enlargement along right side of neck is most likely 3. Larynx (voice box): formed by nine cartilages that
either a tumor or an enlarged cervical lymph node are connected by muscles and ligaments:
(lymphadenopathy) along sternocleidomastoid (SCM) a. Three largest cartilages are thyroid, cricoid,
muscle of right side. epiglottis.
2. If involving the lymph nodes, such severe enlargement b. Vocal folds (cords): present in central region;
is indicative of severe infection (local or systemic) air passing through vibrates vocal folds to pro-
or malignancy (cancer). Severe infection should be duce sound. Hoarseness indicates inflammation
painful; lack of pain is prognostically threatening and or possibly cancer if chronic.
points to its being an early malignancy. B. Lower respiratory tract:
3. Treatment planning procedures should include thor- 1. Trachea (windpipe): framework supported by car-
ough intraoral examination for possible causes of tilage; divides into right and left primary bronchi.
swelling (e.g., pulpal infection, periodontal infection) 2. Bronchial tree:
and panoramic radiograph (shows entire jaw areas). a. Bronchi enter lungs and subdivide; branching
If no evidence suggests dental causative factors, oral pattern continues into smaller passages until
prophylaxis should be rescheduled until patient has terminates in alveoli with alveolar sac; permits
seen his physician. Imperative that he seek immediate rapid diffusion of oxygen and CO2.
medical consult. (Later, Hodgkin’s lymphoma, a can- 3. Lungs: formed by bronchial tree and alveoli.
cer of lymph nodes, was diagnosed in this patient.) a. Right lung: three lobes; shorter, broader, and
has greater volume than left lung.
Respiratory System b. Left lung: two lobes and indentation (cardiac
Respiratory system includes the upper and lower re- notch) to accommodate apex of heart.
spiratory tracts and the lungs (Figure 3-15). It functions c. Air flows into and out of lungs through conduct-
to move gases to and from exchange surfaces, where ing passages because of pressure differences
­diffusion can occur. Defends body against pathogens, between atmosphere and gases inside alveoli.
permits speech, helps regulate acid-base balance in This gas exchange is passed onto the rich vas-
body. cular network within the lung ­interstitium.
Anatomy, Biochemistry, and Physiology 51

2
3

7
8
9

10

11
Alveoli 11
Alveolar duct 10

Pulmonary artery 9

Pulmonary vein 8
Bronchiole 7

Lung 6 Larynx 3
Primary bronchi 5 Pharynx 2
Trachea 4 Nasal cavity 1

Lower tract Upper tract

Figure 3-15  Respiratory tract. (From Fehrenbach MJ, ed: Dental anatomy coloring book, St. Louis, 2008,
Saunders/Elsevier.)

Digestive System e­ limination of waste products, immunological functions


Digestive system includes gastrointestinal (alimentary) (e.g., secretory IgA).
tract (GIT; includes oral cavity, pharynx, ­ esophagus, • See Chapter 4, Head, Neck, and Dental Anatomy: oral
stomach, small intestine, large intestine, rectum, anus) cavity structure.
and accessory organs (liver, gallbladder, pancreas) A. Pharynx (throat): muscles in wall contract dur-
(Figure 3-16). Embryologically and physiologically, the ing deglutition (swallowing) to propel food along
mouth is the beginning of GIT. Functions include in- ­passageway into esophagus, then into stomach.
gestion, mechanical digestion, chemical digestion, se- B. Esophagus: carries liquids and solids from pharynx
cretion, mixing and propelling movements, absorption, to stomach:
52 Saunders Review of Dental Hygiene

1. Located in upper left quadrant of abdomen, ante-


rior to spleen; divided into fundus, cardiac region,
body, pyloric region.
2. Mucosal lining has folds (wrinkles) called rugae,
which increase surface area for absorption, and glan-
1 8 dular cells; parietal cells secrete hormone gastrin that
9 stimulates gastric acid production (hydrochloric acid)
2 and intrinsic factor (necessary for the absorption of
10 vitamin B12); also causes chief cells to secrete pep-
3
sinogen, which then is converted to pepsin by acids.
4
3. Chyme: semifluid mixture of ingested food, liquids,
and gastric juice that passes through pyloric sphinc-
ter into small intestine; pH of contents as low as 2.
D. Small intestine: extends from pyloric sphincter to
ileocecal valve; materials require ~5 hours to pass
through and into the large intestine; divided into duo-
11 denum (MOST enzymes), jejunum, ileum:
5
1. Glandular cells in mucosa produce peptidase,
which acts on proteins; maltase, sucrose, and lac-
6 12
tase, which act on disaccharides; lipase, which acts
on neutral fats.
13 2. Produces two hormones, secretin and cholecysto-
7
kinin; secretin stimulates pancreas to produce fluid
rich in bicarbonates; cholecystokinin stimulates
gallbladder to secrete bile and stimulates pancreas
14 to secrete digestive enzymes.
15
E. Large intestine: divided into cecum, colon, rectum,
anal canal; MAIN functions are to absorb water, elec-
trolytes, and vitamins; compact feces; and store fecal
Parotid salivary gland 8 material before defecation:
1. Cecum: short region inferior to ileocecal valve;
vermiform appendix is attached to cecum.
Anus 15 Large intestine 7
Rectum 14 Gallbladder 6
2. Rectum and anal canal: terminal portions of diges-
Small intestine 13 Liver 5 tive system.
Pancreas 12 Submandibular salivary gland 4 F. Accessory organs:
11 Stomach Sublingual salivary gland 3 1. Pancreas: extends from duodenum to spleen:
a. Composed MAINLY of exocrine cells, which
produce enzymes amylase, trypsin, peptidase,
Esophagus 10 Tongue 2
Pharynx 9 1 Mouth (oral cavity) lipase; pancreatic enzymes are carried to duo-
Figure 3-16  Organs of the digestive system. (From Fehren- denum by pancreatic duct (of Wirsung).
bach MJ, ed: Dental anatomy coloring book, St. Louis, 2008, Saun- b. Endocrine cells (islets of Langerhans) produce
ders/Elsevier.) insulin and glucagon.
2. Liver: largest visceral organ; located in upper right
quadrant of abdomen:
1. Located posterior to trachea and anterior to ver- a. Lobule consists of hepatocytes arranged like
tebral column; passes through diaphragm, enters spokes of a wheel around central vein and sepa-
cardiac region of stomach. rated by sinusoids.
2. Lower esophageal (cardiac) sphincter: controls b. Blood is brought to liver by portal vein and he-
passage of food from esophagus into stomach. patic artery, flows through sinusoids into cen-
C. Stomach: involved in temporary storage of in- tral vein and then into hepatic veins.
gested food, maceration and mixing of ingested c. Functions include production of bile (yellow color
materials, secretion of enzymes that begin breaking from bilirubin, breakdown of ­hemoglobin), produc-
­chemical bonds (digestion), production of intrinsic tion of plasma proteins, detoxification of harmful
factor, ­absorption of fluoride (IMPORTANT with substances, filtering of blood, nutrient metabolism,
toxicity): immunological response via Kupffer cells.
Anatomy, Biochemistry, and Physiology 53

straight portions in medulla, convoluted por-


tions in cortex.
c. Juxtaglomerular apparatus associated with
1
nephron secretes renin, which becomes angio-
tensin and increases blood pressure.
2
d. Kidneys excrete fluoride.
B. Ureters: enter urinary bladder; urine is formed in
kidney, leaves kidney through ureters, is transported
to urinary bladder; bladder is posterior to sym-
physis pubis in pelvic cavity and stores urine until
3
micturition.
C. Urethra: transports urine from urinary bladder to
4
­exterior.
D. Urine: fluid produced by nephrons of kidney, usually
1 to 2 L per 24 hours.
E. Blockages (renal calculi, kidney stones) along any
Urethra 4
of these structures result in increased pressure, may
Bladder 3 damage kidneys.
Ureter 2 Reproductive System
1 Kidney Reproductive system consists of the gonads (testes),
Figure 3-17  Components of the urinary system. (From ducts, accessory glands, penis in the male and gonads
F­ ehrenbach MJ, ed: Dental anatomy coloring book, St. Louis, 2008, (ovaries), uterine tubes, uterus, vagina, external genitalia,
Saunders/Elsevier.) mammary glands in the female.
A. Female (Figure 3-19):
1. Ovaries: located in pelvic cavity on each side of
uterus; ovum production occurs monthly in ovar-
3. Gallbladder: attached to visceral surface of liver ian follicles as part of ovarian cycle; follicle cells
by cystic duct; joins hepatic duct to form common produce estrogen.
bile duct; empties into duodenum; functions to 2. Uterine tubes (fallopian tubes, oviducts): extend
store, concentrate, secrete bile produced by liver. laterally from uterus; site for fertilization.
Contracts in response to cholecystokinin, hormone 3. Uterus: consists of a fundus, body, and cervix;
secreted by duodenum. superficial layer is sloughed off during men­
struation.
Urinary System 4. Vagina: muscular tube that extends from cervix to
Urinary system consists of kidneys, ureters, urinary exterior.
­bladder, urethra (Figure 3-17). Functions to rid wastes, 5. Mammary glands located in breast; consist of lob-
regulate fluid volume, maintain electrolyte concentra- ules of glandular units that produce milk; estrogen
tions, control blood pH, secrete renin and erythropoietin and progesterone stimulate breast development;
responsible for long-term maintenance of blood pressure. prolactin stimulates milk production; oxytocin
A. Kidney: one of a pair that are located behind perito- causes ejection (let-down) of milk.
neum (retroperitoneal) between twelfth thoracic and B. Male (Figure 3-20):
third lumbar vertebrae: 1. Testes: begin development in abdominal cavity
1. Enclosed by capsule; blood vessels enter at ureter and then descend into scrotum shortly before birth;
and leave at hilum. physically outside of the body because body tem-
2. Peripheral region is cortex; inner region is me- perature impairs sperm production.
dulla; left kidney higher than right one. 2. Testes: composed of seminiferous tubules, which
3. Nephron: functional unit of kidney; consists of produce sperm, and interstitial cells, which pro-
corpuscle and tubule. duce testosterone.
a. Corpuscle in cortex includes glomerulus and
Bowman’s capsule; blood filtration occurs in BIOCHEMISTRY  
glomerulus and filtrate passes into capsule Biochemistry is the study of living matter or organisms
(Figure 3-18). at a molecular level. Requires understanding of how mol-
b. Reabsorption of water and electrolytes to adjust ecules are structured, bonding to form MORE complex
volume and content of urine occurs in tubule; structures and thus affect living matter. Hydrocarbons
54 Saunders Review of Dental Hygiene

6
5

3
2

13 9

11
10

12

7 Macula densa
13 Descending loop 6 Juxtaglomerular cells
12 Nephron loop 5 Afferent arteriole
11 Ascending limb 4 Afferent arteriole
10 Distal convoluted tubule 3 Efferent arteriole
9 Collecting duct 2 Glomerulus
8 Ascending limb 1 Proximal convoluted tubule

Figure 3-18  Structure of the nephron. (From Applegate EJ: The anatomy and physiology learning ­system,
ed 3, Philadelphia, 2006, Saunders/Elsevier).

are carbon- and hydrogen-based molecules that exist in 2. Ethers: used as general anesthetics.
several different forms and have various properties. Found 3. Aldehydes: variety of uses, e.g., formaldehyde
within several products used daily (including in the dental used for preservation of biological samples.
office) and also form major components of the body. 4. Ketones: include progesterone and testosterone
• See CD-ROM for Chapter Terms and WebLinks. (sex hormones); produced in uncontrolled DM.
A. Parent hydrocarbons have chemical characteristics 5. Carboxyl groups (salicylic acid, constituent of aspirin).
that can be greatly influenced by replacement of hy- 6. Organohalogens: several uses; found in insecti-
drogen atoms with one or MORE functional groups: cides and BEST for treatment of brain cancer.
1. Alcohols: components of several natural products, 7. Amines: treatment of several ailments, including
including menthol (throat lozenges). migraine headaches, hypertension, inflamma­tion.
Anatomy, Biochemistry, and Physiology 55

13
12

11
2
10

3
9 4
8 5
7
6

7 Labia minora

13 Uterine (fallopian) tubes 6 Labia majora

12 Ovary 5 Urethra

11 Urinary bladder 4 Vagina

10 Symphysis pubis 3 Rectum

9 Mons pubis 2 Cervix


8 Clitoris 1 Uterus

Figure 3-19  Organs of the female reproductive system. (From Applegate EJ: The anatomy and physiol-
ogy learning system, ed 3, Philadelphia, 2006, Saunders/Elsevier).

8. Esters: used MAINLY topically; NO longer used • See Chapter 7, Nutrition: biomolecules.
as injected local anesthetics; frequent allergies to A. Carbohydrates: transport, energy, and structure of
ester agents (e.g., PABA) have stopped use as in- the cell:
jectables. 1. Monosaccharides: basic units of complex carbohy-
9. Amides: used MAINLY as injected local anesthetic drates (e.g., glucose, provides energy to brain).
agents or topically (but at lower levels than topical 2. Disaccharides: two monosaccharides (e.g., lactose,
esters); LESS allergenic than esters. milk sugar).
B. Saturated hydrocarbons: 3. Oligosaccharides: consists of 2 to 10 monosaccha-
1. Molecules that contain hydrogen atoms and car- rides (e.g., sucrose-6 monosaccharides); GREAT-
bon atoms connected by single bonds. EST effect on caries process.
2. Can be straight chain, branched, or cyclic; straight 4. Polysaccharides: many monosaccharides (e.g.,
chain hydrocarbons have higher boiling points than starch); LEAST effect on caries process.
branched hydrocarbons because MORE surface con- B. Lipid (fat): energy and structure of the cell:
tact is allowed between straight chain hydrocarbons. 1. Fatty acids: carboxylic acids with hydrocarbon
C. Unsaturated hydrocarbons: carbon- and hydrogen- side chains; exist as major component of other
based molecules that contain at least one double or naturally existing lipids.
triple bond between carbon atoms (e.g., vitamin A). 2. Triacylglycerols: three fatty acids that are esteri-
fied to a glycerol molecule; exist as a form of en-
Biomolecules ergy storage.
Biomolecules are found in living organisms and are clas- 3. Structural lipids: found in biological membranes
sified as belonging to one of four major groups: carbohy- and also work in transport of other lipids in the
drates, lipids, proteins, nucleic acids. circulatory system.
56 Saunders Review of Dental Hygiene

16
2
15
3
4
14
5
13
6
12 7

8
11
9
10

16 Bladder 8 Epididymis

15 Pubic symphysis 7 Bulbourethral gland

14 Ductus deferens 6 Membranous urethra

13 Prostate 5 Ejaculatory duct

12 Spongy urethra 4 Prostatic urethra

11 Penis 3 Rectum

10 Scrotum 2 Seminal vesicle

9 Testis 1 Ureter

Figure 3-20  Structures of the male reproductive system. (From Applegate EJ: The anatomy and
­physiology learning system, ed 3, Philadelphia, 2006, Saunders/Elsevier).

4. Steroids, prostaglandins, leukotrienes: involved in 4. Nucleic acids: encode genetic information and
regulation of body functions. are the molecular basis of heredity; made up
5. High-density lipoproteins (HDL): carry fatty acids of nucleotides, which consist of nitrogenous
and cholesterol from tissues to the liver (“good” base, sugar, phosphate; allow cellular repro-
cholesterol). duction:
6. Low-density lipoprotein (LDL): transports cho- a. Adenosine monophosphate (AMP).
lesterol and triglycerides from liver to tissues and b. Guanosine monophosphate (GMP).
regulates cholesterol synthesis at sites (“bad” cho- c. Cytosine monophosphate (CMP).
lesterol). d. Thymidine monophosphate (TMP); found
C. Amino acids, peptides, and proteins: components in ONLY in deoxyribonucleic acid (DNA).
the development of molecules used by the body for e. Uridine monophosphate (UMP); found ONLY
forming cell structure, catalyzing reactions, and trans- in ribonucleic acid (RNA).
mitting signals. 5. Include RNA and DNA; located in nucleus (DNA
1. Amino acids: building blocks (chemical struc- and RNA) and in cytoplasm (RNA):
tures) of peptides and proteins; thus proteins are a. Structure and replication of DNA is a double
degraded into amino acids when denatured. helix with two strands of DNA that run comple-
2. Peptides: molecules that consist of more than one mentary (antiparallel) to each other:
amino acid linked by peptide bond. (1) The two strands are held together by forma-
3. Proteins: functional molecules that consist of tion of hydrogen bonds between nucleotide
one or more peptide molecules present in the on one strand of DNA and nucleotide on
cells of all living organisms; important com- other strand of DNA.
ponents in chemical and physical cell func­ (2) Deoxy AMP forms base pair by forming
tioning. two hydrogen bonds with deoxy TMP.
Anatomy, Biochemistry, and Physiology 57

(3) Deoxy GMP forms base pair by forming A. Catabolism:


three hydrogen bonds with deoxy CMP. 1. Process of breaking down large complex mol-
(4) Semiconservative replication: ecules into simpler molecules.
(a) During replication, two complementary 2. Produces energy for physical activity; helps main-
strands of DNA are separated. tain body temperature and elimination of body
(b) Separation allows protein DNA poly- waste.
merase to synthesize new strand of B. Anabolism:
DNA complementary to each original 1. Process of building larger, more complex mol-
strand of DNA. ecules from smaller, simpler units.
(c) Replication of one double-stranded 2. Important in new cell development and in older
DNA molecule results in two double- cell maintenance.
stranded molecules of DNA, identical C. Enzymes (most): functional proteins involved in cata-
to original molecule of DNA. lyzing specific reactions.
(d) Process of DNA replication is necessary D. Adenosine triphosphate (ATP): main form of energy
for cell division; when cell divides into storage:
two, each daughter cell receives one of 1. Energy released from exothermic reactions can be
DNA molecules. stored in the form of ATP by addition of inorganic
b. Structure and function of RNA: phosphate (Pi) to molecule of ADP.
(1) RNA is similar to DNA in structure EX- 2. Energy stored in the form of ATP can be donated
CEPT that: to an endothermic reaction by hydrolysis of the
(a) RNA contains ribose sugar instead of inorganic phosphate (Pi) from the ATP molecule,
deoxyribose sugar. resulting in a molecule of ADP.
(b) RNA contains UMP instead of TMP. 3. Glycolysis: pathway for catabolism of the simple
(c) RNA is a single-stranded rather than a sugar glucose.
double-stranded molecule. 4. Pyruvate dehydrogenase: enzyme that catalyzes
(2) Overall function of mRNA, tRNA, rRNA is conversion of molecule of pyruvate into acetyl
the synthesis of proteins. CoA, which can be used in the tricarboxylic acid
c. Transcription: transfer of genetic informa- cycle (TCA cycle) for further production of energy.
tion from DNA to messenger RNA (mRNA) 5. Tricarboxylic acid cycle: pathway for further ca-
­transcript; genetic information from an mRNA tabolism of glucose to produce energy.
transcript is used as a guide for protein 6. Electron transport and oxidative phosphorylation:
­synthesis. pathway by which electrons proceed down the
d. Translation: transferring genetic information electron transport chain and result in the reduc-
from mRNA to synthesis of proteins. tion of O2 to H2O and eventually the production of
e. Mutagenesis: alteration in the DNA that ATP.
changes the encoded genetic information; 7. Overall goal of these metabolic pathways: use
chemicals, viruses, radiation are three classes food taken into the body for production of ATP,
of agents that cause mutation. enabling body to store energy for later use.
f. Gene expression: regulated by a number of pro-
teins called transcription factors, regulated by PHYSIOLOGY  
other internal and external stimuli. Physiology is the study of function of organ systems.
g. Genetic engineering: using specific techniques Lends meaning to anatomical structure; conversely,
for manipulating genetic information; e.g., gene anatomy is what makes physiology possible. Homeo-­
for human insulin can be placed in bacteria, al- stasis refers to the relative constancy or equilibrium of
lowing expression of large quantity of human the internal environment. Maintained by negative feed-
insulin (protein can then be purified from bacte- back mechanisms, under regulation from nervous and
ria and used for diabetic therapy). ­endocrine systems. When lacking, results in illness or
h. Gene therapy: type of genetic engineering; in- disease.
volves compensating for damaged gene respon- • See CD-ROM for Chapter Terms and WebLinks.
sible for causing illness by expressing foreign
gene in affected cells. Cell Physiology
Cell physiology is study of general and specific functions
Metabolism of cells and requires understanding of cell structure and
Metabolism is the process of producing and using energy environment.
and includes BOTH catabolism and anabolism. • See Chapter 2, Embryology and Histology: cell structure.
58 Saunders Review of Dental Hygiene

A. Physiological functions of cell: (1) Amount of water that moves depends on


1. Response to environment: concentration of water relative to solute and
a. Production of substances (e.g., mucus in re- relative permeability of membrane to solute.
sponse to tobacco smoke). (2) Isotonic solution has same osmotic charac-
b. Movement (e.g., of cell part, such as cilia, or teristics on both sides of the membrane; no
entire cell, such as lymphocyte). net movement of water occurs.
c. Ingestion of substances (e.g., nutrients or bacte- (3) Hypotonic solution contains MORE water
ria). outside the cell than inside the cell; water
d. Generation of electrical impulses (e.g., action moves into the cell and exerts a pressure;
potential). with increased volume, cell may burst.
2. Synthesis of proteins for structural and functional (4) Hypertonic solution contains LESS water
requirements. outside than inside cell; movement of water
3. Use of membrane transport for: out of cell makes it shrink (crenation).
a. Separation of internal and external cell environ- 3. Carrier-mediated transport:
ments. a. Uses molecules to move substances across
b. Communication between cells. plasma membrane.
c. Intracellular communication. b. Exhibits selectivity for a substance, saturation,
d. Fluid volume regulation. or a transport maximum (limit to how much can
4. Duplication for: be moved); competition for transporting sub-
a. Growth (mitosis). stances that are closely related chemically.
b. Reproduction (meiosis). c. Types of carrier-mediated transport:
5. Differentiation and specialization as: (1) Facilitated diffusion for molecules, like glu-
a. Muscle cells (skeletal, smooth, and cardiac). cose, moves down a concentration gradient;
b. Nervous cells (neurons). uses no energy.
c. Epithelial cells (lining of organs and tissues). (2) Primary active transport: moves substances
d. Connective tissue cells (fibroblasts, WBCs). against their concentration gradients; uses
B. Specific cell functions: energy in the form of ATP (e.g., Na+, K+
1. Transport of substances across the plasma mem- transmembrane ATPase pumps).
brane: (3) Secondary active transport (more than one
a. Permeability: ability of substance to move molecule is moved); uses energy:
across membrane. (a) Moves molecules against their concen-
(1) Completely permeable: anything can cross. tration gradients.
(2) Impermeable: nothing can cross. (b) Creates gradient to move second sub-
(3) Semipermeable: ONLY some substances stance with the energy used to move one
can cross. ion (e.g., coupled transport of sodium
2. Diffusion mechanism of transport: with amino acids in the intestine).
a. Random movement of molecules from a higher C. Cell communication:
to a lower concentration gradient; once con- 1. Examples of communication between cells:
centrations differences are eliminated, NO net a. Hormones: made by one cell type are trans-
movement occurs. ported in blood and affect cells at distant site.
b. Following factors affect the diffusion rate: b. Neurotransmitters: chemicals produced by one
(1) Higher the temperature, greater the diffu- neuron that affect neurons or muscle cells.
sion. c. Paracrine secretion: involves one cell secreting
(2) Smaller the molecule, faster it diffuses. a chemical (e.g., neurotransmitter, cytokine, gas
(3) Shorter the distance across the membrane, such as nitrous oxide) that affects nearby cell.
faster the rate of diffusion. d. Tight (nexus) junctions: specialized connec-
(4) Greater the available surface area for diffu- tions between cells that restrict movement of
sion, higher the rate of diffusion. substances between cells.
(5) MORE lipid soluble the substance, MORE 2. Examples of communication from the outside of a
readily it moves across the membrane. cell to its interior:
c. Movement of ions depends on their charge (dif- a. Channels: allow fluxes of ions (e.g., Na+, Cl−,
ferent charges attract, similar ones repel) and K+) or water into or out of a cell.
their concentration gradients. b. Receptors:
d. Osmosis: net movement of water down its con- (1) Bind chemicals from outside of the cell
centration gradient. that couple to membrane-bound enzymes
Anatomy, Biochemistry, and Physiology 59

(e.g., adenosine cyclase), affect intracellu-


lar second messenger systems (e.g., elevate
calcium levels).
(2) Large intracellular response occurs (am-
plification) when ONLY a small number of
membrane receptors are activated.
D. Membrane potential:
1. Occurs when there is a difference in the numbers
and charges of ions across plasma membrane.
2. Does NOT exist when the number and charge
characteristics of ions are the same on either side
of the membrane. A
3. Increases as both the concentration gradient and
difference in charge across the membrane in-
crease.
4. Affected by the permeability of a membrane for an
ion:
a. Membranes are “leaky” for K+ but “tight” for B
Na+. Figure 3-21  Skeletal muscle cell (A) and its components
b. Proteins have a negative charge and are trapped and the sarcomere (B), the contractile unit.
within the cell; K+ that leaves the cell because
of “leakiness” results in a relative negative
charge inside the cell compared with outside b. Endoplasmic reticulum (ER), involved in
the cell. ­energy generation, is well developed; actin and
myosin filaments make up myofibrils.
Neurophysiology 3. Contractile unit of the muscle is the sarcomere:
Neurophysiology is the study of the nervous system; a. “A” band: myosin (thick filaments composed of
includes central (CNS) and peripheral nervous sys- heads and tails) and actin (thin filaments); “M”
tems (PNS). Systems are responsible for sensing and line: ONLY myosin.
responding to both external and internal stimuli. For b. Actin filaments: surrounded by tropomyosin, to
more on organization of nervous system, see earlier which troponins are attached; these molecules
discussion. interact with calcium ions and allow actin to
• See Chapters 14, Pain Management: nerve anatomy bind to myosin heads; also allow filaments to
and physiology; 9, Pharmacology: products involved slide (causing contraction of the muscle).
in nervous systems; 16, Special Needs Patient Care: c. The “I” band: contains actin filaments and other
neural disease. proteins linked to “Z” line; the “H” zone con-
tains ONLY myosin filaments.
Skeletal Muscle Physiology B. Skeletal muscle function:
Skeletal muscles are relatively large cells and are in- 1. Neuromuscular junction and excitation-­contraction
volved in locomotion, work, heat production. See earlier coupling:
discussion on muscular system. a. Motoneurons release acetylcholine, which
• See Chapter 16, Special Needs Patient Care: muscle binds to receptors on muscle endplate.
diseases. b. Receptors form channels that respond to bind-
A. Basic structural components of skeletal muscle (Fig- ing of acetylcholine by allowing influx of
ure 3-21): cations into muscle cell, resulting in action po-
1. Connective tissue surrounds muscles, fascicles tential that travels along and into muscle fiber
(functional unit of the muscle), and fibers: by means of the “T” tubules.
a. Muscles are surrounded by epimysium. c. Action potential causes release of calcium from
b. Each muscle consists of fascicles surrounded sarcoplasmic reticulum (SR); calcium binds to
by perimysium. troponin and results in a conformational change
c. Muscle fibers are surrounded by endomysium. of tropomyosin, allowing actin to bind to myo-
2. Individual muscle cells or fibers are surrounded by sin heads activated with ATP.
sarcolemma, which invaginates into cell, forming d. Interaction of actin and myosin results in SR
“T” tubules: shorting (contraction); actin and myosin fila-
a. Muscle cells are large, multinucleated. ments require ATP to separate.
60 Saunders Review of Dental Hygiene

e. If calcium levels continue to be high within the c. Bind to plasma proteins.
cell, contraction continues; if NOT, relaxation 3. Steroid (cholesterol derivatives):
occurs because of calcium reuptake in SR. a. Lipophilic and easily enter cells; bind to both
2. Factors involved in muscle contraction: membrane and intracellular receptors.
a. Active tension develops from interaction of b. Can activate genes to initiate protein synthesis
actin and myosin against connective tissue out- and may activate second messengers.
side and proteins within muscle fibers. c. Produced in adrenal cortex, gonads (testes and
b. Motor unit consists of a motoneuron and all of ovaries), placenta.
the muscle fibers it innervates. C. Hormones produced by CNS:
c. Types of fatigue: 1. Pineal produces melatonin:
(1) Muscle fatigue: a. Induces sleep; involved in modulation of body
(a) Inability of a muscle to continue to work rhythms.
at a given level. b. Affects reproduction: increased levels lead to
(b) May be caused by increases of waste decreased reproduction.
products (lactic acid), lower energy c. Acts as antioxidant; enhances immune system.
stores (phosphates), muscle disease 2. Hypothalamus stimulates release:
(muscular dystrophy). a. Thyrotropin-releasing (TRH): thyroid-­stimulating
(2) Neuromuscular junction fatigue: inability hormone (TSH) from anterior pituitary, regu-
to manufacture enough acetylcholine pre- lates production of thyroid hormones.
synaptically. b. Corticotropin-releasing (CRH): adrenocor-­
(3) Central fatigue: inability of the CNS to ticotropin (ACTH), acts on adrenal cortex to
drive muscles; may occur in response to increase cortisol in response to stress.
muscle fatigue. c. Gonadotropin-releasing (GnRH): follicle-
(4) General fatigue at end of day (or at the end ­stimulating (FSH) and luteinizing (LH) from
of this chapter!): may be due to iron defi- anterior pituitary.
ciency (hypochromic anemia). (1)  FSH in females: growth of ovarian follicles,
(5) Chronic fatigue syndrome (CFS): unknown production of estrogen.
etiology; associated with fibromyalgia (2)  FSH in males: sperm development in testes.
(widespread pain of unknown origin). (3)  LH in females: formation of corpus luteum,
production of estrogen and progesterone.
Endocrinology (4)  LH in males: production of testosterone in
Endocrinology is the study of endocrine glands, the hor- testes.
mones secreted, and effects on body. See earlier discus- 3. Pituitary stimulates release:
sion of endocrine system. a. Growth hormone–releasing hormone (GH-RH)
• See Chapters 6, General and Oral Pathology: hormonal stimulates release:
pathology; 9, Pharmacology: hormones. (1) Growth hormone (GH) by anterior pitu-
A. Endocrine system: slower control system compared itary.
with CNS; allows for ability to “fine tune”: (2) GH acts directly or through production of
1. Produces some hormones in periphery and some in somatomedins:
CNS, but functions differ in each location. (a) Increases protein synthesis, influx of
2. Produces hormones in rhythmic manner (e.g., cir- amino acids into cells.
cadian, monthly). (b) Decreases cellular uptake of glucose,
B. Chemical categories of hormones: promotes blood glucose levels.
1. Peptide and protein: (c)  Stimulates insulin secretion and carti-
a. Hydrophilic (water-soluble); transported as free lage and bone growth.
hormones. b. Somatostatin inhibits release of GH.
b. Produced in hypothalamus, pineal, pancreas, c. Prolactin-releasing factor (PRF) stimulates an-
parathyroid, gastrointestinal tract, kidneys, terior pituitary to release prolactin, affects de-
liver, thyroid, heart, and other organs. velopment of breast and milk production.
2. Amines (tyrosine derivatives): d. Antidiuretic hormone (ADH, vasopressin) and
a. Catecholamines (hydrophilic), produced in ad- oxytocin are produced by hypothalamus but
renal medulla. stored in and released from posterior pituitary:
b. Thyroid hormones (iodinated tyrosine deriva- (1) ADH: decreases water excretion from the
tives), lipophilic (lipid soluble), act at genomic kidney, involved in stress responses; con-
level to produce new proteins. tracts blood vessels (EXCEPT in lungs).
Anatomy, Biochemistry, and Physiology 61

(2) Oxytocin: promotes uterine contractions (3) Goiter: enlarged thyroid (can occur with
and aids in milk ejection (let-down) by ­EITHER hypothyroidism or hyperthyroid-
breast with nursing. ism).
D. Hormones produced by PNS: 2. Hormones that affect calcium metabolism:
1. Thyroid hormones: a. Calcium levels in plasma are tightly regulated;
a. Produced and stored in thyroid gland, trans- the active form is free (1.3 mmol/L), NOT
ported in blood bound to plasma proteins. bound to plasma proteins; largest store of cal-
b. Include BOTH thyroxine (T4, tetraiodothyro- cium is in bone calcium and is also stored and
nine) and triiodothyronine (T3): released at the SR.
(1) T4 can be converted to T3 by deiodinase in b. Deviations from normal range of calcium levels
brain, kidneys, liver, brown fat. can affect:
(2)  Major systemic source of T3 is derived by (1) Neuromuscular excitability; low levels (hy­
conversion from T4 in liver and kidneys. pocalcemia) result in contraction and
(3) Production is regulated in negative feed- twitching of skeletal muscles; excitation-
back manner by TSH (Figure 3-22). contraction coupling in cardiac and smooth
c. Major functions: normal is considered euthyroid. muscles.
(1) Regulation of metabolic rate, heat produc- (2) CNS excitability; stimulus-secretion cou-
tion, body temperature. pling of hormones and neurotransmitters.
(2) Increases number of catecholaminergic re- (3) Maintenance of tight junctions between
ceptors on cells, heart rate, force of heart epithelial cells.
muscle contraction. (4) Clotting of blood.
(3) Intermediary metabolism of carbohydrates c. Parathyroid hormone (PTH): regulates cal-
and fats. cium, produced in parathyroid glands (within
(4) Stimulates growth, teeth eruption, CNS de- thyroid) along with phosphorus and magne-
velopment by interaction with GH. sium; vitamin D (cholecalciferol) produced by
d. Thyroid hormone disorders: the skin is necessary for absorption of calcium
(1) Consequences are dictated by severity of as well as phosphorus:
disorder, duration, time of onset. (1) Decreased plasma calcium levels stimulate
(2) Hypothyroidism (decreased hormones) and release of PTH.
hyperthyroidism (increased hormones). (2) PTH stimulates kidneys to conserve cal-
cium and phosphate.
(3) In conjunction with vitamin D, PTH increases
intestinal reabsorption of calcium and phos-
phate and increases bone reabsorption.
Hypothalamus (4) LOW levels of PTH can affect tooth devel-
opment and may result in delayed tooth erup-
tion along with defects in matrix, enamel
TRH
mineralization, and dentin production.
d. Calcitonin: affects BOTH calcium and phos-
Anterior pituitary phate handling; produced by thyroid C (inter-
follicular) cells (within thyroid):
(1) Increased levels of calcium increase release
TSH
of calcitonin.
(a) Calcitonin (short-term increase): de-
Thyroid gland creases movement of calcium from
bone into plasma.
(b) Calcitonin (long-term increase): de-
T3 and T4
creases bone reabsorption by inhibiting
­osteoclasts, thereby decreasing plasma
Target organ calcium and phosphate levels.
3. Hormones of pancreas: BOTH exocrine and endo-
Physiological response
crine functions:
Figure 3-22  Negative feedback in the endocrine system. a. Exocrine glands (acini): secrete enzymes (pro-
TRH, Thyroid-releasing hormone; TSH, thyroid-stimulating teases, amylase, lipases) and aqueous bicarbon-
hormone; T3, triiodothyronine; T4, thyroxine. ate-rich fluid into intestine.
62 Saunders Review of Dental Hygiene

b. Cells in the isles of Langerhans: produce insu- (3) Increases production in muscles; increases
lin, glucagon, somatostatin, pancreas polypep- long bone growth.
tide: b. Inhibin: produced by Sertoli cells; inhibits FSH
(1) Insulin: produced in beta cells: decreases production.
blood glucose, fatty acid, amino acid levels; 6. Ovarian hormones and the menstrual cycle (see
increases storage of these nutrients as gly- birth control pills and hormone replacement ther-
cogen, triglycerides, and protein; increases apy in Chapter 9, Pharmacology):
uptake of glucose: a. Hormones involved in female health:
(a) Release is stimulated MAINLY by el- (1) Estrogens (estradiol [MOST important],
evations in blood glucose levels. ­estrone, estriol):
(b) Release also occurs with increased blood (a) Essential for follicular (egg) maturation
amino acid levels and PNS stimulation. and release; stimulate development and
(2) Glucagon: produced in alpha cells; coun- maintenance of the female reproductive
teracts many of the effects of insulin; in- tract; induce production of progester-
creases breakdown of glycogen to glucose; one and oxytocin receptors.
produces ketone bodies from breakdown of (b) Increase fat deposition, which then de-
fatty acids; inhibits liver protein synthesis creases blood cholesterol levels.
and increases protein degradation; excess (c) Close epiphyseal plates and increase
glucagon increases effect of insulin defi- bone density.
ciencies (e.g., DM). (d) Act as antioxidants and increase vaso-
4. Hormones of adrenal glands: cortical and medul- dilation.
lary regions have different endocrine functions: (2) Inhibin inhibits production of FSH during
a. Hormones secreted by adrenal cortex: menstrual cycle.
(1) Mineralocorticoids (MAINLY aldoste- (3) Progesterone: prepares uterus for develop-
rone): produced in zona glomerulosa; in- ing fetus; inhibits GnRH production; stimu-
creases sodium retention and potassium lates alveolar development in breasts and
elimination by the kidneys, thus involved inhibits milk-secreting action of prolactin;
in long-term regulation of blood volume stimulates ventilation.
and pressure; secretion is stimulated by re- b. Hormonal changes during menstrual cycle:
nin-angiotensin system; decreases plasma ovarian cycles begin at puberty (~12 years,
sodium levels; increases potassium levels. “menarche”) and continue until menopause
(2) Glucocorticoids: produced in zona (~50 years), with possible breaks during preg-
­fasciculata; include cortisol, regulated by nancy and lactation.
ACTH, which increases gluconeogenesis, c. Hormones involved in fluid volume and blood
increases protein degradation in muscle pressure regulation:
(wasting), facilitates lipolysis; have permis- (1) Renin: produced by the kidney and stimu-
sive effect on actions of catecholamines; lated by:
exert antiinflammatory and immunosup- (a) Low plasma sodium levels, decreased
pressive effects. extracellular fluid volume.
(3) Sex steroid hormones (discussed later): (b) Decrease in blood pressure as sensed
produced in zona reticularis; dehydroepi- by the kidney; SNS stimulation.
androsterone (DHEA) is MOST important (2) Angiotensin I and II:
form of androgen; production peaks at 25 to (a) Renin is converted via enzyme angio-
30 years and then decreases. tensinogen (made in the liver, released
b. Medulla secretes MAINLY epinephrine and into plasma) into angiotensin I (AI).
some norepinephrine; control of medullary se- (b) In the lungs, angiotensin-converting en-
cretion is through the SNS. zyme (ACE) converts AI to angiotensin
5. Testicular hormones: II (AII).
a. Testosterone: produced in the testes under the (c) Potent vasoconstrictors; also increase
stimulation of LH, in conjunction with FSH: production of aldosterone.
(1) Increases production of RBCs. (3) ADH produced by hypothalamus in the
(2) Increases production of sperm; involved in CNS, NOT peripherally:
growth and maturation of male reproductive (a) Stimuli for its release include AII, de-
organs at puberty; induces development of crease in extracellular fluid volume, stress,
secondary male sexual characteristics. increase in extracellular fluid osmolarity.
Anatomy, Biochemistry, and Physiology 63

(b) ADH increases water retention by the 5. Defense against inhaled particles (airways and
kidney and is a potent vasoconstrictor. nose) and emboli (pulmonary circulation).
(4) Atrial natriuretic peptide is produced in the 6. Means for drug delivery because of the lung’s
right heart chamber and in the lungs: large surface area and ready access to circulatory
(a) Production is influenced by increase in system.
extracellular fluid volume. 7. Enhancement of venous return.
(b) Increases water excretion (diuretic); in- B. Specific functions of the respiratory system:
creases sodium excretion (natriuretic). 1. Process of ventilation:
(c) Increased plasma levels are found in a. Tidal volume (TV, minimum flow rate): volume
patients with congestive heart failure of breath that depends on force of ­ respiratory
(CHF). muscle contractions and elasticity of lung; IM-
d. Hormones that regulate satiety (feeling of full- PORTANT during administration of gases such
ness, disappearance of hunger): as nitrous oxide during dental treatment.
(1) Leptin is produced peripherally by white (1) Part of breath goes to alveolar sacs for gas
adipose cells: exchange.
(a) Transported in the blood across the (2) Part of breath remains in conducting branches
blood-brain barrier by selective carri- of lung, forming “dead space” volume.
ers; in brain acts on receptors located (3) During inspiration, alveolar pressures are
in the hypothalamus and brainstem re- below atmospheric pressure and suck air
gions to affect a number of physiologi- into lung.
cal functions. (4) During expiration, alveolar pressures are
(b) Also regulates reproductive behaviors above atmospheric pressures and push air
and sleep. out of lungs.
(2) Other neuromodulators and neurotransmit- (5) Frequency of breathing in adults is between
ters associated with feeding include opi- 14 and 20 breaths/min.
oids, galanin, neuropeptide. (6) Minute ventilation is product of TV and fre-
quency of breaths per minute.
Respiratory Physiology b. Multiple factors drive ventilation: elevated lev-
Respiratory physiology is the study of the respiratory els of CO2 in plasma are sensed MAINLY by
system, which controls gas exchange, acid-base balance, neurons within ventral medulla of brain:
and vocalization and enhances venous return. See earlier (1) Decreased levels of O2 are sensed by ­carotid
discussion on the respiratory system. bodies.
• See Chapters 6, General and Oral Pathology: respira- (2) Increased levels of hydrogen ions are sensed
tory disorders; 16, Special Needs Patient Care: respira- by carotid bodies.
tory disabilities; 14, Pain Management: nitrous oxide (3) Within airways and lung, receptors are sensi-
administration. tive to stretch, congestion (e.g., pneumonia,
A. Functions of the respiratory system: pulmonary emboli), irritation (e.g., smoke).
1. Gas exchange of oxygen (O2) and carbon dioxide c. Ventilation increases with:
(CO2): (1) Increased metabolic demands, e.g., exer-
a. Exchange with environment in the lungs (venti- cise.
lation). (2) Use of stimulants, e.g., caffeine and proges-
b. Exchange between lungs, circulatory system, terone.
and cells (diffusion and transport). (3) Decreased levels of O2 found at high alti-
c. Exchange in cells involves use of O2 and pro- tudes and with cardiopulmonary disease
duction of CO2 (respiration). (COPD).
2. Acid-base balance with maintenance of blood pH (4) Acidosis with uncontrolled DM, results in
at ~7.4: ketone body formation.
a. Rapid response system that alters plasma levels d. Ventilation decreases with:
of CO2. (1) Metabolic alkalosis.
b. Works in conjunction with other buffering sys- (2) Overdose of some drugs (e.g., morphine,
tems, including blood, kidneys, bones, other heroin).
cells. (3) Respiratory muscle failure.
3. Regulation of airflow involved in vocalization. (4) Use of supplemental O2 by patients with
4. Metabolism and elimination of substances via the COPD; low levels of O2 provide ONLY
pulmonary circulation (e.g., conversion of AI to AII). chemical drive to breathing, whereas in
64 Saunders Review of Dental Hygiene

healthy people the major drive is elevations


+60
in CO2 levels.
+50
2. Lung mechanics and volumes:
+40
a. Compliance: measure of distensibility of lung. +30

Membrane potential (mv)


(1) Increases in emphysema (type of COPD)

Na+ conductance
+20

K+ conductance
because of destruction of connective tissue. +10
(2) Decreases in pneumonia (increased lung 0
water) and in diseases that decrease levels -10
of lung surfactant (e.g., adult respiratory -20
distress syndrome [ARDS] and cystic fibro- -30
sis [CF]). -40
b. Resistance: measure of the ease with which air -50
-60 Threshold
moves in or out of the lung, related to radius of
-70
airways; small decreases in radius have large
-80
effects on airway resistance: -90 Resting potential
(1) Decreased by excess mucus (bronchitis),
airway edema, tumors within or around 1 msec Time
­airways. Figure 3-23  Lung volumes and capacities in a normal
(2) Increased by agents that cause contraction adult. Residual volume and any capacities containing resid-
of airway smooth muscles (e.g., acetylcho- ual volume cannot be determined using a spirometer.
line, histamine, leukotrienes).
(3) Changes in mechanics, such as decreased
compliance (lung fibrosis with emphysema
or lung cancer) or increased resistance c. Affected by thickness of alveolar-capillary bar-
(asthma), can increase the work of breath- rier; ­increases during lung edema.
ing, resulting in breathlessness even when 5. Regulation of acid-base balance by lungs occurs
performing minor tasks. by increasing or decreasing ventilation of CO2:
(4) Lung volumes: influenced by lung and a. If amount of CO2 produced by body is equal to
chest wall mechanics (Figure 3-23): ventilation, pH remains constant (7.4).
(a) Smaller: pulmonary fibrosis (possibly b. If amount of CO2 produced by body is greater
an aftereffect of infection, with added than ventilation, pH decreases and acidosis re-
factors such as smoking, environmental sults (pH <7.35).
pollutants). c. If ventilation is greater than amount of CO2
(b) Larger: hyperinflation caused by em- produced by body, pH increases and alkalosis
physema. results (pH >7.45).
3. Handling of O2 and CO2 by the body: d. Hydrogen ions are produced by metabolism and
a. Major sources of O2 are lungs and blood during periods of low O2 and buffered by bicar-
(~8 L), transported in dissolved state, bound to bonate ions (HCO3−) produced from reaction of
hemoglobin or myoglobin. CO2 and H2O (see above).
b. The CO2 is produced by aerobic metabolism, e. Resulting decrease in bicarbonate ions can be
transported in blood, some dissolved, with addressed by the kidneys ONLY.
­majority transported as bicarbonate; at level 6. Decreased delivery to or use of O2 occurs when:
of the lung, this reaction is reversed and CO2 is a. Alveolar po2 is decreased (hypoxic hypoxia).
released by lungs. (1) In high altitudes (decreased barometric
4. Diffusion of gases across lungs: pressure).
a. Occurs in areas that both are ventilated and per- (2) During lung disease and respiratory failure.
fuse with blood. b. Amount of functional hemoglobin decreases
b. Depends on solubility of gas in plasma (anemic hypoxia):
­membranes; CO2 is 20 times MORE soluble (1) In anemia caused by decreased RBC pro-
than O2. duction, abnormal forms of hemoglobin
(1) Arterial blood: partial pressure (“P,” ten- (sickle cell anemia), or blood loss.
sion) of O2 is 100 mm Hg; of CO2 is 40 mm (2) In carbon monoxide (CO) poisoning, pre-
Hg. vents hemoglobin from binding to O2 (CO
(2) Mixed venous blood: pCO2 45 mm Hg; pO2 binds to hemoglobin 150 times greater than
40 mm Hg. does O2).
Anatomy, Biochemistry, and Physiology 65

c. Poisoning of enzymes involved in using O2 to d. Blood flow to the tissues decreases (ischemia or
form ATP (histotoxic hypoxia): stagnant hypoxia) as a result of:
(1)   CO. (1)   Arteriosclerosis.
(2) Cyanide. (2)   Congestive heart failure (CHF).
(3) Stroke (cerebrovascular accident [CVA]).
(4) Effect of DM on blood vessels.

clinical study  

Age 14 YRS SCENARIO

Sex ☒  Male   ☐  Female Patient’s mother is calling to cancel his recall


appointment. He has developed an upper respira-
Height 4’3” tory infection (URI) with symptoms of cough, sore
throat, fever, and dyspnea (difficulty breathing).
Weight 80 LBS
Now she says that his end tidal pco2 has increased
Chief Complaint “He can’t make his dental appoint- from 45 to 55 mm Hg. His maximal inspiratory
ment since he is so sick today.” pressure (MIP) and maximal expiratory pressures
(MEP) dropped from 30 cm H2O to 10 cm H2O; vital
Medical History Duchenne muscular dystrophy (MD)
capacity also dropped. He has had this situation
Slight cognitive disability
Wheelchair bound before, and they anticipate that it should clear up
in 2 weeks’ time.
Current Medications prednisone (Deltasone) 15 mg qd
deflazacort (Calcort) 6 mg qd

Social History Grade school student at a special


school

1. Why would the pco2 increase in the patient? 3. Resolution of increased pco2 and decreased respira-
2. What could be the reason for decreased respiratory tory muscle strength occurs as URI clears up and fac-
muscle strength exemplified by decrease in MIP, MEP, tors mentioned above disappear.
and vital capacity? 4. Superior cervical nodes would be involved because of
3. The decrements in lung function were reversible. What drainage from infection of pharynx (throat), ­especially
does this say about the effects of the URI in the patient? jugulodigastric (tonsillar or sentinel node), below pos-
4. What lymph nodes can be affected by his sore throat? terior belly of digastric muscle. Hypothalamus is body’s
Which organ is responsible for his fever, and what thermostat and regulates temperature; can produce a fe-
does it mean? ver, in which body temperature is elevated by at least
5. What type of MD is this? How wide does the entry full degree; occurs with infection, acute inflammation,
to the dental operatory have to be for him to get his shock, and/or changes in temperature exposure.
wheelchair through when he is again able to visit the 5. Duchenne MD is a form characterized by decreasing
dental office? muscle mass and progressive loss of muscle function
in male children; caused by a mutation in a specific
gene within the X chromosome. The patient takes
1. Increase in pco2 indicates alveolar hypoventilation, the corticosteroid medications to increase energy
which may be associated with marked respiratory and strength and defer severity of some symptoms.
weakness (normal values of MIP and MEP would be Minimum 32-inch door opening is needed to enter the
more than double the pre-URI values in this case) and dental operatory, with at least 18 inches of clear wall
increased airway resistance (decreased airway diam- space on pull side of door next to handle. The door
eter) caused by inflammation and edema. Increased handle must be no higher than 48 inches above the
pco2 may also occur because of increased metabo- floor and of a lever type that can be operated with a
lism (fever) and an inability of the weakened ventila- closed fist.
tory muscles to compensate.
2. Marked decrease in respiratory muscle strength during Renal Physiology
URI may be caused by release of cytokines, muscle Renal physiology is the study of the urinary system; con-
injury from free radical production, or anorexia; all sists of kidneys, ureter, bladder, urethra. Kidneys selec-
­affect respiratory muscle performance. tively excrete substances, reabsorb nutrients (via active
66 Saunders Review of Dental Hygiene

transport) and water, affect blood pressure, influence acid- 2. Processes performed by nephron:
base balance. See earlier discussion on the urinary system. a. Glomerular filtration rate (GFR) involves indis-
• See Chapter 14, Pain Management: metabolism of criminate filtration of plasma; depends on:
drugs in kidneys. (1) Blood pressure. See later discussion.
A. Major functions of the kidneys: (2) Leakiness of glomerular (fenestrated) capil-
1. Conservation of water, electrolytes (especially laries and Bowman’s capsule.
­sodium), and nutrients. (3) Afferent versus efferent vasoconstric-
2. Regulation of acid-base balance in conjunction tion: afferent leads to decreased GFR and
with other systems (lungs, blood, bone). ­decreased perfusion of the rest of nephron;
3. Secretion of erythropoietin (hormone that stimu- efferent leads to increased GFR and in-
lates production of RBCs). creased perfusion of the rest of nephron.
4. Elimination of waste and foreign products (includ- (4) Tubular reabsorption: selective, high-vol-
ing excretion of local anesthetic agents, fluoride). ume process dependent on:
5. Regulation of plasma volume, which affects blood (a) Energy: Na+ reabsorption (water and Cl−
pressure. follow passively; glucose, phosphate,
6. Secretion of renin, enzyme that produces AI from amino acids are transported secondarily).
angiotensinogen, resulting in peripheral vasocon- (b) Concentration of substances: normally
striction. higher in proximal tubule than in peri-
7. Production of active form of vitamin D involved in tubular vessels.
calcium handling by body. (c) Hormones: aldosterone (for sodium),
B. Nephron is functional unit of kidney: shorter nephrons ADH (for water), atrial natriuretic pep-
are called cortical nephrons; longer nephrons involved tide (for both sodium and water).
in concentrating urine are juxtamedullary nephrons. (d) Tubular maximum for the substance:
1. Basic components of nephrons (structures listed in once this is reached, substance is
series): “dumped” into urine (e.g., higher
a. Vascular components: plasma glucose levels in DM result in
(1) Afferent arteriole: delivers blood to nephron. glucose in urine; ALL glucose in neph-
(2) Glomerulus: dense capillary bed formed from ron is reabsorbed back into the blood).
arteriole that acts to filter plasma, allowing (5) Tubular secretion results in movement of
ALL substances in the blood ­EXCEPT pro- substances from peritubular capillaries into
teins and cells to enter nephron. nephron for excretion.
(3) Efferent arteriole: carries blood from (a) Secretion of hydrogen and bicarbonate
­glomerulus. ions depends on acid-base status.
(4) Peritubular capillaries: formed from efferent (b) Potassium secretion is dependent on
arteriole and supply blood to rest of nephron. amount of aldosterone present.
(5) Juxtamedullary apparatus (JMA): vascular (c) Organic ions and cations (especially
and nephron tubular components, macula some drugs and endogenous substances
densa, juxtaglomerular cells of afferent such as histamine, epinephrine, creati-
arteriole; produces chemicals that affect nine) are secreted.
blood flow to kidneys. (6) Plasma clearance and urinary excretion:
b. Tubular components: (a) Clearance of substance by kidneys re-
(1) Bowman’s capsule: composed of two epithe- fers to volume of plasma that is com-
lial layers (visceral and parietal) that enable fil- pletely cleared of substance per unit
tration of plasma. Filtered plasma enters here. of time.
(2) Proximal convoluted tubule (PCT): reab- (b) Inulin is filtered, NOT reabsorbed or
sorbs MOST filtered constituents; secretion secreted; completely cleared by kidney,
of H+ and organic ions may occur. measured clinically to determine GFR.
(3) Loop of Henle: concentrates urine in con- (c) Glucose in healthy person is filtered but
junction with distal convoluted tubule completely reabsorbed; therefore NO
(DCT) and collecting duct; loops are espe- glucose is cleared by the kidneys.
cially long in medullary nephrons. 3. Excretion is the measure of the amount of urine
(4) DCT and collecting duct: variable control output per unit of time:
(by means of hormones such as aldosterone, a. Difference between what is filtered and what is
ADH, atrial natriuretic peptide) of sodium, reabsorbed plus what is secreted; normal excre-
potassium, hydrogen ions, water. tion rates are 1 mL/min.
Anatomy, Biochemistry, and Physiology 67

b. Excretion rates depend on the concentration of d. The SV is influenced by:


the urine, modulated by osmolarity of plasma (1) Amount of blood returning to the heart (ve-
and hormonal status; rates decreased during de- nous return), increased by an increase in
hydration, decreased blood pressure, renal dis- blood volume, decreased right atrial pres-
ease. sure, skeletal muscle contractions, increas-
c. Acid-base handling by the kidney involves re- ing venous tone.
absorption or secretion of H+ and HCO3−; if (2) Contractility of heart muscle is increased
disturbance is caused by kidney or metabolic by the SAME factors that increase HR.
source, respiratory system tries to compensate: (3) Condition of the heart; as heart fails, CO
(1) Healthy: metabolically produced acids are decreases.
buffered by HCO3−, replaced by the kidneys. 2. The TPR is influenced by:
(2) Metabolic acidosis: H+ is secreted and a. Diameter of blood vessels, especially “resis-
HCO3− is reabsorbed; increase in ventila- tance” vessels and arterioles that have large
tion eliminates CO2 produced by combin- amount of smooth muscle; diameter is affected
ing the excess H+ with HCO3−. by:
(3) Respiratory acidosis (hypoventilation): HCO3− (1) SANS stimulation, leads to constriction.
and H+ are generated from CO2 and H2O; H+ (2) Local conditions, e.g., decreased O2 and in-
is secreted and HCO3− is reabsorbed. creased metabolites, cytokines, adenosine,
(4) Respiratory alkalosis (hyperventilation): and histamine increase dilation.
excess HCO3− is secreted by the kidneys (3) Circulating hormones, ADH and angioten-
and H+ is retained; CO2 levels in blood are sin, are constrictors and atrial natriuretic
LOWER. peptide is a dilator.
(5) Metabolic alkalosis: handled by the kidney b. Blood viscosity affected by:
in a similar manner to respiratory alkalosis, (1) Dehydration; leads to increased viscosity
but metabolic alkalosis depresses ventila- and resistance.
tion and CO2 builds up in the plasma to (2) Number of RBCs; viscosity is increased
compensate. in polycythemia (increased RBCs) and de-
creased in anemia (decreased RBCs).
Blood Pressure (3) Pancytopenia: decreased number of total
Blood pressure is the force that drives blood through circulating blood cells.
vessels to carry nutrients and oxygen to tissues and waste C. Short-term regulation of blood pressure:
products and metabolized substances away from tissues. 1. The CNS regulates blood pressure for the short term.
Mean arterial pressure (MAP) is a term used to describe a. Pressure and volume sensors are located in:
an average blood pressure. (1) Carotid sinus and aortic arch (pressure).
• See Chapters 6, General and Oral Pathology: high blood (2) Right atrium and lungs (volume).
pressure; 11, Medical and Dental Emergencies: blood 2. Afferent pathways to the CNS include medulla
pressure readings, classifications, related emergencies. oblongata, hypothalamus, cortex.
A. MAP (cardiac output [CO] × total peripheral resis- 3. CNS responses to decreased blood pressure
tance of blood vessels [TPR]). include:
B. Blood pressure medications act on one of these three a. Increased SANS outflow.
variables; factors that affect MAP: b. Release of hormones such as ADH from the
1. The CO is the amount of blood pumped by the pituitary.
heart per minute: c. Stimulation of renin release by the kidneys.
a. Equal to the heart rate (HR) multiplied by the 4. Systemic consequences include:
stroke volume (SV) (CO = HR × SV). a. Increased CO.
b. The HR is the number of beats per minute and b. Increased TPR.
the SV is the volume pumped per beat (e.g., 70 c. Production of AII and aldosterone.
beats per minute [HR], 70 mL blood is ejected D. Long-term regulation of blood pressure:
with each beat [SR]). 1. Influenced by the kidneys; retain and/or excrete
c. The HR: sodium and water.
(1) Increased by sympathetic division (SANS) 2. Involves measurement of MAP by means of:
stimulation and release of norepinephrine a. Direct use of indwelling catheter (thank good-
and epinephrine. ness we do not have to do this to our patients!).
(2) Decreased by parasympathetic division b. Indirect use of pressure cuff, sphygmomanom-
(PANS) stimulation. eter, stethoscope:
68 Saunders Review of Dental Hygiene

(1) Systolic: 120 mm Hg in a resting, healthy 4. Consequences of HBP include:


adult. a. Increased work by the heart; may lead to cardiac
(2) Diastolic: 80 mm Hg in a resting, health failure and then heart attack (myocardial infarc-
adult. tion [MI]).
(3)  MAP = DP + 1⁄3 (SP – DP). b. Stresses on blood vessels; may lead to athero-
E. Blood pressure abnormalities according to the Joint sclerosis and then stroke (cerebrovascular ac-
National Committee 7 on Prevention, Detection, cident [CVA]).
Evaluation, and Treatment of High Blood Pressure 5. Hypotension (low blood pressure) caused by:
(see CD-ROM for guidelines): a. Cardiovascular disease (CVD) that leads to de-
1. Prehypertension: 120/80 to 139/89 mm Hg; life- creased CO.
style changes are needed to prevent hyperten- b. Decreased SANS function or β-adrenergic re-
sion. ceptor blockade (drugs, such as propranolol
2. Hypertension (high blood pressure [HBP]): with [Inderal]).
systolic/diastolic of >140/>90 mm Hg; can involve c. Shock related to:
systolic and/or diastolic. (1) Blood loss (such as by hemorrhage [exces-
3. May be caused or increased by: sive bleeding]), termed hypovolemic shock.
a. Excess fluid retention cause by hormonal abnor- (2) Septic (bacterial) infections.
malities (e.g., increased levels of aldosterone). (3) Neurogenic causes (fainting [syncope]).
b. Stress and obesity. (4) Anaphylactic reactions (e.g., allergy to bee
c. Renal disease and DM. stings results in the release of mediators
d. Aging (may instead lead to hypotension). such as histamine from mast cells).
clinical study  

Age 66 YRS SCENARIO

Sex ☒  Male   ☐  Female The patient was admitted to the hospital while
visiting his dental office. The dental staff sent him
Height 5’11” by ambulance, since he had a sudden onset of
pain ­affecting his left arm right before his dental
Weight 260 LBS ­appointment. Patient had stopped taking his pre-
scribed daily medication, since he did not like the
BP 210/120 side effect of coughing.
Chief Complaint “My wife could not get me out of bed
this morning.”

Medical History Essential hypertension


Angina pectoris

Current Medications nitroglycerin, sl prn


captopril (Capoten) 25 mg bid

Social History Retired bar owner who used to smoke

1. What does a blood pressure reading mean? What is an average blood pressure (heart rate × stroke volume ×
MAP? What organ maintains the long-term regulation total peripheral resistance), with a systolic pressure
of blood pressure? of <120 mm Hg and a diastolic pressure of <80 mm
2. What level of high blood pressure is the patient expe- Hg in a resting, healthy adult. Long-term regulation of
riencing? blood pressure is influenced by the kidneys, which ei-
3. What may be happening to the patient because of his ther retain or excrete sodium and water.
high blood pressure? Did the dental office need to 2. Patient has a history of hypertension (high blood
have him sent to the hospital? ­pressure [HBP]), with past readings of systolic/dia-
stolic >140/90 mm Hg. With this latest reading, he is
at stage 2, severe HBP (≥180/≥110 mm Hg), so the
1. Blood pressure is force that drives blood through ves- dental office needs to activate the emergency medical
sels to carry nutrients and oxygen to tissues, and waste services (EMS) system and treat as emergency.
products and metabolized substances away from tissues. 3. Patient could again be having angina pectoris, chest
Mean arterial pressure (MAP) is a term used to describe pain caused by decreased oxygen flow to the heart,
Anatomy, Biochemistry, and Physiology 69

or could be having an MI, since pain can spread from residues, fungi, parasites, viruses (also in
chest to other areas. The pain is of greater intensity and tears, sweat, gut fluid).
duration than angina pectoris, and the pain he is feel- (4) Histatins function as antifungal.
ing in left arm could be an indicator of MI. Consider- b. Salivary amylase: enzyme that begins to
ing the symptoms, the dental office acted correctly digest starches in the oral cavity (such as
by sending him to hospital by ambulance for emer- ­carbohydrates; see earlier discussion) to dex-
gency care. A delay of 4 to 6 weeks after this cardiac tran and maltose (disaccharide).
episode, as well as follow-up medical consult, will be 3. Pharynx allows movement of gases into the lungs and
needed before dental care (either emergency or elec- chewed food into esophagus; during swallowing, glot-
tive), and patient must meet the 4 METs to determine tis closes to prevent foodstuff from entering lungs.
his FC levels to reduce future cardiac risk during dental 4. Esophagus is tubelike and consists of both smooth
treatment. and skeletal muscle:
a. Moves chewed food from the mouth toward the
Gastrointestinal Tract Physiology stomach by peristaltic contractions.
Gastrointestinal (alimentary) tract (GIT) consists of the b. Mucus produced by cells lining the lumen is
oral cavity, esophagus, stomach, small and large intes- protective and helps movement of food.
tines. MAINLY responsible for the digestion of food into c. After the gastroesophageal sphincter relaxes,
MORE usable forms of nutrients for the cells of the body. food enters the stomach.
See earlier discussion about GIT. 5. Stomach functions:
• See Chapter 4, Head, Neck, and Dental Anatomy: oral a. Storage of ingested food until it is emptied into
cavity structure. the small intestine.
A. Functions of GIT: b. Secretion of HCl and enzymes that initiate di-
1. Digestion of food into nutrients that can be ab- gestion.
sorbed and used by cells for energy, growth, repair, c. Pulverization and mixing of food to produce
heat production, differentiation: chyme (thick liquid mixture).
a. Motility: movement and mixing of food along d. Major secretions of stomach (and locations):
GIT. (1) Oxyntic mucosa (fundus and body):
b. Secretion: production and release of enzymes, (a) Mucous neck cells produce thin mucus.
hormones, fluids, mucus, or electrolytes that aid (b) Gastrin secreted by the duodenum initi-
in digestion. ates gastric acid production.
c. Digestion: breakdown of ingested food into nu- (c) Chief cells produce pepsinogen, which
trients that may be used by the body. can be converted to enzyme pepsin un-
d. Absorption: uptake of digested materials across der influence of chloride to initiate pro-
GIT into blood and lymphatics for delivery to cells. tein digestion.
2. Defense against ingested substances that may be (d) Parietal (oxyntic) cells produce HCl
harmful to the body. and intrinsic factor, which help absorb
B. Anatomical structures and functions of digestion: vitamin B12 in ileum.
1. Oral cavity is the site for the ingestion of foods: (e) Epithelial cells secrete thick mucus to
a. Tongue mixes food with saliva and contains protect stomach from autodigestion.
taste buds; fluid in saliva helps dissolve foods (2) Pyloric area contains G cells that secret gas-
to be “tasted.” trin, hormone that stimulates parietal and
b. Teeth chew food into more manageable pieces, chief cells.
allowing better mixing of food with saliva, e. Emptying of chyme from stomach:
which acts as a lubricant. (1) Depends on volume and fluidity of chyme;
2. Salivary glands and accessory digestive glands initiates gastrin production and reflexes (by
produce secretions: way of vagus nerve and ­ intrinsic nervous
a. Saliva (fluid, mucus, lysozyme, bicarbonate): system) that stimulate antrum ­ contraction,
(1) Production is tonically stimulated by the leading to stomach emptying.
PANS, by smell or sight of food or antici- (2) Presence of acid and/or fat in duodenum
patory reflex. initiates enterogastric reflex and release of
(2) Fluid and lysozymes have antibacterial ac- secretin, cholecystokinin, and gastric in-
tions, as well as helping to reduce acids to hibitory peptides that prevent emptying.
help prevent caries. (3) Vomiting (emesis) is caused by active con-
(3) Contains sIgA (secretory), which provides traction of respiratory muscles that act on
immune resistance against bacteria, food stomach emptying.
70 Saunders Review of Dental Hygiene

f. Gastric secretion is stimulated during phases: (2) Absorption of vitamin B12 and bile salts oc-
(1) Cephalic phase: smell or thought of food curs in terminal ileum.
leads to secretion of HCl, gastrin, pepsinogen. (3) Absorption of calcium and iron depends on
(2) Gastric phase: large volume of food, caf- body’s needs.
feine, or alcohol leads to gastrin and HCl 8. Processing of substances NOT absorbed in small
secretion. intestine by large intestine (colon, cecum, appen-
(3) Intestinal phase: chyme enters duodenum, lead- dix, rectum) includes electrolytes and water.
ing to decreased gastrin and HCl secretion.
g. Stomach absorbs fluoride at about 86% to 97%.
6. Functions of pancreas and liver (accessory organs): Review Questions
a. Pancreas:
(1) Produces enzymes (e.g., trypsin [activated
by enterokinase], chymotrypsin, amylase,   1 Some cells that line the respiratory tract produce mucus,
lipase) and an alkaline watery solution, se- which traps inhaled particles and subsequently is propelled
creted into small intestine; lipase acts on upward for expectoration. This upward movement is pro-
fats to become fatty acids and glycerol. duced by the action of
(2) Endocrine organ that produces insulin, glu- A. ribosomes.
B. cilia.
cagon, somatostatin (see section on endo-
C. flagella.
crinology). D. centrioles.
b. Liver produces bile, cholesterol, lecithin, which   2 One of the following is NOT a type of connective tissue.
facilitate absorption of fats; bile produced by Which one is the EXCEPTION?
the liver is stored in gallbladder. A. Blood
c. Additional functions of liver NOT associated B. Bone
with digestion include: C. Cartilage
(1) Detoxification of waste products and for- D. Muscle
eign substances.   3 Which of the following describes the type of membrane lin-
(2) Storage of glycogen. ing the respiratory tract?
(3) Production of fibrinogen and prothrombin, A. Mucous
B. Cutaneous
involved in blood clotting.
C. Serous
(4) Synthesis of amino acids and proteins. D. Parietal
(5) Production of RBCs in fetus and destruc-   4 An abrasion on the surface of the skin heals when new cells
tion of RBCs in adult. form to replace the damaged cells. The layer of skin that
(6) Immune surveillance via Kupffer cells (re- produces the replacement cells is the stratum
ticuloendothelial cells). A. lucidum.
d. All blood that leaves the gut passes through liver B. corneum.
by way of portal vein, allowing additional pro- C. granulosum.
cessing of nutrients and blood detoxification. D. basale.
7. Role of small intestine in digestion and absorption:   5 A fracture of the radius is also a fracture of the
a. Segmentation, results in mixing and moving of A. axial skeleton.
B. medial side of the forearm.
chyme:
C. brachium or arm.
(1) Initiated by pacemaker cells in GIT. D. lateral side of the forearm.
(2) Responds to presence of chyme in stomach    6 Diarthrotic joints (diarthroses)
and production of gastrin by duodenum. A. have a synovial membrane lining the fibrous joint capsule.
(3) Stimulated by PANS and inhibited by B. are immovable joints.
SANS. C. include the intervertebral discs.
b. Digestion of chyme occurs by: D. are fibrous joints.
(1) Pancreatic enzymes released into the small    7 One of the following muscles is NOT associated with the
intestine. lower extremity. Which one is the EXCEPTION?
(2) Intracellular enzymes. A. Adductor longus
(3) Crypts of Lieberkühn (between villi), which B. Sternocleidomastoid
C. Biceps femoris
secrete water and electrolytes, constantly
D. Gastrocnemius
replace epithelial cells that secrete mucus.
c. Small intestine has large surface area for ab-
sorption (presence of villi and microvilli):
(1) MOST occurs in duodenum and jejunum.
Anatomy, Biochemistry, and Physiology 71

  8 One of the following is NOT a part of the central nervous 18 During which of the following situations can an Rh factor
system. Which one is the EXCEPTION? reaction occur?
A. Cranial nerves A. Anti-Rh agglutinins of the fetus come in contact with
B. Cerebrum agglutinogens from an Rh– mother.
C. Cerebellum B. Rh+ agglutinogens of the fetus come into contact with
D. Spinal cord anti-Rh agglutinins from an Rh– mother.
  9 Which of the following statements is related to the function C. Rh+ agglutinogens of the fetus come into contact with
of the cerebellum? Rh– agglutinogens from an Rh– mother.
A. Contains centers that help regulate breathing move- D. Anti-Rh agglutinins of the fetus come into contact with
ments anti-Rh agglutinins from an Rh– mother.
B. Regulates heart rate and blood pressure 19 Which of the following describes the blood in the hepatic
C. Is important in maintaining posture and equilibrium portal vein?
D. Contains centers for hearing and vision A. Lower oxygen content than blood in the hepatic artery
10 The afferent division of the peripheral nervous system B. Higher oxygen content than blood in the descending aorta
A. consists entirely of cranial nerves. C. Higher oxygen content than blood in the left atrium
B. consists entirely of spinal nerves. D. Lower glucose content than blood in the hepatic artery
C. carries sensory impulses to the central nervous system. 20 Into where does the lymph from the right inguinal lymph
D. carries motor impulses from the central nervous system nodes flow?
to the effectors. A. Thoracic duct, then into the right subclavian vein
11 One of the following descriptors is NOT true about the au- B. Thoracic duct, then into the left subclavian vein
tonomic nervous system. Which one is the EXCEPTION? C. Right lymphatic duct, then into the right subclavian
A. Visceral efferent system vein
B. Divided into sympathetic and parasympathetic divi- D. Right lymphatic duct, then into the left subclavian vein
sions 21 The MOST diffusion of oxygen and carbon dioxide occurs
C. System effectors are skeletal muscles across the walls of the
D. Part of the peripheral nervous system A. trachea.
12 Which of the following is CORRECT about taste and B. bronchi.
smell? C. alveoli.
A. General senses D. hypopharynx.
B. Localized and closely related 22 Which of the following is one of the differences between
C. Type of proprioception the right lung and the left lung?
D. Detected by Meissner’s and pacinian corpuscles A. Left lung has a greater volume because it is longer than
13 Where are the rods and cones of the eye located? the right lung.
A. Innermost eye layer B. Right lung is covered by a serous membrane but the left
B. Cornea lung is not.
C. Ciliary body C. Right lung has an indentation for the base of the heart
D. Lens but the left lung does not.
14 The organ of Corti D. Right lung has three lobes but the left lung has two.
A. is located in the middle ear. 23 One of the following is NOT secreted by the mucosal cells
B. contains the malleus, incus, and stapes. of the stomach. Which one is the EXCEPTION?
C. functions to provide equilibrium. A. Hydrochloric acid
D. contains the receptors for hearing. B. Gastrin
15 Which of the following describes the hormone aldoste- C. Pepsinogen
rone? D. Amylase
A. Reacts with receptors on cell membrane surface 24 Cells of the small intestine secrete both enzymes and hor-
B. Considered sex hormones in the body mones. Secretin is a hormone produced by the small intes-
C. Produced by the adenohypophysis tine that stimulates
D. Derivative of cholesterol A. contraction of the gallbladder.
16 Which of the formed elements of the blood contain hemo- B. production of pancreatic enzymes.
globin and transport oxygen? C. secretion of an alkaline fluid from the pancreas.
A. Thrombocytes D. secretion of hydrochloric acid from the stomach.
B. Leukocytes 25 One of the following statements is NOT true about the liver.
C. Monocytes Which one is the EXCEPTION?
D. Erythrocytes A. Blood is drained from the liver by the hepatic portal vein.
17 Which of the following transfusions can be given to a per- B. Liver produces bile for digestion.
son with type A blood? C. Liver is located in the upper right quadrant of the
A. Type A or type AB ­abdomen.
B. Type A or type O D. Oxygenated blood is brought to the liver by the hepatic
C. Type B or type AB artery.
D. Type AB or type O
72 Saunders Review of Dental Hygiene

26 Which of the following sequences BEST describes the di- 35 Which of the following can be used to describe the auto-
rectional flow of urine to outside the body? nomic nervous system?
A. Urinary bladder, urethra, ureter, kidney A. Not part of the peripheral nervous system
B. Kidney, urethra, urinary bladder, ureter B. Innervates only smooth and cardiac muscles
C. Kidney, ureter, urinary bladder, urethra C. Consists of the sympathetic and parasympathetic ner-
D. Urinary bladder, ureter, urethra, kidney vous systems
27 If a male has had mumps, what side effect is possible? D. Innervates skeletal muscles
A. Emesis E. Uses the neurotransmitter glutamate
B. Orchitis 36 All of the following are true of skeletal muscles, EXCEPT
C. Inflamed prostate gland one. Which one is the EXCEPTION?
D. Fluid from the penis A. Contain actin and myosin microfilaments
28 What is the basic structure of the helix of DNA? B. Require external Ca+2 for contraction
A. Single C. Use acetylcholine as the neurotransmitter
B. Double D. Contain an extensive sarcoplasmic reticulum
C. Triple E. Need energy for both contraction and relaxation
D. Quadratic 37 Which of the following statements is CORRECT concern-
29 All the following are ways that RNA is dissimilar to DNA, ing cardiac muscles?
EXCEPT one. Which one is the EXCEPTION? A. Need no external Ca+2 for contraction
A. Formed as single stranded B. Have action potentials that last more than 200 msec
B. Contains uridine monophosphate C. Contain few mitochondria
C. Assists in protein translation D. Contract in response to acetylcholine
D. Involved in transcription E. Exhibit summation
30 What is the process of breaking large, complex molecules 38 How can hormones be divided into classes of chemicals?
into smaller, simpler units called? A. Amines, peptides, and steroids
A. Anabolism B. Free fatty acids, peptides, and amines
B. Catabolism C. Steroids, free fatty acids, and phospholipids
C. Electron transport D. Amines, phospholipids, and steroids
D. Oxidative phosphorylation 39 Which of the following is related to the role of parathyroid
31 The products of aerobic metabolism include all of the fol- hormone in the body?
lowing, EXCEPT one. Which one is the EXCEPTION? A. Decreases Ca+2 plasma levels in response to hypocalce-
A. H2O mia
B. Heat B. Acts on the kidney to conserve Ca+2
C. Lactic acid C. Acts with vitamin D to decrease intestinal absorption
D. ATP of Ca+2
E. CO2 D. Affects only phosphate metabolism in muscle
32 According to Fick’s law of diffusion, the greatest net move- 40 Which of the following is the role of the pancreas in the
ment of a substance across a membrane will occur if the body?
A. concentration gradient is small. A. Only an endocrine gland within the body
B. thickness of the membrane is great. B. Produces glucagon, which decreases glucose levels
C. temperature is low. C. Produces insulin, which promotes cellular uptake of
D. available surface area is large. plasma glucose
E. membrane permeability for a substance is low. D. Secretes hormones that break down carbohydrates and
33 Which of the following statements can describe the process proteins
of osmosis? 41 Which of the following describes the hormone aldoste-
A. Associated with movement of H2O across a membrane rone?
B. Unrelated to membrane permeability A. Produced in the adrenal medulla
C. Associated primarily with movement of solutes across a B. Stimulated by increased Na+ plasma levels
membrane C. Regulation of blood volume by means of effects on the
D. Unrelated to the concentration gradient kidneys
34 All are characteristics of carrier-mediated transport EX- D. Stimulated by a rise in blood pressure or a fall in K+
CEPT one. Which one is the EXCEPTION? plasma levels
A. Movement of substances across membrane E. Stimulated by high estrogen levels only
B. Specificity 42 Glucocorticoids such as cortisol have all of the following
C. Saturation or transport maximum characteristics, EXCEPT one. Which one is the EXCEP-
D. Competition TION?
E. Requirement of energy (ATP) A. Promote muscle breakdown
B. Promote gluconeogenesis
C. Involved in stress responses
D. Stimulate the immune system
E. Are regulated by ACTH
Anatomy, Biochemistry, and Physiology 73

43 Which of the following statements describes the role of tes- 52 Secretions produced by the stomach include all of the fol-
tosterone in the body? lowing, EXCEPT one. Which one is the EXCEPTION?
A. Produced by Leydig cells in testes A. Mucus
B. Stimulates production of sperm in Sertoli cells B. Hydrochloric acid (HCl)
C. Decreases red blood cell production C. Intrinsic factor
D. Decreases protein production in muscles D. Vitamin B12
44 Estrogen is a hormone that E. Pepsinogen
A. induces production of testosterone receptors in the uterus. 53 Which of the following is considered a function of the
B. is essential for follicular maturation. liver?
C. promotes oxygen production. A. Detoxification of wastes
D. decreases bone density. B. Production of red blood cells in adults
45 Hormones involved in fluid volume regulation include all C. Synthesis of steroid hormones
the following, EXCEPT one. Which one is the EXCEP- D. Production of vitamin A
TION? E. Synthesis of fats
A. Angiotensin II 54 Functions of the small intestine include all of the following,
B. Aldosterone EXCEPT one. Which one is the EXCEPTION?
C. Antidiuretic hormone A. Absorption of digested foodstuff
D. Inhibin B. Production of fluids and electrolytes
E. Atrial natriuretic peptide C. Production of enzymes released into the intestine
46 How does gas exchange occur across the lung? D. Movement of chyme through the intestine
A. Depends on alveoli that are ventilated and perfused by E. Absorption of vitamin B12 and bile salts
blood 55 Which of the following is a function of the large intestine?
B. Is similar for oxygen and carbon dioxide A. Absorbs food
C. Occurs in the bronchi B. Produces water and electrolytes
D. Depends on a large pressure gradient for CO2 C. Produces mucus for protection and lubrication
E. Occurs primarily in the pulmonary artery D. Absorbs hormones
47 Which of the following statements can be used to describe E. Produces intrinsic factor
the pleural space? 56 Major functions of the kidneys include all of the following,
A. Normally contains a large amount of fluid EXCEPT one. Which one is the EXCEPTION?
B. Contains positive (above atmospheric) pressure A. Production of erythropoietin
C. Helps attach the chest wall and lung for ventilation B. Production of renin
D. Produces surfactant C. Production of angiotensin II
48 Which of the following is a factor related to ventilation of D. Regulation of blood volume
the lungs? E. Acid-base regulation
A. Determined by the product of tidal volume and fre- 57 What can occur with a blockage of the ureter?
quency of breathing A. More pressure within kidney
B. Little influenced by the body’s O2 demands B. Increased urine excretion
C. Stimulated by high levels of CO2 C. Increased overall size of nephrons
D. Stimulated by anoxia D. Formation of prostatic hypertrophy
49 How is MOST of the carbon dioxide (CO2) transported in 58 Components of a nephron include all of the following,
blood? ­EXCEPT one. Which one is the EXCEPTION?
A. In the form of bicarbonate ions A. Glomerulus
B. Dissolved in the blood B. Juxtaglomerular apparatus
C. Bound to plasma proteins C. Bowman’s capsule
D. Bound to hemoglobin D. Urethra
E. In the form of H2CO3 E. Loop of Henle
50 Decreased access to oxygen by cells of the body can occur 59 Which of the following statements is CORRECT concern-
in all of the following conditions, EXCEPT one. Which one ing the glomerular filtration rate?
is the EXCEPTION? A. Increased by an increase of blood pressure
A. Inhalation of carbon monoxide B. Decreased by constriction of the efferent arteriole
B. Decreased hemoglobin levels C. Decreased by increasing the leakiness of the Bowman’s
C. High altitude capsule
D. Increased blood flow to the cells D. Increased by constriction of the afferent arteriole
E. Cyanide poisoning 60 Which of the following occurs during respiratory acidosis?
51 What is the role of alveolar hyperventilation in the body? A. Kidneys play no role.
A. Decreases O2 levels in blood B. Kidneys compensate by producing chloride.
B. Increases CO2 levels in blood C. Kidneys compensate by reabsorbing HCO3−.
C. Decreases the pH of blood D. Kidneys compensate by reabsorbing H+.
D. Decreases H+ in blood
74 Saunders Review of Dental Hygiene

61 Which of the following can cause a change in cardiac 69 Loss of the sense of smell could result from injury to which
­output? of the following cranial nerves?
A. Increased by an increase in contractility of the heart A. I
B. Increased when the heart rate decreases B. III
C. Decreased by an increase in venous return C. V
D. Decreased by an increase in stroke volume D. VII
62 Which of the following can occur with changes in total pe- E. IX
ripheral resistance? 70 What is the largest visceral organ of the body?
A. Increased as the diameter of blood vessels decreases A. Skin
B. Decreased by angiotensin II B. Liver
C. Independent of blood viscosity C. Stomach
D. Increased by increased production of metabolites D. Gallbladder
E. Decreased with dehydration 71 Which of the following provides energy to the brain?
63 Blood pressure or volume is sensed by receptors in all of A. Glucagon
these locations, EXCEPT one. Which one is the EXCEP- B. Glucose
TION? C. Glycogen
A. Right atrium D. Glycolic acid
B. Carotid sinus 72 Alpha and beta cells are found in which of the following
C. Carotid body organs?
D. Aortic sinus A. Pancreas
64 Which of the following can happen when there is a decrease B. Liver
in blood pressure? C. Kidney
A. Release of acetylcholine D. Adrenal glands
B. Increased sympathetic nervous system discharge 73 What is the primary form of carbohydrate storage?
C. Decreased heart rate A. Glucose
D. Decreased cardiac output B. Sucrose
65 Which of the following statements is CORRECT concern- C. Glycogen
ing hypertension? D. Starch
A. Decreases the work of the heart 74 Starch digestion is initiated in the
B. Contributes to the formation of atherosclerotic plaques A. oral cavity.
C. Affects only the pulmonary circulation B. stomach.
D. Is age independent C. small intestine.
E. Is lower in obese people D. large intestine.
66 Hypotension may occur in all the following situations, 75 Which of the following cranial nerves is also designated as
­EXCEPT one. Which one is the EXCEPTION? the trigeminal nerve?
A. Heart failure occurs. A. V
B. Rapid release of epinephrine occurs. B. VII
C. Blood is lost. C. X
D. Septic shock occurs. D. XII
E. Sympathetic nervous system–blocking drugs are
taken.
67 Functions of the plasma membrane include all of the fol-
lowing, EXCEPT one. Which one is the EXCEPTION?
Answer Key And Rationales
A. Selectively allows movement of substances into the cell
interior
B. Contains receptors that allow communication between 1 (B) ���������������������������������������������
Both cilia and flagella are involved in move-
cells ment; flagella move cells and cilia move substances
C. Synthesizes proteins across cell surface, and ciliary action moves mucus
D. Is involved in cell division with trapped particles upward for subsequent removal
E. Forms vesicles for endocytosis from the body. Ribosomes are involved in protein
68 Which of the following organelles produces ATP? synthesis, and centrioles function in cell division.
A. Mitochondria 2� (D)  Four main categories of tissue are epithelial, con-
B. Golgi apparatus nective, muscle, and nervous. Blood, bone, cartilage,
C. Peroxisomes
along with areolar tissue, adipose tissue, tendons, and
D. Lysosomes
E. Vesicles
ligaments, are examples of connective tissue.
3� (A)  Serous membranes line body cavities that do
NOT open to the outside and consist of parietal and
visceral layers. Cutaneous membrane is the skin.
Mucous membranes line body cavities that open to
Anatomy, Biochemistry, and Physiology 75

the exterior, such as the respiratory and digestive or ­ orientation, detected by Golgi tendon organs and
tracts. muscle spindles. Meissner’s and pacinian corpuscles
4� (D)  Stratum basale is the actively mitotic layer that are receptors that detect the general senses of touch
produces new cells. Other layers consist of dead or and pressure. Taste and smell are special senses that
dying cells and have no mitotic ability, with EXCEP- are localized and closely related.
TION of stratum spinosum, which has limited mitotic 13� (A)  Rods and cones are visual receptors (photorecep-
ability. tors) located in the retina, the innermost layer of the eye.
5� (D)  Radius is bone of appendicular skeleton and is Cornea is the clear portion of outermost fibrous layer.
located on lateral side of forearm. Humerus is bone Ciliary body changes shape of the lens to focus light rays
in brachium or arm. on visual receptors of the retina.
6�� (A)  Diarthrotic joints are freely movable joints 14 (D)  Middle ear contains malleus, incus, and stapes,
whose range of motion is limited by adjacent mus- three ossicles that transmit sound waves to inner ear.
cles, bones, and ligaments. Joint has a fibrous joint Receptors for hearing are located in organ of Corti,
capsule lined with a synovial membrane that secretes part of cochlea of the inner ear. Receptors for equilib-
synovial fluid into the joint cavity for lubrication. Im- rium are located in vestibule and semicircular canals
movable joints are synarthroses. Intervertebral discs of inner ear.
are located between vertebral bodies to form slightly 15 (D)  Aldosterone is a steroid hormone that helps reg-
movable joints or amphiarthroses. ulate fluid and electrolyte balance and is produced by
7 (B)  Adductor longus, biceps femoris, and gastrocne- adrenal cortex. Steroids are derivatives of cholesterol
mius are associated with the lower extremity. Adduc- and react with receptors inside the cell. Protein hor-
tor longus adducts thigh at hip joint; biceps femoris mones react with receptors on cell membrane.
is one of the hamstrings on posterior side of the thigh 16 (D)  Monocytes are a type of leukocytes, cells that do
and flexes the knee; gastrocnemius is located on pos- NOT contain hemoglobin. Thrombocytes or platelets are
terior leg and plantar flexes the foot. Sternocleido- fragments of cells and function in blood clotting. Eryth-
mastoid (SCM) is a neck muscle. rocytes are red blood cells (RBCs), which contain hemo-
8�� (A)  Central nervous system (CNS) includes brain globin and transport oxygen.
and spinal cord; cerebrum and cerebellum are parts 17 (B)  Blood types are determined by the agglutinogens
of brain. Peripheral nervous system (PNS) consists on the surface of the RBC (red blood cell). Type A
of 12 pairs of cranial nerves and 31 pairs of spinal blood has A agglutinogens and anti-B agglutinins. In
nerves. transfusion reactions the recipient’s agglutinins re-
9�� (C)  Pons in the brainstem contains apneustic and act with the agglutinogens of the donor. The anti-B
pneumotaxic centers, which regulate rate and depth agglutinins of the type A blood (recipient) will react
of breathing. Heart rate and blood pressure are es- with type B agglutinogens found in type B and type
tablished by vital centers in the medulla oblongata. AB blood; therefore these two blood types CANNOT
Auditory cortex is in temporal lobe, and visual cortex be used as donors. Type O blood has no agglutino-
is in occipital lobe; both are in cerebrum. Cerebellum gens on the RBC surface and is considered the uni-
is motor area that coordinates skeletal muscle activity versal donor.
and is important in maintaining muscle tone, posture, 18 (B)  Under normal conditions, no anti-Rh agglutinins
and equilibrium. are present in either Rh+ or Rh– blood on the red
10�� (C)  Afferent division of peripheral nervous system blood cells (RBCs). An Rh– person who comes into
(PNS) carries sensory impulses to the central ner- contact with Rh+ blood, however, develops these ag-
vous system, and the efferent division carries mo- glutinins. If such a woman becomes pregnant with an
tor impulses from central nervous system (CNS) Rh+ fetus, some of her anti-Rh agglutinins may cross
to muscles and glands (effectors). Some, but NOT the placenta and react with the Rh+ agglutinogens of
all, cranial nerves are sensory; some are motor, and the fetus. Reaction may cause breakdown of the fetal
others have BOTH sensory and motor components. RBCs, resulting in hemolytic disease of the newborn
All spinal nerves have BOTH sensory and motor (erythroblastosis fetalis).
­components. 19�� (A)  Descending aorta and left atrium contain blood
11 (C)  Autonomic nervous system (ANS) is the visceral that has been freshly oxygenated in the lungs. Venous
efferent portion of peripheral nervous system (PNS). blood, such as that in the hepatic portal vein, has lower
Includes sympathetic (SANS) and parasympathetic oxygen content. Hepatic portal vein receives blood
(PANS) divisions, and its effectors are cardiac mus- from digestive system and has higher nutrient (glucose)
cle, smooth muscle, and glands. content than other vessels.
12�� (B)  General senses are widely distributed in the 20 (B)  Right lymphatic duct collects lymph from up-
body. Proprioception is a general sense of position per right quadrant, which then drains into the right
76 Saunders Review of Dental Hygiene

subclavian vein. Lymph from the remaining three ­ nabolism is a process of building large complex
A
quarters, including inguinal lymph nodes, enters the molecules from smaller molecules. Electron trans-
thoracic duct, which carries it to the left subclavian port and oxidative phosphorylation constitute the
vein. pathway by which electrons are donated from NADH
21 (C)  Pharynx, trachea, and bronchi are conducting and FADH2 to a series of proteins, which results in
passages for air. Diffusion of gases, oxygen, and car- production of ATP.
bon dioxide occurs across the simple squamous epi- 31 (C)  Lactic acid is a product of anaerobic metabo-
thelial walls of the alveoli. lism. Constantly produced from pyruvate by way of
22 (D)  Right lung is shorter and wider and has a enzyme lactate dehydrogenase (LDH), in process of
greater volume than the left. BOTH lungs are fermentation during normal metabolism and exer-
covered by the pleura, a serous membrane. Left cise. Does NOT increase in concentration until the
lung is divided into two lobes and has an inden- rate of lactate production exceeds the rate of lactate
tation for the apex of the heart. Right lung is di- removal; this is governed by a number of factors, in-
vided into three lobes by oblique and horizontal cluding monocarboxylate transporters, concentration
fissures. and isoform of LDH, and oxidative capacity of tis-
23 (D)  Amylase is an enzyme that hydrolyzes starches sues. Concentration of blood can rise during intense
into disaccharides. It is produced by both salivary exertion.
glands and the pancreas but NOT by the stomach. 32�� (D)  According to Fick’s law of diffusion, the great-
Hydrochloric acid, gastrin, and pepsinogen are all est net movement of a substance across a membrane
produced by stomach mucosa. will occur if the available surface area is large. If the
24 (C)  Cholecystokinin (pancreozymin) stimulates concentration gradient was great, the thickness of the
contraction of the gallbladder and production of pan- membrane was small, the temperature was increased,
creatic enzymes. Gastrin is a hormone that increases and the permeability was high, such conditions would
stomach activity. Secretin is a hormone produced by ALL facilitate diffusion.
the small intestine that stimulates the pancreas to se- 33�� (A)  Osmosis is associated with water movement
crete a fluid that is rich in bicarbonate ions. Alkalin- across a membrane. Membrane permeability may af-
ity of the bicarbonate ions neutralizes the acid chyme fect solute movement, which in turn drags water with
from the stomach so that pancreatic enzymes can it, affecting osmosis. Osmosis is IMPORTANT in bi-
function. ological systems, since many biological membranes
25 (A)  Oxygenated blood is transported to the liver by are semipermeable. These membranes are imperme-
hepatic artery. Hepatic portal vein delivers nutrient- able to organic solutes with large molecules, such as
rich blood from the digestive tract to the liver. Venous polysaccharides, while permeable to water and small,
blood is drained from the liver by the hepatic vein. uncharged solutes. Others are NOT true of osmosis.
Bile (gall) is yellow-green alkaline fluid secreted by 34�� (E)  Facilitated diffusion does NOT require ATP. Fa-
hepatocytes (cells) from the liver. cilitated diffusion (facilitated transport) is a process
26 (C)  Urine is produced in the kidney, flows into the of diffusion, a form of passive transport facilitated by
ureter, and transports urine to the bladder for stor- transport proteins. Facilitated diffusion may occur ei-
age. During micturition, urine flows from the bladder ther across biological membranes or through aqueous
through the urethra to the exterior. compartments of organism.
27 (B)  Orchitis is an acute inflammatory reaction of 35�� (C)  Autonomic nervous system (ANS) is part of the
the testis caused by infection. MOST cases are as- peripheral nervous system (PNS) and innervates NOT
sociated with a viral infection like mumps. Emesis is only smooth and cardiac muscles, but also glands.
­vomiting. Prostate becomes enlarged as a result of in- Major neurotransmitters include acetylcholine (para-
fection and/or cancer. No glandular cells in the penis sympathetic) and norepinephrine (sympathetic).
contribute to the seminal fluid. 36�� (B)  NO external Ca+2 is needed for contraction. ALL
28 (B)  Basic structure of the helix of DNA is a double Ca+ release is regulated by sarcoplasmic reticulum.
helix with two strands of DNA running antiparallel to 37�� (B)  Unlike skeletal muscles, which have contrac-
each other. RNA is a single-stranded molecule. tion times of 60 msec, cardiac muscles have greater
29 (D)  Both RNA and DNA are involved in transcrip- ­contraction times of 200 msec. This length of time
tion, the process of transferring genetic information ensures that this muscle will NOT exhibit summa-
from DNA to mRNA. RNA is dissimilar to DNA in tion. Cardiac muscles contain MORE mitochondria
that it is single stranded, contains uridine monophos- and contract in response to norepinephrine but NOT
phate, and is involved in translation of protein. ­acetylcholine.
30 (B)  Process of breaking large complex molecules 38�� (A)  Three chemical classes of hormones are ­ami­nes,
into smaller, simpler units is called catabolism. peptides, and steroids. Hormone is a chemical
Anatomy, Biochemistry, and Physiology 77

messenger that carries a signal from one cell (or group in hormone therapy for transsexual women. Like all
of cells) to another via the blood. Regulates function steroid hormones, estrogens readily diffuse across the
of target cells, i.e., cells that express a receptor. Action cell membrane; inside the cell, they interact with es-
(net effect) of hormones is determined by a number trogen receptors.
of factors, including pattern of secretion and response 45�� (D)  Inhibin is a peptide that inhibits production of
of the receiving tissue (signal transduction response). follicle-stimulating hormone. Thus it participates in
Endocrine molecules are secreted (released) directly the regulation of the menstrual cycle. Produced in go-
into the bloodstream, whereas exocrine molecules are nads, pituitary gland, placenta, other organs.
secreted directly into the duct and from the duct flow 46�� (A)  Alveolar perfusion by blood is a major factor as-
either into the bloodstream or from cell to cell by dif- sociated with normal gas exchange. Because of dif-
fusion. ferences in solubilities for CO2 and O2 (GREATER
39�� (A)  Parathyroid hormone decreases Ca+2 plasma lev- for CO2 than O2), gas exchange for oxygen is MORE
els in response to hypercalcemia. Acts with vitamin D limited than CO2 movement. Thus gradient for O2 is
to increase intestinal absorption of Ca+2. It may influ- 60 mm Hg and gradient for CO2 is 5 mm Hg. No gas
ence phosphate handling, but NOT metabolism, by exchange occurs in bronchi or pulmonary artery.
kidneys. 47�� (C)  Pleural space normally contains ONLY a small
40�� (C)  Pancreas is BOTH endocrine and exocrine gland. amount of fluid. Pressure within is negative (below
Produces glucagon, elevates plasma glucose levels. atmospheric) because of opposing forces of lung and
Exocrine function results in ­ secretion of enzymes chest wall and fluid movements out of the space. Sur-
(NOT hormones) that break down carbohydrates and factant is produced by type II cells in lungs.
proteins. 48�� (A)  Metabolism generally is coupled with ventila-
41�� (C)  Aldosterone is produced by the adrenal cortex, tion; thus as O2 demands increase, ventilation in-
and its secretion is stimulated by LOW plasma lev- creases. Ventilation is depressed by high levels of
els of Na+ and high levels of K+. Low blood pressure CO2 and by anoxia.
through the renin-angiotensin system also stimulates 49�� (A)  Bicarbonate ions are ~72% of the CO2 that is
its production. transported.
42�� (D)  Glucocorticoids suppress the immune system. 50�� (D)  Increased blood flow to the cells is the ONLY
Class of steroid hormones characterized by an ability listed condition that would increase O2 transport or ac-
to bind with the cortisol receptor and trigger similar cess. Inhalation of carbon monoxide would decrease
effects. Distinguished from mineralocorticoids and the amount of O2 bound to hemoglobin; decreased
sex steroids by the specific receptors, target cells, and hemoglobin levels would decrease the amount of he-
effects. Corticosteroid refers to BOTH glucocorti- moglobin available; high altitudes would decrease
coids and mineralocorticoids but is often used as syn- the alveolar and arterial po2 and cyanide poisons en-
onym for glucocorticoid. Cortisol (hydrocortisone) is zymes that use O2 to make ATP.
MOST important. Essential for life, and regulates or 51�� (D)  Hyperventilation results in the “blowing off ” of
supports a variety of important cardiovascular, meta- CO2 and also causes a decrease in H+ and increase in
bolic, immunological, homeostatic functions. level of O2. State of breathing faster and/or deeper
43�� (B)  Testosterone production is stimulated by lutein- than necessary thereby reduces CO2 concentration of
izing hormone, stimulates protein production, and in- blood below normal. Can cause such symptoms as
creases red blood cell production, resulting in higher numbness or tingling in hands, feet, and lips, light-
hematocrit values (number of RBCs in blood) in men headedness, dizziness, headache, chest pain, slurred
compared with those in women. Testosterone is a ste- speech, and sometimes fainting, particularly when
roid hormone from the androgen group. Testosterone accompanied by the Valsalva maneuver.
is secreted MAINLY in the testes of males and the 52�� (D)  Intrinsic factor is essential to absorption of vi-
ovaries of females, although small amounts are also tamin B12. Stomach does NOT produce vitamin B12,
secreted by the adrenal glands. MAIN male sex hor- but parietal cells of stomach produce glycoprotein,
mone and anabolic steroid. which is necessary for absorption of vitamin B12 later
44�� (B)  Estrogen induces the production of progesterone in the terminal ileum. Upon entry into stomach, vi-
receptors in the uterus, acts as antioxidant, and in- tamin B12 becomes bound to one of two B12-binding
creases bone density. Estrogens are a group of ste- proteins present in gastric juice. In the LESS acidic
roid compounds, named for importance in the estrous environment of small intestine, these proteins dis-
cycle, and functioning as the MAIN female sex hor- sociate from vitamin, enabling it to bind to intrinsic
mone. Estrogens are used as part of some oral con- factor and enter portal circulation through a receptor
traceptives (birth control pills [BCP]), in estrogen in ileal mucosa specific for the B12–intrinsic factor
replacement therapy of postmenopausal women, and complex.
78 Saunders Review of Dental Hygiene

53�� (A)  Liver produces red blood cells in the embryo 61�� (A)  Cardiac output (CO) is increased by increase in
and fetus. Synthesizes factors needed for clotting and contractility of heart. Other factors that increase car-
amino acids. Also stores glycogen, iron, copper, and diac output are increased heart rate, venous return,
vitamins A and D. Steroid hormones are synthesized stroke volume. The CO is volume of blood being
MAINLY in gonads and in adrenal cortex. pumped by heart, in particular by a ventricle in a min-
54�� (C)  Enzymes that digest chyme in the lumen are ute.
produced by the pancreas. Small intestine produces 62�� (A)  Factors that influence total peripheral resistance
ONLY intracellular enzymes. include diameter or radius of blood vessels (where
55�� (C)  Neither food NOR hormones are absorbed by R = 1⁄4) and viscosity (increased viscosity leads to in-
large intestine. Major functions include reabsorption creased resistance). Thus as diameter of blood vessels
of water and electrolytes. Intrinsic factor is produced decreases, total peripheral resistance increases. Radii
in the stomach. of blood vessels are increased by production of me-
56�� (C)  Angiotensin II is produced by endothelial cells tabolites, cytokines, histamine. Viscosity is increased
in lung. Angiotensin is an oligopeptide in blood that during dehydration.
causes vasoconstriction, increased blood pressure, 63�� (C)  Chemoreceptor for O2, CO2, pH; all others sense
release of aldosterone from adrenal cortex. Powerful pressure or volume changes. Sensory receptor that
dipsogen. Derived from precursor molecule angio- transduces a chemical signal into an action potential.
tensinogen, serum globulin produced in liver. Plays Detects certain chemical stimuli in the environment.
IMPORTANT role in renin-angiotensin system. An- 64 (B)  Decreased blood pressure causes increased SNS
giotensin I is converted to angiotensin II through discharge.
removal of two terminal residues by the enzyme an- 65�� (B)  Hypertension contributes to formation of athero-
giotensin-converting enzyme (ACE, kinase), found sclerotic plaques. Also increases work of heart; tends
MAINLY in capillaries of lung. Angiotensin II acts as to increase with BOTH age and obesity.
endocrine or autocrine, paracrine, or intracrine hor- 66�� (B)  All of these situations, EXCEPT when epineph-
mone. Angiotensin II is degraded to angiotensin III rine is released rapidly, result in decrease in blood
by angiotensinases that are located in red blood cells pressure.
and vascular beds of MOST tissues. Has a half-life 67�� (C)  Ribosomes that produce protein are located on
of around 30 seconds in circulation but as long as 15 rough endoplasmic reticulum (RER) or in cytoplasm,
to 30 minutes in tissue. Angiotensin II thus increases NOT plasma membrane.
blood pressure; ACE inhibitor drugs are major drugs 68�� (A)  Mitochondria produce ATP. Others are involved
against hypertension (high blood pressure). in metabolic handling of internally produced chemi-
57�� (A)  Blockage of ureter increases pressures within cals (Golgi apparatus) and/or externally produced
kidney and decreases urine excretion. Ureters are chemicals (peroxisomes and lysosomes) but do NOT
ducts that carry urine from kidneys to the urinary produce ATP.
bladder. They are muscular tubes that can propel 69�� (A)  First (I) cranial nerve (olfactory) is FIRST of 12
urine along by motions of peristalsis. cranial nerves. Specialized olfactory receptor neu-
58 (D)  Urethra is a part of urinary system but is NOT rons of the olfactory nerve are located in the olfac-
part of nephron. Others are parts of nephron. Ure- tory mucosa of the upper parts of the nasal cavity.
thra is tube that connects bladder to the outside of Nerve consists of collection of sensory nerve fibers
body. Has excretory function in both genders to pass that extend from olfactory epithelium to the olfactory
urine to outside, and also reproductive function in the bulb, passing through many openings of cribriform
male, as passage for sperm. plate, sievelike structure. Sense of smell (olfaction)
59�� (A)  Efferent arteriole constriction increases GFR. arises from stimulation of olfactory receptors by ac-
Increase in the leakiness of the Bowman’s capsule tivation from gas molecules that pass by nose during
also increases GFR. Constriction of afferent arteriole respiration. Resulting electrical activity is transduced
decreases GFR. into olfactory bulb, which then transmits electrical
60�� (C)  During respiratory acidosis, kidneys retain activity to other parts of olfactory system and rest of
HCO3− and excrete H+. Respiratory acidosis is aci- central nervous system by way of olfactory tract.
dosis (abnormal acidity of the blood) caused by de- 70�� (B)  Liver is LARGEST visceral organ (internal or-
creased ventilation of pulmonary alveoli, leading gan of the body); skin is LARGEST external organ.
to elevated arterial carbon dioxide concentration 71�� (B)  Glucose provides energy to brain. Monosaccharide
(Paco2). Clinical disturbance that is due to alveolar (simple sugar) is an important carbohydrate. Insulin
hypoventilation. Production of CO2 occurs rapidly, reaction and other mechanisms regulate concentration
and failure of ventilation promptly increases level of of glucose in blood; high fasting blood sugar level is
Paco2. indication of prediabetic and diabetic conditions.
Anatomy, Biochemistry, and Physiology 79

72�� (A)  Pancreas has both alpha and beta cells. Alpha 75�� (A)  Trigeminal is also designated as fifth (V) cranial
cells release glucagon, and beta cells produce insulin. nerve (V). Seventh (VII) cranial nerve is the facial.
73�� (C)  Glycogen provides a food storage system for all Tenth (X) cranial nerve is the vagus. Twelfth (XII)
forms of animal life. One area of storage is in liver, cranial nerve is the hypoglossal. However, others in-
which assists in regulating blood sugar. Glycogen nervate important structures of the oral cavity. Facial
storage also occurs in the muscle, where glycogen nerve is responsible for sensation in the face. Sensory
serves as an energy source for muscle contraction. information from the face and body is processed by
74�� (A)  Initiation of starch digestion begins in the oral parallel pathways in the central nervous system. Tri-
cavity during chewing. Enzyme salivary amylase be- geminal is MAINLY a sensory nerve from the oral
gins to break starch into disaccharides such as malt- cavity, but it also has certain motor functions (biting,
ose. Stomach acids hydrolyze maltose and sucrose, chewing, swallowing), such as innervating muscles
but ONLY to a small extent. Digestion continues in of mastication.
the small intestine; there, enzyme pancreatic amylase
(from the pancreas) breaks down the remaining di-
saccharides into monosaccharides.
C ha p ter 4

Head, Neck, and Dental Anatomy


REGIONS OF THE HEAD   2. Temporomandibular joint (TMJ): inferior to zy-
Regions of the head include frontal, parietal, occipital, gomatic arch and just anterior to ear, where upper
temporal, orbital, nasal, infraorbital, zygomatic, buccal, skull forms a joint with lower jaw.
oral, mental regions. Specific landmarks are noted for 3. Cheek: forms the side of face between nose, mouth,
each region. and ear; composed MAINLY of fat (buccal fat pad)
• See CD-ROM for Chapter Terms and WebLinks. and muscles, including masseter muscle; angle of
• See Chapter 3, Anatomy, Biochemistry, and Physiol- mandible, sharp angle of lower jaw that is inferior
ogy: general anatomy. to earlobe.
A. Frontal region: supraorbital ridge under each eye- G. Oral region: includes lips, oral cavity, palate, tongue,
brow; frontal eminence forms the prominence of the floor of mouth, parts of pharynx:
forehead. 1. Nasolabial sulcus: groove running upward ­between la-
B. Parietal and occipital regions: covered by scalp, which bial commissure and ala of nose; labiomental groove:
may be covered by hair. horizontal groove to which lower lip ­ extends and
C. Temporal region and external ear: include auricle and which separates lower lip from chin in mental region.��
external acoustic meatus (EAM): 2. Vermilion zone: darker portion of lips than sur-
1. Helix: superior and posterior free margin of au- rounding skin; vermilion border: outlines lips from
ricle, ends inferiorly at lobule (earlobe). surrounding skin, transition zone.��
2. Tragus: part of auricle anterior to external acoustic 3. Philtrum: vertical groove on midline of upper lip,
meatus; antitragus: flap of tissue opposite tragus extending downward from nasal septum; tubercle
(radiographic landmarks). of upper lip: thicker area where philtrum termi-
D. Orbital region: includes eyeball and supporting struc- nates; labial commissure: where upper and lower
tures within orbit (bony socket): lips meet at each corner of mouth.�����
1. Lateral canthus: outer corner where upper and H. Mental region: chin’s mental protuberance (promi-
lower eyelids meet; medial canthus: inner corner nence), with possible midline depression (dimple)
of eye. �� that marks underlying bony fusion of lower jaw.
2. Upper and lower eyelids: cover and protect each
eyeball; lacrimal gland: located behind each upper Oral Cavity
eyelid and within orbit and produces lacrimal fluid Oral cavity is the inside of the mouth and includes jaws,
(tears). palate, tongue, floor of the mouth, pharynx, and all asso-
3. Sclera: white area of eyeball; iris: circular central ciated oral mucosal tissues.
area of color; pupil: central opening in iris.�� • See Chapters 2, Embryology and Histology: oral cavity
4. Conjunctiva: membrane that lines inside of eyelids histology; 3, Anatomy, Biochemistry, and Physiology:
and front of eyeball.����� pharynx, digestion.
E. Nasal region and external nose: root of nose is located A. Structures labeled according to location: (1) lingual,
between eyes: if closest to tongue; (2) palatal, if closest to palate;
1. Nasion: midpoint landmark; bridge of nose is infe- (3) facial, if closest to facial surface; (4) buccal, if
rior. closest to inner cheek; (5) labial, if closest to lips.
2. Nares (nostrils): separated by midline nasal sep- B. Mucous membranes (oral mucosa): line inner oral
tum, bounded laterally by alae. cavity; parts of the lips are lined by labial mucosa;
3. Apex of nose: tip. buccal mucosa lines inner cheek, covers buccal fat
F. Infraorbital region: inferior to orbital region and lat- pad, including parotid papilla, and contains a duct
eral to nasal region; zygomatic region overlies cheek- opening from parotid salivary gland.
bone; buccal region is composed of soft tissues of C. Maxillary and mandibular bones (jaws): lie beneath
cheek: the respective lips and contain primary and/or per-
1. Zygomatic arch: extends from just below lateral manent teeth, including incisors, canines, premolars,
margin of eye to upper part of ear. molars, depending on dentition present.

80
Head, Neck, and Dental Anatomy   81

1. Maxillary and mandibular vestibules: upper and 5


lower spaces between cheeks, lips, and gingival
tissues.
2. Alveolar mucosa: lines each vestibule and meets 6
1 Base
labial or buccal mucosa at the mucobuccal fold 7
(depth or height is location for local anesthetic 8
nerve injections given into soft tissue).
3. Labial frenum: fold of tissue located at the midline 2
between the labial mucosa and alveolar mucosa of 9
Body
the maxilla and mandible.
4. Gingival tissues (discussed later): surround maxil- 10
3
lary and mandibular teeth.
D. Palate (roof of mouth):
1. Anterior part: hard palate and includes median
palatine raphe (median palatal suture), incisive pa- 4
pilla, and palatine rugae, which are irregular ridges 10 Fungiform lingual papillae
of tissue directly posterior to the incisive papilla;
posterior part: soft palate and includes uvula.
9 Median lingual sulcus

2. Pterygomandibular fold: extends from junction 8 Sulcus terminalis


of the hard and soft palates down to mandible, 7 Foramen cecum
just posterior to the most distal mandibular
tooth; separates cheek from pharynx; retromo-
6 Palatine tonsil

lar pad is located just distal to last tooth of the 5 Epiglottis


mandible. 4 Apex of the tongue
E. Tongue (Figure 4-1):
1. Body: anterior two thirds and includes the tip
3 Filiform lingual papillae

(apex). 2 Circumvallate lingual papillae


2. Base: posterior one third, which attaches to floor 1 Lingual tonsil
of the mouth; does NOT lie within the oral cavity
but rather in the oral part of pharynx. Figure 4-1  Tongue and tonsillar tissues. (From Fehrenbach
3. Lateral surfaces: located on each side; dorsal sur- MJ, ed: Dental anatomy coloring book, St. Louis, 2008, Saunders/
face is located on top and includes median lingual Elsevier.)
sulcus; ventral surface (underside) has visible deep
lingual veins (varicosities in elderly) and plicae
fimbriatae, with fringelike projections. 2. Sublingual fold: V-shaped ridge of tissue on floor
4. lingual papillae: elevated structures of special- of mouth; contains duct openings from sublingual
ized mucosa on tongue surface, some of which salivary gland.
are associated with taste buds (see Chapters 2, 3. Sublingual caruncle: small papilla at anterior end
Embryology and Histology; 3, Anatomy, Bio- of each sublingual fold; contains duct openings
chemistry, and Physiology: lingual papillae, taste from BOTH submandibular and sublingual sali-
sensation). vary glands.
5. Sulcus terminalis: V-shaped groove located poste- G. Pharynx (throat): muscular tube that serves BOTH
riorly on dorsal surface that separates base from respiratory and digestive systems, divided into naso-
body. pharynx, oropharynx, laryngopharynx.
6. Foramen cecum: pitlike depression in center of sul-
cus terminalis; point of attachment of thyroglossal REGIONS OF THE NECK
duct, embryological connection for development neck is divided by the cervical muscle, sternocleido-
of thyroid gland. mastoid muscle (SCM), diagonally each side into ante-
7. Lingual tonsil: irregular mass of tonsillar tissue rior cervical and posterior cervical triangles.
­located more posteriorly than sulcus terminalis on A. Anterior region: corresponds to two anterior cervical
the dorsal surface. triangles, separated by midline, and subdivided into
F. Floor of the mouth: located inferior to ventral surface submandibular and midline submental triangle by di-
of tongue (Figure 4-2): gastric muscle (Figure 4-3).
1. Lingual frenum: midline fold of tissue between B. Posterior cervical triangle: considered lateral region
ventral surface of tongue and floor of mouth. and is posterior to the SCM on each side.
82   Saunders Review of Dental Hygiene

1 6
3
1 7
2
2 8
4 3
9
4 10

Sublingual ducts 4

Sublingual gland 3
10 Muscular triangle Submandibular triangle 5
Sublingual caruncle 2
Thyroid cartilage 9 Omohyoid muscle 4
Sublingual fold 1
Hyoid bone 8 Sternocleidomastoid muscle 3
Figure 4-2  Floor of the mouth. (From Fehrenbach MJ, ed:
Dental anatomy coloring book, St. Louis, 2008, Saunders/Elsevier.) Submental triangle 7 Carotid triangle 2
Digastric muscles 6 Mandible 1

C. Thyroid cartilage: prominence of larynx at anterior


midline; vocal cords (ligaments) are attached to pos-
terior surface.
D. Hyoid bone: suspended in anterior midline; supe-
rior to thyroid cartilage, attached to many muscles;
controls position of base of tongue (see Chapter 3,
Anatomy, Biochemistry, and Physiology).

SKULL
Skeletal system serves as a base during palpation of
soft tissues and as a marker during location of soft tis-
1
sue lesions, administration of local anesthesia, radio-
graphic procedures. Bones may be a factor in spread
of dental infection and may undergo a disease process 2
themselves.
Skull (22 bones), with single and paired bones. Im- 3
movable, with EXCEPTION of mandible and temporo- 5
4
mandibular joint (TMJ). Has movable articulation with
6
bony vertebral column in neck area. Contains bony open-
ings for important nerves and blood vessels (Table 4-1).
Has paranasal sinuses within that serve to lighten bony Clavicle 6 Trapezius muscle 3
mass. Has many associated processes that are involved Omohyoid muscle 5 Occipital triangle 2
in important structures. To study its landmarks, important Subclavian triangle 4 Sternocleidomastoid muscle 1
to view from superior, lateral, inferior, anterior. Its bones
are divided into three categories: cranial, facial, hyoid, Figure 4-3  Regions of the neck. (From Fehrenbach MJ, ed:
which are noted on each view; each category is discussed Dental anatomy coloring book, St. Louis, 2008, Saunders/Elsevier.)
separately next.
Head, Neck, and Dental Anatomy   83

Table 4-1  Bony openings in the skull and their associated nerves and blood vessels

Bony opening Bony location Nerves and vessels

Carotid canal Temporal Internal carotid artery


Cribriform plate with Ethmoid Olfactory nerves
­foramina
External acoustic Temporal (Opening to tympanic cavity)
meatus
Foramen lacerum Sphenoid, occipital, (Cartilage)
temporal
Foramen magnum Occipital Spinal cord, vertebral arteries, eleventh cranial
nerve
Foramen ovale Sphenoid Mandibular division of fifth cranial nerve
Foramen rotundum Sphenoid Fifth cranial nerve
Foramen spinosum Sphenoid Middle meningeal artery
Greater palatine Palatine Greater palatine nerve and vessels
foramen
Hypoglossal canal Occipital Ninth cranial nerve
Incisive foramen Maxilla Nasopalatine nerve and branches of sphenopalatine
artery
Inferior orbital fissure Sphenoid and maxilla Infraorbital and zygomatic nerves, infraorbital artery,
ophthalmic vein
Infraorbital foramen and Maxilla Infraorbital nerve and vessels
canal
Internal acoustic meatus Temporal Seventh and eighth cranial nerves
Jugular foramen Occipital and temporal Internal jugular vein and ninth, tenth, eleventh
cranial nerves
Lesser palatine Palatine Lesser palatine nerve and vessels
foramen
Mandibular foramen Mandible Inferior alveolar nerve and vessels
Mental foramen Mandible Mental nerve and vessels
Optic canal and Sphenoid Optic nerve and ophthalmic artery
foramen
Petrotympanic fissure Temporal Chorda tympani nerve
Pterygoid canal Sphenoid Area nerves and vessels
Stylomastoid foramen Temporal Seventh cranial nerve
Superior orbital fissure Sphenoid Third, fourth, sixth cranial nerves and ophthalmic
nerve and vein

From Fehrenbach MJ, Herring SW: Illustrated anatomy of the head and neck, ed 3, St. Louis, 2007, Saunders/Elsevier.

• See Chapter 3, Anatomy, Biochemistry, and Physiol- 1. Orbits: contain and protect eyeballs:
ogy: skeletal system. a. Orbital walls: orbital plates of frontal, ethmoid,
A. Superior view of skull: major bones and sutures lacrimal; orbital surfaces of maxilla; ­zygomatic
(joints between bones of the skull are immovable; bone; and orbital surface of greater wing of
fontanelles [“soft spots”] on an infant’s head). sphenoid.
1. Coronal suture: frontal and parietal bones. b. Orbital apex: lesser wing of sphenoid and palatine
2. Sagittal suture: paired parietal bones. bones; optic canal is opening in orbital apex.
3. Lambdoidal suture: single occipital bone and c. Superior orbital fissure (SOF): located ­between
paired parietal bones. the greater and lesser wings of sphenoid and
B. Anterior view of skull: orbits and nasal cavity (Figure connects orbit and cranial cavities; carries oculo-
4-4): motor, trochlear, abducens, ophthalmic vessels.
84   Saunders Review of Dental Hygiene

2. Temporal fossa: formed by several bones of skull,


contains body of temporalis; infratemporal fossa:
inferior to temporal fossa; pterygopalatine fossa:
deep to infratemporal fossa.
1 3. Zygomatic arch: formed by union of temporal pro-
2 cess of zygomatic bone and zygomatic process of
temporal bone.
3 9
4. TMJ: ONLY freely movable joint in the head, be-
4 tween temporal bone and mandible.
3
5 D. Inferior view of external surface of skull (Figure 4-6):
6 10 1. Hard palate: forms floor of nasal cavity and roof
of mouth at anterior section; formed by palatine
7 11
­processes of maxilla and horizontal plates of pala-
tine bones; bordered by alveolar process of maxilla
and maxillary teeth (Figure 4-7).
8 12 2. Median palatine suture: midline articulation be-
tween palatine processes of maxillae anteriorly as
well as horizontal plates of palatine bones posteri-
orly, noted clinically as the median palatal raphe,
6 MORE prominent and thicker in the soft palate
region; transverse palatine suture articulates with
12 Maxilla Lacrimal bone
11 Inferior nasal concha Ethmoid bone 5 palatine processes of maxillae and horizontal
10 Zygomatic bone Nasal bone 4 plates of palatine bones.
3. Jugular foramen: opening through which internal
jugular vein and glossopharyngeal, vagus, acces-
Temporal bone 9 Sphenoid bone 3

Mandible 8 Parietal bone 2


sory nerves pass; foramen lacerum is large, irregu-
Vomer bone 7 Frontal bone 1 larly shaped, filled with cartilage.
E. Superior view of internal surface of skull (Figure 4-8).
Figure 4-4  Anterior view of skull. (From Fehrenbach MJ,
ed: Dental anatomy coloring book, St. Louis, 2008, Saunders/ Cranial Bones
Elsevier.)
Cranial bones (8) form cranium and include occipital,
frontal, parietal, temporal, sphenoid, ethmoid. See earlier
d. Inferior orbital fissure (IOF): located between views of skull.
greater wing of sphenoid and maxilla; con- A. Occipital: single that is located in MOST posterior
nects orbit with infratemporal and pterygopala- part of skull:
tine fossae; carries infraorbital and zygomatic 1. Foramen magnum on external surface: largest skull
nerves, branches of maxillary, infraorbital, and opening; carries spinal cord, vertebral arteries, ac-
inferior ophthalmic vessels. cessory nerve.
2. Nasal cavity: has piriform aperture for anterior 2. Occipital condyles: pair located lateral and anterior to
opening; lateral boundaries are formed by maxillae: foramen magnum; movable articulation with atlas.
a. Nasal septum: divides nasal cavity into two 3. Basilar part: four-sided plate; pharyngeal tubercle
parts; formed by nasal septal cartilages, perpen- is midline projection anterior to foramen magnum,
dicular plate of ethmoid bone, vomer.�� with paired hypoglossal canals that carry hypo-
b. Nasal conchae: located on each lateral wall; glossal nerve.
superior nasal concha and middle nasal concha B. Frontal: single that forms forehead and superior part
are formed from ethmoid bone; inferior nasal of orbits:
concha is separate facial bone���. 1. Frontal sinuses: pair contained within, superior to
c. Nasal meatus: grooves beneath each concha, have nasal cavity; each sinus communicates with and
openings through which paranasal sinuses and na- drains into nasal cavity by means of frontonasal
solacrimal duct communicate with nasal cavity�������. duct to middle nasal meatus.
C. Lateral view of skull (Figure 4-5): 2. Supraorbital ridges: elevations over superior part of
1. Superior temporal line: superior ridge; inferior orbit, and supraorbital notch is located on medial part
temporal line: superior boundary of temporal of the supraorbital ridge; lacrimal fossa is located just
fossa; squamosal suture is between temporal and inside lateral part of supraorbital ridge and contains
parietal bones. lacrimal gland, which produces lacrimal fluid.
Head, Neck, and Dental Anatomy   85

1 12

2 13
14 1
3
15
4 2
16
5
17 3
6
18 4
7
19
8

9 20

21
10 11

11 Pterygoid process Maxillary sinus 4


10 Styloid process Sphenoidal sinus 3
21 Mental foramen
of temporal bone Ethmoidal sinuses 2
20 Mandible Mastoid process 9
Frontal sinus 1
19 Maxilla Condyloid process 8
18 Zygomatic bone External occipital protuberance 7
17 Nasal bone External auditory meatus 6
16 Lacrimal bone Occipital bone 5
15 Ethmoid bone Lambdoidal suture 4
14 Sphenoid bone Temporal bone 3

13 Frontal bone Parietal bone 2

12 Coronal suture Squamosal suture 1

Figure 4-5  Lateral view of skull. (From Fehrenbach MJ, ed: Dental anatomy coloring book, St. Louis, 2008, Saunders/Elsevier.)

C. Parietal bones: paired and BOTH articulate with each maxillary division of the same nerve; foramen spi-
other at sagittal suture. nosum: near spine and carries middle meningeal
D. Temporal bones: paired and form lateral walls of skull: artery into cranial cavity.
1. Tympanic part: forms MOST of external acoustic 2. Body: middle position; contains sphenoidal sinus,
meatus; petrous part contains mastoid process and which communicates with and drains into nasal
carotid canal, which carries internal carotid artery; cavity through opening superior to each superior
mastoid notch is medial to mastoid process; styloid nasal concha:
process is bony projection. a. Lesser wing of sphenoid: anterior and forms
2. Stylomastoid foramen: carries seventh cranial base of orbital apex; greater wing of sphenoid
nerve (facial), located between styloid process and bone: posterolateral process with spine at each
­mastoid process; internal acoustic meatus is on in- corner, divided by infratemporal crest.
tracranial surface, carries vestibulocochlear nerve b. Pterygoid process: inferior and consists of lat-
and seventh cranial nerve (facial). eral and medial plates, between which is ptery-
E. Sphenoid (“butterfly”): single in midline; MOST goid fossa; hamulus: inferior termination of
complex; articulates with frontal, parietal, ethmoid, medial pterygoid plate.
temporal, zygomatic, palatine, occipital bones, max- F. Ethmoid: single in midline and adjoins vomer:
illae, vomer: 1. Two unpaired plates that cross each other, with
1. Foramen ovale: oval opening that carries man- perpendicular plate midline and vertical, cribri-
dibular division of fifth cranial nerve (trigeminal); form plate horizontal, perforated by foramina to
foramen rotundum: round opening that carries allow passage of olfactory nerves.
86   Saunders Review of Dental Hygiene

9 1 8
1 10 Hard palate 2
2 11 9
3
3 12
4
13
5
14
6 4 10
15
7
16 5
11
8
17 6
18
foramen Sphenoid bone 6
19 11 Lesser palatine
Maxillary tuberosity 5
foramen
10 Greater palatine Palatine bones 4

Maxillae 9 Median palatine suture 3


11 Palatine bone (horizontal plate)
10 Alveolar process Incisive foramen 8 Alveolar process of maxilla 2

9 Maxilla (palatine process) Maxillary teeth 7 Palatine process of the maxilla 1

Figure 4-7  Inferior view of hard palate. (From Fehrenbach


19 Occipital bone 8 Mastoid process
18 Temporal bone 7 Jugular foramen MJ, ed: Dental anatomy coloring book, St. Louis, 2008, Saunders/
Elsevier.)
17 Foramen magnum 6 Carotid canal
16 Occipital condyle 5 Foramen lacerum 1. Lacrimal: � paired that form small part of anterior
15 Stylomastoid foramen 4 Foramen ovale medial wall of orbit; nasolacrimal duct is formed at
14 Styloid process 3 Sphenoid bone junction of lacrimal and maxillary bones; lacrimal
fluid from gland drains into inferior nasal ­meatus.
2. Nasal: �paired that form�������������������������������
bridge of nose and articu-
13 Mandibular fossa 2 Zygomatic bone
12 Vomer bone 1 Zygomatic process of maxilla late with each other in the midline above piriform
Figure 4-6  Inferior view of external surface of skull. (From aperture; paired inferior nasal conchae project off
Fehrenbach MJ, ed: Dental anatomy coloring book, St. Louis, 2008, maxilla to form part of lateral walls of nasal cavity.
Saunders/Elsevier.) 3. Inferior nasal conchae: project off maxilla to form
part of lateral walls of nasal cavity.
C. Zygomatic: paired that form cheek bones (malar surfa­
2. Crista galli: vertical midline continuation of per- ces), each composed of frontal, temporal, maxillary
pendicular plate into cranial cavity; ethmoidal processes:
sinuses: in the lateral masses, which open into 1. Orbital surface of frontal process forms anterior
superior and middle meatus of nasal cavity, lateral orbital wall; temporal process of zygomatic
forms superior and middle nasal conchae in nasal bone joins zygomatic process of temporal bone to
cavity. form zygomatic arch; orbital surface of maxillary
process forms part of lateral orbital wall.
Facial Bones D. Palatine: paired that consist of horizontal and vertical
Facial bones (14) create facial features and serve as base for plates; articulate with each other and with maxillae
the dentition. Include vomer, lacrimal, zygomatic, palatine, and sphenoid:
maxilla, mandible. See earlier skull views. Many landmarks 1. Horizontal plates: form posterior part of hard pal-
of the facial bones are used for local anesthetic delivery. ate, also floor of nasal cavity; ���������������������
posterior part of me-
• See Chapter 5, Radiology: radiographic descriptions of dian palatine suture is located at articulation of two
bones. horizontal plates; ��������������������������������������
vertical plates: form part of lateral
A. Vomer: single that forms posterior part of nasal sep- walls of nasal cavity; BOTH plates contribute to
tum; located in midsagittal plane, inside nasal cavity; orbital apex.
has free posteroinferior border. 2. Greater palatine foramen: located in posterolateral
B. Smaller bones (but they do just fine unless they par- region of each of the palatine bones, distal to maxil-
ticipate in any bar fights!): lary third molar, carries greater palatine nerve and
Head, Neck, and Dental Anatomy   87

8
1
9
10

11
2 1
12 7
3 2
13 8
4
14
5 9
15
6 3
16
17 4
7
18 10
5
19
6

10 Cribriform plate 10 Sphenoid bone Canine fossa 5

19 Occipital bone Crista galli 9 Infraorbital sulcus 9 Zygomatic process of maxilla 4

18 Parietal bone Ethmoid bone 8 Ethmoid bone 8 Infraorbital foramen 3

17 Hypoglossal foramen Foramen magnum 7 Lacrimal bone 7 Nasal bone 2

16 Hypoglossal canal Jugular foramen 6 Canine eminence 6 Frontal process of maxilla 1

15 Internal auditory meatus Temporal bone 5 Figure 4-9  Maxilla. (From Fehrenbach MJ, ed: Dental anatomy
coloring book, St. Louis, 2008, Saunders/Elsevier.)
14 Foramen lacerum Foramen ovale 4

4. Canine fossa: depression posterosuperior to maxil-


13 Sella turcica Foramen rotundum 3

12 Optic foramen Greater wing of sphenoid 2 lary canine roots; canine eminence is facial ridge
11 Sphenoid bone Frontal bone 1 over maxillary canine.��
5. Lateral wall of nasal cavity: made up mainly of max-
Figure 4-8  Superior view of internal surface of skull. (From illa; floor of nasal cavity is formed by palatine process
Fehrenbach MJ, ed: Dental anatomy coloring book, St. Louis, 2008, of maxilla anteriorly, also anterior part of hard palate.��
Saunders/Elsevier.)
6. Anterior part of median palatine suture: articu-
lation of two fused maxillae (two palatine pro-
blood vessels; lesser palatine foramen: nearby to cesses); incisive foramen in anterior midline part
carry own vessel to soft palate and tonsils. of palatine process, just posterior to maxillary cen-
E. Maxilla: upper jaw, consists of two maxillary bones tral incisors; carries branches of BOTH right and
(maxillae) fused together; articulates with frontal, left nasopalatine ­vessels.
lacrimal, nasal, inferior nasal concha, sphenoid, eth- 7. Alveolar process: contains roots of maxillary teeth;
moid, palatine, zygomatic bones, vomer (Figure 4-9); maxillary tuberosity on posterior part is perforated
each maxilla has a body and frontal, zygomatic, pala- by posterior superior alveolar foramina, allowing
tine, alveolar processes: entry for posterior superior alveolar vessels.
1. Body of maxilla contains maxillary sinus located F. Mandible: lower jaw, single, ONLY freely movable
just posterior to maxillary canine and premolars; bone of skull; articulates with temporal bones at each
largest sinus; drains into middle meatus.�� TMJ (Figure 4-10):
2. Inferior orbital fissure: separates each orbital sur- 1. Mental protuberance: prominence of chin; man-
face from sphenoid; carries infraorbital and zygo- dibular symphysis is ridge where bone was formed
matic nerves, infraorbital artery, inferior ophthalmic by fusion.
vein.�� 2. Mental foramen: on outer (lateral) surface of man-
3. Infraorbital sulcus groove in floor of orbital sur- dible, between apices of mandibular premolars; al-
face: becomes infraorbital canal, then terminates as lows entrance of mental nerve and blood vessels
infraorbital foramen; carries infraorbital vessels.�� into mandibular canal.
88   Saunders Review of Dental Hygiene

1 2 3 4 5 6 ­sublingual fossa contains sublingual gland; sub-


mandibular fossa contains submandibular gland.

MUSCLES OF HEAD AND NECK  


Head and neck muscles include cervical muscles, mus-
cles of facial expression, muscles of mastication, hyoid
muscles, muscles of the tongue.
7
• See Chapter 3, Anatomy, Biochemistry, and Physiol-
ogy: skeletal muscles.

8 Cervical Muscles
9 Cervical muscles include SCM and trapezius.
A. The SCM: paired that serves as MAIN muscular
­landmark, divides neck region into anterior and pos-
terior cervical triangles (Figure 4-3):
1. Originates from clavicle and sternum, inserts on
10 11 12 13 14 mastoid process of temporal bone.
2. If ONLY one SCM contracts, the head and neck
Mandibular teeth 7 bend to the same side and the face and front of the
neck rotate to the opposite side; if BOTH SCMs
14 Neck
of condyle
13 Angle Articulating surface 6 contract, head flexes at the neck and extends at the
12 Ramus Mandibular notch 5 junction between the neck and skull; innervated by
accessory nerve.
B. Trapezius: paired that covers the lateral and posterior
11 Mental foramen Coronoid process 4
10 Body Coronoid notch 3 surfaces of the neck:
Mental protuberance 9 Pterygoid fovea 2 1. Originates from occipital bone and posterior mid-
Alveolar process 8 line of the cervical and thoracic regions; inserts on
clavicle’s lateral third and parts of scapula.
External oblique line 1

Figure 4-10  Mandible. (From Fehrenbach MJ, ed: Dental 2. Acts to lift clavicle and scapula; innervated by ac-
a­natomy coloring book, St. Louis, 2008, Saunders/Elsevier.) cessory nerve and third and fourth cervical nerves.

MUSCLES OF FACIAL EXPRESSION  


3. Mandibular foramen: on inner (medial) surface of Muscles of facial expression act in various combinations
­ramus, from which inferior alveolar vessels exit man- to alter appearance of face; innervated by the facial nerve,
dible after traveling in mandibular canal. seventh (VII) cranial nerve (Figure 4-11).
4. Lingula: sharp spine that overhangs mandibular fo- A. Epicranial muscle (epicranius), located in scalp re-
ramen and gives attachment to the sphenomandib- gion, with frontal and occipital bellies, which are
ular ligament; at its lower and back part is a notch separated by a scalp tendon, epicranial aponeurosis;
from which the mylohyoid groove runs obliquely raises eyebrows and scalp to express surprise.
downward and forward and lodges the mylohyoid B. Orbicularis oculi encircles eye, closes eyelid; orbicu-
vessels and nerve. laris oris encircles mouth, closes lips.
5. Alveolar process: contains roots of the mandibular C. Corrugator supercilii is deep to superior part of orbi-
teeth; ramus extends upward and backward from cularis oculi muscle; originates on frontal bone and
the body on each side and terminates in coronoid inserts into skin tissue of eyebrow; causes vertical
process and coronoid notch. wrinkles in forehead (frowning).
6. Body: inferior to alveolar process. D. Buccinator forms anterior part of cheek; originates
7. External oblique line (ridge): outer crest where ra- from alveolar processes of maxilla, mandible, and
mus joins the body; ramus extends from angle to pterygomandibular raphe (ligament that extends from
condyle and articulates with TMJ; mandibular notch hamulus and attaches to mylohyoid line); pulls angle
is located between coronoid process and condyle. of mouth laterally and shortens cheek BOTH vertically
8. Genial tubercles: midline of medial surface; retro­ and horizontally; keeps food in CORRECT ­ position
molar triangle is at lateral end of each alveolar during chewing.
process. E. Risorius originates from fascia superficial to masseter
9. Internal oblique (mylohyoid) line (ridge): ex- and inserts in skin tissue at angle of mouth; widens
tends posteriorly and superiorly on inner surface; mouth to stretch lips.
Head, Neck, and Dental Anatomy   89

angle of upper lip and pulls it laterally; zygomaticus


minor originates on zygomatic bone and inserts in
skin tissue of upper lip; raises upper lip for smiling.
I. Levator anguli oris originates superior to root of max-
illary canine teeth and inserts in skin tissues at angle
1
of mouth; raises angle of mouth; depressor anguli oris
originates on mandible and inserts in skin tissue at
2 angle of mouth; lowers angle of mouth for smiling
and then frowning.
3
J. Depressor labii inferioris originates from mandible and
4 inserts in skin tissues of lower lip; lowers lower lip.
K. Mentalis originates near mandible midline and inserts
5 in skin tissue of chin; raises chin, causing lower lip
6
protrusion and narrowing of oral vestibule.
L. Platysma runs from the neck to the mouth, covering
7
10 anterior cervical triangle; originates in skin tissue su-
perficial to the clavicle and shoulder and inserts on
11 mandible and muscles surrounding mouth; raises skin
8
12 of the neck and pulls down the corner of mouth (mak-
ing anyone a Mr. Grimace, so stretch it out now as
9 you study anatomy).

Muscles of Mastication
Muscles of mastication are paired muscles that attach to
mandible and work with TMJ to accomplish movements
of mandible: depression, elevation, protrusion, retraction,
lateral deviation (Figures 4-12 and 4-13). Innervated by
12 Mentalis
11 Depressor labii inferioris mandibular division of fifth (V) cranial nerve (trigeminal).
10 Orbicularis oris Include masseter, temporalis, medial pterygoid, lateral ptery-
goid. Involved in temporomandibular joint disorder (TMD).
See later discussion on occlusion in regard to masseter.
Depressor anguli oris 9

Platysma 8 • See Chapter 6, General and Oral Pathology: TMD dis-


Buccinator 7 cussion.
A. Masseter: MOST superficially located and one of
strongest muscles; located anterior to parotid gland:
Levator anguli oris 6
Zygomaticus major 5 1. Two heads, superficial and deep, originate from
Zygomaticus minor 4 different areas of zygomatic arch and insert on
mandible (two heads are better than one).
2. Superficial head inserts on lateral surface of angle;
Levator labii superioris 3

Levator labii superioris alaeque nasi 2 deep head inserts on ramus.


Orbicularis oculi 1 3. When bilateral contraction occurs during closing
of jaws, mandible raised.
Figure 4-11  Muscles of facial expression. (From Fehrenbach 4. Can become increased in size (enlarged) with
MJ, ed: Dental anatomy coloring book, St. Louis, 2008, Saunders/
clenching and/or grinding (bruxism) of teeth.
Elsevier.)
B. Temporalis: originates from entire temporal fossa, in-
serts on coronoid process of mandible:
F. Levator labii superioris originates from infraorbital 1. When entire muscle contracts during closing of
region of maxilla and inserts in skin tissue of upper jaws, mandible raised.
lip; raises upper lip. 2. When only posterior part contracts, muscle moves
G. Levator labii superioris alaeque nasi originates from mandible backward, causing retraction of jaw
frontal process of maxilla and inserts into skin tissue (usually during jaw closing).
of ala of nose and upper lip; raises upper lip and di- C. Medial pterygoid: SIMILAR to but deeper to masseter:
lates ala of nose for sneering. 1. Originates from pterygoid fossa, on medial surface
H. Zygomaticus major originates from zygomatic bone of lateral pterygoid plate; inserts on medial surface
and inserts in skin tissue at angle of mouth; raises of angle of mandible.
90   Saunders Review of Dental Hygiene

3
1 3
2 4

Coronoid process of mandible 3


Superficial head of masseter 4 Temporalis 2
Zygomatic arch 3 Inferior temporal line 1
Ramus of mandible 2
Figure 4-13  Muscles of mastication. (From Fehrenbach
Deep head of masseter 1 MJ, ed: Dental anatomy coloring book, St. Louis, 2008, Saunders/
Elsevier.)
Figure 4-12  Muscles of mastication. (From Fehrenbach MJ,
ed: Dental anatomy coloring book, St. Louis, 2008, Saunders/Elsevier.)
2. Contraction of anterior muscles causes mandible
to lower and jaws to open when hyoid is stabilized
2. Raises mandible during closing of jaws. by contraction of both posterior suprahyoid and
D. Lateral pterygoid: two separate heads of origin, su- ­infrahyoid muscles.
perior and inferior, separated by slight interval ante- 3. Categorized according to anterior or posterior po-
riorly; heads fuse posteriorly, within infratemporal sition relative to hyoid:
fossa, deep to temporalis: a. Digastric: anterior and posterior bellies (sepa-
1. Superior head originates from greater wing of rate); anterior belly is part of anterior supra-
sphenoid; inferior head originates from lateral hyoids; posterior belly is part of posterior
pterygoid plate; BOTH heads unite and insert on suprahyoids:
mandibular condyle. (1) Each muscle demarcates superior part of
2. Has tendency to lower mandible during opening of anterior cervical triangle, forming (with the
jaws; when BOTH muscles contract, serve to bring mandible) submandibular triangle on each
mandible forward, causing protrusion of mandible side of neck.
(during opening); when ONLY one is contracted, (2) Right and left anterior bellies form midline
mandible shifts to opposite side, causing lateral submental triangle; anterior belly arises
deviation of mandible. from intermediate tendon and inserts close to
symphysis on inner surface of mandible,
Hyoid Muscles and post­erior belly arises from mastoid
Hyoid muscles are located superficially in neck tissues, process and inserts on intermediate tendon;
attached to hyoid. Assist in the actions of mastication and anterior belly innervated by mylohyoid
swallowing (Figure 4-14). nerve; posterior belly innervated by poste-
• See Chapter 3, Anatomy, Biochemistry, and Physiol- rior digastric nerve.
ogy: hyoid. b. Mylohyoid: located deep to the digastric; its fi-
A. Suprahyoids: paired that are located superior to bers run transversely between the two sides of
­hyoid: mandible, where it originates from mylohyoid
1. Act to raise hyoid and larynx by contraction of line (on inner surface of mandible):
muscles of mastication when mandible is stabi- (1) Unite medially, forming floor of mouth, and
lized, which occurs during swallowing. most posterior fibers insert on body of hyoid.
Head, Neck, and Dental Anatomy   91

8
1

2 9
3
4
10
5
11

Sternothyroid 6
Thyrohyoid 5
11 Sternohyoid
of omohyoid
10 Thyroid cartilage Superior belly 4

Posterior belly of digastric 9 Hyoid bone 3

Anterior belly of digastric 8 Stylohyoid 2

Inferior belly of omohyoid 7 Mylohyoid 1

Figure 4-14  Hyoid muscles. (From Fehrenbach MJ, ed: Dental anatomy coloring book, St. Louis, 2008,
Saunders/Elsevier.)

(2) Helps raise tongue; also raises hyoid bone and inserts on thyroid cartilage and lowers thyroid
or lowers mandible; innervated by mylohy- cartilage and larynx; does NOT directly lower
oid nerve. ­hyoid.
c. Stylohyoid: anterior and superficial to posterior 2. Sternohyoid: superficial to sternothyroid, thyroid,
belly of digastric: cartilage; originates from sternum and inserts on
(1) Originates from styloid process of temporal hyoid.
bone and inserts on body of hyoid. 3. Omohyoid: lateral to BOTH sternothyroid and
(2) Innervated by stylohyoid nerve. thyrohyoid; has superior and inferior bellies:
d. Geniohyoid: originates from medial surface of a. Inferior belly: originates from scapula and
mandible: passes beneath SCM, where it attaches by short
(1) Inserts near the symphysis at genial tub­ tendon to superior belly.
ercles. b. Superior belly: originates from short ten-
(2) Innervated by first cervical nerve by way of don attached to inferior belly and inserts on
twelfth (XII) cranial nerve (hypoglossal). hyoid.
B. Infrahyoids: paired that are located inferior to hyoid; 4. Thyrohyoid: covered by omohyoid and ster-
innervated by second and third cervical nerves; gen- nohyoid; originates from thyroid cartilage and
erally act to lower hyoid unless indicated: inserts on hyoid; appears as a continuation of
1. Sternothyroid: superficial to thyroid gland, deep sternothyroid; raises thyroid cartilage and larynx
and medial to sternohyoid; originates from ­sternum in addition to lowering hyoid.
92   Saunders Review of Dental Hygiene

4
3
1

5
4
1
2
6
5 7
2
3
8

Angle of mandible 6 Lateral pterygoid muscle 8


Stylomandibular ligament 5 Lower synovial cavity 7
Styloid process of temporal bone 4 Joint disc 6
Spine of sphenoid bone 3 Articular eminence 5
Sphenomandibular ligament 2 Upper synovial cavity 4
Joint capsule 1 Condyle 3
Blood vessels 2
Postglenoid process 1

Figure 4-15  Temporomandibular joint. (From Fehrenbach MJ, ed: Dental anatomy coloring book, St. Louis,
2008, Saunders/Elsevier.)

Muscles of the Tongue ­Patients may experience a temporomandibular joint dis-


Tongue performs complex movements during mastica- order (TMD) of one or BOTH of these joints. Bones of
tion, speaking, and swallowing as a result of combined the TMJ are discussed in skeletal system section.
action of muscles. Muscles of the tongue are divided into A. Joint capsule: completely encloses TMJ; membranes
intrinsic and extrinsic. Muscles innervated by twelfth that line inside of joint capsule secrete synovial fluid,
(XII) cranial nerve (hypoglossal). which helps lubricate joint.
A. Intrinsic: located mainly inside the tongue; change B. Disc: located between temporal bone and condyle,
shape of tongue: completely divides the TMJ into two synovial cavi-
1. Named according to their orientation. ties, upper and lower, filled with synovial fluid:
2. Include superior longitudinal, transverse, vertical, 1. Attached to mandibular condyle anteriorly and NOT
inferior longitudinal. to temporal bone, except indirectly through capsule.
B. Extrinsic: originate outside the tongue, yet insert in- 2. Divided posteriorly into two areas where it blends
side tongue: with capsule: upper division is attached to post-
1. Move the tongue while suspending and anchoring glenoid process, lower division is attached to
tongue to mandible, styloid process, hyoid. ­condyle.
2. Include genioglossus, styloglossus, hyoglossus. 3. Nerves and blood vessels enter joint in posterior
area of attachment to capsule.
TEMPOROMANDIBULAR JOINT   C. Ligaments associated with TMJ:
Temporomandibular joint (TMJ) is located on each 1. TMJ ligament: on lateral side of each joint and re-
side of the head and allows movement of the mandible inforced capsule; prevents excessive retraction of
for speech and mastication (Figure 4-15). Innervated mandible.
by mandibular division of fifth cranial nerve (trigemi- 2. Stylomandibular: variable ligament that is formed
nal) and blood supply from the external carotid artery. by thickened cervical fascia; runs from styloid
Head, Neck, and Dental Anatomy   93

­process of temporal bone to angle of mandible; be- from joint; runs from angular spine of sphenoid
comes taut when mandible protrudes. to lingula; becomes taut when mandible protrudes
3. Sphenomandibular: NOT considered a part of and can prevent diffusion of local anesthetic agent
TMJ; on medial side of mandible, some distance during inferior alveolar nerve block.

CLINICAL STUDY  

Age 58 YRS SCENARIO

Sex ☐  Male   ☒  Female The first afternoon patient has not had a dental
examination since her last dentist retired 2 years
Height 5’6” ago. An initial extraoral examination indicates
Weight 75 LBS that her zygomatic region is firm and enlarged;
she currently has no pain or discomfort in the jaw
BP 115/68 area. She does admit to frequently grinding her
teeth at night.
Chief Complaint “My jaws really ache when I wake up.”

Medical History Hysterectomy 5 years ago


Hypothyroidism (no goiter formation)

Current Medications levothyroxine sodium (Synthroid)


1 mg qd

Social History Internal revenue investigator


Likes hockey

1. What caused the patient’s firmly enlarged zygomatic on side of the body; other arteries of the head and neck
region? Why is there no jaw discomfort except in the are symmetrically located on each side of body. Vas-
morning? cular system supplies tissues with nutrients. May be-
2. Does the condition of cheek firmness and enlarge- come compromised by disease process or during dental
ment require treatment? If so, what may be recom- ­procedure.
mended? • See Chapter 3, Anatomy, Biochemistry, and ­Physiology:
vascular system.
1. Patient’s firm zygomatic region and enlarged cheeks A. Subclavian:
are caused by an overuse of masseter muscle with 1. Located lateral to the common carotid artery.
grinding during sleep. The muscle underwent hyper- 2. Has branches that supply BOTH cranial struc-
trophy (enlargement by increase in size of cells, not tures.
number of cells). Jaw discomfort that is a dull ache B. Common carotid:
only upon awakening is indicative of nighttime brux- 1. Branchless and travels up the neck, lateral to the
ism (grinding) and/or clenching habit. That is why trachea and larynx, then to thyroid cartilage.
there is no pain, since now she is at an afternoon 2. Contained in a sheath beneath the SCM, along with
appointment. the internal jugular vein and vagus nerve, until it
2. Her teeth should be checked for the presence of oc- ends by dividing into internal and external carotid
clusal wear (attrition) and cusp fracture. The condition arteries at level of larynx.
may require noninvasive treatment initially with a flat- 3. Bifurcates just past location of carotid sinus, a swell-
plane splint (mouthguard), which prevents full closure ing (dilation); provides MOST reliable arterial pulse
and allows the masseter muscle to relax. It also helps (used during emergencies by trained personnel).
distribute forces and prevent further wear to the oc- C. Internal carotid:
clusal surfaces of the teeth. 1. After leaving common carotid, is hidden by the
SCM; has no branches in neck but continues adja-
ARTERIAL BLOOD SUPPLY OF THE HEAD cent to internal jugular vein within carotid sheath
AND NECK   to the skull base, where enters cranium to supply
MAJOR arteries that supply the head and neck are the intracranial structures.
subclavian and the common carotid. Their paths from 2. Source of ophthalmic artery, which supplies the
the heart to the head and neck are different, ­depending eye, orbit, lacrimal gland.
94   Saunders Review of Dental Hygiene

D. External carotid: 5
1. Arises from common carotid; supplies extracranial
tissues, including oral cavity. 1
2. Has anterior, medial, posterior, terminal branches.
3. Anterior branches of external carotid:
a. Superior thyroid branches into infrahyoid, ster­
2
nocleidomastoid, superior laryngeal, and superior
and inferior thyroid arteries, which supply tissues 6
3
inferior to hyoid, including infrahyoid muscles, 7
8
SCM, muscles of larynx, thyroid gland. 4
9
b. Lingual arises above superior thyroid, at level of
hyoid; travels anteriorly to tongue apex; ­supplies
10
tissues superior to hyoid, including suprahyoid
muscles and floor of mouth by the dorsal lingual,
deep lingual, sublingual, suprahyoid branches;
also ­supplies tongue.
c. Sublingual supplies mylohyoid muscle, sublin-
gual gland, floor of mouth; suprahyoid branch 11 12 13

supplies suprahyoid muscles.


d. Facial arises slightly superior to lingual as it
13 Mylohyoid

branches off anteriorly and has a complicated Branches of inferior alveolar 12 Mental
path; supplies the face in the oral, buccal, zy- 11 Incisive
gomatic, nasal, infraorbital, orbital regions with
its major branches, including ascending pala-
10 Inferior alveolar

tine, submandibular and submental, inferior Buccal 9


labial, superior labial, angular (may share com- Pterygoids 8
mon trunk with lingual).
4. Medial branch of external carotid includes ascend- Masseteric (cut) 7

ing pharyngeal artery and pharyngeal and menin- Middle meningeal (cut) 6
geal branches. Deep temporals 5
5. Posterior branches of external carotid:
a. Occipital includes muscular, SCM, auricular, (portion cut)

meningeal branches.
Posterior superior alveolar 4

b. Posterior auricular arises superior to the occipi- (branch of infraorbital)


tal branch and stylohyoid muscle, level with the
Anterior superior alveolar 3

tip of the styloid process. Infraorbital 2


6. Terminal branches of external carotid: Sphenopalatine (cut) 1
a. Superficial temporal arises within parotid gland
and may be clinically visible in temporal re- Figure 4-16  Maxillary artery. (From Fehrenbach MJ, ed: Den-
gion; transverse facial branch supplies parotid; tal anatomy coloring book, St. Louis, 2008, Saunders/Elsevier.)
middle temporal branch supplies temporalis;
frontal and parietal branches supply scalp.
b. Maxillary gives off many branches within in- canal; in canal, branches into mandibular pos-
fratemporal and pterygopalatine fossae, such as terior and alveolar (dental) branches to supply
middle meningeal and inferior alveolar arteries, teeth, periodontium, and associated gingiva.
and has branches near the muscles supplied, d. Mylohyoid arises before main artery enters
including deep temporal, pterygoid, masseteric mandibular canal by way of mandibular fora-
artery, and buccal muscles, then branches into men and travels in the mylohyoid groove to
inferior alveolar (IA), posterior superior alveo- supply floor of mouth and mylohyoid.
lar (PSA), infraorbital (IO), greater (GP) and e. Mental arises from main artery and then exits
lesser palatine (LP) ­(Figure 4-16). mandibular canal by way of mental foramen to
c. The IA arises from maxillary artery in infratem- supply the chin tissues; anastomoses with infe-
poral fossa, turns inferiorly to enter mandibular rior labial artery.
foramen, then enters mandibular canal and IA, f. Incisive branches off main artery and remains in
branches into mylohyoid before it enters the mandibular canal, divides into dental branches
Head, Neck, and Dental Anatomy   95

to supply mandibular anteriors and alveolar


branches to supply periodontium; anastomo-
ses with alveolar branches of incisive artery on
other side of mouth.
g. The PSA artery is given off just as maxillary
2
artery leaves infratemporal fossa and enters
pterygopalatine fossa, enters PSA foramina, and 7
6
gives off dental branches (to supply the maxil-
lary posteriors) and alveolar branches (to supply 8
periodontium and maxillary sinus); ­anastomoses 3
9
with anterior superior alveolar artery.
h. The IO artery branches from the maxillary in the 4
pterygopalatine fossa and may share common 1
5
trunk with the PSA artery; enters orbit through
10
the inferior orbital fissure and travels through
the IO canal; gives off orbital branches to the or-
bit; gives off anterior superior alveolar (ASA).
i. The ASA arises from IO artery and gives off
dental branches to supply the maxillary anteri-
ors and alveolar branches to supply periodon-
tium; anastomoses with the PSA; after giving
off these branches in infraorbital canal orbit,
emerges onto the face from IO foramen to IO
10 Internal jugular Submental 5

region of face; anastomoses with facial. Retromandibular 9 branches Inferior labial 4


j. The GP and LP arteries arise from maxillary in Pterygoid plexus of veins 8
Facial
Superior labial 3
the pterygopalatine fossa, which travels to the
palate through the pterygopalatine canal and
Cavernous venous sinus 7 Supraorbital 2

GP and LP foramina to supply hard and soft Ophthalmic 6 Facial 1


palate, respectively.
Figure 4-17  Venous drainage of head and neck. (From
k. Maxillary artery ends by becoming sphenopala- F­ ehrenbach MJ, ed: Dental anatomy coloring book, St. Louis, 2008,
tine, which supplies nasal cavity and gives rise Saunders/Elsevier.)
to posterior lateral nasal branches and septal
branches, including nasopalatine branch that
accompanies nasopalatine nerve through inci- 2. Receives branches from same facial areas supplied
sive foramen. by facial artery and anastomoses with deep veins,
such as the pterygoid plexus in infratemporal fossa
VENOUS DRAINAGE OF THE HEAD AND NECK   and retromandibular vein, before joining internal
Veins are symmetrically located but have greater variabil- jugular vein at level of hyoid.
ity in location than do arteries. Veins anastomose freely 3. Has oral tributaries:
and generally are larger and MORE numerous than arter- a. Superior labial, drains upper lip; inferior labial,
ies in same tissue. Internal jugular drains brain and other drains lower lip.
tissue; external jugular drains only some extracranial b. Submental, drains chin tissues and submandib-
tissues, with many anastomoses between them. Internal ular region.
and external jugular are major venous drainage vessels c. Lingual, including dorsal lingual, which drains
of head and neck (Figure 4-17). Leaving the head near dorsal surface of tongue; deep lingual veins,
the base of neck, veins become larger. Vascular system is which drain ventral surface; and sublingual
capable of spreading infection or cancerous cells in head veins, which drain floor of the mouth; may join
and neck area because valveless veins control direction to form a single vessel or may empty into larger
of blood flow. vessels separately, either indirectly into facial
A. Facial: includes superior labial, inferior labial, sub- or directly into internal jugular.
mental, lingual; drains into internal jugular: B. Retromandibular:
1. Begins at medial canthus, at junction of supra- 1. Formed by union of superficial temporal and max-
trochlear and supraorbital, which anastomoses with illary; emerges from parotid; drains areas similar
ophthalmic that drains orbit tissues and provides to those supplied by superficial temporal and max-
communication with cavernous venous sinus. illary arteries.
96   Saunders Review of Dental Hygiene

2. Divides below parotid; anterior division joins fa- B. Salivary glands (Figure 4-18):
cial, and posterior division continues and is then 1. Produce saliva, which lubricates and cleanses oral
joined by posterior auricular vein, which drains cavity and aids digestion; include BOTH major
scalp behind ear and becomes external jugular: and minor glands, defined by their size; exocrine
a. Superficial temporal drains lateral scalp; drains into glands with ducts that drain saliva directly into
and forms retromandibular, along with maxillary. oral cavity where it is used; controlled by ANS.
b. Maxillary: 2. With connective tissue of gland divided into cap-
(1) Begins in infratemporal fossa; drains ptery- sule, which surrounds outer part, and septa (plu-
goid plexus near maxillary artery and re- ral of septum), each septum helps divide inner
ceives middle meningeal, posterior superior part of the gland into larger lobes and smaller
alveolar, inferior alveolar, and other veins, lobules.
such as those from nose and palate (areas 3. Major glands are large paired glands; ducts are
served by maxillary artery). named after them; include parotid, submandibular,
(2) Merges with superficial temporal to drain sublingual.
into and form retromandibular. 4. Parotid:
(3) Can be pierced during PSA block, causing a. Largest encapsulated gland; provides only 25%
hematoma, when needle overreaches target of total volume; has purely serous secretion; di-
of apices of maxillary posteriors. vided into two lobes: superficial and deep.
C. Pterygoid plexus of veins: b. Parotid duct (Stensen’s), which emerges from
1. Collection of small anastomosing vessels around anterior border of gland, pierces buccinator,
pterygoid muscles and surrounding maxillary ar- then opens into oral cavity at parotid papilla;
tery, protecting it from compression in infratempo- occupies parotid fascial space, posterior to ra-
ral fossa; drains blood from maxillary and middle mus, anterior and inferior to the ear; extends
meningeal veins, which drain blood from menin- irregularly from zygomatic arch to angle of
ges and deep facial parts. mandible.
2. Anastomoses with BOTH facial and retromandibu- c. Innervated by parasympathetic nerves of the
lar veins; may be involved in spread of infection to otic ganglion of ninth cranial nerve (glosso-
cavernous venous sinus (needletrack infections); pharyngeal), as well as afferent nerves from
also drains PSA vein, formed by union of dental auriculotemporal branch of the fifth cranial
branches of maxillary teeth and alveolar branches nerve (trigeminal); drains into deep parotid
of periodontium. nodes; supplied by branches of external carotid
3. Also drains inferior alveolar (IA) vein, which is artery.
formed by union of dental branches of mandibu- d. Becomes enlarged and tender with mumps (uni-
lar teeth, alveolar branches of periodontium, and lateral or bilateral parotitis, inflammation of
mental branches that enter mental foramen after gland), viral disease that because of introduc-
draining chin area, where they anastomose with tion of a vaccine is NOT a common childhood
branches of facial vein. disease; the salivary gland MOST commonly
4. Can be pierced during PSA block, causing hema- involved in tumorous growth, which can change
toma, when needle overreaches target of apices of consistency and cause unilateral facial pain on
maxillary posteriors; can also involve serious in- involved side (seventh [VII] cranial nerve [fa-
fection if needle-track infection is involved (caus- cial] travels through gland).
ing cavernous sinus thrombosis). e. Trauma can also occur to nerve from accidental
overreaching of needle during inferior alveolar
GLANDULAR TISSUE   nerve block, causing unilateral transient fa-
Glandular tissues include the lacrimal, salivary, thyroid, cial paralysis, temporary loss of movement of
parathyroid, thymus glands. muscles of facial expression on affected side;
• See Chapters 6, General and Oral Pathology: glandular patient CANNOT close one eye, smiles asym-
diseases or conditions; 9, Pharmacology: secretions. metrically, has drooping lip on that side.
A. Lacrimal glands: 5. Submandibular:
1. Paired exocrine glands that secrete lacrimal fluid a. Second largest encapsulated gland; provides
(tears) for lubrication of conjunctiva, which leaves 60% to 65% of total volume with mixed secre-
gland through tubules. tion.
2. After passing over eyeball, drained through a hole b. Submandibular duct (Wharton’s), which arises
in each eyelid, gland terminates in nasolacrimal from deep lobe and remains inside mylohy-
sac, structure behind medial canthus. oid, travels along anterior floor of the mouth
Head, Neck, and Dental Anatomy   97

7
1

Sublingual ducts 7
Parotid papilla 6
Parotid duct 5
Parotid gland 4
Sublingual caruncle 3
Submandibular duct 2
Submandibular gland 1

Figure 4-18  Salivary glands. (From Fehrenbach MJ, ed: Dental anatomy coloring book, St. Louis, 2008,
Saunders/Elsevier.)

and then opens into oral cavity at the sublin- c. Innervated by parasympathetic fibers of chorda
gual caruncle; tortuous travel may be reason tympani and submandibular ganglion of sev-
that it is gland MOST often involved in stone enth (VII) cranial nerve (facial); drains into
­formation. submandibularnodes; supplied by sublingual
c. Occupies submandibular fossa in submandibu- and submental arteries.
lar fascial space, mostly superficial to the my- 7. Minor salivary glands:
lohyoid; deep lobe wraps around the posterior a. Smaller than major glands but MORE numer-
part and is posterior to the sublingual. ous, exocrine glands with unnamed ducts that
d. Innervated by parasympathetic fibers of chorda are shorter than major glands; scattered in buc-
tympani and submandibular ganglion of sev- cal, labial, and lingual mucosa, soft palate,
enth (VII) cranial nerve (facial); drains into lateral parts of hard palate, floor of the mouth;
submandibular nodes; supplied by branches of include von Ebner’s glands (associated with
facial and lingual arteries. circumvallate lingual papillae).
6. Sublingual: b. Secrete MAINLY mucous saliva, EXCEPT von
a. Smallest and ONLY unencapsulated gland; pro- Ebner’s glands, which secrete only serous se-
vides only 10% of total volume with mixed se- cretions.
cretion. c. Innervated by seventh (VII) cranial nerve (fa-
b. NOT just one major duct; sublingual ducts cial); drain into various lymph nodes; supplied
(Bartholin’s) open directly into oral cavity by various arteries.
through gland and have other ducts that open 8. Histology of salivary glands:
along the sublingual fold; located in sublingual a. Secretory cells: produce saliva; two types of
fossa in sublingual fascial space at the floor of epithelial cells in glands:
the mouth; superior to mylohyoid, medial to (1) Mucous cells: cloudier-looking cytoplasm;
body of mandible, anterior to submandibular. produce mucous secretory product.
98   Saunders Review of Dental Hygiene

(2) Serous cells: clearer cytoplasm; produce 1


­serous secretory product.
b. Acinus: single layer of cuboidal epithelial cells
surrounding lumen, where saliva is deposited
after being produced; MOST acini match with 2

type of cell, but mucoserous acini have mucous


cells surrounding lumen, with serous demilune
3
located superficially.
c. Myoepithelial cells: on surface of some acini to 4 6 8
help flow of saliva (squeeze play). 5
d. Duct system: intercalated duct is associated
9
with acinus, connected to striated duct, then ex- 7
cretory duct.
C. Thyroid gland (Figure 4-19): Esophagus 9
1. Located in anterior and lateral regions of neck, Parathyroid glands 8
inferior to thyroid cartilage, at junction between
larynx and trachea.
Trachea 7

2. Produces thyroxine, secreted directly into blood, Left lobe of thyroid gland 6
stimulating metabolic rate. Right lobe of thyroid gland 5
3. LARGEST endocrine gland; has NO ducts but is
encapsulated; consists of two lateral lobes, right
Isthmus 4

and left, connected anteriorly by isthmus. Cricoid cartilage 3


4. Innervated by sympathetic nerves through cervical Thyroid cartilage 2
ganglia; drains into superior deep cervical nodes;
supplied by superior and inferior thyroid arteries.
Hyoid bone 1

5. May become enlarged during disease process, pro- Figure 4-19  Thyroid and parathyroid glands. (From Feh-
ducing goiter. renbach MJ, ed: Dental anatomy coloring book, St. Louis, 2008,
D. Parathyroid glands (Figure 4-19): Saunders/Elsevier.)
1. Located close to or inside thyroid on posterior sur-
face; consist of four endocrine glands, two on each
side, with NO ducts; produce parathyroid hormone
secreted directly into blood to regulate calcium and 2. Part of immune system and fights disease process;
phosphorus levels. T-cell lymphocytes mature in gland in response to
2. Innervated by sympathetic nerves through cervical stimulation by thymus hormones.
ganglia; drain into superior deep cervical nodes; 3. Innervated by branches of tenth cranial nerve
supplied MAINLY by inferior thyroid arteries (vagus) and cervical nerves, with lymphatics that
3. May alter thyroid if involved in disease process. arise within the substance of gland and terminate
E. Thymus gland: in internal jugular vein; supplied by inferior thy-
1. Ductless endocrine gland located in thorax and an- roid and internal thoracic arteries.
terior region of base of neck, inferior to thyroid, 4. Grows in size from birth to puberty, then stops
deep to sternum; its muscles, superficial and lateral growing and starts to shrink; thyroid cancer may
to trachea, consist of two lateral lobes in close con- develop as a result of mistaken past radiation ther-
tact at midline. apy to shrink gland.
Head, Neck, and Dental Anatomy   99

clinical study  

Age 49 YRS SCENARIO

Sex ☒  Male   ☐  Female The intraoral examination of the patient


reveals a number of carious lesions, and the
Height 6‘5” oral mucosa appears quite dry.
Weight 175 LBS

BP 112/62

Chief Complaint “My mouth is so sore when I eat!”

Medical History Recent radiation therapy for low-grade


adenoid cystic carcinoma of the submandi­bular
gland after surgical removal

Current Medications None

Social History Basketball coach at high school


Past history of spit tobacco use

1. What is the cause of the patient’s dry mouth? What submandibular fascial space, mostly superficial to the
type of tumor is this? Why is the patient not taking any mylohyoid; deep lobe wraps around the posterior part
medications? and is posterior to the sublingual.
2. What are recommendations for treatment of dryness?
3. What types of cells were most damaged during radia- TRIGEMINAL NERVE AND SENSORY ROOT  
tion treatment provided? Trigeminal nerve (fifth cranial nerve [V]) is formed by
ophthalmic, maxillary, mandibular branches.
1. Primary cause of xerostomia is radiation treatment the • See Chapters 3, Anatomy, Biochemistry, and Physiol-
patient received for tumor. Radiation treatment causes ogy, and 14, Pain Management: nervous system, oral
glandular tissue fibrosis or atrophy, resulting in par- cavity nerves; 6, General and Oral Pathology: trigemi-
tial or total loss of secretory function of the subman- nal neuralgia.
dibular salivary gland (hypofunction). Submandibular A. Ophthalmic division (ophthalmic nerve, V1) is FIRST
is the second largest encapsulated gland; provides and smallest division, carried by way of superior or-
60% to 65% of total saliva volume with mixed secre- bital fissure; includes frontal, lacrimal, nasociliary.
tion, thus most of the volume of saliva. Discomfort B. Maxillary division (maxillary nerve, V2) of fifth cra-
and difficulty speaking and swallowing are common nial nerve (trigeminal) (Figure 4-20):
complaints of those suffering from inadequate sa- 1. Second division of trigeminal, with nerve trunk
liva production. Adenoid cystic carcinoma (ACC) formed in pterygopalatine fossa by convergence of
is uncommon except in salivary glands, is usually a MANY nerves; enters skull through foramen ro-
painless, slow-growing mass, and often appears to tundum; contains branches: zygomatic, infraorbital
be a low-grade tumor; hard to get rid of completely (IO), anterior superior alveolar (ASA), middle su-
and very often comes back after surgery. There is no perior alveolar (MSA), posterior superior alveolar
effective medication or chemotherapy for this type of (PSA), greater palatine (GP), lesser palatine (LP),
tumor. nasopalatine (NP).
2. Patient may use salivary substitute throughout the 2. Zygomatic: union of zygomaticofacial and zygo-
day to provide moisture for the mouth when speaking maticotemporal in orbit and is afferent; conveys
and swallowing. Sipping water frequently, sucking on postganglionic parasympathetic fibers to lacrimal
sugar-free lozenges, and humidifying air in the home gland; courses posteriorly along lateral orbit floor,
have also been found to be beneficial. Daily use of enters pterygopalatine fossa through inferior or-
mineralizing fluoride and calcium product helps pre- bital fissure, then joins maxillary.
vent formation of caries. 3. The IO: passes into IO foramen; then travels
3. Secretory cells that make up submandibular glands through IO canal, along with infraorbital blood
are serous and mucous cells, since it is a mixed gland, vessels; then joined by ASA, which is afferent and
and both types of cells can be damaged by radiation is formed by a union of cutaneous branches from
treatment. Gland occupies submandibular fossa in the upper lip, medial part of cheek, lower eyelid,
100   Saunders Review of Dental Hygiene

and interradicular branches, forming dental


plexus; all internal branches exit from PSA
foramina.
b. BOTH external and internal branches ascend
along maxillary tuberosity, which forms in pos-
terolateral wall of maxillary sinus, to join IO or
1
maxillary.
5 7. Greater palatine (GP, anterior palatine): located
2 6 between mucoperiosteum and bone of anterior
hard palate; communicates with terminal fibers of
3 7 nasopalatine (NP):
a. Enters GP foramen in palatine bone to travel
4 8
in pterygopalatine canal, along with GP blood
vessels and lesser palatine (LP) and blood ves-
sels, ascends through pterygopalatine canal, to-
ward maxillary nerve in pterygopalatine fossa;
afferent for posterior hard palate and posterior
Dental plexus 4 lingual gingiva.
b. The LP (posterior palatine): afferent for soft
Middle superior alveolar 8
alveolar
Posterior superior alveolar 7 Anterior superior 3 palate and palatine tonsillar tissues; enters LP
Greater and lesser palatine 6 Infraorbital 2 foramen in the palatine bone, along with LP
blood vessels; then joins GP nerve and blood
vessels in pterygopalatine canal; then ascends
Pterygopalatine ganglion 5 Zygomatic 1

Figure 4-20  Maxillary division of trigeminal nerve. (From through pterygopalatine canal toward maxillary
Fehrenbach MJ, ed: Dental anatomy coloring book, St. Louis, 2008, nerve in pterygopalatine fossa.
Saunders/Elsevier.) 8. The NP: originates in the mucosa of anterior hard
palate; afferent for anterior hard palate, lingual
gingiva of maxillary anteriors, nasal septal tissues;
side of nose; then passes from IO canal and groove communicates with GP; has right and left nerves;
into pterygopalatine fossa through inferior orbital BOTH enter incisive canal by way of incisive fora-
fissure; after leaving IO groove, and within ptery- men; then travel along nasal septum.
gopalatine fossa, receives PSA. C. Mandibular division (mandibular nerve, V3) of fifth
4. The ASA: ascends along anterior wall of maxil- cranial nerve (trigeminal) (Figure 4-21):
lary sinus to join IO in IO canal; afferent for max- 1. Largest of three divisions that form trigeminal and
illary anteriors by way of dental and interdental innervates ALL muscles of mastication:
branches; forms dental plexus and innervates a. Has meningeal and muscular branches, which
overlying facial gingiva; crossover can occur over arise from trunk before separation into two
midline. trunks; main trunk is formed by union of ante-
5. The MSA: afferent for maxillary premolar teeth rior and posterior trunks in infratemporal fossa,
and possibly mesial buccal (MB) root of maxillary before passing through foramen ovale; muscu-
first molar; originates from dental, interdental, in- lar branches arise from motor root of trigeminal;
terradicular branches, forming dental plexus; then have deep temporal nerves that are efferent for
ascends to join IO by running in lateral wall of temporalis and include masseteric nerve, which
maxillary sinus and communicates with BOTH the is efferent for masseter, and sensory branch to
ASA and PSA; may or may not be present; if NOT TMJ and lateral pterygoid nerve, which is effer-
present, area is innervated by BOTH the ASA and ent for lateral pterygoid.
PSA, but MAINLY the ASA. b. Anterior trunk formed by union of buccal nerve
6. The PSA joins IO (or maxillary directly) in the and muscular branches; posterior formed by
pterygopalatine fossa; afferent for most parts of union of auriculotemporal, lingual, inferior al-
maxillary molar teeth (possibly NOT MB root of veolar (IA); mandibular division joins ophthal-
maxillary first molars) and maxillary sinus: mic and maxillary to form trigeminal ganglion;
a. Some branches remain external to posterior includes buccal, muscular, lingual, IA, mental,
surface of maxilla and provide innervation incisive, mylohyoid, and auriculotemporal,
for buccal gingiva overlying maxillary mo- which is efferent for the external ear and scalp;
lars; ­others originate from dental, interdental, then joins posterior trunk.
Head, Neck, and Dental Anatomy   101

7 8 9

10
11
2

4
5 12

12 Branches to tongue Submandibular ganglion 6

11 Nerve to medial pterygoid muscle Mylohyoid 5

10 Nerve to tensor veli palatini muscle Lingual 4

Pterygopalatine ganglion 9 Inferior alveolar 3

Otic ganglion 8 Auriculotemporal 2


Motor root of trigeminal nerve 7 Middle meningeal 1

Figure 4-21  Mandibular division of trigeminal nerve. (From Fehrenbach MJ, ed: Dental anatomy coloring
book, St. Louis, 2008, Saunders/Elsevier.)

2. Buccal (long buccal): not to be confused with buccal 5. Lingual:


nerve that serves the buccinator muscle of the cheek: a. Formed from afferent branches from the body
a. Located on surface of buccinator and travels of tongue that travel along its lateral surface;
posteriorly in cheek, deep to masseter, then afferent for general sensation of body of tongue,
crosses in front of the anterior border of the ra- floor of the mouth, lingual gingiva of mandibu-
mus, between two heads of lateral pterygoid, to lar teeth.
join anterior trunk. b. Then passes posteriorly from medial to the lat-
b. Afferent for skin of cheek, buccal mucous mem- eral side of submandibular gland and commu-
branes, buccal gingiva of mandibular posteriors. nicates with submandibular ganglion located
3. Mental: external branches that are afferent for chin, superior to deep lobe of submandibular gland,
lower lip, and labial mucosa near mandibular an- which is part of the parasympathetic efferent
teriors; then enters mental foramen to merge with innervation for sublingual and submandibular
incisive to form IA in mandibular canal. glands (chorda tympani travels along with lin-
4. Incisive: afferent; composed of dental branches from gual nerve).
the mandibular anteriors and interdental branches, c. At the base of tongue, ascends and runs between
which form dental plexus; then merges with mental, medial pterygoid and mandible, anterior and
just posterior to mental foramen, to form the IA in slightly medial to IA, then continues to travel
mandibular canal; crossover can occur over midline. upward to join posterior trunk.
102   Saunders Review of Dental Hygiene

6. The IA: travels lateral to m­edial pterygoid, between


a. Afferent for mandibular posteriors, formed from sph­enomandibular ligament and ramus and
union of mental and incisive, then continues to within pterygomandibular space, posterior
travel posteriorly through mandibular canal, and slightly lateral to lingual; then joins pos-
along with IA blood vessels. terior trunk.
b. Joined by dental, interdental, and interra- 7. Mylohyoid: branch of IA nerve after it exits man-
dicular branches from the mandibular pos- dibular foramen, then pierces sphenomandibular
teriors, forming a dental plexus; then exits ligament; runs in mylohyoid groove and then onto
mandible through mandibular foramen, lower surface of the mylohyoid; efferent for mylo-
where it is joined by mylohyoid nerve; then hyoid and anterior belly of digastric.

CLINICAL STUDY  

Age 25 YRS SCENARIO

Sex ☒  Male   ☐  Female The dental hygienist previously performed a localized


scaling of #14 and #15, with local anesthesia. Since
Height 5‘8” the area continued to be sore at the patient’s next
Weight 190 LBS appointment 2 weeks later, the supervising dentist
referred him to an oral surgeon, with tooth #16
BP 118/78 circled on the referral form. However, after extraction
1 week ago, he is now presenting with a fever and
Chief Complaint “My upper left jaw is still really sore.” swollen lymph nodes and poor oral hygiene at extrac-
tion site. Antibiotics have been prescribed.
Medical History Laser eye surgery 2 years ago
Broken nose as child

Current Medications None

Social History Computer graphics engineer

1. What three local anesthesia blocks would have to have 2. Tooth #16 (wisdom tooth, maxillary left third molar)
been done to make the patient completely anesthetized was extracted. Crown has no standard form; smallest
for his localized scaling? molar and most variable in shape in permanent denti-
2. Which tooth was extracted by oral surgeon? Which tion. Occlusal table with heart-shaped occlusal outline
nerves should be anesthetized to extract tooth #16 is most common; similar to maxillary second molar
comfortably? but with more supplemental grooves; has only three
3. What types of roots might oral surgeon encounter cusps (MB, DB, and ML). With rhomboid occlusal
when extracting tooth #16? From which bone and its outline, has four cusps; no oblique ridge on small DL
process did the extraction of #16 take place? cusp. Both PSA and GP nerves must be anesthetized to
4. Which blood vessels supply the area of extraction extract tooth #16.
site? 3. Tooth #16 was extracted from alveolar process of max-
5. Which primary and secondary lymph nodes might be illa, the tooth-bearing part of upper jaw where each
involved in infection of tooth #16? alveolus (tooth socket) is located. Posterior superior
alveolar artery (branch off maxillary artery) and vein
1. For the patient to be completely anesthetized on (begins as maxillary vein) supply tooth #16 and area.
#14 and #15, the following nerve blocks would be 4. Roots of #16, as third molar, may be partially or fully
needed: posterior superior alveolar (PSA) and mid- fused, poorly developed, and/or curved distally; may
dle superior alveolar (MSA) (both used for anesthe- have accessory root. May even have dilaceration, a cur-
sia of tooth and its pulp, associated periodontium, vature of the root that is developmental anomaly and
overlying buccal gingiva), as well as greater palatine may be caused by trauma during tooth development;
(GP) (for overlying anesthesia of lingual gingiva). this presents the greatest challenge to the oral surgeon.
All nerves anesthetized are branches of maxillary 5. Primary lymph nodes that directly drain tooth #16 are
nerve (maxillary division, V2) of fifth (V) cranial superior deep cervical nodes, located deep beneath
nerve (trigeminal). sternocleidomastoid muscle (SCM), superior to where
Head, Neck, and Dental Anatomy   103

omohyoid muscle crosses internal jugular vein. Drain muscle, and other two nerves supply stylohyoid
posterior nasal cavity, posterior part of hard palate, muscle and posterior belly of the digastric, respec-
soft palate, base of tongue, maxillary third molars, tively.
esophagus, trachea, thyroid gland. Then drain into 5. Branches to muscles of facial expression are ef-
secondary nodes, inferior deep cervical nodes, or can ferent branches of facial, which originate in pa-
drain directly into jugular trunk. rotid gland and pass to the muscles innervated;
include temporal, zygomatic, buccal, (marginal)
FACIAL NERVE   mandibular, cervical; RARELY seen as five in-
Facial nerve (seventh cranial [VII]) innervates muscles dependent nerves; vary in number and connect
of facial expression. Carries BOTH efferent and affer- irregularly.
ent nerves. Branches include greater petrosal, chorda
tympani, posterior auricular, stylohyoid, posterior di- LYMPH NODES OF HEAD AND NECK  
gastric. Lymph nodes are part of immune system and help to fight
• See Chapters 6, General and Oral Pathology: Bell’s disease. Network of lymphatic vessels that link lymph
palsy, CVA; 14, Pain Management: transient facial nodes throughout MOST of body. NOT palpable unless
­paralysis. involved in a disease state.
A. Facial: emerges from brain and enters internal acous- • See Chapters 3, Anatomy, Biochemistry, and Physiol-
tic meatus; gives off efferent branches to middle ogy: lymphatics; 11, Clinical Treatment: extraoral ex-
ear muscle and greater petrosal and chorda tympani amination.
nerves, which carry parasympathetic fibers. A. Lymph nodes of head are either superficial or deep:
B. Main trunk then emerges from skull through stylo- 1. Superficial nodes of head (Figure 4-22):
mastoid foramen, gives off posterior auricular nerve a. Occipital: bilaterally in occipital region; drain
and branch to posterior belly of digastric and stylohy- part of scalp, then empty into inferior deep cer-
oid muscles, then passes into parotid gland and forms vical nodes.
numerous branches to supply ALL muscles of facial b. Retroauricular (mastoid, posterior auricular):
expression (but NOT to the parotid): posterior to ears; drain surrounding area, then
1. Greater petrosal: branches off facial before exit- empty into superior deep cervical nodes.
ing skull; has efferent fibers that are preganglionic c. Anterior auricular: anterior to each ear; drain
parasympathetic fibers to pterygopalatine ganglion surrounding area, then empty into superior deep
in pterygopalatine fossa. cervical nodes.
2. Has postganglionic fibers that arise in pterygo- d. Superficial parotid (paraparotid): superficial to
palatine ganglion, then join with branches of parotids; drain surrounding area, then empty into
maxillary division of trigeminal, carried to lac- superior deep cervical nodes (anterior auricular
rimal gland (by way of zygomatic and lacrimal and superficial parotid may be grouped).
nerves), nasal cavity, minor salivary glands of e. Facial: along length of facial vein to drain area
hard and soft palate; afferent for taste sensation with descending subgroups: malar (infraorbital),
in palate. nasolabial, buccal, mandibular; drain into each
3. Chorda tympani: branch of facial and is parasym- other, then into submandibular nodes.
pathetic; efferent for submandibular and sublingual 2. Deep nodes of head (too deep to palpate):
glands and afferent for taste sensation for body of a. Deep parotid: deep in gland; drain middle ear,
tongue: auditory tube, gland, then empty into superior
a. After branching off facial, within petrous part deep cervical nodes.
of temporal bone, crosses tympanic membrane b. Retropharyngeal: near deep parotid; drain
(eardrum); exits skull by way of petrotympanic pharynx, palate, paranasal sinuses, nasal cav-
fissure, which is located immediately posterior ity; then empty into superior deep cervical
to TMJ. nodes.
b. Then travels with lingual along floor of the B. Cervical nodes: in neck, divided into superficial
mouth in same nerve bundle; in submandibu- and deep.
lar triangle, appears alongside of lingual nerve 1. Superficial cervical nodes (Figure 4-23):
and has communication with submandibular a. Submental: inferior to chin in submental fascial
ganglion. space, superficial to the mylohyoid, near mid-
4. Posterior auricular, stylohyoid, posterior digastric line; drain BOTH sides of chin, lower lip, floor
nerves: given off by facial nerve after it exits sty- of the mouth, apex of tongue, mandibular in-
lomastoid foramen; are all efferent; posterior au- cisors, then empty into submandibular or deep
ricular nerve supplies occipital belly of epicranial cervical nodes.
104   Saunders Review of Dental Hygiene

4
2
3 5

Anterior auricular node 6

Occipital nodes 5

Retroauricular nodes 4

Facial nodes 3

Superficial parotid nodes 2

Facial nodes 1

Figure 4-22  Superficial nodes of the head. (From Fehrenbach MJ, ed: Dental ­ anatomy coloring book,
St. Louis, 2008, Saunders/Elsevier.)

b. Submandibular: inferior border of the ramus, 2. Deep cervical nodes (Figure 4-24):
superficial to submandibular gland and within a. Superior deep cervical: deep beneath SCM, supe-
submandibular fascial space; drain the cheeks, rior to omohyoid muscle crosses internal jugular
upper lip, body of tongue, anterior part of hard vein; drain posterior nasal cavity, posterior part of
palate, and ALL teeth (EXCEPT mandibular hard palate, soft palate, base of tongue, maxillary
incisors and maxillary third molars); may be third molars, esophagus, trachea, thyroid gland:
secondary nodes for the submental and facial (1) May be secondary nodes for ALL other
regions because they also drain sublingual and nodes of head and neck (EXCEPT occipi-
submandibular glands and then empty into su- tal and inferior deep cervical nodes), then
perior deep cervical nodes. empty into inferior deep cervical nodes or
c. External jugular (superficial lateral cervical): directly into jugular trunk.
on side of the neck along external jugular vein, (2) Jugulodigastric (tonsillar or sentinel node):
superficial to SCM; may be secondary nodes for below posterior belly of digastric; EASILY
the occipital, retroauricular, anterior auricular, becomes palpable when palatine tonsils and
superficial parotid; then empty into the superior pharynx are inflamed.
or inferior deep cervical nodes. b. Inferior deep cervical: continuation of supe-
d. Anterior jugular (superficial anterior cervical): rior deep cervicals, located deep to SCM, at
on side of the neck, along the length of the ante- level where omohyoid crosses internal jugular
rior jugular vein, anterior to SCM; drain infra- vein or inferior to this point; extend inferiorly
hyoid region of neck, then empty into inferior into supraclavicular fossa, superior to each
deep cervical nodes. clavicle.
Head, Neck, and Dental Anatomy   105

1
2
4

External jugular nodes 4

Anterior jugular nodes 3

Submental nodes 2

Submandibular nodes 1

Figure 4-23  Superficial cervical nodes. (From Fehrenbach MJ, ed: Dental anatomy coloring book, St. Louis,
2008, Saunders/Elsevier.)

(1) Drain posterior part of scalp and neck, su- A. Palatine tonsils: two rounded masses of variable
perficial pectoral region, part of arm; may size, between anterior and posterior tonsillar pillars
be secondary nodes for occipital and supe- (Figure 4-1).
rior deep cervical. B. Lingual tonsil: indistinct layer of lymphoid tissue lo-
(2) Jugulo-omohyoid: located at crossing of cated on base of dorsal surface of tongue (Figure 4-1).
omohyoid and internal jugular vein; drains C. Pharyngeal tonsils (adenoids): on posterior wall of na-
tongue and submental region. sopharynx; normally temporarily enlarged in children.
3. Accessory: along accessory nerve; drain sc­alp and D. Tubal tonsils: in nasopharynx, posterior to openings
neck, then empty into supraclavicular nodes. of eustachian (auditory) tubes.
4. Supraclavicular (transverse cervical): located
along clavicle; drain lateral cervical triangles, then clinical study  
empty into either one of jugular trunks or into right Scenario: An intraoral examination of the 12-year-old
lymphatic or thoracic duct; communicate with ax- patient reveals a 6-mm circular vesicle on the hard palate,
illary lymph nodes that drain breast region (may be near his maxillary anteriors. Bilaterally enlarged tonsils,
enlarged and hard if involved with breast cancer). near the throat, are also noted. He has a cold and is on
C. Efferent vessels form jugular trunk, one of tributaries cough medicine.
of right lymphatic duct (on right side) and thoracic
duct (on left). 1. Which nerves are involved in the patient’s discomfort
from the blister?
Tonsillar Tissue 2. Which bone underlies the tissue in the sore area?
Tonsils are masses of lymphoid tissue that drain into the 3. Identify the unique intraoral landmark located near or
superior deep cervical nodes, especially jugulodigastric within the sore area.
(tonsillar or sentinel) lymph node. Includes palatine, lin- 4. Which tonsils are involved in the enlargement? Which
gual, pharyngeal, tubal tonsils. Enlargement (lymphade- primary lymph nodes would one expect to find en-
nopathy) occurs in disease states. larged during the extraoral examination?
106   Saunders Review of Dental Hygiene

1 5

Supraclavicular node 6

Accessory nodes 5

Inferior deep cervical nodes 4

Jugulo-omohyoid node 3

Superior deep cervical nodes 2

Jugulodigastric node 1

Figure 4-24  Deep cervical nodes. (From Fehrenbach MJ, ed: Dental anatomy coloring book, St. Louis, 2008, Saunders/Elsevier.)

1. Nasopalatine nerve is involved in patient’s discomfort • See Chapters 6, General and Oral Pathology: dental
from vesicle (blister) from the thermal burn. disorders; 11, Clinical Treatment: dental charting.
2. Maxillary bone underlies soft tissues in burn area. A. Primary (deciduous) dentition is the FIRST dentition;
3. Unique intraoral landmark located near or within burn consists of 20 teeth (8 incisors, 4 canines, and 8 ­molars).
ar­ea is incisive papilla, centrally located on the hard 1. Are exfoliated (shed) and replaced by permanent
palate, around 10 mm posterior to the maxillary ­central dentition (Figure 4-25).
incisors. 2. Begins on average with eruption of the mandibular
4. Palatine tonsils are enlarged between anterior and pos- central incisor at 6 to 10 months and is completed
terior tonsillar pillars. Palpation of neck would reveal with the eruption of the maxillary second molar at
enlargement (lymphadenopathy) of primary nodes for 25 to 33 months.
the region, superior deep cervical lymph nodes, espe- B. Permanent (adult) dentition is the second dentition;
cially the jugulodigastric (tonsillar or sentinel node), replaces primary dentition; consists of 32 teeth (8 in-
below posterior belly of digastric. cisors, 4 canines, 8 premolars, and 12 molars).
1. Anterior and premolar teeth are succedaneous
Dental Anatomy and Dentitions (have primary predecessors); molars are nonsuc-
The dentition consists of the natural teeth in the jaw cedaneous (do NOT have primary predecessors)
bones and includes BOTH primary and permanent denti- (Figure 4-26).
tions. It is understood that the teeth being discussed on 2. Begins on average with the eruption of the man-
examinations are permanent unless otherwise designated dibular first molar or central incisor at 6 to 7 years
(as on NBDHE). and is completed with the eruption of the third mo-
• See CD-ROM for Chapter Terms and WebLinks. lars at 17 to 21 years.
Head, Neck, and Dental Anatomy   107

Tooth Types Tooth Surfaces


Tooth types with their specific anatomy are related to mas- Includes facial (buccal), lingual, occlusal, and interpro-
ticatory function of the tooth and to its role in speech and mixal surfaces. Interproximal surfaces are complex (as is
esthetics (Figure 4-26). Form and function of each type the care of the area).
are somewhat similar for BOTH primary and permanent A. Contact point: point on the proximal surface where
dentitions. Types include incisors, canines, premolars, two adjacent teeth actually touch each other (if not
and molars; premolars (bicuspids) ONLY in permanent present, may have an open contact).
dentition. B. Interproximal space: area between two teeth.
A. Incisors: bite and cut food because of triangular prox- 1. Part of the interproximal space is occupied by the
imal form. interdental papilla.
B. Canines (cuspids): pierce or tear food because of 2. Part of the interproximal space not occupied is
prominent cusp and tapered shape. called the embrasure.
C. Premolars (bicuspids): assist molars in grinding food C. Embrasure: area bordered by interdental papilla,
because of broad occlusal surface and prominent proximal surfaces of the two adjacent teeth, and con-
cusps and assist canines in piercing and tearing food tact point.
with their cusps. 1. If there is no contact point between the teeth,
D. Molars: grind food, assisted by premolars, because of the area between is a diastema instead of an em­
broad occlusal surfaces and prominent cusps. brasure.

8 9
7 10

6 11
E F
5 12
1
D G 2
4 3 13

C H 3
1 14

2 15
I 2
B 4
3
1 Maxillary arch 16
A J Right Left
Maxillary arch
32 Mandibular arch 17
Right Left

31 4
18
T Mandibular arch
K

30 19
3
3
S 2 L 29 2 20
1
1 28 21
R M
27 22
Q N 26 23
25 24
P O
Molars 4
Posterior teeth Molars 3 Posterior teeth
Premolars 3
Canine 2
Anterior teeth Canine 2
Incisors 1 Anterior teeth
Incisors 1

Figure 4-25  Primary dentition. (From Fehrenbach MJ, ed: Figure 4-26  Permanent dentition. (From Fehrenbach MJ, ed:
Dental anatomy coloring book, St. Louis, 2008, Saunders/Elsevier.) Dental anatomy coloring book, St. Louis, 2008, Saunders/Elsevier.)
108   Saunders Review of Dental Hygiene

D. Height of contour (crest of curvature): b. Occurs just after 12 years and includes eruption
1. Located on the mesial and distal surfaces at the of all permanent teeth.
contact area (also greatest elevation of the tooth B. Eruption sequence: see Tables 4-2 and 4-3 for
either incisocervically or occlusocervically on a ­approximate eruption times and Figure 4-27 for
specific surface of the crown). ­favorable sequence of eruption for permanent teeth.
2. Also located on the facial and lingual surfaces as is C. Premature eruption: early loss of primary tooth and
easily seen when viewing the tooth’s profile from premature eruption of permanent, can be caused by:
the proximal. 1. Not enough root for stability in primary tooth.
2. Gross caries in primary molars, resulting in early
Tooth Designation loss.
Two systems of tooth designation are widely used. These 3. Bone resorption beneath primary tooth resulting
systems offer standardized method of identifying teeth from abscess.
for purposes of treatment, identification, documentation. D. Variables thought to influence permanent tooth erup-
Palmer method is older method used by orthodontists and tion:
uses quadrant and position within it in numbering. 1. Genetic: familial, race, sex (females slightly ear-
A. Universal Tooth Designation System: used in the lier than males).
United States; adaptable to electronic data transfer 2. Environmental:
(used on the NDHBE): a. Low-birth-weight babies have delayed erup-
1. Primary teeth are designated by the capital letters tion, especially primary dentition.
A through T, consecutively, starting with the max- b. Nutrition has little or no effect on eruption.
illary right second molar, moving in a clockwise c. Systemic factors:
fashion, and ending with the mandibular right sec- (1) Delayed: hypopituitarism, hypothyroidism,
ond molar (Figure 4-25). low levels of growth hormone.
2. Permanent teeth are designated by numbers #1 (2) Premature exfoliation: hyperpituitarism,
through #32, consecutively, starting with the max- hyperthyroidism, cherubism, localized ag-
illary right third molar, moving in a clockwise gressive periodontitis, Papillon-Lefèvre
fashion, and ending with the mandibular right third syndrome, leukemia, cyclic neutropenia.
molar (Figure 4-26).
B. International Standards Organization Designation PERMANENT TEETH  
System (ISO System) by the World Health Organiza- Permanent dentition makes up the second dentition. Re-
tion: used internationally; adaptable to electronic data places primary dentition; some are succedaneous (hav-
transfer; teeth are designated by a two-digit code; first ing primary predecessor), others nonsuccedaneous.
digit indicates quadrant, second indicates the tooth in A. There are 32 permanent teeth, 16 per dental arch:
quadrant.
Table 4-2  Approximate eruption and shedding
Dentition Periods and Eruption Timetable ages for primary teeth

Dentition present may be within three periods: primary, Eruption


mixed, permanent. Tooth (months) Shedding (years)
• See Chapter 2, Embryology and Histology: dental de-
MAXILLARY
velopmental.
A. Dentition periods: Central incisor 8-12 6-7
1. Primary dentition period: Lateral incisor 9-13 7-8
a. Begins with eruption of primary mandibular Canine 16-22 10-12
central incisor.
First molar 13-19 9-11
b. Occurs approximately from 6 months to 6 years;
ends when first permanent tooth, mandibular Second molar 25-33 10-12
first molar, erupts. MANDIBULAR
2. Mixed dentition period: Central incisor 6-10 6-7
a. Follows primary dentition period; occurs ap- Lateral incisor 10-16 7-8
proximately from 6 to 12 years.
Canine 14-18 9-12
b. Begins with eruption of first permanent tooth
(mandibular first molar); ends with shedding of First molar 17-23 9-11
last primary tooth. Second molar 23-31 10-12
3. Permanent dentition period: From Bath-Balogh M, Fehrenbach MJ: Illustrated dental embryology and
a. Begins with shedding of last primary tooth. anatomy, ed 2, Philadelphia, 2006, Saunders/Elsevier.
Head, Neck, and Dental Anatomy   109

Table 4-3  Approximate eruption and root completion


ages for permanent teeth (in years)

Tooth Eruption Root completion

Maxillary
Central incisor 7-8 10
Lateral incisor 8-9 11
Canine 11-12 13-15
First premolar 10-11 12-13
Second premolar 10-12 12-14
First molar 6-7 9-10 Figure 4-27  Favorable sequence of eruption per arch of
Second molar 10-12 14-16 the permanent dentition. (From Bath-Balogh M, Fehrenbach
MJ: Illustrated dental embryology and anatomy, ed 2, Philadelphia,
Third molar 17-21 18-25 2006, Saunders/Elsevier.)
Mandibular
Central incisor 6-7 9 B. Proximal: contact areas are centered labiolingually on
Lateral incisor 7-8 10 proximal surfaces, have smaller area than contact ar-
eas of posteriors but greater cementoenamel junction
Canine 9-10 12-14
(CEJ) curvature than posteriors.
First premolar 10-12 12-13
C. Lingual: cingulum with raised, rounded area on cervi-
Second premolar 11-12 13-14 cal third, in varying degrees of development; ridges
First molar 6-7 9-10 are bordered mesially and distally by marginal ridges.
Second molar 11-13 14-15 1. May have a fossa, which is a shallow, wide depres-
Third molar 17-21 18-25 sion; also may have pits located in deepest part of
fossa.
From Bath-Balogh M, Fehrenbach MJ: Illustrated dental embryology and 2. May have developmental groove, which marks junc-
anatomy, ed 2, Philadelphia, 2006, Saunders/Elsevier.
tion among developmental lobes, and supplemental
groove, which is MORE shallow and ­irregular.
1. Include anterior teeth: 4 incisors and 4 canines, D. Root: one.
and posterior teeth: 8 premolars and 8 molars; la-
beled in Universal Tooth Designation System by Permanent Incisors
numbers, #1 to #32. Incisors are of two types, central and lateral (Figure ­
2. Eruption of the FIRST permanent tooth is the 4-28). When newly erupted, may contain three mame­
mandibular first molar at 6 years of age (NOT lons (rounded enamel extensions) on the incisal ridge,
usually noticed, since it appears similar to the which are extensions from labial developmental lobes.
primary second molar anterior to it); completed at The two incisal angles are formed from the incisal ridge
17 to 25 years, after third molars have erupted. on each proximal surface. Incisal ridge is flattened and
3. By the age of 13, ALL primary teeth have been becomes an incisal edge through attrition. Cingulum,
replaced; takes ∼3½ years for primary to lose its lingual fossa, and marginal ridges are located on lingual
roots and be replaced by permanent tooth. surface at different levels, depending on type of incisor.
B. Larger and with yellower enamel than primary teeth; A. Maxillary incisors: maxillary central and lateral in-
jaws grow in size to accommodate difference. cisors resemble each other MORE than resemble
similar type of incisors of opposing arch; maxillary
Permanent Anteriors central is larger than lateral incisor but has SIMILAR
Anteriors include incisors and canines. Composed of four form; BOTH are wider mesiodistally than labiolin-
developmental lobes, three labial and one lingual, and gually.
two vertical labial developmental depressions. Succeda- 1. Crown: larger in ALL dimensions, especially me-
neous teeth replace primary teeth of same type. siodistally, compared with mandibular incisors; la-
A. Crown: long with incisal masticatory surface: bial surfaces are more rounded from incisal aspect,
1. Outline is trapezoidal from labial and lingual and tri- with tooth tapering toward lingual.
angular from its proximal; compared with posteriors, 2. Lingual: MORE prominent features than on the
anteriors are wider mesiodistally than labiolingually. mandibular; incisal edge is just labial to long axis
2. Height of contour for the labial and lingual sur- of root from either proximal; shovel-shaped form
faces is in cervical third. has greater prominence of marginal ridges, deeper
110   Saunders Review of Dental Hygiene

fossa; pit susceptible to caries, and presence of ra- C. Maxillary lateral incisor (#7, #10):
dicular lingual groove (RLG) can increase risk of 1. Crown: greater variation in form than any other
periodontal disease. permanent, EXCEPT for third molars.
3. Roots: SHORT compared with other maxillary a. Resembles maxillary central incisor but has
teeth; do NOT have concavities. smaller and slightly more rounded crown; is
B. Maxillary central incisors (#8, #9): largest incisors. frequently confused with small mandibular ca-
1. Crown: widest of any anterior, and outline when nine, yet has NO depressions on proximal sur-
viewed from labial or lingual is trapezoidal, with face as is common on mandibular canine.
lingual surface narrower overall than labial. b. Outline is more rounded or oval from incisal,
2. Lingual: horizontally placed groove; may have NOT triangular as central; mesiodistal measure-
linguogingival groove or pit. ment is wider than labiolingual measurement;
3. Proximal: CEJ curvature on mesial is deep inci- labial surface is more rounded than central.
sally and has greatest depth of curvature of any 2. Lingual: horizontal groove is more common
tooth surface in permanent dentition; largest height and better developed than on central, with pit
of contour for BOTH labial and lingual surfaces. common.
4. Root: conical shape, which is smooth and slightly 3. Root: conical in shape, relatively smooth and
straight; narrows through middle to blunt apex. straight, yet may curve slightly to the distal.
a. Approximately same length or shorter but wider a. Same length or longer than central, yet thinner;
than lateral of same arch and triangular in cervi- apex is NOT rounded like central but rather is
cal cross section because wider on labial. sharp.
b. Pulp cavity has only one canal and large cham- b. Pulp cavity is simple in form; one canal.
ber, three sharp horns.

1
6

7
2
4 3
Labial 8 Lingual

10 9

Incisal

Height Height
of contour of contour

Mesial Distal

10 Distal incisal angle Marginal ridges 5

Mesial incisal angle 9 Distal incisal angle 4

Incisal ridge 8 Mamelons 3

Lingual fossa 7 Mesial incisal angle 2

Cingulum 6 Imbrication lines 1


Figure 4-28  Permanent incisors. (From Bath-Balogh M, Fehrenbach MJ: Illustrated dental embryology and
anatomy, ed 2, Philadelphia, 2006, Saunders/Elsevier.)
Head, Neck, and Dental Anatomy   111

4. MOST common for partial microdontia (peg lat- E. Mandibular central incisor (#24, #25):
eral) and MOST common for partial anodontia 1. Crown: difficult to distinguish between right and left.
(congenitally missing). 2. Root has pronounced proximal root concavities and
D. Mandibular incisors: smallest and MOST symmetri- pulp cavity is simple; one canal and three horns.
cal, uniform teeth. F. Mandibular lateral incisor (#23, #26): NOT as sym-
1. Crown: may have attrition to incisal edge, chang- metrical as that of central; from BOTH labial and lin-
ing symmetrical form. gual appears tilted or twisted distally in comparison
2. Root: elliptical on cervical cross section; narrow with the long axis.
on labial and lingual; wide on BOTH proximal 1. Crown: slightly larger than central, yet resembles it.
surfaces; proximal concavities are also pres- 2. Root: pronounced proximal concavities, especially
ent and, if deep enough, give double-rooted on distal surface, varying in BOTH length and
ap­pearance. depth; pulp cavity is simple; one canal, three horns.

CLINICAL STUDY  

Age 20 YRS SCENARIO

Sex ☐  Male   ☒  Female The patient comes into the office with-
out an appointment because of concern
Height 6’2“ about her teeth. During the intraoral
Weight 125 LBS ­examination, she says her palate feels sore
and her face seems to be changing too,
BP 90/54 with some “downy” hair on it. However,
the photo­graphers like her more round
Chief Complaint “My teeth look thinner and transparent and face and square jaw.
even more gray in my recent photographs!
How can I fix them?”

Medical History Breast augmentation 2 years ago


Liposuction 1 year ago

Current Medications Laxatives for constipation

Social History Model

1. What has happened to the patient’s teeth? What teeth is also salivary gland enlargement caused by the bu-
are involved and why just them? Why is her palate limia, with change in shape of face (round) and jaw
sore? (square) and with “downy” hair occurring on her face.
2. What could be the barriers to care for her now and Her palate is sore from forced vomiting (by fingers,
later? knuckles, or other objects).
3. After working past any denial problems, what should 2. Initially, lack of communication could be a barrier
her patient education emphasize? to oral care because of denial, guilt, fear of gaining
weight, and lack of compliance; also results from an
1. Patient may have bulimia, a compulsive disorder that inability to gain trust and confidence. She really needs
involves periods of starvation and bingeing and per- to be referred to her physician to deal with her disor-
ceived lack of control over eating behavior; affected der. Economic barriers may come into play later be-
individuals engage in an average of two episodes a cause of cost of repairing oral damage.
week (see Chapter 7, Nutrition). Cause is unknown but 3. Discussion of oral and medical problems associated
is likely to be stress related, affecting young women. with purging, diuretics, and laxative use. Also, she
Enamel erosion (perimolysis) occurs because of stom- needs to neutralize vomit acid by rinsing with tap
ach acids from vomiting on maxillary anteriors, with ­water, or possibly mix of water and sodium ­bicarbonate
dishing of lingual surfaces. This would involve the or ­ magnesium hydroxide, as well as to avoid tooth-
lingual surfaces of her maxillary lateral and central brushing and flossing immediately after vomiting to
on both sides of her mouth (teeth #7, #8, #9, #10), reduce abrasion. Use of saliva substitutes or sugarless
since that is where the vomit always contacts. It would gums (possibly with xylitol) to increase salivary flow
make the teeth thinner, thus appearing grayer. There and daily use of mineralizing fluoride and calcium
112   Saunders Review of Dental Hygiene

products should be explained. Nutritional concerns 2. Root: longest in maxillary arch; with blunt apex
may enter into discussion. and developmental depressions on BOTH ­proximal
surfaces, especially on the distal; one canal and
Permanent Canines large chamber, with only one horn.
Maxillary and mandibular canines (cuspids) are simi- B. Mandibular canines (#22, #27):
lar (Figures 4-29 and 4-30). Have LARGER cingulum 1. Crown: can be as long as or even longer than max-
and marginal ridges on their lingual surfaces, which are illary canine; rarely has lingual pits or grooves.
narrower than the labial surfaces, and crowns that taper 2. Root: may be as long as that of maxillary canine
lingually. Longest teeth; each has a long, thick root, ex- but is somewhat shorter; longest mandibular root,
ternally manifested by canine eminence on maxillary with slight mesial inclination.
arch. Proximal root concavities are on BOTH proximal a. More pronounced and often deeper mesial de-
root surfaces and are ovoid on cervical cross section, with velopmental depression than in the maxillary
wide facial and proximal surfaces that show increased canine.
convergence to narrow lingual surface. b. Distal developmental depression similar to me-
A. Maxillary canines (#6, #11): sial, giving double-rooted appearance.
1. Crown: similar in length to or even shorter than c. Pulp cavity resembles that of maxillary canine,
maxillary central incisor. with only one horn; may have two separate ca-
a. Outline is asymmetrical from incisal; distal part nals (one labially and one lingually), may join
appears thinner than mesial and gives impres- at the apex or have separate apical foramina.
sion of being “stretched” to make contact with
first premolar. Permanent Posteriors
b. Arch space is often partially closed and may Posteriors include premolars and molars.
erupt labially or lingually to surrounding teeth A. Crowns: trapezoidal from the buccal and lingual.
or may NOT erupt and remain impacted.

1
2
5
3 2
3
6

6
4
5 9

7
6 8
Labial View Labial View 10

Cusp slopes 6 Lingual View Lingual View

Cusp tip 5 10 Cusp tip Marginal ridges 5

Labial ridge 4 Marginal ridges 9 Cingulum 4

Labial ridge 3 Lingual fossae 8 Lingual fossae 3

Cusp tip 2 Lingual ridge 7 Lingual ridge 2

Cusp slopes 1 Cingulum 6 Cusp tip 1

Figure 4-29  Labial view of permanent right canines. Left, Figure 4-30  Lingual view of permanent right canines. Left,
Maxillary; right, mandibular. (From Bath-Balogh M, Fehrenbach Maxillary; right, mandibular. (From Bath-Balogh M, Fehrenbach
MJ: Illustrated dental embryology and anatomy, ed 2, Philadelphia, MJ: Illustrated dental embryology and anatomy, ed 2, Philadelphia,
2006, Saunders/Elsevier.) 2006, Saunders/Elsevier.)
Head, Neck, and Dental Anatomy   113

1. Height of contour for the buccal surface is in cer-


vical third and for lingual surface is in middle or
occlusal third.
2. Wider labiolingually than mesiodistally, except
for mandibular molars, when compared with
­anteriors.
B. Proximal: 4
1. Contact areas are wider for posteriors than those 1
of anteriors and are located to the buccal of center;
also are closer to same level on each side.
2. CEJ curvature is less pronounced on posteriors 5
than on anteriors, may be quite straight.
C. Occlusal table: bordered by raised marginal ridges lo- 3 2 6
cated on the distal and mesial and have two or more Mesial marginal ridge 6
cusps (see Figures 4-32 to 4-39 for comparison to
­anterior teeth). Mesial marginal groove 5

1. With four cusp ridges descending from each cusp Mesial developmental depression 4
tip and inclined cuspal planes between them, trian- Distal cusp slope 3
gular cusp ridges descend from cusp tips toward
central part. Mesial cusp slope 2

2. May have a transverse ridge that occurs with the Imbrication lines 1
joining of two triangular ridges that cross occlusal
table transversely or from the labial to lingual out- Figure 4-31  Views of the permanent maxillary right first
premolar. (From Bath-Balogh M, Fehrenbach MJ: Illustrated dental
line. embryology and anatomy, ed 2, Philadelphia, 2006, Saunders/
3. Shallow and wide fossae; central fossae are lo- Elsevier.)
cated at converging of cusp ridges in central point,
where there is junction of grooves and triangular
fossa that appears to have triangular shape at con- root concavities. MOST susceptible to abfraction and la-
vergence of cusp ridges, associated with termina- bial cervical recession and enamel breakage caused by
tion of triangular grooves. oral habits.
4. Developmental grooves or primary grooves are A. Maxillary premolars: BOTH types resemble each
sharp, deep, with V-shaped linear depressions; MOST other MORE than mandibular premolar; first is larger
prominent is central groove, which travels mesiodis- than second.
tally and separates occlusal table buccolingually. 1. Crown: two cusps of almost equal size, centered
5. Marginal grooves cross marginal ridges, and trian- over long axis of tooth from proximal.
gular grooves separate marginal ridge from trian- a. Shorter occlusocervically than maxillary ca-
gular ridge of a cusp, forming triangular fossa at nines, yet slightly longer than molars.
termination. b. Unlike crowns of mandibular premolars, cen-
6. Supplemental grooves or secondary grooves are tered over root and show no lingual inclina-
shallower, more irregular linear depressions. tion.
7. Developmental pits, where two or more grooves c. Greater buccolingual than mesiodistal width
meet; can be located in deepest parts of fossa, compared with mandibular premolars.
making it susceptible to caries. 2. Proximal: trapezoidal.
3. Occlusal table: somewhat hexagonal.
Permanent Premolars: First and Second 4. Roots: shorter than maxillary canines and same
Premolars (bicuspids) are succedaneous and replace pri- length as molars.
mary first and second molars; they are found ONLY in a. Show a slight lingual and distal inclination.
permanent dentition. May be extracted in each quadrant b. On cervical cross section, appear elliptical; ap-
for orthodontic purposes to improve dental arch spacing pearance may be slightly altered by proximal
(first premolar). Shorter crown than anteriors and buc- concavities.
cal surface is rounded, with prominent buccal ridge and c. May penetrate maxillary sinus during acciden-
buccal developmental depressions. Height of contour tal trauma or during tooth extraction because of
labially is in cervical third, similar to anteriors, and lin- close proximity; discomfort from sinusitis may
gually is in middle third. MOST have one root, except be confused with tooth-related discomfort or
for maxillary first, which has two roots. Have proximal vice-versa.
114   Saunders Review of Dental Hygiene

B. Maxillary first premolars (#5, #12) (Figure 4-31): 1. Crown has two types, three-cusp and two-cusp
1. Crown: widest mesiodistally of all premolars; wide crowns.
at the level of contact areas and more narrow at a. The three-cusp (tricuspidate) develops from
CEJ. Occlusal table: transverse ridge perpendicu- three lobes:
lar to central groove, mesial and distal pits. (1) MOST common; one large buccal cusp and
2. Roots: ONLY premolar with two roots, two two smaller lingual cusps.
branches or bifurcated in apical third; one buccal (2) Grooves form distinctive Y-shaped pattern on
and one lingual or palatal; may be fused or lami- the occlusal table, resembling a small molar.
nated. b. The two-cusp (bicuspidate) develops from two
a. DISTINCT mesial concavity is present on lobes.
trunk, extending from contact area to bifurca- (1) Similar to that of mandibular first premo-
tion; increases periodontal risk, since allows lars, one larger buccal cusp and one smaller
increased deposit level. lingual cusp;
b. Pulp cavity shows two horns (one for each cusp) (2) Central groove is crescent or U shaped, ap-
and two canals (one for each root). pears rounded from occlusal.
C. Maxillary second premolar (#4, #13): 2. Root: larger and longer than first premolar, yet
1. Crown: less angular and more rounded in shape shorter than maxillary premolars.
than the maxillary first, with more variations. a. Pronounced proximal concavities.
2. Occlusal table: numerous supplemental grooves b. Pulp cavity for three-cusp type has three pointed
radiating from a central groove. horns; two-cusp type has two horns.
3. Roots: has one; may have two, however; less pro-
nounced greater length of second mesial concav- Permanent Molars: First, Second, and Third
ity; pulp cavity has one canal and two horns. Molars are nonsuccedaneous and do NOT replace primary
D. Mandibular premolars: do NOT resemble each other teeth (Figures 4-33 and 4-34). They show evidence of de-
as much as those of maxillary premolars; first smaller velopmental lobe separation in developmental grooves
than second. on occlusal table; do NOT exhibit buccal developmental
1. Crowns: buccal outline shows strong lingual in- depressions. Have large crowns in comparison with rest
clination proximally; occlusally, appears almost of the permanent dentition and are shorter occlusocervi-
round, with strong buccal ridge; may have more cally than teeth anterior to them.
than two cusps; lingual are always smaller than A. Crown: three or more cusps, of which at least two are
buccal. buccal cusps, and its buccal surface has a prominent
2. Proximal: outlines are rhomboidal, with lingual cervical ridge running mesiodistally in cervical one
incline; contact areas are nearly on same level and third.
have similar CEJs. B. Occlusal table: bordered by cusp ridges and marginal
3. Root: has one, with a slight distal inclination; in ridges.
cervical cross section is either ovoid or elliptical; 1. MORE complicated than premolars because there
may be slightly altered by proximal concavities, are more developmental grooves, supplemental
MOST frequently found on mesial surface. grooves, occlusal developmental pits.
E. Mandibular first premolar (#21, #28): shows transi- 2. Grooves (fissures) and pits are located on
tion in dental arch from canine to molarlike second occlusal and lingual surfaces of maxillary and
premolar. on occlusal and buccal surfaces of mandibular
1. Crown: resembles mandibular canine in many molars (Figure 4-35).
more ways than it does mandibular second premo- C. Roots: multirooted; maxillary molars usually have
lar, smaller overall than canine. three branches; mandibular usually have two.
2. Occlusal table: lingual cusp is very small, no more 1. Cervical cross section of trunk follows form of
than half height of buccal, with four lingual cusp crown but then divides into number of branches.
ridges, four lingual inclined cuspal planes, and lin- 2. Furcations lie between two or more of these
gual triangular ridge. branches before dividing from trunk; furcation
3. Root: smaller and shorter than mandibular second crotches are the spaces between roots at furcation
premolar. and concavities are found on many branches and
a. May have deep groove on distal. furcal surfaces.
b. Pulp cavity consists of one canal and two horns; 3. With the loss of periodontal tissue support from
each horn is located within cusp; buccal horn advanced periodontal disease, furcations, furca-
more pronounced. tion crotches, and concavities of the molars may
F. Mandibular second premolars (#20, #29) (Figure 4-32): lose their bony coverage in varying degrees and
Head, Neck, and Dental Anatomy   115

1 10 5

9 6
DL
ML
8
4 3 2 7

Three-cusp type Two-cusp type Three-cusp occlusal view

14 11 16 15
19 18

ML DL 12
13

Mesial pit Distal pit 20 17

Three-cusp occlusal view U-shaped H-shaped


groove pattern for two-cusp groove pattern for two-cusp

20 Mesial pit 10 Central groove


19 Distal pit Mesial marginal ridge 9
18 Central groove Central pit 8

17 Mesial pit Lingual groove 7

16 Distal pit Distal marginal ridge 6


15 Central groove Distobuccal triangular groove 5
14 Mesiobuccal triangular groove Mesiolingual cusp 4
13 Mesial marginal groove Distolingual cusp 3

12 Distal marginal groove Lingual cusp 2

11 Distobuccal triangular groove Buccal cusp 1

Figure 4-32  Occlusal view of the two types of permanent mandibular right second ­premolars:
three-cusp and two-cusp. Occlusal table, Y-shaped groove pattern, ­ fossae, and U- and H-shaped
groove patterns shown. (From Bath-Balogh M, Fehrenbach MJ: ­Illustrated dental embryology and anatomy,
ed 2, Philadelphia, 2006, Saunders/Elsevier.)

p­ resent a challenge during instrumentation and a. Wider buccolingually than mesiodistally; from
performance of oral hygiene. occlusal, outline is rhomboidal, and from proxi-
D. Localized enamel hypoplasia from congenital syphi- mal, trapezoidal.
lis can result in mulberry molars, when molars have b. Possible lingual pit that is susceptible to
one or more tubercles or accessory cusps; dilacera- caries.
tion can also occur, making extraction and endodontic 2. Occlusal table:
treatment difficult. a. Has four major cusps, with two cusps on buccal
E. Maxillary molars: and two on lingual (Figure 4-36).
1. Crown: shorter occlusocervically than teeth ante- b. ONLY teeth with oblique ridge, transverse ridge
rior to them, but larger in all other measurements formed by union of triangular ridge of DB cusp
than other maxillary teeth. and the distal cusp ridge of ML cusp.
116   Saunders Review of Dental Hygiene

2 3 1

4 2
cuspal plane
Inclined 4 3

Cusp tip 3 4

Cusp ridge 2
1 1
Marginal ridge 1 5
8

6
Figure 4-33  Example of occlusal surface of permanent
­molar. (From Bath-Balogh M, Fehrenbach MJ: Illustrated dental em-
bryology and anatomy, ed 2, Philadelphia, 2006, Saunders/Elsevier.)
7
3. Roots: ONLY teeth with three roots, trifurcated into Marginal groove 8
three branches (MB, DB, and lingual [palatal]).
a. Lingual branch is largest and longest; when
Triangular fossa 7

located farther (distally) in the maxillary arch, Triangular ridge 6


have shorter and more varied roots in size, Triangular fossa 5
shape, and curvature but are less divergent.
b. Show great lingual and moderate distal inclina-
Occlusal pit 4

tion and have three furcations, located on the me- Triangular groove 3
sial, buccal, and distal surfaces, that begin near Central groove 2
the junction of the cervical and middle thirds.
c. May have concavities on the mesial surface of
Supplemental groove 1

MB root, on lingual surface of lingual root, and Figure 4-34  Example of occlusal table on permanent ­molar.
on all three furcal surfaces. (From Bath-Balogh M, Fehrenbach MJ: Illustrated dental embryology
d. May penetrate the maxillary sinus, from acci- and anatomy, ed 2, Philadelphia, 2006, Saunders/Elsevier.)
dental trauma or during tooth extraction owing
to close proximity, MOST commonly associ-
ated with discomfort from sinusitis. 2. Occlusal table:
e. Teeth MOST commonly involved in concres- a. Rhomboid type: cusps similar to major cusps of
cence in permanent dentition. maxillary first molar.
F. Maxillary first molars (#3, #14): five developmental b. Heart-shaped type: has DL cusp that is small,
lobes: two buccal and three lingual. sometimes absent.
1. Crown: largest among those of the permanent dentition; 3. Roots: show furcation notches that are narrower
more complex form than nearby maxillary premolars; than first molar.
LEAST variable form among maxillary molars. a. Greater chance of fusion, especially of buccal
2. Occlusal table: roots or all three roots.
a. ML cusp is largest cusp, with rounded cusp tip b. Pulp cavity consists of chamber, three main ca-
(Figure 4-37). nals, four horns.
b. May have minor (fifth) cusp of Carabelli, with H. Maxillary third molars (#1, #16) (wisdom tooth):
its groove; smallest cusp. 1. Crown: no standard form; smallest molar and
c. Has three triangular grooves (MB, ML, D) and MOST variable in shape in permanent dentition.
oblique ridge. 2. Occlusal table: two outline forms, heart shaped or
3. Roots: larger and more divergent than those of rhomboidal.
second molar, more complex in form than roots of a. Heart shaped: MOST common; similar to max-
maxillary premolars. illary second molar but with more supplemental
a. Twice as long as crown; thus furcations are well grooves; only three cusps (MB, DB, ML).
removed from cervical area. b. Rhomboid occlusal: four cusps; no oblique
b. Pulp cavity has one horn for each major cusp; ridge on small DL cusp.
roots have three main pulp canals. 3. Roots: trifurcated and fused, either partially or
G. Maxillary second molars (#2, #15): fully.
1. Crown: two outline types, either rhomboid or heart a. Poorly developed and shorter than second mo-
shaped. lar, curved distally; may have accessory root.
Head, Neck, and Dental Anatomy   117

8
6

3
1
9
4
7
10
2
5

Root concavity 9 Furcation crotch area 5

Furcation 9 Furcation 4

Furcation crotch area 8 Root concavity 3

Root trunk 7 Roots 2

Roots 6 Root trunk 1

Figure 4-35  Features of roots of permanent molars. (From Bath-Balogh M, Fehrenbach MJ: Illustrated dental
embryology and anatomy, ed 2, Philadelphia, 2006, Saunders/Elsevier.)

b. Pulp cavity may have a chamber and three ca- by cusps and MB, DB, lingual grooves; NO trans-
nals; number of horns varies, depending on the verse ridges (Figure 4-38).
number of cusps present. 3. Roots: widely separated buccally; trunk is shorter
4. May show partial microdontia (peg third molar) than second and pulp cavity has three canals and
or partial anodontia (congenitally missing) or may five horns.
fail to erupt and remain impacted. K. Mandibular second molars (#18, #31):
I. Mandibular molars: 1. Crown: smaller than first molar and rectangular
1. Crown: wider mesiodistally than buccolingually, occlusally.
similar to anteriors. 2. Occlusal table: cross-shaped groove pattern
a. From occlusal, have an outline that is rectan- formed by well-defined grooves that divide into
gular or pentagonal; buccal view shows strong four parts, with four nearly equal-sized cusps
lingual inclination proximally (like nearby pre- (Figure 4-39).
molars) and is rhomboidal. 3. Roots: smaller, shorter, less divergent, and closer
b. Have four or five major cusps; always with two together than in first molar, and pulp cavity can
lingual cusps of approximately same width. have two canals (one for each root).
2. Roots: two roots or bifurcated, with mesial and L. Mandibular third molars (#17 and #32) (wisdom
distal; show distal inclination. tooth):
a. Have two furcations located on buccal and lin- 1. Crown: NO standard form; often has oval occlusal
gual surfaces, midway between proximal sur- outline; can be smaller or same size as first molar
faces. of same arch.
b. Concavities on mesial surface of mesial root 2. Occlusal table: two mesial cusps, larger than
and on furcal surfaces of BOTH mesial and two distal cusps; irregular groove pattern, with
distal roots; concavities on mesial root are numerous supplemental grooves and occlusal
especially prominent if this root also has two pits.
canals. 3. Roots: two roots OFTEN are fused, irregularly
J. Mandibular first molars (#19, #30): curved, shorter than mandibular second molar;
1. Crown: widest mesiodistally of any permanent pulp cavity is similar to second molars, with four
tooth because of fifth major cusp; has three buccal horns and two or three canals.
and two lingual cusps. 4. May fail to erupt and remain impacted or partially
2. Occlusal table: MOST complex groove pattern of erupted; may exhibit partial anodontia (congeni-
all mandibular molars: Y-shaped groove pattern tally missing).
118   Saunders Review of Dental Hygiene

8 1
5 1

MB DB
D

2 7 2
ML DL

4 3 3

Distobuccal cusp 6 4
5
5
Distolingual cusp 4
Central pit 4 Mesiobuccal groove 8
Mesiolingual cusp 3
Distal pit 3 Mesial marginal ridge 7
Oblique ridge 2
Distal marginal ridge 2 Mesial pit 6
Mesiobuccal cusp 1
Distobuccal groove 1 Lingual groove 5
Figure 4-36  Occlusal view of permanent maxillary molar.
(From Bath-Balogh M, Fehrenbach MJ: Illustrated dental embryol- Figure 4-38  Occlusal table of permanent mandibular right
ogy and anatomy, ed 2, Philadelphia, 2006, Saunders/Elsevier.) first molar. (From Bath-Balogh M, Fehrenbach MJ: Illustrated den-
tal embryology and anatomy, ed 2, Philadelphia, 2006, Saunders/
Elsevier.)
6 1
7 1
DB MB

MB DB
5 2
6 2

ML 5
DL 3
3
4 ML DL

4
6 Buccal groove

5 Distal marginal ridge


Lingual groove 4
Distal marginal ridge groove 4
Transverse ridge 3 Central pit 7
Cusp of Carabelli 3
Distal pit 2 Mesial pit 6
Mesial marginal ridge 2
Buccal groove 1 Transverse ridge 5
Mesial marginal ridge groove 1
Figure 4-39  Occlusal table of permanent mandibular right
Figure 4-37  Occlusal table of permanent maxillary right first second molar. (From Bath-Balogh M, Fehrenbach MJ: Illustrated
molar. (From Bath-Balogh M, Fehrenbach MJ: Illustrated dental em- dental embryology and anatomy, ed 2, Philadelphia, 2006, Saun-
bryology and anatomy, ed 2, Philadelphia, 2006, Saunders/Elsevier.) ders/Elsevier.)

PRIMARY TEETH   3. Eruption of primary mandibular central incisor oc-


Primary teeth make up first dentition. Exfoliated (shed) curs FIRST at average age of 8 months; thus NO
and replaced by permanent dentition (Figure 4-40). primary teeth are visible at birth.
A. There are 20 primary teeth, 10 per dental arch. 4. Root formation takes from 2 to 3 years to be com-
1. Include 4 incisors, 4 canines, and 8 molars; labeled pleted, including completion of roots of maxil-
in Universal Tooth Designation System by capital lary second molar; 6-month delay or acceleration
letters, A through T. is considered normal (see earlier ­discussion).
2. Begin calcification at 13 to 16 weeks’ gestation; by B. Smaller and whiter enamel than permanents (par-
18 to 20 weeks’ gestation have started to ­calcify. ents can be so jealous); may demonstrate ­ extensive
Head, Neck, and Dental Anatomy   119

extrinsic green staining because of Nasmyth’s Mesial View

membrane.
C. Crowns:
1. Shorter relative to the total length and MORE con-
stricted (narrower) at the CEJ, which makes them
appear bulbous.
Root CEJ
2. Cervical ridges on BOTH labial and lingual sur-
CEJ
faces of anteriors and buccal surfaces of molars;
masticatory surfaces may show high levels of at-
trition.
Crown
D. Roots:
1. Narrower and longer when compared with overall
crown length and may show partial resorption. Primary Maxillary Permanent Maxillary
2. Hold eruption space for succedaneous permanent First Molar First Molar

teeth; early assessment for appropriate preventive Figure 4-40  Features of primary teeth. (From Bath-Balogh
orthodontic intervention is important; supervising M, Fehrenbach MJ: Illustrated dental embryology and anatomy,
ed 2, Philadelphia, 2006, Saunders/Elsevier.)
adults and child patients sometimes discount the
importance of these teeth.
E. Pulp cavity: NOT as constricted at CEJ as primary maxillary
1. Relatively large in proportion to the permanent central.
dentition, especially larger mesial horns of the b. Labially, appears wide in comparison with per-
molars, increases risk of exposure during cavity manent successor; has mesial and distal outlines
preparation. that taper evenly from the contacts.
2. Thinner enamel and dentin than permanent teeth; c. Lingual appears smooth and tapers toward
risk of endodontic complications is greater. prominent cingulum, which has less pronounced
marginal ridges and shallow lingual fossa.
Primary Incisors d. Mesially, much wider labiolingually than
Primary incisors include primary maxillary and mandibu- ­permanent successor.
lar central and lateral. 2. Root: one, long and slender; labial and lingual are
A. Primary maxillary central incisor (E, F): rounded, yet proximals are slightly flattened.
1. Crown: wider mesiodistally than incisocervically D. Primary mandibular lateral incisor (Q and N):
(opposite of permanent successor). 1. Crown: similar in form of mandibular central in-
a. Appears thick even at incisal third (can be al- cisor, wider and longer than central, but NOT as
tered by attrition); incisal edge is nearly straight; symmetrical.
no mamelons and no lingual pits, unlike its per- a. Cingulum more developed; offset toward distal.
manent successors. b. Deeper lingual fossa than central; incisal edge
b. Cingulum and marginal ridges are more promi- slopes distally; distoincisal angle more rounded.
nent and lingual fossae are deeper than in per- 2. Root: may have distal curvature in apical third and
manents. distal longitudinal groove.
2. Proximal: the CEJ curves distinctly toward incisal,
but not as much as on permanent successor. CLINICAL STUDY 
3. Root: one; is round and tapers evenly to apex. Scenario: During an emergency dental appointment for
B. Primary maxillary lateral incisor (D, G): a 2-year-old, the patient’s mother tells how he slipped in
1. Crown: similar to central incisor but is much the bathtub and hit his upper front teeth. He is bleeding
smaller; longer incisocervically than mesiodistally profusely, and his upper lip is severely swollen. Both pri-
and incisal angles that also are more rounded than mary maxillary central incisors are intruded into the max-
those of the central incisor. illa. “My son’s front teeth are pushed into the gums!”
2. Root: is also similar to that of central incisor, but
apex is sharper. 1. How many teeth can the patient have at present, and
C. Primary mandibular central incisor (O, P): what dentition period is present?
1. Crown: more like that of primary mandibular lat- 2. At 2 years of age, which teeth should be fully erupted?
eral incisor than permanent successor or any other 3. Identify the blood and nerve vessels that are most
primary maxillary incisor. likely to be injured in this situation.
a. Symmetrical tooth, similar to permanent suc- 4. What effect might this trauma have on the ­ patient’s
cessor, with incisal edge centered over root; oral development?
120   Saunders Review of Dental Hygiene

1. He is in the primary (deciduous) dentition period in to them (may make it hard to see permanent’s
which 20 teeth are usually present. eruption, so that there is failure of oral homecare
2. At 2 years, primary maxillary and mandibular cen- and dental care).
tral and lateral incisors, canines, first molars are 2. Shorter occlusocervically than mesiodistally.
fully erupted. Primary second molars erupt at 27 to B. Occlusal table:
29 months, depending on arch. 1. More constricted buccolingually than permanent
3. Infraorbital (IO) artery supplies upper lip, and anterior molars, with buccal and lingual surfaces flatter
superior alveolar (ASA) nerve innervates maxillary occlusal to CEJ.
anterior teeth and bone. 2. Anatomy of cusps NOT as pronounced as on per-
4. Intrusion injury occurs when the tooth root is forc- manent successors.
ibly compressed into alveolar bone. This severe injury C. Roots:
caused damage to primary maxillary central incisor 1. Have a short trunk, creating more space for devel-
roots and alveolar bone. When injury occurs in primary oping permanent premolar crowns.
dentition, displacement of tooth can also cause injury 2. Flare beyond crown outlines; widely separated to
to adjacent developing permanent maxillary teeth. create additional space between roots for develop-
Enamel development of the permanent maxillary cen- ing permanent premolar crowns.
tral and lateral incisors begins 3 to 4 months after birth 3. Greater spread of roots, along with narrow shape
and continues to age 5; therefore permanent maxillary and lack of trunk, making it easier to fracture dur-
central incisors may possibly exhibit slight to moderate ing extraction procedures.
enamel defects (hypoplasia and/or hypocalcification). D. Primary maxillary first molar (B, I):
1. Crown: does NOT resemble other crowns of either
Primary Canines dentition.
Primary canines include primary maxillary and mandibu- a. From buccal, mesial and distal outlines are
lar canines. rounded and constricted at CEJ, and on mesial
A. Primary maxillary canine (C, H): half of buccal surface curves around prominent
1. Crown: relatively longer, with sharper cusp than buccal cervical ridge.
permanent successor, when first erupts. b. Height of contour on buccal is at cervical one
a. Rounder mesial and distal outlines that greatly third and for lingual is at middle one third.
overhang cervical line; mesial cusp slope is 2. Occlusal table: can have four cusps (MB, ML, DB,
long­er than distal. DL).
b. Cingulum, lingual ridge, and marginal ridges a. Mesial cusps are largest and distal cusps are
are well developed; tubercle often present on very small; frequently has only three cusps; DL
cingulum. cusp may be absent.
c. Lingual ridge divides lingual into shallow ML b. Has prominent transverse ridge and oblique
and DL fossae. ridge running between ML cusp and DB cusp,
d. Incisally, diamond shaped; cusp tip offset distally. although NOT as prominent as on permanent
2. Root: twice as long as crown, more slender than counterpart.
permanent successor, inclined distally. c. Has H-shaped groove pattern and central, me-
B. Primary mandibular canine (M, R): sial triangular, and distal triangular fossae, with
1. Crown: resembles primary maxillary canine. central groove connecting the central, mesial,
a. Smaller labiolingually; incisal edge straight and and distal pits; buccal groove separates MB and
centered labiolingually. DB cusps, and distal triangular fossa contains
b. Distal cusp slope longer than the mesial and disto-occlusal groove.
lingual is smoother than maxillary canine, with 3. Roots: SAME number and position as those of per-
shallow lingual fossa. manent dentition.
2. Root: long, narrow, almost twice crown length; a. Short trunk, with three branches that are thinner
shorter and more tapered than maxillary canine. and have greater flare than on permanent molar.
b. The MB root is wider buccolingually than DB
Primary Molars root; lingual root is longest and MOST diver-
Primary molars include primary maxillary and mandibu- gent.
lar first and second molars. E. Primary maxillary second molar (A, J):
A. Crown: 1. Crown and occlusal table: MOST closely resem-
1. Does NOT resemble any other tooth in either den- bles the permanent maxillary first molar.
tition; primary second in BOTH arches does re- a. Yet is smaller in all dimensions.
semble permanent first molars that will erupt distal b. Has cusp of Carabelli, minor fifth cusp.
Head, Neck, and Dental Anatomy   121

F. Primary mandibular first molar (L, S): (3) MOST important when mounting radio-
1. Crown: unlike any other tooth of either dentition. graphs.
a. Has prominent buccal cervical ridge on mesial b. Curve of Wilson: lateral concave curve of the
half of buccal. occlusal plane produced by the alignment of the
b. Height of contour on the buccal is at cervical posteriors.
one third and for lingual is in middle one third. (1) Noted when a frontal section is taken through
c. The ML line angle is rounder than any other. each set of both maxillary and mandibular
2. Occlusal table: four cusps; mesial cusps are larger; molars: the firsts, seconds, and then thirds.
ML cusp is long, pointed, and angled; transverse (2) Formed by the lingual inclination of the
ridge runs between MB and ML cusps. posteriors because the lingual cusps are
3. Roots: BOTH positioned similarly to other pri- lower than the buccal cusps.
mary and permanent mandibular molars. C. Centric occlusion (CO) (habitual occlusion):
G. Primary mandibular second molar (K, T): 1. Voluntary position of dentition that allows maxi-
1. Crown: larger than primary mandibular first mo- mum contact when teeth occlude, equalizing forces
lar; MOST closely resembles form of permanent of impact; serves as basis for reference.
mandibular first molar that erupts distal to it (may 2. In CO, relation of permanent maxillary first molar
make it hard to notice permanent’s eruption, so to mandibular arch is established when ML cusp
that there is failure of oral homecare and/or dental interdigitates with the central fossa of mandibular
care). first molar (see following discussion of occlusal
2. Occlusal table: five cusps; three buccal cusps evaluation).
of nearly equal size, with overall oval occlusal
shape. Occlusal Evaluation
3. Roots: positioned similarly to those of other pri- Normal occlusion is an ideal occlusion. However, it rarely
mary and permanent mandibular molars. exists; the concept provides a basis for treatment. Occlu-
sal disharmony may lead to occlusal trauma, which could
OCCLUSION   be an adverse factor in periodontal disease development;
Occlusion is the contact relationship between the must be considered during dental treatment; occlusal ad-
maxillary and mandibular teeth when the jaws are in justment involving removal of restorative, prosthetic, or
a fully closed position. Also refers to the relationship natural tooth material may be needed.
between teeth in the same arch. Occlusion develops as A. American Association of Orthodontists (AAO) rec-
the primary teeth erupt. Involves interrelated ­ factors ommends that all children have orthodontic evalua-
in development, such as associated muscu­lature, tion no later than age 7.
neuromuscular patterns, temporomandibular joint 1. Posterior occlusion is established when the first
functioning. molars erupt; evaluation of the anteroposterior and
• See Chapter 13, Periodontology: occlusal trauma. transverse occlusal relationships can be performed
A. Mandibular rest position: physiological rest posi- and any functional shifts discovered.
tion of mandible when relaxed; 2 to 3 mm of inter- 2. Incisors have begun to erupt, and problems such
occlusal clearance between arches; failure may mean as crowding, habits, deep bites, open bites, some
parafunctional habits, which may be involved in oc- facial asymmetries can be detected.
clusal problems. 3. Early intervention and/or early treatment presents
B. Occlusal plane: maxillary and mandibular teeth in the opportunity to:
centric occlusion along BOTH anteroposterior and a. Influence jaw growth in a positive manner and
lateral curves (noted by the linea alba). harmonize width of the dental arches.
1. Arches do NOT conform to flat planes but are b. Improve eruption patterns and lower risk of
curves; maxillary arch is convex occlusally, and trauma to protruded maxillary incisors.
mandibular arch is concave: c. Correct harmful oral habits and improve esthet-
a. Curve of Spee: anteroposterior concave curve ics and self-esteem.
of the occlusal plane produced by the alignment d. Simplify and/or shorten treatment time for later
of the posteriors. corrective orthodontics and reduce likelihood
(1) Noted when viewing posteriors from the of impacted permanent teeth.
buccal. e. Improve some speech problems and preserve or
(2) Beginning at the tip of the mandibular ca- gain space for erupting permanent teeth.
nine, following the buccal cusps of the nat- B. Malocclusions: misalignment of teeth and jaws; can
ural premolars and molars, and continuing be a factor in oral health and hygiene; crowding is
to the anterior border of the ramus. MOST common type (Figure 4-41):
122   Saunders Review of Dental Hygiene

1. Overbite: vertical overlap between maxillary and C. Angle’s classification of malocclusion: serves to ini-
mandibular anterior teeth, normal overbite is only tially address malocclusion (Table 4-4).
2 to 3 mm or one third overlap. D. Malposed dentition factors:
a. Measure vertical overlap with a periodontal 1. Oral musculature force (tongue thrusting, cheek or
probe and record in millimeters; described as lip biting, mouth breathing, abnormal tongue posi-
slight (in the incisal one third), moderate (in the tion) and tooth loss:
middle one third), or severe (in the gingival one a. Tongue exerts outward pressure, lips and cheeks
third). provide balancing inward force.
b. Over closed, deep bite, or severe overbite: max- b. Strong forward thrust of tongue can force teeth
illary incisors overlap into gingival one third of out of position, also happens when adult contin-
the mandibular incisors. ues swallowing patterns of childhood.
2. Overjet: horizontal distance between maxillary 2. Foreign object-to-tooth parafunctional habits (hold­
and mandibular incisors; normal overjet is only 2 ing pin or pipe between teeth, nail biting, finger
to 3 mm. and/or thumb sucking).
a. Measured as horizontal distance between lin- a. Outward pressure of a finger-sucking habit
guals of maxillary anterior teeth and facials of pushes maxillary anterior and jaw forward and
mandibular anterior teeth with a periodontal out of alignment.
probe and recorded in millimeters. 3. Tooth-to-tooth parafunctional habits (clenching
b. Excessive overjet is second MOST common and/or bruxism [grinding]).
form of malocclusion. 4. Teeth that are in contact with opposing teeth (oc-
3. Openbite: lack of incisal or occlusal contact or clusal antagonist) in opposing jaw undergo super-
overlap between maxillary and mandibular teeth; eruption (increased eruption) that can decrease al-
usually canines, laterals, centrals. veolar bone support and thus increase mobility.
4. Edge-to-edge bite (end to end): NOT any vertical 5. Iatrogenic factors: poorly contoured restorations,
overlap between the arches (incisal edge of maxil- improper fit of removable appliance.
lary teeth meets incisal edge of mandibular teeth); E. Primary dentition occlusal evaluation: tooth eruption
occurs with anteriors and/or posteriors. usually begins at 6 to 10 months of age and is com-
5. Anterior crossbite: maxillary incisors are lingual pleted at 2½ to 3 years of age; MOST variability in
to mandibular incisors. eruption for dentitions.
6. Posterior crossbite: maxillary posteriors are lin- 1. Primate spaces: normal developmental spaces,
gual or facial to ideal position (buccal or lingual to usually located between maxillary lateral incisors
maxillary arch). and canines and between mandibular canines and
7. Labioversion: tooth lies labial to normal position; first molars; necessary for proper alignment of fu-
linguoversion: tooth lies lingual to normal position. ture permanents.

Figure 4-41  Common misalignments. (From Bath-Balogh M, ­Fehrenbach MJ: Illustrated dental embryology
and anatomy, ed 2, ­Philadelphia, 2006, Saunders/Elsevier.)
Head, Neck, and Dental Anatomy   123

Table 4-4  Angle’s classification of malocclusion*

Class Model Arch relationships Descriptions

Class I Molar: MC cusp of the maxillary first Dental malalignment(s) present (see
occluding with the MB groove of text), such as crowding or spacing;
the mandibular first ­mesognathic profile; facial profile is flat
Canines: maxillary occluding with with chin in same plane as forehead
the distal half of the mandibular
canine and the mesial half of the
­mandibular first premolar

Class II Division I Molar: MB cusp of the maxillary first Division I: maxillary anteriors protruding
occluding (by more than the width facially from the mandibular anteriors,
of a premolar) mesial to the MB with deep overbite; retrognathic profile
groove of the mandibular first Division II: maxillary central incisors either
Canines: distal surface of the upright or retruded, and lateral incisors
mandibular canine distal to the me- either tipped labially or overlapping the
sial surface of the maxillary canine central incisors with deep overbite;
by at least the width of a premolar mesognathic profile.
Facial profile is convex with chin retruded
Division II

Class III Molar: MB cusp of the maxillary first Mandibular incisors in complete crossbite;
occluding (by more than the width prognathic profile; facial profile is
of a premolar) distal to the MB concave with chin protruded
groove of the mandibular first
Canines: distal surface of the
mandibular mesial to the mesial
surface of the maxillary by at least
the width of a premolar

From Bath-Balogh M, Fehrenbach MJ: Illustrated dental embryology and anatomy, ed 2, Philadelphia, 2006, Saunders/Elsevier.
MB, Mesiobuccal.
*Note that this system deals with the classification of the permanent dentition.

2. Terminal plane: permanent first molars are guided 1. Succedaneous (successional) replace primary
by distal surface of primary second molars into teeth; nonsuccedaneous (accessional) erupt distal
preferred positioning. in the arch and do NOT replace any primary teeth.
a. Mesial step: mandibular second molar is me- 2. Leeway space: sum width of MD widths of pri-
sial to maxillary molar; ideal molar relationship mary canines through primary second molars.
will occur. a. Enhances development of a normal occlusion in
b. Flush terminal plane: maxillary second molars permanent dentition.
lie end to end with mandibular second primary b. Usually greater on the maxillary arch; usually
molars; ideal molar relationship may occur. greater than the space needed for successional
3. Distal step: mandibular second molar is distal to teeth.
maxillary second molar, NOT type of terminal c. If primary is lost (caries or trauma) before per-
plane relationship; ideal molar relationship does manent is ready to replace it, space maintainer
NOT usually occur. (fixed or removable) can keep leeway space
4. Slight to moderate amounts of attrition owing to open.
bruxism; BOTH deep overbite and overjet are of- G. Permanent dentition occlusal evaluation: eruption of
ten present. all permanents (except third molars) is usually com-
F. Mixed dentition occlusal evaluation: BOTH pri- pleted by age of 12:
mary and permanent teeth are present (around 6 to 1. Centric occlusion (CO): maximum intercuspation
12 years): when jaws are fully closed (discussed earlier).
124   Saunders Review of Dental Hygiene

2. Centric relation (CR): MOST retruded (un-   5 Which of the following BEST describes the groove pattern
strained) relationship of mandible to the maxilla on the occlusal table of the mandibular second molar?
and MOST reproducible relationship; position A. Linear
from which lateral excursions are made. B. Snake eyes
3. Contact areas: protect interdental gingiva and sta- C. Crescent
D. Cross
bilize each tooth in arch; open contact allows food
  6 Which of the following teeth is considered succedaneous?
impaction and reduces mesiodistal stability. A. #13
4. Centric stops: occur in three places: (1) between B. #14
the two arches where the teeth contact; (2) at the C. #18
height of cusp contour of supporting cusps; (3) at D. #19
marginal ridges and central fossae.   7 Which of the following features is located on the lateral
5. Centric prematurity: occlusal factor that is deter- surface of the mandible?
mined when guiding the mandible into centric re- A. Lingula
lation closure. B. Submandibular fossa
6. Protrusive excursion: forward movement of lower C. Genial tubercles
jaw until mandibular anterior incisors meet the D. External oblique line
E. Mental foramen
maxillary incisors edge to edge.
  8 Which of the following bones of the skull is paired?
7. Lateral excursion: movement of the mandible from A. Sphenoid
centric occlusion to the right or left until canines B. Ethmoid
on the same side meet cusp tip to cusp tip. C. Occipital
D. Vomer
E. Parietal
Review Questions   9 Which of the following bones of the skull is considered a
facial bone?
A. Occipital
  1 How many erupted primary teeth does a 4-year-old child B. Parietal
have? C. Sphenoid
A. 10 D. Zygomatic
B. 12 E. Frontal
C. 16 10 Which of the following landmarks of the temporomandibu-
D. 18 lar joint is located on the mandible?
E. 20 A. Articular eminence
  2 Which of the following are considered functions of the B. Coronoid process
­incisors? C. Articular fossa
A. Biting and tearing D. Postglenoid process
B. Grinding and cutting 11 Which of the following lymph nodes are initially affected
C. Biting and cutting in a patient who develops an infection in the lower lip after
D. Tearing and grinding trauma from an accident?
  3 Which of the following structures or features are found in A. Submandibular
numbers of four in the oral cavity only during the perma- B. Deep cervical
nent dentition period? C. Submental
A. Molars D. Buccal
B. Dental arches E. Malar
C. Canine cusp slopes 12 Which of the following nerves serves the parotid salivary
D. Maxillary premolars gland and can be affected by medication, thus causing xero-
E. Quadrants stomia?
  4 When viewed proximally, the crown of the mandibular first A. Facial
molar is inclined B. Trigeminal
A. buccally. C. Glossopharyngeal
B. lingually. D. Chorda tympani
C. mesially. 13 Which of the following arteries of the head and neck pro-
D. distally. vides the MOST reliable arterial pulse of the body?
A. Internal carotid
B. Common carotid
C. Lingual
D. Facial
E. Superior thyroid
Head, Neck, and Dental Anatomy   125

14 Which muscle can become enlarged in the presence of the 23 Which of the following methods is MOST commonly used
parafunctional habit of bruxism? in the United States for the designation of teeth?
A. Mentalis A. Universal Tooth Designation System
B. Masseter B. Palmer Method for Tooth Designation
C. Orbicularis oris C. International Standards Organization Designation System
D. Risorius D. World Health Organization system
E. Epicranial 24 Which of the following statements is CORRECT concern-
15 Which of the following muscles is palpated during an extra- ing the pterygoid plexus of veins?
oral examination of the posterior cervical triangle? A. Located around the infrahyoid muscles
A. Suprahyoid B. Protects the superficial temporal artery
B. Infrahyoid C. Contains valves to prevent the backflow of blood
C. Sternocleidomastoid D. Injury can lead to hematoma
D. Temporalis 25 Which of the following teeth has an oblique ridge?
16 Which teeth may cause sensations that suggest a carious A. Mandibular second molar
and/or endodontic situation when only a sinus infection is B. Maxillary first molar
diagnosed? C. Mandibular third molar
A. Maxillary anteriors D. Maxillary third molar
B. Maxillary posteriors 26 Which tooth has a buccal pit that is susceptible to caries?
C. Mandibular anteriors A. Mandibular first molar
D. Mandibular posteriors B. Maxillary first molar
17 Which of the following salivary glands is MOST commonly C. Mandibular second molar
involved in stone formation? D. Maxillary second molar
A. Parotid 27 Which of the following structures is located just posterior to
B. Submandibular the MOST distal molar of the mandibular dentition?
C. Sublingual A. Maxillary tuberosity
D. Submandibular and sublingual B. Median palatine suture
18 Which of the following structures divides the tongue into C. Incisive foramen
the body and the base? D. Greater palatine foramen
A. Circumvallate lingual papilla E. Retromolar triangle
B. Sulcus terminalis 28 The mandibular third molar, compared with the mandibular
C. Foramen cecum second molar, is
D. Median lingual sulcus A. larger in the crown.
E. Lingual tonsil B. less smooth on its occlusal surface.
19 One of the following situations present in a primary denti- C. less variable in form.
tion will NOT allow the efficient eruption of a normal per- D. longer in the roots.
manent dentition. Which one is the EXCEPTION? 29 Which of the following nerves supplies the muscles of
A. Presence of primate spaces mastication?
B. Mesial step A. Hypoglossal
C. Distal step B. Vagus
D. Flush terminal plane C. Facial
20 Which of the following terms is used to describe the maxil- D. Chorda tympani
lary incisors overlapping the mandibular incisors in centric E. Trigeminal
occlusion? 30 The submandibular duct opens into the oral cavity
A. Overbite A. opposite the maxillary second molar.
B. Crossbite B. on the sublingual caruncle.
C. Overjet C. on the buccal mucosa.
D. End to end D. at the base of the mandibular labial frenum.
21 Which of the following nerves or branches of a nerve causes 31 Which of the following teeth has two roots?
discomfort during disorders of the temporomandibular A. Maxillary first molar
joint? B. Maxillary second premolar
A. Lingual C. Maxillary first premolar
B. Auriculotemporal D. Maxillary third molar
C. Inferior alveolar 32 Which of the following teeth has two pulp canals?
D. Buccal A. Maxillary first premolar
22 Which pair of bones forms the floor of the nasal cavity? B. Mandibular first premolar
A. Frontal and ethmoid C. Maxillary first molar
B. Ethmoid and lacrimal D. Mandibular first molar
C. Lacrimal and maxillary
D. Maxillary and palatine
E. Zygomatic and palatine
126   Saunders Review of Dental Hygiene

33 Which of the following muscles, when fully contracted, 40 Which of the following arteries is a branch of the maxillary
helps close the jaws? artery?
A. Lateral pterygoid A. Facial
B. Platysma B. Lingual
C. Mentalis C. Superior thyroid
D. Buccinator D. Inferior alveolar
E. Temporalis E. Ascending pharyngeal
34 Accumulation of food in the left vestibule might suggest 41 Which of the following lymph nodes receives lymphatic
malfunction of which of the following muscles? drainage from maxillary teeth?
A. Buccinator A. Buccal
B. Risorius B. Submental
C. Orbicularis oris C. Infraorbital
D. Medial pterygoid D. Submandibular
E. Levator anguli oris 42 Which of the following nerves innervates the mandibular
35 Damage to the facial nerve within the parotid salivary gland posterior teeth?
with local anesthesia may cause A. Mental
A. drooping of the lips. B. Buccal
B. spasm of the muscles of mastication. C. Incisive
C. temporary facial paralysis on the affected side. D. Inferior alveolar
D. deviation of the tongue to the affected side. 43 Which nerve listed is affected if a patient complains of be-
36 Which of the following structures are found in the pterygo- ing unable to experience touch, pain, hot, cold, or pressure
mandibular space? on the anterior two thirds of the tongue?
(1) Inferior alveolar nerve A. Vagus
(2) Lingual artery B. Lingual
(3) Hypoglossal nerve C. Hypoglossal
(4) Inferior alveolar artery D. Chorda tympani
(5) Sphenomandibular ligament E. Glossopharyngeal
A. (1), (2), and (3) 44 The nasopalatine nerve enters the oral cavity by way of
B. (1), (2), and (4) which of the following foramina?
C. (1), (4), and (5) A. Mental
D. (2), (3), and (4) B. Incisive
E. (2), (3), and (5) C. Pterygopalatine
37 Which of the following are contained within the sheath of D. Lesser palatine
the lingual nerve as it passes medial to the mandible, ante- E. Greater palatine
rior to the mandibular foramen? 45 Which of the following is supplied by the hypoglossal
A. Sensory fibers to the lip nerve?
B. Motor fibers to the masseter muscle A. Sublingual salivary gland
C. Parasympathetic motor secretory fibers to the subman- B. Muscles of the tongue
dibular gland C. Mucous membrane of the floor of the oral cavity
D. Special sense fibers to the anterior two thirds of the D. Muscles of facial expression
tongue 46 The rest position of the temporomandibular joint is NOT
E. Somatic sensory fibers to the posterior one third of the with the teeth biting together because during mastication of
tongue food the mandible returns to the center.
38 The floor of the mouth and the tongue both receive their A. Both the statement and reason are correct and related.
blood supply by way of which of the following arteries? B. Both the statement and reason are correct but NOT
A. Facial ­related.
B. Lingual C. The statement is correct, but the reason is NOT.
C. Mylohyoid D. The statement is NOT correct, but the reason is correct.
D. Maxillary E. NEITHER the statement NOR the reason is correct.
39 Which of the following nerves exits the cranium through 47 Which of the following nerves or branches of a nerve con-
the foramen ovale? tains pain fibers affected by disturbances of the temporo-
A. Facial mandibular joint?
B. Maxillary A. Chorda tympani
C. Ophthalmic B. Auriculotemporal
D. Mandibular C. Zygomaticotemporal
E. Glossopharyngeal D. Temporal branch of the facial nerve
Head, Neck, and Dental Anatomy   127

48 The parotid duct pierces which of the following muscles 57 The tooth that has the longest crown is the
before entry into the oral cavity? A. maxillary lateral incisor.
A. Masseter B. maxillary central incisor.
B. Mylohyoid C. mandibular canine.
C. Buccinator D. maxillary first molar.
D. Medial pterygoid 58 Blood vessels are more numerous than lymphatic vessels
49 The lateral pterygoid muscle inserts into the in the head and neck; however, the venous vessels mainly
A. coronoid process. parallel the lymphatic vessels in location.
B. articular eminence. A. Both statements are true.
C. mandibular condyle. B. Both statements are false.
D. angle of the mandible. C. The first is true, the second is false.
E. internal oblique line. D. The first is false, the second is true.
50 Pain impulses from the periodontal ligament are carried by 59 Which of the following premolars frequently lacks a trans-
which of the following cranial nerves? verse ridge?
A. I A. Maxillary first
B. II B. Maxillary second
C. V C. Mandibular first
D. VII D. Mandibular second
E. IX 60 How does the mandibular second molar differ in related
51 Mandibular teeth are vascularized by branches of which of numbers from the mandibular first molar?
the following arteries? A. Cusps
A. Facial B. Roots
B. Labial C. Lingual grooves
C. Lingual D. Marginal ridges
D. Maxillary 61 Where is the cingulum normally located on the teeth indi-
52 During instrumentation, longitudinal developmental gro­oves cated?
would probably be noted on which of the following root A. Incisal third of the lingual surface of anterior teeth
surfaces of a maxillary first premolar? B. Middle third of the lingual surface of posterior teeth
A. Lingual C. Cervical third of the lingual surface of anterior teeth
B. Mesial D. Occlusal third of the lingual surface of posterior teeth
C. Facial 62 In centric occlusion the normal relation of the maxillary
D. Palatal first molar to the mandibular arch is established when the
53 The roots of which of the following premolars present the A. maxillary first molar occludes with the mandibular sec-
greatest difficulty in endodontic therapy? ond premolar and first molar.
A. Maxillary first B. distal surface of the maxillary first molar is in the same
B. Maxillary second plane as the distal surface of the mandibular first molar.
C. Mandibular first C. distofacial cusp of the maxillary first molar falls in the same
D. Mandibular second plane as the distal surface of the mandibular first molar.
54 Which of the following occlusal factors is determined when D. mesiolingual cusp of the maxillary first molar falls in
guiding the mandible into centric relation closure? the central fossa of the mandibular first molar.
A. Canine rise 63 Where is the height of contour of the buccal surface of the
B. Degree of overbite mandibular first molar located?
C. Centric prematurity A. Junction of the occlusal and middle thirds
D. Working interference B. Center of tooth surface
E. Balancing interference C. Junction of the cervical and middle thirds
55 Which of the following is a normal pattern for eruption of D. Lingual of tooth surface
primary teeth? 64 The pulp horns MOST likely to be exposed accidentally in
A. Maxillary central incisors erupt before mandibular cen- the preparation of a Class II cavity in the maxillary first
tral incisors. molar are the
B. Maxillary canines erupt before maxillary lateral incisors. A. mesiobuccal and mesiolingual.
C. Maxillary first molars erupt before maxillary canines. B. mesiolingual and distolingual.
D. Mandibular canines erupt before mandibular first molars. C. distolingual and distobuccal.
E. Mandibular second molars erupt before mandibular first D. distobuccal and mesiobuccal.
molars. 65 What does the vermilion border consist of ?
56 Which of the following premolars often has three cusps? A. Internal lining of the lip
A. Maxillary first B. Inner epithelial lining of the cheeks
B. Maxillary second C. Junction of the lip with the skin
C. Mandibular first D. Formed by palatopharyngeus muscle
D. Mandibular second
128   Saunders Review of Dental Hygiene

66 Where is the soft palate located? 74 During the extraoral examination, feeling inferior to and
A. Dorsal aspect of the oral cavity medial to the angles of the mandible is important because
B. Just posterior to the hard palate this will allow the dental hygienist to effectively palpate the
C. Center of the hard palate hyoid bone.
D. Within the pillars A. Both the statement and reason are correct and related.
67 Where is the median palatine raphe more prominent? B. Both the statement and reason are correct but NOT
A. Hard palate ­related.
B. Soft palate C. The statement is correct, but the reason is NOT.
C. Tonsillar pillars D. The statement is NOT correct, but the reason is correct.
D. Near oropharynx E. NEITHER the statement NOR the reason is correct.
68 Where is the lingual tonsil located? 75 The lymphatics are a portion of the immune system, and
A. Posterior to the circumvallate lingual papillae they help fight disease processes. Another component of
B. Posterolateral border of the tongue the lymphatic system is the thymus gland because it works
C. Along the sulcus terminalis on the tongue within the immune system.
D. Lateral border and anterior tip of the tongue A. Both statements are true.
69 The thyroid gland produces thyroxine, which it secretes B. Both statements are false.
directly into the blood. Thyroxine is a hormone that stimu- C. The first is true, and the second is false.
lates the metabolic rate. D. The first is false, and the second is true.
A. Both statements are true. 76 The dental professional must be thoroughly familiar with
B. Both statements are false. the surface anatomy of the head and neck to examine pa-
C. The first statement is true, the second is false. tients because features of the surface provide essential land-
D. The first statement is false, the second is true. marks for many deeper anatomical structures.
70 Which of the following glands is unencapsulated? A. Both the statement and reason are correct and related.
A. Submandibular B. Both the statement and reason are correct but NOT
B. Thyroid ­related.
C. Parotid C. The statement is correct, but the reason is NOT.
D. Sublingual D. The statement is NOT correct, but the reason is correct.
71 The temporal bone and the mandible come together to form E. NEITHER the statement NOR the reason is correct.
the temporomandibular joint because a joint is defined as a 77 An extrinsic tongue muscle that retracts the tongue is the
junction or union between two or more bones. A. palatoglossus muscle.
A. Both the statement and reason are correct and related. B. inferior longitudinal muscle.
B. Both the statement and reason are correct but NOT re- C. styloglossus muscle.
lated. D. genioglossus muscle.
C. The statement is correct, but the reason is NOT. 78 Occlusal evaluation defines the contact relationship ­between
D. The statement is NOT correct, but the reason is correct. the maxillary and mandibular teeth when the
E. NEITHER the statement NOR the reason is correct. A. anterior teeth are contacting end to end.
72 The lymphatics of the right side of the head and the neck B. teeth are fully closed.
converge by way of the right jugular trunk. Then these lym- C. mandibular teeth are in the most protruded position.
phatics join the lymphatics from the right arm and thorax to D. maxillary and mandibular first molar interproximal
form the thoracic duct. spaces coincide.
A. Both statements are true. E. contact occurs only on posterior teeth.
B. Both statements are false. 79 Overbite presents with which of the following situations?
C. The first is true, the second is false. A. Vertical overlap between maxillary and mandibular an-
D. The first is false, the second is true. terior teeth
73 The superior and posterior free margin of the auricle, known B. Horizontal overlap between maxillary and mandibular
as the helix, ends inferiorly at the lobule. The lobule is a anterior teeth
small flap of tissue, which is the part of the auricle anterior C. Vertical overlap of the first molars when the anterior
to the external acoustic meatus. teeth are missing
A. Both statements are true. D. Horizontal overlap of the first molars when the anterior
B. Both statements are false. teeth are missing
C. The first is true, the second is false. 80 In the primary dentition, when the maxillary second molars
D. The first is false, the second is true. occlude end to end with the mandibular second molars, it is
referred to as which of the following?
A. Mesial step
B. Distal step
C. Flush terminal plane
D. End-to-end occlusion
E. Class I occlusion
Head, Neck, and Dental Anatomy   129

l­ocated on temporal bone. Coronoid process is a thin,


Answer Key and Rationales triangular eminence, flattened from side to side.
11 (C)  Lower lip drains directly into submental lymph
nodes, which serve as primary nodes during an infec-
1 (E)  All of 4-year-old child’s primary teeth would tion. Submandibular and cervical nodes would serve
have erupted into oral cavity because average age for as secondary nodes if infection progressed. Buccal
primary (deciduous) dentition completion is approxi- and malar nodes drain upper and middle cheek.
mately age 3. There are 20 teeth in the primary denti- 12 (C)  Parotid gland is supplied by ninth cranial nerve
tion. (glossopharyngeal, IX) (with preganglionic parasym-
2 (C)  Incisors function as instruments for biting and pathetic innervation). Even though seventh cranial
cutting food during mastication because of incisal nerve (facial, VII) travels through parotid, does NOT
ridge, triangular proximal form, and arch position. supply it. Chorda tympani supplies submandibular
There are eight incisors, two of each type, central and and sublingual salivary glands, which may also be af-
lateral. fected by drugs to produce xerostomia (dry mouth).
3 (D)  Only during permanent dentition period are four Trigeminal (fifth cranial nerve [V]) serves many other
maxillary premolars (bicuspids) present, two of each oral cavity structures but NOT the parotid.
type, first and second. The two types of molars, first 13 (B)  Common carotid artery provides MOST reliable
and second molars, are found during primary denti- carotid pulse from carotid sinus, the swelling of the
tion period. The three types of molars are found dur- common carotid before it bifurcates into internal and
ing permanent dentition period. Only two arches are external carotid arteries. Others are branches from in-
found in BOTH primary and permanent dentition ternal carotid artery after bifurcation. Carotid pulse is
periods. Only two canine cusp slopes per tooth are used in emergencies ONLY by emergency personnel,
found during BOTH periods. There are four quad- and radial pulse is used when taking vital sign of a
rants, present during BOTH primary and permanent baseline pulse at other times.
dentition periods. 14 (B)  Masseter muscle can become enlarged (hyper-
4 (B)  ALL permanent mandibular molars, including trophied) in a patient who habitually grinds (brux-
first molar, show strong lingual inclination when ism) or clenches the teeth. Action of masseter muscle
viewed proximally. This inclines crown lingually on during bilateral contraction of the entire muscle is to
root base, bringing cusps into proper occlusion with raise or elevate the mandible, thus raising the lower
maxillary antagonists and the distribution of forces jaw. Elevation of the mandible occurs during closing
along the long axis. of the jaws or grinding of the teeth. ALL others are of
5 (D)  Cross-shaped groove pattern is formed on oc- facial expression and NOT of mastication.
clusal table of permanent mandibular second molar 15 (C)  Sternocleidomastoid muscle (SCM) divides each
when the well-defined central groove is crossed by side of the neck into the anterior and posterior cervi-
buccal groove and lingual groove, dividing the oc- cal triangle. Posterior cervical triangle is located on
clusal table into four parts that are NEARLY equal. side of the neck, and the anterior cervical triangle
6 (A)  Tooth #13 (permanent second premolar) is suc- ­corresponds to the anterior region of the neck that
cedaneous for primary second molar. ALL others are contains both the suprahyoid and infrahyoid muscles.
permanent molars, and ALL molars are nonsucceda- Temporalis is located in the temporal region on the
neous because they do NOT have primary predeces- lateral surface of the skull.
sors and erupt distal to primary second molar. 16 (B)  Discomfort from sinusitis can be confused with
7 (E)  Mental foramen is located on lateral surface of tooth-related discomfort from the maxillary posteri-
mandible. ALL others are located on internal (me- ors because the roots are in close proximity to max-
dial) surface of the mandible. illary sinus. Others are NOT near maxillary sinus,
8 (C)  Occipital bone is the ONLY paired bone of skull. the sinus MOST commonly involved in sinus infec-
Others are single bones of skull. tions.
9 (D)  Zygomatic bone is considered a facial bone be- 17 (B)  Duct for submandibular gland is the submandib-
cause it helps create facial features and serves as base ular duct (Wharton’s duct); long duct travels along
for the dentition. Vomer, lacrimal, palatine, maxilla, the anterior floor of the mouth. Its tortuous upward
mandible are also facial bones. ALL others are con- travel for a considerable distance during its course
sidered cranial bones because they form cranium. may be the reason that the gland is MOST commonly
Cranial bones form cranium and include occipital, involved in salivary stone formation.
frontal, parietal, temporal, sphenoid, ethmoid bones. 18 (B)  Sulcus terminalis divides tongue into posterior
10 (B)  Coronoid process is located on mandible and base and anterior body. Dorsum (top) surface is con-
is part of TMJ. ALL others are part of joint but are vex and marked by sulcus terminalis; this sulcus ends
130   Saunders Review of Dental Hygiene

behind, about 2.5 cm from the root of the tongue, in When it is pierced, small amount of blood escapes
a depression, foramen cecum, from which a shallow and enters the tissues, causing a hematoma. Can be
groove, terminal sulcus, runs lateralward and forward pierced when posterior superior alveolar (PSA) block
on either side to the margin of tongue. Others are also is administered and needle overreaches for target of
on dorsal surface. apices of maxillary posteriors.
19 (C)  Distal step will NOT allow efficient eruption of a 25 (B)  Maxillary first molar has an oblique ridge that is
normal permanent dentition. Distal step occurs when formed by union of triangular ridge of the DB cusp
primary mandibular second molar is distal to maxil- and distal cusp ridge of the ML cusp, crossing occlu-
lary second molar and thus is NOT in a terminal plane sal table obliquely. All others do NOT have oblique
relationship. Mesial step and flush terminal plane are ridges.
BOTH types of terminal plane relationships that al- 26 (A)  Mandibular first molar’s MB groove almost
low efficient eruption, along with primate spaces. always ends in the buccal pit, susceptible to caries
Flush terminal plane occurs when primary maxillary because of increased dental biofilm retention and be-
and mandibular second molars are in an end-to-end cause of the thin enamel that forms the walls of the
relationship; mesial step occurs when primary man- pit. Enamel sealants can be placed in buccal pit to
dibular second molar is mesial to maxillary molar. If protect the tooth from caries. All others do NOT have
primate spacing exists in primary mandibular arch buccal pits.
after eruption of permanent first molar, this tooth will 27 (E)  Retromolar triangle is located just posterior to
put pressure on primary second and first molars, caus- MOST distal molar of mandibular dentition. ALL oth-
ing forward movement of primary mandibular canine ers are found on maxilla, near maxillary dentition.
and first molar. 28 (B)  Generally, the MORE posterior the tooth, the
20 (A)  In centric occlusion, maxillary incisors over- MORE supplemental grooves are present, such that
lap mandibular incisors, causing an overbite. When occlusal table appears more “wrinkled.” Thus man-
maxillary dental arch naturally “overhangs” the man- dibular third molar is LESS smooth on its occlusal
dibular arch facially, an overjet is caused. Crossbite surface than third molar of same arch. Others refer to
occurs when mandibular tooth or teeth are placed fa- second molar and NOT the third molar.
cially to maxillary teeth. End-to-end bite occurs when 29 (E)  Muscles of mastication are four paired muscles
teeth occlude WITHOUT maxillary teeth overlapping attached to the mandible; include masseter, tempora-
mandibular teeth. lis, medial pterygoid, lateral pterygoid. Mandibular
21 (B)  Auriculotemporal nerve serves as an afferent division of fifth (V) cranial nerve, trigeminal, inner-
nerve for the external ear and scalp near TMJ. Au- vates ALL muscles of mastication. Seventh (VII) cra-
riculotemporal nerve is a branch of mandibular nerve nial nerve is the facial. Tenth (X) cranial nerve is the
that runs with the superficial temporal artery and vein vagus. Twelfth (XII) cranial nerve is the hypoglossal.
and provides sensory innervation to various regions Chorda tympani is a branch of the facial nerve (sev-
on the side of head. ALL others are afferent for oral enth [VII] cranial nerve). However, others innervate
structures. important structures of the oral cavity.
22 (D)  Horizontal plates of palatine bones and the pala- 30 (B)  Submandibular duct opens up into the oral cavity
tine process of maxilla together form floor of nasal on the sublingual caruncle, on the floor of the mouth.
cavity. Also forms the anterior portion of hard pal- Parotid duct opens opposite maxillary second molar on
ate (roof) of the oral cavity. ALL others form orbital buccal mucosa. Mandibular labial frenum has ONLY
walls. minor salivary gland ducts nearby in labial mucosa.
23 (A)  Universal Tooth Designation System is the 31 (C)  Maxillary first premolar has two roots. ALL max-
MOST used system in United States for the designa- illary molars have three roots (trifurcated); however,
tion of BOTH dentitions because it is adaptable to sometimes the roots of the third molar are so close
electronic data transfer. Palmer Method for Tooth together that they are fused roots, either partially or
Designation is used in orthodontic offices. Interna- fully, giving appearance of one root. Maxillary sec-
tional Standards Organization Designation System is ond premolar has one root and only occasionally has
used MAINLY internationally by World Health Orga- two roots (bifurcated).
nization (WHO) system. 32 (A)  Pulp cavity for maxillary first premolar has two
24 (D)  Pterygoid plexus of veins is around pterygoid pulp canals, even if there is ONLY one undivided
muscles and surrounds maxillary artery on each side root. Mandibular first premolar has one canal. BOTH
of the face in infratemporal fossa. Protects maxillary maxillary and mandibular first molars MOSTLY have
artery from being compressed during mastication and three canals.
may be involved in the spread of infection to the cav- 33 (E)  BOTH lateral pterygoid and temporalis are mus-
ernous venous sinus because it does NOT have valves. cles of mastication that affect the movement of the
Head, Neck, and Dental Anatomy   131

jaws. If the entire temporalis muscle contracts, its orbital fissure, second division (maxillary, V2)
MAIN action is to elevate the mandible, thus rais- through foramen rotundum (round shaped), and the
ing the lower jaw. Elevation of the mandible occurs third (mandibular, V3) through foramen ovale (oval
during the closing of the jaws. MAIN action when shaped). Fissure and foramen are located in the sphe-
BOTH lateral pterygoid muscles contract is to bring noid. Fissure also caries third, fourth, sixth cranial
the lower jaw forward, thus causing the protrusion of nerves and ophthalmic vein.
the mandible. Protrusion of the mandible often occurs 40 (D)  Maxillary artery is one of the two terminal
during the opening of the jaws. If ONLY one lateral branches of the external carotid artery. Branches sup-
pterygoid muscle is contracted, the lower jaw shifts ply teeth and supporting structures of BOTH arches.
to the opposite side, causing a lateral deviation of the Blood supply to mandibular teeth and supporting
mandible. ALL other muscles listed are muscles of structures is MAINLY from inferior alveolar branch
facial expression. of the maxillary artery.
34 (A)  Buccinator muscle is the muscle of the cheek 41 (D)  Submandibular lymph node receives lymphatic
and aids in the cleaning of food from the oral ves- drainage from maxillary teeth. Facial nodes are po-
tibule. Because the buccinator is innervated by the sitioned along the length of the facial vein to drain
buccal branch of the facial nerve (seventh cranial the area and include the buccal and infraorbital.
nerve [VII]), injury to nerve will affect the action of However, do drain into each other and then into the
the muscle (e.g., Bell’s palsy). submandibular nodes. Submental nodes drain BOTH
35 (C)  Damage to facial nerve (seventh cranial nerve sides of the chin, lower lip, floor of the mouth, apex
[VII]) within the parotid gland may cause unilateral of the tongue, mandibular incisors, then empty into
transient facial paralysis of the facial muscles on the submandibular nodes or deep cervical nodes.
affected side. Temporary situation can occur when a 42 (D)  Inferior alveolar (IA) nerve innervates the
local anesthetic agent is injected by overreaching the mandibular posteriors. Buccal (long buccal) nerve
needle during a inferior alveolar block into the parotid is afferent for the skin of the cheek, buccal mucous
gland, since it contains the facial nerve; nerve serves membranes, buccal gingiva of mandibular posteriors.
the muscles of facial expression on each side. Thus Mental nerve is composed of external branches that
patient will have a drooping lip on one side but NOT are afferent for the chin, lower lip, and labial mucosa
both sides. Contacting medial surface of the ramus near the mandibular anteriors.
of the mandibular bone during the injection prevents 43 (B)  Lingual nerve is affected if a patient complains
this from occurring. of being unable to experience touch, pain, hot, cold,
36 (C)  Following structures are found in pterygoman- or pressure on the anterior two thirds of the tongue,
dibular space: inferior alveolar (IA) nerve, IA ar- which is a branch of the mandibular nerve from fifth
tery, sphenomandibular ligament. Injection site for (V) cranial nerve (trigeminal). Ninth (IX) cranial nerve
IA nerve block. Lingual artery will be superficial to (glossopharyngeal nerve) covers the general sensation
this site. Hypoglossal (twelfth cranial nerve [XII]) is from the base of the tongue. Tenth (X) cranial nerve
found in the hypoglossal canals of occipital bone. (vagus nerve) is efferent for muscles of the soft palate,
37 (D)  Special sense fibers to anterior two thirds of the pharynx, and larynx. Chorda tympani is a branch of
tongue are contained within the sheath of lingual the facial nerve (seventh cranial nerve [VII]) and af-
nerve as it passes medial to the mandible (and infe- ferent for taste sensation for the body of the tongue.
rior alveolar nerve), anterior to mandibular foramen. 44 (B)  Nasopalatine (NP) nerve enters the oral cavity
Tongue thus becomes anesthetized as the local anes- by way of incisive foramen on hard palate. Lesser
thetic agent diffuses to lingual nerve FIRST during palatine (LP, posterior palatine) nerve enters LP fora-
the IA nerve block. Lingual nerve shock can also oc- men in the palatine bone. Mental nerve enters men-
cur when the needle touches lingual nerve during the tal foramen. Greater palatine (GP, anterior palatine)
injection. nerve enters the GP foramen in the palatine bone.
38 (B)  Floor of the mouth and tongue receive blood 45 (B)  Muscles of the tongue are supplied by hypoglos-
supply by way of lingual artery. Facial artery with its sal (twelfth cranial nerve [XII]), which is efferent
major branches supplies the face in the oral, buccal, for intrinsic and extrinsic muscles of the tongue and
zygomatic, nasal, infraorbital, and orbital regions. exits the skull through the hypoglossal canal. Facial
Mylohyoid artery supplies the floor of the mouth and (seventh cranial nerve [VII]) serves muscles of facial
the mylohyoid muscle. Maxillary artery has branches expression. Lingual nerve serves floor of the mouth,
near the muscles supplied. as well as submandibular gland (parasympathetic ef-
39 (D)  Mandibular nerve is the third of the three divi- ferent innervation) and branch off mandibular nerve.
sions of the fifth (V) cranial nerve (trigeminal). First 46 (B)  Both statements are true but are not re-
division (ophthalmic, V1) exits through ­ superior lated. Statement describes the rest position of the
132   Saunders Review of Dental Hygiene

­temporomandibular joint (TMJ), while the reason de- roots; has one buccal and one lingual (palatal) root.
scribes the power stroke, which occurs during mas- May be fused or laminated. In addition, crown is wid-
tication. Teeth are NOT occluding or biting together est mesiodistally of all premolars.
when the TMJ is at rest. Mandible is brought back 54 (C)  Centric prematurity is an occlusal factor that is
to the center during mastication owing to the power determined when guiding mandible into centric rela-
stroke or the teeth crunching the food. tion closure. Centric stops occur in three places be-
47 (B)  Auriculotemporal nerve contains pain fibers af- tween the two arches where the teeth contact: at the
fected by disturbances of the temporomandibular height of cusp contour of supporting cusps, at the
joint (TMJ). Also serves as an afferent nerve for the marginal ridges, and at the central fossae. With canine
external ear and scalp near the TMJ. Chorda tympani rise, the tooth should be the only tooth in function dur-
is a branch of the facial nerve (seventh cranial nerve ing lateral occlusion. Overbite occurs as the maxillary
[VII]) and afferent for taste sensation for the body incisors overlap the mandibular incisors to allow max-
of the tongue. Zygomatic nerve is composed of the imum contact of the posteriors. Overjet occurs when
union of zygomaticofacial and zygomaticotemporal the maxillary dental arch naturally overhangs the
in orbit and is afferent; conveys postganglionic para- mandibular arch facially. BOTH are measured in mil-
sympathetic fibers to the lacrimal. limeters with a probe. Evaluation of lateral occlusion
48 (C)  Buccinator muscle is pierced by parotid duct is made by moving the mandible laterally left until the
before entry into the oral cavity, after it emerges canines on that side are in a cusp-to-cusp relationship.
from the anterior border of the gland. Parotid duct Side to which the mandible has been moved is the
(Stensen’s) serves the gland; it opens up into the oral working side; opposite side is the balancing side.
cavity at the parotid papilla on the buccal mucosa, 55 (C)  Primary maxillary first molars (B, I) erupt at 13
opposite maxillary first molar. to 19 months, before maxillary canines (C, H) (16 to
49 (C)  Lateral pterygoid inserts into the mandibular 22 months), which is a normal pattern for eruption of
condyle; this includes BOTH heads. Superior head primary (deciduous) teeth. Unless designated other-
originates from greater wing of the sphenoid; inferior wise, the teeth in the exam SHOULD be considered a
head originates from lateral pterygoid plate. Tempora- part of the permanent (adult) dentition.
lis inserts on the coronoid process of mandible. Mas- 56 (D)  Mandibular second premolar (#20, #29) has three
seter inserts on the lateral surface of the angle. Medial cusps in one of its two types. Its tricuspidate form has
pterygoid inserts on medial surface of the angle of the one large buccal cusp and two smaller lingual cusps;
mandible. ALL are muscles of mastication. grooves form a distinctive Y-shaped pattern on the oc-
50 (C)  Fifth (V) cranial nerve (trigeminal) carries pain clusal table; resembles small molar. Other premolars
impulses from the periodontal ligament (PDL) of ALL have two cusps.
the teeth to the brain by its two MAJOR nerve branches, 57 (C)  Mandibular canine (#22, #27) has longest crown
maxillary and mandibular nerves, which serve (respec- in the permanent dentition; can be as long as or even
tively) the maxillary and mandibular teeth. LONGER than maxillary canine.
51 (D)  Mandibular teeth are vascularized by branches 58 (D)  First statement is false: blood vessels are LESS
of the maxillary arteries, by way of inferior alveo- numerous than lymphatic vessels. Second statement
lar (IA) artery, as well as the maxillary teeth. The is true: venous vessels MAINLY parallel lymphatic
IA arises from the maxillary artery in infratemporal vessels in location.
fossa, turns inferiorly to enter the mandibular fora- 59 (B)  Mandibular second premolar (#20, #29) has
men, and then enters the mandibular canal and the either two or three cusps. The two-cusp type has a
inferior alveolar nerve; it branches into mylohyoid transverse ridge, the MORE common three-cusp type
before it enters the canal. In the canal it branches into does NOT. Bicuspidate form is similar to that of the
mandibular posterior and alveolar (dental) branches mandibular first premolars; has one larger buccal
to supply the teeth and periodontium of these teeth, cusp and one smaller lingual cusp; central groove is
and also associated gingiva. crescent or U shaped; appears rounded from occlusal.
52 (B)  Longitudinal developmental grooves (root con- 60 (A)  Mandibular second premolar (#20, #29) differs
cavity) would probably be noted on mesial root sur- from the mandibular first molar (#19, #30) by the
faces of maxillary first premolar (#5, #12). Because number of cusps; has either two or three cusps. Molar
of its depth, there may be increased deposits in this has five cusps. BOTH teeth have two roots and two
area that may require instrumentation. marginal ridges, and BOTH have a lingual groove
53 (A)  Roots of the maxillary first premolar (#5, #12) that cuts the occlusal outline on the lingual surface, if
present GREATEST difficulty in endodontic therapy the premolar has the bicuspid form.
as compared with all the other premolars, whether 61 (C)  Cingulum is normally located in cervical third of
first or second or even mandibular, because of its two the lingual surface of anteriors. Includes incisors and
Head, Neck, and Dental Anatomy   133

c­ anines, BOTH maxillary and mandibular teeth. Raised, papillae are mostly located on the lateral border and
rounded area; varying degrees of development. anterior apex (tip) of the tongue.
62 (D)  In centric occlusion the normal relation of the 69 (A)  Both statements are true. Thyroxine (T4) is the
permanent maxillary first molar to the mandibular MAJOR hormone secreted by the follicular cells of
arch is established when the mesiolingual cusp of the the thyroid gland. T4 is transported in blood. T4 is in-
maxillary first molar falls in the central fossa of the volved in controlling the rate of metabolic processes
mandibular first molar. in the body and influencing physical development.
63 (C)  Height of contour of the buccal surface of the Increases rate of chemical reactions in cells and helps
mandibular first molar (#19, #30) is located at the control growth and development. Thyroxin is version
junction of the cervical and middle thirds. Height of manufactured in the laboratory; used to treat thyroid
contour is the greatest elevation of tooth, either inci- disorders.
socervically or occlusocervically, on a specific sur- 70 (D)  Sublingual gland is NOT capsulated; others
face of the crown. (submandibular, thyroid, parotid) are encapsulated.
64 (A)  Pulp horns MOST likely to be exposed ac- Gland is located in the sublingual fossa in sublingual
cidentally in the preparation of a Class II cavity in fascial space at the floor of the mouth. Superior to
the maxillary first molar are the MB and ML. Cavity mylohyoid, medial to the body of mandible, and an-
preparation would involve the occlusal surface of the terior to submandibular gland.
tooth. Tooth has three horns, one for each major cusp. 71 (A)  Statement is correct: temporomandibular joint
The ML and MB are BOTH very large horns, and the (TMJ) is a junction between temporal bone and man-
MB is large on all molars and can be exposed more dible. Reason is also correct: joint is defined as union
easily during cavity preparation than the other horns. between two or more bones. Thus both statement and
65 (C)  Vermilion border is smooth, well-delineated, reason are related.
and slightly raised junction of the lip with the skin. 72 (C)  First statement is true: lymphatics of right side
Internal lining of the lip is the labial mucosa. Inner of head and neck converge by way of right jugular
epithelial lining of the cheeks is the buccal mucosa. trunk. Second statement is false: lymphatics on right
Posterior tonsillar pillar is formed by palatopharyn- side of head and neck converge by way of right jugu-
geus muscle. lar trunk to join lymphatics from right arm and thorax
66 (B)  Soft palate is just posterior to the hard palate. to form right lymphatic duct, which drains into ve-
Loc­ation of the hard palate is on the dorsal aspect of nous system at junction of right subclavian and right
the oral cavity. Median palatal raphe is in the center internal jugular veins. Lymphatic vessels of left side
of the hard palate. Tonsils (palatal), which are lym- of head and neck converge into left jugular trunk, a
phoid tissue, lie between the tonsillar pillars. Soft short vessel, and then into thoracic duct.
palate is the soft tissue constituting the back of the 73 (C)  First statement is true: superior and posterior
roof of the mouth; distinguished from the hard palate free margin of the auricle, known as the helix, ends
at the front of the mouth in that it does NOT contain ­inferiorly at lobule. Second statement is false; this
bone. is the definition of the tragus, NOT the lobule. Thus
67 (B)  Median palatal raphe is MORE prominent and tragus is a small flap of tissue, which is the part of
thicker in the center of soft palate, clinical evidence auricle anterior to external acoustic meatus.
of the median palatine suture line showing the fusion 74 (A)  Both the statement and the reason are correct
of the palatal processes from the maxillary process and related: hyoid bone is located in anterior mid-
during the development of the secondary palate. line, superior to thyroid cartilage, where the angles of
Median palatal cyst is a rare cyst that may ­ occur the mandible are located. Angle of mandible is also
anywhere along the median palatal raphe; may pro- the landmark used to locate the hyoid bone. Palpa-
duce a swelling because of infection, treated by ex- tion of angles of mandible is also part of extraoral
cision or surgical removal. Soft palate is located in examination.
the posterior part of the palate (roof) of the oral cav- 75 (C)  First statement is true: lymphatics are a portion
ity; soft tissue constituting the back of the roof of of the immune system and help fight disease pro-
mouth; distinguished from hard palate at the front of cesses. Second statement is false: thymus gland is
mouth in that it does NOT contain bone. NOT a component of lymphatic system even though
68 (A)  Lingual tonsil is located on dorsal lateral aspect it works within the immune system.
of the tongue, posterior to circumvallate lingual pa- 76 (A)  Both the statement and reason are correct and
pillae. Lingual tonsil is an irregular mass of tonsillar are related. Dental professionals must have knowl-
tissue. Location for the foliate is on the posterolateral edge of normal or healthy structures to identify
border of the tongue. Location of circumvallate is and locate deeper anatomical landmarks, which are
along the sulcus terminalis on the tongue. Fungiform needed to perform certain dental procedures such as
134   Saunders Review of Dental Hygiene

­administration of local anesthetic or radiographic ex- 80 (C)  Flush terminal plane occurs in the primary denti-
posures. tion when the maxillary second molars occlude end
77 (C)  Styloglossus muscle moves the tongue supe- to end with the mandibular second molars; ideal mo-
riorly and posteriorly. Palatoglossus elevates the lar relationship may occur. Mesial step occurs when
tongue against the soft palate during swallowing. In- mandibular second molar is mesial to maxillary mo-
ferior longitudinals are intrinsic tongue muscles, and lar; ideal molar relationship will occur. Distal step is
the genioglossus acts to protrude the tongue. when mandibular second molar is distal to maxillary
78 (B)  Teeth have to be fully closed for evaluation of second molar, NOT type of terminal plane relation-
occlusion. ship; ideal molar relationship will rarely occur. An
79 (A)  Vertical overlap between maxillary and man- end-to-end occlusion occurs when the buccal cusps
dibular anterior teeth is called overbite. Overjet is the of maxillary and mandibular molars occlude with one
horizontal overlap between maxillary and mandibu- another. A Class I occlusion applies to the permanent
lar anterior teeth. dentition.
CH A PTER 5

c0005 Radiology
s9000 FUNDAmental radiation physics   b. Some forms are ionizing and others are not, o0220
Understanding radiation physics requires a basic knowl- ­ depending on their energy (wavelengths) (e.g.,
edge of atomic structure, radiation types, properties of x-rays have short wavelengths and are ionizing).
radiation. c. Arranged in a spectrum according to its ener- o0230
• See CD-ROM for Chapter Terms and WebLinks. gies, which is demonstrated by its wavelengths
A. Atomic structure: nucleus and electrons in orbiting and frequencies; wave concept suggests that
shells of an atom function in state of equilibrium until electromagnetic radiation is much like waves or
ionization occurs (Figure 5-1): ripples of water and involves wave frequency
1. Nucleus: and length.
o0030 a. Contains protons, with positive charge. (1) Wavelength:
o0040 b. Contains neutrons, with NO electrical charge. (a) Distance between the peaks or crests of
2. Orbiting shells: a wave.
o0060 a. Contain electrons, with negative charge, and (b) Determines energy of radiation; longer
circle the nucleus in shells. the distance, LESS energetic the radia-
o0070 b. Have binding energy, which keeps electrons in tion, LESS ability it has to penetrate
their shells as a result of centripetal force and objects.
attraction of opposites. (c) Measured according to distance be-
3. Ionization: tween peaks; longer wavelengths are
o0090 a. Occurs when an atom loses an electron from measured in meters; shorter wave-
one of its shells and becomes part of ion lengths are measured in nanometers
pair (not unlike the celebrity world with its (0.01 to 0.05 nm).
changing pairs); this process elicits chemical (2) Frequency:
changes in matter. (a) Number of peaks or crests that occur in
o0100 b. Ion pair is created when an x-ray photon ejects a given amount of time.
a negative electron from its shell and neutral (b) Frequency of the peaks determines en-
atom becomes positive. ergy of the wavelength; MORE frequent
o0110 c. Ionizing radiation is any radiation that is the peaks, shorter and MORE energetic
­capable of producing ions. the radiation.
B. Types of radiation: particulate and electromagnetic C. Properties of x-rays:
radiation are types of ionizing radiation: 1. X-rays are ionizing forms of radiation that have
1. Particulate (corpuscular) radiation: unique properties.
o0140 a. Involves mass and particles that travel in 2. Bundles of pure energy (short wavelengths) that
straight lines at high speeds. have NO electrical charge.
o0150 b. Four types: 3. Travel at speed of light (3 × 108 m/sec) (The
(1) Electrons: originate from radioactive ­atoms; Flash!).
called beta particles. 4. Invisible, weightless, undetectable by any of the
(2) Protons: heavily charged particles (namely, senses.
hydrogen nuclei). 5. Absorbed by matter according to atomic structure
(3) Neutrons: have mass but NO electrical of material that is exposed.
charge. 6. Cause ionization, which produces biological
(4) Alpha particles: large and emitted from change in interaction with matter.
nucleus of heavy metals; usually unable to
penetrate tissue. X-Ray Machine and Components
2. Electromagnetic radiation: X-ray machine has basic operable components and inter-
o0210 a. Comprises electric and magnetic fields of ­energy nal components that are important for safe operation and
that move through space in wavelike motion. functioning.

135
136   Saunders Review of Dental Hygiene

Tungsten
Copper sleeve X-ray tube filament Oil
Tungsten target

Nucleus Metal housing of
+ + Proton x-ray tubehead
Neutron
+
Unleaded
– glass window Filter Focusing cup
– Electron Collimator
PID "cone"

Figure 5-1  Atomic structure.


Figure 5-2  Components of x-ray machine.

A. Basic operable components: include control panel, 2. Power supply:


tubehead, position-indicating device (PID): a. Electricity that provides energy to unit to pro-
1. Control panel: duce x-rays and is described by its current, am-
a. Operated by visible on-off switches, exposure perage, voltage.
button, indicator light. b. Electric current (or flow) of electrons through a
b. Controls milliamperage, time, kilovoltage set- wire in a given point of time:
tings by dials on panel. (1) When a direct current (DC), electrons flow
2. Tubehead (Figure 5-2): in one direction.
a. Heavy metal housing for transformers and (2) When an alternating current (AC), elec-
x-ray tube that serves as attachment for PID, trons flow in one direction and then change
filter, collimator. to flow in another direction.
b. Contains insulating oil, which surrounds x-ray c. Usually measured in amperes; however, in den-
tube and transformers to prevent overheating. tistry, milliampere (1⁄1000 of ampere) is used.
3. Position-indicating device (PID): d. Uses voltage as the unit of force between two
a. Attaches to the tubehead. points; measured in kilovolts (1000 volts).
b. Lead-lined, open-ended cylinder or rectangle 3. Circuit:
that directs x-ray beam to object and film. a. Route the electrical current takes.
B. Internal components: include x-ray tube, power sup- b. Filament and high-voltage are two types of cir-
ply, circuit, transformer: cuits:
1. X-ray tube: (1) Filament circuit:
a. Lead glass vacuum tube that is 1 inch by several (a) Adjusts flow through a low-voltage
inches long. ­circuit.
b. Contains two types of electrodes: (b) Controls the heating of a filament and
(1) Cathode (negative) electrode: quantity of available electrons.
(a) Has tungsten filament, which is a wire (c) Controlled by milliamperage (mA) set-
that is source of electrons when heated. ting.
(b) Has molybdenum cup, semicircular in (2) High-voltage circuit, uses 65,000 to
shape, to focus direction of electrons 100,000 volts to provide high voltage nec-
toward anode. essary to propel electrons and produce
(c) Contains milliamperage (mA) control. x-rays.
(2) Anode (positive) electrode: 4. Transformer:
(a) Has tungsten plate (or target) that elec- a. Mechanism used in electrical circuit to increase
trons hit (when they leave the cathode) or decrease the voltage.
to produce x-rays and heat. b. May be a step-up, step-down, or autotrans-
(b) Has copper stem with an anode embed- former type:
ded in it to dissipate excessive amount (1) Step-down transformer:
of heat produced. (a) Needed to produce electrons by heating
(c) Has focal spot, hit by the electrons leav- the tungsten filament in a cathode.
ing cathode. (b) Decreases incoming line voltage (110
(d) Contains kVp control to 220) to required 3 to 5 volts.
Radiology   ���
137

(c) Has MORE wires in primary coil (in- b. Characteristic radiation:


put coil) than in secondary coil (output (1) NOT generated frequently; involves elec- o1070
coil). trons dislodging electrons from the K/L
(2) Step-up transformer: shell of tungsten atom.
(a) Provides energy needed to propel elec- (2) Requires filling of a vacancy by lower o1090
trons from the negative pole to positive ­energy shell electron, resulting in an x-ray
pole to produce x-rays. that comprises the binding energy between
(b) Increases incoming line voltage (110 these two different shell electrons.
to 220) to required 65,000 to 100,000
volts. Interaction of X-Rays with Matter
(c) Has MORE wires in secondary coil An X-ray weakens whenever reacts with matter. When
than in primary coil. x-rays pass through atoms of tissue but do NOT react
c. Autotransformer: changes voltage input into with the tissue, NO interaction occurs.
primary coil of step-up transformer. A. Photoelectric effect:
1. Ionization that occurs when x-ray photon interacts
Generation of X-Rays with inner shell electron.
X-rays are produced in x-ray tube through complex pro- 2. When photons are absorbed and electrons are
cess that involves negative and positive poles and differ- ej­e­cted, effect ceases to exist (NO penetrating
ing types of x-rays produced. power).
A. X-ray process: involves use of both cathode and 3. Responsible for ~30% of all interactions in dental
­anode: x-rays.
1. Cathode: negative pole in x-ray tube: B. Compton scatter (effect):
a. Contains tungsten filament that is heated by 1. Ionization that occurs when x-ray photon interacts
filament circuit and generates electrons: with an outer shell electron.
(1) Heating temperature is controlled by mil- 2. Occurs when electron is ejected and x-ray photon
liamperage. scatters in a different direction.
(2) Available electrons are “boiled off” from 3. Responsible for ~62% of all scatter in dental
filament by process called thermionic x-rays.
­emission. C. Thompson (coherent) scatter:
b. Includes focusing cup, made of molybdenum; 1. Results without the occurrence of ionization.
cup surrounds filament and directs electrons to 2. Occurs when low-energy photon reacts with
focal spot on anode. an outer shell electron and NO loss of energy
2. Anode: positive pole in x-ray tube: occurs.
a. Has copper stem that serves as thermal conduc- 3. X-ray photon scatters in a different direction.
tor and dissipates heat away from the anode.
b. Includes target made of tungsten, embedded in Units of Radiation Measurement
the copper stem: Two units of radiation were established to measure
(1) Target’s purpose is to convert the electrons ­exposure, dose, and dose equivalent by the International
from cathode into x-rays. Commission on Radiation Units and Measurements. Two
(2) Process results in generation of x-rays (1%) systems are used for each unit: Standard and Système
and heat (99%). ­International (SI).
B. X-ray production: A. Exposure:
1. Produced in an x-ray tube when kinetic energy 1. Radiation quantity that measures ionization in air;
from electrons hits the target in the anode. described in roentgens or SI units.
2. Types include Bremsstrahlung and characteristic a. Intensity measured by the roentgen (R).
radiation: b. In other SI units, equivalent to coulombs per
a. Bremsstrahlung radiation: kilogram (C/kg).
(1) Means “braking action” in German; MA- B. Dose:
JOR source of x-rays produced in dentistry. 1. Amount of radiation energy absorbed by tissue:
(2) Results when high-energy electrons come a. Rate is amount of radiation absorbed per unit
close to the nuclei of tungsten atoms but of time.
are slowed down by the positive pull of the b. Often identified with radiation absorbed dose
­nuclei. (RAD):
(3) X-ray production results from the slowing (1) Traditional unit of dose.
down of the electrons, which releases energy. (2) The SI equivalent is the gray (Gy) or J/kg.
138   Saunders Review of Dental Hygiene

(3) Conversion: 2. Chronic exposure: small amounts of ionizing


(a) 1 Gy = 100 rad ­radiation given over longer period; results in less
(b) 1 rad = 0.01 Gy effect on tissue.
(c) 1 J/kg = 100 rad 3. Latent period: time (hours, days, or months)
C. Dose equivalent: ­between radiation exposure and observed clinical
1. Has differing biological effects when compared effect.
with different types of radiation. B. Exposure area: amount of the body exposed to radia-
2. Quality factor is a specific number, according to ex­ tion, also determines biological effect of radiation on
posure effect, that is given to each type of radiation. the tissues.
3. Roentgen-equivalent-man (REM): 1. Whole-body exposure: exposure of entire body,
a. Traditional unit of absorbed dose. results in MORE severe biological effects; can be
b. The SI equivalent is the sievert (Sv). acute or chronic.
c. Conversion: a. Acute: occurs in nuclear warfare and with occu-
(1) 1 Sv = 100 rem pational hazards; can result in such symptoms
(2) 1 rem = 0.01 Sv as nausea, vomiting, bleeding, diarrhea.
b. Chronic: exposure to natural background radia-
Mechanisms of Radiation Injury tion; largest single cause is cosmic rays.
Two theories attempt to explain biological effects of 2. Limited-area exposure: irradiation to specific body
­radiation that result in injury to living tissues, and dose- area; can be acute or chronic.
response curve is a graphic means of showing those a. Acute: to higher doses directed at specific area,
­effects. such as occurs during radiation therapy (see
A. Direct theory: ­effects on cells and organs listed below).
1. Radiation damage to tissues is caused by a direct b. Chronic: to lower doses, such as occurs during
hit on the DNA molecule of a cell, which causes dental radiography.
cell death. C. Cell types: effects of radiation are determined by cell
2. Seldom causes radiation injury. maturity and cell type:
B. Indirect theory: 1. Law of Bergonié and Tribondeau states that the
1. Radiation damage to tissues involves the forma- greatest radiation effects occur in cells that are
tion of free radicals. immature, are NOT highly specialized, and fre-
2. Applies when x-ray photon is absorbed in a cell, quently divide.
which creates toxins (free radicals) and damages a. Cell maturity:
the cell. (1) Immature cells are nonspecialized; experi-
3. Indirect injuries are common because of the reac- ence rapid cell division.
tion with water (70% to 80%) in cells (i.e., forma- (2) Mature cells are specialized in function;
tion of hydrogen peroxide). ­divide at slower rate or do NOT divide at all.
C. Dose-response curve: b. Cell types:
1. Graphic means of plotting biological response (1) Somatic tissue cells are NOT inherited and
(damage) to radiation exposure (dose) to deter- NOT involved in reproduction (e.g., skin,
mine acceptable levels of radiation exposure. kidney cells).
2. For low doses of radiation, very little information (2) Genetic tissue cells are reproductive tissue
is available about acceptable levels. cells (e.g., ovaries in woman and testes in
3. Graphically, linear nonthreshold curve occurs, man).
which indicates that response is seen at any dose. D. Radiosensitivity: human tissue cells have different
4. Results indicate that there is NO true safe level of sensitivities to radiation:
­radiation exposure. 1. Radiosensitive tissues and cells are highly sensi-
tive to radiation (e.g., bone marrow, reproductive,
Factors That Determine Radiation Injury small lymphocyte).
Several factors determine amount of injury that occurs 2. Radioresistant tissues and cells are MORE resis-
from radiation: total dose, amount of area exposed, types tant to radiation (e.g., salivary glands, lungs, kid-
of cells exposed, cell sensitivity. neys, muscle).
A. Total dose: amount of radiation absorbed in the tissue 3. In order of MOST sensitive to least sensitive:
that determines effect. blood-forming and reproductive cells, gastrointes-
1. Acute exposure: ionizing radiation given in shorter tinal tract, immature bone, epithelial (glandular)
amount of time; results in MORE dramatic biolog- and muscle tissues, adult bone and nerve tissue;
ical effects. blood type does NOT affect sensitivity.
Radiology   ���
139

E. Critical organs and radiation: A. Patient: patient exposure to radiation MUST BE reduced
1. When sensitive organs and tissues are exposed by using filtration, collimation, positioning, shielding:
to radiation during dental irradiation, significant 1. Filtration involves filtering-out of nonproductive
­decrease in quality of person’s life may occur. x-rays; inherent filtration plus added filtration
2. Include skin, eye lens, thyroid, bone marrow equals total filtration for the machine:
­(hematopoietic): a. Inherent filtration is inside tubehead (i.e., oil,
a. Skin irradiation may lead to erythema: FIRST glass window, tubehead seal).
clinical sign of overexposure (250 rad in 14-day b. Added filtration includes aluminum disc (disk)
period). placed around seal of machine:
b. Eye irradiation can lead to cataract formation if (1) Used to filter out longer, nonproductive
exposure to eyes is 200,000 mrad. wavelengths, resulting in primary beam
c. Thyroid irradiation can result in increased can- with MORE energy.
cer risk, particularly when radiation exposure (2) Federal and state laws dictate thickness of
of 6000 mrad or more occurs. total filtration according to kilovolt peak
d. Bone marrow irradiation poses increased can- (kVp) setting of the machine (≤70 kVp =
cer risk (particularly leukemia) at exposures of 1.5-mm thickness of aluminum; ≥70 kVp =
5000 mrad or more of radiation. 2.5-mm thickness).
e. Organs MOST affected by continued low doses 2. Collimation restricts size and shape of x-ray beam
of radiation in order from MOST sensitive to (reducing film fog) and therefore restricts patient
least sensitive are thyroid, skin, brain. exposure (scatter exposure):
a. Collimator: lead diaphragm placed over open-
Radiation Exposure and Risks ing of tubehead.
Exposure and risks associated with radiation include b. Exposure reduction is possible by limiting
those from environment (background) and those associ- size of x-ray beam at end of PID to no more
ated with direct irradiation of organs. (See the WebLink than 2.75 inches in diameter, using rectangular
on the CD-ROM for the ADA and FDA Guide to Patient PID, and placing tubehead 1½ to 2 inches from
Selection for Dental Radiographs.) ­patient.
A. Background radiation: GREATEST contribution to 3. The PID (beam-indicating device [BID]): aiming
radiation is from naturally occurring sources in envi- device that is used to direct the x-ray beam:
ronment: a. Cone-shaped PID: closed, pointed, plastic de-
1. External radiation: from cosmic and terrestrial vice that increases scatter radiation and should
sources; comprises ~15% of the radiation exposure NOT be used.
to the population. b. Round PID: open ended, circular, and lead
2. Internal radiation: from exposure to radionuclides lined; standard lengths are 8 inches and 16
through inhalation and ingestion; comprises ~67% inches; 16 inches is MAINLY used because
of radiation exposure; radon is the LARGEST sin- provides MORE parallel rays.
gle contributor to natural radiation. c. Rectangular lead-lined PID: highly recom-
B. Artificial radiation: includes such sources as medical mended because offers MOST reduction in ra-
and dental radiation: diation exposure to the patient; standard lengths
1. Comprises ~17% of radiation exposure. are 8 inches and 16 inches.
2. Exposure is equivalent to few days’ worth of 4. Lead aprons or lead equivalent (contains 0.25 to
background radiation or environmental radiation 0.3 mm of lead thickness) shields; aprons worn by
exposure, or similar to dose received during cross- patients to protect tissues from scatter radiation;
country airplane flight. mandated by law in many states; should be used in
conjunction with thyroid collar for intraoral films
Radiation Protection (questionable use now).
p0090 Proper use of equipment, patient and clinician position- 5. Fast film: one of the MOST effective means of re-
ing, and technique can provide acceptable limits to inci- ducing patient exposure; F-speed film is fastest and
dental radiation exposure. Advances in dental radiograph is MORE than twice as fast as D-speed film; F-speed
technology have reduced scatter radiation, the reason for film needs 60% LESS exposure time and E-speed
protective boxes; lead-lined radiograph storage boxes are film needs 50% LESS exposure time than D-speed
thus unnecessary and should be discarded, since they film; thus E- and F-speed films recommended.
contaminated the film. There is controversy over need 6. Film-holding devices: method necessary to stabi-
for lead (or lead equivalent) aprons for patient protection. lize film and ultimately reduce additional exposure
Note that term “clinician” is used for operator. to patient.
140   Saunders Review of Dental Hygiene

B. Clinician: measures MUST be taken to protect clini- (6) Maximum accumulated dose = 5 rem ×
cian from unnecessary radiation; include proper posi- N −18 (N = age).
tioning, use of monitors, attention to proper methods c. The ALARA concept: As Low As Reasonably o2290
of taking radiographs: Achievable.
1. Positioning: (1) Philosophy of radiation protection that is o2310
a. Involves awareness of distance; remaining at currently practiced by all radiation workers.
least 6 feet from the source of radiation and in (2) Implies that every effort will be made to o2320
a safe quadrant (at right angles to the primary keep radiation exposure, whether occupa-
beam) is one of the MOST effective ways to tional or nonoccupational, as LOW as pos-
eliminate unnecessary clinician exposure. sible (see later discussion of guidelines for
b. Clinician SHOULD never hold the tubehead or patient radiographic examination).
film in patient’s mouth during exposure.
c. Clinician should stand behind lead shield or bar- Digital Imaging
rier whenever possible (cinderblock or 2½ inches With digital imaging, nonfilm sensor is placed inside the
of drywall provides adequate protection). mouth; electronically attached to computer, immediately
2. Monitoring: producing image on monitor.
o2160 a. Involves using personnel monitoring devices A. Principles:
and following national guidelines when mea- 1. Requires conventional equipment to generate
suring occupational radiation exposure; film x-rays; see earlier discussion.
badges that measure exposure to low doses of 2. Image captured is SIMILAR to conventional film.
radiation should be worn at waist level; two 3. When sensor is exposed to radiation, electronic
types of monitoring available. chart is produced on its surface.
(1) Metal filters: located inside badge, measure 4. Direct or indirect (scanned):
amount of radiation but can be worn for a. Direct: sensor in rigid case, either charge-
ONLY 1 month; hard radiation shows faint ­coupled device (CCD) or complementary
shadow; soft radiation casts pronounced metal-oxide semiconductor (CMOS), is used;
shadow. immediate image and immediate reuse.
(2) Thermoluminescent dosimeter (TLD): de- b. Indirect: photostimulable phosphor (PSP) plate
vice that contains lithium fluoride crystals is used to store image, which is then scanned,
that absorb x-ray energy; MORE sensitive read, and erased; conventional films can also be
and can be worn for 3 months but is MORE scanned and converted to digital.
expensive. B. Advantages:
b. Maximum permissible dose (MPD): devel- 1. Images can be stored, retained as a hard copy, or
oped by National Council on Radiation Protec- sent to a different site.
tion (NCRP): 2. Images can be displayed immediately, which is use- o2420
(1) Relates to amount of radiation exposure ful for intraoperative procedures such as endodon-
that is permissible through occupational tic treatment; eliminates darkroom procedures.
exposure. 3. Provides less radiation exposure to the patient but
(2) Considered to be maximum dose that does poorer resolution, costs MORE, large sensors,
NOT produce significant injury in a lifetime. manipulated films, MORE exposures because of
(3) Formula represents whole-body dose small sensor size.
equivalent of ionizing and electromagnetic 4. Associated software using a database can analyze
radiations and is expressed in sieverts. changes in radiographic density to identify demin-
(4) The MPD for occupationally exposed per- eralized areas and determine probability that caries
sons: is present.
(a) Per year = 50 mSv (5 rem, 5000 mrem). C. Disadvantages: initial cost, uncomfortable sensors,
(b) Per month = 4 mSv (400 mrem). infection control of sensors by plastic sleeves and
(c) Per week = 1 mSv (100 mrem). NOT heat sterilization.
(5) The MPD for nonoccupational (whole-
body) exposure: 10% of that for worker, or
5 mSv (500 mrem) per year.
Radiology   ���
141

clinical STudy  

Age 42 YRS SCENARIO

Sex ☐  Male   ☒  Female The new patient’s intraoral examination reveals


several suspect areas and fractured restorations on
Height 5’4”� the maxillary arch. It has been at least 6 or 7 years
since patient has had radiographs taken. The dental
Weight 170 LBS
hygienist recommends a full-mouth series (FMX).
BP 118/76 However, the patient is reluctant because of what
she has heard about the dangers and misuse of
Chief Complaint “Boy, the fillings in my upper back radiation.
teeth are so sensitive! They feel like
they are moving when I bite down.”

Medical History Has had a bad cold for 2 weeks with


postnasal drip
Mother died of breast cancer
Has mammograms every 6 months
because of high risk

Current Medications None

Social History Bus driver


Single mom with two children
Likes bowling

1. What is the most probable diagnosis of the patient’s 3. Devices on x-ray machine such as filter, collimator,
condition? Identify all possible causes for her symp- lead-lined PID reduce radiation exposure to patient.
toms. What other information should be gathered to Using lead apron or lead equivalent with thyroid collar
determine the cause of the ­sensitivity? and film-holding devices with fast-speed film (F-speed
2. What issue must be addressed to provide this patient film is fastest) keeps patient exposure to minimum
with the best treatment? while providing excellent-quality diagnostic radio-
3. What procedures are used to protect the patient from graphs.
unnecessary radiation?
CHARACTERISTICS OF RADIATION IMAGES  
1. Sinus infection and carious lesions are present. Radiation images are characterized by their quantity,
Sensitivity of maxillary teeth could be related to the quality, intensity, clarity.
closeness of the roots of maxillary teeth to enlarged, A. Quantity: number of x-rays that are produced; mea-
inflamed sinuses; such sensitivity should dissipate sured in amperes, milliampere-seconds, density of the
as sinusitis clears up. Movement of restorations radiographs.
is more serious in nature and is indicative of frac- 1. Ampere: number of electrons that flow through a
tured restorations. Ill-fitting restorations often have filament:
marginal leakage and/or recurrent caries. Other a. Milliampere (mA) is equal to 1⁄1000 of an ampere.
information can be gained by asking: “What are b. Milliamperage controls temperature of the cath-
the teeth sensitive to (e.g., temperature, pressure, ode filament.
sweets)?”, “How long have they been sensitive?”, 2. Milliampere-seconds (mA-s): combination of mil-
and “Does anything seem to exacerbate or alleviate liamperes and seconds:
the ­discomfort?” a. Milliamperes have a direct effect on the number
2. Benefit versus risk of having radiographs taken of electrons produced.
should be discussed with patient. Clinical examina- b. Seconds have the same effect on the quantity of
tions warrant FMX radiographs to confirm exact electrons produced.
diagnosis and direction of proper treatment. Although c. To maintain similar density:
minimal amount of radiation exposure occurs, ben- (1) Increased seconds, milliamperage must
efit of diagnosis far outweighs risk of allowing cari- be decreased.
ous lesions to continue without diagnosis and then (2) Decreased seconds, milliamperage must
­treatment. be increased.
142   Saunders Review of Dental Hygiene

3. Density: overall darkness of radiograph: c. MAINLY influenced by patient and tubehead


a. Affected by milliamperage and by quantity of movement; either will reduce image sharpness.
electrons produced. 2. Magnification: enlargement of actual image:
(1) Increased mA, film will be darker. a. Influenced by target-film distance; increased
(2) Decreased mA, film will be lighter. distance from target in the x-ray tube to film
B. Quality: energy and penetrating power of the beam; allows MORE parallel rays to hit film, LESS
measured in kilovolts; affects image contrasts: magnification.
1. Kilovolt (kV): equal to 1000 volts and is measure- b. Affected by object-film distance; closer the
ment used to determine energy in a tubehead: tooth to film, LESS the magnification.
a. Kilovoltage peak (kVp) represents peak or 3. Distortion: disfigurement of the shape and size of
maximum voltage; determines speed and ulti- an image:
mately energy of electrons. a. Affected by the object-film alignment; film and
b. With increased kVp, primary beam will have long axis of the tooth MUST be parallel to each
MORE energy and density will increase. other (so that rays will hit at a right angle) to
2. Contrast: difference between lighter and darker decrease distortion.
shades (grays) on radiograph: b. Influenced by beam direction; beam MUST be
a. Increased kVp (≥90) results in many shades of directed perpendicular to both the film and the
gray on the film (long-scale contrast); BEST for tooth to record accurate, distortion-free image.
detecting bone abnormalities. 4. Pneumatization: with free airspace in a structure
b. Decreased kVp (65 to 70) results in MORE or tissue, one area is MORE radiolucent than an-
black and white areas on a radiograph (short- other area (frequently seen when maxillary sinus
scale contrast); BEST for detecting caries. has dropped).
C. Beam intensity: affected by mA, kVp, exposure time,
distance: RADIOGRAPHIC EXAMINATION  
1. The mA affects intensity by controlling the num- Radiographic examination involves intraoral, extraoral,
ber of electrons produced; higher the mA, MORE and special imaging techniques to produce high-quality
intense the beam. radiographs for use in examination, interpretation, diag-
2. The kVp controls energy of electrons traveling nosis of dental conditions and diseases. (See the CD-ROM
from the cathode to the anode; higher the kVp, for WebLink to the ADA/FDA Guide to Patient Selection
MORE intense the beam. for Dental Radiographs, which includes the chart Guide-
3. Exposure time affects the intensity in the same man- lines for Prescribing Dental Radiographs.) These guide-
ner as mA; increase in time = MORE intense beam. lines will allow for ALARA concept, although each case
4. Distance also affects beam intensity; as beam trav- is subject to clinical judgment.
els longer distance to film or tooth, becomes LESS
intense: Intraoral Radiographic Views
a. Inverse square law states that “intensity of ra- Views include periapical (PA), bitewing (BW), occlusal,
diation is inversely proportional to square of the full-mouth series (FMX).
distance from the source of radiation”; explains A. The PA view: captures from cementoenamel junction
how the beam loses its intensity as it travels far- (CEJ) to root on the film; included in FMX; uses size
ther from the source. 1 to 2 films:
b. Half-value layer describes the reduction in 1. Used for diagnosis of a pathological changes in a
beam intensity by one half with the use of alu- specific tooth by its root and surrounding bone.
minum filters; filters remove LESS penetrating, 2. Especially if there is a need for endodontic therapy
longer wavelengths in the beam. (late finding).
D. Accurate image (geometric) formation: BEST pro- B. The BW view: visualizes crowns, contacts, height of
duced by controlling image sharpness, magnification, alveolar bone in relation to CEJ; included in FMX;
distortion: uses size 1 to 2 films:
1. Sharpness (detail, definition): umbra is increased 1. Used for diagnosis of a series of teeth as they con-
clarity or distinctness of outlines of object; penum- tact on the occlusal.
bra is fuzziness or LACK of sharpness in image. 2. Used for diagnosis of dental caries (ONLY inter-
a. Influenced by focal spot size; tungsten target of proximal early, with occlusal and root late) and peri-
anode should be small to increase sharpness. odontal disease (ONLY bone loss on interproximal
b. Associated with film composition; size of crys- alveolar crest is shown, NOT facial or lingual plates).
tals on film determines sharpness; larger the 3. Can be either horizontal or vertical in placement;
crystals, LESS sharp the image. vertical used to evaluate alveolar bone loss.
Radiology   ���
143

C. Occlusal view: reveals bone surrounding tooth, floor b. Positive vertical angle of 65°; central ray is
of the mouth, or presence of sialolith (stone) in pa- ­directed just superior to tip of nose.
rotid (Stensen’s) duct; NOT included in FMX; uses 3. For cross-sectional maxillary occlusal:
size 4 films. a. Sagittal plane perpendicular to floor; occlusal
D. FMX: complete set of intraoral radiographs: number plane parallel to floor.
depends on size of films: b. Central ray directed to film at 90°, with PID
1. Use size 1 for anteriors and size 2 for others: centered between the eyebrows.
20 films, 4 BWs and 16 PAs. 4. For cross-sectional mandibular occlusal:
2. Use only size 2: 20 films, 4 BWs and 14 PAs. a. Head tilted backward until ala-tragus line is al-
most perpendicular to floor.
Intraoral Techniques b. A 90° vertical angle, central ray directed 3 cm
Intraoral radiography includes bisecting, paralleling, and inferior to the chin.
occlusal techniques. D. Buccal object (SLOB: Same Lingual, Opposite ­Buccal) o3260
A. Bisecting (angle) technique (Figure 5-3): rule: shows whether artifact or object is lingual or
1. Based on rule of isometry: two triangles are equal buccal; in second radiograph, if central ray is moved
if they have equal angles and share common side. and object moves in same direction, then lingual, if it
2. Requires bisection of the angle, forms two equal moves in opposite direction, buccal; used mainly in
triangles; angle formed by plane of film and plane endodontics, since on PAs, roots are often superim-
of long axis of tooth. posed on one another and require separation for proper
3. If technique is strictly followed, directs primary beam identification so as to visualize lengths and anatomy.
90° to bisected line, resulting in accurate image.
4. MORE distortion than paralleling technique Intraoral Technique Errors
­because of position of the palate. Proper recognition of errors can help clinician correct
B. Paralleling technique: a technique problem before it is repeated, possibly sub-
1. Based on geometric figures of parallelism. jecting patient to unnecessary radiation and producing
2. Requires that film be placed parallel to long axis of nondia­gnostic radiographs.
the tooth. A. Overlap (Figure 5-4):
3. Requires film holder for proper film placement. 1. Adjacent tooth structures superimposed on each other.
4. Directs primary beam perpendicular to film and 2. Corrected by redirecting primary beam through
long axis of the tooth. contacts (change in horizontal angulation).
C. Occlusal technique: B. Foreshortening (Figure 5-5):
1. For anterior topographical mandibular occlusal: 1. Teeth shorter than actual size because of excessive
a. Head tilted backward, using negative 55° vertical angulation.
­vertical angle. 2. Corrected by decreasing the vertical angulation.
b. Central ray directed through point of the chin.
2. For anterior topographical maxillary occlusal:
a. Sagittal plane perpendicular to floor; occlusal
plane parallel to floor.
Long axis
of tooth

Bisector

Central ray

Film

Figure 5-4  Overlapping of tooth structures caused by


Figure 5-3  Bisecting (angle) technique. faulty horizontal angulation.
144   Saunders Review of Dental Hygiene

Figure 5-5  Foreshortening from excessive vertical angu­


lation.

Figure 5-7  Radiolucent mark associated with creased or


bent film.

Figure 5-6  Elongation from inadequate vertical angu­


lation.

C. Elongation (Figure 5-6):


1. Teeth distorted and larger than actual size because Figure 5-8  Cone cut caused by inadequate position-
of insufficient vertical angulation, and apices of ­indicating device (PID) coverage of the film.
the teeth are NOT included.
2. Corrected by increasing the vertical angulation. F. Backward placement (Figure 5-9):
D. Bent film (Figure 5-7): 1. Film appears much lighter, with a herringbone or waf-
1. Radiolucent mark across film; caused by creasing fle patterned effect on both sides; caused by backward
or improperly placing film. placement of the film in the mouth, with lead foil in
2. Corrected by NOT creasing film corners and by the packet closest to (facing) tooth and x-ray source.
positioning film to keep the surface of film in the 2. Corrected by placing the film with the smooth side
same plane. next to the tooth (pebble side away).
E. Cone cut (Figure 5-8): G. Movement (Figure 5-10):
1. Radiopaque area with circular or rectangular 1. Blurred or unclear image; caused by movement of
­border; caused because PID did NOT cover entire the patient and/or tubehead during exposure of film.
surface of film. 2. Corrected by giving the patient MORE specific direc-
2. Corrected by redirecting the PID to cover the tions about NOT moving and by checking to be sure
­surface of the film. the tubehead is stabilized before exposing the film.
Radiology   ���
145

Figure 5-11  Double exposure occurs when a single film is


exposed to radiation twice.

Tubehead movement

Figure 5-9  Film placed backward in the mouth has a waf-


fle or herringbone appearance.

Film movement
Figure 5-12  Focal trough used in panoramic imaging.

­cephalometry, Waters projection techniques. (See the


Figure 5-10  Blurred image is caused by movement of Chapter 5 Extras on the CD-ROM for discussion of pan-
­either patient or tubehead during film exposure. oramic imaging.)
A. Panoramic imaging: movement of film cassette and
H. Double exposure (Figure 5-11): radiation source around the head (Figure 5-12).
1. Darker film with distinct outlines of many teeth; 1. Uses focal trough or plane of acceptable focus in
caused by NOT organizing film order during expo- shape of the dental arches.
sure or by exposing a single film twice. 2. Any images inside this trough are clearer; images
2. Corrected by keeping film in order of exposure and outside the trough are blurred; thus CORRECT
by keeping exposed film in a different place. ­patient positioning is crucial.
3. Evaluates impacted teeth, eruption patterns,
Extraoral Techniques growth; used to detect diseases, conditions of the
During extraoral techniques, film is placed outside of jaw, trauma.
the mouth during exposure; MOST of these require 4. Errors in panoramic technique are common:
use of screen film (if NOT using digital method). a. Lip and tongue placement: IMPORTANT in
­Includes panoramic imaging, lateral jaw exposures, quality of film.
146   Saunders Review of Dental Hygiene

Palate

Patient’s
tongue not
on roof of
mouth

Spine

Figure 5-13  Panoramic film demonstrating improper tongue placement.

(1) NOT closing lips; lips must be closed to


prevent appearance of dark shadow on an-
teriors.
(2) Tongue NOT on palate; tongue must be
placed on the palate to prevent appearance
of radiolucent areas over apices of maxil-
lary teeth (Figure 5-13).
b. Chin placement: MUST be parallel to floor.
(1) If chin is tipped up, “frown” results, with
blurring and magnification of maxillary
teeth.
(2) If chin is tipped down, “exaggerated smile”
results, with blurring and magnification of
mandibular incisors.
(3) Shadow of chin rest. Figure 5-14  Lateral jaw exposure is useful for examining
c. Placement of anterior teeth: MUST be placed traumatic injury to the posterior jaw.
correctly in focal trough.
(1) If anterior teeth are placed forward to the
focal trough (notch in the bite-block), will
appear thinner.
(2) If anterior teeth are placed behind the focal
trough (notch in the bite-block), will appear
thicker and wider.
B. Lateral jaw exposure: film cassette placed laterally
to jaw during exposure (Figure 5-14):
1. Used to examine posterior portion of mandible or
for those who CANNOT open the mouth because
of swelling or possible fractures.
2. Also, lateral areas of the mandible are too large for
PA film.
C. Lateral cephalometric projection: film cassette
placed on the side of head to image outline of face;
evaluates trauma, facial growth and development, de- Figure 5-15  Lateral cephalometric projection is often used
velopmental abnormalities (Figure 5-15). to detect developmental or traumatic disturbances.
Radiology   ���
147

Figure 5-16  Posteroanterior projection is used to evaluate


the frontal and ethmoidal sinuses, the eye orbits, and the
nasal cavity. Figure 5-17  Waters projection is useful for examining
maxillary sinuses.

D. Posteroanterior projection: film cassette placed E. Waters projection: film cassette placed on face, with
on face, with forehead and nose touching it; shows chin touching it, with head tipped back so that tip of
frontal and ethmoidal sinuses, orbits, nasal cavity the nose is 1 inch from it; specifically evaluates max-
(Figure 5-16). illary sinus area (Figure 5-17).
clinical study  

Age 8 YRS Scenario

Sex ☒  Male   ☐  Female At his first dental appointment, the patient demon-
strates nervousness and a short attention span.
Chief ­Complaint His guardian asks, “My foster
child often has bad breath. Is it
due to cavities?”

Medical ­History Fetal alcohol syndrome (FAS)


Moderate cognitive disability

Current Medications None because of cost factors at


this time

Social History Does not attend school at


the present time because of
mother’s recent incarceration

1. What is FAS and what are the oral manifestations of nose or extremely wide bridge, cleft lip with or without
this disorder? cleft palate, underdeveloped maxilla, micrognathia.
2. What radiographs are recommended and why? 2. Panoramic radiograph with accompanying BWs is
3. A double image appeared on one of the two films. recommended because of the FAS and lack of drugs
What caused this? for ADD or hyperactivity, since he will have trouble
sitting still for a long period needed for any other ra-
1. FAS is caused by maternal ingestion of alcohol during diographs. In addition, his small jaw size would make
pregnancy. Alcohol ingestion during the early stages of numerous intraoral films difficult to obtain.
pregnancy often leads to intellectual disability (mental 3. Blurred image on a film is caused by movement of
retardation) and other developmental disabilities in either tubehead or patient during exposure. In this
the child. Hyperactivity and attention deficit disorder situation, most likely it is caused by patient movement
(ADD) often accompany the syndrome, as they do in because this patient is not taking any drugs for hyper-
this case. Oral manifestations may include flattened activity that accompanies FAS.
148   Saunders Review of Dental Hygiene

(4) Potassium bromide: restrainer that inhibits


Special Imaging Techniques development of unexposed silver halide
Magnetic resonance imaging, computed tomography, crystals.
­sialography are specialized imaging techniques. b. Fixing takes ~10 minutes, or TWICE devel-
A. Magnetic resonance imaging (MRI): derives energy oping time; clears all unexposed silver halide
from powerful magnetic field: crystals from film emulsion by means of four
1. H ydrogen nuclei in the body are realigned by ingredients in solution:
radiofrequency pulses, received by the sensor (1) Sodium thiosulfate: clears unexposed silver
and transmitted to computer to generate mage. halide crystals from emulsion.
2. BEST for imaging soft tissue, especially within (2) Sodium sulfite: prevents the breakdown of
temporomandibular joint. sodium thiosulfate.
B. Computed tomography (CT scanning, CAT scan): (3) Potassium alum: shrinks and hardens emul­
computer generated; uses ionizing radiation as energy sion.
source: (4) Acetic acid: acidifier that keeps medium
1. MUST be performed in special CT unit in the hos- acidic and stops additional development.
pital; special software is required. 2. Automatic processing: automatically processes
2. Allows multiple images to be fed into a computer, film; roller transport system moves film through
resulting in image on monitor or film. solutions in order: developer, fixer, water, drying
3. Useful in dentistry for implant planning and soft chamber.
tissue lesions, especially in salivary glands. a. Developing and fixing solutions are different
o9000 C. Cone beam computed tomography (CBCT): from those used in manual processing.
computer-generated imaging of hard tissues of (1) Developing solution works at higher tem-
maxillofacial region, providing three-dimensional perature and MORE rapidly.
representation of skeleton with minimal distortion; (2) Fixing solution has a hardening agent to
high diagnostic quality of resolution, with short scan- keep the film sturdy as it continues through
ning time and radiation dosages lower than CT scans; the rollers.
highly recommended for implant placement, preop- b. Rollers MUST be clean, or radiolucent bands
erative and postoperative endodontic procedures with will appear on films.
cracked teeth and missing canals, and apical surgery. 3. Rapid (hot) processing method: concentrated or
o3850 D. Sialography: used ONLY for radiographic examination higher temperature solutions process in 1 minute
of ductal system of salivary glands by use of contrast or less; BEST for endodontic therapy or emer-
agent to check for salivary stones and other blockage. gency procedures; quality of film is less than with
standard methods.
Image Production B. Darkroom lighting: important in film processing:
Producing a good image requires knowledge of proper 1. Without any light leaks to prevent fogging of
film processing and darkroom techniques, processing the film.
problems and solutions, film reproduction methods, in- 2. Filter that removes blue-green wavelengths; red both
fection control practices. intraoral and extraoral, orange intraoral ONLY.
A. Film processing: transformation of latent image into 3. Safelights (direct illumination) must be spaced at
visible image by means of chemical processing; can least 4 feet from working surface with low-wattage
be manual or automatic: bulb (7½/15 watts).
1. Manual processing (time-temperature method) in- 4. LED safelights: replace conventional filtered
volves, in order: manual developing, washing, fix- lights; offer MORE efficient light.
ing, washing, drying. C. Film duplication: copying or reproducing a second
a. BEST developing takes ~5 minutes at 68° F; film or set of films without reexposing the patient to
reduces exposed silver halide crystals into radiation:
black metallic silver through four ingredients in 1. Requires special duplicating film that must be used
­solution: under safelight conditions.
(1) Hydroquinone (gives contrast) or elon a. Duplicating film is coated on ONLY one side.
(gives detail): developing agent that con- b. Duplicating film is direct positive film:
verts energized crystals into metallic silver. (1) Darker film requires LESS exposure time.
(2) Sodium sulfite: preservative that prevents (2) Conversely, lighter film requires MORE
rapid oxidation of developer. exposure time.
(3) Sodium carbonate: softens film emulsion 2. Used for third-party payment, patient transfer, re- o4150
and speeds up action of developing agents. ferral, litigation.
Radiology   ���
149

o4160 3. Procedure requires placing original film next to the 2. Radiographer expertise: ONLY individuals skilled
light source, placing duplicate film on top of it, and in radiographic procedures and techniques should
exposing the films to the light source for specified expose patients to x-rays.
amount of time. 3. Processing solutions (manual and automatic):
D. Film mounting: processed radiographs are organized a. Mixed and replenished according to the manu-
and placed in a frame for ease of reading, to protect facturer’s recommendations and dated.
films from wear, to reduce interpretation errors: b. Changed as needed according to degree of use
1. Depending on the teaching method, radiographs and dated.
are mounted either “dots-in” or “dots-out.” c. Can be evaluated by:
a. Dots-in or lingual method (as if inside the (1) Stepwedge (constructed of aluminum lay-
mouth looking out): films are mounted with the ers) to provide standard radiograph for
patient’s left side films on left side of the mount evaluating density on daily basis.
and right side films on right side of the mount; (2) Dental radiographic normalizing device
less common method. (DRND) to compare density of film and
b. Dots-out or labial method (as if outside the monitor strength of the solution.
mouth looking in): films are mounted with 4. Darkroom:
the patient’s right side films on the left side of a. Safelight is evaluated every 6 months (see ear-
the mount and the left side films on the right lier discussion).
side of the mount; MORE common method. b. Evaluated for light leaks around doors and vents
2. Mounting procedure: once films are dry, placed on by means of coin test.
clean viewbox surface with dots up; use of magni- 5. Cassette intensifying screens: inspected and cleaned
fying viewer is helpful. monthly.
a. Films are initially separated into maxillary, F. Administration:
mandibular, BW sections: 1. Assign duties to appropriate personnel; write de-
(1) BWs are mounted FIRST, using curve of scriptions of the plan and its expectations.
Spee so that the “smile” is directed up and 2. Maintain records for equipment; create forms to
distally, to guide placement of posterior PA keep records of monitoring procedures.
films; maxillary molars have three roots 3. Document all monitoring procedures.
and mandibular molars have two roots.
(2) Posterior films with most posterior struc- Processing Problems and Solutions
tures are placed in molar areas; subse- Many problems occur when processing technique and pro-
quently, posterior films with least posterior cedure errors are made even with automatic processing.
structures are placed in premolar areas. A. Overdeveloped film too dark (Figure 5-18):
(3) Anterior films with both central incisors are 1. Overdevelopment (excessive development time).
placed in appropriate maxillary and man- 2. Developing solution at a temperature that is
dibular central anterior sections. too high.
(4) Remaining right- and left-of-central ante- 3. Developing solution that was incorrectly mixed or
rior films are placed in appropriate sections too concentrated.
to either side of the central films. B. Underdeveloped film too light (Figure 5-19):
b. Films are then arranged by the anatomical 1. Underdevelopment (inadequate development time).
landmarks as if facing the dental arches: max- 2. Developing solution at a temperature that is
illary posterior right, posterior left, and ante- too low.
rior; mandibular posterior right, posterior left, 3. Exhausted (overused) developing solution.
anterior.
E. Quality assurance program: systematic procedure
used to guarantee that high-quality radiographs are
produced with minimal exposure to the patient:
1. X-ray equipment:
a. Evaluated annually as recommended by Ameri-
can Academy of Oral and Maxillofacial Radiol-
ogy (AAOMR).
b. Inspected by specifically trained inspectors
who check kVp, mA output, half-value layer,
timer, collimation, beam alignment, tubehead
­stability. Figure 5-18  Overdeveloped film appears too dark.
150   Saunders Review of Dental Hygiene

Figure 5-19  Underdeveloped film appears too light.

Figure 5-21  Developer cutoff has straight white border.

Figure 5-20  Developer contamination.

C. Reticulation: cracked emulsion; caused by sudden


temperature change between developer and fixer
solution.
D. Contamination from chemicals:
1. Developer contamination: darker areas; caused
when developing solution comes into contact with
film before processing procedure (Figure 5-20).
2. Fixer contamination: lighter areas; caused when
fixer solution comes into contact with film before
processing procedure.
E. Other errors:
1. Exhausted solutions or insufficient time in solu-
tions: yellow-brown stains.
2. Developer cutoff: straight white border; caused by
a low level of solution (Figure 5-21).
3. Fixer cutoff: straight black border; caused by in-
complete immersion of film into fixer solution
(Figure 5-22).
4. Overlapped films: white and dark areas in the
shape of outline border of another film; caused
when films contact each other in solutions
(Figure 5-23). Figure 5-22  Fixer cutoff has straight black border.
Radiology   ���
151

Figure 5-23  Overlapped films have an outline shape of a


second film.

Figure 5-24  Static electricity on this half of a panoramic


film caused a black, branchlike image.
5. Air bubbles: white spots; caused by air that is
trapped on surface of film during processing. 7. Scratched films: white lines; caused by removal of
6. Static electricity: thin, black, branchlike lines; emulsion during processing process.
caused by low humidity and opening film packet 8. Fogged films: gray hue; caused by improper safelight-
too quickly (Figure 5-24). ing, outdated film, or light leaking into darkroom.

clinical study  

Age 85 YRS SCENARIO

Sex ☒  Male   ☐  Female The patient fell while getting dressed yesterday and
hit his head on the headboard of his bed. Visual
Height 5����
’���
10�
” examination at his winter dental office during the
emergency appointment reveals a large bruise on
Weight 178 LBS
the right side of his face along the lower jaw line.
BP 110/69 Teeth #30 and #31 are missing their anatomical
crowns.
Chief Complaint “My jaw is sore and I can’t open my
mouth!”

Medical History Shielded pacemaker placed 10 years


ago

Current Medications warfarin (Coumadin) 10 mg qd to


prevent blood clots from forming

Social History Retired fireman


Travels to Arizona during the winter
months

1. Which type of radiograph would provide the best 3. During examination of the processed film, a pyramid-
assessment of this patient’s jaw and teeth? What in- shaped radiopacity is observed in the center of
formation should be obtained from the radiograph? the panoramic image. What is it, and how was it
Describe the appearance of such information. caused?
2. After the film is processed, a 9-mm semicircular radi- 4. “Branch-out” radiolucent marks appear on several
opaque mark is found on the lower right corner of the areas of the film. What are they, and what caused
radiographs. What is the mark, and what caused it? them?
152   Saunders Review of Dental Hygiene

1. Panoramic radiograph should be taken to allow as- 2. All uncovered surfaces (e.g., armrests, chair, coun-
sessment of a larger area of the jaws and teeth; also tertops) that might be contaminated SHOULD be
is easier for patient, given that he has difficulty disinfected with EPA-registered disinfectant; clini-
opening his mouth. Indications of jaw fracture (ver- cian SHOULD wear utility gloves.
tical radiolucent line) should be looked for on right
side; possible tooth discoloration, noncontinuous Normal Anatomical Landmarks
lamina dura, and infected retained root tips on right Familiarity with normal anatomical landmarks of
side. the oral cavity is essential for the development of
2. The 9-mm semicircular radiopaque mark located proper radiographic technique, assessment, diagnosis.
on the lower edge of film is most likely a fingernail Radiopaque or radiolucent appearance of image
scratch mark caused by improper handling of film dur- helps determine specific landmarks; however, NOT all
ing processing. Scratching of emulsion caused the ra- landmarks are seen on every FMX. When a lesion is
diopaque artifact on processed film. suspect, appropriate landmarks for that site should be
3. Pyramid-shaped opacity is a ghost image of spine ruled out.
caused by slumped position of patient during expo- • See Chapter 4, Head, Neck, and Dental Anatomy: nor-
sure. Patient should be required to stand erect so that mal landmarks.
spinal column is not superimposed on the film. A. Tooth tissues and periodontium (in order of density or
4. These “treelike” marks are artifacts caused by static mineralization; lighter [highly mineralized] to darker
electricity; usually occur because of low humidity, as [nonmineralized]): enamel, dentin, cementum, bone
is the case in Arizona. (first cortical bone as lamina dura, then supporting bone
as spongy), periodontal ligament (PDL space), pulp.
Radiology Infection Control B. Maxillary films have 11 common intraoral land-
Disinfection and sterilization of equipment and clinical marks:
areas involve proper attention to infection control proce- 1. Incisive foramen (Figure 5-25):
dures and standards. a. Passageway for nasopalatine nerves and vessels
• See Chapter 8, Microbiology and Immunology: stan- in middle of hard palate, posterior to central in-
dard precautions. cisors.
A. Equipment and supplies: b. Centrally located radiolucent oval-shaped ob-
1. Chair, headrest, tubehead, PID SHOULD be cov- ject between roots of central incisors.
ered with disposable barrier material such as plas- 2. Median palatal suture (intermaxillary suture)
tic bag. (Figure 5-25):
2. Controls on the machine, including activating a. Radiolucent thin line between roots of central
switch, SHOULD be covered with clear wrap or a incisors, with thin lines of cortical bone on each
disposable item. side.
3. Lead apron or shielding equivalent SHOULD be b. Runs vertically from alveolar crest to hard
sanitized by misting with disinfectant and wiping ­palate.
surface; should NOT be contaminated during pro­ 3. Nasal fossae (Figure 5-25):
cedure. a. Centrally located air-filled spaces.
4. Surface of the work area SHOULD be covered b. Radiolucent oval shapes superior to central in-
with disposable barrier material. cisors, outlined by radiopaque bone.
5. Film holders SHOULD be sterilized, and film 4. Nasal septum (Figure 5-25):
should have protective coverings or be contained a. Thin wall that divides two spaces, formed by
in disposable cup to prevent contamination of ethmoid and vomer bones and cartilage.
other surfaces. b. Centrally located radiopaque vertical strip that
6. If lead barrier is used, barrier SHOULD have pro- separates the nasal fossae.
tective coverings on all handled areas. 5. Anterior nasal spine (Figure 5-25):
B. Clinician preparation (includes personal protective a. Pointed projection of bone anterior and inferior
equipment [PPE]): to nasal cavity, between central incisors.
1. Hands SHOULD be washed and gloved; mask and b. Radiopaque triangular shape at median palatal
eyewear are optional. suture where nasal septum and fossae meet.
2. Clinician SHOULD wear a gown or protective 6. Maxillary sinus (Figure 5-26):
outerwear. a. Hollow spaces in bone superior to molar and
C. Exposure procedures: premolar apices.
1. Contaminated items (e.g., paper cups, BW tabs, b. Large, oval radiolucent areas (multilocular struc-
cotton rolls) SHOULD be carefully discarded. ture) outlined with thin lines of cortical bone.
Radiology   ���
153

Nasal fossa

Nasal
septum

Anterior
nasal Nasopalatine
spine foramen

Median
palatal
suture

Figure 5-25  Anatomical structures commonly seen on maxillary anterior radiographs.

Zygomatic
process

Maxillary
sinus

Figure 5-26  Maxillary sinus and zygomatic process commonly appear on maxillary posterior radiographs.
154   Saunders Review of Dental Hygiene

b. Circular radiopacities inferior to central incisor


apices.
2. Lingual foramen (Figure 5-30):
a. Exit for incisive vessel branches.
b. Radiolucent circle inside opaque genial tubercles.
3. Mental ridge (Figure 5-31):
a. Ridge of bone located on anterior surface of
mandible.
b. Bilateral radiopaque lines, starting inferior to
premolar apices, extending anterior to midline.
4. Mental foramen (Figure 5-32):
Inverted a. Opening for mental nerve and vessels inferior
Y-junction to premolar apices.
b. Round radiolucent area that can be mistaken for
periapical disease or condition.
5. External oblique line (ridge) (Figure 5-33):
a. Raised superior linear area of bone on external
surface of mandible.
b. Radiopaque line running anterior from ramus
across molars.
6. Internal oblique (mylohyoid) line (ridge) (Figure
Figure 5-27  Inverted Y-junction is the meeting point of the 5-33):
nasal fossa and maxillary sinus. a. Raised inferior linear area of bone on internal
surface of mandible.
b. Radiopaque line running anterior along the pre-
7. Inverted Y-junction of two anatomical landmarks molar and molar apices, from anterior part of
(Figure 5-27) (Let’s all go to the YMCA!) ramus to anterior part of alveolar process.
a. Point where nasal fossa and maxillary sinus 7. Mandibular canal (Figure 5-34):
meet. a. Canal for incisive and then inferior alveolar o5440
b. Two oval radiolucent areas outlined by thin line nerves and vessels; its posterior opening is the
of cortical bone, superior to canine apices. mandibular foramen (NOT visible on radio-
8. Maxillary tuberosity (Figure 5-28): graphs), and its anterior opening is the mental
a. MOST distal portion of alveolar process. foramen (visible).
b. Rounded, radiopaque elevation distal to third b. Radiolucent band; outlined with a thin line of
molar region. cortical bone inferior to premolar and molar
9. Hamulus (Figure 5-29): apices.
a. Extension of medial pterygoid plate of sphenoid 8. Submandibular fossa (Figure 5-34):
bone. a. Thin, depressed area of bone on internal surface
b. Radiopaque, hooklike protrusion posterior to of mandible that contains submandibular sali-
maxillary tuberosity. vary gland.
10. Zygomatic process (Figure 5-26): b. Radiolucent band inferior to internal oblique
a. Together with zygomatic bone forms zygomatic (mylohyoid) line (ridge) and premolar and mo-
arch. lar apices.
b. Starts as U-shaped band superior to molar api- 9. Nutrient canals: parallel radiolucent bands between
ces, extends posteriorly continuing as a radi- the teeth; may be mistaken for bone fractures.
opaque band.
11. Coronoid process of mandible (Figure 5-28): RADIOGRAPHIC INTERPRETATION  
a. Anterior portion of ramus. Radiographic interpretation involves the explanation and
b. Radiopaque triangular projection, usually su- clarification of what is seen on dental radiograph. Teeth
perimposed over maxillary tuberosity. appear lighter (see earlier discussion) than MOST patho-
C. Mandibular films have eight common intraoral land- logical conditions because LESS radiation penetrates
marks: them to reach the film; dental caries and other pathologi-
1. Genial tubercles (Figure 5-30): cal conditions, such as changes in the bone density and
a. Four bony spines used for muscle attachment the periodontal ligament, appear darker because x-rays
for genioglossus and geniohyoid muscles. readily penetrate these LESS dense structures.
Radiology   ���
155

Coronoid
process

Hamulus

Maxillary
tuberosity

Figure 5-28  Coronoid process appears superimposed over the maxillary tuberosity.

Hamulus

Figure 5-29  Hamulus is a radiopaque protrusion posterior to the maxillary tuberosity.

• See Chapters 6, General and Oral Pathology: diagnos- a. Radiolucent shadows under enamel as progres-
tic procedure, developmental presentations; 11, Clini- sion occurs.
cal Treatment: diagnosis of caries; 13, Periodontology: b. NOT apparent on radiographs until large.
diagnosis of periodontal disease; 15, Dental Biomateri- c. MORE effectively diagnosed by clinical exami-
als: restorations. nation.
A. Dental caries: loss of tooth structure caused by acid 3. Root surface caries: exposed roots, preceded by
by way of diet or microorganisms; radiolucent. Only bone loss and gingival recession; increased levels
about 50% to 75% sensitive on detection, NOT in older adults (Figure 5-36):
“gold standard”; MUST be accompanied by clinical a. Ditched-out radiolucent area or dot on the root.
detection. b. NOT apparent on radiographs until large.
1. Interproximal caries located on mesial and distal c. MORE effectively diagnosed by clinical exami-
surfaces (Figure 5-35): nation.
a. May progress from incipient to advanced, de- B. Restorations and associated materials:
pending on length of time. 1. CANNOT determine if material used is gold, sil-
b. Radiolucencies that advance as caries becomes ver, or metal alloy (amalgam); can be determined
larger. ONLY by size and contour of restoration.
2. Occlusal caries: located on biting surfaces of pos- 2. CANNOT determine surface of the tooth the resto-
terior teeth: ration is on; image of one can be superimposed on
156   Saunders Review of Dental Hygiene

Mental
processes
Figure 5-31  Mental processes (radiopaque lines) extend from
the apices of the mandibular premolars to the central incisors.

Lingual Genial
foramen tubercles
Figure 5-30  Lingual foramen (radiolucent) is surrounded
by the genial tubercles (radiopaque).

Mental
foramen

Figure 5-32  Mental foramen, located inferior to mandibular premolar apices, may be mistaken for
periapical pathological condition.
Radiology   ���
157

External
oblique
ridge

Internal
oblique
ridge

Figure 5-33  Internal and external oblique ridges are radiopaque structures that run from the ramus
through the molar region (external) or to the anterior region (internal).

Mandibular
canal

Submandibular
fossa
Figure 5-34  Mandibular canal and submandibular fossa are located inferior to apices of the
­mandibular molars and premolars.

Root
caries

Figure 5-35  Interproximal caries (arrows) commonly ap- Figure 5-36  Root caries appears as ditched-out radiolu-
pears as radiolucencies at the contact areas between adja- cent areas located at the cervical third of the exposed root
cent teeth. surface.
158   Saunders Review of Dental Hygiene

another restoration, giving appearance of ONLY (b) Vertical bone loss appears along one
one instead of two. side of a tooth in angular form; NOT
3. May appear lighter (radiopaque) or darker (radio- parallel to CEJ of adjacent teeth.
lucent) depending on density of material. (3) Bone loss is NOT the only diagnostic fac-
a. Radiopaque (lighter): tor; also evaluated are calculus, overhang-
(1) Retention pins appear as short rods near ing margins of restorations, caries, root
larger radiopaque casting restorations of resorption, as well as periapical and patho-
gold, silver, or alloy (amalgam). logical lesions.
(2) Whereas first-generation tooth-colored b. Radiographic appearance is NOT the only diag-
(composite) restorations were radiolucent, nostic factor; clinical examination is also used
second-generation materials feature high for diagnosis:
level of radiopaque fillers. (1) Does NOT show loss on facial and lingual
(3) Some restorative cements can be differ- places, ONLY on interproximal alveolar
entiated ONLY by location and degree of crest.
radiopacity; these include zinc phosphate, (2) When becomes apparent, as much as 20%
ZOE, calcium hydroxide cements. of bone may have already been lost.
(4) Amalgam tattoo results from amalgam par- E. Traumatic lesions, injuries caused by external forces:
ticles embedded into tissues during restor- 1. Occlusal forces:
ative procedures. a. Trauma: widening of radiolucent PDL space sur­
(5) Gutta-percha and silver pins appear as rods rounding root, outlined by radiopaque ­ lamina
within pulp canals.
(6) Implant(s) with varying degrees of sur­
rounding osteointegration opacity depend-
ing on length of time of placement in jaws
(Figure 5-37).
b. Radiolucent (darker):
(1) Glass ionomers appear MORE radiolucent
than surrounding teeth.
(2) Porcelain, acrylic resins, sealants, older
composites.
4. Distinguishing cement-retained single-unit im-
plant restoration from natural tooth is often diffi-
cult without accompanying radiograph.
C. Torus (plural, tori): slow-growing bone mass on man-
dibular alveolar process, unilateral or bilateral radi-
Figure 5-37  Implants appear as radiopaque areas with
opacity superimposed superior to or inferior to apices varying degrees of osseous integration opacity (arrows).
on mandible on PAs; palatal, central radiopacity on (From Bath-Balogh M, Fehrenbach MJ: Illustrated dental embryol-
occlusal films of palate. ogy and anatomy, ed 2, St. Louis, 2006, Saunders/Elsevier.)
D. Periodontal disease: affects periodontium (supporting
structures of teeth) (Figure 5-38).
1. Gingivitis: inflammation that affects marginal
­gingiva.
a. NOT apparent on radiographs.
b. Diagnosed ONLY by clinical examination.
2. Periodontitis (chronic): inflammation that involves
alveolar bone loss.
a. Appears as radiolucency (but ONLY in two di-
mensions):
(1) FIRST noted as fuzzy or ditched-out area
where destruction of cortical alveolar crest
of bone occurs.
(2) In MORE advanced stages as horizontal or
vertical bone loss (Figure 5-39): Figure 5-38  Periodontal disease is typified on radiographs
(a) Horizontal bone loss appears radiograph- by fuzzy appearance of alveolar crest bone; arrows indicate
ically parallel to CEJ of adjacent teeth. areas of advanced bone loss.
Radiology   ���
159

dura, possibly excess opaque cementum at apex instrumentation and debris that irritate sulcus
of root(s) (hypercementosis). area, something left in sulcus after initial peri-
b. Lack of occlusal contact: narrowed radiolucent odontal scaling, or primary tooth’s root portion
PDL space (may also have extrusion or hyper- left after shedding.
eruption). d. Condensing osteitis (chronic focal sclerosing
2. Tooth fractures: osteomyelitis): radiopaque area attached to
a. Crown fracture usually involves anteriors and apex of a nonvital tooth with a low-grade infec-
results from fall or blows to face or mouth; tion (Figure 5-42).
space from missing structure appears radiolu- e. Sclerotic bone (osteosclerosis): radiopaque, de-
cent on radiograph. fined area around apices of asymptomatic, non-
b. Root fracture occurs MOST frequently in carious, nonvital tooth; MOST likely associated
anteriors; appears as radiolucent horizontal line with inflammation (Figure 5-43).
on root. G. Malignancy (cancer): destructive radiolucency usu-
3. Resorption: loss of tooth structure with NO ally centrally in mandible (molar or premolar), occa-
­resultant clinical recognition; either internal or sionally apex of tooth.
­external:
a. External resorption, which starts FIRST on
the root surface with loss and a flat rather than
conical appearance radiographically; bone and
lamina dura appear normal; can result from
orthodontic therapy when teeth have been Vertical
bone loss
moved TOO rapidly (Figure 5-40).
b. Internal resorption, which appears as asymp-
tomatic radiolucency; results from trauma or
pulp capping; begins in pulp and travels into
dentin. Horizontal
bone loss
4. Enamel pearl: round, radiopaque calcifications
MOST commonly seen externally on multirooted
teeth in furcation area or at CEJ. Figure 5-39  Vertical or horizontal bone loss is assessed by
5. Cementicle: round, radiopaque calcifications on determining whether the bone level maintains parallelism
with the cementoenamel junction of associated teeth; paral-
the root surface within the PDL space. lelism is maintained with horizontal bone loss but not with
6. Extraction: radiolucent space in bone that fills in vertical bone loss (often appears angular).
with time.
7. Avulsion: complete accidental removal of the tooth
from the bone; radiolucent space in bone.
F. Pulpal lesions: commonly seen on radiographs:
1. Pulp stones: round, radiopaque calcifications
MOST commonly seen in molars (Figure 5-41).
2. Pulp erasure: disappearance of usually radiolucent
canal because of cavity trauma caused by caries,
abrasion, or abnormal forces; appears radiopaque
because of secondary dentin.
3. Periapical lesions: usually located around apex but
can be at ANY point around root because of lateral
canals; ONLY a late finding on radiograph; clinical
examination used to detect early lesions.
a. Periapical abscess: radiolucent area located at
end of apex of symptomatic tooth with infected
pulp; appears as widening of PDL space and
obliteration of lamina dura.
b. Periapical granuloma: radiolucent area at the
apex of nonvital, usually asymptomatic tooth
(caused by pulp death or necrosis).
c. Periodontal abscess: radiolucent area located Figure 5-40  External resorption appears as a blunted or
at end or lateral part of root; may be result of flat appearance of the apices.
160   Saunders Review of Dental Hygiene

Pulp stones

Figure 5-41  Pulp stone calcifications in the pulps of


­molars. Figure 5-43  Sclerotic bone near the root apex of asymp-
tomatic vital tooth.

3. CANNOT determine whether it is a primary or


secondary tumor from radiographic presentation
alone.
H. Other pathological lesions with radiographic pre-
sentations (see also Chapter 6, General and Oral
Pathology):
1. Ameloblastoma: aggressive benign tumor with
multilocular radiolucent bone destruction having
“soap bubble” appearance.
2. Cherubism (familial fibrous dysplasis): disorder o6210
with bilateral, symmetrical, well-defined, well-
corticated multilocular radiolucency in posterior
Figure 5-42  Condensing osteitis at the root apex of a mandible and displacement of teeth.
nonvital tooth. 3. Fibrous dysplasia: disorder with unilocular or
multilocular radiolucency having “ground glass” ap-
pearance because of the many C-shaped trabeculae.
4. Multiple myeloma: cancer that on a skull radio-
1. Diffuse “moth-eaten,” “ratty”; NO corticated graph shows radiolucent “punched-out” lesions.
­outline; adjacent teeth being displaced, loosened, 5. Osteoradionecrosis: postradiation complication
and/or absorbed; certain breast and prostate me- that shows wide spectrum of presentations from
tastases may be radiopaque owing to osteogenic normal to osteolysis (dissolution or degeneration
qualities of tumor, with areas of bone production of bone tissue); does NOT always correlate with
and sclerosis. severity or detect presence.
2. Primary: squamous cell carcinoma; secondary: 6. Paget’s disease: disorder with an enlarged maxilla,
from breast, bronchus, prostate, kidney, or thyroid overall loss of lamina dura, “cotton-wool” appear-
cancers metastasized to jaws. ance of bone, patchy opacities.
Radiology   ���
161

clinical study  

Age 32 YRS SCENARIO

Sex ☐  Male   ☒  Female The patient recently had her orthodontic bands
removed. Visual intraoral examination reveals a large
Height 5���
’��
2�
” restoration on tooth #30, recession on the right side
of her mouth, and no apparent caries or other resto-
Weight 130 LBS
rations. During examination of her past radiographs,
BP 119/78 well-­defined radiopacity is seen apical to tooth #30.
Also, the apices of teeth #27 and #28 appear blunted
Chief Complaint “My lower right back teeth are and roots appear shorter than normal.
­sensitive when I eat.”

Medical History Second month of pregnancy


No complications except morning
sickness

Current Medications Folic acid and prenatal vitamins qd

Social History Stay at home mom with twin 4-year-


old boys

1. What is (are) the most likely reason(s) for the patient’s 3. Radiopaque area most likely indicates condensing oste-
sensitive teeth? What homecare recommendations can itis (chronic focal sclerosing osteomyelitis), which re-
be made? sults from a low-grade infection or mild irritation. Tooth
2. Given the patient’s history, what types of radiographs is most likely nonvital; teeth that contain large restora-
are recommended? tions are the most frequently affected. Condensing os-
3. What is the radiopacity noted with #30, and how does teitis most commonly occurs in mandibular first molars
it relate to her sensitivity? in adults. Because this condition is asymptomatic, no
4. What does the condition noted with #27 and #28 most treatment is necessary. Pulp testing is also ­indicated.
likely indicate? 4. This most likely indicates external resorption of apices
of teeth #27 and #28 from mechanical forces placed on
1. The patient’s sensitivity may be caused by gingival them by orthodontic movement. This type of resorp-
recession, recent removal of orthodontic bands, or tion does not have any signs or symptoms and can-
erosion associated with morning sickness. Further not be detected clinically. No treatment for external
­information must be gathered to determine cause. Be- resorption is needed.
cause patient did not present clinically with caries and
has only one restoration, the best approach is to provide
oral hygiene instructions on toothbrushing technique
Review Questions
(modified Stillman’s method) to reduce ­ incidence of
recession and encourage use of mineralizing fluoride 1 Which one of the following energies in the electromagnetic
 ���������
and calcium products (for sensitivity). She should spectrum is considered ionizing radiation?
be examined in 1 month to evaluate effectiveness of A. Radar rays
homecare procedures in reducing sensitivity. B. Radio waves
2. Because the patient is pregnant and has no obvious C. Infrared rays
signs of decay, radiographs can be postponed until af- D. Gamma rays
ter pregnancy unless sensitivity does not abate after 2 What is the source of electrons used for x-ray production?
 ������
practice of homecare recommendations. Although use A. Tungsten filament
of a lead apron or lead equivalent with a thyroid collar B. Molybdenum cup
reduces chance of radiation reaching developing fe- C. Tungsten target
D. Copper stem
tus, most dental practitioners prefer to take only those
3 What part of the circuit provides the energy to propel the
 ������
radiographs that are believed to be absolutely neces- electrons and produce x-rays?
sary to the safety and health of the pregnant patient. A. Filament circuit
If the tooth discomfort continues at the 1-month visit, B. High-voltage circuit
specific areas of sensitivity must be identified and C. Step-down transformer
­appropriate radiographs must be exposed. D. Milliampere
162   Saunders Review of Dental Hygiene

4 Where does the generation of x-rays in the x-ray tube actu-


 ������ 14 Which one of the following is used to restrict the size of the
ally occur? x-ray beam?
A. Tungsten filament A. Filtration
B. Focusing cup B. Collimation
C. Copper stem C. Film-holding device
D. Tungsten target D. Fast film
5 When x-rays interact with an outer shell electron of an atom,
 ������ 15 If the source-to-film distance is changed from 8 to 24 inches,
causing ionization and scattering in a different direction, it the intensity of the beam becomes
is called A. three times as intense.
A. the photoelectric effect. B. nine times as intense.
B. Thompson scatter. C. one-ninth as intense.
C. Compton scatter. D. one-third as intense.
D. no interaction. 16 Magnification occurs when there is a
6 Which theory BEST describes the free radical formation
 ������ A. long object-film distance.
that is so injurious to living tissue? B. small focal spot.
A. Direct theory C. parallel object and film.
B. Indirect theory D. short target-film distance.
C. Dose-response curve 17 Which one of these film-processing ingredients converts
D. Buccal object rule energized crystals into metallic silver?
7 What unit of radiation represents the amount of radiation
 ������ A. Sodium carbonate
energy absorbed by the tissue? B. Sodium thiosulfate
A. Roentgen C. Potassium alum
B. Sievert D. Hydroquinone
C. Dose rate 18 What is the processing order that films undergo in an auto-
D. Gray matic film processor?
8 What factor is MOST important in a consideration of the
 ������ A. Water, developer, water, fixer, dry
biological effects of radiation used in dentistry? B. Developer, water, fixer, dry
A. Whole-body exposure C. Developer, fixer, water, dry
B. Limited area exposure D. Developer, water, fixer, water, dry
C. Acute exposure (limited area) 19 Which of the following is LEAST likely to cause fogged film?
D. Latent period A. Improper safelighting
9 Which one of the following cell types is MOST sensitive to
 ������ B. Developer spots
radiation? C. Outdated film
A. Highly specialized cells D. Light leakage
B. Mature cells 20 Which radiopaque maxillary anatomical landmark is MOST
C. Slowly dividing cells likely to be seen on a maxillary incisor periapical radio-
D. Immature cells graph?
10 All of the following reduce patient exposure to radiation, A. Nasal septum
EXCEPT one. Which one is the EXCEPTION? B. Median palatal suture
A. Filtration C. Zygomatic process
B. Collimation D. Genial tubercles
C. Positioning 21 Which radiolucent mandibular anatomical landmark is
D. Lead apron MOST likely to be seen on a mandibular molar periapical
11 Which one of the following reduces operator exposure to radiograph?
radiation? A. Lingual foramen
A. Fast film B. External oblique line
B. Distance C. Submandibular fossa
C. PID D. Mental foramen
D. Film-holding device 22 Which is the FIRST step in the radiograph mounting
12 Which radiographic technique has LESS distortion and mag- ­process?
nification and creates a MORE accurate image on film? A. Organizing films by anatomical area
A. Bisecting-angle B. Separating maxillary from mandibular films
B. Occlusal C. Placing films dot-side up
C. Paralleling D. Mounting premolar films first
D. Panoramic 23 Root caries of a tooth is o7440
13 Which one of these extraoral techniques is used specifically A. typically located at the middle third of the root.
to evaluate the maxillary sinuses? B. identified as well-circumscribed radiopacity.
A. Waters C. associated with bone loss and gingival recession.
B. Lateral cephalometric D. found more often in adolescents.
C. Panoramic
D. Lateral jaw
Radiology   ���
163

24 If partially formed third molars are noted on a panoramic 34 Which of the following situations increases penumbra?
radiograph, how old is the patient? A. Increased object-to-film distance
A. 6 to 10 years of age B. Reduced focal spot size
B. 11 to 15 years of age C. Faster-speed film
C. 16 to 20 years of age D. Movement of film
D. 21 to 25 years of age 35 The lingual foramen should be evident on the
25 Overlap on a left molar bitewing radiograph is caused by A. maxillary posterior film.
improper B. maxillary anterior film.
A. exposure time. C. mandibular posterior film.
B. horizontal angulation. D. mandibular anterior film.
C. film placement. 36 On a mandibular molar film the MOST superior radiopaque
D. patient movement. anatomical landmark is the
26 When placed backward, a film exposed in the mouth will A. external oblique ridge.
­exhibit B. coronoid process.
A. fogginess. C. internal oblique line.
B. no image. D. mental process.
C. waffle pattern. 37 Exposure of a radiograph for 3⁄5 of a second is equal to how
D. cone cut. many impulses?
27 A patient of record complains of temperature sensitivity A. 3
in the maxillary right second molar region of the mouth. A B. 15
full-mouth series of radiographs was taken 2 years previ- C. 22
ously. Which radiographs should be prescribed? D. 36
A. Panoramic 38 A white area with a right-angle-shaped border on a film is
B. Horizontal bitewings caused by
C. Vertical bitewing A. rectangular PID.
D. Periapical B. cylindrical PID.
28 The “coin test” is a method of assessing C. fixer cutoff.
A. film freshness. D. developer cutoff.
B. proper kVp settings. 39 What is the inverted Y-junction in a radiograph?
C. contamination by developer. A. Triangular shape at median palatal suture where nasal
D. proper safelighting. septum and fossa meet
29 If films were properly exposed at 6 mA for 4 seconds, films B. Rounded, radiopaque elevation distal to the third molar
exposed for 3 seconds would have an mA of region
A. 2. C. Radiopaque, hooklike protrusion posterior to the maxil-
B. 8. lary tuberosity
C. 16. D. Point where the nasal fossa and maxillary sinus meet
D. 24. 40 Which of the following diseases has a “ground glass” ap-
30 Which of the following situations created the error of a ra- pearance on radiographs?
diolucent shadow that obscures the apices of the maxillary A. Osteoradionecrosis
dentition on a panoramic film? B. Multiple myeloma
A. Lips were not closed. C. Fibrous dysplasia
B. Midsagittal plane was not perpendicular to the floor. D. Cherubism
C. Tongue was not touching the palate. 41 Root fracture occurs MOST frequently in the anterior por-
D. Teeth were not in the focal trough. tion of the mouth. Root fracture appears as a radiopaque
31 If the x-ray machine is operating at 70 kVp, what thickness horizontal line on the root.
of filtration is necessary? A. Both statements are true.
A. 1.5 mm B. Both statements are false.
B. 2.0 mm C. The first statement is true, the second is false.
C. 2.5 mm D. The first statement is false, the second is true.
D. 3.0 mm 42 Film duplication is the process of copying or reproducing a
32 The lead diaphragm in the tubehead is referred to as the second film or set of films. With film duplication, there is no
A. target. need to reexpose the patient to radiation.
B. filament. A. Both statements are true.
C. filter. B. Both statements are false.
D. collimator. C. The first statement is true, the second is false.
33 Developing solution splashed on the film before processing D. The first statement is false, the second is true.
will result in
A. white spots.
B. dark spots.
C. brown stain.
D. yellow stain.
164   Saunders Review of Dental Hygiene

43 Contrast is the difference between lighter and darker shades Answer Key and Rationales s0350
(grays) on the radiograph. Decreased kVp (90 or higher)
results in many shades of gray on the film.
A. Both statements are true.
B. Both statements are false. 1 (D)  Gamma rays are ionizing forms of radiation in
C. The first statement is true, the second is false. electromagnetic spectrum. Radar rays and infrared rays
D. The first statement is false, the second is true. are NOT high-energy forms of radiation and thus do
44 Optimum developing takes approximately 10 minutes at NOT cause ionization. Radio waves have long wave-
50° F. Developing reduces exposed silver halide crystals into lengths and therefore CANNOT cause ­ionization.
black metallic silver through one ingredient in solution. 2 (A)  Heating tungsten filament is necessary to pro-
A. Both statements are true. vide a source of electrons for generation of x-rays.
B. Both statements are false.
Molybdenum cup directs the electrons to the tungsten
C. The first statement is true, the second is false.
target in the anode. Tungsten target is the spot in an-
D. The first statement is false, the second is true.
45 The bitewing view visualizes crowns, contacts, and height ode that electrons hit to produce x-rays. Copper stem
of alveolar bone. The bitewing view is not included in a in the anode dissipates the heat that is produced.
full-mouth series. 3 (B)  High-voltage circuit provides the voltage neces-
A. Both statements are true. sary to propel electrons to the anode. Filament cir-
B. Both statements are false. cuit controls the heating of the filament and quantity
C. The first statement is true, the second is false. of available electrons. Step-down transformer is
D. The first statement is false, the second is true. necessary to decrease incoming voltage to heat the
46 Overdeveloped film that is too dark may occur because de- filament. Milliampere is 1⁄1000 of an ampere and deter-
veloping solution is at too high a temperature. mines quantity of electrons produced.
A. Both the statement and reason are correct and related.
4 (D)  Electrons hit tungsten target and convert kinetic
B. Both the statement and reason are correct but NOT related.
energy into heat and x-rays. Tungsten filament has to
C. The statement is correct, but the reason is NOT.
D. The statement is NOT correct, but the reason is correct. be heated to provide electrons for x-ray production.
E. NEITHER the statement NOR the reason is correct. Focusing cup in cathode surrounds filament and directs
47 Fogged films with white streaks may occur because there is electrons to target. Target is embedded in copper stem,
improper safelighting in the darkroom. helping keep target cool.
A. Both the statement and reason are correct and related. 5 (C)  Compton scatter occurs when an outer shell elec-
B. Both the statement and reason are correct but NOT related. tron is removed and scatters in a different direction.
C. The statement is correct, but the reason is NOT. Photoelectric effect occurs when an inner shell elec-
D. The statement is NOT correct, but the reason is correct. tron is removed and is absorbed so that it ceases to
E. NEITHER the statement NOR the reason is correct. exist. Coherent or Thompson scatter occurs when an
48 Genial tubercles are an exit for incisive vessel branches.
outer shell electron scatters in a different direction but
Genial tubercles appear as radiolucent circle inside opaque
no ionization takes place. No interaction occurs when
lingual foramen tubercles.
A. Both statements are true. x-rays pass through the atoms of tissue but NEVER
B. Both statements are false. react with them.
C. The first statement is true, the second is false. 6 (B)  Indirect theory explains tissue damage by
D. The first statement is false, the second is true. ­positing that x-ray photons are absorbed into cells,
49 Lateral cephalometric projection evaluates causing the production of free radicals (toxins);
A. posterior portion of mandible only. this damage is believed to be caused by the water
B. lateral areas of mandible that are too large for ­periapical content in cells. Direct theory explains damage as
film. ­occurring from direct hit to the target within a cell;
C. trauma and facial growth and development. this ­ damage seldom occurs. Dose-response curve
D. frontal and ethmoidal sinuses only.
graphically ­demonstrates that any level of radiation
50 Compton scatter
exposure causes a response from the tissue. Buccal
A. is ionization that occurs when x-ray photon interacts
with an inner shell electron. object (SLOB) rule is whether artifact or object is
B. occurs when electron is ejected and x-ray photon scat- lingual or buccal: in second radiograph, if central ray
ters in a different direction. is moved and object moves in same direction, then
C. is responsible for approximately 10% of all scatter in object is lingual, if it moves in the opposite direction,
dental x-rays. buccal.
D. is a result without any occurrence of ionization. 7 (D)  Gray is (SI) unit that represents amount of en-
ergy absorbed by tissue. Roentgen is standard unit for
­measuring the quantity of ionization in air. Sievert
is SI unit for dose equivalent and is used to compare
Radiology   ���
165

different types of radiation. Dose rate is amount of 15 (C)  According to the inverse square law, intensity
radiation absorbed per unit of time. decreases as distance from the source increases;
8 (B)  Limited area exposure refers to radiation in this would be adding the same amount of dis-
small area, such as occurs in dental radiation. Whole- tance each time when it really should be squared.
body exposure is MORE extensive but is NOT used Intensity of the beam does NOT increase as this
in dentistry. Acute exposure (limited area) refers to suggests, but decreases; because distance is three
exposure to high doses of radiation in a specific area. times the original, intensity CANNOT be one-third
Latent period is the time between exposure and the the original.
observable effect. 16 (A)  Increased target-film distance allows MORE par-
9 (D)  Immature cells divide rapidly and therefore are allel rays to hit the film; thus less magnification oc-
the MOST sensitive to radiation. Highly specialized curs. Small focal spot size in the x-ray tube increases
cells are MORE radioresistant to radiation. Mature sharpness. If the object and film are parallel to each
cells do NOT change; because they are MORE spe- other, distortion is decreased. Short target-film dis-
cialized in function, they are LESS sensitive to radia- tance increases magnification because fewer parallel
tion. Cells that divide at a slow rate have LESS cell rays hit the film.
activity and LESS reaction to radiation. 17 (D)  Hydroquinone (or elon) in developing solu-
10 (C)  Positioning patient reduces risk of exposure to tion converts energized crystals into metallic silver.
operator but NOT to patient. Filtration reduces radia- Sodium carbonate softens emulsion and speeds up
tion by filtering out the nonproductive x-rays. Colli- action of developing agents. Sodium thiosulfate, in
mation reduces radiation exposure by restricting size fixer solution, clears unexposed silver halide crystals
of the beam directed at the patient’s face. Lead apron from the emulsion. Potassium alum, in fixer, shrinks
or equivalent protects patient from unnecessary non- and hardens emulsion.
productive scatter radiation. 18 (C)  Processing order in which films go through the
11 (B)  Distance of at least 6 feet from the primary beam automatic processor is “developer-fixer-water-dry.”
is essential for reducing operator exposure. Faster film In automatic processing, a reduced number of water
reduces exposure to the patient. The position-indicat- baths and the addition of hardeners in the developer
ing device (PID) should be lead lined and rectangular and fixer solutions allow film emulsion to remain
to reduce patient exposure. Film-holding devices re- firm throughout the automated procedure. Film ex-
duce exposure to the patient’s hand. posed to too much moisture will become soft and
12 (C)  Paralleling technique is BEST and provides a prone to stick to the machine roller. In manual pro-
MORE accurate image because the primary beam cessing, water baths between developing and fixing
is at right angles to the tooth and the film. Bisecting are necessary and film softness is NOT an issue be-
(angle) technique is NOT as accurate because the use cause the chemical concentrations and temperatures
of bisecting line allows MORE errors. Occlusal tech- used are NOT as high as those used in automatic
nique results in a large amount of distortion, which processing.
is caused by the dramatic angles used and the need 19 (B)  Developer spots occur when the developing so-
for greater coverage. Panoramic technique leads to lution contacts the film before processing procedures
greater distortion because the need for a larger survey begin. Proper safelighting is necessary. safelights
leads to blurred images outside the focal trough. (direct illumination) MUST be spaced at least 4 feet
13 (A)  Waters projection specifically shows maxillary si- from working surface with low-wattage bulb (71⁄2/15
nus area. Lateral cephalometric technique shows trauma, watts). The expiration date on the film SHOULD be
facial growth, and development. Panoramic projection checked, and the film SHOULD be stored in a cool,
is used to evaluate impactions, diseases, growths of the dry environment. Because film is sensitive to white
jaw. Lateral jaw exposure shows posterior part of the light, the darkroom should be “white-light tight.”
mandible and is used for patients who are unable to 20 (B)  The nasal septum is a vertical opaque strip that
open their mouths. separates the nasal fossae. The median palatine suture
14 (B)  Collimation is a lead diaphragm that restricts is a thin radiolucent line between the maxillary cen-
size of the x-ray beam directed at the patient’s face tral incisors. The zygomatic process is a radiopaque
to 2.75 inches. Filtration uses aluminum disc to fil- U-shaped band located superior to the apices of the
ter out the nonproductive, longer wavelengths from maxillary molars. Genial tubercles are four radiolu-
primary beam. Film-holding device is used to hold cent bony spines located inferior to the apices of the
the film in place but does NOT restrict the size of mandibular central incisors.
the primary beam. Fast film (F-speed film is fastest) 21 (C)  Submandibular fossa is a radiolucent band that
reduces radiation exposure to the patient but does runs inferior to mylohyoid and apices of molars. Lin-
NOT restrict the size of the beam. gual foramen is a radiolucent circle inside the genial
166   Saunders Review of Dental Hygiene

tubercles, inferior to apices of mandibular central in- tones of a film. Contamination by developer solution
cisors. External oblique line is a radiopaque line that is identified by dark spots on the film.
runs anterior from ramus across mandibular molars. 29 (B)  Multiplying milliamperage and time gives total
Mental foramen is a radiolucent oval inferior to api- number of x-rays produced (mA × seconds = mAs).
ces of premolars. Total number of x-rays produced affects film den-
22 (C)  Films should be placed dot-side up on a lighted sity; therefore increase in one (mA/time) requires
viewbox first. Next, films should be organized by decrease in the other (time/mA) to maintain a similar
anatomical area, separated into maxillary and man- density.
dibular groups, and subsequently mounted in the ap- 30 (C)  Patient’s tongue must be touching the palate or
propriate area. Premolar films are mounted after the the resulting airspace will appear as a radiolucent
molar films are in place. line, obscuring apices of the anterior teeth. Open
23 (A)  Root caries is associated with bone loss and gingi- lips result in a radiolucent shadow that obscures
val recession, is located on exposed root surfaces (of- anterior teeth. Improper midsagittal alignment re-
ten in cervical areas), and is identified as a ditched-out sults in unequal magnification of ramus and pos-
radiolucency; found MORE often in older adults (who terior teeth. Distortion of anterior teeth (wide and
experience MORE root exposure and are at risk of thin appearance) occurs if the teeth are outside the
­medication-induced xerostomia). focal trough.
24 (B)  Partially formed third molars on a panoramic ra- 31 (C)  According to federal and state regulations, filtra-
diograph indicate that the patient is MOST likely to tion (thickness of aluminum) for machines operating
be 11 to 15 years old. Roots of third molars develop below 70 kVp is 1.5 mm. Machines operating at 70
approximately 6 years before their expected eruption kVp and above are required to have filtration thick-
at age 18 to 21. Third molars may NOT be present ness of 2.5 mm.
in individuals 6 to 10 years old. Individuals 16 years 32 (D)  Collimator, located in tubehead, is a lead dough-
and older should show well-formed third molars. nut or diaphragm. Target refers to tungsten target in
25 (B)  Overlap on left molar bitewing radiograph is anode. Filament is the tungsten filament in the cathode.
caused by improper horizontal angulation. Improper Filter refers to aluminum filter that is used to eliminate
exposure time affects the darkness and lightness of longer wavelengths from beam.
film; poor film placement affects which teeth are ex- 33 (B)  Recontamination of film with developing solu-
posed or how well film is centered; patient movement tion results in an overdevelopment of the affected
results in blurring. areas and causes dark spots. Recontamination with
26 (C)  Film that is exposed when placed backward in fixer solution clears film of silver halide in emulsion,
the mouth will have a waffle or herringbone pattern. resulting in white spots. Insufficient washing of films
Fogginess is associated with old film; no image in- after fixing results in brown and yellow stains.
dicates lack of exposure; cone cut is associated with 34 (B)  Reducing the focal spot size increases penum-
cone placement. bra (fuzziness, less sharpness); film movement and
27 (D)  For a patient of record with a specific complaint increase in the object-to-film distance or film speed
(temperature sensitivity in the maxillary right second decrease penumbra.
molar region of the mouth) and a recent FMX, a periapi- 35 (D)  Because the lingual foramen is located inferior to
cal (PA) radiograph is the BEST choice. Temperature apices of the mandibular anterior incisors at the mid-
sensitivity is often associated with pulpal pathology, line, it can be seen ONLY on mandibular anterior film.
and PA provides coverage of the entire tooth struc- 36 (A)  External oblique ridge is the MOST superior
ture and the bone surrounding the apex of the specific radiopaque anatomical landmark on mandibular mo-
site. Panoramic film is NOT recommended because it lar film. Coronoid process appears ONLY on maxil-
may NOT provide the clarity required to detect pulpal lary molar films. Internal oblique line is the inferior
pathological or other site-specific conditions and it re- ­radiopaque landmark on a mandibular film, and the
quires exposure to greater amounts of radiation than mental process is a radiopaque landmark located infe-
a PA film. Horizontal and vertical bitewing films do rior to mandibular incisors.
NOT provide coverage of apices (root apex) and sur- 37 (D)  Converting seconds to impulses requires cross-
rounding bone and thus are NOT as helpful. multiplication. It is necessary to know that each im-
28 (D)  The “coin test” is a method of assessing improper pulse equals 1⁄6 of a second (3/5 = x/60; 5x = 180; x =
safelighting caused by excessive bulb wattage, im- 36). Therefore 3⁄5 of a second equals 36 impulses.
proper distance, or inadequate filtration. Film fresh- 38 (A)  Right-angle shape is the border of a rectangular
ness is confirmed by the expiration date and by the PID. Circular border is caused by a cylindrical PID.
absence of film fogginess. Proper kVp settings affect Fixer cutoff has a straight black border, developer
film quality as determined by the gray, black, white cutoff has a straight white border.
Radiology   ���
167

39 (D)  Inverted Y-junction in a radiograph is a point 45 (C)  First statement is true, second is false. Bitewing
where the nasal fossa and maxillary sinus meet. (BW) view visualizes crowns, contacts, and height
Rounded, radiopaque elevation distal to the third mo- of alveolar bone, included in a full-mouth ­ series
lar region is the retromolar pad. Radiopaque, hooklike (FMX).
protrusion posterior to the maxillary tuberosity is the 46 (A)  Both statement and reason are correct and related.
hamulus, extension of medial pterygoid plate of the Overdeveloped film that is too dark may be caused
sphenoid bone. Anterior nasal spine is the triangular by developing solution at too high a temperature. It
shape at the median palatal suture where nasal septum could also be due to excessive development time.
and fossa meet. 47 (D)  Statement is not correct, but reason is correct.
40 (C)  Fibrous dysplasia shows unilocular or multilocular Fogged films with a gray hue may occur because
radiolucency with “ground glass” appearance caused there is improper safelighting in the darkroom.
by many trabeculae. Osteoradionecrosis ­demonstrates 48 (B)  Both statements are false. Genial tubercles are
wide spectrum of presentations from normal to oste- four bony spines used for muscle attachment. They
olysis. Skull radiograph of multiple myeloma shows appear as circular radiopacities inferior to apices of
“punched-out lesions.” Cherubism shows well-de- central incisors. Lingual foramen is an exit for incisive
fined, well-corticated, multilocular, radiolucent, bilat- vessel branches and appears as radiolucent circle in-
eral, symmetrical lesions in posterior mandible. side opaque genial tubercles.
41 (C)  First statement is true, second is false. Root fracture 49 (C)  Lateral cephalometric projection is used to eval-
occurs MOST frequently in anterior portion of mouth, uate trauma and facial growth and development. Pos-
appearing as radiolucent horizontal line on root. teroanterior projection is used to evaluate frontal and
42 (A)  Both statements are true. Film duplication is ethmoidal sinuses, as well as orbits and nasal cav-
process of copying or reproducing second film or set ity. Lateral jaw exposure is used to evaluate posterior
of films without reexposing patient to radiation. portion of the mandible or lateral areas that are too
43 (C)  First statement is true, second is false. Contrast large for periapical film.
is the difference between lighter and darker shades 50 (B)  Compton scatter is ionization that results when x-
(grays) on radiograph. Increased kVp (≥90) results in ray photon interacts with outer shell electron. Occurs
many shades of gray on the film. when electron is ejected and x-ray photon scatters
44 (B)  Both statements are false. Optimum developing in a different direction. Responsible for approxi-
takes approximately 5 minutes at 68° F. Developing mately 62% of all scatter in dental x-rays. Thompson
reduces exposed silver halide crystals into black me- (­coherent) scatter results without the occurrence of
tallic silver through four ingredients in solution. any ionization.
CHAPTER  6

General and Oral Pathology

PATHOLOGY   b. Type of cell injured; different types of cells


p0010 General pathology includes the study of the destruction have variable responses to injury.
and repair of bodily tissues. Oral pathology is the study c. Biochemical considerations, including integ-
of the destructive conditions associated with the mouth, rity of cell membranes, adenosine triphosphate
head, neck. Can be either primary or secondary (follows (ATP) production systems, protein synthesis
and results from an earlier disease), or there can be comor- mechanisms, genetic apparatus.
bidity, with more than one pathological condition pres- B. Response to injury: inflammation:
ent. MOST conditions and diseases go through periods of 1. (Cardinal) signs: heat, redness, swelling, pain, loss
exacerbation (relapse, attack, flare-up), with an increase of function.
in the severity of a disease or its symptoms, as well as 2. Pathogenesis involves circulatory changes, altera-
periods of remission (absence of disease activity). tions in blood vessel permeability, leukocyte re-
In many cases etiology (cause) of disease is unknown. sponse, chemical mediators.
Pathogenesis is the course of a disease or how it progresses. 3. Types:
Even if etiology and pathogenesis are known, many condi- a. Acute: immediate response to injury, with in-
tions and diseases still can be treated ONLY by palliative creased blood flow, increased vessel perme-
methods, care concentrated on reducing severity of symp- ability, movement of leukocytes into the tissue
toms (see following discussion). Risk factors for NOT (MAINLY neutrophils [polymorphonucleo-
achieving oral health are listed so that the medically com- cytes, PMNs], FIRST at site of injury).
promised patient will receive correct dental hygiene care. b. Chronic: low-grade inflammation that lasts
• See CD-ROM for Chapter Terms and WebLinks. weeks to years, during which inflammation, in-
• See Chapters 9, Pharmacology: specific usage; 10, jury, healing occur all at once, with migration of
Medi­cal and Dental Emergencies: emergencies in the phagocytes (monocytes, macrophages), plasma
dental setting; 11, Clinical Treatment, and 16, Special cells, lymphocytes.
Needs Patient Care: related medical disabilities. 4. Functions:
a. Neutralize, destroy, isolate, remove injurious
INJURY, INFLAMMATION, AND REPAIR   agent.
Disease causes various injuries to the body. ALL diseases b. Remove necrotic debris.
include processes of inflammation and repair. Inflamma- c. Initiate repair and regeneration.
tion is always FIRST response to injury; it can be BOTH C. Response to injury:
protective and destructive process. Repair occurs in re- 1. Regeneration: replacement of injured cells with
sponse to inflammation; involves BOTH regenerative cells of same type, dependent on cell type.
and reparative processes. 2. Repair: restoration of cell function and morphol-
A. Injury can have various causes: ogy (form) after injury:
1. Hypoxia (lack of oxygen): from chemical, physi- a. Response to severe or persistent injury; repair
cal, or microbial agents, e.g., heart attack (myocar- CANNOT be accomplished by regeneration
dial infarction). alone.
2. Immunological factors: such as anaphylactic reac- b. Usually accomplished by fibrous connective
tions and reactions to self-antigens, e.g., asthma. tissue (scar).
3. Genetic defects: result in pathological changes, 3. Factors:
e.g., sickle cell anemia. a. Angiogenesis: proliferation of new blood ves-
4. Nutritional imbalances: including lack of sufficient sels.
proteins or other nutrients and dietary excesses, b. Remodeling: secretion of collagen-degrading
e.g., high intake of animal fats. enzymes.
5. Mechanisms of injury: c. Fibrosis: repair by excessive fibrous connective
a. Type, duration, severity of injury. tissue.

168
General and Oral Pathology   169

D. Latent period: time elapsed between exposure to


injury and first appearance of symptoms and signs. Infectious Diseases
Infectious diseases may be bacterial, viral, or fungal.
Diagnostic Process Note that hepatitis is also discussed under metabolic
Diagnosis is translation of data gathered by clinical and ­disorders.
radiographic examination into organized, classified defi- • See Chapters 8, Microbiology and Immunology: infec-
nition of conditions or diseases present. Preliminary di- tious diseases and microorganisms; 9, Pharmacology:
agnosis may be made with one or more components of drug usage for infections.
diagnostic process. Differential diagnosis requires use A. Bacterial infections (include common diseases and
of diagnostic components to rule out other possibilities. infective agents):
Dental hygiene diagnosis involves interpreting data into 1. Acne: multiple facial lesions because of plugged
coherent description of condition or disease in terms that sebaceous glands and hair follicles.
can be addressed by dental hygienist. a. MOST common in adolescence.
• See Chapter 11, Clinical Treatment: standards for clini- b. Involves heredity and hormonal factors; aggra-
cal dental hygiene practice, contraindications, ­treatment vated by stress and drugs.
planning. 2. Abscess: raised pocket of necrotic tissue and puru-
A. Patient history: medical, dental, lesion histories. lent exudate.
B. Clinical assessment: identification of lesion color, a. MOST are around trauma sites or associated
size, shape, location. with obstructed skin appendage on skin.
1. Sign: objective evidence of disease that can be dis- b. Also called boil (furuncle) on the skin (tooth-
cerned by clinician and/or patient. associated discussed later).
2. Symptoms: subjective information about disease 3. Impetigo: infectious pustules with itchy, yellow
by patient. scabs.
C. Lab assessment: includes microscopic analysis, blood a. MOST are around mouth and face.
tests, biopsy, radiographs (BEST for osseous and b. Highly contagious; can be either staphylococ-
periapical pathological conditions). cal or streptococcal infection.
D. Surgical assessment: visual examination of surgical B. Viral infections (include common diseases and infec-
site; therapeutic trial uses effective treatment to assist tive agents):
in diagnostic process. 1. Localized viral diseases: include warts, caused by
strains of human papillomavirus (HPV).
Prognosis, Treatment, and Care 2. Involve discrete proliferation of squamous epithe-
Prognosis is prediction of how condition or disease lium; may disappear spontaneously.
will progress and whether there is chance of recovery
or cure with treatment. However, treatment does NOT Metabolic Disorders
always involve cure of condition or disease. Palliative Metabolic disorders often manifest themselves as overpro-
care is any form of medical care or treatment that con- duction or underproduction of secretions. Hyper­secretion
centrates on reducing severity of disease symptoms, (overproduction) occurs because of a tumor of gland; hy-
rather than providing a cure; prevents and relieves suf- posecretion (underproduction) occurs because of agenesis
fering and improves quality of life for serious, complex (failure of an organ to develop), atrophy (partial or complete
illness. Nonhospice palliative care is NOT dependent wasting away), or destruction of secretory cells. Treatment
on prognosis and is offered in conjunction with curative for hypersecretion is removal or destruction of all or part
and all other appropriate forms of medical treatment. of gland; hormone supplementation is prescribed for hypo-
Hospice care delivers palliative care to those at the end secretion. Clinician will need a medical consult regarding
of life. metabolic disorder or its history before dental care.
• See Chapters 3, Anatomy, Biochemistry, and Physiology:
Inflammatory Diseases physiological processes; 8, Microbiology and Immunol-
Inflammatory diseases include several common forms ogy: infectious agents involved in hepatitis; 9, Pharmacol-
of dermatitis, acute or chronic inflammation of skin. ogy: hormones and replacement therapy; 9, Pharmacology,
Etiology can involve chemicals (drugs), radiation, and 11, Clinical Treatment: alcoholic patient, hyposecre-
trauma, metabolic and immunological factors. Types tion of salivary gland with geriatric patient.
include eczema (nonspecific lesions and itching), seb- A. Endocrine gland disorders:
orrheic dermatitis (inflammation of sebaceous glands, 1. Pituitary:
redness, itching, scaling), and psoriasis (raised papules a. Hyperpituitarism: gigantism and acromegaly
or patches with silvery scales and thickened skin on from overproduction of growth hormone (GH),
­extremities). caused primarily by pituitary adenoma; oral
170   Saunders Review of Dental Hygiene

­hypermetabolism, can result in death; vaso-


constrictors should NOT be used.
(5) No barriers to care; professional care and
homecare: thorough extraoral and intraoral
examination for increases in thyroid gland
and lymph node size; home and professional
fluoride to reduce incidence of caries as well
as calcium products; frequent examination
and oral prophylaxis to manage bone loss,
caries, periodontal infection.
(6) Patient or caregiver education: instruction
in dental biofilm removal methods to pre-
vent periodontitis; adequate diet to reduce
risk of osteoporosis; mineralizing fluoride
and calcium products.
c. Hypothyroidism:
(1) Associated with hyposecretion of thyroxine;
MOST common is Hashimoto’s thyroiditis,
autoimmune disorder that attacks gland; LESS
common are secondary forms (iodine therapy,
thyroid surgery, developmental disturbances,
Figure 6-1  Goiter formation from thyroid gland disorder. iodine deficiency [uncommon in United States,
(From Fehrenbach MJ, Herring SW: Illustrated anatomy of the head where salt is iodized], pituitary disease).
and neck, ed 3, St. Louis, 2008, Saunders/Elsevier.) (2) Also associated with cretinism, congeni­
tal lack of thyroxine; causes dwarfism with
signs: macroglossia and mandibular prognath­ stocky stature, protruding abdomen, under-
ism (enlarged jaw to continued bone growth). developed sex organs, misshapen face.
b. Hypopituitarism and panhypopituitarism: as- (3) In children: delayed development (physical
sociated with diabetes insipidus (sometimes and mental).
mistakenly confused with diabetes mellitus); (4) In adults: symmetrically enlarged thyroid
urine production is copious because of lack of gland, may have nodules; signs and symp-
antidiuretic hormone (ADH) from pituitary. toms include myxedema (fatigue and weight
2. Thyroid: gain from slow metabolism), skin dryness,
a. BOTH disorders can lead to goiter formation, hair coarseness, face puffiness, intolerance
thyroid gland enlargement (Figure 6-1). to cold temperatures, constipation, muscle
b. Hyperthyroidism: MAINLY associated with cramping, mental slowness, low blood pres-
Graves’ disease, autoimmune hypertrophy of sure, slow pulse; sometimes confused with
gland, with hypersecretion of thyroxine; causes Sjögren’s syndrome.
exophthalmos (protruding eyes), tachycar- (5) Oral signs: o0670
dia (increased pulse rate), severe weight loss, (a) In children: both enlarged tongue and
sweating, tremors, heat intolerance, frequent lips, with delayed tooth eruption and
bowel movements: possible enamel hypoplasia.
(1) In children: premature primary exfoliation, (b) In adults: enlarged tongue; chronic, se-
early eruption of permanents. vere periodontal disease; slow, hoarse
(2) In adults: aggressive periodontal disease, speech; possibly gingival hyperplasia.
dental caries, osteoporosis of jaws, burning (6) Treatment: thyroid hormone supplementa-
tongue, increased salivation. tion (e.g., with levothyroxine [Synthroid,
(3) Treatment: radioactive iodine (MOST com- Levoxyl, Levothroid, Unithroid]).
mon) or antithyroids, such as methimazole (7) No significant risk factors; professional
(Tapazole) or propylthiouracil (PTU); pos- care and homecare: thorough extraoral and
sibly partial surgical gland removal. intraoral inspection for increases in thyroid
(4) Risk factors: thyroid storm (thyrotoxi- gland or lymph node size, frequent oral ex-
cosis) if undiagnosed or uncontrolled; amination and recare.
emergency with fever, elevated blood (8) Patient or caregiver education: instruction
pre­ssure, heart dysrhythmia, tachycardia, in thorough dental biofilm control and need
General and Oral Pathology   171

for frequent recare to reduce incidence of (4) Daily fluoride and calcium product supple-
periodontal destruction. mentation as needed.
3. Adrenal: (5) Dealing with possible intoxication and dif-
a. Hyperadrenalism: Cushing’s syndrome with hy- ficulty in keeping appointments.
persecretion of glucocorticoid hormones; causes h. Patient or caregiver education: need for frequent
obesity, striae, easy bruising, red face, pathologi- recall, fluoride and calcium supplementation,
cal fractures, hyperglycemia, hypertension; drug saliva substitutes, good homecare practices be-
therapy (corticosteroids), which can result in cause of risk of infection.
“moon face,” “buffalo hump,” and osteoporosis. 2. Viral hepatitis: caused by several viruses known as
b. Hypoadrenalism: Addison’s disease with hy- hepatitis A, B, C, D, or E (see Chapter 8, Microbi-
posecretion of adrenal cortex (NOT medulla) ology: related discussion).
hormones; causes dehydration, electrolyte im- a. MOST prevalent liver disease in world; vac-
balances, hypotension (low blood pressure), cines for hepatitis A and B.
weight loss, pigmented skin and oral mucosa; b. Systemic signs: anorexia,
��������������������������������
malaise, jaundice, en-
orthostatic hypotension risk. larged liver, high levels of aspartate aminotrans-
4. Parathyroid: hyperparathyroidism that causes ra- ferase (AST, enzyme released into blood after
diolucencies in mandible and mobile teeth. liver injury), dark urine (contains bilirubin).
B. Liver disorders: c. Types and pathogenesis:
1. Cirrhosis: chronic destruction of liver cells with (1) Hepatitis A (HAV):
fibrous, nodular regeneration. (a) Transmission: fecal-oral route (con-
a. Etiology: drugs, alcohol, viral infections; may taminated food and water and sewage),
be unknown. includes shellfish.
b. Systemic signs: jaundice (yellow skin, sclera), (b) Incubation period: 15 to 50 days, with
puffy eyes, facial redness, hepatic encephalopa- short-lived, slight flulike symptoms, rapid
thy, ascites, splenomegaly, bleeding. recovery, NO long-term consequences.
c. Oral signs: xerostomia, reduced ability to taste, (2) Hepatitis B (HB): dental healthcare per­sonnel
glossitis, increased risk of caries, periodontal (DHCP) are at highest risk for contracting.
disease, oral cancer, alcohol breath, enlarge- (a) Transmission: exposure to contami-
ment of parotid gland, facial and dental trauma nated blood and blood products or by
from falls and injuries. sexual contact; at risk for contracting
d. Treatment for alcoholics: alcohol abstinence; hepatitis D.
occasionally involves surgical operations to re- (b) Incubation period: 40 to 180 days;
lieve portal circulatory disruptions. MOST have clinically unrecognizable
e. Risk factors: nutritional deficiencies, infec- symptoms; MOST recover fully.
tions, trauma, oral cancer (especially if tobacco (3) Hepatitis C:
is also used); relative contraindication when ad- (a) Transmission: exposure to contami-
ministering amide local anesthetics (lowering nated blood and blood products or by
amount) and nitrous oxide sedation (metabo- sexual contact.
lism difficulties). (b) Incubation period: 15 to 150 days; symp-
f. Barriers to care: toms are SAME as HB but LESS severe;
(1) Communication difficulties; may not however, progresses to chronic hepatitis
­appear for appointments when actively in HALF of all infected persons.
­drinking. (4) Hepatitis D:
(2) Transportation problems, if the alcoholic (a) Transmission: exposure to contami-
does not drive or has lost driving privileges nated blood and blood products or by
and must rely on others. sexual contact.
(3) Economic problems, if unable to hold a job (b) Incubation period: 30 to 50 days; virus
or on a fixed income. depends on HB virus (HBV) for its rep-
g. Professional care and homecare: lication; infection with BOTH viruses
o0910 (1) Need for frequent oral cancer evaluations. occurs at same time (thus considered a
o0890 (2) Nonalcoholic mouthrinses (Rembrandt, “piggy-back” virus).
Biotene). (5) Hepatitis E:
o0900 (3) Possible bleeding problems from damaged (a) Transmission: fecal-oral route, contam-
liver, need INR (international normalized inated food and water and sewage (not
ratio) number. shellfish like hepatitis A).
172   Saunders Review of Dental Hygiene

(b) Incubation period: 14 to 60 days and slight Box 6-1  Monitoring Levels of Blood Glucose
or asymptomatic course; no long-term
consequences in MOST cases (EX-
<70 mg/dL: Too low; tendency toward hypoglycemia. Take
CEPT liver necrosis develops in many 15 mg of carbohydrates (glucose source) and wait
acutely infected pregnant women). 15 minutes. If this level continues, need for immediate
d. Treatment: palliative. medical consult. Risk for emergency.
C. Pancreatic disorders: 70-99 mg/dL: Normal levels.
1. Diabetes mellitus (DM, hyperglycemia): multi-
factorial disease leading to high blood glucose 100-125 mg/dL: Prediabetes; recommend medical consult.
­levels. 126-240 mg/dL: Higher levels; monitor infection level, insu-
a. Types and pathogenesis: can be present for lin intake, stress level, food level.
10 to 20 years before diagnosis. >240 mg/dL: Too high; tendency toward hyperglycemia. If
(1) Prediabetes: blood glucose levels are this level continues, need for immediate medical consult.
higher than normal but NOT yet at the high Risk for emergency.
levels of type 2; includes BOTH impaired
glucose toleration and impaired fasting Levels can vary per person and situation (before and after meals and
­medications); only average levels noted here.
glucose.
(2) Type 1 (insulin-dependent diabetes mellitus ­parotid gland enlargement, increased slight car-
[IDDM], juvenile onset): ies rate from dry mouth, as well as increased
(a) MORE serious type; autoantibodies moderate to severe periodontal disease, tooth
­ag­ainst insulin produce pancreatic beta loss and mobility.
cells so pancreas fails to secrete ad- d. Systemic complications: related neural, visual,
equate insulin. and kidney disorder because of fibrosis of blood
(b) Many serious complications over time. vessels.
(c) Requires daily insulin injections. e. Diagnostic and monitoring tests (Box 6-1):
(3) Type 2 (non-insulin-dependent diabetes (1) Self-monitoring blood glucose (SMBG): o1390
mellitus [NIDDM], mature onset): recommended for ALL types; MORE fre-
(a) LESS serious type; target cells fail to quently required for those on intensive in-
respond to insulin. sulin therapy or pregnant, having hard time
(b) May have many serious complications controlling levels, with increased ketones
over time, MORE in females; associ- in urine, or with intercurrent illness.
ated with obesity. (2) Glycosylated hemoglobin (GHb): measures o1380
(c) May be controlled by dietary changes, how much glucose attaches to hemoglobin
but may need drug therapy, including (Hb) on red blood cells (RBCs) (HbA1c or
insulin injections. A1c test); reflects average blood glucose
(4) Latent autoimmune diabetes of adulthood levels over 3 to 4 months; > 9% to 20% in-
(LADA, type 1.5): slowly progressive form dicates poor control and the need to reassess
of type 1 DM, often diagnosed as type 2 the present treatment.
DM, but positive for pancreatic islet anti- (3) Fasting plasma glucose (FPG): measures o1370
bodies, especially to glutamic acid decar- blood glucose levels after fast; ≥126 mg/dL
boxylase (GAD); does not immediately is diagnostic; casual (nonfasting) of ≥200
require insulin for treatment; patients often mg/dL with classic symptoms is diagnostic but
not overweight, have little or no resistance is confirmed on separate day; impaired glucose
to insulin UNLIKE type 2. metabolism (prediabetes) has <126 mg/dL.
(5) Gestational (pregnancy); usually does NOT (4) Blood urea nitrogen (BUN) and serum o1330
continue after birth of child. ­creatinine: to determine any related renal
b. Signs and symptoms: polyuria (increased uri- damage.
nation), glycosuria (sugar in urine), polypha- (5) pH levels: to determine severity of diabetic o1340
gia (increased hunger), polydipsia (increased ketoacidosis.
thirst), increased amount of ketone bodies in (6) Serum electrolytes: to determine biochemi- o1350
the blood, acidosis (increased acidity of blood cal imbalances that interfere with muscle
plasma), weight loss, poor wound healing, risk and cardiovascular system (CVS) function.
for infections. (7) Oral glucose tolerance test (OGTT): for diag- o1360
c. Oral complications (ALL at increased risk nosing gestational (pregnancy) type ­diabetes.
with poorly controlled blood sugar): Candida f. Treatment (See Chapter 9, Pharmacology: drug
albicans infections, xerostomia (dry mouth), therapy for DM):
General and Oral Pathology   173

g. Risk factors: j. Patient or caregiver education:


(1) Disturbance in balance between glu- (1) MUST understand that controlling oral dis-
cose and insulin occurs when diabetes is ease means controlling blood sugar levels
­uncontrolled, which can lead to �����������
poor wound and overall need for drug therapy and even
healing, which in turn can increase the risk reduced complications.
of bacterial, viral, and fungal ­infections. (2) Preventive oral health procedures and need o1530
(2) Local anesthetics with vasoconstrictors are for frequent recall; MUST control any oral
recommended for ALL but the most com- infections.
plicated procedures. (3) Daily mineralizing fluoride and calcium
h. Barriers to care: NO greater than others unless product supplementation.
complications exist. 2. Pancreatitis: inflammation of gland.
i. Professional care and homecare: a. Etiology: protein- and lipid-digesting enzymes
o9000 (1) Thoroughly assessing oral health, as well as in pancreas that digest organ itself; severe cases
monitoring diabetic control; including use of are associated with alcoholism.
diabetes risk assessment test kit (BIOSAFE), (1) Acute: causes severe abdominal pain with
FDA-approved home use or professional use sudden onset, nausea, vomiting, elevated
diabetic test kit (includes immediate FPG and serum levels of amylase and lipase, rapidly
HbA1c test from self-collected finger-prick developing shock, pancreatic enzymes in
blood sent to lab, with patient consent form; urine.
professional lab report is faxed to the dental (2) Chronic: causes severe pain, malabsorp­
office and paper mailed to the patient). tion, DM.
o1500 (2) Antibiotic premedication may be advised in b. Treatment: acute centers on controlling sys-
cases of uncontrolled diabetes or extensive temic consequences of shock; chronic centers
infection before dental care (determined on treating symptoms.
during medical consult). 3. Pancreatic cancer: early diagnosis difficult because
o1470 (3) Scheduling short dental appointments soon of nonspecific and varied symptoms; MOST not
after morning meals (1½ hours). diagnosed until advanced; prognosis is generally
o1490 (4) Preparing for diabetic emergency (glucose regarded as poor; MAINLY in males, smokers,
source MUST be available in emergency kit). persons with periodontal ­disease.

clinical study  

Age 37 YRS Scenario

Sex ☐  Male   ☒  Female A patient new to the dental practice


is at the end of the 2-hour afternoon
Height 5’10” ­appointment. She starts complaining
Weight 210 LBS of a headache, reports feeling weak,
and becomes more nervous and
BP 112/68 ­somewhat confused.

Chief Complaint “I was so nervous about this dental appointment


that I forgot to eat lunch today.”

Medical History Diabetes mellitus (well controlled) for 2 years


Osteopenia for 1 year

Current Medications 250 mg chlorpropamide (Diabinese) qd


OTC calcium supplement with vitamin D qd

Social History Interior decorator

1. What type of diabetes does the patient have? What 3. What emergency has begun to occur during this
type of drug is she taking? When should it be taken? ­appointment?
2. What is the American Society of Anesthesiologists 4. Describe the steps that should be followed to manage
(ASA) classification of this patient at the beginning of this situation.
the appointment? 5. How could this emergency have been avoided?
174   Saunders Review of Dental Hygiene

1. Patient has type 2 diabetes mellitus (DM), most often A. Anemias: low hemoglobin and oxygen concentration
controlled by diet and exercise; in some more mod- in blood.
erately involved cases by oral hypoglycemic drug(s) 1. Etiology: excess bleeding, hemolysis, iron or other
such as chlorpropamide (Diabinese); even by insu- ­nutrient deficiency, improper formation of new RBCs.
lin in severe cases. DM occurs when body does not 2. Forms and pathogenesis:
make enough insulin (type 1) or when insulin that is a. Pernicious (B12 deficiency, type of megalo-
produced no longer works properly (type 2). Insulin blastic anemia): too few RBCs; factor for RBC
works by helping sugar get inside the body’s cells, formation is missing; treated with injections of
where it is used for energy. Physician will advise the missing intrinsic factor.
patient of the dosage schedule depending on the time b. Sickle-cell (sickle-cell disease): genetic hemo- o1780
of onset of the particular insulin used to achieve best lytic anemia; usually confined to African-
control over blood sugar levels. Americans; RBCs are sickle (crescent) shaped
2. Physical status shows that mild systemic disease such and CANNOT function properly; lower level of
as type 2 DM equates to an ASA II classification. The anemia present with sickle-cell trait (person has
ASA physical status classification determines risks ONLY one copy of mutation).
and necessary modifications before dental treatment. c. Hypochromic (iron deficiency anemia): too o1760
3. Emergency that has occurred is hypoglycemia (low little hemoglobin per RBC; treated with iron
blood sugar); she is now at ASA IV level. Condition is supplementation.
most often associated with lack of food intake; in this d. Hemolytic: rupture of RBCs; may be genetic or o1770
case she missed lunch because of anxiety associated environmental; NO effective treatment for genetic
with dental appointment. Headache, weakness, ner- forms; treatment for environmentally induced
vousness, confusion are symptoms of hypoglycemia. forms centers on control of environmental factors.
Condition typically occurs rapidly for patients who re- e. Aplastic: faulty bone marrow; treated with
quire insulin; onset of symptoms is slower for patients transfusion or stem cells from bone marrow
taking oral hypoglycemics. transplantation.
4. Dental treatment must be stopped, and patient must f. Secondary: from another disease; cell numbers
be positioned upright. Source of glucose (orange juice are normal but hemoglobin level is low.
or other glucose source such as glucose tablets or gel) 3. Oral signs: bleeding oral tissues, angular cheilitis,
should be given if patient is conscious. Source of glu- pale skin and oral mucosa, erythematous burning
cose should be stored in easily accessible dental office tongue with loss of BOTH filiform and fungiform
emergency kit. If situation is allowed to progress, pa- lingual papillae.
tient may become unconscious, in which case IM or IV 4. Risk factors: excessive bleeding and trauma.
administration of glucagon by the supervising dentist is 5. Barriers to care: nonexistent; in severe cases fa-
indicated, if available, after EMS system is activated. tigue may keep patient from appointments.
5. Both lack of lunch and stress related to appointment 6. Professional care and homecare: observation of signs
contributed to occurrence of hypoglycemia. It is es- and symptoms of anemia and appropriate referral;
sential to explain all dental procedures to this patient with sickle-cell anemia, needs medical consult for
to alleviate anxiety about future appointments and en- possible antibiotic premedication before dental care.
courage eating before the appointment; use of morn- 7. Patient or caregiver education: appropriate care of
ing appointments after she eats a good breakfast and anemia by following physician’s orders and practic-
takes her prescription drugs is an effective way to deal ing good oral hygiene to decrease risk of infection.
with problem. May also want her to have a prescrip- B. Polycythemia: overproduction of circulating RBCs.
tion to ease her dental anxiety (diazepam [Valium]) 1. Primary: hyperactive bone marrow and elevated
and help reduce risk of hypoglycemia. hematocrit, blood viscosity, blood pressure, car-
diac workload; treatment involves bloodletting
Blood Disorders and radiation therapy to reduce RBC numbers.
Basic categories of blood disorders include red (RBC) and 2. Secondary: associated with need to compensate for
white (WBC) blood cell disorders and clotting disorders. inadequate blood oxygen levels caused by heart
They are related to abnormal levels, function, or structure of disease, emphysema, living at high altitude, or in-
some blood components. Note that hemophilia is discussed creased oxygen demand by muscles (in athletes);
under inherited disorders. Will need a medical ­consult for treated by controlling symptoms.
ALL types of blood disorders before dental care. C. Thrombocytopenia: FEWER platelets (thrombocytes)
• See Chapters 3, Anatomy, Biochemistry, and Physi- in blood with SAME clinical signs as anemia; can be
ology; 8, Microbiology and Immunology: vascular caused by decreased production in bone marrow, in-
­system and immunology. creased destruction, disease, or as a side effect of drugs.
General and Oral Pathology   175

D. Diseases of WBCs and bone marrow: (2) Chronic: excessive buildup of relatively


1. Agranulocytosis: severe reduction in number of mature, but still abnormal, WBCs that takes
granulocytes (basophils, eosinophils, neutrophils); months and years to progress.
results in greatly reduced immune system. c. Systemic signs: pallor (pale color) of skin and
2. Leukopenia: abnormally low levels of WBCs. lining mucosa, bone and joint pain, abnormal
3. Neutropenia: abnormally low levels of neutrophils bleeding that leads to bruising, enlarged lymph
(granulocytic WBCs): nodes, hepatomegaly, splenomegaly, anemia.
a. Etiology: congenital (discussed later), immuno- d. Oral signs: red, tender gingival tissues, spon-
logical, chemicals (drugs) or radiation therapy taneous sulcular bleeding, petechiae and
that suppresses bone marrow; acute disease is ­ecchymosis, risk for infections.
adverse reaction to toxic levels of drug or to e. Treatment: chemotherapy.
bone marrow–suppressing radiation therapy. 5. Multiple myeloma (MM): cancer of plasma cells,
b. Systemic signs: lymphadenopathy; fever and which are immune system cells in bone marrow
chills; prostration; jaundice; bleeding ulcers of that produce antibodies and immunoglobulins
rectum, oral cavity, vagina; proper identifica- (MAINLY IgG).
tion of disease is by blood test; WBC counts a. Etiology: unknown, does not have hereditary
drop to one fifth to one tenth. basis, but family members may have increased
c. Measured by absolute neutrophil count (ANC), risk.
which would be <1800/mm3. b. Systemic signs: elevated calcium level, renal
d. Oral signs: infections, painful ulcerations, bleeding failure, anemia, bone lesions (skull radiograph
gingival lesions with rapid periodontal destruction. shows “punched-out” radiolucencies), with bone
e. Infections can be fatal, so must be treated pain and increased risk of infection (Figure 6-2).
quickly; invasive oral procedures, including c. Treatment: includes bisphosphonates to treat
oral prophylaxis, should NOT be performed. fractures and erythropoietin to treat anemia, as
f. Treatment: transfusions to restore WBC counts, well as chemotherapy and transplants.
antibiotics to control infections. 6. Burkitt’s (lymphoma): cancer of B lymphocytes;
4. Leukemia: cancer of red bone marrow, lymphatic one type associated with HIV and mononucleosis
tissue, or WBCs. (Epstein-Barr virus [EBV]).
a. Etiology: largely unknown, although radiation, 7. Langerhans cell histiocytosis X (eosinophilic
genetics, viruses implicated. granuloma, Hand-Schüller-Christian disease): rare
b. Types and pathogenesis (Table 6-1): clonal proliferation of Langerhans cells, abnormal
(1) Acute: rapid increase of immature WBCs cells from bone marrow, capable of migrating from
that crowd out normal cells. skin to lymph nodes; affects children.

clinical study  

Age 12 YRS SCENARIO

Sex ☐  Male   ☒  Female During her initial intraoral examina-


tion, gingival bleeding, poor oral
Chief Complaint “My grandmother is so scared since my gums are hygiene, three carious lesions, and two
bleeding so much!” nearly exfoliated teeth are observed.
Medical History Recent diagnosis of leukemia with intensive che- Physician wants oral cavity checked for
motherapy scheduled in 1 week signs of infection.

Current Medications None at this time

Social History Grade school student studying ballet


Grandmother is guardian

1. What information must be obtained from the pa- 3. When during the several months of chemotherapy
tient’s oncologist before any dental treatment can be should dental treatment be scheduled?
given?
2. What dental procedures should be completed before 1. Before dental treatment, oncologist should be con-
chemotherapy? sulted regarding blood tests (WBC count), clotting and
176   Saunders Review of Dental Hygiene

Table 6-1  Comparison of leukemia types

Type Occurrence 5-Year survival rate Overall treatment

Acute lymphocytic Most in young children; also affects 85% in children; 50% Bone marrow and systemic
leukemia (ALL)* adults, especially those ≥65 in adults disease control, prevention
of spreading, e.g., to CNS
Chronic lymphocytic Most in adults >55; sometimes in 75% Incurable
leukemia (CLL)* younger adults, but almost never
children; most are men
Acute myelogenous More in adults than in children; 40% Bone marrow and systemic
leukemia (AML)† most are men ­disease control, specific
treatment of CNS, if in-
volved
Chronic myelogenous Most in adults; small number of 90%
leukemia (CML)† children
CNS, Central nervous system.
*Also known as lymphoblastic.
†Also known as myeloid or nonlymphocytic.

3. Dental treatment can be scheduled whenever blood


counts have returned to safe levels. Safe levels are
most common just before scheduled round of che-
motherapy. Bleeding levels monitored by INR; INR
≤2.5 is safe for invasive dental work; older test is pro-
thrombin time (PT). Antibiotic premedication will be
needed before invasive dental procedures if catheter
placement is permanent.

Bone Diseases
Diseases of the bone include osteoporosis, Paget’s dis-
ease, fibrous dysplasia, cherubism, cleidocranial dysos-
tosis (note that latter two are discussed under inherited
disorders).
A. Osteoporosis: MOST common bone disease.
1. MORE common in women and with age; other
factors include low levels of sex hormones (e.g.,
menopause), low body weight, ethnicity (e.g., white
or Asian), family history, low ­dietary intake of cal-
cium and/or vitamin D, smoking, alcoholism.
2. Drugs such as anticonvulsants or glucocorticoids
can also increase risk.
Figure 6-2  Skull radiograph of multiple myeloma showing 3. Reduced bone quantity and quality; loss in bone
“punched-out” radiolucencies. mass and decrease in bone density (i.e., increased
porosity) cause the bone to weaken.
bleeding factors (INR), placement of central venous 4. EARLY loss of bone density with osteopenia; as ��� o2240
catheter, when chemotherapy will begin, expected disease progresses, bone becomes MORE frag-
length of immunosuppression. ile and even slightest trauma can cause fracture
2. If blood counts are adequate and time is available (e.g., hip fracture); resulting spinal compression
before chemotherapy, all loose teeth and teeth close ­fractures may result in loss of height with stooped
to exfoliation (shedding) should be removed, carious posture (dowager’s hump).��
teeth should be restored, oral hygiene instructions and 5. May be related to oral bone loss with periodontal o2270
need for meticulous oral hygiene should be stressed, disease, since quality and quantity of the jaws ap-
and oral prophylaxis should be completed. Since this pear to be affected.
is a child patient, it is important that her guardian be 6. Diagnosis: dual-energy x-ray absorptiometry o2280
advised of all that is needed to maintain oral and re- (DXA) measures density of spine, hip, or total
lated systemic health. body (gold standard).
General and Oral Pathology   177

o2290 7. Treatment: diet and exercise; also selective estro- C. Classification:


gen receptor modulators (SERMs) such as raloxi- 1. Benign neoplasia (suffix “-oma” added after tis-
fene (Evista). See notes on bisphosphonates and sue type [e.g., osteoma]):
osteoradionecrosis complications. a. Single, discrete mass; solid organs or connec-
B. Paget’s disease (osteitis deformans): second MOST tive tissue, with formation of capsule around
common bone disease after osteoporosis. mass.
o2310 1. Systemic signs: BOTH destruction and formation of b. Close resemblance to tissue of origin; mature
bone in patients >50 years of age; ��������������
often asympto- cells with minimal to moderate cellular atypia.
matic; patients may have blindness (closure of neural c. However, effects of benign tumors are NOT al-
foramina) or deafness; MOST striking is progressive ways so safe:
enlargement of spine, femur, and skull, MAINLY (1) With endocrine growth, may cause uncon-
the maxilla, with tendency for bone fracture.�� trolled hormone hypersecretion.
2. Oral signs: teeth may loosen and migrate with (2) With cellular growth, may compress adjacent
jaw expansion; dentures NO longer fit (NOR do tissue (brain, spinal cord) or cause blockage
hats). of lumen (trachea, intestines), causing fur-
3. Radiographs: “cotton-wool” appearance, with ther serious or even life-­threatening compli-
patchy opacities, root resorption with loss of lam- cations.
ina dura, enlarged maxilla. (3) Examples:
o2360 4. Diagnosis: elevated serum alkaline phosphatase (a) Angioma: tumor composed of blood
level. vessels (birthmark).
o2370 5. Treatment: none. (b) Lipoma: soft, fatty tumor that develops
C. Fibrous dysplasia: abnormal proliferation of fibrous in adipose tissue.
connective tissue; COMMON in children; two ����������
types: (c) Adenoma: tumor of glandular tissue.
1. Monostotic: ONLY one bone involved. (4) Treatment: includes surgical removal and
o2410 2. Polyostotic (McCune-Albright syndrome): MOST radiation therapy.
severe form, affecting many bones, with “café-au-lait 2. Malignant neoplasia (cancer) (“carcinoma” added
spots” (localized skin pigmentation). to tissue type for epithelial tissue [e.g., squamous
o2420 3. Oral signs: painless swelling of jaw; teeth may be cell carcinoma] and “sarcoma” for nonepithelial
displaced. tissue [e.g., osteosarcoma]):
o2430 4. Radiographs: unilocular or multilocular radiolu- a. Cells are neoplastic (new growth); capacity ex-
cency, with “ground glass” appearance because of ists for invasion (going deeper in tissues) or me-
many irregular C-shaped trabeculae. tastasis (spread beyond original site).
b. Frequency and significance:
Neoplasia (1) In both sexes, MOST affected are lungs,
p0140 Neoplasm is uncontrolled cellular growth resulting in a digestive organs (including pancreas), sex
tumor; general term, includes either benign or malignant organs.
(cancer). Will need medical consult for ALL with neo- (2) In females, breast cancer is second MOST
plasm history before dental care. See������������������������
later discussion of common; in males, prostate cancer.
biopsy procedures and lung cancer. Treatment after radia- c. TNM classification of malignant tumors: can-
tion therapy and chemotherapy for patients in the dental cer staging system that allows determination of
office is also discussed later. prognosis and treatment (T = size of tumor, N =
• See also Chapter 9, Pharmacology: chemotherapy. nodes involved, M = metastasis).
A. Etiology: exposure to adequate concentrations of car- d. Treatment (may be combined during therapy):� o2660
����������������������������������
cinogens (cancer-causing agents) that alter cellular includes surgical incision, chemotherapy, radia-
DNA and proteins: tion therapy, stem cells from the blood or bone
1. Exogenous factors: chemicals (including drugs), marrow transplantation, donor lymphocyte
radiation, viruses, other environmental factors. ­in­fusion (DLI); if treatment is ineffective in
2. Endogenous factors: genetic factors (genome of eliminating cancer, disease can be fatal.
cells), hormonal factors, immune system factors, e. Paraneoplastic syndromes (such as fever, an- o9010
nutritional factors, aging. orexia) occur with active cancer and those in
B. Tissues at risk for neoplastic change: tendency for remission after treatment; clinical pictures may
cells to undergo neoplasia is roughly related to cell’s vary greatly.
involvement in physiological replacement; MUST D. Vascular neoplasias:
have adequate time for initiation, transformation, 1. Hemangioma: benign vascular lesion that devel-
growth of mass. ops at birth or during early childhood.
178   Saunders Review of Dental Hygiene

a. Exophytic lesion that occurs on skin and rarely


in oral mucosa (tongue or buccal mucosa).
o9020 b. MOST are compressible and some are
­emptiable.
o2700 c. Treatment: may require surgical removal; bleed-
ing can be a problem.
2. Kaposi’s sarcoma (KS): malignant vascular lesion
MOST often associated with HIV/AIDS (see Fig-
ure 13-2).
a. Associated with CD4+ cell counts <200 cells/
mm3; AIDS diagnostic lesion.
b. Dark purple, flat, or exophytic, may be ulcer-
ated and bleeding; various intraoral (palate and
gingiva) and extraoral sites (see later discussion Figure 6-3  Ameloblastoma, aggressive benign jaw tumor,
with “soap bubble” appearance in posterior mandible.
of vascular lesions).
3. Treatment: surgery, radiation, chemotherapy.
E. Osseous neoplasias:
1. Osteoma: small benign tumor that can occur any-
where, including oral cavity.
2. Osteosarcoma: MOST common malignant osseous
neoplasia; poorly defined jaw radiolucency.
a. MOST in adults 30 to 40 years old; may cause
pain, swelling, paresthesia (numbness).
b. Treatment: chemotherapy followed by surgical
resection.
F. Muscle neoplasia:
1. Rhabdomyosarcoma: rare cancer that involves
skeletal muscle, including in the head and neck
area; MOST common in children.
G. Soft tissue neoplasias: MOST often benign and with- Figure 6-4  Squamous cell carcinoma of the floor of the
out serious pathogenesis; occur in variety of tissues, mouth.
including adipose (lipoma), muscle (myoma), blood,
lymphatic:
1. Fibroma (true): mesenchymal tumor associated I. Epithelial squamous cell neoplasias:
with excessive growth of connective tissues. 1. Papilloma: benign, pedunculated or sessile, white
a. Exophytic lesion with color matching surround- warty lesion associated with squamous cells.
ing tissues (unless traumatized); may occur a. MOST on palate; may be pedunculated (have a
on any oral tissue; MOST on buccal or labial stalk).
­mucosa. b. Treatment: surgical removal to rule out cancer.
b. When associated with gingiva and cells of the 2. Squamous cell carcinoma (SCC): malignancy as-
periodontal ligament, considered a peripheral sociated with squamous cells that invades through
odontogenic fibroma. the basement membrane; MOST common oral
2. Lipoma: small benign tumor of fatty tissue. cancer (90%) (Figure 6-4).
H. Odontogenic neoplasias: a. Etiology: chronic tobacco and/or alcohol use,
1. Ameloblastoma: aggressive benign epithelial tu- human papillomavirus (HPV); because of HPV,
mor associated with cells of enamel organ. may occur in younger group as well as those
a. MOST in posterior mandible; associated with ≥40 years; varies in appearance:
presence of unerupted teeth, painless swelling, (1) Skin and oral signs: red and/or white pap-
thinning periosteum with “egg shell” cracking ule, plaque, or ulcer that does NOT heal;
when palpated. pain and bleeding possible.
b. Radiographs: multilocular bone destruction with (2) Jaw signs: swelling, pain, numbness (pares-
“soap bubble” appearance; ameloblast-like cells thesia).
in biopsy (Figure 6-3). (3) Pharynx and larynx signs: dysphagia (dif-
c. Treatment: wide surgical removal, including ficulty swallowing), continued hoarseness,
jaw. lump (mass).
General and Oral Pathology   179

(4) Salivary gland signs: small, persistent uni- 5. Malignant melanoma: dark-colored skin malig-
lateral lump, little to moderate pain. nancy; occasionally occurs on masticatory oral
b. Location: MOST on lower lip, lateral border mucosa (maxillary alveolar ridge and palate).
or base of tongue, floor of mouth, soft palate a. Etiology: excessive exposure to sunlight; usu-
complex (including tonsillar region); MAINLY ally at site of preexisting nevus, but can be ini-
hidden areas the clinician should access thor- tial lesion; greater risk with family history.
oughly during an intraoral examination; how- b. Pathogenesis: grows aggressively, with wide-
ever, if HPV involved, can be present in ANY spread metastases; exhibits “ABCD” changes:
location. asymmetry, border, color, diameter; may have
o3030 c. Diagnostic dental office tests: brush biopsy bleeding or ulcerated surface.
(Oral CDx), ultraviolet (UV) light kit, toluidine c. Treatment: surgery, radiation, chemotherapy.
blue dye kit; exfoliative cytology (not recom- d. Prevention: avoidance of sun, use of sunscreen,
mended anymore); ALL noninvasive tests do routine examination of existing lesions.
NOT take place of surgical biopsy. J. Salivary gland neoplasms:
d. Pathogenesis: does NOT heal or show response 1. Pleomorphic adenoma: benign (MOST common type)
to palliative treatment; may metastasize to cer- with mixture of epithelial and connective tissue.
vical lymph nodes. a. Asymptomatic mass associated with major or mi-
e. Verrucous carcinoma: type of SCC that is slow nor glands of palate; MOST with parotid gland.
growing; associated with spit (smokeless) b. MOST in adults ≥40 years.
­tobacco. c. Treatment: surgical removal.
o3060 (1) Associated with buccal and labial mucosa 2. Adenoid cystic carcinoma: slow-growing malig-
(related to placement of tobacco); NO vi- nancy associated with major or minor gland.
sual difference from “chewer’s” lesions, so a. “Swiss cheese–like” appearance microscopically.
biopsy is necessary. b. Treatment: surgical removal.
(2) Pathogenesis: does NOT invade basement 3. Mucoepidermoid carcinoma: malignancy of epi-
membrane; unlikely to metastasize, thus thelial and mucous cells.
with prognosis better than other SCC. a. Slow-growing mass associated with palate and
f. Treatment: surgery, radiation, chemotherapy. parotid.
3. Basal cell carcinoma (BCC): noninvasive malig- b. Treatment: surgical removal.
nancy with increase in basal epithelial cells; asso- K. Breast cancer: second MOST common cancer in
ciated with sun exposure (actinic damage). women after lung cancer that starts in the cells of the
a. Pathogenesis: begins as small, elevated pap- breast.
ule and then becomes more fibrous and ulcer- 1. Adenocarcinoma that is classified by histological
ates centrally to become small cratered lesion appearance; presents as metastatic disease.
with indurated (hard) rolled borders on exposed 2. FIRST symptom is lump that feels different from
­areas. surrounding tissue; mammography or ultrasound
b. MOST in older adults; common on exposed tests for presence.
­areas, especially lower lip. 3. May secondarily involve unilateral supraclavicular
c. Usually does NOT undergo metastasis unless lymph nodes palpated during extraoral examina-
less common infiltrating type. tion, metastasis to jaws noted on radiographs as
d. Treatment: surgery, radiation, chemotherapy. either radiolucency or opacity.
4. Melanocytic nevus (plural, nevi) (mole): blue- 4. Treatment: surgery, radiation, chemotherapy.
brown pigmented benign lesion. L. Prostate cancer: second MOST common cancer in men
a. Pathogenesis: never changes shape (symmetri- after lung cancer, within gland of reproductive system;
cal), border, color, diameter. may cause pain, difficulty in urinating, erectile dys-
b. Treatment: may require a biopsy to rule out ma- function (ED); if undergoes metastasis to jaws, may be
lignancy, melanoma. noted on radiographs as either radiolucency or opacity.
180   Saunders Review of Dental Hygiene

Clinical study  

Age 50 YRS SCENARIO

Sex ☐  Male   ☒  Female The lesion noted on intraoral


­examination of the patient
Height 5����
’���
10” ­appears to be a preexisting mole
Weight 140 LBS that is 5 mm in diameter and has
a raised, irregular, and reddened
BP 105/70 border on its anterior surface.

Chief Complaint “My mole becomes sore and bleeds and I think it is
getting larger!”

Medical History Past history of repeated urinary tract infections


Gallbladder removed 5 years ago

Current Medications OTC ginseng qd

Social History High school teacher who spends many hours sunbath-
ing every summer and uses tanning booths in the win-
ter to maintain her tan, since she has very pale skin

1. What is a likely diagnosis of the patient’s condition? OTC drug, since ginseng can promote bleeding and
2. Which of the usual signs and predisposing factors for delay clot formation. Prevention includes reducing ex-
this condition does she present? posure to sun, using sunscreen, routine examination of
3. How would this condition treated? What could the pa- existing lesions.
tient have done to prevent this condition?
Biopsy Procedures
1. Malignant melanoma, which is a malignancy of skin Surgical biopsy is a definitive diagnostic technique for p0160
that occasionally occurs on masticatory oral mucosa (at- many diseases and conditions such as neoplasia. A tissue
tached gingiva and hard palate); most occur on the skin sample surgically taken from the patient is microscopi-
at a site of existing mole (nevus) but can occur as ini- cally examined and evaluated for structure and cell type.
tial lesion. Grows aggressively; dark in color; exhibits See also earlier discussion of neoplasia.
changes ����������������
(ABCDs) ��������
such as asymmetry, border, color, or A. Indications:
diameter. Caused by excessive exposure to sunlight as is 1. Any unusual or suspected lesion that CANNOT be
shown by her lesion, which is on an exposed portion of clinically identified with certainty or has persisted
her skin. Nevus, by contrast, is benign tumor of melanin for more than 2 weeks without healing.
and does not show any change in manner as this lesion 2. Any surgically excised tissue should be routinely
does. evaluated.
2. Asymmetry of lesion (flat and raised portions); irregu- B. Types:
lar borders; variation in color; relatively large diam- 1. Excisional: entire suspect lesion is removed and
eter; preexisting nevus; prolonged, frequent exposure microscopically examined.
to the sun and pale skin that burns easily. 2. Incisional: representative section of suspect lesion
3. Treated by surgery, radiation, chemotherapy; if sur- is removed and microscopically examined; used
gery, patient may want to let surgeon know about her for large lesions.
General and Oral Pathology   181

CLINICAL STUDY�  

Age 65 YRS SCENARIO

Sex ☒  Male   ☐  Female The patient indicates that he noticed the sore several
months ago. Intraoral examination reveals a 10-mm
Height 6’2” ulcerated red lesion located on the left lateral border of
Weight 195 LBS the tongue.

BP 112/72

Chief Complaint “This sore in my mouth really


hurts!”

Medical History Smokes a pack of cigarettes a day


Started smoking when he was 15
Consumes alcoholic beverages on
the weekends

Current Medications None

Social History Farmer


Volunteer firefighter

1. What additional information is required for a proper A. Localized oral and/or pharyngeal cancer concerns:
diagnosis for the patient? What is the most likely di- 1. Surgery:
agnosis of the lesion? a. Acute (short-term) complications: pain, fistula
2. Describe the histological features of this type of formation, speech alteration, nutritional defi-
­lesion. ciency, infection, and psychosocial impact.
3. Is this lesion likely to spread to other areas? b. Long-term complications: pain, disfigurement,
4. What risk factors are important in the formation of the malocclusion, temporomandibular disorder,
lesion? speech alteration, nutritional deficiency, infec-
tion, and psychosocial impact.
1. Surgical tissue biopsy is required for proper diagnosis. 2. Radiation therapy to the oral cavity and salivary
Surgical biopsy may confirm a squamous cell carci- glands:
noma (SCC) that is noted by its clinical signs. a. Acute (short-term) complications: oral muco-
2. Histologically, neoplastic squamous cells invade sitis (discussed later), loss of taste, xerostomia
through epithelial basement membrane. Thus this is (caused by salivary hypofunction), infection,
a carcinoma, since it involves the epithelium, unlike nutritional deficiency.
a sarcoma that involves nonepithelial tissues such as b. Long-term complications: continued xerosto-
connective tissue like bone. mia, caries from radiation, trismus, osteora-
3. The SCCs of oral cavity are most likely to metastasize dionecrosis (discussed later); altered growth
to lymph nodes of neck, cervical nodes, on the same and arrested tooth development occur in
side as the lesion. children.
4. Tobacco and alcohol use are two key risk factors associ- 3. Chemotherapy:
ated with this case, as well as age; it is important to note a. Acute (short-term) complications: mucositis
that patients �����������������������������������������
usually����������������������������������
smoke or drink almost double the (discussed later), myelosuppression, infection.
amount they report to their healthcare professionals. b. Long-term complications: NONE after chemo-
therapy is completed and acute complications
are resolved.
Cancer Patient Care B. Systemic cancer concerns (requiring chemotherapy
Cancer will develop in just less than half of Americans. and/or bone marrow transplantation, monoclonal an-
Oral cancer rates are increasing, and younger populations tibody therapy):
are being affected. Dental care is IMPORTANT part of 1. Chemotherapy for systemic malignancies: higher o3760
overall treatment protocols for cancer therapy. Will need risk for oral complications with leukemia, lym-
medical consult before dental care. phoma, solid tumors than in chemotherapy for
182   Saunders Review of Dental Hygiene

other malignancies (new therapies are using short- 2. Process is gradual and may take many months or
term higher dose therapy in some cases). years to appear.
2. Monoclonal antibody therapy by infusion (along 3. Surgery to the affected area (including oral sur-
with drugs, radioactive materials): made in labora- gery) may not allow healing.
tory from a single type of immune system cell to 4. Treatment: antibiotics and passibly hyperbaric
kill or stop cancer growth; complications same as oxygen therapy that produces new blood vessels in
with other treatments. the irradiated area and stimulates wound healing.
3. Blood cell or bone marrow transplant: stem cells E. Barriers to oral care:
(immature blood-forming cells) are transplanted by 1. Oncologists often recommend not brushing the teeth
infusion to replace cells that were destroyed during when blood counts are low and discourage dental
cancer treatment; for complications see below. visits to avoid the risk of infection (controversial).
4. Complications associated with intensive chemo- 2. Psychosocial concerns, complex treatment sched-
therapy and/or bone marrow transplantation: ules, fatigue disrupting dental recall schedules.
a. Cytotoxic drug effects include temporary de- F. Dental hygiene care:
struction of healthy tissues, including oral mu- 1. Before cancer treatment:
cosa (mucositis). a. Assess condition of teeth, periodontal struc-
b. Myelosuppression results in low WBC count, tures, oral hygiene, previous interest in dental
which increases risk for oral and systemic in- care, psychosocial factors.
fections; hemorrhage in response to low plate- b. If patient is scheduled for radiation therapy to
let counts and clotting factors may also occur; the jaw bones:
measured by ANC. (1) Teeth can NEVER be extracted safely from
c. Include temporary xerostomia, loss of appetite, irradiated bone.
nausea, pain, fatigue, graft-versus-host disease (2) Determine, with supervising dentist, which
in bone marrow transplant. teeth can be maintained.
C. Oral mucositis (OM): MOST common side effect of c. Oral hygiene education: dental biofilm control
radiation and/or chemotherapy; painful inflammation and gentle (nonalcoholic) mouthrinses.
and ulceration of mucous membranes lining digestive d. Nutrition and dietary counseling essential; to-
tract, including oral cavity. bacco cessation recommended.
1. Treatment is MAINLY palliative; palifermin (Kepiv- e. Mineralizing fluoride and calcium products:
ance), human keratinocyte growth factor (KGF) for (1) If undergoing radiation therapy to salivary
severe OM, has been shown to enhance healing; glands, needs daily prescription-strength
sucking crushed ice (cryotherapy) is alternative. fluoride gel in brush-on form or custom
2. Complications caused by ulcerations: fluoride trays, as well as calcium products,
a. Can become infected by virus, bacteria, or fun- for mineralization.
gus; may act as site for local infection and por- (2) With temporary xerostomia during chemo-
tal of entry for oral flora that, in some instances, therapy, may benefit from OTC mineraliz-
may cause septicemia (especially in immuno- ing fluoride and calcium products.
suppressed). f. Instrumentation SHOULD include scaling,
b. Pain and loss of taste perception make it MORE removal of overhangs, smoothing of rough
difficult to eat, which leads to increased weight ­­restorations.
loss. 2. During cancer treatment:
c. In approximately half with OM, it becomes so a. Design alternative dental biofilm control mea-
severe that it is dose limiting; planned cancer sures for those with OM and forbidden by on-
treatment must then be modified, thus compro- cologist to use toothbrush and floss.
mising prognosis. b. Counsel daily cleansing and denture removal
D. Osteoradionecrosis: bone (can include jaws, onj) before sleep.
that has died as complication of radiation therapy; c. Suggest measures to relieve xerostomia and topi-
damage to the small arteries reduces circulation, de- cal anesthetics or coating agents for OM; since xe-
priving bone of oxygen and other necessary nutrients. rostomia is not drug related, can recommend use
1. Increased risk in cancer patients who are receiv- of prescription cholinergic agonist drugs such as
ing high doses and longer therapy with IV (pos- pilocarpine (Salagen) and cevimeline (Evoxac).
sibly oral) bisphosphonates for repair of primary d. Assist with selection of nonirritating and non-
and metastatic bone lesions and have undergone cariogenic foods and tobacco cessation.
dentoalveolar trauma (such as with extractions or e. Encourage continued fluoride and calcium
periodontal disease). product application.
General and Oral Pathology   183

3. After cancer therapy: (5) Discuss nutritional care and provide to-


a. If received radiation therapy to salivary glands bacco cessation counseling.
and/or jaw bones: (6) Caution to avoid all future surgical proce-
(1) Encourage frequent recalls, scheduled ac- dures to irradiated bone.
cording to individual need, for prophylaxis b. If received chemotherapy and/or bone marrow
and homecare evaluation. transplantation:
(2) Encourage palliative measures for xerosto- (1) Monitor blood counts until recovers from
mia and stimulation of saliva or consider immunosuppression; place on regular den-
prescriptions (discussed earlier). tal hygiene recall.
(3) Reinforce need to continue daily fluoride (2) Encourage excellent oral hygiene.
and calcium product applications; encour- (3) Consider mineralizing fluoride and cal-
age excellent homecare. cium products until xerostomia is re-
(4) Recommend daily jaw exercises to prevent solved.
trismus. (4) Provide tobacco cessation counseling.

CLINICAL STUDY �  

Age 55 YRS Scenario

Sex ☒  Male   ☐  Female The patient has a well-fitting


maxillary partial denture. Teeth
Height 5’11” #6 through #11 have localized
Weight 222 LBS moderate chronic periodontitis.
and teeth #19 through #29 have
BP 120/80 localized severe chronic periodon-
titis. Dental biofilm level is moder-
Chief Complaint “What can I do to make sure my teeth survive this therapy?” ate, with heavy calculus and stain.
He brushes his teeth twice daily
Medical History Stage III squamous cell carcinoma of the retromolar pad and
and rinses with Listerine once
left lateral tongue, with metastasis to left side of the ­ neck
each day.
Scheduled to receive a 7000-cGy dose of radiation therapy
Drinks alcohol occasionally and smokes two packages of ciga-
rettes daily

Current Medications None

Social History Mission minister who has been overseas for 20 years

1. Identify five acute complications that this patient may hypofunction), osteoradionecrosis (nonhealing tis-
experience as result of radiation therapy. sues and bone), trismus that limits mouth opening,
2. Name common long-term complications associated temporomandibular joint pain, and rampant caries in
with radiation therapy to the head and neck. remaining teeth.
3. What dental and dental hygiene treatments should be 3. Before initiation of radiation therapy, all mandibular
completed before the initiation of radiation therapy? teeth should be extracted and bone and tissues should
4. Two years after radiation therapy the patient complains be allowed to heal; all remaining teeth should be
of pain in the left mandible and an open sore beneath thoroughly scaled as needed to remove deposits and
his denture. What may be the problem? promote healing. Maxillary denture must be cleaned
twice daily, and patient should be instructed on the
1. Five acute complications that the patient may experi- importance of meticulous homecare, including tooth-
ence as result of radiation therapy include thickened brushing and flossing. Custom fluoride trays should be
saliva (which causes dry mouth), alteration in taste fabricated and tobacco cessation encouraged. Patient
sensation, oral mucositis (OM, inflamed tissue), Can- should be encouraged to use another mouthrinse that
dida infection, muscle tightness (trismus). does not contain alcohol (Listerine does unless labeled
2. Common long-term complications associated with otherwise), although gentle mechanical removal of
radiation therapy to the head and neck include dental biofilm is the most important factor in keeping
xerostomia (dry mouth resulting from salivary his oral health.
184   Saunders Review of Dental Hygiene

4. The patient later complains of pain in the left mandible may have significant effects on many organs, includ-
and open sore beneath denture, most likely caused by ing heart, kidneys, blood vessels, brain:
denture irritation and osteoradionecrosis, respectively. 1. Essential (idiopathic, primary) with unknown eti-
Patient should be immediately referred to oral surgeon ology; secondary results from such diseases as re-
who is aware that patient has undergone radiation nal disease and damage and DM.
therapy to the area. 2. Treatment: drug therapy and environmental
­manipulation.
Diseases of Cardiovascular System D. Congenital heart disease: malformation of the heart and
Coronary heart disease, cerebrovascular disease, and blood vessels that occurs during fetal development:
hypertension are the three MOST common diseases of 1. Includes defects of valves, ventricular septum,
CVS. American College of Cardiology (ACC) and AHA atrial septum, and patent ductus arteriosus, dis-
suggest the evaluation of cardiac risk during dental care placement of the great vessels of the heart.
using major clinical predictors, rather than relying on 2. Etiology: heredity; infection with rubella or taking
time passed since cardiac event. They recommend deter- of drugs during pregnancy.
mining functional capacity (FC), which is measured in 3. May have difficulty with exertion, inhibited
metabolic (MET) equivalents such as ability to (1) climb growth.
flight of stairs; (2) walk up a hill; (3) run short distance 4. May require antibiotic premedication before inva-
(these activities that are all equal to 4 METs). Will need sive dental procedures.
a medical consult for ALL with history or presence of E. Cardiac dysrhythmia: irregular heartbeat caused by
cardiovascular disease (CVD) before dental care. See unreliable sinoatrial node, cardiac arrest, various dys-
CD-ROM for a WebLink to ACC/AHA 2007 guidelines rhythmias, heart blockage, or other disease:
on perioperative cardiovascular evaluation and care for 1. Treatment consists of drugs and/or implantation of
noncardiac surgery. pacemaker.
• See Chapters 8, Microbiology and Immunology: CVS 2. Antibiotic premedication during FIRST 6 months
infections; 9, Pharmacology: cardiac drugs; 10, Medi- after pacemaker implantation.
cal and Dental Emergencies: CVS emergencies, vital 3. NO contraindication to ultrasonic because of shield-
signs; 16, Special Needs Patient Care: stroke patient. ing (ALL are shielded, ONLY metal detectors are a
A. Coronary artery atherosclerosis (coronary heart dis- problem); AVOID electronic pulp testing.
ease [CHD]): F. Congestive heart failure (CHF): failure of diseased
1. Etiology: primary injury to the endothelium of heart to meet needs of the body; causes fluids to build
artery leads to deposition of platelets and serum up in body tissues:
lipoproteins and to proliferation of smooth muscle 1. Etiology: related to underlying heart valve dam-
cells in artery wall. age, ischemic heart disease.
o9050 2. May have anginal episodes; considered unstable or 2. Symptoms: shortness of breath with minimal exer-
severe if not relieved by rest. tion, fatigue, ankle and/or abdominal swelling.
o4360 3. Complications: plaque formation may lead to oc- 3. Causes of sudden death: dysrhythmias, pulmonary
clusion of coronary arteries, which may result in embolism, acute myocarditis, acute hypertensive
myocardial infarction (MI, heart attack); MI con- crisis.
sidered a MAJOR clinical predictor of increased G. Oral signs: no greater than for others unless patient is
perioperative CVD risk; acute if less than 7 days severely debilitated; certain drugs (calcium channel
previously and recent if more than 7 days but less blockers) are associated with gingival hyperplasia.
than 1 month. H. Risk factors: infection (rheumatic or congenital heart
4. Risk factors: sex, heredity, diet, hypertension, disease), side effects of drugs, vasoconstrictors (low-
smoking, obesity, DM. ered amounts), increased risk of emergency.
5. Treatment: drugs to lower serum lipid levels and I. Barriers to care: transportation difficulties when the
blood pressure, reduction of risk factors, bypass condition restricts mobility; may need to rely on
surgery, heart transplant. ­others; economic issues may be present if income is
o4370 6. Association of periodontal disease with established restricted because of disability or fixed income.
CHD, thus the emphasis on oral health to decrease J. Professional care risks are determined after medical
risk of further damage to the CVS. consult:
B. Cerebrovascular disease (CVD): atherosclerosis of 1. Antibiotic premedication may be needed before o9060
blood vessels in the brain; may result in cerebrovas- invasive dental procedures.
cular accident (CVA, stroke). 2. Withhold emergency or elective surgery (or peri- o9070
C. Hypertension (high blood pressure [HBP]): multifac- odontal procedures by dental hygienist) for 4 to
torial disease that involves heredity and environment; 6 weeks after MI and determine FC.
General and Oral Pathology   185

o9080 3. Dental care should be delayed unless patient can 2. Treatment: adrenergic agonists (beta2 agonists),
meet 4 METs capacity and/or further medical test- corticosteroid inhalers, methylxanthines, anticho-
ing has been completed to quantify level of cardiac linergics, cromolyn (Intal, Nasalcrom).
risk in treatment. B. Chronic irreversible airway obstruction (COPD): oc-
o9090 4. Use of stress reduction protocol that includes ad- curs with BOTH chronic bronchitis and emphysema.
equate pain control, pretreatment with anxiety 1. Bronchitis:
premedication, use of nitrous oxide sedation (see a. Acute: bronchitis: infection caused by viruses
Chapter 14, Pain Management). and bacteria; may last several days or weeks
o9100 5. Continued use of concurrent drug therapy, which and lead to chronic condition.
possibly includes statins, as well as antiplatelet b. Chronic: cigarette smoking or breathing fumes
therapy. and dusts causes chronic inflammation of air-
o9110 6. Caution when administering local anesthetics ways and excess sputum, with cough (pro-
because of possible interactions with drugs (see ductive, yellow or greenish mucus), dyspnea
Chapter 14, Pain Management). (shortness of breath), chest tightness.
o9120 7. Adjusting chair appropriately (more upright, 45°) 2. Emphysema: destruction of alveoli; airspace en-
because some are NOT able to recline in supine largement and airway collapse.
position. a. Etiology: MOST commonly cigarette smoking.
o9130 8. Long appointments should be avoided with CVD; b. Chronic cough, trouble breathing during exer-
blood pressure is lowest in the afternoon so ap- cise, hard to catch breath, barrel-chest appear-
pointments then or when patient is well rested are ance (caused by using accessory muscles for
preferred. respiration).
K. Patient or caregiver education: daily meticulous c. Dental management focuses on prevention of
homecare, frequent dental recall, need to take respiratory depression.
­antibiotic premedication before appointments and (1) More upright chair position may be neces-
cardiac drugs as prescribed. sary (45°).
(2) Relative contraindication for nitrous oxide
Respiratory Diseases sedation from respiratory depressive ef-
Asthma and chronic obstructive pulmonary disease (COPD) fects of high oxygen administration (NO
are significant respiratory diseases. Lung cancer and cystic contraindication to local anesthetic use);
fibrosis are also discussed. Will need medical consult with need medical consult.
respiratory disease or its history before dental care. (3) Rubber dam use may result in the feeling
• See Chapters 8, Microbiology and Immunology: tu- of compromised air exchange; provide low
berculosis; 9, Pharmacology: respiratory disorder and continuous flow of oxygen during treat-
disease drugs; 10, Medical and Dental Emergencies: ment.
respiratory emergencies in the dental setting. (4) Concurrent CVD may require special man-
A. Asthma: severe bronchoconstriction that results in agement.
airway obstruction. (5) Oral signs: drug related and may include
1. Dental complications: Candida infection and xerostomia.
a. Local anesthetics that contain vasoconstrictors d. Treatment: palliative and to prevent complica-
(sulfite preservatives, vasoconstrictor antioxi- tions; may include tobacco cessation, reduction
dants) or sulfite compounds, such as articaine of respiratory allergens and irritants, inhala-
(Septocaine, Zorcaine), should NOT be admin- tion therapy with bronchodilators (inhalers),
istered because of risk of allergic reactions. oxygen, drugs (adrenergic agonists, methylxan-
b. Can be sedated with nitrous oxide because of thines, corticosteroids, anticholinergics), possi-
increased levels of oxygen present and relax- bly surgery.
ation effects. C. Lung cancer: squamous cell carcinoma, adenocar-
c. May use inhaler for acute attacks; patient should cinoma, small cell carcinoma, or large cell undif-
be instructed to rinse the mouth thoroughly af- ferentiated carcinoma; either develops in lungs or
ter using and expectorate to avoid increasing metastasizes to lungs from lymph nodes or other sites
systemic absorption and reduce tooth staining (e.g., oral cavity).
and taste alteration; bring to appointments in 1. Etiology: associated with tobacco (cigarette, pipe
case of emergency need. smoking, spit [smokeless] tobacco) or other pollut-
d. Use acetaminophen (FIRST choice) as substi- ants (e.g., uranium, asbestos).
tute for its analgesic and antipyretic effects to 2. Symptoms include chronic coughing, coughing up
avoid aspirin-induced attack. blood, frequent pulmonary infections, Cushing’s
186   Saunders Review of Dental Hygiene

syndrome, clubbing of fingers, dyspnea, weight 5. Overall health and options for fatal prognosis of
loss. lung cancer and CF.
3. Treatment: tobacco cessation, surgical removal I. Patient or caregiver education: need for meticulous
of cancerous site, chemotherapy and/or radiation oral hygiene, daily mineralizing fluoride and calcium
therapy. supplementation as needed, saliva substitutes, in-
D. Cystic fibrosis (CF): autosomal recessive hereditary creased water intake for xerostomia.
disease that affects mainly lungs and digestive sys-
tem, causing progressive disability: Renal Disease
1. Thick mucus production and incompetent immune Renal disease is disease of the kidney, which alters filter-
system; results in frequent lung infections. ing ability. It is caused by infection, autoimmune disease,
2. Diminished secretion of pancreatic enzymes causes or developmental disorder. Patients have unusually high
poor growth, fatty diarrhea, fat-soluble vitamin ���
de- levels of toxic urine components in blood. As disease
ficiencies���. worsens, may need dialysis to clean the blood and pre-
3. Treatment: no cure; early death (twenties to thir- vent death. With renal disease or its history, will need a
ties); lung transplant as worsens. medical consult before dental care; may need antianxiety,
E. Oral signs of respiratory disease: xerostomia and can- antibiotic, or steroidal premedication.
didiasis from inhalants or other drugs, ulcerations, • See Chapter 3, Anatomy, Biochemistry, and Physiol-
lymphadenopathy. ogy, renal system.
F. Risk factors for respiratory disease: A. Oral signs:
1. Infections, especially with long-term steroid use. 1. Mucositis, candidiasis, hemorrhage, petechiae.
2. Exposure to asthma provokers, such as sulfites and 2. If oral hygiene is poor, increase in dental caries, o5180
stress. calculus development, periodontal disease.
3. Dyspnea breathing in supine position, may need 3. Halitosis, enamel hypocalcification. o5170
to sit MORE upright (especially during emergency B. Risk factors: infections, malnutrition, possible drug
care) at 45°. interactions, metabolism.
4. Persistent coughing. C. Barriers to care: economic; finances may be limited
5. Risk of lung infection with aspiration of aerosols because of high medical costs and inability to main-
from ultrasonics because of immunosuppression; tain employment.
aerosol use should be minimized; condition should D. Professional care and homecare:
be observed at all times. 1. Need to check vital signs and prevent drug interac-
6. Relative contraindications (need medical con- tions; do NOT use same arm as for shunt.
sult): 2. May need antianxiety, antibiotic, or steroidal pre-
a. Use of sedatives, antihistamines, aspirin (EX- medication because of shunt or organ transplant
CEPTION is asthma). (all determined with medical consult).
b. Tranquilizers, nitrous oxide sedation, general 3. Tendency for bleeding; dental treatment after di-
anesthetics (EXCEPTION is asthma). alysis and NOT before; NO treatment on day of
G. Barriers to care for respiratory diseases: dialysis because of drugs given (anticoagulants).
1. Lack of communication because of embarrass- 4. Possible interactions with local anesthetics (me-
ment about condition (e.g., tuberculosis, emphy- tabolism difficulties); may need reduced amounts.
sema). 5. Slow healing.
2. Transportation difficulties, if walking even short E. Patient or caregiver education:
distances is difficult. 1. Meticulous oral hygiene to reduce the incidence of
3. Weather may exacerbate certain respiratory con- infection.
ditions and may require rescheduling of appoint- 2. Frequent recall appointments for examination and
ments. prophylaxis.
H. Professional care and homecare for respiratory dis- 3. Daily mineralizing fluoride and calcium product
eases: supplementation.
1. Recognition of communicability of tuberculosis 4. Dentifrices with antigingivitis, anticaries, antical-
and need to avoid aerosol production. culus properties, unless mucosal irritation results.
2. Awareness of and ability to deal with respiratory
difficulties associated with asthma and COPD. Gastrointestinal Disease
3. Need for relaxed dental environment to prevent Gastrointestinal (GIT) diseases include gastroesophageal
asthmatic and other complications. reflux disease (GERD), ulcers. Both diagnosed by upper
4. Placement of drugs, especially bronchodilators, in gastrointestinal (GI) radiographic series (barium) and/or
close proximity. endoscope. May need a medical consult for all patients
General and Oral Pathology   187

with GIT disease if there are any questions before dental a­ ctivating pain regions or deactivating pain inhibitory
care. regions in brain, associated with maxillary and man-
• See Chapter 3, Anatomy, Biochemistry, and Physiol- dibular divisions of fifth cranial (trigeminal) nerve.
ogy: GIT system. 2. MOST in women ≥50 years.
A. GERD: mucosal damage produced by abnormal re- 3. Treatment: surgery reserved for severe conditions;
flux in esophagus: ice pack over area to numb it; drugs:
1. Caused by transient or permanent changes in bar- a. Anticonvulsants are MOST effective; pain re-
rier between esophagus and stomach. lievers usually do NOT help; may be poten-
2. Heartburn caused by acid in esophagus, burning tiated with adjuvant such as muscle relaxer
discomfort behind breastbone (sternum), cough, and antispastic agent baclofen (Kemstro and
hoarseness, voice changes, chronic earache, burn- Lioresal) or clonazepam (Klonopin); baclofen
ing chest pains, nausea, or sinusitis. may also help patient eat more normally if jaw
3. Complications: stricture formation, Barrett’s movement tends to aggravate symptoms.
esophagus, and esophageal ulcers; could even b. If anticonvulsants and surgical options have
lead to esophageal cancer, especially in adults >60 both failed or are ruled out as to use, pain
years old. may be treated long term with opioid such as
4. Treatment: physicians recommend lifestyle modi- methadone (Dolophine); low doses of some an-
fications (weight loss and diet change, sleeping tidepressants can be effective in treating neuro-
on left side and elevating head) in addition to or pathic pain; may try Botox injections.
instead of H2-receptor antagonists and/or proton B. Bell’s palsy (facial palsy): temporary unilateral facial
pump inhibitors. paralysis, with drooping on affected half.
B. Peptic ulcer disease (PUD): sore on lining of stomach 1. Unknown etiology; may be an infection involving
or duodenum, beginning of small intestine: nerve tissue; usually resolves in 2 weeks.
1. Caused by bacterial infection (Helicobacter 2. Malfunction of seventh cranial nerve (facial) that
pylori) or use of long-term nonsteroidal antiinflam- controls the muscles of facial expression.
matory drugs (NSAIDs), such as aspirin (Empirin) 3. Treatment: palliative, possibly prednisone (Delta­ o5540
and ibuprofen (Advil, Motrin, Pamprin). sone and/or acyclovir (Zovirax); eye care is im-
a. H. pylori weakens protective mucous coating, perative to prevent complications.
allowing acid to get through to sensitive lin- C. Epilepsy (seizure disorder): common chronic neuro-
ing beneath, with both acid and bacteria irritat- logical group of disorders; with recurrent unprovoked
ing lining and causing ulceration; ����������
diagnosed seizures:
through blood, breath, stool, tissue tests.�� 1. Unknown etiology, with episodic abnormal electri-
2. Dull, gnawing ache comes and goes for several cal activity in brain.
days or weeks in empty stomach (2 to 3 hours af- 2. Treatment: anticonvulsant drugs (antiepileptic
ter meal, middle of night); relieved by eating and drugs [AEDs]) can be taken daily to prevent sei-
­antacids. zures altogether or reduce frequency.
3. Complications: perforation, bleeding, or obstruc- a. ALL have side effects that are idiosyncratic and
tion of gastric portions. others that are dosage dependent.
4. Treatment: combination of proton pump inhibitors b. NOT possible to predict who will suffer from
with two antibiotics (e.g., clarithromycin [Biaxin], side effects or at what dose the side effects will
amoxicillin [Amoxil, Larotid, Polymox], metroni- appear.
dazole [Flagyl]), and possibly three if severe case. 3. Systemic signs: o5600
a. Aura (predication) for oncoming seizure (in-
Neurological Disorders cludes lights, sounds, smells).
Many diseases affect the nervous system, especially the b. Seizures:
central nervous system (CNS) and its cranial nerves. May (1) Petit mal: simple type with staring, trem-
need medical consult with neurological disorder or his- bling, or jerking motions with NO loss of
tory if there are any questions before dental care. consciousness.
• See Chapters 10, Medical and Dental Emergencies: sei- (2) Grand mal: complex type with a trance-
zure; 16, Special Needs Patient Care: neurological and like stage, impairment of consciousness,
orthopedic disabilities. involuntary motor movements followed by
A. Trigeminal neuralgia (tic douloureux): unilateral se- confusion, irritability, extreme fatigue, NO
vere pain triggered by touch (trigger zone[s]). memory of the seizure.
1. Unknown etiology, with damage to myelin sheath, (3) Status epilepticus: life-threatening condition
results in erratic or excessive electrical impulses, in which the brain is in a state of persistent
188   Saunders Review of Dental Hygiene

seizure, with either continuous unremitting repetitive, stereotyped, nonrhythmic movements,


seizure lasting longer than 5 to 10 minutes such as eye blinking, coughing, throat clearing, sniff-
or recurrent seizures without regaining ing, facial movements) and at least one vocal (phonic)
consciousness between seizures for >30 tic. In most cases medication is unnecessary; no ef-
minutes; associated with poor compliance fective medication for every case of tics, but there are
(adherence to medication regimen), alcohol medications and therapies that can help when use is
withdrawal, metabolic disturbances, and warranted, although many have severe side effects.
possibly a tumor or abscess. E. Glaucoma: group of eye diseases of second (II) cranial
o5660 4. Oral signs and concerns: nerve (optic nerve), involve loss of retinal ganglion
a. Gingival hyperplasia resulting from phenytoin cells in characteristic pattern of optic neuropathy.
(Dilantin) use; dental biofilm control is vital 1. Etiology: raised intraocular pressure is significant
for prevention and limitation of gingival over- risk factor, as well as age, family history, race
growth. ­(African-American, Asian), diabetes mellitus, or
b. Trauma from seizure activity such as cheek, gender (female); may be due to unstable ocular
tongue, or lip biting, falling, and tooth chip- blood flow, possibly related to hypertension (high
ping (from biting instruments or clenching blood pressure).
teeth). 2. Visual signs: involves loss of visual field; often
c. Risk factors: occurs gradually and may be recognized ONLY
(1) Trauma from seizure activity during dental when advanced; once lost, damaged visual field
procedures. can never be recovered; glaucoma is second lead-
(2) Development of gingival hyperplasia be- ing cause of ­blindness.
cause of phenytoin (Dilantin) use; surgical 3. Diagnosis: dilated eye examination to check nerve;
excision of gingival overgrowth may be includes measurements of intraocular pressure via
­required. tonometry, changes in size and shape of eye, and
(3) Drowsiness from drug usage; may inhibit examination of nerve to look for visible damage;
memory. if damage is suspected, formal visual field test
(4) Drug interactions (e.g., between phenytoin should be performed.
and barbiturates, doxycycline, others). 4. Treatment: if intraocular pressure is present, can
d. Barriers to care: be lowered with drugs, usually eye drops (like pi-
(1) Economic cost, particularly if disability af- locarpine), or surgery if severe; poor compliance
fects employability. with drugs and follow-up visits is MAJOR reason
(2) Lack of transportation, if unable to drive. for vision loss. Some drugs may have “hidden beta
(3) Lack of communication, if fear of or em- blockers”; therefore patient might experience xe-
barrassment about having a seizure in pub- rostomia (dry mouth).
lic is strong. F. Neurofibromatosis type 1 (NF-1) (von Recklinghau-
e. Professional care and homecare: sen’s disease): rare genetic (autosomal dominant)
(1) Frequent (even monthly) oral prophylaxis, disorder affects nerve cell growth, with multiple cu-
depending on severity of the gingival taneous and subcutaneous tumors that appear in late
­condition. childhood and medium-brown, flat discolorations of
(2) Calm atmosphere, with careful preparation the skin, known as café-au-lait spots.
for dental appointments; medical consult
may be necessary. Immunological Disorders
(3) Demonstration and explanation of thorough Immunological disorders include hypersensitivity, ac-
homecare procedures, including sulcular quired immune deficiency, erythema multiforme, lichen
brushing and flossing. planus, and various autoimmune diseases. Will need
f. Patient or caregiver education: medical consult for patients with immunological disorder
(1) Discussion of oral health and need for ex- before dental care.
cellent oral biofilm control. • See Chapter 8, Microbiology and Immunology: immu-
(2) Repetition of instruction if memory is im- nology and HIV/AIDS.
paired by drugs. A. Hypersensitivity (allergy):
(3) Positive reinforcement to bolster self- 1. Allergic response that varies from a slight cell-
­esteem. ­mediated response to anaphylaxis.
o9140 D. Tourette syndrome (TS): inherited neuropsychiatric 2. Mucositis and dermatitis (rash, hives) on oral mucosa
disorder with onset in childhood, characterized by and skin; reaction from rubber latex contact may oc-
presence of multiple physical (motor) tics (sudden, cur in BOTH patients and healthcare providers.
General and Oral Pathology   189

Figure 6-5  Lichen planus with Wickham’s striae on the Figure 6-6  Systemic lupus erythematosus with “butterfly”
buccal mucosa. rash over bridge of nose.

3. Treatment: ranges from none to drugs such as di- b. Corticosteroids may be injected directly into a
phenhydramine (Benadryl) and epinephrine and lesion.
medical assistance for emergency conditions (see c. Topical retinoic acid cream (Retin-A, form of
Chapter 10, Medical and Dental Emergencies). vitamin A) and other ointments and creams
B. Erythema multiforme (EM): may affect both oral mu- with occlusive dressings may reduce itching
cosa and skin (epidermoid bullosa). and inflammation and aid healing.
1. Lesions on skin resemble a target, consisting of d. Lidocaine mouthwashes may anesthetize mouth
concentric rings of erythema (redness), alternating temporarily, making eating more comfortable
with normal color. with erosive forms.
2. Oral: red macules and ulcerations with hemor- e. Ultraviolet (UV) light therapy.
rhagic crusting of lips. D. Hodgkin’s lymphoma: malignancy (cancer) of lymph
3. MOST in young adults and males; severe form is tissue found in lymph nodes, spleen, liver, bone marrow.
Stevens-Johnson syndrome. 1. Etiology: unknown; MOST common at 15 to 35
4. Treatment: topical corticosteroids. and 50 to 70 years old.
C. Lichen planus: benign, chronic disease that affects 2. FIRST sign is enlarged lymph node, which ap-
skin and oral mucosa (Figure 6-5). pears without a known cause; can spread to nearby
1. Forms: atrophic, reticular, erosive, plaquelike: lymph nodes and later may spread to the lungs,
a. Reticular form: thin white lines called Wick- liver, or bone marrow.
ham’s striae MAINLY on buccal mucosa; con- 3. Diagnosis: lymph node biopsy with Reed-Stern-
sist of white papules in linear form. berg cells.
b. Erosive form: large, painful ulcerations. 4. Treatment: radiation and chemotherapy.
2. Associated with hepatitis C virus; develops after E. Autoimmune diseases: common in women:
exposure to potential allergens such as drugs, dyes, 1. Scleroderma: chronic disease with excessive de-
other chemicals. posits of collagen in the skin and other organs.
3. Symptoms increased with emotional stress, possi- a. MOST evident symptom is hardening of the
bly because of related changes in immune ­system. skin and oral mucosa with associated scarring;
4. Biopsy reveals that epithelial layer has separated can also have Raynaud’s syndrome, vasocon-
from connective tissue in erosive and bulbous stricting disorder affecting fingers and toes.
forms. b. Can be fatal as result of heart, kidney, lung, or
5. Treatment: ranges from palliative to help speed intestinal damage.
healing of the skin lesions to drug therapy: c. Presence of detectable antinuclear antibody.
a. If symptoms are slight, NO treatment may be 2. Systemic lupus erythematosus (SLE): chronic and
needed. progressive disease that affects oral mucosa, skin,
b. Possible risk of cancerous change. organs.
6. Drug therapy: a. Displays classic sign of “butterfly” rash over
a. Antihistamines, topical corticosteroids (such as bridge of nose, also arthritis, dermatitis, glo-
triamcinolone [Azmacort]) or oral corticoste- merulonephritis, anemia, inflamed spleen and/
roids (such as prednisone [Deltasone]) to reduce or lymph nodes; if ONLY rash, discoid form
inflammation and suppress immune responses. (Figure 6-6).
190   Saunders Review of Dental Hygiene

b. Autoantibodies to own deoxyribonucleic acid a. Intraepithelial formation of bullae that do NOT


(DNA) (antinuclear antibodies). rupture easily, possibly positive Nikolsky’s
c. MOST common cause of death: kidney failure sign (MORE common than with pemphigoid),­
(degeneration of kidneys). BMMP); smear shows Tzanck cells.
3. Benign mucous membrane pemphigoid (BMMP) b. Biopsy reveals breakdown of cellular adhesion
(cicatricial pemphigoid): rare chronic, vesicular ­between epithelial cells (acantholysis affecting
disease of oral cavity. desmosomes and hemidesmosomes), which al-
a. Produces bullae (large vesicles) that rupture lows symptoms to develop; MOST in middle age.
easily; cause discomfort; MOST often on gin- c. Treatment: corticosteroids.
giva (desquamative gingivitis). 5. Sjögren’s syndrome: affects salivary and lacrimal
b. May produce positive Nikolsky’s sign (tissue glands, resulting in decrease in BOTH saliva and
desquamation with gentle pressure, LESS com- tears.
mon than with pemphigus), areas of ulceration; a. Has xerostomia and xerophthalmia (dry eyes).
some forms leave scarring (less common in oral b. May have parotid gland enlargement and Rayn-
­cavity) that can cause blindness. aud’s syndrome, vasoconstricting disorder affect-
c. Affects older adults, women MORE than men; ing fingers and toes (pain with bluish ­coloring).
biopsy reveals oral epithelium separated from c. Rheumatoid arthritis also present or other auto-
lamina propria. immune disease; positive reaction to rheuma-
d. Treatment: a team of medical specialists is re- toid (RA) factor, antibody to IgG (see Chapter
quired for overall care; primary aim is to stop 16, Special Needs Patient Care).
vesicle formation, promote healing, and pre- d. Treatment: palliative; nonsteroidal antiinflam-
vent scarring (cicatricial form is difficult to treat matory agents are used for arthritis; in severe
­because it can affect so many different parts of cases gold injections, corticosteroids, other im-
the body). munosuppressive drugs; can recommend use of
4. Pemphigus vulgaris: autoimmune disease that prescription drugs such as pilocarpine (Sala-
affects skin and oral mucosa (NOT as much as gen) and cevimeline (Evoxac), since dryness is
BMMP). not drug related.

s9000 CLINICAL STUDY�  

t9000 Age 65 YRS Scenario

Sex ☐  ­Male   ☒  Female The patient states that this condition has occurred
several times during the past few years. Visual examina-
Height 5’2” tion of her oral cavity reveals several erythematous areas
surrounding grayish white lesions on the gingiva.
Weight 121 LBS
A negative Nikolsky’s sign is also noted during the
BP 110/74 ­intraoral examination. Patient does not mention any
other new health problems.
Chief Complaint “My mouth burns and my gums are
bleeding all the time now. And when
I eat, the blisters in my mouth break
open.”

Medical History Past history of polio as a child

Current Medications ibuprofen for backaches pm

Social History Dry cleaning business owner, widow

o9150 1. What is the most likely diagnosis of this patient’s con- 5. What other indicators assist in the differential dia­ o9190
dition? Why? gnosis?
o9160 2. Identify the common signs and symptoms associated
with this condition. 1. Condition most likely is benign mucous membrane o9200
o9170 3. Describe the treatment for this condition. pemphigoid (BMMP), autoimmune disorder that re-
o9180 4. How does the presence of a negative Nikolsky’s sign sults in development of vesiculobulbous lesions on
distinguish this condition from pemphigus ­vulgaris? the gingiva, floor of the mouth, hard palate. Lesions
General and Oral Pathology   191

easily rupture, giving appearance of desquamative 5. Treatment: full coverage if restorations and/or
gingivitis, causing soreness. Lesions eventually heal, tooth structure is compromised.
sometimes causing scarring. Tends to occur in older B. Abrasion: slow wear of dentition associated with
adults; chronic condition with periods of exacerbation abrasive substances, abnormal habit.
and remission. 1. Caused by improper use of toothbrush and/or
o9210 2. Common signs and symptoms of disorder include ­abrasive dentifrice; causes notch at exposed
vesicles that easily rupture, reddened and ­ ulcerated root surface, particularly at canine and premolar
­gingiva, burning, mouth soreness, recurrence of area.
­condition after remission, and scarring (however, less 2. May also be caused by repetitive habit (bobby
in oral cavity). Scarring can involve blindness. pins, needles, musical instruments, pipe smokers),
o9220 3. Treatment is palliative. During active stages, tooth- especially on incisors.
brushing may be impossible; thus 0.2% chlorhexidine 3. May lead to hypersensitivity because of exposed
rinse is recommended. Topical anesthetics may be dentin and increased risk of caries.
needed to encourage proper nutrition. Topical and/or 4. Prevention: discussion of oral hygiene methods
systemic corticosteroids are prescribed based on se- and habits.
verity of condition. 5. Treatment: patient education and restorative care if
o9230 4. Both BMMP and pemphigus vulgaris may exhibit a indicated.
positive Nikolsky’s sign; noted histologically as sepa- C. Erosion: loss of tooth structure because of a chemical o6650
ration of surface epithelium from underlying con- action differing from soft tissue crosion after rupture
nective tissue; clinically shows tissue desquamation of vesicle or bulla.
(removal) with gentle pressure. However, BMMP is 1. In bulimia or chemotherapy, result of emesis (vom-
associated with it less often than pemphigus vulgaris. iting) of stomach acids, causes loss of enamel on
o9240 5. Although biopsy would confirm diagnosis of BMMP maxillary lingual surfaces of teeth (perimolysis)
rather than pemphigus vulgaris, the fact that she has and rising of existing restorations.
had previous oral lesions with no mention of skin 2. May be acid from industrial work (battery, plating,
lesions or other concerns indicates that BMMP is soft drink manufacturing).
most likely diagnosis. Other autoimmune disorders 3. Also is influenced by diet (citrus sucking, cocaine
such as Sjögren’s syndrome and lupus erythemato- or methamphetamine use, soft drink overuse, espe-
sus (systemic [SLE] and discoid) may occur in as- cially diet soft drink because of low pH) and can
sociation with pemphigus vulgaris, as well as skin be noted in early childhood caries (ECC).
lesions. 4. Prevention: discussion of diet and recommenda-
tion for counseling if eating disorder is present.
TRAUMATIC ORAL LESIONS   5. Treatment: removing cause; fluoride and mineral
Traumatic injury, by either chemical or physical means, can (topical calcium replacement) application; rinsing
result in oral lesions. Includes attrition, abrasion, erosion, the mouth with water (NOT brushing in acids) and
burns, tobacco-associated lesions, salivary gland obstruc- thoroughly cleaning the teeth immediately after
tion, connective tissue hyperplasias, ulcerations, and others. vomiting episodes also lessen effects of acid; may
• See Chapter 13, Periodontology: occlusal trauma. need full-­coverage restorative therapy for esthetics
A. Attrition: physiological wear of dentition accelerated and function.
by abnormal grinding (bruxism) and/or clenching. D. Abfraction: wedge-shaped lesions at the cervical
1. FIRST sign is disappearance of mamelons and then ­areas of teeth (especially premolars).
MAINLY wear facets on cusp tips and incisal edges 1. May be related to fatigue, flexure, fracture, and
of anteriors and facets on occlusal cusps of posteri- deformation of tooth as the result of biomechanical
ors; also staining of dentin-exposed worn surfaces. forces such as grinding [bruxism] and/or ­clenching,
2. May also result from abrasion from porcelain torquing of tooth and/or bite discrepancy.
crowns, bridges, or denture materials on surfaces 2. Weakened tooth structure is more at risk for abra-
of opposing teeth (unrestored, amalgam, resins, sion, hypersensitivity because of exposed dentin
etc.), as well as the use of spit (smokeless) tobacco and caries; mainly in older adults.
(sand). 3. Prevention: occlusal splint (nightguard).
3. Bruxism may also be associated with gingival reces- 4. Treatment: restorative care with composite or
sion, temporomandibular disorders (TMD), hypertro- glass ionomer materials, but forces may result in
phy (enlargement) of masseter muscle, mandibular dislodging the restoration.
tori, hypersensitivity because of exposed dentin and E. Temporomandibular joint disorder (TMD): con-
unsightly stained tooth material. ditions that affect musculoskelature of joint area;
4. Prevention: occlusal splint (nightguard) and dis- result in pain and dysfunction of masticatory
cussion of habits. system.
192   Saunders Review of Dental Hygiene

1. May affect muscles ONLY (extracapsular) or joint H. Crack cocaine smoking:


(intracapsular). 1. Lesions from hot smoke located at midline of
2. Etiology is multifactorial; stress is a frequent fac- hard palate; vary from ulcers to keratotic lesions
tor; others include arthritis, psychological prob- to exophytic reactive lesions (will also have nasal
lems, macrotrauma or microtrauma to joint. ulcerations and saddle nose appearance if snorting
3. Four primary diagnostic categories: crack).
a. Muscle and facial disorders of masticatory sys- 2. Necrotic ulcers of the nose, tongue, and epiglot-
tem; include trismus, myalgia, spasms, dyski- tis related to smoking free-base cocaine have also
nesia (abnormal movement), bruxism. been reported; epistaxis (nosebleeds) or discharge
b. Disorders of the TMJ; include conditions that (constant wiping); saddle nose from chronic use.
cause internal derangements impeding function 3. Treatment: substance abuse therapy.
of joint (e.g., arthritis) (see Chapter 16, Special I. Salivary gland duct obstruction: obstruction of a sali-
Needs Patient Care). vary gland duct by salivary stone (sialolith) and/or
c. Disorders of mandibular mobility, which include severance of the duct to prevent drainage.
joint adhesions, ankylosis, and muscular fibrosis. 1. Translucent or bluish (because of fluid within) su-
d. Disorders of maxillomandibular growth, which perficial swelling:
are less common and include both neoplastic a. Mucocele: smaller on labial mucosa of lower
and nonneoplastic conditions. lip or tongue, involving minor gland.
4. Oral habits can be significant factors in produc- b. Ranula: larger on one side of the floor of the
tion, especially in acute cases: mouth; resembles a “frog’s belly” with submen-
a. Amount of damage produced by habit is rela- tal swelling related to eating, involving mainly
tive to intensity and duration. submandibular and sublingual.
b. Bruxism, clenching (see earlier discussion). 2. Treatment: removal of the obstruction; surgery if
5. Signs and symptoms: necessary.
a. Pain and tenderness in muscles of mastication. J. Oral connective tissue hyperplasia (reactive):
b. Pain and tenderness of the joint itself. 1. Pyogenic granuloma (pregnancy tumor, if female):
c. Functional limitation of the joint. proliferation of connective and vascular tissue (see
6. Prevention and treatment: bite plane splint, an- Figure 11-7).
tidepressants, physical therapy; surgery is last a. Located on gingiva; also on lip, tongue, buccal
­resort. mucosa.
F. Oral burns: initiated by heat or chemical exposure, b. Formation may be related to hormonal changes
causing tissue necrosis of oral mucosa; MAINLY (MOST during pregnancy) combined with
from hot foods (bagels in the East, pizza in the Mid- chronic irritants such as dental biofilm.
west, coffee in the West): c. SIMILAR in clinical appearance to peripheral
1. Can also involve “aspirin burn” from aspirin giant cell granuloma; however, this lesion con-
placed on oral mucosa to ease dental pain, phenol tains multinucleated giant cells and may lead to
from dental procedures, live electric cords, use of bone destruction.
hydrogen peroxide if not diluted or strong whiten- d. Treatment: if NOT spontaneously reduced (af-
ing products. ter birth of child), surgical excision.
2. Include white or ulcerated lesions of the oral mu- 2. Denture-induced hyperplasia: proliferation of con-
cosa; thermal burns on the palate are red. nective tissue associated with removable prosthesis.
3. Treatment: analgesics, topical anesthetics; severe a. Two forms:
cases, surgery. (1) Epulis fissuratum: folds of tissue near den-
G. Tobacco-related oral lesions: ture flange (edge) from ill-fitting dentures.
1. Spit (smokeless) tobacco (chewer’s) lesions: (2) Inflammatory papillary hyperplasia: nipple-
symptomatic, wrinkled, white lesions located in like fibrous papillary projections of tissue
mucobuccal fold at tobacco placement; clinical located on palate.
presentation does NOT indicate level of carcino- b. Treatment: surgical removal of excess tissue and
genic changes. a prosthesis reline or new denture, if necessary.
2. Nicotinic stomatitis: whitened hard palate with 3. Gingival hyperplasia: proliferation of free or at-
raised red dots (minor salivary gland inflamma- tached gingival tissue, in response to local irritants
tion); caused by heat of smoking and/or hot foods. (e.g., dental biofilm, calculus).
3. Treatment: tobacco cessation if needed and pos- a. Varies from soft to fibrous gingiva; color may
sible biopsy to assess for SCC, spit tobacco also vary from pink to red because of increased
lesions. vascularity.
General and Oral Pathology   193

masticatory oral mucosa (attached keratinized


such as attached gingiva, hard ­palate).
b. MOST often recur and may be associated with
history of trauma (sharp food, toothbrush edge)
or emotional stress; may be immune or bacte-
ria related; increased incidence during tobacco
cessation.
c. Forms:
(1) Minor: single lesion ≤1 cm in diam-
eter; MOST common form; treated by
Figure 6-7  Irritation fibroma on the buccal mucosa. ­antibacterial topical application with occlu-
sal ­dressing.
(2) Major: single lesion ≥1 cm in diameter;
b. Result of hormonal changes, including puberty more destructive and can cause scarring;
and pregnancy, hypothyroidism, or heredity; treated by application of topical steroids
exacerbated by poor oral hygiene. during prodromal and preulcerative period.
c. Side effect of certain drugs: phenytoin (Dilan- (3) Herpetiform: small clusters of lesions; may o7410
tin) for seizures with epilepsy, calcium channel coalesce into larger irregular lesions.
blocker such as nifedipine (Procardia, Adalat, d. Prevention: avoiding causative factor.
Nifedical) for cardiac disease caused by hyper- e. Treatment: protection of lesion and topical anti-
tension (high blood pressure) or angina, immu- septic to reduce secondary infection.
nosuppressant cyclosporine to prevent rejection 2. Traumatic ulcers: result of occlusal trauma (bit-
after transplant of organs and tissue. ing), irritation from orthodontic and removable
d. Can occur with pericoronitis when associated appliances, trauma from food.
with partially erupted third molars (“wisdom a. Diagnosed using history; heal within 2 weeks
teeth”) as a flap of tissue (operculum); has more (rule out carcinomatous growth).
inflammation and bleeding present. b. Treatment: removing cause of trauma; may
e. Treatment: gingivoplasty, gingivectomy (in- need biopsy if clinically suspect.
cluding removal of operculum), and oral hy- 3. Blood-related lesions with NO treatment because they
giene instruction; in addition, debridement and resolve by healing (may need to reassure patient):
irrigation, as well as antibiotic therapy, may be a. Hematoma (“blood blister”): red to purple mass
used initially to treat pericoronitis. produced by accumulation of blood within tis-
4. Irritation fibroma: dense connective tissue growth sue resulting from trauma to a large blood ves-
related to chronic irritation (Figure 6-7): sel (e.g., lesion after local anesthetic injection,
a. Asymptomatic, exophytic growth with color especially inferior alveolar nerve block, or after
similar to that of adjacent tissues. extraction of a difficult tooth).
b. Located on the buccal or labial mucosa, gin- b. Ecchymosis (“bruise”): reddened to purple area
giva, tongue. with slight height; changes color during break-
c. Treatment: surgical removal and possibly bi- down of clot.
opsy to rule out malignancy. c. Petechiae: red spots caused by escape of small
5. Frictional keratosis: chronic rubbing and friction amount of blood.
against an oral mucosal surface may result in hy- L. Necrotizing sialometaplasia: benign necrotic condi-
perkeratosis. tion of salivary glands.
a. Results in an opaque, white appearance and 1. Moderately painful prolonged swelling and ulcer-
represents a protective response. ation at junction of hard and soft palates.
b. Treatment: identification of trauma, eliminating 2. Caused by blockage of the blood supply to the area
cause, observing resolution; may take awhile to of the lesion that heals slowly; deep biopsy may be
resolve on keratinized surfaces (e.g., attached needed to confirm diagnosis.
gingiva), possible biopsy to rule out malignancy. 3. Treatment: palliative.
K. Oral ulcerations: M. Traumatic neuroma: injury to peripheral nerve result-
1. Recurrent aphthous ulcers (RAU, “canker sores”) ing from anesthetic injection, oral surgery, or other
that heal within 7 to 10 days: trauma.
a. Painful oral lesions on lining mucosa (unattached 1. Painful lesion on palpation or constant pain
nonkeratinized such as buccal and labial mucosa); ­possible in adults; MOST common in mental
in contrast, herpes lesions (“fever ­ blisters”) on ­foramen region.
194   Saunders Review of Dental Hygiene

2. Diagnosis based on a biopsy and microscopic 2. Asymptomatic; late radiographic change indicates
­examination. distinct periapical radiolucency, similar to peri-
3. Treatment: surgical removal; recurrence is rare. apical granuloma; associated with badly decayed
N. Amalgam tattoo: teeth or endodontically treated teeth (in contrast,
1. Flat, bluish gray lesion of oral mucosa caused periodontal cyst usually results from trauma and is
by amalgam particles embedded into tissues dur- around a healthy tooth).
ing restorative procedures; may have radiopaque 3. May be differentiated from periapical granuloma
­appearance. ONLY by biopsy.
2. MOST on the gingiva and edentulous alveolar 4. Treatment: endodontic therapy, extraction; resid-
ridge of mandibular posterior region. ual cyst can form.
3. Treatment: none; biopsy may be necessary to rule D. Peripheral cemental dysplasia (cementoma[s]):
out pigmented malignancy. disordered increased production of cementum and
bone.
Periapical Inflammation and Pathology 1. Radiolucent to radiopaque appearance that changes
Inflammation and pathological changes of the periapical over time; can be similar to periapical disease, but
tissues occur as periapical abscesses, cysts, and granu- tooth is vital and asymptomatic; noted FIRST on
lomas, condensing and alveolar osteitis, chronic hy- radiograph.
perplastic pulpitis. Abscess is a localized collection of 2. MOST in anterior teeth and in adults ≥30 years.
suppuration (pus) in a cavity formed by disintegration of 3. Treatment: none indicated; may need to take radio-
tissues. A TRUE cyst is an abnormal closed epithelium- graphs and perform pulp testing over time to rule
lined cavity in the body, containing liquid or semisolid out periapical disease.
material. Developmental cysts are discussed later. E. Condensing osteitis (focal sclerosing osteomyelitis):
• See Chapters 11, Clinical Treatment: pulpal evalua- 1. Indicated by formation of dense periapical bone,
tion; 13, Periodontology: periodontal and pericoronal often in response to low-grade dental infection.
­(pericoronitis) abscesses. 2. Asymptomatic; may have radiopacity in periapi-
A. Periapical (endodontic) abscess: acute inflammatory cal region inferior to roots of teeth, with possible
condition that develops in the pulp or surrounding radiolucency either surrounding or central to it.
periapical tissues. 3. MOST with permanent mandibular first molar.
1. Etiology: trauma, caries, spread of infective mi- 4. Treatment: none; biopsy rules out other lesions.
croorganisms to pulp; MOST cases result in pulpal F. Chronic hyperplastic pulpitis (pulp polyp): asymp-
necrosis. tomatic hyperplasia of pulp:
2. May cause severe pain, exudate with fistula forma- 1. Often is associated with a large carious lesion in
tion; may have suppuration (pus); often is difficult children and young adults.
to distinguish from acute periodontal abscess. 2. Treatment: pulpotomy, extraction.
3. Radiographic late finding, changes ranging from G. Alveolar osteitis (“dry socket”): painful lesion in
slight to distinct periapical radiolucency; located area of extraction with foul odor, bad taste, increased
usually at apex but may follow lateral canal. bleeding.
4. In MOST cases, involves a nonvital tooth; pain 1. Usually mandibular third molar (“wisdom tooth”).
differs from that of periodontal abscess in that it 2. Result of lack of proper blood clot formation after
is sharp and intermittent rather than dull and con- extraction; may NOT have followed postextraction
tinuous; may or may NOT test positive on electric recommendations (NO hot food, drink through
pulp tester for vitality (vitalometer with a scale of straw).
1 to 10; the closer to 1, the healthier the tooth). 3. Treatment: irrigation and placement of medicated
5. Treatment: endodontic therapy (root canal), dressing, possibly antibiotic coverage.
­extraction. H. Ludwig’s angina: serious, potentially life-threatening
B. Periapical granuloma: involves chronic growth of cellulitis involving infection of soft tissues of floor of
granulation tissue located in periapical region. the mouth.
1. Asymptomatic; late radiographic changes range 1. Usually bacterial infection, Streptococcus or
from slight to distinct periapical radiolucency. Staphylococcus; route of infection from mandibu-
2. Treatment: endodontic therapy, extraction. lar third molars and/or pericoronitis (opposed to
C. Periapical cyst (radicular cyst): epithelial-lined tissue cavernous sinus thrombosis, which can also be
cavity associated with periapical granuloma: related to spread of dental infection but involves
1. Result of proliferation of the epithelial rests of infected thrombus transported into venous sinus
Malassez; tissue remains from Hertwig’s epithelial of the brain, usually by way of infected maxillary
root sheath (HERS). teeth).
General and Oral Pathology   195

2. Usually occurring in immunocompromised adults


with untreated or invasive dental infections.
3. Pain and swelling of the tongue, which raises it
from the floor of the mouth, with bilateral swelling
of submandibular and sublingual spaces, malaise,
fever, dysphagia (difficulty swallowing), and in se-
vere cases stridor (difficulty breathing).
4. Treatment: appropriate antibiotic drugs, moni-
toring and protection of airway in severe cases,
and where appropriate, emergency maxillofacial
­surgery.

DEVELOPMENTAL DISORDERS OF ORAL CAVITY 


Developmental disorder is a disturbance or lack of devel-
Figure 6-8  Globulomaxillary cyst, pear-shaped radio-
opment of a structure; such disorders of the oral cavity lucency between maxillary lateral and canine, with root
include odontogenic and nonodontogenic cysts and other ­divergence.
abnormalities within the oral cavity. See earlier discus-
sion of periapical cysts and associated lesions.
• See Chapters 3, Anatomy, Biochemistry, and Physiol- a. Radiolucency; MOST in mandibular canine or
ogy: systemic developmental disorders; 4, Head, Neck, premolar area.
and Dental Anatomy: orofacial development and dis- b. Treatment: surgical removal.
orders; 16, Special Needs Patient Care: developmental B. Nonodontogenic cysts: related to facial development
disabilities. (fissural), NOT to tooth development.
A. Odontogenic cysts, related to tooth development: 1. Nasopalatine cyst: cystic transformation of epithe-
1. Eruption cyst: bluish soft tissue swelling around lial remnants in nasopalatine ducts; MOST com-
newly erupting crown (NO radiographic fea- mon of group.
tures): a. Asymptomatic, heart-shaped midline radiolu-
a. MOST with a primary tooth; guardians may be cency, within incisive papilla or duct, like large
concerned and need reassurance. incisive foramen; teeth test vital, no divergence
b. Treatment: none (dissipates during eruption of roots.
process). b. MOST in adults between 40 and 60 years.
2. Dentigerous (follicular) cyst: associated with re- c. Treatment: surgical removal.
duced enamel epithelium; leads to cyst formation 2. Globulomaxillary cyst: cystic transformation of
around crown of unerupted tooth. epithelial remnants trapped at line of fusion be-
a. Well-defined, unilocular radiolucency associ- tween globular process of median nasal process
ated with a tooth; MOST with impacted third and maxillary process (Figure 6-8):
molar (“wisdom tooth”). a. Asymptomatic; teeth test vital, indicating that
b. Treatment: surgical removal of cyst and periapical pathology is NOT likely.
­extraction. b. Pear-shaped radiolucency between maxillary
3. Primordial cyst develops in place of a tooth and is lateral and canine; may cause root divergence.
caused by disturbance of enamel organ. c. Treatment: surgical removal.
a. Well-defined, unilocular or multilocular radio- 3. Nasolabial cyst: cystic transformation of epithelial
lucency; MOST in third molar region. remnants trapped between fusion of globular, lat-
b. Treatment: surgical removal. eral nasal, and maxillary processes:
4. Odontogenic keratocyst: lined by BOTH epithe- a. Soft-tissue cyst located in nasolabial fold caus-
lium and keratin; capable of moving and resorbing ing facial swelling; no radiographic features.
teeth. b. MOST in 40- to 50-year-olds.
a. Type of primordial and dentigerous cyst. c. Treatment: surgical removal.
b. Well-defined, multilocular radiolucency; MOST 4. Median palatine cyst (rare): cystic transformation
in mandibular third molar region. of epithelial remnants trapped between fusion of
c. Treatment: surgical removal; high rate of recur- palatal process of maxilla:
rence (because smaller cysts bud off). a. Midline asymptomatic, fluctuant swellings.
5. Lateral periodontal cyst: asymptomatic soft tissue b. Well-defined, unilocular, midline radiolucent le-
or osseous cyst indicated by swelling in interdental sion that is slightly posterior to incisive ­papilla.
papillae: c. Treatment: surgical removal.
196   Saunders Review of Dental Hygiene

5. Epidermoid and dermoid cysts: consist of ONLY (1) Mesiodens: between maxillary central
epithelial cells (epidermoid cyst) or a combina- ­incisors.
tion of epithelium, mesoderm, endoderm (dermoid (2) Distomolars: distal to third molar.
cyst): (3) Paramolars: buccally or lingually by ­molars.
a. BOTH occur in the mandible; noted MORE in b. Treatment: extraction if it will NOT damage
young adults. erupted teeth.
b. Treatment: surgical removal. 4. Gemination: single tooth bud forms into two joined
C. Other cysts: teeth.
1. Simple bone cyst (traumatic bone cyst): unknown a. Has one root with canal and two crowns; falsely
etiology, possibly trauma. macrodontic.
a. Well-defined, unilocular or multilocular radio- b. MOST in primary mandibular incisors; normal
lucency; located between mandibular molar number of teeth in arch.
roots. c. Treatment: asymptomatic and does NOT re-
b. MOST in young adults. quire treatment; however, may cause crowding
c. Treatment: surgical removal. with resultant need for orthodontics.
2. Aneurysmal bone cyst: possibly a localized exag- 5. Fusion: joining of two tooth germs to form a single
gerated vascular proliferative response to repair tooth.
bone. a. MORE than one root canal may be present;
a. Soap bubble radiographic appearance; may falsely macrodontic.
cause bone expansion. b. MORE in primary than permanent dentition;
b. May follow previous lesion; MOST in young associated with anterior teeth; one LESS tooth
adults. in arch.
c. Treatment: surgical removal (bleeding noted). c. Treatment: asymptomatic and does NOT re-
3. Static bone cyst (lingual mandibular bone concav- quire treatment; however, may cause crowding
ity): NOT considered a true cyst because has no with resultant need for orthodontics.
epithelial lining: 6. Dens in dente (dens invaginatus): invagination of
a. Depression in the medial side of the mandible, the enamel organ into dental papilla:
location for sublingual salivary gland. a. Occurs along lingual surface (cingulum);
b. Radiolucency in the posterior mandible; MOST in maxillary lateral incisor.
MOSTLY discovered accidentally on radio- b. Increased risk of pulpal exposure and endodon-
graphs. tic therapy.
c. Treatment: none; may want to monitor by c. Treatment: none but MUST monitor for pulpal
­radiographs. changes.
D. Dental developmental abnormalities: MOST do not 7. Microdontia: smaller tooth (teeth).
require treatment but can cause possible problems a. Rare to occur in entire dentition; genetic predis-
with esthetic concerns (may need coverage by crowns position.
or veneers, possibly whitening) and spacing in denti- b. MOST with single teeth but bilaterally, espe-
tion or endodontic therapy complications. cially maxillary lateral incisors (peg lateral) or
1. Hypodontia: lack of one or more teeth (tooth molars (peg molar).
bud[s]). c. Treatment: none, but spacing may be a prob-
a. Partial lack is fairly common condition; MOST lem; full-coverage crown for esthetic purposes.
in third molar regions, maxillary laterals, man- 8. Enamel pearl (projection): exophytic area of
dibular premolars of permanent dentition; lat- enamel on root surface.
eral incisors of primary dentition. a. Result of displaced ameloblasts during root
b. Treatment: placement of fixed or removable ­formation.
prostheses (e.g., implants, bridges, partial b. Located near cementoenamel junction (CEJ);
­dentures). MOST in mandibular molars, especially in fur-
2. Anodontia (complete lack of tooth formation or cation areas.
buds): rare; complete dentures are placed as the c. May be mistaken for calculus and can be noted
jaws grow. as radiopacity on root.
3. Supernumerary teeth: formation of extra teeth d. Treatment: surgical removal if needed.
(bud[s]). 9. Enamel hypoplasia: incomplete enamel formation
a. Often smaller in size (microdont) and remain (quantity affected) that is caused by disturbances
unerupted; extra tooth in arch; MOST in maxil- or damage to ameloblasts during enamel formation
lary arch: (very sensitive Barney-like cells).
General and Oral Pathology   197

a. May vary from pits to complete grooving of d. In mild cases, few white flecks or small pits;
teeth; can be noted on radiographs as thin or more severe, may be brown stains; Dean’s fluo-
absent enamel. rosis index is MOST commonly used classifica-
b. Turner’s tooth: single affected tooth, usually ca- tion system.
nine or premolar, as result of trauma. e. By the time the physician and/or dentist rec- o9260
c. Also noted with congenital syphilis, tetracy- ognizes fluorosis in the permanent teeth, it is
cline stain, amelogenesis imperfecta; can be too late to prevent its appearance on most of
combined with hypocalcification with fluo­ the other teeth, including those yet to erupt,
rosis. because fluoride has already been incorporated
10. Enamel hypocalcification: incomplete enamel into their enamel; therefore, providing the ap-
calcification (quality affected) that is cause by propriate amount of fluoride during the first 6
a ­ disturbance or damage to ameloblasts during years of life is the best method to prevent both
enamel formation. caries and fluorosis.
a. May vary in color from white to brown (mottled f. MORE prevalent in rural areas where drinking
enamel); NOT noted on radiographs. water is derived from shallow wells and hand
b. Turner’s spot (“sparkle spot”): single affected pumps; also MORE likely to occur in areas
area. where the drinking water has a fluoride content
c. Also noted with amelogenesis imperfecta of >1 ppm and calcium intake is poor.
(MOST common type); can be combined with 12. Dilaceration: curvature of the root; may be caused
hypoplasia with fluorosis. by trauma during tooth development.
o8880 11. Dental fluorosis: excessive intake of fluoride a. MOSTLY with third molars (“wisdom teeth”).
through fluoride added to the water supply or from b. Treatment: none; a challenge during extraction
other sources (see Chapter 17, Community Oral and endodontic therapy.
Health, for more information). 13. Taurodontism: formation of elongated pulp and
a. Damage in tooth development occurs between short root (looks like bull’s teeth).
ages of 6 months to 5 years; usually permanent a. Results from improper invagination of HERS.
teeth affected; occasionally primary teeth may b. Radiographs show teeth with elongated pulps
be involved. and short roots; increased risk in molars of
b. Noted with yellowing of teeth, white spots, and ­Native Americans and Eskimos.
pitting or mottling (multicolor disturbance) of c. Treatment: none.
enamel; combination of BOTH enamel hypo- 14. Concrescence: joining of teeth by cementum:
plasia and hypocalcification is possible. a. Result of trauma and crowding in the area
o9250 c. In children, fluorosis appears most frequently ­during development, MOST with molars.
as “snowcapping,” a parchment-white-colored b. Treatment: none indicated; can complicate
area on the incisal or occlusal surface. ­extraction.

clinical study�  

Age 26 YRS Scenario

Sex ☒  Male   ☐  Female On visual intraoral examination,


acute inflammation is noted in the
Height 6’�����
3”� gingival tissues surrounding his
newly erupting terminal molar, with
Weight 185
moderate levels of dental biofilm.
BP 100/74 A panoramic radiograph is taken,
and development of the third
Chief Complaint “I really hurt in my lower right jaw. What is going on?” molar is assessed. In addition, a
nickel-sized well-defined corticated
Medical History Multiple broken bones resulting from parasailing
radiolucency is noted between the
Testicular cancer when a teenager
Father recently died of pancreatic cancer roots of the maxillary right lateral
incisor and ­canine; the roots of the
Current Medications Tylenol for headaches prn teeth are also spread apart from
each other.
Social History Physical therapist taking care of patient’s invalid mother
with early-onset Alzheimer’s
198   Saunders Review of Dental Hygiene

1. Name the condition the patient is experiencing in 2. Treatment: none indicated; however, ALWAYS at
the molar regions of his oral cavity. What has caused increased risk for bleeding; prophylactic treatment
his condition? What can this condition lead to if not is sometimes given before surgery.
treated properly? B. Hemophilia: type A (MOST common, classic) is X
2. What can be done to alleviate the patient’s ­discomfort? linked; transmitted from unaffected daughters to
3. What is indicated by the radiolucency located between grandsons, thus affects ONLY men:
the roots of the maxillary lateral ­incisor and canine? 1. Deficiency in blood clotting factors: type A, factor
4. Why does the lesion have radiolucency? How is the VIII deficiency; type B, factor IX deficiency; type
lesion treated? C, factor XI deficiency.
2. Type B (Christmas disease) and others are LESS
1. The patient is experiencing pericoronitis (pericoronal common forms, but ALL affect only men.
abscess) with gingival hyperplasia in molar regions. 3. Oral signs: spontaneous gingival bleeding, pete-
Condition occurs when bacteria such as with dental chiae, ecchymoses (see earlier notes on bleeding
biofilm collect beneath flap of hyperplastic gingival disorders such as anemia).
tissue (operculum) that commonly covers most dis- C. Cyclic neutropenia: autosomal dominant disorder:
tal surface of newly erupting molars (especially third results in decreased neutrophil (polymorphonu-
molars, “wisdom teeth”) and cause acute infection, cleocyte [PMN]) production that tends to occur
with pain and inflammation. If not treated promptly, every 3 weeks (cyclic) and lasts 3 to 6 days at a
can lead to Ludwig’s angina, a serious, potentially time.
life-threatening infection of soft tissues of floor of the 1. Caused by changing rates of cell production by the
mouth. bone marrow.
2. Discomfort can be relieved by gentle irrigation of 2. Oral signs: severe ulcerative gingivitis, tongue and
sulcus with saline solution, warm water, or diluted oral mucosa ulcers, increased risk of periodontal
hydrogen peroxide. Debridement of the area at ini- disease.
tial appointment, using topical anesthetic, is recom- D. Papillon-Lefèvre syndrome: autosomal recessive dis-
mended if patient can tolerate it. If not, debridement order: results in aggressive periodontal disease with
should occur during second visit, when inflammation advanced bone destruction and tooth loss.
has subsided somewhat. Use of systemic antibiotics E. Hereditary fibromatosis: associated with several dis-
may also be indicated if there are systemic signs of orders (e.g., MAINLY Laband’s syndrome):
infection. Surgical removal of the tooth or operculum 1. Causes gingival fibromatosis, nail dysplasia, joint
may be necessary to prevent recurrence. Prevention hypermobility; disease develops soon after tooth
includes discussion of oral hygiene care in patients eruption.
who are at risk for condition, such as having a partially 2. May be present as a generalized enlargement
erupted third molar. of the gingival tissue, which eventually may
3. Radiolucency located between roots of maxillary right cover the dentition completely; tissue is firm
lateral and canine is most likely a globulomaxillary and slightly lighter in color than surrounding
cyst. Asymptomatic; thus teeth remain vital. Roots of ­gingiva.
adjacent teeth often diverge because of pressure from 3. Treatment: oral hygiene instruction, surgical re-
the cyst. moval of excess tissue.
4. Radiolucency is from this developmental disorder, F. Osseous disorders:
since it is noncalcified with no bony involvement. 1. Cherubism: autosomal dominant disorder that de-
Treated by surgical excision. velops early in life.
a. Bilateral facial swelling, multilocular, soap
Inherited Disorders bubble bilateral radiolucency of the mandible.
Several inherited disorders have an effect on the oral cav- b. Associated with abnormal tooth development
ity. Included are disorders that affect the periodontium and eruption.
and teeth. 2. Cleidocranial dysostosis (dysplasia): auto-
• See Chapter 16, Special Needs Patient Care: muscular somal dominant disorder; does NOT affect
dystrophy, Down syndrome. ­intelligence.
A. Von Willebrand disease (VWD): hereditary coagulation a. Hypoplasia of clavicles (touch shoulders to-
abnormality; MOST common of all inherited bleeding gether); underdeveloped premaxilla; low nasal
disorders and can affect both women and men. bridge; occipital, parietal, and frontal bossing;
1. Deficiency of multimeric protein factor that is re- short forearms and fingers.
quired for platelet adhesion; ALL clinical signs of b. Supernumerary teeth, malformed teeth, abnor-
bleeding when organs are stressed or traumatized. mal eruption sequence.
General and Oral Pathology   199

3. Osteogenesis imperfecta: autosomal dominant dis- 3. Treatment: none unless surgical removal for
order; presence of multiple bone fractures and loss a dental prosthesis or growth interferes with
of enamel because of abnormal dentin ­formation. speech.
G. Dental disorders: C. Pigmentation (physiological, melanosis): asymptom-
1. Amelogenesis imperfecta: autosomal recessive dis- atic, symmetrical, brown lesion located on oral mu-
order: malformation of enamel, thin, pitted, mottled, cosa, MOST on gingival tissues:
snow-capped or yellowish, and poorly calcified. 1. Greater incidence in dark-skinned individuals;
a. Causes: heredity, childhood illness, nutritional nonphysiological pigmentation is also associated
deficiency, drugs. with Addison’s disease, Peutz-Jeghers syndrome,
b. Treatment: often requires placement of esthetic smoking, melanomas.
veneers. 2. Treatment: none, although diagnostic tests for
2. Dentinogenesis imperfecta: autosomal dominant associated diseases and syndromes or to rule out
disorder: defect in the odontoblasts. melanoma may be appropriate.
a. Type I with osteogenesis imperfecta; type II D. Linea alba: asymptomatic, bilateral line (band) of
is indicated by opalescent, blue to brown raised white tissue at level of occlusal plane on buc-
­dentition. cal mucosa; also on the labial mucosa:
b. Noted in both primary and permanent denti- 1. Caused by chronic grinding (bruxism) and/or
tions; may cause enamel to fracture easily be- clenching habit that produces hyperkeratosis and
cause of unsupportive dentin. epithelial hyperplasia.
c. Lack of pulp chamber and root canal formation, 2. Treatment: none, but does help in evaluation of
bell-shaped crowns, severe incisal or occlusal habit.
wear noted on radiographs. E. Leukoedema: generalized, opalescent white film cov-
3. Ectodermal dysplasia: X-linked recessive disorder: ering buccal mucosa bilaterally; caused by cellular
a. Conical or missing incisors. edema in prickle cell layer:
b. Lack of body hair and sweat glands; looks aged 1. MOST in ethnic groups (e.g., African-Americans);
(old person appearance). disappears with pressure on tissues.
4. Dentin dysplasia: autosomal dominant condition 2. Treatment: none.
described as type I or type II: F. Lingual thyroid nodule: small, nodular mass located
a. Type I: short roots with nearly complete oblit- slightly posterior to circumvallate lingual papillae:
eration of pulp noted on radiographs. 1. Caused by entrapped thyroid tissue.
b. Type II: similar to type I on radiographs; how- 2. Treatment: surgical removal possible if concern;
ever, also has large coronal pulps filled with ab- possibly biopsy.
normal dentin. G. Fissured tongue: deep grooves or fissures on dorsal
c. Either type may cause tooth loss because of tongue surface:
short roots and periapical lesions. 1. Increased with certain syndromes such as Down
syndrome.
Oral Variations of Normal 2. Treatment: oral hygiene instruction on cleaning
Oral variants of normal are conditions that are NOT al- tongue.
ways evident but so frequently noted that they are NOT H. Geographic tongue (benign migratory glossitis): map­
considered pathological. like appearance of erythema, surrounded by a white
A. Fordyce granules (Fordyce’s spots): asymptomatic or yellow border on dorsal or lateral surface of the
clusters of yellow lobules of sebaceous glands: tongue as result of desquamation of filiform lingual
1. MOST on buccal mucosa and lower lip; MOST in papillae:
adults ≥20 years. 1. Caused by sensitivity of tissue, MOST likely be-
2. Treatment: none. cause of stress, heredity, or nutritional factors.
B. Tori (single, torus): either palatal or mandibular exo- 2. Treatment: none; may want to stress gentle oral
phytic nodular osseous growth on hard palate or bilat- hygiene of the tongue (must remove all maps of
erally on lingual surface of mandible; asymptomatic, Italy-ah!).
slow growing: I. Median rhomboid glossitis (central papillary atro- o9270
1. Palatal occur MORE often in women (with LESS phy): flat or slightly raised benign lesion on tongue
risk of osteoporosis); mandibular appear on radio- dorsum:
graphs as radiopaque areas in mandibular premo- 1. NO filiform present; related to chronic candiasis o9280
lar region. infection.
2. Etiology related to genetics and possibly grinding 2. Thus no treatment with antifungals because of o9290
(bruxism). ­refractory nature.
200   Saunders Review of Dental Hygiene

o9740 J. Black hairy tongue: hypertrophy (elongation) of fili-    6 Which of the following statements is CORRECT concern-
form lingual papillae on ventral surface: ing tissue regeneration?
o9750 1. May be caused by long-term use of alcoholic A. Regeneration is accomplished by the formation of a
mouthrinses, broad-spectrum antibiotics, steroids; ­fibrous connective tissue scar.
color change may be attributed to bacteria, chemi- B. In terms of tissue function, repair is more desirable than
regeneration.
cals (hydrogen peroxide, antacids), tobacco use,
C. Regeneration is the replacement of damaged cells with
foods, or drugs. parenchymal cells of the same type.
o9750 2. Treatment: oral hygiene instruction on cleaning D. The type of cell injured has little impact on whether the
tongue (without dentifrice); removal of cause; re- tissue heals by regeneration or repair.
ferral to physician, if needed.    7 One of the following is NOT a necessary condition for neo-
K. Lingual varicosities: vascularities that appear as red plasia formation. Which one is the EXCEPTION?
or purple areas on the ventral or lateral surface of the A. Exposure to adequate concentrations of carcinogens
tongue: B. Susceptible host tissue
1. MOST in older adults. C. Adequate time for establishment
2. Treatment: none. D. Existence of a genetic defect
   8 Which of the following statements is CORRECT regarding
the designation for neoplasia?
A. For benign neoplasia, the suffix “-oma” is added to the
Review Questions tissue name.
B. For malignant neoplasia, the suffix “-oma” is added to
the tissue name.
   1 Which of the following is TRUE regarding the cellular C. Benign neoplasia of squamous epithelium is called a
­response to injury? squamous cell carcinoma.
A. Duration of an injury has little impact on the cellular D. Malignant neoplasia of bone tissue is called an osteoma.
response to injury.    9 Which one of the following is considered an endogenous
B. Response to injury is the same in all cell types. carcinogen?
C. Cell necrosis will occur if cell membrane integrity is A. Radiation
compromised. B. Viruses
D. Cell necrosis will occur only if the cellular genetic C. Chemicals
­apparatus is damaged. D. Genetic anomalies
   2 Which of the following causes an irreversible injury that 10 The process by which cells with cancer potential establish
leads to cell death? themselves and give rise to cancer is called
A. Swelling of cytoplasm A. initiation.
B. Fragmentation of nucleus B. transformation.
C. Condensation of cytoplasm C. metastasis.
D. Brief hypoxia D. promotion.
   3 The cardinal signs of inflammation include all of the 11 What is the MOST common site for cancer in women?
­following, EXCEPT one. Which one is the EXCEPTION? A. Lungs
A. Pain B. Uterus
B. Heat C. Breast
C. Redness D. Colon
D. Swelling 12 A 14-year-old boy was hospitalized for an osteosarcoma
E. Odor that metastasized to the lungs. Which one of the following
   4 Which one of the following factors involved in tissue repair statements regarding this case is CORRECT?
is CORRECTLY defined? A. Cancer in the lungs most likely originated in the liver.
A. Angiogenesis is the formation of new blood vessels. B. Osteosarcoma is a benign tumor.
B. Fibrosis is the formation of a new epithelial layer. C. Metastasis of the osteosarcoma was most likely by
C. Remodeling is the formation of a fibrous collagen blood.
patch. D. Metastasis is the best criterion for classifying a tumor as
D. Chemotaxis is the secretion of collagen-degrading benign.
­enzymes.
   5 What circulatory system changes take place during inflam-
mation?
A. Decrease in blood vessel permeability
B. Release of chemical mediators such as histamine
C. Constriction of arterioles
D. Inhibition of diapedesis of leukocytes
General and Oral Pathology   201

13 Which of the following statements concerning psoriasis is 21 Which of the following tumors is considered the MOST
CORRECT? common cause of hyperparathyroidism?
A. Refers to many acute or chronic inflammations of the A. Pituitary adenoma
skin B. Parathyroid adenoma
B. Presents as raised papules or patches with silvery scales C. Thyroid adenoma
and thickened skin D. Medullary carcinoma of adrenal gland
C. Bacterial infection with multiple facial lesions from 22 One of following is NOT a complication of cirrhosis of the
plugged sebaceous glands liver. Which one is the EXCEPTION?
D. Most commonly located on the trunk and neck A. Hyperglycemia
14 A 16-year-old male presents with numerous pus-containing B. Ascites
pimples on his face and neck. All of the following are noted C. Splenomegaly
regarding this condition, EXCEPT one. Which one is the D. Jaundice
EXCEPTION? 23 Characteristics of type 1 diabetes mellitus include all of the
A. Condition is acne of the skin. following, EXCEPT one. Which one is the EXCEPTION?
B. Condition is caused by a viral infection of the sebaceous A. Insulin dependence
glands. B. Mature onset
C. Hormonal factors are involved in this condition. C. Juvenile onset
D. Condition is caused by a bacterial infection of the seba- D. More serious than type 2
ceous glands. 24 Which of the following is a manifestation of chronic pan-
15 Which of the following statements BEST describes the rela- creatitis?
tionship between acne and seborrheic dermatitis? A. Nutrient malabsorption
A. Both acne and seborrheic dermatitis are skin diseases B. Chronic constipation
that affect the sebaceous glands and hair follicles. C. Severe jaundice
B. Both acne and seborrheic dermatitis are bacterial skin D. Diverticula of the intestine
infections. 25 What is another term for hypochromic anemia?
C. Acne is a bacterial skin infection, and seborrheic derma- A. Secondary anemia
titis is a viral skin infection. B. Hemolytic anemia
D. Both acne and seborrheic dermatitis are autoimmune C. Iron-deficiency anemia
disorders. D. Aplastic anemia
16 Which of the following conditions would be considered 26 Which type of anemia is characterized by the rupturing of
with the presence of a yellow, itchy, raised lesion on the erythrocytes?
skin around the face and neck of a 3-year-old? A. Pernicious
A. Acne B. Hypochromic
B. Seborrheic dermatitis C. Aplastic
C. Impetigo D. Hemolytic
D. Psoriasis 27 One of the following statements is NOT correct regarding
17 Pituitary hyperfunction is a characteristic of which of the hemophilia. Which one is the EXCEPTION?
following diseases? A. Low blood hemoglobin concentration is present.
A. Acromegaly B. Genetic blood disease is present.
B. Diabetes insipidus C. Blood disease is characterized by improper blood clot-
C. Graves’ disease ting.
D. Cushing’s syndrome D. Treatment includes transfusion and administration of
18 Which of the following is a manifestation of hypopituitarism? clotting proteins.
A. Oversecretion of growth hormone 28 Patients with type 1 diabetes mellitus
B. Acromegaly A. have non-insulin-dependent diabetes.
C. Hyperactive thyroid gland B. can control the disease with oral insulin.
D. Copious urine production C. are at increased risk for infection.
19 Which of the following glands is associated with Graves’ D. typically have low blood glucose levels.
disease? 29 One of the following is NOT a common oral manifestation
A. Pituitary gland of diabetes mellitus. Which one is the EXCEPTION?
B. Thyroid gland A. Parotid enlargement
C. Pancreas B. Candida infection
D. Adrenal glands C. Xerostomia
20 Which of the following could be considered a causal factor D. Atrophic tongue
for Cushing’s syndrome? E. Periodontitis
A. Administration of glucocorticoid steroids by a physi-
cian
B. Destruction of the thyroid gland as treatment for disease
C. Atrophy of the adrenal glands
D. Autoimmune hypertrophy of the parathyroid glands
202   Saunders Review of Dental Hygiene

30 When treating the patient with diabetes mellitus, a dental 38 Patients who undergo chemotherapy for acute leukemia are
professional should do all of the following, EXCEPT one. NOT at an increased risk for oral infections. All patients
Which one is the EXCEPTION? who undergo chemotherapy for cancer will experience oral
A. Schedule appointments before a meal complications that are directly related to drug activity.
B. Consult with the treating physician A. Both statements are true.
C. Limit the length of the appointment B. Both statements are false.
D. Be prepared for an emergency C. The first statement is true, the second is false.
31 All of the following are correct regarding primary polycy- D. The first statement is false, the second is true.
themia, EXCEPT one. Which one is the EXCEPTION? 39 Angular cheilitis, pale oral mucosa, and glossitis are signs
A. Primary polycythemia is an increase in the number of of
circulating erythrocytes. A. anemia.
B. Hyperviscosity of the blood is a manifestation of pri- B. leukemia.
mary polycythemia. C. hemophilia.
C. Emphysema is a cause of primary polycythemia. D. jaundice.
D. Treatment for primary polycythemia includes ­bloodletting. 40 When treating the patient with hemophilia, the dental pro-
32 Which of the following conditions is noted for bone mar- fessional should do all of the following, EXCEPT one.
row infiltration by malignant cells, an increased number of Which one is the EXCEPTION?
immature leukocytes in the blood, and treatment by chemo- A. Consult with the patient’s physician
therapy? B. Treat with replacement factor
A. Leukemia C. Prevent infectious disease transmission
B. Primary polycythemia D. Instrument in small segments over several appoint-
C. Secondary polycythemia ments
D. Hemophilia E. Scale the entire mouth in one appointment
33 Which of the following is the MOST common oral manifes- 41 The patient with a cardiac condition
tation of renal disease? A. may have difficulty breathing in a supine position.
A. Herpes B. should not be administered local anesthetics.
B. Lichen planus C. is at high risk for oral candidiasis.
C. Halitosis D. may have difficulty communicating.
D. Angular cheilitis 42 For which condition would radiographic diagnosis be con-
34 A medical consult is necessary before treating a patient with clusive?
renal disease. One of the following is NOT a reason for con- A. Alveolar osteitis
sultation for this patient. Which one is the EXCEPTION? B. Periapical abscess
A. Premedication needs C. Periapical granuloma
B. Bleeding tendency D. Condensing osteitis
C. Drug interactions 43 Which of the following lesions closely resembles or is
D. HIV status slightly lighter in color than the tissue on which it is lo-
35 Common causes of respiratory disease include all of the fol- cated?
lowing, EXCEPT one. Which one is the EXCEPTION? A. Pyogenic granuloma
A. Cigarette smoking B. Oral fibroma
B. Allergy C. Papillary hyperplasia
C. Cystic fibrosis D. Peripheral giant cell granuloma
D. Infection 44 Which of the following lesions is noted to be a bilateral,
E. Diabetes mellitus lobulated, exophytic bone located in the lingual mandibular
36 The dental patient with respiratory disease is at risk when premolar area?
exposed to all of the following, EXCEPT one. Which one is A. Condensing osteitis
the EXCEPTION? B. Genial tubercles
A. Aerosols C. Mandibular tori
B. Sulfites D. Ranula
C. Stress 45 Which of the following lesions is observed as a radiopaque
D. Inhalants area at the apex and is often associated with a restored or
37 All of the following are risk factors associated with oral carious tooth?
cancer, EXCEPT one. Which one is the EXCEPTION? A. Periapical granuloma
A. Denture irritation B. Focal sclerosing osteomyelitis
B. Heavy alcohol use C. Periapical abscess
C. Age D. Compound odontoma
D. Tobacco
General and Oral Pathology   203

46 Which of the following is noted clinically as a targetlike 56 Hutchinson’s incisors and mulberry molars are both associ-
lesion? ated with which condition listed below?
A. Lichen planus A. Congenital syphilis
B. Ectodermal dysplasia B. Amelogenesis imperfecta
C. Erythema multiforme C. Ectodermal dysplasia
D. Lupus erythematosus D. Tuberculosis
47 Which of the following is noted for a loss of lingual tooth 57 What developmental disorder is located between tooth #8
structure associated with bulimia? and #9?
A. Attrition A. Turner tooth
B. Erosion B. Dens in dente
C. Bruxism C. Microdont
D. Abrasion D. Mesiodens
48 Which of the following is a cluster of sebaceous glands ob- 58 Improper invagination of the Hertwig’s epithelial root
served on the buccal mucosa? sheath may cause which of the conditions listed below?
A. Lipomas A. Enamel pearl
B. Linea alba B. Peg lateral
C. Mucoceles C. Talon cusp
D. Fordyce granules D. Taurodontism
49 One of the following could NOT occur in response to a hy- 59 Which tooth is MOST likely involved in microdontia?
persensitivity to latex. Which one is the EXCEPTION? A. Maxillary central incisor
A. Contact mucositis B. Mandibular lateral incisor
B. Angioedema C. Maxillary lateral incisor
C. Contact dermatitis D. Mandibular third molar
D. Lichen planus 60 What is the pseudocyst that forms in the mandibular pre-
50 Which term BEST describes a freckle? molar or molar region and is caused by a bone depression
A. Pedunculated adjacent to a salivary gland?
B. Sessile A. Simple bone cyst
C. Macule B. Static bone cyst
D. Papule C. Salivary cyst
51 Which of the following lingual papillae has desquamation D. Traumatic bone cyst
and is associated with geographic tongue? 61 All of the following may cause gingival hyperplasia, EX-
A. Fungiform CEPT one. Which one is the EXCEPTION?
B. Filiform A. Cyclosporine
C. Circumvallate B. Tetracycline
D. Foliate C. Nifedipine
52 Acantholysis and positive Nikolsky’s sign are both as- D. Phenytoin
sociated with which one of the following conditions listed 62 Which of the following X-linked recessive disorders pres-
below? ents with a lack of body hair and sweat glands and may be
A. Lupus erythematosus associated with missing or cone-shaped incisors?
B. Pemphigus vulgaris A. Dentin dysplasia
C. Tuberculosis B. Dentinogenesis imperfecta
D. Pemphigoid C. Amelogenesis imperfecta
53 Which cyst is MOST commonly located between the maxil- D. Hypohidrotic ectodermal dysplasia
lary canine and lateral incisor teeth? 63 Which of the following is associated with the loss of lamina
A. Dentigerous cyst dura and a cotton-wool radiographic appearance of bone?
B. Radicular cyst A. Fibrous dysplasia
C. Globulomaxillary cyst B. Type 1 diabetes mellitus
D. Lateral periodontal cyst C. Osteogenesis imperfecta
54 What is the union of two teeth by cementum? D. Paget’s disease
A. Gemination 64 Which of the following is an autosomal dominant disorder
B. Fusion characterized by opalescent blue or brown crowns?
C. Dilaceration A. Amelogenesis imperfecta
D. Concrescence B. Ectodermal dysplasia
E. Dens in dente C. Dentin dysplasia
55 What is the MOST likely etiology for basal cell carcinoma? D. Dentinogenesis imperfecta
A. Heredity
B. Tobacco
C. Alcohol
D. Sunlight
204   Saunders Review of Dental Hygiene

65 What is a macule? 74 A biopsy allows an examination of both surface and deep or


A. Uncovered wound of cutaneous or mucosal tissue that internal cells of the lesion. Exfoliative cytology allows an
exhibits tissue disintegration and necrosis examination of only the surface cells of the lesion.
B. Soft tissue lesion in which the epithelium above the A. Both statements are true.
basal layer is denuded B. Both statements are false.
C. Permanent mark or cicatrix remaining after a wound C. The first statement is true, the second is false.
heals D. The first statement is false, the second is true.
D. Circumscribed area of mucosa distinguished by color 75 Which one of the following is an advantage of exfoliative
from its surroundings cytology?
66 What is a papule? A. Is effective on keratinized lesions
A. Superficial, elevated, solid lesion that is less than 1 cm B. Indicates need for biopsy
in diameter C. Gives false-negative results
B. Flat, solid raised area that is larger than 1 cm in diam- D. Does not involve cutting into the tissue
eter
C. Solid mass of tissue larger than 1 cm in diameter that
has the dimension of depth
D. Solid mass of tissue smaller than 1 cm in diameter that
Answer Key and Rationales s0470
has the dimension of depth
67 Squamous papilloma presents as what type of lesion in the
oral cavity?
1 (C)  Cellular response to injury depends on multiple
A. Macule factors, including the type, duration, and severity of the
B. Nodule injury; cell type; and biochemical considerations such
C. Plaque as cell membrane, genetic apparatus, protein ­integrity.
D. Papule 2 (B)  Damage to nucleus of the cell and therefore to ge-
68 A circumscribed, fluid-filled elevation in the epithelium that netic apparatus of the cell is considered irreversible cell
is less than 1 cm in diameter is termed a injury. Cytoplasmic changes that occur without loss of
A. pustule. cell membrane integrity are normal signs of reversible
B. vesicle. cell injury. Prolonged hypoxia may cause irreversible
C. bulla. cell damage. Brief hypoxia is manifested as cytoplas-
D. cyst.
mic swelling without damage to cell ­membrane.
69 What is a soft tissue variant of the dentigerous cyst that
forms around an erupting tooth crown called?
3 (E)  Odor is NOT one of the (cardinal) signs of inflam-
A. Eruption cyst mation, which include heat, redness, pain, swelling,
B. Congenital epulis loss of function.
C. Dental lamina cyst 4 (A)  Angiogenesis is the proliferation of local blood
D. Natal teeth vessels into site of tissue damage. Fibrosis is forma-
70 What are teeth that are considerably smaller than normal tion of fibrous collagen patch in and around damaged
called? tissue. Remodeling is generation of collagen-degrad-
A. Macrodontia ing enzymes. Chemotaxis is the movement of white
B. Fusion or gemination blood cells (WBCs) toward chemical substance that
C. Dens invaginatus attracts them.
D. Microdontia
5 (B)  Pathogenesis of inflammation has circulatory
71 A bony exostosis that is located midline of the hard palate is
called what?
and cellular changes; includes increases in blood flow
A. Maxillary abscess and blood vessel permeability because of release of
B. Palatal torus chemical mediators. Cellular changes include emigra-
C. Nasopalatine duct cyst tion (diapedesis) of leukocytes (WBCs) and phagocy-
D. Pleomorphic adenoma tosis.
72 Which of the following is a central nervous system disorder 6 (C)  Injured tissues heal by regeneration and repair.
present with a loss of consciousness? Regeneration is replacement of injured cells with pa-
A. Autism renchymal cells of the same type. Repair is healing
B. Cerebral palsy with formation of a connective tissue scar. Type of
C. Epilepsy cell injured is primary factor that determines whether
D. Mental retardation
tissue healing is by regeneration or repair. Some tis-
73 An aura is commonly associated with
A. epileptic seizure.
sues, such as epithelial tissues, have high capacity for
B. myocardial infarction. regeneration. Other tissues, such as cardiac muscle
C. anorexia nervosa. and nervous tissue, have very limited capacity for
D. Sjögren’s syndrome. ­regeneration and healing by repair.
General and Oral Pathology   205

7 (D)  Neoplasia formation occurs ONLY if certain 17 (A)  Gigantism and acromegaly occur when the pi-
conditions are met, including exposure to adequate tuitary gland oversecretes growth hormone. ­Diabetes
concentrations of carcinogens; susceptible host tis- insipidus is caused by pituitary hyposecretion.
sue; and adequate time for neoplasia initiation, trans- Graves’ disease is caused by hypersecretion of the
formation, and growth. Existence of a genetic defect thyroid gland. Cushing’s syndrome is caused by hy-
is NOT a necessary condition. persecretion of the adrenal glands.
8 (A)  For a benign neoplasia, the suffix “-oma” is 18 (D)  Hypopituitarism has a hypoactive pituitary gland.
added to name of the tissue (with EXCEPTION of Caused by agenesis, atrophy, and/or destruction of
­malignant melanoma, which is ALWAYS cancerous). ­secretory cells in the pituitary gland. Lack of ­antidiuretic
For malignant neoplasia in nonepithelial tissue, term hormone (ADH) from the pituitary leads to copious
­“sarcoma” is added to tissue name. For malignant (large amount of) urine production. Oversecretion of
neoplasia in epithelial tissue, term “carcinoma” is growth hormone, acromegaly, and hyperactive thyroid
added to tissue name. ­Malignant neoplasia of bone is gland are ALL characteristics of hyperpituitarism.
an osteosarcoma. 19 (B)  Graves’ disease is associated with hyperthyroid-
9 (D)  Endogenous carcinogens include genetic factors, ism and is an autoimmune hypertrophy of the thyroid
hormonal imbalances, immunological and nutritional gland. Manifestations (signs) include hypersecretion
factors, and aging. Radiation, viruses, and chemicals of thyroxine, which leads to exophthalmos, tachycar-
are considered exogenous carcinogens. dia, severe weight loss.
10 (B)  Transformation occurs when cells with cancer 20 (A)  Cushing’s syndrome is the hypersecretion of ad-
potential establish themselves and give rise to cancer. renal gland hormones, such as glucocorticoids. Ad-
Initiation occurs when cells undergo an alteration or ministration of glucocorticoid steroids by a physician
series of alterations to acquire autonomous growth in inappropriate amounts could lead to elevated blood
potential. Metastasis is spread of cancerous cells to levels of glucocorticoids and Cushing’s syndrome.
new site. Promotion is NOT part of process of cancer 21 (B)  Parathyroid adenoma is the primary cause of hy-
development. perparathyroidism. Adenoma is a tumor in secretory
11 (C)  MOST common site for malignancy (cancer) in epithelium that often leads to hypersecretion by af-
women is the breast. In men, MOST common site for fected epithelial cells.
malignancy (cancer) is the prostate gland. Second 22 (A)  Hyperglycemia is NOT a complication of cirrho-
MOST common site in both is the lungs. sis of the liver. MOST common complications include
12 (C)  Osteosarcoma is a malignant tumor of bone tis- jaundice, hepatic encephalopathy, ascites (fluid accumu-
sue. Metastasis is the primary criterion for classify- lation in abdominal cavity), splenomegaly, bleeding.
ing a tumor as malignant. Osteosarcomas metastasize 23 (B)  Characteristics of type 1 diabetes mellitus (DM)
by blood, and secondary tumors often develop in the include insulin dependency and juvenile onset, more
lungs. serious than type 2. Mature onset is typical of type 2
13 (B)  Psoriasis is a type of dermatitis with raised pap- DM.
ules or patches with silver scales. Cause is unknown, 24 (A)  Pancreatitis may lead to an inadequate secretion
although tends to run in families. Lesions are MOST of pancreatic juices, necessary for digestion of nutri-
commonly located on extremities and lead to thick- ents in the intestine. Malabsorption is KEY manifes-
ened skin. tation (sign) of chronic pancreatitis.
14 (B)  Condition is MOST likely acne, a bacterial infec- 25 (C)  Anemia is a blood disorder with low blood oxy-
tion of the sebaceous glands. MOST common during gen and hemoglobin levels; there are several types.
adolescence, has hormonal and hereditary compo- Hypochromic has too little hemoglobin per erythro-
nents, and may be aggravated by stress and/or drugs. cyte (red blood cell [RBC]); often is caused by iron
Viruses do NOT cause acne. deficiency. Hemolytic has RBCs that rupture too
15 (A)  Acne and seborrheic dermatitis are BOTH skin easily. Pernicious has too few RBCs present. Aplas-
diseases that affect the sebaceous gland and hair folli- tic has faulty red bone marrow. Secondary occurs as
cles. Acne is caused by bacteria and is thus considered consequence of another condition.
an infectious disease. Seborrheic dermatitis is a multi- 26 (D)  Hemolytic anemia has RBCs that rupture too
factorial disorder and is NOT an infectious disease. easily and is due to hemolysis, abnormal breakdown
16 (C)  Impetigo is a highly contagious infectious disease of RBCs either in blood vessels (intravascular he-
that is common in children. Appears as yellow, itchy, molysis) or elsewhere (extravascular). Numerous
raised pustules around the head and neck. Acne lesions ­possible causes, ranging from relatively harmless to
are raised but are NOT yellow or itchy. ­Seborrheic der- life threatening. General classification is either ac-
matitis exhibits redness, itching, and scaling; psoriasis quired or inherited. Treatment depends on cause and
lesions are raised, scaly patches on the extremities. nature of breakdown.
206   Saunders Review of Dental Hygiene

27 (A)  Hemophilia is the improper clotting of blood. 37 (A)  Denture irritation has NEVER been proved to
Genetic disease with prolonged bleeding after even cause squamous cell carcinoma (SCC). With oral
minor injuries. Treatment includes blood transfusion cancer can experience denture sores or trauma from
and ­ administration of blood-clotting proteins. Low intraoral changes associated with cancer. Age, to-
blood hemoglobin concentration is a characteristic of bacco, and heavy alcohol use have been identified as
­anemia. MAJOR risk factors for oral SCC.
28 (C)  With type 1 DM, patients are at increased risk 38 (B)  Both sentences are false. With acute leukemia,
for infection. Require daily injections of insulin to patient undergoes myelosuppressive chemotherapy,
reduce HIGH blood glucose levels. which causes suppression of the immune system.
29 (D)  Atrophic tongue is NOT a common oral manifes- LESS than half who receive chemotherapy experi-
tation (sign) of DM. Tongue often is edematous, and ence oral complications. Certain chemotherapy drugs
parotid gland may be enlarged. Xerostomia, Candida have little effect on the mucosal tissues and do NOT
infection, periodontitis are common. cause significant immunosuppression.
30 (A)  Schedule appointments for a patient with DM after 39 (A)  Angular cheilitis, pale oral mucosa, glossitis are
morning meal, when blood glucose is MORE stable, to signs of anemia. Leukemia is manifested as gingival
reduce possibility of emergency. Before treatment, med- bleeding, risk for infection, and gingival hypertrophy.
ical consult with treating physician regarding level of Hemophilia is associated with excessive gingival
blood glucose control and need for antibiotic premedica- bleeding. Jaundice is associated with yellow skin and
tion (prophylaxis). Appointments should be short. Keep sclera (white of eye) tones from hemoglobin break-
orange juice or glucose tablets or gel as glucose replace- down and liver disease.
ment in emergency kit in case of DM emergency. 40 (E)  When providing dental care with hemophilia,
31 (C)  Primary polycythemia is oversecretion of eryth- especially when procedures may cause bleeding or
rocytes by hyperactive bone marrow. Factors such as when inflammation is present, the entire dentition
emphysema, heart disease, and living at high altitude should NOT be scaled in one appointment. To mini-
may lead to secondary polycythemia. mize bleeding, instrumentation should occur in a
32 (A)  Leukemia is a cancer of bone marrow. MOST series of small segments over several appointments.
common manifestations (signs) are malignant cells Before treatment, medical consult is needed, replace-
in the bone marrow, increased number of immature ment factor should be given (according to physician’s
leukocytes in the blood, complications such as joint recommendation), and infection control protocols,
pain, unusual bleeding, hepatomegaly, splenomegaly. using standard precautions, should be in place.
Polycythemias involve red blood cells, and hemo- 41 (A)  With a cardiac condition, may have difficulty
philia involves blood-clotting factors. breathing in a supine position, thus patient chair
33 (C)  Halitosis (bad breath) is a COMMON oral mani- should be adjusted upright (45°). Caution should be
festation (sign) of renal disease. Herpes is a common taken when administering local anesthetics because
viral infection that is NOT specifically associated of possible drug interactions; however, MOST indi-
with renal disease. Lichen planus is a chronic inflam- viduals with cardiac conditions have NO difficulty
matory disease associated with systemic diseases with local anesthetics and vasoconstrictors if the lat-
such as hypertension and diabetes mellitus. Angular ter is reduced in amount. Patients with cardiac con-
cheilitis is associated with nutritional deficiency, can- ditions are NOT at high risk for oral candidiasis or
didal infection, and immunocompromised states but communication difficulties.
NOT specifically with renal disease. 42 (D)  Condensing osteitis and focal sclerosing oste-
34 (D) Medical consult is necessary before treating pa- itis have a characteristic radiopaque appearance. Dry
tients with renal disease. May be indication for need socket (alveolar osteitis) would NOT indicate peri-
for antibiotic, steroidal, or antianxiety premedication, apical change. BOTH periapical abscess and peri-
as well as tendency to bleed and any drug interactions. apical granuloma may exhibit similar radiographic
The HIV status is ONLY of peripheral significance. appearance, ranging from slight radiolucency to very
35 (E)  COMMON causes of respiratory disease include distinct periapical radiolucency.
tobacco use such as cigarette smoking, allergy, cys- 43 (B)  Fibroma is similar in color to adjacent tis-
tic fibrosis, and infection. Diabetes mellitus (DM) is sues. Because of increase in vascularity, pyogenic
NOT associated with respiratory disease. ­granuloma, papillary hyperplasia, and peripheral gi-
36 (D)  Inhalers contain medicaments that help dilate the ant cell granuloma are all MORE erythematous than
bronchioles and result in easier breathing. Dental pa- the surrounding tissue.
tient with respiratory disease is at risk for breathing 44 (C)  Mandibular torus is a lobulated overgrowth of
difficulty when exposed to aerosols (from ultrasonic dense bone located in mandibular premolar region. Con-
scaler), sulfites (which provoke asthma), and stress densing osteitis is NOT exophytic; ranula is associated
(which can also provoke asthma). with a blocked salivary gland duct and would be located
General and Oral Pathology   207

in the floor of the mouth; genial tubercles are normal 54 (D)  Two neighboring teeth joined ONLY by ce-
anatomical landmark located inferior to the mandibular mentum is concrescence. Germination is incomplete
central incisors on the lingual surface of the jaw. ­formation of two teeth from one; fusion is combina-
45 (B)  Focal sclerosing osteomyelitis is observed as an tion of BOTH cementum and dentin; dilaceration is
apical radiopacity. BOTH periapical granuloma and abnormal root curvature; dens in dente is invagina-
abscess would be observed as a radiolucency; com- tion of enamel organ during tooth development.
pound odontoma would exhibit collection of tooth- 55 (D)  Basal cell carcinoma (BCC) is associated with
like radiopacities between two teeth. sun exposure. Heredity, alcohol, and tobacco are as-
46 (C)  ONLY erythema multiforme (EM) exhibits tar- sociated with squamous cell carcinoma rather than
getlike lesion on the skin but occasionally on oral basal cell carcinoma. BCC is the MOST common
mucosa. EM is a skin condition of unknown etiology form of skin cancer; can be destructive and disfigur-
but may be mediated by deposition of immune com- ing. Risk for BCC is increased for individuals with a
plex (mostly IgM) in the superficial microvasculature family history of the disease and with high cumula-
of the skin and oral mucous membrane that usually tive exposure to UV light via sunlight or exposure to
follows antecedent infection or drug exposure. carcinogenic chemicals, especially arsenic. Treatment
47 (B)  Chronic bulimia presents with erosion of maxil- is surgery, topical chemotherapy, radiation, cryosur-
lary anterior lingual tooth structure, caused by em- gery, photodynamic therapy; rarely life threatening.
esis (vomiting) of gastric acids (chemical wearing of 56 (A)  Irregularly shaped mandibular incisors (Hutchin-
tooth). Tooth loss associated with attrition and brux- son’s) and mandibular first molars (mulberry) may
ism is caused by physiological wear of the incisal develop as a result of congenital syphilis. Both ecto-
edge or occlusal surface and is NOT associated with dermal dysplasia and amelogenesis imperfecta may
tooth loss because of acid exposure. Tooth loss as- cause defects in enamel, but they are NOT confined
sociated with abrasion occurs at the location of the to incisors and first molars. Lesions associated with
repetitive habit (e.g., cementoenamel junction of the tuberculosis do NOT affect dental hard tissue but in-
buccal surface from toothbrush abrasion). stead soft tissue (Ghon and ulcer lesions).
48 (D)  Fordyce granules (Fordyce’s spots) appear as a 57 (D)  Supernumerary (extra tooth) located between
yellow cluster on the buccal mucosa; this is a nor- the maxillary incisors (#8 and #9) is a mesiodens, a
mal variation. Linea alba, also a normal variation, is a developmental disorder. Microdont is a small tooth
white line on the buccal mucosa corresponding to the that could be located in a variety of locations. Dens-
occlusal plane. Mucocele is a clear vesicle caused by in-dente would NOT be located between central
trauma to minor salivary gland, and lipoma is a single ­incisors.
neoplasia composed of mature fat cells. 58 (D)  Taurodontism is caused by a decrease in nor-
49 (D)  Lichen planus is an autoimmune disease. Con- mal invagination of Hertwig’s epithelial root sheath
tact mucositis, dermatitis, and angioedema are ALL (HERS) and is located ONLY in multirooted teeth.
examples of a hypersensitivity response; may occur Peg lateral is a microdontic maxillary lateral incisor;
after latex exposure in sensitive individual. talon cusp is a tooth with accessory cusp; enamel
50 (C)  Macule is a flat lesion. Pedunculated lesion has a pearl is a small formation of enamel on root surface
stalk. Sessile lesion is a broad-based lesion. Papule is caused by misplaced ameloblast activity.
an elevated lesion. 59 (C)  Maxillary lateral incisor is MOST common
51 (B)  Migratory, reversible desquamation of the fi- microdont; maxillary third molar is also a common
liform lingual papillae is noted with geographic microdont. Both mandibular third molar and lateral
tongue (benign migratory glossitis). No changes in incisor are seldom associated with microdontia.
fungiform, foliate, and circumvallate lingual papilla. 60 (B)  Static bone cyst is identified on a radiograph as a
Name is due to condition resembling a map. small, well-defined radiolucency in the lower poste-
52 (B)  Pemphigus vulgaris is a separation of the epithelial rior region of the mandible, NOT a true cyst because
tissue; removal of epithelium may be noted when gauze it is a bone depression caused by the adjacent salivary
is rubbed across the lesion (positive Nikolsky’s sign). gland and is NOT lined by epithelium. Simple and
Lesions associated with systemic lupus ­erythematosus, traumatic bone cysts are true cysts. Salivary gland
tuberculosis, and pemphigoid do NOT exhibit acanthol- cysts do occur in the molar and premolar areas; NOT
ysis (separation of epithelial cells) and Nikolsky’s sign specific to these areas. Blockage of a salivary gland
is less likely positive for pemphigoid than pemphigus. duct may also be noted with salivary gland cysts.
53 (C)  Globulomaxillary cyst is located between maxil- 61 (B)  Tetracycline taken during tooth formation may
lary canine and lateral incisors. Dentigerous and ra- cause intrinsic tooth discoloration but does NOT cause
dicular cysts may be located in a variety of locations; gingival hyperplasia. Nifedipine (Adalat, Nifedical,
lateral periodontal cyst is located in mandibular ca- Procardia), cyclosporine, and phenytoin (Dilantin)
nine and premolar area. have been shown to cause gingival ­hyperplasia.
208   Saunders Review of Dental Hygiene

62 (D)  Missing or conical incisors are associated with the the oral mucosa, as well as dermis and even subcuta-
hereditary disorder ectodermal dysplasia. Lack of sweat neous tissue. Cysts result from entrapment of epithe-
glands and thinning body hair are also characteristic. lium or remnants of epithelium that grow to produce
Dysplasia of tooth structure is noted in dentin dysplasia, a cavity.
amelogenesis imperfecta, and dentinogenesis imper- 69 (A)  Eruption cyst is soft tissue variant of the dentiger-
fecta; however, cone-shaped incisors and ectodermal ous cyst that forms around an erupting tooth crown.
changes are NOT characteristics of these disorders. Children <10 years are MOST commonly affected.
63 (D)  Radiographic evidence reveals cotton-wool appear- Appears as a small, dome-shaped, translucent swell-
ance and loss of lamina dura with Paget’s disease. With ing overlying an erupting primary tooth. Congenital
type 1 diabetes mellitus, may show evidence of bone epulis is a benign, soft tissue polypoid growth arising
loss associated with periodontal disease; with fibrous from the edentulous alveolar ridge. Remnant of the
dysplasia, may show a diffuse radiopacity. Osteogenesis dental lamina that does NOT develop into a tooth bud
imperfecta presents with abnormally formed bones that may degenerate to form dental lamina cyst. Natal teeth
fracture easily and abnormal dentin formation. are present at birth or erupt within 30 days of birth.
64 (D)  Dentinogenesis imperfecta type II, also known 70 (D)  Microdontia teeth are smaller than normal.
as hereditary opalescent dentin, exhibits crowns that Condition is usually noted bilaterally and is seen in
appear opalescent brown to blue. Anodontia, NOT families; may occur as an isolated finding, in relative
opalescent teeth, is characteristic of ectodermal dys- condition, or in a generalized pattern. Occurs MOST
plasia. Dentin dysplasia (radicular type) exhibits nor- with maxillary lateral incisor (peg lateral) and third
mal tooth color. Coronal dentin dysplasia exhibits molars. Macrodontic teeth are larger than normal. Fu-
amber translucent teeth in the primary dentition and sion is the union of two tooth buds at the level of the
normal color in the permanent dentition. Amelogen- dentin to form one tooth. Germination occurs when
esis imperfecta has poorly calcified, mottled, or thin single tooth bud forms into two teeth (one root with
enamel that appears yellowish brown. canal and two crowns). BOTH fusion and gemination
o14230 65 (D)  Macule is a circumscribed area of mucosa dis- can leave teeth that are larger (false macrodontia).
tinguished by color from its surroundings. Ulcer is an Dens invaginatus (dens in dente) is a developmen-
uncovered wound of cutaneous or mucosal tissue that tal anomaly in which enamel organ invaginates into
exhibits tissue disintegration and necrosis. Erosion is ­dental papilla along lingual aspect; MOST are in
a soft tissue lesion with denuded epithelium above maxillary lateral incisors.
basal layer that leaves a depressed, moist, glistening 71 (B)  Maxillary abscess can form on palate but would
lesion. Scar tissue is permanent mark remaining after consist of soft tissues, UNLIKE the palatal torus,
wound heals. which is hard bony exostosis. Nasopalatine duct cyst
66 (A)  Papule is a superficial, elevated, solid lesion that is heart-shaped midline radiolucency, within incisive
is ≤1 cm in diameter. May be of any color and may be papilla or duct. Pleomorphic adenoma can ALSO
attached by a stalk or firm base. Plaque is a flat, solid form on palate but is a benign salivary gland tumor
raised area that is ≥1 cm in diameter. Tumor is a solid with mixture of epithelial and connective tissue cells.
mass of tissue larger than 1 cm in diameter that has 72 (C)  Loss of consciousness is associated with epi-
the dimension of depth. Nodule is a solid mass of tissue lepsy, as are convulsive episodes. Individuals with
≤1 cm in diameter that has the dimension of depth. cerebral palsy, mental retardation (intellectual dis-
67 (D)  Squamous oral papilloma presents a papule type ability), or autism alone are NOT prone to convul-
of lesion. Papule is a superficial, elevated, solid le- sions or loss of consciousness.
sion that is ≤1 cm in diameter. May be of any color 73 (A)  Aura (predication) often precedes an epileptic sei-
and may be attached by a stalk or firm base. Mac- zure and is NOT typically associated with myocardial
ule is a circumscribed area of mucosa distinguished infarction, anorexia nervosa, or Sjögren’s syndrome.
by color from its surroundings, such as a melanoma. 74 (A)  Both statements are true. Biopsy examination
Nodule is a solid mass of tissue that is ≤1 cm in diam- includes BOTH surface and deep internal cells of le-
eter and has the dimension of depth. Plaque is a flat, sion. Exfoliative cytology permits ONLY examination
solid raised area that is ≥1 cm in diameter like early of surface cells, allowing potential for false negative.
lesion of basal cell carcinoma. Statements highlight one of MAIN differences between
68 (B)  Vesicle is a circumscribed, fluid-filled elevation the two types of tests and show advantage of biopsy.
in the epithelium that is <1 cm in diameter. Fluid of 75 (D)  Patients respond favorably to exfoliative cytol-
the vesicle consists of lymph or serum but may con- ogy, since it does NOT involve cutting the tissue.
tain blood. Pustule is a circumscribed elevation filled Unfortunately, it has a number of significant disad-
with purulent exudate resulting from an infection, ≤1 cm vantages as a diagnostic tool, such as limitations on
in diameter. When diameter of a vesicle is ≥1 cm, it kinds of lesions evaluated and frequent false nega-
is a bulla. Cyst is an epithelially lined mass located in tives, and is NOT considered definitive.
CHAPTER  7

Nutrition

NUTRIENTS AND DEFICIENCIES   A. Types:


Nutrients are substances obtained from food and pro- 1. Amino acids (20) held together by peptide bonds;
mote growth, maintenance, or repair. Six classes of nutri- carboxyl (COOH) group of one amino acid is
ents include macronutrients (biomolecules) of proteins, linked to an amino (NH2) group of another.
carbohydrates, and lipids, as well as vitamins, minerals, a. The 9 essential (indispensable) amino acids
and water. For Acceptable Macronutrient Distribu- (EAAs) CANNOT be synthesized by the body
tion Ranges (AMDRs), adults should obtain 10% to and must be obtained from the diet; nitrogen
35% of calories from protein, 45% to 65% from carbo- balance can be achieved from proper propor-
hydrates, and 20% to 35% from lipids (fat). Acceptable tions and adequate amounts of these.
ranges for children are SIMILAR, except that infants b. The 11 nonessential (dispensable) amino ac-
and younger children need higher proportion of fat (25% ids (NEAAs) can be synthesized by the body
to 40%). ONLY when nitrogen is present.
For vitamins and minerals, Dietary Reference Intakes 2. Complete proteins (high-quality): foods that con-
(DRIs), issued by the Institute of Medicine of the Na- sist of ALL the essential amino acids in sufficient
tional Academy, are the average daily dietary intake level amounts; completely supply the needs of the body
sufficient to meet nutrient requirements of nearly ALL for maintenance, repair, and growth; are found in
(~98%) healthy individuals (includes Recommended Di- meat, fish, poultry, eggs, cheese, milk.
etary [Daily] Allowance [RDA]) (Table 7-1). RDIs (Ref- 3. Incomplete proteins (low-quality): foods that lack
erence Daily Intakes) and DVRs (Daily Recommended one or more essential amino acids; include plant
Values) are comparable standards and can be combined to proteins; when ONLY food eaten, can support life
create DVs (Daily Values), as published by the FDA. but NOT normal growth.
Malnutrition is an imbalance between nutrients the 4. Complementary proteins: two or more proteins
body needs and nutrients it receives. Includes overnu- that combine to compensate for deficiencies
trition (consumption of too many calories or too much in amino acid content; use of whole grains
of specific nutrient) and undernutrition (deficiency IMPORTANT.
MAINLY of calories or protein). Deficiencies of vitamins B. Physiology and metabolism of proteins: see Chapter 3,
and minerals are usually considered separate disorders. Anatomy, Biochemistry, and Physiology
When calories are deficient, however, vitamins and min- C. Functions of proteins:
erals are also likely to be. Calorie or kilocalorie (kcal) is 1. Role in overall body needs; assist in growth, main-
a unit of measurement for energy, in this case food en- tenance, construction, and repair of body tisue.
ergy. Energy is measured by heat expenditure; 1 kilocalo- 2. Provide essential components of hormones and
rie is the amount of heat produced to raise temperature of ­enzymes and provide precursors of antibodies.
1 kilogram of water 1° C. 3. Provide energy source when carbohydrate and fat
• See CD-ROM for Chapter Terms and WebLinks. intake is inadequate: 4 kcal per gram (g).
• See Chapters 3, Anatomy, Biochemistry, and Physiol- 4. Regulate acid-base balance.
ogy: biomolecules; 6, General Pathology, and 16, Spe- D. Role in oral health:
cial Needs Patient Care: nutrition-related conditions 1. Essential for cell growth; thus IMPORTANT in
and diseases. growth and development of the oral cavity.
2. Maintain pulp.
Proteins 3. If absent, result is crowded and/or rotated teeth
Proteins are organic compounds that are composed of ­because of inadequate growth of bone.
amino acids, building blocks of proteins. Contain carbon, 4. Deficiency can lead to disturbances in tooth de-
hydrogen, oxygen, nitrogen. MAIN function is to build velopment such as with kwashiorkor or marasmus
tissue and replace cells. Presence of proteins affects ALL (see later discussion) or damage to the periodon-
body activities. tium (as with kwashiorkor).

209
210 Saunders Review of Dental Hygiene

Table 7-1  Acceptable macronutrient distribution


ranges (AMDRs) and dietary reference lntakes
(DRIs) for adults per day*

Macronutrient Females Males


Protein 46 g 56 g
Carbohydrates 130 g Same
Lipids None Same
Vitamin A (retinol) 700 μg 900 μg
Vitamin D 5 μg (AI)† Same
Vitamin E 15 mg‡ Same
Vitamin K 90 μg (AI) 120 μg
Thiamine (vitamin B1) 1.1 mg 1.2 mg
Riboflavin (vitamin B2) 1.1 mg 1.3 mg
Niacin (vitamin B3) 14 mg 16 mg
Pantothenic acid 5 mg (AI) Same
(vitamin B5)
Pyridoxine (vitamin B6) 1.3 mg Same
Cobalamin (vitamin B12) 2.4 μg Same
Vitamin C (ascorbic acid) 75 mg§ 90 mg§
Biotin 30 μg (AI) Same
Folate (folacin, folic acid) 400 μg Same Figure 7-1  Kwashiorkor is a severe protein deficiency even
with calories from carbohydrates, without severe wasting of
Calcium 1000 mg (AI) Same body fat, but with apathy, failure to grow, and edema.
Phosphorus 700 mg Same
Magnesium 310 mg 400 mg G. Nutrient sources of proteins:
Sodium 500 mg Same 1. Complete proteins: meat, fish, poultry, eggs, milk.
Chloride 750 mg Same 2. Incomplete proteins: legumes, grains, vegetables,
Potassium 2000 mg Same soybeans.
H. Nutritional deficiency and diseases related to proteins:
Sulfur None Same
1. Phenylketonuria (PKU):
AI, Adequate intake. a. Liver CANNOT metabolize essential amino
*Includes adequate intake or Estimated Average Requirement (EAR), where

NO DRI has been established, expected to satisfy needs of 50%.


acid phenylalanine into nonessential amino
†5 μg = 200 IU (International Units).
acid tyrosine.
‡Tolerable upper intake limit (UL) is 1000 mg/day.
§Smokers should add 35 mg to these values. b. Toxic by-products build up in the body and dam-
age developing nervous system of infant, caus-
ing intellectual disability (mental retardation).
5.  Can help neutralize acids produced by dental bio- c. Managed by restricting phenylalanine to only
film (dental plaque). enough to support normal growth; supplement
E. Role in periodontal health or disease: tyrosine; avoid aspartame (Nutrasweet, Equal).
1. IMPORTANT in maintaining health of periodon- 2. Protein-energy (calorie) malnutrition (PEM, PCM)
tium. or undernutrition:
2. Essential for cellular defenses against bacteria in a. Marasmus: inadequate food intake; MOST
subgingival dental biofilm. common in children 6 to 18 months of age in im-
3. Essential for healing and repair of tissue after peri- poverished nations; impairs brain development
odontal trauma or surgery. and learning; MAINLY causes severe wasting
F. Dietary requirements for proteins: 0.8 g per kg of and weakening of muscles; NO edema.
body weight for adults: b. Kwashiorkor: severe protein deficiency even
1. DRI for adults: 46 g (female) and 56 g (male) per day. with calories from carbohydrates; begins at ap-
2. Increased during periods of growth, pregnancy, proximately 2 years; WITHOUT severe wasting
lactation. of body fat; symptoms include apathy, failure to
3. Increased during episodes of trauma, fear, anxiety, grow and gain weight, listlessness, changes in
surgery, fever. hair color, edema in abdomen and legs (causes
Nutrition 211

swelling); associated with necrotizing peri- serves MAINLY as fuel for the brain; found
odontal disease (Figure 7-1). in honey, fruits, corn syrup; however, ALL
c. Starvation: MOST extreme form of marasmus sugars are converted to glucose, so do NOT
(and undernutrition), results from partial or total need these to feed the brain.
lack of essential nutrients for a long time (see self- (4)  Fructose (levulose, fruit sugar) is the
starvation under patient with eating disorder). sweetest of ALL sugars; closely related to
d. Cachexia: severe wasting away of muscle and glucose structurally; found in honey, fruits,
fat tissue as a result of excessive production of corn syrup; changed to lactic acid by Strep-
substances called cytokines, produced by im- tococcus mutans.
mune system in response to a disorder; MOST (5)  Galactose, derived from hydrolysis of lac-
common with cancer and AIDS. tose; constituent of many plant polysac-
e. Adult PEM (PCM): alcoholics (with nutritional charides; during lactation, body converts
liver disease), long-term bedridden or hospital glucose to galactose in mammary tissue to
patients, persons taking certain drugs that de- synthesize lactose in breast milk.
crease appetite (diuretics) or increase metabo- b. Disaccharides (double sugars; two monosac-
lism (thyroxine). charides):
(1)  Oral effects of PEM (PCM): (1)  Lactose (milk sugar) is composed of glu-
�������������������������������������������
   ����������������������������������������
(a) Tooth development decreased. cose and galactose; obtained from milk
����������������������������������������
    ������������������������������������
(b) Salivary composition and flow ����������������
affected. products.
�������������������������������������������������
   ����������������������������������������������
(c) Connective tissue and bone develop- (2)  Sucrose (table sugar; cane and beet sugar)
ment affected. composed of glucose and fructose; ob-
����������������������������������������������
   �������������������������������������������
(d) Acid production increased. tained from sugar beets, sugarcane, or ma-
3. Overconsumption of proteins (e.g., Atkins diet, ple syrup.
with low carbohydrates): (3)  Maltose (plant sugar) is composed of two
a. May affect calcium balance if inadequate molecules of glucose; forms as starch in
­calcium. grains and breaks down during germina-
b. Increase in water consumption. tion; ferments alcohol.
c. Additional fat stores. 2. Complex carbohydrates (long chains of sugars):
d. Stress to liver and kidneys owing to presence of a. Polysaccharides:
excess ketones, especially if already damaged. (1) Starch (mixture of amylose and amylo-
e. Halitosis. pectin) is the MOST important carbohy-
4. Possible deficiency in poorly planned strict vegan drate. Digestible by humans; stores energy
diets (consisting ONLY of plant foods); adequate in plants; breaks down at slower rate than
as long as caloric intake is adequate and variety of monosaccharides and disaccharides; found
foods is eaten. in rice, wheat, corn, rye, potatoes, legumes.
a. Easier to achieve with diets that include dairy (2) Glycogen is the animal equivalent of starch;
products and eggs (lacto-ovo-vegetarian). provides food storage system for ALL forms
b. Harder to achieve with diets consisting ONLY of animal life; stored in liver, where it regu-
of fruits and vegetable oils (fruitarian diets). lates blood sugar, and in muscle, where it
serves as an energy source for muscle con-
Carbohydrates traction.
Carbohydrates are organic compounds (contain carbon)  �����������������������������������������������������
  ���������������������������������������������������
(3) Insulin is produced by pancreas; stores
that also contain the elements hydrogen and oxygen. Pro- energy.
vide energy during metabolism (you burn 20 calories per (4) Dextran forms a substrate for dental biofilm
hour chewing gum). (dental plaque); serves as energy source for
A. Types of carbohydrates: dental caries–producing bacteria such as
1. Simple carbohydrates: Streptococcus mutans; makes dental bio-
a. Monosaccharides (single sugars): film sticky.
(1)  Pentoses (C5) act as coenzymes in energy b. Fibers:
production (e.g., ribose, deoxyribose). (1)  Cellulose: provides fibrous framework for
(2)  Hexoses (C6) have MAJOR nutritional im- plants; good source of fiber; NOT digest-
portance. ible and therefore provides roughage and
(3)  Glucose (blood sugar or dextrose) is the bulk to aid in peristalsis and elimination
form used MOST efficiently by the body; of water; found in fruits, legumes, ALL
obtained from ALL plant carbohydrates; vegetables.
212 Saunders Review of Dental Hygiene

(2)  Hemicellulose: group of fibers that are in- 7. Alternative sweeteners:


soluble (do NOT easily dissolve in water); a. Sugar alcohols are sugarlike compounds (nutri-
MAIN constituent of cereal fibers. tive sweeteners) that, like carbohydrates, yield
(3)  Pectin: responsible for thickening of 4 kcal/g of energy; metabolized MORE slowly
jams and fruit preserves; keeps salad dress- than sucrose by bacteria in the mouth and
ing from separating; found in vegetables and therefore do NOT promote caries; CANNOT
fruits, especially citrus fruits and apples. be used at same time as disulfiram (Antabuse)
B. Physiology and metabolism of carbohydrates: see prescribed for treatment of alcohol abuse and
Chapter 3, Anatomy, Biochemistry, and Physiology. alcohol dependence.
C. Functions of carbohydrates: b. Xylitol: derived from cellulose products such as
1. Provide energy (4 kcal/g). wood straw or pulp cane; may produce diarrhea
2. Spare proteins so they can supply energy when when taken in excess; equivalent to sucrose in
necessary, but MAIN function is to build tissue, sweetness; used in sugarless gums and foods,
replace cells. since does NOT promote caries; also reduces
3. Aid in oxidation (burning) of fats to prevent keto- bacterial tooth adherence and secretion of acids
sis (partial fat breakdown; leads to increased ke- and promotes tooth remineralization; must fol-
tone levels in bloodstream). low manufacturer’s instructions when used for
4. Furnish fiber for peristalsis; also provide bulk, re- anticaries purposes.
duce risk of cardiovascular disease, prevent consti- c. Sorbitol: from glucose by hydrogenation; used
pation and diverticular disease. in foods, especially diabetic foods and sugar-
5. Aid in formation of intercellular substance (ground less gums; NOT readily absorbed by the small
substance) and collagen. intestine and therefore can cause diarrhea.
6. Aid in formation of nonessential amino acids. d. Mannitol: from mannose and galactose by
D. Dietary requirements for carbohydrates: hydrogenation; used in sugarless gums and
1. DRI: 130 g/day; infants require MORE carbohy- candies; NOT readily absorbed by the small in-
drates to prevent use of protein for energy. testine; may cause diarrhea.
2. Maximum intake level of ≤25% from added 8. Artificial sweeteners include noncarbohydrate,
­sugars. noncaloric, or low caloric:
3. Dietary fiber recommended intake: ≤50 years 25 g a. Sucralose (Splenda): 600 times sweeter than su-
(female) and 38 g (male)/day; >50 years is 30 and crose, twice as sweet as saccharin and four times
21 g/day, respectively, owing to decreased food as sweet as aspartame; stable under heat and can
consumption. be used in baking and products that require lon-
E. Nutrient sources of carbohydrates: ger shelf-life; NOT recognized by body as car-
1. Milk, cereals, breads provide starch. bohydrate, so does NOT affect blood glucose
2. Leafy vegetables provide cellulose and hemicellu- levels or insulin response (useful for diabetics);
lose. essentially inert molecule that passes through
3. Vegetables such as root tuber and seed (potatoes, body without being broken down for calories.
beans, beets, squash) provide starch and sucrose. b. Aspartame (Nutrasweet, Equal): composed of
4. Fruits provide MAINLY glucose and fructose. two amino acids (phenylalanine and aspartic
5. Sugar, honey, corn syrups provide monosaccha- acid) and methanol; used ONLY in beverages,
rides and disaccharides. gelatin desserts, and sugarless gum; CANNOT
6. Sweeteners: be used in baking (low-calorie sweetener yields
a. Sucrose: disaccharide found in table sugar, beet 4 kcal/g).
sugar, cane sugar. c. Saccharin: derived from coal-tar compounds;
b. High-fructose corn syrup (HFCS): 40% to 90% 500 times sweeter than sucrose; widely used
fructose; made by treating corn starch with acid in soft drinks and table sweeteners; may cause
and enzymes to break it down into glucose, cancer, and therefore products must contain a
which then is changed into fructose; possibly warning label.
tied to obesity. d. Acesulfame-K: 200 times sweeter than sucrose;
c. Turbinado sugar: partially refined version of used in sugarless gum, powdered drink mixes,
raw sugar; has a slight molasses flavor. gelatin puddings, nondairy creamers; can be
d. Brown sugar: white sugar that contains some used in baking.
molasses; molasses is either added or not totally F. Nutritional management of disease and deficiency o1120
removed. ­related to carbohydrates. See Chapter 6, General and
e. Maple syrup: boiled sap from sugar maple trees. Oral Pathology.
Nutrition 213

1. Diabetes mellitus (DM): metabolic endocrine dis- 4. Dental caries:


order with high blood glucose level as result of in- a. Dental biofilm (dental plaque) bacteria break
sufficient or ineffective insulin function; results in down sucrose and glucose into lactic acid as
tissue damage; diet considerations will be guided result of enzymatic activity of streptococci
by diabetic nutrition counselor. and lactobacilli.
a. Non-insulin-dependent (MAINLY type 2): (1) Streptococcus mutans initiates process;
(1)  Restrict calories and reduce weight. synthesizes polysaccharides (glycogen,
(2)  Engage in regular exercise (assists muscles dextran, levan) for future fermentation.
in taking up MORE glucose).  ��������������������������������������������������
  ������������������������������������������������
 ����������������������������������������������
(a) Polysaccharides are used for energy
(3)  Space meals evenly throughout the day. when needed.
(4)  Avoid alcohol consumption. ���������������������������������������������������
   ������������������������������������������������
(b) Dextrans form substrate and serve as
b. Insulin-dependent (MAINLY type 1): energy source.
(1)  Requires careful nutritional assessment; �����������������������������������������������������
   ��������������������������������������������������
(c) Demineralization begins at 5.5 pH.
SHOULD eat regular meals with precise (2) Lactobacillus acidophilus extends pro-
carbohydrate-to-protein-to-fat ratio. cess.
(2)  Intensive insulin therapy, including regular b. Fermentable carbohydrate ingestion: concen-
blood glucose monitoring. tration of sugars in food is KEY factor in car-
(3)  Regular exercise. ies development.
2. Reactive hypoglycemia (low blood glucose (1) Sugar-rich foods: cariogenic activity
­levels): of sucrose, glucose, fructose is similar;
a. Occurs in reaction to ingestion of food, 1 to physical form of food also affects activ-
4 hours after meal: ity (oral clearance time of liquids is faster
(1)  Initial symptoms include weakness, rapid than that of solids and retentive sweets;
heartbeat, sweating, anxiety, hunger; thus sugared liquids are generally less car-
caused by release of epinephrine (hor- iogenic).
mone), which is triggered by falling blood (2) Concentration of 0.8 M must be present
glucose level. for su­gar to pass through 1 mm of dental
(2)  Fasting sets in gradually and affects the biofilm and undergo harmful fermenta-
brain and central nervous system (CNS); tion (e.g., 1-M solution of sugar contains
symptoms then include headache, fatigue, 342.3 g of sugar in water).
blurred vision, confusion; are NOT related (3) Starch-rich foods: if left on teeth for long
to epinephrine release. periods, will degrade to organic acids and
b. Prevention: eat regular meals with protein, fat, can contribute to acid levels.
complex carbohydrates that contain soluble fi- c. Salivary flow rate: inadequate salivary flow (e.g.,
ber; AVOID concentrated sweets. xerostomia) interferes with oral clearance time
c. Management: eat balanced meals at regular in- of cariogenic foods, decreasing natural salivary
tervals (this is the diet for NDHBE time!). buffers and initiating caries development.
3. Lactose intolerance: intestinal condition with re- d. Daily food intake frequency: development re-
duced lactose digestion because of deficiency of lated to between-meal snacking; eating sweets
digestive enzyme lactase, which hydrolyzes lac- with a meal makes them less cariogenic.
tose to glucose and galactose in the intestine.
a. When lactase is absent in small intestine, lac- clinical study   s9000
tose travels to large intestine, where bacteria Scenario: The 8-year-old patient is in the dental office p9000
break it down into gas and acids. for a 6-month oral prophylaxis maintenance appointment.
b. Symptoms: cramps, abdominal distention, gas When asked more about the child’s health, the mother
symptoms, diarrhea after consumption of dairy said the girl even wet her bed a couple of nights ago. The
products. mother also notes that she has recently had an increased
c. Management: appetite yet has lost weight. “My daughter had a severe
(1)  Eat smaller servings of milk products. chest cold about 2 months ago and now she is waking up
(2)  Include fat in meals; slows digestion and frequently during the night to urinate.”
leaves MORE time for lactase action.
(3)  Eat cheese; lactose is lost when milk is pro- 1. What is the most likely diagnosis of the patient’s
cessed into cheese. ­condition?
(4)  Consume yogurt with active bacteria cul- 2. Identify the usual causes and predisposing factors for
tures; lactose is digested by yogurt. this condition.
214 Saunders Review of Dental Hygiene

3. What is the best treatment plan for this condition? (2)  Omega-3 fatty acids are linolenic; found in
4. Is this condition preventable? And if so, how? fatty fish oils, green leafy vegetables, soy-
5. What type of nutritional management would be bean products (tofu).
­recommended for the patient? d. Essential fatty acids (i.e., linoleic and linole-
nic acids) must be obtained from diet; regu-
1. Type 1 diabetes mellitus (DM). late blood pressure, assist in clot formation,
2. Condition that may trigger onset of type 1 DM is a maintain cell membranes, make hormone-
viral infection. Infection may affect immune response like substances, support functioning of the
that attacks the pancreas, leading to inability to make retina.
insulin. Symptoms include polyuria (frequent urina- 2. Compound lipids are compounds added to glycerol
tion), increased appetite (polyphagia), thirst (polydip- and fatty acids.
sia), weight loss. a. Phospholipids are built on backbone of glycerol,
3. Treatment plan would include insulin therapy, re­ with one fatty acid replaced by a compound that
gular blood glucose monitoring, careful nutritional contains phosphorus.
­assessment. (1)  Lecithin is found in cells and participates
4. DM cannot be prevented but can be controlled by in fat digestion in the intestine; acts as an
drugs and diet. Although viral infection may play a emulsifier by mixing oil with water (e.g., in
part in the development of type 1 DM, its determina- salad dressings, mayonnaise); found in egg
tion as a cause cannot always be made. yolk.
5. The patient requires careful nutritional assess- b. Lipoproteins are produced in the liver and al-
ment by a diabetic nutrition counselor; should eat low cholesterol, triglycerides, phospholipids to
regular meals with precise carbohydrate-to-pro- be transported in the bloodstream.
tein-to-fat ratio; needs intensive insulin therapy, (1)  Chylomicrons are made in the intestine af-
including regular blood glucose monitoring and ter fat absorption; contain triglycerides and
regular ­exercise. a small amount of protein.
���������������������������������������������������
   ������������������������������������������������
(a) Transport newly absorbed lipids from
s0070 Lipids intestinal cells to the bloodstream by
p0080 Lipids (fats) are organic compounds composed of car- way of the lymphatic system.
bon, hydrogen, oxygen. Types include triglycerides (fats �����������������������������������������������
   ��������������������������������������������
(b) During circulation, cells remove
and oils), phospholipids, sterols (cholesterol). MAIN lipid content and therefore become
function is to provide energy. smaller.
A. Types of lipids:    �����������������������������������������������
(c) Liver picks up and breaks down chylo-
1. True fats are composed of glycerol (trihydroxy al- microns and assembles new lipoproteins
cohol) that is attached to one, two, or three fatty known as very low-density lipoproteins
acids to form monoglycerides, diglycerides, or tri- (VLDLs).
glycerides, respectively. (2)  VLDLs: the liver coats cholesterol and
a. Saturated fatty acid carries maximum number triglycerides with a shell of protein and
of hydrogen atoms; remains solid at room tem- ­lipids.
perature; increases serum cholesterol. ���������������������������������������������������
   ������������������������������������������������
(a) VLDLs leave the liver, and triglycer-
(1) Stearic acid in beef and lard. ides are broken down into fatty acids
(2) Palmitic acid in animal fat and palm oil. and glycerol by the enzyme lipase.
(3) Myristic and lauric acid in coconut oil. �����������������������������������������������
   ��������������������������������������������
(b) VLDLs become much heavier as tri-
b. Monounsaturated fatty acid contains a point glycerides are released.
of unsaturated linkage (with NO hydrogen ��������������������������������������������������
   �����������������������������������������������
(c) VLDLs gather cholesterol from other
atom); viscous in form; NO effect on serum lipoproteins and become low-density
cholesterol. lipoproteins (LDLs).
(1)  Oleic acid sources include olive oil, short- (3)  Low-density lipoproteins (LDLs): com­
ening, lamb, canola oils. p­osed MOSTLY of cholesterol; contain a
c. Polyunsaturated fatty acid contains two or few triglycerides; larger, lighter, MORE
more points of unsaturation; liquid in con- lipid filled than VLDLs (“bad guys”) (see
sistency (includes oils); decreases serum note with nicotinic acid later).
­cholesterol. (a) Cells absorb LDLs from the bloodstream o1850
�����������������������������������������������������
   ��������������������������������������������������
(1)  Omega-6 fatty acids are linoleic; found in and break them down;   ���������������
MOST LDLs
corn, cottonseed, safflower, and sunflower are taken up by liver cells when a diet is
oils, tofu. low in saturated fats and cholesterol.
Nutrition 215

o1870 �����������������������������������������������������
 ����������������������������������������������������
 (b) Scavenger cells in blood vessels engulf 3. Polyunsaturated: ~10% total kcal.
LDLs that are NOT taken up; over time, 4. Essential fatty acids: 1% to 3% total kcal.
cholesterol builds up on inner blood ves- 5. Cholesterol: ≤300 mg/day.
sel walls and plaque develops. E. Nutrient sources of lipids:
o1890 �������������   �����������
(c) Blood ���������������������������������������
supply to organs is subsequently 1. Saturated: animal fat and palm oil.
cut off (ischemia), which may result 2. Monounsaturated: olive oil, shortening, lamb.
in cardiovascular disease (CVD), myo- 3. Polyunsaturated: soybean, cottonseed, and veg-
cardial infarction (MI, heart attack), or etable oils.
cerebrovascular accident (CVA, stroke). 4. Cholesterol: animal foods.
(4)  High-density lipoproteins (HDLs): produced 5. Essential fatty acids: vegetable oils; recommenda-
by liver and intestine; smaller, denser; con- tion is 1 tablespoon per day.
tain high amount of protein (“good guys”). F. Nutritional management of deficiency and disease re-
������������������������������������������������
  ����������������������������������������������
(a) HDLs travel in the bloodstream and lated to lipids:
pick up cholesterol. 1. Atherosclerosis: degenerative disease that pro-
  �����������������������������������������������������
(b) Cholesterol is transported by way of other duces hardening of large and medium arteries.
lipoproteins back to liver for excretion. a. Formation of plaques:
�������������������������������������������������
  �����������������������������������������������
(c) High HDL levels slow development of (1)  Includes formation of atheromata, deposits
CVD. on inner walls of arteries.
3. Cholesterol (sterol): (2)  With age, plaque becomes fibrotic and nar-
a. Capable of forming esters with fatty acids. rows blood vessels.
b. Makes IMPORTANT hormones (estrogen, tes- (3)  Eventually ischemia occurs, leading to de-
tosterone). creased blood supply, development of CVD
c. Makes bile, emulsifier needed for digestion. with complications.
d. Found ONLY in animal foods. b. Risk factors:
e. Liver can make what the body needs; high lev- (1)  Total blood cholesterol >240 mg/100 mL.
els raise total plasma cholesterol level. (2)  LDL cholesterol >160 mg/100 mL.
4. Fat replacements (artificial fats): (3)  HDL cholesterol <35 mg/100 mL.
a. Olestra: made by adding fatty acids to sugar; yields (4)  Men >45 years; women >55 years.
NO energy to the body; may lower blood serum (5)  Smoking and family history.
cholesterol levels; reduces absorption of vitamin E; (6)  Hypertension.
has NO kilocalories; may cause ­diarrhea. c. Management:
b. Simplesse: whey protein product; feels like fat (1)  Reduce saturated fat in diet (intake <10%
in the mouth but contains NO fatty acids; CAN- of total kilocalories lowers LDL choles-
NOT be used in cooking; contains 1.3 kcal/g. terol level); maintain polyunsaturated and
c. Hydrogenation: process that reduces amount of monounsaturated fat intake at ~10% of to-
fat per serving (e.g., diet margarine). tal kilocalories.
d. Gums: derived from plant sources and added to (2)  Increase consumption of dietary fiber (20 to
thicken products such as diet salad dressings. 35 g per day).
o2050 B. Physiology and metabolism of lipids: see Chapter 3, (3)  Lose weight; then maintain desirable body
Anatomy, Biochemistry, and Physiology, and Chapter 9, weight.
Pharmacology, for control of LDLs. (4)  Stop smoking.
C. Functions of lipids: (5)  Eat LESS cholesterol (<300 mg per day).
o2070 1. Provide energy (9 kcal/g) (used for energy by body (6)  Replace foods rich in animal fat, butter, co- o2440
after readily available carbohydrates are used up). conut oil, hydrogenated (solid) fat with LESS
2. Store energy. hydrogenated forms; low-fat (or fat-free, non-
3. Insulate to maintain body temperature and protect fat in some cases) food recommendations:
organs.   ����������������������������������������������������
(a) Meat, fish, poultry: lean cuts; trim fat;
4. Transport fat-soluble vitamins to small intestine grill, roast, or bake; tuna packed in water.
and aid in absorption.   �������������������������������������������������
(b) Dairy: low-fat or nonfat milk, cheese,
5. Produce satiety (feeling of fullness). yogurt, nonfat frozen yogurt.
6. Provide flavor and texture to foods.   ����������������������������������������������������
(c) Fruits and vegetables: nonfat salad dress-
D. Dietary requirements of lipids: ≤25% to 35% of total ing; steamed vegetables; fruit for dessert.
kcal (AMDR); NO DRI. (d) Breads and cereals: jelly on bread in-
1. Saturated: <10% total kcal. stead of butter or margarine; avoid
2. Monounsaturated: ~10% total kcal. croissants, coffee cake, sweet rolls.
216 Saunders Review of Dental Hygiene

2. Obesity (see weight control section): 2. Vitamin D (calciferol):


a. Decrease fat and calorie intake. a. Functions as a hormone:
b. Excess weight of 30% increases risk of CVD, (1)  Absorbing calcium and phosphorus.
high blood pressure (HBP), DM, cancer. (2)  Promoting mineralization of teeth and
bones.
Vitamins (3)  Regulating proper serum levels of calcium
Vitamins are organic nutrients needed by the body in and phosphorus.
small quantities. They do NOT contribute energy to the b. DRI: 5 μg (adequate intake [AI]).
body but are facilitators of body processes. Fat-soluble c. Nutrient sources:
vitamins include A, D, E, and K; water-soluble vitamins (1)  Sunlight: liver makes a vitamin D precursor
include the Bs and C. Note that DRI is for adults. (7-dehydrocholesterol) that surfaces on the
A. Fat-soluble vitamins: soluble in fats and fat solvents; skin, is activated by sunlight, and is con-
mineral oils interfere with absorption; NOT readily verted into vitamin D3 (cholecalciferol).
excreted and so can build up to toxic levels; stored in (2)  Ultraviolet light.
liver and fatty tissues: (3)  Fortified milk, butter, cod-liver oil, fatty
1. Vitamin A (retinol): fish.
a. Function: (4)  Vitamin D2 (ergocalciferol): derived from
(1)  IMPORTANT for night vision; retinol plants, especially yeasts and fungi.
forms rhodopsin (visual pigment). d. Nutritional deficiency and disease:
(2)  Facilitates transcription of DNA to RNA. (1)  Rickets in children: softening of the bones
(3)  Antioxidant properties help prevent dam- (failure to calcify normally); bowed legs;
age to cells by free radicals. enlarged head, joints, rib cage; deformed
o2600 (4)  Necessary for immune defenses. pelvis.
o2610 (5)  Promotes normal growth and development. (2)  Osteomalacia in adults: calcium is taken
o2580 (6)  Facilitates absorption of calcium at intes- from bones to make up for insufficient ab-
tine. sorption by the intestine; bowed legs, bent
o2590 (7)  Assists in maintenance of epithelial cells posture, and pain in ribs, pelvis, legs.
such as those of oral mucosa. e. Toxicity: occurs with high blood calcium levels;
(8)  Assists in formation of ameloblasts and nausea, weight loss, constipation, weakness,
odontoblasts. vomiting, loss of appetite, mental confusion.
o2630 b. DRI: 700 micrograms (μg or mcg) (female) and 3. Vitamin E (tocopherol):
900 μg (males). a. Functions:
c. Nutrient sources: (1)  Antioxidant: prevents oxidation of vitamin
(1)  Preformed vitamin A (retinoids; animal A, beta-carotene, vitamin C, unsaturated
form) is found in liver, body fat of fish, egg fatty acids.
yolk, butter, cheese, cereal, crackers, vita- (2)  Protects RBCs from damage.
min A–fortified milk. (3)  Necessary for neural and muscular structure.
(2)  Provitamin A (carotenoids; from plant- (4)  Necessary for cellular respiration.
­d­e­rived precursor carotene) is converted in b. DRI: 15 mg/day, 1000 mg/day upper limit (UL);
body to vitamin A; MOST potent form is NO proven need for supplementation.
beta-carotene (antioxidant); found in orange- c. Nutrient sources:
yellow and dark green fruits and vegetables (1) Plant (vegetable) oils: safflower, cotton-
(e.g., carrots, spinach, apricots, broccoli). seed, peanut.
d. Nutritional deficiency and disease: night blind- (2) Green leafy vegetables.
ness (nyctalopia); dry, rough skin or dry oral (3) Legumes, nuts, whole grains.
mucosa; glossitis; pulp calcification; enamel hy- (4) Apricots, apples, peaches.
poplasia; defective dentin; acne and skin prob- d. Nutritional deficiency and disease: erythrocyte
lems treated with isotretinoin (­Accutane), which hemolysis (breakdown of RBCs), anemia.
can dry oral cavity. e. Toxicity: relatively safe; increased levels may
e. Toxicity: irritability; enlarged liver and lead to increased bleeding, especially in those
spleen; dry skin; bone and joint pain; xero- prone (drugs, medical history, etc.).
stomia; ­ damage to red blood cells (RBCs) 4. Vitamin K:
and ­ lysosomes; increased risk of hip frac- a. Function: synthesizes blood-clotting factor pro-
tures ­because of ­stimulation of osteoclasts and thrombin (PT).
­inhibition of ­osteoblasts. b. DRI: 90 μg (AI) (female) and 120 μg (male)/day.
Nutrition 217

c. Nutrient sources: green leafy vegetables; meat s­ ustained release) assists in reducing LDL
and dairy; also synthesized by intestine. cholesterol and tryglycerides, increasing
d. Nutritional deficiency: hemorrhage; for fe- HDL cholesterol levels.
males, reduced bone mass. b. Properties: amino acid tryptophan from B6 can
e. Toxicity: eating large amounts in foods can be converted to niacin, one of the MOST stable
counteract benefits of blood-thinning agents vitamins.
such as warfarin (Coumadin), heparin, or aspi- c. DRI: 14 mg (female) and 16 mg (male)/day.
rin for CVD. d. Nutrient sources:
B. Water-soluble vitamins: easily excreted; LESS likely (1) Meat, poultry, fish.
to reach toxic levels; NOT stored in body; need daily (2) Enriched breads and cereals.
amounts. (3) Legumes (peanuts, soybeans), seeds, nuts.
1. Thiamin (B1): (4) Milk, eggs.
a. Functions: e. Nutritional deficiency and disease:
(1) Essential in metabolism of carbohydrates (1) Pellagra: dementia, diarrhea, dermatitis (rough,
and aids in metabolism of fats and proteins. painful skin), death (considered the “4 Ds”).
(2) Assists in proper functioning of nervous (2) Gastrointestinal disturbances, loss of appetite.
and cardiovascular systems. (3) Stomatitis, glossitis, inflamed gingiva, fis-
b. Properties: sensitive to heat; avoid overcooking. sured tongue.
c. DRI: 1.1 mg (female) and 1.2 mg (male)/day. f. Toxicity: blood vessel dilation (flushing) caused
d. Nutrient sources: whole grains (wheat germ), by increased blood flow (reduced by use of an-
pork, sunflower seeds, nuts, legumes (peanuts, algesics), headache, nausea, vomiting, blurred
soybeans), organ meats. vision, fainting.
e. Nutritional deficiency and disease: causes beri- 4. Pantothenic acid (B5):
beri, which used to be present only in those a. Functions:
whose diet was polished white rice in Asia but (1) Aids in metabolism of carbohydrates, fats,
is now rare and associated ONLY with alcohol- proteins for energy.
ics, because drinking heavily can lead to poor (2) As coenzyme A, assists in fatty acid me-
nutrition, making it harder to absorb and store tabolism and initiates fatty acid synthesis.
vitamin; two types: wet, affecting cardiovascu- (3) Aids in formation of hormones and nerve-
lar system, and dry, affecting nervous system. regulating substances.
f. Presents with limb swelling, elevated pulse, b. Properties: easily destroyed by heat.
heart failure; Wernicke-Korsakoff syndrome c. DRI: 5 mg/day (AI).
(jerky gait, disorientation, impaired short-term d. Nutrient sources: liver, eggs, whole grains, le-
memory) occurs among alcoholics. gumes, broccoli.
2. Riboflavin (B2): 5. Pyridoxine (B6):
a. Functions: aids in metabolism of carbohydrates, a. Functions:
fats, and proteins; assists in adenosine triphos- (1) Involved in carbohydrate, protein, fat
phate (ATP) formation. ­metabolism.
b. Properties: sensitive to light; milk SHOULD be (2) Converts tryptophan to niacin.
stored in cardboard or opaque containers. (3) Aids in synthesis of hemoglobin and neu-
c. DRI: 1.1 mg (female) and 1.3 mg (male)/day. rotransmitters.
d. Nutrientsources:milkproducts,grains,organmeats, (4) Regulates blood glucose.
green vegetables (e.g., broccoli, turnip greens, b. Properties: stable to heat.
asparagus), meat, poultry, fish, enriched food. c. DRI: 1.3 mg/day.
e. Nutritional deficiency and disease: glossitis, an- d. Nutrient sources:
gular cheilosis, dermatitis, anemia. (1) Animal sources: meat, fish, poultry.
3. Niacin (B3): (2) Fruits and vegetables: bananas, cantaloupe,
a. Functions: broccoli.
(1) Coenzyme that assists in energy metabolism. (3) Whole grain products.
(2) Aids normal functioning of CNS. e. Nutritional deficiency and disease: weakened
o3240 (3) Maintains healthy skin and oral mucosa; immune system, weakness, irritability, insomnia,
however, essential for growth of cariogenic dermatitis, anemia, angular cheilitis, glossitis.
bacteria. f. Toxicity: uncommon; long-term megadoses
o3260 (4) Supplemental nicotinic acid (2 to 3 g/day; ­result in sensory nerve damage, numbness in
>1000 mg may cause facial flush if NOT extremities, walking difficulties.
218 Saunders Review of Dental Hygiene

6. Cobalamin (B12):
a. Functions:
(1)  Assists folate metabolism.
(2)  Maintains myelin sheaths that insulate
nerve endings.
(3)  Essential for the proper functioning of ALL
cells.
b. Properties:
(1)  Intrinsic factor (protein): made in stomach;
needed for absorption; prevents pernicious Figure 7-2  Scurvy from water-soluble vitamin C defi-
anemia. ciency, with swollen and bleeding gingiva.
(2)  Extrinsic factor: must be obtained through
foods; prevents pernicious anemia.
c. DRI: 2.4 μg/day. from toxins and generally have lower blood
d. Nutrient sources: found exclusively in foods of levels of vitamin C.
animal origin (meat, liver, eggs, cheese). f. Toxicity:
e. Nutritional deficiency and disease: (1)  Nausea, abdominal cramps, diarrhea.
(1)  Can occur in strict vegetarians, those with (2)  May interfere with cobalamin (B12) absorption.
diets consisting ONLY of plant foods (see (3)  Rebound scurvy can occur after megadoses
earlier discussion under protein deficiency), are reduced.
persons who chronically abuse nitrous ox- 8. Biotin:
ide and alcohol. a. Functions: coenzyme form assists in energy
(2)  Loss of absorption with stomach surgery metabolism and serves as carrier of carbon di-
(such as weight loss surgery) (see later dis- oxide, thus promoting synthesis of glucose and
cussion). fatty acids.
(3)  Deficiency: pernicious anemia (type of b. Properties: avidin, protein that is present in raw
megaloblastic anemia) with weakness, egg whites, can bind to biotin and interfere with
glossitis, hemorrhagic gingiva, stomatitis, its absorption.
xerostomia, bone loss, apathy. c. DRI: 30 μg/day (AI).
7. Vitamin C (ascorbic acid): d. Nutrient sources: egg yolk, yeast, liver, cereals;
a. Functions: also synthesized by intestinal bacteria.
(1)  Promotes synthesis of protein collagen in e. Nutritional deficiency and disease:
connective tissue, bone, teeth, blood ves- (1)  Glossitis, papillary atrophy, oral mucosal
sels; assists in wound healing. pallor.
(2)  Acts as antioxidant, reducing formation of (2)  Severe deficiency may lead to nausea, mus-
cancer-causing nitrosamines in stomach; cle pain, loss of appetite and sleep.
keeps folate coenzymes intact. 9. Folate (folacin, folic acid):
(3)  Promotes iron absorption. a. Functions:
(4)  Protects the body from infections. (1)  Assists in forming DNA and protein.
(5)  Maintains integrity of the capillaries (re- (2)  Manufactures RBCs.
duces capillary fragility). b. Properties: easily destroyed by processing and
b. Properties: CANNOT be stored in the body for heating.
long periods. c. DRI: 400 μg/day.
c. DRI: 75 mg (female) and 90 mg (male)/ d. Nutrient sources: liver, dark green leafy vegeta-
day. bles, fruits (oranges, orange juice, cantaloupe).
d. Nutrient sources: citrus fruit (oranges), toma- e. Nutritional deficiency and disease:
toes, cantaloupe, strawberries, mangoes, broc- (1)  Birth defects: neural tube disorders (NTDs)
coli, peppers. such as spina bifida.
e. Nutritional deficiency and disease: (2)  Megaloblastic anemia.
(1)  Scurvy: deficiency with ruptured blood (3)  Impaired cell division and immune response.
vessels, swollen and bleeding gingiva with (4)  Gastrointestinal tract (GIT) deterioration
“scorbutic” glossitis, delayed wound heal- and diarrhea.
ing, rough skin (Figure 7-2). (5)  Glossitis and periodontal disease.
(2)  DRI for smokers is 35 mg/day higher, since (6)  Lowered resistance to Candida albicans.
they are under increased oxidative stress f. Toxicity: kidney damage.
Nutrition 219

CLINICAL STUDY  

Age 27 YRS SCENARIO

Sex ☐  Male   ☒  Female The patient is in the dental office for an oral
­prophylaxis appointment. Her oral cavity shows
Height 5’4” signs of slight gingivitis.
Weight 135 LBS

BP 114/58

Chief Complaint “I am concerned for the health of


my baby.”

Medical History Two weeks pregnant but has not


seen her physician
History of spina bifida in family

Current Medications None

Social History Lawyer

1. Identify the typical causes of and predisposing factors (4)  Maintains normal functioning of the ner-
for spina bifida. vous system.
2. How can this condition be prevented? (5)  Regulates cellular metabolism.
3. What food sources are essential in preventing spina b. Properties:
bifida? (1)  Proper absorption requires vitamin D (most o4300
supplementations include it).
1. Folic acid (folate, folacin) deficiency during preg- (2)  Parathyroid hormone (PTH) maintains nor-
nancy is associated with increased risk of spina bifida. mal level of serum calcium.
Because folate is necessary for nucleic acid synthesis, (3)  DRI: 1000 mg/day (AI).
deficiency may impair cell growth, causing anomalies (4)  Nutrient sources:
of fetus. Defects usually occur within first 6 weeks af- �����������������������������������������������������
  ���������������������������������������������������
 (a) Dietary: dairy products, milk, yogurt,
ter conception. cheese.
2. The patient must see a physician as soon as possible. �������������������������������������������������
   ����������������������������������������������
(b) Calcium-fortified orange and fruit
Most women are encouraged to increase folate con- drinks; calcium supplements.
sumption even before conceiving. Adequate intake of (5)  Nutritional deficiency and disease: women
foods containing folate is important; however, prena- are MOST likely to be at risk.
tal vitamins containing folate may be prescribed. ����������������������������������������������������
   �������������������������������������������������
(a) Osteoporosis: bone disease that devel- o4410
3. Food sources high in folate include fruits and vegeta- ops MORE commonly in women with
bles, especially green leafy vegetables. Raw vegeta- decreased bone density (osteopenia is
bles are usually better than cooked vegetables because early indication of loss of bone density).
cooking easily destroys folate. Orange juice is also a �����������������������������������������������������
   ��������������������������������������������������
(b) Calcium tetany: failure of the muscles o4370
good source of folate. to relax after contraction.
�������������������������������������������������������
   ����������������������������������������������������
(c) Rickets in children: deficiency leads to o4380
Minerals stunted growth.
Minerals are inorganic nutrients used to build the body ����������������������������
   �������������������������
(d) Affects HBP. o4390
and regulate functions. They yield NO energy to the body ���������������������������������������������������
   ������������������������������������������������
(e) Weight loss surgery may adversely af- o4400
but assist in regulating the release of energy. fect absorption.
A. Macrominerals (major minerals): needed in amounts ���������������������������������������������������
   ������������������������������������������������
(f) Prevention: vitamin D hormone or hor-
>100 mg/day. mone calcitonin, meeting DRI require-
1. Calcium (MOST abundant mineral in the body): ments, exercising regularly, AVOIDING
a. Functions: alcohol and/or smoking.
(1)  Forms and maintains bones and teeth. (6)  Toxicity: nausea, vomiting, severe consti-
(2)  Aids in blood coagulation. pation, kidney stone formation, irregular
(3)  Assists in muscle contraction or relaxation. ­heartbeat, tingling, xerostomia, fatigue,
220 Saunders Review of Dental Hygiene

increased blood pressure; may inhibit iron c. DRI: 500 mg/day.


and zinc uptake. d. Nutrient sources: table salt, animal meat, salt-
2. Phosphorus: second MOST abundant mineral in water fish, eggs, dairy, processed foods.
body. e. Nutritional deficiency and disease are rare in
a. Functions: United States, but with restriction of intake:
(1)  Assists in formation of bones and teeth. muscle cramps, mental apathy, dizziness, de-
(2)  Involved in the transfer and release of high- creased appetite, nausea.
energy phosphates. 5. Chloride:
(3)  Regulates acid-base balance. a. Functions:
(4)  Releases energy from carbohydrates, pro- (1)  Aids in maintaining normal acid-base and
tein, fat metabolism. fluid balance.
(5)  Plays active role in cell protein synthesis. (2)  Component of hydrochloric acid in the
b. Properties: absorption is affected by vitamin D stomach and necessary for digestion.
and PTH. (3)  Assists in maintenance of nerve func-
c. DRI: 700 mg/day. tions.
d. Nutrient sources: milk, cheese, bakery prod- b. Properties: negative ion of extracellular
ucts, meats. fluid.
e. Nutritional deficiency and disease: uncommon; c. DRI: 750 mg/day.
risk is greater among elderly, strict vegetarians, d. Nutrient sources: table salt, processed foods,
alcoholics; presents with hypocalcification and water.
muscle ­weakness. e. Nutritional deficiency and diseases:
3. Magnesium: (1)  Caused by starvation, fever, diarrhea, vom-
a. Functions: iting.
(1)  Calcium homeostasis. (2)  Muscle cramps, mental apathy, decreased
(2)  IMPORTANT to structural integrity of appetite.
heart muscle. 6. Potassium:
(3)  Assists in mineralization of bones and a. Functions:
teeth. (1)  Maintains fluid-electrolyte balance and cell o4940
(4)  Facilitates operation of enzymes. integrity.
(5)  Assists in production of energy. (2)  Assists in the transmission of nerve o4930
b. Properties: vitamin D enhances its absorption. ­impulses.
c. DRI: 310 mg (female) and 400 mg (male)/day. (3)  Aids in muscle contraction (including heart o4970
d. Nutrient sources: nuts, legumes, whole grains, muscle) and electrical conductivity of
dark green leafy vegetables. heart.
e. Nutritional deficiency and disease: risk is (4)  Assists in carbohydrate and protein metab- o4960
greater among women: olism.
(1)  Vomiting, diarrhea, tetany, weakness, con- (5)  Reduces adverse effects of sodium on blood o4990
vulsions. pressure.
(2)  Alveolar bone reduction, hypoplasia of (6)  Reduces risk of kidney stones and possibly o5000
enamel and dentin. reduces bone loss.
4. Sodium: b. DRI: 2000 mg/day.
a. Functions: c. Nutrient sources:
(1)  Retains body water. (1)  Inside ALL living cells, both plant and
(2)  With chloride and potassium, aids in main- ­animal.
tenance of acid-base and fluid balance. (2)  Fresh fruits (banana, oranges) and vegeta-
(3)  Facilitates transmission of nerve impulses bles (yams).
and muscle contraction. (3)  Milk, meat, whole grains, dried beans, le-
b. Properties: possibly contributes to HBP, which gumes (peanuts, soybeans).
can lead to CVD and complications (MI and d. Nutritional deficiency and disease:
CVA); HBP may be controlled by lifestyle man- (1)  Hypokalemia: caused by dehydration, dia-
agement in its early stages and managed in its betic acidosis, vomiting, diarrhea, diuret-
later stages by: ics, steroids.
(1)  Limiting intake of processed foods. (2)  Leads to muscle weakness, cramps, loss of
(2)  Restricting intake of sodium to 3 g/day (sea appetite, constipation, mental confusion,
salt has LESS sodium). apathy.
(3)  Exercising. e. Toxicity: muscle weakness, cardiac arrest.
Nutrition 221

7. Sulfur: 2. Fluorine: helps make teeth resistant to caries by


a. Functions: hardening outer enamel.
(1)  Aids in maintaining normal acid-base bal- 3. Zinc: involved in growth, healing of wounds, male
ance. sexual development.
(2)  Stabilizes shape of proteins by forming 4. Iodine: IMPORTANT in function of thyroid gland; io-
bridges between sulfur molecules (creates dine deficiency causes endemic goiter and enlarged
rigid proteins in hair, nails, skin). gland and can result in learning deficiencies; iodine
b. Properties: component of amino acids and deficiency uncommon because of addition to table
­vitamins biotin and thiamin (B1). salt (not usually included in sea salt, unless noted).
c. DRI: none. 5. Cobalt: contained in cobalamin (B12).
d. Nutrient sources: ALL protein-containing 6. Copper: deficiency can cause anemia and bone dis-
­sour­ces; found in preserved foods and naturally ease, changes in hair color, problems with immune
on certain fruit (grapes). system.
e. Nutritional deficiency and disease: protein defi- 7. Manganese: vital for bones, reproductive system,
ciency occurs before sulfur deficiency. nervous system.
B. Trace elements: remaining elements necessary for ­he­alth, 8. Chromium: involved in metabolism of sugar.
and have safe adequate intakes (SAEs) of <100 mg/day. 9. Also small amounts of boron, molybdenum, sele-
1. Iron: IMPORTANT in hemoglobin in RBCs, bonds nium, tin, nickel, etc.
to oxygen in lungs and carries it throughout body; C. Lead poisoning can result from ingestion of old paint
deficiency causes hypochromic anemia, with risk of chips by children; will cause anemia and blue lines in
bleeding, feeling of tiredness at the end of the day. ­gingiva.

CLINICAL STUDY  

Age 62 YRS SCENARIO

Sex ☐ Male   ☒ Female After an initial examination of the patient, a


panoramic radiograph is taken and ­thinning
Height 5’2” of the posterior border of the mandible
Weight 112 LBS is ­noted. The patient is then questioned
­regarding her diet. She states that she has
BP 112/74 never been a milk drinker.

Chief Complaint “My lower partial denture does not fit


as well anymore.”

Medical History Fractured radius from fall 6 months ago


Menopausal for 1 year
Smokes pack and half a day of
­cigarettes

Current Medications None

Social History Photographer for travel magazine

1. What is the most likely diagnosis of the patient’s is a predictor for the development of osteoporosis
­condition? (note that menopause is earlier for smokers by at least
2. Identify the most common causes of and predisposing 5 years). Other contributing factors include deficien-
factors for this condition. cies in calcium and/or vitamin D, lack of physical ac-
3. What is the best treatment plan for this condition? tivity, cigarette smoking, excessive intake of protein
4. Is this condition preventable? If so, how? and/or caffeine.
3. Traditional treatment plan includes estrogen replace-
1. Osteoporosis, a bone disease that develops more com- ment (controversial), calcium and/or vitamin D sup-
monly in women with decreased bone density (osteo- plements, regular exercise; exercise (weight bearing)
penia is early indication of loss of bone density). may increase bone mass.
2. Genetics is a factor in the development of osteoporosis 4. Adequate calcium intake is necessary throughout indi-
and in the attainment of bone mass. Early menopause vidual’s life; DRI 1000 mg/day for adults.
222 Saunders Review of Dental Hygiene

C. Vitamin A maintains the integrity of the tissues:


Water 1. Synthesizes protein matrix for tissues, also for
p0130 Water, main constituent of the body, makes up 50% to enamel and dentin.
60% of total body weight. MOST of water is located in 2. Sources: carrots and squash.
the cells. Required on a daily basis to maintain neces-
sary levels; have enough if urine is clear enough to read Repair Process
through. Speed and effectiveness of the repair process are greatly
A. Functions: influenced by nutrients such as proteins, vitamins, miner-
o5380 1. Removes waste products, including urea (by- als (see later discussion on postsurgical diet). Repair is an
­product of protein metabolism, contains nitrogen) important part of posttreatment of the periodontium and
and stabilizes ALL body fluids. other dental tissues.
2. Maintains body temperature. A. Zinc speeds wound healing and the repair process;
o5410 3. Serves as solvent for minerals, vitamins, glucose, sources include oysters, shrimp, crab, bran cereal,
amino acids and acts as lubricant around joints. lean pork, lamb, ham, hamburger.
o5430 4. Participates in chemical reactions and transports B. Calcium, phosphorus, vitamin D all promote bone
inorganic nutrients. density and calcify the protein matrix of cemento-
o5450 5. Maintains normal kidney function and electrolyte blasts, ameloblasts, and odontoblasts; sources include
balance, reducing risk of kidney stones. milk and hard cheeses.
B. Dietary requirements: ~3.7 L/day for adults. C. Protein and vitamin C are involved in connective
C. Nutrient sources: MORE water and beverages. tissue formation; sources for protein include animal
D. Nutritional deficiency and disease: survival without wa- sources and water-packed tuna; sources for vitamin C
ter is limited to 2 to 3 days. Dehydration is caused by: include citrus fruits and green vegetables.
o5520 1. Lack of water intake.
o5510 2. Vomiting and/or diarrhea. Immune Mechanisms
o5490 3. Blood loss. Proteins are needed for the immune system and aid in
o5500 4. Malfunctioning kidneys. controlling infection, important factors in periodontal
o9000 E. Toxicity (by dehydration) is characterized by:
o5530 health and protection against periodontal disease.
1. Dizziness and nausea.
o5540 2. Sodium retention. WEIGHT CONTROL AND ENERGY  
o5550 3. Hypertension. Weight control is achieved when the calories gained
from food equal the energy needs of body. One kilocalo-
EFFECTS OF NUTRIENTS ON CELLS OF ORAL rie is amount of heat produced to raise the temperature of
TISSUES   1 kg of water 1° C. Weight control MUST be done with
Nutrients significantly affect the function and health of healthy intent.
oral tissues by providing components for synthesis, re- A. Basal metabolism rate (BMR):
pair, immunity. 1. R ate of energy utilization in the resting state o9010
is MOST closely related to lean body mass;
Periodontal and Dental Health measured from O2 consumption (CO2 produc-
Several nutrients are needed for the formation and main- tion) of a person awake, at rest, after an overnight
tenance of the health of the periodontium and other dental fast.
tissues. MANY deficiencies are noted in the tongue as glos- 2. Includes energy needed for breathing, beating o5750
sitis (smooth inflamed tongue), since it affects the sensitive of the heart, circulation, muscle tone, body tem-
lingual papillae on dorsum as well as other dental tissues. perature; influenced by such factors as age, gen-
A. Iron, vitamin C, zinc are needed for collagen synthe- der, body size, infection, injury, surgery, hormone
sis and wound healing: levels. (~70% of total energy expenditure is due
1. Collagen is MAIN component of periodontium to basal life processes; ~20% comes from physical
and hard dental tissues. activity, ~10% from thermogenesis, or digestion of
2. Sources: food).
a. Iron: beef, liver, beans. 3. Thyroid gland is the MAJOR gland that regulates o5770
b. Vitamin C: citrus, broccoli, peppers. metabolism; higher thyroid hormone levels in-
c. Zinc: red meats, shellfish. crease the metabolic rate.
B. Folate and protein are needed for cell formation: B. Degree of physical activity is the voluntary compo-
1. Junctional epithelial cells, which have a rapid turnover. nent of energy.
2. Sources: 1. Varies from sedentary to strenuous activity (we
a. Folate: asparagus, broccoli, liver. weigh less on top of mountain than at sea level, so
b. Protein: meat, fish, poultry. let’s all move!).
Nutrition 223

2. Influenced by such factors as intensity and length 3. Purging is responsible for MORE of the oral and me­
of activity and size of individual. dical complications associated with this ­ disorder,
C. Specific dynamic activity (SDA): including erosion, anemia, cardiac problems, GIT
1. Energy required to digest and absorb food. disturbances, renal failure, failure to ovulate.
2. Contributes ~10% of total kilocalories consumed. 4. Diuretic use can cause reduction in salivary flow
and development of angular cheilitis; laxative use
Eating Disorders can lead to metabolic acidosis and dependence on
Include serious eating disorders such as anorexia, bulimia, laxatives for adequate bowel function.
bulimorexia, compulsive overeating, obesity. Requires 5. Body weight is normal or slightly overweight, with
multidisciplinary care (dental, medical, psychosocial, nu- malnutrition, dehydration, cardiac dysrhythmias,
tritional consults). calluses noted on knuckles.
A. Anorexia nervosa: syndrome that involves extreme 6. Family history of alcoholism and drug abuse is
loss of weight (20% to 40% below average) as re- common; may have depression.
sult of self-starvation (protein-energy malnutrition 7. Oral signs:
[PEM], protein-calorie malnutrition [PCM]), exces- a. Dental erosion (perimolysis) and thermal sensitiv-
sive exercise, aversion to food, altered eating habits, ity on maxillary anteriors from vomiting, dishing
and/or chronic use of laxatives or suppositories in re- of lingual surfaces, raised appearance of resto-
sponse to a distorted body image. ration margins, appearance of anterior open bite
1. Etiology: unknown, but disorder is associated with (See Chapter 6, General and Oral ­Pathology).
psychosocial pressures; occurs MAINLY in young b. Dental caries from gastric acids.
women with low self-esteem; can prove fatal. c. Xerostomia from prescribed antidepressants,
2. Outpatient treatment, in conjunction with antide- laxative and/or diuretic abuse.
pressant and antianxiety drugs, is common. d. Palatal trauma lesion from forced vomiting.
3. Decreased heart rate, malnutrition, dehydration, con- e. Angular cheilosis; enlarged interdental papillae
stipation, diarrhea; often occurs in individuals with as result of constant irritation from acidic vomitus.
family history of depression or bipolar ­disorder. f. Parotid salivary gland enlargement, with change
4. May cause dry skin and nails, with excessive in shape of face (round) and jaw (square).
growth of facial and appendage hair (hirsutism), 8. Risk factors for oral health: denial of the problem,
called lanugo (“downy”), which helps to insulate emotional stresses, xerostomia, vomit acid.
the body against heat loss. 9. Barriers to care:
5. Amenorrhea: absence of a menstrual cycle in re- a. Lack of communication because of denial,
sponse to hormonal changes. guilt, fear of gaining weight, and lack of com-
6. Oral signs: dental caries, xerostomia, and oral le- pliance; also results from clinician’s inability to
sions from malnutrition. gain patient’s trust and confidence and failure to
7. Risk factors for oral health: medical crises and refer patient to physician and/or psychiatrist.
emotional stress. b. Economic barriers may be involved because of
8. Barriers to care: lack of communication because of cost of repairing damage (Figure 7-3).
denial, guilt, fear of gaining weight, lack of com-
pliance; may include economic barriers because of
cost of repairing damage.
9. Professional care and homecare: fluoride in custom
tray and use of mineralizing calcium products are
recommended if xerostomia or vomiting is problem.
10. Patient education: prevention of further damage to
the teeth; discussion of influence of diet on caries;
recommendation for daily mineralizing fluoride
and calcium products.
B. Bulimia nervosa: compulsive disorder that involves
periods of starvation, bingeing, purging; person has a per-
ceived lack of control over eating behavior; affected in-
dividuals engage in an average of two episodes a week.
1. Etiology: unknown but is likely to be stress related,
affecting young women.
Figure 7-3  Loss of enamel on lingual surfaces as result of
2. Bingeing may be followed by purging activity bulimia. Repaired using veneer crowns. (From Bath-Balogh M,
(vomiting with laxative and diuretic abuse) in Fehrenbach MJ: Illustrated dental embryology and anatomy, ed 2,
those who are fearful of gaining weight. St. Louis, 2007, Saunders/Elsevier).
224 Saunders Review of Dental Hygiene

10. Professional care and homecare: restoration of car- F. Obesity: excessive body weight caused by excessive
ious teeth with glass ionomer restorative (to leach nutrient intake (overnutrition) and sedentary lifestyle.
fluoride ions) where possible, sealing of eroded ar- 1. Evaluated in absolute terms by measuring body
eas with composite resins, assessment of progres- mass index (BMI).
sion of erosion (whether by study models and/or 2. Etiology:
intraoral photos), use of mineralizing neutral so- a. Positive energy balance (kilocalorie intake ex-
dium fluoride and calcium products. ceeds expenditure).
11. Patient education: b. Genetics: child with one obese parent has HIGH
a. Discussion of oral and medical problems asso- risk of becoming obese, child with two obese
ciated with purging, diuretics, and laxative use. parents has HIGHER risk.
b. Need to neutralize vomit acid by rinsing with c. Development of excess fat cells during child-
tap water, or possibly a mix of water with hood; theory includes that:
­sodium bicarbonate or magnesium hydroxide (1)  During childhood, increased number of fat
may be used. cells develops in response to excess kilo-
c. Discouragement of toothbrushing or flossing calorie intake (hyperplasty).
immediately after vomiting to reduce abrasion. (2)  After puberty, fat cells enlarge (hypertrophy).
d. Use of saliva substitutes and sugarless gums d. Set-point theory: body chooses a weight and
(possibly with xylitol) to increase salivary flow. defends its set-point by internal factors.
e. Daily use of mineralizing fluoride and calcium e. Psychological factors (e.g., finding comfort in
products. eating).
12. Daily vitamin and mineral supplementation may f. Physical activity (i.e., lack of energy expenditure).
be recommended when gingival tissue appears un- 3. Prevention and weight control:
healthy or when angular cheilitis is present. a. Losing weight involves: o6530
C. Bulimorexia: combination of anorexia and bulimia (1)  Controlling energy intake and eating low- o6540
(binge eating followed by starvation). fat carbohydrate foods such as fresh fruits,
1. Involves signs and symptoms of both anorexia and vegetables, whole grains, and some nonfat
bulimia. (fat free).
o6230 2. Medical complications, oral signs, risk factors, (2)  Drinking adequate water to satisfy thirst o6560
barriers to care, professional care, and patient edu- and fill the stomach.
cation are the same as for anorexia nervosa and (3)  Still meeting DRI nutritional needs; avoid- o6570
bulimia. ing diet drinks that contain high amounts
D. Diabulimia: bulimia in insulin-dependent diabetics of acid and may falsely increase need for
who purge to keep sugar levels under control. “sweet” items.
E. Compulsive overeating: chronic uncontrolled episodes b. Behavior modification (changing eating behav-
of overeating without other signs of eating disorder. ior):
1. Overeating in response to stress, feelings of anxi- (1)  Become aware of current behaviors.
ety, or depression. (2)  Stimulus control; change environment to
2. Bingeing on foods that are easy to eat in large por- minimize stimuli for eating (e.g., shopping
tions (e.g., noodles, rice). after eating to avoid poor choices).
3. Eating foods categorized as “junk” foods, such as (3)  Cognitive restructuring; altering one’s state of
ice cream and chips. mind regarding eating (e.g., avoiding excuses
4. Characteristics: for overeating such as having difficult day).
a. Eating large quantities in isolation to induce a (4)  Self-monitoring (i.e., keeping diary of
sense of well-being. foods and eating patterns); complies BEST
o6310 b. Grazing in some individuals with lack of preoc- when involved in decision making.
cupation with weight. (5)  Contingency management; developing plan
o6330 c. Difficulty expressing and dealing with feelings. for responding in environment where over-
5. Management: eating is MOST likely to occur (e.g., party
a. Dietary: where snacks are served).
(1)  Eat until full (a must) but not beyond. c. Physical activity:
(2)  Avoid diets that may encourage more ­bingeing. (1) Increases energy output and controls appe- o6650
b. Psychological: tite.
(1)  Focus on responding to hunger rather than (2) Aids in stress reduction and increases basal o6670
emotions. metabolism.
(2)  Identify personal needs and express them. (3) Increases self-esteem. o6690
Nutrition 225

4. Risk for CVD (especially HBP, high blood cho- 4. Counseling approaches:
lesterol, triglyceride levels [combined hyperlip- a. Directive: decisions are made by counselor; pa-
idemia]), DM type 2, sleep apnea, osteoarthritis; tient is passive.
possibly periodontal disease. b. Nondirective: counselor assists in understanding
5. May have media-driven diet with all its restrictions and needs and recommends changes; patient is in-
implications (discussed earlier under each ­nutrient, volved in planning, implementing, evaluating diet.
especially noted is overconsumption of proteins). B. Principles of diet management:
6. May undergo or have had bariatric procedure 1. Food diary:
(weight loss surgery [WLS]; gastric bypass or a. Patient keeps a 3-, 5-, or 7-day food diary
Lap-Band) performed to modify GIT to reduce nu- (including weekend); records ALL meals and
trient intake or absorption; need to have nutritional ­between-meal snacking (MOST common;
blood tests done regularly. BEST representation of normal diet).
a. Diet is soft and limited by smaller size stomach. b. Patient keeps a 24-hour food diary (NOT repre-
b. Loss of stomach lining may adversely affect ab- sentative of normal diet).
sorption of calcium, zinc, cobalamin (B12), and c. Food frequency checklist represents consump- o9020
other nutrients; may increase risk for osteopenia, tion of certain foods per week; MORE repre-
osteoporosis, and ultimately osteomalacia. sentative than 24-hour but not as representative
c. May have acidic oral cavity with certain surger- as food diary, since limited in scope.
ies and poor diet that increases caries risk. 2. Interview:
d. Poor fit of dentures because of rapid weight loss a. Assess MAIN problem or complaint.
after surgery. b. Ascertain routine and habits.
e. AVOID drugs that can cause ulceration or c. Review medical history for any changes or
trauma to GIT such as nonsteroidal antiinflam- problems; may reveal systemic conditions or
matory agents or drugs (e.g., NSAIAs, NSAIDs, drug intake that can influence medical health,
ibuprofen). including ability to digest and/or metabolize
food.
NUTRITIONAL COUNSELING   d. Evaluate diet.
p0200 Counseling for weight control and proper health and (1)  Review adequacy of food intake.
nutrition involves diet counseling, diet management, (2)  Determine amount of foods that contain
nutritional counseling. Nutritional counseling by dental sugar and frequency of intake.
hygienists MAINLY involves maintenance of a healthy (3)  Look for food items or herbal supplements
oral cavity; patients with serious systemic nutritional that can interfere with prescription drugs
problems, such as DM or eating disorder, need to be (e.g., grapefruit juice).
­referred initially to a nutritional counselor. e. Assess clinical signs of deficiencies or diseases:
A. Diet counseling: (1)  Dental caries. o7090
1. Patient selection: (2)  Gingivitis and periodontal diseases, espe- o7100
a. Patient MUST be willing to change and/or im- cially necrotizing forms.
prove eating habits. (3)  Other signs such as dry skin, hair loss, sto- o7070
b. Patient has a need for dietary improvement. matitis, angular cheilosis, glossitis may need
2. Communication techniques are necessary to create to be referred to a nutritional counselor.
motivation in patient. f. Dental hygiene diagnosis: try to determine pos-
a. Maintain good eye contact. sible causes for clinical signs.
b. Use effective verbal and nonverbal methods of 3. Follow-up:
communication; tone of voice, gestures, and fa- a. Review a second 3-, 5-, or 7-day diet plan.
cial expressions can communicate sincerity and b. Compare second plan with first.
concern to patient. c. Emphasize good changes patient has made.
c. Adapt education to patient’s needs and under- C. Nutritional management:
standing. 1. Dental caries:
3. Patient interview: a. Cariogenic factors associated with diet:
a. Purpose: (1)  Frequency of between-meal snacking;
(1)  Understand the problem. MORE frequent the exposure to sugar,
(2)  Determine factors that contribute to the MORE cariogenic the diet (BEST to eat
problem. sugar-rich foods at mealtime).
(3)  Understand the personality and motivation (2)  Physical form of food; solid and retentive
of the patient. forms have SLOWER oral clearance time
226 Saunders Review of Dental Hygiene

and therefore are MORE cariogenic than ����������������������������������������������������


   �������������������������������������������������
(a) Recommend increased intake of vitamin
liquid forms; sticky food forms (raisins C 1 week before to condition tissues to
etc.) have slower oral clearance as well. heal; sources include supplementation,
(3)  Amount of sugar added to foods and bever- orange juice, pepper, broccoli.
ages (e.g., to cereals, coffee). ������������������������������������������������
   ���������������������������������������������
(b) Recommend adequate amounts of car-
(4)  Acidic foods also promote demineraliza- bohydrates to spare protein.
tion (e.g., soft drinks and sports drinks, es- ����������������������������������������������������
   �������������������������������������������������
(c) Avoid OTC herbal supplements that can
pecially citrus and/or diet). promote bleeding such as gingko, garlic.
b. Dietary recommendation: (3)  Postoperative diet before nonsurgical and
(1)  Eat nutritionally balanced diet and elimi- surgical periodontal therapy:
nate sugary snacks. ���������������������������������������
   ������������������������������������
(a) First postoperative day: fluids ������������
such
(2)  Restrict sugar to mealtimes and avoid pro- as water, juices, broths; recommend
longed exposure to dietary acids. MORE frequent feedings.
(3)  Eat hard cheeses to help neutralize pH. �������������������������������������������������������
   ����������������������������������������������������
(b) Second postoperative day: blended fruits
(4)  Use fluoride rinses and calcium products to and vegetables, oatmeal, ice cream, milk,
promote remineralization process. milk shakes, eggs, and/or meat broths.
(5)  Use xylitol in form of gum and candy (ac- 3. Necrotizing periodontal disease, bacterial infec-
cording to manufacturer’s instructions). tion of the periodontium, associated with poor oral
c. Dietary modifications: use USDA’s MyPyramid hygiene.
and the Dietary Guidelines for Americans 2005 a. Diet consists of empty-calorie and sugar-rich
(see CD-ROM): foods because of decreased ability to chew.
(1)  Helps to determine recommended servings b. Dietary recommendation:
and establish needs to meet body’s require- (1)  Recommend food sources or supplementa-
ments for essential nutrients. tions that contain protein, calcium, folate,
(2)  Allows modification of the American diet iron, zinc, vitamins A and C.
with four themes: variety, proportionality, (2)  Patient should eat six to eight small meals
moderation, activity. per day (one or two foods at each meal).
2. Chronic periodontitis: c. Dietary modification:
a. Dietary recommendation: (1)  Use USDA’s MyPyramid and the Dietary
(1) Considerations: Guidelines for Americans 2005 for serving
(a) Evaluate information obtained for any recommendations (see CD-ROM).
factors that may interfere with nutri- (2)  Choose good food sources high in nutrients.
tional status. (3)  Initially, menus should consist of liquids
(b) Determine eating patterns and food and soft foods.
habits. d. Avoid spicy foods.
(c) Patient should assist in determining 4. Diet after dental whitening (bleaching): avoidance o7650
­revised diet. of foods that may cause dentinal hypersensitivity
b. Procedure: (hot and spicy) or staining (red wine, fast food col-
(1) Patient determines deficiencies and makes orings) until teeth have had time to rehydrate by
appropriate choices to improve diet. ­saliva; 24 to 48 hours recommended.
(2) Direct patient to choices that can benefit
periodontium.
(3) Stress importance of decreasing sugar-rich Review Questions
foods in diet.
(4) Instruct patient to add variety to meal plan-
ning and to focus on nutritionally sound   1 Which of the following characterizes a complete protein?
foods that the patient likes. A. Adequate in all the essential amino acids
c. Dietary recommendations: B. Synthesized by the body
C. Provided through plant foods
(1)  Goals:
D. A dispensable amino acid
������������������������������������������������������
   ���������������������������������������������������
(a) Assist patient in meeting stress of sur-   2 All of the following conditions are examples of negative
gery. nitrogen balance, EXCEPT one. Which one is the EXCEP-
����������������������������������������������������
   �������������������������������������������������
(b) Aid wound healing and recovery time. TION?
�����������������������������������������������������
   ��������������������������������������������������
(c) Increase resistance to infections. A. Infection
(2)  Preoperative diet before nonsurgical and B. Anorexia nervosa
surgical periodontal therapy: C. Blood loss
D. Pregnancy
Nutrition 227

  3 Phenylketonuria is a condition that 13 Overconsumption of the following foods can increase the
A. involves protein-energy malnutrition. risk of atherosclerosis, EXCEPT one. Which one is the
B. restricts the consumption of aspartame. ­EXCEPTION?
C. ineffectively metabolizes phenylalanine to threonine. A. Butter
D. involves a disorder in carbohydrate metabolism. B. Coconut oil
  4 Kwashiorkor is a disease that C. Beef
A. occurs most commonly in children in underdeveloped D. Skim milk
countries. 14 Risk factors associated with atherosclerosis include all of
B. presents without severe wasting of body fat. the following, EXCEPT one. Which one is the EXCEP-
C. occurs in children 6 to 12 months of age in underdevel- TION?
oped countries. A. Hypertension
D. is not characterized by edema. B. LDL cholesterol of 120 mg/100 mL
  5 What is the MAIN function of carbohydrates? C. Total blood cholesterol greater than 240 mg/100 mL
A. Repair body tissues D. Smoking
B. Neutralize acid-base 15 Fat-soluble vitamins include all of the following, EXCEPT
C. Provide energy one. Which one is the EXCEPTION?
D. Regulate metabolism A. A
6 One gram of carbohydrate yields
  ���� B. E
A. 9 kcal. C. C
B. 7 kcal. D. K
C. 5 kcal. E. D
D. 4 kcal. 16 Preformed vitamin A is found in which of the following
7 Sorbitol, which is a sugar alcohol used in diabetic foods and
  ���� food sources?
sugarless gum, is made from A. Carrots
A. fructose. B. Body fat of fish
B. glucose. C. Broccoli
C. galactose. D. Cauliflower
D. mannitol. 17 Rickets is a disease associated with which vitamin deficiency?
8 Which of the following foods would be MOST cariogenic?
  ���� A. A
A. Dried apricots B. D
B. Soda pop C. E
C. Cake D. C
D. Milk 18 Which of the following is a function of vitamin E?
  9 Which of the following allows lipids to be transported in A. Aids in the prevention of night blindness
the blood? B. Promotes the mineralization of teeth
A. Bile C. Acts as an antioxidant
B. Phospholipids D. Synthesizes prothrombin
C. Lipoproteins 19 Which of the following is a function of water-soluble
D. Water vitamins?
10 A high level of which lipoprotein actually slows down the A. Aid in the metabolism of energy-producing nutrients
development of cardiovascular disease? B. Assist with immune defenses
A. Chylomicron C. Regulate serum levels of calcium and phosphorus
B. Very low-density lipoprotein D. Absorb calcium and phosphorus
C. Low-density lipoprotein 20 The amino acid tryptophan can be converted into which
D. High-density lipoprotein ­water-soluble vitamin?
11 All of the following are characteristics of cholesterol, A. Riboflavin
­EXCEPT one. Which one is the EXCEPTION? B. Pantothenic acid
A. Is a phospholipid C. Thiamin
B. Can be made by the body D. Niacin
C. Acts as an emulsifier 21 Interference with the intrinsic-extrinsic factor leads to a de-
D. Increases the risk of heart disease ficiency in which vitamin?
12 Important functions of dietary lipids include A. Cobalamin
A. providing essential fatty acids and aiding in absorption B. Folate
of B vitamins. C. Biotin
B. aiding in absorption of B vitamins and providing satiety. D. Pantothenic acid
C. providing essential fatty acids, aiding in absorption of B 22 A strict vegetarian diet MOST likely will be deficient in
vitamins, and providing satiety. which vitamin?
D. providing essential fatty acids, providing satiety, and A. Cobalamin
aiding in absorption of vitamins A, D, E, and K. B. Folate
E. providing essential fatty acids, preventing night blind- C. Biotin
ness, and aiding in absorption of B vitamins. D. Thiamin
228 Saunders Review of Dental Hygiene

23 A disease caused by vitamin C deficiency is 33 Which of the following is the major gland that affects the BMR?
A. beriberi. A. Thyroid
B. pernicious anemia. B. Endocrine
C. scurvy. C. Hypothalamus
D. pellagra. D. Sebaceous
24 One of the following characteristics is NOT true concerning 34 For an individual to lose weight, a reduction in energy in-
calcium. Which one is the EXCEPTION? take must take place. The individual should also decrease
A. Aids coagulation physical activity.
B. Assists in muscle contraction and relaxation A. Both statements are true.
C. An intake of 400 mg/day is recommended B. Both statements are false.
D. Vitamin D assists in its absorption C. The first statement is true; the second is false.
25 Which of the following is the MOST abundant mineral in D. The first statement is false; the second is true.
the body? 35 The individual with anorexia nervosa is usually
A. Calcium A. male.
B. Phosphorus B. female and exercises excessively.
C. Copper C. female and aware that her behavior is abnormal.
D. Fluorine D. female, and condition is often difficult to diagnose be-
E. Magnesium cause of a lack of physical signs.
26 Which of the following is the BEST food source of magne- 36 Bulimia is an eating disorder that involves
sium? A. weight loss 40% to 60% below desirable body weight.
A. Citrus fruits B. mostly male political candidates.
B. Dairy products C. episodes of binge eating followed by purging.
C. Dark yellow vegetables D. decreased heart rate in response to a decreased metabolism.
D. Whole grains and nuts 37 Compulsive overeating is associated with
27 The functions of sodium include all of the following, A. eating large quantities of food and purging.
­EXCEPT one. Which one is the EXCEPTION? B. eating large quantities of food in response to stress or an
A. Nerve impulse conduction emotional outlet.
B. Maintenance of acid-base balance C. an aversion to all forms of food.
C. Retention of body water D. a fear of becoming overweight.
D. Used to invoke an immune response 38 The BEST counseling technique for increasing patients’ in-
28 Which of the following minerals is used to preserve foods? volvement and responsibility in making their own decisions
A. Sodium A. is directive.
B. Chloride B. is nondirective.
C. Sulfur C. involves motivation.
D. Phosphorus D. involves listening.
29 Body water possesses all of the following characteristics, 39 Which of the following is the BEST example of a nondirec-
EXCEPT one. Which one is the EXCEPTION? tive approach?
A. Is found extracellularly as well as intracellularly A. Counselor makes all recommendations for a diet modi-
B. Makes up 85% of total body weight fication program and the patient accepts.
C. Assists in maintaining body temperature B. Patient is involved in the diet analysis and evaluation
D. Acts as a lubricant around joints and the counselor makes recommendations for diet
30 Several nutrients have an effect on the cells of the oral tis- modification.
sues. Which of the following nutrients assists in speeding C. Patient is involved in the diet analysis, evaluation, and
up wound healing? diet modification program and the counselor explains
A. Zinc each process to the patient.
B. Folate D. Counselor provides information regarding the diet anal-
C. Calcium ysis, evaluation, and diet modification and the patient
D. Phosphorus accepts all recommendations.
31 With regard to energy measurement, 1 kcal is the amount of 40 The recommended number of days for recording a diet diary is
heat produced to raise the temperature of A. 2 to 3 days.
A. 1 g of water 1° C. B. 4 to 7 days.
B. 1 kg of water 1° F. C. 7 to 10 days.
C. 1 kg of water 1° C. D. 14 days.
D. 1 g of water 1° F. 41 The basic diet for patients recovering from periodontal sur-
32 One of following activities is NOT included in the measure- gery is a
ment of the basal metabolism rate (BMR). Which one is the A. diet that is nutritionally adequate and modified for the
EXCEPTION? patient’s chewing ability.
A. Breathing B. clear liquid diet with supplements of vitamins A and E.
B. Maintaining body temperature C. diet high in fiber with six to eight glasses of water.
C. Beating of the heart D. diet that consists of foods recommended in the Food
D. Digesting food Guide Pyramid and high in vitamin C and cobalamin.
Nutrition 229

42 The MOST common barrier to care for the patient with an


eating disorder is Answer Key and Rationales s0260
A. communication.
B. transportation.
C. economic. 1 (A)  Animal proteins are considered complete (high-
D. physical.
quality) proteins. Composition of human tissue MORE
43 The MOST devastating oral effects of eating disorders are
related to
closely resembles that of animal tissue than that of
A. excessive exercising. plant tissue. Tissue similarities enable the human
B. binge eating. body to use animal proteins MORE effectively for its
C. frequent purging. MAIN functions: body maintenance, repair, growth.
D. malnutrition. Animal proteins contain ALL the essential amino ac-
E. laxative use. ids in amounts that are sufficient for the body.
44 Which of the following should immediately follow a purg- 2 (D)  Nitrogen balance refers to balance between input
ing incident? and output of nitrogen. If the body is NOT growing or
A. Rinsing with tap water in need of extra protein to recover from illness or infec-
B. Brushing with baking soda tion, ONLY enough protein is needed to equal input to
C. Brushing with fluoride gel
output. Negative protein balance exists when output is
D. Flossing
45 Which of the following is correct concerning the condition
greater than input (as occurs with semistarvation, infec-
hypokalemia? tion, fever, blood loss). However, if the body is grow-
A. Restriction of the consumption of aspartame ing, as during childhood, pregnancy, or recovery from
B. Ineffective metabolism of phenylalanine to threonine illness, MORE protein will be necessary to supply the
C. Lack of potassium body’s needs for tissue repair. Increased need for pro-
D. Lack of calcium tein intake is defined as positive protein balance.
46 An adult patient who is deficient in vitamin D could suffer 3 (B)  Phenylketonuria (PKU) is a disease in which the
from osteomalacia. A patient deficient in calcium could suf- liver is NOT able to metabolize the essential amino acid
fer from osteosclerosis. phenylalanine into the nonessential amino acid tyro-
A. Both statements are true. sine. By-products of phenylalanine build up in the body,
B. Both statements are false.
damaging the nervous system and therefore causing in-
C. The first statement is true, the second is false.
D. The first statement is false, the second is true.
tellectual disability (mental retardation). Recommended
47 Mineral oils interfere with the absorption of which of the that individuals with PKU NOT consume aspartame be-
following vitamins? cause it contains high levels of phenylalanine.
A. Vitamin A 4 (B)  Kwashiorkor is a disease caused by protein-energy
B. Vitamin C malnutrition and characterized by a severe protein defi-
C. Vitamin B6 ciency. Affected children who are approximately 2 years
D. Vitamin B12 old and raised in underdeveloped countries show evi-
48 Two types of food diaries are commonly used today. The dence of edema in the abdomen and legs but NO severe
24-hour food diary and the 7- to 10-day food diary are the wasting of body fat. Other signs include listlessness,
diaries MOST commonly used today. failure to grow and gain weight, changes in hair color.
A. Both statements are true.
5 (C)  Main function of carbohydrates is to provide
B. Both statements are false.
C. The first statement is true, the second is false.
energy. Although proteins can provide energy, use of
D. The first statement is false, the second is true. carbohydrate energy spares proteins for MAIN func-
49 Water makes up 50% to 60% of total body weight but is tion, to repair body tissues. Carbohydrates also aid in
NOT the main constituent of the body. Water is required on oxidation of lipids to prevent ketosis. ­Polysaccharides
a daily basis. and fibers provide for normal peristalsis. One of main
A. Both statements are true. functions of water-soluble B vitamins is to regulate
B. Both statements are false. metabolism of carbohydrates. Minerals such as phos-
C. The first statement is true, the second is false. phorus help regulate acid-base balance.
D. The first statement is false, the second is true. 6 (D)  Energy yield for carbohydrates is 4 kcal/g (same
50 Which of the following is a major mineral? as for proteins). However, fats can be broken down
A. Vitamin A
and used as energy source, yielding 9 kcal/g.
B. Boron
C. Selenium
7 (B)  Sorbitol is a sugar alcohol made from glu-
D. Calcium cose through hydrogenation. Sugar alcohols are
230 Saunders Review of Dental Hygiene

c­ arbohydrates and yield approximately same energy applying MORE pressure to circulate the blood because
as sucrose (4 kcal/g); both are nutritive sweeteners. of narrowing of walls of arteries from plaque buildup.
Advantage that sugar alcohols have over sucrose is Smoking is also risk factor for cardiovascular disease
that they do NOT promote caries because they are (CVD), as is low-density lipoprotein cholesterol (LDL)
NOT easily metabolized by bacteria in the mouth. reading ≥160 mg/100 mL and total blood cholesterol
Fructose is a monosaccharide and sweetest of ALL >240 mg/100 mL. Men who are >45 years and women
sugars; found in honey, fruits, and corn syrup. Man- who are >55 are also at greater risk for this disease.
nitol is another sugar alcohol made from mannose and 15 (C)  Fat-soluble vitamins include A, D, E, and K.
galactose. These vitamins are soluble in fat and NOT readily
8 (A)  Consistency of food plays a role in caries devel- excreted, so they can build up in toxic amounts. ALL
opment. Food with sticky consistency will remain B vitamins and vitamin C are water soluble.
on the teeth longer and have slower clearance time. 16 (B)  Preformed vitamin A (retinoid) is found in ani-
Dried apricots, like raisins, are a dried fruit and re- mal sources. Provitamin A (carotenoid) is found in
main on the teeth longer than other foods. Retention plant foods, especially deep green and yellow fruits
is a factor contributing to the initiation and progres- and vegetables.
sion of dental decay. Although soda pop, cake, and 17 (B)  Rickets is a bone disease that affects children as
milk contain moderate to large amounts of sugar, they a result of vitamin D deficiency. Bones soften because
are cleared from the oral cavity MORE quickly. of failure to calcify normally; include bowed legs.
9 (C)  Lipoproteins are a means of transportation 18 (C)  Vitamin E is a fat-soluble antioxidant; protects
through the bloodstream for cholesterol, triglycer- other substances from oxidation. Also protects RBC
ides, and phospholipids. Lipoproteins include chylo- membranes.
microns, very low-density lipoproteins, low-density 19 (A)  Water-soluble vitamins must be consumed regu-
lipoproteins, and high-density lipoproteins. larly because they are easily excreted or lost in cook-
10 (D)  High-density lipoproteins (HDLs) are capable of ing and therefore readily dissolved. Although vitamins
transporting cholesterol back to the liver, where it can do NOT provide energy, they play an active role in
be disposed of. Contain higher level of protein, mak- energy metabolism. Carbohydrates, lipids, proteins
ing them smaller and denser than very low-density require vitamin input for energy metabolism.
and low-density lipoproteins. HDLs are often referred 20 (D)  Niacin is found in protein foods such as fish, beef,
to as “good” cholesterol because elevated levels of turkey, and chicken. However, 1 mg of niacin can be
HDLs are associated with low risk of heart and vessel converted from 60 mg of the amino acid tryptophan.
disease. 21 (A)  Intrinsic factor is made in the stomach and is
11 (A)  Cholesterol is a sterol, one of three main types of needed for absorption of cobalamin (B12). If intrinsic
lipids. It is synthesized in the liver and makes bile to factor becomes inadequate or halts because of sur-
aid in the digestion (emulsification) of fats. Because gical removal of the stomach (weight loss surgery),
dietary cholesterol elevates blood cholesterol, it may absorption will be affected. Results in deficiency,
increase risk of coronary disease. causing pernicious anemia. Deficiency in extrinsic
12 (D)  Dietary lipids have essential functions in the factor, which must be obtained from food, also causes
body. They provide a concentrated source of energy: pernicious anemia but can be prevented by altering
9 kcal/g. In addition, they assist with the absorption diet to include animal foods.
of fat-soluble vitamins and provide insulation, es- 22 (A)  Strict vegetarian will be deficient in cobalamin
sential fatty acids, flavor and texture to foods, and a (B12) because major source is almost exclusively ani-
sense of satiety. Deficiency in fat-soluble vitamin A mal-derived foods.
may cause night blindness. 23 (C)  Scurvy is a disease caused by vitamin C defi-
13 (D)  Skim milk is considered nonfat food source. But- ciency. Deficiency in vitamin C is NOT uncommon
ter, coconut oil, beef contain high levels of saturated because this water-soluble vitamin is NOT stored for
fat. High intake of saturated fats (>30% of daily intake) long periods. FIRST sign of a deficiency is bleeding
is associated with development of ­ atherosclerosis, gingiva. Subsequently the muscles begin to ­degenerate
disease characterized by plaque buildup on arterial and the skin takes on a dry and scaly appearance.
walls, which increases risk of cardiovascular disease. Takes approximately 10 mg of vitamin C per day
To reduce amount of fat from diet, reduce amount of to prevent overt scurvy; however, DRI for adults is
saturated fats. Can be accomplished by eating such 90 mg/day (males) and 75 mg/day (females).
foods as fish, skinless poultry, low-fat or nonfat dairy 24 (C)  Although MAJOR function of calcium is to
products. form and maintain bones, it has other IMPORTANT
14 (B)  Atherosclerosis obstructs blood flow along arterial ­functions. Aids in the contraction of muscles and in
walls. High blood pressure (HBP) is evidence that heart is the blood coagulation process. Also may reduce the
Nutrition 231

risk of colon cancer. MOST of its absorption is in the 35 (B)  Anorexia nervosa is an eating disorder that is
upper intestine, with assistance of active vitamin D characterized by self-starvation (PEM, PCM). Indi-
hormone. Recommended daily allowance for both vidual, usually female, has a distorted body image
calcium and phosphorus is 1000 mg/day. (believes self to be overweight). Tendency to exer-
25 (A)  Calcium is MOST abundant mineral found in the cise excessively, and weight is usually 20% to 40%
body (followed by phosphorus). The bones and teeth below desirable body weight. Individual has low self-
store 99% of the body’s calcium. Plays a vital role esteem, is competitive in nature, and is NOT aware of
in keeping bones healthy and in preventing bone dis- problem.
ease, such as osteoporosis, in later life. 36 (C)  Bulimia is an eating disorder that involves epi-
26 (D)  Magnesium facilitates the operation of enzymes sodes of binge eating followed by purging by means
and assists with the relaxation of muscles after con- of vomiting and/or use of diuretics. Individual is usu-
traction. Foods rich in magnesium include dark green ally female, maintains normal body weight, is aware
leafy vegetables, whole grain breads and cereals, that behavior is abnormal.
nuts, legumes, and seafood. 37 (B)  Compulsive overeating is uncontrolled episodes
27 (D)  Sodium, MAJOR mineral, is active in maintain- of overeating without other signs of eating disorder.
ing the acid-base balance in the body. Along with cal- Individual eats foods that are easy to consume in
cium, potassium, and magnesium, plays a vital role large portions, such as noodles, rice, and foods that
in nerve conduction. Sodium also helps retain water are categorized as “junk” foods. MOST who compul-
by triggering a thirst reaction when it is consumed. sively overeat do so in response to stress and feelings
Thirsty individual will drink MORE water to help of depression.
balance the sodium-water ratio. 38 (B)  MOST effective counseling approach when deci-
28 (C)  Sulfur is present in ALL proteins and in the vi- sions are needed is nondirective. Counselor assists the
tamins biotin and thiamin. Aids in maintaining a nor- patient in understanding the needs and changes recom-
mal acid-base balance in the body. Sulfur compounds mended, and subsequently patient is active participant
preserve foods. in planning, implementing, evaluating the changes.
29 (B)  Water is the MAIN constituent of the body and 39 (C)  Directive approach in diet counseling involves a
comprises 50% to 60% of total body weight. Re- passive participant and decisions made by the coun-
quired on a daily basis and flows in and out of cells selor. In a nondirective counseling approach, counselor
through cell membranes. Roles include removal of assists with the process but patient is active participant
waste products, maintenance of body temperature, in analysis, evaluation, modifications of diet.
transport of inorganic nutrients, lubrication around 40 (B)  Diet diary should be recorded for 5 days and
joints, maintenance of normal kidney function. should include a weekend. This gives sufficient data
30 (A)  Adequate zinc intake is essential for the proper for a review of adequacy of food intake and determi-
development of sexual organs and bone. Also is nation of foods that contain sugar and frequency of
needed for DNA and protein metabolism and assists their consumption.
with wound healing. 41 (A)  Diet after recovering from periodontal surgery
31 (C)  Energy is measured by heat expenditure. One ki- should include MORE frequent feedings and MOSTLY
localorie is the amount of heat produced to raise the fluids on first day. On FIRST day after surgery, should
temperature of 1 kilogram of water 1° C. consume plenty of water, juices, and broths. Diet on
32 (D) Basal metabolism requirement (BMR) is ~60% second day can include blended fruits and vegetables,
to 70% of the total energy used by the body. Minimal oatmeal, dairy products such as milk shakes, ice cream.
amount of energy is necessary to maintain life at rest. For protein, may have meat broth and eggs. Comfort
The BMR involves circulation, beating of the heart, in chewing will determine what can be consumed.
breathing, maintaining body temperature. Does NOT 42 (A)  Communication is MOST common barrier
include specific dynamic activity, which is the energy to care with eating disorder. Denial, shame, guilt,
required for the digestion of food. lack of compliance, fear of weight gain are typical
33 (A)  MAJOR gland that affects the BMR is the ­thyroid ­reasons for NOT wishing to discuss eating disorder.
gland. Higher thyroid hormone levels increase the ­Transportation and physical barriers are insignificant,
metabolic rate. but economic considerations may be problematic
34 (C)  First statement is true; the second is false. Re- when dentition has been damaged from behav-
duction in weight can be accomplished by a reduction iors associated with eating disorders, ­ particularly
in energy input together with an increase in physical ­vomiting.
activity (energy output). Increase in physical ­activity 43 (C)  Frequent purging (vomiting) is responsible for
controls the appetite, aids in stress reduction, in- the MOST devastating oral effects of eating disor-
creases the BMR, raises self-esteem. ders. Purging is associated with severe erosion of
232 Saunders Review of Dental Hygiene

enamel surfaces, high incidence of caries, sensitiv- 47 (A)  Vitamin A is a fat-soluble vitamin, and others are
ity to sweets and temperatures. Excessive exercising, water soluble. Mineral oils interfere with absorption
bingeing, malnutrition, laxative use ALL have fewer of fat-soluble vitamins.
direct effects on the oral cavity. 48 (C)  First statement is true, and second statement is
44 (A)  Immediately after purging incident, should rinse false; 3-, 5-, or 7-day food diary is the MOST com-
with tap water (water mixed with sodium bicarbonate or mon today.
magnesium hydroxide may be used) to neutralize vomit 49 (D)  First statement is false, second is true. Water
acid. Brushing and flossing are contraindicated immedi- makes up 50% to 60% of total body weight but is
ately after vomiting because the tooth enamel is weak- main constituent of the body; however, water is re-
ened and brushing and flossing may abrade surface. quired on a daily basis. The body loses water ALL the
45 (D)  Lack of potassium causes hypokalemia. Lack time, and it must be replaced daily.
of calcium would cause osteopenia and osteoporo- 50 (D)  Calcium is a major mineral (macromineral) es-
sis. Liver CANNOT metabolize essential amino acid sential for strong bones and teeth. Boron is MAINLY
phenylalanine into nonessential amino acid tyrosine an essential plant nutrient, ONLY trace mineral in
with phenylketonuria (PKU). animals; physiological role in animals is poorly un-
46 (C)  First statement is true, and second statement derstood. Selenium is also a trace mineral. Vitamin A
is false. Osteosclerosis results from living in an is a vitamin essential for vision.
­overfluoridated area for a very long period; lacking
calcium puts a person at risk of osteoporosis.
Chapter  8

Microbiology and Immunology


MICROBIOLOGY    1. ALL are parasites and require viable (living) host
Microorganisms are life forms that normally CANNOT cell for replication; replicate once inside the cell
be seen with the unaided human eye. Possess character- and NOT by binary fission.
istics common to ALL cellular life in terms of physiol- 2. Classification by shape (symmetry), type of nu-
ogy, morphology, reproduction. Inhabit MOST niches of cleic acid, type of host cell, type of vector (agent
environment. Normal inhabitants of human body (normal of transmission), and/or associated disease pro-
flora). A medically IMPORTANT microorganism is one duced.
that inhabits human host and is considered a pathogen
(causes ­disease). Microbial Metabolism
• See CD-ROM for Chapter Terms and WebLinks. Microbial metabolism includes sum total of all biochemi-
cal reactions that occur in the cell.
Microbiology Classification • See Chapter 3, Anatomy, Biochemistry, and Physiol-
Microbiology spectrum includes prokaryotes, eukaryotes, ogy: body metabolism.
viruses. Classification of prokaryotes and eukaryotes em- A. Exoenzymes:
ploys Latin binomial system of genus and species. Ultra- 1. Enzymes that are excreted into external cellular
structures have important MAIN differences. environment.
A. Prokaryotes (bacteria) possess a relatively simple 2. Break down polymers and other molecules into
cellular structure and are MORE primitive cells. subunits that are small enough to be transported
1. Propagate by binary fission (split into two). into the cell.
2. Operationally classified into divisions and classes 3. May be potent exotoxins (e.g., Clostridium botuli­
based primarily on morphological and physiologi- num and C. tetani produce lipases that affect cen-
cal characteristics; divided into two major taxa: tral nervous system [CNS]).
eubacteria and archaeobacteria. 4. Examples:
a. Eubacteria (true bacteria): comprise bacteria a. Carbohydrases degrade polysaccharides to in-
of MOST importance to medicine; smallest dividual sugar residues.
bacteria. b. Proteases degrade proteins to amino acids.
(1) Mycoplasmas: smallest free-living bacteria. c. Lipases degrade lipid molecules.
(2) Rickettsias and chlamydials, obligate in- d. Hemolysins lyse red blood cells.
tracellular parasites: smallest members of B. Transport of molecules for metabolism:
bacteria. 1. Includes transport of individual molecules to the
b. Archaeobacteria: primitive nonpathogenic cell and concentration of these molecules within
bacteria normally associated with extreme or the cell.
unusual environments (e.g., thermophilic, halo- 2. Includes four MAIN transport mechanisms:
philic, methanogenic bacteria). a. Passive diffusion: random movement of mol-
B. Eukaryotes: possess relatively complex cellular ecules into and out of the cell, progressing from
structure, MORE highly evolved than prokaryotes. higher to lower concentration gradient (e.g.,
1. Fungi: NOT photosynthetic; include number of diffusion of water); cellular energy and carrier
pathogenic molds and yeasts. proteins are NOT involved.
2. Protozoa and helminths: NOT photosynthetic; b. Facilitated diffusion: transport of molecules,
include some pathogens. such as sugars, mediated by carrier protein that
3. Algae: photosynthetic; do NOT include patho- follows a concentration gradient; cellular en-
gens. ergy is NOT expended.
C. Viruses: acellular life forms that are principally c. Active transport: mediated by a carrier protein,
composed of proteins and nucleic acid; NOT a requires expenditure of cellular energy to move
cell, so cannot be classified as either prokaryote or molecules into the cell against a concentration
eukaryote. gradient.

233
234   Saunders Review of Dental Hygiene

d. Group translocation (phosphotransferase sys- 6. The reduced NADH2 molecule MUST be able to
tem): moves molecules into the cell against donate its electrons to an acceptor to regenerate
concentration gradient: oxidized NAD; thus defines fermentation and res-
(1) Carrier protein and energy (form of phos- piration:
phoenol pyruvate) are involved, and sub- a. Fermentation: organic molecule serves as final
strate (e.g., glucose molecule) becomes electron acceptor.
phosphorylated in the process. b. Respiration: inorganic molecule serves as final
(2) Molecule becomes negatively charged electron acceptor.
(e.g., glucose-phosphate), then retained in- F. Fermentation: energy-inefficient process that results
side the cell. in the production of two ATPs from the partial oxida-
(3) MOST efficient and MOST common form tion of one glucose molecule.
of transport in bacteria. 1. Strictly anaerobic process:
C. Metabolism: a. Glucose (electron donor) enters glycolytic path-
1. Involves metabolic pathways; once inside the cell, way.
a molecule (e.g., glucose) enters metabolic path- b. Glycolysis occurs, resulting in oxidation of
ways. 1 glucose molecule (C6) into 2 pyruvate mol-
2. Includes two major classes of metabolic reac- ecules (C3) and net production of 2 reduced
tions: NADH2 molecules and 2 molecules of ATP,
a. Catabolic reactions: degradative reactions that: which is referred to as substrate level phosphor­
(1) Produce energy (adenosine triphosphate ylation.
[ATP]). c. Pyruvate, organic compound, is central “hub”
(2) Produce carbon skeletons. of fermentation reactions and serves as final
(3) Produce reducing power. electron acceptor in simplest fermentation; re-
b. Anabolic reactions: biosynthetic reactions that: duced NADH2 donates its electrons directly to
(1) Use up energy (ATP). pyruvate, reducing it to lactic acid (C3), which
(2) Use up carbon skeletons. is released into external environment.
(3) Use up reducing power. 2. Accomplished by microorganisms capable of car-
D. Oxidation-reduction reactions: employed in cellular rying out fermentations that result in a wide vari-
metabolism: ety of metabolic end products, including CO2, H2,
1. Compound is oxidized when gives up or loses organic acids, and alcohols.
­electrons; reduced when receives or gains elec- 3. Ability or inability of organism to ferment differ-
trons. ent substrates is one of KEY features that enable
2. Reactions are simultaneous; when molecule is oxi- distinction among bacterial species.
dized, another is simultaneously reduced. G. Respiration: energy-efficient process that results in
E. Energy production (catabolism): production of 38 ATPs from complete oxidation of
1. Bacteria produce cellular energy, MAINLY in 1 glucose molecule.
form of ATP, by passing pair of electrons through 1. Can be either aerobic or anaerobic process:
the cell. a. Glucose (C6) enters the glycolytic pathway, with
2. Molecule that gives up the pair of electrons and same result as in fermentation: 2 ATP, 2 NADH2,
becomes oxidized is called the electron donor. and 2 pyruvate (C3) molecules are formed.
3. Molecule that ultimately receives these electrons b. Pyruvate becomes decarboxylated and bound to
after they pass through the cell is called the final coenzyme A, resulting in acetyl-CoA, CO2, and
electron acceptor. production of 1 reduced NADH2 molecule.
4. Molecules that accept and transfer these electrons c. Acetyl-CoA feeds into tricarboxylic acid (TCA)
(within the cell) from the donor to the final elec- cycle; one turn of the TCA cycle results in pro-
tron acceptor are called electron carriers; typical duction of 2 CO2 molecules and 4 reduced
electron carrier involved in energy metabolism is NADH2 molecules.
nicotine adenine dinucleotide (NAD). d. Second pyruvate molecule generated by gly-
5. When the electron donor (e.g., glucose) becomes colysis undergoes same series of reactions; at
oxidized, NAD receives the pair of electrons (and this point glucose is completely oxidized to 6
protons) and becomes reduced to NADH2; how- molecules of CO2; 2 ATPs have been produced
ever, the cell is limited in number of oxidized and a total of 12 reduced NADH2s have been
NAD molecules; soon all would become re- formed; cells then regenerate oxidized NAD
duced to NADH2 and energy metabolism would and generate MORE energy by means of the
cease. electron transport system.
Microbiology and Immunology   235

e. Electron transport chain is made up of a series


of electron transfer molecules (cytochromes) Microbial Observation
that are located in cytoplasmic membrane: Observation of microorganisms may be either macro-
(1) Reduced NADH2 donates its electrons and scopic or microscopic, depending on total number of
protons to electron transport chain, becom- organisms.
ing reoxidized to NAD. A. Macroscopic observation is necessary when organ-
(2) Passage of the electrons through electron isms are in large numbers:
transport chain results in production of 1. Turbidity (cloudiness) in liquid cultures.
3 ATP molecules, producing oxidative 2. Colonial morphology occurs when grown on solid
phosphorylation. surface such as agar agar Petri dish:
(3) At the end of electron transport chain the a. Defined as mass of cells that arise from single
electrons and protons combine with O2, re- cell.
ducing it to H2O. b. May provide distinguishing characteristics of
(4) Overall, there are a total of 12 reduced organism in terms of color (pigmentation), tex-
NADH2 molecules, which results in the ture, size, shape.
production of 36 ATPs (3 × 12) in addition B. Microscopic observation of individual cells is re-
to the 2 ATPs produced from glycolysis, for quired because of small size:
a total production of 38 ATPs from 1 glu- 1. Bacteria may range in size from ∼10 μm (size of
cose molecule. a red blood cell) to 0.1 μm (micrometer = 10-6 of
H. Differences between aerobes and anaerobes: a meter); viruses are even smaller; range in size
1. Aerobic respiration: from ∼0.25 μm to as small as 15 nm (nanometer =
a. Carried out by bacterium that can use O2 as the 10-9 of a meter).
final electron acceptor in respiration. 2. Compound microscopes (two or more lenses) pro-
b. Involves a combination of O2, electrons, and vide enlargement (magnification) and allow distin-
protons at the terminus of the electron transport guishing of fine details (resolving power).
chain, resulting in the formation of toxic inter- 3. Resolving power is dependent on wavelength of
mediates, superoxide anions (O2-), and hydro- light and employed with decreasing wavelength,
gen peroxide (H2O2). resulting in higher resolving power.
c. Requires bacterium that can grow aerobically to C. Bright field microscopy (via compound microscope):
possess two enzymes for removal (conversion) uses visible light and oil immersion:
of these toxic products. 1. MOST commonly used microscopy type (magnifi-
(1) Superoxide dismutase: converts O2- into cation = ∼1000× and resolution = ∼0.25 μm).
H2O2. 2. Specimens MUST be stained to be observed.
(2) Catalase (peroxidase): subsequently con- D. Dark field microscopy: observes living, unstained
verts H2O2 into H2O and O2. specimens; oblique light increases contrast so speci-
2. Anaerobic respiration: mens appear bright against dark background.
a. Carried out by bacteria that possess electron E. Phase-contrast microscopy: observes unstained speci-
transport chain but lack either or both su- mens; contrast is amplified by detection of small dif-
peroxide dismutase and catalase and are un- ferences in refractive indices between specimen and
able to remove toxic by-products of aerobic surrounding medium; some dental offices offer this to
respiration. show dental biofilm during oral hygiene discussion.
b. Uses oxidized inorganic compounds other than F. Fluorescence microscopy: visualizes objects that flu-
O2 as final electron acceptor; some common oresce, i.e., emit light, when exposed to light of dif-
inorganic compounds used in anaerobic respi- ferent wavelength:
ration include sulfate (SO4), sulfur (S), nitrate 1. Detects fluorescent objects illuminated by ultra-
(NO3), and carbon dioxide (CO2). violet or near-ultraviolet light; specimens absorb
I.   Anaerobic fermentation enables bacteria that lack light and emit visible light.
components of, or do NOT have, electron transport 2. Fluorescent dyes conjugated with specific antibod-
chain to grow anaerobically by means of fermen­ ies are basis of immunofluorescence; employed
tation. in identification of microorganisms and BEST for
J.  Biosynthesis (anabolism): fermentation and res- tracking of antigen-antibody complexes.
piration produce energy for driving biosynthetic G. Electron microscopy (EM): takes advantage of short
(anabolic) reactions of the cell for necessary syn- wavelength of electrons:
thesis of nucleic acids, proteins, carbohydrates, and 1. Greatly increases magnification and resolving
lipids. power.
236   Saunders Review of Dental Hygiene

2. BEST for observation of cellular ultrastructure and


viruses. Bacterial (Prokaryotic) Structure and Function
a. Transmission EM: BEST for thin sections of Small bacteria possess variety of shapes and sizes and
specimens. internal or external structures that aid in survival and are
b. Scanning EM (SEM): BEST for whole cells useful in identification of different bacteria. Survival is
and cells attached to surfaces. KEY to involvement with infectious processes.
A. Cellular shape:
Specimen Preparation and Staining 1. Coccus (plural, cocci), spherical:
Specimens MUST be prepared and then stained to be a. Diplococci (pairs): Streptococcus pneumoniae.
seen under light microscope because refractive index b. Chains of cocci: Streptococcus mutans, Strepto­
of MOST organisms is clear. Basic dyes (e.g., crystal coccus pyogenes.
violet, methylene blue, safranin) are positively charged c. Random clusters: Staphylococcus aureus.
and combine with negatively charged cell constituents. 2. Bacillus (plural, bacilli), cylindrical rods:
Acidic dyes (e.g., eosin, nigrosin, basic fuchsin) possess a. Large rods: Bacillus subtilis, Clostridium botu­
negative charge and combine with positively charged cell linum.
components. b. Small rods: Escherichia coli, Salmonella,
A. Specimens MUST be heat fixed to slide for bright ­Shigella.
field microscopy so that cells adhere and are NOT c. Chains aligned end to end: Streptobacillus
washed off during staining procedures. ­moniliformis.
B. Simple staining: employs single stain, such as methy- d. Fusiform types (threadlike): rods with pointed
lene blue or crystal violet; useful ONLY to distinguish ends.
cellular morphology. e. Filamentous types (threadlike): branching ba-
C. Differential stains: employ two different-colored cillus rods.
stains to indicate cellular morphology and distin- 3. Spirillum (plural, spirilla) (spirochetes), cylindri-
guish between groups of organisms and/or cellular cal with an amplitude (spiral with axial fibrils):
structures. a. Leptospira, Treponema, Borrelia.
1. Gram stain: separates ALL bacteria into two b. Vibrios have a comma shape: Vibrio cholerae.
groups: gram-positive (purple-blue) cells or 4. Palisade arrangement (“snapping division”):
gram-negative (red-pink) cells; stain reflects dif- ­Corynebacterium diphtheriae.
ferences in cell wall structure. 5. Pleomorphic (lack a defined cell shape): Myco­
a. MOST important staining procedure in micro- plasma pneumoniae.
biology, first step in bacterial identification. B. Cellular ultrastructure:
b. Procedure has four steps: 1. Internal cellular structures:
(1) Crystal violet (purple) is primary stain. a. Chromosomal DNA: circular and haploid; pos-
(2) Iodine is mordant that helps to “fix” pri- sesses MOST cellular genes and genetic infor-
mary stain to cells. mation.
(3) Alcohol is wash that decolorizes ONLY b. Plasmids: autonomously self-replicating, small,
gram-negative cells. circular DNA molecules, may be transferred
(4) Safranin (red) is counterstain that enables from cell to cell and may contain genes for
observation of colorless gram-negative ­resistance to multiple antibiotics; NOT present
cells. in all bacteria.
2. Acid-fast stain: BEST used to differentiate myco­ c. Ribosomes: composed of 30S and 50S subunits,
bacteria, causative agents of tuberculosis, which which make up 70S ribosomal RNA required
appear red, from other bacteria, which appear blue. for protein synthesis, distributed throughout the
a. Primary stain MUST be heated into cell and is cytoplasm.
NOT washed out, even with acid alcohol, be- d. Inclusion bodies: noted in some bacteria; con-
cause of hydrophobic waxy nature of cell wall; sist of energy storage molecules, such as poly-
thus the stain is acid “fast.” beta-hydroxybutyrate, that form granules or
b. Procedure has three steps: refractive bodies.
(1) Carbolfuchsin (red) is primary stain heated 2. Cytoplasmic membrane encompasses cytoplasm:
into cell. a. Composed of lipids, proteins, ions; provides
(2) Wash with 3% HCl in alcohol that decolor- hydrophobic matrix; exists as bilayer.
izes non-acid-fast cells. b. Provides permeability barrier and support for
(3) Methylene blue is counterstain that enables transporting proteins, as well as electron trans-
observation of the non-acid-fast cells. port system.
Microbiology and Immunology   237

3. Cell wall surrounds the cytoplasmic membrane, may remain viable for thousands of
which provides cellular shape and rigidity. years (the Dick Clark of the microbial
a. Cell wall: prevents cell from lysing because of world).
increased internal osmotic pressure. (b) Bacillus (aerobic) and Clostridium (an-
b. Periplasmic space: region between cytoplasmic aerobic) are the two major genera of
membrane and cell wall; contains various en- spore-forming bacteria.
zymes and proteins associated with transport. C. Chemical differences in cell wall structure between
4. Cell surface structures external to the cell wall: gram-positive and gram-negative bacteria:
a. Capsule: generally composed of polysaccha- 1. Peptidoglycan (murine) is present in BOTH gram-
rides; thick, tightly bound structure found in positive and gram-negative cells:
some; when present, may be antigenic and may a. Three-dimensional macromolecular net that
protect against phagocytosis and desiccation. surrounds the cytoplasmic membrane.
b. Slime layer: composed of polypeptides; ­exists b. Composed of a polysaccharide backbone (re-
as relatively thin layer over the cell surface peating N-acetyl glucosamine/N-acetyl mu-
of some; loosely bound and in some insta­­n­ ramic acid subunits) to which amino acid chains
ces MUST be present to provide virulence in are attached (to muramic acid residues).
­bacteria. c. Amino acid chains are further cross-linked to
c. Glycocalyx: surrounds the surface of some; amino acid chains on adjacent peptidoglycan
composed of a macromolecular network of pri- molecules by additional amino acid residues,
marily polysaccharide molecules; provides cel- thus providing three-dimensional structure.
lular adhesion to surfaces such as tooth enamel (1) Gram-positive bacteria possess thick,
(e.g., Streptococcus mutans). highly cross-linked peptidoglycan layers
d. Flagella: long (as long as 70 μm), thin protein because of high internal osmotic pressures;
(flagellin) filaments extending from the cell sur- stains purple (e.g., Streptococcus and Ac­
face; provide bacterial motility by turning in a tinomyces species; susceptible to antibiotic
circular motion. cillins and sporins).
(1) Allow movement toward or away from (2) Gram-negative bacteria possess thin,
chemical substances (chemotaxis). sparsely cross-linked peptidoglycan layers
(2) Bacteria without flagella MUST depend on because of relatively low internal osmotic
brownian motion (random activity by van pressures with lipopolysaccharides and li-
der Waals forces) for movement. poproteins; stay red-pink (e.g., Escherichia
e. Pili (singular, pilus) and fimbriae (singular, coli, Salmonella, Neisseria gonorrhoeae).
fimbria): short, proteinaceous, hairlike struc- 2. Gram-positive wall macromolecules:
tures that extend from the cell surface; associ- a. Teichoic acid: polymers of repeating glycerol
ated with gram-negative bacteria. phosphate and ribitol phosphate residues at-
(1) Pili: involved with the cell-to-cell transfer tached to peptidoglycan; may serve as antigenic
of genetic material between bacteria. determinants or receptor sites for bacterial
(2) Fimbriae: provide cell-to-cell adhesion viruses.
(e.g., pellicle formation, clumping together b. Lipoteichoic acids: structures similar to teichoic
of cells). acid, but with diglyceride residue(s) on one end
f. Endospore: large, generally spherical struc- that provide an anchor into the cytoplasmic
ture that forms within the cytoplasmic inte- membrane.
rior of the cell; one spore is formed per cell c. Various polysaccharides may bind to peptido-
and one bacterium is produced per spore upon glycan.
germination. 3. Gram-negative wall macromolecules: lipid-
(1) Spore coat: contains large amounts of dip- ­enriched outer membrane is formed externally to
icolinic acid and divalent cations (Ca++). the peptidoglycan layer by the presence of lipo-
(2) Spores: means of survival and NOT repro- polysaccharides and lipoproteins.
duction; formation is induced by deteriorat- a. Lipopolysaccharides (LPS) (endotoxins):
ing growth conditions. complex lipid; molecule linked to a polysac-
(a) Spores resist high temperatures, desic- charide chain oriented on the outer surface of
cation, ultraviolet light, and disinfec- the peptidoglycan, with polysaccharide chains
tants and are major reason for elaborate extending into the environment.
and harsh sterilization procedures; (1) Antigenic and may serve as receptor sites
prevalent throughout ­environment and for bacterial viruses.
238   Saunders Review of Dental Hygiene

(2) Important NOT only in systemic infections d. Death (decline) phase: cells begin dying expo-
but also in initiation and progression of nentially; now the nonviable cells outnumber
periodontal disease. the viable ones and nutrients are depleted.
b. Lipoproteins: lipid and extended amino acid B. Nutritional requirements for bacterial growth:
chains oriented in a manner similar to that of 1. Include liquid water, inorganic elements, carbon,
LPS. energy source for growth and reproduction.
c. Porins: globular hollow proteins that form pores 2. Bacteria may be physiologically classified based o2000
in the lipid-rich outer membrane. on how they derive their carbon and/or energy
D. Mycobacterial (acid-fast) cell walls differ from those (sorry, they do not drink coffee to keep them going
of typical gram-positive and gram-negative cells: like some of you are doing right now!).
o1860 1. Waxes (wax D) and mycolic acids cover peptido- a. Heterotrophs: obtain carbon from an organic
glycan layer; creates hydrophobic surface layer compound (e.g., glucose).
correlated with staining reaction of the bacteria b. Autotrophs: obtain carbon from inorganic
(e.g., causative agent of tuberculosis). source (e.g., CO2).
o1880 2. Basis for the formulations of many disinfectants is c. Phototrophs: derive energy from light.
whether they can break down this strong cell wall d. Chemotrophs: derive energy from the oxida-
(see later discussion in chapter). tion of chemical compounds.
3. Bacteria may be further subdivided based on the
Bacterial Growth and Nutrition carbon plus the energy source:
Bacterial growth is the increase in the numbers of bacte- a. Photolithotrophs: obtain energy from light and
rial cells by binary fission. Growth is influenced by the carbon from CO2.
physical environment and the availability of essential nu- b. Photoorganotrophs: obtain energy from light
trients. This is important NOT only in pathogenesis of and carbon from organic compounds.
systemic infections but also with development of infec- c. Chemolithotrophs: obtain energy from inor-
tious oral disease (periodontal disease and caries) and its ganic compounds (e.g., sulfur, iron) and carbon
MAIN etiological factor, dental biofilm (dental plaque). from inorganic CO2.
A. Bacterial growth: d. Chemoorganotrophs: obtain BOTH energy and
1. Generation time: amount of time it takes a single carbon from organic compounds, often from the
cell to divide into two cells (∼30 minutes for a typ- same molecule (e.g., glucose).
ical bacterium growing under ideal conditions). e. Saprophytes: grow on dead organic matter.
a. Cell numbers expressed as log to the base 10 f. Parasites: grow on living organic matter.
(e.g., 1 × 107 = 10 million cells) for convenience C. Media for bacterial growth:
because of the large number of bacterial cells 1. May be liquid or solidified by addition of 1.5%
that may be attained under ideal growth condi- agar agar (obtained from kelp); inert to bacterial
tions. metabolism.
b. Measured by visually observing turbidity of a 2. May be identified as one of several media based on
culture, increase in optical density, and increase its composition:
in protein concentration, or by direct cell counts a. Defined medium: exact concentration and exact
in a Coulter counter. composition of all compounds added are known.
2. Bacterial growth curve has four phases: b. Complex medium: exact concentration of com-
a. Lag phase: on inoculation into a new medium, ponents added is known; exact composition of
cells do NOT immediately divide; cells swell, all constituents is NOT known.
begin metabolic preparation for growth in the c. Selective medium: agents are added that inhibit
new medium, and use ATP. growth of one group of bacteria but NOT another.
b. Exponential (logarithmic, log) phase: cells di- d. Differential medium: distinguishes between
vide and generation time is at a minimum; ALL different organisms.
cells are viable (alive) and use nutrients; Gram e. Differential and selective medium: contains
stain is MOST reliable during this phase; anti- components of BOTH selective and differential
biotics and infection control agents such as dis- mediums.
infectants are MOST effective. f. Enriched media: for fastidious (difficult to cul-
c. Stationary phase: number of dying cells equals ture) organisms; contain such complex compo-
number of dividing cells (constant population nents as serum and red blood cells.
numbers); endospore formation may begin; g. Reducing media and special techniques are neces-
­nutrients are being depleted (used up) and toxic sary for the cultivation of anaerobic organisms that
metabolic products are increasing. CANNOT be grown in the presence of oxygen.
Microbiology and Immunology   239

(1) Reducing agents such as thioglycollate, b. Spontaneous mutations occur infrequently in a


hydrogen sulfide, cysteine are added to re- population, but recipient bacteria may instantly
move oxygen from the medium. acquire new genes, which allows changes to the
(2) Air may be replaced with nitrogen, hydro- genetic constitution and capabilities of the or-
gen, and/or carbon dioxide in special jars; ganism.
hydrogen generator and palladium catalyst c. Incoming DNA may be degraded by nucleases
used to remove oxygen; employed for cul- of the recipient or may be exchanged and inte-
tivating organisms on agar agar plates. grated with existing sequences in the recipient
D. Factors that influence microbial growth: DNA by recombination.
1. Oxygen requirements for growth and reproduc- 2. Types of transfer between bacteria:
tion: a. Transformation: gene transfer mediated by free
a. Aerobes (need O�2): require and use oxygen. “naked” DNA.
b. Anaerobes (NO O2): grow in absence of oxy- b. Transduction: gene transfer mediated by bacte-
gen; exposure to oxygen may be toxic. riophages (bacterial viruses).
c. Facultative anaerobes (okay O�2 or NO O2): c. Conjugation: plasmid-mediated gene transfer,
use oxygen or grow in absence oxygen. which requires cell-to-cell contact.
d. Microaerophiles (low O2): require and use ox- 3. Significance of conjugation and gene transfer:
ygen but in low concentrations. NOT only can these plasmids become donated
2. Hydrogen ion concentration (pH): MOST bacteria and spread like an infection throughout a bacterial
grow optimally near neutrality, pH 7.0 (see Chap- population, but they may be transferred between
ter 3, Anatomy, Biochemistry, and Physiology). different genera of bacteria and between species of
a. Acidophiles: grow ONLY at acidic values, ∼0 the same genus.
to 4.0 pH.
b. Alkalophiles: grow ONLY in alkaline INFECTIOUS DISEASES   
values, > 9.0 pH. Microorganisms generally MUST breach host defense p0080
c. Aciduric: tolerate an acid environment (e.g., barriers to initiate disease-causing reactions and trans-
Streptococcus mutans). fer to new hosts, whether it involves bacteria or vi-
d. Acidogenic: produce weak acids by fermenta- ruses. EXCEPTION is transmission by ingestion of
tion (e.g., Streptococcus mutans). preformed microbial (bacterial) toxin that can cause
3. Osmotic pressure: MOST bacteria grow optimally toxemia (poison in the bloodstream) (see later discus-
at salt concentrations near salinity (0.9%). sion). There are many overlaps between the routes of
E. Microbial growth relationships: transmission of infectious materials, since the over-
1. May be beneficial, harmful, or benign metabolic whelming goal of the microorganism is to survive at
interactions between microorganism and host; ANY means; there are also nonspecfic and specific host
interactions (Table 8-1) (just like the cast of defenses.
“Friends”). Each route of transmission is discussed here with the
2. Involve growing on or within other organisms MOST common examples of infectious disease that have
(hosts), including humans. practical association with major risks for exposure in
F. Bacterial genetics: microorganisms vary in the com- the dental setting. Risk factors for NOT achieving oral
position of their DNA and RNA and in the way their health are also listed when involved. Infection control
genetic codes are expressed, similar to the cells of the protocol and standard precautions to prevent transmis-
human body. sion are discussed later.
1. Genetic transfer between bacteria: A. Nonspecific host defenses: constitute first-line
a. Involves the passage of DNA from a donor bac- ­barriers and reactions against invading microorgan-
terial cell to a recipient cell. isms:
1. Physical defenses of skin cells and hair.
Table 8-1  Microbial interactions
2. Chemical defenses:
a. Acidic pH present in stomach (acid secretion),
Term Microorganism Host
vagina (microbial flora), and skin (lipids).
Symbiosis Benefits Benefits b. Microbe-digesting enzymes at mucosal sites
Commensalism Benefits Benign (e.g., lysozyme), including oral mucosa.
c. Blood proteins termed “acute phase” proteins,
Parasitism Benefits Benign or
harmed which harm microbes by being deposited on
their surface and facilitating clearance by
Pathogenesis Benefits Harmed
phagocytosis.
240   Saunders Review of Dental Hygiene

3. Biological defenses: antibiotic premedication patient; 17, Community Oral


a. Phagocytic cells, which internalize and degrade Health: epidemiology.
potential pathogens. A. Group A Streptococcus (e.g., “strep throat”) infection:
b. Complement: series of proteins that damage caused by S. pyogenes transmitted by ANY route, but
microbial membranes and facilitate clearance most commonly from infected person (or carrier)
by phagocytosis. via respiratory droplets or skin contact; nasal dis-
B. Specific host defenses: immune system recognizes charge is IMPORTANT source.
and responds to microbes and microbial toxins (dis- 1. Pathogenesis: site dependent, with rapid spread-
cussed later). ing; attachment to pharyngeal epithelium in the
C. Common ways for infection transmission: case of sore throat, with disease ranging from local
1. Contact: skin infections to fatal bacteremia and toxic shock
o2620 a. Direct contact with other infected host(s), which syndrome.
occurs during patient care; especially involves 2. Diagnosis: strep throat, pharyngitis, tonsillitis, red-
hands, such as NOT washing after restroom use ness and edema of mucous membranes, purulent
and between patients. exudate, fever, enlarged cervical lymph nodes; iso-
o2610 b. Nondirect contact: occurs with a contaminated lation of streptococci in smears or cultures; rapid
intermediate object, usually inanimate, e.g., detection of group A streptococcal antigen from
contaminated instruments or gloves that are throat swabs is available; may have symptoms of
NOT changed between patients. rheumatic fever (see below), including carditis,
o2630 2. Droplet (airborne): travel on dust particles or polyarthritis, subcutaneous nodules.
respiratory droplets that may become aerosolized 3. Prevention: unavailable.
(sneeze, cough, laugh, or exhale); increased with 4. Treatment: penicillin (Pen-Vee K) for 10 days.
aerosol production generated in dental practice, 5. Epidemiology of rheumatic fever: in a few cases, un-
e.g., ultrasonic use. treated strep throat may progress to rheumatic fever;
3. Vehicle: transmitted to host by contaminated mechanism responsible is antigenic cross-reaction
items, e.g., food, water, drugs, devices, ­equipment; with heart tissue (see later discussion of infective en-
includes fecal contamination of food, water, or docarditis), then immune response becomes directed
hands. against heart antigens (rheumatic heart disease).
4. Many infections are transmitted from insects or B. Tuberculosis (TB): caused by Mycobacterium tuber­
other living carriers (vector borne). culosis transmitted by respiratory route (droplets pro-
D. MORE common infection transmission routes: duced by coughing).
1. MOST common route is via respiratory system, 1. Pathogenesis: chronic infection spreads throughout
oral mucosa, eyes (do not ever touch your eyes the body via lymphatics and blood, with growth
at workplace). of bacteria in monocytes and intracellular growth,
a. Facilitated by the absence of protective skin cells. which stimulates T-cell lymphocytic response and
b. Potential access to various body cells (e.g., epi- results in cell-mediated hypersensitivity and asso-
thelial cells, lymphocytes, macrophages). ciated tissue damage, causing fibrosis and the cal-
2. Second MOST common route is fecal-oral (see cification of granulomas in the lungs and possibly
handwashing discussion). liver and kidneys.
3. Skin protection can be breached by cuts or abra- 2. Symptoms: fever, night sweats, chills, malaise,
sions of unprotected skin, e.g., reschedule patients cough, lymphadenopathy of cervical and subman-
with open infected lesions until healed. dibular lymph nodes; weight loss is common.
4. Other infections are transmitted by sexual mode; a. With lung, coughing and spitting blood.
protection afforded by intact skin is bypassed. b. With CNS: may include meningitis.
5. Perinatal route from the mother to the fetus and/or c. Oral: ulcer on tongue (common and highly in-
newborn (congenital infections). fectious lesion).
3. Diagnosis: by fever, fatigue, and specific organ
Bacterial Infections involvement; examination of smears and culture;
Many infections are bacterial in origin, but many have tuberculin (“TB”) skin testing using PPD (purified
vaccinations developed against them or are sensitive protein derivative) reveals positive cell-mediated
to antibiotics and other infection control procedures. response to antigens.
­However, resistant strains are now a developing problem 4. Prevention: bacille Calmette-Guérin (BCG) vac-
in healthcare. See later discussion on antibiotics. cination, NOT recommended because of ques-
u0030 • See Chapters 6, General and Oral Pathology: oral in- tionable efficacy, also now renders tuberculin
fections, infectious peptic ulcers; 9, Pharmacology: skin test false positive; avoidance of close contact
Microbiology and Immunology   241

with ­infectious persons is first defense; long-term


therapy with antibiotics is second defense.
a. Can be transmitted from patient to patient if
infection control procedures such as standard
precautions are NOT followed.
b. Avoid elective dental care for patients with ac-
tive TB; includes first 2 weeks of therapy or un-
til tests are negative.
5. Treatment: antibiotics (isoniazid [INH, Nydra-
zid], rifampin [Rifadin, Rimactane], pyrazinamide
[PZA], or ethambutol [Myambutol]); requires long
course (6 to 9 months) to eliminate disease; addi-
tional drugs if antibiotic resistance.
6. Epidemiology: close contact and massive exposure
enhance transmission rate; immunosuppression,
such as with acquired immunodeficiency syndrome
(AIDS), leads to enhanced susceptibility.
o14860 a. Increased risk for healthcare personnel, includ-
ing dental healthcare personnel.
o14870 b. Patients with human immunodeficiency virus
(HIV)-positive history and undiagnosed pulmo-
nary infection SHOULD be suspected of hav-
ing TB.
c. Poor compliance has resulted in emergence of
multidrug-resistant strains (MDR-TB), espe- Figure 8-1  Facial abscess caused by infection with Actino-
myces israelii.
cially with HIV infection.
C. Actinomyces infection: caused by Actinomyces israelii
(MOST common) and A. naeslundii, gram-positive,
rod-shaped, anaerobic or facultatively anaerobic; part b. Lab analysis of cerebrospinal fluid (CSF) for
of normal oral flora. cells, elevated protein level, and depressed
1. Pathogenesis: opportunistic infection (see later glucose level.
discussion in this chapter) that follows injury, in- c. Lab identification of bacteria present in CSF
troducing contaminated debris into tissues in im- ­is critical for rapid institution of antibiotic therapy.
munocompromised; with facial swelling (“lumpy 3. Prevention:
jaw”) and abscess formation in soft tissues be- a. Vaccines for several causative microbes (e.g.,
comes fibrotic (“wooden”) and can then discharge H. influenzae, S. pneumoniae, N. meningitidis).
tangled masses of bacteria (“sulfur granules”) b. Prophylactic use of antibiotics for close contacts.
(Figure 8-1). 4. Treatment:
2. Treatment: antibiotics such as penicillin (Pen-Vee a. Prompt use of appropriate antibiotics, antifun-
K) or amoxicillin (Amoxil, Larotid, Polymox) for gals, or antiparasitics.
6 months to year; surgery if extensive. b. Antiviral therapy when involving herpes sim-
D. Meningitis: inflammation of the membranes (me- plex virus.
ninges) of brain and spinal cord; MAINLY caused 5. Epidemiology:
by group B streptococci, Escherichia coli, Neisseria a. Occurs in all age groups; neonatal meningitis
meningitidis, Haemophilus influenzae, and Strepto­ (group B streptococci or E. coli) has HIGH
coccus pneumoniae (but also caused by enteroviruses, ­fatality rate.
mumps virus, fungi, parasites), with transmission by b. Use of H. influenzae vaccine has greatly re-
contact with another individual or other outside source duced incidence of childhood form caused by
or as result of invasion by microorganisms normally H. influenzae.
present (normal flora). c. AIDS patients are at risk for meningitis caused
1. Pathogenesis: occurs after microbial invasion of by Cryptococcus neoformans.
CNS. E. Legionnaire’s disease: caused by gram-negative ba-
2. Diagnosis: cillus; ­ Legionella pneumophila is MAIN species
a. Clinical, including fever, irritation, headache, transmitted to humans by way of infected water sup-
neurological signs. ply, primarily by aerosol.
242   Saunders Review of Dental Hygiene

1. Pathogenesis: infection of lower respiratory tract; period) following implantation or continually for
results in toxic effects on lung cells and ­stimulation persons at risk such those with comorbidities and
of inflammatory responses; exacerbates tissue immunocompromised health (e.g., diabetes melli-
damage. tus, systemic lupus erythematosus).
2. Diagnosis: lab culture, detection of bacterial anti- 2. Diagnosis: redness, warmth, and inflamma-
gen and seroconversion. tion around affected area, which may be stiff,
3. Prevention: decontamination of contaminated wa- drain pus, lose range of motion; radiographs
ter supplies by heating or chlorination. and magnetic resonance imaging (MRI) scans,
4. Treatment: antibiotic erythromycin (Robimycin, as well as blood samples and aspirated fluid for
E-Mycin, E.E.S.) for symptomatic infections. culturing.
5. Epidemiology: bacterium common in aquatic sys- 3. Prevention: antibiotic premedication before inva-
tems; dissemination by aerosols (e.g., from evap- sive procedures in high-risk patients, excellent oral
orative condensers, cooling towers, nebulizers) hygiene, monitoring past history of disease (high
plays important role in outbreaks; immunosup- risk for recurrence).
pression appears to play a role in susceptibility; 4. Treatment: early intervention is KEY; prolonged
risk factors are greater for healthcare facilities. bactericidal antimicrobial therapy, administered
F. Infective (bacterial) endocarditis (IE): inflammation parenterally in high dosages (see discussion below
of lining of heart, MAINLY from infected valves (es- on MRSA).
pecially mitral) that may lead to massive sepsis (blood H. Methicillin-resistant Staphylococcus aureus (MRSA)
infection), with all signs of infection (mainly fever); infections: caused by gram-positive S. aureus
ends in fatality. (“staph”) bacteria transmitted by contact.
1. Pathogenesis: presence of artificial (prosthetic) 1. Pathogenesis: deep wound infection in older
valves is MAJOR risk factor, with or without heart adults, chronically ill, persons with weakened im-
transplant; valves need replacement because of mune systems; leads to serious skin and soft tissue
congenital disease, IE, rheumatic heart disease, or infections, pneumonia.
drugs (phen-fen). 2. Diagnosis: small red bumps that resemble pimples,
a. Bacteremia (bacteria in the blood) from inva- boils, or spider bites, quickly turning into deep,
sive procedures, e.g., surgery, dentistry, intra- painful abscesses that require surgical draining;
vascular medical or drug abuse devices, that tissue sample, nasal secretions, detection of organ-
allow entry of infectious agents. ism’s DNA.
b. Caused by Staphylococcus aureus (MOST com- 3. Prevention: isolation of infections in healthcare
mon) or Streptococcus viridans; however, other settings, standard precautions for healthcare per-
microorganisms can be involved; thus name is sonnel with emphasis on washing hands, basic
now “infective” instead of bacterial. hygiene practices after sports, special precautions
c. Risk factors besides valvular complication: past with immunocompromised, testing if suspected,
history of IE, cancer, diabetes mellitus (DM), CORRECT use of basic antibiotics.
corticosteroid use, IV drug use, ­ alcoholism, 4. Treatment: antibiotic vancomycin and others
renal failure, systemic lupus erythematosus already proved effective against particular strains,
(SLE). such as gentamicin (Garamycin); resistant to all
2. Diagnosis: repeated blood cultures. β-lactam antibiotics (penicillins [Pen-Vee K],
3. Treatment: early intervention is KEY; prolonged amoxicillin [Amoxil], cephalosporins [Keflex]).
bactericidal antimicrobial therapy, parenterally in 5. Epidemiology: now community acquired
high dosages (see discussion below on MRSA). (CA-MRSA), NOT only in healthcare setting (i.e.,
4. Prevention: antibiotic premedication (prophylaxis) hospital).
before invasive procedures in high-risk patients, I. Osteomyelitis: infection of bone and bone marrow,
excellent oral hygiene, monitoring past history of usually caused by pyogenic bacteria or mycobacteria.
disease (high risk for recurrence). 1. Subclassified on basis of causative organism,
G. Prosthetic (artificial) joint infection: caused by Staph­ route, duration, anatomical location; Staphylococ­
ylococcus aureus or S. epidermidis (NOT viridans cus aureus MOST common.
group streptococci), the major oral bacterial spe- 2. Pathogenesis: microorganisms may be dissemi-
cies involed. nated to bone hematogenously (i.e., via the blood-
1. Pathogenesis: bacteremias from invasive proce- stream).
dures cause hematogenous seeding of total joint im- a. Spread contiguously to bone from local areas
plants (ALL joint surfaces are replaced), BOTH in of infection, such as cellulitis, or are intro-
early postoperative period and for 2 years (­critical duced by penetrating trauma, including such
Microbiology and Immunology   243

iatrogenic causes as joint replacement, internal areas of devitalized infected bone, known as se-
fixation of fractures, or endodontically treated questra, form the basis of a chronic infection.
teeth. c. The body commonly tries to create new bone
b. Leukocytes release enzymes that lyse bone around area of necrosis; new bone is an involu-
while trying to heal area; pus spreads into the crum.
bone’s blood vessels, impairing the flow, and 3. Treatment: prolonged antibiotic therapy via IV.

Clinical study  

Age 42 YRS Scenario

Sex ☒  Male   ☐  Female The patient presents at a walk-in medical and


dental clinic with weight loss, fever, oral sore.
Height 5’������
10”� A sputum smear shows the presence of acid-fast
bacilli. A chest radiograph shows evidence of
Weight 140 LBS
pulmonary lesions. Two days later, a test the
BP 98/67 patient took on his skin is determined to be
­positive; however, he says he has never
Chief ­Complaint “I am coughing all the time now.” received any vaccinations since he was a baby.

Medical ­History Serious car accident 5 years ago


Smokes cigars when he can get them
Used to abuse alcohol

Current Medications None

Social History Homeless for the last 5 years


Past work as a taxi driver

1. Does the patient pose a risk to healthcare personnel, 3. Positive TB test indicates active immunity (with
and if so, what precautions could be taken? T-cell lymphocytes and ­ cell-mediated immunity
2. What oral lesion may be present in his mouth? [CMI]) to tubercle bacillus, Mycobacterium tubercu­
3. What is the significance of the patient’s positive skin test? losis. Based on the symptoms and results of the other
4. How would therapy for condition be handled? lab tests and no other past medical history of vaccina-
tion, active infection with TB is likely.
1. Close contact and heavy exposure enhance tuber- 4. Therapy would consist of immediate antibiotic
culosis (TB) transmission by respiratory droplets, therapy and monitoring of compliance with drug
so healthcare personnel are at greater risk. Limiting therapy, which consists of pyrazinamide (PZA)
droplet exposure by wearing facemasks helps reduce combined with INH (Nydrazid) and rifampin (Rifa-
the risk of acquiring TB. Also, using a high- to inter- din, Rimactane). Ethambutol (Myambutol) is being
mediate-level disinfectant against the microorgan- added to the usual three-drug therapy regimen. Ther-
ism (tuberculocidal) on surfaces that are not covered apy requires long course (6 to 9 months) to eliminate
with disposables will help prevent transmission, as disease; additional drugs may be added if antibiotic
will other standard precautions. However, the patient resistance develops. Compliance is very important
should not be treated differently from other patients; for eradication of the infection but will be difficult
standard precautions should be strictly followed for for this patient to maintain in his present homeless
all patients. Unless it is necessary, such as with emer- situation.
gency care, elective dental care should wait until he
is on drug therapy and no longer contagious. This in- Bacterial Toxins
cludes the first 2 weeks of drug therapy or until tests Some bacteria may cause disease in humans because of
are negative. toxic (poisonous) properties present in protein molecules
2. Common oral sign of secondary TB is a tongue ulcer (toxins), which they carry and release, producing a tox-
with irregular border, which is a highly infectious le- emia. Disease from toxin exposure may NOT require
sion. Confirmed by biopsy and culturing, it resolves bacterial replication in the body, because in some cases
after treatment. exposure to the toxin alone is sufficient.
244   Saunders Review of Dental Hygiene

A. Enterohemorrhagic Escherichia coli infection: from b. In addition to paroxysmal cough, encepha-
toxin-producing enteric gram-negative rod, transmit- lopathy and bronchopneumonia occasionally
ted via contaminated food by fecal-oral route (food develop; disease has been stemmed by vaccina-
poisoning). tion in developed countries ONLY; recovery is
o3560 1. Pathogenesis: production of toxins, which cause associated with immune protection; increased
hemorrhagic colitis and hemolytic-uremic syn- numbers are due to LACK of vaccination by
drome (HUS, with kidney failure, hemolytic ane- choice.
mia, thrombocytopenia); NOT usually the harmless D. Toxic shock syndrome (TSS): toxin (TSST) from
strains that are part of normal gut flora, producing some strains of Staphylococcus aureus; tampon-
vitamin K or preventing establishment of patho- ­associated disease; toxin absorbed into blood after
genic bacteria within intestine. growth of bacteria in tampon; however, any staphy-
2. Diagnosis: strain-specific immunological detec- lococcal infection that produces enterotoxin or TSST
tion and growth in culture. may cause TSS.
3. Prevention: thoroughly cooking ground beef, 1. Pathogenesis: by activation of large numbers of
which is MOST common source. T lymphocytes by TSST; superantigen causes re-
4. Treatment: rapid administration of antibiotics, re- lease of inflammatory cytokines.
constitution of fluid and electrolyte imbalances, 2. Diagnosis: based on fever, shock, rash; lab detec-
treatment of disseminated intravascular coagulation. tion of bacteria and/or enterotoxin; also indicated
5. Epidemiology: fatal cases may occur; children and by organ system failure.
debilitated are MOST susceptible. 3. Prevention: use less absorbent tampons, change
B. Staphylococcal food poisoning: toxin (enterotoxin): more often.
from half of Staphylococcus aureus strains. 4. Treatment: antibiotics and supportive measures.
1. Pathogenesis: toxin action on gut nerves, stimu- 5. Epidemiology: recognition of relationship be-
lates vomiting and diarrhea. tween TSS and highly absorbent tampons has led
2. Diagnosis: by symptoms and epidemiology, may to ­ reduced incidence; sporadic cases continue to
be assayed. occur and require rapid recognition.
3. Prevention: complicated by normal presence of
S. aureus on skin and mucous membranes; use of ORAL MICROORGANISMS   
antiseptics and good personal hygiene may help Many microorganisms are found in a healthy oral cavity.
control spread to food; antibiotic therapy for carri- Vary according to age and location as noted in the Human
ers has limited use because of emergence of drug Oral Micro-biome Database (HOMD).
resistance and need for long-term therapy; heating A. Appearing soon after birth: coliform bacteria (disap-
of contaminated food may be sufficient to destroy pear soon after), Streptococcus, Staphylococcus, Lac­
the bacteria, although toxin remains active. tobacillus, Neisseria.
4. Treatment: symptomatic because of short incuba- B. Appearing with tooth eruption: Actinomyces spe-
tion period (1 to 8 hours) after exposure to toxin. cies (teeth-occlusal) and streptococci: S. sanguis
C. Whooping cough (pertussis): toxin from Bordetella per­ (MAINLY tongue dorsum, teeth-occlusal), S. sali­
tussis, gram-negative bacterium transmitted in respira- varius (MAINLY saliva, tongue dorsum), S. mutans
tory droplets. (teeth—MAINLY smooth surfaces), S. mitior (ALL
1. Pathogenesis: colonization of ciliated cells in the re- nonkeratinized tissue).
spiratory mucosa, followed by production of bacte- C. Dental biofilm (dental plaque): see Chapter 13, Peri-
rial toxins; MOST likely play a role in cell damage. odontology, for complete description and discus
2. Diagnosis: lab culture, demonstration of specific sion.
IgA antibodies, molecular detection of DNA.
3. Prevention: vaccination with whole-cell vaccines, Viruses
effective but occasionally toxic; LESS toxicity Viruses are the smallest and simplest infectious agents.
with acellular vaccines. Genome consists of RNA or DNA (NOT both). Structur-
4. Treatment: antibiotic erythromycin (Robimycin, ally, composed of protein molecules; some proteins are
E-Mycin, E.E.S.) reduces infectious period to 5 to glycosylated or associated with lipids. Obligate intracel-
10 days but has LITTLE effect on the disease. lular parasites, requiring host cell’s metabolic apparatus
5. Epidemiology: for reproduction. Two major groups: (1) infect prokary-
a. Period of highest infectivity occurs soon after otic (bacterial) cells, bacteriophages; (2) infect ONLY
infection, when levels of bacteria are high- eukaryotic cells.
est, but remains infectious for as much as A. Composition and morphology of virions (complete
5 weeks. infectious virus particles):
Microbiology and Immunology   245

1. Nucleic acid may be single-stranded (ss) or double­- 2. Genetic effects on the host (e.g., carcinogenesis).
stranded (ds) forms of either DNA or RNA. 3. Induction of harmful host immune responses (im-
2. Viral capsid: protein that surrounds nucleic acid; munopathology) as infected host attempts to fight
capsomers: capsid subunits. the virus.
3. Some viruses have an envelope that surrounds the E. Detection by culture of viruses in appropriate host
capsid and contains lipid acquired from membrane cells (bacterial, plant, or animal) grown in lab:
of host cell; BOTH viral proteins and glycopro- 1. Important tool for detection of viruses and deter-
teins are embedded in the lipid envelope, serving mination of their concentration by virus titration.
as antigens and virulence factors. 2. May cause microscopically visible damage and
B. Classification of viruses: increasingly based on nu- cytopathic effects.
cleic acid sequence homology; periodically updated 3. By structural analysis: electron microscopy BEST
as molecular data become available. used for definitive identification of viruses.
1. Traditional classification has been based on size, 4. By genetic technology: allows specific identifi-
symmetry, type of nucleic acid, and virus-host in- cation of viral nucleic acid sequences in cells or
teractions such as cellular location of replication tissues (e.g., by polymerase chain reaction [PCR],
(nucleus or cytoplasm), type of disease associated using viral-specific probes to amplify and detect
with the virus (e.g., hepatitis), tissue site from viral nucleic acid).
which the virus was isolated. 5. By presumptive identification of specific virus in-
2. Includes different families of viruses, classified ac- fection: determining specific host immune response
cording to criteria above; clinically IMPORTANT that occurs during infection (e.g., determination of
examples: HIV infection by identifying HIV-specific antibod-
a. Bacteriophages: plant viruses, animal viruses. ies in the blood of infected individual).
b. Picornaviruses (small “RNA” viruses): polio F. Antiviral therapy: see later discussion with each in-
and a common cold virus (rhinovirus). fectious disease discussed; generally designed to
c. Reoviruses (dsRNA): rotavirus, which causes interfere with viral replication (e.g., viral-coded en-
diarrheal illness. zyme) and not actually destroy viral particle.
d. Orthomyxoviruses (ssRNA): influenza virus.
e. Rhabdoviruses (ssRNA): rabies virus. Viral Infections
f. Retroviruses (ssRNA): human immunodefi- The closer that an infectious agent’s physiology is p0150
ciency virus (HIV). to ours, the MORE difficult it is to treat. Viruses, al-
g. Herpesviruses (dsDNA): herpes simplex virus though simple, tend to be difficult to treat because they
(HSV). become part of our cells and take over the cells’ ma-
h. Hepadnaviruses (dsDNA): hepatitis B virus chinery. Attacking the virus ultimately means attack-
(HBV). ing our cells, since they replicate once inside the cell.
C. Replication within host cell requires several basic See later discussion of antivirals. Persistent virus
steps (similar to creature in the movie “Alien”): infection is an infection that may involve latency or
1. Attachment (or adsorption): virus attachment some other mechanism that promotes continuous in-
to host cell surface, mediated by interaction be- ternal infection.
tween viral proteins and host cell surface receptor • See Chapter 9, Pharmacology: specific antivirals.
­molecules. A. Influenza (flu): caused by RNA virus (orthomyxovi-
2. Penetration: virus enters the cell through endocy- rus) transmitted by aerosols of respiratory secretions.
tosis or direct passage through cell membrane. 1. Pathogenesis: virus replication in airway cells; in-
3. Uncoating: virus disassembles and releases its nu- flammatory immune response contributes to tissue
cleic acid inside cell. damage.
4. Multiplication: viral nucleic acid replication and 2. Diagnosis: fever, malaise, headache, cough; de-
production of viral proteins, as virus takes over tecting virus antigens in cells from nasop­harynx.
some of the host cell biochemical machinery. 3. Prevention: vaccination; protects some but NOT
5. Maturation of virus particles: viral nucleic acid all individuals; vaccine preparation is complicated
and proteins are packaged into mature virions and by mutations and antigenic strains that arise.
released from the cell through rupture or budding 4. Treatment: amantadine (Symmetrel) and rimanta-
of cell membranes. dine (Flumadine), antivirals that hasten recovery
D. Pathogenesis can be divided into the following from some cases if started early.
­categories: 5. Epidemiology: yearly epidemics (higher rate than
1. Harmful effects of virus replication on host cells expected) and occasional worldwide pandemics
(structural and biochemical damage). (epidemic that spreads over a specific region).
246   Saunders Review of Dental Hygiene

B. Polio: caused by RNA virus (picornavirus) transmit- 6. Epidemiology: occurs sporadically in United
ted by oral route, from virus present in water or from States; vaccine has greatly reduced incidence.
close contact with carriers who shed the virus in pha- E. Mumps: caused by RNA virus (paramyxovirus) trans-
ryngeal secretions and feces. mitted by respiratory route (droplets), local virus rep-
1. Pathogenesis: infection of tonsils and small intes- lication, viremia, systemic dissemination.
tine cells, invasion of CNS, damage of nerve cells, 1. Pathogenesis: virus replication in parotid gland, testes
which leads to muscle paralysis; MAINLY self- (after puberty), ovaries, brain, kidneys, other organs.
limiting infections; immunity to the same strain of 2. Diagnosis: malaise, MAINLY parotid (parotitis) or
virus occurs after recovery; reactivation in adult as other salivary gland enlargement; specifically, by
postpolio syndrome (PPS). isolation of virus or seroconversion.
2. Diagnosis: symptoms ranging from minor to ma- 3. Complication: orchitis, painful inflammation of
jor, including paralysis; isolation of the virus or testicles, possibly with sterility.
demonstration of seroconversion. 4. Prevention: immunization is effective; combina-
3. Prevention: vaccination with BOTH live-­attenuated tion vaccine for measles-mumps-rubella (MMR).
and killed vaccines. 5. Treatment: none, supportive.
4. Treatment: none, supportive. 6. Epidemiology: worldwide epidemics (primarily
5. Epidemiology: worldwide pandemic; susceptibil- children); vaccine use has greatly reduced inci-
ity of children. dence and complications.
C. Rubeola (“measles”): caused by RNA virus (paramyxo- F. Varicella (chickenpox) and herpes zoster (shingles):
virus) transmitted by respiratory route; involves local caused by varicella zoster virus (VZV).
infection followed by viremia and systemic spread. 1. Pathogenesis:
1. Pathogenesis: replication of virus in various body a. Chickenpox caused by acute infection with VZV.�� o14890
cells, including skin (rash), respiratory tract, con- b. Shingles caused by reactivation of dormant o14900
junctiva; virus may persist for years; rarely, causes virus in tissue of cranial nerve V (trigeminal)
fatal brain infection. (SIMILAR to reactivation of herpetic gingivo-
2. Diagnosis: Koplik spots (early-appearing red spots stomatitis and/or recurrent herpes labialis from
with blue-white centers on buccal mucosa), fever, dormant virus in nerve tissue).
sneezing, cough, skin rash (head and neck moving c. Chickenpox: skin lesions; oral lesions may o4480
downward); virus isolation or seroconversion. ­occur; shingles: unilateral painful vesicles or
3. Prevention: immune protection after natural in- ulcers on oral/genital mucosa; facial area served
fection is lifelong; vaccination with measles im- by ophthalmic division of trigeminal is MOST
munoglobulin is highly effective and passive commonly affected; may be associated with im-
prophylaxis. munodeficiency.
4. Treatment: none, supportive. d. Complications: in eyes can cause blindness or o14910
5. Epidemiology: by age 12, MOST unvaccinated on skin may result in neuralgia (painful area).
children have had measles and thus become im- 2. Prevention: vaccines for BOTH chickenpox and
mune; vaccination has greatly reduced the inci- shingles.
dence in developed countries. 3. Treatment: supportive, possibly antivirals such as
D. Rubella (“German measles”): caused by RNA virus acyclovir (Zovirax).
(togavirus) transmitted by contact with respiratory se- G. (Infectious) mononucleosis (“mono”, “kissing dis-
cretions or directly to fetus. ease”): caused by Epstein-Barr virus (EBV), DNA
1. Pathogenesis: virus replication in respiratory and herpes virus (HHV-4) transmitted by saliva to phar-
other body cells. ynx and/or salivary glands.
o14880 2. Complication: congenital rubella with growth re- 1. Pathogenesis: 30- to 50-day incubation period, vi-
tardation; mental retardation; malformations of rus replicates in epithelial and B-lymphoid cells,
the heart and eyes; deafness; and liver, spleen, and development of enlarged lymph nodes and spleen,
bone marrow problems. persistence of virus in B lymphocytes, suppression
3. Diagnosis: typical rash (starts on face, then pro- of EBV by immunity.
ceeds downward) and lymphadenopathy; specifi- 2. Diagnosis: headache, malaise, fatigue, sore throat,
cally by virus isolation and seroconversion. palatal petechiae (red spots), enlarged spleen and
4. Prevention: protective immunity occurs after in- lymph nodes; virus isolation and seroconversion.
fection or vaccination; live-attenuated rubella vi- 3. Prevention: vaccine NOT available.
rus vaccine NOT for use during pregnancy, must 4. Treatment: none, supportive.
be given before. 5. Epidemiology: COMMON infection, establishes
5. Treatment: none, supportive. lifelong persistence; associated with two types of
Microbiology and Immunology   247

cancers, Burkitt’s lymphoma and nasopharyngeal 3. Treatment of progressive disease: antifungal am-
carcinoma. photericin B (Fungilin, Fungizone, Fungisome,
Amphocil, Amphotec); ONLY half is achieved in
Molds and Yeasts (Fungi) immunocompromised.
Molds and yeasts are eukaryotic cells that may repro- 4. Epidemiology: fungus resides in soil and is en-
duce either asexually or sexually. MOST are saprophytes demic in central United States as well as tropical
(grow on dead organic matter), but many are parasites world areas, transported to air and inhaled so that
(grow on living material), which are pathogenic for fungal spores may spread from alveoli to lymph
plants and animals. Some produce potent mycotoxins nodes, resulting in disseminated disease, which
or aflatoxins that may cause liver tumors. Others are may appear years after initial infection in immu-
responsible for such afflictions as ringworm, athlete’s nocompromised hosts.
foot, trench mouth, and thrush. Mold and yeast isolation B. Candidiasis: caused by overgrowth of Candida albi­
and culture generally employ techniques similar to those cans; however, normal flora present in oral cavity.
used for bacteria. 1. Pathogenesis: long-term antibiotics, corticoste-
A. Molds (mycelium): grow by production of hyphae; roid use, DM (sugar in blood), immunodeficiency
mycelia: forming cottonlike tangled mass. (see later discussion); serious infection that can be
1. Two types: life threatening and may occur in immunocom-
a. Nonseptate: long, stringlike cells that do NOT promised such as patients with cancer, transplant,
contain cross-walls. AIDS.
b. Septate: similar but contain perforated cross- 2. Types:
walls. a. Acute pseudomembranous candidiasis (thrush):
2. Identification: based primarily on morphology and curdlike superficial plaque easily removed,
modes of reproduction (e.g., types of asexual and leaving raw, bleeding area of oropharyngeal re-
sexual spores produced). gion.
B. Yeasts: special class of egg-shaped, elliptical, single-celled b. Angular cheilitis: cracking and/or redness
molds, larger than bacteria; do NOT produce mycelia. with labial commissures (corners of mouth);
1. Reproduce asexually by cell division called bud- may be mixed infection with staphylococcal
ding; asexual spores are NOT produced; sexual bacteria.
spores (ascospores) may be produced by some c. Chronic hyperplastic candidiasis: MORE pen-
upon two-cell mating. etrating and enlarged tissue form.
2. Can grow anaerobically by fermentation, with pro- d. Acute atrophic candidiasis: reduced tissue form
duction of alcohol (ethanol) and CO2. associated with antibiotic use.
3. Examples: e. Chronic atrophic candidiasis: associated with
a. Saccharomyces: important in bread, wine, and full or partial dentures; flat red outline mimics
beer production. denture shape.
b. Candida albicans: agent of thrush and oral can- f. Systemic: MAINLY genital but can be involved
dida (see discussion below). any body tissues.
4. Diagnosis: by symptoms, microscopic examina-
Fungal Infections tion, culturing.
Metabolically versatile and grow on typical media as well 5. Treatment: antifungal topical (e.g., nystatin [My-
as on cork, hair, paint, even polyvinyl plastics. Fungal costatin]), with caries risk from sugar content,
spores are everywhere and are MAINLY responsible for and/or use of systemic drugs such as amphotericin
contamination of bacterial cultures and media, as well as B (Fungilin) and clotrimazole (Diflucan); correct
infections (there is a fungus among us). denture care to reduce incidence of fungal infec-
A. Histoplasmosis: caused by Histoplasma capsulatum tions (see Chapter 15, Dental Biomaterials).
transmitted by inhalation. 6. Epidemiology:
1. Pathogenesis: pulmonary infection; with intracel- a. Using long-term antibiotics can lead to elimi-
lular growth of fungus, may disseminate to lymph nating natural competitors for resources and
nodes, liver, bone marrow, or brain, causing life- increase severity.
threatening infection in immunocompromised per- b. Solely treating with drugs may NOT give de-
sons and infants. sired results, and other underlying causes re-
2. Diagnosis: culture from blood, sputum, bone mar- quire consideration; may need to control health
row, CSF, or by biopsy and histological exam­ factors (e.g., DM, HIV), change environmental
ination. pH, hygiene, diet, other factors.
248   Saunders Review of Dental Hygiene

Clinical study  

Age 68 YRS Scenario

Sex ☒  Male   ☐  Female The patient visits his dental office for an oral
prophylaxis of his mandibular teeth. Intraoral
Height 5’�����
6”� examination reveals several new white patchy
­lesions that adhere to his buccal mucosal
Weight 225 LBS
surfaces and erythematous mucosa underlying
BP 115/95 his upper denture. Microscopic examination of
a small specimen mixed with saline and placed
Chief ­Complaint “My upper denture still feels sore even on a slide under low-power resolution reveals
after last week’s adjustment.” ovoid cells, some of which appear singly and
some of which are budding. When the specimen
Medical ­History Tired and seeming to gain weight in last
is cultured in the lab where it was sent by the
year but his whole family is overweight
dentist, irregular cream-colored colonies rapidly
Current Medications None appear. Further investigations by the patient’s
family physician after referral reveal a fasting
Social History Plumber blood sugar concentration of 200 mg/dL.

1. The patient is diagnosed with a fungal infection. Which any systemic infections. Risk of caries from sugar in
of the data mentioned are important to the diagnosis? drugs (if physician is comfortable prescribing a drug
What organism is the likely cause of the infection? with sugar for this patient), as well as correct denture
2. What is the likely source of the fungal infection? care, should be discussed and then monitored by the
3. What is the most important factor in the patient’s sus- dental staff. Annual intraoral examination would reveal
ceptibility to this oral infection? recurrent or resistant oral fungal lesions in the future.
4. What strategies for treating the patient would be ap-
propriate? PARASITES  
Parasites are eukaryotic animals that are found in two
1. Gross and microscopic appearance of lesion and subkingdoms, protozoa and metazoa. Protozoa are unicel-
growth in culture are consistent with oral Candida lular organisms. Metazoa comprise all the parasites other
albicans infection. Patient’s type is chronic atrophic than protozoa. Important group of the metazoa are the
candidiasis, which is associated with full or partial helminths, which are worms. Classification of parasites is
dentures and has a flat red outline mimicking denture based on morphology, cytoplasmic structure, locomotion,
shape. May be misdiagnosed as a denture sore from organelles, life cycles, reproduction.
physical irritation by denture; however, it does not A. Protozoa: unicellular but may be found in groups as
heal after the denture has been adjusted. colonies; range from ∼10 to 200 μm in length.
2. C. albicans is ubiquitous (environmental exposure is 1. Nutritionally holozoic; ingest solid food particles
common), and likely source was endogenous (from through mouth or opening, reproduce asexually or
Candida harbored in or on his body). C. albicans thus sexually.
represents opportunistic pathogen and causes clinical 2. Classified based primarily on mode of locomotion,
disease during breakdown in host defenses. transmitted by ingestion but sometimes by pen-
3. Fasting blood glucose level of 200 mg/dL suggests di- etration of skin; cyst formation may occur (helps
agnosis of diabetes mellitus (DM) type 2 (normal = with transmission of parasitic forms).
70 to 110 mg/dL). DM is known to increase suscep- 3. Protozoa classes:
tibility to some infections, including opportunistic a. Mastigophora (flagellates) move by means of
C. albicans. flagella; major pathogens include:
4. Several antifungals, both topical and systemic, includ- (1) Giardia lamblia: giardiasis, intestinal in-
ing amphotericin B (Fungilin), nystatin (Mycostatin), fection with cramps and diarrhea; fecal-
clotrimazole (Diflucan), may be useful in helping to oral transmission.
clear oral infection. DM must be evaluated by physi- (2) Trichomonas vaginalis: trichomoniasis,
cian and treated by nutritional therapy through diabetic genitourinary infection; sexually transmit-
nutritional counselor and possibly appropriate drug ted disease (STD).
therapy for both fungal infection and uncontrolled (3) Trypanosoma gambiense: African sleeping
DM. Medical examination may want to look signs of sickness; tsetse fly transmission.
Microbiology and Immunology   249

b. Sarcodina: flexible amoebas; major pathogen is b. Damage to macrophages and other body cells in
Entamoeba histolytica, which causes amebic dys- which T. gondii replicates.
entery and has fecal-oral or sexual transmission. 2. Diagnosis: prenatal toxoplasmosis in pregnant
c. Sporozoa: nonmotile; major pathogens include: woman, which is associated with blindness and
(a) Plasmodium malariae: malaria; trans- other defects, including neurological damage, in
mission from mosquito to the blood- newborn; microscopic examination, animal inocu-
stream. lation, and seroconversion.
(b) Toxoplasma gondii: toxoplasmosis, 3. Prevention: avoidance of cat feces; proper cooking
fatigue syndrome; transmission by cat of meats.
feces or undercooked meat. 4. Treatment: antibiotics, including sulfonamides (sulfa
d. Ciliata (ciliates): move by means of cilia; ma- drugs, sulfisoxazole [Gantrisin] and trimethoprim-
jor pathogen is Balantidium coli, which causes sulfamethoxazole [TMP-SMX, Bactrim, Septra])
severe dysentery and has fecal-oral transmi­s­ and pyrimethamine (Daraprim).
sion. 5. Epidemiology: MOST natural infections occur from
B. Helminths: multicellular complex organisms; elon- eating undercooked meat containing tissue cysts or
gated and bilaterally symmetrical worms; macro- from eating food contaminated with cat feces.
scopic in size.
1. Classes: include tapeworms, flukes, roundworms. Vector-Borne Infections
2. Pathogens: Trichinella spiralis causes trichinosis Many infections are transmitted from a vector (insect such p0210
in intestine and muscles; transmitted by ingestion as tick, mosquito, or flea that spreads infection). Carriers
of larvae in meat, especially pork. acquire an infectious agent from an infected individual or
waste and pass it to a susceptible individual, either directly
Parasitic Infections (e.g., bite) or indirectly (e.g., food or environment). Others
Medical parasitology involves study of parasitic proto- than those discussed here include Lyme disease (deer tick)
zoa, helminths, anthropoids. and West Nile (mosquitoes).
A. Giardia lamblia infection: caused by protozoan found A. Malaria: tropical disease caused by mosquito-borne
in small intestine of humans; transmission occurs by transmission of parasite (genus Plasmodium), with
ingestion of fecally contaminated food or water that transmission ­ occurring in cycle, from mosquito to
contains Giardia cysts; may survive for weeks or human.
months. 1. Pathogenesis: internal growth of parasite through
1. Pathogenesis: attachment of the parasite to wall of life cycle, resulting in chronic infection and dam-
small bowel; occurrence of persistent infections, age to red blood cells and other body sites; those
with weakness, abdominal pain, weight loss; how- with sickle-cell disease NOT affected (reason for
ever, may be asymptomatic. maintenance of disease in populations).
2. Diagnosis: microscopic examination of stool 2. Diagnosis: examination of stained blood speci-
­specimens. mens; hematological and liver function tests may
3. Prevention: personal hygiene and disinfection of be BEST.
water through filtration, boiling, and chlorination. 3. Prevention: avoidance of mosquito bites, eradica-
4. Treatment: antibiotic quinacrine hydrochloride tion of mosquitoes, prophylactic use of antimalarial
(Atabrine) and/or metronidazole (Flagyl). drugs, treatment of infected individuals; vaccines
5. Epidemiology: epidemics in institutional settings under development.
and outbreaks in wilderness areas (which suggest 4. Treatment: antimalarial chloroquine, drug of
transmission from animal sources [zoonosis]). choice for acute attacks; resistance may occur,
B. Toxoplasma gondii infection: caused by protozoan neces­sitating other drugs (suffix “-ine”) for ther-
that infects wide range of animals, including the cat apy or prophylaxis.
family; transmitted to humans by exposure to cat fe- 5. Epidemiology: has been resistant to eradication
ces (which carry oocyst), by eating undercooked meat in tropical areas; may be transmitted via contami-
that contains tissue oocysts, or from mother to fetus nated needles, by blood transfusions, from mother
(in utero). to fetus (in utero).
1. Pathogenesis: asymptomatic infection in humans; B. Hepatitis A infection: caused by RNA virus (hepatovi-
development of congenital toxoplasmosis when rus, HAV) transmitted via fecal-oral route; commonly
nonimmune mothers are infected during preg- caused by contamination of water or food, including
nancy. shellfish (unlike hepatitis E).
a. Immunosuppression, associated with toxoplas- 1. Pathogenesis: target organ is liver; causes necrosis
mosis. of liver cells and inflammatory changes.
250   Saunders Review of Dental Hygiene

2. Diagnosis: elevated levels of liver enzymes (ami- (STIs), microbial exposure of fragile mucous membranes
notransferase); anti-HAV antibodies. in body cavities allows direct infection of target cells near
3. Prevention: prophylaxis by administration of im- the site of exposure and also facilitates access to blood,
munoglobulins, type of passive immunity; NO particularly in cases of concurrent infection or trauma.
vaccine is available. Blood transfusions and sharing of contaminated needles
4. Treatment: none, supportive. during recreational drug injection also facilitate cross-
5. Epidemiology: common occurrence in families, ­infection between individuals.
institutions, camps, and armed forces; short in- • See Chapter 13: Periodontology: acute herpetic gingi-
cubation period of 15 to 45 days; onset is abrupt, vostomatitis; 6, General and Oral Pathology: hepatitis
with recovery by means of immune response. discussion.
A. Gonorrhea: caused by Neisseria gonorrhoeae; sexu-
s9000 clinical study   ally transmitted; infects MOSTLY mucous mem-
p9000 Scenario: The 5-year-old patient’s mother, contacted by branes of urethra in males or cervix in females.
phone by the receptionist, wants to wait to schedule the 1. Pathogenesis: invasion of epithelial cells and cell
child’s routine dental examination. Three weeks after start- damage associated with inflammatory response;
ing at her new daycare center because her mother just started dissemination to various body sites (e.g., brain,
back to work, she abruptly developed fever and general mal- heart, joints, peritoneal cavity).
aise of several days’ duration. After obtaining some lab data, 2. Diagnosis: gram-negative diplococci, culture of
the patient’s physician diagnoses hepatitis A virus infection. oxidase-positive colonies, immunological studies
Her mother says, “My daughter is just not feeling well.” (fluorescent antibody staining; agglutination), or
reaction with DNA probes.
1. What was the likely source of the hepatitis A virus in- 3. Prevention: NO vaccine available; condoms are ef-
fection for the patient? fective in preventing transmission.
2. What lab tests would help in the diagnosis of this in- 4. Treatment: cephalosporin (ceftriaxone [Rocephin]
fection? and quinolone (ciprofloxacin [Cipro]) antibiotics
3. What is the likely outcome of this case for the patient? (quinolone resistance is becoming widespread).
When will the patient be feeling better enough for her 5. Epidemiology: STD or STI often acquired from
dental appointment? asymptomatic sex partner.
4. Are other family members or other children at risk for B. Syphilis: caused by Treponema pallidum; sexually
this infection? If so, what might help prevent infection? transmitted, and ALL oral lesions are highly conta-
gious at each stage.
1. Transmission of hepatitis A virus (HAV) is primarily 1. Pathogenesis: complex; may involve almost ANY
by fecal-oral route. Case history suggests likely expo- tissue in the body; three stages possible unless
sure in institutional daycare setting, most likely from treated:
contaminated food or water. a. Primary: occurs at site of entry (usually mucous
2. Lab determination of elevated blood levels of ami- membrane); characterized by oral and/or geni-
notransferase; increases in antibodies to HAV would tal chancre (ulcer) that eventually heals.
confirm the diagnosis. b. Secondary: follows weeks to months later by
3. Complications of HAV infection are uncommon; dissemination of bacteria to various tissues; re-
within a few weeks, her healthy immune system will sults in development of new contagious ­lesions,
clear infection. which heal within several weeks after appear-
4. Her family and other children are at risk for HAV in- ance because of immunological defenses;
fection. Because contamination occurs through fecal- presence of oral mucous patch, painless ulcers
oral route, handwashing and aseptic handling of foods (Figure 8-2).
and drink are essential. Family members may receive c. Tertiary: occurs years later; infection reemerges
antibodies or immunoglobulins (Igs) as preventive or in cardiovascular system (CVS) or CNS in body
prophylactic measure, a type of passive immunity. tissues as gumma (soft tissue lesion) with resul-
tant dementia and may be fatal.
Infections Transmitted Sexually or by   2. Diagnosis: physical symptoms, microscopic iden-
Blood and Body Fluids tification of spirochetes, positive serological find-
A variety of microorganisms have adapted to sexual trans- ing (detection of antisyphilis antibodies).
mission, whereby protection afforded by intact skin is by- 3. Oral signs in infected offspring (congenital form): o5580
passed and exchange of body fluids that contain ­infectious Hutchinson’s triad, which consists of Hutchin-
agents or infected cells may occur. In these sexually trans- son’s teeth (mulberry molars, with constricted
mitted diseases (STDs) or sexually transmitted infections or poorly developed crowns and incisors, with
Microbiology and Immunology   251

fever, malaise, gingival inflammation; specific vi-


rus isolation and seroconversion.
3. Treatment: antivirals acyclovir (Zovirax), valacy-
clovir (Valtrex), and famciclovir (Famvir).
4. Epidemiology: widespread (HSV-1 greatest inci-
dence); LESS common: encephalitis, meningitis,
neonatal herpes.
D. Human papillomavirus (HPV) infection: caused
by DNA viruses that initially infect epithelial cells
of skin and/or mucous membranes, transmitted by
­intimate contact.
Figure 8-2  Oral mucous patch on dorsum of tongue from
1. Pathogenesis: range of clinical lesions, include
infection with Treponema pallidum with syphilis. oral and skin wart, plantar wart, flat wart, genital
condyloma, oral and laryngeal papilloma; tumor
resulting from virus infection of epithelial cells;
screwdriver or bell-shaped shaped crowns and linked with premalignant or malignant (cancer)
notched incisal edge), eighth cranial nerve deaf- oral and/or genital lesion(s) mainly in young
ness (labyrinthine disease), interstitial keratitis. persons and transmitted sexually.
4. Prevention: vaccines are NOT yet effective; trac- 2. Diagnosis: clinical appearance and/or viral type
ing of contacts and prophylactic treatment with detection.
penicillin (Pen-Vee K); condoms are effective in 3. Prevention: avoidance of sexual contact effective
preventing transmission. for genital infection; vaccination (Gardasil, Cer-
5. Treatment: antibiotic penicillin (Pen-Vee K). varix) of certain populations.
C. Herpes simplex infection: caused by two subtypes of 4. Treatment: removal or destruction of wart lesion
DNA virus family, which are HSV type 1 (HSV-1) by cryotherapy, surgery, cone biopsy, electrosur-
and HSV type 2 (HSV-2); transmitted BOTH non- gery (LEEP).
sexually (primarily HSV-1) and sexually (primarily 5. Epidemiology: majority of genital cancers are as-
HSV-2) through breaks in the skin or direct mucosal sociated with types 16 and 18.
contact with infected lesions; weeping of lesions may E. Hepatitis B infection: caused by DNA virus (HBV)
easily spread infection to other sites (intraoral �������
and/���
or transmitted by blood transfusion, IV drug use, sexual
extraoral) or other persons. and/or oral, mother to fetus (in utero), occupational ex-
1. Pathogenesis: virus-induced cell death and inflam- posure (healthcare personnel); present in blood, saliva,
mation; two forms possible: semen, vaginal fluids.
a. Primary infection (acute herpetic gingivosto- 1. Pathogenesis: long onset, chronic liver infection,
matitis): skin and/or oral mucous membranes; leads to liver dysfunction and cancer.
dormant virus in nerve tissues; reactivation 2. Diagnosis:
presents as edematous, red, painful gingival tis- a. By symptoms; vary and include jaundice, rash,
sues, with vesicles (blisters), MOST in children and arthritis.
6 months to 6 years.
b. Secondary infection: reactivation from dormant
nerve presents as recurrent vesicles that break to
ulcer(s) (“cold sores,” “fever blisters”); MOST
often on lip (labialis; Figure 8-3) or perioral tis-
sues or on fingers (whitlow); however, lesions
may also be noted on keratinized oral mucosa
(e.g., palate and gingiva).
o5650 c. Reactivation from dormant virus in nerve tissue
is SIMILAR to that of the pathogenesis of her-
pes zoster (chickenpox in childhood to herpes
zoster in adulthood); reactivation triggered by
tissue trauma (physical, chemical, or by exces-
sive sunlight), as well as an immunocompro-
mised or stressed state.
2. Diagnosis: by symptoms as noted; however, ini- Figure 8-3  Herpes labialis lesion on lower lip from infec-
tially may be asymptomatic or may present with tion with herpes simplex virus (HSV-1).
252   Saunders Review of Dental Hygiene

b. By specific immunological tests for viral an- a. Prolonged asymptomatic incubation period (∼10
tigens and antivirus antibodies or detection of years average) for MOST HIV-infected people,
elevated blood levels of some liver enzymes. during which the immune system becomes pro-
3. Transmission in dental setting, from patient to den- gressively impaired from reduction in CD4+ cells,
tal professional, through: leading to neurological and physical symptoms.
a. Injuries from contaminated sharps. b. Development of AIDS-defining infections and/
b. Splatter of contaminated blood and saliva onto or cancer follows, resulting in death if untreated,
open oral mucosa or skin. generally within few years.
c. Spread of contaminated blood to open lesions c. Early-stage symptoms: weight loss (wasting),
by ungloved hands or torn gloves. fever, diarrhea, generalized lymphadenopathy
4. Prevention: postexposure prophylaxis with passive (enlarged lymph nodes), night sweats, nausea;
­injection of hepatitis B immune globulin (HBIG) later stages: encephalopathy, dementia, neopla-
without seroconversion after vaccination; Recom- sia, opportunistic infections, gastrointestinal
bivax is currently used HBsAg, derived from yeast (GIT) disorders.
cells; avoidance of exposure to body fluids; HBV 2. Oral signs:
vaccination. a. Persistent generalized lymphadenopathy (PGL),
5. Treatment: none, supportive; specific therapy NOT angular cheilitis, dental erosion from frequent
available. vomiting (side effect of AZT use), xerostomia
6. Epidemiology: MOST recover completely; 5% to (drug induced), caries, bleeding.
10% become chronic carriers with higher risk for b. Hairy leukoplakia: opportunistic infection with
liver cancer. Epstein-Barr virus (EBV), DNA herpes virus,
F.  Hepatitis C infection: caused by RNA virus (HCV) presenting as bilateral white patch on lateral
transmitted like HBV, by blood transfusion, IV drug tongue with corrugated or “hairy” appearance
abuse, mother to fetus (in utero), sexual or oral, oc- (Figure 8-4); NO treatment needed unless for
cupational exposure (healthcare personnel); present esthetic purposes (acyclovir).
in blood, saliva, semen, vaginal fluids. c. Other opportunistic infections (especially with
1. Pathogenesis: little is known; replicates over time, LOW CD4+ lymphocyte counts): candidal, se-
causing mild disease in ∼10% of recipients. vere and recurrent aphthous and/or herpetic infec-
2. Diagnosis: viral nucleic acid, using virus-specific tions, severe and chronic gingival and aggressive
cDNA probe and seroconversion. periodontal infections (with linear gingival ery-
3. Transmission in dental setting: SAME as HBV. thema [LGE] of the marginal tissues), oral warts.
4. Prevention: NO vaccine. d. Kaposi’s sarcoma (see Chapter 6, General and
5. Treatment: antiviral interferon (INF)-alpha and Oral Pathology, and Figure 13-2).
ribavirin; type of natural protein (cytokine) pro- 3. Risk factors: vomiting and xerostomia from infec-
duced by immune system can be used to inhibit tion and/or drug use and risk of infection.
viral replication in infected cells. 4. Diagnosis:
6. Epidemiology: a. Case-defining infections (e.g., cytomegalovi-
a. MOST common cause of transfusion-associ- rus [CMV], Pneumocystis carinii, candidiasis,
ated hepatitis; blood donor screening currently
performed to eliminate antibody-positive
­donors.
b. Over 75% of infected persons become chroni-
cally infected carriers.
c. Chronic active hepatitis develops in ∼50%; can
progress to cirrhosis and liver cancer.
G. AIDS: caused by HIV retrovirus (contains reverse
transcriptase, which mediates transcription of viral
RNA into DNA); various strains exist; MOST AIDS
patients in North America are HIV-1 infected, with
transmission occurring via sexual contact from con-
taminated body fluids, by IV from contaminated
needles or surgical implements that penetrate skin, or
from mother to fetus (in utero). Figure 8-4  Hairy leukoplakia on the lateral tongue from
1. Pathogenesis: HIV replication in CD4+ T cells, opportunistic infection from Epstein-Barr (EBR) virus in per-
macrophages, and other body cells. son with HIV infection and AIDS diagnosis.
Microbiology and Immunology   253

histoplasmosis, TB, toxoplasmosis, cryptococ- d. Avoiding creation of aerosols during instrumen-


cosis); one or more opportunistic infections. tation (especially with sonics and ultrasonics).
b. Presence of HIV RNA in blood and reduced e. Diet counseling, if xerostomia and vomiting oc-
blood CD4+ cells; positive enzyme-linked cur, should emphasize adequate nutrition with
­immunosorbent assay (ELISA) screen for anti- noncariogenic foods to reduce caries risk.
HIV antibodies in blood (i.e., serocon­ve­rsion). f. Tobacco cessation counseling, as needed, to re-
c. Confirmation of anti-HIV antibodies in blood duce periodontal disease risk.
by immunoblot (MORE specific antibody g. Treating patient and caregivers with respect,
test), history of exposure; acute disease occurs kindness, compassion.
2 to 4 weeks after infection and is characterized 9. Patient or caregiver education:
by fever, adenopathy, pharyngitis, rash. a. Meticulous, frequent daily homecare, including
5. Prevention: flossing and brushing; self-care may be impos-
a. Avoidance of contact with blood and other sible during later stages of disease.
body fluids (tears, saliva, semen, vaginal secre- b. Prevention of dental disease to promote oral
tions, amniotic fluid, urine, cerebrospinal fluid) and systemic health.
of HIV-infected individuals; screening of blood c. Use of daily chlorhexidine mouthrinse and
used for transfusion. systemic antibiotics if severe gingivitis or peri-
b. Virus is easily killed outside the body (MORE odontitis is present.
easily than HBV or Mycobacterium tubercu­ 10. Epidemiology:
losis). a. Worldwide epidemic; highest transmission
6. Treatment of HIV infection: groups in North America include homosexual
a. Frequent assays of HIV RNA level in blood and males and IV drug users who share needles;
determination of blood CD4+ cell counts. in Africa greatest risk is via heterosexual
b. Highly active antiretroviral therapy (HAART), ­transmission.
type of combination therapy, includes reverse b. Fatality rate is high; more than half are due to
transcriptase inhibitors, such as AZT, D4T, ddI, immunodepressed state.
or ddC, and one or more viral protease inhibi- H. Cytomegalovirus (CMV) infection: caused by DNA
tors; extends length of disease-free period of herpes virus transmitted by sexual contact and close oral
infection by suppressing viral replication and and respiratory contact (saliva and urine shedding).
encouraging blood CD4+ count increases. 1. Pathogenesis: subclinical, 4- to 8-week incubation
7. Barriers to care: period, followed by infectious mononucleosis–like
a. Lack of communication: patient may be afraid syndrome.
to provide accurate or thorough medical infor- a. Involves congenital and perinatal infections, may
mation; attitudes and fears of dental profession- result in severe fetal and newborn infections.
als may interfere with communication. b. Causes severe infection in immunosuppressed
b. Transportation problems, if debilitated. hosts, such as transplant recipients, patients
c. Economic difficulties because of the high costs with AIDS or cancer.
of drugs, medical visits, and hospitalizations; 2. Diagnosis: malaise, myalgia, fever, lymphocyto-
also may be caused by loss of employment. sis, atypical lymphocytes; virus isolation and sero-
8. Professional care and homecare: conversion.
a. Medical consult regarding blood counts and 3. Prevention: unavailable.
risk of bleeding; know blood CD4+ cell counts 4. Treatment of CMV retinitis: ganciclovir (Cyto-
and current INR, if needed. vene) and foscarnet (Foscavir), Cidofovir (Vis-
b. Careful orofacial examination for signs of infec- tide), Valganciclovir (Valcyte).
tion and focus on maintenance of oral health. 5. Epidemiology: prevalence, determined by anti-
c. Frequent professional oral prophylaxis and body (demonstration of immune response to infec-
dental examination to reduce opportunity for tion), is ∼50% to 100%; latent infection is lifelong,
secondary (opportunistic) infections. with intermittent shedding.
254   Saunders Review of Dental Hygiene

clinical study  

Age 69 YRS Scenario

Sex ☒  Male   ☐  Female During an extraoral examina-


tion of a patient of record in
Height 6’�����
4”� the dental office, a cluster
of blisters in noted on lower
Weight 225 LBS
lip; lymphadenopathy is also
BP 115/95 noted.

Chief ­Complaint “I had a slight fever a few days ago and my lower lip felt
odd.”

Medical ­History Prostate cancer in remission after 10 years


Cataract surgery 5 years ago

Current Medications None

Social History Semiretired with landscaping business

1. What are the most likely diagnosis and cause of the appear on nonkeratinized lining mucosa. Herpetic
patient’s condition? ­condition lasts for 1 to 2 weeks.
2. What common signs of inflammation are associated with 5. Prodromal conditions, such as pain, burning, or ­tingling,
the condition? Is this an acute or chronic inflammation? occur before vesicle development; these are symptoms
3. Identify the major functions of inflammation. During of the condition, since they are subjective information
the healing process of the patient’s condition, are tis- about disease by patient. ­ Landscaping job is a factor
sues restored by regeneration or by repair? for condition because excessive exposure to sunlight is
4. What other locations might exhibit the condition? one of several stimuli known to trigger viral activity.
How long does this condition last?
5. Are prodromal conditions present? Are they signs or IMMUNOLOGY  
symptoms of the condition? Was the patient’s job a Immunology is controlled by the immune system, which
factor in this condition? protects the body from both harmful infections and the
development of some tumors. Bone marrow stem cells
1. In recurrent herpes simplex (herpes labialis, “cold generate the immune cells on a continuous basis through-
sore,” “fever blister”), etiological agent (cause) is out life. Body produces BOTH specific and nonspecific
herpes simplex virus (HSV). Virus exists in a latent or immune factors. Thus either immunosuppression or
quiescent state in trigeminal ganglion, and infection ­immunodeficiency predisposes the individual to infec-
follows stimulus. tions and some types of cancers.
2. Common signs of inflammation include fever, pain, • See CD-ROM for Chapter Terms and WebLinks.
swelling. Functioning of lip is often limited because of
discomfort. These are signs (not symptoms) because Antigens
they are objective evidence of disease that can be dis- Antigens (immunogens) are molecules that are recog-
cerned by clinician or patient. Acute inflammation that is nized as foreign and potentially harmful by the immune
immediate response to viral attack and results in redness, system. Present on all invading microorganisms; are con-
vesicle formation (increased vessel permeability), move- stituents of bacterial toxins; are targets for specific pro-
ment of leukocytes to the area (mainly neutrophils). tective immune reactions.
3. Major functions of inflammation are neutralization or
destruction of injurious agent, cleansing of necrotic Lymphocytes
debris, initiation of repair and regeneration. During Lymphocytes are the KEY antigen-specific cells of the
healing process, most epithelial tissues are restored by ­immune system. Antigen receptors on their surface
regeneration. recog­nize the presence of foreign microbial invaders,
4. Herpes labialis occurs on vermilion border of lips; which triggers their responses. Produce antigen-spe-
however, recurrent herpes may occur on keratinized cific molecules that focus immune responses on foreign
masticatory mucosa, such as hard palate and gingival ­microorganisms that have entered the body. Help activate
tissues. In contrast, aphthous ulcers (canker sores) antigen-nonspecific inflammatory responses produced
Microbiology and Immunology   255

by macrophages, neutrophils (polymorphonuclear leu- (d) IgE: attacks parasites and harms
kocytes [PMNs]), and other cells. Combined immune MOSTLY by causing allergies.
responses neutralize or destroy agents of pathogenic in- (e) IgD: participates in immune regulation.
fections, as well as some tumors. All cells of immune sys- (3) Antibodies may bind to surface of microbes
tem originate from bone marrow stem cells. and thereby trigger:
• See Chapter 3, Anatomy, Biochemistry, and ­Physiology: (a) Neutralization.
white blood cells. (b) Destruction by way of complement acti-
A. Lymphoid tissue and cell interactions: vation or phagocytosis by macrophages
1. B lymphocytes are highly dependent on bone mar- and PMNs.
row because much of their maturation occurs there b. T cells specifically recognize microbial antigens
(Figure 8-5). and carry out cell-mediated immunity (CMI).
2. T lymphocytes mature in the thymus. (1) T cells are also involved in:
3. B and T lymphocytes migrate to secondary lym- (a) Helping B cells respond by secreting cy-
phoid tissue (e.g., lymph node, spleen). tokines (regulatory protein molecules),
a. Secondary lymphoid tissues are sites of interac- which stimulate B cells.
tion with trapped antigen, also considered sites (b) Killing virus-infected cells by recogniz-
of initiation of immune responses (Figure 8-6). ing the presence of viral antigen on the
b. Accessory cells (e.g., macrophages, dendritic surface of the infected target cells; kill
cells) present antigen to lymphocytes, thereby tumor cells by a similar mechanism.
facilitating immune responsiveness. (c) Stimulating phagocytosis of microbes
4. Three functional types of lymphocytes: by macrophages and PMNs through re-
a. B cells produce antibodies (immunoglobulins, lease of activating cytokines.
class of blood proteins that contains antibodies) c. NK cells (natural killer cells) mature also in the
from subpopulation of plasma cells. bone marrow; large cells that are involved in
(1) An immunoglobulin (Ig) or antibody first line of nonspecific host defense; kill ­tumor
chemically binds to specific antigenic de- or virally infected cells and thus are NOT con-
terminants known as epitopes. sidered part of the immune response.
(2) Humoral immunity is mediated by anti­
bodies: Development of Immunity
(a) IgG: major blood antibody; crosses Immune responses occur when antigens are recognized
­placenta to protect newborn. by lymphocytes. Primary immunity describes the first
(b) IgA: protects mucosal sites (e.g., gas- contact with an antigen; may occur by natural infec-
trointestinal and genitourinary tracts); tion or by vaccination; takes time (varies, several days
present in tears, saliva, breast milk to weeks) for immunity to develop after primary immu-
­(secretory type), or blood (serum type). nization. Secondary immunity is the subsequent rapid,
(c) IgM: first antibody produced during highly vigorous immune response to a second contact
­immune response. with the same antigen, by way of reexposure or booster
­vaccination.

Vaccination
Antigenic determinant (epitope) Vaccination (active immunization) is exposure to antigens
in a manner designed to stimulate protective immunity.
See later discussion of required or suggested vaccinations
for dental personnel.
A. Killed vaccines: heat- or chemical-treated microorgan-
Ab on surface of isms or toxins; safe but in some cases may be LESS
B lymphocyte effective than live-attenuated vaccines.
B. Live-attenuated vaccines: genetically altered micro-
organisms that have lost virulence but still undergo
B cell
limited replication.
C. Molecular vaccines: composed of critical antigenic
determinants, derived as recombinant (from cloned
bacteria or yeast) or synthetic peptides; adjuvants
may be used to stimulate immune responses (types I
Figure 8-5  B lymphocyte.
and III—see later discussion).
256   Saunders Review of Dental Hygiene

(CD4 or CD8)

Antigenic peptide sequence


Cytokines
T cell cytotoxic T cells

Antigen- CD4+
presenting T cell Cytokine release
cell APC

B cell Antibodies
T cell receptor

(Plasma cell
precursor)
Figure 8-6  Cellular interactions of the immune system, including B and T lymphocytes.

Passive Immunization (3) Powdered rubber latex gloves may facili-


Passive immunization is the transfer of protective anti- tate inhalation of rubber latex molecules
bodies (immunoglobulins) into individual. Protection bound to cornstarch powder, which be-
is immediately established but MAINLY short lived come airborne, setting stage for an allergic
because of antibody catabolism. A common form of (hypersensitivity) response to inhaled rub-
postexposure prophylaxis after possible exposure to in- ber latex molecules; occasionally, severe
fection (e.g., hepatitis viruses A/B, HBIG) or toxin (e.g., or life-threatening hypersensitivity reac-
snakebite). Also occurs naturally during pregnancy, when tions occur and require emergency medi-
­maternal antibodies are transported into fetus, and during cal treatment (use of epinephrine or other
nursing, when breast milk antibodies are ingested. drugs).
(4) DHCP should be familiar with the signs
Allergies and symptoms of rubber latex sensitivity;
Allergies (hypersensitivities) are immune reactions with DHCP exhibiting symptoms of this type of
pathological inflammatory side effects. rubber latex allergy need medical consult
• See Chapter 10, Medical and Dental Emergencies: al- because further exposure could result in a
lergic emergencies in a dental setting. serious allergic reaction.
A. Types of hypersensitivity reaction based on cells in- (5) Procedures should be in place for mini-
volved: mizing rubber latex–related health prob-
1. Type 1: IgE-mediated humoral reaction induces lems among DHCP and patients while
basophils and mast cells to release chemical me- protecting them from infectious materi-
diators of inflammation (includes histamine). als; should include reducing exposures
a. Asthma and hay fever are common. to rubber latex–containing ­ materials by
b. Anaphylaxis is an emergency that requires im- using appropriate work practices, train-
mediate treatment with epinephrine; drugs (e.g., ing and educating DHCP, monitoring
penicillin, Pen-Vee K) and stings and bites may symptoms, and substituting other prod-
cause anaphylaxis. ucts without rubber latex where appro-
c. Rubber latex allergy: priate.
(1) Increasing incidence; may occur in anyone, 2. Type II and III: IgG antibodies involved in hu-
but at-risk groups include healthcare per- moral reactions:
sonnel (including dental healthcare person- a. By attaching to cell surface antigens and acti-
nel [DHCP]) and other people who work vating complement to injure or kill cells.
with rubber latex; MORE women than b. By forming circulating immune complexes,
men. which lodge in capillaries to then incite inflam-
(2) Contact with hospital gloves, catheters, matory damage.
tourniquets, dental dams, balloons, baby 3. Type IV (delayed-type hypersensitivity): T cells
bottle nipples, condoms, or other rubber la- release inflammatory cytokines, which stimulate
tex items may sensitize individual to rubber tissue damage as seen in cell-mediated reactions
latex molecules. in tuberculosis (also BOTH TB skin test and TB
Microbiology and Immunology   257

vaccine) and transplant rejection; includes allergy 2. Avoidance of allergen (e.g., food allergy such as
to nickel in restorations. peanuts, use other products that do not contain rub-
B. Treatment of allergies: ber latex in dental setting).
1. Drugs that suppress inflammation. 3. Desensitization: exposure to allergen by a different
a. Antihistamines. route, which alters immune response (e.g., modulating
b. Corticosteroids. from IgE to IgG) such as by use of allergy injections.

Clinical study  

Age 27 YRS Scenario

Sex ☒  Male   ☐  Female The patient has returned from his ­travels
late for his 6-month recall appoint-
Height 6’�����
2”� ment. Because he has not had ­dental
radiographs taken during the past 18
Weight 205 LBS
months, four bitewing radiographs are
BP 116/67 taken. Upon returning from discussing
the radiographs with the supervising
Chief ­Complaint “I don’t want to wear dentures like my dad.” dentist, the dental hygienist observes
that the patient’s lips appear swollen and
Medical ­History Allergy to milk, eggs, bee stings, pollen he complains that he is having difficulty
Carries EpiPen (self-administered epinephrine)
breathing. His breathing appears labored
Current Medications cetirizine (Zyrtec) 10 mg qd and is noisy (wheezing).

Social History Semipro golfer

1. What is the most likely cause of the patient’s symp- 4. Use of powder-free, hypoallergenic gloves helps
toms? to prevent rubber latex allergies. Residual rubber
2. What should be done immediately? latex proteins are the allergens responsible for the
3. What office preparations are necessary to ensure that rubber latex allergic response. These proteins bind
the patient does not have the same experience during to powder used to facilitate glove application and
his next dental appointment? are then transmitted to the wearer. Hypoallergenic
4. DHCP can also have this response. What precautions gloves are specially treated to remove most of re-
can minimize the risk its development? sidual rubber latex proteins responsible for allergic
response. DHCP must wash hands after wearing
1. The patient appears to be having anaphylactic reac- any rubber latex gloves. Oil-based lotions (e.g.,
tion. Swelling of lips indicates that he has had con- petroleum, mineral oil, lanolin, coconut oil) should
tact with an allergen, since he has a strong allergenic not be worn because these oils break down glove
history. Because rubber latex is a common allergen, barrier and cause release of more allergens, as do
exposure to powders on rubber latex gloves is a likely same lubricants on lips.
cause of this response.
2. Activate dental office emergency plan. Inform the su- Autoimmunity
pervising dentist, activate emergency medical service Autoimmunity occurs when an individual’s immune
(EMS) system, and provide oxygen if necessary. Su- system develops a breakdown in the normal control
pervising dentist must determine whether ap­propriate mechanisms that prevent immune reactions against
drugs from the emergency kit should be administered. “self ”-antigens. Spectrum of autoimmunity ranges from
Patient may self-administer epinephrine with use of mild, transient responses to potentially fatal responses.
the EpiPen; recommended to also have in kit. May be organ specific (e.g., directed against a particular
3. DHCP must identify all rubber latex–containing instru- endocrine gland, such as the thyroid) or more generalized
ments, equipment, and protective devices (anything that (e.g., directed against DNA). May be successfully treated
contacts patient) before seeing this patient again. DHCP with immunosuppressive and/or antiinflammatory drugs
should order replacement devices that do not contain such as corticosteroids or aspirin.
rubber latex to ensure safe care for all their patients. • See Chapter 6, General and Oral Pathology: autoim-
Many dental settings are becoming rubber latex free. mune disorders.
258   Saunders Review of Dental Hygiene

A. Body is self-tolerant: Table 8-2  Immunodeficiency types and clinical


1. When lymphocytes that are reactive for self- associations
­antigens develop in the body, they are automati-
cally eliminated, which provides protection from Name or type Clinical associations
internal immune attack.
Selective IgA Mild to severe infections at
2. If mechanisms that normally eliminate self-reactive deficiency mucosal sites ­(respiratory,
lymphocytes break down, surviving self-reactive gastrointestinal, genito­
lymphocytes may lead to autoimmune disease (i.e., urinary) because of low
clinically apparent immune reaction to self-antigen). levels of IgA
3. Reasons for breakdown in self-tolerance are NOT X-linked hypogam- Reduced levels of circulating
clear; some autoimmune disease may have a ge- maglobulinemia antibodies (immunoglobu-
netic basis because the major histocompatibility lins) leading to chronic
bacterial infections
complex (MHC, gene locus present in all mam-
mals that codes for immunologically important DiGeorge ­syndrome Absence of the thymus
leading to T-lymphocyte
cell surface proteins; this protein class is human deficiency, viral infections,
leukocyte antigen [HLA] in humans) governs im- and increased susceptibility
mune recognition and regulation; viral or bacterial to some cancers
infection may trigger some autoimmune diseases. Severe combined Absence of functional T and
B. Examples: immunodeficiency B lymphocytes is poten-
1. Hashimoto thyroiditis: hypothyroidism with anti- (SCID) tially fatal
thyroid autoantibodies.
2. Type 1 diabetes mellitus (DM): autoantibodies
against pancreatic insulin-producing beta cells.
3. Multiple sclerosis (MS): etiology unknown, but 3. May involve any of the molecules, cells, tissues,
autoimmune responses are caused by demyelin- or organs that constitute the complex immune de-
ation of CNS. fenses, including B and T lymphocytes, phagocytic
4. Myasthenia gravis (MG): muscle weakness with cells, complement molecules.
autoantibodies that disrupt nerve connections to a. Primary: intrinsic (genetic) defects in immu-
muscle. nity; caused by inheritance or mutation:
5. Rheumatoid arthritis (RA): etiology unknown, but (1) Severe combined immunodeficiencies (NO
has autoantibodies against antibodies (“rheuma- functional B or T lymphocytes).
toid factors”). (2) Variety of disorders linked to the X chro-
6. Systemic lupus erythematosus (SLE): anti-DNA mosome (generally B-cell disorders).
antibodies are present. b. Secondary: result of an insult to the immune
7. Sjögren’s syndrome: affects salivary and lacrimal system:
glands, resulting in a combination of dry mouth (1) Malnutrition.
and dry eyes. (2) Trauma from burns or toxin exposure.
(3) Virus infection (e.g., HIV with AIDS).
Immunodeficiencies (4) Long-term antibiotic or immunosuppres-
Immunodeficiency is a breaksown of one or more im- sant therapy.
mune system components; leads to NOT being able to (5) Radiation and chemotherapy.
fend off infection or cancer. Severity varies: may NOT be B. Complications:
clinically apparent (detected ONLY by lab test), may be 1. Opportunistic infection: infection with microbes of
mild and treatable with drugs, or may be severe enough normally low virulence that usually do NOT cause
to cause death as noted with some more uncommon types pathological changes, such as ­ actinomycosis,
(Table 8-2). MRSA, candidiasis (see earlier discussions).
A. Severity is often related to the location of the cellular 2. Reactivated infection: previous exposure does
defect: NOT stimulate lifelong protective immunity, such
1. Failure of lymphocyte stem cells to develop as HBV.
­properly leads to a wide-ranging immunodefi- 3. Incidence of some cancers is higher.
ciency. C. Treatment: immunotherapy:
2. Defect late in the developmental pathway of lym- 1. Passive administration of pooled Igs (also used for
phocytes may lead to a selective immunodefi­ciency prophylaxis for some viruses after contamination,
(e.g., the inability to produce ONLY a single im- such as hepatitis A or B [HBIG after needlestick
munoglobulin isotype, such as IgA). injury]).
Microbiology and Immunology   259

2. Transplantation of human stem cells (e.g., from (b) Can cause serious allergy (includ-
bone marrow or other sources), used with severe ing anaphylaxis); original molecule
combined immunodeficiency (SCID). was chemically modified in attempt to
3. Immune stimulators (e.g., cytokines) (also treat overcome resistance and allergy prob-
some cancers). lems, resulting in derivatives such as
4. Immunosuppression, drugs or antibodies directed ­ampicillin (Polycillin), methicillin
against immune cells (may also treat autoimmunity (Staphcillin), cloxacillin (Tegopen,
or graft rejection). Cloxapen), dicloxacillin (Dynapen,
Pathocil).
MICROORGANISM CONTROL BY ANTIBIOTICS (c) Ampicillin (Polycillin) is used for anti-
AND OTHER CHEMOTHERAPEUTIC AGENTS   biotic premedication for infective endo-
p0350 Antibiotic is a natural chemotherapeutic compound that carditis (IE) if NO history of ­allergy.
is produced by a bacterium or mold (may also be synthet- (2) Cephalosporins, monobactams, carbape­nems:
ically produced), inhibits growth of or even kills bacteria (a) Other classes of lactam antibiotics;
or molds, and may be taken internally or used topically. some have broader spectrum.
Similarly, antimetabolites are involved against bacteria’s (b) Cephalosporins: cephalexin (Keflex),
normal functions and antifungals inhibit or destroy fun- ceftriaxone (Rocephin), cefazolin (An-
gal growth. In contrast, antivirals do not kill the virus but cef), cefadroxil (Duricef), cefaclor (Ce-
interfere with viral replication so that the virus cannot clor), cefuroxime (Ceftin).
continue to reproduce to the level at which normal cel- (c) Cephalexin (Keflex), ceftriaxone (Ro-
lular functions are disabled. cephin), or cefazolin (Ancef) is used as
• See Chapter 19, Pharmacology: specific chemothera- antibiotic premedication for infective
peutic agents. endocarditis (IE) in individuals who are
A. Antibiotics: allergic to penicillin (Pen-Vee K).
1. Ideal antibiotic: 3. Antibiotics that act on cell membranes:
a. Broad spectrum; has ability to inhibit wide a. Agents are few in numbers because bacterial
range of microorganisms. and eukaryotic membranes are very similar,
b. Prevents development and spread of organisms which makes targeting cell membrane difficult.
resistant to antibiotic. b. Include topically applied agents:
c. Selective for pathogenic organism and NOT (1) Polyene antibiotics:
detrimental to human host (e.g., does NOT (a) Molecules that contain many double
cause allergy or toxicity). bonds; complex with sterols in the
d. Does NOT eliminate the normal flora of the hu- membrane, causing disruption.
man host. (b) BEST for fungal infections because
e. Has NOT been found. bacteria do NOT contain sterols in the
2. Antibiotics that act by inhibiting cell wall biosyn- membrane.
thesis: (c) Include amphotericin B (Fungilin) and
a. Includes IMPORTANT group of agents because nystatin (Mycostatin).
bacteria possess peptidoglycan but eukaryotic (2) Polymyxins (polymyxin B):
(human) cells do NOT; without a complete pep- (a) Small polypeptides that disrupt mem-
tidoglycan, bacterial growth will stop or cells branes by a detergent-like action.
will lyse. (b) MOST often combined with other
b. Inhibitors of earlier stages of peptidoglycan agents to provide broad-spectrum anti-
biosynthesis: cycloserine, vancomycin, ristoce- biotics.
tin, bacitracin. (c) BEST for stopping infection with severe
c. Beta-lactam antibiotics inhibit peptidoglycan burns; highly neurotoxic and nephro-
cross-linking, last stage of peptidoglycan bio- toxic, very poorly absorbed from GIT.
synthesis: active site of all of these antibiotics is 4. Antibiotics that act on DNA synthesis; inhibitors
the lactam ring, internal amide bond; effective of DNA gyrase:
primarily on gram-positive bacteria: a. Toxic to BOTH bacteria and the human host
(1) Penicillin (Pen-Vee K): because of the universal structure of DNA and
(a) First antibiotic discovered; can be ren- its biosynthesis; bacteria do differ from eukary-
dered ineffective by penicillinase, enzyme otes in that the DNA is circular and the enzyme
pro­duced by resistant bacteria that opens DNA gyrase is required for supercoiling and
lactam ring on penicillin molecule. DNA stability.
260   Saunders Review of Dental Hygiene

b. Include nalidixic acid, basis for class called qui- clarithromycin (Biaxin) are used for
nolones and novobiocin. antibiotic premedication for infective
5. Antibiotics that act on protein synthesis: endocarditis (IE).
a. Inhibits bacteria at the level of protein syn­thesis. (c) Clindamycin (Cleocin), spectrum simi-
b. Antibiotic action involving transcription: lar to the macrolides; used in dental
(1) Bacteria and eukaryotic cells differ in medicine as substitute for penicillin
structure of their ribosomal RNA; bacte- (Pen-Vee K) and for Bacteroides infec-
ria possess 70S ribosomes made up of 30S tions; used for antibiotic premedication
and 50S subunits and eukaryotes possess for infective endocarditis (IE); however,
80S ribosomes made up of 40S and 60S may cause pseudomembranous colitis.
­subunits. 6. Reasons for judicious (careful) use of antibiotics:
���������������������� o8020
(2) Prolonged use of these antibiotics may a. Have potentially harmful side effects (e.g., o8030
have detrimental side effects because mi- allergy, liver damage, bone deposition, nerve
tochondria of eukaryotic cells possess 70S damage).
ribosomes. b. Viruses are NOT affected by antibiotics. o8040
(3) Inhibitor, so few clinically useful antibiot- c. Indiscriminate use of antibiotics and unneces- o8050
ics are produced because of universal na- sary placement into the environment ultimately
ture of the transcription process. select for organisms that become increasingly
(a) Rifampin (Rifadin, Rimactane) in- resistant to the antibiotic, thereby reducing use-
hibits initiation of mRNA synthesis fulness (see later dicussion).
by ­ inhibiting RNA polymerase; pri- B. Antimetabolites: are NOT strictly considered antibiot-
mary agent when combined with oth- ics but are chemically derived chemotherapeutics that
ers in therapy for mycobacterial (TB) stereochemically resemble metabolic intermediates
­infections. that block important biosynthetic pathways in ­bacteria.
c. Antibiotic action involving translation: 1.  Sulfonamides (sulfa drugs, sulfisoxazole [Gan-
(1) Binding to 30S ribosome: trisin] and trimethoprim [combined SMX-TMP:
(a) Aminoglycosides (e.g., streptomycin, Bactrim, Septra]), which resemble paraaminoben-
neomycin [Mycifradin], kanamycin zoic acid (PABA); intermediate; reversibly block
[Kantrex], gentamicin [Garamycin], (bacteriostatic) folic acid biosynthesis in bacteria;
tobramycin) contain unusual amino highly allergenic.
sugars in their structures and are bacte- 2. Isoniazid (INH): stereochemically resembles nia-
ricidal because they bind irreversibly to cin and nicotinamide; appears to block synthesis
the 30S ribosomal subunit. of these two coenzymes; used in treatment of TB
(b) Tetracyclines (e.g., tetracycline [Achro- with other drugs.
mycin, Sumycin], doxycycline [Vibra- C. Antifungals and antivirals: see Chapter 19, Pharma- o14910
mycin], minocycline [Minocin]) are cology, for more discussion.
broad-spectrum antibiotics that contain D. Chemotherapeutic drug resistance:
a series of cyclohexyl rings in their 1. Mechanisms for antibiotic resistance:
structure and are bacteriostatic because a. Enzymatic inactivation (e.g., penicillinase).
of binding reversibly to 30S ribosomal b. Modification of the agent’s binding site.
subunit. c. Decreased cell permeability or loss of ability to
(2) Binding to 50S ribosome: inhibits by transport the agent into the cell.
blocking transpeptidation or translocation; d. Overproduction of biosynthetic intermediates
generally are bacteriostatic. that are inhibited by the agent.
(a) Chloramphenicol, quite toxic; re- 2. Origins of antibiotic resistance:
served for typhoid fever and infectious a. Random natural genetic mutations followed by
agents that are resistant to LESS toxic ­selection.
­anti­b­iotics. b. Gene transfer: multiple antibiotic resistance
(b) Macrolides (e.g., erythromycin [Robi- genes are carried on plasmids, which may be
mycin, E-mycin, E.E.S.], clarithromy- transferred from an antibiotic resistant donor bac-
cin [Biaxin], azithromycin [Zithromax]) terium to antibiotic sensitive bacterium, render-
are effective on gram-positive bacte- ing recipient immediately resistant to antibiotics.
ria and antibiotic of choice for myco- 3. Increasing antiviral and antifungal agent resistance o14920
plasmal infections and Legionnaire’s is also occurring (CDC monitors changing influenza
­disease; azithromycin (Zithromax) and ­viruses).
Microbiology and Immunology   261

­ olecules in the cell; when hit by a gamma


m
CONTROL OF MICROORGANISMS BY CHEMICAL ray, water molecule is converted into hydroxyl
AND PHYSICAL AGENTS   radical (•OH) and hydride radical (•H), which
Microbial growth is controlled by the use of both chemi- are potent oxidizing and reducing agents, re-
cal and physical agents. Agents range from those that in- spectively.
hibit or slow growth to those that kill microorga­nisms. b. BEST for sterilizing many items that would be
destroyed by high temperatures, such as plastic
Infection Control Agents Petri dishes, sutures, prosthetic devices, bra-
Effectiveness of these agents depends on a number of chioscopes.
­factors. 4. Ultraviolet (UV) light:
• See Chapter 12, Instrumentation: instrument process- a. Absorbed by thymine molecules present in the
ing and sterilization, biological monitoring. DNA of the cell; subsequently forms thymine
A. Effectiveness of controlling agents is modified by: dimers that inhibit DNA replication.
1. Number of microorganisms present. b. NO effect on bacterial endospores, which are
2. Concentration of the agent. resistant; does NOT penetrate glass, plastic, or
3. Temperature during treatment. other materials.
4. Duration of exposure to the agent. c. BEST for disinfecting surfaces and sometimes air.
5. Environment (e.g., microorganisms trapped in pus 5. Filtration:
or dried blood). a. Employs membrane filters with pore sizes in the
B. Cell death by chemical or physical agents: approximate range to retain bacteria and viruses.
1. Does NOT occur instantly or simultaneously b. BEST for sterilization of sensitive material that
for members of a microbial population; usually would be destroyed by other means, such as an-
­exponential; may take an extended time to elimi- tibiotic solutions, vitamins, serum, biological
nate the last remaining cell or endospore. fluids.
2. Resisted MOST by endospores. c. High-efficiency particulate air (HEPA) filters o14930
C. Physical agents and effects: are used to filter the air flowing into aseptic en-
1. Moist heat: vironments and out of potentially contaminated
a. Pasteurization: process of disinfection applied ones (e.g., containment facilities).
to dairy products, wines, beers. D. Chemical agents and effects:
(1) Items are heated to temperature high 1. MOST chemical agents are disinfectants because
enough to kill only known pathogenic or- endospores may NOT be killed by these agents.
ganisms that might be present. 2. Many are mild enough to be used as topical skin
(2) Milk, for example, is pasteurized when antiseptics.
heated at 65° C for 30 minutes, eliminates 3. Detergents: disrupt cellular membrane.
possible presence of Mycobacterium tu­ a. When cationic, possess a net positive charge;
berculosis (agent of TB), Brucella abortus examples: cetylpyridinium chloride and benzal-
(agent of abortion), and Coxiella burnetii konium chloride, representatives of quaternary
(agent of Q-fever); all can be transmitted amine compounds.
through milk. b. When anionic, possess net negative charge; ex-
(3) Many bacteria survive pasteurization, which amples: soap and sodium lauryl sulfate.
is why milk eventually sours. c. When nonionic, lack a charge; examples: Tri-
b. Autoclaving: process of sterilization that uses ton-X and Tween compounds, which are fairly
steam under pressure. ineffective disinfectants.
c. Dry heat: process of sterilization that uses elec- 4. Phenolic compounds: work by disrupting mem-
tric dry air oven. branes and ultimately precipitating proteins.
2. Cold temperatures: a. Based on the phenol nucleus, which by itself is
a. Refrigeration ONLY slows microbial growth caustic.
and metabolism. b. Types:
b. Freezing stops microbial growth and kills (1) Cresols: methylated phenol derivatives in
some but NOT all cells of a microbial popula- mixture form; example: Lysol.
tion; deep freezing at -70° C is also the BEST (2) Halogenated diphenyls: two phenol mol-
means of preserving many microbial cultures. ecules linked together with additional chlo-
3. Ionizing radiation (gamma rays): rine molecules; examples: hexachlorophene
a. Penetrates cells, can react with macro- (pHisoHex) and hexylresorcinol (ST-37 in
molecules, but reacts primarily with water mouthwash).
262   Saunders Review of Dental Hygiene

5. Alcohols, which disrupt cellular membranes and DHCP SHOULD become familiar with the hierarchy of
precipitate cellular proteins. controls that categorizes and prioritizes prevention strate-
a. A 100% alcohol is NOT effective because gies. Personnel subject to occupational exposure SHOULD
ONLY serves to dehydrate the cell. receive infection control training on initial assignment,
b. A 70% isopropyl alcohol is MOST commonly whenever new tasks or procedures affect their occupational
used, made by dilution with water. exposure, and at a minimum annually. Education and train-
6. Heavy metal compounds: ing should be appropriate to the assigned duties of specific
a. Contain mercury or silver molecule; react DHCP. Dental practices SHOULD develop written infec-
reversibly with sulfhydryl groups of pro- tion control program to prevent or reduce the risk of disease
teins to form mercaptide bonds; are bacte- transmission. This should include establishment and imple-
riostatic. mentation of policies, procedures, practices (in conjunction
b. Examples: mercury-containing merbromin with selection and use of technologies and products) to pre-
(Mercur­ochrome) and merthiolate (Metaphen), vent work-related injuries and illnesses among DHCP, as
silver nitrate. well as healthcare-associated infections among patients.
7. Oxidizing agents: Program SHOULD include principles of infection
a. React irreversibly with active hydrogen mol- control and occupational health, reflect current science,
ecules of proteins, such as sulfhydryl groups, and adhere to relevant federal, state, and local regulations
to form covalently linked disulfide bonds; are and statutes. Infection control coordinator (e.g., dentist
bactericidal. or other DHCP) knowledgeable or willing to be trained
b. Include iodine, chlorine, hypochloric acid should be assigned responsibility for coordinating the
(bleach), hydrogen peroxide. program. Health status of DHCP can be monitored by
8. Alkylating agents: maintaining records of work-­related medical evaluations,
a. React with reactive hydrogen molecules in screening tests, immunizations, exposures, postexposure
the cell, such as sulfhydryl groups, by adding management; records MUST be kept in accordance with
(alkylating) hydroxy methyl or hydroxy ethyl all applicable state and federal laws.
groups to these residues; are bactericidal. • See CD-ROM for related guidelines and terminology.
b. Include formaldehyde (solution), glutaralde­ A. Dental patients and DHCP can be exposed to patho-
hyde (solution), ethylene oxide (gas). genic microorganisms:
(1) Chemical vapor: uses combination of 1. High to moderate risk: cytomegalovirus (CMV),
chemicals (alcohol, formaldehyde, ketone, hepatitis B (HBV), hepatitis C (HCV), herpes
acetone, and water) instead of water alone simplex virus types 1 and 2 (HSV-1, -2), Myco­
to create a vapor for sterilizing. bacterium tuberculosis (TB), staphylococci, strep-
(2) Ethylene oxide: major gaseous sterilant tococci, other viruses and bacteria that colonize or
for sterilizing nonbiological materials at infect oral cavity and respiratory tract.
room temperature. 2. Low risk: HIV transmission in dental settings in
United States compared with HBV/HCV and others.
INFECTION CONTROL PROTOCOL   a. Risk increases with exposure to large volume of
Planned system of preventing disease transmission is blood, as indicated by deep injury with a device
an important element of dental setting protocol. MOST that was visibly contaminated with patient’s
diseases that are transmissible in the dental setting are blood, or procedure that involved needle placed
difficult or impossible to cure; consequently, prevent- in a vein or artery.
ing transmission from patient to patient, from patient to b. Risk increases if exposure was to blood from
clinician or dental healthcare personnel (DHCP), and patient with terminal illness, possibly reflecting
from DHCP to patient is of MOST importance. DHCP higher titer of HIV in late-stage AIDS.
refers to ALL paid or unpaid personnel in dental health- B. Pathogenic microorganisms transmitted in dental set-
care setting who might be occupationally exposed to tings through:
infectious materials, including body substances and con- 1. Direct contact with blood, oral fluids, or other pa-
taminated supplies, equipment, environmental surfaces, tient secretions.
water, or air; includes dentists, dental hygienists, dental a. Splatter involved in delivery (rinsing mouth,
assistants, dental laboratory technicians (in-office, com- using prophy angle to polish, debridement with
mercial), students and trainees, contractual personnel, sonic and ultrasonic scalers).
and other persons NOT directly involved in care but po- b. Contaminants through capillary action for next
tentially exposed to infectious agents (e.g., administra- patient:
tive, clerical, housekeeping, maintenance, or volunteer (1) Ultrasonic and sonic scalers and high-speed
personnel). handpieces and air-polishing devices.
Microbiology and Immunology   263

(2) Air-water syringes and dental unit water- from pathogenic microorganisms that can be spread by
lines (may serve as contaminated biofilm blood or any other body fluid, excretion, or secretion.
reservoir). Standard precautions apply to contact with (1) blood; (2)
(3) Self-contained water units, antiretraction all body fluids, secretions, and excretions (except sweat),
valves, and backflow preventive devices regardless of whether contaminated by blood; (3) non-
help eliminate risk that patient will draw intact skin; and (4) mucous membranes. Saliva has AL-
back contaminated water. WAYS been considered a potentially infectious material
c. Needlestick and sharps injuries; open skin in dental infection control; thus NO operational differ-
wound or mucous membrane. ence exists in clinical dental practice between “universal”
2. Inhalation of airborne microorganisms that can re- precautions and standard precautions.
main suspended in the air for long periods. • See CD-ROM for related information about standard u9000
a. Aerosols created by: precautions.
(1) Coughing, sneezing, or breathing. • See Chapters 5, Radiology: radiology practice ­protocol;
(2) Prophy angles, brushes, or cups. 11, Clinical Treatment: clinical practice protocols; 15,
(3) Using BOTH controls on air-water sy- Dental Biomaterials: lab control protocols.
ringes. A. Taking and reviewing detailed patient medical
(4) Ultrasonic and sonic scalers; high-speed histories:
handpieces and air-polishing devices. 1. Complete, updated histories are important for the
(5) Failure to use high-volume evacuation protection of all parties involved in providing and
(HVE, high-speed suction). receiving dental care.
b. Containing and reducing the production of 2. Medical histories identify prescription and over-
aerosols MUST be considered (see later discus- the-counter (OTC) drugs.
sion). 3. Drugs the patient is taking can give clues to
3. Indirect contact with contaminated objects (e.g., present medical status and enable dental care
instruments, equipment, or environmental sur- professionals to explain possible impact on
faces). oral health that the condition or the drugs may
4. Contact of conjunctival, nasal, or oral mucosa with have.
droplets (e.g., spatter) containing microorgan- 4. Combination of written and oral questions gives
isms generated from an infected person and pro- patient an opportunity to explain condition.
pelled short distance (e.g., by coughing, sneezing, 5. Noting specialties of listed physicians gives clues
or talking). as to medical conditions the patient is monitoring.
C. Infection through any of these routes requires that all a. Increased trust between patient and provider
of the following conditions be present: promotes patient honesty in divulging medical
1. Pathogenic organism of sufficient virulence and in history.
adequate numbers to cause disease. b. Dental office MUST maintain the confidential-
2. Reservoir or source that allows the pathogen to ity of patients’ medical records (see Chapter 18,
survive and multiply (e.g., blood). Ethics and Jurisprudence).
3. Mode of transmission from the source to the host. B. Appropriate immunization of DHCP (see earlier dis-
4. Portal of entry through which the pathogen can en- cussion of immunology and the CD-ROM):
ter the host. 1. HBV vaccination:
5. Susceptible host (i.e., one who is NOT immune). a. Full-time, part-time, temporary, and probation-
D. Occurrence of these events provides the chain of in- ary employees MUST begin series during first
fection; effective infection control strategies such as 10 working days of employment.
standard precautions prevent disease transmission by b. New employees can continue to provide pa-
interrupting one or more links in the chain. tient care during 6 months needed to complete
E. Avoiding occupational exposures to blood is the series; new employees may refuse, but MUST
MAIN way to prevent transmission of HBV, HCV, read and sign “Refusal for Hepatitis B Vaccina-
and HIV to DHCP in healthcare settings. tion” form.
(1) Vaccinations MUST be made available to
Standard Precautions employee free of charge and at reasonable
Standard precautions as discussed by the Centers for time and location; licensed physician or
Disease Control (CDC) in “Guidelines for Infection Con- other healthcare professional MUST super-
trol in Dental Health-Care Settings—2003” integrate and vise administration.
expand the elements of past “universal” precautions into (2) Employees who receive vaccination MUST
a standard of care designed to protect DHCP and patients have appropriate documentation in medical
264   Saunders Review of Dental Hygiene

records; primary immunization with E. Personal protective equipment (PPE) of DHCP:


­hepatitis B vaccine usually consists of DHCP MUST remove PPE before leaving patient-
three IM doses; second and third doses care areas.
should be given 1 and 6 months after first, 1. Gloves:
respectively. a. Single-use examination (nonsterile): ­ rubber
(3) Vaccine is administered unless employee ­latex, vinyl, or nonvinyl materials; should
has proof of HBV antigens or antibodies ­extend over the cuffs of long-sleeved treatment
to HBV antigens; vaccinated individuals gowns. Rubber latex allergy discussed earlier.
should be tested after last injection to verify (1) Single-use sterile: BEST used in conjunc-
positive immune response. tion with surgical procedures.
2. Vaccination recommended for influenza, mea- (2) Single-use plastic overgloves: worn over treat-
sles, mumps, rubella, varicella (chickenpox) (see ment gloves to prevent cross-­contamination
CD-ROM). (e.g., of charts, drawers, writing instru-
3. Note that CDC does NOT recommend routine im- ments).
munization against TB (i.e., inoculation with bac- b. Multiple-use utility: heavy ��������������������
rubber �������������
latex or neo-
ille Calmette-Guérin [BCG] vaccine) or hepatitis prene; BEST used during handling of contami-
A (HAV); NO vaccine exists for HCV. nated instruments, cleaning of operatories, or
C. DHCP handwashing hygiene: handling of chemicals.
1. MUST be washed between patients, before don- c. Glove use protocols:
ning and after removing gloves; using antimicro- (1) MUST be used whenever contact with
bial liquid soap provides additional protection by blood, saliva, mucous membranes, or
leaving protective layer. blood-contaminated objects is a possibility.
a. Short handwashing method: wash hands twice (2) Hands MUST be washed before and after
(rinsing in between) for 15 seconds at begin- wearing gloves; defective gloves MUST
ning of each day; between patients, wash for be changed immediately (accompanied by
15 seconds. appropriate handwashing) and between pa-
b. Surgical hand scrub: before surgical procedures, tients or during long appointments.
scrub hands and forearms for 5 minutes, repeat- (3) MUST not be reused between patients;
edly washing with antimicrobial soap and soft MUST not be washed or disinfected and
brush and rinsing off debris; dry with a sterile then reused.
towel. (4) Prevent herpetic whitlow (digital herpes o14950
c. Whenever gloves are punctured or torn, hands simplex infection) (Figure 8-7).
MUST be washed and covered with new gloves. 2. Masks: glass fiber mat and synthetic fiber mat are
d. If the hands are NOT visibly soiled, alco- MOST effective types.
hol-based hand rub is adequate; rubs are rap- a. Dome masks with elastic bands provide space
idly germicidal when applied to the skin but between the mask and mouth and reduce moist-
should include such antiseptics to achieve ening of the mask (increasing effectiveness);
persistent activity as chlorhexidine, quater- surgical masks (ear-loop or tie-on type) filter
nary ammonium compounds, octenidine, or smaller particles BETTER but become moist
triclosan. quicker, thus compromising clinician protection.
2. Keeping nails short; majority of microflora on the
hands are found under and around fingernails.
a. Nails should be short enough to allow DHCP
to thoroughly clean underneath them and pre-
vent glove tears; sharp nail edges or broken
nails are also MORE likely to increase glove
failure.
b. Long artificial or natural nails can make don-
ning gloves MORE difficult and can cause
gloves to tear MORE readily.
3. Jewelry worn on hands should NOT interfere with
glove use (e.g., impair ability to wear correct-sized
glove or alter glove integrity).
o14940 D. Respiratory hygiene or cough etiquette (additions
added): see CD-ROM for more information. Figure 8-7  Herpetic whitlow.
Microbiology and Immunology   265

b. American Society for Testing and Materi- c. Protective clothing protocols: o9530
als (ASTM) identifies masks according to (1) Wear whenever contamination with blood o9540
three levels of protection (low, medium, or bodily secretions is likely (e.g., from
or high); high-performance masks provide aerosolization of blood, saliva, respiratory
MOST protection for use in procedures where secretions, and microorganisms during ul-
heavy to moderate amounts of fluid, spray, trasonic scaling or polishing of teeth).
and/or aerosols are produced, especially with (2) Change daily or more often if visibly soiled; o9550
high- and low-speed handpieces and ultrasonic MUST avoid wearing outside of healthcare
scalers. facility.
c. Mask use protocols: (a) Limit handling as much as possible af- o9560
(1) Place and adjust before handwashing and ter wearing; place in bags for transpor-
change between patients. tation to laundering facility.
(2) Avoid touching during appointment and (b) MUST be thoroughly laundered be- o9570
continuing to wear after completion of tween wearings separately from other
­procedures for protection against ­particulate clothing and using commercial laundry
matter or microorganisms in ambient air. detergent, highest water temperature
(3) Avoid contamination with disinfectants or (60° to 70° C), machine dried at highest
other chemicals; avoid touching of mask temperature (>110° C).
proper, dangling mask around neck, or re-
moving over the head by elastic or ties. Procedures Related to Standard Precautions
3. Protective eyewear and/or face shield: Certain procedures or protocols MUST be performed
a. Prescription eyewear offers LESS protection on a daily basis to maintain standard precautions in the
because of open top, sides, and bottom. dental setting.
b. Safety glasses and goggles cover all areas • See Chapter 12, Instrumentation: instrument ­processing.
around the eye, offer MOST protection from A. Infection control protocol methods: involve steriliza-
splatter, and are shatter resistant. tion and/or disinfection.
c. Face shields ensure maximum coverage, but 1. Sterilization (destruction of ALL forms of life)
mask MUST also be worn for protection from of reusable dental instruments; recommended
inhalation of contaminants. for ALL dental instruments in critical category
d. Reasons for wearing protective eyewear: (e.g., scalers, burs, bone chisels, scalpels) and in
(1) Prevention of disease transmission (e.g., semicritical category (if heat stable) (discussed
conjunctivitis, ocular herpes, HBV). later).
(2) Protection against penetrating eye injury. 2. Disinfection (destruction of pathogens but NOT of
(3) Protection against chemical splatter (e.g., spores) of instruments unable to withstand heat of
disinfectants, chemicals with a low pH). sterilization; MUST be high-level disinfection for
(4) Patient protection (e.g., against chemical heat-unstable instruments in semicritical category;
splatter, oral debris, dropped instruments). can be used for noncritical surfaces (discussed
e. Eyewear use protocols: later).
(1) Wear eyewear for ALL clinical appoint- a. Disinfection of environmental surfaces in the
ments; clean and disinfect between operatory:
­patients. (1) Disinfect according to categories of critical
(2) AVOID touching protective eyewear dur- surfaces (Table 8-3).
ing care delivery to reduce possibility of (a) Critical surfaces: items that are used to
contamination. penetrate soft tissue or bone (e.g., scal-
4. Protective clothing: pels); MUST be sterilized.
a. Types of protective clothing: reusable or dispos- (b) Semicritical surfaces: touch intact mucous
able gowns or uniforms, surgical gowns (BEST membranes or oral fluids but do NOT
choice), laboratory coats. penetrate tissues (e.g., scalers); require
b. Selection of protective clothing: sterilization or high-level disinfection.
(1) Any protective clothing MUST cover street (c) Noncritical surfaces: contact ONLY the
clothing, fit closely around BOTH neck skin (do NOT contact mucous mem-
and wrists, cover the arms. branes) (e.g., lead [or lead equivalent]
(2) Protective clothing made from synthetic apron); require intermediate disinfection.
material is MORE protective, since it is (2) Involves identifying surfaces that MUST be
MORE fluid resistant. disinfected after treatment (e.g., patient chair,
266   Saunders Review of Dental Hygiene

Table 8-3  Levels of disinfection

Level of disinfection Action Use

High Kills some but not all bacterial spores Critical instruments that have touched mucous
and is tuberculocidal; disinfectant ­membranes, have penetrated soft tissue but not
and sterilizant agents registered by touched bone, or will not withstand sterilization
the Environmental Protection
Agency are high-level disinfectants
Intermediate Kills HBV, HIV, Mycobacterium Noncritical instruments that have contact with
­tuberculosis (var. bovis); does not intact skin.
kill spores
Low Kills most bacteria, some fungi, some General cleaning
viruses; does not kill spores or
M. tuberculosis (var. bovis)

clinician chair, dental light, countertops, tray contamination during patient care, re-
tables, handpiece and air-water syringe sup- covered between patients to break chain
ports, ALL tubing, knobs, drawers used); of infection; plastic coverings should be
some can be isolated using plastic wrap or removed and disposed of properly, while
bags to reduce time and effort needed to pre- DHCP are still gloved.
pare treatment area for next patient. B. Reducing the oral microflora population with prepro-
b. Disinfection technique (Table 8-3): cedural antimicrobial mouthrinse before treatment:
(1) MUST wear multiple-use utility gloves 1. Includes mouthrinses with alcohol, chlorhexidine
(heavy rubber, nitrile, or neoprene) and gluconate, essential oils, or povidone-iodine.
PPE when disinfecting treatment areas. 2. Can reduce level of oral microorganisms in aero-
(2) Uncovered or contaminated surfaces sols and spatter generated during routine dental
SHOULD be precleaned using cleaner procedures with rotary instruments (e.g., dental
(detergent) or cleaner-disinfectant; BEST handpieces, ultrasonic scalers).
to use high- to intermediate-level tubercu- C. Minimizing aerosolization of microorganisms with use o9830
locidal disinfectant registered by the En- of high-volume evacuation (HVE, high-speed suc-
vironmental Protection Agency (EPA) for tion) with evacuation of debris, as well as ultrasonic
BOTH cleaning step and disinfecting step. and polishing use; AVOID use of both controls on air-
(3) Surfaces should be wiped to dislodge ac- water syringe and use of dental dams, if possible.
cumulations; then SHOULD wiped again D. Care and disinfection of dental unit waterlines:
with the disinfectant and remain wet for 1. All retraction valves SHOULD be replaced with anti-
recommended amount of time (≤10 ­minutes retraction valves; should be checked to ensure proper
depending on manufacturer). function; may become stuck (open) over time.
(4) Criteria for agent(s) used for disinfection 2. Waterlines SHOULD be flushed at beginning of
(see later discussion): each day for 3 to 5 minutes to reduce bacterial
(a) MUST be water-based disinfectant and counts (e.g., for handpieces, ultrasonic scalers, air-
EPA-registered hospital disinfectant la- water syringes).
beled viricidal and fungicidal (high to 3. Waterlines SHOULD be flushed for 20 to 30
intermediate level); MUST kill Myco­ seconds between patients to eliminate bacte-
bacterium tuberculosis (TB organism rial accumulation (if unit has been NOT been
very resistant to disinfection because of used for some time, should be flushed for 3 to
tough cell wall). 5 minutes).
(b) MUST be compatible with treatment E. Appropriate disposal of dental sharps and biohazard-
area surfaces and remain active in pres- ous waste generated during care delivery to prevent
ence of organic matter. the transmission of disease (discussed later).
(c) MUST be accepted by ADA Council on
Scientific Affairs as effective disinfec- clinical sTudy  
tant for dental environment. Scenario: A new dental assistant is employed in the
(5) Environmental surfaces that CANNOT be dental office, after graduating from high school last
disinfected should be covered to prevent week at age 18. She has had no education or experience
Microbiology and Immunology   267

in dental ­assisting but has held a dental office reception- and Health ­ Administration (OSHA) have further cat-
ist position for 3 months. Between patients, she care- egorized waste generated in the dental setting into
fully washes her gloves. She wears the same facemask two types, biohazardous waste and biomedical waste.
throughout the day, even though it appears moist and These guidelines are ­ categorized under Occupational
has a small bloodstain on it. At the end of the day, the Exposure to Bloodborne Pathogens and Hazard Con-
dental assistant walks out of the dental office without trol (2001-2003).
changing her clothes. A. Biohazardous waste:
1. Blood, blood-soaked items, blood-caked items,
1. What training should the dental assistant have had be- and items soaked or caked with other poten-
fore participating in patient treatment? tially infectious material, including teeth and
2. What immunization issues must be addressed if the other body tissues NOT required for microscopic
dental assistant is to continue employment at the den- ­examination.
tal office? 2. Sharps: include used and unused syringe needles,
3. Identify mistakes that the dental assistant made on her glass and plastic local anesthetic cartridges, bro-
first day of work. ken instruments, scalpel blades, burs, disposable
4. What implications do these mistakes have regard- syringes, broken glass, and suture needles (see
ing the dental assistant’s health and the health of the later discussion).
­patients? 3. Disposal of biohazardous waste:
a. Involves separating this waste from biomedical
1. The assistant should have had comprehensive training waste.
in the dental office’s infection control policy and proce- b. Involves determining which disposal options
dures, including discussion of standard precautions, be- (incineration, burial, sterilization) are available
fore she began seeing patients. She does not understand and which is the MOST cost effective.
the concepts of cross-contamination, chain of infection, c. Involves labeling infectious waste with bio-
or use of personal protective equipment (PPE). hazard label to alert those who will handle
2. The assistant must review her personal immunization the waste container and inform them of its
record to verify her present status. She must be up to ­contents.
date with immunizations against measles, mumps, ru- d. Involves discarding sharps in a puncture-
bella, tetanus. She must also begin the hepatitis B vac- ­resistant container for disposal.
cination series (or sign a waiver of refusal) within 10 B. Biomedical waste:
days of employment to maintain employment in the 1. Waste generated during course of care delivery
dental office. She can continue employment while she that does NOT qualify as biohazardous.
receives the three-dose regimen. She would also be 2. Includes gloves, masks, patient napkins (bibs),
well advised to receive influenza vaccine to help pre- surface barriers, paper towels that do not fit criteria
vent future sickness and missed workdays. noted above.
3. The assistant was washing and reusing her rubber 3. Disposal of biomedical waste involves same pro-
­latex gloves. She was not changing her mask between cedures as disposal of regular trash, unless state
patients or when it became soiled or damp. She wore and local regulations stipulate different handling
her office uniform home after work. Because she was procedures.
violating these infection control protocols, she may
have been violating others and contaminating the Handling Sharps and Occupational Exposure
­dental office with infectious agents. Incident (Needlestick Protocol)
4. Her health was in danger because of her lack of Every sharp dental instrument should be considered po-
­knowledge of both infection control protocols and tentially infectious and should be handled with safety and
standard precautions. She has been endangering the injury prevention in mind. If occupational exposure oc-
health and well-being of both patients and fellow staff curs, protocol MUST be followed as referenced in 2004
members since her employment began. Her family CDC document “Exposure to Blood: What Health Care
and friends are also at risk for infection from contami- Workers Need to Know.”
nated items and surfaces (including her uniform and • See Chapter 12, Instrumentation: instrument processing.
her skin) that she brought outside of the dental office. A. Sharps:
1. Include used and unused syringe needles, scalpels,
Care and Disposal of Waste burs, broken instruments, suture needles, local an-
Infectious waste, according to the EPA, consists of esthetic cartridges, broken glass, sharp wires.
blood, blood-soaked items, tissue, extracted teeth, 2. Considered biohazardous (infectious) waste and
and sharps. The ADA and the Occupational Safety therefore MUST be disposed of in a ­puncture-proof,
268   Saunders Review of Dental Hygiene

leak-resistant container kept within ­ operatory clinical study  


according to local, state, or federal regulations Scenario: A new dental hygienist has been employed for
(whichever take precedence). 9 months after taking a year off to have a child. She is
3. MUST be handled carefully to avoid occupational nearly an hour behind schedule because of unforeseen
exposure (puncture of skin with contaminated scheduling problems. Her final appointment of the day
sharps). is with the dentist’s sister, who is scheduled to receive
B. Needle recapping: MUST use safe method/device. two quadrants of nonsurgical periodontal therapy. The
1. Occupational exposures often happen during han­ dental hygienist helps the supervising dentist administer
dl­ing of syringe after local anesthesia administra- local anesthesia. While she is recapping the needle, the
tion. cap, which she is holding in her left hand to put over the
o10170 2. AVOID recapping needles using ONLY two- needle, slips and she accidentally sticks herself in her left
handed technique; do NOT bend or break needles index finger. A pool of blood the size of a dime forms
after use on trays or in tops of sharps containers under her glove. She proceeds with completion of the ap-
because of risk of needlestick. pointment and subsequently dismisses the patient.
C. Occupational exposure incident (including needle-
stick protocol): 1. What is the primary cause of this needlestick injury?
1. Consists of eye, mouth, mucous membrane, nonin- 2. How should the dental hygienist have handled the
tact skin, or parenteral contact with blood or other situation immediately after the injury occurred?
potentially infectious materials during the perfor- 3. What is the protocol that must be followed after a
mance of an employee’s duties. needlestick injury?
2. Wounds and skin sites that have been in contact 4. Is any follow-up required after a needlestick injury?
with blood or body fluids should be washed with 5. When do most needlestick injuries occur?
soap and water; mucous membranes should be
flushed with water. 1. This needlestick injury was a direct result of using a
3. Exposure incidents might place DHCP at risk for two-handed recapping technique.
hepatitis B virus (HBV), hepatitis C virus (HCV), 2. After the injury occurred, the hygienist should have
or human immunodeficiency virus (HIV) ­infections, stopped treating the patient, carefully removed and
therefore should be evaluated immediately fol- disposed of her gloves in biomedical waste, washed
lowing treatment of the exposure site by qualified the needlestick area thoroughly with soap and water,
healthcare professional. and applied a bandage to the injury. She subsequently
4. Employers should follow all federal (including should have asked the patient to immediately accom-
OSHA) and state requirements for recording and pany her to a medical setting where both could have
reporting occupational injuries and exposures. blood drawn to test for infectious agents.
Should include the following details: 3. Immediately after an occupational exposure incident
a. Details about the exposure source: whether such as a needlestick, the hygienist as an employee
the patient was infected with hepatitis B vi- must complete an exposure incident form, be evalu-
rus (HBV) and hepatitis B e antigen (HBeAg) ated by a healthcare professional, and possibly have
status, hepatitis C virus (HCV), or human im- herself and the patient tested for HBV, HCV, and HIV.
munodeficiency virus (HIV); if source was Her dentist employer must send her job description,
infected with HIV, stage of disease, history of a report of incident, relevant medical information for
antiretroviral therapy, viral load, if known. both the hygienist and patient, and a copy of OSHA’s
b. Details about exposed person (e.g., hepatitis B bloodborne pathogens standard to the healthcare pro-
vaccination and vaccine response status). vider. She should be counseled on test results, infec-
c. Details about counseling, postexposure man- tious status, and associated risks. She should be given
agement, follow-up. recommendations for postexposure management.
d. If this information is NOT known from medical 4. The hygienist must be provided with a report containing
record, source patient should be asked to obtain her healthcare provider’s written opinion regarding the
serological testing for HBV, HCV, HIV. incident. Depending on the circumstances surrounding
5. After conducting this initial evaluation of the oc- the incident, she may need to given prophylactic drug
cupational exposure, qualified healthcare profes- therapy and be medically reexamined in 6 months.
sional MUST decide whether to conduct further 5. Most needlestick injuries occur during the recapping
follow-up on individual basis using ALL of infor- of used needles. Must recap using a safe method or
mation obtained. device, unlike what the hygienist did this time.
Microbiology and Immunology   269

Review Questions 11 A facultative anaerobe grows in the presence of


A. oxygen.
B. no oxygen.
C. oxygen or no oxygen.
  1 Candida albicans has all the following characteristics
D. low concentrations of oxygen.
­EXCEPT one. Which one is the EXCEPTION?
12 What is the antibiotic of choice in the treatment of a topical
A. Reproduced by budding
yeast or fungal infection?
B. Yeast organism
A. Nystatin
C. Causative agent of thrush
B. Hexachlorophene
D. Inhibited by penicillin
C. Tetracycline
  2 The rigidity and shape of the bacterial cell are caused by the
D. Penicillin
A. capsule.
13 What is the principal agent currently used for gaseous
B. peptidoglycan layer.
­sterilization?
C. lipopolysaccharide layer.
A. Formaldehyde
D. teichoic acid layer.
B. Isopropyl alcohol
  3 Which of the following sequences is correct for staining a
C. Gamma rays
bacterial smear with the Gram stain?
D. Ethylene oxide
A. Crystal violet, alcohol, iodine, safranin
14 Identification of bacterial species depends on all of the fol-
B. Crystal violet, iodine, alcohol, safranin
lowing, EXCEPT one. Which one is the EXCEPTION?
C. Safranin, iodine, alcohol, crystal violet
A. Presence of plasmids
D. Safranin, alcohol, iodine, crystal violet
B. Nutritional requirements
  4 One of following is NOT a characteristic of mycobacteria.
C. Biochemical characteristics
Which one is the EXCEPTION?
D. Staining and morphology
A. Acid fast
15 Transduction as a means of gene transfer between bacteria
B. Lacks cell wall
is mediated by
C. Possesses mycolic acids and waxes in the wall structure
A. bacteriophages.
D. Causes tuberculosis
B. cell-to-cell contact.
  5 The term “sterilization” means the killing of
C. free DNA.
A. spores.
D. yeasts.
B. microbes that cause disease.
16 What are the enzymes needed for the breakdown of hydro-
C. all life.
gen peroxide to H2O and O2?
D. fungus.
A. Autotrophs
  6 Multiple antibiotic resistance may be rapidly spread through
B. Heterotrophs
bacterial populations by
C. Aerobes
A. transformation.
D. Anaerobes
B. transduction.
17 A virus can be distinguished from all other microorganisms
C. high-frequency recombination.
by virtue of its
D. plasmids.
A. ability to be isolated from the blood.
  7 What does penicillin effectively inhibit?
B. need to replicate intracellularly.
A. Cell wall synthesis
C. resistance to antibodies.
B. Cytoplasmic membrane
D. possession of only one type of nucleic acid.
C. Nucleic acid
18 Prokaryotic cells differ in structure from eukaryotic cells in
D. Transcription
that prokaryotes each possess
  8 How is a bactericidal agent recognized?
A. a nuclear membrane.
A. Action is irreversible
B. mitochondria.
B. Does not kill bacteria and only stops bacterial growth
C. 70S ribosomes.
C. Effective only on dead cells
D. 80S ribosomes.
D. Action is reversible
19 One of following statements regarding the control of micro-
  9 The central intermediate that leads to the production of dif-
organisms is INCORRECT. Which one is INCORRECT?
ferent fermentation products by bacteria is
A. Widespread and indiscriminate use of antibiotics may
A. pyruvate.
select for bacteria that are resistant to antibiotics.
B. ATP.
B. An antibiotic that inhibits a large variety of different or-
C. lactate.
ganisms is said to possess a broad spectrum.
D. NADH.
C. Tetracyclines are inhibitors of bacterial cell wall biosyn-
10 One of the following statements is INCORRECT regarding
thesis.
bacterial respiration. Which one is INCORRECT?
D. Viruses are not affected by antibiotics.
A. May occur anaerobically
20 By what means do bacteria usually reproduce?
B. Organic compound serves as the final electron acceptor
A. Endospores
C. May occur aerobically
B. Mitosis
D. Produces more energy in the form of ATP than does
C. Binary fission
­fermentation
D. Budding
270   Saunders Review of Dental Hygiene

21 What is the source of carbon for a heterotrophic bacterium? 31 Antibodies are BEST characterized by which of the following?
A. Glucose A. Lack of antigen specificity
B. Hydrogen gas B. Production of T lymphocytes
C. Carbon dioxide C. Specific interaction with antigenic determinants
D. Hydrogen sulfide D. Absence from blood
22 In which phase of bacterial growth is the number of newly 32 B lymphocytes have which important characteristic?
formed cells equal to the number of dying cells? A. Absence of cell wall cytokine receptors
A. Lag B. Inability to produce antibodies
B. Exponential C. Maturation centered in the thymus
C. Stationary D. Clonal expansion after immunization or infection
D. Death 33 All of the following are TRUE about immunoglobulin G,
23 What do aminoglycoside antibiotics (e.g., streptomycin) EXCEPT one. Which one is the EXCEPTION?
inhibit? A. Predominant antibody class found in blood plasma
A. Transcription B. Mediator of specific antibacterial and antiviral immunity
B. Cell wall biosynthesis C. Produced by plasma cells
C. Translation D. Failure to bind to Fc receptors
D. DNA replication 34 Which of the following statements can be applied to phago-
24 Elaborate and harsh procedures are required for sterilization cytic cells?
because of the possible presence of A. Have no role in the generation of specific immunity
A. viruses. B. Capable of ingesting foreign molecules in the absence
B. endospores. of antibodies
C. capsules. C. Capable of synthesizing antibodies
D. flagella. D. Do not bear surface Fc receptors
25 Which one of the following antibiotics is different from 35 All of these are TRUE about the complement system,
ALL of the others in its mode of action? EXCEPT one. Which one is the EXCEPTION?
A. Erythromycin A. Consists of many blood proteins
B. Tetracycline B. Helps defend the body against infection
C. Cephalosporin C. Facilitates attachment and clearance of foreign mol-
D. Streptomycin ecules by the body
26 Yeasts possess all of the following characteristics, EXCEPT D. Genetic deficiencies do not occur
one. Which one is the EXCEPTION? 36 Allergic reactions are BEST characterized by which of the
A. Reproduce asexually following?
B. Reproduce sexually A. Subclinical symptoms
C. Unicellular organisms B. Susceptibility that excludes genetic inheritance
D. Produce mycelia C. Lack of T-lymphocyte participation
27 Which type of microorganism may be found growing deep D. Mediation by IgE
inside a wound and on its surface? 37 The spectrum of antigens recognized by the immune
A. Aerobe system is
B. Anaerobe A. restricted to immune recognition of membrane lipid
C. Facultative anaerobe molecules.
D. Microaerophile B. highly diverse; caused by genetic mechanisms that reg-
28 Biologically active solutions such as antibiotics and ulate antigen receptor molecules.
vitamins are BEST sterilized by C. very narrow because most B and T cells carry the same
A. autoclaving. antigen receptor molecules.
B. ethylene oxide. D. unaffected by B and T cell antigen receptor molecules.
C. filtration. 38 Immune mechanisms of host defense against infections in-
D. gamma radiation. clude all of the following, EXCEPT one. Which one is the
29 All of the following infective agents would have penicillin EXCEPTION?
as antibiotic of choice for treatment, EXCEPT one. Which A. IgG-mediated neutralization of viruses
one is the EXCEPTION? B. Stimulation of bacterial phagocytosis by antibodies and/
A. Streptococcus pneumoniae or complement (opsonization)
B. Mycoplasma pneumoniae C. IgD-mediated neutralization of bacteria
C. Staphylococcus aureus D. Complement-mediated degradation of bacteria
D. Bacillus subtilis 39 Graft rejection is characterized by
30 Tuberculosis is characterized by all of the following, A. immunological tolerance to the grafted tissue.
EXCEPT one. Which one is the EXCEPTION? B. immune recognition of powerful transplantation anti-
A. Rapidly cured by antibiotic therapy gens on the graft.
B. Emergence of drug-resistant M. tuberculosis C. failure of B and T cells to become activated.
C. Susceptibility is enhanced by HIV infection D. failure of the complement system to become activated.
D. Efficient airborne transmission
Microbiology and Immunology   271

40 Which of the following is CORRECT concerning the 49 Determination of which of the following characterizes spe-
human major histocompatibility complex? cific serological diagnosis of an infection?
A. Known as the human leukocyte antigen complex A. Seroconversion after vaccination
B. Encodes antibody molecules B. Blood B-cell counts
C. Identical among all humans C. Specific blood antibody response to antigens
D. Encodes T-cell receptor molecules D. Elevated total blood IgG levels
41 Genetic typing of a person’s HLA genes 50 For which of the following is the ELISA test used as a
A. has no role in controlling the likelihood of graft re­jection. screening tool?
B. has no role in the diagnosis of certain autoimmune A. HIV RNA
diseases. B. CD4+ T cells
C. can be performed using cellular, serological, or genetic C. Cytotoxic T cells
technology. D. Anti-HIV antibodies
D. cannot yet be performed because HLA alleles are 51 Which of the following is a CORRECT statement concern-
­uncharacterized. ing bone marrow transplantation?
42 Which of the following are known characteristics of A. Replaces body stem cells, leading to regeneration of the
CD4+ T cells? immune system
A. Greater in number in individuals with AIDS B. Effective only for B-cell regeneration; T cells and
B. Known as “helper” T cells phagocytes are not affected
C. Nonresponsive to vaccine injections C. Treatment of choice for anaphylactic reactions
D. Unable to produce interleukin-2 D. Cannot introduce infections into the recipient
43 Cytokines are cells that are 52 Important oral signs of HIV infection include all of the fol-
A. protein molecules produced by lymphocytes, macro- lowing, EXCEPT one. Which is the EXCEPTION?
phages, and other cells. A. Aphthous ulcers
B. normally not involved in regulating immune responses. B. Dental caries
C. not involved in the inflammatory process. C. Necrotizing ulcerative gingivitis
D. produced only during humoral immune responses. D. Oral hairy leukoplakia
44 Cytotoxic T cells are cells that 53 Current therapy for HIV infection is based on all of the fol-
A. recognize and kill virus-infected and tumor cells. lowing, EXCEPT one. Which one is the EXCEPTION?
B. release killer antibody molecules. A. Treatment with viral reverse-transcriptase inhibitors
C. are not antigen specific. B. Management with viral protease inhibitors
D. bear the CD4 cell surface marker. C. Reduction of viral load
45 Immunological protection of a newborn is D. Human bone marrow transplant
A. unnecessary because of vigorous nonspecific and innate 54 Which of the following characterizes the secondary im-
protection. mune response?
B. transferred to the baby only after birth by contact with A. Predominance of IgM
foreign antigens. B. High-affinity IgG
C. passively transferred to the baby in utero by maternal C. Absence of IgA
IgG antibodies. D. Unrelated to booster vaccination
D. unaffected by nursing. 55 Which of the following is a CORRECT statement concern-
46 Systemic lupus erythematosus is characterized by all of the ing the process of antigen presentation to CD4+ T cells?
following, EXCEPT one. Which one is the EXCEPTION? A. Not necessary for helper T-cell activation, because
A. Autoimmunity CD4+ T cells recognize soluble antigen
B. Circulating anti-DNA antibodies of unknown origin B. Occurs only in the thymus
C. Lack of effect by sex hormones C. Necessary for the immune response to T-dependent
D. Immune complex–mediated damage to the kidneys ­antigens
47 All of the following describe immune complexes, EXCEPT D. Does not depend on recognition of MHC molecules
one. Which one is the EXCEPTION? 56 All of the following are B cell–dependent mechanisms of
A. Are found during infections ­cytotoxicity, EXCEPT one. Which one is the EXCEPTION?
B. Are formed in response to virus infections A. IgM complement activation
C. Are associated with complement activation B. ADCC
D. Result in tissue repair when deposited in blood vessels C. CTL activity
48 Hemolytic disease occurs in newborns of Rh-negative ­ D. K cells
mothers and is characterized by all of the following, 57 Monoclonal antibodies may have all of the following prop-
­EXCEPT one. Which one is the EXCEPTION? erties, EXCEPT one. Which one is the EXCEPTION?
A. Father whose erythrocytes bear the Rh+ phenotype A. Products of single clone of B cell o13150
B. Mother whose erythrocytes bear the Rh+ phenotype B. React with a single antigenic determinant o13160
C. Maternal antibodies stimulated by paternal erythrocyte C. Have little antitumor activity
antigens, which are recognized as foreign by the mother D. Specific for one epitope
D. Antibody-mediated attack against red cells made by the
newborn
272   Saunders Review of Dental Hygiene

58 The hemagglutination test of blood erythrocytes from 66 Laundering of contaminated protective clothing requires
the following individuals would be positive in all cases, A. cool water temperatures, a commercial laundry deter-
­EXCEPT one. Which one is the EXCEPTION? gent, and a cool dryer.
A. Blood group A–positive person tested with anti-A antibody B. tepid water temperatures, a commercial laundry deter-
B. Blood group A–negative person tested with anti-A gent, and a warm dryer.
­antibody C. warm water temperatures, a commercial laundry deter-
C. Newborn with hemolytic disease gent, and a warm dryer.
D. Person with autoimmune hemolytic anemia D. hot water temperatures, a commercial laundry detergent,
59 Which of the following is the BEST treatment for a patient and a hot dryer.
with severe combined immunodeficiency? 67 Waterlines in the dental office are BEST disinfected by
A. Transplantation with purified donor T cells flushing for
B. Transplantation with purified donor stem cells A. 1 to 3 minutes at the beginning of the day and for 10 to
C. Passive immunization with pooled human gamma globulins 15 seconds between patients.
D. Periodic injections of interleukin-2 B. 3 to 5 minutes at the beginning of the day and for 20
60 Secretory component is a substance that is to 30 seconds between patients.
A. produced by T cells; stabilizes the T-cell receptor. C. 5 to 10 minutes at the beginning of the day and for 35 to
B. required for IgE binding to mast cells. 45 seconds between patients.
C. required for cross-linking of IgM monomers. D. 10 to 15 minutes at the beginning of the day and for 50
D. produced by epithelial cells; protects IgA. to 60 seconds between patients.
61 The serum protein electrophoresis pattern of a patient with 68 Biomedical waste includes all of the following, EXCEPT
a history of recent infections shows abnormally elevated one. Which one is the EXCEPTION?
gamma globulins. Subsequent serum immunoelectrophore- A. Extracted teeth
sis shows an elevation of the IgM band. The MOST likely B. Gloves
diagnosis of this patient is C. Patient napkins
A. hypogammaglobulinemia. D. Facial masks
B. DiGeorge syndrome. 69 Biohazardous waste is
C. Waldenström’s macroglobulinemia. A. disposed of with regular trash.
D. T-cell lymphoma. B. incinerated, buried, or sterilized.
62 What condition is associated with multi-drug-resistant C. placed in plain packaging.
­tuberculosis? D. disposed of with biomedical waste.
A. Systemic lupus erythematosus 70 All of the following are common oral manifestations of
B. Acquired immunodeficiency disease syndrome AIDS, EXCEPT one. Which one is the EXCEPTION?
C. Trisomy 21 A. Hairy leukoplakia
D. Bullous pemphigoid B. Candidiasis
63 All of the following routes are involved in HIV transmis- C. Kaposi’s sarcoma
sion, EXCEPT one. Which one is the EXCEPTION? D. Angular cheilitis
A. Exposure to contaminated blood, blood products, or E. Lichen planus
blood-contaminated body fluids 71 The major barrier to care for the patient with HIV/AIDS is
B. Infected mother to unborn child A. communication.
C. Intimate sexual contact involving the exchange of ­semen B. transportation.
or vaginal secretions C. economic.
D. Casual contact with a recently seroconverted HIV- D. physical.
­positive individual 72 One of the following does NOT meet the criteria for a diag-
64 An important deterrent against the transmission of disease nosis of acquired immune deficiency syndrome. Which one
in the dental office is is the EXCEPTION?
A. requiring that patients undergo HIV testing. A. Kaposi’s sarcoma
B. refusing to treat patients from high-risk groups. B. Histoplasmosis
C. vaccinating office staff who are exposed to bloodborne C. Herpes simplex
pathogens for HBV. D. Mouth rinsing using both controls on an air-water syringe
D. asking whether patients have AIDS during the medical E. Lymphoma of the brain
history. 73 Which of the following lesions is produced by herpes zoster
65 All of the following are methods of reducing airborne trans- and eventually ulcerates and causes intense pain?
mission of infectious agents, EXCEPT one. Which one is A. Vesicle
the EXCEPTION? B. Pustule
A. Having the patient use an antimicrobial mouthrinse C. Bulla
B. Using high-volume suction during ultrasonic scaling D. Cyst
procedures
C. Wearing protective clothing, gloves, and mask
o13580 D. Mouth rinsing using both controls on an air-water
­syringe
Microbiology and Immunology   273

74 All of the following are statements regarding acquired im- a recipient by conjugation. Thus a bacterium sensi-
mune deficiency syndrome, EXCEPT one. Which one is the tive to antibiotics may receive plasmid from donor
EXCEPTION? cell and become immediately resistant to antibiot-
A. Fatal syndrome that develops after infection by virus. ics. Transformation, transduction, high-frequency
B. Best classified as an autoimmune disease. recombination are mechanisms for transfer of chro-
C. Contracted by contact with contaminated blood.
mosomal genes.
D. Late-stage symptoms include increased opportunistic
infections.
7 (A)  Penicillin (Pen-Vee K) specifically inhibits bio-
75 In treatment of a patient with AIDS who has pseudomem- synthesis of bacterial cell wall peptidoglycan. Inhib-
branous candidiasis, which of the following drugs would be its cross-linking of peptidoglycan chains, disrupting
MOST likely be prescribed? integrity of three-dimensional structure; effective
A. Acyclovir ONLY on actively growing cells.
B. Corticosteroid 8 (A)  Bactericidal agent is one that binds irreversibly
C. Ketoconazole with microorganism, resulting in death of bacterium.
D. Chlorhexidine Bacteriostatic agent binds reversibly to bacterium,
stopping growth, but does NOT kill organism.
9 (A)  Pyruvate is end product of glycolytic pathway in
Answer Key and Rationales which sugar such as glucose (C6) is broken down into
two molecules of pyruvate (C3), resulting in net pro-
duction of two ATP molecules and two reduced NADH2
1 (D)  Penicillin (Pen-Vee K) is NOT effective in treat- molecules. During simplest fermentation, pyruvate is
ment of Candida infections because organism is a the organic compound that serves as final electron ac-
yeast, eukaryotic cell. Thrush is a Candida infection ceptor and is thus reduced to fermentation product, lac-
common in infants. Penicillin inhibits peptidoglycan tate, central “hub” in fermentation reactions.
biosynthesis in bacterial cell walls, and peptidoglycan 10 (B)  By definition, inorganic compound serves as fi-
is found ONLY in prokaryotic cells. nal electron acceptor in respiration, whereas organic
2 (B)  Peptidoglycan is three-dimensional macromolecu- compound serves this function in fermentation. Bac-
lar structure of the cell wall that surrounds cytoplasmic terial respiration may occur aerobically, employing
membrane and provides shape and rigidity to bacterial oxygen, or may occur anaerobically, using other in-
cells. Because of high osmotic pressure within the cell, organic compounds (e.g., sulfate, nitrate); produces
bacteria normally would lyse without this structure. MORE energy from one glucose molecule in the form
3 (B)  Correct order of reagents used in Gram stain is of ATP (38 net) than does fermentation (2 net).
crystal violet (primary stain), iodine (mordant), al- 11 (C)  Facultative anaerobe is bacterium that may grow
cohol (which decolorizes gram-negative cells), and either aerobically, in presence of oxygen, or anaero-
safranin (counterstain). Gram-positive bacteria retain bically, in absence of oxygen. Aerobe grows ONLY
primary stain and appear purple-blue. Gram-negative in presence of oxygen, and anaerobe grows ONLY in
bacteria are decolorized and take up counterstain; absence of oxygen; microaerophile requires limited
­appear red-pink. amounts of oxygen for growth.
4 (B)  Mycobacteria possess a peptidoglycan cell wall 12 (A)  Nystatin (Mycostatin) and amphotericin B (Fun-
covered with mycolic acids and waxes. Hydrophobic gilin) are polyene antibiotics (antifungals) that com-
surface is responsible for positive acid-fast staining plex with sterol, thereby disrupting membranes of
reaction characteristic of causative agent of tubercu- eukaryotic cells such as yeast. Bacterial cells are NOT
losis, Mycobacterium tuberculosis. Mycoplasmas are affected by these antibiotics because sterol is absent.
organisms that LACK cell wall and are smallest free- Tetracyclines such as tetracycline (Achromycin, Su-
living bacteria. mycin), doxycycline (Vibramycin), and minocycline
5 (C)  Sterilization is killing of all life, including bac- (Minocin) and penicillin (Pen-Vee K) act ONLY on
terial endospores. Major sterilization agents include prokaryotic cells. Hexachlorophene (Nabac, Phiso-
high temperatures (use of an autoclave or dry air hex, Septisol) is NOT an antibiotic but a disinfectant.
oven), gamma radiation, ethylene oxide. Disinfec- 13 (D)  Ethylene oxide is BEST used for room-
tants are usually harsh agents, including chemical ­temperature gaseous sterilant for materials that may
agents and ultraviolet (UV) light, that kill MOST but be destroyed by other means. Used as a sterilant for
NOT all organisms. foods and medical supplies. Colorless flammable gas
6 (D)  Plasmids are small, circular, self-replicating or refrigerated liquid with a faintly sweet odor.
DNA molecules that may contain multiple genes 14 (A)  Plasmids are NOT present in all microorgan-
that code for resistance to different antibiotics. Plas- isms and NOT routinely useful for purposes of iden-
mids may be transmitted from a donor bacterium to tification. Identification depends on Gram stain and
274   Saunders Review of Dental Hygiene

­morphology (e.g., rods, cocci), biochemical character- to the 30S ribosomal subunit. Antibiotics such
istics (such as ability to grow on different substrates), as ­ rifampicin inhibit protein synthesis at level of
metabolic end products, requirement for oxygen, spe- ­transcription. Nalidixic acid is inhibitor of DNA syn-
cific nutritional requirements. thesis, and beta-lactam antibiotics (e.g., penicillin,
15 (A)  Transduction occurs upon replication within host Pen-Vee K) inhibit cell wall biosynthesis.
bacterium when piece of host cell chromosomal DNA 24 (B)  Bacterial endospores are MOST resistant form of
is mistakenly incorporated into phage capsid instead of life known. They resist heat, ultraviolet light, desic-
phage DNA. Upon infection of new host cell, this piece cation. Viruses are killed by much milder conditions;
of DNA is injected and thus transferred to the new capsules and flagella are bacterial structures that are
bacterial cell. Conjugation is process of gene transfer easily destroyed by physical and chemical procedures.
mediated by cell-to-cell contact, and transformation is 25 (C)  Cephalosporins such as cephalexin (Keflex),
incorporation of free “naked” DNA into recipient cell. cefadroxil (Duricef), cefaclor (Ceclor), cefuroxime
16 (C)  For bacteria to grow aerobically, using oxygen as (Ceftin), cefazolin (Ancef), ceftriaxone (Rocephin)
final electron acceptor (respiration), enzymes catalase are beta-lactam antibiotics that inhibit cross-linking
and superoxide dismutase are required to break down in peptidoglycan biosynthesis. Erythromycins such
toxic intermediates that form at end of electron trans- as Robimycin, E-Mycin, E.E.S.; tetracyclines such
port chain. Superoxide dismutase converts superoxide as tetracycline (Achromycin, Sumycin), doxycycline
anions (toxic) to hydrogen peroxide (toxic), and the per- (Vibramycin), and minocycline (subgingival: Arestin,
oxide is converted to water and oxygen. Without these oral: Minocin); and streptomycin are ALL inhibitors
enzymes, bacteria grow anaerobically. Autotrophs are of bacterial protein synthesis.
bacteria that use carbon dioxide as carbon source; het- 26 (D)  Yeasts do NOT produce mycelia; they are unicel-
erotrophs derive carbon from organic compounds. lular fungi. Yeasts may reproduce asexually, by bud-
17 (D)  Distinguishing feature of viruses is that they con- ding, or sexually, by the production of ascospores.
tain either DNA or RNA but NOT both as do cellular 27 (C)  Facultative anaerobes may grow on surface of
lifeforms. Able to replicate themselves from informa- wound, in presence of air, or deep in a wound, where
tion contained in DNA or information in RNA. anaerobic conditions exist. Anaerobes grow only deep in
18 (C)  Prokaryotes (bacteria) possess 70S ribosomes. wound, and aerobes grow only on surface. Microaero-
Others are characteristic of MORE complex eukary- philes might be expected to grow just below surface.
otic cells. MANY useful antibiotics specifically inhibit 28 (C)  Sterilization by filtration is necessary for biologi-
bacterial protein synthesis because they specifically cally active solutions. Alternative methods such as au-
bind to 70S ribosomes, but NOT to 80S ribosomes of toclaving, ethylene oxide, or gamma radiation would
eukaryotic cells. be TOO severe and would destroy biological activity.
19 (C)  Tetracyclines such as tetracycline (Achromycin, 29 (B)  Penicillin (Pen-Vee K) inhibits cell wall biosyn-
Sumycin), doxycycline (Vibramycin), minocycline thesis. Mycoplasmas are bacteria that LACK a cell
(subgingival: Arestin, oral: Minocin) inhibit bacterial wall and are therefore NOT affected by any of the cell
protein synthesis. Penicillins (Pen-Vee K) and other wall antibiotics. Others possess typical cell walls and
beta-lactam antibiotics inhibit cell wall biosynthesis. are inhibited by penicillin.
Others are all CORRECT and should be remembered 30 (A)  Drug resistance is significant problem in treat-
when administering antibiotics. ing tuberculosis (TB), as is drug compliance, because
20 (C)  Bacteria reproduce by binary fission. Eukaryotic patient may feel better soon after initiating therapy;
cells reproduce by mitosis, and yeasts do so by bud- however, eradication of M. tuberculosis may take
ding. Endospores are NOT a mode of reproduction in MANY months of treatment. TB is a growing prob-
bacteria but are a means of survival. lem in immunocompromised, including those with
21 (A)  Organic compounds serve as the carbon source for human immunodeficiency virus (HIV) infection. In-
heterotrophic bacteria. Carbon dioxide is the carbon fected may cough up large numbers of organisms, and
source for autotrophic bacteria. Although hydrogen and these may survive for long periods of time because
hydrogen sulfide may serve as energy source for chemo- of outer waxy coat of this bacterium, which makes
lithotrophic bacteria, they do NOT contain carbon. airborne transmission relatively efficient.
22 (C)  Dividing and dying bacteria are in balance dur- 31 (C)  KEY property of antibodies (produced ONLY by
ing stationary phase of growth. NO growth during the plasma cells after these cells are produced by B cells)
lag phase. Rapid growth and division of the cell occur is specific reactivity with antigens, which allows tar-
during exponential phase, and cells die exponentially geting of specific pathogens while avoiding autoim-
during death phase. mune responses. Antibodies are soluble proteins that
23 (C)  Aminoglycoside antibiotics specifically inhibit circulate in blood, facilitating contact with pathogens
protein synthesis at level of translation by binding that invade the body.
Microbiology and Immunology   275

32 (D)  Production of antibodies by B lymphocytes is is characterized by active immunity against anti-


greatly influenced by cytokines (e.g., IL-2, produced gens carried by the graft; MOST potent appear to be
by T cells). B lymphocytes are generated by bone ­transplantation (MHC) antigens, which activate T and
marrow and mature there and in secondary sites such B cells to react against the graft. Complement may
as lymph nodes. Contact with antigens, either natu- play IMPORTANT role in graft rejection, either
rally or by immunization, selects specific clones of through direct activation by graft cells or through
B lymphocytes for expansion. ­antibodies that react against graft.
33 (D)  Plasma cells represent terminal stage of B-cell 40 (A)  MHC is generic term for gene complex, also
differentiation; secrete antibodies that find way into known as human leukocyte antigen complex (HLA).
blood (IgG has highest concentration in blood) and Carries alleles (specific DNA coding sequences),
other body sites (e.g., mucosal sites where antibody is which are highly diverse. Antibodies and T-cell recep-
MAINLY IgA, secretory); may mediate immune reac- tors are encoded on separate and distinct gene regions
tions against bacteria, viruses, and other pathogens. outside HLA complex.
Fc receptor binding is MAJOR mechanism that helps 41 (C)  HLA typing is IMPORTANT method used to
phagocytic cells clear pathogens. match graft donors and recipients and to help diagnose
34 (B)  Phagocytic cells (e.g., macrophages, neutrophils) autoimmune diseases (and some cancers) that have
help in generation of specific (B and/or T cell) immu- been linked to certain MHC alleles. Classical cellular
nity by processing antigen molecules, presenting antigen and serological typing techniques are being replaced
fragments to B and T cells, and releasing cytokines. How- with the molecular detection of HLA gene sequences.
ever, phagocytic cells do NOT synthesize antibodies, al- 42 (B)  CD4 denotes the “helper” T-cell phenotype.
though they are rich in surface Fc receptors, which helps CD4+ T cells are destroyed in AIDS (MOST likely
interaction with antibodies and immune complexes. because HIV replicates within these cells). Because
35 (D)  Complement system is complex biochemical CD4+ T cells are IMPORTANT for immune responses
system that comprises MORE than 30 protein mole- and in the production of cytokines such as IL-2, their
cules. When activated by contact with antigens and/or destruction leads to susceptibility to infections.
antibodies, complement helps stimulate inflammation 43 (A)  Many cell types, including B and T lymphocytes
and magnify immune reactions; also attaches to and and macrophages, produce cytokines. Cytokines are
helps clear foreign molecules and pathogens from the IMPORTANT regulators of immunity and participate
body. Various complement proteins may be adversely in ALL phases of immune responses.
affected by genetic mutations. 44 (A) Cytotoxic (CD8+) T cells are capable of killing
36 (D)  Clinical spectrum of allergy is broad and includes other cells by a specific protein-killing mechanism
potentially fatal allergic reactions (e.g., anaphylaxis unrelated to antibodies, which are NOT produced by
caused by food or drug allergy). Genetic factors that T cells. Antigen-specific T-cell killing allows these cells
affect susceptibility to allergy include HLA genes. to be directed against virus-infected or tumor cells.
Although T cells may participate in some types of al- 45 (C)  Immune system of a newborn is NOT fully ma-
lergies (e.g., delayed-type hypersensitivity), some of ture. Without passive maternal protection, acquired
the MOST severe allergies are caused by antibodies in utero (maternal IgG crosses placenta) and during
(immunoglobulin [Ig]). nursing (from breast milk antibodies and cells), new-
37 (B)  Diversity is hallmark of immunity, being gener- born would NOT be able to cope effectively with ex-
ated by recombination at V(D)J genes that encode for posures to infection. By ∼6 months of age, immune
antibodies and T cell receptors. Results in the genera- system has reached stage in which it is likely to re-
tion of antibodies that specifically recognize antigens spond well to contact with MOST foreign antigens.
of virtually ANY molecular structure. 46 (C)  Anti-DNA antibodies are found in the blood of
38 (C)  Immune mechanisms of host defense are mul- persons with systemic lupus erythematosus (SLE),
tifactorial. Include antibody reactions (e.g., direct MOSTLY female; thus sex hormones exert influence
neutralization and complement activation) and op- on development of SLE, although it is NOT known
sonization (facilitation of clearance of bacteria and why anti-DNA antibodies develop. Clinical spectrum
other foreign invaders by phagocytic cells). IgG is of this autoimmune disease is broad. In MORE se-
major mediator of virus neutralization. Although bac- vere cases, kidney failure may occur from BOTH the
teria may be inhibited by IgG, IgA, or IgA antibodies, deposition of immune complexes in kidney and the
binding to critical receptors, IgD has NO known di- stimulation of inflammatory responses.
rect antimicrobial function. IgD is involved in devel- 47 (D)  As in the example of systemic lupus erythemato-
opmental regulation of immune responses. sus (SLE), immune complexes may precipitate tissue
39 (B)  Tolerance is LACK of immune reactivity to spe- damage, and the activation of complement is a major
cific antigens (e.g., self-antigens). Graft ­ rejection pathway for this occurrence. Nonpathogenic immune
276   Saunders Review of Dental Hygiene

complexes circulate at low levels (e.g., as result of the tongue involving filiform lingual papillae) caused
immune responses to microorganisms and their anti- by Epstein-Barr virus (EBV), treatable (if desired, not
gens) and are cleared in healthy individuals without needed) by acyclovir.
causing dangerous immune reactions. 53 (D)  Bone marrow transplant from a human donor
48 (B)  When an Rh-negative mother is exposed to pater- has NOT been developed as a human deficiency virus
nal Rh+ antigen during pregnancy (usually because of (HIV) treatment because the donor cells might be in-
expression of RhD antigen by erythrocytes of the de- fected with HIV. Central goal of current HIV therapy
veloping fetus, which leak into the maternal circulation is to reduce the viral load present in the body (mea-
before or during birth), the mother may become sensi- sured by the HIV RNA copy number in the blood) by
tized to the RhD antigen. During subsequent pregnan- using highly active antiretroviral therapy (HAART);
cies, maternal IgG anti-RhD antibodies may enter the this includes reverse transcriptase inhibitors, such as
fetus and damage fetal red blood cells, resulting in he- AZT, D4T, ddI, or ddC, and one or more viral prote-
molytic disease of the newborn. Prophylactic treatment ase inhibitors and extends length of disease-free pe-
consists of injecting mother with anti-RhD antibodies riod of infection by suppressing viral replication and
at approximately time of birth; passively administered encouraging blood CD4+ count increases.
antibodies bind to paternal antigen and inhibit devel- 54 (B)  IgG is present MAINLY during the secondary
opment of maternal anti-RhD antibodies. Reactions to immune response. IgM is present MAINLY during
blood group antigens other than RhD may also occur. primary immune response (first Ig produced), but
49 (C)  Determination of specific antibody responses to the affinity of ALL antibody isotypes, including IgA,
pathogens (e.g., anti-HIV detection by enzyme-linked increases during secondary immune response, which
immunosorbent assay [ELISA]) is powerful tool for de- for some vaccinations requires a “booster shot.”
tecting human exposure and infection. However, specific 55 (C)  Antigen presentation to CD4+ T cells occurs at
antibody response that occurs as result of vaccination many lymphoid sites. Necessary for the generation of
would NOT necessarily indicate infection because im- immunity to T-dependent antigens and the activation
munization itself could be responsible for seroconversion. of T-helper activity because CD4+ T cells CANNOT
Blood B-cell counts and IgG levels are nonspecific mea- recognize soluble antigen. CD4+ T cells recognize
sures of immunity and do NOT reveal specific exposures. antigen in combination with MHC molecules that are
50 (D)  Enzyme-linked immunosorbent assay (ELISA) present on the antigen-presenting cell surface.
screening for HIV infection detects specific immunity 56 (C)  Cytotoxic T-lymphocyte (CTL) activity is indepen-
that occurs during HIV infection, as represented by dent of antibody, but IgM activation of complement, an-
circulating IgG or IgM anti-HIV antibodies. False- tibody-dependent cell-mediated cytotoxicity (ADCC),
positive ELISA reactions may occur because of blood and K cells all require antibody for role in cytotoxicity.
antibodies that cross-react with HIV antigens, even 57 (A)  Monoclonal antibodies are obtained from cultures o14670
in the absence of HIV infection. Blood HIV RNA de- of cloned cells; products of a single clone of B cells that
tection by molecular techniques and measurement of react with a single antigenic determinant or epitope. Epi­
CD4+ T cells in the blood by cytochemistry BOTH tope, also known as antigenic determinant, is the part of a
provide important correlates of disease progression macromolecule that is recognized by the immune system,
but are NOT carried out by ELISA. specifically by antibodies and B cells. Epitopes are useful
51 (A)  By providing a new source of functional stem as an antitumor treatment in some cases of leukemia.
cells, bone marrow transplantation may effectively 58 (B)  For the cases listed in A, C, and D, erythrocytes
treat a variety of immune system disorders, includ- will carry either antigen (the A antigen for A) or anti-
ing those that affect B and T cells and phagocytes. bodies (maternal anti-RhD for C; autoantibody for D),
Transmission of infection—particularly but NOT ex- which results in positive hemagglutination test.
clusively virus infection—from the donor to the re- 59 (B)  Transplantation with stem cells (or stem cell–
cipient is one of the risks of this procedure. ­containing tissue) provides the ONLY mechanism for
52 (B)  Dental caries is NOT indicative of human defi- reconstituting the immune system with severe combined
ciency virus (HIV) infection, although it may occur immunodeficiency (SCID). Administration of T cells
when oral hygiene is inadequate or may be caused ONLY puts patient at risk for graft-versus-host disease.
by xerostomia (dry mouth) from drug use. Aphthous Passive immunizations or cytokine injections have too
ulcers (canker sores) are common in HIV infection. limited an effect to provide significant benefit with SCID
LESS common but still prevalent in HIV infection are because all functional B and T cells are absent.
necrotizing ulcerative gingivitis (NUG), necrotizing 60 (D)  Secretory component is produced by epithelial
periodontal disease with anaerobic infection, treatable cells located near mucosal sites, becomes attached to
with systemic metronidazole (Flagyl), and oral hairy IgA, and provides protection from degradation at mu-
leukoplakia (white lesions on the lateral surfaces of cous membranes (such as oral mucosa in oral cavity).
Microbiology and Immunology   277

61 (C)  Waldenström’s macroglobulinemia is B-cell tu- 67 (B)  Recommended flushing times are 3 to 5 minutes
mor characterized by overproduction of IgM, which at beginning of the day (or if the unit has NOT been
in this case was observed on protein electrophoresis used for a few hours) and 20 to 30 seconds between
and isoelectric focusing. Hypogammaglobulinemia is patients. Longer rinse times are NOT harmful but are
B-cell deficiency, and DiGeorge syndrome is absence NOT an efficient use of time.
of the thymus, neither of which would result in el- 68 (A)  Extracted teeth, blood-soaked sponges and
evated serum IgM. gauze, sharps, and tissue removed from patients are
62 (B)  Multi-drug-resistant tuberculosis is ALWAYS considered biohazardous wastes. ALL others fall into
associated with AIDS and is resistant to rifampin category of biomedical waste.
(Rifadin, Rimactane) and isoniazid (INH, Nydrazid). 69 (B)  Biohazardous waste is treated differently from
Trisomy 21 (Down syndrome) is a genetic disorder biomedical waste and MUST be incinerated, buried,
with extra chromosome 21. Affected persons have or sterilized. Biomedical waste can be disposed of
NO epicanthic eye fold, shorter stature, heart abnor- with regular trash.
malities, intellectual disability (mental retardation), 70 (E)  Hairy leukoplakia, candidiasis, Kaposi’s sarcoma,
and greater tendency to develop periodontal disease. and angular cheilitis are common oral manifestations
Systemic lupus erythematosus (SLE) is chronic au- of AIDS. Lichen planus is chronic inflammatory dis-
toimmune disease of unknown cause. Skin lesions ease of unknown cause that is often associated with
are MOST common sign of SLE (butterfly rash over systemic diseases such as diabetes mellitus and hyper-
nose, erythematous lesions on fingertips), and oral le- tension but NOT commonly associated with AIDS.
sions include bleeding gingiva and reddened lesions 71 (A)  Communication is MAJOR barrier to care for
with white striae radiating from center. patient with HIV/AIDS. Fear of healthcare providers’
63 (D)  HIV is commonly transmitted through contact with attitudes may prevent patient from fully disclosing
blood, from infected mother to unborn child or fetus (in health status. Transportation, economic, physical bar-
utero), and through secretions of intimate sexual contact. riers may arise as patient becomes MORE debilitated
HIV is transmitted ONLY through the exchange of body with disease.
fluids (e.g., saliva, semen, vaginal secretions, blood). 72 (C)  Herpes simplex infection would NOT be con-
Casual contact with HIV-infected individual (carrying sidered an indicator of acquired immune deficiency
on conversation, standing nearby, riding in elevator) is syndrome (AIDS) unless herpes simplex infection
NOT a risk factor for transmission of HIV virus. persisted longer than 1 month. Others are IMPOR-
64 (C)  Staff immunizations against hepatitis B (HBV) TANT in AIDS diagnosis.
are effective way to deter the transmission of com- 73 (A)  Herpes zoster produces vesicle (blister) as a
municable disease in the dental office. Human immu- virus-infected, circumscribed, fluid-filled elevation
nodeficiency (HIV) testing provides professional with in epithelium ≤1 cm in diameter. Acne can produce
ONLY momentary view of HIV status because detec- pustule, circumscribed elevation filled with purulent
tion of seroconversion takes several weeks to months exudate resulting from infection. Pemphigus vulgaris
and because test does NOT detect any other disease. produces bulla (plural, bullae); diameter of large
Testing has NO impact on spread of communicable vesicle is ≥1 cm. TRUE cysts result from entrapment
disease in the dental office. Discriminating against of epithelium or remnants of epithelium that grow to
patients from perceived “high-risk” groups is illegal produce a cavity, such as that of nasopalatine cyst;
and unethical. One CANNOT tell whether a person is MUST be lined by epithelium.
infected with HIV by appearance. Although knowing 74 (B)  Acquired immune deficiency syndrome (AIDS)
whether patient has acquired immunodeficiency syn- is NOT considered autoimmune disease because im-
drome (AIDS) may be important when considering mune system is NOT attacking own tissues. Rather,
treatment choices and regimens, knowing HIV status AIDS is syndrome that develops as result of infection
has NO effect on transmission of communicable dis- by human immunodeficiency virus (HIV).
ease in the dental office. Standard precautions protect 75 (C)  Recommended treatment for pseudomembra-
BOTH patients and healthcare providers from infec- nous candidiasis consists of either topical or systemic
tious diseases transmissible in the dental office. ingestion of an antifungal agent, such as nystatin
65 (D)  Mouth rinsing using BOTH controls on an air- (Mycostatin), ketoconazole (Nizoral), or fluconazole
water syringe increases aerosolization of infectious (Diflucan). Acyclovir (Zovirax) is an antiviral re-
agents in the treatment area. Others help to reduce served for viral infections, and Lidex (fluocinonide), a
airborne transmission of infectious agents. corticosteroid, is often used to soothe the pain of oral
66 (D)  Contaminated protective clothing should be ulcerations. Chlorhexidine (Periogard) is frequently
washed with hottest water temperatures available and prescribed in HIV/AIDS to assist in controlling the
dried at highest temperature the garment can withstand. intraoral bacterial load.
CHAPTER  9

Pharmacology

PHARMACOLOGICAL PRINCIPLES 3. Metabolism: irreversible transformation of sub-


AND CONCEPTS   stance and its daughter metabolites.
Understanding a pharmacological effect on a patient and 4. Excretion: elimination of substances from the
possible interaction with other drugs is essential to safe body.
dental care. Thorough medical history helps identify ad- D. Indications (therapeutic uses):
verse effects and potential drug interactions with current 1. Valid reasons to use drug (test, drug, procedure, or
prescription (Rx) drugs, dental drugs, over-the-counter surgery).
(OTC) drugs. Dental settings are advised to have the cur- 2. Strictly regulated by Food and Drug Administra-
rent Physician’s Desk Reference (PDR) Index for refer- tion (FDA).
ence information concerning drug information by brand E. Reactions to drugs:
and generic names, product categories, manufacturers. 1. Potency: strength of a drug in terms of ability to
• See CD-ROM for Chapter Terms and WebLinks. achieve particular result.
• See Chapters 6, General and Oral Pathology: disease 2. Produces BOTH desired (therapeutic effect) and
states; 14, Pain Management: physical classification adverse effects (reaction, side effect) on the body.
system; 8, Microbiology and Immunology: antibiotics; a. Desired effect(s):
10, Medical and Dental Emergencies: allergy-related (1) Pharmacological effect(s): act on target
emergencies in a dental setting; 11, Clinical Treatment: organs to achieve therapeutic response.
contraindications, xerostomia. (2) Therapeutic effect(s): dose related, pre-
A. Dosage: dictable.
1. Effective dose: produces 50% of maximum response b. Adverse effect(s) (reaction[s]):
or specific response in 50% of subjects (ED50). (1) Side effect(s) or unintended consequence(s)
2. Lethal dose: kills 50% of subjects (LD50); certainly specifically arising from drug therapy; un-
lethal dose (CLD): amount of drug likely to cause predictable and dose related (e.g., xerosto-
death. mia [dry mouth], gingival hyperplasia).
3. Therapeutic index (TI): measurement of safety (2) Act on nontarget organs (e.g., vomiting
(= LD50/ED50); larger the number, safer the drug. with antibiotic erythromycin [Robimycin,
4. Maximum recommended dose (MRD): estab- E-Mycin, E.E.S.] and topical fluoride prod-
lished by manufacturer (in milligrams per pound). ucts).
B. Administration: latest is time-released drug with in- c. Interactions:
gredients released at predictable rate: (1) Synergism: combination action of two
1. Topical: local effect applied directly where ac- drugs gives a greater total effect than sum
tion is desired; includes eye drops and ear drops, of individual drug’s effects (e.g., codeine
­epicutaneous, enema, vaginal. with acetaminophen for pain relief, alcohol
2. Enteral: desired effect is systemic (nonlocal), and diazepam to cause respiratory depres-
given by way of the digestive tract; includes oral, sion).
by tube, sublingual, rectal administration. (2) Agonist: drug that causes an effect by in-
3. Parenteral: desired effect is systemic, given by teracting with a receptor.
routes other than digestive tract; includes transder- (3) Antagonist: drug that counteracts another
mal, intradermal, intravenous (IV), subcutaneous drug’s effect on a receptor (e.g., at neuro-
(SC), intramuscular (IM), intrathecal, intraperito- muscular junctions).
neal, inhalation. d. Tolerance: Decreased susceptibility to effects
C. Pharmacokinetics (“ADME” scheme): of a drug because of repeated use (e.g., pain-
1. Absorption: process of a substance entering the body. killers, intoxicants, or antibiotics); when oc-
2. Distribution: dispersion or dissemination of sub- curs, MORE drug is needed to produce same
stances throughout fluids and tissues of the body. pharmacological effects.

278
Pharmacology    279

3. Toxicology: 2. Extrarenal excretion routes include lungs, gastro-


a. Unpredictable and dose related; acts on target intestinal (alimentary) tract (GIT), saliva, breast
organs and involves extended pharmacological milk, sweat.
effect(s) (e.g., hepatotoxicity with antineoplastic 3. Lactating: many drugs are excreted into breast milk,
methotrexate [Rheumatrex Dose Pack, Trexall]). such as tetracyclines, and can affect newborn, medical
b. Drug allergy reaction(s): immunological response; consult needed; safest drugs SHOULD be considered.
NOT predictable or dose related; four types:
(1) Type I (immediate hypersensitivity): occurs Prescription Writing
immediately (within minutes) after exposure Effective prescription writing requires accurate diagnosis;
to previously encountered antigen (e.g., al- proper selection of drug, dosage form and route of admin-
lergy to penicillins [Pen-Vee K), preserva- istration; proper size and timing of dose; precise dispens-
tive [sodium metabisulfite] for epinephrine ing; accurate labeling; and correct packaging. Prescription
in local anesthetics); can be life threaten- is provided by a physician, osteopath, dentist, or other ap-
ing if produces anaphylaxis, must activate proved practitioner. Note that OTC drugs do not require a
emergency medical services (EMS) system. prescription but must be included with drug history.
(2) Type II (cytolytic or cytotoxic): inhibits or A. Prescription form and written contents:
prevents cell function; results in cell de- 1. Prescriber’s full name, address, telephone number, o0490
struction; may be result of blood transfu- DEA number (can be preprinted); group practices
sion or drug-induced anemia. may contain multiple prescribers’ names to be cir-
(3) Type III (Arthus): occurs within 6 to 8 hours cled or checked.
after exposure (SIMILAR to serum sick- 2. Superscription: dated order includes name, ad- o16510
ness); symptoms include urticaria (hives, dress, and age of patient; with symbol ] (means
wheals), lymphadenopathy, fever (e.g., al- prescription, capital R with a cross on diagonal)
lergy to topical benzocaine, penicillins, that separates superscription from inscription.
­sulfonamides). 3. Inscription: body of prescription; contains name
(4) Type IV (cell-mediated delayed hypersensi- and amount and/or strength of each ­ingredient for
tivity): occurs within 48 hours, results in an prescribed drug.
inflammatory reaction such as contact derma- 4. Subscription: directions to pharmacist in short
titis (e.g., allergy to penicillins, poison ivy). ­sentence.
(5) Idiosyncratic (individual) reactions: abnor- 5. Signatura (sig or signa, “write”): contains directions
mal and have genetic causes. to patient; can be written out or abbreviations can
F. Contraindications: be used (Table 9-1), with prescriber’s signature.
1. Conditions and factors that increase risks involved
in using particular drug (carrying out medical pro- Table 9-1  Pharmacological abbreviations  
cedure or engaging in particular activity); may be for prescription writing
absolute or relative; opposite of indication.
a. Absolute contraindication: situation that makes Abbreviation Meaning
particular treatment or procedure inadvisable.
Example: aspirin [Bayer, Ecotrin, Empirin, Buf­ a.c. Before meals
ferin] is an absolute contraindication for an in- b.i.d. Twice daily
fant or child because of danger that it can cause h.s. At bedtime
Reye’s syndrome; can substitute acetaminophen. q.d. Once per day
b. Relative contraindication: situation that makes q.i.d. 4 times per day
particular treatment or procedure somewhat in- q.o.d. Every other day
advisable but does NOT rule it out. Example:
q6h Every 6 hours
nitrous oxide sedation during pregnancy is a
p.c. After meals
relative contraindication because of concerns
for developing child; should not be used unless p.o. By mouth
deemed absolutely necessary because it is pref- p.r.n. As needed
erable to other general anesthetics; NOT classi- sl. Sublingual
fied in Pregnancy Risk Category (PRC); need stat Immediately
medical consult (discussed later). supp Suppository
G. Excretion: t.i.d. 3 times per day
1. MOST drugs are excreted by means of renal
u.d. As directed
route.
280   Saunders Review of Dental Hygiene

Table 9-2  Controlled substances*

Schedule Abuse potential Medical use Examples

I High None except research Heroin, hallucinogens,


opium derivatives
II: Requires prescription; High (severe dependence Legal medical uses Oxycodone, amphet-
nonrefillable ­possible) amines, morphine
III: Requires prescription; Lower than Schedule I   Legal medical uses Codeine, anabolic ste-
five refills in 6 months and II drugs (moderate roids
­dependence possible)
IV: Requires prescription; Low (limited dependence Legal medical uses Benzodiazepines,
five refills in 6 months ­possible) ­barbiturates
V: Requires prescription;   Lower than Schedule IV drugs Legal medical uses Codeine cough syrups
five refills in 6 months

*Drug schedules (classifications) issued as part of the Controlled Substances Act in 1970. The Drug Enforcement Administration determines which drugs are added
to and removed from the schedules.

6. Labeling: when drug name is to be included, A. The SANS enables the body to function during emer-
­“label” box is checked. gencies or respond to stressful situations (engage in
7. Refills: designated by number. “fight-or-flight”):
8. Proprietary (brand name, trade) vs. nonpropri- 1. By means of the release of its neurotransmitters,
etary (generic name): indication of desire or which include norepinephrine and epinephrine.
willingness to dispense type of drug known by 2. These neurotransmitters subsequently stimulate
chemical name (brand, generic). the SANS receptors, which include alpha (α)- and
B. Laws affecting prescription writing by types of beta (β)-receptors:
drugs: a. Stimulation of α-receptors causes vasoconstric-
1. Restricted drugs: available without prescription tion of the skin and skeletal muscles.
but kept at pharmacy service counter because of b. Excitation of β1-receptors causes stimulation of
labeling or abuse risk (e.g., drugs containing pseu- the heart muscle and glycogenolysis.
doephedrine, such as Claritin-D). c. Stimulation of β2-receptors causes smooth mus-
2. Legend drugs: may NOT be dispensed by a phar- cle relaxation, which results in bronchodilation
macist without prescription from physician, osteo- of the lungs.
path, dentist, or other approved practitioner; FDA B. The PANS enables the body to maintain bodily func-
and state drugs are legend; label carries the legend, tions during normal daily activities (engage in “stay-
“Caution! Federal law prohibits dispensing with- and-play”):
out a prescription.” 1. By means of the release of the neurotransmitter
3. Controlled drugs: require further safeguards for acetylcholine (ACh).
storage, in addition to requiring a prescription; 2. The ACh subsequently stimulates its receptor sites
refills are also limited; BOTH state and federal (muscarinic and nicotinic) to enhance body ­function.
government agencies promulgate (make public) C. Prolonged stressful situations can cause chronic o016520
the regulations (see Table 9-2 for Controlled Sub- SANS stimulation and depression of PANS with di-
stances Act). arrhea, constipation, difficulty maintaining sexual
a. Federal agency is Drug Enforcement Adminis- arousal, depressed immune system (see Chapter 16,
tration (DEA). Special Needs Patients: mental illness).
b. State agency is Division of Narcotics and
­Dangerous Drugs (DHHR). ADRENERGICS  
Adrenergics stimulate α- and β-receptors and include epi-
DRUG EFFECTS ON AUTONOMIC NERVOUS nephrine, levonordefrin, isoproterenol, ephedrine, dopa-
SYSTEM   mine III.
MANY drugs exert their effects on the autonomic nervous • See Chapters 11, Medical and Dental Emergencies:
system (ANS). There are two ANS systems: sympathetic emergency kit for the dental setting; 14, Pain Manage-
division (SANS) and parasymphathetic division (PANS). ment: vasoconstrictor use.
• See Chapter 3, Anatomy, Biochemistry, and Physiol- A. Pharmacological effects:
ogy: ANS. 1. Excitation of CNS.
Pharmacology    281

2. Effects on cardiovascular system (CVS): B. Therapeutic uses:


a. Hypertension (high blood pressure [HBP]), dys- 1. α-Blockers:
rhythmias, tachycardia (increased pulse rate). a. Peripheral vascular disease (PAD) or Raynaud’s
b. Blood vessel vasodilation with β2-receptors, syndrome.
vasoconstriction with α1-receptors. b. Pheochromocytoma, catecholamine-secreting
3. Hyperglycemia from β1 stimulation increases gly- tumor of adrenal medulla.
cogenolysis, decreases release of insulin. 2. β-Blockers and α1-selective blockers:
4. Bronchodilation, mydriasis (pupil dilation), xero- a. HBP, angina pectoris, dysrhythmias, migraine
stomia (dry mouth). headaches, alcohol withdrawal, myocardial in-
B. Therapeutic uses: farction (MI, heart attack) prophylaxis, anxiety
1. Vasoconstriction: syndromes.
a. Increases duration of local anesthetic actions C. Adverse reactions:
and decreases systemic location and toxicity. 1. Few occur because receptor effects are MORE site
b. Controls bleeding through hemostasis, nasal specific.
decongestants (due to α-receptors). 2. Xerostomia, nausea, vomiting.
2. Treatment of cardiac arrest by epinephrine (be- 3. Hypotension (low blood pressure).
cause of β1-stimulation). D. Drug interactions (see also antiadrenergics under
3. Bronchodilation (because of β2-receptors) for ­antihypertensives):
treatment of asthma and emphysema. 1. Effects of β-blockers:
4. Stimulation of central nervous system (CNS) by a. Decreased by sympathomimetics, nonsteroidal
methylphenidate (Ritalin) or pemoline (Cylert) for antiinflammatory drugs (NSAIAs/NSAIDS),
treating attention deficit–hyperactivity disorder rifampin (Rifadin, Rimactane).
(ADHD). b. Increased by calcium channel blockers.
C. Adverse reactions: E. Contraindications:
1. CNS stimulation, resulting in anxiety and ­tremors. 1. Hypersensitivity, asthma.
2. Heart palpitations and dysrhythmias. 2. Congestive heart failure (CHF).
3. HBP (if drug affects α-receptors).
D. Drug interactions: CHOLINERGICS  
1. Decreased pressor effect with haloperidol, pheno- Cholinergics stimulate body functions by direct action
thiazines, thioxanthenes, diuretics. (choline esters) or indirect action (inhibition of ace-
2. Cardiac dysrhythmias with halogenated general tylcholinesterase). Include pilocarpine (Salagen) and
anesthetics, cardiac glycosides, tricyclic antide- bethanechol (Urecholine).
pressants (TCAs). A. Pharmacological effects:
3. HBP with β-blockers from blocking β-adrenergic 1. Effects on the CVS:
effects of epinephrine. a. Bradycardia (slow pulse rate), hypotension (­direct
E. Contraindications: effect on heart), tachycardia (rapid pulse rate).
1. Narrow-angle glaucoma from an increase in intra- b. HBP (indirect effect on blood vessels).
ocular pressure. 2. Nausea, diarrhea, dyspepsia (indigestion).
2. Cardiac dysrhythmias. 3. Miosis (pupil constriction), decreased intraocular
pressure.
ADRENERGIC BLOCKERS   B. Therapeutic uses:
Adrenergic blockers inhibit α- and β-receptors, including 1. Xerostomia.
(1) α-adrenergic blockers such as tolazoline (Priscoline) 2. Glaucoma by decreasing intraocular pressure.
and phentolamine (Regitine); (2) β-adrenergic blockers, 3. Urinary retention after surgery by increasing se-
including nonspecific blockers such as propranolol (In- cretions and activity of GIT.
deral) and nadolol (Corgard) and specific blockers such 4. Myasthenia gravis (MG) by decreasing muscle
as metoprolol (Lopressor) and atenolol (Tenormin); (3) α- weakness.
and β-adrenergic blockers such as labetalol (Normodyne, C. Adverse reactions: increased salivation, lacrimation,
Trandate). See later discussion of patient with CVD. urination, defecation.
A. Pharmacological effects: D. Drug interactions:
1. α-Blockers inhibit vasoconstriction in blood ves- 1. Reduced effects with anticholinergics.
sels (α-receptor effects). 2. Enhanced effects with cholinergics.
2. β-Blockers competitively block the effects of ad- E. Contraindications:
renergics on β-receptors and may inhibit specific 1. Severe cardiovascular disease (CVD) from its
β-receptors or be nonspecific receptor site ­inhibitor. CVS effects.
282   Saunders Review of Dental Hygiene

2. Peptic ulcer, asthma, obstruction of GIT or urinary


tract by increasing secretions. ANXIETY AND PAIN MANAGEMENT  
3. Uncontrolled hyperthyroidism, with impaired drug Management of pain and anxiety during dental treat-
metabolism. ment uses antianxiety drugs, analgesics, anesthetics,
4. Patients with MG taking cholinesterase inhibitors ­sedatives.
(neostigmine [Prostigmin]).
Antianxiety Drugs
ANTICHOLINERGICS   Main purpose of antianxiety (tranquilizer) drugs is to de-
Anticholinergics (parasympathomimetics) inhibit body crease anxiety through depression of the CNS. Drugs are
functions by blocking muscarinic cholinergic receptors. also noted with therapeutic uses.
Include atropine (oral, Sal-Tropine; ophthalmic, Atro- A. Benzodiazepines (BZDs) (suffixes “-lam,” “-pam”):
pair) and propantheline bromide (Pro-Banthine). 1. Pharmacological effects:
A. Pharmacological effects: a. Antianxiety and anticonvulsant effects and
1. The CNS effects are dose dependent: muscle relaxation.
a. High doses result in stimulation. b. Anterograde amnesia (temporary memory im-
b. Therapeutic doses result in depression. pairment).
2. The CVS effects are dose dependent: c. Wide therapeutic index makes overdose poi-
a. High doses result in tachycardia (vagal block- soning rare (compared with barbiturates).
ade). 2. Therapeutic uses:
b. Small doses result in bradycardia. a. Anxiety associated with depression and panic at-
3. Smooth muscle relaxation results in bronchodila- tacks with diazepam (Valium), alprazolam (Xanax),
tion and constipation. temazepam (Restoril), triazolam (Halcion).
4. Exocrine gland by reducing secretions in the respi- b. Insomnia by means of CNS depression with flu-
ratory tract, genitourinary tract, GIT. razepam (Dalmane).
5. Include mydriasis and cycloplegia (blurred vision). c. Alcohol withdrawal (agitation, tremors [delir-
B. Therapeutic uses: ium tremens, DTs]) with chlordiazepoxide (Lib­
1. Preoperative drug to inhibit salivary and bronchial rium), lorazepam (Ativan), oxazepam (Serax).
secretions, producing dry field. d. Epileptic seizures with clonazepam (Klonopin),
2. Eye examinations with relaxation of the lens and diazepam (Valium).
mydriasis. e. Antianxiety premedication in dentistry with di-
3. Blocking of vagal slowing of the heart in response azepam (Valium) or alprazolam (Xanax); also
to general anesthetics. memory loss of stressful dental procedures.
4. Parkinson-like tremors caused by smooth muscle 3. Adverse reactions:
relaxation from antipsychotics. a. Xerostomia.
5. Motion sickness; also sleep aid with depression of b. Depression of CNS, which results in sedation
CNS. and fatigue.
6. GIT disorders with decreasing secretions and hy- c. Thrombophlebitis, if given IV.
permotility (e.g., gastric ulcers). d. Possible physical and psychological addiction;
C. Adverse reactions: occurs less often than with barbiturates.
1. Xerostomia, photophobia (light sensitivity), cy- e. Possible teratogenicity, especially during first
cloplegia (paralysis of the lens). trimester.
2. Signs of CNS stimulation, tachycardia, hyperpy- 4. Drug interactions:
rexia (high fever). a. Increased effects with alcohol and CNS depres-
3. Urinary and GIT stasis. sants.
D. Drug interactions: b. Decreased effects with smoking.
1. Increased anticholinergic effects with TCAs, anti- c. Avoid with antifungals.
histamines, antidepressants, opioids, antipsychotics. d. Increases effects of digoxin (Lanoxin) and phe-
2. Decreased absorption of ketoconazole (Nizoral). nytoin (Dilantin) (particularly with clonazepam
E. Contraindications: [Klonopin]).
1. Prostatic hypertrophy or intestinal or urinary ob- 5. Contraindications:
struction or retention because drug increases uri- a. Drug addiction, narrow-angle glaucoma, hyper-
nary retention. sensitivity, pregnancy.
2. For CVD because of tachycardia, vagal blockade. b. Psychosis (experience CNS stimulation),
3. Glaucoma (narrow-angle) from an increase in in- chronic pain (because of possible abuse and
traocular pressure. ­addiction).
Pharmacology    283

B. Barbiturates (among first antianxiety drugs): (2) High: unpredictable emotional reactions
1. Classified (by length of action): and mental confusion, severely impaired
a. Ultrashort acting: methohexital (Brevital) and judgment, mood swings.
thiopental sodium (Pentothal). b. Mental effects depend on amount and strength;
b. Short acting: pentobarbital (Nembutal) and person falls asleep but drug remains in system
secobarbital (Seconal). for long time.
c. Intermediate acting: amobarbital (Amytal) and 5. Drug interactions:
butabarbital (Butisol). a. Decreased effects of acetaminophen (Tylenol),
d. Long acting: phenobarbital (Luminal). β-blockers, oral contraceptives, doxycycline
2. Pharmacological effects: (Vibramycin), phenytoin (Dilantin), steroids,
a. Depression of CNS, which results in sedation. TCAs, warfarin (Coumadin).
(1) Higher doses result in greater CNS depression, b. Increased effects with disulfiram, monoamine oxi-
including respiratory and CVS depression. dase inhibitors (MAOIs), propoxyphene (Darvon).
b. Anticonvulsant effect. c. Increased CNS depression with alcohol and
c. Stimulate microsomal liver enzymes to stimulate other CNS depressants.
metabolism of other drugs metabolized by liver. 6. Contraindications:
3. Therapeutic uses (by duration of action): a. Hypersensitivity.
a. Ultrashort-acting barbiturates: short-term gen- b. Porphyria (excess amount of porphyrins); can
eral anesthesia. stimulate and increase the production of por-
b. Short-acting and intermediate-acting barbitu- phyrins.
rates: replaced by benzodiazepines for antianxi- c. Chronic pain and drug addiction (because of
ety and insomnia. risk of further abuse and addiction).
c. Long-acting barbiturates: treatment of seizures C. Chloral hydrate (orally): sedation of children.
with epilepsy. D. Meprobamate: dental anxiety disorders and as mus-
4. Adverse reactions: cle relaxant for acute temporomandibular disorders
a. Like alcohol, barbiturates are intoxicating: (TMD) with muscle spasms.
(1) Mild: slurred speech, loss of coordination, E. Zolpidem (Ambien): nonbenzodiazepine sedative
stumbling, staggering, shallow breathing, hypnotic that is used as short-term sleep aid for in-
fatigue, frequent yawning, irritability. somnia: SAME adverse effects and drug interactions
as other sedatives.

clinical study  

Age 45 YRS Scenario

Sex ☒  Male    ☐  Female The patient is scheduled for an oral


prophylaxis today. He has general-
BP 105/57 ized dental biofilm and slight calculus
deposits throughout, with only slight
Chief “I hate being here but I know that I have to keep up  
Complaint my mouth.” to moderate gingivitis.

Medical Myasthenia gravis (MG)


History Rheumatoid arthritis (RA)

Current prednisone (Deltasone) 20 mg qod  


Medications neostigmine (Prostigmin) 5 mg qd  
diazepam (Valium) 10 mg before dental appointments

Social History Science fiction writer

1. What types of drugs are prednisone (Deltasone) and 4. Why does the patient especially need to avoid stress
neostigmine (Prostigmin), and why is he taking them? that a dental appointment might bring on, as well as
2. What type of drug is diazepam (Valium)? Identify its avoid any dental infections?
common uses.
3. What are the oral signs of MG that are possible for 1. Prednisone (Deltasone) is a corticosteroid, antiinflam-
him? matory drug. It treats various diseases of ­ allergic,
284   Saunders Review of Dental Hygiene

inflammatory, autoimmune origin. Patient takes


­prednisone to treat RA, an inflammatory joint dis- Analgesics
ease. Neostigmine (Prostigmin) is an anticholinester- Analgesics inhibit perception of pain. Nonopioids reduce
ase muscle stimulant that markedly improves muscle pain perception by inhibiting prostaglandin synthesis;
strength by improving muscle contraction; he takes it MORE effective if they are taken before onset of pain.
to treat MG, disease of muscular weakness. Opioids depress pain perception in CNS by binding with
2. Diazepam (Valium) is a benzodiazepine (BZD), opioid receptors (Tables 9-3, 9-4, and 9-5).
­antianxiety, anticonvulsive drug. He takes it as an • See Chapter 11, Medical and Dental Emergencies:
­antianxiety premedication to help cope with dental emergency kit (aspirin).
anxiety. A. Nonopioids:
3. MG affects facial and cervical musculature, leading 1. Salicylates: aspirin (Bayer, Ecotrin, Empirin, Buff-
to loss of control of facial muscles and resulting in erin) and long-acting diflunisal (Dolobid).
difficulty with smiling, eating, swallowing, speaking, a. Pharmacological effects:
vision; when severe, can cause respiratory distress. (1) Analgesia for mild-to-moderate pain relief;
4. Crisis can occur as result of infection (oral included), antipyretic effect (reduces fever).
emotional stress (such as dental care), and surgery. (2) Antiinflammatory effect (from inhibition of
It could involve a myasthenic crisis, which includes prostaglandin synthesis), decreases vasodi-
loss of swallowing and speaking ability and difficulty lation and swelling.
breathing and seeing; caused by undermedication, (3) Antiplatelet effect (from reduction of plate-
underlying illness, risk factors, or worsening of dis- let aggregation), results in reduced clotting.
ease. Or it could involve a cholinergic crisis: related to (4) Uricosuric effect; high doses (MORE than
overmedication with anticholinesterase, occurs within 5 g/24 hours) cause increase in uric acid
hour of drug use, causes increased muscle weakness, ­secretion in urine.
gastrointestinal upset, respiratory difficulties. Both b. Therapeutic uses:
crises require the dental office to provide basic life (1) Analgesia for mild-to-moderate pain.
support and activate EMS system. (2) Reduction of fever (antipyretic effect).

Table 9-3  Drug interactions with commonly used analgesics

Drug Interactions

NSAIA Decrease antihypertensive effects of ACE inhibitors (captopril), beta blockers, loop diuretics,  
and thiazide diuretics
Increase effects of anticoagulants
Increase effects of digoxin
Increase effects of phenytoin
Increase effects of lithium
Increase methotrexate toxicity
Increase effects of sympathomimetics
Nephrotoxicity with cyclosporine
Probenecid increases NSAIA concentrations
Salicylates decrease NSAIA concentrations
Salicylates (aspirin Alcohol increases gastrointestinal ulceration and bleeding time
[Bayer, Ecotrin, Corticosteroids decrease salicylate concentrations
­Empirin, Buff- Decrease antihypertensive effects of ACE inhibitors (captopril), beta blockers, loop diuretics,  
erin]) and thiazide diuretics
Decrease NSAIA concentrations
Decrease uricosuric effect of probenecid and sulfinpyrazone
Increase effects of anticoagulants
Increase effect of sulfonylureas and exogenous insulin, thereby increasing hypoglycemic effect
Dose >2 g per day displaces VA from binding site, thereby increasing VA effect
Nizatidine increases salicylate concentrations
Acetaminophen Alcohol increases hepatotoxicity
­(Tylenol) Beta blocker (propranolol) increases effects of acetaminophen
Oral contraceptives decrease half-life

ACE, Angiotensin converting enzyme; NSAIA, nonsteroidal antiinflammatory agent; VA, valproic acid.
Pharmacology    285

Table 9-4  Nonopioids: comparison of analgesic efficacy

Drug Dose (mg) Dosing interval (hr) Peak (hr)

Acetaminophen (Tylenol) 325-650 4-6 0.5-2


Aspirin (Bayer, Ecotrin, Empirin, ­Bufferin) 650 4-6 1-2
Ibuprofen (Advil, Motrin, Pamprin) 400 4-6 1-2
Diflunisal (Dolobid) Loading dose: 1000; ­  8-12 1-2
subsequent doses: 500
Naproxen sodium (Aleve, Anaprox) 250-500 6-8 2-3

Table 9-5  Analgesic drug combinations and uses

Drug name Opioid component Nonopioid component Dental use

Hydrocodone a/APAP (Bancap HC,   Hydrocodone Acetaminophen Mild to moderate pain;


Dolacet, Vicodin) ­posttreatment pain
control
Acetaminophen w/codeine   Codeine Acetaminophen Mild to moderate pain;
(Capital w/codeine, Phenaphen   ­posttreatment pain
w/codeine, Tylenol w/codeine) control
Propoxyphene   Propoxyphene   Acetaminophen Mild to moderate pain;
N/APAP (Darvocet-N 100,   ­napsylate ­posttreatment pain
Wygesic, Roxicet) control
Percocet (Tylox) Oxycodone Acetaminophen Moderate to severe pain;
posttreatment of  
postoperative pain
Hydrocodone w/aspirin   Hydrocodone Aspirin Mild to moderate pain;
(Lortab ASA, Alora 5/500,   ­posttreatment pain
Azdone) control
Oxycodone w/aspirin (Percodan,   Oxycodone Aspirin Moderate to severe pain;
Codoxy, Percodan-Demi,   posttreatment of  
Roxiprin) postoperative pain

(3) Reduction of pain and swelling (antiinflam- (2) Increased risk of GIT complaints with
matory effect); also treats inflammatory NSAIAs, corticosteroids, alcohol.
conditions such as arthritis. (3) Increased risk of hypoglycemia with oral
(4) Prevention of post–myocardial infarction antidiabetics.
(MI, heart attack) (because of antiplatelet (4) Decreased effects of gout drugs such as
effect) at low-dose (LD) levels. probenecid (Benemid, Probalan) and sul-
c. Adverse reactions: finpyrazone (Anturan).
(1) GIT irritation, MOST common (caused by (5) Increased risk of toxicity with methotrexate
direct irritation of stomach mucosa). (Rheumatrex Dose Pack, Trexall), lithium
(2) Bleeding levels monitored by international (Lithobid, Eskalith), zidovudine (AZT;
normalized ratio (INR); INRs ≤2.5 are safe Retrovir).
for invasive dental work; older test is pro- e. Contraindications:
thrombin time (PT). (1) Hypersensitivity.
(3) Asthmatics; use acetaminophen (FIRST (2) GIT bleeding and bleeding disorders (be-
choice) as substitute for its analgesic and cause of GIT irritation and antiplatelet
antipyretic effects to avoid aspirin-induced effect); peptic ulcers (because of GIT
attack. ­irritation).
d. Drug interactions: (3) Children with flulike symptoms (Reye’s
(1) Increased risk of bleeding with anticoagu- syndrome [abnormal accumulations of
lants. fat begin to develop in the liver and other
286   Saunders Review of Dental Hygiene

o­ rgans of the body, along with a severe in- (1) Analgesic and antipyretic effects SIMI-
crease of pressure in the brain] can occur LAR to aspirin (Bayer, Ecotrin, Empirin,
during some viral infections). Bufferin) and NSAIAs.
(4) Can be toxic if larger doses taken, espe- (2) Little or NO antiinflammatory effect occurs
cially if expired (will smell like vinegar). (UNLIKE other NSAIAs).
2. NSAIAs (NSAIDs): naproxen (Naprosyn; b. Therapeutic uses: reduction of mild to moder-
OTC: Aleve, Anaprox), ibuprofen (OTC: Advil, ate pain and fever; as substitute for its analgesic
Motrin, Pamprin), etodolac (Lodine), ketorolac and antipyretic effects to avoid aspirin-induced
(Toradol), indomethacin (Indocin), celecoxib attack in asthmatics.
(Celebrex). c. Adverse reactions:
a. Pharmacological effects: (1) Hepatotoxicity and nephrotoxicity (associ-
(1) Analgesic, antipyretic, antiinflammatory ated with long-term use, excessively high
effects SIMILAR to aspirin (Bayer, Eco- doses).
trin, Empirin, Bufferin). (a) Minimum toxic dose is 10 g (140 mg/kg),
b. Therapeutic uses: control of mild to moderate but liver damage occurs with one 5.85-g
pain, fever, inflammatory conditions such as dose.
RA. (b) Chronic excessive use (MORE than 4 g/
c. Adverse reactions: day) can cause transient hepatotoxicity.
(1) Stomach mucosa irritation SIMILAR to (c) Safe dose for chronic alcoholics should
aspirin’s effect on GIT; xerostomia. NOT exceed 4 g/day; should be 2 g/day
(2) Inhibition of platelets; occur ONLY for or less if hepatotoxicity has occurred (oc-
drug’s duration. curs in some chronic alcoholics who take
(3) Depression of the CNS, including sedation, the therapeutic dose).
dizziness, depression. d. Drug interactions: rare at therapeutic doses.
(4) Teratogenicity (can interfere with closure e. Contraindications:
of ductus of fetal heart [ductus arterio- (1) Hypersensitivity.
sus]). (2) Hepatic and renal disease.
d. Drug interactions: B. Opioids: morphine (MS Contin), hydromorphone
(1) Decreased effects of antihypertensives. (Dilaudid), meperidine (Demerol), ­ propoxyphene
(2) Increased effects of oral anticoagulants, (Darvon), codeine (various cough preparations), oxy-
lithium (Lithobid, Eskalith), digoxin (Lan- codone (OxyContin; with acetaminophen [Percocet],
oxin), phenytoin (Dilantin), sympathomi- with aspirin [Percodan]) (Table 9-4).
metics. 1. Pharmacological effects:
(3) Increased risk of toxicity with methotrex- a. Analgesic effect for relief of moderate to severe
ate (Rheumatrex Dose Pack, Trexall) and pain; sedative effect for relief of mild anxiety.
cyclosporine. b. Antitussive effect (cough suppression); antidi-
(4) Raises serum concentration of hypoglyce- arrheal effect (caused by increase in smooth
mic agents. muscle tone in GIT); constipating.
(5) May cause GIT ulcers with corticosteroids. c. Depress CNS by interacting with opioid
e. Contraindications: ­receptors.
(1) Hypersensitivity to ANY antiinflammatory 2. Therapeutic uses: analgesia for moderate to severe
drugs. pain, cough suppression, sedation, anxiety relief.
(2) Blood clotting problems (because of anti- 3. Adverse reactions are related to the drug’s potency
platelet effect). (greater potency, greater risk of adverse reactions):
(3) Renal disease (caused by higher risk of ad- a. Addicting; can be abused.
verse renal reactions); GIT disorders, such b. Drowsiness, sedation, and (occasionally) stim-
as peptic ulcer; asthma. ulation; decreased rate and depth of respiration
(4) Celecoxib (Celebrex) is a COX II inhibitor: (causing death with overdose).
increased risk of MI/CVA (cardiovascu- c. Increased intracranial pressure (because of cra-
lar accident, stroke) but can be taken with nial vasodilation); bradycardia and orthostatic
low-dose aspirin (Bayer, Ecotrin, Empirin, hypotension.
Bufferin). d. Nausea, emesis (vomiting), constipation; uri-
3. Acetaminophen (N-acetyl-para-amino-phenol nary retention.
[NAPP]; OTC: Tylenol). e. Miosis (pupil constriction); may be used to de-
a. Pharmacological effects: termine abuse or overdose.
Pharmacology    287

4. Drug interactions: increased sedation and respira- (1) Esters can result in topical allergies (from
tory depression with other CNS depressants such PABA) (see earlier discussion).
as MAOIs, TCAs, alcohol. (2) Overall, can increase plasma levels with
5. Contraindications: overuse and lead to toxicity.
a. Hypersensitivity. b. Periodontal gel: 2.5% lidocaine, 2.5% prilo-
b. Respiratory problems, including asthma and caine (Oraqix) for site-specific soft tissue an-
emphysema (because of respiratory center esthesia; see state regulation if can be used by
­depression). dental hygienist:
c. Chronic pain (abuse and addiction may occur). (1) LOW levels in plasma, NO allergenic po-
d. Head injury (caused by increase in intracranial tential; MRD 5 cartridges or 8.5 mg, with
pressure). 1 cartridge usually used per quadrant.
C. Semisynthetic opioids: hydrocodone (Vicodin, Lorcet): (2) Used with caution in patients with severe o16530
1. Derived from two of the naturally occurring opi- impairment of renal or hepatic function;
ates, codeine or thebaine. NOT used with pregnant woman or under
2. Combined with paracetamol (acetaminophen, Ty- age 18.
lenol), aspirin (Bayer, Ecotrin, Empirin, Bufferin), (3) Indications: nonsurgical periodontal ther- o16540
ibuprofen (Advil, Motrin, Pamprin), homatropine apy, substitution for injection when injec-
methylbromide (hydrocodone compounds). tion is contraindicated.
3. Purpose of the noncontrolled drugs in combina- (4) Technique: o16550
tion: (a) Apply on gingival margin around o16560
a. Provide increased analgesia because of synergy. selected teeth using blunt-tipped ap-
b. Limit intake by causing unpleasant and often un- plicator; wait 30 seconds and fill
safe side effects at higher than prescribed doses. periodontal pockets until becomes
4. Pharmacological effects and therapeutic uses: nar- visible.
cotic analgesic and antitussive. (b) Wait another 30 seconds before starting
5. Adverse so reactions, drug interactions, contrain- treatment.
dications: SAME as opioids. c. Lidocaine transoral delivery system (Denti-
a. Addicting, so can be abused. Patch): 46.1 mg of lidocaine for site-specific
b. Liver problems caused by larger amounts of soft tissue anesthesia.
­acetaminophen (Tylenol). (1) Onset of action is 5 to 10 minutes; duration
D. Opioid antagonists: of action is ∼45 minutes.
1. Short-acting naloxone (Narcan): used to reverse (2) Indications: preinjection anesthesia (espe-
respiratory depression caused by opioids. cially in palatal areas), nonsurgical perio-
2. Long-acting naltrexone (ReVia, Trexan): used for dontal therapy, substitution for injection
opioid addiction after detoxification. when injection is contraindicated.
(3) Technique:
Anesthetics and Sedatives (a) Remove plastic backing.
Anesthetic, local (with or without vasoconstrictors) and (b) Isolate and dry targeted area for
systemic, as well as sedatives to control pain. 30 seconds.
• See Chapter 14, Pain Management: local anesthetics, (c) Apply with pressure for 30 seconds.
vasoconstrictors, drug for reversal of local anesthesia, d. Injected for soft and hard tissue anesthesia:
nitrous oxide sedation. (1) Esters:
A. Local anesthetics: (a) Procaine (Novocaine) and propoxy-
1. Pharmacological effects: caine.
a. Reversibly blocked peripheral nerve conduc- (b) Withdrawn from market (because of in-
tion. creased allergenic potential).
b. Depressed cardiac conduction and excitability (2) Amides:
that results in hypotension. (a) Shorter-acting injectables: 2% lido-
2. Therapeutic uses: caine (Xylocaine, Anestacaine), 2%
a. Topical: 20% benzocaine (preinjection or surgi- and 3% mepivacaine (Carbocaine),
cal gel, liquid, or spray, Americaine, Lanacaine, 4% prilocaine (Citanest), 4% articaine
Hurricane; oral gel, Anbesol, Orajel) and 5% li- (Septocaine, Zorcaine).
docaine (preinjection or surgical gel, Topicaine; (b) Longer-acting injectables: 2% bupi-
patch, Lidoderm; preinjection or anesthetic vacaine (Marcaine), 1.5% etidocaine
cream 2.5% with prilocaine 2.5%, EMLA): (­Duranest).
288   Saunders Review of Dental Hygiene

3. Adverse reactions: 4. Drug interactions:


a. Talkativeness, apprehension, tremors, seizures a. HBP, when used in conjunction with nonselec-
(sometimes followed by CNS depression, respi- tive β-blockers.
ratory and CVS depression, coma). b. Vasoconstriction is enhanced with TCAs when
b. Hematoma at site of injection (especially if epinephrine is given IV.
piercing pterygoid plexus of veins, maxillary c. Increase in blood glucose with oral antidiabetics.
artery). 5. Contraindications:
c. Allergy, ranging from a rash to anaphylaxis; a. Absolute: hypersensitivity to sulfite compounds
allergic reactions to esters occur MORE often that serve as preservative antioxidants may
than those to amides; if there is an allergy to cause allergic reactions in asthmatics and/or
BOTH esters and amides, first-generation anti- sensitivity to sulfites (red wine, aged cheese).
histamine diphenhydramine (Benadryl) can be b. Relative: CVD patient who undergoes elective
used for a local anesthetic effect. dental treatment should:
d. Metabolism of amides occurs primarily in liver; (1) Receive a local anesthetic with a vaso-
esters hydrolyzed by pseudocholinesterase, a constrictor to increase pain control and its
plasma enzyme. duration, which lowers patient stress level
e. BOTH amides and esters are excreted by kidneys. (decreasing endogenous epinephrine).
4. Drug interactions: additive effect with CNS de- (2) Receive an MRD of 0.04 mg of vasocon-
pressants. strictor per appointment (equal to one
5. Contraindications: cartridge of 1:50,000, two cartridges of
a. Hypersensitivity; sulfite compounds in articaine 1:100,000, or 4 cartridges of 1:200,000).
(Septocaine, Zorcaine). C. General anesthetics:
b. May cause allergic reactions in asthmatics and/ 1. Pharmacological effects: depress CNS.
or sensitivity to sulfites. a. Guedel’s stages:
B. Vasoconstrictors (inclusion in some local anesthet- (1) Stage I: analgesia (reduced pain sensation).
ics): epinephrine, levonordefrin (Neo-Cobefrin). (2) Stage II: delirium and excitement (uncon-
1. Pharmacological effects: sciousness and involuntary movement).
a. Cardiac stimulation and lipolysis in response to (3) Stage III: surgical anesthesia.
β1-receptor stimulation. (4) Stage IV: respiratory and medullary paraly-
b. Bronchodilation in response to β2-receptor sis (death occurs if NOT reversed).
stimulation. b. Flagg’s phases (better description of anes-
c. Epinephrine causes vasoconstriction of blood thesia levels):
vessels in response to α1 receptor stimulation. (1) Induction phase: occurs before surgery.
2. Adverse reactions: tremors, anxiety, palpitations, (2) Maintenance phase: begins when depth
HBP. of anesthesia necessary for operation is
3. Therapeutic effects: achieved; continues until procedure is com-
a. Epinephrine is available with ALL amides EX- pleted.
CEPT mepivacaine, which is available ONLY (3) Recovery phase: occurs at end of operation,
with levonordefrin (Carbocaine with Neo- continues through postoperative period un-
Cobefrin). til patient is fully responsive.
(1) Available at 1:20,000 (with levonordefrin), 2. Adverse reactions:
1:50,000, 1:100,000, 1:200,000 (with epi- a. Dysrhythmias, cardiac arrest, HBP, hypoten-
nephrine). sion.
(2) An MRD of 0.2 mg per appointment for b. Respiratory depression and arrest.
healthy patients. c. Teratogenicity, with chronic exposure.
(3) Used MAINLY at 1:100,000 for nerve d. Hepatotoxicity, with chronic exposure.
blocks and 1:50,000 for infiltrations; higher D. General anesthetics:
levels do NOT increase effectiveness, only 1. Nitrous oxide, inhalation gas (relatively odorless,
hemorrhage control. colorless).
b. Increased duration of local anesthetics because a. Pharmacological effects:
of decreased absorption into CVS. (1) Analgesia and amnesia, affecting the CNS,
c. Decreased absorption of local anesthetics into with NO significant effect on the respira-
CVS because of blood vessel constriction. tory system.
d. Epinephrine has hemostatic properties (stop- (2) Used instead in MOST cases for mild to o3990
page of bleeding) by blood vessel constriction. moderate (conscious) sedation and NOT
Pharmacology    289

at usual general anesthetic levels (less than a. Pharmacological effects: used IV for con- o4210
50%, standard of care) during nonsurgical scious sedation, short-duration sedation; often
dental treatment. used for oral surgery; also used as adjunctive
b. Adverse reactions: drug in balanced anesthesia technique or for
(1) Peripheral vasodilation. ­antianxiety premedication before appoint-
(2) Nausea and vomiting, especially if patient ment.
consumes large meal before. b. Adverse reactions: thrombophlebitis with diaz-
(3) Diffusion hypoxia may result in headache epam (Valium); using water-soluble midazolam
or other adverse reactions if patient does (Versed) reduces risk.
NOT receive 5 minutes of 100% oxygen 3. Halogenated ether, enflurane (Ethrane).
when sedation is terminated. a. Pharmacological effects: produces greater skel-
(4) Neuropathy and paresthesia result with etal muscle relaxation than halothane.
abuse; symptoms include numbness of b. Adverse reactions:
hands and legs, as well as liver and kidney (1) Respiratory depression, hypotension, myo-
problems. cardial contractility depression.
(5) Inhalation volatile liquids. (2) Dysrhythmias (occur less often than with
c. Absolute contraindications: halothane).
(1) Recent ophthalmic surgery during which (3) Depressed renal function.
gas was administered (C3F8, perfluoropro- (4) Possible seizure activity.
pane or SF6, sulfur hexafluoride). 4. Halogenated hydrocarbon, halothane (Fluothane).
(a) Gas bubble in the eye is associated with a. Pharmacological effects:
perfluoropropane use within the past 8 (1) Fruity odor; is nonflammable, nonexplo-
weeks or with sulfur hexafluoride use sive.
within the past 14 days. (2) Safe for asthmatics because does NOT ir-
(b) Can disrupt the bubble and cause blind- ritate bronchioles.
ness. b. Adverse reactions:
(2) Cystic fibrosis. (1) Incomplete muscle relaxation; depression
(3) Lack of patient cooperation or existence of of renal function.
communication barrier (such as differing (2) Uterine muscle relaxation; possible brady-
language). cardia, hypotension, dysrhythmias.
(4) Patients with head trauma or shock. 5. Halogenated ether, isoflurane (Forane).
(5) Patients who fear nitrous oxide or those a. Pharmacological effects:
who must ALWAYS be in control (NEVER (1) Does NOT cause liver toxicity.
talk anyone into use). b. Adverse reactions:
d. Relative contraindications: (1) Respiratory depression, hypotension, mus-
(1) Respiratory obstruction and congested air- cle relaxation.
way (NO exchange of gas possible through (2) Deeper levels of anesthesia result in respi-
nasal passages). ratory acidosis.
(2) Chronic obstructive pulmonary disease 6. Ultrashort-acting IV barbiturates: thiopental
(COPD) (CANNOT tolerate receiving ad- (Pentothal), methohexital (Brevital), thiamylal
ditional oxygen). (­Surital):
(3) Emotional instability, mental illness (al- a. Pharmacological effects: IV results in rapid
tered sense or euphoria may occur); poor onset of action; barbiturates are NOT anal-
patient communication. gesics; local anesthetics are required for pain
(4) Pregnancy because of concerns for devel- control.
oping fetus unless it is absolutely necessary b. Adverse reactions: bronchospasm, laryngo-
because other general anesthetics cannot spasm, hiccups, increased muscle activity, de-
be used; NOT classified in Pregnancy Risk lirium on recovery.
Category (PRC); chronic exposure reduces c. Contraindications: porphyria and status asth-
fertility in women; spontaneous abortion or maticus.
miscarriage may occur. 7. Ketamine (Ketalar): chemically related to phency-
(5) Contagious disease (e.g., tuberculosis, hep- clidine (PCP):
atitis) if nonautoclavable tubes are used. a. Pharmacological effects: produces dissociative
2. Benzodiazepines (BZDs): diazepam (Valium), anesthesia (patient fails to respond to environ-
midazolam (Versed): ment but is NOT asleep).
290   Saunders Review of Dental Hygiene

b. Adverse reactions: (1) Acyclovir (Zovirax): topically, orally, par-


(1) HBP and tachycardia, nausea, excessive enterally.
salivation; atropine (Sal-Tropine) may be (2) Valacyclovir (Valtrex): orally; longer dura-
required for dry field. tion of time than acyclovir; converted to
(2) Can cause hallucinations and delirium dur- acyclovir in body.
ing recovery. (3) Penciclovir: topically (Denavir) and
c. Contraindications: HBP and CVD. by IV.
8. Opioids: sufentanil (Sufenta), fentanyl (Subli- (4) Famciclovir (Famvir): longer duration of
maze), alfentanil (Alfenta), morphine (MS Con- time than acyclovir (Zovirax); converted to
tin). penciclovir in body.
a. Pharmacological effects: does NOT cause sig- 2. Adverse reactions: suprainfections, resistant
nificant peripheral vascular resistance or CVD strains.
changes. B. AIDS antivirals:
b. Adverse reactions: respiratory depression, 1. Include combination drug therapy, highly active
which is reversible with naloxone, opioid an- antiretroviral therapy (HAART) with reverse tran-
tagonist. scriptase inhibitors, such as AZT, D4T, ddI, or ddC,
9. Innovar: neuroleptics, combination of antipsychotic and one or more viral protease inhibitors; HAART
(droperidol) with opioid analgesic (­fentanyl). extends length of disease-free period of infection
a. Pharmacological effects: droperidol causes by suppressing viral replication and encouraging
sedation and catatonia; drug combination pro- blood CD4+ count increases.
duces wakeful anesthetic state. 2. Adverse reactions; these toxicities LIMIT use:
b. Adverse reactions: a. Headache, nausea, diarrhea, anemia.
(1) Boardlike chest, which requires ventilatory b. Peripheral neuropathy and oral ulcers.
assistance for respiratory depression. C. Foscarnet (Foscavir), ganciclovir (Cytovene), cido-
(2) Tremors. fovir (Vistide), valganciclovir (Valcyte) in treatment
c. Contraindications: of cytomegalovirus (CMV) retinitis in immunosup-
(1) Parkinson’s disease. pressed patients, primarily HIV-positive patients and
(2) Pulmonary insufficiency. transplant recipients.
D. Interferons (IFN): type of natural proteins (cytokines)
INFECTION MANAGEMENT   produced by immune system; can be used to inhibit
Infection management drugs include oral antimicrobials, viral replication in infected cells; treat hepatitis C and
antibiotics, antifungals, antivirals, antituberculins. Dis- multiple sclerosis.
cussion of antibiotic premedication follows this discus- E. Amantadine (Symmetrel) and rimantadine (Fluma-
sion. dine): antivirals that hasten recovery from some influ-
• See Chapter 11, Clinical Treatment: oral antimicro­ enza cases if started early.
bials.
Antifungals
Antivirals Antifungals inhibit or destroy fungal growth.
Antivirals (suffix “-vir”) inhibit replication of viral DNA, • See Chapters 6, General and Oral Pathology; 8, Micro-
which is necessary for viruses to reproduce themselves. biology and Immunology: fungal infections.
These drugs reduce rate of viral growth but will NOT de- A. Nystatin (Mycostatin), amphotericin B (Fungilin),
stroy inactive virus already present. NOT curative, and clotrimazole (Mycelex), ketoconazole (Nizoral), flu-
must be used either prophylactically or early in develop- conazole (Diflucan) (suffix “-zole”):
ment of infection. However, drug therapy must normally 1. Therapeutic uses: candidiasis:
be initiated within 48 hours of onset of infection to pro- a. Cream, lotion, solution to apply to the skin; oral
vide any benefit. Tend to be narrow in spectrum and have suspensions.
limited efficacy. b. Lozenges (troches) to dissolve in the mouth.
• See also Chapter 8, Microbiology and Immunology: for c. Vaginal tablets and vaginal cream.
more information on viruses and related infections. 2. Adverse reactions: troches may contain sugar for
A. Antiherpetic virals: taste compliance, which increases the risk for
1. Pharmacological effects and therapeutic uses: ­caries.
a. Infections with herpesvirus types 1 and 2 and B. Ketoconazole (Nizoral):
varicella-zoster virus (chickenpox, shingles). 1. Adverse reactions (occur MORE often):
b. Should be used at the first signs of an out- a. GIT irritation, MOST common (nausea and
break: vomiting).
Pharmacology    291

b. Hepatotoxicity, MOST serious reaction. 3. Penicillinase-resistant penicillins: cloxacillin


c. Headache, photophobia, allergic reactions. (Tegopen, Cloxapen), methicillin (Staphcillin),
2. Drug interactions: dicloxacillin (Dynapen, Pathocil), still narrow
a. Decreased effects with anticholinergics, H2 spectrum.
blockers, antacids, rifampin (Rifadin, Rimac- a. Therapeutic uses: infections caused by penicil-
tane). linase-producing bacteria.
b. Cardiac toxicity occurs with terfenadine and 4. Extended-spectrum (broad- or wide-spectrum)
­astemizole (Hismanal). penicillins: as ampicillin (Polycillin, Omnipen,
3. Contraindications: hypersensitivity. Totacillin) and amoxicillin (Amoxil, Larotid, Poly-
C. Fluconazole (Diflucan): mox) (suffix “-cillin”).
1. Therapeutic uses: a. Therapeutic uses:
a. Prophylactic for immunocompromised patients (1) Infections caused by gram-positive and
to prevent candidiasis. some gram-negative bacteria (Escherichia
b. Nonresponsive candidal infections; treatment coli and Haemophilus influenzae), but NOT
of vaginal infections. penicillinase-resistant.
o16570 D. Echinocandins (caspofungin [Cancidas]): inhibit syn- (a) Ampicillin (Polycillin, Omnipen, To-
thesis of cell wall glucan (similar to penicillin when tacillin): drug used in the 2007 American
given for bacteria, but for fungi), which can be used Heart Association (AHA) antibiotic pre-
in case of azole-resistant Candida, with long half-life medication regimen for patients unable
and low side effects. to take oral drug (adults 2 g, children
50 mg/kg IM or IV, 30 to 60 minutes be-
Antibiotics fore procedure) to prevent infective en-
Antibiotics prevent or control bacterial growth; they are docarditis (IE) in certain at-risk patients.
seldom dual prescribed. Oral antibiotics decrease ef- (b) Amoxicillin (Amoxil, Larotid, Polymox)
fectiveness of oral contraceptives (birth control pills is used MORE often in dentistry; uses
[BCP]); female dental patients of childbearing age when SIMILAR to those of Penicillin VK; first
prescribed should be asked to use alternative birth con- drug of choice for the 2007 AHA antibi-
trol method until the start of the next menstrual cycle. otic premedication regimen for patients
Also, care must be used when prescribing oral antibiotics able to take oral drug (adults 2 g, children
to patients with history of lower GIT disorders (colitis, 50 mg/kg PO, 30 to 60 minutes before
diverticulitis, inflammatory bowel disorder [IBS]), since procedure) to prevent IE in certain at-
they are MORE likely to develop serious diarrhea. Su- risk patients; multidrug therapy for pep-
perinfections (such as community-associated methicil- tic ulcers caused by Helicobacter pylori
lin-resistant Staphylococcus aureus [CA-MRSA]) and bacteria.
resistant strains are other recent considerations with use, (c) Amoxicillin with clavulanic acid (CA)
since they lower the spectrum of the drug. (See later dis- (Augmentin): CA combines with peni-
cussion of antibiotic premedication.) cillinase so drug is NOT inactivated.
• See Chapter 8, Microbiology and Immunology: drugs b. Adverse reactions:
and related microorganisms. (1) SIMILAR to other antiinfectives: aller-
A. Penicillins: three subgroups: narrow-spectrum, peni- gic reaction, GIT irritation, superinfection
cillinase-resistant, extended-spectrum (broad- or (because of resistance from penicillinase-
wide-spectrum). ­producing bacteria).
1. Pharmacological effects: (2) Candidiasis, glossitis, stomatitis, taste al-
a. Interfere with bacterial wall synthesis. teration, xerostomia, black hairy tongue.
b. Bactericidal. c. Drug interactions:
2. Narrow-spectrum penicillins: penicillin G and (1) Increased anticoagulant effect (especially
penicillin VK (Pen-Vee K, V-Cillin K). with warfarin [Coumadin]).
a. Therapeutic uses (Penicillin VK, MOST fre- (2) Decreased antimicrobial effectiveness with
quently used antibiotic in dentistry; better tetracyclines and erythromycin (Robimy-
oral absorption): cin, E-Mycin, E.E.S.).
(1) Ear, skin, respiratory, and urinary tract in- (3) Increased plasma levels with disulfiram
fections. (Antabuse) for alcoholism and probenecid
(2) Aggressive periodontal disease, periodon- (Benemid) for gout.
tal abscesses, soft tissue infections, osteo­ d. Contraindications: hypersensitivity (range from
myelitis. rash to anaphylaxis; see Chapter 8).
292   Saunders Review of Dental Hygiene

B. Tetracyclines: tetracycline (Achromycin, Sumycin),


Box 9-1  Patients at Potential Increased Risk  
doxycycline (Vibramycin), minocycline (subgingi- of Hematogenous Total Joint Infection
val: Arestin, oral: Minocin) (suffix “-cycline”).
1. Pharmacological effects: All patients during first 2 years after prosthetic joint
a. Inhibits protein synthesis. ­replacement
b. Bacteriostatic. Immunocompromised and immunosuppressed patients:
2. Therapeutic uses: • Inflammatory arthropathies (e.g., rheumatoid arthritis,
a. Sexually transmitted diseases and infections ­systemic lupus erythematosus)
• Drug-induced immunosuppression
(STDs/STIs) such as syphilis, gonorrhea. • Radiation-induced immunosuppression
b. Doxycycline at low-dose (LD) levels: Patients with comorbidities:
(1) Reduced level for antiinflammatory effects • Previous prosthetic joint infections
(reduced activity of collagenase) and NOT • Malnourishment
for antibiotic activity. • Hemophilia
• HIV infection
(2) Used for dermatological lesions such as • Insulin-dependent (type 1) diabetes
acne and rosacea (Oracea) and periodontal
disease (Periostat), since it concentrates in
gingival crevicular fluid.
3. Adverse reactions: 1. Pharmacological effects:
a. Tetracycline causes permanent intrinsic yellow a. SAME as for penicillins: interfering with bacte-
or brown staining of teeth if taken during de- rial cell wall synthesis.
velopmental period involving enamel calcifica- b. Bactericidal.
tion; minocycline produces bluish gray stain in 2. Therapeutic uses:
adults (see below). a. Infections caused by gram-positive, penicillin-
b. Systemic minocycline can cause black pigmen- ase-producing, and some gram-negative bacte-
tation of jaws (including palate); pigment ap- ria such as Salmonella and E. coli; respiratory
pears bluish through mucosal tissues. and urinary tract infection; otitis media.
c. GIT irritation, including nausea, vomiting, diar- b. BOTH cefazolin (Ancef) and ceftriaxone (Ro-
rhea, xerostomia. cephin) can be used for the 2007 AHA antibi-
d. Superinfection. otic premedication regimen if patient is unable
e. Increased risk of liver damage when tetracy- to take oral drug and allergic to penicillin (Pen-
cline is given IV. Vee K) or ampicillin (Polycillin) (adults 1 g,
f. Nephrotoxicity (with older tetracyclines). children 50 mg/kg IM or IV, 30 to 60 minutes
g. Photosensitivity (because of exaggerated re- before procedure) to prevent IE in certain at-
sponse of skin and eyes to sun exposure). risk patients.
4. Drug interactions: c. Cephalexin (Keflex) or other cephalosporin can
a. Older tetracyclines show chelation and thus less be used for the 2007 antibiotic premedication reg-
effectiveness when concomitantly administered imen if patient is allergic to penicillin (Pen-Vee
with dairy products (Ca), mineral supplements K) or ampicillin (Polycillin) (adults 2 g, children
(Fe, Ca, fortified foods), antacids (Ca, Mg, Al). 50 mg/kg PO, 30 to 60 minutes before procedure)
b. LESS pronounced effect with dairy products to prevent IE in certain at-risk patients.
with the newer synthetic forms doxycycline d. Antibiotic premedication (prophylaxis) for pre-
(Vibramycin) and minocycline (Minocin), but vention of hematogenous total joint infection if
AVOID concomitant use of mineral supple- patient has had surgery within past 2 years, is
ments and antacids. ­immunocompromised or immunosuppressed, or
c. Enhanced effect of oral sulfonylureas, which has a medical history of comorbidities (Box 9-1).
results in hypoglycemia. 3. Adverse reactions:
d. Increased effect of anticoagulants. a. SAME oral effects as penicillins.
5. Contraindications: b. Allergic reactions (ranging from rash to ana-
a. Hypersensitivity. phylaxis; see Chapter 8).
b. Should NOT be prescribed for children younger c. Local reactions: localized pain and swelling in
than 8 years or pregnant or nursing women (be- response to IM.
cause of intrinsic stain of developing teeth). d. GIT irritation; includes diarrhea (can be pseu-
C. Cephalosporins: cephalexin (Keflex), cefadroxil (Du- domembranous colitis), nausea, vomiting.
ricef), cefaclor (Ceclor), cefuroxime (Ceftin), cefazo- e. Nephrotoxicity; hemostasis impairment; super-
lin (Ancef), ceftriaxone (Rocephin). infections.
Pharmacology    293

4. Drug interactions: decreased effectiveness with 4. Drug interactions:


tetracycline and erythromycin (Robimycin, a. Increased serum concentrations with di-
E-Mycin, E.E.S.). goxin (Lanoxin, leading to toxicity), warfarin
5. Contraindications: hypersensitivity (including ­(Coumadin, increased bleeding), carbamaze-
cross-sensitivity reaction in patients allergic to pine (Tegretol), cyclosporine (Neoral, Sandim-
penicillin [Pen-Vee K]). mune, Gengraf).
D. Clindamycin (Cleocin): 5. Contraindications: hypersensitivity.
1. Pharmacological effects: inhibit protein synthesis; F. Aminoglycosides (suffix “-mycin”; do NOT confuse
bacteriostatic. with macrolides):
2. Therapeutic uses: 1. Pharmacological effects:
a. Infection by Bacteroides fragilis and gram- a. Inhibit protein synthesis.
­positive organisms. b. Bactericidal.
b. Drug of choice for the 2007 AHA antibiotic pre- 2. Therapeutic uses (MOST given IM/IV):
medication regimen for patients allergic to pen- a. Gentamicin (Garamycin) for gram-negative in-
icillin (Pen-Vee K) or ampicillin (Polycillin), fections that require hospitalization (e.g., bone,
either able to take oral drugs or NOT (adults respiratory tract, urinary tract infections), in-
600 mg, children 20 mg/kg PO, IM, or IV, 30 to cluding methicillin-resistant Staphylococcus
60 minutes before procedure), to prevent IE in aureus (MRSA).
certain at-risk patients. b. Neomycin (Mycifradin) as topical for skin in-
3. Adverse reactions: fections.
a. Allergic reactions, GIT irritation. c. Also includes streptomycin, kanamycin (Kan-
b. Superinfections. trex), tobramycin (eye drops).
4. Drug interactions: 3. Adverse reactions (excessive blood levels): oto-
a. Decreased effectiveness with erythromycin toxicity for cranial nerve VIII (auditory); BOTH
(Robimycin, E-Mycin, E.E.S.). balance and auditory functions affected.
b. Increased effects of nondepolarizing muscle G. Metronidazole (Flagyl):
­relaxants. 1. Pharmacological effects:
5. Contraindications: a. Inhibits protein synthesis.
a. Hypersensitivity. b. Bactericidal.
b. Ulcerative colitis, enteritis (because of GIT ir- 2. Therapeutic uses:
ritation). a. Dentistry: infections caused by anaerobes such
E. Macrolides: erythromycin (Robimycin, E-Mycin, as Bacteroides organisms and aggressive peri-
E.E.S.), clarithromycin (Biaxin), azithromycin (Zith- odontal disease treatment.
romax, Z pack, descending doses) (suffix “-mycin”; b. Medical: intestinal amebiasis, trichomoniasis,
do NOT confuse with aminoglycosides). giardiasis.
1. Pharmacological effects: 3. Adverse reactions:
a. Interfere with protein synthesis; wide distribu- a. Metallic taste, black hairy tongue, xerostomia.
tion, long-lasting effects. b. Effects on the CNS, including dizziness, head-
b. Bacteriostatic. ache, vertigo.
2. Therapeutic uses: c. Nausea, vomiting, diarrhea.
a. Mild to moderate respiratory tract infections, d. Renal toxicity; carcinogenicity in animals but
otitis media, Legionnaire’s disease, STDs. has been discounted.
b. Alternate choice for orofacial infections caused 4. Drug interactions:
by aerobic gram-positive bacteria. a. Disulfiram (Antabuse) reactions occur with
c. BOTH clarithromycin (Biaxin) and azithro- alcohol and alcohol-containing products; anti-
mycin (Zithromax) can be used for the 2007 microbial rinses (e.g., chlorhexidine, Listerine)
AHA antibiotic premedication regimen for that contain alcohol are contraindicated.
patients allergic to penicillin (Pen-Vee K) or b. Decreased effects with phenobarbital (­Luminal).
ampicillin (Polycillin) (adults 500 mg, chil- 5. Contraindications:
dren 15 mg/kg PO, 30 to 60 minutes before a. Hypersensitivity.
procedure) to prevent IE for certain at-risk pa- b. CNS disorders (because of CNS effects).
tients. c. Renal disease (because of renal toxicity).
3. Adverse reactions: H. Sulfonamides (sulfa drugs): sulfisoxazole (Gantrisin)
a. GIT irritation with standard therapeutic doses. and trimethoprim-sulfamethoxazole (TMP-SMX,
b. Allergic reactions are uncommon. Bactrim, Septra).
294   Saunders Review of Dental Hygiene

1. Pharmacological effects: prevent use of PABA to b. Decreased effects of ketoconazole (Nizoral).


make folic acid. 4. Contraindications: hypersensitivity to drug(s).
2. Therapeutic uses: C. Pyrazinamide (PZA): combined with INH (Nydrazid)
a. Drug of choice for treating Pneumocystis jir- and rifampin (Rifadin, Rimactane).
oveci (carinii) pneumonia with acquired immu- D. Ethambutol (Myambutol): with other drugs noted
nodeficiency syndrome (AIDS). above; contraindicated for patients with optic neuritis
b. Urinary tract infection, otitis media. because of optic neuropathy.
3. Adverse reactions: E. Rifapentine (Priftin): taken once a week during last
a. Skin rash, photosensitivity. 4 months of drug therapy.
b. Hematological reactions (e.g., leukopenia,
MANAGEMENT OF SPECIAL PATIENTS  
thrombocytopenia).
4. Contraindications: hypersensitivity. Patients Receiving Antibiotic Premedication
I. Quinolone such as ciprofloxacin (Cipro): The American Heart Association (AHA) has 2007 guide-
1. Pharmacological effects: inhibits bacterial portion. lines on antibiotic premedication or prophylaxis for patients
2. Therapeutic uses: effective against gram-negative at risk, Recommendations for the Prevention of Infectious
bacteria. ­Endocarditis (Box 9-2 and Table 9-6; also see CD-ROM).
3. Adverse reactions: AVOID dental light in eyes. Patients with cardiac conditions associated with highest risk
of adverse outcome from infective endocarditis (IE), such
Antituberculins as patients with artificial heart valves, history of endocardi-
Antituberculins inhibit spread of tuberculosis (TB) in pa- tis, certain serious congenital heart conditions, or hypertro-
tients; usually three or more are given concurrently. May phic cardiomyopathy, as well as heart transplant patients in
be involved in directly observed therapy short-course whom problem with heart valve develops, should receive
(DOTS), in which healthcare professional administers antibiotic premedication (prophylaxis) before invasive den-
medication for compliance. tal procedures (may have wallet card). For patients with his-
• See Chapters 6, General and Oral Pathology, and 8, tory of pathological (organic) heart murmur or documented
­Microbiology and Immunology: tuberculosis. case of such heart murmur, premedicationwith antibiotics is
A. Rifampin (Rifadin, Rimactane): NOT necessary before invasive dental procedures. Specific
1. Therapeutic uses: when administered with other antibiotic coverage has already been discussed.
antituberculosis drugs, reduces development of The American Academy of Orthopaedic Surgeons
antituberculosis drug resistance. (AAOS) and the American Dental Association (ADA)
2. Adverse reactions: published an advisory statement regarding use of ­antibiotic
a. Stomatitis, discolored (red-orange) saliva and premedication (prophylaxis) to prevent ­infection in per-
urine, nausea, vomiting, diarrhea. sons with total joint replacements who are undergoing
b. Thrombocytopenia (low platelet count). dental procedures, Antibiotic Prophylaxis for Dental Pa-
3. Drug interactions: tients with Total Joint Replacements (see CD-ROM and
a. Increased hepatotoxicity with alcohol and acet- Box 9-1). Antibiotic prophylaxis is NOT indicated for
aminophen (Tylenol). dental patients with pins, plates, and screws. Antibiotic
b. Decreased effect of ketoconazole (Nizoral). prophylaxis is NOT routinely indicated for MOST dental
4. Contraindications: hypersensitivity to drug(s). patients with prosthetic joints.
B. Isoniazid (INH) (Nydrazid): Patients who have received total prosthetic (artifi-
1. Therapeutic uses: cial) joints (hips, shoulders, knees) need antibiotic pre-
a. Treatment and prevention. medication before invasive dental procedures ONLY for
b. Can be used alone for prevention or with ri- the 2 years after the joint has been placed or if they
fampin (Rifadin, Rimactane) and pyrazinamide continue to have ­immunocompromised or immunosup-
(PZD) for treatment (see below). pressed tendencies or problem, rejection, or infection in
2. Adverse reactions: joint after placement. Recommendation for prophylaxis
a. Xerostomia. includes those with inflammatory arthropathies such as
b. Effects on the CNS: peripheral neuropathy, rheumatoid arthritis (RA) and systemic lupus erythema-
toxic encephalopathy, convulsions. tosus (SLE), drug- or radiation-induced immunosup-
c. Hepatotoxicity, vitamin B6 deficiency. pression, or comorbidities such as previous prosthetic
d. Hematological effects: hemolytic anemia, joint infections, malnourishment, hemophilia, HIV
thrombocytopenia, agranulocytosis. infection, type 1 insulin-dependent diabetes mellitus
3. Drug interactions: (DM), malignancy (cancer).
a. Increased hepatotoxicity with alcohol or acet- • See Chapter 8, Microbiology and Immunology: infec-
aminophen (Tylenol). tive endocarditis, prosthetic joint infection.
Pharmacology    295

Box 9-2  Antibiotic Premedication (Prophylaxis) for Cardiac Conditions and Dental Procedures to Reduce Risk  
of Infective Endocarditis

CARDIAC CONDITIONS NEEDING ANTIBIOTIC PREMEDICATON


Prosthetic cardiac valve
Previous infective endocarditis
Congenital heart disease only in the following categories:
• Unrepaired cyanotic congenital heart disease, including those with palliative shunts and conduits
• Completely repaired congenital heart disease with prosthetic material or device, whether placed by surgery or catheter intervention,
during first 6 months after procedure
• Repaired congenital heart disease with residual defects at site or adjacent to site of prosthetic patch or device (which inhibits
­endothelialization)
Cardiac transplantation recipients with cardiac valvular diseases, since endothelialization of prosthetic material occurs within 
6 months after procedure

CARDIAC CONDITIONS NOT NEEDING ANTIBIOTIC PREMEDICATION


Heart murmur (functional and pathological)
Bypass graft surgery history
Pacemaker
Mitral valve prolapse with or without insufficiency
Rheumatic heart disease
Bicuspid valve disease
Calcified aortic stenosis
Congenital heart conditions such as ventricular septal defect, atrial septal defect, and hypertrophic cardiomyopathy

dental procedures for which antibiotic prophylaxis is recommended in patients with cardiac
conditions listed above
All dental procedures that involve manipulation of gingival tissue or the periapical region of teeth, or perforation of the oral mucosa

Dental procedures or events for which antibiotic prophylaxis is NOT recommended


Routine anesthetic injections through noninfected tissue; taking dental radiographs; placement of removable prosthodontic or
­orthodontic appliances; adjustment of orthodontic appliances; placement of orthodontic brackets; shedding of primary teeth and
bleeding from trauma to the lips or oral mucosa.

Modified from Wilson W, Taubert KA, Gewitz M, et al: Prevention of infective endocarditis: guidelines from the American Heart Association Rheumatic Fever,
Endocarditis, and Kawasaki Disease Committee, Council on Cardiovascular Disease in the Young, and the Council on Clinical Cardiology, Council on Cardiovascular
Surgery and Anesthesia, and the Quality of Care and Outcomes Research Interdisciplinary Working Group, Circulation 116:1736, 2007.

CLINICAL STUDY  

Age 60 YRS Scenario

Sex ☒  Male    ☐  Female The patient is due to return for a recall exami-
nation and oral prophylaxis appointment. To
Medical History History of infective endocarditis (IE) facilitate the appointment, his chart is reviewed
Allergy to penicillin (Pen-Vee K) before scheduling and he is called to update his
Current Medications digoxin (Lanoxin) 0.5 mg qd medical history. He is taking two drugs now,
OTC loratadine (Claritin) 10 mg prn and he also has additions to his list of cardiac  
complications.
Social History Chef

1. The patient takes digoxin (Lanoxin) for the treatment be an addition that he needs to discuss and add to his
of which type of condition? What type of drug is this? medical history.
2. Does the patient require prophylactic antibiotic treat- 2. Yes, he will be undergoing an invasive dental
ment before his appointment? If so, what antiinfective procedure(s); the 2007 AHA Recommendations for
agent should be prescribed and why? the Prevention of Infectious Endocarditis should be
3. Why is the patient taking loratadine (Claritin)? What followed because of his history of IE, since he is at
type of drug is this? increased risk for IE. Amoxicillin (Amoxil, Larotid,
Polymox) should NOT be used because of his his-
1. Digoxin (Lanoxin) is used to treat congestive heart tory of penicillin (Pen-Vee K) allergy. However,
failure (CHF) and is a cardiac glycoside. This must azithromycin (Zithromax) or clarithromycin (Biaxin),
296   Saunders Review of Dental Hygiene

Table 9-6  American Heart Association recommendations for the prevention of infectious endocarditis: antibiotic  
premedication (prophylactic) regimens recommended for dental procedures

Regimen: single dose 30 to 60 minutes before


procedure

Situation Agent Adults Children

Oral Amoxicillin 2g 50 mg/kg


Unable to take oral medication Ampicillin 2 g IM or IV 50 mg/kg IM or IV
Cefazolin or ceftriaxone 1 g IM or IV 50 mg/kg IM or IV
Allergic to penicillins or ampicillin:   Cephalexin*,† 2g 50 mg/kg
oral regimen Clindamycin 600 mg 20 mg/kg
Azithromycin or   500 mg 15 mg/kg
clarithromycin
Allergic to penicillins or ampicillin and Cefazolin or ceftriaxone† 1 g IM or IV 50 mg/kg IM or IV
unable to take oral medication Clindamycin 600 mg IM or IV 20 mg/kg IM or IV

Modified from Wilson W, Taubert KA, Gewitz M, et al: Prevention of infective endocarditis: guidelines from the American Heart Association Rheumatic Fever,
Endocarditis, and Kawasaki Disease Committee, Council on Cardiovascular Disease in the Young, and the Council on Clinical Cardiology, Council on Cardiovascular
Surgery and Anesthesia, and the Quality of Care and Outcomes Research Interdisciplinary Working Group, Circulation 116:1736, 2007.
IM, Intramuscular; IV, intravenous.
*Or other first- or second-generation oral cephalosporin in equivalent adult or pediatric dosage.
†Cephalosporins should NOT be used in individual with history of anaphylaxis, angioedema, or urticaria with penicillins or ampicillin.

which can usually be used in these patients, should be 2. General therapeutic uses:
avoided for this particular patient because of poten- a. Control of HBP.
tial drug interactions. Azithromycin (Zithromax) and 3. General adverse reactions:
clarithromycin (Biaxin) belong to the macrolide fam- a. Xerostomia.
ily of antibiotics. Digoxin (Lanoxin) in combination b. CNS depression.
with these antibiotics tends to increase drug availabil- c. Orthostatic hypotension.
ity, thereby ­ increasing the risk of digoxin (Lanoxin) 4. Diuretics: first-line intervention for HBP (suffixes
toxicity. Therefore the drug of choice in the AHA regi- “-mide,” “-nide”).
men, clindamycin (Cleocin) (for adults 600 mg PO, 30 a. Thiazide-type diuretics: hydrochlorothiazide
to 60 minutes before procedure) could be prescribed (HCTZ) (Lopressor):
by the supervising dentist, since the patient can take (1) Specific pharmacological effect:
oral drugs. However, a medical consult would be war- (a) Increase secretion of sodium and water.
ranted in this case. (b) Thus lower blood volume and blood
3. Loratadine (Claritin), an H1 receptor antagonist, is pressure.
used to treat rhinitis and seasonal allergies. This must (2) Specific therapeutic uses:
be the other medical condition that he wants to add to (a) Used in combination with other antihy-
his medical history. pertensives (see below).
(3) Specific adverse reactions:
Patients with Cardiovascular Disease (a) Hypokalemia (low blood potassium);
Patient with cardiovascular disease (CVD) has damage thus must have potassium supplement.
or disease of the heart and blood vessels. Includes hy- (b) Hyperglycemia (high blood glucose).
pertension (high blood pressure [HBP]; MOST common (c) Hyperuricemia (excessive uric acid in
CVD), congestive heart failure (CHF), dysrhythmias, an- the body), increased urination.
gina pectoris. (4) Drug interactions:
• See Chapter 6, General and Oral Pathology: CVD; 11, (a) Decrease in hypotensive effect with
Medical and Dental Emergencies: emergency kit drugs NSAIAs.
in dental settings. b. Loop diuretics: furosemide (Lasix).
A. Antihypertensives: (1) Specific pharmacological effects:
1. General pharmacological effect: (a) Inhibit absorption of sodium and chlo-
a. Lower blood pressure by relaxing the blood ves- ride in loop of Henle and distal renal
sels so heart does NOT have to pump as hard. tube.
b. Control chest pain by increasing supply of (b) Thus increase excretion of water and
blood to heart. ­sodium.
Pharmacology    297

(2)  Specific therapeutic uses: (b) Decrease glycogenolysis and glucagon


(a) Also CHF and edema present with secretion; NOT preferred for insulin-
heart, liver, or kidney disease. ­dependent DM.
(b) Impaired kidney function. (3) Drug interactions: HBP and bradycardia
(c) Used if patient does NOT respond to occur when used with epinephrine (need
thiazides. to reduce amounts with local anesthesia
(3) Specific adverse reactions and drug interac- use).
tions: c. Selective (cardioselective) β-blockers: ateno-
(a) SAME as thiazides. lol (Tenormin), metoprolol (Lopressor, Toprol
c. Potassium-sparing diuretics: spironolactone XL):
(Aldactone). (1) Specific pharmacological effects: block
(1) Adverse reactions: MORE β1 receptors at lower doses.
(a) Hyperkalemia (high blood potassium). (2) Therapeutic uses:
(b) Increased urination. (a) NO effect on insulin; preferred for insu-
(c) Tumors in lab animals. lin-dependent DM.
(2) Drug interactions: SAME as thiazides. (b) NO bronchoconstrictor effects; used for
d. Thiazides combined with potassium-sparing patients with HBP along with impaired
­diuretics: Maxzide, Dyazide. pulmonary function such as COPD and
(1) Specific therapeutic uses: reduce incidence asthma.
of hypokalemia. (c) Less likely to interact with vasocon- o16580
5. Adrenergic blockers (see also under earlier discus- strictors in local anesthetics than nonse-
sion of adrenergic blocker): lective β-blockers.
a. β-blockers (suffix “-olol”): (3) Drug interactions: reduced antihypertensive o16590
(1) Specific pharmacological effects: effects with COX inhibitors like ibuprofen
(a) Reduce the workload of the heart by de- (Advil) and naproxen (Aleve).
creasing arterial pressure. d. Centrally acting antiadrenergics stimulate cen-
(b) Thus decrease venous return, decrease tral α-adrenergic receptors: clonidine (Cata-
preload and myocardial oxygen demand. pres) and methyldopa (Aldomet).
(2) Specific therapeutic uses: (1) Specific adverse reactions: sedation.
(a) Also angina pectoris, migraine head- (2) Drug interactions:
ache prophylaxis, MI prophylaxis. (a) Increase CNS depression when used
(3) Specific adverse reactions: with CNS depressants.
(a) Fatigue, sleep disturbances, sexual dys- (b) Decrease hypotensive effect when used
function (impotence). with NSAIAs and sympathomimetics.
(b) If discontinued suddenly, spontaneous e. α1-Selective blockers: prazosin (Minipress, Hy-
rebound HBP. povase).
(4) Drug reactions: 6. Angiotensin-converting enzyme (ACE) inhibitors:
(a) Decrease effects of sulfonylureas (oral captopril (Capoten), enalapril (Vasotec), lisinopril
hypoglycemics). (Zestril, Prinivil) (suffix “-pril”):
(b) Decrease antihypertensive effects with a. Specific pharmacological effects:
NSAIAs, barbiturates, penicillins, ri- (1) Prevent formation of angiotensin II, power-
fampin (Rifadin, Rimactane), salicylates. ful body vasoconstrictor.
(c) Mask tachycardia from hypoglycemia (2) Inhibit breakdown of bradykinin; result
caused by insulin and hypoglycemics. is vasodilation, lowering of blood pres-
(d) Slow metabolism of lidocaine (Xylo- sure.
caine); may lead to toxicity (need to (3) Decrease secretion of aldosterone, which
limit dose). decreases retention of sodium and wa-
b. Nonselective β-blockers: propranolol (Inderal): ter; thus act like diuretic to lower blood
(1) Specific pharmacological effects: block ­pressure.
BOTH β-receptors. b. Specific therapeutic uses: used when first-line
(2) Specific adverse reactions and contraindi- interventions (diuretics, β-blockers) are NOT
cations: effective or are contraindicated.
(a) Bronchospasm; NOT used with patients c. Specific adverse reactions:
who have HBP with impaired pulmonary (1) Neutropenia (abnormal decrease in neutro-
function such as COPD and asthma. phils in blood).
298   Saunders Review of Dental Hygiene

Table 9-7  Drugs that cause gingival hyperplasia 8. Angiotensin II receptor blockers: irbesartan
(Avapro), losartan (Cozaar), valsartan (Diovan)
Generic (suffix “-sartan”).
name Brand name Drug classification a. Specific pharmacological effects (SIMILAR to
Diltiazem Cardizem, Dilacor, Calcium channel ACE inhibitors):
Tiazac blocker (1) Block binding of angiotensin 1 to II.
Nifedipine Procardia, Adalat, Calcium channel (2) Block vasoconstriction and aldosterone-
Nifedical blocker ­secreting effects of angiotensin II, lowering
Verapamil Isoptin, Calan, Calcium channel blood pressure.
Bosoptin blocker b. Specific therapeutic uses:
Phenytoin Dilantin Hydantoin (1) Alternatives to ACE inhibitors because of
fewer side effects.
(2) Reduce kidney damage that can occur
(2) Cough, dysgeusia (altered taste that may with DM; thus better for patients with
lead to caries), xerostomia. DM.
(3) Hypotension, teratogenicity. c. Adverse reactions:
d. Drug interactions: decreased effect with in- (1) Oral lesions.
domethacin (Indocin) and possibly other (2) Nausea, vomiting, dyspepsia.
NSAIAs. (3) Orthostatic hypotension in early stages
7. Calcium channel blockers: diltiazem (Cardizem, of use.
Dilacor, Tiazac), nifedipine (Procardia, Adalat, d. Drug reactions: decrease effects when given
Nifedical), verapamil (Isoptin, Calan, Bosoptin), with antifungal ketoconazole (Nizoral).
amlodipine (Norvasc; with atorvastatin [Lipitor], 9. Vasodilators: minoxidil (Loniten) and hydralazine
Caduet; with valsartan [Diovan], Exforge; with (Apresoline).
benazepril, Lotrel) (suffix “-dipine”). a. Specific pharmacological effects: relax smooth
a. Specific pharmacological effects: muscles of arterioles.
(1) Inhibit calcium ions from entering voltage- b. Adverse reactions: hirsutism (hair growth, re-
sensitive channels with vascular smooth versing baldness).
muscle and myocardium during depolariza- c. Drug interactions: toxicity with sympathetic
tion, relaxing the muscles. blockers, β-blockers, diuretics.
(2) Vasodilate coronary and peripheral arteri- B. Digitalis glycoside: digoxin (Lanoxin):
oles, lowering blood pressure. 1. Pharmacological effects: increases cardiac con-
o06600 (3) Reduce spasm in coronary arteries, pro- tractility and output.
mote vasodilation. 2. Therapeutic uses:
b. Specific therapeutic uses: a. Commonly prescribed.
(1) BEST in patients with asthma, DM, Pel- b. Used in treatment of CHF (inability of the heart
izaeus-Merzbacher disease. to pump efficiently).
(2) Also used for angina pectoris and dysrhyth- 3. Adverse reactions:
mias. a. Sensitive gag reflex, nausea, vomiting.
c. Adverse reactions: b. CNS effects: headache, drowsiness, fatigue,
(1) Hypotension, dizziness, lightheadedness, muscular weakness.
nausea. c. Blurred vision and photophobia; dysrhythmias;
o7590 (2) Gingival hyperplasia (Table 9-7); LESS hypotension.
common with amlodipine (Norvasc). d. Narrow therapeutic index.
o16610 (3) Inhibition of platelet function; increased 4. Drug interactions:
bleeding should be monitored during inva- a. Possible dysrhythmias when used in conjunc-
sive or surgical dental procedures. tion with epinephrine or levonordefrin.
d. Drug interactions: b. Digoxin (Lanoxin) toxicity when used in con-
(1) Increased effects with antifungals, clar- junction with macrolides or tetracyclines.
ithromycin (Biaxin), doxycycline (Vi- 5. Contraindications:
bramycin), erythromycin (Robimycin, a. Hypersensitivity.
E-Mycin, E.E.S.). b. Ventricular fibrillation and tachycardia.
(2) Decreased effects with indomethacin (In- C. Antiarrhythmics: disopyramide (Norpace, Rythmo-
docin), ibuprofen (Advil, Motrin, Pam- dan), lidocaine (Xylocaine; used parenterally), pro-
prin), possibly other NSAIAs. pranolol (Inderal):
Pharmacology    299

1. Pharmacological effects: suppression of dysrhyth- (1) Headaches, flatulence (excess gas in GIT),
mias by various mechanisms, depending on drug abdominal pain, constipation, diarrhea.
classification. (2) May have myalgia (muscle soreness) that o8240
2. Therapeutic uses: treat cardiac dysrhythmias (al- would affect oral care; adjustments in pa-
tered rhythmic contractions of cardiac muscle). tient positioning in dental chair and/or
D. Antianginals: nitroglycerin (NTG, Nitrostat), iso- shorter appointments.
sorbide dinitrate (Isordil), β-blockers (see anti- (3) Elevated liver enzyme levels (monitored).
hypertensives), calcium channel blockers (see d. Drug interactions:
antihypertensives). (1) Myalgia (diffuse muscle pain) and myo-
1. Pharmacological effects: reduction of oxygen con- sitis (muscle tissue inflammation), lead-
sumption, which decreases work of heart and thus ing to rhabdomyolysis (muscle weakness
relieves anginal pain. and deterioration) with acute renal failure
2. Therapeutic uses: (dose-related) with macrolide antibiot-
a. Treat angina pectoris, caused by insufficient ox- ics (erythromycin [Robimycin, E-Mycin,
ygen supply to myocardium, resulting in chest E.E.S.], clarithromycin [Biaxin]), azole
pain. antifungals (itraconazole, ketoconazole),
b. Patient’s own supply should be brought to each or combined with other antihyperlipid-
dental appointment but may NOT be current. emics or large quantities of grapefruit juice
c. Thus NTG (metered spray or tablet form) should (guess it does not pay to want to watch your
be part of dental office emergency kit (do NOT ­waistline!).
refrigerate). (2) Increased effects of levothyroxine (Syn-
3. Adverse reactions to NTG: throid), digoxin (Lanoxin), ethinyl estra-
a. Xerostomia, headache. diol; AVOID taking with alcohol.
b. Orthostatic hypotension. 3. Ezetimibe (Zetia); in combination with simvas-
4. Drug interactions: increased hypotensive effects with tatin: Vytorin:
alcohol, opioids, benzodiazepines, phenothiazines, a. Pharmacological effects:
erectile dysfunction (ED) drugs (discussed later). (1) Inhibits absorption of cholesterol within
5. Contraindications: hypersensitivity to NTG (Ni- small intestines.
trostat) or nitrites. (2) Reduces dietary cholesterol, resulting in
o8090 E. Antihyperlipidemics (hypolipidemics, lipid-lowering decrease in hepatic cholesterol stores.
drugs [LLD]): MOST common type of drug pre- (3) Increases cholesterol clearance from blood,
scribed in recent years: causing reductions in LDL, total levels and
1. General pharmacological effects: triglycerides, increases in HDL.
o8110 a. Result in lowered plasma cholesterol during b. Therapeutic uses: high cholesterol levels.
treatment of hyperlipidemias. c. Adverse reactions: SAME as simvastatin.
b. Reduce arteriosclerosis (plaque stabilization), 4. Fibrates: bezafibrate (Bezalip), ciprofibrate (Mo-
improve coronary endothelial function, inhibit dalim), gemfibrozil (Lopid), fenofibrate (TriCor).
platelet thrombus formation. a. Specific pharmacological effects:
2. Statins (HMG-CoA reductase inhibitors): lovas- (1) Class of amphipathic carboxylic acids.
tatin (Mevacor, Altocor), atorvastatin (Lipitor), (2) Increase desired high-density lipoproteins
simvastatin (Zocor) (suffix “-statin”). (HDL); combined therapy with statins.
a. Specific pharmacological effects: b. Adverse reactions and drug interactions: SAME
(1) Inhibit enzyme HMG Co-A reductase in liver, as statins.
rate-limiting step in cholesterol synthesis. 5. Niacin (vitamin B3):
(2) Decrease synthesis of cholesterol. a. Specific pharmacological effects: blocks break-
(3) Clearance of harmful low-density lipopro- down of fats (decreasing free fatty acids in the
teins (LDL) and triglycerides. blood).
(4) Increase plasma levels of desired high- b. Adverse reactions: facial flushing (decreased
­density lipoproteins (HDL) in some patients. with NSAIAs).
b. Specific therapeutic uses (drug of choice): F. Antithrombotics include anticoagulants and antiplate-
(1) Lipitor: primary CVD: risk of MI and CVA, lets. American Dental Association has stated that these
multiple risk factors (HBP and/or smoking, rarely need to be discontinued before MOST dental
low HDL), type 2 DM. procedures to prevent adverse reactions of bleeding
(2) Zocor: secondary CVD. (see below); there is a greater risk of thromboem-
c. Adverse reactions: bolic events than uncontrollable bleeding if drugs are
300   Saunders Review of Dental Hygiene

t­emporarily stopped. If risks of drug are too great, a. Pharmacological effects:


medical consult is needed. (1) Inhibits platelet aggregation.
1. Anticoagulants: warfarin (Coumadin): (2) Thus reduce risk of thrombosis and embo-
a. Pharmacological effects: lism formation.
(1) Interfere with blood clotting factors (those b. Therapeutic uses:
dependent on vitamin K). (1) Clopidogrel (Plavix) and ticlopidine (Ti-
(2) Thus reduce risk of thrombus and embo- clid) are MORE effective than low-dose
lism formation. aspirin compounds at preventing MI and
b. Therapeutic uses: CVA in patients at risk.
(1) Post-MI treatment. (2) Clopidogrel (Plavix) used for those
(2) Treatment of pulmonary emboli (blood unable to tolerate GIT effects of aspi-
clots in the lung). rin; has replaced ticlopidine (Ticlid) for
(3) Treatment of thrombophlebitis (vein in- those who are allergic to or intolerant of
flammation). aspirin.
(4) Treatment of atrial dysrhythmias. (3) Adverse reactions: SAME as for warfarin
c. Adverse reactions: (Coumadin) and aspirin, but lower GIT ef-
(1) Gingival bleeding and hemorrhage; can be fects; see earlier notes.
serious problem during dental work, es- c. Drug interactions:
pecially with surgical procedures (extrac- (1) Toxicity with some herbal supplements and
tions); however, see note above. NSAIAs.
(2) Monitored by international normalized ra- (2) Increased effects also with aspirin (Bayer,
tio (INR); INRs ≤2.5 are safe for invasive Ecotrin, Empirin, Bufferin), with other
dental work; older test is prothrombin time herbal supplementals, and with other
(PT). ­NSAIAs, antiplatelets, anticoagulants.
d. Drug interactions: (3) Decreased effects of atorvastatin (Lipi-
(1) Increased effects with salicylates, metroni- tor) and macrolides (erythromycin [(Ro-
dazole (Flagyl), erythromycin (Robimycin, bimycin, E-Mycin, E.E.S.], clarithromycin
E-Mycin, E.E.S.), NSAIAs. [­Biaxin]).
(2) Decreased effects with barbiturates and G. Erectile dysfunction (ED) drugs: sildenafil citrate
­vitamin K. (­Viagra, Revatio), tadalafil (Cialis), vardenafil
e. Contraindications: blood disorders. (­Levitra), ALL resulting in increased inflow of blood;
2. Antiplatelets: clopidogrel (Plavix), ticlopidine (Ti- ­concurrent use of NTG (Nitrostat) or amyl nitrate
clid), aspirin (Bayer, Ecotrin, Empirin, Bufferin) would lead to serious hypotension, so monitor BP
(also low-dose compounds, see earlier discus- during dental appointment; may have hearing loss or
sion): vision impairment with chronic use.

clinical study  

Age 29 YRS SCENARIO

Sex ☐  Male    ☒  Female A recall patient apologizes for missing her last
6-month recall appointment. Today at this
Height 5’9” ­appointment, her mandibular anterior gingiva
is visibly hemorrhagic.
Weight 205 LBS (previous 180 LBS)

BP 148/98 (previous 120/76)

Chief Complaint “My gums bleed when I brush them.”

Medical History Took dexfenfluramine (Redux) in past


Artificial mitral valve placed 4 months ago

Current Medications None

Social History Party planner


Pharmacology    301

1. What type of drug is dexfenfluramine (Redux), and • See Chapters 6, General and Oral Pathology; 16, Spe-
why was it prescribed? cial Needs Patient Care: discussion on specific mental
2. What might explain the change in the patient’s blood illnesses.
pressure readings? Why did the drug cause her heart A. Antipsychotics:
valve to be replaced? 1. Phenothiazines: chlorpromazine (Thorazine), me-
3. Should the patient receive dental treatment at this re- soridazine (Serentil), prochlorperazine (Compa-
call appointment? zine), thioridazine (Mellaril).
2. Phenylbutylpiperadines: haloperidol (Haldol) and
1. Dexfenfluramine (Redux) is an anorexiant. It suppresses pimozide (Orap).
appetite and was prescribed as an aid to weight loss. 3. Dibenzapine derivatives: clozapine (Clozaril),
2. Although one adverse effect of dexfenfluramine (Redux) loxapine (Loxitane), olanzapine (Zyprexa), and
is hypertension (high blood pressure [HBP]), this effect quetiapine (Seroquel).
usually disappears after the drug is discontinued. Mod- 4. Benzisoxidil group: risperidone (Risperdal) and
erate weight gain after discontinuing drug might be her ziprasidone (Geodon).
cause of HBP. A more serious cause might be primary 5. Pharmacological effects:
pulmonary HBP, which is a side effect of the drug and is a. Calm emotions and slow down psychomotor
associated with dyspnea, angina, syncope, edema of the responses.
lower extremities, and possible congestive heart fail- b. Act as antiemetics (antivomiting drugs) by de-
ure (CHF). Her diastolic reading is considered stage 1 pressing chemoreceptor zone; mainly prochlor-
(mild) HBP. Fenfluramine (Pondimin) and dexfenflura- perazine.
mine (Redux) in combination with phentermine (com- 6. Therapeutic uses:
monly referred to as phen-fen) were both removed from a. Treatment of psychoses (e.g., schizophrenia,
the market because of their adverse effects on the heart obsessive-compulsive disorder [OCD], bipolar
valves when taken long term, causing disease. In the disorder [manic-depressive illness]).
case of valve disease associated with fenfluramine and b. Preoperative relaxation; nausea and vomiting.
dexfenfluramine, leakiness is problem; valvular damage 7. Adverse reactions:
may ultimately produce severe heart and lung disease. a. Orthostatic hypotension and tachycardia.
Drug caused so much permanent damage to her mitral b. Sedation (tolerance develops).
valve that it needed to be replaced with artificial valve. c. Extrapyramidal effects (body movements) such
3. She should not receive invasive dental treatment (in- as Parkinson-like symptoms, e.g., tremors, ri-
cluding oral prophylaxis) at this time because of pos- gidity, akathisia (increased motor restlessness);
sibility of significant oral bleeding. This is because of acute dystonia: muscle spasms of the face,
her artificial heart valve, as well as having had cardiac tongue, back, and neck; tardive dyskinesia: in-
surgery within the last 6 months. Patient must be pre- voluntary, abnormal movement of tongue, lips,
medicated with antibiotics before any invasive dental face, and neck; seizures (because of lowered
procedure according to 2007 American Heart Asso- seizure threshold).
ciation (AHA) ­ Recommendations for the Prevention 8. Drug interactions:
of Infective Endocarditis (IE) (see recommendations a. Additive effect with CNS depressants, includ-
below). Patient should be also immediately referred ing barbiturates, alcohol, general anesthetics,
by the dentist to her cardiologist because of her HBP opioid analgesics.
reading, especially in consideration of her complex b. Hypotension and tachycardia with epinephrine
medical history and signs of oral inflammation. IV; epinephrine can be safely administered as a
vasoconstrictor in local anesthetic.
Patients with Mental Illness c. Increased anticholinergic effects (xerostomia)
p0290 Patients with mental illness or disability include those with anticholinergics.
who are treated for mental disorders that affect ability to d. Increased photosensitivity with tetracycline.
recognize reality. MOST medications for these disabili- 9. Contraindications: hypersensitivity.
ties cause xerostomia and taste alterations, which must B. First-generation antidepressants, TCAs: amitriptyline
be taken into account to reduce risk of caries; bruxism (Elavil), nortriptyline (Pamelor, Aventyl), imipramine
(grinding) may also be present. In some cases, antide- (Tofranil) (suffix “-triptyline”).
pressants may be associated with worsening symptoms of 1. Pharmacological effects (onset of action may take
depression or suicidal thoughts or behavior, particularly several weeks):
early in treatment or when there is a change in dosage; be a. Mild sedation and fatigue in normal patients.
sure to refer patient to healthcare providers if there are b. Elevation of mood and decreased depression in
any of these signs. depressed patients.
302   Saunders Review of Dental Hygiene

2. Therapeutic uses: a. Fluoxetine (Prozac): does NOT have adverse


a. Treatment of depression. interactions with vasoconstrictors.
b. Combination with phenothiazines. b. Trazodone (Desyrel, Molipaxin, Trittico, Thom-
c. Nortriptyline is prescribed if less sedation is bran, Trialodine): CNS stimulation, NOT de-
­desired. pression; nausea, diarrhea, aphthous sto­­matitis.
3. Adverse reactions: c. Bupropion (Wellbutrin, Zyban: tobacco ces-
a. Drowsiness (tolerance may develop), ­confusion, sation): increases seizures; xerostomia, vivid
sometimes tremors. dreams (does develop tolerance).
b. Blurred vision and constipation (tolerance does d. Sertraline (Zoloft, Lustral): dizziness and nau-
develop). sea; may increase effects of certain drugs.
o9120 c. Avoid with narrow-angle glaucoma, enlarged e. Paroxetine (Paxil): libido difficulties, anticho-
prostate (benign prostatic hyperplasia), or cer- linergic effects.
tain types of heart disease (HBP, dysrhythmias, f. Escitalopram (Lexapro): used for MAJOR
MI, since higher risk of heart attack). de­press­ive disorders and generalized anxiety
o16610 d. May affect blood sugar levels; with DM, check disor­ders.
blood sugar levels more often. D. Other antidepressants:
4. Drug interactions: 1. Monoamine oxidase inhibitors (MAIOs): moclobe-
a. Increased anticholinergic effects with antihista- mide (Aurorix, Manerix), phenelzine (Nardil), iso-
mines and phenothiazines. carboxazid (Marplan).
o9150 b. Increased effects of epinephrine and CNS a. Pharmacological effects:
­depressants; used with caution if history of sei- (1) Brain’s three neurotransmitters, known as
zures or thyroid problems. monoamines (serotonin, norepinephrine,
C. Second-generation antidepressants, selective sero- dopamine), are broken down by mono-
tonin reuptake inhibitors (SSRIs) (suffix “-ine”): amine oxidase, a liver and brain enzyme.
1. General pharmacological effects: (2) The MAIOs inhibit the activity of mono-
a. Increase extracellular neurotransmitter sero- amine oxidase, thus preventing breakdown
tonin by inhibiting its reuptake into presynaptic of monoamine neurotransmitters and in-
cell. creasing availability to ease depression.
b. Increase serotonin available to bind to the post- b. Therapeutic uses: powerful older antide­
synaptic receptor. pressant.
c. Result in elevated mood, improved sleep, sati- (1) Reserved as last line of defense (because of
ety after eating. potentially lethal dietary and drug interac-
2. General therapeutic uses: tions) when antidepressant drugs (e.g., SSRIs
a. Used in the treatment of depression, anxiety and TCAs) have been tried unsuccessfully.
disorders (social anxiety, panic attacks, OCD, (2) Patch: selegiline (Emsam) applied transder-
eating disorders), chronic pain (acute temporo- mally; drug does NOT enter GIT, decreas-
mandibular disorder [TMD]). ing dangers of dietary interactions.
b. Personality disorders; also premature ejacula- c. Adverse reactions:
tion problems. (1) React with many foods containing tyramine
3. Adverse reactions: (e.g., aged cheeses, fish, wines, smoked
a. Fewer CVD and CNS problems than with meats).
TCAs. (2) Nausea and vomiting, HBP, fever.
b. Must be tapered off when discontinued. (3) NOT for patients with CVD, epilepsy, bron-
4. Drug interactions: chitis, asthma, or HBP or those who resist
a. Contraindicated with concomitant use of following stringent diet.
MAOIs (serotonin disorder) or other CNS an- d. Drug reactions:
tidepressants such as alcohol. (1) The SSRIs (serotonin disorder), TCAs, di-
b. Increased bleeding complications with sulfiram, β-blockers (life-threatening reac-
NSAIAs. tions).
5. Contraindications: (2) AVOID use of levonordefrin (vasoconstric-
a. Liver impairment, pregnancy. tor with mepivacaine local anesthetic agent,
b. Box warning for suicide risk in children and Carbocaine with Neo-Cobefrin).
adolescents is required. 2. Lithium (Lithobid, Eskalith): treats bipolar disor-
6. Specific drugs’ pharmacological effects and thera- der (manic-depressive illness); works by decreas-
peutic uses: ing abnormal activity in brain; toxicity can be
Pharmacology    303

avoided if blood levels are monitored; NSAIAs a. Hepatotoxicity, increased bleeding time; bleed-
increase drug’s toxicity. ing levels monitored by international normalized
3. Venlafaxine (Effexor): bicyclic antidepressant; ratio (INR); INRs ≤2.5 are safe for invasive den-
NO adverse interactions with vasoconstrictors; tal work; older test is prothrombin time (PT).
box warning for suicide risk. b. Constipation.
4. Nefazodone (Serzone): NO adverse interactions 2. Drug interactions:
with vasoconstrictors; serious and possibly fatal a. Additive effect with CNS depressants.
reactions with use of MAOIs. b. Increased bleeding when taken with salicylates
and NSAIAs.
Patients with Seizure Disorders D. Phenobarbital (Luminal): used as anticonvulsant
Patients who suffer from seizure disorders such as epilepsy agent by itself or in combination with other anticon-
are treated with anticonvulsants (antiepileptics [AEDs]) vulsants (e.g., phenytoin [Dilantin]).
to depress the CNS, preventing seizures. ­Adverse reac- E. Benzodiazepines (BZDs): clonazepam (Klonopin)
tions include CNS depression and stimulation (in young and diazepam (Valium); diazepam (Valium) used to
and elderly individuals), xerostomia, gingival hyperpla- treat recurrent tonic-clonic seizures and untreated sta-
sia, nausea and vomiting, teratogenicity, gingival hyper- tis epilepticus (see earlier discussion on BZDs).
plasia (Table 9-7).
• See Chapters 6, General and Oral Pathology: seizures, Patients with Endocrine Disorders
epilepsy, gingival hyperplasia; 13, Periodontology: Endocrine disorders include adrenal gland disorders, diabetes
surgery for gingival hyperplasia; 16, Special Needs mellitus (DM), thyroid disorders, female hormonal disorders.
Patient Care: related discussion on specific seizure • See Chapters 2, Anatomy, Biochemistry, and Physiol-
­disorders. ogy; 6, General and Oral Pathology: related ­discussion
A. Phenytoin (Dilantin): on specific endocrine disorders.
1. Adverse reactions: A. Adrenocorticosteroids (corticosteroids):
o16620 a. Gingival hyperplasia (Table 9-7). 1. Types:
b. Vitamin D and folate deficiencies. a. Glucocorticoids: hydrocortisone, prednisone
c. Rashes; can cause Stevens-Johnson syn- (Deltasone), triamcinolone (Kenalog, Kenacort,
drome. Aristocort, Atolone), dexamethasone (Decad-
2. Drug interactions: ron); used MORE often.
a. Decreased effects with barbiturates and carba- b. Mineralocorticoids, which affect water and
mazepine (Tegretol). electrolyte balance (hydrocortisone has SAME
b. Decreases effects of steroids and doxycycline effect) (suffixes “-sone,” “-one,” “-ide”).
(Vibramycin). 2. Pharmacological effects:
B. Carbamazepine (Tegretol): a. Affect carbohydrate metabolism, which de-
1. Other pharmacological effects: creases inflammatory response.
a. Anticholinergic, antiarrhythmic, antineuralgic, b. Allergic reactions, immunosuppression.
antidiuretic, muscle relaxant. c. Osteoporosis (thinning bone).
2. Therapeutic uses: also treats trigeminal neuralgia 3. Therapeutic uses:
(tic douloureux). a. Replacement drug therapy for Addison’s dis-
3. Adverse reactions: ease (adrenal gland insufficiency).
a. Glossitis, constipation. b. Cushing’s syndrome (hyperadrenocorticism)
b. Aplastic anemia, thrombocytopenia, leukocyto- with resulting “moon face,” “buffalo hump,”
sis; lab monitoring of blood is necessary. truncal obesity.
c. Rashes, photosensitivity, erythema multiforme, c. Antiinflammatory effects treat dermatitis, acute
abnormal liver function. asthma, systemic lupus erythematosus (SLE),
4. Drug interactions: scleroderma, rheumatoid arthritis (RA), severe
a. Decreased effect of phenobarbital (Luminal), and acute allergic reactions, TMD.
benzodiazepines, doxycycline (Vibramycin), d. Used to treat emergencies such as shock and
warfarin (Coumadin), oral contraceptives. adrenal crisis; dental patient may need MORE
b. Increased effect with erythromycin (Robimy- steroids: medical consult needed.
cin, E-Mycin, E.E.S.), isoniazid (Nydrazid), e. Used to treat aphthous stomatitis and noninfec-
propoxyphene (Darvon), calcium channel tious inflammatory oral lesions (e.g., erythema
blockers, cimetidine (Tagamet). multiforme, lichen planus, pemphigus).
C. Valproic acid (VPA, Depakene, Depakote). 4. Adverse reactions:
1. Adverse reactions: a. Decreased resistance to infections.
304   Saunders Review of Dental Hygiene

b. Delayed wound healing. with glipizide (Metaglip), with ­ glyburide (Glu-


c. Development of osteoporosis. covance), with rosiglitazone (Avandamet), with
d. Adrenal crisis (including hypertension, circu- pioglitazone (Actoplus Met); meglitinide class
latory collapse, and death) as a result of abrupt (repaglinide (Prandin), nateglinide (Starlix).
drug withdrawal and stress; may be avoided d. Exenatide (Byetta): mimics GLP-1 incretin, in- o10270
by premedication with additional steroids. sulin secretagogue with glucoregulatory effects;
e. Weight gain and hyperglycemia; increased used in monotherapy or in combination, injected
stomach acid. with prefilled pen (from ugly Gila monster spit—
f. Behavioral changes, including agitation, eu- who would believe it!); unlike sulfonylureas and
phoria, depression. meglitinides, increases insulin synthesis and se-
5. Drug interactions: cretion in presence of glucose only, lessening hy-
a. Decreased effects with barbiturates and ri- poglycemia risk and causing significant weight
fampin (Rifadin, Rimactane). loss; being used to treat insulin resistance.
b. Increased adverse reactions with alcohol, sa- e. Sitagliptin (Januvia): class of dipeptidyl pep- o16630
licylates, NSAIAs. tidase–IV (DPP-4) enzyme inhibitors that de-
6. Contraindications: grade GLP-1 and GIP incretin hormones, which
a. Psychosis because of effects on behavior during stimulate insulin release in response to increased
presence or withdrawal of the drug. blood glucose levels following meals to enhance
b. Infection because of antiinflammatory effects glycemic control; indicated for type 2 DM as
may mask symptoms of infection; also may de- monotherapy or in combination; lower side ef-
crease resistance to infection. fects (e.g., less hypoglycemia, less weight gain)
c. Peptic ulcers because of stimulation of gastric in control of blood glucose values.
acid secretions. 3. Pharmacological effects:
d. CVD because of HBP associated with miner- a. Reduce hepatic glucose production, decrease
alocorticoids. intestinal absorption of glucose.
e. Do NOT use topical preparations on open b. Improve insulin receptor sensitivity, improving
wounds, burns, or large areas; dermal absorp- glucose utilization in the periphery.
tion from use in these areas may approach c. Metformin also reduces LDL and raises HDL;
SAME effects as oral dose. useful for patients with CVD.
B. Drugs for the treatment of DM: 4. Adverse reactions:
1. For type 1 DM: insulin (MOST given SC/SQ: a. Hypoglycemia (when insulin overdose results in
thigh, abdomen, upper arm; rotating sites): insulin shock); especially noted with combination
a. Rapid acting: Humulin R. therapy; meglitinide less than for sulfonylureas.
b. Intermediate acting: Humulin L and Humulin N b. Hyperglycemia (insufficient insulin); less com-
(NPH). mon, treated in the hospital.
c. Long acting: Humulin U. c. GIT complications.
d. Mixed: Humulin 70/30; MOST common com- 5. Drug interactions:
bination is Humulin R and NPH. a. Increased hypoglycemia with salicylates,
(1) Insulin pumps: external for certain patients NSAIAs, alcohol.
who need intensive insulin therapy, sy- b. Hyperglycemia with corticosteroids and epi-
ringe filled with a predetermined amount of nephrine.
short-acting insulin, plastic cannula, needle 6. Contraindications:
and pump that periodically deliver the de- a. Hepatic and/or renal disease.
sired amount. b. Cardiac and/or respiratory insufficiencies.
2. For type 2 DM: oral hypoglycemics (suffixes c. Alcohol abuse, severe infections, pregnancy.
“-ide,” “-gliatazone”): C. Thyroid gland replacements: levothyroxine (Syn-
a. First-generation: sulfonylureas class: tolbuta- throid, Levoxyl, Levothroid, Unithroid) and liotrix
mide (Orinase), chlorpropamide (Diabinese). (Euthroid, Thyrolar).
b. Second-generation: sulfonylureas class: glybu- 1. Pharmacological effects:
ride (DiaBeta, Micronase), glipizide (Glucotrol). a. Maintain the function of organ systems.
c. Third-generation: biguanides class: metformin b. Regulate metabolism and control energy use.
(Glucophage, Fortamer, Riomet); α-­gluocosidase 2. Therapeutic uses:
inhibitors class: acarbose (Precose), miglitol (Gly- a. Hypothyroidism: hypofunction that requires
set); thiazolididinediones (TZD) class: pio­glitazone thyroid replacement therapy for treatment; sup-
(Actos), rosiglitazone (Avandia) and metformin presses thyroid-stimulating hormone (TSH).
Pharmacology    305

3. Drug interactions: c. Adverse reactions:


a. Increased sensitivity to opioids and sedatives. (1) Nausea and vomiting (tolerance does
b. Increased effects of anticoagulants (warfarin ­develop).
[Coumadin]). (2) Edema and weight gain.
c. Toxicity with TCAs. (3) HBP; increased with tobacco use.
d. Decreased absorption with antacids, sucralfate (4) Increased incidence of dry socket with
(Carafate). ­extraction, increased crevicular gingival
e. Decreased serum levels with antiseizure drugs fluid and gingivitis levels.
and rifampin (Rifadin, Rimactane). (5) Uterine bleeding and thrombophlebitis
f. Decreased effects of sulfonylureas. (inflammation of vein); increased with to-
D. Antithyroid drugs: methimazole (Tapazole), iodide, bacco use.
radioactive iodine; used to treat hyperthyroidism d. Drug interactions: decreased effectiveness of
(such as Graves’ disease); hyperfunction results from BCP when taking antibiotics.
excess hormones circulating in the blood (thyrotoxi- 2. Estrogens (synthetic: Premarin, Estraderm), oral
cosis, thyroid storm, emergency condition). tablets, creams, transdermal patches; includes es-
E. Female sex hormones: tradiol.
1. Progesterones: synthetic, medroxyprogesterone a. Pharmacological effects:
(Provera, Depo-Provera) oral and parenteral, levo- (1) Female sex characteristics, puberty changes,
norgestrel (Norplant) implants. reproductive development, preparation for
a. Pharmacological effects: prepares uterus for conception.
implantation after fertilization. (2) Increased fat deposition, salt and water re-
b. Therapeutic uses: tention, bone cell activity (osteoblasts).
(1) Contraception (birth control pill [BCP]); b. Therapeutic uses:
occurs by inhibition of ovulation and thus (1) Contraception (see progesterone discussion
pregnancy. above) with BCP.
(a) Inhibits release of follicle-stimulating (2) Treatment of menstrual disturbances (e.g.,
hormone (FSH) and luteinizing hor- dysmenorrhea, dysfunctional uterine bleed-
mone (LH). ing).
(b) Combination with estrogen or alone. (3) Short-term treatment of acute symptoms of
(2) Treatment of endometriosis, dysmen- menopause (hot flashes, sleep disturbances)
orrhea, dysfunctional uterine bleeding, from a depletion of estrogen.
­premenstrual tension. c. Adverse reactions: SIMILAR to progesterone.
(3) Treatment of postmenopausal symptoms d. Drug interactions: increase effects of cortico-
(in combination with estrogen). steroids.

clinical study  

Age 34 YRS SCENARIO

Sex ☐  Male    ☒  Female The patient is scheduled for four appoint-


ments for quadrant scaling of nonsurgical
Height 5’8” periodontal therapy after diagnosis of
generalized moderate chronic periodon-
Weight 110 LBS (previous 135 LBS)
titis based on the AAP classification of
BP 130/95 (taken at physician’s office) periodontal disease. These procedures will
require the administration of local anesthe-
Chief Complaint “So that is my medical history! When are we go- sia. Patient calls to make the appointments
ing to start with the therapy?” and provide an update of her medical
history.
Medical History Recent diagnosis of Graves’ disease, poorly con-
trolled
Seasonal allergies
GIT distress

Current Medications methimazole (Tapazole) 40 mg tid  


famotidine (Pepcid) 20 mg qd

Social History Ballroom dancer


306   Saunders Review of Dental Hygiene

1. What gland in the body does Graves’ disease involve, B. Adverse reactions:
and what are some common symptoms of the dis- 1. GIT complications: difficulty swallowing, erosion
ease? and inflammation of esophagus, gastric ulcer.
2. Why does the patient take methimazole (Tapazole) 2. Hypocalcemia.
and famotidine (Pepcid)? 3. Rare: jaw osteonecrosis (damage to blood sup-
3. Are there any changes in dental treatment for this pa- ply causes bone to die), especially noted with in-
tient in regard to local anesthetic? jectables in cancer patients; risk is considerably
lower in people who take them orally for osteo-
1. Graves’ disease is a thyroid disorder that causes hy- porosis.
perthyroidism; more common in females and diag- 4. Forteo may cause impaired thinking and reactions;
nosed between 30 and 40 years of age. Symptoms avoid smoking.
include enlarged thyroid gland (goiter), exophthalmos 5. Evista listed in pregnancy category “X” for terato-
(forwardly displaced eyes), hyperactivity, weight loss, genicity.
tachycardia. C. Drug interactions: calcium, antacids, and iron can
2. Methimazole (Tapazole) is an antithyroid agent that ­reduce absorption.
is used to suppress hyperactivity of the thyroid gland
that is present with Graves’ disease. Famotidine (Pep- Allergic Patients
cid) is an H2-receptor antagonist that treats GIT dis- Patients who suffer from allergies use drugs to control
orders such as heartburn and acid indigestion; acts by symptoms. See later discussion of respiratory drugs.
decreasing gastric secretions. • See Chapter 6, General and Oral Pathology, and Chap-
3. Changes in dental treatment include avoidance of epi- ter 8, Microbiology and Immunology: allergic reac-
nephrine in local anesthetic until hyperthyroidism of tions; 11, Medical and Dental Emergencies: allergic
Graves’ disease is controlled, since drug may cause emergencies in dental setting.
thyrotoxicosis, thyroid storm. Thyroid storm can be A. First-generation H1-receptor antagonists: diphen-
fatal and associated with fever, tachycardia, arrhyth- hydramine (Benadryl), clemastine (Tavist), dimen-
mia, hypermetabolism, and other serious complica- hydrinate (Dramamine), doxylamine (Unisom),
tions. Treatment may have to wait until the patient has chlorpheniramine (Chlor-Trimeton, Piriton), hy-
moved from ASA IV status to ASA III and able to have droxyzine hydrochloride (Atarax). Diphenhydramine
elective dental treatment. Medical consult would con- is part of dental office emergency kit.
firm this need to wait. 1. Pharmacological effects:
a. Decreased vasodilation, bronchodilation, pain,
Osteoporotic Patients itching (antihistamine effects).
Postmenopausal osteoporosis is prevented and treated b. Antiemesis, xerostomia, sedation (stimulation
with bisphosphonates (suffix “-dronate”). Also used to can occur).
treat bone loss that results from glucocorticoids (pred- c. Diphenhydramine provides local anesthetic ­effect.
nisone, cortisone) therapy and Paget’s disease. Those 2. Therapeutic uses:
given orally include alendronate (Fosamax), risedro- a. Control of symptoms of allergic reactions; how-
nate (Actonel); those given by IV (injectables): iban- ever, if anaphylaxis occurs, epinephrine rather
dronate (Boniva), pamidronate (Aredia), and zoledronic than antihistamine should be used for broncho-
acid (Zometa). Orals are taken on empty stomach with dilation.
water ONLY; injectables are for those who CANNOT b. OTC sleep aids (e.g., diphenhydramine in
sit upright. May be combined with vitamin D and/or Nytol, Phendry, Compoz, Sleep-Eze, Sominex,
calcium supplementation. Teriparatide (Forteo) and Unisom).
raloxifene (Evista) offer alternatives to bisphosphonate c. Preoperative sedation; may also be used as anti-
therapy. emetics.
• See Chapter 12, Clinical Treatment: menopausal and 3. Adverse reactions:
geriatric patient. a. Drowsiness.
A. Pharmacological effects: b. Xerostomia, nausea, constipation.
1. Alendronate and risedronate increase bone mass, 4. Drug interactions:
reducing spine, hip, and other fractures. a. Increased effects of CNS depressants, including
2. Ibandronate reduces spine fractures. alcohol.
3. Teriparatide increases bone density and strength b. Increased anticholinergic effect occurs with an-
(synthetic parathyroid hormone). ticholinergics, phenothiazines, TCAs.
4. Raloxifene alters cycle of bone formation and bone 5. Contraindications:
tissue loss. a. Hypersensitivity.
Pharmacology    307

b. Acute asthma attack; epinephrine is MORE D. Antidiarrheals: reduce GIT motility to treat diarrhea (e.g.,
­effective for latter. Lomotil, Imodium, any opioid); may have xerostomia.
B. Second-generation (selective) H1-receptor antago- E. The H2-receptor antagonists (antihistamines, OTC):
nists: loratadine (Claritin), cetirizine (Zyrtec); third- cimetidine (Tagamet), ranitidine (Zantac), famotidine
generation (selective) H1-receptor antagonists: (Pepcid), nizatidine (Axid) (suffix “-tidine”).
desloratadine (Clarinex), fexofenadine (Allegra). 1. Pharmacological effects:
1. Pharmacological effects: a. Decreased gastric acid secretions; pain is re-
a. Antihistaminic and anticholinergic actions lieved.
SIMILAR to first-generation drugs but without b. NO further damage to esophageal lining from
sedation. gastric secretions.
b. NO CVS effects. 2. Therapeutic uses:
c. Little evidence for advantage of second- over a. Treat gastric ulcers resulting from NSAIAs and
third-generation that have been developed from gastroesophageal reflux (GERD).
second-generation. 3. Adverse reactions:
2. Therapeutic uses: a. Nausea, diarrhea, constipation.
a. Treatment of rhinitis. b. Cimetidine (Tagamet) and ranitidine (Zantac)
b. Seasonal allergy symptoms. may cause tachycardia and bradycardia.
3. Adverse reactions: 4. Drug interactions:
a. Xerostomia. a. Decreased absorption with ketoconazole (Ni-
b. Constipation. zoral).
4. Drug interactions: SAME as for first-generation. b. Cimetidine (Tagamet) results in increased blood
C. Leukotriene-receptor antagonist: montelukast (Singu- levels of metronidazole (Flagyl), alcohol, lido-
lair). caine (Xylocaine), opioids.
1. Pharmacological effects: c. Cimetidine (Tagamet) inhibits diazepam (Va-
a. Blocks leukotriene receptor sites. lium), warfarin (Coumadin).
b. Prevents leukotrienes from producing airway d. AVOID ulcerogenics such as NSAIAs and glu-
edema and inflammation, causing bronchocon- cocorticoids.
striction. F. Proton pump inhibitors (PPIs): first-generation
2. Therapeutic uses: (OTC): omeprazole (Prilosec); second-generation:
a. Treats mild or early forms of asthma as alterna- lansoprazole (Prevacid), esomeprazole (Nexium),
tive to steroids (carrying rescue bronchodilators pantoprazole (Protonix) (suffix “-prazole”).
still recommended). 1. Pharmacological effects:
b. Treats season allergy symptoms. a. Bind to proton pump in parietal cells of stom-
3. Adverse reactions: dream abnormalities, drowsi- ach to reduce gastric acid secretions; pain is re-
ness, irritability, restlessness. lieved.
4. Drug interactions: b. Neutralize gastric acid after it has been released;
a. Chewable form contains phenylalanine (contra- protect gastric mucosa from damage.
indicated for phenylketonuria). 2. Therapeutic uses:
b. Seizure drugs may decrease effects. a. Treat erosive esophagitis and GERD.
b. Treat gastric ulcers associated with use of
Patients with Gastrointestinal Tract Disorders NSAIAs, maintenance therapy for healed ul-
Patients with GIT disorders present with vomiting, nausea, cers, used with antibiotics for ulcers caused by
constipation, diarrhea, gas, gastric acid complications. Helicobacter pylori.
• See Chapter 6, General and Oral Pathology: related dis- c. Lansoprazole (Prevacid) for limited nighttime
cussion on GIT disorders. and pantoprazole (Protonix) for daytime and
A. Emetics: induce vomiting (emesis) to treat poisoning nighttime heartburn relief and with long-term
(e.g., OTC syrup of ipecac); abused with eating disor- use of NSAIAs to prevent GIT ulceration.
ders. 3. Adverse reactions:
B. Antiemetics: reduce nausea and vomiting (e.g., pro- a. Xerostomia, esophageal candidiasis, mucosal
chlorperazine [Compazine]); are used with nitrous atrophy of tongue.
oxide and other general anesthetics if there is risk of b. Nausea, diarrhea, constipation.
these adverse reactions; can cause CNS sedation and 4. Drug interactions:
xerostomia. a. Decrease absorption of weak bases that require
C. Laxatives: increase GIT motility to treat constipation acid for absorption such as antiretrovirals, iron
(e.g., milk of magnesia); abused with eating disorders. salts, systemic azole antifungals.
308   Saunders Review of Dental Hygiene

b. Esomeprazole (Nexium) increases levels b. Increased risk of malignant arrhythmias with


of some benzodiazepines (e.g., diazepam inhaled anesthetics.
[V­alium]). c. Decreased effects with nonselective β-blockers
c. Pantoprazole (Protonix) increases effects of (propranolol, Inderal).
SSRIs, some oral hypoglycemics, phenytoin B. Methylxanthine: aminophylline (Phyllocontin, Tru-
(­Dilantin), warfarin (Coumadin). phylline), theophylline (Theo-dur, Slo-bid), caf-
feine.
Patients with Respiratory Disease 1. Pharmacological effects: bronchodilation, CNS
Asthma and chronic obstructive pulmonary disease stimulation, diuresis.
(COPD) are serious respiratory diseases. COPD includes 2. Adverse reactions:
chronic bronchitis and emphysema. For these patients it is a. Nausea, vomiting.
difficult to make analgesic choice: aspirin (Bayer, Ecotrin, b. Anxiety, cardiac palpitations, increased respira-
Empirin, Bufferin) compounds may precipitate attack, tions.
so NSAIAs may be contraindicated and opioids cause 3. Drug interactions:
bronchoconstriction; acetaminophen may be the drug of a. Increased effects with erythromycin (Robimy-
choice, possibly mixed with a weak opioid. Patients us- cin, E-Mycin, E.E.S.).
ing inhaler should be instructed to rinse thoroughly after b. Increased dysrhythmias with CNS stimulants.
using and expectorate, which avoids increasing systemic c. Decreased effects with barbiturates, carbam-
absorption, reduces tooth staining, reduces risk for can- azepine (Tegretol), ketoconazole (Nizoral).
didal infection (with inhalers containing steroid), and re- C. Corticosteroid inhalers (metered dose inhalers
duces taste alteration. Patient must bring own inhalers [MDIs]): triamcinolone (Azmacort, Aerobid), be-
to appointments; however, many personal inhalers may clomethasone (Qvar, Beclovent, Vanceril); used for
NOT be current, so should be provided for in the dental acute and maintenance therapy; MDI provides drug
office emergency kit. directly to pulmonary tissues, lowering dose neces-
• See Chapters 6, General and Oral Pathology, and 8, Mi- sary for effect.
crobiology and Immunology: respiratory disorders and D. Cromolyn (Intal, Nasalcrom): effective ONLY for
diseases; 11, Medical and Dental Emergencies: respira- prevention of acute bronchospasm; NOT a treatment.
tory emergencies in a dental setting. E. Anticholinergics: atropine (past asthma drug) and
A. The β2-adrenergic (sympathomimetic) agonists by ipratropium bromide (Atrovent): newer drug causes
way of tablets, liquids, inhalers: albuterol (Proventil, fewer adverse reactions than atropine, used MAINLY
Ventolin), metaproterenol (Alupent), salmeterol (Ser- for emphysema.
event inhaler with fluticasone, Advair Diskus).
1. Pharmacological effects: Pregnant Patients
a. Bronchodilation by means of β2-receptor stimu- Many drugs can pass through the placental barrier, such as
lation in the lungs. tetracyclines that affect the newborn. However, some med-
b. NO antiinflammatory effects. ical conditions such as gestational diabetes, hyperthyroid-
2. Therapeutic uses: ism, or hypertension may require drug therapy to ensure
a. NOT used alone for treatment of asthma and optimal health of mother and fetus. Important to AVOID
COPD. any unnecessary drugs with pregnant patient and obtain
b. For rescue use ONLY with immediate relief a medical consult if use of any drugs is needed in a den-
during an attack. tal setting. See FDA Pregnancy Risk Category (PRC) and
3. Adverse reactions: dental drug examples of each category in Table 9-8. See
a. Xerostomia, nervousness, insomnia, tachycar- Chapter 11, Clinical Treatment, for a related discussion.
dia with selective adrenergic β2-agonists.
b. However, fewer adverse reactions occur with Patients with Cancer
these selective drugs than with nonselective Cancer patients may undergo chemotherapy (antineo-
adrenergic agonists (epinephrine and isoproter- plastics) and/or radiation therapy to eradicate cancerous
enol) that can be administered during an acute (malignant) growth (see Chapter 6, General and Oral Pa-
attack. thology: cancer treatment).
c. Overuse can lead to hyperreflexive airway, re-
sulting in drug failure during respiratory crisis. Immunocompromised Patients
4. Drug interactions: Patients undergoing immunosuppressive therapy or with
a. Increased CVS effects with MAOIs, antide- autoimmune disease may be taking certain potent drugs.
pressants, sympathomimetics (amphetamines, • See also Chapters 6, General and Oral Pathology, and 8,
dopamine). Microbiology and Immunology: ­immunocompromised
Pharmacology    309

Table 9-8  FDA use-in-pregnancy ratings with dental drug examples*

Rating Possible negative


PRC notation Category interpretation Dental drug example* outcome†

A Controlled Adequate, well-controlled studies in preg- None


studies nant women have failed to demonstrate
show no a risk to the fetus in any trimester of
risk pregnancy.
B No evidence Adequate, well-controlled studies in preg- Local anesthetics: etidocaine
of risk in nant women have not shown increased ­(Duranest), lidocaine (Xylo-
humans risk of fetal abnormalities despite adverse caine), prilocaine (Citanest)
findings in animals, or, in the absence of Pain relievers: acetaminophen Delayed labor and
adequate human studies, animal studies ­(Tylenol), ibuprofen (Advil, prolonged  
show no ­fetal risk. The chance of fetal Motrin, Pamprin), naproxen pregnancy in
harm is remote but remains a possibility. (Aleve, Anaprox)‡ mother
Antibiotics: amoxicillin (Amoxil,
Polymox), cephalexin (Ke-
flex), clindamycin (Cleocin),
erythromycin: base (E-Mycin),
estolate (Ilosone), ethylsuc-
cinate (E.E.S.), metronidazole
(Flagyl), penicillin V-potassium
(Pen-Vee K)
C Risk cannot Adequate, well-controlled human stud- Local anesthetics: bupivacaine Fetal bradycardia
be ruled ies are lacking, and animal studies have (Marcaine), mepivacaine
out shown a risk to the fetus or are lacking as ­(Carbocaine)
well. There is a chance of fetal harm if the Pain relievers: aspirin (Bayer, Postpartum
drug is administered during pregnancy, ­Ecotrin, Empirin, Bufferin) hemorrhage
but the potential benefits may outweigh and delivery
the potential risks. complications
Centrally acting opioid analge- Neonatal respira-
sics: codeine with acetamino- tory depression
phen (Tylenol with codeine),   and opioid
hydrocodone and acetamino- withdrawal (co-
phen (Vicodin), oxycodone deine also has
with acetaminophen (Percocet) multiple birth
defects)
Antibiotics: gentamicin Potential ototoxic-
­(Garamycin)‡ ity in fetus
D Positive Studies in humans, or investigational or Sedatives: diazepam (Valium), Possible oral clefts
evidence postmarketing data, have demonstrated alprazolam (Xanax) in fetus with
of risk fetal risk. Nevertheless, potential benefits prolonged
from the use of the drug may outweigh exposure
the potential risk. For example, the drug Antibiotics: doxycycline Instrinsic tooth
may be acceptable if needed in a life- ­(Vibramycin, Periostat) stain
threatening situation or serious disease
for which safer drugs cannot be used or
are ineffective.
X Contrain- Studies in animals or humans, or investi- Antibiotics: chloramphenicol Maternal toxicity  
dicated gational or postmarketing reports, have (Chloromycetin) and fetal death
in preg- demonstrated positive evidence of fetal
nancy abnormalities or risks, which clearly
outweighs any possible benefit to the
patient.

FDA, Food and Drug Administration; PRC, pregnancy risk category.


*Nitrous oxide is not rated.
†Data from Moore PA: Selecting drugs for the pregnant dental patient, J Am Dent Assoc 129(9):1281, 1998.
‡Risk for use during third trimester.
310   Saunders Review of Dental Hygiene

disorders or autoimmune diseases and immune sys- 4. Risk of bacteremia is higher in IV drug us-
tem. ers, thus antibiotic premedication may be
A. Immunosuppressants: cyclosporines, tacrolimus, si- ­indicated.
rolimus, etanercept (Enbrel). C. Barriers to care:
1. Pharmacological effects: prevent activity of im- 1. Communication problems related to denial of drug
mune system. abuse and disordered behavior (careful attention to
2. Therapeutic uses: behavior and physical appearance may help iden-
a. Prevent rejection of transplanted organs and tis- tify condition).
sues (e.g., bone marrow, heart, kidney, liver). 2. Transportation problems, if patient is unable to
b. Used to treat autoimmune diseases (e.g., RA, drive.
MG, SLE, Crohn’s disease, ulcerative colitis) 3. Economic problems related to drug habit costs and
and other nonautoimmune inflammatory dis- possible unemployment.
eases (e.g., long-term allergic asthma control). D. Professional care and homecare:
3. Adverse reactions: 1. Careful assessment of patient behavior and ap-
a. Risk of infections and cancer (MAJORITY of pearance (needle marks, sniffing, agitation, dull
them act nonselectively). expression, careless dress and hygiene, dilated or
b. HBP, hyperglycemia, peptic ulcers, liver and constricted pupils, bloodshot eyes); this needs to
kidney injury (especially with cyclosporines). be done even if patient claims to be in abstinence,
c. Gingival hypertrophy with cyclosporines, may a drug-free state.
be even MORE severe if patient is also taking a 2. Thorough intraoral examination with possible in-
calcium channel blocker. crease in risk of oral cancer owing to cohabit of
4. Drug interactions: many interactions noted; needs tobacco and alcohol use.
medical consult. 3. Awareness that patient may request pain or nitrous
oxide sedation because of drug habit.
Patients with Substance Abuse a. Seek drugs through dental offices by requesting
Substance (drug) abuse can be described as use of drugs drugs for pain relief.
despite adverse consequences to user or others. Dental b. Suspicions of drug abuse should be aroused
patients with substance (drug) addiction or drug depen- when specific drugs are requested.
dence may be problematic: (1) they may seek dental care c. Eliminating source of pain and prescribing
to obtain legal but addictive drugs; (2) patients with drugs NSAIAs if pain control is necessary, preferable
in bloodstream may be at risk for adverse drug interaction to prescribing addicting drugs.
with dentally prescribed drug. Drugs that can be abused are d. Short-term prescribing when necessary with
discussed next, and alcoholic patient is discussed later. medical consult; NOT prescribing with history
p9000 This category does NOT include drugs that cause of drug addiction.
a discontinuation syndrome when treatment stops 4. Caution with use of local anesthetics to prevent
abruptly or several doses are missed; can cause similiar reactions with illegal drugs (e.g., vasoconstrictor
withdrawal-like symptoms (“brain zaps,” “brain shocks,” with cocaine use can lead to fatality).
“brain shivers,” or “head shocks”) unless medication is E. Patient or caregiver education should emphasize:
gradually tapered off (e.g., occurs with antidepressants 1. Responsibility for maintaining good oral hygiene
such as SSRIs). and for keeping scheduled recall appointments.
• See CD-ROM for related terms. 2. Prevention of oral infection is particularly impor-
A. Oral signs: trauma, mucositis, xerostomia, extrinsic tant in IV drug users because of risk of bacteremia.
stain, high rates of dental caries, gingival lesions, F. Abused drugs: many drugs can be abused, and dental
periodontal disease, often from poor oral ­hygiene. professional must be aware of this possibility; alcohol
B. Risk factors: abuse is considered in next section.
1. May be be undergoing withdrawal, with psycho- 1. Abuse of CNS stimulants: nicotine, caffeine, am-
logical dependence and tolerance, which could phetamines, cocaine.
affect overall health and attitude. a. Caffeine (coffee, tea, sports and soft drinks):
2. Drug interaction with local anesthetics (AVOID (1) Physical dependence can occur with con-
vasoconstrictor use in cocaine and marijuana us- sumption of two to three cups of caffein-
ers) and ineffectiveness of local anesthetics in drug ated drink daily.
abusers. (2) Tolerance can develop.
3. Increased risk for contracting disease such as HIV, (3) Withdrawal can occur 24 hours after last
hepatitis, STD, infective endocarditis (IE) through intake, with headache, lethargy, irritability,
IV drug use (IVDA). anxiety, constipation.
Pharmacology    311

b. Nicotine (tobacco products): (2) Tolerance to the drug does NOT occur;
(1) Serious diseases can result from use (e.g., withdrawal symptoms also do NOT occur.
CVD, oral and lung cancer); use is linked (3) HBP, MI, CVA, seizure, or dysrhythmias
with increased risk of infant clefts and car- may occur.
ies, as well as endodontic complications. (4) Use of vasoconstrictors (such as epineph-
(2) Withdrawal symptoms: rine in local anesthetic) is absolutely con-
(a) Different for each person. traindicated; could be fatal.
(b) Aphthous ulcers (canker sores) as kera- (5) Pregnancy category “X” drug.
tinization is reduced. 2. Abuse of CNS depressants: opioids, barbiturates,
(3) Drugs can be used to reduce symptoms dur- benzodiazepines, volatile solvents (huffing glue,
ing cessation (behavioral modification can gasoline), nitrous oxide, alcohol.
help process): a. Opioids: heroin, hydromorphone (Dilaudid),
(a) Nicotine replacement therapy (NRT): morphine (MS Contin), methadone (Dolo-
chewing gum (Nicorette), lozenges phine), oxycodone (OxyContin; with acetamin-
(Commit), patches (Nicoderm, Nico- ophen [Percocet], with aspirin [Percodan]).
trol, Prostep, Habitrol), inhaler and (1) Pharmacological effects: elevated mood,
spray (Nicotrol) may be used to reduce euphoria, suppressed hunger, reduced sex-
physical dependence. ual drive, slowed respirations, constipation,
(b) Reduce psychological dependence by urinary retention, peripheral vasodilation.
continuing to release low levels of do- (2) Physical dependence and tolerance de-
pamine (antidepressant bupropion, Zy- velop; MORE drug is needed to achieve
ban); can be used with NRT. “high” feeling.
(c) Blocking receptors so smoking is less (3) Opioid overdose: respiratory depression,
pleasurable (nicotinic receptor partial miosis, coma; overdose treated by adminis-
agonist varenicline [Chantix] box label- tration of short-acting naloxone (Narcan).
ing involving mental states); NOT used (4) Withdrawal effects: lacrimation, rhinorrhea
with NRT. (runny nose), chills, diaphoresis, tremors,
(4) Quitlines have been shown to be effective tachycardia, HBP.
in introducing cessation efforts (healthcare (5) Treatments:
professionals MAINLY use 5 As; ADHA (a) Methadone maintenance programs.
uses three steps: “Ask, Advise, Refer”). (b) Counseling such as Narcotics Anony-
c. Amphetamines: dextroamphetamine (Dexe- mous (NA).
drine), methylphenidate (Ritalin), methamphet- b. Sedative-hypnotics: benzodiazepines (Valium),
amine (crystal meth) (Desoxyn). barbiturates, meprobamate (Miltown), Quaalude,
(1) Provide sense of euphoria and increased Librium.
energy; tolerance to euphoria develops. (1) Symptoms: resemble those of alcohol intox-
(2) Toxic symptoms: anxiety, hallucinations, ication, with MORE respiratory depression,
paranoia, aggressiveness. decreased GIT, urinary, cardiac output.
(3) Withdrawal symptoms: psychological de- (2) Prolonged abuse: paranoia and suicidal ten-
pression and suicidal tendencies. dencies.
(4) Acute overdose: mydriasis, HBP, tachycar- (3) Withdrawal symptoms are SIMILAR to
dia, cardiac dysrhythmias. opioids but can be life threatening (with-
(5) Abuse of meth may be seen in ALL age groups, drawal replacement drugs may include
including children; high lasts long time (12 benzodiazepines).
hours compared with 1 hour from ­cocaine). c. Nitrous oxide: hallucinations, paresthesias of
(6) Methamphetamine produces “meth mouth,” the extremities, mood swings; chronic abuse
erosion based on xerostomia, extended peri- can cause death.
ods of poor oral hygiene, frequent consump- 3. Psychedelics (hallucinogens): marijuana, lysergic
tion of high-calorie carbonated beverages, acid diethylamide (LSD), phencyclidine (PCP).
bruxism and clenching; smoking MOST a. Effects of psychedelics are psychological;
damaging form of drug for causing dental disturb reality perception; tolerance develops
destruction. quickly.
d. Cocaine (crack, rock): b. Abuse can cause lasting psychological distur-
(1) Creates sense of euphoria, short duration of bances, range from panic attacks to schizo-
action (1 hour). phrenic episodes to life-threatening depression.
312   Saunders Review of Dental Hygiene

c. Marijuana (cannabis, “weed”): other street drugs; associated with a high inci-
(1) Increases pulse rate; reddening of eyes, eu- dence of “bad trips.”
phoria. 4. Oral signs of drug abuse:
(2) Active ingredient has antiemetic effect; can be a. Rampant caries and tooth loss, halitosis (bad
used by prescription in some states to treat nau- breath), xerostomia.
sea that occurs during cancer chemotherapy. b. Trauma to the mouth, teeth, tissues; bruxism
d. LSD overdose symptoms: mydriasis, HBP, vi- (grinding).
sual distortions, hallucinations, paranoia; flash- c. Tissue ulceration, gingival recession, periodon-
backs can occur years after using. tal disease.
e. PCP (peep, angel dust) users exhibit bizarre d. Increased risk of oral cancer.
behavior, HBP, tachycardia; used alone or with

clinical study  

Age 18 YRS SCENARIO

Sex ☐  Male    ☒  Female The patient presents for her oral prophylaxis appointment
in an agitated state. Her behavior is in direct contrast to her
BP 112/65 (previous 100/60) behavior at previous oral prophylaxis appointments. She asks
the dental hygienist if he ever partied after work with all the
Chief Complaint “I want some of that
­laughing gas!” drugs available in the dental office.

Medical History None

Current Medications None

Social History High school student

1. What might be the cause of the patient’s unusual of patient’s behavior. Suspicions of drug abuse should
­behavior? be aroused when specific drugs are requested, as well
2. What is “laughing gas”? Identify its drug classifica- as awareness that patient may request pain or nitrous
tion and common uses. Should it be used at this ap- oxide sedation when not needed (procedure does not
pointment? entail much pain in a teenager’s oral cavity) because of
3. What is the significance of the slightly higher blood drug habit.
pressure at this appointment?
Alcoholic Patients
1. Patient’s unusual behavior could have several possible Alcoholism is a chronic but treatable disease that involves
causes, including anxiety, emotional disturbance, ill- compulsive abuse of ethanol-containing substances. Causes
ness, fatigue, substance abuse. may include genetic, psychological, environmental factors.
2. “Laughing gas” is slang term for nitrous oxide and A. Systemic complications:
oxygen analgesia (sedation). Classified as CNS de- 1. Impaired judgment, mydriasis, slurred speech,
pressant, along with opioids, barbiturates, benzodiaz- ataxia, seizures, coma, even death may occur.
epines, and alcohol. In dentistry, used to relax anxious 2. Nutritional deficiencies and cirrhosis of the liver. o16640
patients during dental treatment. Before gas is admin- 3. May have DTs during withdrawal. o16650
istered, careful medical and dental history interview 4. Chronic consumption during pregnancy causes
and physical assessment must be conducted to deter- fetal alcohol syndrome (FAS) with mild to severe
mine whether patient is using any drugs; necessary developmental disturbances; thus pregnancy cat-
to avoid potential problems such as drug interactions egory “X” drug.
(e.g., with phenothiazines, lithium [Lithobid, Eska- B. Orofacial signs:
lith], TCA) and CNS suppression. 1. Jaundiced (yellow) skin and sclera, puffiness of
3. Significance of slightly higher blood pressure at ap- the eyes.
pointment may be one indicator of substance abuse, 2. Reddened nose (rhinophyma), forehead, and
emotional distress, or a similar problem. Alone, vital cheeks.
signs offer little information; however, when com- 3. Xerostomia, reduced ability to taste, glossitis, odor
bined with other information, may lead to explanation of alcohol on breath.
Pharmacology    313

4. Enlargement of the parotid gland and the   3 What is an adverse reaction to a drug that is both predictable
tongue. and dose related and that acts on target organs?
5. Facial and dental trauma from falls and injuries. A. Side effect
6. Increased risk of BOTH caries and periodontal dis- B. Toxic reaction
ease from poor oral hygiene. C. Type I allergic reaction
D. Type III allergic reaction
7. Increased risk of oropharyngeal cancer.
E. Type IV allergic reaction
C. Treatment:   4 A patient reveals a history of asthma and uses the following
o13000 1. Counseling such as Alcoholics Anonymous (AA). drugs: propranolol, acetaminophen, cephalexin, and isopro-
o13010 2. Disulfiram (Antabuse) may be used during treat- terenol. Which of these drugs is used to treat asthma?
ment; ingestion of alcohol while taking causes the A. Propranolol (Inderal)
patient to become nauseated (i.e., to vomit); must B. Acetaminophen (Tylenol)
AVOID alcohol-containing mouthrinses. C. Cephalexin (Keflex)
o13020 3. Use of β-blockers. D. Isoproterenol (Isuprel)
D. Risk factors for oral health: nutritional deficiencies,   5 At his recall appointment, John complains that his oral tis-
interactions with drugs, infections, trauma, oral ­cancer sues and teeth are sensitive. He states that he chews gum
(especially if tobacco is also used); caution should be more often because his mouth is dryer. Upon examination, a
caries problem is detected. John reports a drug change since
used when administering amide local anesthetics and
his last appointment. Which one of the following drugs
nitrous oxide sedation. could account for the change in his oral condition?
E. Barriers to care: A. Nadolol (Corgard)
1. Communication difficulties; patient may NOT ap- B. Digoxin (Lanoxin)
pear for appointments when actively drinking. C. Temazepam (Restoril)
2. Transportation problems, if patient does NOT D. Sulfinpyrazone (Antazone)
drive or has lost driving privileges and must rely   6 A recall patient returns after receiving radiotherapy for cancer
on others. of the head and neck. The dental hygienist recognizes that the
3. Economic problems if the patient is unable to hold patient has salivary gland hypofunction in the form of xero-
a job or is on fixed income. stomia. The drug that may be prescribed for this condition is
F. Professional care and homecare: A. atropine (Atropair).
B. diflunisal (Dolobid).
1. Use of nonalcoholic mouthrinses.
C. pilocarpine (Salagen).
2. Possible bleeding problems from liver damage. D. prilocaine (Citanest).
3. Need for increased level of oral cancer evalua-   7 An analgesic, antipyretic, and antiinflammatory effect oc-
tion. curs with all of the following drugs, EXCEPT one. What is
4. Daily supplementation with fluoride and calcium the EXCEPTION?
product as needed. A. Diflunisal (Dolobid)
5. Dealing with patient’s intoxication and difficulties B. Acetaminophen (Tylenol)
in keeping appointments. C. Naproxen (Aleve)
G. Patient or caregiver education should emphasize need D. Ibuprofen (Advil)
for frequent recall, fluoride and calcium product sup-   8 Which of the following is a nonopioid drug that patients
plementation, saliva substitutes, need for practicing take in low doses after a myocardial infarction?
A. Aspirin (Empirin)
good homecare because of the risk of infection.
B. Acetaminophen (Tylenol)
C. Ibuprofen (Advil)
D. Naproxen (Aleve)
Review Questions   9 A recall patient who is in pain because of a dental abscess is
prescribed Tylenol #3. The patient requires diazepam (Va-
lium), an antianxiety drug, before her appointments. A pos-
  1 The strength of a drug with regard to its ability to achieve a sible drug interaction to be aware of is
desired effect is termed A. decreased sedation.
A. efficacy. B. increased sedation.
B. potency. C. increased antitussive effect.
C. therapeutic effect. D. decreased analgesia.
D. tolerance. 10 When respiratory depression occurs because of an overdose
  2 All of the following are sites of drug elimination, EXCEPT of an opioid analgesic agent, a drug that will reverse the
one. Which one is the EXCEPTION? respiratory depression is
A. Liver A. morphine (MS Contin).
B. Kidney B. epinephrine.
C. Colon C. naloxone (Narcan).
D. Lungs D. ammonia inhalants.
E. Pancreas E. lidocaine (Xylocaine).
314   Saunders Review of Dental Hygiene

11 All of the following are TRUE about tetracyclines, ­ 18 Diazepam (Valium) is used in dentistry for all of the fol-
EXCEPT one. Which one is the EXCEPTION? lowing therapeutic uses, EXCEPT one. Which one is the
A. Are used to treat aggressive periodontal disease. EXCEPTION?
B. Photosensitivity is a side effect of their use. A. Analgesic
C. Chelation occurs when they are combined with calcium, B. Antianxiety
magnesium, or aluminum. C. Anticonvulsant
D. Tooth discoloration is not an adverse effect of D. Muscle relaxant
their use. 19 Phenobarbital (Luminal) is used for its anticonvulsant effect
E. Use decreases the effectiveness of oral contraceptives. and belongs to which one of the following drug groups?
12 A patient being treated for aggressive periodontal disease A. Benzodiazepines
complains of a metallic taste, black hairy tongue, and xero- B. Barbiturates
stomia. The drug that is causing these conditions is C. Nonbarbiturates
A. amoxicillin (Amoxil). D. Phenothiazines
B. metronidazole (Flagyl). E. Monoamine oxidase inhibitors
C. tetracycline (Tetracyn). 20 A patient reports that he is taking prochlorperazine (Com-
D. cephalexin (Keflex). pazine). All of the following are adverse reactions of Com-
E. clindamycin (Cleocin). pazine, EXCEPT one. Which one is the EXCEPTION?
13 A patient reports no medical conditions on her medical his- A. Orthostatic hypotension
tory but indicates that she takes isoniazid (Nydrazid) and ri- B. Seizures
fampin (Rifadin). These medications indicate that she might C. Xerostomia
have D. Tardive dyskinesia
A. tuberculosis. E. CNS stimulation
B. human immunodeficiency virus infection. 21 A patient reports having manic-depression. Which one of
C. osteomyelitis. the following drugs is used to treat this illness?
D. candidiasis. A. Phenelzine (Nardil)
14 A patient has aggressive periodontal disease and takes B. Amitriptyline (Elavil)
chlorpropamide (Diabinese) for type 2 diabetes. The drug C. Lithium (Lithobid, Eskalith)
of choice for treating the periodontal disease and avoiding D. Chlordiazepoxide (Librium)
adverse interaction with Diabinese is 22 All of the following are second-generation antidepressants,
A. doxycycline (Vibramycin). EXCEPT one. Which one is the EXCEPTION?
B. metronidazole (Flagyl). A. Fluoxetine (Prozac)
C. penicillin V (Pen-Vee K). B. Amitriptyline (Elavil)
D. cephalosporin (Keflor). C. Trazodone (Desyrel)
15 Harry requires prophylactic antibiotic coverage for an ar- D. Bupropion (Wellbutrin)
tificial heart valve before dental treatment. He also takes 23 Sarah reports that she has epilepsy but cannot remember
digoxin (Lanoxin) to treat congestive heart failure. He is the drug she takes to treat it. She notices that her gingiva is
NOT allergic to penicillins. The safest antiinfective agent enlarged. The drug she is MOST likely taking is
for Harry that follows AHA recommendations is A. cimetidine (Tagamet).
A. tetracycline (Achromycin). B. fluconazole (Diflucan).
B. clindamycin (Cleocin). C. phenytoin (Dilantin).
C. azithromycin (Zithromax). D. diazepam (Valium).
D. amoxicillin (Amoxil). 24 Carbamazepine (Tegretol) is used as an anticonvulsant
16 Propranolol (Inderal) is used to treat all of the following agent. Its other pharmacological effects include all of the
conditions, EXCEPT one. Which one is the EXCEPTION? following, EXCEPT one. Which one is the EXCEPTION?
A. Angina pectoris A. Antihypertension
B. Congestive heart failure B. Anticholinergic
C. Moderate to severe hypertension C. Antidepressant
D. Migraine headaches D. Sedative
E. Myocardial infarction prophylaxis E. Muscle relaxant
17 A patient reports that he is being treated for high choles- 25 Prednisone (Deltasone) is prescribed to treat all of the
terol. He is MOST likely taking which one of the following following conditions, EXCEPT one. Which one is the
drugs? ­EXCEPTION?
A. Propranolol (Inderal) A. Crohn’s disease
B. Metoprolol (Toprol XL) B. Rheumatoid arthritis
C. Simvastatin (Zocor) C. Addison’s disease
D. Atenolol (Tenormin) D. Cushing’s syndrome
E. Nifedipine (Procardia) E. Systemic lupus erythematosus


Pharmacology    315

26 A patient with type 1 diabetes mellitus complains of pain 33 When nitrous oxide is used during nonsurgical dental treat-
after professional prophylaxis. The dental hygienist rec- ment, which of the following is the desired effect?
ommends a nonopioid analgesic. Which of the following A. Loss of consciousness
medications would have the LEAST interaction with the B. Conscious sedation
patient’s medication? C. General anesthesia
A. Aspirin (Empirin) D. Respiratory depression
B. Acetaminophen (Tylenol) E. Local anesthesia
C. Ibuprofen (Advil) 34 After terminating the use of nitrous oxide, a patient com-
D. Diflunisal (Dolobid) plains of a headache. Which of the following has occurred?
27 A recall patient reveals a recent history of taking Premarin. A. Diffusion hypoxia
She states that she has gained weight, and the dental hy- B. Neuropathy
gienist discovers that the patient’s blood pressure is much C. Allergic reaction
higher than during previous visits. These symptoms may be D. Toxic reaction
the result of 35 All of the following are inhalation volatile liquids that are
A. drug interactions. used as general anesthetics, EXCEPT one. Which one is the
B. allergic reactions. EXCEPTION?
C. adverse reactions. A. Halothane (Fluothane)
D. toxic reactions. B. Enflurane (Ethrane)
28 A patient who reports taking levothyroxine (Synthroid) has C. Methohexital (Brevital)
which of the following conditions? D. Isoflurane (Forane)
A. Hypothyroidism 36 The MAIN adverse reaction that occurs with the use of an
B. Hyperthyroidism opioid agent as a general anesthetic is
C. Cushing’s syndrome A. significant cardiovascular change.
D. Addison’s disease B. excessive salivation.
E. Cancer C. hypertension.
29 When Tom arrives for his dental appointment, he complains D. respiratory depression.
of itching and a rash on his arm. He reports that he took E. bronchospasm.
his prophylactic antibiotic an hour before his appointment. 37 Abuse occurs with all of the following central nervous
Which of the following drugs will treat this mild type IV system depressant drugs, EXCEPT one. Which one is the
allergic reaction? ­EXCEPTION?
A. Loratadine (Claritin) A. Alcohol
B. Clemastine fumarate (Tavist) B. Morphine (MS Contin)
C. Prednisone (Deltasone) C. Diazepam (Valium)
D. Ibuprofen (Advil) D. Nicotine (Camels)
E. Diphenhydramine (Benadryl) E. Oxycodone (Percodan)
30 A patient with cardiovascular problems has seasonal aller- 38 Antineoplastics usually produce all of the following oral
gies. Which antihistamine agent has little or no effect on conditions, EXCEPT one. Which one is the EXCEPTION?
the cardiovascular system and can be safely used for this A. Mucosal pain
patient? B. Gingival recession
A. Nizatidine (Axid) C. Gingival hemorrhage
B. Loratadine (Claritin) D. Xerostomia
C. Hydroxyzine (Atarax) E. Mucosal ulceration
D. Famotidine (Pepcid) 39 An asthmatic patient returns for a recall appointment. The
31 When oral surgery is performed, a long-acting local anes- patient forgets to bring her inhaler but remembers that it is a
thetic is usually needed. Which of the following would be corticosteroid inhaler. The active agent in the inhaler is
the drug of choice? A. triamcinolone (Azmacort).
A. Procaine (Novacaine) B. albuterol (Proventil).
B. Bupivacaine (Marcaine) C. metaproterenol (Alupent).
C. Lidocaine (Xylocaine) D. isoproterenol (Isuprel).
D. Prilocaine (Citanest) 40 Asthmatic patients may have allergic reactions when they
E. Mepivacaine (Marcaine) are given local anesthetics that contain vasoconstrictors.
32 Adding a vasoconstrictor to a local anesthetic does all of the Such reactions may be caused by which of the following in
following, EXCEPT one. Which one is the EXCEPTION? dental cartridges?
A. Creates hemostasis A. Epinephrine
B. Constricts blood vessels B. Lidocaine (Xylocaine)
C. Increases duration of anesthetic C. Sodium bisulfite
D. Increases absorption into cardiovascular system D. Sodium sulfate
316   Saunders Review of Dental Hygiene

41 Common oral manifestations of substance abuse include all 49 Diazepam (Valium) is a benzodiazepine given as premedi-
of the following, EXCEPT one. Which one is the EXCEP- cation before dental treatment. A history of infective endo-
TION? carditis requires premedication before dental treatment.
A. Xerostomia A. Both statements are true.
B. Trauma B. Both statements are false.
C. Lichen planus C. The first statement is true, the second is false.
D. Extrinsic stain D. The first statement is false, the second is true.
E. Mucositis 50 Contraception using birth control pills occurs because
42 In the treatment of a patient who is a suspected substance of inhibition of ovulation and thus pregnancy.
abuser, the safest pain control modality is A. Both the statement and reason are correct and related.
A. local anesthetic. B. Both the statement and reason are correct but NOT
B. local anesthetic with vasoconstrictor. ­related.
C. nitrous oxide analgesia. C. The statement is correct, but the reason is NOT.
D. general anesthesia. D. The statement is NOT correct, but the reason is correct.
43 All of the following are conditions associated with alcohol- E. NEITHER the statement NOR the reason is correct.
ism, EXCEPT one. Which one is the EXCEPTION?
A. Reddened nose
B. Puffy eyes
C. Yellow skin
Answer Key and Rationales
D. Dry mouth
E. Fruity breath
44 Prophylactic antibiotics are required for the prevention of 1 (B)  Potency of a drug refers to strength, which en-
infective endocarditis in all of the following conditions, ables it to achieve desired effect. Efficacy is ability,
EXCEPT one. Which one is the EXCEPTION? NOT strength, to produce desired effect. Therapeutic
A. Mitral valve prolapse with valvular regurgitation effect is clinically desirable action of a drug. Toler-
B. Past history of infection ance occurs when no effect takes place because of
C. Prosthetic cardiac valves decreased susceptibility to drug after continuous use.
D. Valve disorders after heart transplant 2 (E)  Pancreas aids in digestion of foods and regula-
45 Diffusion hypoxia may result with headache or other ad- tion of carbohydrate metabolism; however, unlike
verse reactions because patient did not receive 5 minutes of
liver, kidney, colon, and lungs, does NOT function to
100% oxygen when nitrous oxide sedation terminated.
A. Both the statement and reason are correct and related.
eliminate drugs from body.
B. Both the statement and reason are correct but NOT ­related. 3 (B)  Toxic reactions and side effects (adverse reac-
C. The statement is correct, but the reason is NOT. tions), unlike allergic reactions, are predictable and
D. The statement is NOT correct, but the reason is correct. dose related. However, toxic reactions, unlike side ef-
E. NEITHER the statement NOR the reason is correct. fects, act on target organs because they are extension
46 Ketamine produces dissociative anesthesia because the pa- of drug’s pharmacological effect.
tient continues to respond to environment but is not asleep. 4 (D)  Isoproterenol (Isuprel), adrenergic (sympatho-
A. Both the statement and reason are correct and related. mimetic) agent, affects β2-receptors in lungs to en-
B. Both the statement and reason are correct but NOT ­related. hance bronchodilation. Propranolol (Inderal) is a
C. The statement is correct, but the reason is NOT. ­nonselective β-adrenergic blocker that treats car-
D. The statement is NOT correct, but the reason is correct.
diovascular diseases. Acetaminophen (Tylenol) is
E. NEITHER the statement NOR the reason is correct.
47 Antimicrobial rinses (e.g., chlorhexidine, Listerine) are
a nonopioid analgesic, and cephalexin (Keflex) is a
contraindicated in a recovering alcoholic taking disulfiram cephalosporin antiinfective agent.
(Antabuse) because reactions occur with antifungal-con- 5 (A)  Nadolol (Corgard) is a nonspecific β-adrenergic
taining products. blocker. Common adverse reaction is xerostomia,
A. Both the statement and reason are correct and related. which can cause an increase in caries and sensitiv-
B. Both the statement and reason are correct but NOT ity. Digoxin (Lanoxin), a cardiac glycoside, temaze-
­related. pam (Restoril), a benzodiazepine, and sulfinpyrazone
C. The statement is correct, but the reason is NOT. (Antazone), a uricosuric agent, are NOT associated
D. The statement is NOT correct, but the reason is correct. with xerostomia as an adverse reaction.
E. NEITHER the statement NOR the reason is correct. 6 (C)  Pilocarpine (Salagen) is a cholinergic (para-
48 Digitalis glycoside (Lanoxin) is used in treatment of glau-
sympathomimetic) agent that mimics action of ACh
coma because it decreases kidney output.
(a neurotransmitter) on parasympathetic receptors,
A. Both the statement and reason are correct and related.
B. Both the statement and reason are correct but NOT ­related. thereby increasing salivary excretion. Atropine (At-
C. The statement is correct, but the reason is NOT. ropair) is an ophthalmic anticholinergic agent that
D. The statement is NOT correct, but the reason is correct. decreases tears. Diflunisal (Dolobid) is a nonsteroi-
E. NEITHER the statement NOR the reason is correct. dal antiinflammatory agent (NSAIA) that is used
Pharmacology    317

for ­analgesia. Prilocaine (Citanest) is an amide local (Amoxil, ­Larotid, Polymox) and digoxin (Lanoxin),
­anesthetic. and patient is NOT known to be allergic to penicil-
7 (B)  Acetaminophen (Tylenol) is a nonopioid analge- lin. Combining digoxin (Lanoxin) with azithromy-
sic with antipyretic but no antiinflammatory effects. cin (Zithromax) may cause increased blood levels
Others are NSAIAs, nonopioid analgesics that pos- of digoxin (Lanoxin), which leads to toxicity, since
sess ALL three stated effects. digoxin belongs to family of macrolide antibiotics.
8 (A)  Aspirin (Bayer, Ecotrin, Empirin, Bufferin) is Tetracyclines are NOT drug choice in AHA regi-
a nonopioid agent that irreversibly reduces platelet men.
adhesiveness. When used at low dose (other com- 16 (B)  Propranolol (Inderal) is an antiadrenergic nonse-
pounds), it prevents unwanted clotting and helps to lective β-blocker that treats angina pectoris, hyperten-
prevent occurrence of myocardial infarction (MI, sion, and migraines and can be used for myocardial
heart attack). Others are also nonopioids but do NOT infarction (MI, heart attack) prophylaxis. NOT a
have irreversible platelet effects. digitalis glycoside; CANNOT treat congestive heart
9 (B)  Tylenol #3 contains codeine, an opioid analgesic failure (CHF).
that depresses CNS. Diazepam (Valium) is a benzo- 17 (C)  Simvastatin (Zocor) is an antihyperlipidemic
diazepine that depresses CNS. When combined, pro- used to treat hyperlipidemia (high cholesterol). Pro-
duce additive effect of CNS depression. pranolol (Inderal), an antiadrenergic nonselective
10 (C)  Naloxone (Narcan) is an opioid antagonist that β-blocker, metoprolol (Lopressor) and atenolol (Te-
competes with opioids at receptor sites and thus re- normin), antiadrenergic β1-blockers, and nifedipine
verses opioid respiratory depression. Morphine (MS (Procardia), calcium channel blocker, ALL treat car-
Contin) is an opioid analgesic, NOT antagonist. Epi- diovascular diseases, NOT hyperlipidemia.
nephrine, lidocaine (Xylocaine), and ammonia inhal- 18 (A)  Diazepam (Valium) is a benzodiazepine (seda-
ants also are NOT opioid antagonists. tive-hypnotic) agent that is used for anxiety control,
11 (D)  Permanent tetracycline staining does occur if for antianxiety premedication, for treatment of sei-
the drug is given in the period of enamel calcification zures, as muscle relaxant; it has NO analgesic ef-
during tooth development. fects.
12 (B)  Metronidazole (Flagyl) is an antiinfective agent. 19 (B)  Phenobarbital (Luminal) is a long-acting barbi-
Drug of choice for aggressive periodontitis that may turate that treats epilepsy.
produce ALL three oral effects. Amoxicillin (Amoxil) 20 (E)  Prochlorperazine (Compazine), a phenothiazine,
and tetracycline (Tetracyn) are antiinfectives that treat is an antipsychotic agent that depresses, rather than
aggressive periodontitis but that do NOT have ALL suppresses, central nervous system.
three oral effects. Cephalexin (Keflex) and clindamy- 21 (C)  Lithium (Lithobid, Eskalith) is an antipsychotic o16210
cin (Cleocin) are antiinfectives but are NOT used to agent that treats manic-depressive illness, now con-
treat aggressive periodontitis. sidered bipolar disorder, which cycles between moods
13 (A)  Tuberculosis treatments often combine isonia- of depression or mania. Phenelzine (Nardil), antide-
zid (Nydrazid) and rifampin (Rifadin, Rimactane) pressant monoamine oxidase inhibitor (MAOI), and
for their antituberculosis synergistic effects. Drug amitriptyline (Elavil), tricyclic antidepressant (TCA),
combination also is used prophylactically in patients treat depression other than that with bipolar disorder.
who test positive in response to tuberculosis skin test Chlordiazepoxide (Librium) is a benzodiazepine that
(Mantoux test). treats anxiety.
14 (B)  Drugs of choice for aggressive periodontitis are 22 (B)  Amitriptyline (Elavil) is a tricyclic antidepres-
metronidazole (Flagyl) and tetracyclines such as dox- sant (TCA), NOT a second-generation antidepres-
ycycline (Vibramycin). Tetracyclines combined with sant. Others are second-generation antidepressants.
oral sulfonylureas such as chlorpropamide (Diabi- 23 (C)  Phenytoin (Dilantin) is an anticonvulsant agent.
nese) result in hypoglycemia. ONLY remaining drug Gingival hyperplasia is one of its common adverse re-
of choice for this patient is metronidazole (Flagyl) be- actions. Cimetidine (Tagamet) is an H2 antagonist that
cause penicillins (Pen-Vee K) and cephalosporin are treats ulcers. Fluconazole (Diflucan) is an antifungal
NOT appropriate for treating aggressive periodontal that treats candidiasis. Diazepam (Valium), benzodiaz-
disease. epine, is used as adjunct in seizure control, NOT asso-
15 (D)  Amoxicillin (Amoxil, Larotid, Polymox), NOT ciated with gingival hyperplasia as adverse reaction.
clindamycin (Cleocin), is first drug of choice in 24 (A)  Carbamazepine (Tegretol) can create hyperten-
the 2007 American Heart Association (AHA) regi- sion; NOT classified as cardiovascular disease (CVD)
men for prevention of infective endocarditis (IE). drug.
Patient is at risk for IE from artificial heart valve. 25 (D)  Prednisone (Deltasone), glucocorticoid with
NO adverse drug interactions between amoxicillin antiinflammatory and immune-suppressing ­effects,
318   Saunders Review of Dental Hygiene

treats diseases listed, EXCEPT for Cushing’s syn- 35 (C)  Methohexital (Brevital) is ultra-short-acting
drome. Cushing’s syndrome results from hyper- ­barbiturate used as an IV agent, NOT an inhalation
secretion of glucocorticoids by adrenal cortex; volatile liquid.
therefore prednisone is contraindicated for this 36 (D)  Primary adverse reaction associated with opioids
disease. is respiratory depression, which can be reversed with
26 (B)  NO adverse drug interaction occurs when ac- opioid antagonist such as naloxone (Narcan).
etaminophen (Tylenol) is combined with insulin. 37 (D)  Nicotine is abused, is a CNS stimulant, NOT a
Insulin combined with salicylates (aspirin [Bayer, depressant.
Ecotrin, Empirin, Bufferin] and diflunisal [(Dolobid] 38 (B)  Gingival recession does NOT occur because of
or NSAIAs (ibuprofen [Advil, Motrin, Pamprin]) antineoplastic agent use but because of other factors
may increase hypoglycemia. in patient’s oral cavity.
27 (C)  Premarin, estrogen, is associated with several ad- 39 (A)  Triamcinolone (Azmacort) is a corticosteroid
verse reactions, including nausea, vomiting, uterine inhaler that achieves significant improvement in
bleeding, weight gain, thrombophlebitis, hypertension pulmonary function. Others are adrenergic agonists
(high blood pressure). that create bronchodilation by means of β2-receptor
o16280 28 (A)  Levothyroxine (Synthroid) is a thyroid gland stimulation in lungs.
agent. Administered to patients who have hypothy- 40 (C)  Sodium bisulfite is antioxidant (preservative) for
roidism (insufficient production of thyroid hormone) vasoconstrictor (epinephrine) in dental local anes-
and require thyroid replacement therapy. Cushing’s thetic (such as lidocaine) cartridges. Preservative may
syndrome and Addison’s disease are associated with be catalyst for allergic reactions in asthmatic patients
adrenal cortex abnormalities. Cancers are treated who receive local anesthetics with vasoconstrictors.
with antineoplastics. Sodium sulfate is natural sulfate found in body.
29 (E)  Diphenhydramine (Benadryl) is an H1-receptor 41 (C)  Common oral manifestations (signs) of sub-
antagonist (antihistamine). When a type IV allergic stance abuse include xerostomia, oral and facial
reaction creates a rash by releasing histamine, admin- trauma, extrinsic stain, mucositis. Lichen planus is an
istration of an antihistamine blocks histamine and inflammatory disease of unknown origin associated
decreases allergic reaction. Loratadine (Claritin) and with systemic diseases such as hypertension (high
clemastine fumarates (Tavist) are antihistamines that blood pressure) and diabetes mellitus.
treat seasonal allergies. Prednisone (Deltasone) is a 42 (A)  With suspected substance abuser, AVOID use
steroid, and ibuprofen (Advil, Motrin, Pamprin) is a of epinephrine because may interact with cocaine
nonopioid analgesic. and other drugs. Safest pain control modality is lo-
30 (B)  Loratadine (Claritin) is second-generation cal anesthetic without vasoconstrictor. Nitrous ox-
H1-receptor antagonist (antihistamine). Claritin has ide analgesia (sedation) is safe to use but does have
NO effect on CVS and therefore can be administered potential for abuse and for transmission of infec-
to patients with cardiovascular problems. Nizatidine tious diseases (such as hepatitis B or C) if tubing is
(Axid) and famotidine (Pepcid) are H2 antihistamines NOT sterilized. General anesthesia is NOT recom-
that treat ulcers. Hydroxyzine (Atarax) is an antianxi- mended because of concern for patient safety and
ety antihistamine. dental practice risk management. For general pain
31 (B)  Bupivacaine (Marcaine) and etidocaine (Duran- control, over-the-counter (OTC) pain relievers are
est) are longer-acting amide local anesthetic drugs. recommended.
Others are considered shorter-acting drugs. 43 (E)  Fruity breath is NOT one of conditions associ-
32 (D)  Vasoconstrictor constricts blood vessels, thereby ated with alcoholism. Reddened nose (rhinophyma),
decreasing absorption of local anesthetic agent into puffy eyes, yellow skin (jaundice), and dry mouth
CVS and increasing duration of agent. Vasoconstric- (xerostomia) are associated with alcoholism.
tor also creates hemostasis because of blood vessel 44 (A)  Functional heart murmurs involve NO tissue dam-
constriction. age and therefore NO risk of infective ­endocarditis.
o16330 33 (B)  Nitrous oxide is not used at the level of a gen- American Heart Association (AHA) recommends that
eral anesthetic for most nonsurgical dental treatment antibiotic premedication be given before any invasive
(standard of care recommends use at less than 50%). dental procedure for patients with prosthetic heart
When it is used properly, conscious (moderate) seda- valves, past history of infection, valve disorders after
tion should occur with NO significant effect on respi- heart transplant, and NOT mitral valve prolapse with
ratory system. regurgitation.
34 (A)  Diffusion hypoxia results in a headache if patient 45 (A)  Both statement and reason are correct and re-
does NOT receive at least 5 minutes of 100% oxygen lated. Diffusion hypoxia may result with headache
at the termination of the nitrous oxide sedation. and other adverse reactions because patient did NOT
Pharmacology    319

receive 5 minutes of 100% oxygen when nitrous congestive heart failure (CHF) with its inability of the
oxide sedation terminated. heart to pump efficiently because digitalis increases
46 (C)  Statement is correct, but reason is NOT. Ket- cardiac contractility and output.
amine produces dissociative anesthesia because pa- 49 (A)  Both statements are true. Diazepam (Valium) is o16490
tient continues to fail to respond to environment but a benzodiazepine and antianxiety drug that is given
is NOT asleep. as an antianxiety premedication to help cope with
47 (C)  Statement is correct, but reason is NOT. Antimi- dental anxiety. Infective endocarditis (IE) is a seri-
crobial rinses (e.g., chlorhexidine, Listerine) that may ous infection; patients with past history of it need
contain alcohol are contraindicated in a recovering al- antibiotic premedication before invasive dental
coholic taking disulfiram (Antabuse) because reactions treatment.
occur with alcohol and alcohol-containing products. 50 (A)  Both statement and reason are correct and re-
48 (E)  Neither statement nor reason is correct. Digi- lated. Contraception using birth control pills occurs
talis glycoside (Lanoxin) is used in treatment of ­ by inhibition of ovulation and thus pregnancy.
CHAPTER 10

Medical and Dental Emergencies


EMERGENCY PREVENTION    retaken as needed. Information in the following is taken
The combination of pain, stress, infection, and anesthe- from the “7th Report of the Joint National Committee
sia use in the dental setting may contribute to medical on Prevention, Detection, Evaluation, and Treatment of
emergencies. Evaluation includes completion of a com- High Blood Pressure” (2003).
prehensive health history, in-depth history of present ill- • See Chapter 3, Anatomy, Biochemistry, and Physiol-
ness, assessment of vital signs, physical assessment, and ogy: physiology of blood pressure; Chapter 6, General
thorough ­emergency preparation in the dental setting. In- and Oral Pathology: high blood pressure.
stances of when to request a medical consult regarding a A. Blood pressure: force exerted by circulating blood on
patient and/or when to consider a situation an emergency the walls of blood vessels.
and activate the Emergency Medical Service (EMS) 1. Measured in terms of air pressure required to com-
system are discussed in this chapter. press large artery to point of shutting off blood flow,
• See CD-ROM for Chapter Terms and WebLinks. reading recorded in fractions of millimeters of mer-
• See Chapters 6, General and Oral Pathology: pathologi- cury (mm Hg), although mercury is NO longer used
cal disorders risk for emergencies; 8, Microbiology and in most modern vascular devices because of possible
Immunology: allergic reactions in a dental setting; 9, toxicity. Measurements for healthy adult patient are
Pharmacology, drugs in emergency kit in a dental set- usually <120/80; cannot feel blood pressure level.
ting; 14, Pain Management: local anesthesia and nitrous a. Systolic pressure: heart contracts (works):
oxide administration emergencies in a dental setting. (1) Is <120 mm Hg.
(2) FIRST pulse sound heard when pressure is
HEALTH HISTORY   released from the cuff.
Patient’s health history includes BOTH medical and den- b. Diastolic pressure: heart dilates (rests):
tal history, as well as in-depth history of present illness. (1) Is <80 mm Hg.
Crucial to BOTH the diagnosis and treatment of and (2) Last pulse sound heard when pressure is re-
prognosis for conditions experienced by dental patients. leased from the cuff.
Modifying dental treatment as indicated by the health his- 2. Prehypertension (“high normal”):
tory creates less risk of emergencies in the dental setting. a. 120 to 139/80 to 89 mm Hg.
• See Chapters 9, Pharmacology: antibiotic premedica- b. Treat as usual, but recommend that patient
tion and Physician’s Desk Reference; 11, Clinical Treat- schedule a consult with physician.
ment: health history; 18, Ethics and Jurisprudence: c. Usually involves changes in lifestyle to reduce
related ethical concerns. risk of further complications.
A. Identifies changes in drugs and/or conditions that may 3. Hypertension (high blood pressure [HBP]):
require antibiotic premedication (e.g., valvular replace- a. Indicated by sustained readings that are ≥140/
ment and/or congenital heart disease or ­transplant). 90 mm Hg.
B. Identifies precautions, allergies, or conditions that b. Can occur for either systolic or diastolic or both.
may contraindicate or modify treatment. c. HBP can have two stages:
C. Includes record of vital signs. (1) Stage 1: Mild HBP (140 to 159/90 to
D. May assist in identification of oral manifestations 99 mm Hg):
­associated with some drugs or systemic diseases. (a) Repeat, patient may have “white-coat”
E. Assists in planning treatment according to physiolog- syndrome (elevated arterial pressure
ical and psychological status. specifically during medical exams,
F. Serves as medicolegal document. probably because of anxiety) or may
have raced to appointment.
Vital Signs (b) NO change in dental care treatment plan.
Vital signs include blood pressure reading, pulse rate, res- (c) MUST refer to physician unless other
piration rate, temperature reading (if needed), pupil size. signs of HBP are present or patient is in
Obtained as a baseline for patient. Specific vital signs are denial.

320
Medical and Dental Emergencies    321

(2) Stage 2: Moderate to severe HBP 3. Measurements for healthy patients: adults 60 to
(>160/>100 mm Hg): 90 BPM; children 80 to 110 BPM.
(a) Ask patient if taking prescribed an- 4. Affected by age, exercise, stress, drugs, stimulants,
tihypertensive drug(s); patient may fasting, long-term illness, emotional state.
NOT be compliant, since “feels bet- 5. Unusual pulse rates:
ter.” a. Tachycardia: faster than normal; >100 BPM
(b) May NOT be able to perform elective for adult.
dental care; may need immediate medi- b. Bradycardia: slower than normal; <60 BPM
cal consult. for adult.
(3) If ≥180/≥110 mm Hg: activate EMS system c. Pulsus alternans: beats are alternately weak
and treat as emergency. and strong.
4. Blood pressure affected by: d. Premature ventricular contractions (PVCs):
a. Heart rate (HR), stroke volume, total peripheral longer than normal pause or a skip noted
resistance. ­between beats; if MORE than 5 per minute,
b. Exercise, eating, stimulants, pain, stress, anxi- cause for concern as this may represent a heart
ety, patient positioning. block.
c. Drugs being taken, such as antihypertensives 6. AVOID palpating carotid pulse unless EMS per-
or others (e.g., monoamine oxidase inhibitors sonnel (lack indicates death), since stimulating its
[MAOIs]). baroreceptors with vigorous palpation can provoke
d. Recordings taken from right and left arm may severe bradycardia or even stop the heart in some
differ. sensitive persons; BOTH should not be palpated at
5. Technique for taking blood pressure reading: in- the same time to AVOID risk of fainting or brain
struments used to measure blood pressure include ischemia.
stethoscope and sphygmomanometer (cuff size is C. Respiration rate: obtained by counting number of
determined by arm size, cuff too small or large breaths for 30 seconds (number is multiplied by 2 if
will give false HBP reading); SIMILAR for digital rate is regular), one exhalation and one inhalation to-
devices; need to wait 2 to 3 minutes between read- gether count as one breath; if increased or irregular,
ings for all types. taken again for 1 minute.
a. Cuff placed snugly 1 inch above antecubital 1. Measurement for healthy patient: adults 14 to
fossa, NO clothing between cuff and arm. 20 breaths/min; children 18 to 30 breaths/min.
b. Cuff pumped until radial pulse is NO longer 2. Normal rates can be altered if patient knows that
felt; note this point (estimated systolic number) breathing is being observed; overall rates can be
and release the pressure. affected by anxiety, pain, excitement, infection,
c. Cuff then repumped to 10 to 30 mm Hg above presence of fever.
previously recorded point (maximum inflation 3. Rate, rhythm, quality (if strong, labored, shallow,
level). deep) should be noted MAINLY for those who
d. Pressure released slowly until systole (FIRST present with medical history of respiratory symp-
pulse sound) is heard. toms, e.g., asthma, emphysema, congestive heart
e. Continues to release pressure slowly until dias- failure, or allergic reactions.
tole (last pulse sound) is heard (must listen ALL 4. Unusual respiration rates:
the way down because at times there is a gap in a. Dyspnea: difficult or labored breathing.
the sounds). b. Hyperventilation: increased rate and depth of
f. Patient is informed of reading, which is re- respirations that result in decreased level of car-
corded in patient record. bon dioxide.
B. Pulse rate frequency of the heart beat or rate, mea- c. Kussmaul breathing: heavy, labored breath-
sured in beats per minute (BPM): ing, either rapid or slow; occurs during hyper-
1. Taken from radial artery if patient is conscious glycemic incident.
(easy access) and from common carotid artery in D. Temperature reading: taken with covered thermome-
unconscious patient (more accurate) for adults ter when needed because of signs of clinical infection
and brachial artery on infants but ONLY by EMS or inflammation:
system personnel; fingers used for palpation, since 1. Measurement in healthy patient: 97.0° to 99.6° F
thumb has own pulse. (36.1° to 37.6° C).
2. Taken for 30 seconds if within range (number is 2. Fever: elevated temperature by at least a full
multiplied by 2 if the rate is regular); if increased ­degree; occurs with infection, acute inflammation,
or irregular, taken again for 1 minute. shock, or changes in temperature exposure.
322    Saunders Review of Dental Hygiene

a. Hyperthermia: >105.8° F; with infection or 2. History of uncontrolled acute or chronic disease,


overexposure to heat. which includes unstable angina pectoris, MI/CVA
b. Hypothermia: <96.0° F; with shock or overex- within the last 4 to 6 weeks, severe CHF, moderate
posure to cold. to severe COPD, and uncontrolled HBP, epilepsy,
E. Pupil size: evaluate for signs of constriction, dilation, DM, or thyroid condition.
evenness during extraoral examination. 3. Pose significant risk, since patients in this category
1. Dilated pupils (mydriasis): indication of shock, have a severe medical problem of GREATER
heart failure, hallucinogens, or amphetamine use. importance to the patient than the planned dental
2. Constricted or pinpoint pupils (miosis): with mor- treatment.
phine, heroin, or barbiturate use. 4. Whenever possible, NO elective dental care should
3. Loss of evenness (two pupils with different sizes): be performed; should be postponed until medical
with ­concussion. condition has improved to at least ASA III; will
need medical consult before emergency treatment.
ASA Physical Classification E. ASA V: moribund; NOT expected to live longer than
Emergencies may be minimized or prevented by the pre- 24 hours, hospitalized, terminally ill; elective dental
treatment evaluation of physical classification status. The treatment is definitely contraindicated; however, emer-
ASA (American Society of Anesthesiologists) physical gency care or palliative treatment may be necessary.
status classification is used to determine risks and neces- F. ASA VI: died, and organs are to be harvested.
sary modifications before treatment. G. Emergency operation of any variety modifies classifi-
A. ASA I: healthy. cations (e.g., ASA III-E).
1. Able to walk up one flight of stairs or two level H. Those who demonstrate a MORE extreme anxiety
city blocks without distress. and fear toward dentistry have a baseline of ASA II
2. Little or no anxiety; little or no risk. even before their medical history is considered; situa-
B. ASA II: mild systemic disease or healthy ASA I who tion raises ASA one level each time.
demonstrates a MORE extreme anxiety and fear to-
ward dentistry. PREPARATION FOR MEDICAL EMERGENCIES s0050
1. Able to walk up one flight of stairs or two level IN DENTAL SETTING  
city blocks but will have to stop after completion When emergency situations occur, mandatory that ALL
because of distress. personnel in the dental setting know procedures to be fol-
2. History of well-controlled disease, which includes lowed, location and use of the emergency kit and oxygen,
non-insulin-dependent diabetes mellitus (DM), HBP, possibly a portable defibrillator and pulse oximeter, and
epilepsy, asthma, or thyroid conditions; ASA I with are able to provide basic life support (BLS). MAIN man-
a respiratory condition, pregnancy, active ­allergies. agement of emergency involves BLS; drugs delivered are
3. Minimal risk during treatment. of secondary importance. Kit can be assembled by person-
C. ASA III: severe systemic disease that limits activity nel or purchased as a unit and must not contain expired
but is NOT incapacitating. drugs. Personnel are MORE likely to be familiar with one
1. Able to walk up one flight of stairs or two level they have organized, which is MORE likely to contain
city blocks, but MUST stop enroute because of articles that meet needs of particular dental setting. Kit
distress. MUST be readily available during treatment time.
2. History of controlled chronic disease, which in- ALL staff must be trained to use emergency kit’s con-
cludes angina pectoris, myocardial infarction tents. Periodic emergency drills and posting the telephone
(MI), cerebrovascular accident (CVA), congestive numbers of EMS or other appropriately trained health-
heart failure (CHF) or cardiac surgery more than care providers is also recommended at the dental setting.
4 to 6 weeks in the past, slight chronic obstructive Many states require all dental settings to have portable
pulmonary disease (COPD), controlled ­insulin-de- automated external defibrillator (AED).
pendent DM/HBP (with drugs). A. Basic emergency kit: MUST include:
3. May need medical consult before elective 1. Noninjectable drugs:
­treatment. a. Nitroglycerin (sublingual tablets or nitrolingual
4. If dental care is indicated or allowed, stress reduc- aerosol spray) for angina attacks.
tion protocol and other treatment modifications are b. Bronchodilator (asthma inhaler).
indicated. c. Glucose source (e.g., orange juice, oral glucose
D. ASA IV: severe systemic disease that limits activity gel or tablets).
and is a constant threat to life. d. Aspirin, prevents additional platelet activation
1. NOT able to walk up one flight of stairs or two and interferes with platelet adhesion and cohe-
level city blocks; distress is present even at rest. sion with cardiac events.
Medical and Dental Emergencies    323

o0980 e. Secondary (additional): aromatic ammonia (amyl 4. NOT a complete measure of circulatory sufficiency;
nitrate), can be used to stimulate breathing. example: if insufficient blood flow or insufficient
o0990 2. Injectable (IV) drugs: hemoglobin in the blood (anemia), tissues can suf-
o1000 a. Epinephrine (1:1000; EpiPens are also recom- fer hypoxia despite high oxygen saturation in the
mended) for allergic responses. blood that does arrive.
o1010 b. Diphenhydramine (Benadryl), histamine blocker.
o1020 c. Secondary (additional): glucagon, hormone MEDICAL EMERGENCIES IN DENTAL SETTING    s0060
that converts stored glycogen into glucose and COMMON medical emergencies that may occur in the
releases it into the bloodstream. dental setting include syncope, hyperventilation, airway
3. Oxygen with positive pressure administration obstruction. After emergency, patient should be called at
­capability: home or at hospital later in the day to ensure well-being.
a. Oxygen cylinder (size E) SHOULD be avail- Cardiovascular and other less common emergencies in
able to deliver 100% oxygen for 30 minutes the dental setting are discussed later.
(5 liters/min with a nasal cannula).
b. For patient who is NOT breathing, delivery re- Syncope and Orthostatic Hypotension
quires pocket mask and demand-valve resuscita- A. Syncope (fainting): MOST common dental set-
tor (alternative is Ambu bag); full masks of clear ting emergency; MOST common cause of loss of
material that provide tight seal are necessary. consciousness.
B. AED: portable electronic device that automatically 1. MOST often associated with a form of stress.
diagnoses potentially life-threatening cardiac ar- 2. Caused by decreased oxygen flow to the brain,
rhythmias of ventricular fibrillation and ventricular which in turn is caused by:
tachycardia. a. Psychogenic factors: stress, unwelcome news,
1. Arrhythmia can be treated by application of elec- noxious smell or sight of a needle or blood
trical therapy, which stops arrhythmia, allowing (MOST common cause in dental setting).
heart to reestablish effective rhythm. b. Nonpsychogenic factors: poor health or exhaus-
2. When turned on or opened, unit will instruct user tion.
to connect the electrodes (pads) to patient: 3. Presyncope stage:
a. Once pads are properly attached, everyone a. Pallor, dizziness, and nausea.
should AVOID touching victim to prevent false b. Lowered blood pressure (hypotension) and in-
readings by unit. creased pulse rate (tachycardia).
b. Pads allow unit to examine electrical output 4. Syncope symptoms:
from heart and determine if patient is in viable, a. Loss of consciousness, shallow breathing, di-
shockable rhythm (either ventricular fibrilla- lated pupils.
tion or ventricular tachycardia); if device de- b. Lowered blood pressure (hypotension) and
termines that a shock is viable, will use battery thready pulse.
to charge internal capacitor in preparation to 5. As with ALL emergencies that involve uncon-
deliver shock. scious patient, may lead to airway obstruction and
C. Pulse oximeter: indirectly measures oxygen satura- death if improperly recognized and treated.
tion and changes in blood volume in skin, producing a 6. Treatment:
photoplethysmograph; useful for monitoring patients a. Place in Trendelenburg position (subsupine
during surgical procedures. with feet elevated higher than head), EXCEPT
1. BEST for simplicity and speed for patients with for pregnant patient (on side before feet are ele-
respiratory and cardiac problems. vated); usually ensures recovery (Figure 10-1).
2. Monitor displays percentage of arterial hemoglo- b. Maintain open airway to prevent airway ob-
bin in the oxyhemoglobin configuration (possibly struction by tongue; imperative for unconscious
heart rate); acceptable normal ranges are from 95% patient.
to 100%; for a patient breathing room air at not c. Use of aromatic ammonia inhalant (amyl ni-
far above sea level, estimate of arterial po2 can be trate), as needed, to provide stimulus for breath-
made from blood-oxygen monitor Spo2 ­reading. ing; caution is necessary during use to prevent
3. NOT a complete measure of respiratory suffi- burning of tissue.
ciency; example: patient with hypoventilation d. Administration of oxygen.
(poor gas exchange in the lungs) given 100% e. Carefully monitor until vital signs return to base-
­oxygen can have excellent blood oxygen levels line; mandatory because syncope is LIKELY
while still suffering from respiratory acidosis to recur soon after recovery; vomiting MOST
caused by excessive carbon dioxide. commonly occurs after unconsciousness.
324    Saunders Review of Dental Hygiene

b. Does NOT usually occur in children, since


anxiety is released by other means (e.g., crying,
refusing to cooperate).
c. May lead to syncope and then vomiting; vicious
cycle.
2. Treatment:
a. Calm and place in upright position; respiratory
emergencies SHOULD be managed with pa-
tient upright, if conscious.
b. Restore carbon dioxide:
(1) Have patient try to regulate breathing: in-
hale, hold breath, then exhale.
(2) Have patient breathe (called rebreathing)
into a paper bag or headrest cover (NOT
Figure 10-1  Trendelenburg position with patient in a sub- plastic); patient should hold bag and breathe
supine position and feet elevated higher than head.
6 to 10 times/min.
(3) ONLY emergency situation in which oxy-
gen does NOT benefit patient; further de-
f. NO further dental treatment should take place creases carbon dioxide levels, which then
for 24 hours. slows return to normal.
7. Prevention: place supine during treatment; use of c. Administration of tranquilizer drug such as di-
relaxation techniques and pharmacological means azepam (Valium) by supervising dentist in se-
such as use of antianxiety premedication such as vere cases, if available.
diazepam (Valium) and/or nitrous oxide sedation 3. Prevention: same as syncope. o9000
to relieve stress (see Chapter 14, Pain Manage- D. Airway obstruction or foreign body airway obstruc- o2110
ment). tion (FBAO):
B. Orthostatic (postural) hypotension: 1. Occurs MORE easily in dental setting because of o2120
1. May occur when patient sits up quickly from be- supine position; includes cotton rolls, extracted
ing in a supine position and may result in loss of teeth or fragments, restorative materials, vomit.
consciousness (second MOST common cause of 2. Types of FBAO: o2130
unconsciousness). a. Partial FBAO with adequate air exchange: o2140
a. Sitting upright or standing can lead to a drop (1) Occurs when airway is NOT completely o2150
in blood pressure; with standing, systolic drops blocked, allows patient to cough forcibly
at <25 mm Hg, diastolic drops at <10 mm Hg. and talk.
b. Highest risk: antihypertensives predispose (2) Requires NO treatment but must be moni- o2160
patient (see discussion on angina and nitro- tored closely.
glycerin). b. Partial FBAO with poor air exchange: o2170
c. Also at risk: patients who are elderly, pregnant, (1) Patient is NOT able to cough forcibly, and o2180
have Addison’s disease, or are under nitrous wheezing occurs during inhalation.
­oxide or diazepam (Valium) IV sedation. (2) May lead to complete FBAO and SHOULD o2190
2. Patient usually asymptomatic just before incident be treated the same (see next).
and returns to consciousness after lying down. c. Complete FBAO: o2200
3. Treatment: (1) Patient is NOT able to talk; may exhibit o2210
a. Check vital signs and administer oxygen at 5 universal distress signal of hands clasped at
liters/min through nasal cannula, if needed. throat.
b. Reposition slowly from supine position when (2) Symptoms include cyanosis (blue face from o2220
patient feels recovered. lack of oxygen) and loss of consciousness
4. Prevention: bring ALL patients slowly up after be- when the object is NOT removed.
ing supine; especially if at risk. d. Treatment for conscious adult or child with o2230
C. Hyperventilation: in dental setting usually related to complete or partial FBAO with poor air ex-
anxiety. change is the SAME:
1. Increased rate and depth of respirations, which re- (1) Ask “Are you choking?” and “Can you o2240
sults in decreased level of carbon dioxide. speak?”
a. Other symptoms: tightness of chest, sensation (2) Perform Heimlich maneuver: stand behind o2250
of suffocating, dizziness, tingling in fingers. patient, wrap both arms around the patient
Medical and Dental Emergencies    325

and deliver abdominal thrusts (position flat hand above first, interlacing fingers;
portion of one fist between navel and bot- push inward and upward five times;
tom of rib cage; wrap other hand around then move to head and open airway.
fist), push inward and upward until object is (c) After abdominal thrusts, repeat process o2300
expelled or patient becomes unconscious. of lifting chin, moving tongue, feeling
o2260 (3) For pregnant or obese patient: chest thrusts for and possibly removing the object.
in SAME manner on lower portion of the (d) If airway is still NOT clear, repeat ab- o2310
sternum. dominal thrusts as often as necessary; if
o2270 (4) Activate EMS system if patient becomes object has been removed but patient is
unconscious: still NOT breathing, provide BLS and
o2280 (a) Help patient to the floor or perform ab- start CPR as needed (see Table 10-1).
dominal thrusts in the chair, straddling (5) In ALL cases, referral to hospital emergency o2320
patient’s hips or standing close to hips unit because of risk of ­laryngeal edema.
at one side (if in dental unit). e. Prevention: use rubber dam and suction; exercise care o2330
o2290 (b) Place heel of one hand between navel when working with small objects in the mouth; do
and bottom of rib cage; position other NOT use instruments that can break off easily.

clinical study ��   

Age 33 YRS SCENARIO

Sex ☐  Male   ☒  Female The patient dreads dental appointments but cur-
rently experiences less anxiety because she has
BP 122/82 been following 6-month oral prophylaxis schedule
Chief Complaint “I just hate being here and I hate all and her appointments take less time. Her dental
that happens here!” health is good. After her medical history is updated
and her vital signs are obtained, bitewing radio-
Medical History Varicose veins removed 6 months ago graphs are taken. While the dental hygienist leaves
Knee surgery 5 years ago the operatory to discuss the case with the office
Achilles’ tendon rupture and repair manager, the patient remains seated in an upright
2 years ago position. Upon returning to the patient, the dental
hygienist notes that she is breathing rapidly, and
Current Medications None
she complains of tightness in her chest and tingling
Social History Physician at free clinic in her fingers.
Husband recently died in a freak
accident

1. What is the patient most likely experiencing? paper bag 6 to 10 times/min (known as rebreathing);
2. How should this emergency be handled? important to allow her to hold bag in position.
3. Is oxygen indicated? Why or why not? 3. Hyperventilation is one emergency in which admin-
4. What can further emergency complications can result istration of oxygen is not indicated; although oxygen
from this type of breathing? would not harm her, it would not benefit her.
4. May lead to syncope and then vomiting, which can
1. Probably experiencing hyperventilation, indicated become a vicious cycle.
by increased rate of respiration, dizziness, chest
tightness, tingling in fingers. Most often occurs in Allergic Reactions
the dental setting because of anxiety. Anxiety about Allergic reactions, including asthmatic attacks, are com-
dental appointments was a problem, although the mon in a dental setting. MOST are due to increased sen- p9000
patient believed it had lessened. Her recent personal sitivity to allergen (antigen, foreign) that has entered the
loss and stressful career must also be taken into body (type I [immediate] hypersensitivity).
­consideration. A. Allergic reactions: o9030
2. Treatment includes calming her and keeping her po- 1. Immune system then releases chemicals, such as
sitioned upright. Rapid rate of breathing results in histamine from mast cells, resulting in varying
decreased level of carbon dioxide. To restore carbon degrees of reaction.
dioxide, patient should inhale, hold her breath, and 2. Skin reactions generally are NOT considered true
then exhale. If this is ineffective, should breathe into emergencies; however, MUST be monitored for
326    Saunders Review of Dental Hygiene

potential advancement; reactions may be ­localized (3) With allergy to local anesthetic agent (rare),
or generalized, severity and type of reaction are MUST be determined if true allergic reaction
related to type and amount of allergen to which or some other adverse reaction occurred.
patient is exposed. (a) Allergic reaction occurs MORE often
3. If reaction occurs immediately after exposure, is in response to ester injectable local
MORE severe; when reaction occurs MORE than anesthetic agents (e.g., procaine was
60 minutes after exposure, LESS severe. removed from American market for
4. Types of hypersensitivity and allergic reactions: this reason); allergic reactions to amide
a. Contact dermatitis: occurs when skin is exposed agents are rare; NO longer using in-
to an allergen. jected ester agents, so has become rare
(1) Results in erythema, edema, vesicle forma- with injectables.
tion; usually acute and localized but can (b) Allergy to preservative sodium metabi-
spread. sulfite (antioxidant for the vasocon-
(2) Treatment includes removing cause; refer- strictor) must also be considered;
ral to dermatologist or allergist; supervis- using plain local anesthetic, such as 2%
ing dentist may administer an antihistamine ­lidocaine (Xylocaine), 3% carbocaine
(histamine blocker) such as diphenhy­ (Mepivacaine), or 4% prilocaine (Ci-
dramine (Benadryl) or chlorpheniramine tanest), without any vasoconstrictor will
(Chlor-Trimeton) to reduce reaction in an prevent almost ALL local anesthetic-
emergency situation. related allergic reactions.
b. Urticaria (hives): appears as raised areas of (c) Increased sensitivity to articaine (Sep-
edema, accompanied by pruritus (itching). tocaine) from sulfites in agent for some
(1) Caused by ingestion of food or drug or by patients with history must be consid-
direct skin contact with allergen. ered, MORE in asthmatic population;
(2) Treated by removing cause; referral to der- may want to AVOID use in sensitive or
matologist or allergist; supervising dentist asthmatic patients.
may administer antihistamine (histamine (d) Localized reaction to topical benzo-
blocker), such as diphenhydramine (Benad- caine is also possible, since this is an
ryl) or chlorpheniramine (Chlor-Trimeton), ester; will need to use amide (lidocaine
to reduce reaction. or prilocaine) for topical use instead
c. Angioedema: initially may be mistaken for (see earlier discussion).
hives. (4) Symptoms affect different body systems and
(1) Localized, unilateral, colorless and painless may occur separately or simultaneously:
swellings of face or neck without well-defined (a) Begins rapidly as a skin response (pru-
borders of hives; allergic reaction to food or ritus, urticaria, angioedema); angio-
drug; affects tissues of the hands, face, geni- edema in mouth may result in airway
tals; rarely results in pain or itching. obstruction.
(2) Treated by removing cause; referral to derma- (b) Gastrointestinal system may respond
tologist or allergist, supervising dentist may with spasms, cramps, nausea, diarrhea;
administer antihistamine, such as diphen- respiratory system exhibits varying
hydramine (Benadryl) or chlorpheniramine degrees of laryngeal edema, which is
(Chlor-Trimeton), to reduce reaction; local- MOST common cause of death from
ized, mild allergic reactions should be treated anaphylaxis.
with oral histamine blocker for minimum of (c) Circulatory system involvement in­
3 days because of risk of ­recurrence. clu­des hypotension, shock, cardiac
d. Anaphylaxis (anaphylactic reaction): MOST arrest.
severe allergic reaction. (5) Treatment for anaphylaxis:
(1) Level depends on amount of allergen to (a) Place in supine position, administer ox-
which patient is exposed, acquired sensitiv- ygen at 5 liters/min through nasal can-
ity, route of entry; occurs almost immedi- nula; provide BLS as needed.
ately after exposure; may be fatal despite (b) Activate EMS system; epinephrine ad-
treatment. ministration (0.3 mg every 5 minutes
(2) Can be caused by foods, drugs (MOSTLY for adults, 0.15 mg for children; in
penicillin and related antibiotics), rubber some cases of systemic allergic reac-
latex, or other environmental factors. tions, clinical signs of bronchospasm
Medical and Dental Emergencies    327

and hypotension persist after FIRST


dose of epinephrine) by supervising CLINICAL STUDY   
dentist when diagnosis is definite. Scenario: A healthy 12-year-old boy recently experienced
(c) Because of urgent nature of epinephrine trouble breathing after eating a peanut butter sandwich. His
administration in anaphylactic shock, mother relates this experience at his next dental appoint-
prefilled syringes provide quicker, ment when asked if there has been any change in his medi-
MORE foolproof method for adminis- cal history. She says that his symptoms included wheezing
tering; autoinjectors (EpiPen) are avail- and a swollen face. His parents took him to the local hospi-
able to inject single adult or pediatric tal emergency unit, where his blood pressure was 60/40 mm
dose when pressed against the thigh Hg and his breathing was labored. He received an injection
(even through clothing) by patient or of epinephrine (adrenaline), and minutes later his blood
supervising dentist. pressure normalized and his breathing difficulty abated.
(d) Since MORE than one dose may be
needed, must have MORE than one 1. Describe the pathogenesis and etiology of the young
dose of epinephrine available in the boy’s clinical presentation.
kit (see earlier discussion); referral to 2. What lab tests would help confirm the diagnosis?
emergency unit via ambulance is es- 3. What role did epinephrine play in facilitating his
sential. recovery?
B. Asthma attack: respiratory emergency that affects 4. What preventive measures should he and his mother
individuals of ALL age groups, often is triggered by take in the future?
anxiety; may occur suddenly or slowly and may be
allergic or nonallergic in nature. 1. Patient had an allergic response to peanuts, based on
1. Identification requires distinguishing asthma his symptoms and the case history. Therefore the pre-
(wheezing upon expiration) from airway obstruc- sumptive diagnosis is anaphylactic response. Patho-
tion (wheezing upon inspiration). genesis is that of a type I (immediate) hypersensitivity
2. If medical records indicate history of asthma, pa- response: after having become sensitized to peanut
tient is MORE likely experiencing an asthmatic antigens, upon subsequent exposure he produced an
attack. IgE response that triggered anaphylaxis.
3. Involves narrowing of bronchioles (airway) in re- 2. Tests to confirm and further explore this diagnosis
sponse to overproduction of mucus and smooth would include skin test with peanut antigens (to see
muscle contraction. whether responds with wheal and erythema) and ra-
4. Results in difficulty breathing because IgE causes dioallergosorbent test (RAST), which specifically
histamine release from mast cells, then affects re- seeks out blood IgE antibodies that are reactive with
spiratory system. particular antigens.
5. Symptoms of asthma attack: sweating, coughing, 3. Epinephrine was the solution to his emergency because
nervousness, tightness of the chest, struggling for this drug causes decrease in mast cell degranulation,
air (wheezing on expiration). vasoconstriction, and maintenance of blood pressure
6. Treatment for asthma attack: and respiratory rate, resulting in rapid inhibition of type
a. Position upright. I allergic or hypersensitivity response. Other drugs that
b. Use bronchodilator (patient’s if possible and could be administered as second line of treatment in-
is current; from emergency kit, if needed); clude corticosteroid, such as prednisone, and broncho-
dilates bronchi and bronchioles, increasing dilators in inhalers such as albuterol (Preventil).
airflow; for quick relief, use of short-act- 4. Future medical precautions would include avoid-
ing β2-agonists (SABAs) such as albuterol ing peanuts and carrying self-injectable emergency
(Proventil). epinephrine pen (EpiPen), because peanuts may be
c. Administration of oxygen at 5 liters/min hidden component of some foods (especially ethnic
through nasal cannula, allowing patient to posi- foods) and thus difficult to avoid entirely.
tion mask.
d. Administration of epinephrine by supervising Cardiovascular Emergencies
dentist if bronchodilator is NOT effective (see Although NOT as common as the medical emergencies
above discussion). discussed previously, emergencies related to the cardio-
e. Provide BLS and/or referral to hospital emer- vascular system (CVS) can occur; they are the MOST
gency unit, if necessary. serious. CVS emergencies include angina pectoris, heart
7. Prevention: AVOID trigger for attack if possible; attack, cardiac arrest, and stroke and can occur in the den-
prepare for emergency; reduce patient stress. tal setting. BLS is also discussed (Table 10-1).
328    Saunders Review of Dental Hygiene

Table 10-1  Basic life support (by lay rescuer), ABCDs (see notes below)

ABCDs Adult CPR Child CPR (1 to Infant CPR (less


(8 years and older) 8 years old) than 1 year old)*

Airway Establish unresponsiveness; Establish unresponsiveness; call Establish unresponsiveness; call


activate EMS system, and for help. Open airway using for help. Open airway using
locate AED if possible. head-tilt, chin-lift. head-tilt, chin-lift.
Open airway using head-
tilt, chin-lift.
Breathing† Determine breathlessness Determine breathlessness Determine breathlessness
within 5 to 10 seconds. within 5 to 10 seconds. No within 5 to 10 seconds. No
No breathing: give two breathing: give two ventila- breathing: give two ventila-
ventilations (1 second tions (1 second per breath) tions (1 second per breath)
per breath) with enough with enough volume to with enough volume to
volume to produce visible produce visible chest rise. produce visible chest rise.
chest rise.
Circulation‡ Begin compressions, press- Begin compressions, pressing Begin compressions, pressing
ing 1½ to 2 inches in ¹⁄³ to ½ of chest depth in ½ to 1 inch of chest depth,
center of chest, between center of chest, between just below nipple line, us-
nipples, using two hands, nipples, using two hands, ing two fingers. Give 100
with heel of one hand, with heel of one hand, sec- compressions per minute,
second hand on top. Give ond hand on top. Give 100 allowing chest to recoil,
100 compressions per compressions per minute, 30 compressions followed
minute, allowing chest to allowing chest to recoil; by two breaths. After five
recoil; 30 compressions 30 compressions followed cycles, activate
followed by two breaths. by two breaths. After five EMS system, continue with
Continue until AED ar- cycles, activate EMS. Con- compressions and breathing
rives, there is movement, tinue until AED arrives, there until EMS arrive.
or EMS arrive. is movement, or EMS arrive.
Definitive Use adult pads. When Use child pads if available, Not recommended to use AED
care/defibrillation attempting defibrillation adult pads when not. When because of age.
with AED, deliver one attempting defibrillation
shock followed by im- with AED, deliver one shock
mediate CPR, beginning followed by immediate
with chest compressions. CPR, beginning with chest
Check rhythm after giving compressions. Check rhythm
five cycles (about after giving five cycles
2 minutes). (about 2 minutes).

AED, Automated external defibrillator; CPR, cardiopulmonary resuscitation; EMS, emergency medical services.
*Differentvalues for newborn infant.
†Breathing is performed only on adults who experience lack of oxygen because of drowning, drug overdose, or carbon monoxide poisoning; breathing is still used
on children, since they are more likely to have trouble breathing if they unexpectedly collapse.
‡To maximize effectiveness of compressions, victim should lie supine on hard surface (e.g., backboard or floor), with rescuer kneeling beside victim’s thorax.

A. Angina pectoris: chest pain caused by decreased oxy- (2) Ointment, patch, or pill forms of NTG may
gen flow to the heart (anoxia). be prescribed by physician to prevent or
1. Prevention: remind patients with history of angina reduce number of angina episodes; NOT
to bring own nitroglycerin (NTG) to dental ap- recommended for rapid treatment of angina
pointment (watch expiration of patient’s supply). because work slowly; MUST use sublingual
2. Symptoms: NTG for quick relief.
a. Includes chest pain, which may spread to jaws b. Unstable angina: can occur when victim is at rest.
and teeth, even edentulous arches. (1) Episodes have greater intensity, longer du-
b. Lasts 3 to 5 minutes after cause of the pain is ration, MORE frequent occurrence.
removed; person will remain still in attempt to (2) NOT relieved by administration of NTG.
relieve pain. c. Treatment for either type of angina in dental
3. Types: setting:
a. Stable angina: often caused by emotional upset (1) Calm patient and position as comfortably as
or physical exertion. possible.
(1) With constant duration and intensity of pain, (2) Administer oxygen at 5 liters/min through
quickly relieved by administration of NTG. nasal cannula, as needed.
Medical and Dental Emergencies    329

(3) Administer NTG sublingually (beneath d. Administer oxygen at 5 liters/min through nasal
tongue), as needed; works as a vasodilator; cannula.
emergency kit should contain it: e. Provide BLS as indicated (Table 10-1).
(a) Sublingual tablet has shorter shelf-life; f. May need administration of morphine sulfate
may be used for 6 months after con- IV by supervising dentist to alleviate pain and
tainer opened; metered spray form has attendant anxiety, if available.
longer shelf-life; one metered spray is 6. Considerations after MI:
equal to 1 tablet. a. Withholding emergency or elective surgery (or o2790
(b) Ammonia inhalant (amyl nitrite) can be periodontal procedures by dental hygienist) for
used; provides significant vasodilation 4 to 6 weeks and then determining functional
in 10 seconds (however, side effects are capacity (FC).
greater). b. Dental care should be delayed unless patient o9010
(4) Usually 1 to 3 sublingual tablets (or SAME can meet 4 METs (metabolic equivalents) and/
number of sprays) relieve pain in 1 to 2 min- or further medical testing has been completed
utes; BEST to follow patient’s instructions to quantify level of cardiac risk in treatment;
for NTG use; pain should NOT return. medical consult before ALL dental treatment.
(5) Side effects include orthostatic (postural) c. May start taking drugs, including anticoagu-
hypotension and headache resulting from lants (heparin [Calciparine, Liquaemin] and
antihypertensive effects. warfarin [Coumadin]); antiplatelets (aspirin
(6) If NO relief occurs after tablets or sprays and clopidogrel [Plavix], dipyridamole [Ag-
or if angina returns after relief, MI may be grenox, Persantine]); digitalis (Crystodigin,
occurring. Lanoxin); and NTG; these have associated side
B. Myocardial infarction (MI, heart attack): also known effects (e.g., increased bleeding) (see Chapter
as coronary thrombosis or coronary occlusion (acute 9, Pharmacology).
coronary syndrome) (depending on etiology). C. Sudden cardiac arrest (SCA, sudden death): abrupt,
1. Death of portion of heart muscle (irreversible ne- unexpected cessation of breathing and circulation.
crosis), caused by decrease or lack of oxygen flow 1. Causes of SCA:
to that part of heart muscle (prolonged ischemia). a. Cardiovascular disease (CVD), drug overdose,
2. May be caused by atherosclerosis with occlusion electrocution, drowning.
(blocking) of coronary arteries or by thrombosis b. Anaphylactic shock, hypoxia.
(blood clot) in artery that supplies heart muscle. 2. Symptoms of SCA: NO pulse or respiration, cya-
3. INCREASED risk with stressful situation; how- notic tissues, dilated pupils, unconsciousness.
ever, MOST often occurs while victim is at rest; 3. Treatment of SCA for adult, child, or infant: pro-
may follow preexisting anginal condition. vide BLS through CPR (Table 10-1).
4. Symptoms of MI:
a. Diffuse crushing pain in chest, arm, neck, Cerebrovascular Accident
shoulder, lower jaw, especially left side, since A. Cerebrovascular accident (CVA, stroke): classified
pain can spread from chest to other areas. according to cause, such as cerebral embolism, hem-
b. Pain is of greater intensity and duration than orrhage, infarction, or thrombosis.
angina pectoris; LESS intense symptoms for 1. Symptoms: headache together with confusion, im-
women. paired speech, paralysis, unconsciousness; vary in
c. Patient typically moves around in attempt to al- relation to area affected and type of CVA and do
leviate discomfort; other symptoms include dizzi- NOT regress.
ness, shortness of breath, cold and clammy skin, 2. CVA is NOT a common dental office emergency;
weakness, fatigue, anxiety, denial of possible MI. predisposing factors include HBP, DM, CVD, and
5. Treatment for MI: history of transient ischemic attacks (TIAs).
a. Position however patient is MOST comfortable. 3. Treatment for CVA:
o2730 b. Administer NTG if have not done already; an- a. If patient is conscious, allow to remain upright;
gina pectoris that is NOT relieved by NTG may if unconscious, position supine, with head
lead to cardiac arrest; if conscious, patient can slightly elevated.
be asked to chew aspirin (at least 160 mg and b. Activate EMS system.
up to 325 mg), which prevents additional plate- c. Monitor vital signs (blood pressure will be el-
let activation and interferes with platelet adhe- evated).
sion and cohesion. d. Administer oxygen at 5 liters/min through
c. Activate EMS system. nasal cannula to unconscious patient; for
330    Saunders Review of Dental Hygiene

conscious, administer oxygen ONLY when c. Physical limitations during appointment (under-
respiratory ­ difficulty exists; provide BLS as standing, speech, and movement) and with oral
needed. hygiene care treatment by drugs such as anticoag-
4. Other considerations after CVA: ulants (heparin [Calciparine, Liquaemin], warfarin
a. High incidence of recurrence; thus patients with [Coumadin]), antiplatelets (aspirin and clopidog-
history pose greater risk in the dental office; rel [Plavix], dipyridamole [Aggrenox, Persantine])
management involves recording vital signs at with possible side effects (increased bleeding).
each appointment; high blood pressure readings B. Transient ischemic attack (TIA, temporary or incipi-
are of concern. ent stroke):
b. Withholding emergency or elective surgery (or 1. Similar to a CVA yet lasts for ONLY few minutes;
periodontal procedures by dental hygienist) symptoms regress, NO permanent neurological
for 4 to 6 weeks and/or further medical testing damage results.
has been completed to quantify level of risk in 2. NOT mini-strokes but precursors to CVA; patient
treatment; medical consult before ALL dental remains conscious but experiences confusion.
­treatment. 3. MUST make medical referral.

CLINICAL STUDY�  

Age 54 YRS SCENARIO

Sex ☒ Male��   ☐  Female The patient is in the dental office for his recall oral
prophylaxis appointment. Shortly after the patient
Height 5’10” is placed in a reclining position and treatment is
Weight 195 LBS begun, the patient asks to sit up. He appears pale,
is short of breath, and says that his chest hurts. He
BP 158/94 takes 3 tablets of nitroglycerin sublingually, but his
chest still hurts.
Chief Complaint “I feel fine and can’t wait anymore
to have my teeth cleaned, since it has
been a whole year.”

Medical History Myocardial infarction 5 years ago


Past cigar smoker
Bypass heart surgery 4 years ago
Angina pectoris (occasional)

Current Medications nifedipine ­(Procardia) 10 mg tid


­nitroglycerin sl prn

Social History Tax accountant

1. What term describes the pain in the patient’s chest? While patient is experiencing angina, should be placed
2. How should this condition be treated? in whatever way patient is most comfortable. Oxygen
3. What is indicated by the continued presence of chest should be administered to any patient who is experi-
pain after the patient’s use of nitroglycerin tablets? encing chest pain. Can also use metered NTG spray
4. What type of drug is nitroglycerin? form (1 spray equals 1 tablet). Many patients carry ex-
5. What other drug should be administered? pired drugs, so it is important to have these emergency
6. Describe how this situation is managed if the patient drugs ready in the emergency kit in a dental office.
loses consciousness. 3. When angina pectoris is not relieved after three
NTG tablets or spray cycles, myocardial infarction
1. Chest pain is angina pectoris and is caused by de- (MI, heart attack) must be considered. Although
creased oxygen supply to the heart. Pain may spread this patient does exhibit predisposing factors for a
to jaws and teeth, even edentulous arches. cerebrovascular accident (stroke, CVA), such as hy-
2. Treatment of angina includes administration of nitro- pertension (high blood pressure [HBP]) and cardio-
glycerin (NTG) tablets beneath tongue (sublingual). It vascular disease (CVD), presence of nonresponding
is best to follow patient’s instruction for ­administering angina pectoris is strongly suggestive of MI and not
drug, if possible. Relief often results within 90 ­seconds. of CVA.
Medical and Dental Emergencies    331

4. A vasodilator and the most effective drug in the treat- (3) Postictal stage: patient sleeps soundly after
ment of angina, NTG increases blood flow to coronary seizure activity (10 to 30 minutes); provide
arteries by decreasing coronary artery resistance. It BLS if needed, and may be critical at this
may result in orthostatic (postural) hypotension and/or time; snoring during sleep indicates that
headache. airway is partly blocked; gurgling sound
o9020 5. If conscious, patient can be asked to chew aspirin (at indicates that airway MUST be suctioned
least 160 mg and up to 352 mg), which prevents ad- for vomitus or blood, with confusion and
ditional platelet activation and interferes with platelet disorientation; full recovery takes several
adhesion and cohesion. hours.
6. If patient loses consciousness, management includes b. Petit mal (absence) seizure: less severe type,
BLS of cardiopulmonary resuscitation (CPR). First, less common type.
the dental setting must activate EMS system; posi- (1) Occurs MORE often in children than in
tion patient supine, open airway, assess breathing, adults.
and begin chest compressions as needed. Chest (2) Involves LOSS of awareness of surround-
compressions are given on center of chest, between ings for a few seconds; victim stares
nipples, pressing 1½ to 2 inches and watching chest blankly and experiences twitching or rapid
recoil between compressions; must give 100 com- ­blinking.
pressions per minute. Breathing is performed only c. Partial seizure: jerking of one side of body
on adults who experience lack of oxygen because of because ONLY one hemisphere of brain is
drowning, drug overdose, or carbon monoxide poi- ­involved, trancelike appearance and engage-
soning; breathing is still used on children, since they ment in purposeless activity.
are more likely to have trouble breathing if they un- d. Hysterical seizure:
expectedly collapse. To maximize effectiveness of (1) Occurs when there is an audience and used to
compression, victim should lie supine on hard sur- manipulate others, indicates that this seizure
face (e.g., backboard or floor), with rescuer kneeling can be triggered by emotions and anxiety.
beside victim’s thorax. (2) Involves movements that are MORE con-
trolled (e.g., repeated up-and-down move-
Seizures, Hypoglycemia, and Hyperglycemia ment of one arm).
Other common medical emergencies that may be encoun- e. Status epilepticus:
tered in the dental setting include seizures, ­hypoglycemia, (1) With SAME symptoms present during con-
hyperglycemia. vulsive stage of grand mal, but symptoms
A. Seizures: last longer than 5 minutes; uninterrupted,
1. MOST caused by epilepsy; other causes include with NO recovery between episodes; may
prolonged high fever, alcohol or drug withdrawal, last for hours or days.
trauma to head, congenital abnormalities, imbalance (2) MAJOR cause of death is related directly
of body fluids; in dental setting, seizures caused by to seizures, MUST have immediate medical
epilepsy may be triggered by stress. referral.
2. Seizures are categorized according to the area of 3. Treatment for seizure(s): MAIN objective is to
brain affected and associated symptoms: prevent patient from becoming hurt and provide
a. Grand mal seizure: MOST severe type, MOST support afterward.
common type; has three phases: a. Other considerations:
(1) Prodromal phase (early symptom of dis- (1) Remove dental objects from mouth if pos-
ease): occurs before seizure activity, may sible (removable appliances are NOT re­
result in a personality change, may include commended; may cause trauma or may be
aura (scent, flicker of light, or noise) that is aspirated during seizure activity); do NOT
unique for that person (few seconds to sev- place fingers in the mouth of a patient hav-
eral hours). ing a seizure.
(2) Convulsive (ictal) phase: loss of conscious- (2) If anticipating a seizure (noting aura) dur-
ness, associated with an epileptic cry caused ing prodromal phase ONLY, can place bite
by air rushing from the lungs, with tonic block or mouth prop.
movements that occur as the body becomes (3) Clear area surrounding patient; NO attempt
rigid (10 to 20 seconds) and clonic move- should be made to restrain patient.
ments that result from the contraction and (4) Place conscious patient on side afterward;
relaxation of muscles and cause patient to for unconscious patient, place patient in su-
jerk violently (2 to 5 minutes). pine position and maintain open airway.
332    Saunders Review of Dental Hygiene

(5) Patient should NOT be given anything to be placed in the mouth in mucobuccal
eat or drink; also should NOT be permitted fold.
to drive afterward. (c) When patient is unconscious for >30
(6) Activate EMS system if injury occurs seconds, seizure activity is MORE likely
during seizure or if breathing is NOT ���� to occur; if seizure occurs, must be man-
present. aged before hypoglycemia is treated.
b. Drugs patient might be taking include anticon- c. Then activate EMS system, as needed.
vulsants, such as phenytoin (Dilantin), which C. Hyperglycemia (high blood sugar): LESS common
may result in gingival overgrowth. emergency with DM than hypoglycemia.
B. Hypoglycemia (low blood sugar): increased risk with 1. FIRST indication of disease; reason DM is diag-
history of DM; MORE common emergency with DM nosed in patients.
than hyperglycemia. 2. Occurs in response to little or NO insulin or when
1. Caused by skipped meal, excess insulin, increased glucose level is too high; may occur with DM or
exercise, or change in routine; onset rapid for pa- with undiagnosed DM; urine samples indicate high
tient who requires insulin; symptoms occur MORE glucose level and presence of ketones.
slowly for individuals who take oral hypoglycemic 3. Infection, stress, illness, failure to comply with
drugs. therapy, and other factors that increase need for
2. With cold sweat, headache, trembling, weak- insulin may lead to this condition.
ness, possible personality change, nervousness, 4. At an emergency level, patient has increased thirst
and ­ confusion; confusion may be mistaken for and urination, loss of appetite, nausea, vomiting,
­inebriation. Kussmaul breathing, dry warm skin, and fruity,
3. Can cause potential fatal insulin shock; although sweet odor of the breath; heart rate is rapid and
can be experienced by anyone, MOST occurs with weak, and blood pressure is lower than normal;
type 1 DM. when these symptoms are present, NO further
4. Treatment for hypoglycemia: dental treatment is performed and medical ­consult
a. If patient is conscious: give glucose ­replacement is needed; emergency situation; if untreated,
(see earlier discussion about kit). may lead to unconsciousness (diabetic coma or
b. If patient unconscious: ­ketoacidosis):
(1) Position supine, with feet elevated; ­normally, a. Activate EMS system; patient typically is seen
breathing and circulation are maintained; immediately by physician and hospitalized;
pulse rate is MORE rapid but blood pres- condition may advance to diabetic coma.
sure is NOT affected. b. If patient’s skin is dry and has ketoacidic smell,
(2) Because glucose CANNOT be adminis- administration of glucose IV or IM may be
tered to unconscious patient, administration ­lethal.
of glucagon (hormone that converts stored 5. When it is NOT known whether nature of emer-
glycogen into glucose and releases it into gency is hyperglycemia or hypoglycemia, condi-
the bloodstream) IV or IM by supervising tion is treated as hypoglycemia by EMS personnel,
dentist, if available. since if NOT treated quickly, may lead immedi-
(a) Response occurs in 10 to 15 minutes. ately to serious damage or death; alternatively,
(b) If parenteral administration is NOT pos- permanent disability or death takes much longer to
sible, small amounts of glucose gel can occur with hyperglycemia.
Medical and Dental Emergencies    333

Clinical study�  

Age 65 YRS SCENARIO

Sex ☒  Male   ☐  Female The patient is following a 6-month recall


schedule for oral prophylaxis and ­examination.
Height 6���
’��
2�
” His intraoral examination notes slight to
Weight 245 LBS moderate levels of dental biofilm and calculus
deposits, with slight gingivitis.
BP 144/80

Chief Complaint “I am a little tired from all my travel


this week, and the girls are keeping
me up.”

Medical History Type 1 diabetes mellitus for 45 years,


well controlled
Secondary hypertension for 2 years

Current Medications U-100 insulin (Humulin 70/30) bid


hydrochlorothiazide (Lopressor) 100
mg bid

Social History Sales executive who recently adopted


twin baby girls from overseas with his
second wife

1. Which of patient’s medical conditions are associated 3. If unconsciousness occurs because of hypoglyce-
with potential emergency? mia, patient will be unable to take glucose source
2. Describe potential emergencies the dental hygienist and injection of glucagon IV or IM will be necessary
should be aware of for this patient. by supervising dentist, if available. When chronic
3. Is the blood pressure recorded for this appointment ­hyperglycemia occurs in uncontrolled or noncompli-
within normal range? ant diabetics, patient presents dry warm skin, Kuss-
maul breathing, fruity breath odor, increased thirst and
1. Hypertension (high blood pressure [HBP]), diabetes urination. Factors that increase need for insulin, such
mellitus (DM), and antihypertensive drugs pose risks as stress, illness, or infection, will cause hyperglyce-
and may lead to an emergency. mia. If these symptoms occur, dental treatment should
2. Several emergencies are possible with this patient. be stopped and medical consult should be sought. Will
A common emergency in the dental setting involves pa- result in diabetic coma if not resolved. If the patient’s
tient positioning. Antihypertensive drugs may increase skin is dry and patient has ketoacidic smell, adminis-
chance of loss of consciousness caused by orthostatic tration of glucose IV or IM may be lethal. Presence of
hypotension (lowering of blood pressure from remain- both HBP and DM also is a concern. Cerebrovascular
ing in one position for significant period). Patients may accident (CVA, stroke) does not commonly occur in
experience dizziness, lightheadedness, loss of balance, the dental setting, but having two predisposing factors
or loss of consciousness when raised from supine po- (HBP and DM) does increase a patient’s risk for this
sition or when they stand up on their own. Important occurrence.
to raise patients slowly and allow them to sit upright 4. Blood pressure reading of 144/80 indicates higher than
before being dismissed. Potential for hypoglycemia or normal systolic reading (range is <120 mm Hg) but di-
hyperglycemia (low or high blood sugar, respectively) astolic is within normal range (<80 mm Hg). Because
is also of concern for patients who have DM. Individu- systolic reading is above 140, second blood pressure
als with insulin-dependent DM are more likely to have reading should be taken to check accuracy; patient
hypoglycemia. Conscious patients who have episode may have “white-coat” syndrome or raced to appoint-
of hypoglycemia demonstrate quickly occurring symp- ment. Reading is below level that necessitates a medi-
toms of cold sweat, headache, trembling, weakness, cal consult (160 to 179/100 to 109 mm Hg or greater).
personality change, confusion. Source of glucose, such Patient may have stage 1 (mild) high blood pressure
as orange juice or other glucose source replacement, (HBP) (140 to 159/90 to 99 mm Hg). However, must
should be given to reverse symptoms. Hypoglycemia refer to physician for follow-up medical consult on
may result in insulin shock if not resolved. HBP, especially with patient’s medical history.
334    Saunders Review of Dental Hygiene

c. Bleeding levels monitored by international nor-


COMMON DENTAL EMERGENCIES   malized ratio (INR); INRs ≤2.5 are safe for in-
Dental emergencies occur as result of dental treatment or vasive dental work.
because treatment is needed. Include avulsed tooth, dis- (1) Older test is prothrombin time (PT), broad
placed tooth, postsurgical hemorrhage, broken instrument measurement of time it takes blood to clot;
tip, foreign body injuries, needlestick, burns. normal PT levels were 11 to 13.5 seconds.
• See Chapters 6, General and Oral Pathology: dry (ne- (2) Result of a PT test is now converted into
crotic) socket and Ludwig’s angina; 8, Microbiology standard INR units that can be compared
and Immunology: standard precautions, needlestick regardless of the reagent used; MOST com-
protocol, sharps and waste handling; 13, Periodontol- mon INR target range for patient taking an-
ogy: periodontal and pericoronal (pericoronitis) ab- ticoagulants is 2 to 4; INR of 5 or more is
scesses; 14, Pain Management: needle breakage. typically avoided because risk of bleeding
A. Avulsed tooth: completely removed from socket be- increases significantly.
cause of trauma, may involve moderate pain or NO d. Instruct patient (both written and oral instruc-
pain. tion) to refrain from tobacco and alcohol use
1. Immediate examination in dental setting or emer- and AVOID rinsing area, drinking from straw,
gency hospital unit by a dental professional. or strenuous exercise for 24 hours.
2. Ask patient to bring tooth along in a wet handker- 4. Treatment: control by biting on folded gauze,
chief, milk, or water or in buccal vestibule (pre- folded tissue, or tea bag (theophylline is a vaso-
ferred mode of transport because of maintenance constrictor).
of hydration in saliva); purchased transport me- D. Broken instrument tip: requires removal from mouth.
dium for sport teams is now available. 1. Clinician stops procedures, keeps patient calm,
3. Radiograph of area to identify any remaining frag- and asks patient NOT to swallow.
ments; reimplantation of tooth and placement of 2. Recovery of instrument tip:
splint. a. Visually examine area FIRST, then dry with
4. BETTER prognosis results when there is short cotton roll (NOT with air syringe).
time between avulsion and reimplantation. b. Examine sulcus with gentle strokes of a curet in
5. Root canal therapy, as indicated; observation of a spoonlike manner; tip should NOT be pushed
other trauma and tetanus booster may be indicated. into base of sulcus.
B. Displaced tooth: loosened in socket and visibly c. If tip is NOT located, expose periapical radio-
moves, usually associated with pain. graph of suspected location of broken tip.
1. Immediate examination in dental setting or emer- d. If tip is still not located, must have immediate
gency hospital unit by a dental professional. medical referral and radiographs of chest and
2. Radiograph of area to check for root fracture; ap- gastrointestinal area; policy is essential for the
plying splint depending on severity of displace- management of this emergency; SHOULD in-
ment; instructing NOT to chew or bite with tooth. clude an outline of the steps to be followed;
3. Observation of other trauma; identification of need ALL personnel should be familiar with this
for tetanus booster; application of cold packs to policy.
minimize external swelling. E. Foreign body injuries to patient:
C. Postsurgical hemorrhage: 1. Injuries caused by objects that are foreign to a par-
1. Primary bleeding: within 24 hours after extraction. ticular part of the body; in dentistry, MOST often
2. Secondary bleeding: 24 or more hours after affect the eyes and respiratory tract (see earlier
extraction. discussion).
3. Prevention: 2. Typically involve polishing agents, calculus,
a. Awareness of medical conditions that could cause restorative materials, chemicals, or extracted
problems (updating medical history), may be tak- teeth.
ing anticoagulant drugs (“blood thinners”) such 3. Prevention:
as aspirin, warfarin (Coumadin, Dicumarol). a. Protective eyewear SHOULD be worn by pa-
b. American Dental Association has stated that these tient during treatment.
rarely need to be discontinued before MOST den- b. Children must�������������������������������
�����������������������������������
be attended to at ALL times.
tal procedures because adverse reactions could c. Clinician MUST have knowledge of equipment
occur: there is greater risk of thromboembolic being used and rotation direction of rubber cup
events than uncontrollable bleeding if medica- when polishing.
tion is temporarily stopped; if risks of drug are d. Patient should be informed of ALL procedures
considered too great, need medical consult. and role in prevention.
   Medical and Dental Emergencies    335

e. Throat pack of 4 × 4 gauze should be used to 1. Classified according to their source (e.g., heat or
protect pharynx. chemicals) or depth:
f. Clinician must use care when handling instru- a. Superficial (first-degree) burns involve ONLY
ments and polishing pastes; pastes and other epidermis, top layer of skin, with redness, dry-
solutions should NOT be held directly over ness, pain.
patient’s face. b. Deep (second- and third-degree burns) in-
g. Thin instruments SHOULD be replaced to pre- volve BOTH layers of skin, epidermis and
vent breakage; rubber dam SHOULD be used dermis.
as needed during treatment. (1) With redness and blisters that may open and
4. Treatment: weep, causing a wet appearance, swelling
a. For cases involving aspiration, radiographs of results and is painful, since exposed nerve
the chest and gastrointestinal area may be nec- endings are sensitive to air; scarring may
essary to locate the object. See earlier discus- occur during healing.
sion of respiratory obstruction (FBAO). (2) These wounds often become secondarily
b. If object has entered the eye (reacts by tearing infected.
and blinking): ask patient to look down and po- 2. Basic steps of treatment:
sition upper lid over lower lid for 1 second; then a. Small superficial (first-degree) burns SHOULD
pull lid upward. be immersed in cool water or ice to stop burn
(1) If object or particle can be seen in lower lid, (ice can be used ONLY for this type of burn);
use moistened cotton applicator to retrieve clean area and cover with antibiotic ointment;
object and flush eye using eye cup or eye most often does NOT require medical attention.
washing station. b. Deep (second- and third-degree) burns must
(2) Must have immediate medical referral, es- have immediate medical referral:
pecially if object CANNOT be removed; (1) Cool area with large amounts of cool water.
to prevent rubbing the eye, stabilize gauze (2) Remove clothing from area carefully; how-
over eye with adhesive tape. ever, cloth that sticks to the burn should
F. Wounds that break the skin of clinician: NOT be removed.
1. Slippage of instruments during treatment or (3) Ointments, butter, oil, and other remedies
cleanup; dull instruments; sudden patient move- should NOT be placed on deep burn that
ment; needlestick; bite. requires medical attention.
2. Treatment: (4) Blisters should be left intact to help prevent
a. Clean wound with soap and water; may be nec- infection; dry, sterile bandage should be used
essary to force puncture wound to bleed. to cover the burn loosely to prevent infection.
b. Place ice pack on area to reduce swelling. (5) Must NOT allow patient to become chilled
c. Apply firm pressure to control bleeding of a or overheated; if possible, burned area
laceration. SHOULD be elevated above the heart.
G. Foreign bodies in eye of clinician: see earlier (6) Large deep burn can cause shock from pain
discussion. and body fluid loss.
1. Prevention: (7) Must have immediate medical referral to en-
a. Wear protective eyewear. sure hydration, asepsis, wound management.
b. Maintain stable fulcrum on dried surface during c. Chemical burns:
instrumentation and use sharp instruments. (1) Will continue to burn as long as agent is in
c. Use approved method or device to recap needles. contact with the skin; affected area should
d. Take care when passing instruments; NEVER be flushed with water (but NOT forcefully;
pass instruments over the patient’s face or eyes. vigorous flushing may further damage the
H. When cross-infection or contamination is a concern, area), and soaked clothing removed.
use needlestick protocol for postexposure manage- (2) If an eye is affected, required eye washing sta-
ment (see Chapter 8, Microbiology and Immunol- tion should be used to flush the eyes for 15 to
ogy). 20 minutes; head should be tipped so that the
I. Burns that occur in laboratories and sterilization areas affected eye is lower than the other eye.
are MOST often caused by Bunsen burners, denture (3) If chemicals have been inhaled, must have
torches, autoclaves, or dry heat sterilizers. immediate medical referral.
336    Saunders Review of Dental Hygiene   

clinical study  

Age 42 YRS SCENARIO

Sex ☒  Male   ☐  Female The patient visits the dental office on Wednesday
morning for an extraction of tooth #15. The extraction
Chief Complaint “I am sorry I smoked, but I is simple and no complications are expected. After the
needed to relax after such a try- extraction, sterile gauze is placed over the wound and
ing week.”
patient is instructed to bite the gauze for 30 minutes.
Medical History Smokes one pack of cigarettes Verbal instructions for care of the extraction site are
a day given before he is released. On Thursday afternoon,
he calls the office from the road where he had to dig
Current Medications None himself out of the snow. He is calling because the
extraction site has begun to bleed.
Social History Traveling computer specialist
Tends to use sugared mints for
fresh breath

1. What verbal instructions should have been given to


the patient after the extraction?
Review Questions
2. Is the bleeding that occurred on Thursday considered
primary or secondary bleeding?
1 What is the pressure in the blood vessels termed when the
 �����
3. Describe the appropriate treatment for this ­situation. heart contracts?
4. What could the patient have done to prevent the recur- A. Diastolic
rence of bleeding? B. Systolic
5. What could dental personnel have done to prevent the C. Pulse
situation? D. Ventricular
2 During an emergency, which artery is used by emergency
 �����
1. Patient should have been given verbal instructions to personnel to check the pulse for an unconscious adult?
avoid all of the following for at least 24 hours: strenu- A. Femoral
ous activity, rinsing the mouth, drinking from a straw, B. Brachial
C. Carotid
smoking, drinking alcohol.
D. Radial
2. Bleeding that occurs 24 hours after extraction is
3 The term “premature ventricular contractions (PVCs)”
 �����
considered secondary bleeding; bleeding that occurs refers to
within first 24 hours is primary bleeding. A. longer than normal pause or skip between beats.
3. Treatment of postsurgical hemorrhage includes return- B. slower than normal pulse rate.
ing to the dental office if occlusion on gauze does not C. alternately strong and weak beats.
halt bleeding. Because bleeding has recurred, patient D. faster than normal pulse rate.
should not drink from straw or rinse area with warm 4 Which of the following is administered immediately by the
 �����
saline rinses for 24 hours. supervising dentist when treating an anaphylactic reaction?
4. Smoking after the extraction most likely was the cause A. Oxygen
of secondary bleeding, although strenuous activity B. Epinephrine
C. Corticosteroids
when he traveled may have been involved.
D. Antihistamines
5. In cases of surgical tooth extraction, written instruc-
5 What is the possible side effect of Dilantin that can affect
 �����
tions in addition to verbal instructions should be given dental health?
to the patient. Although patient may indicate that he A. Gingival hyperplasia
understands procedures to be followed, he may not B. Dry mouth
hear or remember all instructions. In this case, he did C. Increased chance of syncope
not adhere to instruction regarding smoking. D. Decreased prothrombin time
Medical and Dental Emergencies    337

6 Erythema and small vesicle formation on the hands in


 ����� 16 What emergency situation occurs when a diabetic patient
response to wearing latex gloves are an example of who requires daily insulin has increased thirst and urina-
A. pruritus. tion, Kussmaul breathing, and a fruity, acetone breath?
B. urticaria. A. Hyperglycemia
C. contact dermatitis. B. Insulin shock
D. angioedema. C. Hypoglycemia
7 Those who are LEAST likely to hyperventilate include
 ����� D. Low blood sugar
A. males. 17 Which of the following procedures applies to the treatment
B. children. of any emergency in the dental office?
C. elderly. A. Patient should always be moved from the dental chair.
D. females. B. Use of a manufactured drug kit ensures greater success.
8 The emergency oxygen tank or cylinder in the dental office
 ����� C. Basic life support is considered primary management.
should include which size? D. Oxygen administration is always necessary for ALL
A. H emergencies.
B. B E. Administration of drugs is considered primary
C. C management.
D. E 18 What should be done in the dental office for a patient with
9 If the mask becomes fogged when oxygen is delivered to a
 ����� partial airway obstruction and poor air exchange?
patient, it indicates which of the following? A. Manage the situation as for a complete airway obstruc-
A. Flowmeter is set too high. tion.
B. Mask does not fit the patient correctly. B. Do not intervene; allow the patient to cough and expel
C. Tank is empty, and no further oxygen is available. the obstruction.
D. Patient has started breathing. C. Administer back blows as needed until the object is
10 What is the primary advantage of using an emergency kit expelled.
assembled by office personnel? D. Do nothing until the patient becomes completely uncon-
A. Office personnel are more familiar with the contents. scious.
B. It is less expensive than manufactured kits. 19 When performing cardiopulmonary resuscitation on an
C. There is no advantage; manufactured kits are always adult, how far down should the chest compressions be on
superior and more effective. the center of the chest, between the nipples?
D. Office personnel know where it is stored. A. 1⁄3 to ½ inch
11 When blood pressure is taken, the cuff is initially inflated to B. 1 to 1½ inches
110 mm Hg, the point at which the pulse can no longer be C. 1½ to 2 inches
palpated. To which level should the cuff be inflated next? D. 2 to 2½ inches
A. 90 to 110 mm Hg 20 The ratio of compressions to breaths for a child victim dur-
B. 100 to 110 mm Hg ing basic life support is
C. 130 to 140 mm Hg A. 15:2.
D. 150 to 160 mm Hg B. 15:5.
12 Which of the following is indicated for the patient who is C. 30:2.
experiencing an asthma attack? D. 30:5.
A. Aspirin or ibuprofen 21 For a child who requires basic life support, breaths should
B. Bronchodilator be given within the last ____ second(s).
C. Glucagon A. 1
D. Diazepam B. 2
13 Which emergency is NOT triggered by stress? C. 5
A. Insulin shock D. 10
B. Hyperventilation E. 15
C. Epileptic seizure 22 After determining that an adult victim is unconscious, the
D. Asthma attack NEXT step in this emergency situation is to
E. Anaphylaxis A. activate EMS system.
14 In the prodromal phase of which type of seizure does a per- B. perform CPR for 1 minute and then activate EMS
son experience an aura? system.
A. Petit mal C. determine whether the victim is breathing.
B. Partial D. open the airway.
C. Grand mal 23 What is used for a child of 7 years who requires chest com-
D. Hysterical pressions?
15 What is the initial treatment for syncope? A. One finger
A. Positioning the patient upright B. Two fingers
B. Administering oxygen C. One hand
C. Using ammonia inhalants D. Heel of one hand
D. Positioning the patient in the Trendelenburg position E. Heel of one hand, second hand on top
338    Saunders Review of Dental Hygiene

24 A patient is being transported to the dental office after an ac- 31 A seizure may be caused by all of the following EXCEPT
cident during volleyball practice that resulted in an avulsed one. Which is the EXCEPTION?
tooth. The BEST method of transporting the tooth is in A. Prolonged high fever
A. a dry cloth. B. Trauma to the head
B. a glass of water. C. Congenital abnormalities
C. patient’s vestibule. D. Drug withdrawal
D. patient’s cleaned hand. E. Myocardial infarction
25 During scaling an instrument tip breaks. What should the 32 Alleviating a patient’s fears of dental treatment will aid in
clinician do immediately? preventing each of the following EXCEPT one. Which is
A. Examine the area closely, after drying it thoroughly with the EXCEPTION?
air. A. Angina pectoris
B. Ask the patient to help locate the tip with his or her B. Myocardial infarction
tongue. C. Anaphylaxis
C. Examine the sulcus, using a curet and gentle strokes. D. Hyperventilation
D. Expose a radiograph of the area. E. Epileptic seizure
26 You have been performing the Heimlich maneuver for a 33 Which of the following is an example of an ASA II medical
victim for some time without being able to dislodge the ob- history of a patient?
struction. As the individual becomes unconscious, you help A. Uncomplicated pregnancy
him to the floor and activate the EMS system. It is now nec- B. Uncontrolled epilepsy
essary to C. Controlled angina pectoris
A. straddle the victim’s hips, administer five abdominal D. Controlled insulin-dependent diabetic
thrusts, and attempt to ventilate. 34 Which of the following is an example of an ASA III medical
B. continue administering the Heimlich maneuver, with the history of a patient?
victim positioned on his side. A. Slight COPD with exercise
C. kneel beside the victim’s hips and administer six to ten B. Myocardial infarction 7 months ago
chest compressions. C. Controlled epilepsy
D. do nothing further until emergency assistance arrives. D. Active allergies
27 For the patient who has experienced a heart attack within 35 Bleeding levels in patients at risk of hemorrhage are MORE
the last 4 weeks effectively monitored by
A. patient positioning is important; the patient should be A. blood glucose testing.
more upright. B. international normalized ratio.
B. the clinician should be ready to perform basic life sup- C. prothrombin time.
port as needed. D. reducing drugs and performing surgery.
C. dental treatment should be performed only after medical 36 A sublingual tablet of nitroglycerin has a long shelf-life; it
consult. may be used for 12 months after the container is opened.
D. the clinician must determine whether angina is present. The metered spray form of nitroglycerin has a much shorter
28 At which of the following blood pressure readings is a shelf-life, and one metered spray is equal to two tablets.
­medical consult indicated? A. Both statements are true.
A. 110/68 B. Both statements are false.
B. 119/76 C. The first statement is true, the second is false.
C. 150/92 D. The first statement is false, the second is true.
D. 164/102 37 ASA IV classification of physical status indicates signifi-
29 Conditions that predispose a patient to cerebrovascular ac- cant risk because the person has moderate systemic disease
cident include all of the following EXCEPT one. Which is that limits activity and is a constant threat to life.
the EXCEPTION? A. Both the statement and reason are correct and related.
A. Diabetes mellitus B. Both the statement and reason are correct but NOT
B. Hypertension related.
C. Transient ischemic attack C. The statement is correct, but the reason is NOT.
D. Cardiovascular disease D. The statement is NOT correct, but the reason is correct.
E. Epilepsy E. NEITHER the statement NOR the reason is correct.
30 An epileptic cry caused by air rushing from the lungs may 38 One of the following does NOT predispose a patient to
occur during which stage of a grand mal seizure? orthostatic hypotension. Which one is the EXCEPTION?
A. Prodromal A. Elderly
B. Convulsive B. Pregnancy
C. Postictal C. Addison’s disease
D. Characteristic of hysterical seizure instead D. Nitrous oxide or diazepam (Valium) IV sedation
E. Getting up slowly from dental chair
Medical and Dental Emergencies    339

39 Presyncope stage is characterized by 5 (A) ������������������������������������������������


Phenytoin (Dilantin) can result in slight to se-
A. thready pulse. vere gingival hyperplasia and may affect patient’s
B. loss of pulse. oral hygiene. Other possible side effects include skin
C. tachycardia. rash, drowsiness, gastric distress, restlessness. Dry
D. bradycardia.
mouth (xerostomia) may result from antihistamines
40 Which type of burn involves only epidermis, the top layer
and other drugs. There is an increased chance of loss
of skin, along with redness, dryness, and pain?
A. Superficial of consciousness from orthostatic (postural) hypoten-
B. Deep sion, which MOST commonly occurs with antihy-
C. Second degree pertensive drugs. Bleeding may occur with increased
D. Third degree use of aspirin. Prothrombin time (PT) is a more gen-
eralized test. Bleeding levels are monitored more
­effectively by international normalized ratio (INR);
Answer Key and Rationales INRs ≤2.5 are safe for invasive dental work.
6 (C) ������������������������������������������������
Contact dermatitis is characterized by erythema
and small vesicle formation on the hands, a localized
1 (B) �����������������������������������������������
Systolic pressure is pressure in blood vessels allergic reaction. Pruritus, urticaria, and angioedema
when the heart contracts (work). Diastolic pressure is are other types of allergic reactions.
pressure in blood vessels when the heart dilates (rest). 7 (B)  I���������������������������������������������������
n the dental setting, hyperventilation in response
Pulse pressure is found by subtracting diastolic read- to anxiety is least likely to occur in children be-
ing from systolic reading. Ventricular pressure ap- cause they release their anxiety in other ways (e.g.,
plies to pressure within ventricular chambers of the crying, refusing to cooperate). State of breathing
heart, NOT within blood vessels. faster and/or deeper than necessary, thereby reduc-
2 (C) ��������������������������������������������
Common carotid artery is ONLY used to check ing the carbon dioxide concentration of the blood to
carotid pulse for an unconscious adult in an emer- below normal. Hyperventilation can cause symptoms,
gency situation by trained EMS system personnel such as numbness or tingling in the hands, feet, and
(if <60 mm Hg, artery is NOT palpable). Radial ar- lips, lightheadedness, dizziness, headache, chest pain,
tery is used during routine appointments to check for slurred speech, and sometimes syncope (fainting).
radial pulse (if <90 mm Hg, artery is NOT palpable) 8 (D) ��������������������������������������������
Oxygen tanks that are part of the emergency
on conscious patient. Note that checking for pulse equipment in dental settings should be at least
such as carotid pulse is NOT a part of basic life sup- size E, which supplies 100% oxygen for 30 minutes,
port (BLS) or cardiopulmonary resuscitation (CPR) administered at 5 liters/min with a nasal cannula.
by nontrained personnel. Brachial artery is used dur- 9 (D) ���������������������������������������
When oxygen mask becomes fogged during
ing an emergency for an unconscious infant ONLY by use, this indicates that patient has started breathing.
EMS personnel. Femoral artery is NOT commonly used Full mask made of clear material should be used;
to check pulse in an adult. Since systolic blood pressure should fit the patient to provide tight seal. Setting
rarely drops low enough that carotid pulse cannot be flowmeter too high, empty tank, or improper fit of
felt, lack of a carotid pulse usually indicates death. mask does NOT cause mask to fog.
3 (A) ����������������������������������������������
Premature ventricular contractions (PVCs) are 10 (A)  O�������������������������������������������
ffice personnel will be MORE familiar with
longer than normal pauses or skips in beat. Brady- the contents of an emergency kit that they have as-
cardia is a slower than normal pulse rate (<60 BPM). sembled to meet the needs of their particular dental
Pulsus alternans is alternate strong and weak beats. setting. Although this kit is likely to be less expen-
Tachycardia is faster than normal pulse rate (>100 sive, the cost is NOT an issue. Effective use of an
BPM). emergency kit is the MOST important factor. Person-
4 (B) �����������������������������������������������
Epinephrine is drug of choice when treating se- nel should ALL know where the kit is stored, NO
vere allergic reactions such as anaphylaxis. Rapid matter if purchased or NOT.
effect after administration makes it extremely use- 11 (C)  When blood pressure is taken, the cuff is initially
ful when time is crucial. Epinephrine is a vasopres- inflated until the radial pulse is NO longer felt. This
sor and bronchodilator and has antihistamine actions is the estimated systolic number (palpatory systolic
(prevents mast cell degranulation). Can be given by pressure) and is mentally noted as aid in hearing first
supervising dentist as an IV or can be given by super- systolic sound; cuff is then reinflated 10 to 30 mm
vising dentist or patient by injectable pen (EpiPen). Hg above this point. If pulse is NO longer palpated
As with MOST emergencies, oxygen can be adminis- at 110 mm Hg, cuff is then inflated to 130 to 140 mm
tered (5 liters/min with a nasal cannula) but will NOT Hg. Inflating to only 90 to 110 mm Hg would result
stop the reaction. Corticosteroids and antihistamines in loss of FIRST systolic reading; inflating to 150 to
can be used in non-life-threatening allergic reactions. 160 is unnecessary and is uncomfortable for patient.
340    Saunders Review of Dental Hygiene

12 (B)  Asthma attack is a respiratory emergency that of- unconscious patient, MUST maintain an open airway
ten is triggered by anxiety. Overproduction of mucus and administer breathing and chest compressions
and contraction of smooth muscles narrow bronchioles, as indicated to supply the necessary oxygen to the
making breathing difficult. Treatment includes use of brain. NOT always necessary to move patient from
a bronchodilator, which acts directly on the bronchial chair; positioning of the patient depends on the type
smooth muscle. Patient should be allowed to use of emergency. Using a manufactured drug kit does
own bronchodilator if possible (it is current and pa- NOT guarantee successful recovery. Oxygen may be
tient knows how to use it). Aspirin or ibuprofen may beneficial but is NOT necessary for ALL emergencies
cause an asthma attack in those who are allergic to it. (e.g., hyperventilation).
Glucagon is administered IV or IM by the supervising 18 (A) ����������������������������������������������
When an individual exhibits partial airway ob-
dentist to an unconscious patient who is experiencing struction and poor air exchange, it indicates that he
hypoglycemia. Diazepam (Valium) can be adminis- or she is unable to cough forcibly in an attempt to
tered IV by the supervising dentist for the treatment of remove the object. This situation must be managed
severe hyperventilation, if available. in the same manner as complete airway obstruction:
13 (E)  Anaphylaxis is caused by exposure to an aller- Heimlich maneuver should be performed until the
gen and is NOT associated with stress. Insulin shock, obstruction is expelled or the victim becomes un-
hyperventilation, epileptic seizure, and asthma at- conscious. Back blows are NOT part of the Heimlich
tack may be triggered by stress; thus it is necessary maneuver; they are part of the protocol for obstructed
to lower anxiety as much as possible by explaining airway of an infant. ONLY when the victim can cough
ALL procedures thoroughly, attending to patients forcibly is it correct NOT to intervene. Doing nothing
promptly, closely monitoring patients. until a patient becomes unconscious is NOT an ap-
14 (C)  Aura, such as flicker of light, certain smell, or propriate choice for any emergency.
noise, may occur during prodromal phase of a grand 19 (C) ���������������������������������������������
For an adult who requires BLS, chest compres-
mal seizure. Characteristic symptom of grand mal sions in cardiopulmonary resuscitation (CPR) are
seizure; is NOT characteristic of petit mal, partial, performed with two hands: heel of one hand, with
or hysterical seizure. Can also occur before migraine the second hand on top, pressing 11⁄2 to 2 inches in
headache; allows epileptics time to prevent injury to the center of the chest, between the nipples. For an
themselves. Time between appearance and onset of a infant, compressions are 1⁄2 to 1 inch of chest depth;
migraine or seizure can be anything from a few sec- for a child, compressions are 1⁄3 to 1⁄2 inch of chest
onds up to an hour; MOST who have auras have the depth. It is recommended that the clinician note if the
same type of aura every time. chest has recoiled from compression to allow the heart
15 (D)  Syncope (fainting) is caused by a decreased ox- to fill with blood. Do NOT allow interruptions in com-
ygen supply to the brain. Placing the patient in the pressions; give ALL 100 compressions per minute.
Trendelenburg position (subsupine with feet elevated 20 (C)  D�������������������������������������������������
uring BLS for children and infants, ratio of com-
higher than head) typically ensures recovery by in- pressions to breaths is 30 to 2.
creasing blood flow to the brain and thereby replen- 21 (A) ����������������������������������������������
For child who requires BLS, breaths should be
ishing oxygen. If such positioning does NOT result given within the last 1 second. Must use enough volume
in recovery, ammonia inhalants (amyl nitrate) can be to see chest rise when giving each breath; use regular
used to stimulate breathing, and oxygen can be ad- breathing before breaths; do NOT use a deep breath.
ministered (5 liters/min with a nasal cannula). 22 (A) ������������������������������������������������
As soon as unconsciousness is established in an
16 (A)  Hyperglycemia (high blood sugar) is an emer- adult victim, the next step is to activate EMS system.
gency that affects patients with diabetes mellitus (DM) Afterward, airway is opened and rescuer determines
who require daily insulin. Results when there is little whether victim is breathing. If the victim is NOT
or NO insulin available and may be caused by failure breathing, chest compressions are begun to start car-
to adhere to therapy, increased exercise, or infection. diopulmonary resuscitation (CPR). Necessary to acti-
With increased thirst and urination, Kussmaul breath- vate EMS as quickly as possible, since during (sudden)
ing, and the presence of a fruity odor on the breath. cardiac arrest (SCA), heart may undergo fibrillation
Loss of appetite, nausea, and vomiting may be pres- and require use of special equipment transported by
ent. Urine samples indicate high glucose levels and EMS personnel. Such equipment defibrillates the
presence of ketones. May result in diabetic coma. Hy- heart and allows effective compressions. Some den-
poglycemia (low blood sugar) and its potentially fatal tal settings may have portable automated defibrillator
end result, insulin shock, occur when level of blood equipment (AED) and be able to start the process.
glucose is low and have different characteristics. 23 (E) ��������������������������������������������������
For the child who requires chest compressions, us-
17 (C) ����������������������������������������������
During treatment of any emergency, BLS is the ing two hands, with heel of one hand, second hand on
MAIN management. Drug usage is secondary. For an top, pressing 1⁄3 to ½ of chest depth. For an adult, it is the
Medical and Dental Emergencies    341

same two hands but pressure of 1½ to 2 inches. For in- grand mal seizure. Aura is characteristic of prodro-
fant, ONLY two fingers are necessary to perform chest mal stage; patient regains consciousness during post-
compressions, pressing ½ to 1 inch of chest depth. ictal stage. Petit mal seizures do NOT have stages.
24 (C) �������������������������������������������
BEST mode of transportation for an avulsed 31 (E) ���������������������������������������������������
Myocardial infarction (MI, heart attack) typically
tooth is in patient’s own buccal vestibule, since the does NOT cause seizures. Seizure may be caused by a
saliva will keep the tooth moist. Other choices in- prolonged high fever, trauma to the head, congenital ab-
clude a wet handkerchief, milk, or water; commercial normalities, alcohol or drug withdrawal. Other causes
preparations are available. For reimplantation with include epilepsy and imbalance of body fluids. Tempo-
the BEST prognosis, do NOT to dry the tooth. rary abnormal electrophysiological phenomenon of the
25 (C) ����������������������������������������������
When an instrument tip breaks during scaling, brain, resulting in abnormal synchronization of electri-
primary objective is to remove the tip from the pa- cal neuronal activity. Can manifest as an alteration in
tient’s mouth. It is important for the clinician to remain mental state, tonic or clonic movements, convulsions,
calm and examine the area using gentle strokes with various other psychic symptoms.
a curet in the sulcus. To dry the area, a cotton roll 32 (C) ��������������������������������������������������
Alleviating fears of dental treatment will aid in
should be used. Patient should NOT swallow or at- preventing angina pectoris, myocardial infarction
tempt to locate broken tip with tongue. If the tip is (MI, heart attack), hyperventilation, epileptic seizure,
NOT retrieved, radiograph should be used to aid in since ALL are risks when the patient is undergoing
the location of the tip. stress. Anaphylaxis, instead, is based on a severe
26 (A) �����������������������������������������������
When airway obstruction results in unconscious- allergy. Angina pectoris is chest pain caused by de-
ness, the victim should be helped to the floor. Operator creased oxygen flow to heart. MI involves death of a
then should straddle the victim’s hips and administer portion of heart muscle; caused by decrease in or lack
five hand thrusts to abdomen with upward and inward of oxygen flow to that part of heart muscle.
motion in attempt to dislodge obstruction. Thrusts are 33 (A)  Uncomplicated pregnancy is example of ASA II;
followed by attempt to ventilate. Routine is contin- others are examples of ASA III.
ued until object is expelled and clinician is successful 34 (A) ��������������������������������������������
Slight COPD with exercise is example of ASA
in ventilating. Two additional ventilations are then III; others are examples of ASA II.
given to stimulate breathing and pulse is checked. 35 (B) �����������������������������������������������
Bleeding levels are monitored by international
o6890 27 (C) ����������������������������������������������
Withholding emergency or elective surgery (or normalized ratio (INR); INRs ≤2.5 are safe for inva-
periodontal procedures by dental hygienist) for 4 to sive dental work. Older test is prothrombin time (PT).
6 weeks after MI and then determining functional ca- Blood glucose testing is done to monitor status of dia-
pacity (FC); dental care should be delayed unless pa- betics. NO need to reduce drugs if patient is at risk of
tient can meet 4 METs (metabolic equivalents) and/or hemorrhage, however, medical consult is indicated.
further medical testing has been completed to quantify 36 (B)  Both statements are false. Sublingual tablet of ni-
level of cardiac risk in treatment. Patient positioning troglycerin (NTG) has short shelf-life, may be used for
is NOT the primary consideration. Patient who expe- 6 months after container opened. Metered spray form has
riences angina pectoris and carries his or her own ni- longer shelf-life; one metered spray is equal to 1 tablet.
troglycerin (NTG) should have easy access to NTG. 37 (C) ������������������������������������������������
Statement is correct, but reason is not. ASA IV
Office personnel should be prepared to provide BLS for classification of physical status indicates significant
any patient, NOT just those with recent history of MI. risk because the person has moderate systemic dis-
28 (D)  Medical consult is indicated when systolic read- ease that limits activity and is a constant threat to life.
ing is >160 and diastolic is >100 mm Hg; this is stage Poses significant risk because patients in this category
2, moderate to severe high blood pressure (HBP). have a severe medical problem of greater importance
Blood pressure reading of 150/92, stage 1, mild HBP, than the planned dental treatment.
should be rechecked for accuracy by taking again; pa- 38 (E) ������������������������������������������������
Orthostatic (postural) hypotension occurs in el-
tient may have raced to appointment or have “white- derly, pregnant women, persons with Addison’s dis-
coat syndrome”; must refer to physician if second ease, and with nitrous oxide or diazepam (Valium) IV
reading is within same range. Readings of 110/68 and sedation. Certain antihypertensives and use of NTG
119/76 are considered within normal range. predispose patient to postural hypotension, as does
29 (E) �����������������������������������������
Epilepsy does NOT predispose a person to getting up quickly from dental chair.
cerebrovascular accident (CVA, stroke). History of 39 (C) ��������������������������������������������
Presyncope stage has tachycardia (increased
diabetes mellitus (DM), hypertension (high blood pulse rate) and NOT slower pulse rate (bradycardia);
pressure), transient ischemic attack (TIA), and car- syncope by thready pulse.
diovascular disease (CVD) increase risk for CVA. 40 (A) �����������������������������������������������
Superficial or first-degree burns involve ONLY
30 (B)  Epileptic cry, which is caused by air rushing epidermis, top layer of skin, with redness, dryness,
from the lungs, occurs during convulsive stage of and pain.
CHAPTER  11

Clinical Treatment
DEFINITION OF DENTAL HYGIENE PRACTICE   6. Documentation: complete and accurate recording
Dental hygiene is the science and practice of recognition, of all collected data, treatment planned and pro-
treatment, prevention of oral diseases. Hygienist is a pre- vided, recommendations, and other information
ventive oral health professional who has graduated from an relevant to patient care, treatment.
accredited program in an institution of higher education; a B. Dental hygiene process of care is a cycle in which
licensed professional who provides educational, clinical, re- the hygienist might pass through each of the rec-
search, administrative, therapeutic services supporting total ommended steps (ADPIED) several times during a
health through promotion of optimal oral health. In practice, course of treatment; over a period of months or years,
hygienists integrate roles of clinician, educator, advocate, a hygienist may have evaluated the process several
manager, researcher to prevent oral diseases and promote times, altering the diagnosis and plan numerous times
health; hygienists work in partnership with dentists. as the patient’s condition changes.
The distinct roles of the hygienist and dentist comple-
ment and augment the effectiveness of each professional
and contribute to a cotherapist environment (see later DENTAL RECORD  
discussion under “Motivation”). Hygienists are viewed The dental record is a medicolegal document that should
as experts in their field, are consulted about appropriate be complete, accurate, and legible. From the record, it
dental hygiene interventions, are expected to make clini- should be possible to recreate the patient’s medical and
cal dental hygiene decisions, and are ­ expected to plan, dental history and oral status at initial presentation, along
implement, and evaluate the dental hygiene component with periodic updates, treatment needs, treatment ren-
of the overall care plan. Hygienist establishes the dental dered, and any recommendations made.
hygiene diagnosis, which is an integral component of the • See Chapters 16, Special Needs Patients: medical dis-
comprehensive dental ­diagnosis established by the den- abilities; 18, Ethics and Jurisprudence: HIPAA, ­informed
tist. Each state has ­defined its own specific regulations for consent.
dental hygiene licensure. A. Financial record demographic and insurance informa-
• See CD-ROM for Chapter Terms and WebLinks. tion.
• See CD-ROM for ADHA Guidelines for the Stan- B. Assessment data:
dards For Clinical Dental Hygiene Practice (SCDHP) 1. Medical history.
(2008). 2. Dental history:
A. Process of care consists of six components ­ a. Significant past experiences.
(ADPIED): b. Chief complaint (CC).
1. Assessment: systematic collection, analysis, docu- c. Attitudes about dentistry and possible anxiety.
mentation of the oral and general health status and d. Dental biofilm control and self-care practices.
patient needs. 3. Test results.
2. Dental hygiene diagnosis: component of the over- 4. Abnormal findings from extraoral and intraoral
all dental diagnosis; identification of existing or ­examination.
potential oral health problem that a dental hygien- 5. Dental and periodontal chart.
ist is educationally qualified and licensed to treat. 6. Radiographs and photographs.
3. Planning: establishment of goals and outcomes C. Diagnosis (treatment needs).
based on patient needs, expectations, values, cur- D. Treatment plan and record of informed consent.
rent scientific evidence. E. Documentation of treatment (progress record):
4. Implementation: delivery of dental hygiene ser- 1. Implementation of specific treatment provided by
vices based on dental hygiene care plan in a man- date and signed by clinician.
ner minimizing risk and optimizing oral health. 2. Evaluation of treatment outcomes.
5. Evaluation: process of reviewing and documenting 3. Referrals and recommendations.
the outcomes of dental hygiene care, which occurs 4. Future plans.
throughout the process of care. F. Correspondence.

342
Clinical Treatment   343

intensity, location, as well as related drug


Health History therapies.
Health history is BEST for gathering information re- (2) May influence or contraindicate certain
garding the medical and dental history, behavior, demo- dental procedures.
graphics, vital signs. Listed below is only overview of d. History of past illness includes:
what may be noted in chart. Dental history is included (1) Diseases that complicate dental ­ treatment
under assessment of the dentition. or might require special precautions or an-
• See Chapters 6, General and Oral Pathology: common tibiotic premedication before ­treatment.
conditions and diseases in dental setting; 8, Microbiol- (2) Allergy or untoward reactions to drugs.
ogy and Immunology: vaccinations, infectious diseases; (3) Diseases and drugs with oral signs.
9, Pharmacology: antibiotic premedication; 10, Medi- (4) Communicable diseases.
cal and Dental Emergencies: medical history, physical (5) Physiological and psychological state.
status classification (ASA), high blood pressure. e. Includes common medical conditions noted
A. Medical history: review of physical health status. on medical history forms; as a result may need
1. Includes statement of chief complaint (CC), his- medical consult.
tory of present and past illness, drug history. (1) Confirming fax is recommended from in- o17780
2. Need for medical consult and antibiotic and/or volved healthcare professionals.
­antianxiety premedication MUST be assessed. (2) Legally similar to sending certified mail, o17790
3. Information obtained by: since cannot be refused by recipient; if there
a. Questionnaire vs. interview: is no response, it is viewed as admission of
(1) Questionnaire: saves time, consistent, poten- silence; no need to be HIPAA qualified be-
tially MORE thorough; however, inflexible cause it is viewed as a telephone call.
and LACKS opportunity for development C. Medically compromised patients who can be seen in a
of rapport with patient; ­impersonal. dental setting but need major modification of den-
(2) Interview: flexible and personable; requires tal treatment and/or need medical consult (drug his-
good communication skills; enhances the tory discussed next):
relationship-building process between pa- 1. Patient with cardiovascular disease (CVD):
tient and clinician; time consuming and a. Has inadequate blood circulation to heart mus-
may cause embarrassment. cle, resulting from arteriosclerosis, blocking or
(3) Combining BOTH interview and ques- narrowing of blood vessels; MUST be ready for
tionnaire achieves practical results and an emergency.
thoroughness by allowing clinician to get b. Possibly reports angina pectoris caused by o0490
additional clarification and information on muscle pain that radiates to left arm and jaw;
positive responses. experiences pain with exertion and anxiety;
b. Subjective vs. objective information: discomfort is relieved by rest and vasodilator
(1) Subjective information: impressions, feel- nitroglycerin (NTG, Nitrostat); request that pa-
ings, attitudes from patient and clinician; tient bring NTG to dental appointment; have
patient reports (symptoms), e.g., nausea, readily available during treatment.
and clinician’s observations of patient, e.g., c. May use antihypertensives (includes diuretics, o17800
flushed skin. adrenergic blockers) that may result in xero­
(2) Objective information: concrete notes, stomia.
facts, data (signs), test results. d. Patient after myocardial infarction (MI, heart o17810
B. Health status: notation of existing and previous con- attack): attack resulted from insufficient blood
ditions. supply to heart muscle caused by atherosclerosis
1. Includes chief complaint (CC) and formation of of blood vessel walls; after cardiac episode and/
physical classification status (American Society of or surgery, requires 4- to 6-week delay ­before
Anesthesiologists [ASA]): emergency or elective dental procedures.
a. CC: statement in patient’s own words of why he e. MUST meet 4 metabolic (MET) equivalents to o17820
or she is seeking dental care. determine functional capacity (FC) to decrease
b. Formation of ASA status: reduces risk of emer- cardiac risk during dental care; continue on
gency occurring during treatment. CVD drug therapy during treatment.
c. History of present illness; may influence future f. May need to lower amount of vasoconstrictor
treatment options and setting. with local anesthesia.
(1) Includes description of current complaint g. Increased risk of periodontal disease, and if
and its signs, symptoms, onset, duration, present, may affect CVD.
344   Saunders Review of Dental Hygiene

2. Patient with hypertension (high blood pressure 6. Patient with diabetes mellitus (DM): has disorder
[HBP]): has or had elevated blood pressure; may of glucose metabolism resulting from relative or
also report symptoms of headache, dizziness, or absolute lack of insulin; has vascular component
nosebleeds: (may have related CVD and/or renal diseases; see
a. Taking antihypertensive and/or diuretic drug(s); later discussion).
may result in xerostomia. a. Two types: type 1, can be severe and unstable;
b. Can progress to arteriosclerosis, impaired renal stems from lack of insulin produced from pan-
function, cardiac enlargement, MI, cerebrovas- creas, and type 2, often NOT as severe but
cular accident (CVA, stroke); MUST be ready ­usually stable; develops slowly with age; fre-
for an emergency. quently associated with obesity; pancreas
c. Monitor blood pressure at each visit; refer produces adequate insulin but there is insulin
to physician when blood pressure is stage insensitivity of the tissue.
1: mild HBP (140 to 159/90 to 99 mm Hg); b. Dental appointment should be scheduled after
with stage 2, moderate to severe HBP (>160/ meal and insulin therapy if taken; MUST be
>100 mm Hg), may NOT be able to perform ready for emergency.
elective dental care; may need immediate c. Need to know current blood glucose levels:
­medical consult. <70 mg/dL, too low; tendency toward hypogly-
d. Do NOT provide dental treatment when blood cemia; >240 mg/dL: too high; tendency toward
pressure is ≥180/≥110 mm Hg, activate EMS hyperglycemia; medical consult since is a risk
system and treat as emergency. for emergency.
3. Patient with congestive heart failure (CHF): occurs d. Risk factor for periodontal disease; MORE fre-
when heart muscle is weak and is unable to pump quent maintenance appointments are required, as
blood at an adequate rate; blood circulation is poor, well as excellent oral hygiene self-care; if peri-
resulting in congestion and pooling of blood in the odontal disease is present, may affect DM control.
organs and lower extremities. 7. Patient with renal disease: renal function is im-
a. Swollen ankles from edema; shortness of paired at various levels, possibly maintained by
breath; fatigue. hemodialysis or kidney transplant.
b. May be taking digitalis glycoside (digoxin, a. Experiences MORE frequent infections, poor
Lanoxin) and diuretics that usually result in xe- healing, bleeding difficulties.
rostomia. b. Surgical systemic shunts require antibiotic pre-
4. Patient with cardiac dysrhythmia: had or has medication.
­irregular heartbeat (too slow or fast). c. Certain drugs remain in circulation longer be-
a. May have pacemaker (with or without defi- cause of poor renal function (metabolism); care
brillator) and report taking digitalis glyco- MUST be taken with drugs delivered (such as
side (digoxin, Lanoxin) or calcium channel anesthetics and preventives) and prescriptions
blocker such as diltiazem (Cardizem, Dilacor, given by the dental office.
Tiazac) or nifedipine (Procardia); may have 8. Patient with blood disorder or with history of dif-
xerostomia and gingival hyperplasia. ficulty with bleeding:
b. With pacemaker, do NOT need to avoid use of ul- a. Patients include those with anemia, deficiency
trasonic scalers, since ALL are shielded, AVOID of red blood cells (RBCs) resulting from vitamin
electric pulp testing (EPT) (discussed later). or iron deficiency, bone marrow malfunction,
c. Antibiotic premedication for first 6 months af- excessive loss of blood, or RBC destruction.
ter placement of pacemaker. b. Also include patients with leukemia, cancer of
5. Patient with valvular heart disorder: white blood cells that do NOT function nor-
a. Has or had deformity of heart valves because mally (cells are immature and excessive in
of past infection, drug use (such as phen-fen for number); thrombocytopenia may develop from
weight control), congenital disorder, or heart chemotherapeutic treatment.
transplant; artificial replacement of valves may c. Prone to infection, weakness, postappointment
have been performed. bleeding, hematomas, or other traumatic vascu-
b. Patient with artificial (prosthetic) valve is at lar lesions; can present with traumatic oral le-
high risk for infective endocarditis (IE), infec- sions, periodontal disease, xerostomia resulting
tion of lining of heart caused by bacteremia from chemotherapy.
(bacteria in blood) from invasive procedures; d. Bleeding levels can be monitored by interna-
­requires antibiotic premedication before tional normalized ratio (INR); INRs ≤2.5 are
­invasive dental procedures. safe for invasive dental work.
Clinical Treatment   345

9. Patient with respiratory infection or disease: 13. Patient with epilepsy: patient can have disturbance of
a. May have a cold, flu, or tuberculosis (TB); electrical brain activity resulting in seizure (convulsions).
may have TB oral lesion (highly contagious); a. Usually reports taking anticonvulsants such as
if contagious, postpone dental treatment until phenytoin (Dilantin); may have related gingival
antibiotic therapy reduces infection that may be hyperplasia.
transmitted. b. Appointments should be made when rested; con-
o17830 b. With emphysema or other forms of chronic firm drug compliance and keep appointments
obstructive pulmonary disease (COPD), may short; MUST be ready for an emergency.
need oxygen during treatment and to be kept 14. Patient with total joint replacement:
upright. a. Patient has replacement for ALL surfaces of
10. Patient with sexually transmitted disease or infec- joint (knee, hip, elbow, fingers) because of past
tion (STD, STI): infection or traumatic disorder, osteoporosis, or
a. Includes patients with syphilis, gonorrhea, osteoarthritis.
and chlamydia acquired through sexual in- b. HIGH risk for prosthetic joint infection 2 years
tercourse; note any oral or pharyngeal lesions after implantation and ALWAYS for immuno-
(highly contagious). compromised (such as diabetics) from bactere-
b. If contagious, postpone dental treatment until mia caused by invasive dental procedures and
antibiotic therapy reduces infection that may be thus requires antibiotic premedication before
transmitted. invasive dental procedures.
11. Patient with hepatitis A, B, C, D, E: patients ALL 15. Patient with xerostomia: if NOT drug related (such
have inflammation of the liver resulting from vi- as with patients who have cancer or Sjögren’s syn-
ral infection; causes fatigue, nausea, tender joints, drome), can recommend use of prescription cholin-
jaundice. ergic agonist drugs such as pilocarpine (Salagen)
a. Follow recommendations for hepatitis B vacci- and cevimeline (Evoxac); drug-related xerostomia
nation for ALL dental providers before treating is discussed in next section.
patients in any clinical setting, since carrier sta-
tus is frequently unknown. Drug History
b. HIGH risk of disease transmission to clinician Drug history provides information on drugs that may af-
and then through chain to other patients; however, fect oral health, dental treatment, and oral hygiene self-
dental providers MUST follow standard precau- care. Often requires use of reference text and/or online
tions at ALL times when treating ALL patients. resource, such as Physician’s Desk Reference (PDR), or
c. May have difficulty with bleeding and metabo- a medical consult and/or pharmacist consult for further
lism of drugs because of liver damage. information. PDR lists all drugs by manufacturer’s name,
12. Patient with allergy: can have reactions to sub- brand name, generic name.
stance that may be as slight as a skin rash or as • See Chapters 6, General and Oral Pathology: specific
severe as fall in blood pressure and anaphylactic conditions and diseases noted in dental setting; 9, Phar-
shock (airway obstruction from tissue swelling macology: drugs that affect dental care.
and/or cardiac arrest). A. Pharmacological record: information regarding drug
a. Reports reaction to known substance (allergen) action, usage, contraindications, adverse reactions,
such as antibiotics, rubber latex, restorative warnings, precautions is reviewed.
metals (especially nickel), other allergens; need 1. Includes record of drugs taken (including over the
to AVOID allergen. counter [OTC]), noting dosage and condition be-
b. May have asthma; need to recommend acet- ing treated; note if drug has impact on oral health
aminophen as substitute for its analgesic and an- and any contraindications for dental ­treatment.
tipyretic effects to avoid aspirin-induced attack; 2. Common drug-induced oral side effects:
may need to alter local anesthetics (AVOID va- a. Xerostomia (dry mouth): associated with
soconstrictor or articaine). cracked lips, sore labial commissures, inflamed
c. May use inhaler for acute asthmatic attacks, smooth tongue (glossitis).
so should be asked to bring to ALL appoint- (1) MOST common reaction to drugs; MOST
ments; instruct to rinse mouth thoroughly after commonly to drugs used to treat depression
using and expectorate, to reduce systemic ab- and anxiety, urinary incontinence, Parkin-
sorption, tooth staining, and taste alteration. son’s disease, as well as antihistamines,
d. MUST determine cause and severity of aller- antihypertensives, antidiarrheals, muscle
gic response; MUST be ready for an emergency, relaxants; BOTH prescription and OTC,
which includes having inhalers in emergency kit. such as for colds, flu, allergies.
346   Saunders Review of Dental Hygiene

(2) Results in thick and ropy saliva, altered slowly and keep seated for several minutes
taste, loss of buffering action; may be as- to reduce dizziness of hypotension:
sociated with burning tongue. (2) Monitor blood pressure at each visit (see
(3) Increases incidence of caries, especially of earlier discussion).
the root; causes difficulty with chewing, (3) Can cause xerostomia (discussed earlier).
swallowing, speaking, bad breath, denture (4) Nifedipine (Procardia), calcium channel
wearing. blocker, can cause gingival hyperplasia
(4) Treatment: relieving symptoms through in- (discussed earlier).
creased water consumption (sip water), sa- (5) May need to alter local anesthetic agent.
liva substitutes, tissue lubricants, chewing (6) Common HBP drugs:
sugar-free gum (especially with xylitol), air (a) Diuretics: hydrochlorothiazide (Lopres-
humidification; AVOID alcohol mouthrinses sor), furosemide (Lasix) (see next dis-
that can dehydrate tissues; to prevent caries, cussion).
use of home fluoride applications and calcium (b) Calcium channel blockers: diltiazem
products for additional remineralization; see (Cardizem, Dilacor, Tiazac), nifedipine
earlier discussion if non–drug related. (Procardia, Adalat, Nifedical).
b. Candidiasis: overgrowth of yeast organism, (c) Beta blockers: propranolol (Inderal),
Candida albicans (see Chapter 8, Microbiology nadolol (Corgard), metoprolol (Lopres-
and Immunology): sor), atenolol (Tenormin).
(1) Increased risk with history of prolonged (d) Angiotensin converting enzyme (ACE)
antibiotics, immunosuppressants, or cor- inhibitors: captopril (Capoten), enalapril
ticosteroid therapy with immunocompro- (Vasotec), lisinopril (Zestril, Prinivil).
mised health, denture or dental appliance b. Diuretics: used to promote renal excretion in
wear. the treatment of CHF and HBP:
(2) Frequently results in painful irritations, cre- (1) Result in frequent urination and xerostomia
ating difficulty in chewing, speaking, wear- (see earlier discussion); affects planning of
ing dental appliances. appointment time and length of appointment.
(3) Treatment: antifungals such as nystatin (2) Common drugs: see antihypertensives and
(Mycostatin), clotrimazole (Mycelex), diuretics.
ketoconazole (Nizoral), fluconazole (Di- c. Anticoagulants and antiplatelets: used for treat-
flucan) that may contain sugar, increasing ment of CVD to increase blood flow by sup-
risk of caries; education given on proper pressing or delaying coagulation of blood:
denture care (see Chapter 15, Dental (1) Discontinuation of drug may be hazardous
Biomaterials). because of embolus, thrombus formation.
c. Drug-induced gingival hyperplasia: associated (2) Side effects can be prolonged; spontaneous
with enlarged, bulbous, and fibrotic gingival internal bleeding can occur; hematoma more
tissues: likely with some local anesthesia injections
(1) Results from anticonvulsant phenytoin (Di- (especially inferior alveolar nerve block).
lantin), calcium channel blocker such as (3) Requires monitoring of bleeding by INR levels
nifedipine (Procardia), or transplant rejec- (see earlier discussion) and medical ­consult.
tion drugs such as cyclosporine. (4) Common drugs: warfarin (Coumadin),
(2) Can be controlled or prevented through ad- clopidogrel (Plavix), aspirin (low dose).
equate oral hygiene self-care, especially if d. Antianginals: used for treatment of angina pec-
begun before the drug regimen. toris to increase oxygen supply to heart muscle
(3) May need surgery to remove excess tissues through vasodilation:
(see Chapter 13, Periodontology). (1) Usage pattern should be established before
d. Additional oral signs of drugs: glossitis, ery- each appointment; keep patient’s NTG tab-
thema signs, hairy tongue, lichenoid eruptions, lets handy during dental appointment; make
trigeminal neuralgia. dental appointment short and in morning.
3. Common drugs that affect dental treatment: (2) When combined with alcohol, can cause
a. Antihypertensives: used to reduce high blood severe hypotension.
pressure (HBP) with cardiovascular disease (3) Common drug: NTG (Nitrostat).
(CVD). e. Cardiac drugs:
(1) Can cause orthostatic (postural) hypoten- (1) Used with CHF; strengthen myocardial
sion (reduced blood pressure): sit patient up contractility.
Clinical Treatment   347

(2) May present with swollen extremities and (d) Lithium (Lithobid): treats manic-
report shortness of breath; seat in upright ­depressive illness (bipolar disorder).
(45°) position to increase ease of breathing. k. Tranquilizers: may be taken daily or ONLY for
(3) Common drugs: digitalis (Lanoxin), enala- antianxiety premedication because of anxiety
pril (Vasotec), captopril (Capoten). and fear of dental care.
f. Anticonvulsant drugs: used to reduce inci- (1) Result in CVS depression; may be drowsy,
dence of seizures with epilepsy or Down fatigued, or sedated.
­syndrome. (2) May cause xerostomia (see earlier discus-
(1) Stimulate gingival growth, resulting in gin- sion).
gival hyperplasia (see earlier discussion). (3) Long-term use can cause tardive dyskine-
(2) Common drugs: phenytoin (Dilantin). sia, involuntary facial movements of facial
g. Vasoconstrictors: used to alleviate symptoms of muscles and/or tongue; may have bruxism
asthma and emphysema; taken as inhalants. with attrition.
(1) Can cause xerostomia (see earlier discus- (4) Be aware of signs of sucidal or depressive o1820
sion); frequently associated with inflamed tendencies; refer immediately to physician.
gingival tissues because of mouth breath- (5) Common drugs: diazepam (Valium), alpra-
ing (associated with asthma). zolam (Xanax).
(2) Need to follow inhaler use instructions
(discussed earlier). PATIENT ASSESSMENT  
(3) May need to alter local anesthetic agent; Patient assessment is necessary to establish baseline in-
may need to avoid use with sulfites (vaso- formation on general health; includes taking vital signs
constrictors, articaine). as well as performing extraoral and intraoral examination
(4) Common drugs: albuterol (Proventil, and dental and periodontal evaluations. Obtaining base-
­Ventolin). line information allows determination of treatment needs
h. Thyroid drugs: treat hypothyroidism (MOST and subsequent development of treatment plan.
common type) by increasing metabolic rate. • See Chapter 10, Medical and Dental Emergencies: vital
(1) Have NO oral side effects; patient may signs.
have goiter or surgical removal of gland.
(2) Common drugs: levothyroxine (Synthroid). Patient Examination
i. Antiinflammatory drugs: depress inflammatory Patient examination includes both extraoral and intra-
response and treat adrenocortical insufficiency, oral examination of the soft and hard tissues of the head
rheumatoid arthritis (RA), and respiratory dis- and neck as part of the assessment of the patient. Recog-
ease (emphysema): nition of abnormal findings may be critical in preserving
(1) High risk for candidal infections (see ear- the overall health. MUST be recorded in chart (see docu-
lier discussion) and oral ulcerations. mentation discussion).
(2) Common drugs: corticosteroids such as • See Chapters 4, Head, Neck, and Dental Anatomy:
prednisone (Deltasone). head and neck anatomy; 6, General and Oral ­Pathology:
j. Antidepressants: taken for mental disorders, ­diagnosis.
weight loss, tobacco cessation, sleep disorders, A. Examination technique: includes visual observation,
temporomandibular disease (TMD). palpation, auscultation, olfaction.
(1) May cause xerostomia (see earlier discus- 1. Visual observation, BOTH direct and indirect
sion) and taste alterations; may cause brux- (mouth mirror), to examine intraoral and extraoral
ism with resultant attrition. structures:
(2) May need to alter local anesthetic agent and a. Adopt organized, sequential pattern to AVOID
AVOID levonordefrin (such as with TCAs; omitting areas.
see next). b. Look for abnormal color (change to red, black,
(3) Common drugs: blue, white, dark brown; spread of color), size
(a) Tricyclic antidepressants (TCAs): ami- (change is sudden or ongoing), or change in ap-
triptyline (Elavil). pearance (irregular margins, elevation, texture
(b) Selective serotonin reuptake inhibi- becomes scaly, crusty, or ulcerated).
tors (SSRIs): bupropion (Wellbutrin), c. Inform patient of the results of examination.
­sertraline (Zoloft), paroxetine (Paxil), 2. Palpation used to examine tissues and underlying
escitalopram (Lexapro). structures:
(c) Monoamine oxidase inhibitors a. Digital: one finger used to examine tissue (hard
(MAOIs): phenelzine (Nardil). palate).
348   Saunders Review of Dental Hygiene

b. Bidigital: tissue grasped between finger(s) and (2) Treatment: agrees to continue treatment.
thumb (cheeks, alveolar ridges, lips, vestibule, (3) Consent: documentation of consent, includ-
tongue, ducts, floor of mouth, larynx). ing risks, benefits, alternative treatment
c. Manual: all fingers of one hand used to grasp ­options.
tissue (anterior deep and superficial cervical (4) Referral: patient referred to specialist by
nodes). supervising dentist.
d. Bimanual: finger(s) and thumb from each hand
applied simultaneously to examine tissues (floor Extraoral Examination
of the mouth). Patient SHOULD be seated in an upright position for
e. Bilateral: bilateral structures examined simul- extraoral examination of the head, face, and neck areas,
taneously to detect differences between sides unless NOT able. Good lighting and exposure of the area
(­temporomandibular joint [TMJ], inferior border being assessed are essential (e.g., patient’s collar and tie
of mandible); place fingers on anterior border of loosened, glasses removed).
jaw, palpate distally on BOTH right and left sides A. Overall physical appearance by visual examination:
at same time. 1. Unsteadiness of gait: CANNOT walk at a normal
f. Circular compression: fingers move in circu- pace and maintain balance because of orthopedic
lar motion while applying pressure (occipital disabilities or influence of drugs.
nodes); AVOID on palate. 2. Restricted mobility: limps while walking, which
3. Auscultation used to listen to body sounds to de- may indicate injury to hip, leg, knee, or foot or
termine a change from normal: systemic disease; investigate possible surgery that
a. TMJ sounds, such as clicking, grinding, or required pins, joint repair, or prosthesis.
popping; make note if pain is associated with 3. Imbalance: CANNOT walk without losing bal-
sounds. ance; may be due to equilibrium problem, inner
b. Hoarseness or cough; make note of how long it ear infection, loss of muscular strength, or damage
has persisted. from CVA.
c. Speech disorders, such as slurred speech, stut- 4. Stature: appears stiff while standing, sitting, or
tering, or extra loud speech. walking; may indicate back problem or arthritis; if
4. Olfaction to detect unusual smells: assumes a slumped position, may be due to lack of
a. Bad breath (halitosis): poor oral hygiene, muscle tone resulting from a CVA.
necrotizing periodontal disease, GIT difficul- 5. Difficulty breathing: appears out of breath while
ties, overconsumption of proteins; MOST walking or right after walking; may indicate CHF,
comes from coated tongue; encourage tongue lung disease, or other systemic disorder.
­cleaning. B. Hair by visual examination and palpation:
b. Acetone breath: indicative of DM, from in- 1. Unusual amount: loss or thinning may be result of
creased processing of ketones. chemotherapy; excessive growth (hypertrichosis)
c. Alcohol breath: recent alcohol consumption, caused by hormone disorder.
­alcoholism. 2. Unusual distribution: as in alopecia, with loss of
B. Documentation (see Chapter 18, Ethics and Jurispru- hair in patches.
dence): 3. Lack of luster: poor nutrition; coarse texture with
1. To record findings that may be significant in the hypothyroidism.
overall diagnosis and recommended treatment. 4. Scalp lesions: psoriasis, previous injury, basal cell
2. As legal record in case of a claim or patient identi- carcinoma (BCC); lice: small insects, nits appear
fication (accident or disaster). as tiny white globules at hair root; spreads easily.
3. As baseline data for subsequent examination and C. Face by visual examination and palpation:
future treatment planning. 1. General expression can indicate general mood
4. Standard of recording: (e.g., happy, sad, angry, indifferent).
a. Requires adequate space to record in a brief, ac- 2. Asymmetry: facial paralysis (along with lack of
curate manner. blinking) with CVA or Bell’s palsy.
b. Includes diagnosis and treatment alternatives 3. Drooling, lack of expression, diminished blinking:
and/or recommendations. Parkinson’s disease.
c. Summarizes patient’s response, listing wants, 4. Unusual muscle twitching and loud vocal sounds:
needs, expectations. Tourette’s syndrome.
d. Indicates treatment to be rendered: 5. Bruising, bleeding, or burned areas: physical
(1) NO treatment: refuses to follow through abuse, anticoagulation therapy, or blood disorder.
with treatment and/or recommendations. 6. Pigmentation: Addison’s disease or melanoma.
Clinical Treatment   349

D. Skin by visual examination and palpation: 4. Border of mandible: bimanual palpation from
1. Color: indicates general health status (e.g., illness, midline to posterior angle for changes in contour
disease, or trauma); note paleness, redness, acne, and pain.
rosacea, sunburn, patches of pigmentation. 5. Mentalis muscle: digital palpation, rolling tissue
2. Textures: unusual ones should be noted, such as over mandible, for smoothness or restriction in
scarring, clamminess, firmness, swelling, acne. swallowing movement.
3. Lesions: should be documented, noting color, size, 6. Lymph nodes: assessed for tenderness or pain,
shape, surface texture (note any changes in pig- swelling, enlargement, unusual hardness, fixed
mented nevus). ­position:
E. Eyes by visual examination and reaction to light a. Occipital: bilateral palpation, with head tilted
source: forward.
1. Pupil size: normal; dilated or pinpoint may be b. Auricular (anterior and posterior), parotid
­related to drug abuse. ­(superficial), facial: bilateral palpation.
2. Sclera color: if yellow, may indicate jaundice; if c. Superficial cervical (submental, submandibular,
red and crusty, conjunctivitis. anterior and external jugular): digital palpation
3. Bulging (exophthalmos) or puffy and swollen eyes with fingers anteriorly from midline of man-
with edematous eyelids: possible thyroid disorder. dible with head down to posteriorly to angle
4. Eyes without palpebral fissures and with broad, of mandible, rolling tissue over jaw, and then
flat nose and low-set ears: Down syndrome. down to and along sternocleidomastoid muscle
F. Nose by visual examination and palpation: (SCM), with patient’s head to side.
1. Obstructed airway: congestion and/or a structural d. Deep cervical (superior and inferior): palpating
defect or ulceration related to drug abuse. deep tissues along SCM with thumb and fin-
2. Secretions: sinus drainage; yellow or green secre- gers, with head to side.
tion: infection or drug use (constant wiping). e. Accessory and supraclavicular: palpating con-
3. Red butterfly-shaped lesion over the nose: sys- tinuing down into clavicular area, with shoul-
temic lupus erythematosus (SLE). ders up and forward.
4. Rhinophyma: overgrowth of sweat and sebaceous 7. Submental and submandibular salivary glands: bi-
glands; appears red and bulbous with increased al- lateral digital palpation for asymmetry, noncontig-
cohol consumption. uous borders, pain, swelling, enlargement, unusual
G. Ears by visual examination and palpation: hardness, difficulty in swallowing.
1. Hearing loss and use of hearing aids: will require 8. Larynx: bimanual palpation for unrestricted
clinician to look at patient while speaking; during ­movement.
use of ultrasonic scaler, patient may need to turn 9. Thyroid gland: displacing gland to one side of
off hearing aid. neck, then combination of bimanual and manual
2. Infections: may affect ability to hear; may also af- palpation inferolaterally to cartilage (“Adam’s
fect balance. apple”) while patient sits upright; ask patient to
3. Tinnitus (ringing): may cause humming or buzzing swallow (may need glass of water) to check for
in ear; may result from long-term use of aspirin, mobility; note asymmetry, nodules, enlargement
infection, or environmental noise trauma. (goiter), or surgical removal.
H. Bones, muscles, lymph nodes, glands using visual
and palpation: Intraoral Examination
1. Parotid glands: bilateral circular compression for Patient should be seated in a supine position (see earlier
pain, swelling, enlargement, hardness; salivary flow position discussion). Use preprocedural antimicrobial
observed at opening of duct across from maxillary mouthrinse and have client remove any pigmented lip-
first molar on buccal mucosa when gland is milked. sticks. Apply nonpetroleum lubricant to cracked and dry
2. TMJ: bilateral palpation with BOTH index fingers areas to make examination more comfortable, and remove
slightly anterior to outer meatus, have patient open any removable appliances. Overall, note level of saliva and
and close several times, slowly; note pain, click- whether dry mouth (xerostomia) is present.
ing, popping, grinding, and restriction in opening A. Lips by visual examination and bidigital palpation:
or closing; note any deviations, including during 1. Changes in size: swelling or allergic reaction (rubber
patient interview; may have TMD. latex).
3. Masseter and temporalis muscles: bilateral circular 2. Chapping: mouth breathing, nutritional defi-
compression for pain, swelling, enlargements, un- ciency or environmental conditions; cracking,
usual hardness; masseter may be enlarged because with angular cheilitis, candidiasis, or vitamin B
of bruxism and/or clenching habits. deficiency.
350   Saunders Review of Dental Hygiene

3. Blistering: ulcers; herpetic lesions; irritations, 5. Recession: loss of periodontal attachment, caused
from lip biting or trauma. by vigorous toothbrushing, abfraction, or gingival
4. Limited motion: scarring, trauma, or paralysis. disease or surgery.
5. Unusual color given overall pigmentation; loss of 6. Mucogingival defects:
vermilion border. a. Lack of attached gingiva: tooth positioning,
6. Abnormal texture, lack of firmness or moistness; hereditary factors, oral habits, poor oral hy-
with dehydration and sun damage. giene.
7. Lower lip: high-risk area for oral cancer, with loss b. Stillman’s clefts (comma shaped): poor oral hy-
of vermilion border, fibrosis, ulceration. giene and occlusal trauma.
B. Labial and alveolar mucosa using visual examination c. High frenum attachments: muscle pull on at-
and bidigital palpation: tached gingiva.
1. Ulcerated lesions: such as herpetic lesions, aph- E. Hard and soft palate using visual inspection and digi-
thous ulcers. tal palpation:
2. Tight frenum attachments: can cause gingival de- 1. Torus (plural, tori): hard protruding bony structure
fects such as loss of attached gingiva (mucogingi- (benign exostosis), NOT of significance unless
val defect). partial or full denture is being constructed, then
3. Amalgam tattoo: blue or black macules of varying may need to be reduced or removed.
size on soft tissues. 2. Ulcerations: fluid-filled lesions surrounded with
4. Enlarged mucous retention cyst (mucocele): red halo with aphthous ulcers, burns, viral infec-
trauma to duct. tion, or autoimmunity (broken blisters).
5. Hyperkeratosis: use of lozenges, drugs, or spit 3. Trauma: mechanical or chemical irritation; should heal
(smokeless) tobacco products held in mandibu- within 10 days, if not, further investigation needed to
lar vestibule; careful evaluation of changes is rule out pathological condition or child abuse.
­essential. 4. Stomatitis: ranges from small, red, petechia-like
6. Smokeless (spit) tobacco lesion: hyperkeratinized lesions (nicotine and denture stomatitis) to gener-
tissue; white, sometimes corrugated, in appear- alized and granular (denture stomatitis) to ulcer-
ance. ative (aphthous stomatitis) to intense redness with
7. Torn or lacerated frenum (lingual frenum too): of- focal bone loss (necrotizing stomatitis).
ten caused by forced feedings, binding, gagging 5. Petechiae: small red dots on mucous membranes,
(abuse). caused by trauma or systemic disease (mononucle-
8. Tissue trauma: biting, abrasion, burns; horizontal osis on palate).
bruises at commissures may indicate binding or 6. Fistulas: pimplelike lesions with periodontal or
gagging with abuse. periapical abscess.
C. Buccal mucosa using visual examination and 7. Soft palate: high-risk area for oral cancer, with red
­palpation: and/or white lesions, bleeding, ulceration; also area
1. Check parotid (Stensen’s) duct using bilateral pal- for trauma from abuse.
pation: pain, enlargements, tumors, calcified areas; F. Floor of mouth using visual inspection and ­palpation:
milk the ducts to confirm function. 1. Check submandibular (Wharton’s) duct: can be-
2. Linea alba: white line paralleling occlusal plane come blocked with mucous plugs or sialoliths
with trauma; also note trauma from toothbrushing, (present on occlusal radiograph).
cheek biting (irritation fibroma). 2. Varicosities: often found sublingually on elderly.
3. Wickham’s striae: thin white lines with lichen 3. Large mucous retention cyst (ranula): blocked or
­planus. traumatized ducts of submandibular and sublin-
D. Gingiva using visual examination and palpation: gual glands; soft, sessile, slowly enlarging mass
1. Tissue trauma: caused by abrasion, burns, or cuts. on one side of floor of mouth; painless; once re-
2. Infection, with redness and swelling (inflamma- moved, may recur.
tion), such as McCall’s festoons (lifesaver mar- 4. Limited tongue movement: ankylosis of tongue;
ginal gingiva): inadequate dental biofilm removal may lead to swallowing and speech disorders.
and presence of periodontal disease, mainly gingi- 5. High-risk area for oral cancer, with red and/or
vitis, possibly periodontitis. white lesions, bleeding, ulceration.
3. Fibrosis: chronic periodontal disease, drug use, or G. Tongue using visual examination and palpation:
traumatic toothbrushing. 1. Coating: varying degree of keratinization of fili-
4. Exostosis (plural, exostoses): small benign form lingual papillae; food, drugs, bacteria (hali-
bone growths projecting from alveolar process, tosis); staining caused by tobacco use; tongue
MAINLY on maxillary facial surface. cleaning needed.
Clinical Treatment   351

2. Fissuring: presence of numerous grooves and e. Tongue thrust (during swallowing): anterior
crevices on dorsal surface and lateral borders of portion pushes between teeth; ideally SHOULD
tongue; entrap food and bacteria; tongue cleaning be positioned on palate behind maxillary central
needed; MORE common with Down syndrome. incisors while swallowing; retraining may be
o3160 3. Varicosities: often found on ventral surface on necessary if thrusting causes speech disorders
­elderly. or tooth positioning problems (malposed teeth).
o17840 4. Lateral border and ventral surface: high-risk area f. White plaques: lichen planus, candidiasis, or
for oral cancer, with red and/or white lesions, other systemic disease; definitive diagnosis is
bleeding, ulceration. needed.
5. Other common lesions of the tongue: g. Black hairy tongue: elongation of filiform with
a. Ulcers: aphthous is MOST common, with sys- dark staining: long-term use of certain drugs or
temic disease such as viral infections or autoim- tobacco; long-term rinsing with undiluted hy-
munities or with smoking cessation. drogen peroxide.
b. Irritation fibroma: trauma. h. Hairy leukoplakia: hyperkeratosis of filiform
c. Geographic: common benign condition involv- caused by Epstein-Barr virus (EBV); results in
ing filiform lingual papillae found on dorsum white corrugated appearance on lateral borders;
and borders; lesions are red areas with white associated with HIV/AIDS.
borders; outline of lesions changes, heals, reap- i. Hemangiomas and lymphangiomas: devel-
pears in different area. opmental benign tumors creating enlarge-
d. Median rhomboid glossitis (central papillary ments.
atrophy): flat or lump benign lesion on dor- j. Macroglossia: symmetrical enlargement; devel-
sum, NO filiform present; related to chronic opmental abnormality, cretinism, acromegaly;
­candidiasis. may be scalloped from pushing against the teeth.

CLINICAL STUDY  

Age 27 YRS SCENARIO

Sex ☒  Male   ☐  Female The patient has minimal cal-


culus and dental biofilm with
Height 5’2” slight gingival inflammation.
Weight 135 LBS

BP 110/65

Chief Complaint “I know I have to keep my mouth healthy, since it impacts all
of my health.”

Medical History Hemophilia


Contracted AIDS 15 years ago from blood transfusion

Current Medications Coagulation therapy prn


Undergoing highly active antiretroviral therapy (HAART)

Social History Rock band member

1. What is hemophilia, and what are its risks and treatment? 1. Hemophilias are a group of disorders involving blood-
2. What additional information should be known before clotting mechanisms. Diseases are genetic, do not pro-
the patient is treated? duce factor VIII (hemophilia A), factor IX (hemophilia
3. Is the patient’s sister likely to have hemophilia? B, or Christmas disease), factor IX (hemophilia C), or
4. Are there any special treatment considerations be- von Willebrand’s factor (vWF). Treatment includes
cause of the patient’s hemophilia or HIV status? coagulation therapy.
5. Why were hemophiliacs at risk for AIDS before 2. Additional information needed before treatment
1985? would include type and severity of hemophilia, treat-
6. What HIV symptoms may be discovered during the ment regimen for control of hemophilia, drugs being
oral examination? taken for hemophilia, HIV status including CD4 cell
352   Saunders Review of Dental Hygiene

counts (lymphocyte subset cell count), presence of any A. Teeth are identified by numbers or symbols:
AIDS-associated conditions (includes oral signs), and 1. Universal Tooth Designation System: MOST
any drugs (both prescription and OTC) being taken widely accepted (and used on NDHBE).
to manage HIV condition. In addition, INR numbers a. Permanent teeth: numbered #1 to #32; primary
would be needed to ascertain bleeding levels; medical (deciduous) teeth: letters A to T.
consult needed. Antifibrinolytic agents (Cyklokapron) b. Chart clockwise for BOTH as if you are looking
effectively prevent oral bleeding when they are com- at patient smiling.
bined with a preventive dose of clotting factor and can 2. Overview of dentition:
be used with oral surgery. a. Missing teeth: determined by clinical absence
3. Hemophilia A, B, and C are rare in women; however, (developmentally or extraction), radiographic
von Willebrand’s disease does occur in both men and findings, reported dental history; charted
women. with single vertical line or with X through
4. Standard precautions are followed for every patient. tooth.
Light debridement (scaling) is only instrumentation b. Unerupted permanent tooth: if completely un-
needed in the patient’s case because of minimal deposits erupted, circled entirely; if partially erupted,
and inflammation; may want to avoid ultrasonic scalers ONLY that portion that is not exposed is circled
and use manual instrumentation, since aerosols created (common with third molars).
may be a health risk for immunocompromised patient. c. Supernumerary tooth is an extra tooth: charted
However, he should be seen for preventive maintenance by drawing in general location, noting as “Su.”
every 3 months because of high-risk status. Excellent d. Root canal has pulp tissue removed (endodon-
oral hygiene self-care and preventive therapy reduce tic therapy): charted by placing vertical line
chances for oral infection and ­bleeding. through root and noting as “RC.”
5. Before 1985, blood stores used for transfusions were e. Full or partial removable denture or removable
not required to be tested for HIV virus, and many he- prosthetic appliance: missing teeth are charted
mophiliacs contracted virus. with vertical lines and area the appliance re-
6. Thorough intraoral and extraoral examination is criti- places is bracketed; labeled “PUD” (partial
cal for noting progression of patient’s HIV status, since upper denture), “PLD” ­(partial lower denture),
many early indicators of disease progression are exhib- “CUD” (complete upper denture), or “CLD”
ited in the mouth. Palpate for any persistent generalized (complete lower ­denture) (Figure 11-1).
lymphadenopathy (PGL) and visually observe for any f. Implant: surgically placed into the bone of jaw,
skin lesions such as Kaposi’s sarcoma, purpura, or her- after healing, prosthesis (crown, bridge, den-
pes. Check for common oral signs of progressing AIDS ture) is constructed over implant; charted by
virus, such as candidiasis, ­Kaposi’s sarcoma, hairy leu- “X” marked on root(s), with prosthesis drawn
koplakia, gingival inflammation (linear gingival ery- in and implant is noted as “I.”
thema [LGE]), or necrotizing periodontal disease. B. Restorations (uses G.V. Black’s Classification)
­(Figures 11-2 and 11-3):
DENTITION ASSESSMENT   1. Temporary: placed on or in tooth for short period;
Along with the medical history, the dental history is includes cement, preformed crowns, or chairside-
taken to complete the patient’s health history. Questions made acrylic crown; charted by outlining, shading,
concerning the patient’s experiences with dentistry, as noting as “T.”
well as needs, should be discussed and reported in the re- 2. Tooth-colored (esthetic) restorations: may be
cords. Then the dentition can be assessed for caries, attri- acrylic, composite resin, glass ionomer cements,
tion, abrasion, erosion, fracture, inflammation, necrosis, porcelain.
restoration, and other conditions. These assessments can a. Composite resins can be placed on anteriors
involve visual, radiographic, and clinical evaluation and or posteriors; material cured by visible (blue)
a variety of diagnostic tests. Dental evaluation includes a light; charted by outlining, shading, noting as
written record of the assessment of all teeth using com- “C,” “CR,” “GIC,” or “R.”
monly acceptable dental notation (listed below in order of b. Acrylic jackets: crowns; charted by outlining,
notation), done manually or using software to save time shading, noting as “AJC.”
and improve efficiency. c. Porcelain: used for a crown, inlay, or veneer;
• See Chapters 4, Head, Neck, and Dental Anatomy: dental charted by outlining, shading, noting as “PJC,”
anatomy, charting, orthodontic evaluation; 6, ­General and “PI,” or “Ven,” respectively; if metal is used
Oral Pathology: oral diseases and conditions; 15, Dental as substructure on crown, noted as “PFM” for
Biomaterials: restorative materials. ­porcelain-fused-to-metal.
Clinical Treatment   353

Dental and Periodontal Charting

1 2 3 4 5 6 7 8 9 10 11 12 13 14 15 16

Date:

FACIAL
LINGUAL

Date:

RIGHT LEFT

Date:

LINGUAL
FACIAL

Date:

32 31 30 29 28 27 26 25 24 23 22 21 20 19 18 17

Legend Additional Findings


#1, impacted; #2, caries, #3, gold onlay; #4, MO
composite restoration; #5 & 6, abfraction; #8, acrylic jacket
crown; #10, root canal and lingual composite; #11 & 12,
abfraction; #13, MOD gold inlay; #14, porcelain-fused-to-metal
crown; #15, occlusal amalgam; #16, impacted; #17, missing
tooth; #18, full gold crown-abutment for bridge; #19, missing
tooth-pontic for bridge; #20, full gold crown-abutment for bridge;
#22, Class V composite; #27, Class V gold foil; #28, occlusal
composite; #29, MOD amalgam; #30, mesio-occusal temporary;
#31, full gold crown; #32, missing tooth

Figure 11-1  Dental charting. Anatomical charting of the permanent dentition, including missing teeth, restored teeth,
carious lesions, and additional notations.

3. Enamel (pit and fissure) sealant: resin (clear or tinted) a. Full high noble, noble, and base metal crowns;
mechanically or chemically bonded (see above); charted by outlining, shading, noting as “FGC”
charted by outlining, shading, noting as “S” or “SL.” or “GC.”
4. Amalgam: alloy used for Class I, II, V; charted by b. Three-quarter crowns; charted by outlining,
outlining, shading, noting as “A.” shading, noting as “¾ GC.”
5. Metal casting: made from a wax pattern of tooth c. Onlay (covers at least one cusp tip); charted by
preparation; classified depending on metal ­content: outlining, shading, noting as “GO.”
354   Saunders Review of Dental Hygiene

Dental and Periodontal Charting

1 2 3 4 5 6 7 8 9 10 11 12 13 14 15 16

Date:

FACIAL
LINGUAL

Date:

RIGHT LEFT

Date:

LINGUAL
FACIAL

Date:

32 31 30 29 28 27 26 25 24 23 22 21 20 19 18 17

Legend Additional Findings


Teeth #'s 1, 2, 3, 4, 5, 12, 13, 14, 15, 16 are missing and
replaced with a partial upper denture.
Teeth #'s 17-32 are missing and replaced with a complete
lower denture.

Figure 11-2  Dental charting of partially edentulous permanent dentition. Anatomical charting that demonstrates bracketing
of teeth included in the partial, as well as missing teeth.

d. Inlay (lies between cusps of the teeth); charted abutment crowns are outlined and shaded as per
by outlining, shading, noting as “GI.” material.
6. Bridge (fixed denture): pontic replaces miss- 7. Gold foil: soft metal condensed used in any cari-
ing tooth; charted by outlining, shading crown ous area, cannot withstand heavy occlusal wear,
portion, and drawing vertical line or an “X” on hard on pulp, seldom used because of technically
root; double horizontal lines are drawn to abut- demanding and time-intensive placement; charted
ments (crowns adjacent to pontic for support), and by outlining, shading, noting as “GF.”
Clinical Treatment   355

(2) Classification by lesion size: o17900


Classification (a) Size 0: initial lesion that can be identi- o17910
fied but has not yet resulted in surface
cavitation; may be possible to undergo
Class I
remineralization. o17920
(b) Size 1: smallest minimal lesion re-
Class II quiring operative intervention; just o17930
beyond remineralization.
(c) Size 2: moderate-sized cavity; still
Class III
sufficient sound tooth structure to o17940
maintain integrity of the remaining
Class IV crown and accept occlusal load.
(d) Size 3: cavity needs to be modified and
enlarged to provide some protection for o17950
Class V
remaining crown from occlusal load;
already split at cusp base or, if not pro-
Class VI tected, a split is likely to develop.
(e) Size 4: cavity is now extensive fol-
Figure 11-3  G.V. Black’s Classification. lowing loss of cusp from posterior
tooth or incisal edge from ­anterior.
8. Overhang: extension of restorative material beyond 2. Recurrent caries: new caries around previously placed
confines of preparation, extra bulk makes oral hygiene restoration; charted by outlining restoration in red.
difficult in the area, may lead to periodontal irritation; 3. Decalcification: area that has started to demineral-
noted as “O” or “OH” and drawn by ­location. ize, chalky and soft; charted by outlining, noted as
C. Dental caries are determined by visual, explorer, laser “decal” or can be cross-hatched.
light, radiographic examination: D. Other traumatic lesions:
1. New caries: charted by outlining and shading 1. Open contact: lack of proximal contact between
in red and/or use of two-digit number. two teeth, can be due to periodontal disease, un-
a. G.V. Black’s Classification (Figure 11-3): restored carious lesion, iatrogenic dentistry, or
(1) Classification by location: discrepancies in alignment of permanent dentition;
(a) Class I: pits and fissures on occlusal, can cause food impaction resulting in increased
buccal, lingual surfaces of posteriors caries and periodontal irritation; noted with two
and lingual of ­anteriors. parallel lines between teeth.
(b) Class II: proximal surface of posteriors. 2. Malposed teeth: located out of normal position in
(c) Class III: proximal surface of anteriors. dental arch; caused by lack of space or unusual
(d) Class IV: proximal surface, including eruption:
incisal edge of anteriors. a. Rotated teeth marked with arrow pointing to-
(e) Class V: gingival one third of facial ward surface rotated, usually marked across fa-
and lingual surfaces of anteriors and cial view.
posteriors. b. Lingual version noted with arrow pointing from
(f) Class VI: cusp tips on molars, premo- incisal edge away from facial view.
lars, canines. c. Labial version noted by arrow pointing toward
(2) Classification by surfaces: occlusal view.
(a) Simple: one tooth surface (e.g., occlusal). 3. Attrition: loss of structure caused by tooth-to-tooth
(b) Compound: two tooth surfaces (e.g., contact on occlusal and incisal surfaces, from ex-
mesio-­occlusal [MO]). cessive horizontal forces such as bruxism; charted
(c) Complex: more than two surfaces by outlining, shading, noting with horizontal line
(e.g., mesio-occlusal-distal [MOD]). across affected portion.
o17850 b. Carious lesion classification (with minimal in- 4. Fractured tooth: result of trauma, confirmed by us-
tervention dentistry) (Figure 11-4) using two- ing radiograph, illumination, and/or reported his-
digit ­numbers: tory; noted with red line that resembles fracture
o17860 (1) Classification by site: configuration.
o17870 (a) Pits and fissures on the occlusal surface. 5. Abrasion: mechanical wearing other than by mas-
o17880 (b) Contact area. tication, such as aggressive brushing or oral habits
o17890 (c) Cervical areas including exposed root like opening pins or biting on pen; charted by out-
surfaces. lining, shading, noting as “abr.”
356   Saunders Review of Dental Hygiene

Size

No cavity Minimal Moderate Enlarged Extensive


Site
0 1 2 3 4

Pit or fissure 1.0 1.1 1.2 1.3 1.4


1

Contact area
2.0 2.1 2.2 2.3 2.4
2

Cervical
3.0 3.1 3.2 3.3 3.4
3
f0040
Figure 11-4  Classification with caries lesions grid (with minimal invasive dentistry).

6.  Abfraction: area at cervical one third that appears dentin); ­ pitting may also be present; caused by
as a V-shaped notch, associated with occlusal trauma or high fever during tooth formation or
forces; charted by outlining, shading, noting as due to localized fluorosis if permanent tooth (from
“abf.” fluoride ingestion); noting as “hycal.”
7.  Erosion: chemical wear, caused by acid sub­ 10.  Dilaceration: distortion of a root and crown linear
stance, involves several teeth and damage over relationship usually caused by trauma; noting as
a broad surface; area is bracketed and noted as drawing in shape of root.
“Ero.” E. Other developmental conditions are charted by mak-
8. Ankylosed teeth: roots that are fused to underly- ing special note or drawing on chart:
ing bone, common on first and second primary 1.  Fusion: union of two tooth buds involving dentin
molars, generally appear lower than occlusal along entire tooth, root, or crown; each has sepa-
plane, sound hollow when tapped; noting as rate pulp canals.
“ankl.” 2.  Gemination: splitting of a single tooth germ; sin-
9. Hypocalcification (enamel hypoplasia, Turner’s gle root and pulp canal.
tooth or “white spot”): area on tooth that has not 3.  Dens-in-dente: enamel organ is invaginated inter-
completely calcified, is white or yellow (exposed nally (“tooth-within-a-tooth” on radiograph).

CLINICAL STUDY  

Age 75 YRS SCENARIO

Sex ☒  Male   ☐  Female Clinical and radiographic examinations


of the patient noted the following:
Height 5’11” (1) tooth #30 has a swelling on the
Weight 168 LBS buccal mucosa, about 6 to 8 mm in
diameter; large existing MOD amal-
BP 130/60 gam; the radiograph demonstrates
distal root caries, as well as a radiolu-
Chief Complaint “Not sure what is going on in my cency located at the apex of the distal
mouth with this gumboil!” root, and (2) teeth #30 and #31 are in
crossbite with #2 and #3.
Medical History Bypass surgery 4 months ago

Current Medications captopril (Capoten) 25 mg tid


hydrochlorothiazide (Lopressor)
25 mg qd

Social History Lived alone since wife died 6 months


ago
Retired chess player
Clinical Treatment   357

1. Does the patient need antibiotic premedication before with ­ speaking, taste, and nutrition. Increase in water
oral prophylaxis? Are there any contraindications with consumption by sipping it, use of artificial saliva, air
his treatment because of his medical history? What is humidification, water-based lubrication, sucking on
his ASA physical classification status at this time? sugarless candies or chewing gum (especially those
2. How should the lesion around tooth #30 be palpated? with xylitol) would all help to increase saliva produc-
3. What are common oral side effects of the patient’s tion or relieve mouth dryness. Brush-on or tray pre-
drugs? What recommendations can be made to help scription–strength fluoride (probably sodium fluoride)
relieve these symptoms? should be recommended for application just before
4. Regarding his crossbite, how are the mandibular teeth bed with directions to expectorate the excess and not
positioned relative to the maxillary teeth? rinse the mouth. Many OTC fluoride mouthrinses con-
tain alcohol and may be contraindicated if the mucous
1. With history of heart bypass surgery, does not require membranes are sensitive from the xerostomia. Re-
any antibiotic premedication to protect from infective placement therapy with calcium products can also be
endocarditis (IE). Bypass surgery does not involve used to remineralize teeth.
heart valve tissues, which when damaged and replaced 4. Mandibular teeth #30 and #31 are buccal to buccal
with artificial valves are at risk for IE. However, there cusps of maxillary teeth #2 and #3. Thus crossbite
is 4- to 6-week delay after cardiac surgery (or any occurs when mandibular teeth are placed facially to
cardiac event or episode) for emergency and elec- maxillary teeth. Normal occlusion would have #2 and
tive dental treatment until heart has healed. Thus any #3 buccal to the mandibular buccal cusps of #30 and
emergency treatment by supervising dentist, possibly #31. Thus permanent maxillary first molar to mandib-
for the lesion on #30, will need medical consult with ular arch is established when ML cusp interdigitates
physician before proceeding. However, presence of with central fossa of mandibular first molar.
oral infection is serious with his medical history. Thus
he is ASA IV because of recent bypass surgery. He Evaluation of Tooth Stains
poses significant risk, since patients in this category Teeth normally vary in color. However, stains are TRUE
have severe medical problem of greater importance to discolorations of teeth. Intrinsic stain is within the tooth
patient than planned dental treatment. Whenever pos- and CANNOT be removed by scaling and/or polishing,
sible, elective dental care should be postponed until although it may be reduced by whitening (bleaching), both
his medical condition has improved to at least ASA internally and externally. Endogenous intrinsic stain oc-
III. His blood pressure reading of 130/86 indicates a curs from internal influences. Exogenous intrinsic stain
prehypertension level and thus may not be fully regu- is caused by environmental influences that result in intrin-
lated with his antihypertensive drug therapy or life- sic stain. Extrinsic stain occurs on the external surface of
style changes; need to mention this to patient so he can the tooth and can be removed by scaling and polishing;
consult on it also with physician. occurs in variety of colors. Over time, however, extrinsic
2. Before palpating tooth #30 and adjacent tissues, ob- stain can become intrinsic because of the uptake by ex-
serve and note any suppuration (pus) or presence of posed dentin that occurs with recession and attrition.
a papule (“gumboil”) that indicates a draining fistula. • See Chapters 12, Instrumentation: scaling and polish-
Use digital palpation with light pressure to examine ing; 15, Dental Biomaterials: tooth whitening.
lesion for tenderness, draining fluid (pus), or any ir- A. Endogenous intrinsic stain:
regular tissue (i.e., hard, firm immovable mass). Bi- 1. Fluorosis: white and/or brown spots, milky opal-
digital compression and circular movements can also escence, and in extreme cases, pitting and mottling
be used to palpate submandibular area for indications caused by excess oral fluoride intake during tooth
of lymph node drainage and swelling. development.
3. Xerostomia from salivary hypofunction is likely prob- 2. Tetracycline (Tetracyn): yellow and/or brown,
lem for this patient because of two antihypertensive caused by ingestion of antibiotic during tooth de-
drugs he takes; captopril (Capoten) is an angioten- velopment.
sin-converting enzyme (ACE) inhibitor, and hydro- 3. Tetracycline derivative, minocycline (Minocin):
chlorothiazide (HCTZ, Lopressor) is a thiazide-type bluish gray stain in adults.
diuretic. Changes in saliva consistency, amount, and 4. Amoxicillin (Amoxil): possibly white opacities
buffering capacity are often mistakenly associated caused by ingestion of antibiotic during tooth de-
with aging adults but are most often associated with velopment.
drug use rather than physiological changes. Other 5. Traumatized or infected pulp: gray or black as re-
problems created by xerostomia include increased risk sult of blood leaching into dentin.
of caries (especially root caries) and irritation or dry- 6. Age: yellow caused by thinning of enamel, allow-
ness of the mucous membrane that causes difficulties ing color of underlying dentin to show through.
358   Saunders Review of Dental Hygiene

7. Genetic and systemic anomalies: widely used system to initially describe the relationship
a. Hypocalcification: white opacities. of the permanent and primary teeth when in centric rela-
b. Amelogenesis imperfecta: white opacities, ­pitting. tion (MOST posterior position of mandible). In addition,
c. Dentinogenesis imperfecta: bluish, translucent. movements of the mandible in relationship to maxilla
d. Dentin dysplasia: bluish, translucent. are evaluated.
e. Porphyria: dark yellow or brown. • See Chapters 4, Head, Neck, and Dental Anatomy: nor-
B. Exogenous intrinsic stain: mal occlusion, occlusal evaluation, malocclusion; 13,
1. Caries: white, brown, black; caused by acid- Periodontology: occlusal trauma.
­producing dental biofilm, diet, vomiting.
2. Pulp necrosis: yellow, black, gray; caused by trauma. Dental Caries Pathogenesis and Diagnosis
3. Restorative materials: gray, black: result of materi- Dental caries is due to irreversible solubilization of tooth
als (amalgam, pins, posts) showing through enamel mineral by acid. (See earlier discussion on classification
or by amalgam staining surrounding tooth. and charting; later discussion on prevention by dental
4. Pulpitis: pink caused by inflammation and internal biofilm removal, fluorides, oral microbials, enamel seal-
bleeding within pulp chamber; internal resorption ants, as well as clinical care throughout a lifetime for pa-
can also appear pink. tients.)
C. Exogenous extrinsic stain: • See Chapters 7, Nutrition: cariogenic diet; 15, Dental
1. Noted mainly at cervical area and facial surfaces Biomaterials: caries treatment; 17, Community Oral
unless otherwise noted. Health: caries epidemiology, community prevention
2. Can become intrinsic over time and affect the incisal programs.
and occlusal surfaces with tooth wear (attrition). A. Etiology:
3. Need to AVOID MOST of these factors post 1. MAIN cause: for coronal lesions, Streptococcus
whitening procedures until tooth has rehydrated mutans with initiation, lactobacilli with progres-
for effectiveness and for long-term maintenance sion of the lesion (root caries discussed later).
(chromogenic agents). 2. Lesser cause: dietary acids and/or other means of
a. Yellow: caused by heavy dental biofilm buildup. acidic contact, such as gastric acids.
b. Green: caused by poor oral hygiene; can be- B. Pathogenesis:
come intrinsic through underlying decalcifica- 1. Tooth surface loses some mineral (calcium, phos-
tion; difficult to remove, but AVOID abrasive phate) from action of the acid at ≤5.0 pH (demin-
scaling or polishing because of possible de- eralization).
calcification; may also be due to staining of a. From dental biofilm bacteria after ingestion of
Nasmyth’s membrane on newly erupted teeth foods containing fermentable carbohydrates.
(MOST brushes off). (1) Low pH selects for aciduric organisms,
c. Black or black line: caused by iron compounds such as S. mutans and lactobacilli, which
from oral fluids that embed in dental biofilm; store polysaccharide (MAINLY S. mutans)
found in clean mouths on lingual surfaces and and continue to secrete acid (acidogenic)
near amalgam restorations. long after food has been removed by swal-
d. Brownish to black: caused by tobacco use. lowing.
e. Orange: rare, associated with chromogenic bac- (2) In addition, when fermentable foods are
teria. eaten frequently, low pH in dental biofilm
f. Tan to dark brown: from diet (food) such as red is sustained.
wine, tea, coffee, colas, vegetables, fruits, nuts, b. From dietary acids, includes citrus products,
candies. soft drinks, and sports drinks (especially
g. Brown or orange-tan: caused by antimicrobials diet).
chlorhexidine (increased with staining diet) and c. From gastric acids because of vomiting (buli-
stannous fluoride. mia, pregnancy, chemotherapy) or reflux (gas-
h. Gray-green: associated with marijuana use. troesophageal reflux disease [GERD]) or after
i. Metallic hues: vary in color and are due to in- bariatric surgery.
gestion of industrial dust and various foods and 2. Normally, tooth minerals are replenished by saliva
water. between meals (remineralization); however, when
low pH is sustained, net loss of mineral from the
Occlusal Evaluation tooth occurs, producing caries.
Occlusion is the contact relationship of the maxillary 3. Addition of dry mouth from drugs or systemic dis-
and mandibular teeth when the teeth are fully closed. orders increases acid damage caused by lessening
Angle’s classification of malocclusion is the MOST of salivary buffering and clearance effects.
Clinical Treatment   359

Box 11-1  Caries Risk Factors


reads out number and audible signal if there is a
lesion.
a. Numbers of 11 to 20 need preventive or
Dental biofilm: quantity and composition
restorative treatment, depending on caries
Previous caries experience
Frequent exposure to fermentable carbohydrates or oral risk; >21 indicates probability of subsur-
acids face lesion and need for restorative treat-
Active periodontal therapy ment.
Saliva: quantity and quality b. NOT able to use on coronal smooth surfaces,
Number of periodontal pockets >3 mm
ONLY noncavitated suspected pit and fissure
Systemic illness or disability, cancer therapy, or stomach re-
gurgitation occlusal lesions.
Number of exposed roots, furcations, and crowded teeth 3. Quantitative light-induced fluorescence (QLF; Ins-
Tobacco use pektor Pro): uses fluorescence to determine demin-
Fewer than nine remaining teeth eralization; used on ALL coronal surfaces except
Low income
interproximally.
Orthodontic appliances or removable partial dentures
E. Clinical signs:
1. Noncavitation (incipient stage): atraumatic white
spot is present because of subsurface demineral-
C. Caries risk factors (Box 11-1). ization.
1. IMPORTANT to determine a patient’s risk factors 2. Early cavitation (early clinical stage): microorgan-
for caries development so as to create a treatment isms penetrate enamel rods.
plan that involves NOT just present concerns but 3. Late cavitation (advanced clinical stage, frank lesion):
also preventive needs (see CD-ROM for caries risk microorganisms enter dentin and approach tooth pulp
levels and treatment concerns). using dentinal tubules; can become painful.
2. There is overlap between caries risk and periodon-
tal disease risk as the patient may have MORE Types of Caries and Treatment
dental biofilm, poor oral hygiene, xerostomia, gin- Caries can be classified by rate of progression or etiology
gival recession. (see earlier discussion with charting). Includes rampant
3. Caries risk assessment methods: or early childhood caries, as well as recurrent or arrested
a. Chairside culture procedures: caries. Can also be classified by location, either pit and fis-
(1) Tests that allow estimate of number of sure or smooth surface (Figure 11-5). With pit and fissure
S. mutans or Lactobacillus organisms in progression, two triangles are noted histologically (one
saliva. triangle in enamel, other in dentin) with bases conjoined
(2) Salivary assessment tests: measure hydra- to each other at dentinoenamel junction (DEJ); includes
tion, conditions, resting and stimulated pH, occlusal, facial, and lingual surfaces. With smooth sur-
salivary buffering capacity, stimulated sali- face progression, the base and apex of the two triangles
vary flow. join; includes interproximal or root caries.
b. Software programs: assess individual patient’s A. Rampant caries: advanced or severe decay on mul-
caries risk based on a database. tiple surfaces of many teeth.
D. Diagnostic methods: explorer, mirror, air, dental light; 1. Professional considerations:
improved with loupes and intraoral camera; other di- a. Discussion of medical implications: vomiting;
agnostic methods: xerostomia induced by drugs, including metham-
1. Radiographs: radiolucencies in enamel and/or phetamine and sugar-laced drugs (nystatin); use
dentin; used MAINLY for interproximal smooth of spit (smokeless) tobacco that contains sugar.
surface caries; NOT considered “gold standard” in b. Close monitoring:
caries diagnosis; do NOT provide definitive diag- (1) Evaluate ability to remove dental biofilm at
nosis and have ONLY around 50% to 70% sensi- 1-week, 1-month, 3-month intervals.
tivity using visual means. (2) Confirm home fluoride and/or calcium use
a. Software associated with digital radiographs at each appointment.
can analyze changes in radiographic density (3) Have patient keep a diet diary for review at
to identify demineralized areas by way of da- next appointment (see Chapter 7, Nutrition).
tabase and determine probability that caries is c. Expose bitewing radiographs at MORE fre-
present. quent intervals.
2. Laser (DIAGNOdent): uses fiber optic transillu- 2. Recommendations to patient or caretaker:
mination and digital imaging fiber optic transillu- a. Dental biofilm removal with brush, floss, or
mination (DIFOTI), which interacts with detector, other aids.
360   Saunders Review of Dental Hygiene

A
Groove
Normal surface
Fissure
Intact surface layer
showing slight
demineralization
Enamel
Subsurface zones
showing increased
demineralization
Dentin

Caries in a Groove

B
Intact surface layer
Enamel showing slight
demineralization

Dentin Subsurface zones


showing increased
demineralization

Normal surface

Smooth Surface Caries


Figure 11-5  Caries process. (From Bath-Balogh M, Fehrenbach MJ: Illustrated
dental embryology and anatomy, ed 2, Philadelphia, 2006, Saunders/Elsevier.)

b. Nutritional counseling: reduce frequency of


ingesting simple-sugar foods; for example, de-
crease from three cans of sweetened, carbonated
beverages daily to one can and drink beverage
in 30 minutes using straw instead of sipping
over 3 hours; use non-citrus-based beverages.
c. Home fluoride use with OTC or prescription
mouthrinses or brush-on or tray application of
foam or gels (see earlier discussion of fluoride
and trays). Acute caries
d. Use of mineralizing calcium products both pro-
fessionally and at home, as well as xylitol prod-
ucts.
B. Early childhood caries (ECC; also, baby bottle car- Figure 11-6  Early childhood caries. (From Bath-Balogh M,
ies tooth decay [BBTD]): shallow acute cervical le- Fehrenbach MJ: Illustrated dental embryology and anatomy, ed 2,
sions of primary dentition caused by dental biofilm Philadelphia, 2006, Saunders/Elsevier.)
(MAINLY S. mutans) and/or dietary acids from
beverages in bottles for infants and young children
(­Figure 11-6): c. Breastfeeding has been proved NOT to be as-
1. Professional considerations: sociated; however, because cariogenic bacteria
a. MAINLY involves primary maxillary anteri- (especially S. mutans) are transmitted soon after
ors (C, D, E, F, G, H) because of contact with FIRST teeth erupt, decreasing guardian’s lev-
nipple; tongue and saliva help cleanse other els may decrease child’s risk; encourage dental
teeth. visit to ensure guardian’s own oral health.
b. Fluoridated water has positive effect, which d. May be at severe levels (S-ECC), which can
may be missed if bottled water used; NOT able progress and cause pulpal exposure, resulting
to use fluoridated dentifrice until child will in large restorations and early tooth loss owing
NOT swallow it. to thinness of overlying enamel and dentin of
Clinical Treatment   361

primary teeth; treatment ranges from compos- factors as buffering, washing, antibacterial,
ites to stainless steel crowns to extraction. immunological (see earlier discussion).
e. Fluoride varnish is BEST method for topical (2) Risk factors for xerostomia:
fluoride application for primary teeth (see later (a) MOST systemic medications.
discussion). (b) Radiation therapy.
f. Prevalence is epidemic, MOST in minority (c) Pathological conditions.
and rural populations (>70%); community pro- b. Root caries may be prevented by:
grams address issue with education and fluoride (1) Effective dental biofilm control.
varnish. (2) Combination fluoride therapy, along with
2. Recommendations to guardians: calcium and xylitol products.
a. Do NOT put infant to bed with baby bottle (3) Noncariogenic diet; reduced intake of car-
filled with formula, milk, juice, soft drinks for iogenic foods to decrease accumulation of
long periods of time; may continue with no- substrate (see earlier discussion).
spill training cup (sippy cup) in toddlers. (4) Frequent examinations of exposed root
b. Plain water is BEST choice for prolonged bottle ­surfaces.
use; educate guardian that infants’ and toddlers’ c. Thus BOTH the geriatric patient and periodon-
teeth SHOULD be brushed starting from first tal maintenance (PM) patient are at high risk for
emergence after every feeding or meal. root caries based on number of significant risk
C. Root caries: soft, leathery progressive, small and factors listed above (discussed later).
shallow lesions of root surfaces caused by dental bio- D. Recurrent caries (secondary caries): appears at loca-
film (MAINLY Actinomyces viscosus, same microor- tion with previous history of caries; frequently found
ganism as involved with occlusal; possibly Veillonella on margins of dental restorations; marginal discolor-
parvulus) and/or dietary acids. ation by itself is NOT a valid sign for dental caries.
1. Professional considerations: E. Arrested caries: brown, smooth, shiny lesion that
a. Root MORE vulnerable to process than enamel was previously demineralized but was remineralized
because cementum begins to demineralize at before causing cavitation; associated with sclerotic
6.7 pH, which is higher than enamel’s critical dentin.
pH; can lead to a rapid loss of teeth despite val- F. Treatment for all carious lesions:
iant recare efforts. 1. For noncavitated initial lesions:
b. Formation: precipitated by conditions that cause a. Minimally invasive dentistry (MID) using pre-
root surface exposure: ventive mode (or nonsurgical management) by
(1) Periodontal disease. use of remineralizing therapy (discussed later)
(2) Malocclusion. instead of repair mode (traditional restorative
(3) Mechanical trauma (toothbrush abrasion). treatment); this therapy can arrest or repair the
(4) Orthodontic tooth movement. initial lesion so that it does not progress to cavi-
(5) Periodontal surgery. tation and need restorative treatment.
o17960 (6) Abfraction caused by bruxism and/or b. Use of enamel (pit and fissure) sealants (dis-
clenching. cussed later).
2. Recommendations to patients or caregivers: 2. For early cavitated lesions: alternative (atrau-
a. Significant risk factors: attachment loss, age, matic) restorative treatment (ART) where NOT all
number of teeth, presence of coronal caries, infected dentin is removed, as long as a secure seal
level of oral hygiene, water fluoridation, years can be achieved between tooth and material, then
of education, xerostomia. MAINLY using glass ionomer restoration with its
(1) Common outcome when xerostomia is pres- fluoride release.
ent; IMPORTANT risk factor because pre- 3. For late cavitated lesions: traditional restora-
ventive action of saliva is diminished by such tions.
362   Saunders Review of Dental Hygiene

CLINICAL STUDY  

Age 19 YRS Scenario

Sex ☐  Male   ☒  Female At the patient’s previous dental examination and oral
prophylaxis, 15 months ago, she had only one occlusal
Height 5’4” restoration placed. At this appointment the clinical and
Weight 150 LBS radiographic examinations reveal: (1) moderately inflamed
gingival tissues, light calculus on the mandibular anteriors
BP 112/67 and posterior molars interproximally; (2) no dental biofilm
deposits or stain; (3) new carious lesions on 13 of her
Chief Complaint “My jaw joint hurts when I open posterior teeth (#2-M, #3-D, #4-M, #5-DO, #12-O, #13-D,
my mouth.” #14-MOD, #15-M, #18-M, #19-D, #20-MOD, #30-M, #31-
MO). Patient shows the dental hygienist a good brushing
Medical History None
technique, brushes once a day, and flosses infrequently.
Current Medications None However, when asked about any dietary changes, she
states she ate well at school at first until the fall when she
Social History College student, comes home at began drinking coffee with sugar and three or four sweet-
breaks ened, carbonated soft drinks sipped all day long to keep
her awake for her studies.

1. What is the most probable reason for the patient’s dra- tooth and be 1 mm coronal to CEJ, and papilla should
matic increase in caries? appear pointed or “knife-edge” in interproximal ar-
2. What techniques should be used to examine the TMJ eas. Gingival tissue is likely to be red, edematous,
area? If positive findings, what recommendation and spongy, with some loss of stippling, and marginal
should be made? contour will be swollen and possibly bulbous, indi-
3. How does the patient’s gingival health vary from nor- cating diagnosis of gingivitis (gingival inflammation).
mal, healthy gingiva with respect to color, consistency, The patient is dental biofilm free at this appointment,
texture, and marginal contour? Why is there tissue in- most likely because she brushed her teeth more thor-
flammation in the absence of any dental biofilm? oughly just before this dental checkup than she typi-
4. What oral hygiene self-care regimen(s) is (are) recom- cally does; previous dental biofilm buildup caused the
mended? What can be planned in the dental office to ­inflammation.
reduce her caries risk? Explain any nutritional recom- 4. At this dental appointment, primary need for treat-
mendations that should be made. ment of dental caries should be addressed. Because
5. What type of instrumentation is needed to complete of increased number of carious lesions, caries process
the oral prophylaxis? should be discussed with patient. She should be encour-
aged to brush two or three times a day for 2 minutes
1. Increase in sugar consumption with coffee and soft each time, using fluoride dentifrice (toothpaste).
drink is the most likely reason for increase in inter- She also needs to add flossing to her daily self-care.
proximal and occlusal caries. Sugar frequency affects Regimen of prescription topical home fluoride appli-
acid production, thereby increasing risk for proximal cations before bedtime (brush-on or tray fluoride) is
and occlusal caries. recommended. Specific directions must include not
2. Use bilateral palpation to assess the TMJ. Bilateral pal- rinsing after fluoride application to allow fluoride to
pation involves placing the index fingers of each hand continue to work in higher concentrations. Because of
just anterior to the outer auditory meatus of the ear and excessive interproximal caries, it is appropriate to re-
feeling the joint area as the patient opens and closes. quest that patient swish with home fluoride mouthrinse
Observation is also required to assess the TMJ for de- daily and use mineralizing calcium products. For her
viations in the functional movements of opening, clos- restorative treatment in the dental office, may want to
ing, and lateral movements. Use auscultation (listening include enamel sealants in any high-risk pit and fissure
for sound) to detect clicking, crepitus, popping, and areas. After her restorative treatment is finished, she
grating. Question patient regarding painful symptoms should schedule another appointment for examination
associated with jaw movements. For positive findings, in 3 months because of her high risk, possibly includ-
refer to supervising dentist for further evaluation. ing bitewing (BW) radiographs and evaluation of car-
3. Healthy gingiva should appear pale pink, firm, and ies management. Discuss the form and frequency of
stippled; marginal contour should hug neck of the sugar exposures from coffee and soft drinks, ­explaining
Clinical Treatment   363

that drinking soft drinks for 1 hour continually bathes 2. Root after scaling (and root planing) is frequently
the teeth with substrate bacteria used to produce an sensitive because removal of cementum exposes
acid environment. The carbonation of the soft drink the dentinal tubules.
provides additional acid attack on the teeth; substrate 3. Post whitening, teeth may be sensitive because
bacteria can actually exist in such an acidic environ- of evaporation of dentinal fluid in the tubules and
ment. Recommend reducing sugar and acid exposures subsequent rehydration (can be reduced by pre-
by limiting amount and duration (length of time) of treatment with desensitizing agents and fluoride
soft drink (whether sugared or sugar free, since latter treatments).
is more acidic, and use of straw) and coffee with sugar 4. Protective smear layer can also be removed by di- o17970
consumption and by rinsing mouth with water after etary and gastric acids, as well as detergents found
eating or drinking to reduce acid environment. Citrus- in toothpastes and rinses.
­containing soft drinks cause the greatest demineraliza- C. Management: performed professionally or at home:
tion of the enamel. 1. Professional application of the following desensi-
5. Manual instrumentation with anterior and posterior tizing agents has proved effective:
sickles, as well as universal or Gracey curets, is suffi- a. Protein-precipitating agents (e.g., silver nitrate,
cient to debride for the oral prophylaxis. Ultrasonic in- zinc chloride, strontium chloride, formalde-
strumentation using thin periodontal inserts could also hyde).
be used. Polishing, even by selective methods, would b. Tubule-occluding metallic salt agents (e.g.,
not be appropriate considering her caries risk and re- calcium hydroxide or phosphate, potassium
lated possibility of increased levels of decalcification nitrate, fluorides, sodium citrate, iontophoresis
of the enamel surface. Thus all instrumentation should with 2% sodium fluoride, potassium oxalate);
avoid the occlusal or other pit and fissured areas, as fluoride varnish is particularly effective because
well as the cervical surfaces of the teeth. Professional its globules fill in the tubules and stay for a long
fluoride application, such as with fluoride varnish, period of time.
should follow instrumentation to replace any loss of c. Neutral noncolored fluoride treatment pre and
surface mineralization. post whitening procedure: gel or foam with
trays.
Dentinal Hypersensitivity
p0170 Dentinal (dentin) hypersensitivity is pain caused by
differing stimuli such as thermal (cold/heat), evapora- STIMULATION
tive (blowing air), osmotic (sweet or sour/acid), tactile
(toothbrushes, flossing, toothpick, dental instruments) on
dentin of root that is exposed by gingival recession and/or
Exposed dentin
on masticatory surfaces as result of attrition. Abrasion,
Dentinal tubule
erosion, and abfraction can further complicate the situa-
Change in
tion. Common occurrence also with periodontal patients dentinal fluid
before and/or after nonsurgical and surgical therapy, as Odontoblastic
well as post whitening. Associated pain may be sharp, process
short, or transient in nature with rapid onset. Hypersen-
sitivity is a chronic condition with acute episodes. Spon-
taneous remission without further intervention has been Possible nerve
shown to occur in ∼20% to 45% in 4 to 8 weeks; also location
becomes LESS noticeable 2 to 3 weeks after nonsurgical Odontoblast cell
body in pulp
periodontal therapy. tissue
A. Etiology is BEST explained by hydrodynamic theory
(Figure 11-7):
1. Stimulation of open dentinal tubules at root surface
causes movement of the fluid within the tubules.
2. This movement transmits signals to the nerves in
the adjacent pulp chamber, resulting in pain.
B. Pathogenesis: Pain message sent to brain
1. Root after periodontal surgery is frequently sensi-
Figure 11-7  Hydrodynamic theory to explain dentinal hy-
tive because of exposure of the root surface and persensitivity. (From Bath-Balogh M, Fehrenbach MJ: ­ Illustrated
eventual dissolution of smear layer approximately dental embryology and anatomy, ed 2, Philadelphia, 2006,
7 days after exposure. ­Saunders/Elsevier.)
364   Saunders Review of Dental Hygiene

2. Physical sealing of the tubules with composites, give similar readings in same individual but may
resins, glass ionomers, varnishes, sealants; may vary widely among individuals.
be used if chemical applications do NOT provide 6. Record results of all tests and other pulpal assess-
­relief. ments in record.
o5470 3. Desensitizing dentifrices (toothpastes) are avail- C. Sources of data for pulpal diagnosis: patient’s subjec-
able for home use and contain one of the follow- tive report of symptoms MUST always be confirmed
ing active ingredients (see later discussion under with objective, clinical information and testing.
dentifrices): strontium chloride, potassium nitrate, 1. Dental history related to tooth or teeth in question,
sodium citrate, amorphous calcium phosphate. as well as trauma, pain, mobility.
Note that fluoride-containing toothpastes do NOT 2. Intraoral and extraoral examination: tooth fracture,
have high enough fluoride concentration to have large carious lesion, large restoration, lymphade-
impact. nopathy, swelling, pustule (or draining fistula),
tenderness.
Pulpal Vitality Testing 3. Radiographic examination by periapical film:
Pulp responds to any form of stimulus, whether electri- pathological radiolucency, periodontal ligament
cal, touch, or temperature, ONLY with pain. Normal pulp (PDL) space widening, fracture, caries, or deep
will typically respond to these pain-producing stimuli restorations (see Chapter 4, Radiology).
in manner consistent with similar teeth in the dentition. 4. Palpation: inflammation in periapical tissues ex-
Pulp tissue can undergo inflammation or pulpitis as result tending into periodontium.
of trauma or infection from either dental or periodontal a. Bidigital palpation of bone in the area of tooth
pathological changes, either reversible or irreversible. apex.
Continuing pulpitis can lead to death of the pulp or necro- b. Use palpation of surrounding (control) teeth for
sis. Tooth varies in its response to pain-producing stimuli comparison.
when undergoing pulpitis and necrosis, as noted during 5. Percussion: test for inflammatory changes in the
pulp vitality testing. Practice acts concerning pulp vital- PDL (horizontal and vertical):
ity testing vary by state. a. Use end of mirror handle or similar instru-
• See Chapters 6, General and Oral Pathology: pulpal pa- ment to gently tap the teeth in both a verti-
thology; 15, Dental Biomaterials: endodontic therapy. cal and horizontal direction; use ONLY finger
A. Diagnostic categories for pulpal diseases: pressure rather than tapping with an instru-
1. Reversible pulpitis: exhibits quick, sharp hyper- ment if patient reports that the teeth are very
sensitive response to thermal stimulus that sub- sensitive.
sides with removal of stimulus. b. Ask the patient if any of the percussed teeth
2. Irreversible pulpitis: usually indicated by lingering feel different or tender in comparison with sur-
response to cold temperatures; may give spontane- rounding (control) teeth.
ous pain without stimulation or may be asymptom- c. Interpretation: positive test indicates inflam-
atic; pulpal damage is NOT repairable. mation in PDL; confirmation of diagnosis by
3. Necrotic pulp: indicated by asymptomatic, non- other clinical tests is needed because inflamma-
vital tooth that does NOT respond to any stimuli; tory changes in PDL are NOT always of pulpal
NOT repairable; requires endodontic therapy (root ­origin.
canal) or extraction. d. Can also have patient bite tongue depressor (lo-
B. General testing technique: decisions are based on a cates fractures, too).
combination of all available data. 6. Thermal tests for pulp vitality: see earlier diagnos-
1. Recognize that individual pulp test can give either tic discussion.
false-positive or false-negative response. a. May involve use of either cold or hot thermal
2. Goal is to make distinction between vital and non- stimulus to test for pulpal response.
vital pulps or determine existence of abnormal (1) Cold test is MORE frequently used.
pulp. (2) Heat test is helpful when the ONLY symp-
3. Recognize that pulpal pathological changes can be tom is heat sensitivity and the offending
of dental (caries) or periodontal origin; combina- tooth is NOT clearly identified.
tion of factors can occur. b. Cold test technique:
4. Test both suspected teeth and control teeth (usually (1) Use ice, carbon dioxide (dry ice), or a cot-
adjacent teeth and contralateral tooth) for compari- ton-tipped applicator coated with refriger-
son data. ant such as ethyl chloride.
5. Compare readings of all teeth tested to determine (2) Place cold agent on the tooth and observe
results, since same types of teeth (e.g., molars) any response.
Clinical Treatment   365

(3) Remove stimulus immediately on patient re- d. Differing response levels do NOT indicate dif-
sponse; leave on tooth a maximum of 10 sec- ferent stages of pulp degeneration.
onds (20 seconds for full crown coverage). (1) May give false-positive reading because
c. Hot test technique: of stimulation of nerve fibers in periodon-
(1) Lubricate selected teeth to prevent material tium.
from sticking. (2) Posteriors may give misleading readings
(2) Heat a small amount of gutta-percha or because of combination of vital and nonvi-
stopping material and place on the tooth. tal root canal pulps.
(3) Remove stimulus immediately on patient (3) Does NOT work on teeth with metal crowns
response; leave on the tooth a maximum of (short-circuiting) or ceramic crowns (insu-
10 seconds (20 seconds for teeth with full lating).
crown coverage).
7. Interpretation of test results: PERIODONTAL EVALUATION
a. Abnormal pulp status is indicated when there is Periodontal evaluation is the examination of the soft and
different response between suspected and sur- hard tissues surrounding the tooth structure. MAIN role
rounding (control) teeth or similar teeth. of the dental professional is to educate patient on charac-
b. Diseased pulp is suspected when: teristics of a healthy periodontium and its maintenance so
(1) NO response. that patient will AVOID periodontal (gum) disease and its
(2) Response is excessively rapid (compara- complication of tooth loss.
tive). • See Chapters 2, Embryology and Histology: anatomy
(3) Response is lingering or prolonged (com- of periodontium; 4, Head, Neck, and Dental Anatomy:
parative). occlusal evaluation; 5, Radiology: radiographic assess-
(4) Response is of increasing intensity (com- ment; 13, Periodontology: dental biofilm and calculus,
parative). periodontal disease, occlusal trauma; 17, Community
(5) Preexisting pain is alleviated when cold is Oral Health: dental biofilm and periodontal indices.
applied. A. Clinical description of healthy periodontium:
8. Electric pulp test (EPT; vitalometer) (forget what 1. Color: uniform pale to coral pink from mucogin-
you have seen in horror movies!): gival attachment to the free gingival margin; may
a. Absolute contraindication for patient with car- be pigmented with melanin (related to complexion
diac pacemaker because of possibility of elec- and race).
trical interference. 2. Contour and size: include free gingiva margins that
b. Technique: fit snugly and follow curved lines around necks of
(1) Place small amount of conducting material the teeth, interdental papillae are knife edged and
(usually dentifrice or toothpaste) on tip. pointed and fill the embrasure space.
(2) Starting with control dial at lowest number 3. Consistency: firm and resilient.
(lowest electric impulse), place electrode 4. Surface texture of attached gingiva: usually stip-
on the tooth and have patient touch metal pled owing to rete pegs, which connect oral epi-
handle with fingers to complete ­circuit. thelium of the oral mucosa to underlying lamina
(3) Depending on equipment, current will propria.
­automatically increase; instruct patient to 5. Free (marginal) gingiva: smooth tissue but NOT
­release handle when he or she feels sensa- bound to underlining tissue; position is 1 to 2 mm
tion, or manually increase current dial un- coronal to the CEJ.
til patient gives prearranged signal; lift tip 6. Bleeding (either spontaneous or on probing [BoP])
from the tooth. and exudate: NOT present.
(4) May want to test surrounding teeth (con- B. Clinical description of diseased periodontium:
trol) or similar tooth for comparison (as 1. Signs of inflammation:
discussed previously). a. Acute inflammation:
c. Interpretation of test results is generally all or (1) Appears bright red because of dilation of
none: scales can range from 1 to 10 or 1 to 50. blood vessels and proliferation of blood
(1) Response at low electric stimulus level, cells.
tooth is vital; the closer to the 1 reading, (2) Begins in area of the interdental papilla;
the healthier the tooth. FIRST site is the col (fragile portion of
(2) If NO response, tooth is nonvital; the far- periodontium).
ther from the 1 reading, the poorer the (3) Tissue is swollen and edematous in early
prognosis. stages.
366   Saunders Review of Dental Hygiene

(4) Consistency is spongy and soft; tissue in- dense, radiopaque, and continuous; NO furcation
dents easily when pressed and hangs away involvement.
from tooth; loss of stippling causes a smooth 2. Diseased periodontium: horizontal and/or vertical
and shiny look because of increased infil- bone loss, widened PDL space, furcation involve-
tration of fluid and inflammatory elements ment (radiolucent); thinning or loss of definition
into tissue cells. in lamina dura (alveolar bone proper); may have
(5) Bleeds easily because of junctional epithe- pulpal complications as well.
lium (JE) ulceration from damage to lamina E. Probe depths of epithelial sulcus from the free gingi-
propria’s blood vessels; BoP is the MOST val margin to the base of the pocket:
objective method of assessing active in- 1. Probing depths (PD): recorded on a periodontal chart
flammation and can identify patients at risk and include six readings per tooth (three on facial;
for disease progression; however, ONLY three on lingual); note BoP for each reading; note
risk and NOT sign of progression; lack of other associated periodontal concerns such as reces-
BoP is a sign of healthy gingiva. sion, furcation, reduced attached gingiva, mobility.
(6) Suppuration (pus) is a sign of acute inflam- 2. Gingival pocket (or pseudopocket): enlargement
mation and infection; exudate may appear of the gingiva that causes the margin of the gingiva
clear, white, or yellow depending on com- to proliferate in a coronal direction; there is NO
position of white blood cells (WBCs) and apical migration; pocket is suprabony and revers-
inflammatory debris. ible with good homecare and professional inter-
b. Chronic inflammation: vention.
(1) Appears dark red to bluish red magenta; 3. Periodontal pocket: JE to migrate apically, al-
however, when extremely chronic, will ap- lowing destruction of epithelial attachment (EA);
pear fibrotic and near normal in color. involves destruction of deeper periodontal struc-
(2) Interdental papilla appears flattened, blunted, tures, cementum, PDL, bone.
or cratered; gingival margins may be rolled; a. Suprabony pocket: JE is apical to the CEJ but
clefts and festoons may be present. coronal to crest of the alveolar bone.
(a) Widening interdental embrasures: with b. Infrabony pocket: JE is apical to crest of the
recession comes loss of interdental alveolar bone (further classified as to three
papillae; classified as type I through types).
type III. F. Recession: level of the marginal gingiva is apical to
(b) Factor to consider when choosing inter- the CEJ, measured in millimeters using periodontal
dental oral care method. probe.
(3) Consistency is tough and fibrotic, with leath- G. Clinical attachment level (CAL): level from probing
ery or hard, nodular-like surface texture. from a fixed point such as the distance from CEJ to
(4) BoP may NOT be present and is NOT as base of the pocket (EA).
obvious as with acute inflammation (espe- 1. When CEJ is covered: to obtain CAL, first mea-
cially with smokers). sured from gingival margin to CEJ, then subtracted
C. Dental biofilm (etiological or causative agent): from overall probing depth.
slight, moderate, or severe levels; supragingival or 2. When CEJ is level with gingival margin: CAL is
­subgingival. measured by probing depth.
1. Combination of dental biofilm and host response 3. With recession: CAL is measured from CEJ to EA
to periodontal pathogens influences initiation and (some add recession and probing depth).
progression of periodontal disease. H. Mucogingival defects: loss of the width of attached
2. Mineralization results in development of calculus gingiva (<1 mm or nonexistent); measurement of at-
deposits on tooth enamel and cementum, restora- tached gingiva is found by subtracting probing depth
tions, prosthetic appliances; acts as attachment from total width of gingiva (gingival margin to muco-
mechanism for new colonies of new dental bio- gingival junction).
film. 1. Type I defects: pockets extend apically to or be-
3. Can be made visible by disclosing solution to iden- yond mucogingival junction, but a firm keratinized
tify areas continually missed or can be measured pocket wall exists.
using an index; two-tone solution stains new red 2. Type II defects: alveolar mucosa acts as marginal
and old blue. gingiva, with NO zone of attached gingiva.
D. Radiographic examination (has limitations): I. Furcation involvement: loss of interradicular bone
1. Healthy periodontium: interproximal alveolar on multirooted teeth; suspected when 4-mm probe
crestal bone is 1 mm apical to CEJ; lamina dura is depth is recorded on a multirooted tooth even with
Clinical Treatment   367

normal gingival contour; classified by extent of bone and antibiotic sensitivity, DNA probes, enzyme im-
destruction: munoassays (enzymes tested may be derived from
1. Class I: soft tissue suprabony lesion with increased host or periodontal pathogens), immunological tests.
pocket depth; can detect concavity but CANNOT L. Documentation protocol:
enter; radiographs show NO radiolucency. 1. Record probing depths, bleeding points, recession,
2. Class II: probe can enter furcation, since there is furcations, mobility, reduced attachment level,
a horizontal component and possibly a vertical or bone level.
infrabony defect, but CANNOT extend through; 2. Determine periodontal status (healthy or American
radiographs may show slight radiolucency or Academy of Periodontology [AAP] periodontal
­furcation arrow for maxillary proximal furcas; type) and periodontal case type.
may involve one or more furcations, but there is 3. Explain status to patient and possible recom-
NO communication between them. mended treatment options to the patient.
3. Class III: probe can pass (depending on soft tissue 4. Record patient’s response and willingness to pro-
location) between facial and lingual, since bone is ceed with treatment.
NOT attached to the roof or dome interdentally of 5. Failure to diagnose and properly treat periodontal
the furca; radiographs show obvious radiolucency disease is the MAIN cause of malpractice lawsuits
or furcation arrow for maxillary proximal furcas; (see Chapter 18, Ethics and Jurisprudence).
Subdivided into two categories: M. Referral protocol: dental hygienist CANNOT make
a. Furca covered by tissue, harder to pass probe referral to specialist, ONLY supervising dentist can.
through. 1. Establish referral guidelines for the dental office.
b. Furca NOT covered by tissue. 2. Explain the need for special care to the patient.
4. Class IV: soft tissue has receded and furcation 3. Assist the patient in scheduling an appointment
is clinically visible because interdental bone has with a specialist.
been destroyed; probe can easily pass through; ra- 4. Communicate with the specialist and patient to
diographs show larger radiolucency. ­ensure continuity of care.
J. Mobility: movement of tooth because of loss of sup- N. If patient does NOT follow through with treatment:
port by periodontium; classified (or graded) by extent 1. Discuss and document reasons for noncompliance.
of horizontal and vertical movement. 2. Answer any additional questions patient may
1. Class I: moves 1 mm or less in any direction. have.
2. Class II: moves more than 1 mm in any direction 3. Continue to see patient for regular maintenance
but CANNOT be depressed. appointments.
3. Class III: moves MORE than 1 mm and can be de- 4. Monitor periodontal status and continue to encour-
pressed in the socket. age patient to seek additional care.
K. Additional diagnostic tests that can be used to assess 5. Confirm patient’s understanding of consequences
periodontal health and disease are culture analysis of noncompliance.

CLINICAL STUDY  

Age 46 YRS SCENARIO

Sex ☒  Male   ☐  Female On completion of the patient’s


dental exam, it is apparent
Height 5’10” that he clenches his teeth; has
Weight 185 LBS moderate gingivitis with fibrotic,
enlarged, and bulbous interden-
BP 110/78 tal papilla; and has dental biofilm
covering more than 50% of his
Chief Complaint “I can’t get floss between my teeth.” teeth.
Medical History Seizure disorder
Allergy to penicillin (Pen-Vee K) and diazepam (Valium)

Current Medications phenobarbital (Luminal) 30 mg qd


phenytoin (Dilantin) 100 mg qd

Social History Food bank director


368   Saunders Review of Dental Hygiene

1. What are common oral side effects of the patient’s I­ nclude devices, dentifrices, fluoride, enamel (pit and fis-
drugs and disease? Into what category do these drugs sure) sealants, mouth protectors.
fall?
2. Given his history of clenching, what clinical signs Oral Hygiene Devices and Self-Care Practice
could be expected in examination of his mouth? Oral hygiene devices include toothbrushes and other de-
3. What recommendations can be made for improving vices for self-care practice.
the patient’s oral hygiene self-care? • See Chapter 16, Special Needs Patient Care: special
4. What should the treatment plan include? oral care devices.
A. Toothbrushes:
1. Phenytoin (Dilantin) can cause gingival hyperplasia 1. Manual brush selection and brushing technique:
(overgrowth of tissue). Because of injury from sei- a. BEST to choose head size that corresponds with
zure trauma, scars on the lips and tongue and possibly size of mouth and tooth accessibility.
fractured teeth may be found. Both drugs, phenytoin b. Handles SHOULD be easy to grip with minimal
(Dilantin) and phenobarbital (Luminal), are anticon- slippage and rotation during use; larger handles
vulsants. increase grasp ability with arthritis, other physi-
2. Clenching frequently results in scalloping of lateral cal disabilities.
borders of the tongue. Occlusal surface of molars will c. Selection of bristle:
demonstrate wear (appear shiny), and dentin may be- (1) Ends should be rounded nylon filaments
come exposed as the enamel is worn away. Wear is (LEAST damaging to gingival tissues).
also common on the incisal edges of anteriors. Patient (2) Filament diameter determines hardness or
may also have mobility, which is the movement of softness (thinner filaments are softer and
tooth resulting from loss of support by periodontium. more resilient).
3. The patient could improve his oral self-care by using (3) Nylon filaments are preferred over natural
shred-resistant floss made of polytetrafluoroethylene bristles because more resistant to bacterial
(PTFE) or thin waxed floss. Powered toothbrush may accumulation, more easily rinsed and dried,
help improve oral hygiene self-care because of built- durable, maintain form; ends are rounded
in timers, ease of technique, and increased access to and closed, which repels water and debris.
hard to reach tooth surfaces. (4) Multitufted bristles support adjacent bris-
4. Treatment plan should include thorough assessment tles, allowing more cleaning action while
of medical history, specifically including most recent being less traumatic to gingival tissues.
seizure, drug regimen, and compliance. Dental exam d. Technique:
should include extraoral and intraoral examination, (1) Use light-to-medium pressure to ensure re-
with special evaluation of existing periodontal condi- moval of dental biofilm.
tion. Complete dental and periodontal chart would be (2) Develop sequence to ensure brushing of
obtained and appropriate radiographs taken. Oral hy- ALL teeth and tooth surfaces.
giene self-care instructions should involve patient by (3) Brush two to three times per day for 2 to
having him give return demonstration of brushing and 3 minutes.
flossing to observe technique. The patient will need e. Methods:
thorough oral prophylaxis including debridement ther- (1) Sulcular/Bass method: for removing at gin-
apy, as well as a reappointment for evaluation with the gival margin.
supervising dentist in 4 to 6 weeks to consider need (a) Filaments are directed apically and an-
for gingivectomy, assess oral hygiene self-care, and gled at 45°, brush is vibrated back and
determine continued care interval. forth with a very short small motion,
keeping brush tip engaged in sulci.
PREVENTION OF DENTAL DISEASE AND INJURY (b) MOST widely accepted technique;
It is IMPORTANT that dental hygienists use evidence- BEST method for ALL patients.
based decisions to support preventive treatment of pa- (2) Roll method: for removing at gingival mar-
tients. Evidence-based decisions are based on verified gin and clinical crown.
research evidence that certain signs or symptoms are pre- (a) Filaments are directed apically, rolled
dictive of certain outcomes. For instance, the finding of toward occlusal surface, forcing fila-
a greater number of S. mutans organisms poses a greater ments interproximally.
risk for future caries development than does a low count. (b) BEST for children, although easy to miss
The following devices and materials are designed to pre- areas if brush is NOT correctly placed.
serve the integrity of the natural dentition and protect (3) Modified Stillman’s method: for removing
the teeth and supporting tissues from disease or ­ injury. at gingival margin.
Clinical Treatment   369

(a) Uses vibratory motion to stimulate tis- (1) Gently inserting the floss between the teeth,
sue addition of the roll stroke, removes wrapping the floss snugly around each
from the buccal and lingual surfaces; proximal tooth surfaces, and using an up-
bristles are directed apically at gingival and-down motion from sulcus to contact to
margin; roll stroke and vibratory mo- remove dental biofilm.
tion are used. (2) Floss handle: BEST for those with larger
(b) Adequate technique for all patients. hands or one functional hand; floss-
(4) Charter’s method: MOST useful for around ­threaders: BEST to get under bridges and
orthodontic bands and brackets, fixed pros- orthodontic wires.
theses, recent postsurgical wounds. 2. Interdental brushes: BEST to cleanse wide embra-
(a) Uses vibratory motion with bristles di- sures (type II to III) or furcations.
rected occlusally. a. Conical and tapered brushes fixed to a wire and
(b) Sides of bristles contact gingival mar- then inserted on plastic handle.
gin and tooth surface. b. Single tufted brush with tapered or flat groups
(5) Circular (Fones’) method: for removal on of filaments inserted on plastic handle (also
clinical crown. called end-tuft brush).
(a) Uses rotational motion on buccal and (1) For MOST effective cleaning, use brush
lingual surfaces. slightly larger than the embrasure.
(b) Good FIRST method for young ­children, (2) Can further enhance effect of antimicrobial
NOT recommended for adults. agent by delivering chemical into embra-
(6) Horizontal or scrub method: for removal on sure area with interdental brush.
buccal, lingual, and occlusal surfaces. 3. Interdental tip: removes excess debris from em-
(a) Uses horizontal scrubbing motion. brasure area and slightly apical to the gingival
(b) Detrimental to gingival tissues, causing ­margin.
abrasion and recession. a. Soft flexible rubber tip is ideal to adapt into
2. Powered toothbrushes: effective as manual with embrasure space, LESS likely to cause trauma
CORRECT use. than plastic tip; trace along gingival margin and
a. Brush motion can be rotational, counterrota- move in and out of the embrasure space.
tional-oscillating, counterrotational-sonic; tech­ b. Toothpick in a holder (e.g., Perio-aid) is BEST
nique is product dependent and SIMILAR to for removing dental biofilm in periodontal
manual techniques. pockets and adjacent to orthodontic appliances;
b. Bristle placement is angled at 45° to gingival round or square-round toothpick is placed in a
margin; smaller brush heads are site specific plastic handle.
and require MORE time to clean all teeth. (1) Trace along gingival margin and move back
c. Automatic timer and audible cues help to and forth in the embrasure space; for peri-
brush for specified amount of time (usually odontal patients, tip is placed on the tooth
2 minutes); useful in encouraging thorough and traced into the col and furcation area.
cleaning. (2) Care is taken NOT to place excessive force
d. Recommended for orthodontic appliances, mal- on gingival tissue; for orthodontic patients,
posed teeth, periodontal disease, arthritis and tip is placed on the tooth and traced around
other grasping disabilities, caregivers. orthodontic brackets or bands.
B. Interdental care: IMPORTANT factor is type of inter- c. Wooden interdental cleaner (e.g., Stimudent):
dental embrasure. in areas where interdental papillae are missing
1. Floss is available in a variety of textures, thick- (type III).
ness, materials and SHOULD be used daily for ef- (1) Triangular; flat part of triangle is placed to-
fective dental biofilm control. ward the tissue.
a. Waxed/polytetrafluorethylene  ( PTE)  c oated (2) Burnishing action is used against one side
floss: BEST for persons with tight contacts, of the embrasure, then the other side.
amalgam overhangs, or rough restorations. C. Tongue cleaning devices: IMPORTANT to reduce
b. Unwaxed floss: can be used by persons with dental biofilm accumulation and malodors (halitosis,
FEW restorations and with normal contact bad breath) caused by production of volatile sulfur
tightness. compounds (VSCs) by bacteria on dorsal surface.
c. Tufted floss or yarn: BEST for large embrasure 1. Plastic scrapers: place on the posterior portion of
spaces or under orthodontic wires. the tongue and pull forward to “scrape” the dental
d. Technique: biofilm coating off.
370   Saunders Review of Dental Hygiene

2. Toothbrushes: also effective tools for removing this 7. Titanium dioxide to make paste opaque and
coating; placed on the posterior portion of the tongue white.
and “swept” forward (four or five times); brush- B. Fluoride-containing (for cavity protection, increased
ing without dentifrice (toothpaste) may reduce the remineralization): source of fluoride for the tooth sur-
gag flex. face; control demineralization and promote reminer-
D. Oral irrigators: reduce gingivitis, reduce dental bio- alization.
film toxicity by altering microflora, deliver antimicro- 1. Active ingredients for OTC: sodium fluoride (NaF,
bial agents (controversial use). 0.24%/1100 ppm) or sodium monofluorophosphate
1. Supragingivally, standard tip provides steady, low- (Na2PO3F, 0.76%/1000 ppm) or stannous fluoride
pressure, pulsating stream of water or other fluid (SnF2, 0.45%) (see later discussion).
agent to dislodge loosely adherent dental biofilm 2. SHOULD be used at least once daily; those at high
at or slightly apical to the gingival margin; tip is risk for caries may use several times per day.
directed at a 90° angle to the tooth at the gingival 3. Should NOT be used on small children; digestion o7630
margin. of fluoride can affect developing permanent tooth
2. Subgingivally, sulcus tip (e.g., Pic Pocket) pro- buds, causing white spots (hypocalcification);
vides pulsating, low-pressure stream of water or once the guardian is sure child is NOT swallowing
chemotherapeutic agent to the sulcus or periodon- it, it can safely be used. See Table 11-1 for amount
tal pocket; can dislodge unattached or loosely ad- ­suggested per age group of children. Can also
herent dental biofilm; tip is directed at a 45° angle cause chronic toxicity; see later discussion under
with the tip directed into sulcus or pocket; gives “Fluorides.”
deeper subgingival penetration of water or antimi- 4. Prescription strength products have higher levels o7640
crobial agent. (NaF, 1.1%/5000 ppm) (Prevident 5000, Fluo-
ridex); used on developed dentitions for patients
Dentifrices with high caries risk.
Dentifrices (toothpastes) are substances used in conjunc- C. Anticalculus (“tartar” control or prevention): binds to
tion with a toothbrush or other applicator to assist in the crystals and prevents them from adhering to the tooth
removal of dental biofilm (dental plaque) and materia alba surface, thereby reducing rate of crystal growth of su-
from the teeth and gingiva. However, mechanical action pragingival calculus by as much as 40%.
from brushing is the MAJOR method for removal of soft 1. Active ingredients: soluble pyrophosphate (5% to
deposits and NOT the use of dentifrices. SHOULD select 1.3%) or zinc chloride.
products with ADA Seal of Acceptance for demonstrated 2. Examples: noted on label.
therapeutic value with safety and efficacy. With accep- D. Antigingivitis and antiplaque (see further discussion
tance, ALL have safe levels of abrasiveness. with oral microbials): reduces gingivitis associated
A. Basic ingredients: with dental biofilm by antimicrobial action against
1. Water and glycerine (to keep paste from drying bacteria.
out) and thickeners (to stay on toothbrush and 1. Active ingredients: stannous fluoride (Sn2, 0.454%)
squeeze out of the tube). or triclosan (0.30%).
2. Detergents to remove fatty films; water softeners 2. Examples: Gum Care, Total, ProHealth.
to make the detergents work better. 3. Essential oils may be added (thymol, menthol, eu-
3. Surfactants to help remineralization process by im- calyptol, methylsalicylate, 25% alcohol); example:
proving wetting on enamel and provide foam (love Listerine (see below).
those bubbles in America!); MOST common are E. Desensitizing: reduce dentinal hypersensitivity,
either sodium lauryl sulfate (SLS) or ammonium MUST be continually used (see earlier discussion).
lauryl sulfate.
4. Abrasives such as calcium pyrophosphate, di-
basic calcium phosphate dihydrate, tricalcium
phosphate, hydrated alumina, hydrated silica, and Table 11-1  Amount of fluoridated dentifrice t9000
(toothpaste) recommended per age group for children
sodium metaphosphate.
5. Binder such as carboxymethylcellulose (gum to
Age Amount
keep the solid and liquid components of paste from
separating); coloring agents such as food dyes; fla- Under age 1 None
voring agents such as specific oils and extracts.
Age 1 to 3 Rice grain-sized amount
6. Sweeteners, preferably nonnutritive such as so-
dium saccharin, so bacterial growth is NOT Age 4 to 5 Pea-sized amount
­encouraged. From National Maternal and Child Oral Health Resource Center.
Clinical Treatment   371

1. Active ingredients: potassium nitrate (KNO3, 0.5%) 4. Alteration of dental biofilm; inhibits bacterial ac-
or sodium fluoride (NaF, 0.243%), or strontium tivity by inhibiting enolase, enzyme needed by
chloride (SrCl2, 10%); casein phosphopeptide; or bacteria to metabolize carbohydrates.
amorphous calcium phosphate (CPP-ACP). C. Types: sodium (NaF), acidulated phosphate (APF),
2. Examples: Sensitivity Protection, Denquel, Senso- stannous (SnF2).
dyne SC. 1. NaF (2% aqueous; pH 9.2) neutral gel or foam:
F. Whitening: have limited ability; include papain (Ci- a. If professionally delivered: four appointments,
troxain), silica, carbamide peroxide. 2 to 7 days apart, to provide MOST topical
G. Other possible ingredients: benefits for caries control and desensitizing
1. Allantoin: relieves the irritation caused by deter- ­therapy.
gents, alkalies, acids. b. Active ingredient in MOST mouthrinses sold
o7780 2. Casein phosphopeptide–amorphous calcium phos- OTC.
phate (CPP-ACP)(MI paste, MI paste plus with c. Also used after whitening (since neutral but o7920
fluoride): can also help with remineralization by must be without coloring).
depositing on tooth surface, then filling in the d. Stable solution; easy to store.
enamel surface defects; especially helpful in high 2. APF (1.23% NaF in 0.1 M orthophosphoric acid;
caries risk as with xerostomia. pH 3.0) gel or foam:
o17980 3. Xylitol: inhibits Streptococcus mutans metabo- a. If professionally delivered: requires ONLY one
lism; may be used in combination with other anti- application at each recall visit for caries ­control.
caries products. b. Requires 4-minute application for MOST fluo-
4. Lauryl sarcosinate: replaces some or all of SLS, ride uptake into enamel.
since it can irritate oral mucosa. c. Absolute contraindication on composite, por-
5. Poloxamer 407 (nonionic foaming agent): to re- celain, sealant materials; causes pitting and
duce soft tissue irritation; does NOT resolve in- roughening.
flammation. d. Gels for home use contain 1.1% concentration.
6. Sodium bicarbonate (baking soda): for taste and 3. SnF2 (8% aqueous; pH 2.1 to 2.3) gel or foam:
mouth feel; combines with acids to release carbon a. If professionally delivered: requires ONLY one
dioxide gas, adding to foam produced by brush- application for caries control and desensitizing
ing; also mild abrasive and may reduce numbers of therapy.
acid-loving bacteria, although lasts ONLY as long b. Unpleasant taste and can cause extrinsic stain-
as mouth stays alkaline. ing (brown, orange, tan).
c. Unstable; MUST be mixed for each
Fluoride Use ­application.
Several chemical formulations and delivery agents pro- 4. The ADA has NOT accepted two-part rinses (APF/
vide a wide variety of ways to deliver adequate fluoride SnF2) as effective, and concerns have also been
to the teeth. Fluoride protection from caries is ONLY by raised regarding the potential of these rinses to be
way of topical use and NOT systemic; also helps reduce easily ingested, which could lead to toxicity.
dentinal hypersensitivity. Discussion in this chapter will 5. Fluoride varnishes contain NaF (e.g., Duraphat,
center on fluoride delivered in the dental office and home Duraflor), possibly xylitol:
use. a. Thin coating of resin (yellow or white) that
• See Chapter 17, Community Oral Health: community is applied to the tooth surface; delivered in
fluoride programs and supplementation (water and ­controlled-release system for caries control and
oral). desensitizes; dries immediately upon contact
A. Topical fluoride is found in dentifrices (toothpastes with saliva.
discussed earlier), gels and foams, mouthrinses, var- b. Does NOT require professional oral prophy-
nishes. laxis before application; requires LESS coop-
B. Topical application of fluoride to the erupted tooth: eration from patient.
works MAINLY on smooth surface remineraliza- c. Brushed on; can eat or drink but MUST refrain
tion (sealants work MAINLY on pit and fissure sur- from brushing, rigorous rinsing, abrasive foods
faces). for 3 to 4 hours.
1. Posteruption maturation of the enamel surface. d. Repeated at 3-month intervals for high risk;
2. Prevention of demineralization and remineraliza- 6-month intervals for lower risk.
tion of early caries. e. BEST for primary caries (see earlier ­discussion);
3. Decreasing enamel solubility and maximizing BETTER than other delivery systems in which
enamel resistance to decay. fluoride is washed away by saliva and eating;
372   Saunders Review of Dental Hygiene

useful for infants, disabled, those with swallow- a. Used with head and neck radiation therapy that
ing difficulties. would cause subsequent damage to the salivary
D. Professional application of topical gel or foam fluo- glands; also for other xerostomic systemic con-
ride: does NOT require professional oral prophylaxis ditions or patients at high risk for dental caries.
before application. b. Formed to cover entire clinical crowns for each
1. Children 4 to 18 years of age need application once arch; filled with neutral sodium fluoride; worn
or twice a year depending on caries risk. for duration of high-risk period; however, with
2. Adults may need fluoride treatment depending on permanent salivary dysfunction have continual
caries incidence, xerostomia, or dentinal hypersen- use.
sitivity. F. Toxicology: concentration of fluoride is regulated by
3. Tray application: do NOT overfill the tray. FDA.
a. Fit Styrofoam trays to the mouth, and place gel 1. Calculation of amount of fluoride ingested:
or foam in the trays. a. NaF: (4.5) × (no. mL swallowed) × (NaF con-
b. Dry teeth with air syringe; place trays in mouth centration [e.g., 0.05%, 1.1%]) = mgF.
for 4 minutes. b. APF: (10) × (no. mL swallowed) × (APF con-
c. Control salivary flow with saliva ejector; ex- centration [e.g., 1.23%]) = mgF.
pectorate fluoride into saliva ejector. c. SnF2: (2.4) × (no. mL swallowed) × (SnF2 con-
d. Floss teeth after tray removal; instruct NOT to centration [e.g., 0.4%, 8%, 10%]) = mgF.
eat or drink for 30 minutes. 2. Metabolism of ingested fluoride:
4. Paint-on application: a. 86% to 97% absorbed in the stomach.
a. Isolate teeth with cotton rolls; dry teeth with air b. Excess excreted through kidneys, sweat glands,
syringe. feces.
b. Paint foam or gel on with cotton-tipped applica- 3. Chronic fluoride toxicity:
tor and leave for 4 minutes. a. Can cause dental fluorosis, skeletal fluorosis,
c. Control salivary flow with saliva ejector; ex- renal damage.
pectorate saliva into saliva ejector. b. Severity can be influenced by increased intake
d. Floss teeth and instruct NOT to eat or drink for of naturally fluoridated water, increased intake
30 minutes. of food with fluoride, nutritional diseases, diets
E. Self-administered topical fluoride: compliance will low in calcium.
affect success of caries reduction. 4. Acute fluoride toxicity:
1. May be in the form of an OTC or prescription a. Can cause nausea, vomiting, heavy salivary
mouthrinse or gel, OTC or prescription dentifrice, flow, stomach pain, diarrhea.
lozenge (discussed later in section on geriatric b. In severe cases, cramping of extremities, breath-
patient). ing difficulty, heart failure, dilated pupils, hy-
2. Gels are brushed on once a day; mouthrinses are perkalemia, hypocalcemia.
used after brushing and before bedtime if possible; 5. Emergency treatment with excessive ingestion of
dentifrices used as often as person brushes. fluoride:
3. SHOULD refrain from rinsing, eating, or drinking a. If fluoride ingestion is <5 mg/kg body
after home administration for at least 30 minutes. weight:
4. Supervision of use is necessary until child has (1) Give milk orally.
demonstrated ability NOT to swallow it; should (2) Observe for several hours.
NOT be swallowed (can lead to chronic toxicity b. If fluoride ingestion is >5 mg/kg body weight:
and hypocalcification of permanent teeth). (1) Induce vomiting.
5. Mouthrinses for home use: daily with 1 tea- (2) Give milk orally.
spoon; swish between teeth for 60 seconds and (3) Admit to hospital and observe several
­expectorate. hours.
a. OTC MAINLY contain NaF (0.05%) (low- c. If fluoride ingestion is >15 mg/kg body
­potency, high-frequency rinse). weight:
b. Prescription-strength fluoride mouthrinses (1) Induce vomiting; activate EMS system.
(high-potency, low-frequency rinse): (2) Admit to hospital immediately; EMS
(1) 0.2% NaF (once a week). personnel will start calcium gluconate
(2) 0.05% NaF (once daily). intravenously and monitor for heart dys-
(3) 0.2% APF (once daily). rhythmias.
6. Fluoride trays (custom or stock): designed to hold 6. Certainly lethal dose (CLD): estimated dosage
fluoride gels in close proximity to teeth. range of a drug that may cause death:
Clinical Treatment   373

a. Child CLD: 0.5 to 1 g (varies with size and 2. Low-substantivity antimicrobial agents used to
weight). alter bacterial cell wall, reduce gingivitis (30%).
b. Adult CLD: NaF is 5 to 10 g taken at one time, 3. Strong taste, stains the teeth, may cause burning
or 32 to 64 mg/kg. sensation in some individuals.
D. Quaternary ammonium compound: contains cetylpy­
Oral Antimicrobials ridinium chloride; may contain domiphen bromide.
Oral antimicrobials (chemical antibiofilm agents) de- 1. May contain as much as 18% alcohol.
stroy oral microorganisms within dental biofilm (bacte- 2. Has slight but NOT substantive antiplaque
ricidal) or prevent growth (bacteriostatic). Substantivity, ­activity.
ability to adhere to tooth structure for long periods, en- E. Triclosan (Total dentifrice) (see earlier discussion):
abling slow release, is also IMPORTANT factor. Can be 1. OTC; second-generation to Listerine.
in form of mouthrinses, dentifrices, or other forms; can be 2. Low substantivity; reduces gingivitis.
delivered with oral irrigation. Includes chlorhexidine, es- F. Sanguinarine (Viadent dentifrice, mouthrinse):
sential oils, quaternary ammonium compounds, triclosan, 1. OTC; originates from bloodroot (Sanguinaria ca-
sanguinarine, SnF2, oxygenating mouthrinses. Must be nadensis) plant.
kept in mind that most are MORE effective on gingivitis 2. Has NO significant effects unless rinse and paste
than periodontitis because action is not able to occur deep are used together; alcohol (10% to 14%).
within periodontal pocket. 3. Minimal substantivity; interferes with bacterial
• See Chapter 13, Periodontology: subgingival antimi- glycolysis and binds to dental biofilm to reduce
crobials. microbial adherence; reduces gingivitis.
A. Basic ingredients in mouthrinses (see earlier discus- 4. NOT as effective as chlorhexidine or phenolics.
sion on dentifrices): G. SnF2: has weak antigingivitis effects, available in
1. Water, alcohol, flavoring agents, sweetener. dentifrice (0.45%) or mouthrinse (0.4%), but may
2. Active ingredient(s): cause exogenous extrinsic orange-tan stain; contains
a. Oxygenating (cleansing and antimicrobial) and NO ­alcohol.
astringent (shrinks tissue). H. Povidone-iodine: used with necrotizing or HIV-
b. Anodynes (relieve pain) and buffering agent ­associated periodontal disease.
(relieves pain, reduces oral acidity, dissolves I. Oxygenating mouthrinses (e.g., Amosan, Orthoflur,
mucinous film). Oxyfresh, hydrogen peroxide [needs to be diluted]).
c. Deodorizing agent (reduces malodor) and an- 1. NO effect on dental biofilm.
timicrobial (reduces oral microbial count and 2. Short-term use intended for oral wound cleansing
bacterial activity). and soothing effects.
B. Chlorhexidine digluconate mouthrinses (at 0.12%, 3. Long-term use has shown serious side effects, in-
Peridex, PerioGard; can be diluted to 0.06%): cluding carcinogenesis, tissue damage, mucosal
1. Requires prescription if dispensed; contains alco- ulcerations, hyperkeratosis, black hairy tongue.
hol (11.6%) unless noted; contraindicated if alco-
hol presents further complications. Enamel Sealants
2. High substantivity and bactericidal for gram- Enamel (pit and fissure) sealants are composed of or-
­positive and gram-negative bacteria and fungi; ganic polymers that bind to the enamel surface MAINLY
reduces gingivitis (as much as 60%); treatment of by mechanical retention to reduce caries.
aggressive periodontal disease, candidiasis, ­herpetic • See Chapter 17, Community Oral Health: community
stomatitis; also used postsurgically. sealant programs.
3. Side effects may include exogenous extrinsic A. Purpose: act as physical barriers to prevent oral bac-
brown staining, increased supragingival calculus, teria and their nutrients from collecting within a pit
poor taste, altered taste sensation, and in some, and fissure on occlusal or facial and lingual surfaces
reversible desquamation with dryness, soreness, (then creating acid environment essential for caries
ulceration of the oral mucosa; LESS effective in ­initiation).
presence of lauryl sulfate, fluoride, blood, protein. 1. NOT very effective in interproximal spaces and on
4. Recommended to rinse with 15 mL twice daily for other smooth tooth surfaces (where topical fluoride
30 seconds; SHOULD be used short term. is MORE effective).
C. Phenolic compound (essential oils) (e.g., Listerine 2. Can reduce occlusal fillings up to 50% on molars,
mouthrinse, dentifrice): 10% on premolars for permanents.
1. OTC, contains thymol, menthol, eucalyptol, meth- 3. Primary teeth are LESS retentive than permanents
ylsalicylate, 25% alcohol; contraindicated for (difference in enamel structure); success has NOT
those who are sensitive to alcohol. been shown.
374   Saunders Review of Dental Hygiene

B. Indications: 2. Photopolymer (visible light or light cured): second


1. Deep occlusal pits and fissures; highest to lowest generation (first generation used UV light, harder
risk for caries: to manipulate, more expensive).
a. First and second permanent molars. a. One component system; does NOT require mix-
b. First and second primary molars. ing; chemical initiator is NOT activated until il-
c. First and second premolars. luminated by light source.
d. Third molars. b. Unlimited working time; less chance of incor-
2. Facial and lingual smooth surface pits; highest to porating air bubbles into materials, causing
lowest risk for caries: ­failure.
a. Buccal pits of mandibular molars. c. However, takes MORE working time to flow
b. Lingual pits of permanent maxillary molars. into pits and fissures (grooves).
c. Lingual pits of permanent maxillary incisors. F. Application:
3. With higher rate of decay, needs to have risky ar- 1. Clean enamel surfaces with pumice or air abrasive
eas sealed as soon after eruption as possible. polisher; ALL types can be placed after fluoride
4. Xerostomic conditions may lead to decay. treatment and/or professional oral prophylaxis
C. Relative contraindications: (even with fluoride paste) because of etching of
1. Open occlusal lesions or caries on proximal sur- enamel.
faces. 2. For hydrophobic resins, isolate area with moisture
2. Adult with low caries and restoration rate. control if needed (rubber dam, cotton roll holders,
3. Well-coalesced pits and fissures. moisture-absorbent dry angles [Dri-Angles], cheek
4. Behavior that does NOT allow a dry field, if needed pads); MOST common cause of failure is moisture
for hydrophobic resins. contamination with these types.
5. Small percentage of the population is known to 3. Dry tooth surface for 30 seconds.
have allergic response to acrylate resins. 4. Etch with phosphoric acid (30% to 50%) for 15 to
6. Short life expectancy of the tooth. 20 seconds for permanents and 20 to 30 seconds
D. Composition: for primary teeth, apply to pits and fissures and
1. Contains organic monomer bisphenol-A diglycid- at least a few millimeters beyond final margin of
ylether methacrylate (bis-GMA), reaction product sealant.
of bisphenol A and glycidyl methacrylate; ADA a. Called “tooth conditioner” by some manufac-
considers exposure to bis-GMA an “acute and in- turers; creates micropores in enamel so that the
frequent event with little relevance to estimating sealant flows and mechanically locks in; some
general population exposures.” also have chemical bonding.
2. Unfilled types: resin component of dental compos- b. Use eye protection for clinician, assistant,
ites; filled types: filler of inorganic material such ­patient.
as glass for strength to reduce occlusal wear from c. May cause burns; avoid contact with oral tis-
chewing and to increase opacity for easier visual- sues, eyes, skin; if accidental contact occurs,
ization. flush affected area with generous amounts of
3. Hydrophobic (“water hating”—must use dry water; in case of contact with eyes, immediately
field) or hydrophilic (“water loving”—saliva rinse with water and seek immediate medical
provides wet environment necessary for activa- attention (see Chapter 10, Medical and Dental
tion) resins; hydrophilic are LESS technique sen- Emergencies).
sitive. 5. Rinse (large amount) for 30 seconds and dry again
4. Fluoride-releasing types (fluoride salt, fluorosili- for 30 seconds; dry tooth is essential for successful
cate glass) or calcium phosphate inclusion may retention of sealant material if hydrophobic; tooth
help protect the tooth structure from caries by is dried until has chalky, frosted ­appearance.
­aiding with remineralization. 6. Etch again if tooth does NOT have chalky, frosted
E. Polymerization: adding together of many molecules appearance, and dry thoroughly again; older per-
to produce larger molecule (polymer), forms highly manent and primary teeth may need re-etch.
cross-linked polymer network. 7. If chemically reactive, mix parts.
1. Autopolymer (chemical reaction, self-curing, liq- 8. Apply sealant material into pits and fissures
uid products contained in two bottles): with brush, cannula, or plastic instrument, most
a. Activator: bis-GMA and initiator: benzyl posterior tooth FIRST; do NOT apply exces-
­peroxide. sive amount (to AVOID occlusal discrepancies),
b. Equal drops of each are mixed together; resin since taste can be offensive if material touches
liquid hardens in 1½ to 2 minutes. tongue.
Clinical Treatment   375

9. If sealant flows onto soft tissue, rinse immediately,


then reapply to tooth. Mouth Protectors
10. If light curing, illuminate according to directions Mouth protectors (mouthguards) are designed to pre-
(UV use, shield from eyes). vent mouth injuries.
11. After has set, wipe sealed surface with wet cot- • See Chapter 15, Dental Biomaterials: impressions.
ton pellet; allows removal of air-inhibited layer of A. Used by participants in sporting activities to protect
nonpolymerized resin; failure to perform this may dentition from direct or indirect trauma.
leave an objectionable taste. 1. MOST sports injuries involve the oral cavity,
12. Use of dental floss is recommended to ensure open MAINLY in contact sports.
contacts, especially if a clear sealant is used. 2. The National Collegiate Athletic Association (NCAA)
D. After application: adopted mouth protector rule; MUST be worn, col-
1. Evaluate sealant both visually and tactilely for ored for compliance and ease of finding if lost.
complete coverage, checking for voids, bubbles, B. Types:
adequate seal; check interproximal surfaces with 1. Stock: made of rubber, polyvinyl chloride, or poly-
floss for any excess material. vinyl acetate–polyethylene copolymer.
2. If any deficiencies in the material, MORE material a. Available at retail shops.
should be applied. b. Manufactured in limited sizes (small, medium,
3. IMPORTANT to evaluate occlusion with ar- large).
ticulating paper for filled resins and make any 2. Mouth-formed: have outer, harder shell, typically
necessary adjustment; unfilled resins will wear made of polyvinyl chloride, and inner liner made of
down naturally and do NOT require occlusal ad- acrylic gel, silicone rubber, or polyvinyl acetate.
justment. a. Available at retail shops.
4. Inform guardian of procedures that have been com- b. Typically are heated and then formed around
pleted, and instruct that chewing ice, hard candies, the teeth.
or other hard objects can fracture sealant, which 3. Custom-made: fabricated from thermoplastic resin
causes leakage; leakage can lead to decay. sheet, heated and vacuum processed onto plaster
5. Reevaluate sealants periodically to ensure still in- model of individual’s teeth.
tact; replace or add to sealant when necessary. a. Require four basic steps to fabricate: (1) tak-
6. Avoid use of APF forms of fluoride, which can ing impression of arch; (2) pouring model;
cause roughing and pitting. (3) forming thermoplastic material over model;
7. If material fails, this is MAINLY result of operator (4) trimming and finishing.
error; MOST likely to occur within first 6 months b. Fit by a clinician; BEST because of reduction
after placement, with greatest loss in first molars, in gagging, irritation of the oral tissues, speech
least loss in premolars. impairment.

CLINICAL STUDY  

Age 13 YRS Scenario

Sex ☒  Male   ☐  Female The patient is in the dental office


for an emergency. Last week, while
Chief Complaint “Wow, I sure got hit hard and my playing football, he was tackled and
teeth got hurt too.” suffered another head concussion.
Medical History Broken ribs last year Visual examination reveals small inci-
sive fractures on teeth #6 and #7. His
Current Medications Concussion 2 years ago 12-year molars are partially erupted.
None

Social History Grade school student

1. What preventive dental measures might have been 2. What might his pupils have looked like with the con-
recommended to the patient and his guardians before cussion?
football season? Give the rationale for these recom- 3. Because of a busy schedule, the dental hygienist left
mendations. the patient’s alginate impression unwrapped on the
376   Saunders Review of Dental Hygiene

laboratory counter until the following day. What result with patients as they move from infants in their guard-
will this have on the final study model? ians’ arms to children coming for dental visits and then
4. Describe the basic steps for fabricating a custom-made to adolescents.
mouth protector. • See Chapter 7, Nutrition: eating disorders.
A. Infants:
1. Use of a mouthguard is recommended for prevention 1. Educate guardian about early childhood caries
of injury to the patient’s teeth and mouth. Mandatory (ECC) (see earlier discussion).
mouthguard and facemask regulations for high school 2. Instruct guardian on use of soft toothbrush or gauze
and junior college football players were enacted be- to clean oral tissues and teeth of infant.
cause of the high facial and oral injury rate. Although 3. NO dentifrice should be used that contains fluoride o9820
use of mouthguards is mandated, compliance must be (see earlier discussion of dentifrices) to reduce risk
achieved for effectiveness. Mouthguard can protect of damage to developing permanent teeth (“white
the dentition from direct or indirect trauma. Clinician spots” or hypocalcification caused by localized
should discuss not only fabrication of mouth protector, fluorosis).
but also consequences of noncompliance in wearing 4. FIRST dental appointment is recommended within
guard during sporting activities (e.g., chipped and bro- 6 months of eruption of first tooth or at 1 year
ken teeth, avulsed teeth). Clinician should explain that of age.
a custom-made mouth protector is designed by clini- B. Children:
cian to specifically fit individual’s teeth. As a ­ result, 1. See Table 11-1 for amount of dentifrice per age
custom-made appliance feels more comfortable, re- group at each brushing to minimize fluoride con-
duces gagging and irritation to oral tissues, decreases sumption if swallowed.
speech impairment. Advantages over the stock, store- 2. Guardian SHOULD be involved with brushing and
bought, or mouth-formed item may contribute to com- flossing until small motor skills develop and child
pliance with use. can become proficient with dental biofilm removal.
2. His pupils would have possibly been different sizes in 3. Short attention span of child demands shorter den-
response to shining a light on them. tal appointments; explain procedure and equip-
3. If alginate impressions are stored for more than ment at level a child can understand.
1 hour in air, they lose accuracy because of process 4. Special attention should be given to AVOID swal-
called syneresis, or loss of water by evaporation and lowing professionally applied fluorides during ap-
exuding of fluid. Final study model in this case will plication, since may cause nausea or cause toxic
be inaccurate, leaving the mouthguard to be possibly effects (in large doses).
ill fitting. C. Adolescents:
4. First, alginate impression of maxillary arch is needed. 1. To promote good hygiene practice, appeal to tooth
Once a good impression is made, study model is fab- appearance and acceptance from peers; treat as
ricated. Maxillary model then must be trimmed and transitioning adults.
a thermoplastic sheet of material must be vacuum 2. Puberty gingivitis is prevalent during these years;
formed over the model, trimmed, and finished by buff- educate teens on role in self-care; age group can
ing the edges. be associated with aggressive periodontitis (espe-
cially localized form).
Patients With Special Concerns   3. Orthodontic appliances common; special tools
Over a lifetime, patients may present with a number of may include powered toothbrush, oral irrigator,
special oral conditions that must be managed as part of tufted toothbrush, floss threaders, Superfloss, and
comprehensive dental hygiene care. The following con- additional fluoride (daily fluoride mouthrinse,
ditions need to be recognized, evaluated, and treated brush-on gel fluoride) or calcium products for re­
appropriately by the dental hygienist, and patients (or mineralization (see later discussion on orthodontic
guardians) need to be educated about role in self-care and fixed and removal appliance care).
disease prevention. These patient groups may be included 4. Transitional period to adulthood: MUST include dis-
in the case-based portion of the NBDHE (Component B). cussion of drug usage (including tobacco use), poor
• See Chapter 16, Special Needs Patient Care: medically dietary habits, eating disorders, oral piercing, STDs,
disabled patients. birth control pills, pregnancy, traumatic injury from
sports and lifestyle (mouthguards).
Pediatric Patient
Pediatric patients can benefit most from working with Pregnant Patient
the dental hygienist, since there is such an opportunity Full-term pregnancy is defined as the 40-week-long de-
to get an early start on oral health. Clinician can work velopmental period of the fetus.
Clinical Treatment   377

• See Chapters 2, Embryology and Histology: human de- c. Frequent intake of fermentable carbohydrates
velopment; 9, Pharmacology: pregnancy, placental bar- for nausea puts woman at HIGH risk for car-
rier, drug usage; 13, Periodontology: pregnancy-related ies; gagging and nausea may lead to inadequate
periodontal disease. performance of homecare.
A. Pregnancy is divided into first, second, third trimes- 2. Many drugs such as tetracycline and its derivatives
ters; each has own focus: are contraindicated during pregnancy to AVOID
1. During first trimester: embryo becomes a fetus and intrinsic staining effects during mineralization of
fetal organ systems develop. the teeth (see earlier discussion under stains); other
a. Lips, palate, and tooth buds form, and miner- antibiotics can be substituted.
alization of the tooth tissues starts toward the 3. Inadequate oral homecare can predispose patient
end. to gingival inflammation; although pregnancy it-
b. Nearly ALL drugs pass through by way of the self does NOT cause the inflammation, elevated
placenta to enter the circulation. hormonal influences of estrogens and progesterone
c. Thus is MOST critical period of development, during pregnancy can exaggerate gingival response
during which birth defects (teratogenic effects) to microorganisms; inflammation is reduced some-
may occur from infection, prescription or illegal what after pregnancy is over.
drug use, alcohol or tobacco use, or nutritional a. Pregnancy gingivitis: generalized gingival en-
deficiency. largement with overall appearance of sore, red-
2. During second trimester: fetal organ systems ma- dened, swollen, bleeding tissues; frequently
ture and fetal growth continues. associated with Prevotella intermedia; dental
3. During third trimester: fetus becomes fully mature and biofilm control instruction and thorough instru-
gains weight; average birth weight is 7 to 8 pounds. mentation are essential.
B. Systemic signs: undergoes systemwide changes dur- b. Pyogenic granuloma (pregnancy tumor): local-
ing pregnancy; NO trimester is without complica- ized area of gingival enlargement, typically in-
tions. volving interdental papillae, appears as painless,
1. During first trimester, may have difficulty with mushroom-shaped hyperplasia that bleeds easily
nausea and vomiting that predispose patient to and typically diminishes after birth (Figure 11-8).
malnutrition because of appetite loss. D. Risk factors for oral health:
a. Caries may result from acid in vomit contact- 1. Need to AVOID radiographs unless necessary to
ing teeth and oral structures; BEST period for the provision of dental care; if needed for emer-
preventive dental examination. gency care, limit the number of radiographs,
b. However, if periodontal disease or caries infec- use lead apron with thyroid collar, use parallel-
tions are severe and NOT treated, may increase ing ­ technique that does NOT require angulations
risk of premature low-birth-weight infant; oral toward the abdomen.
pathogens may stimulate uterine contractions 2. Administer local anesthetics that have been speci-
and cause premature labor. fied for pregnancy (category B: lidocaine and pri-
c. Appointments may have to be shorter but sched- locaine); general and nitrous oxide sedation use is
uled MORE often to accomplish this important a relative contraindication, would need medical
reduction of risk to growing fetus. consult before proceeding.
2. During second trimester, over morning sickness,
risk for developmental disturbances of fetus is
lower (SAFEST trimester), fetus is small enough
that patient can still sit comfortably in dental
chair.
3. During third trimester, may have difficulty sitting
or lying in dental chair for entire appointment.
C. Oral signs:
1. Erosion of enamel may occur in response to fre-
quent vomiting, which accompanies severe morn-
ing sickness.
a. Small, frequent meals of healthy, noncariogenic
foods SHOULD be recommended.
b. Mouth SHOULD be rinsed thoroughly with
water after vomiting, and mineralizing fluoride
and calcium products should be applied daily. Figure 11-8  Pyogenic granuloma (pregnancy tumor).
378   Saunders Review of Dental Hygiene

3. Restorative, scaling (and root planing) procedures a. Allow patient to shift position as needed, pref-
should be performed as needed in light of risk of erably to left side when orthostatic hypotension
dangerous infections; however, elective preven- presents.
tive and restorative dental procedures can be per- b. Supine positioning MUST be avoided if hypo-
formed safely at ANY time but may be delayed for tension persists; breaks for restroom use.
comfort and safety to second and third trimesters. G. Patient or caregiver education:
E. Barriers to care: 1. General health and well-being and a well-balanced
1. During pregnancy: morning sickness (gagging, diet that meets needs of both mother and fetus.
nausea, lack of appetite); inability to sit comfort- a. Adequate consumption of protein, calcium, fo-
ably for long periods (backache, frequent urina- lic acid, and vitamins A, B-complex, C, and D.
tion, fatigue, dizziness); and economic difficulty b. Education regarding dental myth “losing a tooth
because of increased medical costs and possible for every child”; explain how adequate calcium
reduction or loss of employment income. intake during pregnancy will prevent loss of
2. After pregnancy: lack of time (return to work, calcium from the bones; calcium is removed
family obligations) and economic difficulty be- from bones before teeth.
cause of increased costs associated with child 2. Dental biofilm-induced and hormone-­influenced in-
rearing and possible reduction or loss of employ- flammatory effects on gingiva and effect of vomit-
ment income. ing on enamel erosion; rinsing with water instead of
3. Nursing: many drugs are excreted into the breast brushing after vomiting should be recommended.
milk and affect the newborn, SIMILAR to drugs a. Meticulous dental biofilm control; includes
passing through the placental barrier; medical con- proper toothbrushing and flossing daily.
sult indicated. b. Can skip using a dentifrice if causes gagging or
4. Postnatal depression can occur; MUST be noted by use later in evening.
dental providers and referral made for treatment. 3. Fluoride application via dentifrices, gels, or rinses
F. Professional dental care: early intervention is KEY to as needed, depending on risk for caries.
health of both mother and child. a. Topical fluoride supplementation should be
1. Should be based on needs, in terms of both fre- provided ONLY for benefit of the mother.
quency and level of care; more frequent care (every b. NO studies have found a link between systemic
3 months) may be necessary for those with less- prenatal fluoridation and reduction in rate of
than-desirable homecare, those with periodontal caries in offspring.
disease, or those whose morning sickness results 4. Reasonable approach is to AVOID radiographs un-
in frequent vomiting. less diagnosis of oral disease demands attention.
2. Should include routine oral prophylaxis (and 5. Tobacco cessation if necessary owing to lower
root planing); avoid taking radiographs or giving risks of reduced birth weight, spontaneous abor-
drugs. tions, prenatal deaths, sudden infant death syn-
3. Should be provided in limited, shorter appoint- drome; also good time to consider quitting because
ments as required for patient comfort. of second-hand smoke effects on offspring.

CLINICAL STUDY  

Age 28 YRS SCENARIO

Sex ☐  Male   ☒  Female The patient has come to the dental


office because her gums are bleed-
Height 5’3” ing and swollen. She does not
Weight 145 LBS have routine dental examinations;
she prefers to wait until she has a
BP 115/78 dental problem before making an
appointment.
Chief Complaint “My mother lost a tooth for every child she had. Is
that going to happen to me?”

Medical History Five months pregnant, no complications, first child

Current Medications Prenatal vitamins

Social History Assistant principal at local trade school


Clinical Treatment   379

1. In what trimester of pregnancy is the patient? What or episode such as an MI; will need medical
fetal development occurs during this trimester? consult before all dental treatment.
2. What special precautions, if any, are recommended c. MOST serious emergencies in the dental office
when treating a pregnant patient? are related to this disease.
3. What is the most likely cause of the bleeding and swol- d. CVA results in coordination and mobility im-
len gums? What recommendations should be made? pairments that may require adaptive aids and
4. Does she need to worry about losing a tooth for every assistance with personal oral hygiene care.
child? Why? Why not? 2. Dementia and Alzheimer’s disease in some form;
incidence of senility increases with age.
1. She is in second trimester of pregnancy. During this a. Ensuring suitable personal daily oral care is
developmental period, organs of fetus mature and MAIN concern.
growth continues. b. Oral complications and alterations in dental hy-
2. Radiographs generally are postponed during preg- giene care are MORE frequently associated with
nancy, unless emergency condition indicates need the drugs that are used to control the ­disease.
for them. Use of drugs is not recommended at any c. With Alzheimer’s disease, may experience sei-
time during pregnancy unless emergency condition zure disorders and may have phenytoin (Dilan-
­indicates need for them. If there are any questions re- tin)-associated gingival hyperplasia.
garding either radiographs or drugs prescribed by the 3. Arthritis is a common source of discomfort and
dental office, a medical consult with patient’s physi- disability (see later discussion): MOST concerns
cian would be indicated. Special consideration during are with oral care alterations and recommended
dental treatment should be made regarding potential use of modified oral hygiene aids, such as powered
for nausea and/or orthostatic hypotension. toothbrushes and floss holders.
3. Patient appears to be experiencing pregnancy gingivi- 4. Individuals with DM are at increased risk for peri-
tis, influenced by sex hormones, exaggerated response odontal disease.
to dental biofilm. Recommendations should include a. Uncontrolled disease may require antibiotic
oral prophylaxis and meticulous homecare, including premedication before dental treatment.
toothbrushing and flossing. b. Appointments ideally should take place after
4. Loss of a tooth for every child is a myth. If inadequate meal, when blood sugar is stable.
calcium intake occurs during pregnancy, calcium is re- 5. Sensory defects:
moved from the mother’s skeletal and alveolar bones, a. Include visual impairment and hearing loss.
not from the teeth. She should be informed of the need b. Modification of oral hygiene aids and practices is
for adequate calcium in the diet, and myth should be often necessary to accommodate ­impairment.
dispelled. Most teeth are lost because of unrestored 6. Oral adverse drug reactions: SHOULD be advised
caries and/or periodontal disease, most likely explana- and educated about oral side effects.
tion for her mother’s lost teeth. a. High prescription drug use; MOST take at least
one drug with potential adverse reactions.
Geriatric Patient b. Xerostomia is MOST common drug-induced
There are three generally recognized gerontological oral condition.
stages: (1) “young old” (ages 65 to 74); (2) “old” (ages c. Abnormal homeostasis, soft tissue reactions,
75 to 84); (3) “old old” (ages 85+). Occurrence of chronic taste changes, alterations in host responses, gin-
diseases and multiple disease processes increases with gival overgrowth also occur.
age. Many fall into the category of medically disabled 7. Osteoporosis: MOST common bone disease.
because of these diseases. a. Thinning and weakening of skeletal structure,
• See Chapters 6, General and Oral Pathology: chronic which leads to loss of bone density and possible
diseases and conditions; 16, Special Needs Patient fracture.
Care: specific medical and physical disabilities. b. Women are four times MORE at risk for this
A. Common diseases or conditions: condition than men, especially after meno-
1. CVD accounts for MORE than half of all deaths. pause.
o10540 a. Oral complications and treatment plan altera- c. Evidence of loss of bone density may be appar-
tions vary based on the type and severity of the ent in the bones of the jaw in severe cases (pos-
disease; MUST determine functional capac- sibly noted on radiographs).
ity (FC), the patient MUST meet 4 metabolic B. Oral signs:
(MET) equivalents before dental care. 1. Prevention of dental disease has resulted in in-
o17990 b. A 4- to 6-week delay before emergency or elec- creasing number of older adults retaining teeth
tive dental care is needed after cardiac surgery (dentate).
380   Saunders Review of Dental Hygiene

2. Many oral conditions once believed to be a normal (1) LESS than half visit dentist annually.
part of aging currently are recognized as sequelae (2) Individuals with natural teeth are MORE
to disease. than four times as likely to seek dental
3. Age-related oral changes (normal) include darken- care.
ing of the teeth, attrition, gingival recession.
4. Disease-related oral changes: Edentulous Patient
a. Drug-induced oral conditions: MAJORITY of alveolar ridge resorption occurs in the first
(1) Xerostomia (dry mouth). year after extraction, with MORE bone resorption in the
(2) Gingival hyperplasia, oral candidiasis, sto- mandible. Alveolar ridges continue to change (remodel)
matitis, glossitis. throughout life. Patient still needs dental care even if does
(3) Hairy tongue. NOT have any teeth present. Dentures and implants are
(4) Trigeminal neuralgia. discussed next.
b. Periodontal disease: increases in severity with • See Chapters 6, General and Oral Pathology: denture-
age because of accumulated lifelong attach- associated lesions (denture sore mouth, candidiasis, hy-
ment loss. perplasias, angular cheilitis); 15, Dental Biomaterials:
(1) Slight gingival recession (1 to 2 mm) is denture care.
considered a normal part of aging. A. Reasons for an annual dental visit:
(2) Recession in excess of 1 to 2 mm MOST 1. Soft tissue examination for early identification of
likely is result of past or present periodon- cancer and other oral condition(s).
tal disease activity. 2. Cleaning and maintenance of denture and implants,
c. Caries: especially root surfaces because of gin- as well as soft tissues.
gival recession from periodontal disease and 3. Evaluation and reinforcement of good oral self-
improper brushing; xerostomia is often contrib- care for BOTH the denture or implant and oral
uting factor. mucosa.
d. Oral cancer: 4. Dentures need to be replaced or relined periodi-
(1) MORE cases occur; elderly are majority cally (7 to 10 years) as supporting alveolar ridge
of related deaths (however, now occurring changes.
in younger adults because of human papil- B. Dentures SHOULD be left out of the mouth for some
loma virus [HPV]). portion of every day so that the supporting mucosa
(2) MORE common in men than in women, as- can have a recovery (rest) period; standard recom-
sociated with alcohol and/or tobacco use. mendation is to leave them out overnight when pa-
e. Barriers to care: tient sleeps.
(1) Income limitations are an obstacle to dental C. Dentures SHOULD be permanently marked with the
care. wearer’s identification; kept in liquid bath when out-
(2) Few have dental insurance; Medicare does side oral cavity.
NOT reimburse dental services.
(3) Discretionary income is used to cover cost Patient with Dental Prostheses or Appliances
of dental care. Prosthesis is an artificial replacement for a missing body
(4) Education is positively correlated with part. Dental prosthesis typically replaces one or more teeth.
seeking adequate dental care. • See Chapter 15, Dental Biomaterials: dental prosthetics
(a) As general rule, overall have LESS and denture care.
formal education than younger popula- A. Removable prostheses: full denture, overdenture with
tions. either retained natural teeth or implants, and remov-
(b) Young old are BETTER educated and able partial denture and obturator.
are MORE likely to demand high- B. Fixed prostheses: fixed partial denture (bridge), den-
­quality healthcare than old old. tal implant, and implant-supported complete denture
f. Residential status affects ability to seek and re- (if it CANNOT be removed); individual crowns are
ceive dental care. considered in this category.
(1) MAJORITY live in family settings (are C. Appliances are special devices designed for specific
homebound). function or to create specific therapeutic outcome.
(2) Some are homebound but continue to live 1. Removable appliances: orthodontic appliances,
at home with assistance. such as head gear, retainers, or occlusal splints.
(3) About one in four can expect to spend some 2. Fixed appliances: orthodontic appliances, such as
time in assisted living (nursing home). bands, brackets, or palatal expander, periodontal
g. Use of dental services varies. splints, space maintainers.
Clinical Treatment   381

D. Prostheses and appliances are subject to accumulation 3. Special care is needed to prevent abrasions or wear
of dental biofilm, calculus, materia alba, stain; need on acrylic surfaces; avoid bending or breaking par-
BOTH daily cleaning by the patient and routine pro- tial clasps and retainer wires.
fessional maintenance care. 4. Standard precautions are followed to prevent
1. Teeth adjacent to and supporting removable or cross-contamination.
fixed prosthesis or appliance typically are more F. Professional care for fixed orthodontic appliances:
dental biofilm retentive and harder to clean; higher 1. Principles of debridement are SAME, but brack-
risk for caries and gingival inflammation. ets, wires, and ligatures require special adapta-
2. Soft tissues under removable prostheses and appli- tion and care to prevent disturbing or straining
ances are MORE prone to irritation and lesions: metals.
a. To prevent soft tissue damage, removable pros- 2. Patients need MORE personalized and specific
theses and appliances should be kept clean. instruction in dental biofilm control methods to
b. Also left out of the mouth for some portion adapt to all areas of the teeth and appliances.
of each day (e.g., leave denture out overnight 3. Regular or special toothbrushes (narrow shape,
when sleeping). smaller, powered), oral irrigators, floss threaders,
E. Professional care of full or partial removable denture and toothpicks, with or without adapters, are rec-
and orthodontic retainers: ommended.
1. Goal is removal of dental biofilm, calculus, stain 4. Daily self-administered fluoride application or
without damaging the acrylic or metal surface. calcium products are highly recommended to
2. One or more of the following cleaning methods is prevent decalcification (white spot) and caries
used: ultrasonic bath, careful scaling, polishing, around bands and brackets (see earlier discus-
use of denture or other toothbrush. sion).

CLINICAL STUDY  

Age 69 YRS Scenario

Sex ☐  Male   ☒  Female Patient presents to the dental of-


fice with a pain on chewing in the
Height 5’2” mandibular left molar area. She wears
Weight 155 LBS upper denture and lower partial
denture.
BP 115/70

Chief Complaint “My mouth hurts here.”

Medical History Alzheimer’s disease, middle stages


Incontinent
Congestive heart failure (CHF) 2 years ago

Current Medications 10 mg nifedipine (Procardia) t.i.d.


20 mg furosemide (Lasix) t.i.d.

Social History Retired sociologist

1. What are the common oral side effects of the patient’s ­furosemide (Lasix) may contribute to xerostomia and
drugs? its sequelae. Diuretics are the first-line intervention for
2. What are the common mental and physical impair- high blood pressure (HBP). Channel blockers vasodi-
ments with middle-stage Alzheimer’s disease? late coronary and peripheral arterioles, lowering blood
3. Describe the procedures for professional cleaning of pressure.
partial dentures. 2. Alzheimer’s disease, nonreversible dementia, affects
4. Describe common oral manifestations seen in Al- thinking, memory, and personality. Patients in middle
zheimer’s patients. stage can experience disorientation, loss of coordina-
tion, restlessness and anxiety, language difficulties,
1. Calcium channel blocker nifedipine (Procardia) sleep pattern disturbances, progressive memory loss,
can cause gingival hyperplasia, and loop diuretic catastrophic reactions, and pacing.
382   Saunders Review of Dental Hygiene

3. To professionally clean partial dentures, place them in can be physical neglect or abuse, emotional deprivation
a zipper-locked plastic bag with solution. Then soni- or abuse, sexual abuse or exploitation.
cate the bag with dentures in the ultrasonic bath for 15 • See Chapter 18, Ethics and Jurisprudence: reporting
minutes. Wash partial dentures thoroughly with soap, abuse.
and rinse before returning them to the patient. Never A. Types of abuse:
use bleach to clean partial dentures because it can cor- 1. Physical neglect: failure to provide healthy en-
rode the metal framework and clasps. vironment for a dependent includes provision of
4. No specific oral signs are present with patients who adequate food, clothing, supervision, healthcare,
have Alzheimer’s disease; however, oral diseases do living environment, personal hygiene; educational
develop as disease progresses. In the late stages of dis- neglect can be either deliberate or result of igno-
ease, oral hygiene neglect, resultant decay, periodontal rance.
disease, and tissue trauma are common. Because use a. Dental neglect: willful failure of caregiver to
of ­antidepressants is common in this population group, provide appropriate treatment for dental disease
xerostomia can also contribute to oral diseases. when disease causes pain, discomfort, or delays
in growth and development.
Patient with Implant b. Includes rampant caries, untreated oral or facial
Individualized dental biofilm control regimens MUST be trauma, untreated oral infection.
developed for patients with implants because each implant 2. Physical abuse: nonaccidental biting, striking,
system is unique. Depending on whether the prosthesis can burning, lacerating, or other type of conduct that
be removed, dental biofilm control may present difficult results in injury.
challenges for the patient. Homecare cleaning aids that are 3. Emotional deprivation: failure to provide for emo-
effective, safe (must NOT scratch the titanium implant), tional needs of a dependent; includes withholding
and easy to use SHOULD be chosen. Patient’s understand- love, lack of caring, alienation, chronic criticism
ing of proper methods, frequency of use, and techniques of (common in immature guardians).
using cleaning aids and adjuncts SHOULD be evaluated. 4. Emotional abuse: purposeful use of demeaning,
• See Chapter 13, Periodontology: implant surgery, peri- vengeful, demanding, or aggressive behaviors to
implantitis, professional care (instrumentation). control dependent, includes role-reversal, whereby
A. If an overdenture is present, MUST be removed daily child or dependent controls caregiver (common in
for cleansing and soaking, SAME as any denture; immature guardians).
dental biofilm MUST be removed from around abut- 5. Sexual abuse: purposeful use of dependent for
ments. nonconsensual sexual acts, or sexual acts with de-
B. If the prosthesis is NOT removable, dental biofilm pendent who is unable to give informed consent.
removal becomes MORE difficult; requires use of a 6. Sexual exploitation: purposeful use of nonconsen-
variety of oral hygiene aids. sual dependent or dependent who is unable to give
1. Traditional toothbrushes, end-tuft brushes, in- informed consent to provide some form of sexual
terproximal brushes with nylon core tips (rather act for sexual or monetary gain of caregiver.
than metal core tips), and powered interproximal B. Signs of physical neglect:
brushes (Rotadent). 1. Unkempt appearance, including soiled clothing;
2. Tufted floss, floss threaders, yarn, gauze folded into dirty hair, hands, and skin; poor personal hygiene;
a ribbon, floss cords (Postcare) and dental tape or inappropriate clothing or clothing in need of re-
ribbon (G-Floss), plastic elastomeric flange-floss pair; lack of dental or medical care.
(Proxi-floss). 2. Lack of appropriate supervision, which can in-
3. Wooden interdental cleaners, plastic interdental clude allowing dependent unrestricted freedom to
cleaners, rubber-tipped stimulators. roam the streets, leaving dependent home alone,
4. Rinsing and swabbing implant interface with failing to provide needed assistance in proper oral
0.12% chlorhexidine is often recommended for and personal hygiene.
short-term use after abutment connection surgery 3. Improper or inadequate nutrition, which includes
or when inflammation is present. allowing indiscriminate eating and drinking of junk
C. Homecare methods and techniques should be re- foods and beverages, restricting dietary choices, or
viewed thoroughly at each recare appointment. providing too little food to give adequate nutrition.
C. Signs of physical abuse:
Abused Dependent Patient 1. Bruising: on areas of the body where bruising typi-
Clinicians should be aware of the signs of dependent cally does NOT occur.
abuse. Dependents are any persons under the care of an- a. Note that accidental bruising (NOT associated
other and include children, disabled adults, elderly. Abuse with abuse) typically occurs in areas overlying
Clinical Treatment   383

bone (shins, forehead) or areas that protrude 7. Head banging may be evidenced by bruising of
(knees, elbows). back of the head, loss of consciousness, and retinal
b. Nonaccidental bruising (associated with abuse) hemorrhage (shaken baby syndrome).
tends to occur in areas unlike those mentioned 8. Inappropriate behavior is common in the abused:
and may take on the shape of the object used a. Inappropriate fearfulness and crying.
to hit or pinch (fingers, hands, paddle, bat, belt b. Abrupt changes in behavior when separated
buckle, teeth); bruising tends to occur on the from a caregiver.
buttocks, thighs, face, neck, and upper arms; be c. Delays in language and growth development in
suspicious if: infants and children.
(1) Exhibits discomfort when sitting or lying in d. A withdrawn, unhappy character.
the dental chair. D. Signs of emotional deprivation or abuse:
(2) Multiple and multistage bruises are evi- 1. Improper behavior of a caregiver, including dis-
dent. plays of anger, a condescending attitude, and criti-
(3) Bruises are visible ONLY where cloth- cism of a patient in front of others.
ing shifts during movement (e.g., tops of 2. Improper behavior of the patient, including extreme
thighs). displays of fear, dependence, and withdrawal.
(4) Description of an incident does NOT match E. Signs of sexual abuse and exploitation:
the visual evidence. 1. Discomfort of the genital-rectal area when walk-
(5) Wears clothing that is unseasonable and ing or upon sitting.
used to cover bruising (e.g., long-sleeved 2. Sexually explicit behavior in a child.
or legged garments in warm weather). 3. Refusal to allow clinician to enter mouth.
c. Bruising can be in various stages of healing; 4. Severe gag reflex.
may be a strong indicator of abuse because ac- 5. Bruising of the palate NOT associated with acci-
cidental injury typically does NOT occur fre- dental injury.
quently enough to cause bruising in different 6. Oral lesions of sexually transmitted diseases.
stages: F. Oral signs of abuse:
(1) New: reddish purple. 1. Risk factors include a fear of dental procedures, in-
(2) Week-old: greenish. appropriate behavior in the dental office, inability
(3) Two-week-old: yellowish. to control gagging during procedures.
(4) Two-to-four-week-old: brownish. 2. Facial trauma, as evidenced by bruising, lacera-
2. Lacerations tend to occur on lips, eyes, or face; lac- tions, bite marks, or burns in unlikely locations or
erated maxillary labial frenum can indicate forced associated with an unlikely cause.
feeding. 3. Palatal bruising, which is associated with forced
3. Bites anywhere on the body (unless obviously oral sex.
made by toddler) are signs of abuse: 4. Torn maxillary labial frenum, which is often as-
a. Greater than 3 cm from canine puncture to sociated with forced feedings.
canine puncture indicates bites were made by 5. Severe caries or doral infections that remain un-
adult. treated after caregiver is notified of need for
b. Useful for identifying the perpetrator; bite ­treatment.
marks are very individual. 6. Oral lesions associated with sexually transmitted
c. Bites caused by humans create puncture or diseases.
pressure-type wounds that leave mark; animal G. Barriers to care include communication difficulties with
bites typically cause tearing. both patient and caregiver: dental professional may feel
4. Burns that appear suspicious and have NO reason- unprepared or uneasy dealing with abuse situations, pa-
able explanation; often are made by cigarettes, tient may be fearful of reporting abuse to anyone, and
immersion in hot water, or rope (in cases of con- caregiver may be in denial of any wrongdoing; abusive
finement). caregiver who feels threatened with discovery may fail
5. Hitting: purposeful delivery of blows: to keep appointments or may switch dental offices.
a. Associated with bruising, fractures, and internal H. Professional care and homecare:
injuries (brain hemorrhage, retinal hemorrhage, 1. Building trust with patient by explaining all proce-
and laceration or rupture of internal organs). dures and encouraging appropriate beh­avior.
b. To the face and head can result in fractured 2. Immediately reporting suspicion of abuse to super-
jaws, broken or avulsed teeth, lacerated lips. vising dentist or appropriate agency.
6. Hair pulling is indicated by bald patches and thin- 3. Discussing superficial treatment of injured areas;
ning hair. refer to physician as needed.
384   Saunders Review of Dental Hygiene

I. Patient or caregiver education: should include dis- through the nose, bite on bite block, or ­ concentrate
cussion of need to reevaluate injured area at a later on another object; or, in severe cases, applying a local
date. anesthetic gel, salt, or ice to desensitize the area just
before the procedure, as well as using nitrous oxide
clinical study   sedation. Radiograph films should be quickly placed
Scenario: Six-year-old male patient is visiting the den- in the mouth, after tubehead is focused, and removed
tal office for his annual examination, oral prophylaxis, immediately upon exposure. Tray used in fluoride
and fluoride treatment. His father walks him to the dental treatment should be left in the mouth for 1 minute
chair, kisses the top of his head, and returns to the recep- only (or less if not tolerated well), and patient should
tion room. His teeth are very clean and show no evidence be complimented on the ability to maintain the tray
of caries or gingivitis. During inspection of the oral mu- for that long (major benefit of the fluoride is received
cosa, a purplish bruise the size of a quarter is observed and tolerance can be increased as patient’s confidence
along the midline, in the posterior third of the hard palate. rises). Possibly fluoride varnish should be used as an
When asked about the bruise, the patient indicates that he alternative.
is unaware of its cause. He is well behaved while his teeth 3. Palatal lesion should be examined by supervising den-
are being cleaned, although he gags whenever the clini- tist, photographed if possible (no parental permission
cian touches his tongue or gingival tissues in the molar is required if abuse is suspected), described and docu-
regions. Because of his strong gag reflex, taking bitewing mented in the patient record, and examined 1 week
radiographs is difficult and giving a fluoride treatment later for signs of resolution. When abuse of any type is
is impossible. At the end of the appointment, when the suspected, authorities (department of social services,
father is shown the bruised area and asked whether he is police department, or other such agency) should be
aware of its cause, he indicates that he has no idea why notified while child is in operatory, if possible, or im-
the area is bruised. mediately afterward, if not; health professional should
not attempt to confront caregiver regarding concerns
1. What are possible causes of the purplish bruise on the of abuse.
patient’s hard palate? How old is the bruise? What evi-
dence is there to support your theory of the cause? HEALTH PROMOTION AND DISEASE
2. What causes a gag reflex? Identify ways to manage a PREVENTION FACTORS  
gag reflex while taking radiographs or giving a fluo- Promoting health and disease prevention is a primary ob-
ride treatment. jective of the dental professional and is MOST successful
3. Is any follow-up of the palate lesion necessary, and if when the patient is involved. Motivation and its related
so, why? factors, including communication, can work toward this
objective.
1. Palate bruise is caused when a hard object strikes pal-
ate with enough force to cause superficial blood vessels Motivation
to rupture and blood to pool in area. Type of bruise can Motivation is defined as the readiness to act, or the driv-
be caused innocently when person falls with a candy ing force behind our actions. Influenced by a patient’s
sucker, pen, or other hard object in the mouth, forcing perception of responsibility for own health. Internally
object against palate. However, may indicate sexual motivated people believe they can influence their own
abuse, in which penis is forced against hard palate. lives (they are responsible for success or failure), have
Because neither child nor father can recall injury to control over their own health, and will be motivated to
palate, bruising is recent, and strong gag reflex is ex- change long-term health behaviors. Externally motivated
hibited, it is prudent to suspect that bruising may be people feel they have little control over life events, tend
sign of sexual abuse. Guardian and child most likely to believe in fate (feel they are NOT responsible for
would recall recent traumatic incident with enough their health), require MORE supervision and direction,
force to cause large area of bruising. and need an outside source to reinforce positive health
2. Gag reflex is caused when oral tissues, particularly ­beliefs.
in the posterior regions, are hypersensitive to films,
instruments, or fingers that are moved across oral tis- Self-Perception of Need
sues or are retained in place for long periods. May be Patient needs are identified through interview and obser-
associated with previous unpleasant experiences, such vation; focusing on a patient’s expressed chief concern
as poor clinical technique or sexual abuse. Techniques or complaint helps identify patient’s motivational force.
to manage gagging include calming patient by ex- Needs theories are useful in determining the motivational
plaining all procedures; working efficiently and confi- process. Maslow’s Hierarchy of Needs (Figure 11-9) dem-
dently; providing distraction by having patient breathe onstrates that as one need is met, a person is ­motivated to
Clinical Treatment   385

Self
Actualization
Fulfillment of
unique potential

Esteem and Recognition


Self-esteem and the respect
of others: success at work; prestige

Love and Belonging


Giving and receiving affection;
companionship; and identification with a group
Safety
Avoiding harm;
attaining security, order, and physical safety
Physiological
Biological need for food, shelter,
water, sleep, oxygen, and sexual expression

Figure 11-9  Maslow’s Hierarchy of Needs. (From Gaylor LJ: The administrative dental assistant, ed 2,
St. Louis, 2007, Saunders/­Elsevier.)

satisfy the next need higher in the hierarchy; safety, love, C. Sincerity, concern, and rapport with healthcare pro-
and ego are the needs MOST often associated with moti- vider are directly related to level of motivation for
vating a patient. self-care by the patient. Age and level of health belief
Basic physical safety needs MUST be met first (first affect action:
two levels). Love and belonging, feeling a part of a 1. Children: gear toward guardian who assists child
group, and being a part of a significant relationship are with oral hygiene tasks.
typically met at the third level. Self-esteem and ego or 2. Adolescents: gear toward social acceptance.
a belief in one’s self is the fourth level. Self-actualiza- 3. Adults: gear toward social acceptance, health
tion development, fulfillment of one’s ambitions and maintenance, tooth retention, cost factors.
life’s goals, and realizing one’s potential are at the fifth
level. The human needs conceptual model incorporates Communication
concerns about the patient and his or her environment, Communication is an essential component of manag-
specific health and oral health needs, and dental hygiene ing oral hygiene care. It is continuous, inevitable, and
actions (Figure 11-10). irreversible; conducted on several levels; influenced by
physical setting.
Factors That Influence Motivation • See Chapter 13, Periodontology: compliance and non-
Personal and social needs are major factors in motivating compliance.
change. A. Nonverbal communication or behavior skills involve
A. Unmet needs serve as motivation for change: social body orientation, posture, facial expressions, gestures,
acceptance and peer group approval are the MOST touch, distance and space, tone of voice, and hesita-
common motivating forces and are stronger motivat- tion. Expressed message is MOSTLY ­nonverbal:
ing factors than health promotion: 1. Perceived message of nonverbal communication
1. Stain and dental biofilm or calculus accumulated influences relationship-building process between
on teeth affect appearance and serve as motivation patient and healthcare provider.
for removal. 2. Effective nonverbal communication connotes
2. Malador (halitosis, bad breath) is socially unac- interest and involvement, aids in expression of
ceptable and serves as strong motivation for self- thoughts and feelings, expresses the level of com-
care. fort and relaxation, and can give insight into the
B. Pain is ONLY an immediate motivation and does patient.
NOT serve as long-term motivation. Dental health is 3. Body gestures:
frequently neglected because the nature of the disease a. Hand gestures by the patient that give insight
is nonthreatening; once disease becomes more threat- are white-knuckle grasp, finger pointing, and
ening, patient will perceive need for action. extending the hand as sign of welcome.
386   Saunders Review of Dental Hygiene

Wholesome
facial image

Protection
Conceptualization
from health
and understanding
risks

Biologically
Responsibility sound and
Client
for oral health functional
dentition

Skin and mucous


Freedom from
membrane
anxiety and
integrity of the
stress
head and neck
Freedom from
head and neck
pain

Figure 11-10  Human needs related to oral health and disease. (Modified from Harris and ­Christensen,
2003. In Darby ML, Walsh M: Dental hygiene theory and practice, ed 2, St. Louis, 2009, Saunders/Elsevier.)

b. Crossed arms may indicate patient feels de- b. Head nodding and verbal “yes” aid in express-
fensive; arms open and relaxed at side may ing involvement.
convey patient’s willingness to accept infor- c. Feedback involves paraphrasing what the pa-
mation. tient has said; enhances open communication
c. Voice signals include loudness, intonation, or and allows expression of thoughts and feelings;
inflection and can express anger, fear, excite- also gives the patient an opportunity to validate
ment, and cooperation level. and/or correct the perception of verbal and non-
B. Verbal communication involves language and listen- verbal behavior.
ing skills of both healthcare provider and patient: 3. Facilitative or cooperative communication is the
1. Provider’s language SHOULD be kept simple, di- give and take between patient and provider; cre-
rect, and nonthreatening; use verbiage with mean- ates a harmonious relationship, builds rapport,
ing so patient understands the disease process and and gives a sense of empathy, respect, warmth,
consequences. genuineness, and self-disclosure; also enables the
2. Listening skills are necessary to achieve good patient to be involved in process of care and in-
comprehension of patient’s concerns, attitudes, creases patient compliance.
and feelings, as well as specific health beliefs: C. Questioning techniques encourage communication
a. Eye contact conveys interest by listening and process:
being fully attentive; helps patient relax and 1. Open-ended questions: require an explanation or
feel comfortable; clinician communication is description; who, what, where, when, why, and
enhanced when eye level is same. how are BEST word choices to begin.
Clinical Treatment   387

2. Closed questions: require yes, no, or a simple a. The fewer dental cleaning aids introduced, the
nod of the head; BEST way to move conversation greater the chance for compliance (so do NOT
along and to get closure, confirmation, or agree- bling bling bling the patients with what you
ment to an action. have in your operatory!).
3. Answers to questions should NOT be judged; pa- b. Use of dental cleaning aids SHOULD be re-
tient MUST feel that the answers have value. evaluated for need, compliance, and skill dur-
4. Provider’s acceptance of answers will encourage ing maintenance visits.
patient openness. c. Introducing additional dental cleaning aids
D. Learning principles: should be considered ONLY as skill and com-
1. Learning ladder demonstrates the principles of pliance are achieved with present aids.
­instruction (Figure 11-11): E. Planning individualized instruction:
a. Present small amounts of information; organize 1. Assess patient needs:
teaching sessions logically and systematically. a. Ask if patient has any specific concerns or is
b. Use visual aids to enhance learning; let the pa- experiencing any pain; ask what patient consid-
tient set the pace for learning. ers to be current dental needs.
c. Present information at an appropriate age level. b. Evaluate soft and hard tissues for pathological
d. Supervise the patient’s skill and habit develop- changes and document findings.
ment by involving the patient through return c. Assess radiographs for pathological changes
demonstrations. and document findings.
e. Provide immediate and specific feedback. d. Identify risk factors, such as tobacco use, diabe-
2. Active participation requires patient to set goals, tes mellitus, handicaps or disabilities, immuno-
fit behaviors into daily schedule, and decide which compromised conditions, stress.
dental cleaning aids patient is willing to use. e. Identify patient’s health beliefs and dental IQ
and apply to treatment planning.
f. Determine patient’s current oral hygiene tech-
niques and effectiveness by looking at tissue
Rung 6: health and amount of deposits.
Habit
Behavior is modified.
2. Establish a treatment plan by prioritizing needs
and treatment.
a. Plan strategies that account for the patient’s
Rung 5: dental IQ, health beliefs, and behavior.
Action b. Build in reevaluation steps throughout treat-
Action is based on ment.
perceived need.
3. Implement individualized instruction by explain-
ing the rationale and dental techniques to be used;
Rung 4: give treatment options; explain needs and answer
Involvement questions; initiate oral hygiene techniques by the
Motivated to act. patient; initiate treatment.
4. Evaluate treatment by observing and recording
dental biofilm levels, changes in tissue health, and
Rung 3:
skill performance of oral hygiene at each appoint-
Self-Interest
Information has ment; reinforce oral hygiene technique at subse-
personal meaning. quent appointments, modifying as needed, assess
the success of the treatment, and retreat as neces-
sary; document self-care progress or lack of prog-
Rung 2: ress in record.
Awareness
Correct information, lacks Clinical study  
personal meaning.
Scenario: A 37-year-old woman has been a patient at the
dental office for 2 years. She has a high caries risk be-
Rung 1: cause of poor dental care as a child. However, she is not
Unawareness very motivated to floss as needed, even though she has
Incomplete or inaccurate
been shown how to floss and why she should floss. When
information.
it is discussed, she crosses her arms in front of her and
Figure 11-11  Learning ladder. says she is not in pain now so why does it matter?
388   Saunders Review of Dental Hygiene

1. What does it mean when she crosses her arms in front B. Decreased white blood cell formation and function
of her? C. Longer half-life of medications
2. What type of process could help with understanding D. Frequent infections and poor healing
this patient? What can the dental hygienist do to help   5 One of following is NOT used in treating hypertension.
Which one is the EXCEPTION?
motivate the patient? Should the hygienist mention
A. ACE inhibitors
that she could be in pain someday because of caries?
B. Nitroglycerin
C. Beta-blockers
1. It is a nonverbal message. Crossed arms may indicate D. Diuretics
that patient feels defensive, while arms open and re- E. Calcium channel blockers
laxed at side may convey patient’s willingness to ac-   6 All of the following are examination techniques for evalu-
cept information. ating the soft tissues of the head and neck, EXCEPT one.
2. Dental hygienist could use concept of the learning ladder Which one is the EXCEPTION?
to help with motivation of patient. When she first came A. Visual observation
to the office, the patient was on bottom rung, rung of un- B. Percussion
awareness. Dental hygienist has the obligation to educate C. Palpation
D. Auscultation
and motivate this patient into daily flossing to achieve op-
E. Olfaction
timal oral health, since she has a risk of caries, and move
  7 All of the following are true of entries made into the dental
up the ladder to flossing being a habit. It is important to record, EXCEPT one. Which one is the EXCEPTION?
keep in mind that patients typically will not progress to A. Entries should be lengthy, with detailed recordings of
the top rung of the ladder in one or two hygiene visits, but recommended treatments and patient responses.
through regular visits and consistent guidance. Clinician B. Entries should use adequate space to record brief and ac-
needs to remember that sincerity, concern, rapport are di- curate statements of diagnosis and treatment rendered.
rectly related to level of motivation for self-care by the C. Entries should summarize a patient’s response, noting
patient. Discussion should be geared toward individual wants, needs, and expectations.
needs for the patient such as social acceptance, health D. Entries serve as a legal document.
maintenance, tooth retention, cost factors to increase mo-   8 Rhinophyma is BEST described as
A. a butterfly-shaped lesion over the bridge of the nose.
tivation as an adult. Pain is only an immediate motivation
B. an overgrowth of glands causing a red, bulbous nose.
and does not serve as long-term motivation. Discussion
C. numerous infected hair follicles surrounding the nasal
of caries and pain it could bring is not a good idea for septum.
motivating the patient. D. a clear drainage from the nose caused by an extended
sinus infection.
E. an obstructed airway caused by a deviated septum.
Review Questions   9 A maxillary torus
A. is of little significance and need not be recorded as a
clinical finding.
  1 Subjective information is diagnostic. Subjective informa- B. may need to be removed or reduced before the fabrica-
tion is obtained through tests and taking measures. tion of a full denture.
A. Both statements are true. C. is often removed to improve the patient’s ability to clean
B. Both statements are false. the lingual surfaces of the maxillary molars.
C. The first statement is true, the second is false. D. can become malignant in 15% of the female popula-
D. The first statement is false, the second is true. tion.
  2 A patient with coronary artery disease will present with all 10 All of the following are lesions that can be found on the
of the following, EXCEPT one. Which one is the EXCEP- tongue, EXCEPT one. Which one is the EXCEPTION?
TION? A. Fibroma
A. Use of nitroglycerin B. Hemangioma
B. Chest pain with exertion and anxiety C. Mucocele
C. Valvular defect D. Carcinoma
D. Inadequate circulation of blood to the heart 11 One of the following is NOT associated with mucogingival
  3 Patients with pacemakers may defects. Which one is the EXCEPTION?
A. report taking digitalis or calcium channel blockers. A. Lack of attached gingiva
B. present with swollen ankles. B. Clefting
C. report taking warfarin. C. High frenum attachment
D. present with dilated pupils. D. Tissue trauma
  4 Patients with kidney disease experience all of the following,
EXCEPT one. Which one is the EXCEPTION?
A. Need for premedication to prevent IE when a surgical
shunt has been placed
Clinical Treatment   389

12 A pigmented nevus 22 When an enamel organ is invaginated internally during tooth


A. rarely has hair growing from surface. development, it is referred to as which of the following?
B. is generally isolated to the posterior dorsum of the tongue. A. Gemination
C. is rarely elevated above the surface of the skin. B. Dens-in-dente
D. may have brown pigmentation. C. Dilaceration
13 Which of the following is a malignant tumor with etiology D. Fusion
directly related to sunlight exposure? 23 What is the BEST method for identifying a pulpal condition
A. Squamous cell carcinoma in a patient?
B. Juvenile melanoma A. Comparing results from electric pulp tester to standard-
C. Basal cell carcinoma ized scores for that tooth type
D. Intradermal nevus B. Looking for radiolucent areas at the apex of teeth on
14 What is the MOST common oral site for the appearance of periapical films
squamous cell carcinoma? C. Testing both the suspected and several control teeth by
A. Floor of the mouth several methods and comparing the results among teeth
B. Tongue D. Using the cold test (usually ice sticks, dry ice, or ethyl
C. Alveolar mucosa chloride)
D. Palate 24 All of the following are true of a percussion test, EXCEPT
15 Which of the following lesions has the highest mortality one. Which one is the EXCEPTION?
rate of all cancerous lesions? A. It can identify a problem of either periodontal or pulpal
A. basal cell carcinoma origin.
B. squamous cell carcinoma B. If the patient reports a very sensitive tooth, gentle finger
C. malignant melanoma pressure is used at first.
D. juvenile melanoma C. Teeth are tapped or moved in both a horizontal and a
16 Kaposi’s sarcoma is MOST commonly found in patients vertical direction.
with which one of the following diseases? D. Positive results indicate inflammation in the pulp chamber.
A. Uncontrolled type 2 diabetes 25 Which of the following tests is MOST widely used in con-
B. AIDS infection junction with electric pulp testing to provide a definitive test
C. Tertiary syphilis for pulp vitality?
D. Genital herpes A. Heat test
17 A sealant is placed on permanent molars to help prevent B. Cold test
A. Class I caries. C. Palpation
B. Class II caries. D. Percussion
C. Class III caries. 26 An electric pulp tester can be used on patients with all the
D. Class IV caries. following, EXCEPT one. Which is the EXCEPTION?
E. Class V caries. A. Congestive heart failure
18 A metal casting is a restoration made from which of the B. Angina pectoris
­following? C. Rheumatic heart disease
A. Plastic mold D. Cardiac pacemaker
B. Plaster mold 27 Which of the following is TRUE regarding chronic inflam-
C. Metal mold mation?
D. Plastic pattern A. Bleeds spontaneously when probed
E. Waxed pattern B. Appears soft and spongy
19 Hypocalcification is an area on the tooth that C. Appears edematous and bright red
A. needs to be restored. D. Appears leathery and tough
B. has started to demineralize. 28 Which one of the following is a limitation of the use of ra-
C. has started to remineralize. diographs for the assessment of periodontal disease?
D. has not completely calcified. A. Artifacts that may be mistaken for pathological changes
20 Attrition is loss of tooth structure caused by which of the B. A three-dimensional object projected on a one-
following? ­dimensional surface
A. Bacterial influences C. Interproximal areas that are difficult to see on radio-
B. Excessive horizontal forces graphs
C. Chemical influences on the cusp tips and incisal edges D. Consideration of age of the patient
D. Mechanical wearing E. Amount of gingival enlargement
21 All of the following are associated with ankylosed teeth, 29 Which one of the following characteristics would be LEAST
EXCEPT one. Which one is the EXCEPTION? desirable in a toothbrush?
A. First and second primary molars A. Head size that corresponds to mouth size
B. Hollow sound on tapping B. Handle that is easy to grip
C. Need for extraction C. Bristles that are natural
D. Subocclusal location D. Bristles that are multitufted
E. Roots fused to underlying bone
390   Saunders Review of Dental Hygiene

30 What is the MOST widely accepted method of toothbrush- 39 Which fluoride is contraindicated for use on composite and
ing? porcelain restorations?
A. Roll technique A. Sodium fluoride
B. Modified Stillman’s B. Hydrogen fluoride
C. Charter’s C. Acidulated phosphate fluoride
D. Bass D. Stannous fluoride
31 To reduce calculus formation, which one of the following 40 One of the following is INCORRECT regarding fluoride
components is added to dentifrice? varnishes. Which one is INCORRECT?
A. Potassium nitrate A. Varnishes are not washed away by saliva or food.
B. Soluble pyrophosphate B. Application of varnish requires less cooperation from
C. Strontium chloride the patient.
D. Sodium citrate C. Sodium fluoride is one of the frequently used fluorides.
32 All of the following are CORRECT in regard to chlorhexi- D. Application of varnish requires more cooperation from
dine mouthrinse, EXCEPT one. Which one is the EXCEP- the patient.
TION? 41 One of following is INCORRECT regarding fluoride.
A. Active against gram-positive microorganisms Which one is INCORRECT?
B. Approved in the United States in a concentration of A. Promotes demineralization of the enamel
0.12% B. Inhibits acid production in dental biofilm
C. An antimicrobial agent C. Prevents demineralization of the enamel
D. To be used on a long-term basis D. Decreases adherence of dental biofilm
33 Phenolic compounds do which of the following? 42 What symptoms might a child exhibit if he or she ingested
A. Alter the bacterial cell wall a toxic dose of fluoride?
B. Reduce caries A. Caustic burns on mouth and throat
C. Promote oral wound cleansing B. Nausea, vomiting, diarrhea, increased salivation, and
D. Interfere with bacterial glycolysis thirst
34 After application of topical fluoride foam or gel, the patient C. Sloughing of oral mucosa
should be instructed NOT to eat or drink for what period of D. Syncope and falling out of chair
time? 43 Externally motivated people
A. 10 minutes A. believe in fate.
B. 15 minutes B. feel they are responsible for success.
C. 20 minutes C. will be motivated to long-term health behavior.
D. 30 minutes D. require little supervision and direction.
E. 60 minutes 44 According to Maslow’s Needs Hierarchy, which of the fol-
35 Emergency treatment for acute fluoride toxicity if ingestion lowing needs is (are) associated with motivating a patient?
is >5 mg/kg body weight would include all of the following, A. Self-actualization and fulfilling one’s ambition
EXCEPT one. Which one is the EXCEPTION? B. Need for freedom
A. Giving milk orally C. Safety, love, and self-esteem
B. Inducing vomiting D. Need to own space and maintain control
C. Observing patient for several hours 45 Which one of the following influences motivation on a
D. Monitoring patient for irregular heartbeat short-term basis?
36 The acid etching technique used for the placement of pit and A. Esthetics
fissure sealants involves B. Malodor
A. 10% to 20% boric acid. C. Cost
B. 30% to 50% phosphoric acid. D. Pain
C. 40% to 50% maleic acid. 46 Which one of the following body gestures may indicate that
D. 10% to 20% phosphoric acid. the patient has become defensive?
37 Pit and fissure sealants are MOST effective in preventing A. Hands gripping the arms of the chair
caries B. Arms crossed over the chest
A. on occlusal surfaces. C. Hands linked behind the neck
B. on interproximal surfaces. D. Shoulders slumped forward
C. in lingual cingulum. 47 Which one of the following is an example of a “closed ques-
D. in buccal pits of molars. tion”?
38 Which fluoride is the active ingredient in MOST over- A. How many times a day do you brush your teeth?
the-counter fluoride rinses? B. Do you floss your teeth daily?
A. Stannous fluoride C. What kind of toothbrush do you use?
B. Acidulated phosphate fluoride D. When do you floss your teeth?
C. Hydrogen fluoride
D. Sodium fluoride
Clinical Treatment   391

48 According to the Learning Ladder Continuum, at what level 56 Which one of the following oral manifestations is a result of
is behavior modified? normal aging?
A. Awareness A. Xerostomia
B. Self-interest B. Attrition
C. Involvement C. Stomatitis
D. Habit D. Glossitis
49 When planning individual instruction, what is the MOST E. Periodontitis
important question to ask? 57 In older individuals, decay is MOST commonly located on
A. What can we do for you today? _________surfaces.
B. How long has it been since you have seen a dentist? A. occlusal
C. How often do you brush your teeth? B. proximal
D. When do you want to schedule your dental treatment? C. root
50 All of the following are true with regard to the aging pa- D. lingual
tient, EXCEPT one. Which one is the EXCEPTION? 58 The majority of elderly individuals
A. Generally has a softer diet A. live in family homes.
B. Is more sensitive because of recession B. live in nursing homes.
C. Is more likely to experience dry mouth C. have dental insurance.
D. Has more interproximal caries than other age group D. visit the dentist annually.
51 One of the following statements is INCORRECT regarding 59 The failure of a caregiver to provide dental care for a de-
pregnant women. Which one is INCORRECT? pendent when obvious signs of dental decay are present is
A. Oral prophylaxis is an accepted treatment during preg- considered
nancy. A. physical neglect.
B. Pregnant women may have gingivitis and require mul- B. physical abuse.
tiple cleanings. C. emotional abuse.
C. Dental treatment is best performed during the first and D. emotional deprivation.
third trimesters. E. sexual abuse.
D. Pregnant women should be educated on early childhood 60 Which one of the following is commonly associated with
dental health. physical abuse?
52 One of the following is NOT a dental prosthesis. Which one A. Bruising along the shins
is the EXCEPTION? B. Abrasions on the knees
A. Single-tooth dental implant C. Tearing of the maxillary frenum
B. Maxillary denture D. Fracture of the tibia
C. Four-unit bridge 61 Bruising associated with physical abuse often is
D. Space maintainer A. red and purple.
53 All of the following are examples of dental appliances, EX- B. green.
CEPT one. Which one is the EXCEPTION? C. green and yellow.
A. Single-tooth crown D. red, green, yellow, and brown.
B. Periodontal splint 62 Morning sickness puts a woman at risk for
C. Orthodontic retainer A. caries.
D. Occlusal night guard B. malnutrition.
54 All of the following are reasons for an edentulous patient to C. periodontal disease.
have an annual dental visit, EXCEPT one. Which one is the D. constipation.
EXCEPTION? 63 Which of the following is contraindicated during preg-
A. Soft tissue exam could identify early cancer or other nancy?
harmful lesions. A. Daily flossing
B. Dentures need to be relined or replaced annually. B. Tetracycline usage
C. Dentures need periodic cleaning. C. Mouthrinses containing alcohol
D. Self-care of denture needs reinforcement. D. Frequent meals
E. Underlying mucosa of denture needs evaluation. 64 All of the following are associated with morning sickness,
55 All of the following diseases or disorders are associated EXCEPT one. Which one is the EXCEPTION?
with ­aging, EXCEPT one. Which one is the EXCEPTION? A. Vomiting
A. Cardiovascular disease B. Enamel erosion
B. Hypertension C. High-carbohydrate snacks
C. Diabetes mellitus D. Nausea
D. Senile dementia E. Periodontitis
E. Muscular dystrophy
392   Saunders Review of Dental Hygiene

65 When taking radiographs of a pregnant patient, the clinician 74 Dentinal hypersensitivity occurs frequently in periodontal
must do all of the following, EXCEPT one. Which one is patients. Spontaneous remission without therapeutic inter-
the EXCEPTION? vention can be expected in
A. Use the paralleling technique. A. 10% to 20% of patients.
B. Limit the number of radiographs. B. 20% to 45% of patients.
C. Recline the chair in a supine position. C. 50% to 60% of patients.
D. Have patient hold the radiographs. D. 60% to 70% of patients.
66 The “losing a tooth for every child” myth is based on the 75 Agents that have been shown to be effective desensitizers
premise that include all of the following, EXCEPT one. Which one is the
A. calcium needed for the baby’s growth is removed from EXCEPTION?
the mother’s teeth as needed. A. Strontium chloride
B. constant vomiting in the first trimester causes deminer- B. Zinc citrate
alization of tooth enamel. C. Potassium nitrate
C. gagging leads to poor oral hygiene and caries. D. Zinc chloride
D. gingivitis can cause displacement and eventual loss of 76 Mary, a 76-year-old woman with osteoarthritis, presents
teeth. for routine oral care. Her homecare is good, but she is hav-
67 The key to a successful dental biofilm control program is ing some difficulty with care of the lingual surfaces of the
A. accurate instruction. mandibular molars. Based on this information, how many
B. an appropriate length of instruction. times a day should Mary brush and with what type of tooth-
C. patient motivation. brush?
D. multiple reinforcement appointments. A. Two times a day with a manual toothbrush
68 The toothbrushing technique in which the bristles are ap- B. Two times a day with a powered toothbrush
plied to the sulcular area and moved back and forth with C. Once a day with a manual toothbrush
short strokes in a vibratory motion is the D. Once a day with a powered toothbrush
A. Stillman’s technique. 77 Harold, a 54-year-old businessman, presents for oral hy-
B. modified Stillman’s technique. giene instruction. He has a three-unit bridge on teeth #13
C. Charter’s technique. through #15, tooth #14 is missing, and teeth #13 and #15
D. Bass technique. are pontics. In addition, large embrasure spaces on all pos-
69 Furcation areas are BEST cleansed with terior teeth are noted. Deep pockets are noted on DF #30,
A. dental floss and interdental brushes. MF #31, and DF #18. Based on these findings, which of the
B. toothpicks with handles and dental floss. following interdental dental biofilm removal aids would be
C. interdental brushes, toothpicks with handles, and end- the BEST choice for Harold?
tuft brushes. A. Textured floss, rubber tip stimulator, dental tape, and
D. dental floss, interdental brushes, and toothpicks with floss threaders
handles. B. Waxed floss, floss threaders, toothpick tip, and inter-
70 The mouthrinse that has the highest substantivity and also proximal brush
has been shown to reduce dental biofilm and gingivitis is C. Unwaxed floss, threaders, interproximal brush, and
A. chlorhexidine digluconate. wood stick (wedge stimulator)
B. phenolic compounds. D. Dental tape, threaders, wood stick, and toothpick tip
C. cetylpyridinium chloride. 78 At a recent recall appointment, 16-year-old Larry has been
D. sanguinarine. found to have rampant caries after many years of being car-
E. stannous fluoride. ies free. He has always had dry mouth because of using his
71 Dentifrices that contain soluble pyrophosphates or zinc inhaler for his asthma. Recently he has started to work at a
chloride are labeled drive-through window at a fast food restaurant, and he can
A. antigingivitis dentifrices. have all the soft drinks he wants. He has begun to sip them
B. antibiofilm dentifrices. to help him keep awake. Based on his history, what would
C. antisensitivity dentifrices. be the BEST protocol for controlling his caries?
D. antitartar dentifrices. A. Select other snacks from carbohydrate foods.
72 Xerostomia caused by radiation, medications, or a patholog- B. Use a 0.5% fluoride dentifrice before going to bed.
ical condition is of concern to the dental hygienist because C. Use an alcohol-containing mouthwash containing fluo-
it places the patient at greatest risk for the development of ride.
A. oral cancer. D. Chew sugar-free gum containing xylitol.
B. calculus.
C. periodontal disease.
D. root caries.
73 Sodium fluoride compounds, calcium hydroxide, potassium
oxalate, and ferric oxalate are all agents commonly used in
A. calculus-softening products.
B. tooth-desensitizing products.
C. tartar-control dentifrices.
D. anticavity products.
Clinical Treatment   393

79 Harold, an 82-year-old retired fireman, calls the office and 5 (B)  Nitroglycerin (NTG) is used for the treatment of
wonders what he should do. He was recently prescribed angina. ACE inhibitors, beta-blockers, diuretics, and
blood thinners because of a risk of cardiac problems. He calcium channel blockers could ALL be used to treat
states that he is getting bleeding gums sometimes when he hypertension.
brushes and feels maybe he should cancel his appointment 6 (B)  Visual observation, palpation, auscultation, and
next week What should Harold be told?
olfaction are all examining techniques to evaluate
A. Go off the blood thinners and come into the office for
soft tissues. Percussion can be used to evaluate teeth.
his regularly scheduled appointment.
B. When the blood thinner regimen ends, call the office for 7 (A)  Dental record entries should be brief and accu-
an appointment, since the dental care can wait. rate, NOT lengthy, and include detailed statements
C. Stay on blood thinners, and the office will obtain a of diagnosis and treatment rendered. Should summa-
­medical consult before his appointment. rize patient responses and are considered legal docu-
D. Stay on blood thinners, and the office will try to accom- ments.
modate his situation by not performing invasive proce- 8 (B)  Rhinophyma is an overgrowth of sweat and se-
dures. baceous glands of the nose, which makes the nose
80 Mary is your morning patient. She is a 28-year-old librar- appear red and bulbous. Associated with heavy alco-
ian who has just had a baby. She has no family locally, so hol consumption. Lupus erythematosus is associated
she asks many questions about oral care for the newborn.
with a butterfly-shaped lesion over the nose.
Which of the following would you want to include with
9 (B)  Maxillary torus, a benign overgrowth of bone,
your talk with Mary?
A. Use a pea-sized amount of fluoride dentifrice. is NOT of real significance in MOST people. Should
B. Place watered juice in baby bottle at night. fabrication of a denture be needed, a torus may inter-
C. Green stain is an exogenous intrinsic stain. fere with the patient’s ability to successfully wear the
D. She also needs to care for her mouth. denture (rocking or inability to achieve a seal). As a
result, surgical removal or reduction of the torus may
be needed before denture fabrication.
10 (C)  Mucocele is a mucus (saliva)-filled cyst that re-
Answer Key and Rationale sults from blockage of a salivary duct. It is usually as-
sociated with the minor salivary glands found in the
labial and buccal mucosa, but NOT in the tongue. A
o16900 1 (A)  Subjective information (feelings, perceptions, and at- mucocele under the tongue is called a ranula.
titudes) is obtained from the patient. Objective information 11 (D)  Tissue trauma is associated with irritation from
is such items as facts and results of tests and measure- a foreign object, NOT mucogingival defects. Lack of
ments. BOTH objective information and subjective infor- attached gingiva, clefting, and a high frenum attach-
mation are useful in making a diagnosis. Some clinicians ment are associated with mucogingival defects.
use SOAP (Subjective, Objective, Assessment, Plan). 12 (D)  Pigmented nevus lesion may have a brown pig-
2 (C)  Valvular defects do NOT affect coronary artery mentation. Often has a hair growing from the surface,
disease. Coronary artery disease causes the arteries may be scattered anywhere over the body, and may be
of the heart muscle to narrow, reducing blood flow flat or elevated from the surface of the skin.
to the heart and resulting in chest pain that can be 13 (C)  Malignant lesion that is associated with the skin
relieved by nitroglycerin. exposure to the sun is a basal cell carcinoma. Squa-
3 (A)  Digitalis and calcium channel blockers help to mous cell carcinoma is associated with smoking,
regulate irregular heartbeat and are often prescribed alcohol, and syphilis. Juvenile melanoma and intra-
to patients after the pacemaker is inserted. Swol- dermal nevus are NOT malignant lesions.
len ankles are a common physical presentation of 14 (B)  MOST common oral site for a squamous cell
­congestive heart failure. Warfarin (Coumadin) is an carcinoma is the tongue. Less frequently, it may ap-
anticoagulant taken for chronic heart disease to aid in pear on the floor of the mouth, alveolar mucosa, or
blood thinning and results in less pressure being ex- palate.
erted against the arterial walls. Patients with dilated 15 (C)  Malignant melanoma has the highest mortality
pupils may be using cocaine. rate of all cancerous lesions. Mortality rate for squa-
4 (B)  Leukemia is a disease of the white blood cells mous cell carcinoma is NOT as high. Prognosis for
that causes poor function and a decreased formation of recovery from basal cell carcinoma is good when
white blood cells. Kidney disease causes frequent in- treated early. Juvenile melanoma is NOT a malignant
fections and a longer half-life of drugs because of the lesion; benign compound nevus in young people; re-
kidney’s inability to break down drugs efficiently. An- sembles a malignant melanoma histologically.
tibiotic premedication is recommended for renal dis- 16 (B)  Kaposi’s sarcoma is indicative of an AIDS di-
ease patients who have had a surgical shunt placed. agnosis. Uncontrolled diabetes, syphilis, and genital
394   Saunders Review of Dental Hygiene

herpes are NOT specifically associated with Kaposi’s i­nterfere with the patient’s cardiac pacemaker. Use of
sarcoma. EPT is a noninvasive test, so a patient with rheumatic
17 (A)  Enamel sealants are placed to help prevent caries heart disease would NOT be exposed to a potential
in occlusal pits and fissures (Class I caries) of perma- bacteremia. EPT is NOT very painful or stressful,
nent posterior teeth. so should NOT pose particular risk for patients with
18 (E)  Waxed pattern is used in development of a metal heart conditions without a pacemaker.
casting of a restoration. When heated, it evaporates, 27 (D)  Chronically inflamed tissue appears tough and
leaving room for the heated metal to be forced into fibrotic, often having a leathery appearance. It does
the empty space. NOT bleed easily and often looks nearly normal in
19 (D)  Areas that are hypocalcified have never com- color. Acute inflammation appears edematous, soft,
pletely calcified and may never need to be restored. spongy, and bright red.
Decalcified area originally was calcified and at some 28 (A)  Artifacts can be mistaken for pathological
point started to break down. Remineralized area is changes on radiographs. Assessment limitation may
tooth structure that has started to break down but has result from three-dimensional object being projected
reversed its demineralization process and has hard- on a two-dimensional surface. Interproximal areas are
ened. easiest areas to assess bone loss. Buccal and lingual
20 (B)  Excessive horizontal forces, such as those caused areas are difficult to assess because of the density of
by bruxism or clenching, can result in attrition of the the tooth structure. Periodontal disease can occur in
occlusal or incisal tooth surfaces. Bacterial influences people of all ages. Enlarged soft tissue is NOT a ma-
cause caries and periodontal disease. Chemical influ- jor focus and is difficult to detect in the evaluation of
ences, such as those associated with bulimia, cause periodontal disease on radiographs.
erosion (acid destruction of enamel), affecting the 29 (C)  Natural bristles are less desirable than nylon
lingual of the maxillary anterior teeth. Mechanical filaments. Nylon filaments cause less damage to the
wearing is associated with abrasion. tissues and are rinsed and dried more easily, more du-
21 (C)  MOST common ankylosed teeth are primary rable, and more resistant to bacterial accumulation.
first and second molars and may be identified by a When choosing a toothbrush for a patient, consider
hollow sound produced on tapping and by location head size relative to the size of the patient’s mouth
inferior to the occlusal plane. Many ankylosed teeth and handles that are easy to grip.
have NO permanent teeth present to erupt should the 30 (D)  Bass or sulcular method is MOST widely ac-
ankylosed tooth be extracted. Primary tooth is usu- cepted toothbrushing method. Bristles are angled at
ally retained for as long as possible. 45° toward the gingival margin. Efficient in removing
22 (B)  Dens-in-dente is an invagination of enamel or- dental biofilm and BEST method for ALL types of pa-
gan during tooth development. Gemination is divi- tients. Roll technique is useful preparatory instruction
sion of single tooth germ by invagination, resulting for modified Stillman’s method. Modified Stillman’s
in incomplete formation of two teeth. Dilaceration method incorporates a vibratory stroke followed by
is distortion of a root and crown linear relationship, a roll stroke. Charter’s method, which requires that
whereas fusion is the union of two tooth buds. bristles be angled toward occlusal surface, is BEST
23 (C)  There is NO one BEST test for identifying pulp for orthodontic patients.
vitality. ANY individual test can give a false-positive 31 (B)  Anti-calculus-forming (tartar control) dentifrice
or false-negative result. BEST system is to use sev- contains soluble pyrophosphates. Potassium nitrate,
eral types of clinical data, comparing findings among strontium chloride, and sodium citrate are compo-
several of the teeth. nents found in desensitizing dentifrice.
24 (D)  Positive results to percussion test indicate inflam- 32 (D)  Chlorhexidine mouthrinse is intended for short-
mation in periodontal ligament that may originate from term use as an antimicrobial agent active against
the pulp or periodontal problem. Moving tooth in both gram-positive and gram-negative microorganisms, as
directions (horizontal and vertical) tests all of different well as fungi. Approved for use in the United States
fibers that constitute the periodontal ligament. at concentration levels of 0.12%.
25 (B)  Electric pulp testing (EPT) and thermal testing 33 (A)  Phenolic compounds (such as Listerine) alter the
are the MOST specific tests for pulp vitality. Cold is a bacterial cell wall. Fluorides reduce caries. Oxygen-
more versatile and therefore MORE commonly used ating agents are intended to cleanse wounds, and san-
thermal test than heat. Percussion and palpation also guinarine agents interfere with bacterial glycolysis.
provide information regarding the status of the pulp 34 (D)  Patient should NOT eat or drink for 30 min-
but are NOT as definitive as cold and EPT. utes after fluoride application (foam or gel) to avoid
26 (D)  During use of electric pulp tester (EPT), patient disturbing action of the fluoride. A period of 10 or
is exposed to a small electrical current that might 15 minutes is NOT long enough; 1 hour is excessive.
Clinical Treatment   395

35 (D)  Monitoring heartbeat (irregular or arrhythmia) CNS convulsions, and cardiovascular and respiratory
is NOT necessary until fluoride ingestion reaches distress. Emergency treatment includes induction of
>15 mg/kg body weight. Drinking milk, inducing vomiting and ingestion of large volumes of fluoride-
vomiting, and observing patient for several hours are binding liquids such as limewater, milk, and antacids
all appropriate actions to take for fluoride ingestion that contain aluminum or magnesium hydroxide. Ac-
>5 mg/kg of body weight. tivate (EMS) system for acute toxicity.
36 (B)  Etching process involves the use of 30% to 50% 43 (A)  Externally motivated people believe in fate. They
phosphoric acid, applied to the pits and fissures to be are NOT easily motivated to long-term health behavior
sealed. Acid removes inorganic materials and creates and require more supervision and direction. Internally
tiny crevices and micropores into which sealant ma- motivated people believe they are responsible for their
terial can flow, producing mechanical bonding. Me- success and will be motivated to long-term health be-
chanical bonding is force that holds sealant to enamel, havior. They require little supervision and direction.
although some also have a chemical bond. 44 (C)  Safety, love, and self-esteem are generally as-
o17260 37 (A)  The occlusal surfaces of the teeth are the MOST sociated with motivating a patient according to
accessible to acid etching, and therefore the effective- Maslow’s Needs Hierarchy. Self-actualization and
ness of sealants is greatest on these surfaces. fulfilling one’s ambitions are part of the hierarchy but
38 (D)  Sodium fluoride is fluoride MOST commonly are NOT associated with motivating patients. Need
used in OTC rinses. Sodium fluoride is a stable solu- for freedom, need for one’s own space, and need to
tion, which makes it ideal for this type of distribution. maintain control are part of the Dental Hygiene Hu-
Hydrogen fluoride is principal industrial source of man Needs Conceptual Model (Darby and Walsh.)
fluorine and hence the precursor to many important 45 (D)  Pain is a short-term motivating factor. Once pain
compounds, including pharmaceuticals and polymers leaves, the motivation diminishes. The way one looks
(e.g., Teflon). to others (esthetics), smell of one’s breath, and cost of
39 (C)  Acidulated phosphate fluoride has been shown to treatment may be considered long-term motivators.
cause pitting and roughening of composite and porce- 46 (B)  Crossing the arms across the chest indicates
lain materials. Therefore sodium fluoride is indicated that the patient has become defensive. Grasping the
for patients with extensive restorative work that in- arms of the chair may indicate apprehension. Linking
volves these materials. Stannous fluoride can cause the fingers behind the neck shows that the patient is
staining and therefore is also NOT recommended. confident. Shoulders slumped forward may indicate
Hydrogen fluoride is principal industrial source of ­uncertainty or lack of confidence.
fluorine and hence the precursor to many important 47 (B)  The question “Do you floss your teeth daily?”
compounds, including pharmaceuticals and polymers requires a yes or no answer, making it a closed ques-
(e.g., Teflon). tion. Open-ended questions ask the patient who, what,
40 (D)  One of the major advantages of these varnishes where, when, why, or how and require the patient to
is the fact that they do NOT require patient compli- give more than a yes or no answer.
ance with oral care instructions. Semiannual applica- 48 (D)  Habit is formed when behavior is modified.
tions have been used as an alternative for adopting Awareness, self-interest, and involvement are all
the regimen of regular rinsing with fluoride. steps in forming a habit.
41 (A)  Fluoride is effective at preventing caries because 49 (A)  The MOST critical question to ask a patient is
of its ability to inhibit the acid production in dental “What can we do for you today?” Additional questions
biofilm and decrease the adherence of materials to may also need to be asked, but ONLY after the patient’s
fluoride-covered surfaces. Fluoride replaces some of major concern (chief complaint) has been addressed.
the hydroxyl ions in enamel with fluorapatite, which 50 (D)  Aging patient is more likely to experience root
is LESS soluble and MORE resistant to acids. Fluo- caries than any other age group. More likely to expe-
ride ions act as a reservoir of fluoride during deminer- rience dry mouth (xerostomia) and sensitivity (den-
alization to promote the remineralization of enamel. tinal hypersensitivity) because of recession; diet is
42 (B)  Certainly lethal dose (CLD) of fluoride is the often of a softer nature.
amount that is likely to cause death if ingested (when 51 (C)  If dental treatment (other than oral prophylaxis)
NO counteragent therapy is applied). The adult dose is needed, BEST time to provide the treatment is
is 5 to 10 g of NaF taken at one time, and the chil- ­during second trimester. First trimester is NOT the
dren’s dose is 0.5 to 1 g (depending on the size and best time because of the rapidly changing fetus; how-
weight of the child). Signs and symptoms of acute ever, treatment would NOT be detrimental during this
toxicity include gastrointestinal distress (nausea, time; may be necessary to treat periodontal disease
vomiting, diarrhea, abdominal pain, increased saliva- to prevent premature low-birth-weight infant. Restor-
tion, thirst). Systemic distress includes hypocalcemia, ative dental care is generally avoided during the first
396   Saunders Review of Dental Hygiene

trimester unless it is emergency care and involves at least for a while. Few elderly have dental insurance
a serious infection. Third trimester is often avoided or visit dentist annually.
because the patient may be uncomfortable lying in 59 (A)  Failure of a caregiver to provide dental care for a
the dental chair. Multiple recall appointments may be dependent with obvious caries (dental decay) is consid-
necessary during the pregnancy because of hormonal ered dental neglect, a form of physical neglect. Emo-
changes that may cause an inflammatory response of tional abuse and emotional deprivation are harmful
the gingiva. This is also a good time to educate the behaviors that affect the emotional needs of a person.
mother-to-be on early childhood dental care. Sexual abuse involves nonconsensual sexual acts.
52 (D)  Space maintainer (fixed or removable) is an 60 (C)  Tearing of the maxillary frenum, MOST com-
appliance designed for a specific function or thera- monly associated with forced feeding, is a type of
peutic outcome, usually to hold the space of a pre- physical abuse. Bruising along the shins and abra-
maturely lost primary molar, preventing drift of more sions of the knees are common injuries in children,
distal teeth into the open space. When permanent NOT typically associated with abuse. Fractures of the
tooth begins to erupt, appliance will be removed or tibia and other long bones can occur from falls and
NOT worn. Dental prosthesis is defined as artificial are NOT typically a sign of abuse.
replacement for missing body part; good dental ex- 61 (D)  When physical abuse occurs frequently, bruising
amples are dental implant, denture, or dental bridge. in various stages is apparent. Variations in coloration
53 (A)  Single-tooth crown is a fixed prosthesis, an arti- (red, green, yellow, and brown) may indicate numer-
ficial replacement for the crown of the tooth. Dental ous abuse incidents, from days to weeks apart. New
prosthesis replaces single or several teeth. Periodontal bruise is reddish purple, then turns green, then yellow,
splints, orthodontic retainers, and occlusal night guards and finally brown, during the course of 2 weeks.
are all dental appliances. Designed for specific func- 62 (B)  Morning sickness puts a woman at significant
tions or therapeutic outcomes such as stabilizing teeth. risk for malnutrition. Caries can be a result of morn-
54 (B)  Dentures can last many years without replace- ing sickness if vomiting is present and frequent. Peri-
ment or relining, especially once the bone support odontal disease and constipation are NOT directly
has stabilized after the extraction of teeth. Immedi- associated with morning sickness.
ate dentures, however, may need to be relined after 63 (B)  Tetracycline, an antibiotic, is contraindicated
the first year. Annual examinations are IMPORTANT during pregnancy. Associated with yellow-brown ex-
for denture wearers because of the benefits of a soft ogenous intrinsic staining of the dentition, and should
tissue examination, denture cleaning, and evaluation be avoided during pregnancy and early childhood.
and reinforcement of good oral self-care. Flossing and frequent meals (low in simple carbohy-
55 (E)  Aging is associated with many disorders and dis- drates) are recommended. Alcohol mouthrinses are
eases. Cardiovascular disease (CVD), hypertension acceptable for patient use as long as they are NOT
(HBP), diabetes mellitus (DM), and senile demen- swallowed and patient does not have dry mouth or
tia are common among aging individuals. Muscular other sensitivity to alcohol.
dystrophy (MD) is an inherited disorder that causes 64 (E)  Periodontitis is NOT associated with morning
weakness and deterioration of the muscles and is of- sickness, unlike vomiting, enamel erosion, ingestion
ten evident in childhood. of high-carbohydrate snacks, and nausea.
56 (B)  Attrition, darkening of the teeth, and gingival re- 65 (C)  Unnecessary radiographs SHOULD be avoided
cession are the ONLY oral manifestations (signs) that during pregnancy. If radiographs MUST be taken, a
are considered a result of normal aging. Xerostomia, paralleling technique should be used and the num-
­stomatitis, glossitis, and periodontitis are often seen ber of radiographs taken should be limited to ONLY
in the aging population but are associated with illness those that are absolutely necessary for diagnostic pur-
and/or drug use rather than with aging itself. poses. Reclining the chair in a supine position is un-
57 (C)  Decayed (carious) root surfaces are especially necessary and may be uncomfortable for a pregnant
problematic for older individuals and are directly patient.
related to increased incidences of gingival reces- 66 (A)  The “losing a tooth for every child” myth is based
sion, periodontal disease (exposes the roots to oral on the premise that the calcium needed for the baby’s
environment), and xerostomia (often drug induced; growth is removed from the mother’s teeth as needed.
reduces salivary flow and pH-neutralizing effects). Skeletal bones (including the alveolar bones) supply
Caries on occlusal, proximal, and lingual surfaces is calcium for the baby that is NOT provided in the diet.
NOT specifically related to age unless it is located at Constant vomiting in first trimester may indeed cause
or apical to the gingival margin. demineralization of tooth enamel, and gagging may
58 (A)  Majority of elderly individuals live in a family lead to poor oral hygiene; both may increase the risk of
home. Fewer live in assisted living (nursing home), dental caries, but the effects can be reduced by daily use
Clinical Treatment   397

of fluoride gels or rinses. Gingivitis is associated with thorough dental biofilm removal is needed at least once
raised hormonal levels and inadequate dental biofilm a day. Because the patient has osteoarthritis, powered
removal and does NOT cause displacement or loss of toothbrush may be easier for her to manipulate.
teeth; inflammation does regress usually at pregnancy’s 77 (B)  Waxed (or unwaxed) floss and floss threaders are
end. BEST used to clean the bridge. Textured floss also
67 (C)  Patient motivation is the MOST important key to is BEST used for cleaning under bridges. Toothpick
a successful dental biofilm control program. Although tip can be used for teeth with deeper pockets, and
the accuracy of instruction, the amount of time spent, interproximal brush is BEST used for large embra-
and multiple reinforcement appointments are all im- sure spaces. Rubber tip stimulators and wood sticks
portant, all efforts will be futile if hygienist is unable do NOT remove dental biofilm under gumline or
to motivate patient to comply with regimen. between large spaces. Dental tape is BEST used for
68 (D)  Bass method of brushing involves placing bris- contacts that are NOT too tight.
tles in the sulcular area and moving them back and 78 (D)  Xylitol reduces microbial numbers and helps
forth in a vibratory motion. BOTH Stillman’s and with remineralization. The patient can use it on the
modified Stillman’s techniques involve placing the job, and it will promote salivary flow. To reduce car-
bristles on the marginal gingiva and moving them ies, the clinician would instead select other snacks
in a vibratory motion. Charter’s technique advocates from noncarbohydrate foods, recommend prescrip-
pointing bristles away from the gingiva. tion of 1.1% fluoride dentifrice (NaS at 5000 ppm),
69 (C)  Furcation areas are difficult to cleanse with con- and suggest a non-alcohol-containing mouthrinse (al-
ventional toothbrushes and dental floss. Special dis- cohol can promote oral dryness).
posable interdental brushes, toothpicks attached to 79 (C)  “Blood thinners” are anticoagulant or antiplate-
the ends of handles, and end-tuft brushes have been let agents. Used for treatment of CVD to increase
shown to be MOST effective in these difficult-to- blood flow by suppressing or delaying coagulation of
access areas. blood. Discontinuation of drug for dental care may be
70 (A)  Chlorhexidine digluconate significantly reduces ­hazardous because of embolus or thrombus formation.
dental biofilm and gingivitis and exhibits high sub- Side effects can be prolonged; spontaneous or inter-
stantivity. Phenolic compounds, cetylpyridinium nal bleeding can occur; hematoma more likely with
chloride, and sanguinarine have NOT been shown to some local anesthesia injections (especially inferior
exhibit substantivity or significant antibiofilm quali- alveolar nerve block). Requires monitoring bleeding
ties. Stannous fluoride has demonstrated only weak by INR levels. However, with CVD it is important to
antigingivitis effects. have a healthy periodontium, so patient should NOT
71 (D)  Anticalculus agents and tartar control dentifrices wait or have less done than what is needed. Thus he
(toothpastes) contain either soluble pyrophosphates should probably stay on “blood thinners”; office can
or zinc citrate as active ingredient. obtain the medical consult and then see him for his
72 (D)  Xerostomia presents a significant risk for the de- regular appointment and procedures after discussing
velopment of root caries when the preventive wash- it with his physician.
ing action of the saliva is diminished. Oral cancer, 80 (D)  Discussion should include the patient’s need o17690
calculus, and periodontal disease are unaffected by to also take care of her mouth, since decreasing the
xerostomia. parent’s or guardian’s levels of dental biofilm and re-
73 (B)  Sodium fluoride, calcium hydroxide, potassium sultant lowered level of dental disease may decrease
oxalate, and ferric oxalate are compounds that are the child’s risk of dental disease. She should NOT
MOST commonly used for desensitizing root sur- use any dentifrice with fluoride with an infant, since
faces. the newborn can swallow it, possibly affecting devel-
74 (B)  Although dentinal hypersensitivity occurs fre- oping permanent tooth buds, causing “white spots”
quently after periodontal therapy, spontaneous remis- (hypocalcification from localized fluorosis); once she
sion without further intervention has been shown to is sure child is NOT swallowing the toothpaste as it
occur in approximately 20% to 45% of patients in grows older, the toothpaste can safely be used. She
4 to 8 weeks, LESS noticeable 2 to 3 weeks after should place water in the baby bottle at night; breast-
nonsurgical periodontal therapy. feeding is an alternative that does NOT promote
75 (B)  Zinc citrate is MOST effective as an anticalcu- caries. Green stain is an endogenous extrinsic stain
lus agent rather than a desensitizing agent. Strontium from Nasmyth’s membrane, and MOST of it can be
chloride, potassium nitrate, and zinc chloride have brushed off.
demonstrated effective desensitizing properties.
76 (D)  Although toothbrushing traditionally has been
recommended after each meal, now it is suggested that
CHAPTER  12

Instrumentation
PURPOSE OF INSTRUMENTATION   • See Chapters 6, General and Oral Pathology, and 16,
p0010 The purpose of clinical instrumentation is to create an Special Needs Patient Care: medical diseases and dis-
oral environment where periodontal health is maintained abilities and additional modifications.
and tissues can return to health. It is accomplished by A. Advantages of CORRECT patient positioning:
debridement (or scaling) of the teeth to remove hard 1. Improves visibility and access to treatment area.
deposits to discourage dental biofilm (dental plaque) at- 2. Increases treatment efficiency.
tachment, such as calculus and/or stain during an oral 3. Increases patient comfort during appointment.
prophylaxis or nonsurgical periodontal therapy (NSPT), 4. Reduces clinician discomfort and fatigue.
as well as periodontal maintenance (PM). Manual instru- 5. Reduces possibility of occupational injury such
ments, power-driven instruments, or combination of the as cumulative trauma disorder (repetitive strain
two can be used to effectively debride (scale) the teeth injury) to neck, shoulders, wrist, hand (discussed
and accomplish this goal. Loupes and a headlight are later).
BOTH useful for improving instrumentation. Use of the B. Principles of patient positioning for instrumen­
dental endoscope (Perioscope) creates further opportu- tation:
nity for successful NSPT or PM by offering a tool to aid 1. Basic position using supine position from upright
in definitive instrumentation. position for initial patient seating:
Any instrument that means removal of endotoxins a. Chair back is almost parallel to floor; parallel
­(altered cementum) or by-products of microorganisms to floor for maxillary arch and raised 20° for
from root surface is controversial, and use of other means mandibular arch.
for removal of dental biofilm is now stressed. Also contro- b. Patient is almost parallel to floor; head, heart,
versial is the use of traditional root planing with removal and feet are at approximately same height; pa-
of root structure to produce a smooth, glasslike root sur- tient is NOT upright:
face. In addition, soft tissue curettage with removal of (1) Reduces possibility that patient could un-
the pocket lining is no longer recommended for health dergo syncope (fainting), especially during
of the tissue. Section on polishing in this chapter cov- invasive procedures such as local anesthetic
ers the selective method. Discussion of all these issues administration.
is included to allow for changes in testing materials over (2) Relative contraindication: congestive heart
time. The term “operator” may be used here instead of failure, severe breathing difficulties, ad-
“clinician” when discussing instrumentation. The Centers vanced pregnancy, back injuries.
for Disease Control and Prevention (CDC) recommends 2. Patient’s head position:
antimicrobial mouthrinse before any instrumentation. a. Top of head SHOULD be at top of chair to pro-
• See CD-ROM for Chapter Terms and WebLinks. vide clinician access to the mouth.
• See Chapter 13, Periodontology: NSPT and PM discus- b. H ead and neck SHOULD be aligned with
sion. the spine and supported by chair and head-
rest.
Positioning 3. Adjustments to improve visibility and access:
The CORRECT positioning of BOTH patient and cli- a. Treatment of maxillary arch:
nician during instrumentation is critical to long-term (1) Arch SHOULD be positioned so that occlu-
comfort and effectiveness of clinician. Takes into ac- sal plane is perpendicular to floor.
count ergonomic principles and incorporation of neutral (2) Light positioned over chest; shines into
positioning (see later discussion). Transfer of instru- mouth at an angle.
ments is not discussed here, except to emphasize that b. Treatment of mandibular arch:
to prevent injury to patient, instruments should never (1) Arch SHOULD be approximately parallel
be passed over the patient’s face or eyes. Also not dis- to floor.
cussed is concept of work zones that are set up like a (2) Light positioned directly over mouth,
“clock” around the patient (review from class texts). shines straight down.

398
Instrumentation 399

C. Principles of clinician positioning: 3. Preventive behavior:


1. Positioning on stool: a. CORRECT body mechanics or ergonomic con-
a. Feet flat on floor and height adjusted so thighs siderations MUST always be incorporated, us-
are parallel to floor. ing neutral body positions.
b. Sit back on stool so that body weight is fully b. BEST positioning of patient for clinician’s
supported and back is against backrest. comfort and access.
c. Keep knees and legs apart to create tripod effect c. BEST instrumentation to minimize repetitious
between two feet on floor and stool base to give motions; includes using sharp instruments.
stable seated position. d. Varying the activities and positions during ap-
d. Arms on a stool allow clinician to rest arms and pointment to alternate muscles used.
shoulders between procedures.
2. Ergonomics (good body mechanics): designing CLINICAL STUDY  
tasks and work areas to maximize the efficiency Scenario: A dental hygienist is on the first day of his
and quality of work. new position at a large dental clinic. Before taking this
a. Head SHOULD be centered over spine for sup- new position, he was the only dental hygienist for over
port; clinician’s chin is tilted gently downward 10 years at the same small dental practice after gradu-
for visibility, but head is not tilted to right or ating top of his class at age 20. He notices that he is
left. unable to put his feet on the floor when he sits on the
b. Eyestrain is reduced by having mouth at com- stool provided for him. He also has trouble instrument-
fortable focal distance, ∼15 inches from cli- ing the lower arch; it just seems there is not enough
nician’s eyes to mouth; clinician may need to overhead light, especially on the lingual of the ante-
consider an eye examination and/or loupes if riors. He also wants to get closer to his patients; he
this is NOT comfortable distance. has forgotten what the effective distance to a patient’s
c. Back is straight without being rigid, tilting for- mouth is. At the end of the day he notices that his back
ward from hips rather than curling the back or hy- hurts. He is worried about what this means to his future
perextending neck when necessary to get closer. in his profession.
d. Keep body weight evenly centered on stool.
e. Shoulders SHOULD be relaxed and even, nei- 1. What factors does the dental hygienist need to consider
ther raised toward the ears nor rotated forward in order to discover why he is having a backache?
(especially if patient is somewhat upright). 2. What could he have done to help with overhead light-
f. Upper arm SHOULD be kept close to sides, ing during instrumentation of the mandibular arch, es-
­elbows bent, forearms parallel to the floor. pecially the lingual of the anteriors?
g. Patient’s chair height is adjusted to place the 3. What is the distance he should have maintained to
mouth at level of clinician’s elbows. his patient’s mouths? What can he do if this distance
h. Wrist is held straight (neutral position) as much seems too far away?
as possible; other bending of wrist such as 4. If this situation continues for many years without any
pronounced flexion, extension, or deviation to changes to his way of practice, what could the out-
side SHOULD be avoided to prevent excessive come be for him?
stress on the nerves in wrist area.
i. Hands SHOULD be positioned so that palms 1. His backache may result from leaning over too close
are facing inward toward clinician’s midline as to his patients and not putting his feet flat on the floor
much as possible and in same horizontal plane with the stool he used. He should be able to sit back
as forearm (neutral position). on the stool so body weight is fully supported and the
j. Assistant SHOULD be at higher level than back is against backrest. His back should be straight
­operator. without being rigid, tilting forward from hips rather
D. Cumulative trauma disorders (repetitive strain than curling the back or hyperextending neck when
­injuries): necessary to get closer (see later discussion). In his
1. Neuromuscular injuries resulting from the ac- new employment he needs to locate, request, or adjust
cumulation of chronic small traumas to the same a stool that will allow him to practice in a safe and ef-
area are a potential occupational hazard for dental fective ergonomic working position.
personnel. 2. For treatment of mandibular arch, arch should be
2. Injuries to neck, shoulders, hand, wrist, and back approximately parallel to the floor, with overhead
have ALL been reported (e.g., carpal tunnel syn- light positioned directly over mouth, shining straight
drome, entrapment of median nerve in carpal tun- down. He may want to see if he can take advantage
nel leading to pain and numbness). of more indirect vision and/or illumination with his
400 Saunders Review of Dental Hygiene

mirror when he is instrumenting the linguals of the (3) Larger decreases muscle fatigue and cumu-
anteriors. lative traumatic disorder.
3. He needs to keep himself at comfortable focal dis­ (4) Common diameters: 3⁄8, 5⁄16, 1⁄4, 3⁄16. o9980
tance, ∼15 inches from his eyes to patient’s mouth; may b. Varies in surface texture, ranging from smooth
need to consider an eye examination and/or loupes if to rough-textured, knurled, or serrated.
this is not a comfortable distance. (1) Rough reduces slippage in wet environment.
4. If this continues over time, he could be looking at (2) Rough decreases muscle fatigue and cumu-
development of a cumulative trauma disorder, a neu- lative trauma disorder.
romuscular injury resulting from the accumulation of 2. Shank: connects working end to handle.
chronic small traumas to the same area (his back), an a. Functional shank is between working end and
occupational hazard for him. handle.
b. Terminal (lower) shank is between working end
Manual Instruments and first bend.
Manual (hand) instruments are classified as either as- c. Length determines area of access:
sessment or treatment instruments. However, many treat- (1) Shorter shanks used in normal sulcus depth
ment instruments can also be used for assessment during and around anteriors.
treatment. Manual instruments include mirrors, explor- (2) Longer shanks have additional length of
ers, probes, scalers, curets, hoes, files, chisels; available terminal shank; used in deep, narrower
in BOTH basic designs and types and those for specific pockets and around posteriors.
functions. d. Thickness determines shank flexibility, which
determines whether instrument is flexible or
Basic Instrument Design rigid:
Basic manual instrument design determines the intended (1) Thinner, flexible shanks allow greater tac-
purpose and location of use. Specific selections are also tile sensitivity for finer hard deposits.
based on personal preference and/or ergonomic concerns. (2) Thicker, rigid shanks have greater strength
Instrumentation with manually activated instruments, for heavier hard deposits.
BOTH for assessment and for treatment, requires high e. Angles and length determine use; angulation is
degree of precision and control and in some cases power simple or complex:
and force. CORRECT use of instruments can BEST be (1) Simple, straighter shanks on anteriors.
accomplished with consistent application of basic prin- (2) Complex, angled, or bent shanks on poste-
ciples of instrumentation. riors.
A. Basic instrument design (Figure 12-1): 3. Working end: part that does the work.
1. Handle: can be either single or double ended; var- a. Curets and scalers (sickle [Jaquette] scalers) are
ies in composition; either metal or high-density MOST common sharp instruments; cutting edges
plastic. formed by junction of face and lateral sides.
a. Varies in diameter; either narrow or large: b. Probes and explorers are MOST common as-
(1) Larger means BETTER instrument control. sessment instruments; do NOT have cutting
(2) Larger means BETTER transmission of edges; probe tip is dull or blunt, explorer tip is
fine vibrations because of hollowness. sharp; tip of nonsharp instruments is the “nib.”

4
1
3

5
2

Working end 5

Functional shank 4

Handle 3

Terminal shank 2

Working end 1

Figure 12-1  Basic instrument design with part noted.


Instrumentation 401

c. Mirrors and cotton pliers have special functions c. Ends are single or paired (mirror-imaged).
and designs. d. Function as providers of tactile information;
4. Well-balanced instrument is BEST: many need more than one for NSPT:
a. Designed with middle of working end centered (1) Assessment of hard deposits (especially
on long axis of handle. calculus) and irregular tooth surface during
b. Easier to use, more comfortable to hold, pro- ALL phases of dental hygiene treatment.
vides better leverage. (2) Identification of complex tooth anatomy
B. Manual instrument types: functions and area of use and pocket characteristics (e.g., concavi-
are based on design of working end. ties, contours of epithelial attachment).
1. Mirrors: (3) Detection of carious lesions; assessment of
a. Front surface MOST common; reflective coat- restorations for irregularities, lack of mar-
ing on face is easily scratched but does NOT ginal integrity.
produce ghost images. e. Basic types:
b. Plane or flat mirrors cause double or ghost im- (1) Shepherd’s hook (#23): hooked single end,
ages, and concave mirrors magnify images; BEST for caries detection and restoration
may cause some distortion. evaluation.
c. Functions are multiple and may be combined: (2) Pigtail (or cowhorn): curved shank paired o9990
(1) Retraction of cheek, lips, or tongue im- ends, BEST for hard deposit and for explo-
proves access and/or visibility. ration of healthy sulcular areas.
(2) Indirect vision: observation of reflected im- (3) The #17: right-angled single end, straight o10000
age when direct vision is impaired or imprac- and narrow shank with short 2-mm work-
tical; visualization of lingual of mandibular ing end, BETTER for hard deposit and root
posteriors, distal of maxillary posteriors, etc. surface exploration.
(3) Indirect illumination: provides illumination (4) The #3A: long curved shank single end, o10010
of dark area by reflected light bounced off mir- BEST for hard deposit and root surface
ror face; detection of caries and/or calculus. ­exploration.
(4) Transillumination: light reflected from (5) The (ODU) #11/12: area-specific for maxil- o10020
mirror through tooth to observe shadowing; lary molars with paired ends and complex
indication of caries, calculus, pulpal health, shanks (Gracey-like, others available),
especially interproximally. BEST for hard deposit and root surface ex-
2. Explorers (Figure 12-2): ploration.
a. Thin, wirelike working ends tapered to a point 3. Probes (periodontal probes): calibrated measuring
and circular in cross section. tool, usually single-ended but can be combined
b. Shanks are simple (for hard caries detection) with explorers or other probes on one end; blunt
or comp­lex (for hard deposit and root surface tip (Figure 12-3):
­exploration).

Ball tip

f0030
Figure 12-3  Periodontal probes (from left to right): Nabor’s is
Figure 12-2  Three explorers on left are used primarily for curved for furcations, PCP12 with Marquis markings on modi-
calculus detection; shepherd’s hook on right is used for car- fied shank, Williams markings at 1-2-3-5-7-8-9-10, Marquis
ies detection or evaluation of restorations. with 3-6-9 markings, WHO with ball tip used to record PSR.
402 Saunders Review of Dental Hygiene

a. Measurements in millimeters; variations in way


increments are displayed; color coding with
dark or light bands improves ease of reading
1
(some have yellow bands): 2
o1090 (1) Marquis: 3-6-9-12 mm markings; Williams:
1-2-3-5-7-8-9-10 mm markings; Michigan 3
O with 3-6-8, also PCP12 with variable 4
markings on modified Novatech shank,
easier to align with vertical tooth axis.
(2) WHO: 0.5-mm ball tip and 3.5-5.5-
8.5-11.5 mm markings; used ONLY for peri-
odontal screening and recording (PSR).
5
(3) Nabor’s (furcation): paired curved design with
or without millimeter markings at 3-mm inter-
vals; BEST for identification of furcations and Lateral sides 5
to examine root and/or pocket topography. 70°-80° 4
(4) Digital (computer): MORE accurate with
repeatable measurements, since uses same
Cutting edge 3

o10030 amount of force and measures to tenth of Face 2


millimeter. Cutting edge 1
(5) Laser lightbeam (DIAGNOdent): detects
new and residual hard deposits on subgin- Figure 12-4  Sickle scaler working ends may be either
straight or curved. All are triangular in cross section.
gival areas up to 9 mm with up to 80% ac-
curacy (twice that of conventional probe).
b. Functions:
(1) Periodontal assessment: (c) Measurement of oral lesions for diag-
(a) Measures sulcular and pocket probing nostic purposes.
depth (PD), distance from gingival mar- 4. Scalers (sickle scalers, Jacquette scalers, S204)
gin to epithelial attachment (EA) of the (Figure 12-4):
junctional epithelium (JE). a. Working ends may be straight or curved but are
(b) Measures clinical attachment levels always triangular in cross section, with two cut-
(CAL), distance from cementoenamel ting edges.
junction (CEJ) to EA if gingival margin b. Cutting edges converge to form sharp point at
is at or apical to CEJ; distance from gin- the tip.
gival margin to EA if gingival margin is c. Lateral sides converge to form pointed back.
coronal to CEJ. d. Straight shanks for anteriors (single ended) and
(c) Measures recession, distance from CEJ contra-angled for posterior (paired).
to gingival margin (when margin is api- e. Functions:
cal to CEJ). (1) MAINLY for supragingival hard deposits,
(d) Measures width of attached gingiva, can be used 1 to 2 mm below free gingival
distance from gingival margin to mu- ­margin.
cogingival junction, minus sulcus or (2) BEST for initial debridement of heavier
pocket depth. hard deposits because of strength.
(e) Assesses gingival characteristics, in- (3) NOT for removal in contacts and tight
cluding bleeding on probing (BoP), proximal surfaces, because of thin, pointed
with or without indices, presence of tip; pointed tip also increases risk of sub-
suppuration (pus), tissue consistency. gingival laceration or gouging curved tooth
(f) Detects hard deposits and root surface surfaces.
roughness, root and/or pocket topog­raphy. 5. Curets (Figure 12-5):
(2) Other uses: a. Lateral sides that taper to form rounded back on
(a) Overjet assessment: distance from max- working end, with rounded toe.
illary incisal edge to facial of mandibular b. Half-moon shape in cross section.
central; more than 3 mm of overhang. c. Simple shanks for anteriors and complex for
(b) Overbite assessment: distance of over- posteriors.
lap of anteriors in vertical dimension; d. Paired, mirror-imaged working ends (matched
more than one-third overlap. sets, e.g., #11/12, #13/14).
Instrumentation 403

6 8
7
1
2 10 90° 70°
3 9
5
Figure 12-6  Comparison of universal and area-specific
4
­curets. Universal curets have a 90° angle between the face and
10 the terminal shank, which gives them two functional cutting
edges. Area-specific curets have offset angle with a 70° angle
Lateral side

Back 9 between the face and the terminal shank. This makes lower
cutting edge the only one that is used for ­debridement.
Cutting edge 8

Face 7 Table 12-1  Gracey area-specific curets


Cutting edge 6
Gracey curet Designated area of use
Lateral side 5

Back 4 Gracey 1-2 and 3-4 Anterior teeth


Toe 3 Gracey 5-6 Anterior or premolar teeth
Gracey 7-8 and Posterior teeth buccal and
Face 2
9-10 lingual surfaces
Cutting edge 1 Gracey 11-12 and Posterior teeth mesial surfaces
15-16
Figure 12-5  The basic design of all curets is rounded toe,
rounded back, and a half-moon-shaped cross section. Gracey 13-14 and Posterior teeth distal surfaces
17-18

e. Functions:
(1) MOST versatile; BEST for both supra­ (c) Long shanks and offset angles allow
gingival and subgingival hard deposits, de- MOST effectiveness in deep, narrower
pending on thickness, length, angulation of pockets (after fives with about 3 mm lon-
shank, working end. ger shanks and also narrower blades; even
(2) BEST promotion of tissue safety because smaller with micro mini and mini-fives).
of rounded toe and back; BEST manual in- (d) Designed to adapt to specific surfaces and
strument for safe and effective subgingival teeth; treatment of ALL surfaces of poste-
adaptation. riors requires MORE than one instrument.
(3) Controversially: excellent cutting and (3) Turgeon curet: adaptation of Gracey curet
smoothing abilities for traditional root plan- with narrower working end and modified
ing, removal of altered cementum, as well triangular blade in cross section that is eas-
as hard deposits; produces smoothest root ier to sharpen.
surface but removes MOST root structure. (4) Langer curets: hybrid of universals with
f. Two design types: universal and area specific area-specific specifics; long, complex
(Figure 12-6): shank design, with blade face that is 90° to
(1) Universal curets: lower shank, with two parallel working and
(a) Blade face at 90° to terminal (lower) cutting edges; must be used in set to access
shank. entire dentition, and adaptation is difficult
(b) Has two parallel cutting edges per in the posterior with poor access.
working end. (5) O’Hehir area-specific curets: have circular
(c) Treat ALL surfaces with just one instru- discs at working end with extended lower
ment by use of ALL four cutting edges shank; used with push/pull strokes in deep
(two cutting edges per working end and periodontal pockets and furcations.
two working ends per curet). 6. Hoes (hoe scalers) (Figure 12-7):
(2) Area-specific curets: Gracey curet MAINLY a. Bulky working end that gives great strength but
used (Table 12-1): makes subgingival access and adaptation diffi-
(a) Blade face “offset” to 60° to 70° to ter- cult; careful adaptation and instrument control
minal (lower) shank. are critical to prevent damage.
(b) Has one cutting edge per working end b. Straight cutting edge with two sharp corners
(on longer, lower side of blade). that can gouge a root surface if poorly adapted;
404 Saunders Review of Dental Hygiene

8. Chisels (chisel scalers):


a. Straight cutting edge continuous with shank; single
99°-100° or paired ends; blade is flat and beveled at 45°.
Beveled b. Use of push stroke (ONLY push instrument
Face toe ­EXCEPT for O’Hehir curets).
Cutting c. Function:
45° edge (1) Best for supragingival use on heavy inter-
Figure 12-7  The hoe has a straight cutting edge. Since it is proximal; mainly on anteriors; there must
a bulky instrument, used mainly supragingivally.
NOT be any interdental papilla.
(2) Pushing from facial surface through inter-
90° or 105°
proximal surface toward lingual.
(3) Rarely used, replaced by others such as ul-
Cutting
edges trasonics (used when ultrasonics contrain-
dicated).
Back 9. Implant maintenance instruments (see Chapter 13,
Periodontology):
Figure 12-8  Periodontal file has multiple cutting edges. It is
a thin instrument that may be used subgingivally when it fits. a. Made of material that will NOT scratch smooth
surface of implants; working ends made of
straight cutting edge makes adaptation to curved ­various forms of plastic, graphite, resin, or tita-
areas difficult; angulation of blade to shank is nium instruments; ultrasonic tips can use fixed
99° to 100°. or disposable plastic covers.
c. Function: b. Shapes vary; some resemble familiar curets and
(1) BEST for removal of heavy supragingival; others are uniquely shaped to adapt to round
primarily on lingual of mandibular anteri- implant shapes.
ors and distal of MOST posteriors. c. Functions:
(2) Interproximal removal ONLY if adjacent (1) Removal from surfaces without scratching;
tooth is missing. very light, lateral force is ALL that is nor-
(3) Vertical pull strokes and two-point instru- mally needed.
ment contact with BOTH cutting edge and (2) Recommended technique is SIMILAR to
shank adapted to tooth for increased ­control. curet.
(4) Rarely used, replaced by others such as ul-
trasonics (still used when ultrasonics con- Principles of Manual Instrument Use
traindicated). Principles of manual instrument use include grasp, ful-
7. Files (periodontal files) (Figure 12-8): crum, adaption, angulation, stroke.
a. Multibladed hoes that are thin and rounded; A. Grasp:
thick shank gives strength; angulation of blade 1. Pen grasp: SAME way you would hold a pen when
to shank is 99° to 105°. writing, grasping handle with thumb and first fin-
b. Recommended technique includes vertical pull ger while middle finger supports instrument from
stroke with as many blades as possible adapted underneath; used with mouth mirror.
to tooth and root surface. 2. Modified pen grasp: BEST grasp with manual in-
c. Careful adaptation of flat blade is needed when struments for BOTH assessment and treatment.
working subgingivally to avoid gouging and a. Technique:
root damage. (1) Thumb and index finger are placed oppo-
d. Requires special sharpening instrument, either site each other and have primary grasp and
tanged or triangular file sharpener. control.
(1) Rarely used, replaced by others such as ul- (2) Handle rests on hand between second joint
trasonics (used when ultrasonics contrain- of the index finger and webbed area formed
dicated). between thumb and rest of the hand.
e. Functions: (3) Pad of middle finger, usually toward side,
(1) Best for crushing calculus, especially on but NOT on fingernail, rests lightly at junc-
burnished calculus. tion of shank and handle below thumb and
(2) Best for smoothing CEJs and overhangs. index finger.
(3) Best for removal in areas of limited access (4) Thumb and index finger are usually placed
such as deep, narrow pockets because of on the handle near the shank when using a
small size and flat shape. traditional intraoral fulcrum (tooth surface).
Instrumentation 405

(5) Ring finger rests against middle finger and c. Allows greater control over instrument and
is used as fulcrum finger. working stroke, MAINLY stroke length.
(a) For extraoral fulcrum, ALL fingers may d. Allows greater patient safety and comfort.
be moved farther up handle, away from e. Reduces clinician strain and fatigue.
working end. f. Traditional intraoral fulcrum: effective and
(b) Variation of grasp: index finger is bent comfortable for patient in MOST cases: ful-
more and moved higher on handle, giv- crum finger (ring finger) is placed on tooth sur-
ing tripod finger placement. face and remains in contact with middle finger;
b. Advantages: rocking or pivoting motion is used.
(1) Stable grasp with control. g. Alternative intraoral fulcrums: BEST when
(2) Allows instrument to be rolled in fingers used to increase adaptation, improve angula-
easily. tion, or create more effective fulcrum (lever-
c. Used during assessment: age) location or to create more physiologically
(1) Minimum finger pressure SHOULD be ex- sound wrist position: opposite-arch, cross-arch,
erted against instrument. finger-on-finger.
(2) BEST to use light grasp that allows infor- h. However, when forced to separate fulcrum o10040
mation, especially vibrations, to be sensed from instrument grasp fingers (considered split
by fingertips (e.g., feeling explorer ­bumping fulcrum), clinician has a loss of stability and
over hard deposit, into furcations, or probe strength in working stroke, even with palm-up
reaching EA). intraoral rest.
(3) Pad of middle finger is MOST important in i. Extraoral fulcrums: BEST to increase ­ access
sensing vibrations transmitted up shank. and effectiveness by improving adaptation and
d. Used during treatment: angulation, especially for posteriors, in small
(1) Grasp should be light during exploratory mouths, and in deep pockets of ­ periodontally
strokes made with curets and during place- involved teeth; used MAINLY to increase
ment phase of working stroke. power and control during working stroke, since
(2) Grasp should be moderate to firm during whole hand activated:
activation phase of working stroke. (1) MUST be at 9 o’clock position and use ex- o10050
(3) Close correlation between pressure of grasp tended grasp farther down handle.
and amount of lateral pressure against tooth (2) Palm or back of fingers or hand is rested
surface. against the face and reinforced finger rest.
3. Palm thumb grasp: rarely used with debride­­ment (3) Finger or thumb of noninstrumenting hand
instruments; limits mobility and tactile sensi­ is pressed against shank.
tivity. (4) Gracey curet’s lower shank MUST be par- o10060
a. Technique: allel to surface before starting stroke.
(1) Handle is wrapped in palm with ALL four (5) Can also be used with ultrasonics. o10070
fingers around it. C. Adaptation: relationship of working end to the tooth
(2) Working end is toward thumb, and thumb surface.
is left free to act as fulcrum or for special 1. Evaluating adaptation:
function. a. Cutting end SHOULD be positioned so that one
b. Uses: third of working end closest to tip is against the
(1) Air-water syringe and rubber dam clamp tooth (toe third).
holder. b. Point SHOULD be adapted to the tooth as por-
(2) Porte polisher for selected areas (see later tion of toe third; if point is adapted off of the
discussion). tooth, can cause soft tissue laceration and dis-
(3) Instrument sharpening with Neivert Whit- comfort.
tler sharpener or handpiece mounted stone c. Heel of working end SHOULD be kept close
(see later discussion). to the tooth to prevent excessive distention of
B. Fulcrum (finger rest): mechanical leverage point pocket tissue.
that allows greater force to be exerted at working end d. SAME guidelines for adaptation are used with
without increased effort. both assessment and treatment instruments.
1. Purpose: 2. Analyze adaptation difficulties:
a. Both MORE force and MORE lateral pressure a. Worst adaption on curved areas (e.g., line an-
can be exerted. gles) or convex (e.g., mesial surface of maxil-
b. Provides point of stabilization. lary first premolars) or narrow roots (e.g., lingual
406 Saunders Review of Dental Hygiene

mandibular anteriors), with poor access (e.g., 3. Working stroke (scaling stroke; activation phase):
third molars) or deep pockets. used with treatment instruments.
b. Common solutions to adaptation difficulties: a. Tight and controlled grasp with moderate to
(1) Rolling instrument in fingers. heavy lateral pressure, dictated by hard deposit
(2) Moving fulcrum. tenacity; lightest grasp and pressure that will
(3) Adjusting hand orientation on fulcrum. ­remove deposit is BEST.
(4) Changing instruments. b. Overlapping, with various stroke directions.
(5) Altering stroke direction and orientation of 4. Root planing stroke (controversial):
instrument on tooth (e.g., for line angle, use a. Grasp is moderate to light.
oblique stroke with toe toward apex). b. Length becomes progressively longer and lateral
D. Angulation: angle formed between working end and pressure becomes lighter as root is smoothed
the tooth surface. (can be followed by root debridement stroke,
1. Assessment instruments (explorers, probes): even lighter pressure to remove dental biofilm
a. For caries detection, point of explorer is pressed on previously planed root).
into tooth surface with terminal portion meeting c. Direction is multidirectional, with cross-
tooth at 90°. ­hatching.
b. For hard deposit and root surface exploration, 5. Stroke parts:
­working end of explorer is angled about 15° a. Insertion:
from tooth surface with ONLY toe (terminal) (1) Position working end at base of sulcus or
third lightly touching. pocket apical to hard deposit.
c. Periodontal probe is positioned close (almost (2) For bladed instrument, angle for insertion
parallel) to tooth surface with end in contact with of scalers and curets SHOULD be as close
the root surface (also considered adaptation). to flat against tooth surface as possible; NO
d. When probing proximal surface where there is angle.
adjacent tooth, probe MUST be angled 20° to (3) BEST if grasp and lateral pressure are
30° into col (see Chapter 4, Head, Neck, and light.
Dental Anatomy). b. Preparation for activation varies depending on
2. Treatment instruments (scalers, curets): angle type of stroke intended; for working or scaling
formed between face of blade and tooth surface stroke: grasp is tightened, desired angulation is
(see discussion next on strokes). established, lateral pressure is increased.
a. For insertion into sulcus or pocket: angle is c. Activation:
NOT present; as flat against tooth surface as (1) Movement to accomplish purpose of the
possible. stroke (wrist-rock) (e.g., exploration, prob-
b. For scaling and working stroke: angle is 70° to ing, scaling, root planing).
80°; LESSER angle such as <45° results in bur- (2) Stroke length and direction will vary.
nished hard deposits. (3) Hard deposit removal is from MOST sub-
c. For curets ONLY: gingival to MOST coronal.
(1) For root planing stroke: angle becomes 6. Stroke direction:
MORE closed at 60° to 45° (controversial). a. Selection of stroke direction is based on anat-
(2) For soft tissue curettage stroke: angle be- omy of individual tooth, access to tooth and
comes >90° so that cutting edge is directed surface, instrument selection, type of hard
toward epithelial lining of diseased pocket ­deposit.
wall; digitally support tissues (NOT recom- b. Options in stroke direction: vertical, horizon-
mended). tal or circumferential, oblique, cross-hatching
E. Stroke: types differ by amount of pressure applied to (combination vertical, horizontal, oblique).
tooth (lateral pressure) and by length and direction of 7. Stroke length:
movement; goal is to cover ALL tooth surfaces. a. Shorter strokes give more control and used for
1. Exploratory stroke: used with ALL instruments hard deposit removal.
for assessment. b. Longer strokes typically used for assessment
a. Grasp and lateral pressure SHOULD remain (exploring and probing) and root planing.
light throughout, SHOULD be overlapping and c. SHOULD overlap strokes to ensure that all ar-
multidirectional. eas of tooth surface are instrumented.
2. Walking strokes: use of up-and-down motion while d. BEST to treat each root as a separate tooth, if
moving forward around tooth in small steps; used access permits, with combination of horizontal,
with probes since EA varies in depth and contour. vertical, and oblique strokes.
Instrumentation 407

a. Lubricated with a fine oil or petroleum jelly be-


INSTRUMENT SHARPENING   fore each use to prevent glazing (metal particles
Manual instruments become dull with use. Sharpening clogging pores of stone).
is technique of grinding one or both surfaces that join to b. May be sterilized by ALL sterilization methods
form the cutting edge of manual instrument blade until a but may become dry and more brittle with re-
fine, sharp edge is produced. peated exposure to high heat.
A. Objectives: 2. Ceramic stone: hard, synthetic stone with fine to
1. Preserves original instrument shape, maintains medium grit (abrasive particle size).
sharp edge, maintains effectiveness of instrument a. Lubricated with water or used dry (useful for
design, extends useful life of instrument, and re- maintaining sterile stone).
duces instrument replacement costs. b. Tolerates ALL sterilization methods.
2. Increases ease and quality of instrumentation by 3. India stone: synthetic stone with medium grit.
decreasing hand fatigue and potential for occupa- a. Recommended lubricants include oil and water
tional injury. to prevent glazing.
3. Increases patient comfort because of decreased b. Can remove large amount of instrument in short
scaling pressure and reduced number of working amount of time because of increased abrasive-
strokes needed. ness.
B. Need and frequency of sharpening: c. Tends to groove and show wear with exten­
1. ALL instruments with a cutting edge need sharp- ded use.
ening on a regular basis: d. Tolerates ALL sterilization methods.
a. Cutting edge of instrument is formed by junc- 4. Shapes of stones: vary; includes wedge-shaped, cy-
tion of face and lateral side; has length but lindrical, conical stones (use on the instrument face).
should NOT have width. 5. Stone maintenance:
b. With use, sharp cutting edge is worn away, be- a. Protect the stone and use lubricant matched to
comes rounded and dull, needs to be sharpened. the stone type.
2. Rate at which instrument becomes dull is deter- b. Use entire surface of stone to prevent grooving.
mined by: c. Clean stone before sterilization; remove lubri-
a. Number of working strokes taken since last cant and metal particles from stone (glazing)
sharpening. using soap and a brush or ultrasonic bath (if
b. Amount of lateral pressure used. ­already contaminated).
c. Nature of the deposit. d. Sterilize stone between patients.
d. Composition of metal instrument; carbon steel E. Technique: two methods, instrument stable or stone
holds edge longer than stainless steel. stable.
e. When instrument was previously sharpened; 1. General principles:
sterilization in autoclaves (steam) can dull a. Shape of each new instrument SHOULD be
sharp instruments. studied so that it can be preserved.
3. Sharpen at FIRST sign of dullness: b. Each working end has either one cutting edge
a. Amount of work or scaling done since last or two parallel cutting edges that need to be
sharpening; the more working strokes and sharpened.
heavier the deposits, the sooner sharpening is c. Appropriate angles SHOULD be maintained:
needed. (1) Internal angle of 70° to 80° MUST be
b. “Feel” of the instrument on the teeth or deposit; preserved in BOTH curets and scalers for
dull instruments have LESS grab (slip easily), maximum effect of cutting edge.
tend to burnish hard deposits, and have reduced (2) Angle from face of the curet or scaler to stone
tactile sensitivity during exploring and light SHOULD be 100° to 110° (Figure 12-9).
­scaling.
c. Sharpness test shows instrument to be dull (see
Face 110° Face 110°
assessment discussion below).
d. Instruments may need sharpening periodically 70° 70° 70° 70°
during a debridement procedure.
C. Mechanical sharpeners (Sidekick, Periostar): can be
used for ease and MOST consistent ­results, with vari-
Figure 12-9  Instrument sharpening angles. Angulations
ous stones available. for sharpening both curets and sickle scalers (left to right)
D. Hand sharpening with stones and lubricants: are the same. Angle between the instrument face and the
1. Arkansas stone: natural stone with fine grit. stone should be 110° with an internal angle of 70°.
408 Saunders Review of Dental Hygiene

b. Sludge: when oil used as lubricant, accumula-


tion of oil and metal; when water used as lubri-
cant, ONLY metal particles collect.
2. Use of test stick:
a. Apply instrument at same angle for scaling;
press cutting edge into stick and release; how-
ever, shaving stick will dull blade.
b. Sharp instrument SHOULD bite or grab stick
surface when lightly pressed into surface and
released; test entire cutting edge.
c. May be used to evaluate instrument for sharp-
ness or dullness; can be part of standard instru-
Figure 12-10  Instrument sharpening: stone movement ment setup, sterilized between patients.
from heel to toe. The entire lateral side of the instrument 3. Light reflection (glare) test:
should be sharpened. This may require a dramatic change in
position as the instrument is sharpened from the heel to the a. Using lighting, hold cutting edge to be evalu-
toe, as shown with this posterior Gracey curet, or it may be ated so that it is directed toward evaluator’s
a very slight shift from heel to toe in a universal curet. This eye.
adjustment of the stone to instrument relationship is critical b. Sharp cutting edge becomes a fine line and will
for maintaining the original shape of the instrument. NOT reflect light; dull cutting edge has width
and will reflect light.
d. Selected surfaces are sharpened: 4. Magnification (hand or loupes):
(1) Lateral side is the MAIN surface to sharpen. a. May assist in assessment of blade for sharpness
(2) Face may also be sharpened, usually to re- and contour.
move wire edge or to maintain or recover b. Especially helpful with the light reflection test
original shape. and when recontouring instrument.
e. Wire edge: irregular projection that may form G. Instrument recontouring and replacement:
from burnished metal particles that extend be- 1. Recontouring:
yond the surface being sharpened (e.g., when a. Used for instrument that NO longer has its orig-
sharpening lateral surface, wire edge will pro­ inal shape but still has sufficient metal bulk in
ject above face). the blade.
(1) Prevention or removal: b. Goal is to recreate the original shape of the blade
(a) Prevented by ending on a stroke away using same principles and techniques as for reg-
from cutting edge (usually down- ular sharpening; metal is usually removed from
stroke). BOTH the lateral sides and the face.
(b) Honing: removing wire edge by taking 2. Replacement:
one or two light sharpening strokes on a. Replace an instrument when blade width or
adjacent surface (on face when the lat- length has been so reduced that the blade is at
eral surface is sharpened). increased risk of fracture during use.
2. Instrument stable; MOST widely used technique: b. Also recommended when instrument blade ef-
a. Instrument is held steady, either suspended in fectiveness has been decreased by loss of origi-
front of clinician or supported against a counter nal or intended shape.
edge. c. variables for lifespan: frequency of use (number o10080
b. Stone is moved up and down over lateral sides; of setups etc.), difficulty of patients, use of ul-
stone is moved over face, from heel to tip or toe. trasonics, type of metal in instrument.
3. Stone stable; LESS used technique (Figure 12-10):
a. Involves flat stone that is laid on instrument Power-Driven Scaling Instruments
tray or used with honing machines and sharpen- Use of power-driven scaling instruments, combined
ing devices. with manual instruments, may give BETTER results in
b. Instrument is moved across surface to sharpen most cases than either one alone.
lateral sides; may also still move stone over face. A. Indications for use:
F. Assessment (evaluation) for sharpness: 1. Prophylaxis (includes deplaquing for dental
1. Accumulation of metal filings: biofilm and stain removal).
a. During actual sharpening process, metal re- 2. Periodontal debridement (includes scaling and root
moved from instrument collects on face and is planing [controversial—see earlier discussion])
used as general indication that blade is sharp. and/or NSPT, possibly before surgical therapy.
Instrumentation 409

3. Supportive periodontal therapy (periodontal maint­e­ (2) Fluid source and type:
nance [PM]). (a) Water from dental unit SHOULD be
4. Overhang and orthodontic cement removal. purged through dental unit and ­powered-
B. Sonic scalers: frequency range of 2500 to 7000 scaler for 2 to 5 minutes at start of the
cycles/sec (cps). day; from 30 seconds to 2 minutes be-
1. ALL sides are active and MOST effective; ellipti- tween patients (SIMILAR to time used
cal motion. for air-water syringe).
2. Power sources: compressed air, attaches to high- (b) External fluid supplies using ster-
speed handpiece position on dental unit. ile water, antimicrobial solutions, or
3. Advantages: ­temperature-controlled water can have
a. Does NOT generate heat, therefore does NOT advantages for patient treatment.
require fluid lavage as coolant (although lavage (3) Suction: high-speed (high-volume evacu-
is desirable for other reasons). ator [HVE]) is BEST, slow-speed used if
b. Inexpensive, highly portable; small, light. patient is compliant.
4. Disadvantages: noisy and poor deposit removal (4) O-ring (if present): provides rubber seal for o10090
because of LOW power. handpiece coolant.
C. Ultrasonic scalers: frequency range of 18,000 to D. Ultrasonic instrument tips/inserts:
50,000 cps. 1. Technique:
1. Power source: electrical energy that connects to a. Tip parallel to long axis of the tooth with point
standard electrical outlet. pointing toward apical direction.
2. Two types divided on manner of converting elec- b. Last few millimeters of tip are MOST effec-
trical power to mechanical movement (vibrations) tive and SHOULD be positioned against the
at working end: tooth.
a. Magnetostrictive scaler (18,000 to 45,000 c. Point of working end should never be directed
cps): uses magnetic oscillations within metal toward the tooth surface, especially the root
stack or rod; elliptical pattern with ALL sur- surface (can cause gouging).
faces of the tip active (Cavitron, Odontoson). d. BEST to use light grasp to encourage use of
b. Piezoelectric scaler (35,000 to 50,000 cps): light pressure; heavy grasp (pressure) reduces
uses crystal oscillations; linear pattern with two effectiveness of tip, also causes increased
sides of the tip active, limiting adaptation (Sat- ­discomfort to patient, possibility of root dam-
elec, EMS, Symmetry). age, and unnecessary wear.
3. Unit controls: e. If patient is sensitive for one tooth, BEST to
a. Power setting selection: move on to next tooth and use manual scaling
(1) Controls amplitude and size of tip move- on sensitive tooth.
ment; high power gives high amplitude or f. If whole dentition is sensitive, may need to
large tip movement. check warmth of water, pressure of tip on teeth,
(2) Based on treatment needs and tip used: need for local anesthetics and/or manual scaling
(a) Low to medium: used for light or fin- (plus files, hoes, chisels).
ishing debridement, root planing, or de- g. Replace tip with loss of 2 mm from working end. o10100
plaquing, sensitive root surfaces, light 2. Standard (conventional, universal) tips:
and thin tips. a. Thick, strong, shaped like anterior curet or probe.
(b) Medium to high: used for removal of b. BEST used to remove heavier deposits and for
heaver deposits, heavy or thicker tips. initial debridement.
b. Tuning controls frequency of the tip movement: c. May be used for supragingival and subgingival
(1) Autotune: majority have built-in control. debridement but NOT to base of deeper pock-
(2) Manual tune: allows clinician to adjust, ets; possibly in shallow pockets.
giving greater control and thus potentially d. Often used at higher power setting; heavier
increasing patient comfort. metal tip does NOT move as much as a thin tip
c. Fluid: functions as coolant for machines that at the same power setting.
generate heat during operation and provides e. Important to usually use manual instrumenta-
therapeutic benefits. tion afterward if using ONLY standard tips.
(1) Fluid volume: may vary by manufacturers’ 3. Periodontal or thin (slim, narrow-profile) tips:
recommendations, typically, set to give fast a. Longer and thinner than standard, shaped like
drip of water from the tip and halo of fine probe in terminal portion, may be diamond
droplets. coated, beveled, or bladed.
410 Saunders Review of Dental Hygiene

b. Often in sets with right and left contra-angled c. Acoustic (micro) streaming: fluid flow exerts
tips and straight tip. considerable force and turbulence.
c. Used at low power to prevent patient discom- (1) Can disrupt cell walls and kill ­ microor-
fort and damage to tip for magnetostrictive, ­ganisms.
EXCEPT can use high power for piezoele­ctric. (2) MOST effective on highly motile orga­nisms.
d. BEST for removing light deposits or following 4. SIMILAR action as ultrasonic bath for instrument
initial debridement after use of standard tip. processing; healing is comparable between power-
e. May be used to base of deep pockets, giving driven and manual instrumentation.
extensive coverage. 5. Furcations at Class II and III can be MORE effec-
f. Portions of root that CANNOT be accessed tively treated with power-driven instruments than
must be instrumented manually with curets be- with manual instruments.
cause of better adaption. F. Comparison of power-driven to manual instrumentation:
o10110 g. Manual instrumentation produces smear layer 1. Advantages of power-driven instruments:
when debris is moved over subgingival root sur- a. Faster than manual debridement.
face, so after curet is used, ultrasonic should be b. Require light (featherlike) touch SIMILAR to
used on low setting with a thinner insert to provide probing:
final smoothing stroke and remove smear layer. (1) Easier to establish finger rest (fulcrum), but
4. Furcation tips: small ball-like projection at the it is NOT used for leverage or increased
very end of the tip. power of working stroke.
a. May prevent damage to root when working in (2) Less clinician fatigue and possibly less cu-
tight furcation areas, use after manual instru- mulative trauma disorder.
mentation, since may burnish hard deposits. c. NO cutting edges unlike other instruments:
b. Available in right and left, and straight thin and (1) Less soft tissue trauma, creating less dis-
slim style tips. comfort and bleeding.
5. Diamond calculus removal tips: specially designed (2) NO need to achieve effective scaling
to cut through tenacious calculus while protecting ­angulation; effective in stationary position.
the root surface. (3) Effective on ALL sides (less true for piezo-
E. Therapeutic effect: electric, since active mainly on lateral sides,
1. Deposit removal: and sonic instruments) and in any stroke di-
a. For calculus, is effective as with manual in rection.
MOST cases except in deep, narrow pockets (4) NO need for instrument sharpening.
(see earlier discussion). d. Thinner and longer than curets in some cases to
b. For extrinsic stains, very effective (less need for provide BETTER access to deep pockets and
­polishing). furcations.
2. Pattern for treatment: e. Provides oral irrigation:
a. Treatment begins at MOST coronal areas of de- (1) Reduces microorganisms.
posit and moves to MOST apical or subgingi- (2) Washes field for visibility.
val deposits in contrast to manual instruments, (3) Improves patient comfort in many cases
which move from MOST subgingival to MOST (warm water).
coronal. 2. Disadvantages of power-driven instruments:
b. LESS root structure is removed than with man- a. Constant water management (to control heat
ual instruments. and clear areas difficult to access or view be-
3. Fluid lavage: fluid associated with power-driven cause of debris) and evacuation required in
scalers reaches base of the pocket. most cases.
a. Serves as oral irrigator, flushing away the un- b. Aerosol if produced carries microorganisms
attached dental biofilm, fractured calculus, and hemorrhage products; may be health risk to
hemorrhage products (controversial endotoxin certain patients.
detoxicification of root surfaces). c. Access is NOT always achievable, e.g., contact o10120
b. Cavitation: fine mist around the tip of power- areas of mandibular anteriors.
driven scaler, which consists of small bubbles. 3. Relative contraindications:
(1) From the bubbles bursting to produce the a. Respiratory and immunocompromised risk
mist. ­patients:
(2) When occurs near microorganisms, causes (1) Oral microorganisms will be incorporated
small shock wave that can disrupt or lyse into aerosol if produced and will be inhaled
bacterial cell walls. by patient.
Instrumentation 411

(2) Potential for development of respiratory in- (2) Large pulp chambers of primary teeth may
fection exists in susceptible patients. be MORE heat sensitive.
b. Restorations: g. B EST to discontinue use if patient reports
(1) NOT for use on porcelain or composite res- discomfort; substitute manual instrumen-
torations and implants unless implant tips tation; have deaf patient turn off hearing
are used (either fixed or with disposable aids.
plastic covers). 4. NO contraindication for cardiac pacemakers; older
(2) May be used on adjacent tooth structure, ones that were unshielded presented problem, but
but all margins of restorations SHOULD be now ALL shielded; if any questions, obtain medi-
avoided unless purpose is to remove amal- cal consult.
gam overhang. 5. Patient and insert preparation:
c. NOT for use on demineralized areas: may re- a. Provide information to patient about the pro-
move or damage weakened tooth structure. cedure; drape patient to protect from aerosol
d. MORE uncomfortable for patients with pronounced spray.
gag reflex and obligatory mouth breathers. b. Follow standard precautions; use preprocedural
e. Vibrations may be uncomfortable on sensitive antimicrobial rinse.
roots; may remove smear layer over dentinal c. Lubricate O-ring (if present) with water; seat o10130
tubules, creating dentinal hypersensitivity. tip or insert into handpiece with gentle push-
f. Children: twist motion.
(1) Newly erupted teeth MORE susceptible to d. Adjust power for effective debris removal, tip
damage from vibrations. selected, and patient comfort.

clinical study  

Age 44 YRS SCENARIO

Sex ☒  Male   ☐  Female Patient presents with generalized pocket depths of


4 to 5 mm and slight vertical bone loss. There is a
Height 5’10” large band of heavy stained calculus on the lingual
of his mandibular anterior teeth. Radiographs reveal
Weight 235 LBS
amalgam overhangs on #3 and #14. The dental
BP 120/82 hygienist is debating whether to use an ultrasonic
scaler on him but then decides to use it, at least for
Chief Complaint “I am sorry I am bleeding so the initial debridement of the scheduled quadrant
much.” of nonsurgical periodontal therapy. The patient also
told the dental hygienist that he does not have a lot
Medical History Cardiovascular disease
of time to sit in the dental chair today as he is due in
Prehypertension
court. When tooth #7 was instrumented, the patient
Current Medications Low-dose aspirin 86 mg qd felt very sensitive.

Social History Superior court judge


Son works as a barista in a popu-
lar coffee stand, so he gets free
espresso

1. Was the ultrasonic the instrument of choice for scaling less traumatic force and flush the area as it is used. It
the patient’s teeth? What tips should be used? is also quicker than manual instruments and effective
2. What type of stain does this patient have? for removing overhangs and heavy deposits of calcu-
3. Is the ultrasonic scaler contraindicated because of lus and stain, reducing time needed for polishing the
medical history? Why is he bleeding? Why is he tak- stain. Using the piezoelectric with high-speed suction
ing aspirin? is best; it is the fastest of the power-driven at 25,000 to
4. What should the dental hygienist do if the patient has 50,000 cps. Universal or standard tips, best for higher
sensitivity when the ultrasonic is used? settings, would be best for heavy deposits. Hygienist
must remember that two sides of the tip are active with
1. The instrument of choice for this patient was the ultra- this ultrasonic. Should be followed up with manual
sonic scaler, since it will keep bleeding down with its scaling, if needed.
412 Saunders Review of Dental Hygiene

2. This patient has exogenous extrinsic stain from drink- b. Repeated polishings can remove cementum, ex-
ing black coffee; can become intrinsic into the dentin posed dentin, and even enamel, especially inter-
with wearing of tooth enamel such as with attrition or proximally, which can change dynamics of oral
with exposure of root surface. clearance.
3. The patient’s medical history does not present a con- c. Decalcified areas (white and/or brown spot le-
traindication for the use of the ultrasonic scaler, even sions) are more rapidly abraded than intact teeth
with presence of cardiovascular disease (CVD). Cer- and should be NOT be polished with abrasive
tain older pacemakers that are unshielded were a con- pastes.
traindication, but now all pacemakers are shielded and d. Outermost layer of the tooth is the MOST
do not present a problem. He is bleeding because he is ­fluoride-rich layer; if part of this layer is re-

on antiplatelet (low-dose [LD] aspirin) therapy for his moved, SHOULD follow up with topical fluo-
cardiac history, thus reducing risk of thrombosis and ride treatment.
embolism formation. American Dental Association has e. Recently erupted teeth are less mineralized,
stated that drug therapy rarely needs to be discontin- making structure MORE easily removed.
ued before most dental procedures because of adverse 2. Increased roughness to promote dental biofilm
reactions; there is a greater risk of thromboembolic ­reattachment:
events than uncontrollable bleeding if temporarily a. Use of coarse agent and/or excessive pressure
stopped. These events could increase risk that an emer- or use of brush attachment can cause grooving
gency situation can occur with patient care. or roughening of tooth surface; especially true
4. If the patient is sensitive on one tooth, the hygienist of the root, but even enamel can be affected.
should continue scaling unless he experiences sensi- b. Some restorative materials, such as gold, com-
tivity elsewhere. If this is the case, manual instruments posite, or esthetic restorations, can be scratched
should be used instead. or dulled or even suffer loss of material with
rubber-cup or brush; air-powder abrasives have
SELECTIVE POLISHING   been shown to damage ALL restorative mate-
p0120 Selective polishing is polishing ONLY those surfaces of the rials, especially composites and other esthetic
teeth that have extrinsic stain that has NOT been removed materials.
by ultrasonic or manual instrumentation when esthetics is 3. Polishing can cause frictional heat:
valued by patient. Currently MOST accepted approach to a. Teeth with large pulp chambers, such as pri-
polishing. Important to AVOID iatrogenic damage, which mary teeth, are susceptible to heat.
is especially important for root surfaces and restorations. b. Metal restorations can transmit heat to the
This can BEST be accomplished by applying fine grit pulp.
(particle-size) agent with cup attached to prophy angle c. Frictional heat can be reduced by:
on slow-speed handpiece and/or with air-driven polishing (1) Using light pressure against the tooth.
system. Polishing should NOT be considered routine part (2) Rotating cup or brush at slower speed.
of the oral prophylaxis, according to the “Position Paper (3) Decreasing duration of time of contact be-
on Polishing” by ADHA (see CD-ROM). Many clinicians tween the tooth and cup or brush.
are considering polishing first before other instrumenta- (4) Using agent with sufficient wetness to slide
tion to remove loose debris and staining so as to provide a over the tooth surface.
clearer field of vision while working. 4. Contamination from microorganisms:
• See Chapters 6, General and Oral Pathology: intrinsic a. Bacteremia (bacteria in blood) may lead to gin-
stain development; 11, Clinical Treatment: extrinsic gival inflammation.
stain evaluation. b. Aerosol containing microorganisms is created
A. Purposes: by ALL rotary instruments, especially by air-
1. Improve esthetics by removal of extrinsic stain powered abrasive system.
(main purpose; of questionable therapeutic value). c. To reduce contamination:
2. Prepare tooth to receive sealants (NOT used before (1) Use of safety glasses by patient and clini-
all sealants). cian; preprocedural antimicrobial mouth­­
3. Meet patient’s expectations and increase patient rinse.
satisfaction. (2) Complete coverage of clinical surfaces;
B. Relative contraindication: posttreatment surface decontamination.
1. Removal of tooth structure that CANNOT be re- 5. Effect of the abrasive agent: abrasive particles can be
placed: forced into the pocket or even into the tissue itself.
a. Ditching can occur when pressure on the cup is a. When treating patients who have deep pockets
applied unevenly or excessively. or need extensive instrumentation, polishing
Instrumentation 413

precedes scaling or takes place later at reevalu- the least abrasive, and chemical and
ation appointment to prevent agent from being physical properties of the abrasive be-
introduced into the tissue and causing further ing used (e.g., hardness, particle shape,
inflammation. fracture, wear characteristics).
b. Few individuals have negative reaction either to (b) Pumice and silicone dioxide are COM-
the fluoride in commercial pastes or to abrasive MON agents.
particles or other components of agent. (c) Agent must always be used in a moist
C. Polishing methods (older method was by porte pol- paste form, NOT dry.
isher, a wooden stick held in metal handle that was (d) Some rubber cups are already impreg- o10140
rubbed on tooth with agent; may be able to pick one nated with silica pumice granules.
up on eBay or see on “Antiques Roadshow”!): d. Technique for rubber cup or brush:
1. Rubber cup or brush is attached to prophy angle on (1) Use LEAST abrasive agent that will re-
slow-speed handpiece. move stain; do NOT use anything coarser
a. Advantage: BEST method of working agent than medium grit; fine grit is BEST (plus-
over the tooth surface. type is very coarse).
b. Disadvantages (see earlier discussion): (2) Use slowly rotating cup to apply the agent;
(1) Greater speed can cause excessive abrasion use light pressure; do NOT allow the cup to
to tooth surface, resulting in loss of tooth remain on the same area of the tooth.
structure and ditching if the force on cup is (3) Keep the cup parallel to the tooth surface
applied unevenly. to avoid grooving the surface, especially at
(2) Frictional heat can be generated and can the cervical area.
cause discomfort to patient. (4) Keep the duration of contact between tooth
(3) Care must be taken NOT to traumatize gin- and cup to a minimum; flex the cup to adapt
gival tissues or pulp. the cup edges into the interproximals.
c. Equipment: (5) Rinse (irrigate) and suction (high-speed,
(1) Slow-speed handpiece provides rotational high-volume evacuator [HVE] BEST) the
energy to power movement of angle. patient frequently.
(2) Prophy angle: (6) Adapt brush bristles to work the polish-
(a) Right-angled or contra-angled attach- ing agent into the grooves of the occlusal
ment that attaches to the handpiece surfaces.
and holds agent, usually a rubber cup, (7) Use dental floss and polishing agent to
­bristle brush, or rubber polishing point. clean interproximals (controversial).
(b) Available as either disposable product 2. Air-powder polish (air-abrasive polish, air polish- o5450
or reusable product that can be steril- ing [AP]): system that uses special air-powered
ized between uses. machine to deliver slurry that contains abrasive
(3) Polishing device moves agent over the sodium bicarbonate powder, water, and air; pow-
tooth surface. der products of aluminum trihydroxide or calcium
(a) Rubber cup has hollow center to hold carbonate are also available, as well as glycine
agent and thin edges that flare to adapt powder air polishing (GPAP) with LESS abrasive
to various smooth tooth surfaces; flexes qualities.
slightly to conform interproximally. a. Advantages:
(b) Bristle brush and agent are used for oc- (1) BEST for removing extrinsic stain quickly
clusal surfaces and concave areas of the and effectively.
tooth; must NOT be used near the gin- (2) Places LESS stress on clinician’s hand and
giva (controversial). wrist (less pressure and manipulation) than
(c) Rubber polishing points can apply rubber cup or porte polishing.
polishing abrasive to interproximal (3) Possible LESS damage to tooth surface
surfaces if there is adequate space than is caused by manual instruments or
(controversial). rubber-cup polish.
(d) Dental floss or tape can work the (4) BETTER than rubber-cup polish to prepare
abrasive over interproximal surfaces occlusal surfaces for some types of seal-
(controversial). ants.
(4) Abrasive agent: (5) May use GPAP within periodontal pockets o10150
(a) Abrasive quality is controlled by size of up to 4 mm to remove subgingival biofilm
abrasive particle, with smallest particles during PM.
414 Saunders Review of Dental Hygiene

b. Disadvantages: (4) Spray is angled away from gingival


o5520 (1) Causes transient irritation to gingiva NOT margin.
lasting more than few days (not dissimilar (5) Polishing for 5 seconds or less per tooth is
to rubber cup polishing if accidentally con- usually adequate for stain removal.
tacting soft tissue) if using more abrasive 3. Absolute contraindication:
powders. a. Because of possible systemic absorption of so-
(2) Produces aerosol that contains BOTH so- dium bicarbonate powder (if used) for patients
dium bicarbonate (if used) or other powder with a medical history of:
ingredients and oral microorganisms with (1) Hypertension (high blood pressure [HBP]),
potential to be inhaled by both patient and need for low-sodium diet, renal insuffi-
clinician and spread to clinical surfaces (up ciency.
to 6 feet). (2) Addison’s disease, Cushing’s syndrome,
(3) See earlier discussion on contraindications metabolic alkalosis.
and precautions for list of specific medical (3) Taking mineralocorticoid steroids, antidi-
conditions. uretics, or potassium supplements.
o5550 (4) More abrasive powders have been shown to b. Because of inhalation of abrasive particles (if
alter or damage restorative materials, espe- used) and/or microbially contaminated aerosol
cially composites and porcelain; may want for a patient with a history of:
to use less abrasive powders (need to check (1) Respiratory and infectious diseases.
with manufacturers). (2) Immunological diseases and immunosup-
o10160 (5) Still does NOT remove calculus; need pression therapy.
other forms of hard deposit removal; many c. Because of damage inflicted by abrasives (if o5760
use AP, especially GPAP, at start of PM used), NOT on unhealthy periodontium and/
to ­ provide a clearer field of vision while or restorative materials, especially composites
­working. and other esthetic materials; abrasion may be
c. Equipment (Prophy-jet, Cavijet-combined Cavi­ reduced by use of LESS abrasive powders.
tron):
(1) Uses air pressure from dental unit to pro- Instrument Processing
pel slurry of water, sodium bicarbonate, air Care MUST be taken during instrument processing to p0130
through small nozzlelike opening. ensure that reusable instruments are safely cleaned and
(2) Disposable aerosol reduction device is at- sterilized. instruments MUST be exposed to all three pa-
tached to evacuation system at one end and rameters of sterilization: time, temperature, and presence
fits over the nozzle of device at the other of steam, as well as weekly spore testing to ensure effec-
end; adapted to the tooth surface and cap- tive sterilization.
tures aerosol so it can be suctioned away • See Chapter 8, Microbiology and Immunology: infec-
(BEST to use high-speed suction, high- tion control procedures, standard precautions, surface
­volume evacuator [HVE]). disinfection, sharps, and waste handling.
d. Technique: BOTH patient and clinician wear A. Cleaning instruments:
protective eyewear and drapes or garments 1. Manual cleaning (hand scrubbing): BEST for in-
(personal protective equipment [PPE]). struments that would be damaged if exposed to
(1) Patients SHOULD remove contact lenses ultrasonic cleaner (i.e., some handpieces); with
and protect lips with application of lubri- multiple-use utility (heavy rubber or neoprene)
cant. and personal protective equipment (PPE).
(2) CORRECT tip angulation decreases aerosol 2. Ultrasonic cleaner (bath) should MAINLY be
creation; incorrect angulation is MOST com- used, safer than manual cleaning, and instru-
mon cause of excess aerosol production: ments SHOULD be handled with multiple-use
(a) For anteriors, tip is directed at 60° to- utility (heavy rubber or neoprene) gloves and
ward facial and lingual surfaces. PPE.
(b) For posteriors, tip is directed at 80° to- a. Sonicating for time recommended by manufac-
ward facial and lingual surfaces. turer (1 to 10 minutes); cassettes take longer
(c) For occlusal surfaces, tip is directed at (15 minutes).
90° to occlusal plane. b. Acts MAINLY by physical agitation (cavita-
(3) Rapid, sweeping strokes are used with tion with tiny bubbles forming and collapsing,
the tip held about 4 to 5 mm from tooth SAME as ultrasonic scaler) and chemical action
­surface. (solution helps dissolve organic residue).
Instrumentation 415

c. Overloading SHOULD be prevented; solution d. Quick turnover steam (flash) sterilization (e.g.,
MUST contact all surfaces. Statim, Kwikclave sterilizer):
d. Instruments with detachable parts SHOULD (1) Uses GREATER pressure and heat than tra-
be disassembled; hinged instruments MUST be ditional steam sterilizers.
opened. (2) Sterilizes unwrapped instruments in 3 to 12
3. Instruments SHOULD be rinsed and allowed to air minutes; should be used ONLY for instru-
dry (if towel drying, carefully pat dry). ments that are to be used promptly on re-
o5850 B. Packaging instruments: using bags or wrapped cas- moval.
settes, BOTH sealed with indicator tape (chemical 3. Chemical vapor (chemiclave, MDT): uses com-
monitoring) designed for type of sterilization pro- bination of chemicals (alcohol, formaldehyde,
cess, steam or heat, being used; improper packaging ketone, acetone, water) instead of water to create
can melt, char, or even prevent sterilizing agent from vapor for sterilizing in 20 minutes.
reaching instrument pack contents, or may release un- a. Gas vapors MUST penetrate instrument
wanted chemicals into sterilizer chamber. packaging; requires purchasing manufactur-
C. Sterilizing instruments (Table 12-2): er’s chemical solution; increases sterilization
1. Instruments SHOULD be sterilized if they can be costs.
(includes new instruments); otherwise disposable b. Advantages: BEST for corrosion- and rust-free
SHOULD be used; weekly spore testing (biologi- operation and keeps instruments sharp; ease of
cal monitoring) SHOULD be done as indicated, operation; quick drying of instruments.
ONLY way to determine whether sterilization has c. Disadvantages: use of hazardous chemicals;
occurred (see discussion next). MUST have adequate ventilation because of
2. Moist heat (steam under pressure) using autoclave: chemicals.
at 15 lb/in2, generates temperature of 121° C at 15 4. Dry heat (oxidation) (dryclave): electric dry air
to 20 minutes. oven requires materials to be exposed to a tem-
a. Requires that instruments have space around perature of 180° C for 1 to 2 hours.
them to enable steam to reach ALL surfaces and a. Used for instruments that CANNOT be steril-
tight seal against the outside to provide neces- ized safely by steam autoclave and is BEST
sary pressure and temperature. for oils and powders if stable at temperatures
b. Advantages: any item can be sterilized, depend- reached during sterilization and for sharp in-
ing on nature of materials and load. struments such as for surgery.
c. Disadvantages: inability of some instruments b. Depends on rapid heat transfer from circulating
to withstand the heat (plastics melt); possibil- air to instruments (similar to cooking oven).
ity of corroding and pitting high-carbon steel c. Advantages: maintenance of sharp edges on
instruments, slightly dulling sharp instruments; instruments, noncorrosiveness, and usefulness
inability to penetrate oils or powders; distilled on materials unable to withstand steam under
water SHOULD be used instead of tap water, ­pressure.
which often contains minerals and impurities; d. Disadvantages: LESS efficient than moist
this can minimize corrosion and pitting. heat, requires longer exposure and higher

Table 12-2  Methods of sterilization and related spore testing agents

Weekly spore
Method Time Temperature Pressure (psi) testing

Moist heat (steam 15-20 min 250° F 15 Bacillus stearo-


­autoclave): standard 121° C thermophilus
Moist heat (steam auto- 3-12 min 270° F 30
clave): quick 130° C
Chemical vapor (chemi- 20 min 270° F 20-40 Bacillus stearo-
clave) 130° C thermophilus
Dry heat (oven): ­standard 1-2 hr 320° F — Bacillus subtilis
160° C
Dry heat (oven): quick 8-12 min 375° F —
190° C
Ethylene oxide 10-16 hr to 24-48 hr Room temperature — Bacillus subtilis
416 Saunders Review of Dental Hygiene

temperatures, careful loading to ensure steril- e. MUST be maintained and sterilized according
ization, inability of unwrapped instruments to to the manufacturer’s instructions to maximize
stay sterile for long. the life of the handpiece.
e. Quick sterilization using dry heat (e.g., Cox): 2. Prophy angles, contra-angles, air-water syringes:
(1) Maintains internal heat at 375° F. a. MUST be sterilized to prevent transmission of
(2) Sterilizes wrapped instruments in 12 min- infectious agents from patient to patient.
utes, dental handpieces in 8 minutes. b. SHOULD be kept in sufficient supply; treatment
(3) Continuously ready; requires NO warm-up areas SHOULD contain several replacement
time. pieces to facilitate sterilization of contaminated
5. Ethylene oxide (toxic gas): major gaseous steril- handpieces and air-water syringes.
ant for sterilizing nonbiological materials at room c. MUST be maintained and sterilized according
temperature, metal 10 to 16 hours, nonmetal 24 to to manufacturer’s instructions to maximize life
48 hours. of the equipment.
a. Primarily in hospitals and larger clinics; is NOT d. Consideration SHOULD be given to use of dis-
used in dental offices. posables:
b. Advantages: effectiveness without damaging (1) Reusable prophy angles:
MOST instruments, use of low temperatures for (a) MUST be taken apart before sterilization.
effective sterilization, and ability to penetrate (b) Lubricated according to manufacturer’s
different wrapping materials. recommendations, before and after ster-
c. Disadvantages: longer processing times, high ilization.
cost of equipment, use of hazardous chemicals. (2) Reusable air-water syringes:
6. Chemical sterilants and disinfectants (cold steril- (a) MUST be thoroughly flushed before
ization): sterilization.
a. MUST be approved by Environmental Protec- (b) Some types are autoclavable (BOTH
tion Agency (EPA) as a high-level disinfectant syringe and tips).
to be used for semicritical items. e. May have interchangeable tips that can be
b. Reserved ONLY for items (noncritical, semicrit- sterilized.
ical) that CANNOT withstand heat sterilization; 3. Ultrasonic tips: when sterilized in dryclave and
are NOT a substitute for sterilization by steam, NOT with steam, the rubber O-ring (if present)
dry heat, chemical vapor, or ethylene oxide. dries out, becomes ineffective, and needs replac-
c. To achieve high-level disinfection, items MUST ing or leakage will occur.
be immersed for long periods, 6 to 10 hours; E. Storing instruments:
may be used ONLY for 28 to 30 days. 1. Instruments SHOULD be cooled before being
d. LEAST used because of inability to verify ster- stored.
ilization of instruments. 2. Instruments SHOULD be stored in coverings
e. CANNOT be used on packaged instruments; thus such as bags or wrapped cassettes to preserve
maintaining “sterile” state is difficult; after im- sterility for future procedures and NOT free and
mersion, instruments MUST be rinsed with ster- uncovered.
ile water, dried, and stored in sterile ­containers. 3. Instruments MUST be stored in a clean, dry drawer
f. Contain chemicals that are toxic to patients and or shelf, with door to promote instrument sterility.
dental healthcare personnel (DHCP). F. Contamination events: moisture exposure, holes in o10170
D. Specific dental equipment processing: note that spe- packaging, even dust on pouch may contaminate in-
cific areas of instrument processing area need to be struments when pouch is opened; any event requires
specifically laid out to ensure effective processing repackaging and resterilizing instruments before use.
such as demarcating contaminated and sterile areas.
1. Handpieces: Biological Monitoring of Sterilization Process
a. MUST be appropriately sterilized for patient Weekly biological monitoring of the sterilization process
protection. MUST be performed to ensure that the process is effec-
b. Some handpieces can be heat sterilized; this tive. The CDC requires dental offices to verify proper
SHOULD be a priority when purchasing. functioning of sterilization cycles on weekly basis, using
c. Handpieces that CANNOT be adequately steril- biological indicators.
ized require a compromise in the office infec- A. Indicator types:
tion control plan. 1. Chemical (heat-sensitive) indicators verify ONLY
d. May be damaged by disinfectants used in the that sterilizing conditions of correct temperature
dental office (i.e., fiberoptic handpieces). have been reached, NOT that sterilization has been
Instrumentation 417

achieved; used with indicator tapes on paper steril- 4. What could be the cause for the O-ring of the ultra-
ization bags or wrapped cassettes when packaging sonic showing wear?
instruments (indicated by lines turning black). 5. Her ethical principle of wanting sterile instruments for
2. Biological indicators containing specific organ- her patients demonstrates what type of ethical principle?
isms are the ONLY way to verify that sterilization
of the chamber contents has been achieved when 1. Bag color change (black stripes) indicates only that
spores contained in the biological indicator are de- instruments were exposed to heat such as being laid on
stroyed; used because they can withstand chemical top of the autoclave; it is a type of chemical indicator
and mechanical forces but NOT sterilization. only (like instrument tape) and not a biological indica-
a. Bacillus stearothermophilus spores: autoclave tor (such as a spore indicator) that indicates steriliza-
and chemiclave sterilizers. tion. Instrument processing areas should be carefully
b. Bacillus subtilis spores: dry heat and ethylene laid out, and there should be specified areas where
oxide sterilizers. contaminated instruments are placed to ensure proper
B. Spore testing (using biological indicators discussed handling. Instruments should be stored in coverings
above): such as bags and wrapped cassettes to preserve steril-
1. Indicated once a week or during the training of ity for future procedures and should not be kept free
new instrument processing staff. and uncovered.
2. Whenever new packaging material is tried, when 2. She should keep them in the cold sterile (chemical
new sterilizer is used, after a sterilizer has been re- disinfectant) for at least 6 to 10 hours to achieve disin-
paired. fection, depending on the manufacturer’s instructions,
3. When different method of loading the sterilizer is or ask if they can be disposed of; such methods can-
employed. not be used on critical instruments. Since the tips are
o10180 4. Vary placement of spore test in sterilizer chamber disposable, that means that they are plastic and can-
to confirm that sterilization occurs in all areas. not be put into the autoclave or dryclave because they
C. Records of the process and results of biological will melt.
monitoring MUST be kept to comply with federal 3. She should be sure to use ultrasonic cleaner for all in-
regulations and to provide documentation in case of a struments for 1 to 10 minutes (or 15 minutes for cas-
liability suit. settes). This will ensure that debris (such as blood) is
loosened up and will not dry on instruments when they
clinical study   are put in autoclave or dryclave, preventing the instru-
Scenario: A dental hygienist is working as a temporary em- ment from being fully sterilized.
ployee through an agency. When she goes to get instruments 4. Appears that these were sterilized in dryclave, not
out of the drawer, she finds that the autoclave bag markings one with steam, thus drying out the O-rings; if this
have turned color but some instruments have been stored ­continues, the instrument tips will be ineffective and
without coverings. She goes to use the instruments and no- need replacement of the rubber O-rings or leakage will
tices that there are dried-on blood deposits on some of the occur during use.
tips. She also notices that the disposable air-water syringe 5. She is demonstrating nonmaleficence; this is the belief
tips are in the cold solution but are being used frequently all that the action is wrong if harm is inflicted on others.
day long. All this disturbs her. When she goes to sterilize
her instruments in the autoclave, the other dental hygien-
ist tells her to put them on top of the autoclave, next to the Review Questions
dryclave, and they will be taken care of later. The dental hy-
gienist has a problem with this but follows instructions. She
also notices that the O-rings on the Cavitron tips are wear- 1 Correct clinician and patient positioning should have all of
ing out and there is leakage when she uses it. Update: she is the following advantages for the clinician, EXCEPT one.
very disappointed in this office, will not work there again, Which one is the EXCEPTION?
and will let the temporary agency know of her concerns. A. Reduces the chance of upper body pain and/or injury for
the clinician
B. Reduces the number of instruments needed to complete
1. Is the fact that the autoclave bag has changed color
a full-mouth debridement
proof of proper autoclaving? Why does it bother her C. Improves visibility of the treatment area
that the instruments are stored without coverings? D. Reduces the possibility that the patient might faint dur-
2. What should she do with the air-water tips to ensure ing treatment
that they are in fact sterile?
3. What should she do to ensure that there is no dried
blood left on instruments that she uses?
418 Saunders Review of Dental Hygiene

2 A well-positioned patient should present with which of the 8 Mirrors are used for many purposes, including soft tissue
following situations? retraction, indirect vision, indirect illumination, and transil-
A. Top of the head at the top of the dental chair lumination. Transillumination is the illumination of a dark
B. Feet higher than head and heart area of the mouth by bouncing light off the mirror face.
C. Head turned to the right for a right-handed clinician A. Both statements are true.
D. Chair back at about a 45° angle, if the patient has a B. Both statements are false.
strong gag reflex C. The first statement is true, the second is false.
3 When the maxillary arch is treated, the patient is typically D. The first statement is false, the second is true.
asked to tilt the chin downward (or chair back is slightly 9 The cross-sectional shape of the working end of various
raised) to improve access and visibility. When the man- instruments helps define the category and function of the
dibular arch is treated, the patient is positioned with the instrument. All are correctly matched between instrument
maxilla perpendicular to the floor for maximum access and type and cross-sectional shape, EXCEPT one. Which one is
visibility. the EXCEPTION?
A. Both statements are true. A. Explorer—circle
B. Both statements are false. B. Universal curet—half moon
C. The first statement is true, the second is false. C. Sickle—triangle
D. The first statement is false, the second is true. D. Area-specific curet—ellipse
4 All of the following are true of the principles of correct 10 The primary instruments for assessment of the periodontal
clinician positioning, EXCEPT one. Which one is the status and dental hygiene treatment needs of a patient are
­EXCEPTION? various explorers and probes. A Nabor’s probe is designed
A. Both feet are kept on the floor except when seated at specifically for identifying and determining the characteris-
7 or 8 o’clock position, and the inside leg is crossed over tics of furcations.
the outside leg to allow seating closer to the patient for A. Both statements are true.
improved access. B. Both statements are false.
B. The feet and legs are kept apart to create a more stable C. The first statement is true, the second is false.
seated position on the stool. D. The first statement is false, the second is true.
C. The stool height is adjusted so that the clinician’s thighs 11 Which of the following would be the BEST choice for re-
are parallel to the floor. moval of calculus around the contacts and adjacent inter-
D. The body is seated to the back of the stool so that the proximal areas of the mandibular anterior teeth?
spine is supported by the back rest. A. Sickle scaler
5 The incidence of occupational injury can be reduced by the B. Universal curet
use of good body mechanics. Which one of the following is C. Area-specific curet
NOT good use of body mechanics? D. Chisel
A. Keeping the shoulders relaxed and even 12 Universal curets and area-specific curets share a number of
B. Keeping the upper arms near the body and the forearms design features, EXCEPT one. Which of the ones below is
parallel to the floor the EXCEPTION?
C. Tilting forward and hyperextending the neck to improve A. Rounded toe and rounded back
visibility B. Two cutting edges per working end
D. Keeping the patient’s mouth at a comfortable focal dis- C. Simple shanks for anterior teeth and complex shanks for
tance of about 15 inches from the clinician’s eye posterior teeth
6 Curet Q has a shank with complex angles and is designed D. Paired, mirror-imaged working ends
for use in periodontal pockets. Curet Q is excellent for re- 13 Which of the following area-specific curets was designed to
moval of light deposits. Which design characteristic would treat the mesial surfaces of posterior teeth?
need to be changed for instrument Q to be effective in re- A. Gracey 5-6
moving moderate deposits? B. Gracey 11-12
A. Thickness of the shank C. Gracey 13-14
B. Length of the lower (terminal) shank D. Gracey 17-18
C. Toe of the blade 14 Instruments designed for the debridement of implants
D. Length of the blade ­usually
E. Back of the blade A. should be made of plastic.
7 A well-balanced instrument MUST be designed with which B. are standard metal instruments.
of the following? C. have unique shapes.
A. Two cutting edges D. cause light scratches on the surface.
B. Two mirror-imaged working ends, one on each end of
the handle
C. Straight or simple shank
D. Middle of the working end centered on the long axis of
the instrument handle
Instrumentation 419

15 All of the following are true of the modified pen grasp 22 In deep, narrower pockets and hard to reach subgingival ar-
EXCEPT one. Which one is the EXCEPTION? eas, curets are the instrument of choice. Because ultrasonic
A. Thumb and middle finger are placed opposite each other tips do NOT have a specific cutting edge, there is no need
on the handle. to achieve a specific face to tooth working angle as with a
B. Handle rests between the thumb and the second joint of curet or sickle scaler.
the index finger. A. Both statements are true.
C. Ring finger rests against the middle finger and is used as B. Both statements are false.
the fulcrum finger. C. First statement is true, the second is false.
D. Pad of the middle finger rests on the instrument. D. First statement is false, the second is true.
16 All of the following are true of fulcrums, EXCEPT one. 23 While NOT essential for successful treatment, both a stan-
Which one is the EXCEPTION? dard pen grasp and a soft tissue fulcrum are recommended
A. Effective use of a fulcrum can increase control over the for use with ultrasonic instrumentation because they
working stroke. A. foster a lighter working stroke.
B. Ring finger is also called the fulcrum finger. B. improve penetration of fluid to the base of the pocket.
C. Extraoral fulcrum should be used whenever possible. C. are gentler to the soft tissue.
D. Fulcrum is a mechanical leverage. D. prevent clinician fatigue.
17 The optimum angle of the instrument face to the tooth sur- 24 Why is it critical that the point of the ultrasonic tip never be
face for scaling is between 45° and 60°. The optimum angle adapted to the root surface?
for root planing is 70° to 80°. A. Can gouge and permanently damage the root surface
A. Both statements are true. B. Generates too much heat
B. Both statements are false. C. Not effective in deposit removal
C. First statement is true, the second is false. D. Uncomfortable for the clinician
D. First statement is false, the second is true. 25 What is the MAIN reason instrumentation with an ultra-
18 What is the range of movement at the tip of the working end sonic scaler takes less time than manual instrumentation?
for MOST sonic scalers? A. Fewer strokes are needed.
A. 2500 to 7000 cycles/sec B. Clinician tires less easily.
B. 18,000 to 45,000 cycles/sec C. Less pressure is needed.
C. 35,000 to 50,000 cycles/sec D. Strokes are performed more quickly.
D. 18,000 to 50,000 cycles/sec 26 All of the following may be considered a contraindication
19 The two types of machines for ultrasonic scaling are the to the use of an ultrasonic scaler, EXCEPT one. Which one
magnetostrictive and piezoelectric. Magnetostrictive ultra- is the EXCEPTION?
sonics convert electrical power to tip movement through A. Decalcified tooth surface
magnetic oscillations and have a generally elliptical pattern B. Obligatory mouth breather
of tip movement, whereas piezoelectric units use a crystal to C. Shielded pacemaker
convert electricity to movement and generally have a linear D. Newly erupted teeth
pattern of tip movement. 27 During ultrasonic scaling, the clinician should wear
A. Both statements are true. A. eye protection and a facial mask.
B. Both statements are false. B. eye protection and a facial shield.
C. First statement is true, the second is false. C. a facial shield and a facial mask.
D. First statement is false, the second is true. D. a facial shield, eye protection, and a facial mask.
20 Slim- or thin-style ultrasonic tips are the BEST choice for 28 When the instrument is used with reasonable skill and care,
which kinds of deposits? which of the following is generally considered acceptable to
A. Heavy and light calculus treat with an ultrasonic scaler?
B. Heavy calculus A. Amalgam
C. Light calculus B. Composite
D. All deposits C. Porcelain-fused-to-metal crown
21 When beginning ultrasonic instrumentation on a tooth with D. Esthetic veneer
both supragingival and subgingival calculus, one would 29 Which of the following is the BEST rationale for polish-
typically begin on the more apical calculus and end with the ing?
more coronal. Subgingival calculus removal is at least as A. Brightening the patient’s teeth and smile
effective with ultrasonic instruments as with manual instru- B. Removing extrinsic stain
ments if one is thorough with both. C. Removing calculus accumulations
A. Both statements are true. D. Giving the patient’s mouth a clean, fresh feeling
B. Both statements are false.
C. First statement is true, the second is false.
D. First statement is false, the second is true.
420 Saunders Review of Dental Hygiene

30 As a regular part of oral prophylaxis, selective polishing 37 Which of the following BEST indicates that an instrument
has replaced full-mouth polishing for all of the following is dull?
reasons, EXCEPT one. Which one is the EXCEPTION? A. The instrument grabs at the root surface.
A. Repeated polishing can remove cementum, dentin, and B. Light does not reflect off the cutting edge.
even enamel. C. Over 10 working strokes have been taken.
B. Dental biofilm can be removed by other, less damaging D. Calculus is being burnished.
means. 38 Which of the following is CORRECT statement describing
C. Fluoride-rich layer is removed. an Arkansas stone?
D. Treatment time is reduced. A. Natural stone that should be lubricated with oil before
31 Which of the following is a potential problem caused by use and can only be chemically sterilized
polishing? B. Synthetic stone that does not need to be lubricated be-
A. Creation of a bacteremia as a result of abrasive particles fore use and can be sterilized by any means
being absorbed systemically C. Natural stone that should be lubricated with oil before
B. Allergy to ingredients of commercial polishing pastes, use and can be sterilized by any means
which is more common than is generally believed D. Synthetic stone that should be lubricated with water be-
C. Creation of a microbial-laden aerosol that is inhaled by fore use and can be sterilized by any means
the patient and spread to surrounding surfaces 39 W hich of the following is the MOST abrasive sharpening
D. Vaporization of mercury during amalgam polishing pro- stone?
cedures, even when correct technique is used A. Ceramic stone
32 In a rubber-cup polishing system, the slow-speed handpiece B. Arkansas stone
causes the shaft to rotate in the prophy angle, which causes C. India stone
the rubber cup to turn and move the polishing abrasive over D. All have similar abrasive grit
the tooth surface. Regardless of the polishing system used, 40 Which one of the following is NOT a general principle of
it is the abrasive agent that produces the polishing action. instrument sharpening?
A. Both statements are true. A. The angle between the instrument face and the stone is
B. Both statements are false. about 110°.
C. First statement is true, the second is false. B. The original shape of the instrument should be pre-
D. First statement is false, the second is true. served.
33 In an air-powder polishing system, polishing is produced by C. Metal is removed equally from the lateral surface and
a mixture containing abrasive particles to clean the tooth, the face.
water to prevent heating and to dilute the abrasive, and air D. The internal angle of the cutting blade of a curet or a
to provide force to deliver the abrasive to the tooth surface. sickle scaler MUST be maintained at about 70°.
This system causes more damage to composites and to tooth 41 A wire edge is a projection that forms as metal particles are
surfaces than other methods of stain removal. burnished to form an extension beyond the surface being
A. Both statements are true. sharpened. It is important to prevent the formation of a wire
B. Both statements are false. edge because it is difficult to remove after it is created.
C. First statement is true, the second is false. A. Both statements are true.
D. First statement is false, the second is true. B. Both statements are false.
34 To make air-powder polishing MORE effective and com- C. First statement is true, the second is false.
fortable, the angle of the tip to the tooth is important. What D. First statement is false, the second is true.
are the recommended angles? 42 When the lateral sides of a sickle are sharpened with an
A. Anterior teeth 40° and posterior teeth 60° instrument-stable technique,
B. Anterior teeth 60° and posterior teeth 80° A. the face of the instrument is positioned so that it is paral-
C. Anterior teeth 80° and posterior teeth 40° lel with the floor or countertop.
D. Anterior teeth 45° and posterior teeth 15° B. a face-to-stone angle of about 70° is established.
35 All of the following are true of sharpened instruments, C. the stone is moved in an up-and-down motion ending on
­EXCEPT one. Which is the EXCEPTION? the upstroke.
A. Less pressure needed to remove the deposit D. the stone should not rotate or move in any way except
B. More likely to slip during instrumentation up and down.
C. Less chance of occupational injury 43 When comparing sharpening technique between a univer-
D. More likely to produce a clean tooth surface sal and a Gracey curet for an instrument-stable technique,
36 The cutting edges of scaling instruments dull with use. Dull which of the following remains the same?
cutting edges are less effective in removing deposits than A. Rotation of the stone on its long axis is usually the
are sharp cutting edges. same.
A. Both statements are true. B. Stone position or angulation is the same.
B. Both statements are false. C. Terminal shank position is the same.
C. First statement is true, the second is false. D. Number of cutting edges sharpened is the same.
D. First statement is false, the second is true.
Instrumentation 421

o8820 44 The rule of thumb for working on the maxillary arch is that 52 Biological testing during the sterilization process MUST
the be accomplished _______ to ensure the effectiveness of the
o8830 A. arch should be approximately parallel to the floor. process.
o8840 B. light is positioned over the chest and shines into the A. daily
mouth at an angle. B. weekly
o8850 C. arch should be positioned so that the occlusal plane is C. biweekly
perpendicular to floor. D. monthly
o8860 D. light is positioned directly over the nose and shines onto 53 Shorter strokes with a scaling instrument give more control
the chin at an angle. and are used for assessment (exploring). Longer strokes are
45 For BEST control of infection in the dental office, den- typically used for deposit removal.
tal handpieces used for coronal polishing procedures A. Both statements are true.
should be B. Both statements are false.
A. heat sterilized. C. The first statement is true, the second is false.
B. disinfected with chemical agents. D. The first statement is false, the second is true.
C. sonicated in general purpose cleaner. 54 Cold sterilization is widely used because of its ability to
D. wiped with alcohol. verify sterilization of instruments.
46 What is the FIRST step in the process of sterilizing instru- A. Both the statement and reason are correct and related.
ments? B. Both the statement and reason are correct but NOT related.
A. Storing instruments to preserve their sterility for future C. The statement is correct, but the reason is NOT.
procedures D. The statement is NOT correct, but the reason is correct.
B. Packaging instruments in appropriate sterilizer bags E. NEITHER the statement NOR the reason is correct.
C. Steam-sterilizing instruments to kill bacteria, fungi, and 55 Which of the following explorers is considered BEST for
viruses caries detection?
D. Placing instruments in the ultrasonic bath to remove A. Shepherd’s hook
debris B. The #17 style
47 When moving instruments from the ultrasonic bath to the C. The ODU 11/12
sink for rinsing, what type of gloves should be worn? D. Pigtail or cowhorn
A. Multiple-use utility gloves
B. Sterile examination gloves
C. Nonsterile examination gloves
D. Plastic overgloves
Answer Key and Rationale
48 After prophylaxis of an HIV-positive individual has been
completed, the instruments should be
A. soaked in household bleach. 1 (B)  Correct (good) patient and clinician positioning
B. sterilized for 10 minutes longer than the standard steril- may improve efficiency by providing better visibility
ization time. and reducing chance for upper body pain and injury
C. sterilized using the same techniques as for all other ster- to clinician. Does NOT change number of instruments
ilizable instruments. needed to treat all surfaces of the teeth. Correct pa-
D. placed in double sterilizer bags to avoid the cross- tient positioning reduces possibility that patient might
­contamination of other instruments. undergo syncope (fainting) during treatment, mainly
49 The biological indicator Bacillus stearothermophilus is placement in supine position. Other special instances
used to determine effective sterilization in which of the fol-
include semiupright (45° for respiratory disorders)
lowing sterilizers?
A. Moist heat and chemical vapor
and Trendelenburg positions (subsupine, with feet
B. Chemical vapor and dry heat higher level than head, when syncope has occurred).
C. Ethylene oxide and moist heat 2 (A)  Well-positioned patient SHOULD have head at
D. Dry heat and ethylene oxide the top of chair to give clinician optimum access to the
50 Moist heat sterilization mouth from all clock or zone positions. SHOULD be
A. uses steam under pressure. in a supine position MOST of the time with the head,
B. is safe for all instruments. heart, and feet at about the same level. SHOULD turn
C. prevents corrosion of instruments. the head as needed for the clinician to have the best
D. penetrates oils and powders. access. Patients with a strong gag reflex will do best
51 Of the following methods of sterilization, which one re- in a full supine or fully upright position.
quires the longest cycle?
3 (B)  Both statements are false. First statement describes
A. Moist heat
B. Dry heat
CORRECT (good) patient positioning for mandibu-
C. Chemical vapor lar arch, and second statement describes CORRECT
D. Ethylene oxide (good) patient positioning for maxillary arch.
4 (A)  Sitting with legs crossed causes stress along
spine and reduces circulation to the legs. LESS stable
422 Saunders Review of Dental Hygiene

seated position than having the weight evenly distrib- even a particular tooth surface. Gracey 11-12 and
uted, with legs apart and back supported. 15-16 are for mesial surfaces of posteriors. Gracey
5 (C)  BEST body position allows support by the skel- 13-14 and 17-18 are for distals of same teeth. Gracey
etal and muscular system. Includes keeping head cen- 5-6 is for any of the surfaces of anteriors and premolar
tered over the spine, although the chin may be tilted teeth.
down slightly to look into mouth from comfortable 14 (A)  It is critical that implants NOT be scratched or
distance away. SHOULD keep shoulders and arms damaged. Plastic debridement instruments have been
down in relaxed positioned. SHOULD consider mov- shown NOT to scratch, whereas standard metal in-
ing patient, before stressing their own bodies by awk- struments do scratch, May be shaped like curets or
ward positioning. scalers (sickle scalers) or have unique shapes that
6 (A)  Thickness and rigidity of shank MOST directly specifically fit around implant surfaces.
influence quality of deposits (tenacity, volume) the 15 (A)  Thumb and index finger are usually placed op-
instrument will remove. Thicker shank makes instru- posite each other on the instrument handle with the
ment more rigid and better able to remove bulky and middle finger placed below them on or near the in-
dense deposits. strument shank.
7 (D)  Instrument is considered well balanced if rela- 16 (C)  BEST fulcrum is intraoral fulcrum placed on
tionship of working end to the handle makes the tooth surface near working area. Fulcrum is mechani-
­instrument comfortable to hold and easy to use. Oc- cal leverage when used for instrumentation; increases
curs when middle of working end is centered on an the control over working stroke. Ring finger is known
imaginary line that also runs through the handle. as the fulcrum finger.
8 (C)  Four functions of a mirror are soft tissue retrac- 17 (B)  Angles have been reversed between these state-
tion, indirect vision, indirect illumination (reflected ments. MORE open angle is needed for scaling of
lighting), and transillumination. Illumination of a dark larger deposits. BEST is in range of 70° to 80°. Root
area of the mouth with light bounced off mirror face planing is a finishing procedure (controversial), and
is indirect illumination. For transillumination, light is the blade angle becomes progressively more closed.
reflected off mirror and directed through the teeth to Optimum is 45° to 60°.
observe shadows caused by caries and calculus. 18 (A)  MOST sonic scalers operate in the 2500 to 7000
9 (D)  ALL curets, whether universal or area specific, cycles/sec range (cps). MOST magnetostrictive ul-
are half-moon shaped in cross section. Difference be- trasonics operate at 18,000 to 45,000 cps, and piezo-
tween the two is the relationship of the face to the electric operate at 35,000 to 50,000 cps. Range of
terminal shank. Cross-sectional shape of explorer is a ultrasonics is from as low as 18,000 cps to high limit
circle, and (sickle) scaler is a triangle. of 50,000 cps.
10 (A)  Explorers and probes are used as primary instru- 19 (A)  All of this information is accurate and highlights
ments of assessment. Probes, with a variety of nu- MAIN difference between the two types of ultra­
merical demarcations, are used to measure pocket sonics.
depth, clinical attachment levels (CAL), recession, 20 (C)  Slim- or thin-style tips are very effective in re-
and width of attached gingiva and to determine a vari- moving light calculus, NOT as effective for heavy
ety of tissue and root characteristics such as bleeding calculus removal as thicker tips.
on probing (BoP) and root anatomy. Nabor’s probes 21 (D)  Ultrasonic debridement is begun at MOST coronal
have a curved design that BETTER fits into furcation deposits and progresses toward more apical deposits.
areas. Explorers are BEST used to identify calculus BOTH manual and ultrasonic instrumentation is effec-
and other deposits, root texture and anatomy, pocket tive in removing calculus from subgingival areas.
contours, also to identify caries and marginal status of 22 (D)  In deep, narrower pockets and hard-to-reach ar-
restorations. eas like furcations, ultrasonics are generally consid-
11 (A)  (Sickle) scalers taper to a thin point at working ered superior instrument. Long, thin, probelike tips
end. This fits BEST the tight areas around contacts and will reach into deep, narrow pockets easily. More
the interproximal. BOTH types of curets have difficulty advanced furcations have been shown to have lower
fitting into tight contact areas. Chisels (rarely used) are bacterial count after ultrasonics than after manual
BEST for gross debridement of anteriors; NOT suit- instruments. Without a cutting edge, angulation of
able for fine scaling around either contact or gingiva. the face of the blade to the tooth is NOT possible,
12 (B)  Universal curets do have two cutting edges per and this makes it easier to use the ultrasonic instru-
working end; however, area-specific curets have ments. Good tip-to-tooth adaptation is still important.
ONLY one cutting edge per working end. Treating each root as a separate tooth, if access per-
13 (B)  Individual area-specific curets are designed to mits, with a combination of horizontal, vertical, and
instrument a particular type of tooth, or in posteriors, oblique strokes is recommended.
Instrumentation 423

23 (A)  Lighter, more relaxed pen grasp and use of a Polishing will BOTH remove dental biofilm (dental
soft tissue fulcrum place less pressure of the instru- plaque) and give the mouth a clean feeling; however,
ment tip on the tooth, and that results in therapy that potentially damaging to the teeth if repeated too often
is more effective and more comfortable. Each of the and there are other LESS potentially harmful meth-
other answers may occur as a coincidental benefit, but ods of accomplishing these tasks, such as demonstra-
improved therapy is the driving rationale. Increased tion of toothbrushing and other oral hygiene methods,
options in fulcrums may lead to BETTER adaptation use of ultrasonics, and light manual instrumentation.
and therefore deeper instrumentation, which will re- 30 (D)  Time is NOT reduced because dental biofilm
sult in deeper fluid penetration because the fluid fol- MUST be removed by alternative means. Selective
lows the tip. Softer, lighter working stroke will be polishing is BETTER than general polish because it is
more comfortable to the patient’s soft tissue, and the more protective of the tooth surface. Repeated polish-
clinician will be more relaxed and less fatigued. ing can remove tooth structure, especially fluoride-rich
24 (A)  Adapting the tip of ultrasonic against the root surface layer. Biofilm removal, traditional reason for
can result in damage to the root surface. Use of point polish, can be accomplished by other methods that have
is NOT as effective as adapting the side of the tip. less potential to damage the tooth, such as demonstra-
Heat generation is NOT affected, and discomfort is tion of toothbrushing and other oral hygiene methods,
caused to the patient but NOT the clinician. use of ultrasonics, and light manual instrumentation.
25 (D)  Comparable number of working strokes is 31 (C)  Polishing always creates microbial-containing
needed to completely treat the root surface but can aerosol. Bacteremia may be caused when inflamed
be taken more quickly and easily because there is NO gingiva is polished, and oral bacteria, NOT abrasive
need to establish a working angle or lateral pressure particles, are introduced into the body. Allergy to
and also the strokes are effective in BOTH directions. commercial pastes is possible but rare. With COR-
Action of an ultrasonic stroke is easy and uniform RECT (good) polishing technique, including NO
like an eraser motion. Statements that the clinician is frictional heat, mercury should NOT be released from
less tired and pressure is lighter are true but NOT the amalgam restorations.
main reasons for the difference. 32 (A)  Polishing action is accomplished by the abrasive
26 (C)  Shielded pacemaker is shielded (protected) from particles. Rubber cup, brush, wood stick, and force
ultrasonic action and is considered safe for treatment. of the air in air-powder polish are ONLY a means of
Older ones were NOT shielded; ALL pacemakers to- applying abrasive for polishing, NOT cause of the
day are shielded. In demineralized areas, may remove polishing. LESS abrasive powders are available for
or damage weakened tooth structure. Individual who all techniques.
always breathes through the mouth will have difficult 33 (C)  First statement regarding the components and o9750
time breathing with the water spray in the mouth. roles of the ingredients of the polishing slurry is
Newly erupted teeth have large pulps that may be heat correct. Air-powder polishing systems usually do
sensitive and enamel that is NOT fully mineralized. cause MORE damage to composites because the
27 (C)  Ultrasonic scaling, even with high-speed suc- “blasting action” removes some component of the
tion, produces tiny, contaminated droplets in the air composite material; less abrasion is possible with
around the patient’s mouth. Clinician MUST use bar- LESS abrasive powders. However, regarding tooth
rier techniques for protection. Face shield provides structure, air-powder polishing, no matter what
the BEST eye protection but has an opening in the powder is used, removes LESS tooth structure than
chin and neck area that allows aerosols access to the curets when used to remove stain on root surfaces
clinician’s respiratory system. Thus facial mask is and causes less abrasion on enamel than rubber-cup
needed also to complete barrier protection for clini- polishing.
cian. 34 (B)  For anteriors, tip of the nozzle is directed at 60°
28 (A)  Amalgams can be debrided with an ultrasonic toward the tooth and at 80° toward posteriors.
scaler if care is used. It is TRUE that amalgam can 35 (B)  Dull, NOT sharpened, instruments are MORE
be removed; in fact, power-driven scalers are BEST likely to slip during instrumentation. Sharpening im-
instrument to use for overhang removal, but the same proves quality of debridement and makes treatment
could be said of a scaler (sickle scaler). Composites more comfortable and effective for BOTH patient
have shown surface damage from ultrasonic instru- and clinician.
mentation. Porcelain-fused-to-metal crowns and 36 (A)  With use, metal is worn away from cutting edge,
esthetic veneers can be fractured or loosened with making the shape rounded at the junction of the face
ultrasonic use. and lateral side where once they formed a sharp edge.
29 (B)  Polishing is an option for removing extrinsic Rounded cutting edge is NOT as effective in engag-
stain. Will brighten the teeth ONLY, if stain is present. ing and removing deposits, since it is dull.
424 Saunders Review of Dental Hygiene

37 (D)  Dull instrument is MORE likely to slide over disease; older handpieces that CANNOT be sterilized
deposit, causing burnishing. As it dulls, will tend to compromise infection control procedures in dental
slide on the tooth surface and have less bite or grab office.
than sharp instrument. Dull or rounded surface will 46 (D)  FIRST step in sterilization of instruments is to
reflect light. Standard rule CANNOT be made about remove debris by placing in ultrasonic cleaner (NOT
number of strokes to take before sharpening because manually cleaning). Instruments are then packaged
it varies by the amount of lateral pressure being ap- in appropriate sterilization bags or wrapped cassettes
plied, nature of deposit, metal composition. after being rinsed and dried, then sterilized, allowed
38 (C)  Arkansas stone is a natural quarried stone. to cool, and stored to preserve sterility.
SHOULD be lubricated with fine oil before use and 47 (A)  Multiple-use utility gloves provide maximum
may be sterilized by any of the conventional methods, clinician protection against inadvertent occupational
although may become MORE brittle with repeated exposures. Examination gloves and overgloves do
exposures to high heat. NOT provide adequate protection for handling con-
39 (C)  India stone is MOST abrasive, with its medium taminated instruments during sonication and steril-
grit. Although Arkansas and ceramic stones may vary ization procedures.
somewhat, are less abrasive. 48 (C)  Standard Precautions (from CDC), including
40 (C)  Typically, MORE metal is removed from the autoclaving instruments, are designed to protect
lateral surface, rather than the lateral surface and the dental healthcare personnel and patients from blood-
face equally, when sharpening. Besides making in- borne pathogens and infective agents (these latter
strument thinner so that is fits into the sulcus more regulations and guidelines are regulated instead by
easily, it is believed that preserving the face-to-back OSHA). Instruments used for HIV-infected individ-
dimension maintains instrument strength BEST for ual SHOULD be handled as ALL other autoclavable
the pressures and stresses created by scaling. instruments are handled. NO extra time is needed,
41 (C)  First statement is true, the second is false. First and double bagging is NOT necessary. Soaking in-
statement is accurate description of how wire edge is struments in bleach will corrode them and NOT sig-
made. Important NOT to have wire edge when instru- nificantly improve sterilization results.
ment is applied to a tooth because wire edge fractures 49 (A)  Moist heat (autoclave) and chemical vapor steril-
easily and metal can become embedded in gingiva or izers (chemiclave) use spores of B. ­stearothermophilus.
root surface. Easy and equally acceptable to prevent Ethylene oxide and dry heat sterilizers use spores of
or remove wire edge during sharpening procedures. B. subtilis. BOTH are biological indicators of steril-
42 (A)  Scaler (sickle scaler) is held stable with face ization.
parallel to the floor (terminal shank is perpendicular 50 (A)  Moist heat sterilization (autoclave) uses steam
to floor). Internal angel of the curet is 70°, but vis- under pressure. NOT all instruments can withstand
ible angle that is formed between instrument face and conditions in steam autoclave (e.g., metal instruments
stone is 110°. To prevent wire edge, important to end can corrode and plastics melt); does NOT penetrate
sharpening of each area with downstroke so that NO oils or powders.
metal is burnished above the face. 51 (D)  Ethylene oxide requires 10 to 16 hours to be ef-
43 (B)  Position and the movement of stone are the same. fective on metal instruments. Dry heat cycle MUST
Several other aspects are NOT the same because of last 1 to 2 hours, and moist heat cycle MUST last
differences in instrument design. Gracey curets are 15 to 20 minutes (flash sterilization occurs in 3.4 to
typically MORE curved from heel to toe, necessitat- 12 minutes). Chemical vapor cycles require 20 min-
ing more rotation of stone on long axis. Also, because utes for effective sterilization.
blade of Gracey is offset at 70°, the terminal shank 52 (B)  Biological testing (using spores) SHOULD ­occur
is NOT perpendicular to floor during sharpening and on weekly basis to monitor effectiveness of steriliza-
ONLY one cutting edge is used (or sharpened) on tion process as required by CDC.
each working end. 53 (B)  Both statements are false. Shorter strokes dur-
o9860 44 (C)  With treatment of maxillary arch, arch SHOULD ing scaling give MORE control and deposit removal.
be positioned so that occlusal plane is perpendicular Longer strokes are used MAINLY for assessment
to floor and light is positioned over chest; light shines (exploring).
into mouth at an angle. With treatment of mandibular 54 (E)  Neither statement nor reason is correct. Cold
arch, the arch SHOULD be approximately parallel to sterilization is used less because of inability to verify
floor and light positioned directly over mouth; light sterilization of instruments.
shines straight down. 55 (A)  Shepherd’s hook explorer is BEST for caries
45 (A)  Dental handpieces SHOULD be heat sterilized detection and restoration evaluation; the others are
between patients to prevent spread of infectious BEST for deposit and root surface exploration.
CHAPTER  13

Periodontology
PERIODONTIUM 3. Race: African Americans have consistently dem-
Periodontium consists of gingival tissues, periodontal onstrated higher prevalence than Caucasians.
ligament (PDL), cementum, alveolar bone. Functions as 4. Education: inversely related to increasing levels of
attachment mechanism, shock absorber, line of defense education.
against external agents; attaches the tooth to its bony 5. Income: inversely related to increasing levels of
housing (alveolus), provides resistance to forces of mas- income.
tication, speech, and deglutition, maintains body surface 6. Residence: slightly MORE prevalent in rural than
integrity by separating the external and internal environ- in urban areas.
ment, adjusts for structural changes associated with wear 7. Geography: NO significant differences have been
and aging through continuous remodeling and regenera- noted in the United States.
tion, and defends against external harmful factors. B. Major systemic risk factors: MOST able to modify
• See CD-ROM for Chapter Terms and WebLinks. somewhat.
• See Chapters 2, Embryology and Histology: healthy 1. Tobacco use.
periodontium; 6, General and Oral Pathology: peri- 2. Endocrine disorders:
odontal lesions; 8, Microbiology and Immunology: a. Diabetes mellitus.
background information; 11, Clinical Treatment: peri- b. Hyperparathyroidism.
odontal evaluation and charting; 17, Community Oral c. Hormonal factors.
Health: epidemiology of periodontal disease. 3. Nutritional deficiencies.
4. Adverse drug reactions.
Periodontal Diseases and Risk Factors 5. Stress or psychosocial.
Periodontal (gum) diseases, including gingivitis and peri- 6. HIV/AIDS.
odontitis, are serious infections that, left untreated, can 7. Neutrophil (polymorphonucleocyte [PMN]) ab-
lead to tooth loss. Periodontal pathogens alone may NOT normalities.
lead to development of disease. Degree of destruction 8. Genetics: genetic predisposition has been related to
varies greatly from one individual to another, which sug- inherited systemic diseases and to familial occur-
gests that other factors may alter host’s resistance. These rences with some forms (e.g., Down syndrome);
factors have been labeled risk factors. NOT able to modify at this time.
Risk factors affect prevalence, incidence, severity, de-
velopment of a disease—periodontal disease in this case. Tobacco Use Risk
Some of these factors, such as genetics and age, are Tobacco is the MOST important risk factor involved in p0050
­beyond individual’s control but others, such as tobacco periodontal disease. Nicotine and other chemicals embed
use or stress factors, can be modified. Even endocrine on the root surface and act as toxic irritants, in addition to
disorders, nutritional deficiencies, drug reactions, HIV causing constriction of area blood vessels. Result is that
status can somewhat be modified. Increased incidence tobacco users are at greater risk for periodontal disease
of periodontal disease may in turn represent a risk fac- and LEAST likely to heal after treatment. Thus it is the
tor for cardiovascular disease (CVD) or birth of preterm, MOST traumatic habit for periodontium and the hardest
low-birth-weight baby, as well as pose a serious threat to modify, but when tobacco ceases to be a risk factor,
to people whose health is compromised by diabetes mel- it is the MOST effective part of the overall therapy for
litus, respiratory diseases, or osteoporosis. periodontal disease.
A. Major background risk factors: most NOT able to • See Chapter 9, Pharmacology: tobacco cessation.
modify. A. Tobacco smokers: always double amount the patient
1. Age: prevalence increases directly with increasing reports smoking and one cigar is equal to one pack of
age as a result of repeated inflammatory episodes cigarettes.
during lifetime. 1. Tend to exhibit MORE severe levels of disease:
2. Gender: men tend to have higher incidence than deeper pockets, higher periodontal index scores,
women. greater bone loss, higher rates of attachment loss,

425
426   Saunders Review of Dental Hygiene

more calculus (serves as contributing factor in reduced host resistance, increased incidence of
dental biofilm accumulation, prevents pocket heal- periodontal disease.
ing) than nonsmokers. 3. Defective polymorphonucleocytes (PMNs), PMN
2. Increased prevalence of moderate to severe chemotaxis, microangiopathy of periodontal tis-
­periodontal disease, directly related to number of sues, increased collagen breakdown, microbial
cigarettes or cigars smoked per day, number of alterations have been cited as rationales for peri-
years, current smoking status. odontal disease prevalence.
3. May have altered host response to various forms of 4. Oral manifestations: xerostomia, candidiasis, in-
periodontal therapy. creased caries, burning mouth, altered taste.
o15320 a. Carbon monoxide in smoking reduces oxygen 5. Second major risk factor after tobacco use,
concentrations, consequently inhibiting move- MAINLY type 1; type 2 with insulin therapy shows
ment of WBCs in gingival sulcus into periodon- a SIMILAR risk.
tium (chemokinesis, chemotaxis) and depleting 6. In patients with uncontrolled DM, oral manifesta-
capacity to engulf and destroy bacteria (phago- tions include multiple periodontal abscesses, vel-
cytosis). vety red gingival tissues, marginal proliferation of
o15330 b. Smokers also have decreased levels of salivary periodontal tissues.
antibodies (IgA) and serum IgG and IgM anti- B. Hypothyroidism: associated with occasional gingival
bodies to P. intermedia and F. nucleatum, two hyperplasia.
periodontal pathogens. C. Hyperparathyroidism: excessive production of para-
4. Increased risk for recurrent/refractory periodonti- thyroid hormone (PT), which helps control calcium
tis because of failure to respond to treatment (dis- metabolism.
cussed later). 1. Caused by benign adenoma or malignancy and
B. Smokeless (spit) tobacco users: results in osteoporosis, multilocular radiolucent
1. Have LOCAL exposure to high concentrations of jaw lesions, loss of lamina dura, “ground glass”
tobacco products; may play role in localized at- ­appearance of alveolar bone.
tachment loss. 2. Results in MORE rapidly demineralized alveolar
2. At higher risk for various forms of periodontal bone when occurs in presence of periodontitis.
­disease, as well as caries and oral cancer. D. Alterations in hormones: puberty, pregnancy, oral
contraceptives, infertility treatments, perimenopause
Endocrine Disorder Risk and menopause ALL affect periodontium (but ALL
Endocrine glands secrete hormones that regulate cellu- are temporary because hormonal levels change).
lar metabolism and maintain physiological homeostasis. 1. Puberty: involves increased levels of hormones,
Endocrine diseases, such as diabetes mellitus and hyper- which alter capillary permeability and increase
parathyroidism, have been associated with greater risk fluid accumulation in the gingiva, resulting in in-
of periodontitis; MOST can be modified with treatment. creased risk for gingivitis in presence of dental
Fluctuations in hormones (through puberty, menstrua- biofilm.
tion, pregnancy, oral contraceptives, perimenopause or 2. Menstruation: even with normal, cyclical hormonal
menopause) have also been shown to alter tissue response fluctuations has NOT been shown to increase risk
to local factors and therefore are risk factors for devel- of gingivitis; gingivitis has been shown to be actu-
opment of gingivitis and periodontitis, although MOST ally lower than during ovulation and premenstrua-
changes are temporary. tion, when hormones may peak.
• See Chapters 6, General and Oral Pathology: endocrine 3. Pregnancy:
diseases; 11, Clinical Treatment: pregnant patient. a. Strong correlation with development of gingi-
A. Diabetes mellitus (DM): genetically associated, de- val changes such as gingivitis, gingival enlarge-
bilitating endocrine disorder characterized by glucose ment, pyogenic granuloma (pregnancy tumor).
intolerance. b. Results in increased levels of progesterone,
1. Types: altering capillary permeability and tissue me-
a. Type 1: develops before age 30; MUST be con- tabolism, making tissues MORE at risk for
trolled with insulin therapy. inflammatory changes in presence of possibly
b. Type 2: MOST common form occurs after age minimal levels of dental biofilm.
40; may be controlled by diet, oral hypoglycemic c. Associated with high levels of anaerobes, such
agents, or LESS frequently, insulin therapy. as Prevotella intermedia (Pi).
2. Associated with high blood glucose levels that re- 4. Oral contraceptives (birth control pills [BCPs]):
sult from decreased insulin levels and create vari- a. Drugs that have been shown to elevate hormone
ety of systemic effects, including vascular disease, levels and thus mimic pregnancy.
    Periodontology   427

b. Therefore affect gingival tissues in manner 6. Perimenopause and menopause:


SIMILAR to pregnancy; place at risk for gingi- a. Related to mucocutaneous disorders such as
vitis. erosive lichen planus and mucous membrane
c. However, lower doses generally used may NOT pemphigoid.
affect periodontium. b. Risk for osteopenia and osteoporosis (discussed
5. Infertility treatments: women with MORE than later).
three cycles had higher levels of gingival inflamma- c. May affect periodontium if on hormone re­
tion, bleeding, and gingival crevicular fluid (GFC); placem­ent therapy (SIMILAR to pregnancy or
severity strongly associated with duration of drugs. BCPs).

clinical study  

Age 25 YRS SCENARIO

Sex ☐  Male   ☒  Female The patient, whom the dental office has
been seeing regularly every 6 months for
Height 5’10“ oral prophylaxis, presents with pocket
Weight 220 LBS depths that have increased from 3 to
4 mm and a bleeding score of 60%, ac-
BP 120/82 cording to the Ainamo and Bay bleeding
index. Signs of gingival inflammation are
Chief Complaint “I gave up smoking and still my gums are present in the form of marginal erythema,
a mess. Is it my being pregnant?” generalized bulbous interdental papil-
lae, and rolled margins. This finding is
Medical History 4 months pregnant
Quit smoking 4 months ago unusual for the patient because she has
always exhibited meticulous oral hygiene.
Current Medications None Moreover, dental biofilm and calculus are
not present in noticeable quantities dur-
Social History Student nurse ing this visit.

1. What is the most likely American Academy of Peri- 3. Thorough professional debridement, elevated dental
odontology (AAP) classification of the patient’s biofilm control measures, and education regarding
­condition? correlation between the pregnancy and contributing
2. What factors are contributing to her condition? Dis- factors will keep the disease under control.
cuss bleeding index used. 4. Gingivitis associated with pregnancy begins in
3. How should this condition be treated? ­second or third month and is most severe in second
4. What information can be given to the patient to help and third trimesters. Moreover, in pregnant women
prepare her for other possible ­complications? localized enlargements in interdental papillae may
form bleeding growths, referred to as pyogenic gran-
1. Dental plaque (biofilm)-induced gingivitis caused by ulomas (pregnancy tumors). If these lesions ­ occur,
endocrine system changes. Her pocket depths are no most will disappear at end of the pregnancy as hor-
greater than 4 mm and most likely are pseudopock- mones become more regulated. The patient should
ets because signs of inflammation include gingival be ­congratulated for quitting smoking for own health
edema, which causes tissue enlargement. and that of unborn child. Tobacco use is number one
2. Predominant factor contributing to the patient’s condi- factor in periodontal disease; also affects health of
tion is pregnancy. During pregnancy, increased levels child, now during pregnancy and later with pres-
of progesterone have been associated with increased ence of second-hand smoke. However, continued
risk for gingivitis. Also frequently associated with ­gingivitis can still put patient at risk for a preterm
high levels of anaerobes, such as Prevotella interme- low-birth-weight child.
dia (Pi). Gingival bleeding index assesses bleeding of
the gingival margin in response to gentle probing, used Nutritional Deficiency Risk
as indicator of gingival health or disease. To score it, Poor nutrition lowers resistance to periodontal dis-
divide the total number of areas that bleed by the num- ease, which makes deficient individuals MORE at
ber of gingival margins examined, then multiply the risk for infection and severe forms of periodontitis. In
result by 100 to arrive at a score (percentage). United States, nutritional deficiencies are found MOST
428   Saunders Review of Dental Hygiene

c­ ommonly among elderly, lower socioeconomic groups, H. Other nutritional disorders:


drug and alcohol abusers. Food consistency also has 1. Osteoporosis:
been recognized as contributing factor to accumulation a. Loss of bone mass caused by imbalance of
of dental biofilm and thus development of periodontal plasma calcium and phosphorus levels; EARLY
disease. Poor nutrition can be modified with proper loss of bone density with osteopenia.
diet. b. Severity of bone loss increases if periodontitis
• See Chapter 7, Nutrition: diet counseling. is superimposed on osteoporosis.
A. Vitamin A deficiency: effect of deficiency NOT 2. Protein deficiency (undernutrition): when severe,
known even though involved in synthesis of epi- significantly reduces host defenses and wound
thelial cells, proteoglycans, fibronectin, procolla- healing.
gen. a. In MOST severe forms (kwashiorkor, maras-
B. Vitamin B2 (riboflavin) deficiency: may affect the mus, cachexia) exhibits oral changes such as
oral cavity, causing angular cheilitis, glossitis, oral glossitis, angular cheilitis, xerostomia, increased
ulcerations. gingival inflammation, periodontal bone loss.
C. Vitamin B6 (pyridoxine) deficiency: since involved b. Kwashiorkor is also associated with increased
in carbohydrate metabolism; may induce general- incidence and severity of necrotizing periodon-
ized stomatitis, glossitis, gingivitis, and other oral tal disease, resulting in cancrum oris (noma or
­symptoms. oral gangrene).
D. Vitamin B12 (cobalamin) deficiency: c. LESS severe forms still affect oral health
1. May result from diet or may be caused by altered among alcoholics, anorexics (self-starvation),
absorption (pernicious anemia). long-term bedridden and hospital patients,
2. May make gingival tissues MORE at risk for epi- those taking drugs that depress appetite or in-
thelial dysplasia and malignant transformation. crease metabolism.
3. Generally accompanied by folic acid deficiency;
folic acid is of interest because may reduce gin- Stress and Psychosocial Risk
givitis or phenytoin (Dilantin)-induced gingival Increased risk for and severity of periodontal disease have
hyperplasia. been reported during stressful life events such as death,
E. Vitamin C (ascorbic acid) deficiency: divorce, war. Plasma corticosteroid levels become higher
1. Adversely affects periodontal connective tissue, during exposure to stressful stimuli and act to suppress
capillary integrity, and wound healing because protective portions of immune response. Strongest exam-
­vitamin C is essential to collagen biosynthesis. ple of correlation between stress and periodontal disease
2. When prolonged, may cause scurvy, which induces is necrotizing periodontal disease (discussed later). Stress
severe periodontal changes. can be minimized and controlled and in MOST cases is
F. Vitamin D deficiency: temporary in nature.
1. Results in altered levels of plasma calcium and
phosphorus, which interferes with mineralization Adverse Drug Reaction Risk
of organic bone matrix; leads to rickets in children Drugs may have a variety of adverse effects in the oral
and osteomalacia in adults. cavity, ranging from allergic to toxic reactions, includ-
2. Results in loss of lamina dura and thinning of cor- ing gingival hyperplasia and xerostomia. Although drugs
tical plates (radiographically). themselves do NOT cause periodontal disease, may pro-
3. May cause common dental anomalies, such as vide locally irritating conditions that place individual
delayed development of the permanent dentition, at risk. Unless a drug is taken indefinitely, MOST side
enamel hypoplasia, cemental resorption, open api- effects would be temporary and able to be modified by
cal foramina, enlarged pulp chambers, and fre- time.
quent pulp stones. • See Chapters 9, Pharmacology: drugs and side effects;
4. In adults may lead to osteomalacia, which destroys 11, Clinical Treatment: management of xerostomia.
PDL and resorbs alveolar bone; causes replace- A. Gingival hyperplasia (see later discussion for more
ment fibrous dysplasia. specifics and for surgical treatment; Figure 13-1):
G. Antioxidants such as beta-carotene; vitamins A, C, E; 1. Associated with anticonvulsants (phenytoin), im- o1090
selenium may be involved in reducing or preventing munosuppressants for organ and bone marrow
diseases caused by free oxygen radicals; thus may transplants (cyclosporin); calcium channel block-
play role in treating some forms of oral cancer and ers (nifedipine, diltiazem, verapamil); cannabis.
have been shown to be involved in tissue destruction 2. Decreases effectiveness of dental biofilm removal
associated with periodontitis and chronic inflamma- during oral hygiene care; hyperplasia is increased
tory osteoarthritis. with poor oral hygiene.
Periodontology   429

Figure 13-1  Gingival hyperplasia associated with anticon­ Figure 13-2  HIV-associated periodontitis with linear gingi-
vulsant drug therapy (phenytoin [Dilantin]). (From Bath-Balogh M, val erythema (note gingival Kaposi’s sarcoma [KS]).
Fehrenbach MJ: Illustrated dental embryology and anatomy, ed 2,
Philadelphia, 2006, Saunders/Elsevier.)
A. HIV- associated gingivitis:
1. Linear gingival erythema (LGE): well-defined red
B. Xerostomia (see later discussion of benefits): side ef- band along free gingival margin, with NO bleeding
fect of many drugs. on probing (BoP).
2. Punctate or diffuse erythema of attached gingiva.
HIV/AIDS Status Risk B. HIV-associated periodontitis (Figure 13-2):
Oral manifestations of HIV infection often are first signs of 1. Defined by presence of LGE, attachment loss, cra-
AIDS. Dental hygienist may be the first person to identify tering in area of interdental col, severe clefting,
­such manifestations because of seeing patients often over time. which often accompanies recession.
Prevalence of periodontal disease and other oral manifesta- 2. Treated by conventional nonsurgical periodontal
tions in patients with HIV/AIDS varies significantly, depend- therapy (NSPT) in conjunction with meticulous
ing on decline of immune system, lifestyle, early recognition, homecare.
treatment, ALL of which can modify it as a risk factor. C. AIDS-associated necrotizing ulcerative periodontitis
• See Chapter 8, Microbiology and Immunology: HIV/ (NUP): see later discussion.
AIDS.
clinical study  

Age 38 YRS SCENARIO

Sex ☒  Male   ☐  Female Patient has a chronic cough that


he notes has persisted for several
Height 6’4” months. The extraoral examination
Weight 185 LBS reveals bilateral generalized lymph-
adenopathy. Intraorally, several large
BP 105/68 ulcerations that are red and raw are
noted on the hard palate. Addition-
Chief Complaint “My mouth hurts so bad that I can’t ally, a thick white coating appears on
eat. And I have not been feeling well portions of the hard palate and all
lately.” of the soft palate, extending into the
Medical History Tuberculosis 5 years ago pharynx; when the coating is wiped
Epstein-Barr infection 2 years ago off with gauze, raw inflamed tissue
is exposed. The gingival tissues have
Current Medications OTC ginseng qd a definite red band along the facial
gingival margins of the maxillary arch;
Social History Unemployed graphic designer however, no bleeding is detected.

1. What systemic condition is the patient likely to 4. What treatment for the patient’s oral condition should
have? be discussed?
2. What might be causing the severe pain in his mouth? 5. What periodontal maintenance interval is recom-
3. What is the significance of the palatal ­ulcerations? mended, after his acute problems have ­subsided?
430   Saunders Review of Dental Hygiene

1. This patient is likely to have HIV/AIDS, based on B. PMN dysfunctions:


generalized lymphadenopathy, generalized malaise, 1. Adherence defects: include leukocyte adher-
linear gingival erythema (LGE; classic sign of HIV), ence defect (LAD), genetic autosomal recessive
probable candidiasis, oral ulcerations. defect:
2. Severe pain is most likely caused by the open pala- a. Impair successful margination of PMNs, pre-
tal ulcerations. Candidiasis and LGE typically are not venting migration to sites of infection.
painful. b. Clinical condition characterized by recurrent
3. Palatal ulcerations are most likely herpetic lesions, bacterial infections, diminished pus formation,
since they are on tissue overlying bone; aphthous prolonged wound healing, leukocytosis.
­ulcers are on tissues that do not cover bone. c. Dental manifestations are characterized by ag-
4. Tactful discussion should include immediate refer- gressive periodontitis, progressive alveolar
ral to a physician if patient does not reveal his HIV bone loss, premature exfoliation of primary and
status. His oral condition requires prescriptions for permanent teeth, severe gingival inflammation.
antiviral drug, such as acyclovir, for herpetic lesions 2. Chemotaxis defects: involve impairment of direc-
and antifungal agent, such as nystatin (Mycostatin), tional migration of PMNs toward attracting mol-
clotrimazole (Mycelex), ketoconazole (Nizoral), for ecules such as bacteria.
the candidiasis. In addition, povidone-iodine and a. Include several rare diseases and syndromes
chlorhexidine rinses (without alcohol because of open that involve defects in PMN locomotion and
sores) would reduce pain and discomfort the patient is chemotaxis, such as DM, Down syndrome,
experiencing. May need a lidocaine oral rinse to help ulcerative colitis, Job’s syndrome, Chédiak-
with eating. Higashi syndrome, lazy leukocyte syndrome,
o1280 5. All patients with HIV/AIDS should be seen monthly Papillon-Lefèvre syndrome (these rare ones
for periodontal maintenance because oral manifes- discussed later).
tations of the disease, including those of the peri- b. Associated with aggressive periodontitis.
odontium, ­ often occur rapidly and are extremely
destructive. ETIOLOGY OF PERIODONTAL DISEASE  
Many factors are involved in the etiology of periodontal
Neutrophil Abnormality Risk disease, including host response, microbiology, oral con-
The MAIN etiological factor in development of gingivitis tributing factors, occlusal trauma.
and periodontitis is dental biofilm. Also recognized that
bacteria and host response MUST be in balance to avoid Etiology: Host Response
disease. One of the MAIN players in active host response Bacteria interact with host in MOST cases of periodon-
is the PMN. When PMN response is impaired, bacteria tal disease. In healthy individuals, active host response
flourish and disease often becomes MORE severe. Most quickly results in slight inflammation, which destroys the
cases of altered host response are NOT able to be modi- antigens (bacteria). However, periodontal health depends
fied at this time as risk factors, unless resulting from on balance between BOTH protective and destructive
treatment and drug use. systems. When host response is impaired, MORE rapid
• See Chapters 6, General and Oral Pathology: neutrope- destruction of periodontium results. Included in host re-
nias; 8, Microbiology and Immunology: PMN structure sponse are the defense mechanisms of the oral cavity,
and function. inflammatory response, immune response, all discussed
A. Neutropenias: next. See also later discussion of pathogenesis of peri-
1. Associated with decrease in circulating PMNs. odontal disease for host response to etiological factors
2. Manifested as skin infections, upper respiratory in- over time.
fections, otitis media, stomatitis, early exfoliation • See Chapter 8, Microbiology and Immunology, for re-
of the teeth, severe gingivitis with ulceration. lated discussion.
3. May be caused by drugs, radiation, Down syn-
drome, leukemia, DM, tuberculosis, autoimmune Defense Mechanisms of Oral Cavity
disorders. Natural defense mechanisms that exist in the oral cavity
4. Oral manifestations include gingivitis, aggressive include intact epithelium, saliva, gingival crevicular fluid.
periodontitis, oral ulcerations. Mechanisms work together to defend against mechanical,
5. When systemic, MOST often are idiopathic. bacterial, chemical aggression.
a. Cyclic neutropenia is a rare condition charac- A. Intact epithelium provides physical barrier between
terized by neutropenic episodes that persist for external noxious substances (e.g., bacterial enzymes
1 week, occur every 3 weeks, and result in sore such as collagenase and other by-products) and deeper
gingiva, aphthous ulcers, aggressive periodontitis. lamina propria.
Periodontology   431

B. Saliva: 2. Vascular permeability is sustained by additional


1. Functions to mechanically cleanse oral cavity. chemical mediators:
2. Buffers acids produced by bacteria from the dental a. Kinins.
biofilm. b. Prostaglandins.
3. Controls bacterial activity. c. C3 and C5a from complement system.
4. Contains antibacterial components such as lyso- d. Lysosomal enzymes released from leukocytes.
somes and lactoperoxidase. B. Cellular response:
5. Contains secretory IgA for immune resistance ­against 1. MAIN defense cell released in acute phase of in-
bacteria, food residues, fungi, parasites, viruses. flammation is PMN.
C. Gingival crevicular fluid (GCF): a. PMNs leave central stream of blood vessel
1. Increased amount correlated with severity of gin- (margination) and adhere to endothelial walls
gival inflammation. (pavementing).
2. Provides antibacterial action by bringing white b. PMNs escape through endothelial walls by emi-
blood cells (WBCs, leukocytes) and antibodies into gration (diapedesis).
close proximity with bacteria from dental biofilm. c. PMNs move to injured area (migration) by
3. Supplies complement factors that serve to ­initiate ­chemotaxis (directed movement).
BOTH vascular and cellular inflammatory re- d. Main function of PMN is phagocytosis (en-
sponses that can damage the periodontium. gulfment of foreign objects such as debris and
4. Provides sticky plasma proteins in the sulcus that ­bacteria).
serve as adhesive for junctional epithelium (JE), (1) Mature PMNs are capable of living for ONLY
keeping it intact. few hours in highly acidic environment.
5. Contains BOTH cellular and organic elements: (2) After PMNs die, release enzymatic contents
a. Cellular elements include bacteria, desquamated of lysosomes, causing MORE damage.
epithelial cells, PMNs, lymphocytes, monocytes. 2. Considerable debris accumulates, requiring addi-
b. Organic components: tion of defense cells that are capable of living in
(1) C3 and C5a: play role in release of histamine; acid environment.
also facilitate chemotaxis and phagocytosis. 3. Appearance of macrophages and histiocytes (both
(2) IgG and IgM: function in activation of are related blood monocytes in connective tissue)
complement system. signals beginning of next stage of inflammation,
(3) IgG: facilitates phagocytosis. chronic phase.
(4) IgG, IgA, IgM: prevent tissue destruction C. Chronic phase of inflammation, “second line” of ­defense:
by neutralizing bacterial endotoxin. 1. MAINLY involves chronic inflammatory cells, at-
(5) Serum IgA: prevents penetration of anti- tracted to area of inflammation by active proteins
gens across epithelial barriers. called lymphokines that are released by lympho-
cytes to do MORE containment of inflammation.
Inflammatory Response 2. Involves macrophages (within tissue), which func-
Inflammation is the natural response to insult; divided tion to phagocytose or debride area.
into three phases that include acute, chronic, repair. NOT 3. Lymphocytes become MAIN cells, in the form of
only does inflammation seek to fight the insult, it also either T cells or B cells.
adds insult to injury against the periodontium. a. B cells are concerned with humoral immunity;
A. Acute phase of inflammation, “FIRST line” of de- differentiate into plasma cells, which produce
fense; subdivided into vascular response and cellular specific antibodies (immunoglobulins [Ig]) tar-
responses: geted at immobilizing specific antigens.
1. Vascular response: develops rapidly following ini- b. B cells also produce memory cells that are capa-
tial insult. ble of producing MORE antibodies on ­demand.
a. Provides plasma proteins and fluid necessary c. T cells are concerned with cellular immunity
for rapid isolation of irritant. and are classified as T4 activator cells, T8 sup-
b. Fluid produced is responsible for associated tis- pressor cells, killer (cytolytic) cells.
sue edema. 4. Elevated plasma C-reactive protein (CRP) is noted o15340
c. Vascular dilation causes increased blood vol- with periodontal disease; may be future predic-
ume and decreased velocity, resulting in hyper- tor (inflammatory marker) for activity of disease
emia. (similar to cardiovascular disease); periodontal
d. Initial vasoactive response is produced by re- therapy would then seek to modify levels of CRP.
lease of histamine and serotonin by mast cells ­However, it is NOT considered specific enough for
located in lamina propria. use during diagnostic procedures.
432   Saunders Review of Dental Hygiene

D. Repair phase of inflammation: 3. Loss of stippling is LEAST reliable sign of inflam-


1. Begins when macrophages debride site. mation.
2. Involves fibroblasts from surrounding connective 4. Also changes in gingival contour occur as result of
tissue that migrate to injured area and begin to se- either increased tissue fluid (edema) or increased
crete tropocollagen (precursor to collagen) to pro- cellular elements (fibrosis); result in rolled mar-
duce fibrotic (scar) tissue. gins and blunted interdental papillae.
3. Involves endothelial cells that proliferate and pro- E. Changes in gingival position:
vide MORE oxygen tension. 1. ALL relative to CEJ; result in increased height of gin­
4. Involves development of granulation tissue, com- gival margin because of edema and tissue proliferation.
posed of fibroblasts, new immature collagen, new 2. Result in recessed margins, which have numerous
capillaries. causes ranging from edema, to toothbrush abra-
sion, to malposed teeth.
Clinical Signs of Inflammation
Inflammation is identified by several clinical (cardinal) Immune Response
signs, including gingival bleeding and changes in gingi- Immune response is a complex entity that consists of p0190
val color, contour, position. BOTH nonspecific and specific components. Because
A. Gingival bleeding on probing (BoP): of large numbers of bacteria that inhabit the oral cavity,
1. Characteristic of the tissue destruction that occurs ­effective host response is required to minimize disease and
during acute inflammation. tissue destruction. Important to recognize that ­ majority
2. Results from vascular engorgement, increased blood of tissue damage produced during periodontal inflamma-
flow, loss of vascular wall integrity, and microulcer- tion is caused by the host’s response to bacteria. Evidence
ation of the sulcular epithelium (SE) ­initially and then now suggests that periodontal disease is possibly an au-
later also JE, which exposes deeper lamina propria. toimmune disorder, in which immune factors in the body
3. May be spontaneous or may follow provocation. attack the person’s own cells and tissue—in this case,
4. MOST reliable sign of acute inflammation, al- those in the gingival tissues.
though late sign; lack indicates gingival health. A. Nonspecific host responses (innate host defense
B. Gingival color changes: mechanisms):
1. Occur during BOTH destructive and proliferative 1. Inflammatory response:
stages of inflammation. a. Arachidonic acid cascade: produces several
2. Result from thinning of epithelial layers, decrease biologically active products, including prosta-
in keratinization, engorgement of subepithelial glandins, leukotrienes, thromboxane.
blood vessels, capillary proliferation. b. MOST notable effects are vascular permeabil-
3. Bright red color occurs during acute stage of inflam- ity and attraction of phagocytic cells.
mation and is associated with vascular hyperemia. c. Also attract antibodies and complement that
4. Bluish color is associated with chronic inflamma- aid in destruction of bacteria and their by-
tion and venous stasis. ­products.
C. Changes in gingival consistency: d. Products of arachidonic acid cascade are poten-
1. Occur during either destructive or proliferative tially harmful to periodontal tissues and are part
stage of inflammation. of the pathogenesis of periodontal disease.
2. Include edema, softness, friability during acute 2. Complement system:
­inflammation. a. More than 20 serum proteins become activated
3. Caused MAINLY by accumulation of tissue fluid during inflammation and exhibit potent biologi-
during acute stage of inflammation, thinning of the cal activity.
epithelial layers, loss of keratin. b. Activated through classical pathway or alter-
4. Result in increases in density of tissues during nate pathway.
chronic and repair stages of inflammation; these c. Effects of activation (by either pathway):
occur because of proliferation of fibroblasts, which (1) Production of opsonins that enhance phago-
produce collagen in the form of scar tissue, giving cytosis (see below).
firm, fibrotic consistency. (2) Mast cell release of histamine that causes
D. Changes in surface texture: vasodilation.
1. May include loss of stippling if stippling was pres- (3) Release of chemotactic factors that causes
ent during gingival health. migration from the blood system of PMNs
2. Loss of stippling appears as shiny surface; caused and macrophages to specific sites.
by loss of keratin, thinning of epithelium, accumu- (4) Production of factors capable of destroying
lation of connective tissue fluid. bacterial cell walls.
Periodontology   433

3. Phagocytic system: factors, including host response, microbial virulence,


a. Consists primarily of PMNs and macrophages. ­genetics. Periodontal disease is essentially a disease
b. Phagocytes kill bacteria in two different ways: caused by bacteria. However, mere presence of bacteria
(1) Oxygen-dependent system: bacteria are de- does NOT preordain disease. Host defense mechanism of
stroyed in phagolysosome in the cell itself, by the individual is the balancing factor in maintenance of
lysosomal enzyme called myeloperoxidase. health. If BOTH bacterial load and host response are in
(2) Non-oxygen-dependent system: enzymes balance, NO disease occurs. If either a critical mass of
and other cationic proteins (e.g., neutral bacteria is reached or host response is somehow impaired,
proteases, acid hydrolases) in lysosomes of disease occurs. Etiology therefore should be discussed
PMNs and macrophages degrade bacteria from perspective of BOTH host response and bacteria.
after they have been ingested.
B. Specific host responses mediated by lymphocytes: Dental Biofilm
1. T cells provide cellular immune response: Dental biofilm (dental plaque) is a living, highly orga-
o2410 a. Release of cytokines that include potent prod- nized, and complex microbial ecosystem composed of
ucts, such as tumor necrosis factor-alpha (TNF- more than 300 species of bacteria embedded in a gelatinous
alpha) and interleukin-1-beta, that are capable matrix. Classified as either supragingival or subgingival.
of inducing bone resorption. Microflora associated with each type of periodontal dis-
2. B cells provide humoral immune responses by pro- ease classification as noted in the Human Oral Microbiome
ducing plasma cells that in turn produce specific Database (HOMD) are listed in each subcategory, as well
antibodies and immunoglobulins (Igs): as Table 13-1. Presence of dental biofilm (and associated
a. Prevent bacterial cell walls from attaching to calculus) is the MOST common reason for gingivitis (NOT
oral surfaces. more obscure causes such as vitamin C deficiency).
b. Trigger activation of complement system. • See Chapter 8, Microbiology and Immunology: micro-
c. Neutralize toxins and enzymes released by biology overview.
­bacteria. A. Types:
o15350 3. In addition, white blood cells (WBCs) produced 1. Supragingival dental biofilm:
by immune response to bacteria release a fam- a. Found coronal to the margins on clinical crowns
ily of enzymes called matrix metalloproteinases of teeth or within sulcus or pseudopocket:
(MMPs), which break down connective tissue, (1) With healthy gingiva:
such as that within the periodontium. (a) Composed MOSTLY of cocci and rods
that are gram positive and aerobic.
Etiology: Microbiology (b) Equal percentages of gram-positive
Strong relationship exists between microbial dental bio- cocci and Actinomyces; LESS than
film and periodontal disease. However, constant presence 15% are anaerobic.
of microorganisms in the oral cavity and its relation- (2) With early gingivitis: increased numbers of
ship to disease is a complex issue that involves MANY anaerobic, gram-negative cocci and rods.

Table 13-1  Main periodontal pathogens

Microorganism Gram stain reaction, motility Associations

Aggregibacter (previously Gram negative, nonmotile Chronic periodontitis (less than with
Actinobacillus) actinomycetem- Pg) and aggressive periodontitis,
comitans (Aa) both localized and generalized;
can invade tissue
Tannerella forsythensis (Tf) Gram negative, nonmotile Early stages of gingivitis and chronic
(previously Bacteroides forsythus [Bf]) periodontitis
Campylobacter rectus (Cr) Gram negative, motile Chronic periodontitis
Porphyromonas gingivalis (Pg) Gram negative, nonmotile Chronic periodontitis (most preva-
lent) and generalized aggressive
periodontitis
Prevotella intermedia (Pi) Gram negative, nonmotile Gingivitis with pregnancy and
chronic periodontitis
Treponema denticola (Td) NA, motile Chronic periodontitis; can invade
tissue
434   Saunders Review of Dental Hygiene

(3) With chronic gingivitis: increased numbers a­ ssociated with gingivitis and periodon-
of anaerobic, gram-negative, filamentous tal disease.
organisms, fusobacteria, spirochetes. (d) MOST virulent and detrimental portion
(4) Microbial microcosm increases in com- of dental biofilm to periodontium.
plexity by means of progressive, sequential (3) Unattached: free-floating in pocket; since
colonization on clean tooth: pocket has LESS mechanical stress, at-
(a) Formation of acquired pellicle from tachment is NOT vital; at base of pocket
salivary proteins on tooth surface. is MAINLY unorganized gram-negative
(b) Attachment by microorganisms (via rods and spirochetes, separated from inner
specific mechanisms), which colonize pocket lining by WBCs.
over time: first, gram-positive aerobic d. The pH is regulated MAINLY by saliva; from
cocci, then gram-negative anaerobic 6.75 to 7.25 (see more information in Chapter
rods, then motile organisms (i.e., spiro- 11, Clinical Treatment).
chetes). e. There is a consistent distance from dental bio-
b. Within pits and fissures: morphology allows film to alveolar bone that is never <0.05 mm
growth; MOSTLY Streptococcus mutans, Acti- and never >2.7 mm; microorganisms can cause
nomyces naeslundii. destruction of alveolar bone crest only when it
2. Subgingival dental biofilm: located in pockets is <3 mm away from dental biofilm.
apical to the crest of marginal gingiva within peri-
odontal pocket. Bacteria-Mediated Tissue Destruction
a. Diseased periodontium: MORE anaerobic be- Tissue destruction mediated by bacteria is caused by re-
cause subgingival area provides LESS oxygen lease of toxins and enzymes from these microorganisms.
tension, which is MORE conducive to anaero- A. Bacterial enzymes (hyaluronidase, collagenase, ribo-
bic growth. nuclease) affect host cells and intercellular (ground)
o2600 b. During active periodontal disease: MOSTLY an- substance; proteolytic and hydrolytic enzymes de-
aerobes at >90% of organisms present, typically stroy cells and increase permeability of epithelial and
with 75% of them gram-negative ­ organisms; connective tissues.
there are also large numbers of gram-negative, B. Bacterial cell walls of microorganisms that are re-
facultative anaerobic organisms. leased at the time of their destruction:
c. Divided into three zones (LOCATION LOCA- 1. Gram-negative: lipopolysaccharides (LPS, endo-
TION LOCATION—hey, real estate is real toxin):
­estate): a. Activate alternate pathway of immune complex
(1) Tooth-attached: attached along tooth sur- fixation process (via complement).
face from gingival margin almost to the JE b. Side reaction proteins are BOTH chemotactic
at base of pocket. and cytolytic (C3, C5, C5a).
(a) MOSTLY gram positive, although 2. Gram-positive: lymphotoxins (LT, exotoxin):
gram-negative cocci and rods present. a. Inhibit the chemotactic response of the host
(b) Associated with calculus formation, cells.
root caries, root resorption. b. Inhibit host cell phagocytosis.
(2) Epithelium-attached: loosely attached to c. Create resistance to killing and digestion within
inner lining. phagolysosomes of host cells.
(a) Layers closest to pocket lining have d. Release toxins that are cytolytic and/or cyto-
large numbers of motile, gram-nega- toxic.
tive, anaerobic microorganisms and
spirochetes. Etiology: Oral Contributing Factors
(b) Microorganisms are seen invading ul- Although dental biofilm has been recognized as MAIN
cerated inner pocket lining into lamina etiological factor in initiation and progression of gingivi-
propria and have been found on outer tis and periodontitis, other oral contributing factors play
portions of alveolar bone; MAINLY a role in retention of dental biofilm. These factors may be
involves Aggregatibacter (previously divided into local functional factors and local predispos-
Actinobacillus) actinomycetemcomi- ing factors.
tans (Aa) and Treponema denticola A. Local functional factors: weaken the PDL, widen-
(Td). ing the PDL space and thus promoting dental biofilm
(c) Relative proportions are related to accumulation; include missing teeth, malocclu-
nature of disease activity; MOSTLY sions, tongue thrusting and mouth breathing habits,
Periodontology   435

parafunctional habits such as chewing on pencils or ratio of calcium to phosphate and increased
other foreign objects, traumatic occlusion (discussed sodium.
next). (3) Normal mode of attachment to the tooth is
B. Local predisposing factors: harbor dental biofilm by means of salivary acquired pellicle; min-
microorganisms, responsible for promoting oral dis- eralization occurs within 24 to 72 hours,
ease. and maturation occurs in an average of
1. Include calculus, materia alba, dental stains, faulty 12 days.
restorations and overhangs, food impaction, un- (4) May form MORE tenacious attachment by
treated caries, tooth anatomy (e.g., furcas). penetrating cementum and/or mechanically
2. Supragingival and/or subgingival calculus is the locking into surface irregularities, making
MOST significant among this group of factors be- removal difficult (YES!).
cause it BOTH harbors dental biofilm and consists c. Rate of calculus formation varies among indi-
MAINLY of mineralized dental biofilm. viduals, may also vary among teeth; calculus
a. Supragingival calculus: mineralized supragin- may be light, moderate, or heavy.
gival dental biofilm from salivary minerals. 3. Food impaction: forceful wedging of food into
(1) Found coronal to margins on clinical periodontium by occlusal forces, which leads to
crowns of teeth, typically yellowish white; dental biofilm retention.
may darken with age or dietary staining. 4. Food debris:
(2) Found MOST commonly on lingual sur- a. Although rapidly cleared away from oral cavity,
faces of mandibular anteriors and on buc- tends remains on the teeth and oral mucosa.
cal surfaces of maxillary first and second b. Cleared from mouth by salivary flow and me-
molars; BOTH adjacent to salivary gland chanical action of tongue, cheeks, and teeth.
ducts that promote mineralization of dental c. Adherence varies with food composition.
biofilm. 5. Materia alba:
(3) Found LEAST on occlusal surfaces because a. Yellow or grayish white acquired bacterial coat-
of mastication. ing; soft, sticky, LESS adherent than dental bio-
(4) Composed of MAINLY inorganic (70% film; can be flushed away with water.
to 90%) and some organic (10% to 30%) b. Composed of microorganisms, desquamated
­content. epithelial cells, leukocytes, salivary proteins,
(a) Inorganic mineral component is made lipids.
up MAINLY of calcium phosphate 6. Dental stains include a variety of extrinsic stains
(75.9%) and calcium carbonate (3.1%), produced by foods, tobacco products, poor oral hy-
with traces of magnesium, sodium, po- giene; may attach to external tooth surfaces, creat-
tassium, fluoride, zinc, strontium. ing rough surface for retention of dental biofilm
(b) Organic component is mixture of pro- (see Chapter 11, Clinical Treatment).
tein-polysaccharide complexes, des- 7. Faulty dentistry (iatrogenic factor) plays IMPOR­
quamated epithelial cells, leukocytes, TANT role in dental biofilm retention; includes
carbohydrates, lipids, glycosamino- poorly fitted margins of dental prostheses, over-
glycans, various types of microorgan- contoured crowns, rough crown margins, faulty
isms. or overhanging restorations, clasps of partial
(5) Primary crystalline structure is MAINLY ­dentures.
calcium hydroxyapatite, Ca10(PO4)6(OH)2;
includes smaller amounts of octocalcium Etiology: Occlusal Trauma
phosphate, whitlockite, brushite. It is important for the dental hygienist to recognize BOTH
b. Subgingival calculus: mineralized subgingival normal and abnormal occlusal form and function in pro-
dental biofilm from GCF’s minerals, which are gression of periodontal disease. Occlusal disharmonies
originally from associated gingival tissue blood can have a profound influence on periodontal disease risk
supply. and are considered local contributing factors. Recogniz-
(1) Located in sulcus and pockets apical to ing and recording signs and symptoms of occlusal dys-
the crest of marginal gingiva; typically is function and then providing referrals for diagnosis and
dense, hard, dark brown or black, firmly at- treatment are important components of dental hygiene
tached to root surface. care.
(2) Composition differs slightly from that of • See Chapters 4, Head, Neck, and Dental Anatomy: u0210
supragingival calculus, MAINLY by higher occlusion discussion; 6, General and Oral Pathology:
temporomandibular joint disorder (TMD).
436   Saunders Review of Dental Hygiene

A. Hyperfunction: slightly increased forces that result 1. Hypofunction: nonpathological form of adapta-
in nonpathological form of adaptation. tion characterized by slight decrease in occlusal
1. Associated histological changes characterized by forces; slight reduction in BOTH collagen fibers
increase in number of fiber bundles in PDL; result and bone deposition occurs, with narrower PDL,
in increased width of PDL and increased thickness but is NOT seen on radiographs.
of alveolar bone. 2. Disuse atrophy: pathological process when ALL
2. CANNOT be noted clinically; NOT associated occlusal forces are removed; involves extreme
with tooth mobility. weakening of the supporting structures.
B. Occlusal trauma: pathological alterations or adap- a. Requires absence of occlusal antagonist; results in
tive changes that develop in periodontium as result of obvious reduction in BOTH fibers and bone, with
undue forces generated by muscles of mastication. radiographically detectable narrowed PDL space.
1. Defined in two ways: b. Also radiographic evidence of decreased bone
a. Damage to the periodontium caused directly by trabeculation in cancellous bone.
stress. c. Always accompanied by increased tooth mobility;
b. Damage caused indirectly by the teeth of the tooth supererupts into space of opposing arch.
opposing jaw.
2. Effect of occlusal forces on periodontium is ­depend- Pathogenesis of Periodontal Disease
ent on magnitude, direction, duration, frequency. Although gingivitis does NOT necessarily progress to
C. Injury results when occlusal forces exceed adaptive periodontitis, periodontitis is always preceded by gingivi-
capacity of the periodontium. tis. Unknown when transition occurs clinically. Diagnosis
1. Associated histopathological features result from of periodontitis is made when inflammation extends into
areas of pressure and tension between the tooth epithelial attachment (EA, attachment apparatus), result-
root and the alveolar bone; MORE severe level of ing in attachment loss and subsequent periodontal pocket
changes as noted in hyperfunction. formation. Progression of disease occurs in four distinct
2. Can be noted clinically. histological stages; first three result in gingivitis, and
D. Clinical signs and symptoms of occlusal trauma: fourth manifests as periodontitis. This progression has
1. Evidence of wear facets and/or a history of brux- certain clinical pathogenic ramifications.
ism or clenching; possibly abfraction; MAINLY A. Histological pathogenesis:
premolars. 1. Initial lesion (stage I):
2. Tilted or missing teeth and tooth mobility. a. Response of tissues to dental biofilm occurs in
3. Occlusal interferences in excursive mandibular FIRST 2 to 4 days after biofilm invasion; con-
movements. sidered subclinical.
4. Radiographic evidence of widened PDL space of b. Involves initial vasoconstriction followed by sub-
­involved teeth, with loss of continuity of lamina sequent vasodilation and margination; emigration
dura. and migration of PMNs into lamina propria result
5. Radiographic evidence of increased cementum at in slight alteration of JE and increase in GCF.
apices of involved teeth (hypercementosis) and/or 2. Early lesion (stage II): acute gingivitis stage, which
root resorption. occurs within 4 to 7 days after insult.
E. Types of occlusal trauma: either primary or second- a. Results in further invasion of JE, causing ulcer-
ary; may result from hypofunction or disuse atrophy. ation of SE and JE, destruction of connective
1. Primary occlusal trauma: excessive occlusal tissue fibers.
forces applied to teeth with healthy periodontium; b. Involves appearance of chronic inflammatory
alveolar bone levels are supportive. cells, such as macrophages and lymphocytes.
2. Secondary occlusal trauma: 3. Established lesion (stage III): chronic stage of
a. Excessive occlusal forces applied to teeth with inflammation, which occurs 14 days after initial
diseased periodontium; alveolar bone levels are ­insult.
already deficient in support. a. MOSTLY plasma cells, bacterial enzymes, and
b. MOST common in cases of chronic periodon- cell by-products such as collagenase that are de-
titis in which alveolar bone support may be structive to the collagen in periodontal ­tissues.
inadequate to withstand forces of occlusion b. May persist from weeks to years and remain
(whether normal or excessive). stable or may progress to advanced lesion.
F. Hypofunction and disuse atrophy: occlusal forces 4. Advanced lesion (stage IV): occurs ONLY when
are either decreased or removed entirely; decrease in inflammation invades supporting periodontium
­collagen fiber production in PDL and in alveolar bone (PDL and alveolar bone); marks conversion of
formation. gingivitis to periodontitis.
Periodontology   437

a. Depends on host response and presence of spe- (b) Infrabony pocket: base of pocket is
cific bacterial species. ­apical to level of alveolar bone, with
b. Coronal portion of JE becomes detached and vertical bone loss; three distinct types:
migrates along root surface. one-walled, two-walled, three-walled
c. Cementum is altered as it becomes exposed by defects (Figure 13-3).
inflammatory bacterial products, and superficial f. Superficial gingival tissues range from inflamed
layer contains LPS. with its fragility, bleeding, redness to fibrotic
d. FIRST PDL fibers involved are alveolar crest fi- with its whitened and hardened ­appearance.
bers, then later deeper fibers become involved; g. Periods of inactivity (remission) alternating
transseptal (interdental) fibers are NOT involved with periods of activity (exacerbation).
until the tooth is almost lost, they realign more
apically as inflammation occurs, protecting the CLASSIFICATION OF PERIODONTAL DISEASES  
deeper tissues (noted in SAME levels of mobil- Although numerous classification systems exist for peri-
ity that are affected first). odontal diseases, MOST commonly used system was devel-
e. Involves bone destruction initially in the alveo- oped by the 1999 International Workshop for Classification
lar crest that then spreads by blood vessels and of Periodontal Diseases and Conditions sponsored by AAP
bone marrow deeper into the alveolar bone. (see CD-ROM). Classification has numerous subcatego-
f. Destruction results in the response of the peri- ries (especially for gingivitis); only major categories are
odontium by forming either fibrotic tissue or discussed here (Table 13-2). Refractory is NOT a separate
granulation tissue. entity from other forms of periodontitis; term is applied in
(1) Fibrous tissue (scar or cicatricial tissue)
with its excess collagen formation.
(2) Granulation tissue is young collagen tissue
that bleeds easily.
B. Clinical pathogenesis:
1. Gingivitis: inflammation of the gingival tissues.
a. Caused by invasion of JE and SE by dental bio-
film; with vasostagnation, retained tissue fluids
and eventual fibrosis produce gingival enlarge-
ment or edema.
b. Results in gingival pocket (pseudopocket)
where pocket formation is NOT accompanied One-walled
by apical migration.
2. Periodontitis: inflammation of the periodontium.
a. Occurs with apical migration, results in peri-
odontal pocket.
b. As inflammatory process destroys gingival and
PDL fibers, apical proliferation and migration
of JE and SE occur.
c. Disease becomes self-perpetuating with formation
of deeper pockets because dental biofilm, primary
etiological agent, can grow and mature in MORE
Two-walled
anaerobic environment of the deeper pocket.
d. Growth of periodontal pathogens is encouraged
as removal by patient becomes increasingly dif-
ficult because of depth of pocket; if allowed to
persist, chronic infection causes severe destruc-
tion and eventual tooth loss.
e. Pocket continues its progression apically; stim-
ulation of alveolar bone resorption is part of the
inflammatory process.
(1) Classifications of periodontal pockets:
(a) Suprabony pocket: base of pocket is Three-walled
coronal to level of alveolar bone, with Figure 13-3  Infrabony pocket with a one-walled, two-
horizontal bone loss. walled, and three-walled defect.
438   Saunders Review of Dental Hygiene

Table 13-2  American Academy of Periodontology (AAP) classification of periodontal diseases and conditions
(abbreviated version)

Classification Conditions

Periodontal health None


Gingival diseases A. Dental plaque (biofilm)-induced gingival
diseases
B. Nonplaque (biofilm)-induced gingival lesions
Chronic periodontitis* (slight: 1-2 mm attachment loss; A. Localized
moderate: 3-4 mm attachment loss; severe: >5 mm B. Generalized (>30% of sites are involved)
attachment loss)
Aggressive periodontitis* (slight: 1-2 mm attachment loss; A. Localized
moderate: 3-4 mm attachment loss; severe: >5 mm B. Generalized (>30% of sites are
attachment loss) involved)
Periodontitis as manifestation of systemic diseases A. Associated with hematological disorders
B. Associated with genetic disorders
C. N ot otherwise specified
Necrotizing periodontal diseases A. Necrotizing ulcerative gingivitis
B. N
 ecrotizing ulcerative periodontitis
Abscesses of the periodontium A. Gingival abscess
B.  Periodontal abscess
C. P ericoronal abscess
Periodontitis associated with endodontic lesions Combined periodontic-endodontic lesions
Developmental or acquired deformities and conditions A. Localized tooth-related factors that modify or predis-
pose to plaque (biofilm)-induced gingival diseases and
periodontitis
B. Mucogingival deformities and conditions around teeth
C. Mucogingival deformities and conditions on edentu-
lous ridges
D. Occlusal trauma

*Attachment loss and not probe readings determines the clinical attachment levels (CAL) with/without recession and edema.

conjunction with other forms to mean that the case does (1) Gingivitis MAINLY associated with pu-
NOT readily respond to treatment (e.g., refractory chronic berty and pregnancy:
periodontitis, refractory aggressive periodontitis). (a) Caused by dental biofilm; hormonal al-
terations tend to exaggerate inflamma-
Gingival Diseases tory response.
Forms of gingival diseases include dental plaque (bio- (b) During puberty: poor diet and poor oral
film)-induced and nonplaque (biofilm)-induced gingivi- hygiene may also exaggerate response;
tis. Common examples are discussed. affects BOTH males and females.
A. Dental plaque (biofilm)-induced gingivitis (manage- (c) During pregnancy: increased levels of
ment is discussed later): progesterone have been attributed to in-
1. Characteristics: creased risk for gingivitis and presence
a. MOST common form of gingivitis and found to of Prevotella intermedia (Pi).
some extent in MAJORITY of population. (i) Begins in second to third month
b. Exhibits bleeding on probing (BoP); MOST and MOST severe in second and
significant feature. third trimesters.
c. Painless condition; also typically manifested as red- (ii) With localized interdental papil-
ness and swelling of gingival margins and interden- lae enlargements, pyogenic granu-
tal papillae (i.e., rolled margins, rounded interdental lomas (pregnancy tumors); red or
papillae, flabby tissues, loss of stippling), presence magenta, smooth, shiny; disappear
of gingival pockets (pseudopockets) possible. at the end of pregnancy.
2. Subcategories: (d) Clinical characteristics:
a. Gingival diseases modified by systemic factors: (i) Red to bluish red gingiva caused by
endocrine and systemic and blood dyscrasias. increased vascularity.
Periodontology   439

(ii) Enlarged, edematous gingiva (es- (iii) Tendency to bleed spontaneously, with
pecially with pregnancy), with in- gingival pockets (pseudopockets).
creased tendency to bleed. B. Nonplaque (NOT biofilm)-induced gingivitis:
(2) Gingivitis associated with leukemia: dis- 1. Gingival diseases of bacterial, viral, or fungal origin:
ease of blood-forming tissues characterized a. Primary herpetic gingivostomatitis: oral mani­
by production of excessive numbers of im- fetation of a primary infection with HSV-1; occa-
mature WBCs. sionally HSV-2 (genital herpes) (see Chapter 8,
(a) Regardless of the form, dental biofilm Microbiology and Immunology) (Figure 13-4).
is primary etiological agent; histologi- b. Mainly present in infants and children; however,
cal changes associated with leukemia becoming MORE common among young adults
modify and aggravate inflammatory re- infected with HSV-2; symptomatic in ONLY
sponse to local irritants. 10% to 20% of persons infected with HSV-1.
(b) Clinical characteristics: c. Characteristics: fever, malaise, headache, irrita-
(i) Purplish blue color caused by blood bility, lymphadenopathy.
stagnation, with enlargement of (1) Oral lesions that begin as small, yellow vesi-
marginal and interdental papillae in cles and coalesce to form larger, round ulcers
form of tumorlike masses. with gray centers and bright red borders.
(ii) Moderately firm to friable tissues, with (2) Lesions may appear on any keratinized oral
shiny surfaces that may have ulcer- mucous membranes, including lips, tongue,
ations, necrosis, and pseudomembrane. gingival tissues, buccal mucosa, palate.
(iii) Spontaneous bleeding (in MOST cases), (3) Serious to extreme pain that restricts eating
with gingival pockets (pseudopockets). and drinking and may threaten the patient’s
b. Gingival diseases modified by medications and health.
malnutrition. d. Treatment: palliative because disease is viral
(1) Gingivitis associated with vitamin C defi- and typically runs course in 7 to 10 days.
ciency: severe cases result in scurvy (scorbu- (1) Extremely infectious disease; poses major
tic gums); however, this vitamin deficiency risk to clinician because transmission of vi-
alone CANNOT cause inflammation (take rus through inadvertent puncture results in
that, Captain Jack Sparrow!). herpetic whitlow and eye infection (result
(a) Exaggerated because a deficiency of this can be blindness); MUST follow all infec-
vitamin creates increased tendency for tion protocols with standard precautions.
gingival hemorrhage, degeneration of col- (2) Involves patient and parent education in
lagen fibers, edema of gingival tissues. gentle dental biofilm removal and proper
(b) Clinical characteristics: diet; fluid intake to prevent dehydration is
(i) Bluish red gingiva, with enlarge- IMPORTANT.
ment of marginal gingiva. 2. Gingival diseases of genetic origin:
(ii) Soft, friable, smooth, and shiny gin- a. Familial (hereditary) gingival enlargement (id-
gival surfaces, with possible pseudo- iopathic gingival hyperplasia): rare hyperplas-
membrane, if necrosis has occurred. tic disease with unknown etiology.
(1) Enlarged marginal, interdental, attached
gingival tissues; affects facial and lingual
surfaces of BOTH arches.
(a) Affected tissues are pink, firm, resilient,
bulbous, and fibrotic without bleeding;
tissues may obscure occlusion.
(b) May exhibit inflammatory changes in
response to local irritants.
3. Gingival manifestations of systemic condition:
a. Allergic gingivitis: hypersensitivity or abnor-
mal response of tissues to specific agents.
(1) May be oral manifestation of systemic aller-
gic response, if sensitivity is to food or drug.
(2) Also may manifest as localized contact al-
lergy to dentifrices, mouthwashes, topical an-
Figure 13-4  Acute herpetic gingivostomatitis. esthetics, or other dental therapeutic agents.
440   Saunders Review of Dental Hygiene

(3) Antigen (and NOT dental biofilm) is MAIN (d) MORE common in younger patients,
etiological agent. rarely affects edentulous spaces; how-
(4) Redness, soreness, gingival necrosis, edema, ever, increased levels with poor oral
ulceration, possible vesicle formation. hygiene and with increased dental
b. Desquamative gingivitis, gingivitis associated biofilm.
with dermatoses: rare chronic condition that (2) Cyclosporine, immunosuppressive drug
involves interdental, marginal, and attached used to prevent organ transplant rejection
gingival tissues. and to treat type 1 DM and other autoim-
(1) Recently linked to several dermatological mune disorders; drug causes enlargements
diseases and drugs, although exact nature in one third of adults and in even MORE
unknown. children.
(a) Diseases include pemphigus, pemphigoid, (3) Calcium channel blocking drugs, such as
be­nign mucous membrane pemphigoid, li- diltiazem (Cardizem), nifedipine (Procar-
chen planus; MAINLY affects older woman. dia), and verapamil (Isoptin), treatment of
but so do these other disorders (however, angina pectoris; produce gingival enlarge-
NOT direct result of menopause). ments SIMILAR to phenytoin (Dilantin) in
(b) Associated drugs (stomatitis medica- some individuals; more uncommon with
mentosa) include sulfonamides, gold amlodipine (Norvasc).
salts, arsphenamine, aminopyrine, anti-
biotics, barbiturates, salicylates. Chronic Periodontitis
(2) Slight, moderate, or severe; redness of mar- Chronic periodontitis is the MOST common form of
ginal and attached gingival tissues, with periodontitis (“garden variety,” that is why we pick at
periods of exacerbation and remission. it!). Although may begin subclinically in adolescence,
(a) As severity increases, localized areas of does NOT become evident until after age 35. Because
epithelium begin to peel away, expos- of chronic nature, progresses slowly; management is dis-
ing painful connective tissues. cussed later.
(b) In MOST severe form, irregular areas of A. Characteristics: severity is directly related to amount
gingiva are fiery red, smooth, shiny; ex- of hard and soft deposits on tooth surfaces.
posed connective tissues contain blisters 1. NOT associated with abnormal host or systemic
that bleed and are extremely painful. diseases.
(3) Degenerative condition (NOT associated 2. Presence of periodontal pockets, bone loss, even-
with dental biofilm) with ONLY secondary tual tooth mobility.
inflammatory reaction. 3. MOST commonly exhibits horizontal interdental
4. Gingival diseases modified by medications: bone loss, eventually progresses to interradicular
a. Noninflammatory gingival enlargement: may areas (furcations), possibly with a FEW areas of
be produced by certain drugs, with or without vertical interproximal bone loss.
presence of inflammatory condition; NOT as- 4. Degree of periodontal disease activity is directly
sociated with dental biofilm, although inflam- related to amount of bone and PDL destruction;
mation exists (see earlier discussion). determined MORE accurately by measures of
(1) Phenytoin (Dilantin), anticonvulsant drug: clinical attachment level (CAL) than by measures
causes enlargements in over half of pre- of probing depth (Figure 13-5).
scriptions given to patients (Figure 13-1); B. Considered mixed infection, since involves sev-
other anticonvulsants rarely cause it. eral bacterial species (Table 13-1): Porphyromo-
(a) Clinical characteristics: generalized en- nas gingivalis (Pg) typically MOST prevalent;
largement of marginal and interdental with Aggregatibacter (previously Actinobacillus)
gingiva, which causes increased sulcular ­actinomycetemcomitans (Aa); Tannerella forsyth-
depths (gingival pockets, pseudopock- ensis (Tf) (previously Bacteroides forsythus [Bf]);
ets); interdental areas become MORE Prevotella intermedia (Pi); Fusobacterium nuclea-
enlarged and often obscure occlusion. tum (Fn); Campylobacter rectus (Cr); Treponema
(b) Enlarged tissues appear firm, fibrotic, denticola (Td) (Bad boys, bad boys, whacha you
pale pink, and resilient and have less gonna do?).
tendency to bleed. C. Clinical signs, with periods of BOTH exacerbation
(c) Tissues also appear lobed, and although and remission:
enlargement is generalized, frequently 1. Chronically inflamed gingiva, with color that var-
MORE pronounced in anteriors. ies from red to bluish to normal.
Periodontology   441

“Probing Depth 6 mm…”


This statement provides no information
concerning attachment loss or how
much attachment remains on a tooth
(gray columns):
(�3)
A 6 mm probing depth
6
� 3 mm pseudopocket 0
� 3 mm true attachment loss 6 3
3
B 6 mm probing depth
6
� 6 mm true attachment loss 6
C 3 mm gingival recession
9
� 6 mm probing depth
� 9 mm true attachment loss
12
A B C
15

Figure 13-5  Attachment loss determined with measurement of clinical attachment levels with differing gingival margin
levels.

2. Gingiva that varies in consistency from edematous D. Categorized into three MAIN categories based on
to fibrotic, with enlarged appearance. severity, then subcategorized again as localized or
3. Contour that is rounded at the margins and typically generalized, which indicates more or less than >30%
blunted interdentally, exhibiting a loss of stippling. involvement:
4. Evidence of attachment loss and periodontal pock- 1. Slight: slight loss of bone (involving ONLY alveolar
ets, with tendency for BoP. crest) and PDL, with 1 to 2 mm of attachment loss.
5. Radiographic signs of bone loss that may include 2. Moderate: more advanced loss of bone and
lack of definition of the lamina dura, fuzzy crestal PDL, with 3 to 4 mm of attachment loss; may
lamina, horizontal or angular (vertical) bone loss, be accompanied by tooth mobility and furcation
or radiolucencies of bifurcations and trifurcations ­involvement.
(furcation involvement). 3. Severe: further progression of disease; results in
6. Rough root surfaces, calculus, exudate that may be advanced loss of bone and PDL, with >5 mm of
purulent (pus). attachment loss; accompanied by increased tooth
7. Tooth mobility, which varies from none to extensive. mobility and furcation involvement.

CLINICAL STUDY   

Age 44 YRS SCENARIO

Sex ☐  Male   ☒  Female A full-mouth series (FMX) of radiographs


is taken in addition to a complete peri-
Height 5’8” odontal evaluation. Examination reveals
Weight 275 LBS moderate levels of dental biofilm at the
gingival margins, moderate gingivitis,
BP 115/75 heavy subgingival and supragingival cal-
culus, tobacco staining. Molars #3, #19,
Chief Complaint “I am afraid of the dentist but I am so em- #30, and #31 are badly decayed but are
barrassed about my teeth.” no longer painful. Radiographs indicate a
loss of crestal bone throughout posterior
Medical History Two caesarean sections
Past cigarette smoker, quit 2 years ago areas of the mouth. Probe readings are
3 to 4 mm in anterior sextants and 4 to
Current Medications None 6 mm in posterior areas (with attachment
loss at 3 to 4 mm). No recession is noted.
Social History Waitress, employed after being full-time
mom for 15 years
Restaurant employer offers dental insurance
442   Saunders Review of Dental Hygiene

1. Identify a likely diagnosis for the patient’s condition. loss (slight, moderate, severe); then again subcatego-
What does loss of crestal bone indicate? What AAP rized into localized and generalized forms:
classification fits this case? How do you measure at- 1. Localized (LAP): circumpubertal onset (Figure
tachment loss? 13-6).
2. Identify factors contributing to her periodontal condi- a. Periodontal damage localized to permanent first
tion. molars and incisors; however, atypical patterns
3. Discuss treatment options. of affected teeth are possible.
4. What is the prognosis for this disease? b. Associated with periodontal pathogen Aggre-
gatibacter (previously Actinobacillus) actino-
1. MOST likely diagnosis for patient’s condition is peri- mycetemcomitans (Aa), with PMN dysfunction
odontitis, based on loss of crestal bone, age, amount of (function abnormalities).
deposits, lack of routine dental care. The AAP classifi- c. HIGH serum antibody response to infecting
cation is generalized (>30% of sites involved) moder- agents.
ate chronic periodontitis because attachment loss is at 3 2. Generalized (GAP): usually affects people under
to 4 mm, with slight horizontal bone loss evident on ra- 30 years of age, but may be older (Figure 13-7).
diographs in the posterior sextants. With no recession, a. Generalized interproximal attachment loss af-
attachment loss can be determined by clinical attach- fecting at least three permanent teeth other than
ment levels, through measuring pocket depth and then first molars and incisors; occurs in pronounced
subtracting measurement of gingival margin to CEJ. episodic periods of destruction.
2. Factors contributing to periodontal condition include b. Associated with periodontal pathogens Aggre-
tobacco use (smoking), inadequate oral hygiene, lack gatibacter (previously Actinobacillus) actino-
of professional dental care. mycetemcomitans (Aa) and Porphyromonas
3. Appropriate treatment should include thorough peri- gingivalis (Pg), with PMN dysfunctions; MORE
odontal debridement with local anesthetic coverage similar to the microflora of chronic periodontitis.
(preferably by quadrant), oral hygiene instruction, to- c. LOW serum antibody response to infecting
bacco cessation, extraction or restoration of decayed agents.
teeth. Patient should return in 4 to 6 weeks for reevalu- C. Management: given its rapid course and serious con-
ation of periodontal health. Any areas with continued sequences, patients (or guardians, as appropriate)
bleeding should be evaluated and undergo debride- SHOULD be informed of the disease process, thera-
ment if needed to promote healing. Homecare should peutic alternatives, potential complications, expected
be assessed, and additional instruction should be given results, responsibility in treatment.
as needed.
4. Prognosis is the prediction of the duration, course, and
termination of a disease and the likelihood of its re-
sponse to treatment. Prognosis for chronic periodonti-
tis is good, if patient refrains from smoking, practices
good homecare, and complies with appropriate sup-
portive periodontal therapy (typically involving oral
prophylaxis every 3 months).

Aggressive Periodontitis
Diagnosis of aggressive periodontitis is made on clini-
cal, radiographic, and historical findings that show rapid
attachment loss and bone destruction, possible familial
aggregation of disease as described in APP “Parameter on
Aggressive Periodontitis” (2000) (see CD-ROM).
A. Characteristics: systemically healthy, EXCEPT for
periodontal disease.
1. Periodontal tissue destruction greater than would
be expected given level of local factors.
2. MORE periodontal pathogens (see below), as well
as phagocyte abnormalities and increased produc-
tion of prostaglandin E2 and interleukin-1b.
B. Categories are SIMILAR to chronic periodontitis in
number of teeth involved and severity of attachment Figure 13-6  Localized aggressive periodontitis (LAP).
Periodontology   443

1. Consequences of NO treatment should be ex-


plained; failure to treat appropriately can result
in progressive and often rapid loss of periodontal
supporting tissues, may have adverse effect on the
prognosis, and could result in tooth loss.
2. Given this information, patients (or guardians, as
appropriate) should then be able to make informed
decisions regarding periodontal therapy.
3. Therapy consists of medical consult for systemic
factors and 2-week regimen of antibiotics in con-
junction with conventional nonsurgical periodon-
tal therapy; periodontal surgery may be required to
restore periodontal health.
Figure 13-7  Generalized aggressive periodontitis (GAP).

Clinical study  

Age 14 YRS SCENARIO

Sex ☐ Male   ☒  Female Patient of record who is scheduled on


time for her 6-month oral prophylaxis.
Height 5’6” Her gingival tissues continue to ap-
Weight 124 LBS pear healthy, and her mouth is dental
biofilm free. She brushes her teeth
BP 98/58 twice daily and flosses four to five
times a week. Probing depths are 2 to
Chief Complaint “I never had deep pockets like these 3 mm throughout her mouth, with
before. What should I do?” the exceptions of teeth #3, #14, and
#8, which have readings of 6, 6, and
Medical History Parents both have dentures
Always seems to have a cold or flu 9, respectively. Bleeding on probing
(boP) is noted in the pockets only.
Current Medications None

Social History High school student who is interested


in a dental career

1. Identify a likely diagnosis for the patient’s condition. 2. Contributing factors to periodontal condition could
How might radiographs assist in the diagnosis? include elevated levels of Aggregatibacter (previ-
2. Identify factors that might have contributed to her ously Actinobacillus) actinomycetemcomitans (Aa)
periodontal condition. or ­Porphyromonas gingivalis (Pg), phagocyte abnor-
3. Discuss treatment options. malities, increased production of prostaglandin E2 and
4. What is the prognosis for this disease? interleukin-1b.
3. Condition typically is treated with a 2-week regi-
1. Most likely diagnosis for the patient’s condition is local- men of antibiotics in conjunction with conventional
ized aggressive periodontitis, which may be indicated by nonsurgical periodontal therapy; medical consult is
the presentation of vertical bone loss on radiographs of imperative. Periodontal surgery may be required to
teeth #3, #14, and #8 and probing depths of these teeth. restore periodontal health.
Diagnosis of aggressive periodontitis is based on clini- 4. Prognosis is the prediction of the duration, course, and
cal, radiographic, and historical findings that show rapid termination of a disease and the likelihood of its re-
attachment loss and bone destruction, possible familial sponse to treatment. Prognosis good throughout most
aggregation of disease. Except for periodontal disease, of the mouth, except for teeth #3, #14, and #8, because
patients are systemically healthy. The other feature that no bleeding is noted and sulcus depths are normal in
may be present is periodontal tissue destruction greater the rest of the mouth. Prognosis for the areas with
than would be expected given level of local factors pocketing is fair because significant amounts of bone
(healthy patient and mouth, with no dental biofilm). loss already have occurred and there is BoP, showing
444   Saunders Review of Dental Hygiene

active inflammation. However, compliance with home­ 1. Characteristics: sudden onset, fetid metallic breath
care and professional dental care should contain bone (fetor oris), cratered (punched-in) interdental
loss indefinitely. ­papillae, ulcerative necrotic gingival sloughing
(pseudomembrane), with spontaneous gingival
Periodontitis as Manifestation of Systemic bleeding; possible lymphadenopathy, fever.
­Diseases 2. Has been associated with preexisting gingival in-
Systemic diseases that affect immune function, inflam- flammation, severe nutritional deficiency, smok-
matory response, and tissue organization can modify the ing, preexisting systemic disease (e.g., Down
onset and progression of all forms of periodontal disease. syndrome, leukemia, HIV/AIDS, leukopenia, ane-
See earlier discussion of systemic factors with DM, HIV/ mia, ulcerative colitis), stress, fatigue.
AIDS, neutrophil abnormalities. 3. Associated with specific combinations of microbes
• See Chapter 6, General and Oral Pathology: Down syn- that appear to become pathogenic ONLY when the
drome, neutropenias, diabetes mellitus. host defenses are diminished; also, spirochetes and
A. Includes subcategories for hematological disorders fusiform bacillus may be present.
­(acquired neutropenia, leukemias, and other), genetic 4. Management: USUALLY responds rapidly to re-
disorders (familial and cyclic neutropenia, Down syn- duction of oral bacteria by a combination of per-
drome, leukocyte adhesion deficiency syndromes, Pa- sonal dental biofilm control and professional
pillon-Lefèvre syndrome, Chédiak-Higashi syndrome, debridement.
histiocytosis syndromes, glycogen storage disease, in- a. If lymphadenopathy and/or fever accompanies
fantile genetic agranulocytosis, Cohen syndrome, Ehlers- oral symptoms, systemic antibiotics may be
Danlos syndrome types IV and VIII, hypophosphatasia, ­indicated.
and other disorders not otherwise specified (NOS). b. Use of chemotherapeutic rinses by the pa-
1. Papillon-Lefèvre syndrome: inherited autosomal tient may be beneficial during initial treatment
recessive disease characterized by hyperkeratotic stages.
skin lesions of palms, soles, knees, elbows; may c. Patient counseling should include instruction
cause severe destruction of periodontium; onset on proper nutrition, oral care, appropriate fluid
occurs after age 4. intake, tobacco cessation, if needed.
2. Chédiak-Higashi syndrome: rare disease that af- d. Comprehensive periodontal evaluation should
fects production of cellular organelles; produces follow resolution of acute condition.
partial albinism, slight bleeding disorders, recur- e. After acute inflammation of lesion is resolved, ad-
rent bacterial infections, including periodontitis. ditional intervention may be indicated to prevent
3. Hypophosphatasia: rare familial skeletal condition disease recurrence or to correct resultant soft tissue
characterized by rickets, poor cranial bone forma- deformities; needs 3-month PM ­appointment.
tion, premature loss of both primary and perma- B. NUP: rapid necrosis and destruction of the gingiva,
nent dentitions. PDL, alveolar bone (Figure 13-8).
B. Category acknowledges that management of peri- 1. Usually represents extension of NUG with low-
odontal disease should be carried out in conjunction ered host resistance; overall superficial appearance
with management of associated systemic disease; it is SIMILAR to NUG but MORE severe involvement
expected that other diseases will eventually be listed of the interdental papillae (punched out).
as more is known about systemic diseases and their
relationship to periodontal disease.

Necrotizing Periodontal Diseases


Necrotizing periodontal diseases (NPD) include nec-
rotizing ulcerative gingivitis (NUG) and necrotizing
ulcerative periodontitis (NUP). BOTH appear to be op-
portunistic infections resulting from diminished systemic
resistance to bacterial infection and may differ ONLY in
terms of tissue, with NUP extending deeper into peri-
odontium. NOT communicable.
A. NUG: associated with specific bacterial accumula-
tions occurring with LOWERED host resistance (im-
munocompromised, such as with PMN chemotaxis
and phagocytic defects), as well as in undernourished
children (kwashiorkor). Figure 13-8  Necrotizing ulcerative periodontitis (NUP).
Periodontology   445

2. LACK of conventional periodontal pockets be­ 5. Management:


cause of removal of tissue by ulcerative necro- a. Involves debridement, which may be combined
sis. with irrigation with antiseptics (e.g., povidone-
3. Has been reported among BOTH HIV-positive and iodine or diluted 3% hydrogen peroxide) to be
HIV-negative individuals, but true prevalence is used twice daily, antimicrobial oral rinses (e.g.,
unknown. chlorhexidine), administration of systemic anti-
a. With HIV/AIDS-related NUP, presence of nec- biotics (such as metronidazole [Flagyl]).
rotizing stomatitis involving vestibules and b. Later appointments can occur within 1 to 2 days,
palate, with infection extending into bone, pro- and with significant improvement MORE thor-
ducing deep, craterlike lesions (stoma), with ough deposit removal can begin; need to stop
sequestra (bone fragments). chlorhexidine rinses because of staining and cal-
b. May become gangrenous stomatitis (noma). culus buildup and to stop hydrogen peroxide rinses
c. Antibiotic therapy should be used with caution because of development of black hairy tongue.
in HIV-positive patients because of possibility 6. Patient counseling should include instruction on
of inducing opportunistic infections; SHOULD proper nutrition, oral care, appropriate fluid intake,
be seen monthly because oral manifestations tobacco cessation, if needed.
often develop rapidly and are extremely de- 7. Comprehensive periodontal evaluation should fol-
structive; need medical consult. low resolution of the acute condition; need to set
4. MAIN microorganisms are spirochetes and gram- up periodontal maintenance (PM, recare) appoint-
negative organisms. ments to prevent recurrence.

clinical study  

Age 19 YRS SCENARIO

Sex ☒  Male   ☐  Female The patient arrived for an emergency appointment


at his general dentist. After questioning, he indicates
Height 6’2” that he has had a fever and has been feeling run
Weight 175 LBS down since he completed his final exams 2 days ago
and he is back to smoking again instead of eating.
BP 105/68 During visual examination, his gingiva appears fiery
red and swollen. The interdental papillae are
Chief Complaint “My mouth is so sore and I am beat.” grayish white and ulcerated. A strong malodor is
noted when he opens his mouth.
Medical History Cigarette smoking, one pack

Current Medications None

Social History College student, runs track

1. Identify factors that have contributed to the patient’s drinking, as well as inadequate nutrition and dental
periodontal condition. biofilm removal and lack of sleep.
2. Identify a likely diagnosis for his condition. 3. He needs a series of treatments. Immediate concern
3. Discuss treatment options. should be to gently debride affected areas. Accom-
4. What is the prognosis for this disease? plished with ultrasonic and/or sonic scalers to gently
flush the area with oxygenated water. Topical and/or
1. Factors that have contributed to oral disease include local anesthetics may be needed if debridement causes
stress and inadequate oral hygiene (spirochetes and discomfort. If fever and/or lymphadenopathy is present,
gram-negative organisms). Other contributing fac- systemic antibiotics may be prescribed by the super-
tors may include his smoking, fatigue, inadequate vising dentist. A 0.12% chlorhexidine rinse should be
­nutrition. prescribed (for use twice daily for a short time until the
2. The patient most likely has necrotizing ulcerative gin- infection has subsided). Dental biofilm control meth-
givitis (NUG). It is an opportunistic bacterial infec- ods and need for adequate sleep and nutrition should be
tion that often occurs in young people who are under discussed with patient. He should be seen within a few
stress and have not been taking good physical care of days to assess the gingival healing, remove more dental
themselves. Inadequate care may include smoking and biofilm and calculus, and ­reinforce homecare. At a third
446   Saunders Review of Dental Hygiene

appointment, scheduled 3 to 5 days later, an assessment 3. Pericoronal abscess (pericoronitis): direct result of
of healing, removal of remaining dental biofilm and entrapment of food and bacteria under operculum
calculus, and reinforcement of oral hygiene instructions (flap of gingival tissue), which ultimately becomes
should take place. Referral to a periodontist is recom- inflamed and painful.
mended if gingival architecture requires correction as 4. Characteristics:
noted on reevaluation appointment. a. Occurs MOST commonly in distal region of
4. Prognosis for NUG is good. After the patient com- partially or fully erupted mandibular third mo-
pletes therapy and begins to take better care of his lars that are covered, partially or fully, by oper-
health, his condition will improve. In a few cases sur- culum.
gery is required to repair gingival contour. b. MOST commonly occurs in young adults be-
tween the ages of 17 and 26.
Abscesses of Periodontium c. Exhibits as pain, fever, pus (purulent exudate);
Abscess of the periodontium includes gingival, peri- trismus may also be present (MAIN cause).
odontal, pericoronal (pericoronitis), combined. Abscess d. If NOT taken care of promptly (especially in
is a localized collection of suppuration (pus) in a cavity immunocompromised patient), may lead to
formed by disintegration of tissues. Ludwig’s angina (see Chapter 6, General and
• See Chapter 6, General and Oral Pathology: periapical Oral Pathology).
and developmental abscesses. 5. Treatment:
A. Gingival abscess: typically occurs as a result of injury a. Initially gently debride and irrigate the area un-
caused by forceful occlusion of foreign object in the der the operculum with antimicrobial such as
gingiva. diluted hydrogen peroxide.
1. Characteristics: b. If fever is present, antibiotics may be pre-
a. Tends to occur on marginal gingiva; NOT asso- scribed.
ciated with deeper periodontal pockets or with c. Recommend plenty of rest, fluid intake to avoid
disease. dehydration, warm salt water rinses.
b. Raised, painful area of acute inflammation ac- d. Later, if area should show signs of improve-
companied by pus (suppuration). ment, can be flushed again and MORE thor-
2. Treatment consists of: oughly debrided.
a. Incision, drainage, and irrigation with antimi- e. Educate patient regarding dental biofilm control
crobials. in location; may need to have tooth extracted
b. Warm salt water rinses after surgery. and/or operculum surgically removed.
c. Scaling of the teeth in the adjacent area.
B. Periodontal abscess: caused by entrapment of bacteria Periodontitis Associated with Endodontic
in periodontal pocket through occlusion of the pocket ­Lesions
by foreign debris, such as popcorn kernel or incom- Lesions are called combination abscesses or combined
plete scaling (calculus). periodontal-endodontic lesions. Difficult to diagnose;
1. Characteristics: may be acute or chronic, associ- may involve a pulpal abscess that spreads through ac-
ated with preexisting periodontal disease. cessory canals to the periodontium or vice-versa. Causes
a. Initially an acute inflammatory reaction that re- ­severe damage and may cause tooth loss if left untreated.
sults in pus (suppuration), with redness, local- • See Chapters 6, General and Oral Pathology: end-
ized swelling, throbbing or pulsating pain, rapid odontic lesions; 15, Dental Biomaterials: endodontic
onset, rapid bone loss, vital pulp that rules out a ­therapy.
pulpal origin.
b. Acute abscess sometimes stabilizes by draining DEVELOPMENTAL OR ACQUIRED DEFORMITIES
through sulcus and/or fistula in adjacent tissues, AND CONDITIONS  
then becomes chronic abscess that is often pain- Category includes local factors associated with teeth and
less. restorations, mucogingival deformities around teeth and
2. Treatment: on edentulous ridges, and occlusal trauma (see earlier
a. Drainage, typically through the pocket, then discussion).
irrigation with antimicrobial agent such as
­povidone-iodine or chlorhexidine. CLINICAL ASSESSMENT  
b. Antibiotics may be required if fever and/or Clinical assessment is a crucial component of periodontal
lymphadenopathy is present. therapy because it forms the foundation on which an ap-
c. If diagnosed and treated early, prognosis is ex- propriate plan of care can be developed. During clinical
cellent and lost bone typically regenerates. assessment, information about patient’s past and present
Periodontology   447

health from the medical and dental histories and dia- c. Poor positive predictor of periodontal disease,
logue, personal habits, and clinical findings are compiled but conversely a very strong negative predictor;
to identify all risk factors that affect BOTH patient care may NOT indicate periodontal disease; contin-
and disease etiology. Assists in identifying nondental risk ued absence is a strong predictor of continued
factors, such as tobacco use, diet, stress, that may play periodontal health.
a role in the patient’s periodontal condition. Supplemen- 6. GCF: increases during inflammation because of
tal methods to determine risk factors, including dental edema; in MORE severe cases of inflammation
biofilm tests and software programs, can help with the becomes milky, contains numerous PMNs and by-
process. products, referred to as suppuration (pus).
• See Chapters 6, General and Oral Pathology: medi- 7. Periodontal tissue destruction with attachment loss
cal conditions related to systemic factors; 11, Clini- and periodontal pockets:
cal Treatment: recording history, patient assessment, a. Periodontal probing performed to measure
­charting. depth of periodontal pockets and amount of
CAL; considered “gold standard” for determin-
Periodontal Assessment ing extent of periodontal tissue destruction.
Periodontal assessment includes an evaluation of topog- b. While probing, CAL is the probing depth mea-
raphy of the gingiva and related structures that change sured from a fixed point such as the distance
from periodontal health to periodontal disease conditions. from cementoenamel junction (CEJ) to base of
Includes collecting data on surrogate measurements such the pocket (EA) to determine attachment loss
as color, contour, texture. (Figure 13-5).
• See Chapter 11, Clinical Treatment: periodontal evalu- (1) When CEJ is covered: to obtain CAL, first
ation. measure from gingival margin to CEJ, then
A. Involves identifying presence of inflammation in gin- subtract from overall probing depth; prob-
giva: ing depth > CAL.
1. Color: changes from “coral pink,” indicating (2) When CEJ is level with gingival margin:
health (with pigmentation differences), to overt CAL is measured by probing depth; CAL =
signs of redness, indicating gingival ulceration and probing depth.
engorgement of blood vessels that occur during in- (3) With recession: CAL is measured from CEJ
flammatory process. to EA (some add recession and probing
2. Contour: in health follows the contour of the teeth; depth); CAL > probing depth.
margins are knife edged (in normal tooth arrange- (4) Either manual or automated probes; typi-
ment) and interdental papillae appear pointed; cally involve measurements at six sites per
rolled marginal tissues and rounded interdental pa- tooth.
pillae suggestive of inflammation. (5) Even though measurements of probing
a. Other changes in contour related to hyperplastic depth are repeatable to within 1 mm >90%
tissues: of the time, standard deviation of repeated
(1) McCall’s festoons: lifesaver-like enlarge- CAL measurements of the same site by ex-
ments around free gingival margins. perienced examiner with a manual probe is
(2) Stillman’s clefts: narrow, slitlike clefts BETTER at around 0.8 mm.
where marginal gingiva has receded; pro- c. To encourage all dental practitioners to include
duced by atrophy of facial gingiva and ex- routine probing in patient care, periodontal screen-
tensions of adjacent hyperplastic tissue. ing and recording (PSR) system was developed.
3. Texture: LEAST reliable sign of inflammation; may (1) Modification of the Community Periodon-
appear as loss of stippling during inflammation. tal Index of Treatment Needs (CPITN)
4. Consistency: in health is firm and resilient; when developed by World Health Organization
inflamed, marginal and interdental papillae be- (WHO).
come bulbous and flabby, if edematous; bulky and (2) System involves probing each of six rou-
firm from scar tissue accumulation, if fibrotic. tine sites per tooth but recording ONLY
5. Bleeding on probing (BoP): highest reading per sextant and interpreting
a. Although late sign of inflammation, MOST reli- findings with code that determines whether
able and widely accepted sign of active inflam- full-mouth recording of all probing depths
mation. is required; uses WHO probe.
b. Record of bleeding indices establishes baseline d. Presence and distribution of dental biofilm, cal-
measure from which goals can be established culus, stain are identified and recorded to assist
and serves as powerful motivational tool. in setting goals with the patient.
448   Saunders Review of Dental Hygiene

(1) Dental biofilm is the MAIN etiological


agent in development of periodontal dis- Radiographic Assessment
ease; extent and distribution must be deter- Radiographic assessment includes evaluation of inter-
mined to facilitate removal. dental septa, amount and pattern of bone loss, furca-
(2) Several epidemiological indices assist in tion involvement that may be present with periodontal
recording dental biofilm (see Chapter 17, disease.
Community Oral Health). • See Chapter 5, Radiology: radiographic interpretation.
(3) Calculus and stain are contributing factors A. Satisfactory number of diagnostic-quality radio-
for periodontal disease owing to involve- graphs must be interpreted to determine extent of
ment in biofilm retention, and treatment bone loss, includes full-mouth periapical (FMX)
planning needs to include removal. radiographs, vertical bitewings, and possibly pan-
(4) Extent and tenacity of calculus deposits and oramic radiographs to accurately assess periodontal
stains determine length of therapy, thus im- status:
portant components of treatment planning. 1. Fuzziness of alveolar crests and breaks in continu-
e. Proximal contact relationships of teeth should ity of lamina dura (alveolar bone proper) are the
be evaluated; open contacts encourage food FIRST sign of periodontal breakdown.
impaction, and tight contacts prevent effective 2. With further periodontal breakdown, slight to
interproximal cleansing. moderate bone loss can be seen as horizontal or
f. Determining extent of mobility of teeth and MORE extensive loss as vertical or angular; de-
dental implants indicates extent of damage to pendent on pathway of inflammation and disease
supporting structures of periodontium; consid- (Figure 13-9).
ered predictor of attachment loss and thus risk 3. Loss of interradicular bone in furcation areas on
factor for continued progression of periodontal multirooted teeth shows further disease progres-
disease. sion.
(1) Mobility is BEST tested with handles of 4. Widening of PDL space (location of PDL) and hy-
two instruments (or with automated mobil- percementosis at apices indicate occlusal trauma.
ity tester, Periotest). 5. Overhanging margins, interproximal calculus
(2) Several mobility scales exist. spears, root caries may also be present and associ-
(3) Teeth with moderate to advanced mobility ated with periodontal breakdown.
are MORE likely to be lost than teeth with- B. Limitations in the diagnosis of periodontal disease:
out mobility; labeled a risk factor in predic- 1. Destruction of alveolar bone typically is greater
tion of periodontal breakdown. than it appears in radiographs (SIMILAR to calcu-
g. Determining presence and degree of fur- lus, which is also much worse than indicated!).
cation involvement; prognosis is poor for
teeth with extensive furcation involvement
because homecare is almost impossible and
accessibility during therapy becomes a major
obstacle.
(1) Several scales classify furcation involve-
ment.
(2) Curved, calibrated Nabor’s probe is BEST
instrument for detection of furcations.
(3) Teeth with moderate to advanced furcation
involvement are MORE likely to be lost than
teeth without furcations; labeled a risk fac-
tor in prediction of periodontal breakdown.
h. Presence of malocclusion and occlusal patho-
logical changes should be assessed routinely;
BOTH considered risk factors for periodontal
breakdown.
i. Status of dental restorations and prosthetic
appliances should be evaluated to determine
whether overhanging margins and poorly con-
toured crowns exist; considered risk factors for
periodontal breakdown. Figure 13-9  Horizontal (top) and vertical bone loss (bottom).
Periodontology   449

2. Destruction of facial and lingual cortical plates is B. Software programs can help measure and communi-
often NOT fully detectable in radiographs. cate an individual patient’s risk of periodontal disease
from a database.
Supplemental Risk Factor Methods
These supplemental risk factor methods play IMPOR- INTERPRETATION OF EVIDENCE, PRESUMPTIVE
TANT role in the identification of risk factors and cou- DIAGNOSIS, TREATMENT PLANNING,
pled with clinical parameters (surrogate measurements), AND PROGNOSIS  
such as BoP and attachment loss, can be strong predictors Dental hygiene diagnosis involves interpreting data into
of future disease progression. coherent description of condition or disease in terms that
A. Classified by four categories: physical, microbiologi- can be addressed by dental hygienist. These data include
cal, biochemical, or immunological. the medical and dental history and clinical, radiographic,
1. Physical assessment tests include measures of sub- and adjunctive evidence. Then data must be interpreted
gingival temperature and GCF flow rates as they to determine a differential diagnosis by the supervising
relate to inflammation. dentist. ONLY after the differential diagnosis has been es-
a. Higher subgingival temperature correlated tablished can an appropriate plan of care be devised. ALL
to presence of inflammation can be measured evidence collected during the initial assessment MUST be
by an automated, computerized, temperature- considered in developing a differential diagnosis, which is
­sensitive probe (Periotemp System). then used to determine patient’s prognosis and treatment.
b. Higher flow rate of GCF correlated to gingival With diagnosis the patient is classified into one of the
inflammation and useful adjunct for monitoring four periodontal case types (see later discussion). Noted
response to periodontal therapy can be mea- in the reports “Risk Management Series, Diagnosing and
sured (Periotron 6000). Managing the Periodontal Patient” (ADA) and “Current
2. Microbiological tests are ONLY adjuncts for Procedural Terminology for Periodontics and Insurance
identification of bacterial species and are used Reporting Manual” (AAP); recommendations for treat-
MAINLY to determine risk for aggressive ment are available for each case type (see CD-ROM).
forms of periodontal disease but can be used for A. Developing the diagnosis involves:
chronic forms; however, important to recognize 1. Prioritizing immediate extraoral and intraoral
that these tests are NOT diagnostic; SHOULD problems.
be used solely for identification and monitoring 2. Identifying any systemic component of the disease
purposes. process.
a. Culturing of possible periodontal pathogens 3. Reviewing the patient’s clinical signs and symp-
has been considered “gold standard” for toms.
identification and quantification of bacterial 4. Looking for radiographic signs of disease.
species. 5. Evaluating the human needs deficits of the ­patient.
b. Enzymatic methods (BANA test) for identify- B. Establishing the differential diagnosis includes:
ing presence of periodontal pathogens such as 1. Determining the health status of the periodontium.
T. denticola, P. gingivalis, T. forsythensis. 2. Determining whether the gingivitis, if present, is
c. Immunological assays (such as enzyme-linked associated with dental biofilm.
immunosorbent assay [ELISA]) for the identifi- 3. Identifying whether condition is chronic periodon-
cation of specific microbes. titis or more aggressive form.
d. DNA probes for accurate identification and 4. Making presumptive diagnosis, which is the BEST
quantification of microbial pathogens have ad- possibility after examining all the evidence.
vantage of NOT requiring a living specimen; C. Possible diagnostic categories include health, gingivi-
include A. actinomycetemcomitans, P. gingiva- tis, and periodontitis; then a more specific diagnosis
lis, P. intermedia. can be identified (presumptive diagnosis), which is
3. Biochemical assessments are MORE sophisticated used to guide the development of the treatment plan.
laboratory tests to identify various biochemical D. Final diagnosis can be made ONLY after completion
markers found in GCF and are involved in host of initial therapy (phase I) and reevaluation of clini-
­response. cal response to therapy; this response guides further
4. Immunological assessment tests identify anti- treatment planning.
bodies to specific periodontal pathogens in either
blood serum or GCF. Correlations have been found Prognosis
between higher antibody levels and active disease; Prognosis is prediction of the duration, course, and ter-
however, relationship to disease progression has mination of a disease and the likelihood of its response
NOT been established. to treatment.
450   Saunders Review of Dental Hygiene

A. Overall prognosis strongly influenced by patient’s also includes regenerative techniques (discussed
­attitude. later).
B. For gingivitis, is dependent on: 4. Phase III therapy: includes all restorative proce-
1. Other pathological changes besides inflammation. dures, orthodontics, splinting, occlusal therapy
2. Presence of systemic risk factors. (see Chapter 15, Dental Biomaterials).
3. Patient compliance with oral hygiene. 5. Phase IV therapy: recare or periodontal mainte-
C. For periodontitis: nance (PM) or maintenance phase (involves dental
o15360 1. Is dependent on: hygiene plan of care); patient usually remains in
a. Assessment of past bone response. this phase for life (discussed later).
b. Height of remaining bone. a. Occasional relapses may require patient to re-
c. Periodontal attachment loss. peat initial therapy, but patient ALWAYS re-
d. Patient age. turns to PM procedures.
e. Presence of malocclusion. b. Appropriate PM intervals are determined indi­
f. Presence of systemic risk factors. vidually; based on tissue response and home­
2. Is NOT related to compliance with oral hygiene. care efforts of the patient.
D. For individual teeth, is dependent on:
1. Loss of attachment, depth of periodontal pockets, Dental Hygiene Plan of Care
width of attached gingiva. Dental hygiene plan of care is developed by dental hy-
2. Mobility, furcation involvement, tooth morphology. gienist after diagnosis and prognosis of the case. ­Typically
involves phases I and IV of the comprehensive treatment
Determining Appropriate Plan of Action plan of periodontal therapy.
Determining appropriate plan of action for each patient • See Chapter 18, Ethics and Jurisprudence: informed
involves treatment planning, phases of treatment, and consent.
dental hygiene plan of care. A. Plan of care:
A. Treatment planning goals: 1. MUST meet individual patient needs and MUST
1. Developing individualized plans based on the spe- follow an orderly sequence.
cific needs of each patient. 2. Involves determining number of appointments and
2. Incorporating “evidence” in the development of time required for each, depending on needs.
the plan. 3. Always requires reevaluation 4 to 6 weeks (1 month)
3. Developing evidence-based, patient-specific treat- after initial therapy (for all periodontal patients)
ment protocols. to assess tissue response, determine appropriate
4. Determining methods and the sequence for deliv- periodontal maintenance (PM, recare) interval
ering appropriate treatment. (discussed later), evaluate need for referral to a
5. Eliminating and controlling etiological and predis- periodontist, if not the treating dentist.
posing factors. B. May follow a suggested guideline for determining
6. Emphasizing the maintenance of health and pre- number of appointments for various periodontal case
venting disease recurrence. types; however, individual needs may alter model and
B. Phases of treatment are divided into five groups: MUST always take precedence over any suggested
1. Preliminary phase: addresses any dental or peri- protocols:
odontal emergencies and MAINLY involves treat- 1. Case type I (gingival disease): typically is com-
ment of all acute and painful oral conditions (see pleted in one appointment; recare (recall) interval
Chapter 6, General and Oral Pathology). is established at that time.
2. Phase I (initial) therapy: etiological phase that deals 2. Case type II (early periodontitis): often requires more
MAINLY with control or elimination of etiological than one appointment, may require use of local an-
factors and includes dental hygiene plan of care. esthetics; is followed by reevaluation ­appointment.
a. Patient education, dental biofilm control in- 3. Case type III (moderate periodontitis): typically is
struction, risk factor counseling (see Chapter 11, divided into quadrants; requires local anesthesia
Clinical Treatment). and several appointments, including reevaluation
b. Periodontal debridement (including scaling) appointment.
(see Chapter 12, Instrumentation). 4. Case type IV (advanced periodontitis): typically
c. Margination: recontouring of defective mar­ is divided into quadrants or sextants; may require
gins of restorations (see Chapter 15, Dental treatment of individual teeth, depending on extent
Biomaterials). and tenacity of the deposits; local anesthetics are
3. Phase II therapy: surgical phase; involves proce- routinely used and reevaluation appointment is
dures designed to reduce effects of disease and planned.
Periodontology   451

5. Case type V (severe periodontitis): may require i­dentification of dental biofilm retentive factors;
single or multiple visits with local anesthesia, de- may include adjunctive chemotherapeutic agents.
pending on amount of debridement necessary with a. Treat the areas that have discomfort FIRST, un-
adjunctive therapy (see later discussion); more less tissue health is a factor.
frequent recare (recall) appointments are often b. Treat either the quadrant with the fewest teeth or
­required. the one with least severe periodontal disease.
(1) Makes first appointment less complicated.
NONSURGICAL PERIODONTAL THERAPY   (2) Helps orient the anxious patient to clinical
Nonsurgical periodontal therapy (NSPT) is initial ­procedures.
treatment of gingival and periodontal diseases, involves c. Use local anesthesia and/or nitrous oxide seda-
eliminating pathogenic microorganisms through defini- tion based on patient’s previous pain control
tive instrumentation procedures, dental biofilm control in- experiences, severity of infection, type of cal-
struction, other adjunctive procedures. Also called phase I culus, potential for discomfort, and present sen-
periodontal therapy (initial therapy). With oral-systemic sitivity of hard and soft tissues.
disease links that have been established, BETTER clinical d. When two quadrants are treated at the same time,
outcomes of NSPT are MORE important than ever. selecting maxillary and mandibular quadrants
• See Chapters 11, Clinical Treatment: patient motivation minimizes patient posttreatment discomfort.
and dental biofilm removal methods; 12, Instrumenta- 5. Root planing, targeted toward removal of cemen-
tion: deposit removal including selective polishing. tum or surface dentin impregnated with calculus,
A. Goals of NSPT: can be divided into short- and long- endotoxins, or bacteria; current evidence has led
term goals: to MORE conservative approach to this procedure;
1. Short-term goals: thus a glassy, smooth root surface is NO longer
a. Removal of hard and soft deposits through goal of NSPT; questionable procedure.
­instrumentation. 6. Closed gingival curettage techniques to remove
b. Definitive dental biofilm control instruction. diseased soft tissues from within the pocket wall;
c. Control of systemic and other risk factors. currently is believed to have limited value in NSPT
2. Long-term goals: and is used ONLY in selected cases.
a. Reduction or elimination of gingival inflamma- 7. Selective coronal polishing to remove retentive su-
tion. pragingival dental biofilm and stains for cosmetic
b. Elimination of pockets caused by edema. reasons.
c. Restoration of periodontal health. 8. Occlusal evaluation for identification of related
B. Rationales for NSPT: contributing factors.
1. Removal of primary etiological agent, dental bio- D. Effectiveness of NSPT:
film, other local irritants. 1. Effectiveness, whether performed by manual or
2. Initiation of dental biofilm control measures. powered instrumentation, has been well docu-
3. Modification of risk factors, BOTH local and mented.
systemic, wherever possible (e.g., overhang re- 2. Decreased probing depths and reduction in BoP
moval). are expected outcomes of therapy.
4. Reduction or elimination of inflammation. 3. Gains in attachment in deeper pockets are reason-
5. Evaluation of tissue response. able expectations.
6. Establishment of periodontal maintenance (PM, 4. Generally healthier oral environment typically
recare) procedures (discussed later). ­results.
C. Procedures and techniques of NSPT: 5. Close monitoring of all clinical parameters is nec-
1. Oral hygiene instruction, which is comprehen- essary to prevent or arrest progression of disease
sive and individualized, to facilitate daily dental after therapy.
biofilm removal (can be instituted as separate
appointment for tissue conditioning before scaling Adjunctive Therapies
appointment). Because of highly infective microbial population present
2. Scaling for purpose of supragingival and subgingi- in diseased periodontal pocket, other therapies, used con-
val calculus removal. comitantly with periodontal debridement, may be useful
3. Preprocedural mouthwash to lower bacterial adjuncts to NSPT. Include chemical and antibiotic ad-
count in aerosols and decrease the potential for junctive therapies.
­bacteremia. A. Chemical adjuncts:
4. Periodontal debridement; includes scaling to- 1. Include antimicrobial rinses that are given before
gether with subgingival dental biofilm removal and NSPT; NOT shown to have significant benefits.
452   Saunders Review of Dental Hygiene

2. Include subgingival irrigation, which, when com- c. FDA-approved examples:


bined with scaling, may enhance success of NSPT (1) Tetracycline fibers (Actisite, no longer
(controversy regarding benefits). available in United States): controlled-re-
a. Irrigation is considered elective part of therapy. lease impregnated fibers are placed in pock-
(1) To be effective, must meet the following ets greater than 5 mm for 10 days; placed in
requirements: pocket in successive layers, with gingival
(a) Must reach base of pocket. retraction cord packing instrument, then
(b) Bactericidal concentration of agents sealed with cyanoacrylate adhesive; cord is
must be employed. removed after 10 days.
(c) Must exhibit substantivity. (2) Doxycycline hyclate, 50 mg (Atridox): bio-
(2) Methods of delivery absorbable, two-part formulation is mixed
(a) Use of hand syringe with cannula tip. and placed by syringe in a periodontal
(b) Use of pulsated irrigation devices, with pocket, where hardens on contact with sa-
either rubber tip or cannula. liva and slowly releases drug for a week;
b. Commonly used irrigants: ­suppresses bacteria by inhibiting protein
(1) 0.12% chlorhexidine digluconate. synthesis, thus bacteriostatic.
(2) 0.4% stannous fluoride. (3) Chlorhexidinegluconate, 2.5 mg (Perio­Chip):
(3) 0.05% povidone-iodine. biodegradable, rectangular chip placed in
c. Include desensitizing agents for dentinal hy- periodontal pockets greater than 5 mm,
persensitivity, used as adjuncts to NSPT (also releases drug biphasically (40% initially;
postsurgery); recommended because sensitivity ­remainder over 7 to 10 days); ­bactericidal.
after therapy is a commonly reported problem (4) Minocycline hydrochloride, 1 mg (Ares-
(see Chapter 11, Clinical Treatment). tin): powder (microspheres) gives extended
B. Antibiotics: release for up to 21 days, with no inter-
1. Systemic antibiotics: dental cleaning for 10 days, no probing for
a. Sometimes necessary in treatment of MORE seri- 30 days, no retreatment for 3 months; bac-
ous forms of periodontal disease because MOST tericidal.
of these conditions exhibit host impairment. d. Other controlled-release agents include chlor­
b. In all cases, is BEST if the pathogens are h­exidine gels (1% and 2%), metronidazole gels,
identified (by culture and sensitivity, by DNA clindamycin HCl gel.
probing, or by immunofluorescent testing) to
determine which antibiotic agent should be pre- SURGICAL PERIODONTAL THERAPY  
scribed (see earlier discussion). After completion of NSPT, tissue response is evaluated
c. Commonly prescribed adjuncts to NSPT include and need for surgical intervention is determined. IMPOR-
amoxicillin, ampicillin, Augmentin, tetracyclines, TANT for the dental hygienist to understand indications
metronidazole, ciprofloxacin, clindamycin. and contraindications for basic surgical procedures to re-
d. Doxycycline hyclate (20 mg capsule bid) inforce significance to the patient.
(Periostat): use of low-dose (LD) antibiotic A. Rationale: reduction of disease progression when
at subantimicrobial dose (SDD) to suppress nonsurgical interventions have failed.
destructive enzymes such as collagenase pro- B. Goals:
duced during inflammatory process rather than 1. Pocket reduction for BETTER access to homecare
to reduce bacterial numbers by blocking matrix procedures and professional debridement (MOST
metalloproteinases (MMPs); used twice daily important).
for up to 9 months; results in decreased pocket 2. Correction of mucogingival defects and removal
depths and attachment loss; taking a common of inflamed tissues.
nonsteroidal antiinflammatory drug, such as 3. Drainage of periodontal abscesses.
aspirin or ibuprofen (Advil) along with doxy- 4. Providing access for restorative dental proce-
cycline, may enhance the effectiveness of this dures.
treatment. 5. Regeneration of tissue lost to disease.
2. Local delivery of antibiotics (controlled release): 6. Placement of dental implants.
a. Used in cases that do NOT respond to conven- 7. Improved esthetics.
tional NSPT. C. Considerations: several factors must be taken into
b. When used concomitantly with scaling, has consideration before surgical procedure is recom-
been shown to decrease probing depth and re- mended; include probing depths, bone loss, value of
duce BoP and microbes. affected tooth, patient factors.
Periodontology   453

1. Pocket probing depths (PD): specific laser-based treatment is superior to the traditional
a. Must be >5 mm. modalities of therapy.
b. Typically difficult to debride professionally. • See Chapter 7, Nutrition: presurgery and postsurgery
c. Typically make performance of homecare by diets.
patient difficult. A. Gingivectomy: to reduce pocket depths by removal
2. Bone loss: of the soft tissue pocket wall.
a. If MORE than half of the bone remains, osse- 1. Indicated in deep gingival pockets that have fibrotic
ous surgery is viable option. tissue and deep periodontal suprabony pockets.
b. If LESS than half of bone remains, grafting or 2. Example: removal of excess tissue with gingival
regenerative techniques may be required. hyperplasia as result of drug therapy (see earlier
c. Choice of procedure depends on type of bone discussions).
loss and bony defects (e.g., horizontal, vertical, B. Periodontal flap procedures: used during proce-
one walled, two walled). dures that correct mucogingival defects (discussed
3. Value of the tooth: some LESS valuable than oth- later).
ers, may NOT be worth saving (e.g., third molar). 1. Provide access to and visibility of the root surfaces,
4. Dental biofilm control: demonstration of ad- reduce pocket depths surgically, stimulate regener-
equate dental biofilm control measures is neces- ation of lost periodontal attachment in periodontal
sary. pockets.
5. Age of the patient: 2. Indicated in periodontal pockets that extend to or
a. NOT significant, except in determining the beyond mucogingival junction and for infrabony
speed of disease progression. pockets.
b. Healing is similar in BOTH young and old a. Modified Widman flap technique: reposition-
­patients. ing of reflected gingival tissues against the tooth
c. Relationship of surgery to quality of life at approximately the preoperative position.
SHOULD be considered for geriatric patients. b. Apically positioned flap technique: reposi-
6. Health of the patient: tioning the reflected flap MORE apically, com-
a. Poor health contraindicates surgery. pared with original position, to reduce pocket
b. If periodontal disease is contributing to poor depth.
health, surgery may be indicated after medical C. Crown lengthening: used when tooth needs to be re-
consult. stored but there is NOT enough tooth structure coro-
7. Patient preference: nal to the free gingival margin to support direct or
a. Patients MUST be informed of all risks and indirect restoration (crown).
benefits of surgery. 1. Can occur when tooth breaks off at margin or res-
b. Patients must understand the ramifications of toration that has extensive caries underneath is
NOT having surgery. lost, thus exposing MORE of tooth by removing
c. Patients MUST be willing to increase frequency some soft tissue and bone after flap procedure.
of PM appointments and perform MORE dif- 2. Allowed to heal with temporary crown for at least
ficult dental biofilm removal procedures if opt 3 months before tooth is prepared for final crown;
NOT to have surgery. to allow gingival tissues to recede.
D. Presurgical drug notations: NO drugs related to 3. Also used for removal of soft tissue with “gummy”
bleeding 2 weeks before surgery, including over-the- smile; unusually large amount of attached gingiva
counter (OTC) drugs; for MOST healthy patients around maxillary anterior teeth.
reduction of postsurgical swelling starts with taking D. Osseous surgery:
600 to 800 mg of ibuprofen (Motrin, Advil) 1 hour 1. Used to reduce bony walls of infrabony pockets
before treatment; effective analgesic (pain killer) and and to reshape abnormal alveolar contours.
­antiinflammatory. a. Ostectomy: removal of alveolar bone that is
directly attached to the tooth via PDL fibers.
Periodontal Surgical Procedures b. Osteoplasty: recontouring of alveolar bone that
Basic periodontal surgical procedures include gingivec- is NOT directly supporting the tooth.
tomy, periodontal flap procedures, osseous surgery, and 2. Performed in conjunction with periodontal flap
regeneration procedures. Implant surgery discussed later. procedure to treat defects including:
Based on review of the literature, AAP concludes that a. One-walled and two-walled infrabony pockets.
there is a great need to develop an evidence-based ap- b. Interdental osseous craters.
proach to the use of lasers for the treatment of chronic c. Broad three-walled infrabony pockets.
periodontitis and insufficient evidence to suggest that any d. Reversed alveolar bone architecture.
454   Saunders Review of Dental Hygiene

e. Bulbous facial and lingual bony contours: (1) Dehiscence: bursting through bone of
(1) Buttressing bone: abnormal thickening of a root as the tooth continues to erupt so
bone in areas where trabeculae have been that the bone does not extend to its nor-
weakened by resorption. mal proximity to the CEJ; fenestration:
(2) Festoons: wavy contours of bone found be- circumscribed defect that creates a “win-
tween the teeth. dow” through the bone over the prominent
E. Periodontal regenerative procedures in infrabony root.
pockets and class II furcations; includes regeneration (2) Defects NOT present on radiographs and
procedures and/or bone grafting: probing are limited; sounding (pushing
1. Guided tissue regeneration (GTR): placement of probe through anesthetized soft tissue over
various materials to form a barrier that prevents the root) may be used.
JE downgrowth and encourages connective tissue G. Healing after surgery:
­reattachment. 1. Healing of surgical wound commences immedi-
a. Thorough debridement of the area, with flaps ately after surgery with formation of a blood clot
reflected. to protect the wound and allow healing to begin.
b. Placement of barrier material (e.g., Gortex or 2. Epithelial healing is accomplished within 7 days,
collagen) over defect; flap is closed coronally at which time sutures, if NOT resorbable, and
to the material. dressing, if placed, may be removed.
2. Bone grafts: placement of various materials in 3. Gingivectomy wounds take longer to heal than flap
osseous defects before flap closure to encourage procedures because healing is by secondary inten-
regeneration of lost tissue (see also later discussion tion rather than by primary intention.
with implants). 4. Osseous healing does NOT begin until 1 month
a. Autografts: taken from the patient’s hip (iliac after surgery; complete healing and remodeling do
crest) or jawbone. NOT take place until 4 to 6 months after surgery.
b. Allografts: taken from another person (e.g., H. Effectiveness is dependent on patient’s long-term
freeze-dried from cadavers). thoroughness in dental biofilm control.
c. Xenografts: taken from another species (e.g., 1. Thoroughness should be reinforced at each PM
bovine, coral). ­appointment.
d. Alloplasts: synthetic bone minerals (e.g., hy- 2. Compliance with ongoing PM therapy is single
droxyapatite [MAIN form of calcium found in MOST important factor in success of surgical and
bone], calcium sulfate, or ceramics). nonsurgical therapy.
F. Soft tissue grafting for correction of mucogingival I. Dental hygienist’s role in periodontal surgery:
defects: 1. When surgery is discussed, explained, and per-
1. Type I defects: pockets extend apically to or be- formed for patient, discussing advantages and dis-
yond mucogingival junction, but a firm keratin- advantages.
ized pocket wall exists; apically positioned flap is 2. Acting as patient advocate by asking questions and
BEST treatment. providing answers to concerns that patient may
2. Type II defects: alveolar mucosa acts as marginal NOT be able to verbalize.
gingiva, with NO zone of attached gingiva. 3. Participating in postoperative suture and dressing
a. Treatment consists of transfer of masticatory removal (see next section).
mucosa from an adjacent area of the mouth. 4. Removing postsurgical dental biofilm, following up
(1) Lateral sliding pedicle graft: moving kera- with wound care, providing homecare instructions.
tinized tissue from area immediately adja- 5. Motivating patient to engage in long-term mainte-
cent to defect without completely detaching nance of dental biofilm control.
from original spot, sliding it over defect, 6. Providing PM procedures at regular recare intervals.
then attaching with sutures.
(2) Double papillae flap: taking equal amounts Postsurgical Procedures
of tissue from the two adjacent interden- Postsurgical procedures include placement of sutures by
tal papillae, sliding them over to cover the supervising dentist, periodontal dressings, and provision
defective area, and joining the tissues with of postoperative instructions. See state laws for more
sutures. information on local practice acts for dental hygienists
b. Free autogenous soft tissue graft: removing a concerning these duties.
piece of keratinized gingiva from another area A. Sutures: placed at the end of surgical procedure to
of the mouth, such as palate, and suturing it to hold the tissues in place or close (proximate) so the
defective region to completely cover defect. wound can heal by primary intention.
Periodontology   455

1. Made of surgical silk, resorbable gut, or synthetic dressing material, special care in removal
material. ­required.
2. Removed between 7 and 14 days after placement, C. Postoperative instructions can include:
if NOT resorbable, when tissues have undergone 1. Directions for mouthrinses, antibiotics, analgesic
healing. medications.
3. May be interrupted or uninterrupted, but knot al- 2. Instructions to eliminate strenuous activity.
ways faces buccal surface. 3. Directions regarding continued bleeding (both oral
B. Periodontal (surgical) dressing (pack): includes sev- and written).
eral types of dressing materials. 4. Recommendation of soft diet; AVOID spicy foods.
1. Placed over surgical area to protect wound and in- 5. Directions regarding periodontal dressing, if placed.
crease patient comfort. 6. Inform that swelling may occur, can be minimized
2. SHOULD remain in place for approximately with use of ice; 30 minutes on, then 60 minutes off,
1 week. for 48 to 72 hours.
3. Materials: zinc oxide–eugenol or zinc oxide– 7. Directions for homecare, including brushing with
­noneugenol (metallic oxide and fatty acids). soft-bristled toothbrush and gently debriding dress-
a. Eugenol was used to help with adhesiveness of ing area.
the dressing (Coe-pak, Periocare) but made the 8. Reminder to return for suture removal, if needed.
removal challenging, so dressing without eu-
genol is used MORE commonly (nothing to do PERIODONTAL MAINTENANCE  
with any other characteristics of eugenol). Success of periodontal therapy depends on successful
b. Clear, translucent, light-cured dressings (Bar- maintenance of health and prevention of recurring disease.
ricaid VLC) are used for esthetic concerns. To accomplish this healthy state for any length of time,
4. Mixing and application of the dressing: regular periodontal maintenance (PM, recare, continuing
a. Lubricate lips to prevent moist and sticky dress- care) appointments and procedures are required. These are
ing material from adhering during placement. at relatively short intervals to reinforce homecare, control
b. Mix together equal lengths of each component accumulation of dental biofilm–retentive deposits, and deal
paste until well blended and color is homoge- with such problems as root caries and dentinal hypersensi-
neous; create two rolls that approximate length tivity with postsurgical exposure of dentin. Usually patient
of surgical area. is seen every 3 months or four times a year by the dental
c. Using sterile gauze, gently dry area to be hygienist.
­covered. • See Chapter 11, Clinical Treatment: health promotion,
d. Control bleeding before placing by applying disease prevention factors, dentinal hypersensitivity,
pressure to area with sterile gauze sponge until root caries.
bleeding subsides; if this does NOT work, he- A. Role of the dental hygienist:
mostatic agents may be necessary. 1. MAIN provider of PM procedures; role is critical
e. Dressing should be adapted far enough into to the success of periodontal therapy.
interproximal areas that facial and lingual sur- 2. MUST work in collaboration with supervising
faces of the dressing are joined together to form dentist and/or periodontist in provision of high-
mechanical lock between the dressing and the quality oral healthcare.
teeth; extend NO further occlusally than middle 3. MUST function as cotherapist; must encourage
third of the teeth and do NOT allow to interfere patient to assume ultimate responsibility for main-
with normal occlusion. tenance of periodontal health.
5. Patient instructions: B. Rationale: preservation of stability achieved during
a. Avoid eating, drinking, rinsing for FIRST hour active NSPT can be accomplished ONLY through re-
after placement to allow dressing to harden. peated PM visits.
b. Rest of instructions are SIMILAR to those for C. Objectives: preservation of CAL, maintenance of
postoperative (see next). bone height, control of inflammation, control of den-
6. Removing the dressing: tal biofilm.
a. Gently loosen from the soft tissues with pair of 1. Preservation of CAL:
cotton pliers and curet; lift away from wound a. Monitoring probing depths at PM appointments,
site. which can predict attachment loss, signaling re-
b. Rinse area gently with sterile saline to cleanse it turn of disease, or can indicate stability.
of surface debris. b. Recording of CAL, which SHOULD take place
c. If the dressing was placed over sutures, suture periodically at PM appointments for baseline
material may be incorporated into the ­hardened comparison.
456   Saunders Review of Dental Hygiene

2. Maintenance of alveolar bone height: e. Influence of friends and family.


a. Periodic radiographic evaluations of bone levels f. Perceived indifference on part of dental hy­
are necessary for comparison with baseline levels. gienist.
b. Radiographs do NOT indicate active disease; 3. Success of periodontal therapy rests on perfor-
ONLY illustrate historical bone loss. mance of adequate daily dental biofilm control.
c. Evidence of continued bone loss is indicative of a. Dental hygienist is responsible for educating
disease progression. and motivating patient to perform these proce-
d. MAIN goal is to prevent further bone loss, dures routinely.
which can lead to eventual tooth loss. b. Compliance with oral hygiene has been shown
3. Control of inflammation: to diminish within 30 days of commencement.
a. To halt disease progression, inflammation c. The MORE aids that are introduced, the LESS
MUST be eliminated. compliant a patient is likely to be.
b. Repeated evaluation of the gingival tissues for d. Interproximal aids are difficult to use and lead
signs of inflammation at PM appointments en- to LACK of compliance.
ables the dental hygienist and patient to effec- e. Powered toothbrushes have been shown to in-
tively control disease progression. All BoP sites crease patient compliance.
MUST be recorded, since BoP has been shown 4. Noncompliance with either PM appointments or
to be the MOST reliable sign of inflammation. maintenance of oral hygiene results in continued
4. All aspects of oral health MUST be evaluated at disease progression.
each PM appointment; includes identification of F. Techniques for enhancing compliance:
new hard and soft tissue lesions, significant sys- 1. Simplify explanations.
temic disease that might affect oral health, any 2. Accommodate program to suit needs; satisfied
other disharmonies or discrepancies that may af- ­patients are MORE compliant.
fect oral health. 3. Write down oral hygiene regimens to serve as
5. Evaluation and reinforcement of oral hygiene: ­reminders.
a. Compliance with oral hygiene procedures re- 4. Provide positive reinforcement.
quires frequent reinforcement. 5. Remind patients of appointments.
b. Regular observation and recording of dental 6. Identify noncompliers before commencement of
biofilm scores assist in motivating the patient. initial therapy and discuss consequences of non-
c. Direct observation of homecare techniques can compliance.
assist in behavior modification. 7. Keep compliance records to guide behavior modi-
d. Dental biofilm control education is as IMPOR- fication and for legal purposes.
TANT and as difficult as the technical aspects G. Components of appointment:
of therapy. 1. MAIN aims of the PM appointment are to evaluate
D. Effectiveness: stability of the results of initial therapy; to thor-
1. BOTH surgical and nonsurgical periodontal thera- oughly debride all tooth surfaces; to eliminate any
pies have been shown to be effective in elimi- factors that may support persistence of disease-
nating active disease but ONLY if routine PM is producing microorganisms; to evaluate and rein-
performed at regular intervals. force dental biofilm control.
2. Periodontal therapy without regular PM may result 2. Procedures performed at the PM appointment:
in increased attachment and alveolar bone loss. a. Medical and dental history updates.
3. Meticulous dental biofilm control and professional b. Oral and dental examinations.
debridement control disease in MOST cases. c. Evaluation of:
E. Compliance is KEY to successful therapy. (1) Pocket probing depths and CAL to ascertain
1. Major challenge for the dental hygienist, since attachment loss and gingival recession.
there can be LOW compliance rates; MUST spend (2) BoP and suppuration.
time explaining significance of PM appointments (3) Tooth mobility and furcations.
to maintenance of oral health. (4) Mucogingival involvement.
2. Reasons for noncompliance are complex: (5) Root caries and dentinal hypersensitivity.
a. Failure to understand significance. d. Radiographic examination as necessary.
b. Interpretation of chronic disease as NOT life e. Dental biofilm control evaluation, which
threatening. involves:
c. Economic issues; value of care to patient; so- (1) Dental (dental biofilm) index score.
cioeconomic status (SES) of patient. (2) Education, motivation, and reinforcement
d. Fear. regarding dental biofilm control.
Periodontology   457

(3) Provision of oral hygiene instructions, in- c. May be sprayed or coated with plasma of ti-
cluding: tanium, aluminum oxide, hydroxyapatite, or
(a) Review of toothbrushing and interden- single-crystal sapphire.
tal cleansing techniques. 2. Subperiosteal implant: positioned over the bone;
(b) Reinforcement of significance of daily consists of cast framework with projections pro-
homecare. truding through the mucosa that are designed to
f. Periodontal debridement. support complete removable denture.
(1) Removal of all supragingival and subgingi- 3. Transosteal (staple) implant: surgically placed
val dental biofilm and calculus. through the mandible; provides anchorage for
(2) Selective polishing, if rationale exists. ­removable mandibular denture; often made of gold
(3) Fluoride and calcium therapy, if warranted. or Vitallium instead of titanium.
(4) Desensitization, as needed. 4. Mini implants: smaller, LESS invasive implant
g. Referral to an appropriate specialist, if warranted. fixtures used in immediate-loading surgery (dis-
h. Establishment of appropriate PM intervals that cussed next).
meet individual’s needs. C. Relative contraindications:
H. Recurrence of disease is common. 1. Uncontrolled type 1 DM and/or smoking be-
1. Causes of recurrence: cause healing after any type of surgical proce-
a. Insufficient dental biofilm control; incomplete dure is delayed owing to poor peripheral blood
removal of deposits during therapy. circulation.
b. Faulty restorations and/or prostheses that foster 2. Poor volume and height of bone available (often
dental biofilm accumulation; LACK of compli- the complementary procedures sinus augmenta-
ance with regular PM appointment intervals. tion and block graft are needed to provide enough
c. Systemic conditions that affect the host response. bone, discussed later).
2. Sometimes, despite appropriate therapy and com-
pliance, disease remains active or refractory (oc- Implant Surgery
curs in 10% of all periodontal patients). Traditional implant surgery typically can take place either
3. Return of disease requires that initial therapy be 2 weeks or 3 months after extraction (delayed immediate
repeated, results be reevaluated, and decisions be postextraction implant placement) or 3 months after tooth
made regarding surgical alternatives; also requires extraction (late implantation). Consists of two surgeries,
referral to a specialist. and typically takes 2 to 6 months before final permanent
4. Other treatment modalities may include microbio- replacements are placed. During this time patients can
logical monitoring and/or antibiotic therapy. wear removable temporary tooth replacements over the
implant sites.
DENTAL IMPLANTS   However, technology affords some patients ability to
Because dental implants are vulnerable to bacterial inva- have immediate-loading dental implants; immediately
sion and subsequent failure, dental hygienist can play im- placed after tooth extraction, and temporary fixed tooth
portant role in BOTH patient education and professional replacement is attached. MOST patients need the longer
maintenance. treatment plan, which has an excellent history going back
• See Chapters 11, Clinical Treatment: implant charting many years. Only traditional method is discussed here.
and homecare; 12, Instrumentation: professional care. A. Traditional surgical method:
A. Implants are designed for several uses: 1. First surgery:
1. To support dentures in edentulous mouths. a. Osteotomy performed by drilling a hole through
2. As abutments in partially edentulous mouths to the bone to place the implant in the jaw.
support fixed bridgework. b. Covering implant with gingiva and allowing 2
3. To replace single teeth. to 6 months for healing and osseointegration.
B. Types include endosseous, subperiosteal, transosteal 2. Second surgery:
implants: a. Implant is exposed, and placement of the abut-
1. Endosseous implant: placed directly in the bone; ment fixture, which protrudes coronal to the
may be either root form or blade form; MOST gingiva, is made.
commonly used system. b. Final “loading” is performed, which involves
a. Root form is MORE common than blade form; placement of the prosthesis onto the abut-
may be conical, cylindrical, threaded-screw, ment.
perforated, or hollow baskets. (1) Early loading (1 week to 12 weeks).
b. Made MOSTLY of titanium; however, poly- (2) Staged loading (3 to 6 months).
mers, ceramics, and other metals are used. (3) Late loading (more than 6 months).
458   Saunders Review of Dental Hygiene

B. Success is dependent on: a. Typically, implants are placed at least as deeply


1. Careful selection of the candidate; relative contra- into bone as crown or tooth will be coronal to
indications may compromise success: the bone, a 1:1 crown to root ratio.
a. History of blood dyscrasias, DM, smoking, or b. This ratio establishes target for bone grafting in
other diseases that compromise health. MOST cases; if 1:1 or better cannot be achieved,
b. Poor oral health (e.g., gingivitis, periodontitis). the patient is usually advised that only short im-
c. Poor oral hygiene practices. plant can be placed and NOT to expect a long
d. Poor quality and quantity of bone. period of usability.
2. Careful preparation of the implant site: c. Improved bone height, difficult to achieve, is
a. Overheating of the bone should be avoided. particularly important to ensure ample anchor-
b. If bone overheats, necrosis may occur, which age of the implant’s rootlike shape because it
may result in fibrointegration rather than osseo- has to support mechanical stress of chewing,
integration. just like a natural tooth.
3. Accurate sizing of the placement site: d. If an implant is too shallow, chewing may cause
a. Preserving ample keratinized tissue around dangerous jawbone crack or full fracture.
­implant. e. Wide range of grafting materials and substances
b. Parallel placement of multiple implants, which may be used (see earlier discussion).
allows prostheses to be attached with screws.
4. Allowing time for BOTH healing and osseointe­ Professional Implant Maintenance
gration (2 to 6 months) before loading the ­implant. Professional maintenance procedures for implants in-
C. Osseointegration: clude assessing tissues, probing implant sulcus depths,
1. Direct contact or integration of the bone to im- checking for BoP and mobility, assessing dental biofilm
plant; provides MOST stable attachment. and calculus ­ deposits, removing deposits, obtaining an-
2. Fibrointegration: connective tissue grows be- nual radiographs, and discussing homecare with patient.
tween bone and implant; LESS likely to exhibit • See Chapters 11, Clinical Treatment: implant home­
long-term success in contrast to osseointegration. care; 12, Instrumentation: implant scaling, manual and
D. Tissue interface: ultrasonics.
1. Consists of gingiva surrounding implant, SIMI- A. ONLY instruments made specifically for implants
LAR to that which surrounds a normal tooth ex- should be used for removal of hard and soft deposits
cept there are NO gingival or PDL fibers between and for probing; conventional metal instruments can
gingiva and bone. scratch the surface, making them LESS biocompat-
2. Area between JE and the bone is composed of ible with the surrounding tissues and MORE condu-
connective tissue, which touches the implant sur- cive to dental biofilm accumulation.
face directly and contains collagen fibers that run B. Sonic and ultrasonic instrumentation and air-powder
parallel (rather than perpendicular) to the implant polishing should be avoided, unless implant plastic
­surface. tips with ultrasonic scaler, either fixed or disposable,
a. The SE is nonkeratinized, and cells at the base are used.
of the sulcus form a JE that attaches to the im- C. Rubber-cup polishing with fine powder, such as tin
plant by means of a basal lamina and hemides- oxide, or with fine-grit commercial polishing paste
mosomes. may be performed selectively, as with natural teeth.
b. Implant sulcus depths range from 1.3 to 3.8 mm. D. Implant-supported prosthesis should be removed
c. This interface or biological seal prevents pen- yearly to permit thorough debridement and to check
etration of bacteria and their toxins to osseoin- for stability.
tegrated implant surface. E. A 3-month recare (recall) interval should be estab-
E. Complementary procedures: lished for at least first year after implant placement.
1. Sinus lifting: thickens inadequate part of atrophic
maxilla toward the sinus with assistance of bone Implant Failure and Periimplantitis
grafting, creates a BETTER quality bone site. Failure of a dental implant is MOST often related to fail-
2. Bone grafting is necessary in patients who have a ure to osseointegrate correctly. Affected implants have
LACK of adequate maxillary or mandibular bone periimplantitis rather than periodontitis. Dental hygien-
in terms of front to back (lip to tongue) depth or ist SHOULD be able to recognize a failing implant. Treat-
thickness, top to bottom height, and left to right ment requires removal of implant. Considered a failure if
width; sufficient bone is needed in three dimen- lost or mobile or shows postimplant bone loss of greater
sions to securely integrate with the rootlike im- than 1.0 mm in the first year, greater than 0.2 mm a year
plant (see earlier discussion). thereafter.
Periodontology   459

A. Characterized by mobility (MOST important), pock-   5 When vitamin D deficiency occurs in adults, the resulting
eting, BoP, exudate, progression of bone loss, dull effects on the periodontium include all of the following,
sound on percussion, radiographic evidence of peri- EXCEPT one. Which one is the EXCEPTION?
implant radiolucency. A. Resorption of the alveolar bone
B. Periimplant tissues are at risk for invasion by micro- B. Destruction of the PDL
C. Fibrous dysplasia of the bone
organisms the SAME as natural periodontium.
D. Hyperplasia of the gingiva
1. Microbiota found in implant sulcus are SIMILAR   6 Which of the following nutritional deficiencies has been asso-
to those in natural sulcus, in BOTH health and ciated with an increased incidence of necrotizing gingivitis?
­disease. A. Osteoporosis
2. Because of risk for infection, maintenance of im- B. Osteomalacia
plants is a MAIN concern. C. Marasmus
3. Prevention of infection includes excellent mainte- D. Kwashiorkor
nance education with antibiotics before and after   7 Multilocular radiolucent jaw lesions, the loss of lamina
surgery, along with mouthrinses such as chlorhexi- dura, and a ground glass appearance of the alveolar bone
dine. are MOST characteristic of
C. Risk of failure occurs MAINLY with smokers; MORE A. a vitamin D deficiency.
B. hyperparathyroidism.
rarely implant may fail because of poor positioning
C. osteomalacia.
at the time of surgery or may be overloaded initially, D. Paget’s disease.
causing failure to integrate.   8 During puberty, an adolescent is MOST susceptible to the
1. For this reason, frequently placed ONLY after pa- development of gingivitis because of
tient has stopped smoking. A. poor oral hygiene habits.
2. If BOTH smoking and positioning problems ex- B. altered capillary permeability.
ist before implant surgery, clinicians often advise C. decreased levels of hormones.
patients that a bridge or partial denture may be a D. altered chemotactic response.
BETTER solution than an implant.   9 Recent evidence indicates that desquamative gingivitis is
MOST likely a feature of all of the following, EXCEPT
one. Which one is the EXCEPTON?
Review Questions A. Menopause
B. Erosive lichen planus
C. Mucous membrane pemphigoid
D. Pemphigus
  1 Which of the following groups of risk factors for periodon- 10 The rationale for the prevalence of periodontal disease in
tal disease have the MOST capacity to be modified? patients with diabetes mellitus includes all of the following,
A. Age, genetics, and race EXCEPT one. Which one is the EXCEPTION?
B. Smoking, gender, and HIV A. Defective PMN chemotaxis
C. Diabetes, nutritional deficiencies, and neutrophil abnor- B. Microangiopathy of the periodontal tissues
malities C. Increased collagen breakdown
D. Stress, smoking, and pregnancy D. Increased fibrosis of the periodontal tissues
  2 Nutritional deficiencies are considered risk factors for peri- E. Microbial alterations
odontal disease because they 11 Multiple periodontal abscesses, velvety red gingivae, and
A. contribute to accumulation of dental biofilm. marginal proliferation of the gingivae are common oral
B. promote bone loss. manifestations of
C. lower the host response. A. type 1 diabetes.
D. are associated with lower socioeconomic groups. B. type 2 diabetes.
  3 Oral side effects of a deficiency in vitamin B6 (pyridoxine) C. uncontrolled diabetes.
include all of the following, EXCEPT one. Which one is the D. controlled diabetes.
EXCEPTION? 12 A decrease in circulating neutrophils results in
A. Gingivitis A. spontaneous gingival hemorrhage.
B. Glossitis B. oral herpetic lesions.
C. Stomatitis C. an impaired host response.
D. Candidiasis D. leukemia.
  4 A deficiency of which vitamin, responsible for the biosyn- 13 Neutropenia is present in all of the following, EXCEPT
thesis of collagen, will adversely affect periodontal connec- one. Which one is the EXCEPTION?
tive tissue and wound healing? A. Down syndrome.
A. Vitamin A B. Diabetes mellitus.
B. Vitamin B C. Autoimmune disorders.
C. Vitamin C D. CNS dysfunction.
D. Vitamin D
460   Saunders Review of Dental Hygiene

14 Cyclosporine, diltiazem, phenobarbital, and nifedipine are 23 The FIRST cell to appear in chronic inflammation that con-
drugs that stitutes the body’s second line of defense is the
A. belong to the beta-blocker category. A. neutrophil.
B. cause xerostomia. B. macrophage.
C. cause gingival enlargement. C. lymphocyte.
D. suppress the immune system. D. basophil.
15 The primary etiological factor in periodontal disease is 24 All of the following types of lymphocytes are responsible
A. bacteria. for cellular immunity, EXCEPT one. Which one is the
B. host response. ­EXCEPTION?
C. bacteria and host response. A. T4 activator cells
D. poor oral hygiene. B. T8 suppressor cells
16 Dental biofilm associated with gingival health is typically C. Killer cells
composed of D. B cells
A. gram-positive aerobic cocci and rods. 25 The chronic inflammatory cells responsible for humoral im-
B. gram-negative aerobic cocci and rods. munity are
C. gram-positive rods and spirochetes. A. macrophages.
D. gram-negative rods and spirochetes. B. B lymphocytes.
17 With a diagnosis of localized aggressive periodontitis, C. T lymphocytes.
which of the following microorganisms would be present in D. monocytes.
large levels? 26 The appearance of fibroblasts and new capillaries is charac-
A. P. gingivalis teristic of which stage of inflammation?
B. A. actinomycetemcomitans A. Acute stage
C. T. forsythensis B. Chronic stage
D. P. intermedia C. Remission stage
18 Bacterial enzymes that can affect host cells and ground sub- D. Repair stage
stance include all of the following, EXCEPT one. Which 27 The primary tissue destruction that occurs during the in-
one is the EXCEPTION? flammatory phase of periodontal disease is produced by the
A. Hyaluronidase by-products of
B. Collagenase A. bacterial cell walls.
C. Ribonuclease B. complement proteins.
D. Lipase C. antibodies.
19 Bacterial endotoxins are responsible primarily for D. arachidonic acid cascade.
A. destroying host cells. 28 The release of chemotactic factors that cause the migration
B. activating the complement cascade. of neutrophils and macrophages to specific sites is con-
C. releasing enzymes. trolled by the
D. killing bacteria. A. phagocytic system.
20 All of the following are antibacterial components found in B. complement system.
saliva, EXCEPT one. Which one is the EXCEPTION? C. arachidonic acid cascade.
A. Lysozyme D. kinin system.
B. Lactoperoxidase 29 The cytokine released by T cells that is capable of inducing
C. Macrophages bone resorption is
D. Secretory IgA A. arachidonic acid.
21 One of the following is NOT a chemical mediator of vascu- B. prostaglandin.
lar permeability. Which one is the EXCEPTION? C. interleukin-1.
A. Histamine and serotonin D. myeloperoxidase.
B. Kinins and prostaglandins 30 Within 4 to 7 days after dental biofilm accumulation, what
C. Thyroxine histopathological lesion occurs?
D. C3 and C5 A. Initial lesion
E. Lysosomal enzymes B. Early lesion
22 When PMNs begin to adhere to endothelial walls during the C. Established lesion
period of vascular dilation, this is referred to as D. Advanced lesion
A. margination. 31 When a lesion extends into the periodontal ligament and
B. emigration. alveolar bone, it is classified histologically as an
C. migration. A. initial lesion.
D. chemotaxis. B. early lesion.
C. established lesion.
D. advanced lesion.
Periodontology   461

32 The clinical signs and symptoms of occlusal trauma include 41 A 17-year-old male patient presents with purplish blue gin-
radiographic evidence of a giva, tumorlike masses around several interdental papillae,
A. narrowed periodontal ligament space and clinical shiny gingival surfaces with ulcerations, and spontaneous
­evidence of tooth mobility. bleeding. The patient looks pale and says that he has been
B. narrowed periodontal ligament space and clinical evi- tired, has not felt well for a while, and is undergoing some
dence of tooth extrusion. medical tests. A possible diagnosis might be
C. widened periodontal ligament space and clinical A. necrotizing ulcerative gingivitis.
­evidence of tooth mobility. B. primary herpetic gingivostomatitis.
D. widened periodontal ligament space and clinical evi- C. leukemia.
dence of tooth extrusion. D. puberty gingivitis.
33 Damage that occurs when the amount of alveolar bone is 42 Localized, enlarged interdental gingiva that form tumorlike
normal but excessive occlusal forces are present is referred growths may accompany
to as A. leukemic or pregnancy gingivitis.
A. primary occlusal trauma. B. pregnancy gingivitis or necrotizing ulcerative gingivi-
B. secondary occlusal trauma. tis.
C. hyperfunction. C. necrotizing ulcerative gingivitis or diabetic gingivitis.
D. hypofunction. D. diabetic gingivitis or AIDS-associated gingivitis.
34 During inspection of the patient’s mouth, several wear fac- 43 A patient complains that the gums are very sore in the lower
ets on the teeth are noted and determined to be nonfunc- right posterior region of her mouth. Her oral hygiene is me-
tional. This sign is indicative of ticulous, and there is no visible dental biofilm. Her tissues
A. decay. appear very healthy, except for the region surrounding her
B. TMJ disorder. first molar, which appears red and edematous and has sev-
C. trismus eral small vesicles on the marginal gingiva surrounding the
D. bruxism. full nonprecious metal crown. A diagnosis of this condition
35 Subgingival calculus, in the etiology of periodontal disease, might be
is a A. desquamative gingivitis.
A. primary etiological factor. B. menopausal gingivitis.
B. local predisposing factor. C. allergic gingivitis.
C. local functional factor. D. aphthous ulcers.
D. systemic contributing factor. 44 Desquamative gingivitis is frequently linked with all of the
36 Calculus is composed primarily of following EXCEPT one. Which one is the EXCEPTION?
A. inorganic matter. A. Pemphigus
B. organic matter. B. Pemphigoid
C. microorganisms. C. Lichen planus
D. protein polysaccharide complexes. D. Drugs
37 A soft, sticky, yellow or grayish white substance that par- E. Pregnancy
tially adheres to the teeth is referred to as 45 Inflammatory gingival overgrowth is associated with all
A. bacterial dental. of the following, EXCEPT one. Which one is the EXCEP-
B. materia alba. TION?
C. food debris. A. Bacterial dental biofilm
D. acquired pellicle. B. Phenytoin
38 The MOST commonly occurring form of periodontal dis- C. Cyclosporine
ease is D. Calcium channel blockers
A. gingivitis. E. Bulimia
B. chronic periodontitis. 46 Severe enlargement of the entire gingiva, including both
C. aggressive periodontitis. facial and lingual surfaces, occasionally occurs with no
D. periodontal abscess. ­apparent explanation. This condition is MOST likely
39 The MOST significant feature of chronic gingivitis is caused by
A. marginal redness. A. the use of cyclosporine.
B. edema of the gingival margins. B. the presence of type 1 diabetes mellitus.
C. bleeding on probing (BoP). C. a rare hereditary or familial condition.
D. a loss of stippling. D. the presence of epilepsy.
40 A deficiency in vitamin C causes all of the following,
­EXCEPT one. Which one is the EXCEPTION?
A. Gingival inflammation
B. An increased tendency for hemorrhage
C. The degeneration of collagen fibers
D. Poor wound healing
462   Saunders Review of Dental Hygiene

47 A 19-year-old college student presents for emergency care 53 The personal history is important for the identification of
complaining of pain in the gingiva, spontaneous bleeding, all of the following nondentally related risk factors for peri-
and a bad metallic taste in his mouth. During inspection a odontal disease, EXCEPT one. Which one is the EXCEP-
fetid odor, punched-in interdental papillae covered with a TION?
gray pseudomembrane, and the spontaneous bleeding are A. Smoking
noted. This patient MOST likely has B. Alcohol consumption
A. primary herpetic gingivostomatitis. C. Diet
B. leukemia. D. Stress
C. necrotizing ulcerative gingivitis. E. Social status
D. AIDS-associated gingivitis. 54 Lifesaver-like enlargements surrounding the free gingival
48 A periodontal pocket whose base is coronal to the level of margins are BEST referred to as
the alveolar bone is referred to as a(n) A. rolled margins.
A. pseudopocket. B. Stillman’s clefts.
B. relative pocket. C. McCall’s festoons.
C. suprabony pocket. D. marginal hyperplasia.
D. infrabony pocket. 55 The “gold standard” for determining the extent of periodon-
49 A patient with generalized moderate chronic periodontitis tal tissue destruction is the measurement of
received scaling of the maxillary right arch 2 weeks ago. A. clinical attachment level.
The patient returns for treatment of the mandibular right B. periodontal pockets.
quadrant but complains of a dull, throbbing pain around the C. bleeding in response to probing.
maxillary right molar area. During inspection, an 11-mm D. bone loss through radiographic analysis.
pocket depth and accompanying suppuration on the mesial 56 The periodontal screening and recording (PSR) system is a
facial surface of tooth #2 are found. The chart indicates that modification of the
the pocket depth before scaling was 6 mm. This patient A. Ramfjord index.
MOST likely has developed B. Russell index.
A. a periodontal abscess. C. CPITN (WHO).
B. a periapical abscess. D. Loe and Silness index.
C. an endo/perio lesion. 57 It is important for a dental hygienist to examine the condi-
D. pericoronitis. tion of tooth proximal contact areas because
50 A systemic condition characterized by hyperkeratotic skin A. drifting may occur if contacts are not tight.
lesions of the palms, soles, knees, and elbows, severe de- B. contact areas affect a patient’s homecare abilities.
struction of the periodontium, and onset before age 4 is C. contact areas affect a patient’s occlusion.
A. Chédiak-Higashi syndrome. D. extrusion may change a contact point.
B. hypophosphatasia. 58 Tooth mobility, furcation involvement, malocclusions, over-
C. Papillon-Lefèvre syndrome. hanging margins of restorations, and the status of prosthetic
D. Down syndrome. appliances are all important because they are classified as
51 LGE is associated with A. etiological factors for periodontal disease.
A. diabetic gingivitis. B. systemic risk factors for periodontal disease.
B. leukemic gingivitis. C. local risk factors for periodontal disease.
C. AIDS-associated gingivitis. D. primary risk factors for periodontal disease.
D. chronic gingivitis. 59 The use of supplemental periodontal diagnostic tests should
52 You learn at his appointment that thyroid cancer has be reserved exclusively for
been diagnosed in your patient and he must undergo ra- A. the identification of risk factors.
diation therapy. How should you plan his periodontal B. definitive periodontal diagnosis.
therapy? C. treatment planning purposes.
A. Perform all necessary periodontal therapy immediately, D. identifying unusual periodontal diseases.
before his radiation therapy. 60 The BANA test, ELISA test, and DNA probe are all used for
B. Wait until after his radiation therapy has been completed the measurement of
and the cancer has been eliminated before proceeding A. biochemical markers in gingival crevicular fluid.
with his periodontal therapy. B. bacterial enzymes.
C. Ask the patient whether he wishes to have his periodon- C. bacterial species.
tal therapy before or after radiation therapy. D. bacterial antibodies.
D. Do noninvasive therapy before his radiation treatments 61 The phase of periodontal treatment that involves the elimi-
and complete the more invasive care after he is feeling nation of etiological factors is
better. A. phase I therapy.
B. phase II therapy.
C. phase III therapy.
D. phase IV therapy.
Periodontology   463

62 All periodontal patients who have received initial therapy 70 Periodontal surgical dressings are placed over a surgical
require an evaluation after wound for protection and should remain in place for
A. 1 to 2 weeks. A. 3 to 5 days.
B. 4 to 6 weeks. B. 7 to 10 days.
C. 2 to 3 months. C. 12 to 14 days.
D. 4 to 6 months. D. 18 to 21 days.
63 An assessment of Mr. Forbes identifies his condition as 71 Which one of the following is NOT a goal of periodontal
ADA case type III, with average pocket depths of 5 to 6 maintenance therapy?
mm and moderate to heavy subgingival calculus in MOST A. Preservation of clinical attachment levels
areas. His initial therapy should MOST closely resemble B. Maintenance of alveolar bone height
which of the following nonsurgical periodontal therapy C. Control of inflammation
treatment plans? D. Evaluation and reinforcement of the patient’s oral
A. One appointment for completion of all debridement ­hygiene
B. Two appointments without any local anesthesia E. Correction of mucogingival defects
C. Two appointments with local anesthesia 72 Compliance with oral hygiene measures decreases within
D. Four appointments with local anesthesia A. 1 month.
64 Rationales for nonsurgical periodontal therapy include all of B. 2 months.
the following, EXCEPT one. Which one is the EXCEPTION? C. 3 months.
A. The need for surgical access D. 4 months.
B. The modification of risk factors 73 Which of the following procedures is NOT a part of the
C. The removal of bacterial dental (etiological agent) and recare appointment?
other local irritants A. Complete periodontal evaluation
D. The initiation of dental control measures B. Quadrant scaling with anesthesia
E. The evaluation of tissue response C. Oral hygiene instructions
65 Which one of the following is NOT a goal of periodontal D. Tooth desensitization as needed
surgery? 74 During one of Mrs. Smith’s routine recare appointments,
A. Reducing pocket depths for better access to homecare increases in pocket depths in several areas, multiple areas
B. Correcting mucogingival defects of bleeding in response to probing, and moderate to heavy
C. Retaining inflamed tissues calculus deposits in several molar areas are detected. The
D. Regenerating tissues lost to disease BEST approach would be to
66 A surgical procedure that reduces pocket depths by remov- A. review oral hygiene more thoroughly, emphasizing the
ing the soft tissue pocket wall in supragingival pockets is effects of noncompliance.
referred to as a B. treat the areas that exhibit breakdown.
A. gingivectomy. C. repeat initial therapy by quadrant and reevaluate after
B. modified Widman flap. 4 weeks.
C. guided tissue regeneration. D. refer Mrs. Smith to a periodontist for surgical interven-
D. ostectomy. tion.
67 One-walled and two-walled infrabony pockets are BEST 75 The acute periodontal abscess initially is BEST treated
treated surgically with which of the following procedures? with
A. Ostectomy A. drainage of the abscess.
B. Osteoplasty B. antibiotics.
C. Osteotomy C. tooth extraction.
D. Ostectomy and osteoplasty D. tetracycline fiber placement.
E. Osteoplasty and osteotomy 76 A raised, painful area of acute inflammation on the marginal
68 Bone grafting material that is taken from another person gingiva, with no accompanying periodontal pockets and no
(freeze-dried from cadavers) is referred to as a(n) radiographically visible bone loss, MOST likely is a(n)
A. allograft. A. acute periodontal abscess.
B. autograft. B. periapical abscess.
C. xenograft. C. gingival abscess.
D. alloplast. D. combination abscess.
69 Which of the following is a description of a type II muco- 77 If a patient presents with pericoronitis, the dental hygienist’s
gingival defect? role in caring for the patient may include all of the follow-
A. Alveolar mucosa acts as marginal gingiva, without a ing, EXCEPT one. Which is the EXCEPTION?
zone of attached gingiva. A. Irrigation of the area under the operculum
B. Pockets extend to or beyond the mucogingival junc- B. Gentle debridement under the operculum
tion. C. Providing a prescription for antibiotics
C. Gingiva is recessed beyond the mucogingival junction. D. Providing oral hygiene instruction
D. Port-hole defect is present in the alveolar mucosa.
464   Saunders Review of Dental Hygiene

78 The MOST important instruction that a dental hygienist can 85 Approximately 2 weeks ago, four quadrants of periodontal
provide to the parents of a patient with primary herpetic debridement were completed on Catherine, a 34-year-old
gingivostomatitis is woman with diabetes and the mother of two young chil-
A. to keep the child isolated because the virus is conta- dren. She presents for her reevaluation, during which an ap-
gious. propriate maintenance (recare) interval will be determined.
B. to engage in gentle, daily dental biofilm removal. Catherine is removing dental biofilm fairly well and her gin-
C. to strictly follow diet instructions. gival tissue has improved, as evidenced by decreased bleed-
D. to encourage daily fluid intake. ing, inflammation, and pocketing. Slight marginal redness
79 If a patient with necrotizing ulcerative gingivitis (NUG) or ­remains on the lingual surface of the mandibular molars. A
a patient with pericoronitis has been prescribed a hydro- 3-month maintenance interval is recommended. What fac-
gen peroxide rinse, the patient should discontinue the rinse tor was the PRIMARY reason for this recommendation?
­after A. Response to the individualized debridement
A. 3 days. B. Dental biofilm removal ability
B. 7 days. C. Current oral health or disease state
C. 14 days. D. Periodontal disease risk factors
D. 21 days.
80 A root-form or blade-form implant placed directly into the
bone is classified as a(n)
A. endosseous implant.
Answer Key and Rationales
B. subperiosteal implant.
C. transosteal implant.
D. staple implant. 1 (D) Although age, gender, genetics, and neutrophil
81 The MAIN purpose of two-step implant surgery is to allow (PMN) abnormalities are NOT modifiable, possi-
A. fibrointegration. ble to minimize or control stress and quit smoking
B. osseointegration. (HARDEST modifiable factor). Because pregnancy
C. bone regeneration. is a temporary condition, it is considered modifiable.
D. settling to occur. 2 (C)  Poor nutrition results in lowered resistance (host
82 The interface or biological seal found around an implant is response) to infections, including periodontal diseases.
created by the formation of a long junctional epithelium that Individuals with nutritional deficiencies are therefore
is attached to the implant surface by
MORE at risk for severe forms of periodontal disease;
A. glycosaminoglycans.
B. connective tissue.
however, such deficiencies have NOT been shown to
C. hemidesmosomes. contribute to dental biofilm accumulation, bone loss,
D. gap junctions. or lower socioeconomic status (SES).
83 Dental biofilm in a recently cleaned mouth contains 3 (D)  Vitamin B6 (pyridoxine) deficiency is NOT asso-
A. leukocytes, epithelial cells, and a few gram-positive ciated with the development of candidiasis. Vitamin B6
cocci. is involved in carbohydrate metabolism, and depriva-
B. leukocytes, cocci with filaments increasing, and newly tion may result in generalized stomatitis, gingivitis,
formed rods. glossitis, and other oral symptoms.
C. gram-negative anaerobic bacteria, white blood cells, 4 (C)  Vitamin C is essential to collagen biosynthesis; de-
and spirochetes. ficiency adversely affects periodontal connective tissue
D. gram-negative aerobic bacteria, gram-positive cocci,
integrity and wound healing. Vitamin A is associated
and spirochetes.
84 Which statement is CORRECT regarding calculus?
with the synthesis of epithelial cells. Vitamin B is a
A. By itself, can cause disease group of vitamins with multiple roles. Vitamin D is in-
B. Attachment mechanism for oral biofilm volved in maintaining calcium and phosphorus levels.
C. Easily removed with a sharp curet 5 (D) Vitamin D deficiency is NOT associated with
D. Supragingivally appears darker because of blood development of gingival hyperplasia. In adults, is
­pigments called osteomalacia and is characterized by destruc-
tion of PDL, resorption of alveolar bone, replacement
fibrous dysplasia.
6 (D) Severe protein deficiency, or kwashiorkor, has
been associated with increased incidence and severity
of necrotizing gingivitis. Marasmus (general starva-
tion), osteoporosis, and osteomalacia have NOT been
linked with necrotizing gingivitis.
7 (B)  Multilocular radiolucent jaw lesions, loss of lam-
ina dura, and ground glass appearance of the alveolar
bone are characteristics of excessive production of
Periodontology   465

p­ arathyroid hormone and hyperparathyroidism. This 17 (B) Elevated levels of Aggregatibacter (previously


combination of lesions is NOT characteristic of the Actinobacillus) actinomycetemcomitans (Aa) have
­others. been associated with localized aggressive periodontal
8 (B) Increased, rather than decreased, levels of hor- disease. Note also that Tannerella forsythensis (TF)
mones alter capillary permeability and increase tis- was previously Bacteroides forsythus (Bf).
sue fluid accumulation in the gingiva, resulting in 18 (D)  Lipase is an enzyme that aids fat metabolism,
an increased risk for gingivitis in presence of dental NOT a bacterial enzyme. Proteolytic enzymes such as
biofilm. Altered chemotactic response has NOT been hyaluronidase, collagenase, and ribonuclease affect
demonstrated in cases of puberty gingivitis. host cells and intercellular substance (ground sub-
9 (A) Desquamative gingivitis is NOT associated with stance) by increasing permeability of epithelial and
menopause, but MORE likely a feature of mucocuta- connective tissues and outwardly destroying cells.
neous disorders, such as lichen planus, mucous mem- 19 (B) Bacterial endotoxins are responsible for activat-
brane pemphigoid, pemphigus. ing alternate pathway of the immune complex fixa-
10 (D) Increased fibrosis of the periodontal tissues is tion process known as the complement cascade. Side
NOT a common occurrence in diabetic patients and reaction proteins of this process are BOTH chemo-
is NOT related to prevalence. Defective PMN che- tactic and cytolytic.
motaxis, microangiopathy of periodontal tissues, 20 (C) Macrophages are large defense cells that ap-
increased collagen breakdown, microbial alterations pear in larger numbers during chronic inflammation,
have been cited as rationales for prevalence of peri- typically NOT found in saliva. Saliva contains anti-
odontal disease in individuals with diabetes mellitus. bacterial components such as lysosomes and lacto-
11 (C) In uncontrolled diabetes mellitus, oral manifes- peroxidase, as well as secretory IgA.
tations include multiple periodontal abscesses, vel- 21 (C) Histamine, serotonin, kinins, prostaglandins, C3
vety red gingivae, and marginal proliferation of the and C5, and lysosomal enzymes are ALL mediators
periodontal tissues. Although risk for periodontal of vascular permeability, unlike thyroxine (T4) from
breakdown exists in controlled cases of type 1 and the thyroid gland, which is involved in controlling
type 2 diabetes mellitus, oral manifestations can be rate of metabolic processes in the body and influenc-
controlled with frequent periodontal maintenance ing physical development.
(PM, recare) appointments. 22 (A) Process by which PMNs leave the central stream
12 (C)  Although it is well established that periodontal and adhere to the endothelial cell walls of blood ves-
disease is caused by dental biofilm, it is also recognized sels is called pavementing or margination. Emigra-
that impairment in host response increases risk for tion, migration, and chemotaxis follow as cells escape
periodontal breakdown. Because neutrophils (PMNs) through endothelial walls into the connective tissues
are KEY players in the host response, shortage results and move along chemical gradients to the attracting
in increased risk for periodontal infections. microorganisms.
13 (D) Cause of neutropenia is multifactorial but is NOT 23 (B)  Monocyte and connective tissue macrophage sig-
associated with CNS dysfunction. Because term neu- nal the beginning of chronic stage of inflammation.
tropenia refers to reductions in the neutrophil (PMN) Soon afterward, lymphocytes arrive at scene and even-
count, conditions such as Down syndrome, diabetes tually become the predominant cell. Basophils typically
mellitus, and a variety of autoimmune disorders are are present ONLY in allergic conditions. The neutrophil
all capable of producing neutropenic states. (PMN) is the predominant cell in acute inflammation
14 (C) ONLY thing that cyclosporine (immunosuppres- and is first cell on scene.
sant), phenobarbital (an anticonvulsant), and diltia- 24 (D) B cells are responsible for humoral immunity.
zem and nifedipine (calcium channel blockers) have Cells that are responsible for cellular immunity are
in common is ability to produce gingival overgrowth T4 activator cells, T8 suppressor cells, cytotoxic
by hyperplasia. (killer) cells.
15 (A) MAIN etiological agent in the development of 25 (B) The B-cell lymphocyte is responsible for humoral
periodontal disease is dental biofilm, which contains immunity, and the T-cell lymphocyte is responsible
pathogenic bacteria. However, host response is rec- for cellular immunity. Macrophage and/or monocyte
ognized as a balancing factor because mere presence is a phagocytic cell that appears first during chronic
of bacteria does NOT preclude disease. ­inflammation.
16 (A) Dental biofilm associated with gingival health 26 (D) Combination of fibroblasts, new immature colla-
typically is composed of gram-positive aerobic cocci gen, new capillaries constitute granulation tissue that
and rods. Gram-negative microorganisms increase in is formed during repair stage of inflammation.
numbers as gingivitis and periodontitis progress. Spi- 27 (D) Products of the arachidonic acid cascade are po-
rochetes are gram-negative organisms. tentially harmful to periodontal tissues and are part of
466   Saunders Review of Dental Hygiene

the pathogenesis of periodontal disease. BOTH an- matter. Organic component is a mixture of protein
tibodies and complement proteins are later attracted polysaccharide complexes, desquamated epithelial
and assist in the destruction. cells, leukocytes, carbohydrates, lipids, glycosami-
28 (B) Complement system is responsible for the re- noglycans, various types of microorganisms. Pri-
lease of chemotactic factors that cause the migration mary crystalline structure is calcium hydroxyapatite,
of neutrophils (PMNs) and macrophages to specific Ca10(PO4)6(OH)2; includes trace amounts of octocal-
sites. Phagocytic system is composed of PMNs and cium phosphate, whitlockite, brushite.
macrophages. Arachidonic acid cascade and kinin 37 (B) Materia alba is a yellow-gray acquired bacterial
system are responsible for other aspects of the im- coating, soft, sticky, LESS adherent than dental bio-
mune response. film. BOTH dental biofilm and salivary acquired pel-
29 (C) As part of the process of cellular immunity, licle are usually visible ONLY if stained with a dye
T cells release potent cytokines, such as interleukin-1, (disclosing solution). Food debris may be any color
that are capable of causing bone resorption. Arachi- or texture, depending on retained food.
donic acid has the potential for tissue destruction, and 38 (A) MOST commonly occurring form of periodon-
prostaglandin is a mediator of vascular permeability. tal disease is gingivitis, inflammation of the gingival
Myeloperoxidase is found in the saliva and is a pro- tissues. Chronic periodontitis is the second MOST
tective enzyme. common form of periodontal disease; aggressive
30 (B) Initial lesion occurs in the first 2 to 4 days, early periodontitis and periodontal abscess are LESS
lesion occurs in 4 to 7 days, established lesion occurs ­common.
after 14 days, and advanced lesion occurs ONLY after 39 (C) MOST significant feature of chronic gingivitis
inflammation invades supporting periodontal tissues. is marginal bleeding on probing (BoP); marginal red-
31 (D) Advanced lesion is characterized by invasion of ness, edema, loss of stippling may or may NOT be
periodontal tissues, PDL and alveolar bone. Initial, found.
early, and established lesions do NOT typically in- 40 (A) Vitamin C deficiency alone CANNOT cause in-
volve PDL and alveolar bone. flammation; however, deficiency in vitamin C causes
32 (C) Clinical signs and symptoms of occlusal trauma increased tendency for gingival hemorrhage, degen-
always include radiographic evidence of a widening eration of the collagen fibers, poor wound healing.
of periodontal (PDL) space and clinical evidence of 41 (C) Clinical characteristics of leukemia include pur-
tooth mobility. Tooth extrusion typically is related to plish blue gingiva, interdental tumorlike masses, and
the LACK of opposing tooth. Narrowed PDL space is shiny gingival surfaces that may have ulcerations and
associated with a LACK of occlusal contact. spontaneous bleeding. A case of necrotizing ulcer-
33 (A) Primary occlusal trauma is defined as damage ative gingivitis (NUG) should be ruled out because
that occurs when the amount of alveolar bone is nor- purplish blue gingival tissues and tumorlike masses
mal but there is an INCREASE in occlusal forces. are NOT common characteristics. Puberty gingivitis
Secondary occlusal trauma occurs when there is a must be ruled out because the patient is older. Be-
deficiency in the amount of alveolar bone support. cause of his age, primary herpetic gingivostomatitis,
Neither hyperfunction NOR hypofunction results in which typically occurs during the first 5 years of life
irreversible damage. and is NOT associated with tumorlike masses, also
34 (D) Bruxism is MOST common oral habit; involves should NOT be a consideration.
grinding and can be recognized by presence of non- 42 (A) Localized tumorlike growths on the interden-
functional wear facets. Wear facets are NOT associ- tal gingiva are commonly found in BOTH leuke-
ated with decay, TMJ disorder (TMD), or trismus. mic gingivitis and pregnancy gingivitis (pyogenic
35 (B) Dental biofilm is considered MAIN etiological granuloma). However, these enlargements are NOT
factor in development of periodontal disease; how- characteristic of NUG, diabetic gingivitis, or AIDS-
ever, several contributing factors have been identified associated gingivitis.
that make individual MORE prone to oral biofilm 43 (C) Allergic hypersensitivity involves an abnormal
retention and thus disease risk. One such factor is response of the tissues to specific agents, which typi-
subgingival calculus, considered a local predispos- cally manifests as redness, pain, edema, ulceration,
ing factor. Missing teeth, malocclusions, and trau- and possible vesicle formation. Patient has an allergy
matogenic occlusion are considered local functional to nonprecious metal crown on #30. No mention was
factors. Systemic contributing factors include those made of desquamating tissues or tissues that were
conditions that lower host response and raise the risk peeling away, which rules out desquamative gingivi-
of disease. tis. Menopausal gingivitis has NOT been established
36 (A) Calculus is composed of approximately 70% as an acceptable term, and aphthous ulcers should
to 90% inorganic matter and 10% to 30% organic be ruled out because they occur ONLY in the soft
Periodontology   467

tissues (nonkeratinized), NOT on tissues overlying 51 (C) LGE is typically associated with patients who
bone (keratinized). have HIV/AIDS. Characteristic red band of marginal
44 (E) Pregnancy is associated with pyogenic granu- gingiva that rarely bleeds has NOT been demon-
loma (pregnancy tumor) rather than desquamative strated in diabetic or leukemia-associated gingivitis
gingivitis. Although the exact nature of desquamative or even chronic gingivitis.
gingivitis is unknown, recent links have been made to 52 (A) Individuals undergoing cancer radiation therapy
several drugs and dermatological diseases, including are at risk for development of osteoradionecrosis and
pemphigus, pemphigoid, and lichen planus. should have any significant periodontal disease elim-
45 (E) Eating disorder, bulimia, is associated with inated and teeth extracted before radiation therapy.
enamel erosion rather than gingival overgrowth. Continuing periodontal therapy after radiation treat-
Drugs such as phenytoin (Dilantin), cyclosporine, ment begins is extremely risky.
and calcium channel blockers initially cause gingi- 53 (E) Social status is NOT an important characteristic
val overgrowth by hyperplasia. When dental biofilm to gather from the personal history, unlike other non-
accumulates on the enlarged gingival tissues, inflam- dental risk factors such as stress, smoking, diet, alco-
matory response ensues and results in inflammatory hol consumption, which may play a role in patient’s
gingival overgrowth. periodontal condition.
46 (C) Rare hyperplastic disease with unknown etiol- 54 (C) Lifesaver-like enlargements surrounding free
ogy, idiopathic gingival hyperplasia, believed to be gingival margins are McCall’s festoons. Stillman’s
familial or hereditary. NO association has been estab- clefts are narrow, slitlike areas where marginal gin-
lished between this condition and diabetes, epilepsy, giva has receded. Rolled margins are LESS enlarged
or use of drugs. than McCall’s festoons, and marginal hyperplasia
47 (C) Spontaneous bleeding, metallic taste, fetid odor, demonstrates greater overall enlargement.
punched-in interdental papillae, gray pseudomem- 55 (A) Clinical attachment level (CAL) is considered
brane are ALL typical characteristics of necrotizing “gold standard” for determining the extent of peri-
ulcerative gingivitis (NUG). Primary herpetic gingi- odontal tissue destruction. Pocket depths may vary
vostomatitis and leukemia do NOT exhibit the char- according to amount of edema and hyperplasia.
acteristic punched-in interdental papillae. Bleeding on probing (BoP) is indicator of active dis-
48 (C) Periodontal pockets are classified as either su- ease but does NOT indicate extent of damage. Bone
prabony or infrabony. If the base of the pocket is coronal loss through radiographic analysis is less reliable be-
to the alveolar bone, it is a suprabony pocket. However, cause of associated distortion and magnification is-
if the base of the pocket is apical to the alveolar bone sues and extent of visualization.
crest, it is infrabony. BOTH pseudopockets and relative 56 (C) The PSR was developed to encourage general
pockets are gingival and NOT periodontal pockets. practitioners to probe routinely. System is modifi-
49 (A) Rapid pocket destruction (from 6 mm to 11 mm cation of the CPITN, which was developed by the
within a few weeks) and presence of suppuration are World Health Organization (WHO). Other indices
suggestive of periodontal abscess. Entrapment of are measurements of dental biofilm, debris, calculus,
virulent microorganisms after an incomplete scaling gingivitis.
procedure results in pus formation and rapid tissue 57 (B) Condition of tooth proximal contact areas should
destruction. Periapical abscess results in intermittent be evaluated because open contacts encourage food
sharp pain with localized swelling. The combination impaction and tight contacts prevent effective inter-
(endodontic-periodontal) lesion has characteristics proximal cleaning. This is of MAJOR concern to
of BOTH periapical and periodontal abscesses; peri- dental hygienists, who develop homecare strategies
coronitis (pericoronal abscess) is a severe, localized for patients to facilitate cleansing of these difficult
infection associated with third molars. areas.
50 (C) Papillon-Lefèvre syndrome is characterized by 58 (C)  Tooth mobility, furcation involvement, maloc-
hyperkeratotic skin lesions of the palms, soles, knees, clusions, overhanging margins of restorations, and
and elbows; involves severe destruction of the peri- status of prosthetic appliances are considered local
odontium before age 4. Chédiak-Higashi syndrome risk factors for the development of periodontal dis-
produces partial albinism, slight bleeding disorders, ease. Dental biofilm is considered the MAIN etiolog-
and periodontal diseases. Hypophosphatasia is char- ical factor; systemic diseases, host response factors,
acterized by poor bone formation, rickets, and the genetic predispositions are considered systemic risk
premature loss of the dentition. Down syndrome (tri- factors.
somy 21) manifests as mental and growth retardation 59 (A)  Although supplemental diagnostic tests were
and is characterized by a high prevalence of aggres- developed to diagnose specific periodontal diseases,
sive periodontal disease. none is truly diagnostic. However, these tests play
468   Saunders Review of Dental Hygiene

important role in identification of risk factors and, 69 (A)  A type II mucogingival defect refers to a de-
coupled with clinical evidence, can be strong predic- fect in which the alveolar mucosa acts as marginal
tors of future disease progression. gingiva without a zone of attached gingiva. Type I
60 (C)  All tests measure bacterial species. The BANA defects occur when pockets extend to or beyond the
test measures presence of T. forsythensis (previously mucogingival junction but have a firm keratinized
B. forsythus), T. denticola, and P. gingivalis. The pocket wall.
ELISA and DNA tests measure quantities of P. gingi- 70 (B)  Periodontal surgical dressings are placed over
valis, P. intermedia, A. actinomycetemcomitans. the surgical wound for protection and should remain
61 (A)  Phase I therapy is initial stage, deals with elimi- in place for at LEAST 7 days to allow for adequate
nation of etiological factors. Phase II is surgical healing. Dressing and suture removal typically are
phase, phase III is restorative and orthodontic phase, performed concomitantly.
phase IV is periodontal maintenance (PM, recare) 71 (E)  Correcting mucogingival defects is NOT a goal
procedures. of recare or periodontal maintenance (PM) therapy.
62 (B)  All periodontal patients require a 1-month (4 to Such therapy is performed to maintain optimal oral
6 weeks) reevaluation after initial therapy to assess health through preservation of clinical attachment
tissue response and to establish appropriate periodon- levels (CAL), maintenance of alveolar bone height,
tal maintenance (PM, recare) interval or determine control of inflammation, as well as the evaluation and
whether referral to a periodontist for further treat- reinforcement of patient oral hygiene.
ment is necessary. 72 (A)  Compliance studies have shown that oral hy-
63 (D)  For moderate to heavy calculus deposits and 5- giene decreases within 30 days of instruction
to 6-mm pocket depths, quadrant scaling with local (1 month), which suggests a MAJOR challenge for
anesthesia would be the BEST possible treatment op- the dental hygienist regarding patient motivation.
tion (case type IV). 73 (B)  Quadrant scaling with anesthesia is performed
64 (A)  Need for surgical access is a rationale for surgi- during initial therapy, NOT during recare (periodon-
cal therapy. Rationales for nonsurgical periodontal tal maintenance [PM]) appointments. Complete peri-
therapy (NSPT) include modification (control) of odontal examination, oral hygiene instructions, and
risk factors, removal of etiological agents, initia- periodontal debridement are routinely performed at
tion of dental biofilm control, evaluation of tissue recare appointments. Tooth desensitization is typi-
response. cally performed based on need during recare appoint-
65 (C)  Retaining inflamed tissues is NOT a goal of peri- ments.
odontal surgery; goals are to reduce pocket depths, 74 (C)  With the return of active disease, initial therapy
provide BETTER access for homecare, correct mu- should be repeated by quadrant with anesthesia, fol-
cogingival defects, remove inflamed tissues, regener- lowed by a reevaluation in 4 weeks. At the reevalua-
ate tissues lost to illness. tion appointment, appropriate course of action should
66 (A)  Gingivectomy procedure is designed to reduce be determined and may include referral to a special-
pocket depths by removing the soft tissue pocket ist, return to recare (periodontal maintenance [PM])
wall in suprabony pockets. Modified Widman flap intervals, or other treatment options such as antibiot-
exposes the periodontium to provide access for other ics and surgery.
­procedures. Guided tissue regeneration involves 75 (A) Treatment of a periodontal abscess includes
placement of barrier materials to prevent downgrowth drainage and antimicrobial irrigation. Antibiotics are
of JE and stimulate MORE coronal reattachment of prescribed ONLY if lymphadenopathy and/or fever is
connective tissues. Ostectomy involves removal of present. Tooth extraction and tetracycline fiber place-
alveolar bone. ment are rarely considered.
67 (D)  Ostectomy and osteoplasty are used to treat one- 76 (C)  Gingival abscesses are NOT associated with
walled and two-walled defects in infrabony pockets. bone loss or deeper periodontal pockets, but peri-
Osteotomy refers to the sectioning and repositioning odontal abscesses are always associated with BOTH.
of bone and is employed during mandibular advance- Periapical and combination abscesses involve peri-
ment surgery. apical bone loss; combination abscesses may also be
68 (A)  Freeze-dried bone from cadavers that is used as characterized by deeper periodontal pockets.
grafting material is an allograft. Autografts involve 77 (C)  If fever and lymphadenopathy are present, su-
donor bone taken from the patient’s own body, and pervising dentist may decide to prescribe an antibi-
xenografts are bone specimens obtained from other otic. Dental hygienist’s role would involve gentle
species (e.g., cows). Alloplast materials are synthetic debridement, irrigation of the area under operculum
substances that are used for bone grafting proce- with antimicrobial agent, and provision of oral hy-
dures. giene instruction.
Periodontology   469

78 (D)  Painful lesions associated with this condition Hemidesmosomes are part of the basal lamina and
often prevent the child from eating and drinking. De- basement membrane that forms between the tooth
hydration in a child may be life threatening, therefore and the epithelium and therefore are NOT part of
IMPORTANT that daily fluid intake be stressed to the the connective tissue. Gap junctions are cell-to-cell
parent. attachment mechanisms that typically are found in
79 (B)  Use of hydrogen peroxide intraorally MUST be epithelial tissues.
discontinued in 7 to 10 days to prevent the develop- 83 (A)  Dental biofilm in a recently cleaned mouth con-
ment of black hairy tongue. tains leukocytes, epithelial cells, few gram-positive
80 (A)  Endosseous implants are shaped in the form cocci. Biofilm that is 2 to 14 days old will contain
of roots and blades and are directly implanted into leukocytes, filamentous cocci, rods, gram-negative
the bone. Subperiosteal implants are in the form of anaerobic bacteria, gram-positive cocci, white blood
a metal framework that is placed over the alveolar cells, spirochetes.
bone to provide support for complete and removable 84 (B)  Calculus acts as an attachment mechanism for
dentures. Transosteal (staple) implants are surgically dental biofilm; by itself, does NOT cause disease.
placed through the mandible to provide anchor for a May be hard to remove regardless of sharpness of the
removable lower denture. instrument. Subgingival calculus may appear darker
81 (B)  After the initial placement of implant, surgical because of blood pigments.
site is covered for approximately 3 to 6 months to 85 (D)  A patient with insulin-dependent diabetes has a
allow osseointegration (implant to bone integration) compromised immune system, especially if glucose
to take place; this process leads to MOST stable at- levels are unstable. A 3-month recare or periodontal
tachment in most cases. Fibrointegration should be maintenance (PM) interval is recommended because
avoided because implant failure typically follows. diabetes is a risk factor for periodontal disease. Fur-
Bone regeneration and settling are NOT related ther periodontal destruction may result if therapy is
­factors. NOT provided at frequent intervals. Catherine’s re-
82 (C)  Cells at the base of the implant sulcus form a sponse to treatment will be short-term, depending on
junctional epithelium (JE) that attaches to the implant dental biofilm removal ability, length of appointment,
by means of a basal lamina and hemidesmosomes. and control of diabetes. Her risk factors play the
Glycosaminoglycans constitute intercellular “glue.” MOST important role in determining a PM phase.
C H A P TE R   14

Pain Management
Dental Pain   (sympathetic ganglion [SG]) that open and close
Pain is sum total of responses (behavioral, emotional, depending on a number of factors:
motivational, psychological) to actual or impending tis- a. When gates are opening, MORE pain messages
sue damage from noxious stimulus. get through and pain can be intense.
• See CD-ROM for Chapter Terms and WebLinks. b. When gates close, pain messages are prevented
A. Types of dental pain: from reaching the brain and pain may NOT
1. Caused by dental disease or trauma. even be experienced.
2. As result of dental treatment (iatrogenic). F. Responses to pain:
3. After dental treatment. 1. Physiological: increased respiratory rate, increased
B. Pain perception: heart rate (HR), increased blood pressure (BP).
1. Physioanatomical process by which pain is re- 2. Physical: crying out, tapping feet, cold sweat, altered
ceived and transmitted by neural structures from facial expression, white knuckles, inability to sit still.
end organs and pain receptors, through conductive G. Pain control and dental office:
and perceptive mechanisms. 1. Pain is a major factor that brings patients to dental
2. Does NOT differ much from person to person, office, while fear and anxiety about pain are MOST
same in MOST healthy persons, but can be af- common reasons patients fail to seek dental care;
fected by both disease and toxic states. many AVOID dental treatment until forced into
C. Pain reaction: office with an emergency.
1. Manifestation of perception of pain that has been 2. Control of pain and anxiety is therefore essential
perceived by the brain. part of dental practice:
2. Determines what a patient will do about the un- a. To accomplish this objective, various techniques
pleasant experience of pain. are used, including psychological approaches,
3. Differs a great deal from person to person because local anesthetics of various types, and combina-
of past dental experiences and various factors such tions of sedative and general anesthetic agents.
as fatigue, stress, fear, apprehension, age, emo- b. Choice of MOST appropriate modality for par-
tional state, education. ticular situation is based on training, knowledge,
D. Pain threshold: tolerance shown to pain; there are and experience of clinician; nature, severity,
differing levels: duration of procedure; age and physical and
1. Hyporeactive: high pain threshold, tolerates pain psychological status of patient; level of fear
well, shows little reaction to pain. and anxiety; previous responses to pain control
2. Hyperreactive: low pain threshold, does NOT ­procedures.
­tolerate pain well, shows more reaction to pain. c. MUST keep “gates closed” to pain messages to
3. However, MOST patients vary between two types control dental pain (see earlier discussion) (No
depending on factors (discussed earlier). pain, lots of gain!).
E. Gate control theory of pain (GCT): 3. Important part of stress control protocol, especially
1. Explains unusual phenomenon of pain past specific in management of patients with cardiovascular dis-
neural pathway of pain sensation. ease (CVD).
2. Specific neuroanatomical pathway carries impulse
from site of stimulus to cortex of the brain where Anxiety Management
it is perceived as pain and then person receives a Anxiety is the feeling of apprehension and fear char-
painful sensation. acterized by physical symptoms such as palpitations,
3. Thus perception of physical pain is NOT direct result sweating, feelings of stress. Anxiety keeps many peo-
of activation of pain receptor neurons, but instead is ple from receiving necessary dental treatment because
modulated by interaction between different neurons. of fear of pain or discomfort. Fear is excessive appre-
o0400 4. Before pain messages can reach the brain, mes- hension or anxiety. Understanding dental fear can help
sages encounter “nerve gates” in spinal cord in selecting appropriate methods for alleviating patient

470
Pain Management   471

discomfort. Anxiety and fear are common occurrences and the extraction ­proceeded without anesthesia for the
in the dental office and can be managed by a variety of ­patient. Lately her gums have been bleeding and sore.
techniques. Sometimes fear can become excessive and Her ­ husband persuaded her to have a thorough exam
involve a phobia, which promotes inaction (failure to and cleaning. After diagnosis of generalized chronic ­
seek necessary dental treatment). periodontitis, her treatment plan suggests nonsurgical
A. Etiology of dental anxiety and fear: periodontal therapy by quadrant dental hygiene using
1. Anxiety and fear are typically based on past dental ­local anesthesia of the involved regions. On the day of
experiences. her appointment with the dental hygienist, the patient
2. May be based on fearful experiences related by calls and cancels.
others (friends or family) or portrayed by media.
3. Iatrogenic causes arise from personal experiences 1. What is the major problem confronting the ­patient?
with dental situations and personnel (typically dur- 2. Identify the most effective methods for treating the
ing childhood); two greatest dental fears are nee- patient’s condition.
dles and dental drill. 3. What is the difference between fear and a phobia?
4. Feeling a loss of control can increase dental anxi-
ety and fear. 1. Anxiety related to past dental experiences is the prob-
B. Treatment of dental anxiety, fear, phobias: lem the patient is most likely confronting.
1. Using systemic desensitization (small doses of 2. First, the patient must visit the dental office to be o0710
positive experiences) for highly anxious patients fully informed about the necessary dental proce-
increases tolerance for dental encounters. dures. Next, for site-specific soft tissue anesthesia
2. Explaining procedures thoroughly decreases fear the hygienist should help patient identify problems
of unknown. that ­occurred before and explain choices that will de-
3. Increasing patient control during each treatment crease likelihood of similar occurrences during dental
session decreases fear of helplessness and in- hygiene treatment. Patient needs to feel in control of
creases sense of trust (allow patient to help with situation.
suction by holding patient saliva ejector or pick 3. Phobia is excessive fear that leads to inaction, such as
out favorite flavor of topical). failure to seek necessary dental treatment, which can
4. Using relaxation techniques in dental environment, be detrimental to a person’s health and well-being.
includes headphones or other distracters, calming
voice, biofeedback, adhering to time schedule. Sensory Innervation
5. Using pharmacological control of anxiety: Peripheral nervous system (PNS) comprises sensory
a. Antianxiety premedication, either orally or IV, (afferent) nerves that carry sensations of pain to central
with benzodiazepines (BZDs), diazepam (Va- nervous system (CNS), and motor (efferent) nerves that
lium), or alprazolam (Xanax); patient will have transmit messages from CNS to muscles and glands. Un-
LESS memory of stressful dental procedures derstanding of sensory nerve anatomy and physiology
(see Chapter 9, Pharmacology). and action of local anesthetics is essential to pain man-
b. Topical and/or injected local anesthesia agents agement. Sensory nerves are afferent nerves that carry
can remove pain during dental procedures (dis- sensations of pain to the CNS.
cussed later). • See Chapters 3, Anatomy, Biochemistry, and Physiol-
c. Sedation with nitrous oxide can give MORE ogy: nervous system anatomy and physiology; 4, Head,
relaxation, increase pain threshold, decrease Neck, and Dental Anatomy: trigeminal nerve.
awareness of time and procedures, increase A. Anatomy of a nerve:
sense of well-being (discussed later). 1. Myelinated nerves (comprise MOST nerves in
d. Posttreatment with antiinflammatories (such the body), divided into three zones:
as ibuprofen) can reduce inflammation and a. Dendrite (free nerve endings): reacts to stimuli
thus pain, helping with healing (see Chapter 9, in the surrounding tissues.
­Pharmacology). b. Axon: pipeline that delivers impulses to CNS.
c. Terminal nerve endings (arborization): syn-
s0030 Clinical study   apse with CNS nerves.
Scenario: A 35-year-old woman has not been to the 2. Structure of single nerve fiber: myelin sheath cov-
dentist’s office for 10 years. During her last dental ex- ers axon, composed MAINLY of lipid layers (75%)
perience, she had two third molars extracted and the and protein (20%) (Figure 14-1):
local anesthetic initially used did not provide pulpal an- a. Lipid layers: act as barriers to some molecules
esthesia. Repeated injections did not improve the situa- and as binding sites for lipophilic components
tion but did make her more anxious and uncomfortable, of local anesthetics.
472   Saunders Review of Dental Hygiene

11

4
Direc
tion of impulse
10

5 9

7
8

Myelin sheath 6
Synapse with another neuron 11 Node of Ranvier 5
Collateral branch 10 Axon 4
Synapse with myofibers 9 Nucleus 3
Nucleus of Schwann cell 8 Cell body 2

Axon 7 Dendrites 1

Figure 14-1  Structure of a neuron and relationship with other neuron (and muscle tissue). (From
Bath-Balogh M, Fehrenbach MJ: Illustrated dental embryology and anatomy, ed 2, Philadelphia, 2006, Saunders/
Elsevier.)

b. Proteins: act as channels to allow some ions 2. Core bundles: inside of a nerve bundle; receive
(Na+, K+) to pass through nerve membrane. anesthetic last and in lower concentration because
3. Layers of a nerve: of distance from anesthetic source and presence of
a. Epineural sheath: outermost layer; NOT a bar- more blood vessels; innervate distal areas (ante-
rier to anesthetics. rior) and lose anesthetic properties last.
b. Epineurium: connective tissue layer; carries C. Physiology of a nerve:
fasciculi, blood vessels, and lymphatic ves- 1. Nonstimulated nerve has Na+ ions outside the
sels; as anesthetic diffuses through epineural membrane, K+ ions and negative ions inside, and
sheath and blood vessels, begins to eliminate resting potential of −70 mV.
anesthetic. 2. Stimulation of nerve is caused by mechanical (in-
c. Perineurium: surrounds the fasciculi; greatest strument in the soft tissue), chemical, thermal, or
barrier to local anesthetic. electrical means; starts depolarization process;
4. Nodes of Ranvier: constrictions 0.5 to 3 mm permits movement of ions across nerve membrane;
apart; nerve impulses travel from node to node. electrical potential changes from 50 to 60 mV to
B. Structure of a nerve bundle: MANY peripheral nerves +40 mV; as electrical potentials change, impulse
have hundreds to thousands of tightly packed axons in moves along nerve from node to node.
bundles called fasciculi (Figure 14-1). 3. Repolarization occurs when Na+ ions begin to
1. Mantle bundles: outside of nerve bundle, receive move back across nerve membrane to increase
anesthetic FIRST; innervate proximal areas (pos- negative potential inside nerve; from stimulation
teriors) and lose anesthetic properties FIRST. to repolarization, process takes ∼1 msec.
Pain Management   473

LOCAL ANESTHESIA   C. Discussion of pH: mathematical measure of acidity


Dental patients can benefit from removal of pain during and alkalinity, expressed as negative logarithm:
dental procedure with local anesthesia, as well as hemor- 1. Acidic substances have higher concentrations of
rhage control from use of a ­vasoconstrictor, along with hydrogen (H+) ions and therefore can give up more
use of topical anesthesia. Can be used alone or with a H+ ions; alkaline (base) substances have lower
combination of nitrous oxide sedation, which alone does concentrations of H+ ions and can accept more
NOT replace local anesthesia for pain ­ control, since it H+ ions.
is an analgesic and not an anesthetic. Local anesthesia 2. The pH of normal tissue is 7.4; pH of inflamed tis-
administration by dental hygienists is allowed in only sue is lower, between 5 and 6 (more acidic).
some states and usually under the ­ supervision of a 3. Decreased extracellular pH does NOT decrease
dentist. nerve action; internal pH of a nerve is constant;
The CORRECT administration of local anesthesia in- decreased extracellular pH does decrease action of
volves understanding nerve anatomy and physiology, an anesthetic.
pharmacology, armamentarium, technique, and possible 4. The pH of anesthetic without epinephrine (vaso-
complications. Action of local anesthetic agent depends constrictor) is 5.5; anesthetic with epinephrine is
on chemical structure and pH of the solution and body ∼3.3 pH.
tissues. a. Manufacturers acidify anesthetic to inhibit oxi-
A. Mode of action: a local anesthetic prevents or blocks dation (breakdown) of epinephrine.
Na+ channel function in neurons. b. Anesthetic may burn slightly on deposition be-
1. Stabilizes the nerve membrane so that the mem- cause of difference between pH of anesthetic
brane threshold is elevated to point where depolar- and pH of tissues.
ization does NOT occur. c. MORE acidic the anesthetic, slower its onset;
2. Thus, Na+ channels do NOT open and Na+ ion will using cartridge warmers, can break down pH of
NOT enter the axon. local anesthetic, increasing the burning during
3. Depresses all unmyelinated fiber FIRST and larger deposition (also destroys the vasoconstrictor).
myelinated fibers last. D. Dissociation of local anesthetics: ability of a local an-
4. General order of loss of function (first to last): esthetic to dissociate, indicated by pKa number.
pain, temperature, touch, proprioception, motor 1. The pKa number is a constant that characterizes
nerve function. equilibrium of a particular compound; also mea-
B. Chemical formula of agent (Figure 14-2): sures molecule’s affinity for H+ ions:
1. Aromatic group (lipophilic [hydrophobic] compo- a. Equilibrium equation is the pKa equation:
nent): affinity for the lipid portion of the myelin RNH+ ⇔ RN + H+
sheath, which helps local anesthetic agent attach to b. Cation: RNH+ is a positively charged molecule
the nerve membrane and block the nerve impulse. that is responsible for binding at the recep-
2. Intermediate chain: either ester (−COO−R) or tor site and decreasing the Na+ that enters the
amide (NHCO−R) group; determines mode of nerve.
biotransformation or metabolism (see later discus- c. Free base: RN is an uncharged molecule that
sion). is responsible for the diffusion of anesthetic
3. Amino end (hydrophilic component): makes anes- through surrounding tissues and the nerve
thetic agent injectable; amides dissolve poorly in sheath.
water and are unstable on exposure to air; there- 2. Clinical implications of the pKa number, since
fore hydrochloride (HCl) is added to produce a salt each anesthetic has pKa number:
that is MORE soluble and stable. a. Lower the pKa number (below 7.5), the MORE
lipophilic free base molecules (creates greater
diffusion, quicker onset, longer duration),
Lipophilic/ Intermediate Hydrophilic/
aromatic part part amino part but the LESS cations available to bind the
­anesthetic.
b. When the pH of anesthetic is same as the pKa
CH3 O
number, equal amounts of BOTH base and cat-
C2H5 ion exist.
NHCCH2 N c. Equilibrium shifts:
C2H5
(1) Shifts left (cation state) when there are
CH3
MORE hydrogen ions (low pH).
(2) Shifts right (free base state) when there are
Figure 14-2  Chemical formula for a local anesthetic. FEWER hydrogen ions (high pH).
474   Saunders Review of Dental Hygiene

d. The MORE free base molecules available, the c. Have greater potential for OD (alone, without
greater the diffusion of anesthetic through tis- vasoconstrictor) because higher blood levels
sues and membrane, and therefore the faster the occur until agent reaches liver for biotransfor-
onset of action. mation.
3. The pH of anesthetic is lowered with addition of d. Half-life is 50 to 120 minutes.
epinephrine (pH 3.3): (1) EXCEPTION: Articaine’s half-life is dif-
a. When MORE cations exist, more Na+ is bound ferent from other amides.
at the nerve receptor, which creates greater (a) Falls under amide class, but its asso-
binding power. ciated ester group also allows plasma
b. LESS free base leads to less diffusion and metabolism via pseudocholinesterase,
slower onset. purportedly increasing rate of break-
c. Surrounding tissues buffer MORE acidic down and reducing its toxicity.
­solutions. (b) Difference in metabolism gives advan-
4. The pH of inflamed tissues is low (5 to 6), so tage of having a 30-minute half-life,
MORE cations exist; therefore MORE Na+ is in contrast to lidocaine, which has a
bound at nerve receptor site: 90-minute half-life.
a. LESS diffusion of anesthetic into surrounding B. Distribution and elimination of agents:
tissues causes slower onset and leads to ineffec- 1. After entering blood, agents permeate ALL body
tive anesthesia. tissues.
b. Surrounding tissues are NOT able to buffer 2. Are vasodilators; thus vasoconstrictors are added
more acidic solutions because of lower pH. in MOST cases to decrease vasodilation.
c. Patient may need MORE local anesthetic agent 3. Esters and amide agents are BOTH eliminated
because of acidic conditions; block may be mostly through the kidneys.
MORE effective than infiltration, since agent is C. Phentolamine mesylate (Oraverse): injection via
NOT near inflamed tissues. ­syringe, which accelerates return of normal soft tis-
sue sensation (i.e., sensation of lip and tongue), as
Pharmacology of Local Anesthetics well as function following restorative and periodon-
and ­Vasoconstrictors tal ­ procedures, and helps prevent any self-inflicted
Understanding the metabolism, action, dosage calcula- oral trauma that may occur with a long-lasting injec-
tions, and specific functions of topical and local anes- tion of local anesthetic containing a vasoconstric-
thetic agents and vasoconstrictors helps the clinician to tor; contains an alpha-­adrenergic antagonist that
request and use these agents more safely and efficiently. acts as a vasodilator, resulting in faster diffusion of
Clinician MUST prevent an overdose (OD) situation, anesthetic into the ­cardiovascular system and away
which is an accidental or intentional use of a drug in an from site.
amount higher than is normally used.
• See Chapter 9, Pharmacology: local anesthetic and Local Anesthetic Agent Action
­vasoconstrictor pharmacology. Local anesthetic is an anesthetic drug that induces local
A. Types of agents: anesthesia by inhibiting nerve excitation or conduction.
1. Ester agents: NO longer available as injectables, These agents also have effects on central nervous sys-
ONLY topicals (usually 20% benzocaine): tem (CNS), cardiovascular system (CVS), respiratory
a. Biotransformed to paraaminobenzoic acid system.
(PABA) in blood plasma by enzyme pseudo- • See Chapter 10, Medical and Dental Emergencies: local
cholinesterase. anesthesia emergency protocol in a dental setting.
b. Half-life (rate at which 50% of the anesthetic is A. CNS effects:
eliminated from the blood) is 2 to 8 minutes. 1. Low levels have NO effect but can provide anti-
c. Have lower potential for OD (toxicity). convulsive properties by raising seizure threshold;
d. MORE likely to cause allergic reaction (to used to treat epileptic seizures.
PABA). 2. Preconvulsive levels can cause slurred speech,
2. Amide agents: BOTH injectables and topicals shivering, twitching, flushed feeling, dream state,
(NOT as useful at 5%) available: lightheadedness, blurred vision, tinnitus.
a. LESS likely to cause allergic reaction because a. Lidocaine may cause mild sedation or drowsi-
methylparaben, which can cause allergic reac- ness, indication of possible toxic reaction.
tions, has been eliminated as preservative. b. Therefore anesthetized patient should NEVER
b. Biotransformed MAINLY in liver (MUST have be left alone, since onset occurs in 5 to 10
healthy liver tissue). ­minutes.
Pain Management   475

3. Convulsive levels cause convulsions, CNS depres- (2) β2 activation in bronchi, vascular beds, uterus
sion, respiratory depression and arrest. produces bronchodilation and vasodilation.
B. CVS effects: 2. MOST vasoconstrictors used in dentistry exert ac-
1. Low levels produce NO effects. tion on adrenergic receptors:
2. High (non-OD) levels cause mild hypoten- a. Epinephrine: acts on both α and β receptors, but
sion (low blood pressure) by relaxing smooth MAINLY on β receptors.
muscles. b. Norepinephrine: acts on both α and β receptors,
3. At OD levels, can produce profound hypotension, but MAINLY on α receptors.
which causes decreased myocardial contractions, c. Levonordefrin (synthetic): acts on both α and
decreased cardiac output, decreased peripheral β receptors, but MAINLY on α receptors.
resistance. C. Types of vasoconstrictors:
4. Lethal levels lead to CVS collapse caused by mas- 1. Epinephrine (adrenalin): one MOST commonly
sive peripheral vasodilation, decreased heart con- used in United States:
tractions, and decreased heart rate (bradycardia). a. MORE potent; found in BOTH natural and syn-
C. Respiratory system effects: thetic forms.
1. Non-OD levels relax action of bronchial smooth b. In skeletal muscle and blood vessels, produces
muscles. both vasodilation (small amounts act on β2
2. OD levels can lead to respiratory arrest as result of sites) and vasoconstriction (large amounts act
CNS depression. on α receptors).
c. MAINLY used at 1:100,000 for nerve blocks and
Vasoconstrictor Action not 1:50,000 concentrations (usually only infil-
Vasoconstrictors if used in local areas cause constriction trations and intraseptal injections); use of addi-
of the arterioles and capillaries. Vasoconstrictors act on tional dosage does NOT increase duration and/or
alpha (α) and/or beta (β) receptors, depending on the effectiveness, ONLY hemorrhage control.
body tissues. 2. Norepinephrine (Levarterenol): NOT used in
A. Chemical structure of a vasoconstrictor (Figure 14-3): United States, but used in other countries:
1. Benzene ring with two OH groups in the third and a. LESS potent than epinephrine (one fourth
fourth positions is a catechol. as potent) and demonstrates LESS systemic
2. Benzene ring with an amine group in another posi- action.
tion is a catecholamine. b. Activation of α receptors in the smooth muscles
3. MOST vasoconstrictors are catecholamines: of palatal blood vessels can cause ischemia and
a. Epinephrine, norepinephrine, dopamine: ALL then tissue necrosis.
occur in nature. 3. Levonordefrin (Neo-Cobefrin): NOT as commonly
b. Levonordefrin and isoproterenol: BOTH used in United States:
­synthetics. a. LESS potent than epinephrine (one fifth as ­potent);
4. Do NOT use cartridge warmers, since inactivate demonstrates LESS systemic action so used at
vasoconstrictors. higher concentration (1:20,000) when used with
B. Adrenergic receptors: present naturally in MOST agents such as 2% mepivacaine; has same ­onset,
body tissues. depth, duration of anesthesia in both pulpal and
1. Includes two types of receptors: soft tissue as lidocaine with epine­phrine.
a. Alpha (α) activation causes contraction of b. NOT as strong in hemorrhage control as epi-
smooth muscles in blood vessels. nephrine (IMPORTANT in dental procedures
b. Beta (β) activation: involving bleeding and need for hemorrhage
(1) β1 activation in heart and small intestine, control, such as dental hygiene procedures).
responsible for cardiac stimulation and D. Inclusion of vasoconstrictors:
­lipolysis. 1. Medical considerations for use of vasoconstrictors:
a. Absolute contraindications:
(1) Cardiovascular disease (CVD): acute in- o1930
OH
cident (myocardial infarction [MI, heart
attack] or cerebrovascular accident [CVA,
HO C CH2NHCH3 stroke]) within last 4 to 6 weeks or unable to
meet 4 METs (metabolic equivalents); high
H blood pressure (HBP) ≥140/90 mm Hg;
HO unstable or severe angina pectoris that is
Figure 14-3  Chemical structure of a vasoconstrictor. relieved by rest.
476   Saunders Review of Dental Hygiene

(2) With uncontrolled or undiagnosed hyper- Box 14-1  Local Anesthetic Agent Dose
thyroidism, may cause thyroid storm. ­Calculations
(3) Recreational cocaine (crack) user (within
24 hours); could be fatal. Questions/Answers
b. Relative contraindications: A. How many milligrams/milliliter of agent are there
(1) Reduced levels needed with MOST other in solution per percentage concentration of
chronic CVD histories; however, still may anesthetic agent?
need to use a limited amount to ensure pain B. How many milligrams of agent are there per
cartridge?
control (reduces endogenous [own] epineph- C. What is the MRD in number of cartridges of
rine of patient with CVD); see next section. agent allowed per individual patient?
(2) Patients taking tricyclic antidepressants 1. To change % concentration of agent to mg/mL,
(TCAs): NEITHER norepinephrine NOR first divide by 100, then multiply by 1000.
­levonordefrin (Neo-Cobefrin) should be 2. To find mg of agent per cartridge, multiply
mg/mL by 1.8 mL/cartridge.
used; substitute epinephrine if needed. 3. To find MRD of agent for patient, multiply
2. Consideration for duration of appointment: ­patient’s weight by established MRD in mg/
a. Addition increases duration of anesthetic effects. pound.
b. Concentration and type affect duration of a 4. To find mL of agent, divide MRD in mg of agent
­local anesthetic. by mg/mL of agent.
5. To find MRD in number of cartridges for patient,
3. Consideration for hemostasis during appointment: divide mL of agent by 1.8 mL/cartridge.
o2030 a. Epinephrine in large quantities acts as a vaso-
constrictor; in smaller quantities becomes va- Example: Local Anesthetic Agent Dose
sodilator that has potential to increase bleeding Calculation Using 2% Lidocaine for 150 lb
postoperatively. Patient
1. Percentage grams to milligrams/milliliter:
b. Injection SHOULD be close to the area of 2% ÷ 100 = .02 g/mL x 1000 mg/g = 20 mg/mL.
bleeding to be effective; may want to add epi- 2. Milligrams per cartridge: 20 mg/mL x 1.8 mL/
nephrine 1:50,000 levels (available with lido- cartridge = 36 mg/cartridge.
caine agent) as interseptal injection after other 3. Patient MRD: 2 mg/lb (MRD) x patient’s weight
injections. (e.g., 150 lb) = 300 mg.
4. Milliliters of solution: 300 mg ÷ 20 mg/mL
o2050 c. Epinephrine has BETTER hemostatic con- = 15 mL.
trol levels than levonordefrin (Neo-Cobefrin), 5. Number of cartridges per patient: 15 mL ÷
which is important with bleeding that may oc- 1.8 mL/cartridge = 8.3 cartridges.
cur with nonsurgical periodontal maintenance. Fast Tip: Eliminate steps 4 and 5 by dividing mg
E. Vasoconstrictor use causes slight burning upon injec- MRD/patient by 36 mg/cartridge: 300 mg ÷
36 mg/cartridge = 8.3 cartridges.
tion (MORE than with plain) because of presence of
preservative (sodium metabisulfate), since it is acidic
to increase agent’s shelf-life (discussed later) but this MRD, Maximum recommended dosage by ­manufacturer.

side effect does not preclude use or add need for plain
preinjection.
Local Anesthetic Agents and Vasoconstrictors
Dosage Calculations Vasoconstrictors and local anesthetic agents must be cho-
Dosage calculations for local anesthetics are based on the sen carefully, based on the medical concerns and type of
size and general health of patient and on type and concen- dental procedure to be performed. Anesthetic is selected
tration of the anesthetic agent and vasoconstrictor. The based on whether its duration is appropriate to procedure
maximum recommended dose (MRD) is the dose estab- being performed.
lished by manufacturer (in milligrams per pound). A. Selection considerations:
o2060 A. MRD for each anesthetic agent (Box 14-1). 1. Medical concerns (if vasoconstrictor used, see ear-
B. Concentration of vasoconstrictors: ratio of vasocon- lier discussion):
strictor to milliliters of solution (Box 14-2). a. True allergy to local anesthetic agent is rare
1. MRD of epinephrine: (past history may be to esters, usually NOT to
a. MRD for healthy patient is 0.2 mg per appoint- amides); may be allergic to preservative for va-
ment. soconstrictor (sodium metabisulfite); try plain
b. MRD for patient with CVD is HALF as much: anesthetic agent.
0.04 mg per appointment. b. CVD concerns include incident (MI, CVA) o2150
2. MRD of levonordefrin (Neo-Cobefrin) for all within last 4 to 6 weeks or unable to meet
­patients is 1.0 mg per appointment. 4 METs, HBP ≥140/90 mm Hg, unstable or
Pain Management   477

Box 14-2  Calculation of Vasoconstrictor


2. Bupivacaine, articaine, etidocaine are available
­Concentration with vasoconstrictor.

Topical Anesthetics
1. To change grams to milligrams, multiply by 1000,
since ratio equals 1 gram of drug per milliliters of Topical anesthetics are useful for providing light, local- p0160
solution. ized anesthesia to the first 2 to 3 mm of the oral mucosa
2. To find mg of vasoconstrictor drug per mL of as a preinjection agent. May also be used alone before
solution (mg/mL), divide mg of drug by mL of procedures involving soft tissues (nonsurgical periodon-
solution.
3. To find mL of solution, divide MRD* of
tal therapy). Must be placed on oral mucosa for 2 to
­vasoconstrictor in mg by mg/mL. 3 minutes per Materials Safety Data Sheet (MSDS).
4. To find number of cartridges, divide MRD mL of • See Chapter 9, Pharmacology: discussion of topical
solution by 1.8 mL/cartridges. ­anesthesia using gel or patch.
A. Action:
EXAMPLE: CALCULATION OF CONCENTRATION OF
VASOCONSTRICTORS USING 1:100,000 EPINEPHRINE/ML
1. MOST are higher concentrations than injectable
OF SOLUTION anesthetics (20% benzocaine) and thereby increase
1. Ratio: 1:100,000 = 1 g of drug per 100,000 mL diffusion of the active ingredients through oral
of solution. mucosa and open wounds.
2. 1 g = 1000 mg of drug per 100,000 mL of a. Efficient diffusion leads to faster onset.
­solution.
3. 1000 mg ÷ 100,000 mL = 0.01 mg/mL.
b. Increased water solubility increases diffusion
and onset; water insolubility decreases diffu-
EXAMPLE: VASOCONSTRICTOR DOSE CALCULATION sion and onset but increases duration by retain-
FOR HEALTHY ADULTS AND CVD PATIENTS USING ing topical anesthetic at the site.
EPINEPHRINE 2. Do NOT contain vasoconstrictors.
MRD for healthy adult patient: 0.2 mg per
1:100,000 epinephrine.
a. Without vasoconstrictors, duration is decreased
MRD for patient with CVD: 0.04 mg per but potential for OD is increased because of the
1:100,000 epinephrine. faster uptake of agent into blood.
How many cartridges of anesthetic with epinephrine b. Higher concentrations increase potential for
can a healthy adult have? OD and toxicity because uptake of agent into
0.2 mg of drug ÷ 0.01 mg/mL = 20 mL; then 20 mL ÷ 1.8
mL/cartridge = 11.11 cartridges.
vascular system is greater.
How many cartridges of anesthetic with epinephrine c. Controlled use is optimum for safety; NOT to
can a patient with CVD have? be used for more than one or two quadrants per
0.04 mg of drug ÷ 0.01 mg/mL = 4 mL; then 4 mL ÷ 1.8 appointment.
mL/cartridge = 2.2 cartridges. B. Concern for allergenic potential (see earlier discussion).
Note: Levonordefrin [Neo-Cobefrin] allows 1 mg
­vasoconstrictor/any patient/visit.
Local Anesthesia Armamentarium
Preparing the armamentarium for delivery of local an-
MRD, Maximum recommended dosage by manufacturer; clinicians may be
more conservative in use. esthetic involves knowledge of the syringe, needle, car-
tridge, proper setup procedures, care of equipment, safe
handling, and prevention and management of associated
s­ evere angina pectoris, uncontrolled or undiag- problems. Aspiration is the process of removing fluids (or
nosed hyperthyroidism. gases) from the body with a suction device (syringe with
c. Pregnant patients (use category B: lidocaine piston). It allows the clinician to know if the needle tip is
and prilocaine, NOT category C: mepivacaine, in a blood vessel to prevent an intravascular injection.
articaine, bupivacaine). A. Syringe:
2. Onset and duration: 1. Syringe types:
a. Onset is determined by properties of local anes- a. Harpoon syringe: MOST commonly used one
thetic agent, including its dissociation. (Figure 14-4):
b. Duration (short, medium, or long acting) is de- b. Self-aspirating syringe (Figure 14-5):
termined by site of injection, type of anesthetic (1) Metal projection presses on rubber dia-
agent, addition of vasoconstrictor, patient’s phragm of the cartridge; pushing on the
­idiosyncrasies (Tables 14-1, 14-2, and 14-3). thumb ring increases the pressure of pro-
B. Local anesthetic agents with and without vasocon- jection on the diaphragm.
strictors (plain): (2) Thereby increases pressure inside the car-
1. Lidocaine, mepivacaine, prilocaine are available tridge, aspiration occurs, then pressure is
BOTH with vasoconstrictor and without (plain). released.
478   Saunders Review of Dental Hygiene

Table 14-1  Short-acting local anesthetics

Maximum
Duration: Dose/ MRD/Body MRD for cartridges
Onset Duration: soft tissue cartridge weight healthy adult for 150 lb
Local anesthetic (min) pulp (min) (hr) (mg) (mg/lb) (mg) patient

2% lidocaine 2-3 5-10 1-2 36 2 300 8.3


HCl (Xylocaine)
3% mepivacaine HCl 1.5-2 20-40 2-3 54 3* 400* 7.4*
(Carbocaine)
4% prilocaine HCl 2-4 10-20 1.5-2 72 2.7 400 5.5
(Citanest Plain)

Data from Malamed SF: Handbook of local anesthesia, ed 5, St. Louis, 2004, Mosby/Elsevier.
MRD, Maximum recommended dose.
*Manufacturer’s recommendation.

Table 14-2  Medium-acting local anesthetics

Maximum
Duration: Dose/ MRD/Body MRD for cartridges
Onset Duration: soft tissue cartridge weight healthy adult for 150 lb
Local anesthetic (min) pulp (hr) (hr) (mg) (mg/lb) (mg) patient

2% lidocaine HCl, 2-3 1 3-5 36 3* 500* 13.8*


1:50,00/100,000 epi-
nephrine (Xylocaine)
2% mepivacaine HCl, 1.5-2 1-1.5 3-5 36 3* 400* 11*
1:20,000 levonorde-
frin (Carbocaine with
Neo-Cobefrin)
4% prilocaine HCl, 2 1-1.5 3-8 72 2.7 400   5.5
1:200,000 epineph-
rine (Citanest Forte)
4% articaine HCl, 1-9 1 3-5 72 3.2 476  7
1:100,000/200,000
epinephrine
­(Septocaine)

Data from Malamed SF: Handbook of local anesthesia, ed 5, St. Louis, 2004, Mosby/Elsevier.
MRD, Maximum recommended dose.
*Manufacturer’s recommendation.

c. High-pressure syringe (Ligmaject): 2. Handling of syringes: should be checked for rust,


o2390 (1) Used MAINLY for periodontal ligament harpoon sharpness, working piston.
(PDL) (intraligamentous) injection (type a. Leakage during injection: caused by offset
of intraosseous injection), which permits needle placement into rubber diaphragm or by
measured dose of solution and overcomes loose needle.
tissue resistance; for other PDL injection b. Broken cartridge: caused by too much pressure
with traditional syringe, see Table 14-4. when engaging harpoon or by bent ­harpoon.
o2400 (2) May cause trauma to surrounding tissues c. Disengagement of harpoon during aspiration:
if increased solution (under pressure) is caused by dirty, dull, or broken harpoon or
forced into PDL space; if this situation NOT securely engaging rubber stopper in
occurs with primary teeth, can also cause cartridge.
trauma to developing permanent teeth. B. Needles: stainless steel, disposable, presterilized,
d. Safety syringes (UltraSafe, Safety Plus): LESS sharp to decrease potential for cross-contamination
risk of needlestick injury (see later discussion). (Figure 14-6):
Pain Management   479

Table 14-3  Long-acting local anesthetics

Maximum
Duration: Dose/ MRD/Body MRD for cartridges
Onset Duration: soft tissue cartridge weight healthy adult for 150 lb
Local anesthetic (min) pulp (hr) (hr) (mg) (mg/lb) (mg) patient
0.5% ­bupivacaine   3-10 1-2 5-9 9 0.6 90 10
HCL, 1:200,000 epi­
nephrine (Marcaine)
1.5% etidocaine HCL, 1.5-3 1.5-2 4-9 27 3.6 400 14.8
1:100,000 epi-
nephrine (­Duranest)

Data from Malamed SF: Handbook of local anesthesia, ed 5, St. Louis, 2004, Mosby/Elsevier.
MRD, Maximum recommended dose.

Thumb Table 14-4  Periodontal ligament injection technique*


Finger grip ring
Harpoon Piston
Pulpal and soft tissue
Anatomy and nerve endings
anesthetized in area of injection

Needle Window Needle gauge and 27-gauge; short, extra short,


guide
length or ultra short or CLAD
Figure 14-4  Harpoon aspirating syringe.
Depth of Base of pocket until resis-
penetration tance is met
Landmarks Pocket area, mesial of distal
root
Site of penetration Long axis of root on mesial
Thumb
Finger grip ring
or distal with bevel toward
root
Metal projection Piston
Deposition site Base of pocket
Cartridge amount 0.2 mL
(1.8 mL/cartridge)
Needle Window Thumb disk Complications Pain
guide
Figure 14-5  Self-aspirating syringe. Advantages Minimal dose required; no
unnecessary structures
anesthetized; works better
1. Bevel: oblique surface to penetrate soft tissue with CLAD
without resistance; placed toward bone to prevent Disadvantages Leakage of anesthetic; dif-
trauma to overlying periosteum (bone’s surface). ficult to deposit with non-
2. Shank: length is measured from bevel tip to the pressure syringe; not with
inflammation or primary
hub: teeth present
a. Short shank: ∼1 inch/∼25 mm.
b. Long shank: ∼1.58 inches/∼40 mm. *See manufacturer’s directions for use of pressure syringe (Ligmaject); dental
3. Gauge: measure of inside diameter of lumen; hygienists may not be allowed to perform intraosseous injections in some
states.
larger the number, smaller the needle; thus smaller
to larger: 30, 27, 25-gauge.
a. Smaller gauge needles (30-gauge) with smaller
lumen are NOT recommended for less pain: b. Larger gauge needles (27- to 25-gauge)
(1) Deflect MORE when penetrating tissues, with larger lumen are BEST, with long
reduces accuracy because solution is de- shank with 25 gauge and short shank with
posited away from intended site. 27 gauge.
(2) Smaller lumen increases possibility of clog- (1) Deflect LESS when penetrating tissues,
ging needle with blood cells, MORE likely increasing accuracy because solution is de-
to show false-negative aspiration. posited close to intended site.
480   Saunders Review of Dental Hygiene

4. Contents besides agent and vasoconstrictor


(additives):
a. Hydrochloride (HCl): added to agent to cre-
ate acidic salt for BETTER water solubility;
increases diffusion of agent to nerve; makes
­anesthetic injectable.
Hub Shank Bevel b. Sodium chloride: added to make solution
MORE isotonic with soft tissues.
Cartridge Syringe c. Sodium metabisulfite: antioxidant that is added to
penetrating end adapter
preserve vasoconstrictor (if present) by reacting
Figure 14-6  Parts of a needle.
with oxygen to produce sodium bisulfate (more
acidic, slight increase in burning during injection
Rubber stopper [MORE than plain]); can act as allergen.
d. Distilled water: provides remaining volume.
5. Handling of cartridges:
a. SHOULD be checked before use for clear so-
lution without large bubbles, normal stopper,
Rubber noncorroded/rusty cap.
diaphragm (1) Bubbles that are small (1 to 2 mm): accept-
able; however, larger bubbles mean that so-
Glass tube lution was frozen, which causes chemical
containing solution
changes.
(2) Extruded stopper: caused by freezing car-
tridge or by storing in disinfectant.
Metal hub (3) Corroded/rusty cap: caused by cartridge
Figure 14-7  Components of an anesthetic cartridge. breakage in round tin container (10-cartridge
blister packs decrease breakage) or by im-
mersion in disinfectant (metal cap).
(2) Easier to aspirate, since increases true- b. With vasoconstrictors, shelf-life is shorter at
­negative aspiration, LESS likely to show 18 months; without vasoconstrictors, shelf-life
false-negative aspiration. is longer at 24 months.
(3) Short shank needles with 27-gauge are c. Burning during injection: normal response to
used on MOST commonly administered pH of agent; however, those containing disin-
injections, EXCEPT inferior alveolar nerve fectant or vasoconstrictors and overheating (by
block (and thus buccal nerve block done cartridge warmer) can cause increased burning
immediately after) uses long shank with during injection.
25-gauge. d. To maintain viable anesthetic: do NOT auto-
4. Hub or needle adapter: some have bevel indicator clave, soak in disinfectant, or keep in direct
dots. sunlight or in warmers; wipe ends with 70%
5. Cartridge-penetrating end: sharp to penetrate rub- alcohol solution if NOT used.
ber diaphragm of cartridge. D. Computer-controlled local anesthesia delivery (CLAD)
6. Needle cap/sheath. system (CompuDent/Wand, Comfort Control):
C. Cartridges: cylindrical tube of agent, color-coded by 1. BOTH use traditional cartridges and either tradi-
manufacturer to distinguish types, holds 1.8 mL of tional needles or safety (Luer-Lok) needles; provides
anesthetic (Figure 14-7): MORE controlled amounts of anesthetic solution
1. Rubber stopper: coated with silicone to ease delivered for ALL types of injection techniques.
­movement, engaged by harpoon, then pushed by 2. Handpiece is held with a pen grasp and is lighter than
piston to dispel solution; may/may not have rubber traditional syringes; as needle penetrates, clinician
latex. steps on a pedal/uses finger-touch control to activate
2. Aluminum cap: holds rubber diaphragm in flow of anesthetic solution ahead of ­needle path.
place. 3. Has auto-aspiration feature that can be used for
3. Rubber diaphragm: acts as seal to prevent anes- any type of injection; amount deposited for MOST
thetic from leaking around needle, as long as ­needle injections is 0.6 to 0.9 mL.
penetrates the diaphragm squarely and NOT on an 4. MUST be used to perform AMSA block injection
angle, does NOT have rubber latex. (discussed later).
Pain Management   481

Clinical study   2. Have patient rinse with preprocedural antimicro-


Scenario: During an initial dental hygiene appointment bial mouthrinse.
with new patient, the dental hygienist observes blood in B. Armamentarium setup:
a cartridge (positive aspiration) during administration of 1. Select appropriate syringe (harpoon, self-aspirating,
an inferior alveolar nerve (IA) nerve block (2% lidocaine or pressure).
with 1:100,000 epinephrine). A 25-gauge long needle is 2. Select appropriate cartridge:
being used. A large amount of blood fills the cartridge a. According to patient treatment and medical
quickly. The patient with generalized moderate chronic needs.
periodontitis reports no medical conditions and exhibits b. According to date and physical appearance;
no significant need for dental restoration. should have no large bubbles, clear color, rust/
corrosion-free cap, rubber stopper without
1. What is the most likely cause of the positive aspiration? ­needle punctures.
2. How should the clinician handle this situation? 3. Select appropriate needle according to scheduled
3. Are positive aspirations preventable? Was the needle injections.
used appropriate one for this injection? 4. Place hemostat/locking cotton pliers on tray for
potential retrieval of a broken needle (NOT recom-
1. If tip of the needle is located in large artery during aspira- mended as much).
tion, blood will quickly flow into cartridge. Artery that is 5. Select appropriate topical anesthetic and amount
most likely to be penetrated is inferior alveolar (IA) ar- (larger doses can produce toxicity).
tery. Veins are more passive and typically cause blood to 6. Prepare for needle recapping (device/method).
enter the cartridge more slowly and in smaller amounts. C. Syringe setup:
2. Clinician should stop the injection and remove the sy- 1. Place cartridge in syringe; then place needle in
ringe from the patient’s mouth. Cartridge and needle syringe so that cartridge penetrating end passes
should be changed and the injection should be re- through the diaphragm and into anesthetic, squarely
peated until there is negative aspiration. Later, both and NOT at angle.
the ­ cartridge and needle should be placed in the 2. Engage harpoon in rubber stopper with firm, gen-
­biohazardous waste. tle motions so as to not break glass cartridges (self-
3. No, the purpose of choosing a needle with appropri- ­aspirating syringe has NO harpoon).
ate lumen size is for very purpose of getting positive 3. Rotate needle bevel so that on penetration faces
aspiration when needle inadvertently enters a blood the bone (so do NOT scrape periosteum).
vessel. For patient safety, there must not be deposi- 4. Dispel some solution to ensure correct needle
tion of solution in a blood vessel while administering placement in cartridge.
anesthetic. Accomplished by aspirating and checking 5. Make sure that harpoon is engaged (for harpoon
for blood in cartridge before injection, and possibly syringe only) by pulling on the thumb ring with
again in different plane (adjusting bevel) during in- negative pressure.
jection process in highly vascular areas (IA artery D. Site preparation:
with IA nerve block). Needle used (gauge and length) 1. Inspect site for lesions (do NOT inject with abscess
was appropriate for injection, since both lumen size formation).
(gauge) and length (must contact bone) are important 2. Dry tissues to decrease microbial levels with
to achieve safety with injection of IA nerve block. ­patting motion to decrease abrasions.

Local Anesthesia Preparation Local Anesthesia Delivery


It is important for the clinician to properly prepare for the Safe delivery of local anesthetics depends on thorough
local anesthesia procedure. Includes patient preparation, knowledge of proper injection techniques, signs of OD or
proper selection of equipment, anesthetic and vasocon- allergy, proper handling and disposal of equipment.
stricting agents, proper assembly of syringe and needle, • See Chapter 8, Microbiology and Immunology: stan-
preparation of injection site. MUST follow standard pre- dard precautions, sharps/waste handling (needle and
cautions. ­Aspiration MUST be performed to prevent an cartridge); 10, Medical and Dental Emergencies:
intravascular injection. needlestick injury, emergency protocol in a dental
A. Preinjection: review medical history for contraindica- ­setting.
tions, determine appropriate anesthetics, record blood A. During treatment: keep syringe out of view; patient
pressure reading. in supine position to avoid syncope (MOST common
1. Review treatment plan for injections to be per- reaction).
formed to determine appropriate needle lengths/ 1. Larger window should face clinician to ensure
gauges and anesthetic. ability to see positive aspirations.
482   Saunders Review of Dental Hygiene

2. Ensure correct point and depth of penetration; to 8. Observe standard precautions:


prevent loss of broken needle, do NOT bury hub in a. Do NOT inject in presence of abscess or severe
soft tissue. infection (needletrack infection).
3. Aspirate harpoon syringe by retracting piston and b. Avoid touching nonsterile surfaces (patient
harpoon; aspirate self-aspirating syringe by push- napkin) or picking up debris (gauze) on needle.
ing forward and then releasing pressure. c. Use device or method to recap needle to
a. Aspirate in two planes in MORE vascular ­areas ­decrease potential for needlestick injuries.
to ensure that needle has NOT entered a blood B. Monitor patient during and after injection for signs of
­vessel and drawn up the vessel wall during adverse reactions.
­aspiration; may lead to false-negative ­aspiration. 1. Allergic reactions MUST be handled immediately,
(1) Highly vascular areas with higher risk of according to emergency protocol.
positive aspiration and hematoma (in or- 2. OD (toxicity) reactions MUST be handled accord-
der): inferior alveolar, mental and incisive, ing to emergency protocol.
posterior superior alveolar blocks. 3. For treatment or prevention of vasodepressor syn-
b. Blood that enters the cartridge may indicate cope (fainting, MOST common adverse reaction),
that the needle has entered blood vessel or is in patient should be placed in subsupine position
hematoma-induced, blood-filled tissue; change (Trendelenburg position).
needle and cartridge and try again at different C. Dispose of needle and cartridge in an appropriate
site or angle. sharps container (BOTH needles and cartridges are
4. Inject slowly, at rate of ∼1 minute per cartridge, to biohazardous waste because they contain aspirated
decrease potential for OD and for patient comfort. body fluids, including blood).
5. Deposit appropriate amount of anesthetic, accord- D. Posttreatment chart entry:
ing to patient’s treatment plan. 1. Record vital signs, BP, and HR.
6. Change needle when barbed, or after several injec- 2. Record type of injection, including type and amount
tions because the needle becomes dull. of anesthetic (NOT expiration date of cartridge).
7. Evaluate success of the injection; if necessary, re- 3. Record any complications that need immediate or
peat the injection with appropriate adjustments. delayed attention (e.g., hematoma, trismus).

Clinical study  

Age 34 YRS SCENARIO

Sex ☐ Male   ☒ Female The patient is in the dental office for


his first oral prophylaxis appointment
BP 121/84 in 7 years. He appears apprehensive
Chief Complaint “I really hate the drill and needles that at the beginning of the appoint-
you all seem to like using!” ment. After the treatment procedures
for the appointment are explained,
Medical History None the patient asks the dental hygien-
ist to stop because he feels hot and
Current Medications None dizzy. Perspiration is apparent on his
forehead.
Social History Policeman

1. What type of situation is the patient probably in this stage lowered blood pressure (hypotension), in-
­experiencing? creased pulse rate, tachycardia (>100 beats per minute
2. What emergency management or protocol should be for adult) would be present. If this continues, syncope
initiated at this time? may occur, which is characterized by loss of con-
3. If the patient loses consciousness, what steps should sciousness, shallow breathing, ­dilated ­pupils, lowered
be followed? blood pressure (hypotension), thready pulse.
4. What has caused this situation to occur? 2. At this time, he should be positioned in Trendelenburg
position, subsupine with feet elevated higher than head.
1. The patient is most likely is experiencing syncope 3. If becomes unconscious, essential to open airway us-
(fainting), most common emergency in dental ­office. ing head-tilt chin-lift technique. Prevents tongue from
Dizziness indicates that he is still in presyncope stage; obstructing the airway and is indicated for any patient
Pain Management   483

(without a history of cervical spine injury) who is los- very successful. Mandibular cortical plate is thicker than
ing consciousness. Ammonia inhalant can be used to maxillary cortical plate, especially in posterior region;
stimulate breathing, if necessary. Oxygen (5 liters/min block injections are more successful than infiltrations in
with a nasal cannula) may also be administered. Careful this region. Note that intraseptal injection (type of intraos-
monitoring until vital signs return to baseline is essential seous injection) interproximally into interdental bone is
because syncope is likely to recur soon after recovery; not discussed.
vomiting most commonly occurs after unconsciousness, • See CD-ROM for related figure deomonstrating com-
so turning the patient on his left side and having high- mon local anesthesia in the oral cavity.
speed suction (high-volume evacuation) available will • See Chapter 4, Head, Neck, and Dental Anatomy:
minimize aspiration. No further dental treatment should ­anatomy related to each local anesthetic procedure.
be performed at this time (need to wait 24 hours), and
patient should be released. Later in the day, patient Maxillary Injection Procedures
should be called at home to ensure well-being. Maxillary nerve block injections include posterior supe-
4. Stress indicated by apprehension about appointment, rior alveolar (PSA), middle superior alveolar (MSA), an-
causing decreased oxygen flow to the brain, is the likely terior superior alveolar (ASA), infraorbital (IO), greater
psychogenic cause of his syncope. His discussion about palatine (GP), nasopalatine (NP), anterior middle superior
the drill and needle may have brought up ­issues from past alveolar (AMSA). See Table 14-6 for a guide to maxillary
dental experiences. Future ­appointments will have to deal injections. This section discusses specific complications
with his apprehension by education and stress control. for each block.
A. Complications with PSA nerve block:
INJECTION PROCEDURES   1. Deposition site that is too shallow decreases pulpal
p0220 Appropriate injections are based on teeth that need to be anesthetic effect.
anesthetized and anatomical landmarks. Maxillary and 2. Pain occurs when needle contacts bony alveolar
mandibular divisions of fifth (V) cranial nerve (trigemi- process.
nal nerve) are targets for local dental anesthesia. Maxillary 3. Mandibular anesthesia occurs when vertical angle
cortical plate is thin; allows local anesthetic to penetrate (45° to the occlusal plane) is too close to 0° (flat).
through cortical plate to anesthetize maxillary nerves, so 4. Deposition site that is too deep increases potential
injections, both infiltrations (Table 14-5) and blocks, are for hematoma caused by entering pterygoid plexus
of veins and/or maxillary artery.
Table 14-5  Infiltration technique B. Complications with MSA, ASA, IO nerve blocks:
1. Pain occurs when needle contacts bony zygomatic
Single tooth: pulpal and arch, nasal spine, or alveolar process.
facial or lingual soft
Anatomy anesthetized tissue 2. May cause ONLY soft tissue anesthesia without
pulpal anesthesia when deposition sites are either
Needle gauge and length 27-gauge; short too coronal or too far away from cortical plate.
Depth of penetration Depth to above apex C. Complications with GP, NP, AMSA nerve blocks:
Landmarks MB fold, long axis of 1. Since palatal tissue is very tightly adapted to un-
tooth, apex of tooth derlying bone, may be difficult or painful.
Site of penetration Superior to apex at 2. Solution that drips from site of injection has bad
depth of MB fold or taste.
lingual/palatal 3. GP may cause soft palate anesthesia when lesser
Site of deposition Apex of tooth palatine nerve is inadvertently anesthetized.
Cartridge amount 0.45 mL (¼ cartridge) 4. NP (ONLY block that anesthetizes both right and
(1.8 mL cartridge) left sides) may fail to anesthetize canines.
Complications Inadequate pulpal anes- Mandibular Injection Procedures
thesia if below apex of
tooth or too far away Mandibular nerve block injections include inferior alveo-
from cortical plate lar (IA), buccal, mental/incisive (M/I), Gow-Gates (G-G),
Advantages Easy technique; anesthe- Akinosi (AK). See Table 14-7 for a guide to mandibular
tizes only one tooth injections. This section discusses specific complications
Disadvantages No substitute for block for each block.
anesthesia of quad- A. Complications of IA block:
rant; many injec- 1. May include failure of anesthesia:
tions; short duration;
a. Deposition sites that are too low (inferior to
increased volume
mandibular foramen) decrease anesthetic effect.
484   Saunders Review of Dental Hygiene
Table 14-6  Guide to maxillary block injections

Anterior middle
Injection Posterior superior Middle superior Anterior superior Greater palatine Nasopalatine superior alveolar
technique alveolar (PSA) alveolar (MSA) alveolar (ASA) Infraorbital (IO) (GP) (NP) (AMSA)

Anatomy Molars (all but MB Premolars, pulpal Canine, lateral, Area that covers Premolars and Canines, laterals, Area that covers
­anesthetized root first molar*); and buccal soft central; pulpal ASA and MSA molars; lingual centrals, bilater- ASA, MSA,
pulpal and buc- tissue, and MB and facial soft soft tissue only ally; lingual soft NP, GP
cal soft tissue root first molars* tissue tissue only
Needle gauge 27-gauge; short 27-gauge; short 27-gauge; short 27-gauge; long 27-gauge; short 27-gauge; short CLAD only
and length
Depth of 4-8 mm up to 4-6 mm, no bony 4-6 mm, no bony 16 mm (half length); <5 mm, bony <5 mm, bony <5 mm, bony
­penetration 10-16 mm, no contact contact bony contact in- contact contact contact
bony contact fraorbital foramen
roof
Landmarks DB root of second Second premolar, Lateral, canine, Infraorbital notch, Median palatine Incisive papilla, Median pala-
molar, MB fold MB fold canine emi- depression of in- suture, gingival marginal gingi- tine suture,
nence, MB fold fraorbital foramen margin, first val of centrals gingival
(finger pressure and second margin, first
maintained), MB molars and second
fold premolars
Site of Distal buccal Superior to second Superior to Infraorbital foramen Between first and Base of incisive Halfway be-
penetration second molar, premolar at lateral, angled where ASA and second molar papilla, 10 mm tween first
45° to occlusal depth of MB toward canine MSA nerves join and halfway from marginal and second
plane and 45° fold fossa, depth of IO nerve; preinjec- between gingiva of cen- premolar on
to midsagittal MB fold tion depth can be gingival margin trals, with pres- palate and
plane, depth of made by estimat- and median sure anesthesia halfway be-
MB fold ing extraorally palatine suture, tween gingival
using infraorbital with pressure margin and
notch, at depth of anesthesia* median pala-
MB fold tine suture,
with pressure
anesthesia*
Site of Posterior surface of Apex of second In canine fossa Apex of first Just anterior to GP Just anterior to NP Between first
deposition maxilla premolar ­premolar foramen foramen and second
premolars on
palate
Continued
Table 14-6  Guide to maxillary block injections—cont’d

Anterior middle
Injection Posterior superior Middle superior Anterior superior Greater palatine Nasopalatine superior alveolar
technique alveolar (PSA) alveolar (MSA) alveolar (ASA) Infraorbital (IO) (GP) (NP) (AMSA)

Cartridge 1.3 mL (3⁄4 0.45 mL (1⁄4 car- 0.45 mL (1⁄4 0.9 to 1.2 mL (1⁄2 to 0.45 mL (1⁄4 car- 0.3 mL or until >0.3 mL or until
amount (1.8 ­cartridge) tridge) ­cartridge) 2
⁄3 cartridge) tridge) or until blanching blanching
mL ­cartridge)† blanching
Advantages Atraumatic, high Covers premolar Easy technique Covers area of both Less volume and Less volume and Less pain, more
rate of success; area and MB ASA and MSA less needle less needle pen- volume, no
fewer injections root of first with just one penetration etration soft tissue an-
than infiltration molar* injection, less esthesia, cov-
volume ers large area
of quadrant
Disadvantages Hematoma; tris- Nerve not pres- Ballooning of Fear of eye damage Soft palate may Discomfort; solu- Need CLAD,
mus, infection ent in 28% of tissues (unfounded) be anesthe- tion is difficult delayed onset,
population* tized; swallow- to place short dura-
ing is difficult; tion, still need
discomfort PSA to com-
plete quadrant

*Anatomy can vary among patients.


†Amount can vary per patient; amount given is average for 2% agents (usually half as much for 4% agents).

Pain Management   485


486   Saunders Review of Dental Hygiene

Table 14-7  Guide to mandibular block injections

Injection Inferior alveolar Incisive/mental


technique (IA) Buccal (I/M) Gow-Gates (G-G) Akinosi (AK)

Anatomy IAN: Molars to Molars: buccal (I): Premolars Molars to midline, Molars to mid-
­anesthetized midline; pulpal, soft tissue to midline, pulpal and buc- line, pulpal
buccal soft tis- only pulpal with M cal/lingual soft buccal/lingual
sue; premolars included; (M): tissue, mylohy- soft tissue,
to incisors; LN: facial soft tis- oid, auriculo- mylohyoid
lingual soft tissue sue, lower lip temporal nerve
included by dif- and chin only
fusion
Needle gauge 25- or 27-gauge; 27-gauge; 27-gauge; short 27-gauge; long 27-gauge; long
and length long mostly long
Depth of 25 mm (3⁄4 long 2-5 mm, bony 2-5 mm 25-30 mm (3⁄4 25 mm (3⁄4 long
­penetration needle), bony contact long needle to needle)
contact before hub)
Landmarks Contralateral Occlusal plane, Mental foramen, ML cusp of sec- MG junction,
premolar, ptery- buccal cusps height of MB ond maxillary external
gomandibular of most distal fold molar, tragus of oblique ridge
fold and space, molar ear, head and
coronoid notch neck of condyle
Site of Pterygomandibular Lateral to ramus, Between first Distal to terminal Distal to termi-
­penetration space, lateral just inferior premolar and molar at height nal molar at
to the pterygo- to occlusal second pre- of ML cusp of height of MG
mandibular fold, level of buccal molar, slightly second molar junction
medial of anterior cusps of most distal to mental
border of ramus* distal molar foramen
Deposition site At mandibular fora- Lateral to ramus Distal to mental Lateral region of Medial of
men foramen condylar neck ramus, higher
than IAN and
lower than
G-G
Cartridge 1.5 to 3.6 mL (just 0.45 mL (1⁄4 0.9 mL (1⁄2 1.8 to 3.6 mL (1 1.5 to 1.8 mL (just
amount (1.8 less than 1 up to cartridge) ­cartridge) to 2 cartridges) less than 1 up
mL cartridge)† 2 cartridges) to 1 cartridge)
Advantages Single penetration Easy tech- High success rate; Single injection Single injection
permits most nique; many good for cross- for IAN, L, and for IAN, L, LB;
quadrant treat- methods of over innervation LB; used for useful when
ment injection; high accessory nerve mouth cannot
success rate innervation; open
lasts longer
Disadvantages Hematoma; low Uncomfortable May balloon tissue Difficult to see Difficult to see
success rate when bone is or cause hema- landmarks, penetration
(85%); may contacted toma delayed onset site
require two injec-
tions†

*Anatomy can vary among patients.


†Amount can vary per patient; amount given is average for 2% agents (usually half as much for 4% agents).

b. Deposition sites that are too shallow (shallow d. Crossover innervation from opposite side in-
to pterygomandibular space) decrease anes- volving the mandibular anteriors can be elimi-
thetic effect to the IA nerve but may anesthetize nated by means of PDL injection or contralateral
­lingual nerve. mental/incisive injection.
c. May be successful except for mandibular first e. If bone is contacted after penetration of less
molar; may receive accessory innervation from than half of long needle, clinician needs to repo-
mylohyoid nerves; corrected with G-G, lingual sition syringe barrel more over anteriors; needle
infiltration, or PDL injection. tip will now be more posterior.
Pain Management   487

f. If bone is NOT contacted after penetration of 4. If trismus of medial pterygoid muscle (mastica-
more than half to three quarters of long needle, tion) is present and patient CANNOT open mouth,
clinician to withdraw needle and reposition AK is recommended.
­syringe barrel more over posteriors; needle tip B. Complications of buccal block:
will now be more anterior. 1. May have ballooning of solution.
2. Touching the shallower lingual nerve during pene­ 2. Pain upon contacting bone.
tration causes patient to react (sudden ­movement); C. Complications of mental or incisive blocks:
clinician should reassure patient about “tingling sen- 1. May have ballooning of solution.
sation” or lingual shock and continue ­injection unless 2. Patient may feel mental nerve shock.
movement is too great to control injection. 3. May have hematoma from piercing mental blood
3. Bone should be contacted after penetration of three vessels.
quarters or more of the long needle to ensure NOT D. G-G: failure to connect with condylar neck; deposi-
injecting into the parotid gland, which contains the tion of solution in masseter muscle.
seventh (VII) cranial nerve (facial nerve); may cause E. AK: pain when contacting ramus or alveolar
transient facial paralysis lasting 2 hours or less. bone.

clinical study  

Age 30 YRS Scenario

Sex ☐  Male   ☒  Female The patient presents in the dental office for
nonsurgical periodontal therapy. Her peri-
Height 5’6” odontal evaluation reveals generalized severe
Weight 135 LBS chronic periodontitis, with moderate inflam-
mation and generalized 5- to 6-mm pocket
BP 112/78 depths. Only three small caries noted. The den-
tal hygienist begins after the patient has been
Chief Complaint “I haven’t been to the dentist very much.” administered the right inferior alveolar and
buccal (long) nerve blocks. When she proceeds
Medical History None
with debridement in the lower right quadrant,
Current Medications None tooth #30 is sensitive to both manual and ul-
trasonic instrumentation. All other teeth in the
Social History Daycare director, recently divorced quadrant exhibit profound anesthesia.

1. What is the most likely cause of the sensitivity in tooth root of the mandibular first molar. Possibly infil-
#30? tration of the lingual tissues on the medial of the
2. Identify the most effective ways to handle this sensi- mandible of the involved mandibular first molar or
tivity. PDL injection of the tooth would also accomplish
3. What should the dental hygienist do to prevent this anesthesia.
situation in the future? 3. Prevention includes taking thorough dental history
4. Identify other major causes of mandibular anesthesia of past anesthesia and recording sensitivity in patient
failure. treatment record for subsequent appointments, since
this accessory sensory innervation may be bilateral.
1. Most likely cause is accessory sensory nerve innerva- 4. Other major causes of mandibular anesthesia failure
tion (mylohyoid nerve). Mylohyoid nerve branches include anatomical variations, infections, needle de-
off V3 superior to mandibular foramen. Nerve runs viation, and improper type, placement, or amount of
down and forward along mylohyoid groove on me- anesthetic.
dial surface of mandible. Terminal branches may
penetrate the ramus to innervate posterior mandibu- Systemic Complications with Local Anesthesia
lar teeth, especially the mesial root of the mandibular Systemic complications can occur with administration
first molar. of local anesthetics, and others are local, such as needle
2. Alternative mandibular block, such as the Gow- breakage or trismus. MOST concern is with systemic
Gates (G-G) technique, may be used to anesthetize complications; however, they rarely occur in comparison
mylohyoid nerve, which can give accessory inner- with local ones. Specific systemic complications for this
vation to the mandibular molars, especially mesial procedure include OD and allergy.
488   Saunders Review of Dental Hygiene

• See Chapter 10, Medical and Dental Emergencies: syn- 3. Treatment includes placing patient in supine posi-
cope, allergic emergencies in a dental setting. tion, protecting patient from convulsive injury, pro-
A. Types of adverse drug reactions: viding basic life support (BLS) and activating EMS
1. Caused directly by effects of drugs; include side system, administering oxygen; supervising dentist
effects and OD. may inject anticonvulsants (see earlier discussion).
2. Caused by sensitivity that is unique to the patient F. Epinephrine (vasoconstrictor) OD (LESS common than
because of: agent OD because epinephrine is normally in the body):
a. Disease (e.g., methemoglobinemia). 1. Signs and symptoms:
b. Emotional disturbance (e.g., psychogenic syn- a. Fear, anxiety, tenseness, restlessness, head-
cope). ache, tremor, perspiration, weakness, dizziness,
c. Allergy to the local anesthetic (rare). ­pallor, respiratory difficulty.
B. Factors in OD (toxicity) of local anesthetic: b. Increased blood pressure and heart rate, pos-
1. Patient factors include age, weight, sex (e.g., preg- sible cardiac dysrhythmias, including tachycar-
nancy), disease, genetics (e.g., deficient pseudo- dia and fibrillation.
cholinesterase). 2. Likely causes:
2. Drug factors include vasoactivity, concentration of a. Concentration of epinephrine that is too high
the anesthetic, dose, route of administration, rate of (optimum safe concentration is 1:100,000 or
injection, vascularity of site, vasoconstrictor use. lower).
C. Causes of OD of local anesthetic: b. Administering too much epinephrine at one
1. Total dose that is too large; dose is determined by time (in addition to embedded gingival retrac-
weight, physical status, age. tion cord).
2. Rapid absorption into blood; rate of absorption is c. Injecting anesthetic with epinephrine directly
decreased by vasoconstrictor use. into a blood vessel; prevented by aspiration.
3. Intravascular injection; occurs MOST often (in 3. Treatment:
both veins and arteries) with IA, M/I, PSA nerve a. No treatment is necessary if brief in duration.
blocks (areas also with highest positive aspira- b. If prolonged, terminate procedure, seat patient
tions); clinician MUST be sure to aspirate and upright to decrease CVS effects, reassure pa-
inject slowly (60 sec/mL); may want to aspirate tient, give oxygen (unless hyperventilating),
twice and in different plane. provide BLS, activate EMS system.
4. Slow biotransformation (greater than 30 minutes); G. Allergy to local anesthetics (LESS common now be-
caused by systemic diseases such as atypical pseu- cause of exclusive use of amide injectables):
docholinesterase and liver disorders. 1. Signs and symptoms:
5. Slow elimination (greater than 30 minutes); caused a. Dermatological: urticaria, including wheals
by renal disease. (smooth patches), itching, angioedema (i.e., lo-
D. Mild to moderate OD of local anesthetic: calized swelling).
1. Signs and symptoms include talkativeness, excit- b. Respiratory:
edness, slurred speech, apprehension, stutter, mus- (1) Bronchial asthma is a classic sign; includes
cle twitching, increased blood pressure, increased respiratory distress, dyspnea, wheezing,
heart rate, increased respiration, lightheadedness, flushing, cyanosis, perspiration, tachycar-
dizziness, inability to focus, tinnitus, drowsiness, dia, anxiety.
disorientation. (2) Laryngeal edema; associated swelling can
2. Treatment includes reassuring patient, administer- block upper airway.
ing oxygen (prevents acidosis), and monitoring 2. Causes: allergy is a hypersensitive state caused by
vital signs; supervising dentist may administer exposure and subsequent reexposure to allergen.
diazepam (Valium, anticonvulsant) slowly (5 mg/ a. MORE likely to experience allergic reactions
min); if reaction is delayed, hepatic and renal func- to esters than to amides because esters break
tions should be checked; cause should be deter- down into PABA; includes topicals (mainly
mined before anesthetics are given again. ­esters such as 20% benzocaine).
E. Moderate to high OD of local anesthetic: b. MOST likely allergic reaction now is to sodium
1. Signs and symptoms include seizures, decreased metabisulfite, used as antioxidant to preserve
blood pressure, decreased heart rate, unconscious- vasoconstrictor.
ness with or without convulsive seizures, CNS 3. Management of allergy history:
­depression that leads to respiratory depression. a. Assume presence of allergy until it is disproved;
2. MOST likely cause is intravascular injection of postpone routine elective care until cause is
­local anesthetic. known, refer to allergist if necessary.
Pain Management   489

b. After medical consult that allows amide injection, or less); use plain local ­anesthetic agents without
avoid use of topical esters (20% benzocaine), in- vasoconstrictor such as 2% lidocaine (Xylocaine),
stead use amides (lidocaine, prilocaine) for topi- 3% plain mepivacaine (Carbocaine), or 4% prilo-
cal ­administration (although not as strong at 5% caine (Citanest) (not same duration levels).

clinical study  

Age 78 YRS Scenario

Sex ☐  Male   ☒  Female The patient is in the dental office to have


tooth #31 extracted. The tooth crown
Height 5‘4” is badly decayed. Root resection before
Weight 125 LBS removal is anticipated. During a previous
dental office visit, the patient experi-
BP 105/58 enced swollen lips and oral mucosa, with
difficulty breathing after receiving an
Chief Complaint “I am worried about the injection because of ­injection.
the last time!”

Medical History Allergy to local anesthesia

Current Medications None

Social History Retired manager of girls’ softball team


Widow

1. Did the patient have an allergic reaction to a local an- most local anesthetic–related allergic reactions. This
esthetic, or was her reaction psychosomatic? Explain. means that the appointments will have to be brief,
2. Identify the common types of local anesthetic agents since there is little duration (or hemorrhage control)
used in dentistry. Which agent(s) are least likely to with these plain agents. In addition, articaine contains
cause an allergic reaction? Which are most likely to sulfites; may want to avoid this agent for patients with
cause an allergic reaction? allergies, especially asthmatic population. Note that
3. Before the patient is given a local anesthetic, what topical agents used should not contain ester agents
­precautions should be taken? such as 20% benzocaine but lidocaine and/or prilo-
4. Discuss the type of injection(s) that are needed to caine instead, even if at lower percentage. In any case,
anesthetize tooth #31 for extraction. Identify the in- both medical and dental consult is indicated.
traoral clinical anatomical landmarks associated with 3. Following precautions should be taken: (a) consult
the area. with former dentist about type of anesthetic that was
given when patient experienced reaction or refer her
1. Patient showed signs of delayed allergic reaction to for allergy testing; (b) avoid use of topical and/or local
local anesthetic agent, namely swelling of oral ­tissues agents that contain offending agent (probably ester or
and respiratory difficulty. Allergy to anesthetic agents sodium metabisulfite or possibly use of articaine);
is rare. However, emergency protocol in the dental set- (c) consider medical and dental consult information.
ting must be ­instituted. 4. For extraction of tooth #31, 25-gauge long needle
o4340 2. Common local dental anesthetic agents include should be used. Injections should include inferior
amides, least likely to cause allergic reaction and used alveolar (IA) nerve block (also anesthetizes lingual
only for injectables, and esters, which are most likely nerve by diffusion) and buccal (long) nerve block. In-
to cause an allergic reaction but are no longer used traoral clinical anatomical landmarks associated with
for injectables. She probably has an allergenic history area include retromolar pad, coronoid notch, ptery-
based on past use of ester agent because of her age. gomandibular fold (raphe), pterygomandibular space
However, local anesthetic allergic reactions are usu- (triangle), medial surface of anterior border of ramus.
ally related to the presence of the preservative sodium
metabisulfite used as an antioxidant for vasoconstric- Local Complications with Local Anesthesia
tor. Using plain local anesthetic without vasconstrictor MOST local complications from local anesthetic ad-
such as 2% lidocaine (Xylocaine), 3% mepivacaine ministration are short term and are basic risks to local
(Carbocaine), or 4% prilocaine (Citanest) will prevent ­anesthesia administration.
490   Saunders Review of Dental Hygiene

A. Pain or burning during administration: c. If improvement does NOT occur in 48 hours,
1. Causes: careless technique or callous attitude, dull supervising dentist can prescribe antibiotics for
needle with multiple injections, rapid deposition 7 days.
of the anesthetic, difference between pH of tissues d. If improvement does NOT occur after antibiot-
and agent, contamination of cartridges with alco- ics, refer patient to oral surgeon.
hol or sterilants; use of cartridge warmer because E. Transient facial nerve paralysis: temporary paralysis
anesthetic is colder than room temperature, on side of injection, inability to open and close eyes,
2. Prevention: employ proper technique; inject mouth droops on same side.
slowly; use preinjection topical anesthesia; replace 1. Cause: inadvertent injection into posteriorly lo-
barbed needles; use sharp needles; handle unused cated parotid gland during administration of IA
cartridges properly; use local anesthetics at room nerve block because of anesthesia of fifth (V) cra-
temperature. nial nerve (facial nerve), contained in gland.
B. Hematoma: blood into extravascular spaces, espe- 2. Management:
cially noted on PSA nerve block injection. a. Instruct patient to close eyelid manually to keep
1. Cause: nicking a blood vessel; if artery, tissue ves- eye moist (take off contacts).
sel will rapidly increase in size; if vein, perhaps b. Reassure patient that paralysis is temporary and
nothing. will disappear when anesthetic effects wear off
2. Management if large: (within 2 hours).
a. Apply direct pressure, possibly with ice for at 3. Prevention: use CORRECT technique for block by
least 2 minutes. contacting bone before injecting.
b. Inform patient of possible side effects, includ- F. Persistent anesthesia (paresthesia): MAINLY in man-
ing cosmetic effects, soreness, limited move- dible, especially with IA nerve block (affects lingual
ment. nerve MORE than IA nerve).
3. Prevention: 1. Causes: trauma from contaminated (e.g., with
a. Review knowledge of anatomy, strive for ­alcohol) cartridge or by needles; pressure caused
­CORRECT technique, minimize penetrations. by excessive bleeding at site of injection; anatomy
b. Use short needle for PSA nerve block with of site; associated with 4% agents in mandible.
proper depth and angulations. 2. Management:
c. NEVER probe with needle and penetrate a. Reassure patient and explain that affected area
slowly. can require 2 to 12 months to regenerate nerves.
C. Tongue, lips, and cheek trauma (chewing tissue): b. Have patient examined every 2 months; if lasts
1. Cause: long-acting anesthetics, MAINLY with longer than 1 year, refer to oral surgeon and/or
children or mentally disabled or with use of long- neurologist.
acting agent. 3. Prevention: proper handling of cartridges and
2. Management: place cotton roll between the lips proper technique; many also recommend using
and teeth, then warn patient or caregiver about ONLY 2% type of agents in mandible.
need to AVOID hot foods and chewing until G. Needle breakage (rare):
anesthesia wears off. 1. Causes: sudden movement, smaller gauge needles
3. Prevention: using sticker; informing guardians or (30-gauge); up to hub in tissue; bent needles.
caregivers of effects: remind NOT to burn or chew 2. Management: includes use of hemostat or locking
anesthetized tissues. cotton pliers to remove broken needle from tissue;
D. Trismus: motor disturbances of trigeminal nerve, radiographs of embedded needle may be needed
MOSTLY spasms of masseter muscle, cause diffi- and referral to oral surgeon is required (see Chap-
culty in opening mouth. ter 6, General and Oral Pathology).
1. Causes: 3. Prevention: use larger gauge (27- or 25-gauge)
a. Trauma to muscles and vessels caused by re- needles; do NOT bury the hub or bend needles; do
peated penetrations or hitting a nerve. NOT redirect inside tissue.
b. Infection from contaminated cartridges.
c. Increased hemorrhage from a hematoma. NITROUS OXIDE  
d. Excessive volumes of anesthetic injected into Fearful dental patients can benefit from the relaxing
tissue. ­effects of sedation with nitrous oxide. This sedative
2. Management: can be used alone and in combination with local dental
a. Use of heat therapy, analgesics, muscle relaxants. anesthesia. However, nitrous oxide sedation does NOT
b. Encourage to exercise muscles by chewing gum replace local anesthesia for pain control, since is an anal-
(sugarless). gesic and NOT an anesthetic.
Pain Management   491

Pharmacology of Nitrous Oxide b. With TB, difficult to sterilize contaminated


Nitrous oxide is a colorless, tasteless, sweet-smelling items; postpone until infection resolved.
­inorganic gas that is NOT flammable but does support 2. Chronic obstructive pulmonary disease (COPD),
combustion. Commercially prepared by heating ammo- such as emphysema, chronic bronchitis, or lung
nium nitrate crystals in an iron retort at 240° C, then stor- cancer.
ing as a liquid at 750 pounds pressure per square inch a. Depend on lowered blood oxygen level to
(psi). As inhaled gas, does NOT combine chemically ­stimulate respiration, and with increased oxy-
with body tissues but MAINLY affects the CNS. Works gen saturation may go into apnea and stop
by stimulating the release of inhibitory neurotransmitters breathing; match test to check for pulmonary
at the neuropathway junction located within the brain. health.
It is exchanged in the lungs, beginning with the respi- b. Physician discretion for hypoxic drive, medical
ratory bronchioles and ending at the pulmonary alveolus. consult recommended.
It is NOT metabolized in the body but is eliminated by 3. Pregnancy: sedation of choice for short-term use,
diffusion properties from the lungs. Highly soluble in medical consult recommended.
blood plasma, therefore has analgesic properties in con- 4. Epilepsy: hyperventilation may occur, inducing
centrations as LOW as 20%. Note that higher altitudes epileptic fit, medical consult recommended.
increase demand for higher concentrations of use because 5. Other mental disabilities: intellectual disability
of increased need for oxygen in blood. (mental retardation), autism, Alzheimer’s disease,
• See Chapters 3, Anatomy, Biochemistry, and Physi- chemical dependency (alcohol or substance abuse):
ology: respiratory system; 9, Pharmacology: nitrous care MUST also be exercised, medical consult rec-
­oxide pharmacology. ommended.
6. Psychiatric patients and patients taking mood-
Patient Selection for Nitrous Oxide ­altering drugs: care MUST be exercised, medical
Effective administration of nitrous oxide sedation in- consult recommended.
volves both CORRECT patient selection and CORRECT 7. Emotional instability (death, divorce, job loss):
management of the sedation unit. Nitrous oxide adminis- care MUST also be exercised.
tration by dental hygienists is allowed in only some states 8. Children:
and usually under the supervision of a dentist. a. Safe, but NOT all are candidates.
• See Chapter 10, Medical and Dental Emergencies: b. May NOT be effective for children with be-
physical classification system using ASA designations. havioral problems or other special needs;
A. Patient selection using American Society of Anesthe- sedatives administered orally or IV may work
siologists (ASA) physical classification system: BETTER.
1. ASA I and II: considered appropriate candidates.
2. ASA III: greater risk but may be used after medical Procedures for Administration of Nitrous Oxide
consult. For the safe administration of nitrous oxide sedation, pa-
3. ASA IV: high risk; must seek medical consult; tient must be screened and monitored for contraindications
usually NOT appropriate to use with this patient. (see earlier discussion) and sedation unit MUST be checked
B. Absolute contraindications: for safety. National Institute for Occupational Safety and
1. Recent ophthalmic surgery during which gas was Health (NIOSH) has shown that controls can allow LOWER
administered to protect eye after surgery for up nitrous oxide concentrations in dental operatories (see
to 3 months; can disrupt the bubble and cause CD-ROM). MUST follow standard precautions.
­blindness. A. Patient considerations:
2. Cystic fibrosis. 1. Take thorough medical history.
3. Lack of patient cooperation or existence of com- 2. Monitor during administration of gas to determine
munication barrier (such as differing language). consciousness and intolerance.
4. Patients with head trauma or shock. 3. Instruct patient NOT to talk to decrease exhalation
5. Middle ear infection (can experience ear pain of gas into the room air and keep sedation con-
­because of increased pressure). stant.
6. Fear of nitrous oxide or need to always be in con- B. Nitrous oxide–oxygen unit:
trol (NEVER talk anyone into use). 1. Thoroughly check unit for leaks and unsafe equip-
C. Relative contraindications: ment.
1. Respiratory infection such as with sinus or tonsils; 2. Monitor MAIN “fail-safe” features:
cold; active allergies; active tuberculosis (TB). a. Universal color scheme: used for tanks (blue
a. Gas enters lungs through nasopharyngeal pas- for nitrous oxide; green for oxygen); tanks for
sages, so MUST be clear for gas to work. the two gases are NOT interchangeable.
492   Saunders Review of Dental Hygiene

b. Flowmeter: measures how much of BOTH c. Some units automatically determine oxygen
oxygen and nitrous oxide is delivered in liters level when gas level is adjusted.
per minute (Lpm) 4. At 1-minute intervals, decrease oxygen flow by
(1) Minimum flow of oxygen is 2 Lpm and same amount that nitrous oxide flow is increased
­nitrous oxide flow is 0.5 Lpm. until patient acknowledges appropriate effects:
(2) Nitrous oxide shuts off when oxygen quits; a. Process allows nitrous oxide to be titrated
also, clinician can turn off nitrous oxide at (given incremental amounts); current standard
any time. of care.
c. Scavenger system: removes exhaled gases b. Limits amount of drug that is required; allows
from mask (nasal hood) and room air. variety and safe performance of prolonged
d. Flush system: available to provide 8 or more li- ­procedure.
ters of 100% oxygen to patient if there are signs c. Takes 30 to 60 seconds for changes in gas levels
of intolerance. to be felt by patient.
e. Reservoir bag: used to assist or control respi- 5. Once flow is at level where patient is comfortable,
ration during procedure and in the event of an start procedure.
emergency. 6. Increase level of nitrous oxide during painful or
f. Nitrous tank: remains at 750 psi until it is stressful portions of treatment by 5% Lpm from
MOSTLY empty; meter cannot be used to present levels to ensure pain control.
tell how much is left in the tank, UNLIKE the 7. Continue to monitor patient for signs of intoler-
tank of oxygen, which can tell amount left. ance, including nausea, diaphoresis (perspiration),
C. Preprocedural steps: unconsciousness, or changes in behavior (see later
1. Look at chart for last entry involving use and pain discussion).
control issues for patient. a. Patients must NEVER be left unattended;
2. Have patient use restroom, take off contacts (eyes ­otherwise they may feel that they have been
can get dry from air from mask). left alone and may panic, become agitated, or
3. Obtain preprocedure vital signs. remove mask; also, oversedation and toxicity
4. Have patient supine with legs uncrossed to pre- could occur.
vent changes in circulation and paresthesia (numb- b. Once baseline is achieved, level of nitrous ­oxide
ness). should be dropped back 0.5 to 1.0 Lpm and
5. Explain procedure to patient; also explain poten- oxygen turned up by same amount; can also be
tial sensations associated with gas. decreased near end of appointment.
a. Percentage noted is ONLY somewhat relevant 8. When nitrous oxide is NO longer needed, have
for current appointment. ­patient breathe 100% oxygen for at least 5 minutes
b. MOST common mistake is to automatically or until effects of sedation disappear (see later dis-
­deliver a preset percentage to all patients. cussion on importance).
6. Look for any changes in emotional state and listen E. Postprocedural steps:
for any similar verbal cues. 1. Always evaluate recovery: verbal should be same
D. Procedural steps: as if nothing happened.
1. Turn on unit and push in flush valve to allow air for 2. Obtain postprocedure vital signs.
first breaths. 3. Document use in records.
2. Adjust mask for proper secure fit and turn on 4. Thank patient for cooperation, and reinforce
­scavenger system. ­success of appointment.
3. Determine and administer to patient the 5. Have patient wait 15 minutes before leaving if
CORRECT liter flow using 100% oxygen: driving:
a. Considered minimum flow rate (tidal vol- a. Motor skills and attention can be affected for as
ume [TV]) when patient is breathing comfo­rt­ long as 15 minutes after patient stops breathing
ably: gas.
(1) Average adults breathe 4 to 6 Lpm. b. Offense for person to drive while under influ-
(2) Athletes and large adults breathe 7 to ence of drugs; office can be held liable.
8 Lpm.
(3) Children and small adults breathe 4 to Signs and Symptoms Associated with Nitrous
5 Lpm. Oxide Use
b. Make sure reservoir bag is NOT overinflated There are signs and symptoms associated with nitrous
or underinflated while patient is breathing; ­oxide use. However, response varies under sedation.
SHOULD fluctuate as patient breathes. Never liken it to alcohol, since that may upset the ­patient.
Pain Management   493

Table 14-8  Signs and symptoms in response to nitrous Nitrous Oxide Complications
oxide sedation Nitrous oxide is very safe. However, about 15% of
patient experience side effects, including headache, nau-
 oncentration
C sea, vomiting. Other possible side effects are excessive
of N2O (%) Possible response*
sweating and shivering. Behavioral problems (possibly
10-20 Body warmth, tingling of hands sexual dreaming) may occur. If patient experiences side
and feet effects, gas can be turned off and patient can breathe 100%
20-30 Numbness of thighs oxygen for up to 5 minutes. Patient SHOULD tell imme-
diately about any discomfort or other concerns. MOST
20-40 Numbness of hands, feet, and
tongue, droning sounds pres- complications result from excessive levels of gas (more
ent, dissociation begins and than 50%) or not monitoring patients while they are un-
reaches peak, mild sleepiness, der influence of gas.
analgesia (minimum at 30%), • See Chapter 10, Medical and Dental Emergencies:
euphoria
­dental emergencies.
30-50 Sweating, nausea, increased A. Nausea: MORE common than vomiting.
­sleepiness
1. With prompt recognition, can be eliminated; un-
40-60 Dreaming (possibly sexual), recognized, it can lead to vomiting.
laughing, further increase in
sleepiness, tending toward un-
2. Patient monitoring is major means of preventing.
consciousness, increased nausea a. Watching face, arms, and body for any unusual
and vomiting expression or movements.
50 and over Loss of consciousness and light b. Looking for pained features and other signs
­general anesthesia such as pallor, sweating, hands over abdomen.
3. Causes:
*Standard of care states that N2O should be used at <50% for most den- a. Increased depth or length of sedation; concen-
tal procedures because higher levels increase risk of complicating signs and
symptoms. tration of gas.
b. Overwrought patient.
Since MOST nonsurgical dental procedures (such as those c. If patient has inherent tendency to become
performed by a dental hygienist) use minimal levels of nauseated, may be premedicated with prescrip-
­sedation (less than 50% of gas, standard of care), patient tion or OTC antiemetic drugs (dimenhydrinate
remains awake and aware while breathing and can par- [Dramamine]); some should be avoided during
ticipate in treatment. Patient will still be able to respond pregnancy.
to requests and answer questions, but speech may be d. Presence or absence of food in stomach; heavy
slightly slurred and responses may be slower than usual. meal is NOT recommended but does not require
Will be relaxed and cooperative, probably will NOT feel empty stomach.
any injections or other discomforting parts of dentistry, e. Changes in patient’s position; mouth breathing
and will lose track of time. Effective sedation occurs in and prolonged conversation reduce sedation
MOST patients. This level of gas keeps complications level; once patient is resedated with reminder
from occurring and maintains safety (Table 14-8). See to nose breathe, may revert to mouth breathing
later discussion of complications. (“roller coaster”).
A. Signs: 4. Management: by decreasing gas concentration by
1. Patient awake, drowsy, relaxed appearance. ∼5% to 10% until patient feels comfortable.
2. Eye reaction and pupil size, BP, HR, and RR ­normal. B. Vomiting (emesis): prevented by prompt recogni-
3. Flushing of skin with slight perspiration and lacri- tion and management of nausea; LESS common than
mation. nausea.
4. Little or no gagging or coughing; speech infre- 1. MORE common in children; may vomit without
quent and slow. warning.
5. Minimal movement of arms and legs. 2. Caused by oversedation; higher risk during induc-
6. Lessened pain reaction. tion when communication is harder and patient is
B. Symptoms: more likely to mouth breathe.
1. Sense of security, feeling of warmth, pleasant 3. Vomitus may be aspirated because of posi-
floating sensation. tion of head in dental chair; potential to produce
2. Tingling in hands and feet. obstructed airway and its complications.
3. Feeling of heaviness or lightness. 4. Warning signs are nausea, pallor, cold sweat, cold
4. Changes in way sounds are heard (everything and clammy hands, increased salivation, active
sounds far away). swallowing.
494   Saunders Review of Dental Hygiene

5. Immediately turn off gas flow, permitting patient C. Excessive sweating:


to breathe 100% oxygen. 1. May normally become flushed by the peripheral
a. Remove delivery apparatus from face and re- vasodilating properties of gas; minor perspiration
movable dental equipment from mouth. is usually noted on forehead, arms, or hands.
b. Turn head and body to side to allow vomitus to 2. If severe, concentration of gas is slowly reduced by
pool in cheek and NOT pharynx. ∼5% per in attempt to make patient comfortable.
c. Use basin and/or high-speed suction (high- 3. If unable to stop patient’s perspiring, procedure is
­volume evacuator) to remove vomitus. aborted.
6. After incident, have patient breathe 100% O2 again 4. However, if accompanied by pallor, drop in BP,
to reduce further vomiting. and/or increased HR, give patient 100% oxygen,
a. Patient may NOT want to have delivery appara- provide BLS, activate EMS system.
tus on again for fear of becoming sick again, so D. Shivering: NOT uncommon but can be uncomfort-
explain need for oxygen. able; usually develops at end of procedure.
b. Explain that vomiting was an unusual oc- 1. With the flushing and sweating that occur with use,
currence and is UNLIKELY to occur again, core, temperature can be reduced.
to avoid discouraging patient from future 2. Reassure that everything is fine; place blanket over
use. patient to speed warming process.

clinical study  

Age 37 YRS SCENARIO

Sex ☐  Male   ☒  Female The patient has an appointment today for


a crown lengthening on a maxillary molar
Height 6’2” tooth before the final restoration is placed.
Weight 185 LBS To reduce her gag reflex, the clinician would
like to use nitrous oxide sedation during
BP 102/56 the appointment. Near the end of the ap-
pointment, the patient becomes pale, starts
Chief Complaint “I am really nervous about the needle!” to sweat on her forehead, and places her
hands on her abdomen.
Medical History Knee replacement surgery twice
Back surgery last year

Current Medications OTC vitamin preparations qd

Social History Marathon runner

1. What is a crown lengthening procedure? ­ thletes and large persons can be at 7 to 8 Lpm; chil-
A
2. How does the patient’s being an athlete change the dren and small adults are at 4 to 5 Lpm.
treatment using nitrous oxide sedation? 3. Nitrous oxide sedation calms gag reflex, which is es-
3. How does nitrous oxide sedation help the patient with pecially noted when working on the maxillary molar
the gag reflex? teeth as in this patient’s case.
4. How will the clinician respond to her fear of needles 4. Clinician needs to increase nitrous oxide by 5% (0.5
when using nitrous oxide sedation? Lpm) and decrease same amount of oxygen to make her
5. What does it mean when the patient places her hands more comfortable during the injection of local anesthetic
over her abdomen and becomes pale and sweaty? or any other painful, invasive procedure. ­After injection,
amount of gas can be reduced to original base levels or
1. Crown lengthening is used when tooth needs to be less and same amount of oxygen can be increased.
res­tored but there is not enough tooth structure above 5. Patient is becoming nauseated (pale and sweaty with
the free gingival margin to support direct or indirect hands on abdomen and sweating forehead) because
­restoration (crown). higher levels of gas are present during last part of the
2. As an athlete, the patient may need greater volume appointment. To possibly prevent this, clinician needs
of air for her breathing than the average, nonathletic to reduce amount of gas and increase level of oxygen
adult, who would receive 5 to 6 Lpm, so her minimum until patient feels comfortable. To avoid this situation,
flow rate (tidal volume [TV]) during nitrous oxide the clinician should have begun to reduce the nitrous
sedation should be increased if desired by patient. oxide levels near the end of the appointment.
Pain Management   495

Nitrous Oxide Abuse   2 Which of the following local anesthetic agents is MOST
Nitrous oxide can be abused by dental personnel for re­ appropriate if severe and prolonged postoperative surgical
creational use (acute) or because of psychological depen- dental pain is expected?
dence (chronic). A. With 3% mepivacaine (Carbocaine)
• See Chapter 9, Pharmacology: substance abuse patient. B. With 4% prilocaine with epinephrine (Citanest Forte)
C. With 2% bupivacaine (Marcaine) with epinephrine
A. Short-term effects:
D. With 2% lidocaine (Xylocaine) with epinephrine
1. May have slurred speech; difficulty maintaining
E. With 2% mepivacaine with levonordefrin (Carbocaine
balance when walking; delay in responding to ques- with Neo-Cobefrin)
tions; no response to stimulus such as pain, loud   3 Which of the following statements concerning nitrous oxide
noise, or speech; may lapse into unconsciousness. use is CORRECT?
2. Person who is rendered unconscious is likely A. Requires a long time for recovery
to stop breathing within seconds as result of de- B. Used to induce stage III anesthesia quickly
pressed nervous system; if remains conscious and C. Both flammable and explosive
stops breathing gas, recovery can occur within a D. The safest of all analgesics
few minutes.   4 Which descriptions can be used in relation to trismus and
3. Person who remains unconscious and continues to local anesthesia procedure?
A. Extremely rare
inhale pure gas is likely to die.
B. Total lack of sensation
4. Death also occurs when users, in attempt to reach
C. Motor disturbance of the trigeminal nerve
higher state of euphoria, breathe gas in confined D. Sensory disturbance of the trigeminal nerve
space (small room, inside automobile) or by plac-   5 Which of the following is an immediate treatment for
ing head in plastic bag. ­injection-induced hematoma?
5. Long-term exposure (several minutes) is NOT A. Ice and pressure
necessary before death occurs. B. Surgical removal
B. Effects of chronic use: unusual but can occur; may C. Heat placement
cause vitamin B12 (cobalamin) deficiency. D. Aspirin injection
1. Red blood cell count is lowered, resulting in ane-   6 Which of the following situations is CORRECT in regard to
mia and nerve degeneration. the dissociation of local anesthetics?
A. The more base molecules (RN + H+) available, the ­better
2. Painful and/or numbing sensations, unsteady walk,
the diffusion properties.
or irritated appearance.
B. The more cation molecules (RNH+) available, the better
3. May also result in depression of heart muscular the diffusion properties.
functioning and cardiac disturbances. C. The fewer base molecules (RN + H+) available, the
C. Dental personnel recommendations: weaker the binding properties.
1. Exposures SHOULD be minimized to prevent D. The fewer cation molecules (RNH+) available, the
short-term behavioral and long-term reproductive greater the binding properties.
health effects.   7 What is the BEST response when a patient experiences
2. Under OSHA guidelines, accepted level of nitrous vasodepressor syncope with local anesthetic administration?
oxide in the dental operatory should NOT exceed A. Activate emergency medical system.
50 ppm (parts per million). B. Begin cardiopulmonary resuscitation.
C. Place the patient in a subsupine position.
3. Chronic exposure should be avoided by women in
D. Administer 1:1000 injectable epinephrine.
first trimester of pregnancy and infertile individu-
  8 Which of the following explains why, when a vasoconstric-
als using in vitro fertilization procedures (linked tor is added, the pH of a local anesthetic (2% lidocaine),
to spontaneous abortions in surgical nurses), also which was approximately 6.8, drops to approximately 4.2?
those with neurological complaints and immuno- A. Epinephrine (vasoconstrictor) is acidic.
compromised persons because of bone marrow B. Epinephrine (vasoconstrictor) is basic.
suppression. C. Preservative (sodium metabisulfite) is acidic.
D. Preservative (sodium metabisulfite) is basic.
E. Tissue has a pH of 5.5.
Review Questions   9 Which of the following describes the half-life of a local
­anesthetic agent?
A. Rate at which the local anesthetic is eliminated from the
blood
  1 In a sensory neuron, the dendrite zone functions to
B. Rate at which the local anesthetic effects the blockage
A. synapse with CNS nerves.
of the nerve impulse
B. support metabolism.
C. Length of time an anesthetic can be stored before it is no
C. receive a stimulus.
longer effective
D. direct ion movement.
D. Average length of time it takes for anesthesia to wear off
496   Saunders Review of Dental Hygiene

10 Positive aspiration indicates all of the following situations, 19 Which determines the depth of penetration for a maxillary
EXCEPT one. Which one is the EXCEPTION? infiltration local anesthetic injection?
A. Needle is in a blood vessel. A. Length of the needle
B. Hematoma developed. B. Root length of the tooth
C. Poor technique was employed. C. Gauge of the needle
D. Appropriate needle gauge was used. D. Location of the zygomatic bone
11 What is the MOST likely cause of an overdose reaction 20 During the administration of an inferior alveolar nerve
when a patient receives 2% lidocaine with 1:100,000 epi- block, resistance is met after 25 mm. The resistance is
nephrine and experiences symptoms of overdose approxi- MOST likely caused by which anatomical structure?
mately 30 minutes after the injection? A. Medial pterygoid muscle
A. Excessive dose of lidocaine B. Internal oblique ridge
B. Injection of the solution into a blood vessel C. Pterygomandibular raphe
C. Liver dysfunction or renal impairment D. Medial surface of the ramus
D. Rapid injection of the solution 21 During the administration of an inferior alveolar nerve
12 What part of the chemical structure makes a local anesthetic block, resistance is met after 25 mm. Which of the follow-
agent hydrophilic? ing is the BEST way to proceed?
A. Amino group A. Continue to penetrate tissue until the needle insertion
B. Aromatic ring depth reaches the hub.
C. Intermediate chain B. Aspirate and then slowly deposit the solution.
D. Benzene ring C. Withdraw the needle, reinsert the needle superiorly, and
13 A local anesthetic solution must form a salt to penetrate until bone is recontacted.
A. enable the anesthetic to penetrate the nerve membrane. D. Back off slightly, redirect the needle laterally, and pen-
B. enable the anesthetic to diffuse through the tissue to the etrate until bone is recontacted.
nerve. 22 Which of the following muscles may be involved in tris-
C. decrease the acidic properties of the solution. mus during the administration of an inferior alveolar nerve
D. increase the shelf-life of the solution. block?
14 All injectable local anesthetic drugs used in dentistry are A. Buccinator
A. vasoconstrictors. B. Lateral pterygoid
B. vasodilators. C. Masseter
C. neither vasoconstrictors nor vasodilators. D. Medial pterygoid
D. both vasoconstrictors and vasodilators. 23 In which of the following situations is the Akinosi technique
15 What is the MOST likely cause of a patient giving a sud- for local anesthesia indicated?
den jump and complaining of a tingling sensation shoot- A. Alternative to the posterior superior alveolar nerve block
ing downward in the jaw during an inferior alveolar nerve B. When trismus decreases the ability to open the mouth
block? C. For young children or mentally disabled adults
A. Injecting too rapidly for the situation D. When bilateral mandibular anesthesia is required
B. Injecting into the inferior alveolar artery 24 What is the deposition site for the anterior superior alveolar
C. Contacting the lingual nerve nerve block?
D. Contacting the inferior alveolar nerve A. Apical to the lateral incisor
16 Which of the following situations would MOST likely cause B. At the apex of the canine
a leaky anesthetic cartridge? C. In the canine fossa
A. Piercing the cartridge diaphragm on an angle D. Over the central incisor
B. Failing to screw the needle tightly onto the syringe 25 For which of the following nerve block injections does tis-
C. Air bubbles in the cartridge sue blanching indicate an adequate deposition of local anes-
D. Moderate corrosion on the metal cap thetic agent?
17 Performing aspiration in two planes during administration A. Posterior superior alveolar
of a local anesthetic is necessary to B. Anterior superior alveolar
A. prevent the needle from entering a blood vessel and C. Nasopalatine
causing trauma. D. Mental
B. ensure that the bevel is facing the bone to prevent E. Inferior alveolar
trauma. 26 To determine the maximum recommended dose of a local
C. reduce the internal pressure of the cartridge anesthetic for a particular patient, the clinician MUST do
D. ensure that a blood vessel wall is not drawn against the which of the following?
bevel during aspiration A. Multiply the age of the patient by the dose per ­cartridge.
18 An inferior alveolar nerve block does NOT provide B. Multiply the manufacturer’s number for milligrams per
­anesthesia if the solution is deposited too pound by the weight of the patient.
A. medially. C. Divide the milligrams per cartridge of the local anes-
B. laterally. thetic by the weight of the patient.
C. superiorly. D. Divide the maximum cartridges allowed by the milli-
D. inferiorly. grams per cartridge of the local anesthetic.
Pain Management   497

27 The BEST way to approach a fearful dental patient about to 36 Which of the following is the primary site of action for
undergo extensive dental hygiene treatment is to ­nitrous oxide?
A. perform treatment quickly to reduce the time that the A. Central nervous system
patient is stressed. B. Urinary system
B. tell the patient that nitrous oxide–oxygen sedation is C. Respiratory system
available. D. Cardiovascular system
C. give little information about the procedures to avoid 37 About how long does it usually take for the patient to feel
frightening the patient. the changes to the level of the flow of the gases when ni-
D. increase the patient’s control of the treatment session. trous oxide is administered?
28 All of the following are fail-safe components of the nitrous A. 5 to 10 seconds
oxide unit, EXCEPT one. Which one is the EXCEPTION? B. 20 seconds
A. Minimum flow of nitrous oxide is 2 Lpm. C. 30 to 60 seconds
B. Different universal colors are used to label oxygen and D. 2 minutes
nitrous oxide. 38 Which of the following is a common mistake when using
C. Oxygen and nitrous oxide tanks cannot be inter- nitrous oxide in dentistry?
changed. A. Individualizing the amount of nitrous oxide the patient
D. Scavenger system is used to decrease exhaled gases. receives
29 The MAIN goal of nitrous oxide sedation of the dental B. Using the O2 flush button to remove mixture of gases
­patient is from bag
A. decreased chair time for patient care. C. Once baseline is achieved, dropping back the level of
B. stress and anxiety reduction. nitrous oxide
C. compliance for opening the mouth. D. Allowing patients to let the dental staff know how they
D. to put the patient to sleep. feel
30 The PRIMARY safety factor for the patient with nitrous 39 Which of the following signs should be present during
­oxide administration is use of general dental treatment levels of sedation with nitrous
A. oxygen flush button. oxide?
B. disposable masks only. A. Patient is asleep
C. titration technique. B. Lessened pain reaction
D. infection control standards. C. Extra movement of arms and legs
31 Patient indications for nitrous oxide sedation include D. Speech frequent and rapid
A. mild anxiety. 40 Which of the following should be done when introducing
B. emotionally unstable. the patient to the nitrous oxide procedure?
C. severe mental retardation. A. Ask the patient to refrain from mouth breathing
D. severe cerebral palsy. B. Allow only the patient to choose mask fit.
32 Patient contraindications for nitrous oxide sedation can C. Liken the feeling to use of alcohol.
­include D. Have the patient sit upright with legs crossed.
A. language barrier. 41 Which of the following is a pharmacological property of
B. asthma patient. nitrous oxide?
C. overactive gagger. A. Sour smelling
D. cardiac patient. B. Amber colored
33 For BEST sedation appointment results, the patient should C. Explosive
usually receive oxygen alone for ____ at the end of the D. Inorganic
­nitrous oxide administration. 42 The reservoir bag while using nitrous oxide in the dental
A. 1 to 2 minutes office should remain
B. 3 to 5 minutes A. overinflated.
C. 6 to 10 minutes B. deflated.
D. 10 to 20 minutes C. partially inflated.
34 Chronic exposure to nitrous oxide by the dental staff can D. stored in the drawer.
BEST be prevented by use of 43 Which of the following can happen to a patient who feels
A. titration methods. the symptoms of nausea?
B. a pin-index system. A. Hands over stomach
C. a scavenging mask system. B. Feels fine
D. infection control standards. C. Smiling face
35 A scavenging system is designed to D. Red complexion
A. collect exhaled nitrous oxide and return it to the tank. 44 What needs to be done before you begin nitrous oxide
B. collect exhaled nitrous oxide and vent it out through the ­administration on a patient in a dental office?
suction system. A. Have patient use the restroom.
C. control the source of infection in the mask from the B. Have patient put contact lenses in.
­patient. C. Only take pulse reading.
D. control costs of the nitrous oxide gas. D. Have patient wait to use restroom.
498   Saunders Review of Dental Hygiene

45 Which of the following statements concerning nitrous oxide even with vasoconstrictors epinephrine and levonor-
sedation is CORRECT? defrin added, and shorter still without epinephrine.
A. A dental patient can be left unattended for a short time if Bupivacaine is generally NOT used for dental hy-
the patient has had this sedation before. giene care because of its long action.
B. The fail-safe system does not permit the flow of nitrous 3 (D)  Inhaled nitrous oxide–oxygen does NOT com-
oxide if the flow of oxygen is interrupted.
bine chemically with body tissues and thus is ­safest
C. All child patients are excellent candidates for sedation.
D. Patients who are highly anxious about dentistry are
of all analgesics. Gas is taken in and eliminated
­excellent candidates for sedation. in the same state, and recovery is quick; nitrous
46 Which of the following describes an action potential in a oxide supports combustion but is NOT flammable
neuron? or ­ explosive. In the dental office setting, nitrous
A. Local event oxide should NOT be used to induce stage III
B. Small in amplitude ­anesthesia.
C. Travels down an axon toward the synapse 4 (C)  Trismus is a motor, NOT sensory, disturbance
D. Caused by an influx of negative ions of the fifth (V) cranial (trigeminal). It disturbs ­motor
47 Which of the following four types of nerve fibers has the nerves that affect the muscles of mastication and
fastest conduction velocity? ­increases difficulty in mastication and in opening and
A. Unmyelinated small fiber
closing the mouth. Trismus is common and seldom
B. Unmyelinated large fiber
C. Myelinated small fiber
involves a complete loss of sensation.
D. Myelinated large fiber 5 (A)  Tearing a blood vessel causes a hematoma. Initial
48 When local anesthesia is used to anesthetize the inferior treatment includes applying ice to the area with direct
­alveolar nerve, which nerve is secondarily ­anesthetized? pressure for 2 minutes to decrease blood flow. Hema-
A. Posterior superior alveolar toma does NOT need surgical removal because it will
B. Anterior superior alveolar dissipate with time. Initially, heat and aspirin should
C. Lingual NOT be used because they may increase bleeding in
D. Infraorbital the area. Heat may be applied after 4 to 6 hours or the
49 What should a dental professional do immediately if a next day to increase healing.
­patient shivers when under nitrous oxide sedation? 6 (A)  Base molecules (RN) are uncharged ions and are
A. Have the patient use the restroom.
responsible for the diffusion properties of the local
B. Cover the patient with a blanket.
C. Increase tidal volume.
­anesthetic; the more free base molecules available, the
D. Give the patient 100% oxygen. BETTER the diffusion properties. Number of cation
E. Abort the procedure. molecules (RNH) determines the anesthetic’s bind-
50 All of the following should be done immediately after the ing ability; the more cations, the longer the ­anesthetic
administration of nitrous oxide, EXCEPT one. Which one is agent remains in the targeted area.
the EXCEPTION? 7 (C)  Vasopressor syncope is fainting that is caused by
A. Administer 100% oxygen to clear the patient. a pooling of blood in the muscles, which decreases
B. Take blood pressure reading. the blood flow to the brain. Performing procedures
C. Allow the patient to drive. with patient in a supine position helps prevent syn-
D. Thank the patient for cooperation. cope. If syncope does occur, patient should be placed
in a subsupine position (Trendelenburg position),
with feet higher than head to the floor; this increases
Answer Key and Rationales the blood flow to the brain. Activating the emergency
medical service (EMS) system, beginning cardio-
pulmonary resuscitation (CPR), and administering
1 (C)  Dendrite zone is composed of small nerve end- epinephrine are NOT appropriate choices for this
ings at the terminal ends of nerves. These free nerve common emergency.
endings receive stimuli from tissues. When clinician 8 (C)  Agents with vasoconstrictors require preser­ o8880
places a curet in a pocket and nerve endings of den- vatives to extend shelf-life to between 18 and
drite zone receive the stimulus, this sends the im- 24 months (level for plain agents). The manufac-
pulse along the nerve to the CNS, which interprets turer adds preservative (sodium metabisulfite) to
the ­sensation as pain. make the local anesthetic MORE acidic so it will
2 (A)  The 2% bupivacaine (Marcaine) is a long-act- last ­longer.
ing local anesthetic agent that lasts 1 to 2 hours as 9 (A)  Half-life is defined as the rate at which half
an infiltration and 5 to 9 hours as a nerve block for of agent is eliminated from blood. For example,
pulpal anesthesia, with vasconstrictor epinephrine ­lidocaine has half-life of 90 minutes, which means
­concentration at 1:200,000. Others are shorter acting that 50% of agent is eliminated in 90 minutes.
Pain Management   499

10 (C)  Careful technique is important but does NOT s­ urface has been exposed to disinfectant or anesthetic
eliminate chance of positive aspiration. Aspiration is and thus should be discarded.
act of drawing fluids into a syringe. If needle tip is in 17 (D)  If the needle is in a blood vessel during aspira-
a blood vessel, blood will flow into the cartridge. If tion, blood vessel wall can be drawn in against the
needle tip is NOT in a blood vessel, interstitial fluids bevel, giving false-negative aspiration. Rotating
will flow into the cartridge. However, if blood flows needle and aspirating again increases likelihood of
into tissues from a hematoma (tearing of the blood accurately determining needle location. Injecting
vessel) and clinician aspirates, blood will flow into ­local anesthetic into a blood vessel greatly increases
the cartridge, giving a positive aspiration. Positive potential for overdose reactions. Aspiration will alert
aspiration is indication that needle lumen size was the operator of (NOT prevent) possibility of entering
inadequate for aspiration. a blood vessel. Observation of needle bevel before
11 (C)  Slow rise (30 minutes or more) in blood levels insertion is necessary to prevent scraping of perios-
of lidocaine indicates that amide is NOT undergo- teum. Aspiration increases rather than decreases in-
ing proper elimination, typically because of liver ternal pressure of cartridge.
dysfunction or renal impairment. Rapid rise in blood 18 (D)  If the anesthetic is deposited inferior to man-
levels of lidocaine can be caused by excessive dose of dibular foramen, inferior alveolar nerve will NOT be
local anesthetic, by injecting too quickly, or by inject- anesthetized. Deposition of the anesthetic at superior
ing anesthetics into blood vessel. deposition site will provide anesthesia. Deposition
12 (A)  Hydrophilic portion of local anesthetic is amino superiorly, medially, and laterally may slow the speed
group, which determines diffusion properties of of onset but will result in anesthesia.
­anesthetic. If amino group does NOT exist, ­anesthetic 19 (B)  To anesthetize a single tooth using infiltration,
CANNOT be injected into tissues. Aromatic and anesthetic must be deposited at or above root apex.
­benzene rings provide lipophilic portion, which Important to know length of root to provide adequate
­increases nerve penetration. Intermediate chain anesthesia to individual tooth. NEITHER length or
­determines whether anesthetic is ester or amide. gauge of needle NOR location of zygomatic bone
13 (B)  Local anesthetic agents must be combined with is necessary to determine depth of penetration for
hydrochloride to form a salt, which makes anesthetic ­infiltration anesthesia.
more soluble in water (diffuses through tissue) and 20 (B)  One of the first bony landmarks for the inferior
thus more injectable. Does NOT increase anesthetic alveolar nerve (IAN) block is the internal oblique
agent’s shelf-life, affect acidic properties, or enable ridge. If positioning is too low, this landmark will be
anesthetic to penetrate nerve membrane. contacted ALMOST immediately during insertion, or
14 (B)  Local anesthetic agents used in dentistry act as as soon as approximately half of the needle progresses
vasodilators of the blood vessels in surrounding areas. through the tissue (less than 16 mm). Contact with the
Vasoconstrictors are added to INCREASE duration medial pterygoid muscle may occur but ­ offers little
of agent and to decrease the potential toxic effects of resistance. Pterygomandibular fold (raphe) is another
too much anesthetic in blood. landmark used for insertion during the IAN block,
15 (C)  One major complication of the inferior alveolar which would be lateral to this structure but medial
(IA) nerve block is caused by the close proximity of to the target site. Contact with the medial surface of
the more superficial lingual nerve and the potential the ramus should occur when bone is contacted af-
for contacting this nerve during penetration. When ter penetration of three fourths or more of the needle,
the lingual nerve is contacted, patient feels tingling usually 25 mm in depth, for an average adult.
(lingual shock) along the nerve that enters ramus at 21 (D)  Depth of penetration for the inferior alveolar
the mandibular foramen. Injecting TOO rapidly will nerve block is approximately three quarters of long
cause discomfort during the injection, NOT ­tingling. needle, usually 25 mm in depth. Medial surface of the
Injecting into IA artery may cause an overdose ramus should be contacted before any solution is de-
­reaction. posited to prevent deposition into the posterior-placed
16 (A)  Needle must pierce the diaphragm so that it is parotid salivary gland with a resultant transient facial
able to make a tight seal around the needle. If needle paralysis (temporary anesthesia of the facial nerve
pierces the diaphragm on an angle, making oval hole, contained in gland).
faulty seal will result and cause leakage of anesthetic. 22 (D)  On the medial surface of the ramus, the ­medial
Loosely tightened needle may cause needle to fall off pterygoid muscle, muscle of mastication, runs
but will NOT cause leakage of solution. Cartridges ­superiorly from the posterior, inferior angle of the
exposed to freezing temperatures may exhibit bub- ­ramus and covers the mandibular foramen and
bles and SHOULD be discarded. Corrosion of metal ­inferior ­ alveolar nerve. Penetration of this muscle
cap, even when moderate, indicates that its outer may cause trismus. Buccinator is NOT a muscle of
500   Saunders Review of Dental Hygiene

mastication, whereas pterygoid and masseter muscles contraindication is cerebral palsy, especially if
are NOT located near the site of inferior alveolar ­severe.
nerve ­injection. 32 (A)  Patient (relative) contraindications do include
23 (B)  Akinosi technique, used as an alternative man- language barrier, since patient should be able to com-
dibular block, is indicated when the patient CANNOT municate immediately if experiencing any compli-
open mouth. This technique is NOT specifically indi- cations that might be serious. Important indications
cated for young children, mentally disabled adults, or for use of gas include asthma (more oxygen), strong
bilateral mandibular injections. gag reflex (controls it), and cardiac disease (relaxes
24 (C)  The ASA nerve is a branch of the maxillary di- ­patient).
vision of the fifth (V) cranial (trigeminal). Terminal 33 (B)  For best sedation appointment results, the pa-
nerve endings are located within alveolar process in tient SHOULD usually receive oxygen alone for 3 to
the area of the canine fossa. 5 minutes at the end of the nitrous oxide administra-
25 (C) For NP nerve block injection, amount of anes- tion, which allows the gas to be removed from the
thetic deposited should be just enough to blanch tis- body. However, it may be longer for some patients. In
sues. Other injections listed require deposition of addition, patient needs to wait 15 minutes or longer
solution in milliliters. before driving.
26 (B)  For each type of local anesthetic, manufacturer 34 (C)  Chronic exposure to nitrous oxide by dental staff
establishes a maximum recommended dose (MRD) can be prevented by use of scavenging system, which
(in milligrams per pound). To determine MRD for a removes excess gas from operatory. Titration meth-
patient, clinician should multiply MRD per pound ods are for safety, but for the patient, as is use of in-
by weight of patient. The MRD for 2% lidocaine is fection control standards.
2 mg/lb; thus a 140 lb patient could safely receive 35 (B)  Scavenging system is designed to collect ex-
280 mg of lidocaine. haled nitrous oxide and vent it out through the suction
27 (D)  Increasing fearful patient’s control decreases fear ­system.
by allowing patient to determine comfort level. Fear- 36 (A)  MAIN site of action for nitrous oxide is the cen-
ful dental patients require more than quick appoint- tral nervous system, causing sedation (depression of
ments and limited information about procedures and system).
pain control methods. 37 (C)  Takes 30 to 60 seconds for patient to feel changes
28 (A)  Minimum flow for oxygen (NOT for nitrous to the level of flow of the gases when nitrous oxide is
oxide) is 2 Lpm. This fail-safe component ensures administered. Patience is IMPORTANT when deal-
that patient does NOT inhale nitrous oxide without ing with changes made to level until patient feels the
oxygen. Color-coding, noninterchangeable tanks, and difference, especially if any less serious side effects
scavenger systems are ALL fail-safe components. such as nausea are encountered.
29 (B)  Main goal of nitrous oxide sedation is stress and 38 (B)  Using the oxygen flush button to remove mix-
anxiety reduction; clinician does NOT want to put ture of gases from bag is a COMMON mistake when
the patient to sleep but to make patient feel drowsy. using nitrous oxide in dentistry; oxygen flush button
Patient’s ability to open mouth does allow clinician is used for giving the patient the first breaths. Oth-
to know that patient is NOT oversedated. May NOT ers help to reduce complications such as nausea and
want to use mouth blocks for this reason. vomiting that can occur with NOT individualizing
30 (C)  MAIN safety factor for patient with nitrous amount, NOT reducing amount near end of appoint-
­oxide is use of titration technique, current standard of ment (prevents oversedation), and NOT allowing pa-
care. Nitrous oxide is added to oxygen flow in small, tient to let staff know about discomfort.
­incremental amounts. Limits amount of drug that is 39 (B)  Lessened pain reaction is the MOST important
required, allows variety and the safe performance of sign present during general dental treatment levels of
a prolonged procedure. Oxygen flush button allows sedation with nitrous oxide. Patient should NEVER
patient to have air to breathe at first part of adminis- be asleep (could go into deeper stages of sedation),
tration. Infection control, possibly with use of dispos- and with good sedation there is less movement of
able masks, does add to safety by reducing possibility arms and legs and speech is infrequent and slow.
of transfer of infectious material, but it is NOT as im- 40 (A)  Patient must NOT mouth breathe, since this con-
portant as reducing toxicity in patient with titration. taminates the operatory, reduces sedation, and gives
31 (A)  Mild anxiety is an indication for nitrous oxide uneven levels of sedation, which can lead to nausea
sedation. Patients’ emotional instability and severe and then vomiting. Staff must help patient choose the
intellectual disability (mental retardation) are relative mask for a good, comfortable fit. They must NEVER
contraindications for its use; depends on individual liken sedation to alcohol or it may NOT be desired
case as well as medical consult if desired. ­Absolute by the patient. Legs should be uncrossed to allow
Pain Management   501

c­ irculation and even blood pressure, and patient 45 (B)  Fail-safe system does NOT permit the flow of ni-
should be supine to prevent syncope. trous oxide if the flow of oxygen is interrupted. Thus
41 (D)  Nitrous oxide is sweet smelling with NO color, if the unit has been adjusted to allow ONLY nitrous
is NOT explosive, and is inorganic (metal) (this can oxide, the persons involved may be abusing it. Oth-
present a problem when discussing issue with patients ers are NOT correct; a patient should NEVER be left
who love the green, organic way of living; NOT the when gas is being administered, NOT all children are
same meaning as sprouts and such). candidates, and the highly anxious are also NOT can-
42 (C)  Reservoir bag while using nitrous oxide in the didates.
dental office should remain partially inflated and then 46 (C)  Action potentials travel down an axon toward
changes with each breath. This means the patient has the synapse. Action potentials relative to graded po-
the CORRECT tidal volume (TV, minimum flow rate) tentials are large in amplitude, travel, do NOT dec-
for breathing comfortably and the seal of the system rement, and are caused by a relatively large flux of
is tight. TV is set with oxygen before nitrous oxide is positive ions such as Na+ or Ca+2. An action potential
titrated in; this is a standard of care. An overinflated is a “spike” of positive and negative ionic discharge
bag so that patient complains of breathing against the that travels along the membrane of a cell.
rapid flow of air means that TV is too high; if bag is 47 (D)  Myelinated fiber is large, is insulated by myelin,
deflated and patient feels air blowing in eyes, mask and allows saltatory conduction. Type and size of
should be checked for leaks, which may or may NOT fiber both result in an increased conduction veloc-
be corrected by a 2 × 2 gauze square, the mask should ity. Myelin is an electrically insulating phospholipid
be adjusted, or a smaller mask should be provided. layer that surrounds the axons of many neurons. It
However, if patient complains of NOT receiving is an outgrowth of glial cells; Schwann cells supply
enough air with deflated bag, scavenging unit may myelin for peripheral neurons, whereas oligodendro-
need to be adjusted, TV may need to be increased, or cytes supply it to central nervous system.
hoses may have become kinked. 48 (C)  At the base of the tongue, lingual nerve ascends
43 (A)  Hands over stomach indicate that the patient is and runs between the medial pterygoid muscle and
experiencing nausea. Pallor, NOT redness, would mandible, anterior and slightly medial to the inferior
indicate a serious complication, and there is also a alveolar nerve. Thus lingual nerve is anesthetized
change in the patient’s blood pressure, heart rate, and FIRST by diffusion during inferior alveolar (IA) an-
respiration. Clinician needs to abort the procedure esthetic nerve block. Others ALL innervate structures
with gas turned off and oxygen placed at 100% and of maxilla.
provide BLS and institute EMS system. 49 (B)  If patient shivers, cover with a blanket. Have pa-
44 (A)  Patient should use restroom before procedure, tient use restroom before, since voiding can bring a
since this can give a feeling of warmth. Blood pres- feeling of warmth.
sure, NOT just pulse, should be taken beforehand, 50 (C)  Must NOT allow patients to drive until they are
and patient should take out contact lenses, since air completely removed from the effects of sedation,
from mask can be drying to eyes. usually after 15 minutes or more.
C hapter 15

Dental Biomaterials
CHARACTERISTICS OF DENTAL MATERIALS   a. Ultimate strength: highest stress reached before
Understanding of mechanical, physical, electrical, sur- fracture.
face, biological properties of dental materials enables b. Fracture strength: point at which a material
clinician to determine proper use and care of dental ma- breaks.
terials. c. Tensile strength: breaking strength when tested
• See CD-ROM for Chapter Terms and weblinks. in tension.
d. Compression strength: breaking strength when
Mechanical Properties tested in compression.
Mechanical properties include reactions of materials to C. Strain: deformation; change in length or dimension
application of external forces, such as the magnitude of of a dental material because of applied stress.
biting forces. 1. Deformation or change in length (inches per ­meter)/
A. Forces: Original length (inches per meter). ­ Example:
1. Tensile force: tends to pull object apart or elongate ­dental waxes exhibit strain much MORE quickly
object. than amalgam because of differences in stiffness.
2. Compressive force: downward application on ob- 2. The MORE stiff an object, the greater its ability to
ject, compressing (pushing or crushing) structure resist dimensional change.
(i.e., biting by teeth). 3. Strain-related properties:
3. Shear force: rotation, twisting, or sliding of one a. Elastic deformation: when material recovers
portion of material by another portion. after load is released or after material breaks.
4. Bending combination of ALL types of force. b. Plastic deformation: permanent strain or
B. Stress: reaction with which object resists external change in shape of object that results when
force (Figure 15-1). Example: stretching of orthodon- stressed beyond elastic limit.
tic band causes stress because wire is being pulled in c. Elongation: percentage of change in length up
tension; can resist this external force up to point of to point of fracture.
fracture. d. Ductility: ability of a material to withstand sig-
1. Stress = Force kg per mom/Area inches/meters. nificant plastic or permanent deformation under
2. Thus the smaller the area over which a force is ap- stress before fracturing (e.g., gold), opposite of
plied, the larger the value of the stress. brittleness.
3. Stress-related properties: e. Brittleness: ability of material to fracture be-
fore undergoing significant amount of plastic
or permanent deformation when placed under
stress (e.g., glass fiber), opposite of ductility.
f. Malleability: ability of material to be com-
pressed without fracturing; SIMILAR to ductil-
ity but specifically refers to compressive force
applications.
D. Elasticity: permits material to be deformed by ap-
plied load and then assume original shape when load
is removed (just like Gumby and Pokey). Examples:
orthodontic wires resist fracture upon manipulation
by clinician; adjustment of clasps on partial dentures;
ability of fixed partial denture (dental bridge) to with-
stand forces of mastication.
1. Modulus of elasticity: measure of stiffness of
Tensile Compressive Shear material and ability to resist bending or change in
Figure 15-1  Types of stress. shape.

502
Dental Biomaterials   503

Table 15-1  Hardness of restorative materials

Dental material KHN

Stress Acrylic 20
Dentin 70
Calculus on teeth 86
Enamel 350
Porcelain 400-500
Strain
KHN, Knoop hardness number.

Elongation to fracture

grit having higher hardness than denture, to ­prevent


Figure 15-2  Diagram of the stress/strain curve. excessive abrasion of denture because of its low abra-
sion resistance.
1. Hardness tests:
2. Stress/strain curve: graphically shows change a. Knoop: diamond indenter; ONLY one used in
in dimension for given application and identifies dentistry to produce value (Knoop hardness
important points such as elastic limit, proportional number [KHN]).
limit, and permanent deformation; used in dental b. Vickers: square-based diamond point.
research (Figure 15-2). c. Rockwell: diamond cone and hardened steel
3. Elastic limit (proportional limit): maximum stress ball indenter.
that material can withstand without being perma- d. Brinell: steel ball (oldest test).
nently deformed. 2. The higher the hardness number, the harder the
4. Inelasticity: permanent deformation as result of material. Example: porcelain has high resistance to
applied force. abrasion with high hardness number; conversely, it
E. Hardness: resistance of a solid to penetration. Exam- has ability to abrade opposing materials with lower
ple: clinician must instruct denture patients NOT to hardness numbers such as tooth enamel and amal-
use abrasive cleansing materials, such as a paste with gams (Table 15-1).

clInical study  

Age 59 YRS SCENARIO

Sex ☐  Male   ☒  Female The patient is in the dental office because of


a fractured tooth. Visual examination of tooth
Chief Complaint “I bit down on a popcorn kernel and my #18 reveals a fracture of the ML cusp. The
tooth broke.” ­remainder of the tooth holds a large MOD
Medical History Pancreatitis last year (­mesio-occlusal-distal) amalgam restoration.
Past alcoholic, sober for 15 years After the dentist removes all of the old
­amalgam, some caries is discovered that will
Current Medications None result in a near pulpal exposure. Patient also
has amalgam restorations in teeth #2, #14,
Social History Animal groomer #19, and #31, and full gold crowns on teeth
Likes gardening
#3 and #30.

1. What mechanical properties were involved in the frac- 1. Stress from biting on the popcorn kernel created both
ture of tooth #18? compressive and tensile forces on the tooth; smaller
2. The dentist decides to place a cavity liner before the area over which a force is applied, the larger
­restoring this tooth. Explain what material would be the value of stress. In this instance, occlusal surface
appropriate and why. of #18, from contact with the kernel, magnified the
3. How should this tooth be restored? Select an appro- stress. Second property involved was strain (reaction
priate restorative material, and include indications and of tooth #18 to biting stress), which resulted in frac-
contraindications for different restorative materials. ture of ML cusp.
504   Saunders Review of Dental Hygiene

2. Calcium hydroxide is appropriate material for lining 1. Enamel and dentin are poor thermal conductors,
the cavity. Advantages of material: (1) encourages re- compared with gold alloys and amalgam.
covery of the pulp by stimulation of secondary and 2. Cements overlie and insulate inner pulp in areas of
reparative dentin; (2) protects pulp; (3) sufficiently deep cavity preparations covered by metal restora-
strong. Mineral trioxide aggregate (MTA) may also be tions.
effective for lining the cavity. C. Electrical properties: generation of electrical currents
3. Tooth #18 will probably require full-coverage res- through a variety of means.
toration because of extensive loss of tooth structure. 1. Galvanism: small electrical currents that result
Because the tooth that opposes #18 has never been re- from opposing of dissimilar metals across dental
stored, full gold crown is indicated; gold would cause arches (Figure 15-3).
minimal abrasion of #15. Porcelain would be contra- a. Have potential to create tooth hypersensitivity
indicated in this circumstance because has high hard- by irritating the pulp.
ness number (KHN 400-500) compared with enamel b. Roughness and pitting can also occur as result
(KHN 350) and would therefore result in the eventual of galvanic action.
abrasion of tooth #15. 2. Corrosion: deterioration of metal by chemical or
electrochemical reaction.
Physical Properties a. Can be result of adjacent restorations that are
Physical properties depend primarily on the type of atoms composed of dissimilar metals.
and bonding that is present in a material. Allows clini- b. Also may result from chemical attack of metals
cian to understand how given material will withstand oral by components in saliva and food; considered
environment (i.e., temperature fluctuations, biting forces, tarnish.
moisture). D. Color properties: give restorations and prosthetic ap-
A. Thermal properties: reaction to temperature changes pliances appearance of natural teeth and soft tissues
within oral cavity and subsequent expansion and con- (esthetics); important in selecting tooth-colored resto-
traction. Example: rapid change in oral temperature rations and calibrating whitening procedures (digital
would occur when eating ice cream cone immediately machines available).
before drinking cup of hot coffee and affect restora- 1. Hue: dominant color of object (i.e., red, yellow,
tions by expansion. blue).
1. Coefficient of thermal expansion: value or mea- 2. Value: lightness or brightness of color; amount of
surement of change in length per unit length for gray present; MOST important color property.
each degree of temperature change (change in vol- 3. Chroma: intensity of color.
ume in relationship to change in temperature). E. Translucency: how light enters the tooth; affected in
a. The higher the coefficient of expansion, the several ways:
greater the degree of contraction and expansion 1. Part of the light may be transmitted completely
involved when material is exposed to tempera- through the tooth (the MORE light that is transmit-
ture changes. ted through, the MORE translucent the material).
b. Because of differences in expansion and 2. Part of the light may be reflected from the surface
contraction rates of restorative materials and of the tooth and thus may NOT penetrate at all.
tooth structures, microleakage of oral fluids
between the restoration and tooth can occur
(Table 15-2).
B. Thermal conductivity: ability of dental material to
transmit heat.
Aluminum alloy
Table 15-2  Expansion of tooth and dental materials: crown
linear thermal coefficients of expansion
Saliva

Material Coefficient (×10-6/°C) Gold alloy


crown
Tooth 11.4
Amalgam 25.0
Acrylic resin 81.0
Composite resin 35.0
Silicone impression ­material 210.0
Figure 15-3  Galvanism.
Dental Biomaterials   505

3. Part of the light may enter the tooth and then scat- a. MORE tenacious bond than mechanical
ter and be absorbed (refraction). ­bonding.
a. Composites and ceramics (glass) can be made (1) Primary bond: chemical bond between
with varying degrees of translucency by add- atoms and molecules; example: dyeing
ing opaquing agents to block light penetration; cloth.
common problem with older anterior restora- (2) Secondary bond (MOST common): van
tions, especially when photographed. der Waals forces, or physical attraction be-
b. Anterior teeth can be very translucent at the in- tween atoms and molecules that does NOT
cisal edge; extreme or excessive whitening pro- provide as strong a bond as chemical bond-
cedures can actually increase this, creating the ing. Example: materials with polyacrylic
appearance of gray teeth. acid, such as glass ionomer cements and
polycarboxylate cements.
Surface Properties b. Adhesion helps control the problem of micro-
Surface properties are associated with surface of dental leakage around dental restorations.
materials and include surface tension and absorbability. c. Effectiveness of adhesion is determined as fol-
A. Solids: MOST materials in dentistry. lows (Figure 15-4):
1. Have regular arrangement of atoms and molecules; (1) Wetting: degree to which an adhesive will
crystalline or rigid at all temperatures below its spread out on a surface.
melting point. (2) Contact angle: angle between the adhesive
2. In contrast, amorphous structures have NO regu- and surface that is measured to give a value
larity or form; they possess many characteristics of for wetting; the larger the contact area, the
solids but are noncrystalline; those such as glass do BETTER the chance of adhesion.
NOT possess a definite melting or freezing point. (3) Surface energy: the higher the surface en-
­Example: dental waxes are amorphous and gradu- ergy, the greater the attraction of atoms
ally soften without definite melting point. to the surface, resulting in better adhesion
B. Liquids: some dental materials are liquids at some (e.g., metals have high surface energy).
point and then change to solids by cooling or chemi- d. Inhibition or failure of adhesion is affected by:
cal reactions. (1) Dirty or contaminated surfaces: result in
1. Irregular arrangement of atoms and molecules. reduced surface energy of adherent, thus
2. Includes reversible hydrocolloid materials heated reducing ability to attract other atoms and
to liquefaction. molecules.
C. Attaching solid structures to each other: by mechani- (2) Viscosity: adhesives with high viscosity
cal bonding, cohesion, adhesion. have high resistance to flow, are less likely
1. Mechanical bonding:
a. Does NOT require intimate attraction between
atoms and molecules of the substances in-
volved.
b. Examples:
(1) Dental cements with crown: liquid ­cement
penetrates into irregularities on ­internal sur-
face of crown and into porosities in tooth High contact angle
structure, thus creating locking ­mechanism.
(2) Acid etch technique for most enamel seal-
ants and resin materials: minute pores are
produced in enamel and dentin when acid is
applied, allowing spaces for interlocking of
materials.
2. Cohesion: force of attraction between like kinds Moderate contact angle
of atoms and molecules within material, resulting
in tenacious bond. Example: cohesive forces that
hold cement together and prevent fracture within
cement.
3. Adhesion: force of attraction between unlike at- Low contact angle
oms and molecules on two different surfaces when Figure 15-4  Wetting ability: the lower the contact angle,
brought into contact. the greater the adhesion.
506   Saunders Review of Dental Hygiene

to spread out on a surface; if viscosity is too A. Classification of impression materials:


low, though, may be hard to control place- 1. Inelastic materials: rigid impression materials are re-
ment of the adhesive. stricted to applications in which NO undercuts exist.
(3) Shrinkage of adhesives during hardening: a. Compound (modeling plastic): tray compound
causes adhesive to pull away from tooth/ and impression compound.
restoration, thus compromising adhesion. b. Zinc oxide–eugenol (ZOE) impression paste:
LESS use, replaced by rubber impression ma-
Biological Properties terials.
Biological properties refer to the biological response of c. Impression plaster: LESS use because of rigid-
the human body to various materials and the continued ity and ease of fracture; used primarily to mount
examination of the host–foreign body response. Response casts on articulator.
of both patients and clinicians can involve allergy, micro- 2. Elastomeric materials: aqueous elastomeric ma-
leakage, and toxicity factors. terials (hydrocolloid): largely replaced by BOTH
• See Chapter 8, Microbiology and Immunology: allergic alginate and rubber base impression materials:
reactions, including rubber latex allergy. a. Hydrocolloid is a suspension of fine particles
A. Allergic response (immediate or delayed contact al- dispersed in water to produce a viscous solution.
lergic reaction type): b. Irreversible hydrocolloid, such as alginate, and
1. Elicited in patients who are sensitized to nickel- reversible hydrocolloid, such as agar-agar.
containing materials; occurs infrequently. 3. Nonaqueous elastomeric materials (rubber):
2. Possibly also rubber latex allergy for both patient a. Polysulfide (mercaptan rubber).
and clinician from rubber latex rubber dam and b. Silicone rubber; BOTH condensation and addi-
gloves; may be associated with placement of restor- tion (polyvinylsiloxane) types.
ative materials; other restorative, rubber dam (sili- c. Polyether.
cone), and glove (vinyl) materials are available. B. Composition and chemistry:
3. Higher percentage of women are sensitive or al- 1. Compound (modeling plastic): 40% resin, 7% wax,
lergic to nickel and/or rubber latex. 3% organic acid, 50% filler, coloring agent.
B. Microleakage: a. Thermoplastic (material will reversibly soften
1. Inability of materials to adequately seal margins on heating and harden on cooling).
by adhesion, allows leakage of saliva, dental bio- b. Tray compound is in shape of a tray; impression
film (dental plaque), and by-products into the area, compound is in form of sticks and cones.
which can lead to secondary caries, pulpitis, pain, 2. ZOE impression paste: supplied as two-paste
necrosis. system: zinc oxide, eugenol, oils (such as oil of
2. LESS potential with polyacrylic acid, currently cloves), resin, additives.
ONLY material able to adhere sufficiently to avoid 3. Irreversible hydrocolloid (alginate) (Table 15-3).
microleakage.
C. Toxic effects of materials: can occur during and after
restoration of carious tooth, may subject pulp to injury.
Table 15-3  Composition of irreversible hydrocolloid
1. Includes microleakage, thermal shock, galvanism,
alginate impression material
chemical irritation.
2. Can be minimized or eliminated by taking proper
Ingredient Percentage (%) Function
steps (e.g., using insulating bases, tooth isolation
with rubber dam). Diatomaceous 60 Filler
earth
IMPRESSION MATERIALS  Calcium sulfate 16 Reacts with potas-
Function of impression and replication materials is to sium alginate to
create gelation
accurately record hard and soft tissues in the oral cavity.
Impressions produce negative reproduction, which then Potassium 15 Reacts with calcium
­alginate sulfate to create
can be used by clinician for construction of restorations gelation
to replace missing tooth structure and for fabrication of
Zinc oxide 4 Filler
preventive devices. Vary in accuracy and ease of use,
especially in removal from the mouth, such as around Potassium 3 Accelerates setting
tooth undercuts, portion of tooth that lies between its ­titanium of gypsum
­fluoride
height of contour and attached gingiva. Replication
­materials to preserve the impressions are discussed in Trisodium 2 Retardant
­phosphate
next section.
Dental Biomaterials   507

Table 15-4  Composition of reversible hydrocolloid d. Accurate; has low dimensional change during
setting of ∼0.1%.
Ingredient Percentage (%) Function e. Adheres to tray compound or acrylic tray mate-
Agar-agar 8-15 Substance is extracted
rial, eliminating need for tray adhesive.
from a type of sea- 3. Irreversible hydrocolloid (alginate):
weed whose fibrils a. Suspension of molecules (colloid) in some type
form a colloid that of dispersion medium (water); a liquid colloid
results in a partially is a sol.
rigid but elastic gel
(1) Liquid sol is changed into a gel by chemical
Water 80-85 Main component that means, which is an irreversible process.
occupies the spaces
(2) Normal-set alginate sets in not less than
between the agar-
agar fibrils 2 minutes, not more than 4½ minutes (mix-
ing time, 1 minute).
Borax Trace amounts Increases the strength
of the gel (also can (3) Fast-set alginate sets in 1 to 2 minutes (mix-
retard the setting ing time, 30 to 45 seconds).
of gypsum; requires (4) Water/powder (W/P) ratio affects setting
the addition of an time; thinner mixes increase time required
accelerator) for material to set; increased powder de-
Potassium 2 Accelerates the creases setting time.
sulfate ­setting of gypsum b. Has 1.5% permanent deformation.
(1) Increased strength if thick rather than thin
mixes are used.
4. Reversible hydrocolloid (agar) (Table 15-4). (2) Results in increased tear and compressive
5. Polysulfide: supplied as two-paste system: strength if impression is left in mouth until
a. Base: 80% organic polymer-containing reactive fully set.
mercaptan groups and 20% reinforcing agents. (3) Less accurate impression material than
b. Catalyst: MOST often is lead dioxide. agar/rubber impression materials.
6. Silicone rubber: (4) Loses accuracy (dimensional change) if
a. Condensation type: supplied as base and accel- stored for MORE than 10 minutes in air be-
erator; available in light, regular, heavy-bodied cause of syneresis (loss of water by evapo-
consistency and as putty: ration and by exuding fluid); wrap in moist
(1)   Base: silicone liquid, dimethylsiloxane. paper towel or keep in 100% humidity and
(2)  Accelerator: tin organic ester suspension pour as soon as removed.
and alkyl silicate. (5) If stored in contact with water, will absorb
b. Addition: type (polyvinylsiloxane): supplied as water (imbibition), leading to dimensional
two-paste or putty system, with silicone and si- change.
lane hydrogens. c. Procedure for use:
7. Polyether: supplied as BOTH base and catalyst; (1) Mixing:
base is polyether and catalyst is sulfonic acid ester. (a) Fluff alginate powder container or pack-
C. Characteristics, properties, manipulation of each: age; measure appropriate amount of cool
1. Compound: water for required number of scoops of
a. Flow is 85% at 113° F and 6% at mouth tem- alginate; place water in plastic bowl.
perature (98.6° F); hardens in mouth. (b) Add alginate powder to water in bowl,
b. Tray compound is stiffer and has less flow than thus helping to eliminate entrapped air
impression compound. in final mix.
c. Thermoplastic; does NOT involve chemical (c) Stir powder and water vigorously to wet
change. powder completely, strop (wipe) mix
d. LOW thermal conductivity. against side of the bowl for 60 seconds
2. ZOE: to homogenize and remove bubbles.
a. Soft-set ZOE is tougher than and NOT as brittle (d) Visually inspect mixture for a creamy,
as hard-set ZOE. thick consistency.
b. BOTH types are classified as rigid; CANNOT (2) Filling and seating the tray:
be used to record undercut areas. (a) Spray tray with adhesive, then fill with
c. Setting time that is shortened by presence of mixture by spatula, from posterior re-
water, high humidity, high temperatures. gions forward.
508   Saunders Review of Dental Hygiene

(b) Smooth alginate surface with moistened (4)  Causes less tissue reaction than conden-
finger; precoat the occlusal and other sation silicones.
anatomical areas (e.g., vaulted palate, (5)  Very hydrophobic material, can cause prob-
frenums) for BETTER impression. lems with trapped air bubbles on the surface
(c) Seat tray in a posterior to anterior direc- when pouring gypsum casts or dies; surfac-
tion. tants added to make more hydrophilic.
(d) Border mold by pulling lips and cheeks 7. Polyether:
over tray and hold tray in place until al- a. LESS working time than any of the rubber im-
ginate sets. pression materials.
(3) Removing and pouring impression: b. Permanent deformation greater than that of
(a) Lift lips and cheeks away with fingers polysulfides, but NOT as low as that of sili-
to break seal. cones.
(b) Grasp handle and pull tray away from c. High stiffness and low flexibility, thus can
teeth with quick (thrust or snap) motion cause problems during removal from mouth;
for less distortion. trays must be made short of tissue undercuts,
(c) Wash impression under cool water to NOT for removable appliances.
eliminate saliva and blood. d. Lower dimension change than any other rubber
(d) Spray impression with disinfectant and material, EXCEPT for polyvinylsiloxane.
seal in plastic bag for 10 minutes. e. Can cause tissue irritation because of aromatic
(e) Rinse thoroughly and pour impression sulfonic acid ester catalyst.
as soon as possible or store in 100% hu- D. Clinical uses:
midity, 15 minutes maximum. 1. Compound:
4. Reversible hydrocolloid (agar): sol is changed into a. Tray compound: primary impression for den-
gel by physical (cooling) reaction and therefore tures; another impression material, such as rub-
is reversible; gives highly accurate impression; ber base, put into tray to make secondary or
MUST be poured immediately to avoid dimen- corrective impression.
sional changes. b. Impression compounds: border molding for
5. Polysulfide: provides MOST reproduction of sur- denture impressions.
face detail. 2. ZOE impression paste: cementing media; also is
a. Demonstrates shrinkage of impression material used for surgical dressings (after periodontal sur-
during first 24 hours; therefore models and dies gery), temporary restorations, root canal filling
SHOULD be poured within 1 hour. material, bite registration paste, impression mate-
b. Highly compatible with model plaster and stone; rial for dentures.
long shelf-life, improved by refrigeration. 3. Irreversible hydrocolloid:
6. Silicone rubber: a. Diagnostic (study) cast for educational pur-
a. Condensation type: poses, fabrication of fluoride trays and mouth
(1)  LESS working and setting time than poly- protectors, orthodontic appliance construction.
sulfides. b. Preliminary cast for development of com-
(2)  LESS viscosity than polysulfides, making plete and partial dentures (to make trays and
them easier to mix. ­diagnoses).
(3)  Greater dimension change than polysul- 4. Reversible hydrocolloid: SAME uses as irrevers-
fides; reproduces fine details. ible hydrocolloid but produces MORE accurate
(4)  Compatible with model plaster and stone. detail.
(5)   Known to cause allergic reactions. 5. Polysulfide:
b. Addition (polyvinylsiloxane) type: a. Inlay and crown impressions.
(1)  Low dimension change of 0.1%; does NOT b. Development of fixed partial dentures
require immediate pouring, can be delayed (bridge) and secondary (final) impressions for
for up to 7 days. dentures.
(2)  Lowest level of permanent deformation of 6. Silicone:
ALL elastomeric impression materials on a. Polyvinylsiloxane (addition silicone): same
removal from mouth. uses as polysulfide.
(3)  MORE stiff than polysulfides and conden­ b. Condensation silicone: used for inlay and
sation silicones; makes removal from crown impressions as well as fixed partial
under­cuts MORE difficult; NOT as stiff as dentures.
polyethers. 7. Polyether: SAME uses as condensation silicone.
Dental Biomaterials   509

Table 15-5  Water/powder (W/P) ratios of gypsum a. Accelerator is potassium sulfate (salt solu-
materials* tion); reduces setting time from ∼10 minutes to
4 minutes.
Gypsum Water (mL/100 g powder) b. Retarder is borax; increases setting time (blood,
Model plaster (type II) 37-50 saliva, agar, alginate also retard setting).
c. Impressions thoroughly rinsed and disinfected
Dental stone (type III) 28-32
using infection control protocol (remove traces
High-strength dental 19-24 of blood, saliva).
stone (type IV-V)
3. Temperature and humidity:
*Recommended W/P ratios vary among products and manufacturers. a. Higher water temperatures (during mixing) de-
crease setting times.
b. High humidity results in decreased setting time.
Replication Materials D. Types and uses of gypsum:
Impression plaster and stone are examples of replication 1. Impression gypsum (type I): mount casts on articu-
materials that are used to produce positive reproduction lators or as interocclusal record.
of impressions. Standard precautions for lab discussed 2. Model plaster (type II): weakest; lowest com-
later. pressive strength because of excess water, higher
A. Composition: gypsum is calcium sulfate hemihydrate W/P ratio; NOT used intraorally; makes study
(CaSO4 • 2 H2O). casts (models) for documentation:
B. Types: a. To permit clinicians to examine conditions in
1. Model plaster (type II): heating gypsum under at- the mouth from all views; allows study of rela-
mospheric pressure results in a porous, irregularly tionships between adjacent and opposing teeth
shaped material. and permits dentist to examine, measure, and
2. Dental stone (type III): heating gypsum under steam analyze without discomfort to the patient.
pressure results in hemihydrate particles that are less b. Beginning of treatment and progress of in­
porous and MORE uniform in shape. volved or long-term treatment; for drawing,
3. High-strength dental (die) stone (type IV [low ex- com­pleting diagnostic waxing, or performing
pansion], V [high expansion]): dehydrating gyp- proposed treatment on models.
sum in 30% solution of calcium chloride results in c. To illustrate existing conditions to the patient
LEAST porous hemihydrate particles. during case presentation.
C. Characteristics and properties: d. To examine occlusal relationships and chart
1. Hemihydrate: physical differences in hemihydrate existing factors such as wear facets, open con-
particles determine manipulative properties for tacts, rotated teeth, and tissue recession.
mixing particles and use of each. e. To demonstrate individualized homecare pro­
a. Mixing (Table 15-5): cedures; allows practice of techniques on
(1)  Amount of water needed depends on size, ­models.
shape, porosity of the particles. f. Forensic cases.
(2)  Porous, irregularly shaped hemihydrate 3. Dental stone (type III):
crystals require MORE water to facilitate a. Used to make casts of impressions for fabrica-
wetting and mixing. tion of dentures and to produce study casts.
(3)  High-strength stone requires LEAST b. Harder, stronger, MORE durable than model plas­
amount of water. ter (type II); greater strength result of lower W/P
(4)  Too much water prolongs setting time and ratio.
creates weaker cast or die. 4. High-strength dental (die) stone (type IV [low ex-
(5)  Too little water results in increased expansion. pansion] or V [high expansion]):
b. Setting: a. Type IV: used as die material on which wax pat-
(1)  When mixed with water, forms hard sub- terns of inlays, crowns, and other castings are
stance. produced.
(2)  Produces heat (exothermic reaction) through (1) Four times greater compression strength
chemical reaction of hemihydrate with water. than model plaster (type II) because excess
(3)  Allow 45 to 60 minutes to set before re- water has been minimized.
moving impression from the cast or die. (2) Lowest w/p ratio. o10110
2. Accelerators and retarders: more practical method b. Type V: used as die material with higher expan-
of controlling setting time than varying W/P ratio sion to compensate for greater shrinkage that
(added by manufacturer). occurs in higher-melting alloys.
510   Saunders Review of Dental Hygiene

E. Procedure for use:


1. Dispensing: weigh powder and vacuum mix; wa- Temporary Restoratives
ter should be measured in graduated cylinder to Many procedures cannot be completed in one appointment
get consistent results; dimensional accuracy in- because of need for lab work or because of complications
creased when gypsum is weighed and water is of infections. In these instances, temporary (provisional)
measured. restoration that is placed over preparation is indicated.
2. Mixing: performed in a rubber bowl, using a stiff Materials such as acrylics, stainless steel, composites, or
metal spatula, with room-temperature water. cements are used in preparation to maintain relationship
a. Place water in bowl first and then add powder, in occlusion until permanent restoration can be placed.
sift into water slowly. A. Acrylics with compositions similar to denture bases.
b. Spatulate vigorously for 30 to 60 seconds to B. Stainless steel or aluminum:
break up clumps and enhance wetting of pow- 1. Stiff enough to bear occlusal forces when cemented
der, strop (wipe) against side of the bowl. to prepared tooth with temporary cement.
c. Vibrate to remove remaining air bubbles; pres- 2. Easy to cut and adjust; has poor esthetic qualities.
ence of air bubbles results in porosities in cast, C. Composite-type material:
reducing strength. 1. Of intermediate rigidity, somewhat flexible after
d. When complete, mix looks very glossy on the light curing.
surface and has smooth, creamy texture (if sandy, 2. Flexibility allows easy cutting and removal when
grainy, or watery, incorrect W/P ration has been permanent restoration is ready for insertion.
used and mixing is incomplete); can be BEST D. ZOE (type III) (Fynal):
accomplished using vacuum mixer technique. 1. Designed for use as temporary filling when mixed
3. Pouring: to base or puttylike consistency.
a. Surface of the impression is dried, excess water 2. Protects the pulp; reduces pulpal inflammation.
removed.
b. Plaster or stone is dripped into one corner of the Permanent Restorations
impression and gently vibrated to slowly move Permanent restorations are composed of dental materi-
the mix around the arch and express any air to als whose properties allow them to function in the oral
coat impression. cavity for a greater length of service than that of tem-
c. Second increment is added and vibrated; exces- porary restorations. Include both esthetic and amalgam
sive vibration SHOULD be avoided because it restorations.
creates air bubbles in the material. • See Chapter 11, Clinical Treatment: caries classifica-
d. Process is repeated until entire surface of im- tion and dental charting.
pression is coated and teeth imprints are com-
pletely filled; larger increments can be added to Esthetic Restorations
fill the impression completely. Esthetic (tooth-colored) restorations include ceramics,
e. One-pour technique involves initial mix of porcelains, composites.
plaster or stone that is large enough to fill im- A. Ceramics: compounds formed by union of metallic
pression and form base; two-pour technique in- oxides and minerals.
volves second mix of plaster or stone, used to 1. Include glass, fine crystal, gypsum.
form a base after first pour is set. 2. Glass ceramics are used MAINLY as reinforcing
F. Cast trimming: excess gypsum is removed with lab agents and fillers for dental composites and rou-
knife after loss of gloss. tinely as coatings and veneers to improve esthet-
ics of metallic dental restorations; also in several
RESTORATIONS  dental cements and temporary restorations.
A restoration restores or replaces lost tooth structure, 3. Generally considered very brittle materials; CAN-
teeth, or oral tissues. Includes any filling, inlay, crown, NOT be bent or deformed without cracking or
bridge, partial denture, or complete denture. breaking.
4. Characterized by high melting points and low ther-
DIRECT RESTORATIVE MATERIALS  mal and electrical conductivity.
Direct restorative materials can be placed directly in or 5. Inert, NOT chemically reactive, biocompatible.
on a prepared tooth. Advantageous because of immediate B. Porcelains: specific type of ceramic that is used exten-
results and timesaving benefits. Can be either temporary sively in dentistry; fabricated by dental technician.
or permanent restorations. 1. Chosen because of esthetic qualities; matches ad-
• See Chapter 11, Clinical Treatment: enamel (pit and jacent tooth structure in color, intensity, translu-
­fissure) sealants. cence.
Dental Biomaterials   511

2. Unique ability to transmit light when illuminated; slow chemical activation begins, then ex-
gives natural and vital appearance. posure to curing light results in rapid pho-
3. Contains three main components: quartz, feldspar toactivation.
(75% to 80%), kaolin clay (aluminum silicate). 4. Polymerization shrinkage: contraction that ac-
4. High hardness number (Mohs 6 to 7, KHN 400- companies polymerization reaction.
500); may cause rapid abrasion of opposing a. MOST problematic property; even with acid
enamel, which has lower hardness number (Mohs etching of enamel and bonding agents, stresses
5 to 6; KHN 350). from shrinkage can exceed strength of bond be-
5. Used in restorative dentistry to produce denture tween restoration and tooth structure, resulting
teeth, jacket crowns, porcelain-fused-to-metal in marginal leakage.
(PFM) crowns and bridgework, veneers, inlays. b. Two techniques for minimizing shrinkage:
6. Sensitive to acidic preparations, which can cause (1) Insertion and polymerization of composite o10120
pitting and roughening; thus MUST use neutral in layers.
­sodium fluoride therapy, also NOT stannous be- (2) Fabrication of composite inlays. o10130
cause of staining properties; radiolucent. c. Composite can be cured more fully in a lab.
C. Composites: used in ALL classes of restorations, as ve- (1) Produces restoration with superior physical
neers to cover teeth stained by drugs and chemicals such and mechanical properties compared with
as fluoride or tetracyclines, to fill spaces between teeth those cured intraorally.
(diastemas), to enhance contour of misshapen teeth. (2) Extraoral processing eliminates concerns
1. Composed of three major constituents: about shrinkage away from the tooth struc-
a. Organic resin matrix: monomers called dimeth- ture; space between the tooth and inlay
ylacrylates (bis-GMA). is filled by composite cement (after lab
b. Inorganic filler: quartz, glasses, colloidal silica fabrication, inlay is cemented at second
particles. appointment, with dual-cure composite
c. Coupling agent: chemically bonds filler to resin cement).
matrix (silane). 5. Thermal conductivity:
2. Composite materials are classified by particle a. Values closely match those of enamel and dentin.
amount and/or size: b. Provide good thermal insulation for dental pulp.
a. Amount of filler resin: 6. Thermal expansion:
(1) The greater the amount of filler content, the a. The more organic the matrix, the higher the lin-
stronger the restoration. ear coefficient of thermal expansion.
(2) The greater the amount of filler content, the b. Microfilled composites have highest values,
LESS wearing away of the restoration. resulting in greatest dimension changes in re-
b. Size of filler particles: sponse to oral temperature changes.
(1) Smaller size of the filler particles: MORE c. Hybrid composites have the lowest values be-
polishable restoration. cause of high filler content.
(2) Larger size of the filler particles: MORE 7. Water absorption:
difficult to achieve smoothly polished fin- a. Determined by the organic matrix and results in
ish; however, larger filler particles result in discoloration by water-soluble stains.
stronger restoration able to tolerate MORE b. Greatest for microfilled composites because of
abrasion. its high organic matrix content.
3. Polymerization: chemical reaction that converts 8. Radiopacity:
small, individual monomer molecules into long, a. Caused by heavy metal glasses such as those
giant polymer molecules. found in some fine-particle composites and
a. Consists of activated two containers of compos- older hybrid composites.
ite paste: one contains a benzoyl peroxide ini- b. NOT present in newer microfilled composites
tiator, other contains tertiary amine activator. that are filled with silica or in some fine-particle
b. When mixed, amine reacts with benzoyl perox- composites that are filled with quartz.
ide and initiates polymerization. 9. Compression and tensile strength:
(1) Visible light-activated resins: supplied as a. Higher in value when the volume of filler is in-
a single paste that contains both the photo creased.
initiator and amine activator. b. Lower in value in microfilled composites be-
(2) Dual-activated resins: contain ­ activation cause of the percentage of fillers.
chemicals for both chemical and light­ c. Bond strengths between composites and acid-
activated resins; when two pastes are mixed, etched enamel are BETTER (about twice as
512   Saunders Review of Dental Hygiene

great) than the bond strengths between compos- that remains on the dentinal tubules as a re-
ites and acid-etched dentin. sult of the cutting action of a dental drill
10. Enamel and dentin adhesives with composites: (or other means).
a. Enamel adhesives: unfilled resins that are used (a) Object of adhesion is to remove smear
to enhance the adaptation and bond of compos- layer while leaving tubules blocked
ites to etched enamel surfaces. with debris from cavity preparation.
(1) Have BEST flow properties that allow them (b) Removal technique is accomplished
to coat and fill acid-etched enamel (enamel with milder acid (10% vs. 30%) than that
tubules) in a MORE efficient manner than used for enamel etching.
the viscous (thick) composite materials. 11. Basic principles of bonding: provide mechanical
(2) Rely on mechanical interlocking of unfilled retention in cavity preparations where NO enamel
resin into enamel rods for adhesion. is available for etching and retention:
b. Dentin adhesives: necessary for preparations a. Isolate (rubber dam is BEST) surface to be
that extend into the dentin. bonded; maintain a clean surface.
(1) Developed with regard for the following b. Follow manufacturer’s directions carefully and
composition differences between dentin use a fresh bonding agent; close bottle as soon
and enamel: as possible to prevent evaporation.
(a) In etched dentin ONLY 4% of the dentin c. Use a protective liner for deep cavities.
surface near the DEJ may contain tubules; 12. Sensitive to acidic preparations, which can cause
near the pulp about 30% of the surface pitting and roughening; MUST use neutral sodium
area of dentin may contain tubules. fluoride therapy, NOT stannous because of stain-
(b) Enamel is composed almost entirely of ing properties.
inorganic mineral; in contrast, 30% of 13. May need to be replaced because of material loss
weight of dentin is organic. and staining; diet may need to be changed to reduce
(c) Large amount of water exists in dentin staining (eat or drink less chromogenic agents).
tubules; presence of water and organic 14. American Dental Association (ADA) considers
components lowers surface energy of exposure to materials (bis-GMA) to be “an acute
dentin, making bonding MORE difficult. and infrequent event with little relevance to esti-
(2) Attachment to dentin requires removal of mating general population exposures” (companies
smear layer, layer of loosely adhered ­debris state there is no bisphenol-A [BPA] impurity).

clinical study  

Age 47 YRS SCENARIO

Sex ☐  Male   ☒  Female A new patient examination reveals


porcelain veneers on the maxillary
Height 5’9” anterior teeth, several posterior
Weight 177 LBS amalgams that are fractured, and
mandibular anterior composites that
BP 118/69 show signs of microleakage. Patient
states that she is ready for compre-
Chief Complaint “I really have not gone to the dentist a lot lately hensive treatment that will protect
since my husband was unemployed and without her oral investment.
insurance and we kept moving all the time.”

Medical History Breast cancer survivor

Current Medications None

Social History Administrative assistant


Likes scrapbooking

1. What would be the value of fabricating study casts for 3. What properties of composite dental materials lend
this patient? themselves to microleakage?
2. Would finishing and polishing her amalgams be ap- 4. Which topical fluoride would be appropriate for this
propriate care? Explain. patient? Explain.
Dental Biomaterials   513

1. Study casts would allow demonstration of oral condi- a. Lathe-cut: powders composed of small shav-
tions (i.e., fractured amalgams, open margins on com­ ings or filings produced by a cutting lathe; final
posites). In addition, casts could be used to demonstrate powder is blend of various-size particles.
homecare techniques and allow the patient to practice b. Spherical: contain alloy particles in the form
those techniques on the models. of small spheres, result in easier amalgama-
2. Fractured amalgams are a contraindication to finish- tion (combining of mercury and other alloys);
ing and polishing procedures. Appropriate care would accomplished with less mercury than lathe-cut;
involve replacement of the worn amalgams. Polishing do NOT resist condensation forces and there-
of amalgam must be done within the first week after fore require less condensation force and use of
placement in most cases. larger diameter condensers.
3. Polymerization shrinkage and thermal expansion. c. Admixed alloys (mixture of lathe-cut and spher­
Even with acid etching and the use of bonding agents, ical alloys):
stresses from polymerization shrinkage can exceed (1)  Have total copper content that ranges from
the strength of the bond between composite materi- 9% to 20%.
als and tooth structure and fail to prevent marginal (2)  Adapt better to cavity walls and produce
leakage. Thermal expansion of composite materials better contacts with adjacent teeth.
does not match that of tooth structure; therefore a d. Single composition (ONLY high-copper alloys)
differential expansion occurs that results in the mi- contains powder particles of ONLY one compo-
croleakage of fluids between the restoration and the sition (high-copper).
tooth. C. Setting reactions:
4. Neutral sodium fluoride. Acidulated phosphate fluo- 1. Gamma phases:
ride can cause pitting and roughening of porcelain and a. Two main types of amalgam alloy composition
composite materials. Because patient has porcelain are silver-tin alloys with or without significant
veneers on her maxillary anteriors, acidulated phos- amounts of copper; difference between two al-
phate fluoride is contraindicated. Stannous fluoride loys is setting reactions with mercury; reaction
can result in extrinsic staining (orange, brown, or tan) that occurs between mercury and amalgam al-
and thus is also contraindicated. loy is called amalgamation.
(1) Silver-tin phase/gamma (γ) phase: composed
Amalgam Restorations of unreacted alloy particles, strongest phase.
Amalgam is an alloy (combination of two or more met- (2) Silver-mercury phase/gamma 1 (γ1) phase.
als) composed of mercury (ONLY pure metal liquid at (3) Tin-mercury phase/gamma 2 (γ2) phase:
room temperature) mixed with powder of mainly silver weakest component, MORE susceptible to
and tin, with other trace elements. corrosion than either gamma or gamma 1.
A. Has physical properties suitable for dental restor- (a) Low-copper: form weak tin-mercury
atives: phase, resulting in weaker amalgam
1. Silver: increases setting expansion and strength. with inferior properties.
2. Tin: high solubility in mercury, facilitates amalga- (b) High-copper: eliminate tin-mercury phase
mation, decreases setting expansion. by forming copper-tin phase, resulting in
3. Copper: acts much the same as silver; increases stronger, MORE superior restoration.
strength, hardness, setting expansion: 2. Copper content of alloys:
a. Alloys that contain ≤6% copper are low-copper a. Low-copper alloys:
alloys. (1) Silver and tin (Ag3Sn) combination in alloy
b. Alloys that contain >6% copper are called high- particle reacts with mercury during tritura-
copper alloys, MORE superior and perform tion (mixing) in the gamma phase, stron-
BETTER clinically. gest component and phase.
4. Zinc: is often included to minimize oxidation of (2) As mercury becomes saturated with silver
other metals in alloy. and tin, following compounds precipitate
B. Types of amalgam: out: silver-mercury (Ag2Hg3) compound
1. Low-copper: less use because of inferior perfor- (gamma 1) and tin-mercury (Sn8Hg) com-
mance. pound (gamma 2).
2. High-copper: produce alloys with greater strength, b. High-copper alloys:
higher corrosion resistance, less marginal break- (1) Tin has a greater affinity for copper than
down, lower creep; MOST accepted and used for mercury, thus a copper-tin compound is
amalgam alloys; can be made from lathe-cut or formed (eta phase) instead of tin-mercury
spherical particles or may be admixture. compound (gamma 2).
514   Saunders Review of Dental Hygiene

(2) Contains enough copper to suppress forma- h­ ardening; excess mercury left in restoration,
tion of gamma 2 phase; results in superior which leads to reduced strength.
properties. 4. Overmixed (overtriturated): soupy appearance, in-
D. characteristics and properties: ability to hold form, difficult removal from capsule.
1. Dimensional changes: 5. CORRECT mix: shiny appearance, cohesive form,
a. Shrinkage: initially is caused by mercury and easy manipulation for condensation.
alloy particles mixing together: F. Handling:
(1) Results in a slight gap between amalgam and 1. Condensation: adaptation to prepared cavity walls,
tooth; allows some leakage of fluids between matrix, and margins; forms uniform, compact
amalgam and wall of the cavity preparation. mass with minimum voids and reduction of excess
(2) Eventually leads to formation of corrosion mercury content.
products; seals tooth from oral environment. a. Removal of excess mercury during condensation
(3) Results in postoperative sensitivity, often results in LESS creep from chewing forces, stron-
because of fluid movement in unsealed ger and LESS dimensional change during setting.
dentin tubules; gap allows fluids in. b. Longer time lapse between trituration and con-
b. Creep: flow/dimensional change produced in a densation, weaker because of fracturing of crys­
material under a constant stress: tal formation.
(1) Can affect marginal integrity because of c. Builds up amalgam in increments to avoid
chewing forces; creeps or flows into open ­porosities and express excess mercury at each
areas, such as margins, and then fractures. step.
(2) Influenced by gamma 2 because is weak, d. FIRST with smaller-tipped condenser because
easily deformed; high-copper are STRON- greater compacting stress can be generated,
GER because gamma 2 does NOT form. MOST effective in the depth and corners of the
2. Strength and stiffness: preparation; later use of condensers with larger
a. Strong, especially during compression. heads as preparation becomes filled.
b. Stiff because it has relatively high modulus of e. Requires overpacking of restoration; slight ex-
elasticity. cess ensures complete filling.
c. Lower tensile strength; should NOT be placed f. When performed correctly, has a top layer
in thin layers exposed to tensile stress. slightly rich in mercury that is removed during
o3980 3. Studies show that children who receive dental re- carving.
storative treatment with amalgam do NOT score g. Moisture contamination MUST be avoided be-
significantly better or worse on neurobehavioral cause it results in excessive expansion.
and neuropsychological assessments than children 2. Carving: produces CORRECT anatomy; is per-
who receive resin composite material (concerns formed after the amalgam is placed.
about mercury content); however, children who 3. Burnishing: smoothing the surface and margins,
receive restoration with resin may be MORE likely using light pressure; follows carving.
to need additional treatment. However, the FDA 4. Polishing: performed at least 24 hours after place-
has noted that amalgam restorations contain mer- ment; can be performed up to 1 week later; for
cury that may cause health problems in pregnant spherical high-copper alloys may be performed
women, children, fetuses. during same appointment.
E. Mixing: goal is to ensure that mercury and alloy are a. MUST avoid heat generation during finishing
sufficiently mixed to allow the chemical reaction to and polishing to prevent release of mercury and
proceed, thus producing amalgam that condenses and subsequent reduction in strength.
adapts to cavity preparation with minimal porosity. b. MUST use water spray to avoid overheating
1. Requires mechanical amalgamators to triturate al- and damaging or causing discomfort to the pulp
loy and mercury. (odontoblasts, dentinal tubules).
2. Quality is controlled by: G. Bonding amalgam to tooth: performed with specially
a. Time, which ranges from 6 to 20 seconds; man- developed adhesives that bond the amalgam to the
ufacturer’s instructions should be followed. tooth structure.
b. Speed and force, which can be controlled on 1. Advantages: used for MORE conservative prepa-
variable-speed units (medium setting is MOST rations, fracture resistance, increased strength and
often used); efficiency of unit must be periodi- wear resistant.
cally checked to ensure quality. 2. Disadvantages: MORE technique sensitivity,
3. Undermixed (undertriturated): dull appearance, increased time for placement, increased cost as
grainy texture (difficult to manipulate); quick a result of increased chair time and practitioner
Dental Biomaterials   515

training for new technique, also delayed expan- Table 15-6  Uses of cement, bases, liners,
sion if contaminated by saliva, fracturable by ex- and varnishes
cessive occlusion, dimensionally unstable creep
(breakdown of margin), and high thermal con- Material Product Functions
ductivity. Zinc phosphate Flecks Cement, base
H. Amalgam safety: involves controlling mercury ex­
ZOE (type II) SuperEBA Cement
posure:
1. Mercury vapor: colorless, odorless, tasteless; makes ZOE reinforced Fynal Temporary
(type III) restoration
it a hazard for dental personnel; thus recommended
exposure limit (REL) of 0.05 mg/m3 up to 10-hour Glass ionomer Ketac-Cem Cement, base,
­permanent
workday and 40-hour workweek from National Insti- ­restoration
tute for Occupational Safety and Health (NIOSH).
Polycarboxylate Durelon Cement, base
2. Recommended safety precautions:
o10140 a. Well-ventilated operatories without carpet; col- Calcium hydroxide Dycal Base, liner
lection and storage of amalgam scraps in a well- Copal resin Copalite Varnish
sealed container that is kept cool; monitoring of varnish
actual exposure to mercury vapors.
ZOE, Zine oxide–eugenol.
b. Preportioned capsules of amalgam reduce the
risk of mercury exposure; when cutting amal- 4. Varnishes:
gam, SHOULD use water spray and high-speed a. Seal dentin tubules and prevent migration of
suction (high-volume evacuation); if mercury agents into the tooth resulting from marginal
contacts skin, SHOULD be washed immedi- leakage of a newly placed amalgam.
ately with soap and water. b. Prevent the outward migration of metallic ions
c. Amalgam and mercury waste should be dis- from the amalgam to the tooth (leads to darken-
posed of responsibly in accordance with EPA ing of tooth structure adjacent to the amalgam);
regulations for the area in which the practice is older amalgams are MORE likely to corrode
located. and cause staining than the more modern
amalgams.
Cements, Bases, Liners, and Varnishes c. Use of varnish under amalgams is not being
Dental cements are used for a variety of applications in done; instead varnishes are being used with
dentistry. Depending on how mixed, can serve as luting fluoride as preventive measure on outer portion
agents, base materials, or temporary restorations. Lin- of the dentition.
ers are applied in thin layer to seal dentin, and varnishes B. Types and characteristics of cements, bases, liners,
serve as liners under amalgam restorations to seal den- varnishes:
tinal tubules and prevent migration of metallic ions from 1. Zinc phosphate cement (Flecks):
amalgam to tooth. a. Composition and chemistry:
A. General uses (Table 15-6): (1)  Primarily zinc oxide (57%) and magnesium
1. Cements: oxide (10%) in powder form.
a. Luting agents (adhere one surface to another) (2)  Phosphoric acid buffered by alumina in liq-
for inlays, onlays, crowns, bridges, and other uid form (acidic).
structures. (3)  Water (33%) controls rate at which the
b. Adhere orthodontic bands to teeth. powder and liquid react.
c. Cement pins and posts to teeth for purpose of re- b. Uses:
taining restorations (temporary or permanent). (1)  Type I cement (luting agent for crowns, bridges,
2. Bases: orthodontic bands) mixed thin and creamy.
a. Placed in thick layers to provide thermal insula- (2)  Bases are mixed thick and quickly, with
tion under deep metallic restorations. higher powder-to-liquid ratio to obtain
b. Have therapeutic benefit for the pulp (are soo­th­ maximum strength that will enable material
ing). to withstand forces of condensation; results
3. Liners: in MORE rapid setting time.
a. Seal tooth structures against leakage of irritants c. Manipulation techniques:
present in saliva and restorative materials. (1)  Mixing on a cool slab over a wide area to
b. Provide protective covering to the pulp in deep allow escape of heat (exothermic reaction)
cavity preparations (pulp capping) and with and control setting time, slowing down
pulp exposure. chemical reaction.
516   Saunders Review of Dental Hygiene

(2)  Adding powder in small amounts, with b. Uses:


thorough mixing of each increment; extends (1) Luting agent in pediatric dentistry for ce-
setting time; mixing with as much powder mentation of stainless steel crowns because
as possible to reduce acid irritation to pulp. of gentleness (kindness) to pulp.
(3)  Reducing any moisture on mixing slab; mois- (2) Nonirritating base, liner, or temporary ce-
ture accelerates setting time and has negative ment.
effects on the properties of the cement. c. Advantages: bonds to the tooth (directly to
d. Advantages: longest clinical history; useful enamel), gently to pulp, useful in pediatric
for multiple restorative procedures; low film dentistry because of its gentleness to pulp and
thickness, which facilitates easy insertion of ability to bond to stainless steel crowns, useful
restorative work; inexpensive; easy to use; under composites because does NOT interfere
­radiopaque. with polymerization.
e. Disadvantages: low pH of 3.5; can irritate pulp; d. Disadvantages: LESS working time, limited use
unable to adhere to tooth structure; requires because does NOT adhere to porcelain, resins,
mechanical retention; lacks anticariogenic or gold alloys.
properties. 4. Glass ionomer cement (Ketac-Cem):
2. Zinc oxide–eugenol (ZOE) cements: a. Composition and chemistry:
a. Composition and chemistry: (1) Aluminosilicate glass in finely ground
(1)  Zinc oxide powder and eugenol liquid (oil ­powder.
of cloves), can irritate connective tissue; (2) Polycarboxylate copolymer in water.
pH between 7 and 8; may have allergy to (3) Chemical adhesion to tooth structure (simi-
oil of cloves so allergic reaction can occur. lar to that of polycarboxylate cements).
(2)  Cleanup with water unless set, alcohol or (4) Releases fluoride, gentle (kind) to pulp,
orange solvent on instruments will remove translucent, radiolucent.
set materials. b. Uses:
(3)  Type II (reinforced cements) may contain (1) Final cementation of crowns and bridges
EBA (orthoethoxybenzoic acid and alumina). (type I).
b. Uses: (2) High strength base.
(1)  Type I cements: temporary cementation (3) Class V restoration because of fluoride
(Temp-Bond); tensile strength is weaker release (type II); preferred for geriat-
than that of type II, allows temporary restor- ric patients with root caries in alternative
ative work to be easily removed at second (atraumatic) restorative therapy (ART).
appointment so that permanent restoration c. Manipulation:
can be placed. (1) Mixing time of 30 to 60 seconds, working
(2)  Type II cements (alumina EBA): permanent time of ∼2 minutes after mixing.
cementation (SuperEBA). (2) Must NOT contact water or saliva; mois-
(3)  Type III: temporary fillings and thermal in- ture causes cement to set too fast.
sulating bases (IRM). (3) Once hardened, sensitive to becoming
(4)  Type IV: cavity liners. dried out, which results in surface and
c. Advantages: multiple uses, LEAST irritating of interior cracks; two methods to alleviate
dental materials, with pH of 7.0 are sedative problem:
and palliative to pulp, radiopaque. (a) Coating with varnish immediately after
d. Disadvantages: lack strength, inadequate for use initial hardening has taken place.
under composite restorations because they in- (b) Placing layer of unfilled bonding resin
terfere with polymerization process, difficult to over surface and curing it (MORE per-
remove from tissues and mixing surfaces once manent alternative).
they are set, cause sensitivity in some individu- d. Advantages: strong (strength is comparable
als because of eugenol (alternative: noneugenol to zinc phosphate cements), causes LESS ir-
products, such as Nogenol). ritation than zinc phosphate, contains fluoride,
3. Polycarboxylate cement (Durelon): results in anticariogenic property, easy to mix,
a. Composition and chemistry: adheres to tooth structure.
(1)  Zinc oxide in powder form. e. Disadvantages: high solubility, complete set-
(2)  Polyacrylic acid in water or viscous liq- ting takes about 1 day, thus margins (exposed
uid solution, LESS irritating to dentin than cement) MUST be protected for the first
phosphoric acid in zinc phosphate cement. 24 hours, water causes retardation of the set
Dental Biomaterials   517

(­ requires dry field), requires the use of a base 7. Mineral trioxide aggregate (MTA, ProRoot):
in deep lesions. a. Composition and chemistry: mainly calcium
5. Calcium hydroxide (Dycal): and phosphorus (SIMILAR to dentin), trical-
a. Composition and chemistry: cium aluminate, tricalcium oxide, tricalcium
(1)  Base paste: calcium tungstate, calcium silicate, and others, with bismuth oxide added
phosphate, zinc oxide in glycol salicylate. for radiopacity; biocompatible, providing bio-
(2)  Catalyst paste: calcium hydroxide, zinc logical seal.
oxide, zinc stearate in ethylene toluene sul- b. Uses: medicament for various pulpal proce-
fonamide. dures such as pulp capping with reversible pul-
(3)  Together form amorphous calcium disali­ pitis, apexification, repair of root perforations.
cylate when set; pH that varies from 11 to c. Manipulation: mixed with sterile water to pro-
12; radiopaque. vide grainy, sandy mixture; requires moisture
b. Uses: to set (hydrophilic), making absolute dryness
(1)   Direct and indirect pulp capping. NOT only unnecessary but contraindicated;
(2)  Protective barrier under composite restora- takes 4 hours to completely solidify.
tions (does NOT interfere with polymeriza- d. Advantages:
tion). (1)  Compressive strength SAME as ZOE type
c. Manipulation: II and III cements but LESS than amal-
(1)  Two-paste system, dispensing of equal gam.
lengths of catalyst and base onto a mixing (2)  Induces deposition of cementum and bone
pad; mixed to a uniform color. for healing SIMILAR to calcium hydrox-
(2)  Light-cured cements should be cured by ide; bacteriostatic properties.
visible light for 20 seconds for each 1-mm (3)  Sealing ability BETTER than amalgam and
layer. SAME as or BETTER than ZOE type II ce-
d. Advantages: encourages pulp recovery by stim- ments; LESS cytotoxic than ZOE type II or
ulation of reparative dentin, protects the pulp, III cements.
has sufficient strength (stronger than type IV e. Disadvantages: once mixed, difficult to manip-
ZOE cement). ulate; long setting time.
e. Disadvantages: CANNOT be placed if tooth
surface is wet, proportion is difficult to control Finishing and Polishing Materials
with an instrument. Intent of finishing and polishing procedures is to create
6. Varnish (e.g., Copalite): restorations that fit and maintain occlusal harmony and
a. Composition and chemistry: natural gum, copal to produce smooth surface that results in less irritation to
resin dissolved in organic solvent. intraoral tissues, less dental biofilm and calculus adher-
b. Uses: ence, and decreased potential for the corrosion of metal
(1)  Assists in sealing cavity preparation, restorations. Even smallest amount of excess material,
thereby reducing microleakage around the flash, should be removed.
restoration. • See Chapter 12, Instrumentation: selective polishing of
(2)   Seals dentinal tubules. dentition.
(3)  Serves as protective barrier; reduces possi- A. Abrasive procedures:
bility of postoperative sensitivity. 1. Abrasion: wearing away or removal of material
c. Manipulation: application to cavity walls in by rubbing, cutting, or scraping; abrading done by
thin layer with a brush, wire loop, or cotton ap- abrasive.
plicator. 2. Finishing and polishing: BOTH are wear pro­ces­
d. Advantages: seals cavity, reducing microleak- ses but differ in intent and degree.
age; acts as a barrier to cementing material, re- a. Finishing: FIRST restoration or appliance con-
ducing postoperative sensitivity. toured to remove excess material, produce rea-
e. Disadvantages: NOT required with nonirritat- sonably smooth surface.
ing cements (ZOE and polycarboxylates); NOT b. Polishing: follows finishing; refers to removal
used under composite resins because resin will of materials from restoration or appliance with
soften; NOT used with glass ionomer cement intent of producing smooth, reflective surface
because interferes with bonding to tooth and that does NOT contain scratches; surface should
uptake of fluoride; requires base in deep cavi- resemble natural surface.
ties, since does NOT provide ample protection B. Factors affecting finishing:
because of low film ­thickness. 1. Hardness: abrasive’s ability to cut.
518   Saunders Review of Dental Hygiene

2. Size of abrasive: influences speed of cut: a. Particles are attached to paper discs for use.
a. Larger particles abrade surface MORE rapidly. b. In form of beige-colored discs, used in hand-
b. Particles classified by size in micrometers (μm); pieces to finish gold alloys, acrylics, composites.
coarse: 100 μm, medium: 10 to 100 μm, fine: 0 6. Tin oxide: used as polishing agent for metal-
to 10 μm. lic restorations, especially amalgams and gold in
c. Finishing and polishing involves sequential re- the mouth; produces BEST polishing agent for
duction in size of abrasive particles. enamel.
3. Pressure: 7. Pumice: natural glass rich in silica; polishes acryl-
a. Pressure of the force, when greater, results in ics and enamel.
MORE rapid removal of material; creates in- 8. Rouge: iron oxide powder formed into block or
creased temperature and heat. cake; used on rag wheel in dental lathe or hand-
b. Under higher temperatures can lead to distor- piece to polish gold in the lab.
tion or physical changes within appliance or D. Finishing and polishing composites:
restoration; may cause discomfort for patient 1. Finishing composites: with use of plastic matrix
because of transmission of heat to pulp. strip before polymerization, LESS required to pro-
4. Speed: duce smooth, regularly contoured surface.
a. When faster, results in faster cutting rates and a. Indications: roughness, discoloration, flash or
creates higher temperatures, causing dam- overhang, overfilled.
age to pulp, odontoblasts, and/or dentinal tu- b. Contraindications: open margins, fractured, under­
bules. contoured proximal contacts, large overhangs, re-
b. When faster, creates greater danger of overcut- current caries.
ting the appliance or restoration. c. During initial contouring, water is used to
5. Lubrication: avoid heat buildup and damage to the surface;
a. Used MAINLY for less heat buildup. performed with either a carbide bur (12 flutes)
b. Facilitates movement of the cutting edge into or diamond bur (medium-fine) in high-speed
the surface of the appliance or restoration. handpiece, using light pressure to avoid over-
c. Carries away debris so that the cutting edge contouring.
does NOT become clogged. d. Final finishing can be performed dry to facili-
C. Types and composition of abrasives: tate view of margins, may need further finish-
1. Diamond: composed of carbon: ing with sandpaper discs, using either slow- or
a. Hardest substance; efficient abrasive because high-speed handpiece.
does NOT wear down or lose sharpness as eas- 2. Polishing: use of abrasive sandpaper discs in de-
ily as other abrasives. scending order (coarse to fine); alternative tech-
b. Chips (natural and synthetic) of various sizes nique uses rubber points (containing abrasives) on
are bonded to metal shanks (i.e., to create dia- a slow-speed handpiece.
mond burs). a. Accomplished with aluminum oxide and dia-
2. Carbides: include silicon carbide, boron carbide, mond pastes.
and tungsten carbide: b. Contain particles as small as 1 μm in diameter
a. Silicon and boron for finishing instruments sup- to create smooth, reflective surfaces.
plied as particles pressed with binder into discs E. Finishing and polishing amalgam:
or wheels for use on handpiece. 1. Indications: newly placed, irregular margins such
b. Attached to steel shanks as burs or stones (i.e., as flash, creep, and minor ditching, overcontoured,
green stones). poorly defined anatomy, with surface irregularities
3. Aluminum oxide: produced as particles bonded to such as pits, scratches, tarnish, and corrosion.
paper discs and strips or impregnated into rubber 2. Contraindications: fractured, open margins (deep
wheels and points: ditching), flat proximal contour, open contacts, re-
a. Abrasive used for white stones, has fine par- current caries, large and broad overhangs.
ticles of aluminum oxide and diamond. 3. Finishing should be performed 24 to 48 hours after
b. Can be mixed into a paste to produce smooth, placement because of slow setting of amalgam; in
polished surfaces on many types of restorations, order of use (burs and stones always move from
including acrylics and composites. the restoration to the tooth):
4. Zirconium silicate: natural mineral: a. FIRST large round finishing bur is used to de-
a. Used as polishing agent in strips or discs. fine gross anatomical features.
b. Often is used in prophylactic pastes. b. Then green stone removes gross surface irregu-
5. Cuttle: fine particle form of quartz or sand: larities from old restorations.
Dental Biomaterials   519

c. Then white stone is used to refine anatomy of Table 15-7  Revised ADA classification system
older amalgams. for alloys for fixed prosthodontics (2003)
d. Then small round bur is used to define occlusal
anatomy; MUST be followed with a large round Classification Requirement
bur to smooth out scratches from the small bur.
High noble alloys Noble metal content
e. Flame-shaped bur is used on interproximal and >60% (gold + platinum
gingival margins; sandpaper discs are helpful group*) and gold >40%
for smoothing (coarse to fine). Titanium and titanium Titanium >85%
4. Polishing is performed with rubber points con- alloys
taining abrasive particles; interproximal strips are
Noble alloys Noble metal content
available; alternative method uses pumice slurry, >25% (gold + platinum
followed by slurry of tin oxide. group*)
Predominantly base Noble metal content
INDIRECT RESTORATIVES  alloys <25% (gold + platinum
Many dental procedures require that restoration be fabri- group*)
cated outside of the mouth and cementing media be used
*Metalsof the platinum group are platinum, palladium, rhodium, iridium,
for attachment of restoration to appropriate oral structures. osmium, and ruthenium.
Inlays, onlays, veneers, crowns, and bridges (fixed, perma-
nent, permanent fixed denture) are indirect restoratives c. Demonstrate superb esthetic results.
that must be cemented to the teeth. Retention pins may be d. The CEREC technique of computer-assisted
used with cast restorations for increased retention. design (CAD) and computer-assisted machin-
A. Restorations: ing (CAM) designs and cuts restorations such
1. Inlay: used when the portion of the tooth that must as inlays, onlays, and veneers at chairside.
be replaced is within the cusps. (1)  Used because ceramics do NOT crack like
2. Onlay: used when one or more cusps are included other glasses and porcelains.
but entire crown is NOT being replaced. (2)  Produces final restoration that is designed,
3. Veneer: cosmetic facing that is bonded to facial polished, cemented in one appointment.
surface of anteriors to improve appearance; porce- (3)  Equipment is expensive; long-term studies
lain veneers have superior resistance to wear and are needed to evaluate longevity.
staining. 3. Composites:
4. Crowns: two types: a. Two techniques are used:
a. Jacket (older term): constructed of all nonmetal (1)  Indirect technique: requires two appoint-
components; limitation includes low strength; ments, one to make an impression and
brittle and CANNOT withstand posterior occlu- make a die and second to cement inlay onto
sal stresses without breaking. prepared tooth.
b. Porcelain-fused-to-metal (PFM): outer layer of (2)  Direct technique: composite is placed and
porcelain bonded to inner alloy casting; greater cured; cured inlay is removed and given a
strength. secondary cure, using heat to maximize prop-
c. Ceramic: some can be used in the posterior erties, then cemented onto prepared tooth.
area; have a zirconia core that allows for greater b. Curing causes polymerization shrinkage.
forces. (1)  Both inlay techniques complete curing
B. Materials: process outside mouth, thus providing
1. Dental casting alloys: BETTER fit than composite restoration,
a. Metals used for inlays and onlays because of in which curing is accomplished in mouth,
strength and durability. which results in shrinkage while composite
b. Most metals are classified as noble elements is bonding to cavity wall.
based on lack of chemical reactivity; include (2)  Marginal integrity is greater for inlays than
gold, platinum, palladium, and other inert ones. for direct composites.
c. Range in amount of gold present in alloy (Table (3)  Curing in lab allows use of high tempera-
15-7): tures, BETTER degree of cure and me-
2. Ceramics: chanical properties, resulting in stronger
a. Strong and have been used successfully as in- restoration.
lays and onlays. 4. Porcelain:
b. Hard but NOT as abrasive as porcelains, gentler a. MAIN advantage is greater resistance to wear
(kinder) to opposing dentition. and staining.
520   Saunders Review of Dental Hygiene

b. Advantage of natural appearance. C. Patient denture care at home:


c. Strong during compression but weak when ten- 1. MUST be placed in liquid bath when taken out for ex-
sile stresses are applied. tended periods to prevent dehydration of resin; AVOID
d. Can cause abrasion to opposing enamel; MUST hot water because of risk of distortion to base.
be considered when determining appropriate 2. Brushed after each meal or at LEAST once daily
materials for restorations. (both base and teeth), BEST to use denture brush
made for purpose.
PROSTHETICS  a. Should be instructed NOT to use abrasive den-
For patients in whom some or all of the teeth have been tifrices; acrylic plastic can be easily scratched
removed, dentures are fabricated to provide an appliance and worn away.
that will not ONLY facilitate the speaking and chewing b. Clasps on partial denture can be brushed with
processes, but also provide esthetic qualities. tufted end of brush.
u0190 • See Chapter 13, Periodontology: implant placement c. To avoid potential breakage, brush over sink
and concerns; 12, Instrumentation: implant care. lined with towel and/or filled with water.
3. Variety of cleansing and soaking agents can be
Dentures used, including nonabrasive dentifrices, commer-
Complete or full denture is prosthetic device used when cial denture cleansers, soap and water, bleaches;
all the natural teeth in an arch are lost. Removable partial MUST educate on proper use to avoid allergies
denture is used when only a few teeth are missing; clasps and toxicities.
are used to attach partial denture to remaining teeth. Den- 4. Homecare solution: using 1 teaspoon of bleach (al-
tures must be made of materials that are gentle (kind) to kaline hypochlorite) and 2 teaspoons of powdered
soft tissues of the oral cavity, strong enough to withstand water softener in glass of water for occasional
forces of mastication, and yet esthetic in appearance. overnight cleansing; to prevent corrosion, NOT for
• See Chapter 6, General and Oral Pathology: denture­ dentures with metal portions; helps reduce bacte-
related pathology. rial and fungal growth from Candida albicans.
A. Parts and materials: 5. Must still brush and take care of oral tissues in
1. Base (rests directly on the soft tissues): consists of area of prosthesis; recommended prosthesis re- o10150
acrylic resins, polymethylmethacrylate (PMMA); moval during sleep to allow oral tissue rest.
radiolucent and easily subject to abrasion. D. Professional office care:
2. Teeth: made of acrylic resin and porcelain; acrylic 1. Placing in a 10% bleach solution (stain removal
resin is LESS brittle but LESS resistant to wear; solutions are available) and vibrating in ultrasonic
porcelain is MORE resistant to wear but is MORE unit in zip-locked bag; brushing after to remove
brittle; BOTH are radiolucent. debris and residues.
3. Clasps (on removables): silver-colored metals, 2. Remaining calculus on non-tissue-bearing surfaces
composed primarily of nickel, cobalt, and chro- is carefully scaled.
mium; strong and highly corrosion resistant (un- 3. Dental biofilm (dental plaque) control should be o6480
less placed in bleach for cleaning); radiopaque. stressed for both prosthesis and oral tissues, es-
B. Retention: do NOT require use of adhesives if prop- pecially with partial dentures, because of caries
erly fit. and because periodontal disease risk for abutment
1. Retention is determined by: (natural) teeth still exists.
o6320 a. Size of the area between tissue and denture; re- E. Denture liners and conditioners: improve the comfort
tention problems with loss of alveolar process and retention of dentures.
because of age, tooth loss, and/or weight loss. 1. For patients who cannot tolerate hard dental acrylic
b. Surface tension of saliva between tissue and on oral mucosa, those with thin alveolar ridges
denture; thin, ropy saliva with low surface ten- who lack supporting structures for dentures, and/or
sion causes retention problems. those with permanent weight loss.
o6340 c. Ability of the saliva to wet the denture; xerosto- 2. Materials for lining and conditioning include ­resilient
mia can cause retention problem and trauma to substances such as acrylics, silicones, and polymers.
the oral tissue. 3. Cleaning of liners with SAME method that is used
d. Normal suction qualities of oral tissues: maxil- for acrylic dentures; care MUST be taken because
lary arch with broad palate has BETTER reten- of softness and poor abrasion resistance.
tive suction than narrow mandibular arch with 4. Homemade liner use SHOULD be discouraged;
its highly attached muscles. can degrade denture base material, leading to pos-
2. Adhesives: increase confidence of denture wear- sible improper occlusion and subsequent damage
ers, NOT needed if well fitting. to the oral structures that support denture.
Dental Biomaterials   521

CLINICAL STUDY  

Age 68 YRS SCENARIO

Sex ☐  Male   ☒  Female The patient is in the dental office for


an examination. She has not seen a
Height 5’5” dentist for more than 3 years, since
Weight 120 LBS she does not have any teeth. Visual
inspection reveals that her hard palate
BP 113/70 is very red. Her dentures have a heavy
buildup of dental biofilm and materia
Chief Complaint “My dentures do not fit really well.” alba. The patient also sleeps with her
dentures in place and takes them out
Medical History Peptic ulcer, treated with antibiotics
only after meals to clean them.
Current Medications Lost weight (20 lbs) but is trying to regain it

Social History Librarian


Pageant winner at age 22

1. Considering this brief history, what is one possible buildup of dental biofilm (plaque) and materia alba
cause for the patient’s red hard palate? What recom- that is not removed with rinsing or soaking. Should
mendations would you have for this condition? be instructed not to use abrasive dentifrices because
2. What recommendations should be made regarding the can easily scratch acrylic plastic of the dentures. To
fit of her dentures? Should her weight loss and gain be avoid potential breakage, should be instructed to brush
a significant factor in immediate recommendations? dentures over sink filled with water and/or lined with
3. What homecare should the dental hygienist suggest towel.
for her dentures?
Endodontic Therapy and Materials
1. The patient could have denture stomatitis (denture Endodontic therapy and treatment (root canal) allows
sore mouth) resulting from infection with Candida a tooth to be saved. Before endodontic therapy is per-
albicans. Use of antifungal agent is indicated to allevi- formed, however, it MUST be determined first that the
ate fungal overgrowth. In addition, soaking dentures tooth is able to be restored to adequate form and function
nightly in a 10:1 water-to-bleach solution can elimi- and has sufficient support by the periodontium. Increased
nate fungus on the dentures and maintain a surface risk with tobacco use.
compatible with the health of the oral tissues. Patient • See Chapters 2, Embryology and Histology: pulp
should be instructed to rinse the dentures thoroughly structure; 6, General and Oral Pathology: endodontic
before placing them in her mouth. lesions; 11, Clinical Treatment: pulpal evaluation.
2. When denture fit has been compromised because of A. Endodontic therapy: sequence of treatment for ne-
weight fluctuation, appropriate to recommend denture crotic pulp.
adhesive to increase retention. Denture adhesives work 1. End result is elimination of infection and protec-
by forming viscous paste with the saliva; viscosity of tion of decontaminated tooth from future micro-
the paste between the denture and tissue results in bet- bial invasion.
ter retention. Her weight loss and gain is significant 2. Subsequent cleaning and shaping (debridement),
factor in immediate recommendations; a more perma- then decontamination (sterilization) of the pulp
nent solution would be to add a professional denture chamber.
liner, but because patient is regaining her lost weight, a. Use of files and irrigating solutions; MUST use
this should not be recommended. rubber dam for isolation.
3. Because the patient sleeps wearing her dentures, she b. Obturation and filling of decontaminated root ca-
should be instructed to remove her dentures nightly nals with inert filling, such as gutta-percha and usu-
and soak them in a diluted bleach solution (as per an- ally eugenol-based cement; gutta-­percha is natural
swer #1); other soaking solutions are commercially thermoplastic polymer of isoprene, melted and
available, but because she is suffering from fungal injected; barium is added to make radiopaque.
overgrowth, use of diluted bleach is indicated. In c. Radiopaque silver points are NOT deformable
­addition, denture brush and instructions on how to (condensable), so cannot create a complete fill
brush after each meal can be provided to avoid heavy or force sealer paste into lateral canals, thus
522   Saunders Review of Dental Hygiene

NO longer used and even are extracted before a. Microabrasive compounds can remove discol-
refill because of long-term failure problems of ored enamel by use of slow-speed handpiece
microleakage and corrosion (with release of with special mandrel before procedure.
toxic products). b. Gel is placed on enamel 2 mm thick; can be
3. Tooth is now NO longer vital (does NOT respond performed with or without heat and light sys-
to pulp vitality tests). tems; gingival tissues are protected by various
B. Root end surgery (apicoectomy): removal of apex is methods.
performed if infection is spreading, then filled with c. Heat and light systems use powerful light or
calcium silicate–based filling materials. wand that is calibrated to control temperatures;
C. Restoration after therapy: teeth tend to be MORE brit- NO evidence that this provides any benefit.
tle because of loss of blood supply to the teeth, which B. Office-dispensed in-home vital procedure:
nourished and hydrated the tooth. 1. Preoperative photographs and tooth shades are re-
1. Placement of crown- and cusp-protecting cast gold corded; need for restorative treatment to comple-
covering, especially in posterior, has BEST ability ment whitening is discussed.
to seal and protect. 2. With 10% carbamide peroxide solution (equiva-
2. Anterior teeth may receive just simple restoration. lent to a 3% hydrogen peroxide solution).
3. May need core buildup (amalgam and/or compos- a. Alginate impression is made, and vacuum-
ite) to fill if large amount of tooth is lost; posts formed mouth trays are fabricated to hold agent
(made of stainless steel or casting alloy) added to against the teeth; tray is worn at prescribed
retain (NOT strengthen) core and cemented into times per day and then per week.
canal to support core buildup. b. Patient is instructed to report any tissue or tooth
sensitivity (critical).
OTHER SUPPORTIVE PROCEDURES c. Patient must be seen postoperatively to docu-
AND TECHNIQUES  ment changes in tooth shade.
Many patients are requesting tooth whitening (bleach- 3. Retreatment may be necessary, depending on
ing) procedures. Placement of rubber dams and matrices patient’s habits and chromogenic agents (e.g., to-
is important supportive technique that is useful in isolat- bacco, red wine, coffee, tea); patient SHOULD be
ing a tooth or the dentition during restorative procedures. informed of this before initial treatment.
Another supportive technique, margination, is the re- C. Over-the-counter (OTC) whitening products: drops,
moval of excessive amalgam in a restoration. strips, trays are available; may have ONLY slight re-
sults, and acidity may cause enamel damage.
Tooth Whitening
External tooth whitening (bleaching) often is the MAIN Rubber Dams
treatment for lightening vital teeth with extrinsic dis- Rubber dams are sheets of material that attach to clamps,
colorations as well as intrinsic stains (fluorosis, tetracy- which then are clamped onto teeth for retention in
cline). Results for extrinsic stains are BETTER than for working area. Thin sheets of rubber latex, although sili-
intrinsic stains that may require repeated procedures to cone is available for patients with rubber latex allergies.
obtain even slight to moderate results; yellow teeth have When inverted, dam prevents leakage and toxicity to
BETTER results than gray teeth. Effectiveness is depen- surrounding tissues. A MUST for endodontic therapy,
dent on application, time, and dose. In all cases of vital placement of glass ionomers, adhesive procedures such
whitening, agent is applied to the tooth surface. This is as bonding composites. However, communication is
in contrast to internal whitening (bleaching) that places limited during use.
agent inside pulpless or endodontically treated tooth. Tem- A. Indications:
porary post-whitening dentinal hypersensitivity and/or 1. Used to isolate clinical crowns of teeth for restor-
gingival trauma may result from procedure. ative procedures, especially with toxic substances
• See Chapters 6, General and Oral Pathology: intrinsic and/or endodontic therapy.
stains; 11, Clinical Treatment: evaluation of extrinsic 2. Retracts and controls soft tissues, including lips,
stains. cheeks, tongue, with limited gingival retraction.
A. In-office vital procedure: 3. Prevents moisture contamination of the teeth and
1. Preoperative photographs and tooth shades are re- optimum visibility of operative site.
corded; need for restorative treatment to comple- 4. Protects patient against swallowing or aspirating
ment whitening is discussed. dental instruments and restorative materials.
2. With 30% to 35% hydrogen peroxide; different 5. Prevents patient from choking on excess water
techniques involve heat, light, microabrasion; spray; decreases clinician contact with saliva and
MOST agents are gels. blood; decreases aerosol in treatment area.
Dental Biomaterials   523

(3) Retainer is usually positioned on the facial


Matrix band Wooden wedge side of the tooth being worked on (in the
Wedge vestibule).
2. Matrix is placed FIRST and then amalgam; wedge
may be used to hold the matrix in place. Afterward,
the matrix is carefully removed so as to not disturb
the restoration.

Margination
Slot facing gingivally Matrix retainer
Figure 15-5  Properly placed matrix band. Margination (recontouring) is the process of removing
excess restorative material to establish a smooth margin,
to smooth the surfaces of the restorative material, and to
recontour functional tooth anatomy. See earlier discus-
B. Contraindications: sion of finishing composites and amalgam restorations.
1. Patients with severe breathing and psychological A. Consequence of overhanging margins:
problems. 1. Retention of dental biofilm (dental plaque) and
2. If used on patient with chronic obstructive pulmo- other deposits:
nary disease (COPD), must have continuous slow a. Can result in recurrent caries.
flow of oxygen. b. Can cause periodontal problems such as gingi-
3. Partially erupted teeth (clamp will NOT fit). val bleeding and bone loss.
4. Patients with behavioral problems (e.g., small chil- B. Manual techniques:
dren). 1. Amalgam knives are used with short, overlapping,
shaving strokes to remove amalgam in small incre-
Matrices ments and to prevent fracture of the margin (ante-
Matrices are materials used in the restoration of teeth to rior or posterior).
replace missing components of teeth. Allow dental ma- 2. Files include coarse files for removing the bulk
terials to be formed into the shape of the original com- of the amalgam and fine files for smoothing the
ponent or portion of the tooth (proximal or axial). When margins.
matrix is NOT placed properly, there can be leakage of 3. Finishing strips are used after gross amalgam has
materials, causing overhang (Figure 15-5). been removed by an amalgam knife or file.
A. Anterior composite restorations: C. Power-driven techniques:
1. Plastic strips (i.e., Mylar strips) are used to place 1. Finishing burs and stones MUST be kept in con-
anterior composites; MUST be clear to allow stant motion with light, sweeping movement to de-
transmission of light when polymerizing compos- crease possibility of leaving marks and grooves.
ite material. 2. Discs have different abrasivities and are used in
2. Composite is placed FIRST and then the matrix is overlapping strokes diagonally across cavosurface
placed, polymerized, and then removed. margins; disc is rotated from tooth to amalgam to
a. Oxygen inhibits polymerization; covering with avoid ditching restoration.
plastic, nonporous matrix keeps oxygen out and 3. Ultrasonic scalers are BEST for removing over-
allows maximum hardness. hangs; some models have special tips for removal.
b. Provides smooth, regularly contoured surface,
thus LESS time required for finishing resto­ Dental Lab Infection Control
ration. Dental lab infection control SHOULD follow the guide-
B. Amalgam restorations: lines of standard precautions, including that ALL patient
1. Metal matrix bands are used for amalgam restora- items are treated the same, NO matter what the health his-
tions that involve walls of a tooth. tory indicates. Items that ultimately will come into con-
a. Tofflemire matrix retainer and band (Figure tact with the oral cavity MUST be sterilized. Lab items
15-5): that can withstand the heat of sterilization SHOULD be
(1) Small opening of the band MUST always sterilized to increase overall level of infection control in
point toward the cervical portion or root of the dental lab. High-level disinfection of lab products
the tooth. (prostheses) that contact oral mucosa often is sufficient if
(2) Slotted side of the retainer MUST always infection control protocols prevent cross-contamination.
point toward the root of the tooth to allow Routine cleaning and disinfection of lab equipment and
removal of the retainer after the restorative environmental surfaces should be accomplished with the
material has partially set or crystallized. SAME regularity as in the dental office.
524   Saunders Review of Dental Hygiene

• See Chapter 8, Microbiology and Immunology: infec-   3 The modulus of elasticity is the measure of a material’s
tion control protocol, standard precautions. A. hardness.
A. Lab area: B. stiffness.
1. Disposable paper should be placed on counter- C. percentage of elongation.
tops. D. ductility.
  4 The hardness of a material is defined as the resistance of its
2. Disposable plastic coverings should be placed over
surface to
areas that are likely to be touched. A. elastic deformation.
3. Shielding device and high-volume suction used B. strain.
during polishing and grinding to minimize aerosol- C. scratching.
ization of microorganisms and grinding products. D. indentation.
4. Counters should be regularly wiped down with   5 What is the definition of adhesion?
hospital-level disinfectant approved by the Envi- A. Type of bonding that does not require intimate attraction
ronmental Protection Agency (EPA). between the atoms and molecules of two substances
B. Personal protective equipment (PPE) is worn during B. Reaction with which an object resists an external force
work with contaminated lab materials. C. Attraction between like atoms and molecules on two
C. Dental impressions: carry saliva, blood, and microor- different surfaces as they are brought into contact
D. Attraction between unlike atoms and molecules within a
ganisms, must be decontaminated.
given material
1. Rinsed with tap water right after impression is made.   6 Van der Waals forces refer to which type of bond?
2. Soaked in disinfectant that is recommended for A. Cohesion
impression material, for period recommended B. Primary
by manufacturer; different disinfectants may be C. Secondary
needed for different impression materials. D. Chemical
3. Rinsed again with tap water to remove disinfectant   7 Before adhesion can occur between a liquid and a solid, it is
before impression is poured. essential that the solid surface
D. Complete, fixed, and removable partial dentures are A. provide some mechanical interlocking with the liquid.
disinfected (see earlier discussion under dentures). B. be wetted by the liquid.
E. Fixed and removable prosthetic devices are disin- C. exhibit a large contact angle with the liquid.
D. enter into some form of chemical reaction with the
fected after being returned from the lab, after place-
liquid.
ment in the mouth, and after in-office adjustment.   8 Which of the following materials is considered an elastic
1. Disinfectant is checked for compatibility with ma- impression material?
terials used in prosthetic device. A. ZOE impression paste
2. Prosthetic device is rinsed with tap water. B. Reversible hydrocolloid
3. Prosthetic device is disinfected for the time recom- C. Impression plaster
mended for tuberculocidal disinfection. D. Tray compound
4. Device is rinsed again with tap water and allowed   9 The clinician can reduce the potential for permanent de-
to dry. formation when removing an alginate impression from the
5. Professional cleaning of removable prosthetic de- mouth by removing it
vices in the dental office: same as for dentures, but A. slowly and carefully.
B. before it is fully set.
do NOT use solutions that would cause corrosion
C. with patient assistance.
on metal portions. D. with a quick snap.
10 Why do manufacturers include calcium sulfate as a compo-
nent of alginate impression material?
Review Questions A. Reacts with potassium alginate to create gelation
B. Serves as a filler
C. Acts as a retardant
  1 The proportional limit of a material is the stress D. Accelerates the setting of gypsum
A. beyond which elasticity first begins to occur. 11 Which of the following statements concerning agar impres-
B. at which strain hardening ceases to occur. sion material is CORRECT?
C. beyond which plastic deformation begins to occur. A. Composed primarily of diatomaceous earth
D. at which fracture occurs. B. Changed into a gel by a physical reaction
  2 The amount of deformation or change in the length of a C. Changed into a gel by a chemical reaction
dental material is called D. Also known as irreversible hydrocolloid
A. stress.
B. yield point.
C. strain.
D. galvanism.
Dental Biomaterials   525

12 What is a disadvantage for the use of polysulfide rubber im- 21 For dentin adhesives, acid etching is accomplished with
pression material? A. 30% phosphoric acid.
A. Lacks accuracy in recording detail B. 40% maleic acid.
B. Has a poor shelf-life C. 10% phosphoric acid.
C. Lacks compatibility with model plaster and stone D. 37% EDTA.
D. Has strong odor and stains clothing 22 The higher content of which metal increases amalgam
13 Which of the following is NOT considered true for poly- strength and superiority?
ether rubber impression material? A. Zinc
A. Has the longest working time of any of the rubber im- B. Silver
pression materials C. Tin
B. Stiffest of any of the rubber impression materials D. Copper
C. Can cause tissue irritation because of the aromatic sul- 23 For a cavity preparation in which there has been a near pulp
fonic acid ester catalyst exposure, which of the following materials would be the
D. Has great flexibility, which may result in problems when BEST to use to protect the tooth?
it is removed from the mouth A. Polycarboxylate
14 The chemical composition of both plaster and dental stone is B. Zinc phosphate
A. calcium sulfate dihydrate (2 CaSO4 • 2 H2O). C. Glass ionomer
B. potassium sulfate. D. Calcium hydroxide
C. calcium sulfate hemihydrate [(CaSO4)2 • ½ H2O]. 24 Which of the following is CORRECT statement about the
D. potassium alginate. use of zinc phosphate cements?
15 Increasing the water/powder ratio of any of the gypsum ma- A. The powder is added to the liquid all at once.
terials results in B. Moisture on the slab will accelerate the setting time.
A. decreased setting time. C. The cement is mixed over a small area of the slab.
B. stronger cast or die. D. Cooling the mixing slab decreases the setting time.
C. increased setting expansion. 25 In a clinical situation in which a patient requires a deep cav-
D. increased setting time. ity preparation and an amalgam restoration must be placed,
16 What is the reason plaster requires more water for mixing which of the following would be the BEST choice for a
than stone? base?
A. Hemihydrate crystals are more porous and irregularly A. Zinc oxide–eugenol
shaped. B. Glass ionomer
B. Its particles are formed by heating gypsum under steam C. Polycarboxylate
pressure. D. Cavity varnish
C. It has the least porous hemihydrate particles of all of the 26 When using a dental cement mixed for use as a base, which
gypsum materials. of the following is CORRECT?
D. Hemihydrate crystals are more regular in shape and A. The final mix should be thin and creamy.
have a smoother surface. B. A low powder-to-liquid ratio should be employed.
17 Which of the following could lead to the corrosion of re- C. It will have a slow setting time.
storative materials? D. A high powder-to-liquid ratio should be employed.
A. Adjacent restorations constructed of similar metals 27 Which of the following cements actually bonds to tooth
B. Amalgam in close contact with a ceramic restoration structure?
C. Adjacent restorations constructed of dissimilar metals A. Zinc phosphate cement (Flecks)
D. Close contact of a gold onlay and a gold crown B. Glass ionomer cement (Ketac-Cem)
18 Which of the following restorative materials has a thermal C. Calcium hydroxide (Dycal)
expansion closest to that of tooth structure? D. Polycarboxylate cements (Durelon)
A. Acrylic resin 28 The finishing and polishing of amalgam restorations are
B. Amalgam ­indicated for which of the following listed circumstances?
C. Gold A. Open contacts
D. Composite resin B. Fractured amalgams
19 The organic resin matrix used in composite restoration is C. Open margins between tooth and amalgam
composed of D. Surface irregularities
A. polymethyl methacrylate (PMM). 29 When should the polishing of a newly placed amalgam gen-
B. vinyl silane. erally be performed?
C. polystyrene. A. Immediately after placement
D. bisphenol-A glycidyl methacrylate (bis-GMA). B. 24 to 48 hours after placement
20 Polymerization shrinkage, the MOST serious problem as- C. 12 hours after placement
sociated with dental composites, leads to D. 1 month after placement
A. pitting of the composite surface.
B. marginal leakage.
C. wear of the composite.
D. water absorption.
526   Saunders Review of Dental Hygiene

30 Which of the following is a polishing abrasive? 40 Dental alloys become amalgam when mixed with which
A. Tin oxide metal?
B. Sand A. Silver
C. Diamond B. Lead
D. Alumina C. Mercury
31 A restoration that is used when one or more cusps of a tooth D. Magnesium
are in need of replacement but a full crown is NOT indi- 41 What is the preferred consistency of ZOE cement used for a
cated is a(n) temporary restoration?
A. veneer. A. Fluid and creamy
B. inlay. B. Firm and brittle
C. jacket. C. Puttylike
D. onlay. D. Watery
32 Which of the following is a disadvantage of using gold for 42 One of the following metals is NOT found in an amalgam.
restorative procedures? Which one is the EXCEPTION?
A. Lack of esthetic qualities A. Tin
B. Susceptibility to corrosion and tarnish B. Silver
C. Poor biocompatibility with oral tissues C. Copper
D. High strength and durability D. Magnesium
33 Which of the following statements is CORRECT regarding E. Zinc
the use of porcelain in indirect restorations? 43 Alginates must be removed with a quick thrust or snap be-
A. Does not create a natural appearance for the resto­ration cause if NOT they will cause irregularities in the impre­ssion.
B. Can cause abrasion to opposing teeth A. Both the statement and reason are correct and related.
C. Weak in compression B. Both the statement and reason are correct but NOT
D. Stains easily related.
34 Patients who wear dentures should be instructed to C. The statement is correct, but the reason is NOT.
A. use an abrasive dentifrice to remove debris. D. The statement is NOT correct, but the reason is correct.
B. soak dentures in full-strength household bleach. E. NEITHER the statement NOR the reason is correct.
C. use hot water to cleanse dentures. 44 Overhangs can be caused if the ___________ is NOT placed
D. keep dentures in water when not in the mouth. properly.
35 What is the major etiological factor of denture stomatitis? A. wooden wedge
A. S. mutans B. matrix band
B. Papillomavirus C. rubber dam
C. S. salivarius D. anesthetic block
D. C. albicans 45 Which of the following is a contraindication for the use of a
36 For BEST results in casting procedures, water and invest- rubber dam?
ment powders should be A. A patient with severe breathing or psychological
A. cooled. ­problems
B. aerated. B. Prevention of moisture contamination
C. vacuum mixed. C. Decreased operator contact with saliva and blood
D. measured. D. Decreased aerosol in the treatment area
37 Which of the following will cause delayed expansion in an
amalgam restoration containing zinc?
A. Heavy condensation
B. Excess mercury in the mix Answer Key and Rationales
C. Overtrituration of the amalgam
D. Contamination by moisture during manipulation
38 Brinell and Knoop tests both measure
A. hardness. 1 (C)  Proportional limit is the point on the stress/strain
B. ductility. diagram at which stress and strain are directly pro-
C. elasticity. portionate to one another. If stress is removed at this
D. malleability. point, strain recovers and there is NO deformation.
39 Orange solvent is BEST used to clean the instrument after Permanent deformation occurs above the propor-
mixing which of the following? tional limit, when stress and strain are NO longer in
A. Amalgam
proportion to one another and stress is greater than
B. Polycarboxylate
C. Zinc oxide and eugenol
material can withstand (like your head after studying;
D. Bis-GMA feels like it is going to explode).
Dental Biomaterials   527

2 (B)  Strain is defined as deformation or change in and through chemical reaction is changed into gel.
length and dimension of dental material that results This property allows the material to flow around the
from applied stress. Stiffness of object determines oral structures when initially placed in the mouth and
ability to resist dimensional change and strain. to harden into a gel for easy removal without defor­
3 (B)  Modulus of elasticity is a measure of a material’s mation.
stiffness. Stiffness is an IMPORTANT issue in se- 11 (B)  Agar impression material is changed into a gel
lection of restorative materials because large deflec- by a physical reaction that involves water-cooled im-
tions are undesirable under conditions such as biting pression trays. Agar gel is converted to a sol when
forces. When materials MUST withstand the forces heated in water (212° F) and then becomes a gel when
of mastication, preferable to have high modulus of cooled to 110° F. For this reason, agar impression ma-
elasticity. terial is also known as reversible hydrocolloid.
4 (D)  Hardness is material’s resistance to indentation. 12 (D)  Polysulfide was the first rubber impression mate-
Hardness of dental materials is MOST commonly re- rial developed. Although polysulfide has many useful
ported in Knoop hardness numbers (KHN). Knoop properties, such as accuracy of detail, lead dioxide in
hardness test uses diamond indenter and calculations catalyst gives off a strong odor and causes permanent
to create hardness number. The larger the indenta- staining of clothing.
tion, the smaller the hardness number. Enamel (350) 13 (D)  Polyether impression materials are stiffer than
and porcelain (400-500) are two of hardest materials polysulfides and silicones. This stiffness can present
and therefore have two of highest numbers; however, problems in removing impression from undercuts.
porcelain crown will abrade enamel because of dif- One way to compensate is to increase thickness of
ferences in hardness if contact in opposing dental material between impression area and tray.
arches. 14 (C)  Chemical composition of both plaster and den-
5 (C)  Adhesion is force of attraction between the mol- tal stone is calcium sulfate hemihydrate. Gypsum,
ecules and atoms on two different surfaces when rock from which plaster and stone are formed, occurs
brought into contact with one another. For instance, widely as calcium sulfate dihydrate. In manufactur-
when orthodontic bracket is cemented to a tooth, ad- ing process, water is driven off to form calcium sul-
hesion between cement and the tooth surface holds fate hemihydrate. Plaster is made by heating gypsum
bracket in place. at atmospheric pressure; use of steam pressure results
6 (C)  Van der Waals forces refer to secondary bonds, in dental stone.
which involve physical attraction between atoms and 15 (D)  Increasing water/powder (W/P) ratio of gypsum
molecules of adhesive and adherend. Physical attrac- materials produces thinner mix that takes longer to
tion does NOT provide as strong a bond as primary set. Thinner mix results in weaker cast or die.
bonds in which a chemical union occurs. MOST 16 (A)  The MORE porous and irregularly shaped the
common type of adhesion involves secondary bond- hemihydrate crystals, the MORE water required
ing, which occurs with zinc phosphate cement. This for mixing. Plaster is MOST porous and irregularly
cement does NOT involve chemical union between shaped of all of gypsums. Dental stone is LESS
the cement and the tooth. porous than plaster and also has MORE regularly
7 (B)  For chemical and physical adhesion to occur, it shaped particles. High-strength dental stone particles
is essential that adhesive and adherend be in intimate are nonporous, dense, and smooth.
contact. Therefore adhesive is typically liquid that 17 (C)  Corrosion is result of a chemical or electrochem-
can easily flow over entire surface and then come into ical reaction. Electrical current is created by close
contact with all of small roughness on that surface. proximity of dissimilar metals, resulting in dissolu-
Adhesive is said to “wet” the adherend. tion of metals. Dissolution of metals causes rough-
8 (B)  Impression materials can be classified as either ness and pitting of restoration.
elastic or inelastic. Reversible hydrocolloid (agar) 18 (C)  Thermal expansion of tooth structure is 11.4, and
is elastic material with sufficient flexibility to allow thermal expansion of gold is 15 (×10-6/°C). Thermal
removal from undercuts without permanent defor­ expansion of restorative materials often does NOT
mation. match that of the tooth. For example, thermal expan-
9 (D)  BEST to remove elastomeric impression materi- sion of composite resin is ∼35. This differential expan-
als with a quick snap that decreases the stress placed sion becomes problematic in dentistry because it results
on the impression. Reduction of stress decreases like- in microleakage of oral fluids between the tooth and
lihood of permanent deformation. restoration, which can lead to decay and discomfort.
10 (A)  Calcium sulfate reacts with potassium alginate 19 (D)  The bis-GMA is a liquid resin to which inor-
to initiate the gelation of alginate impression mate- ganic component, such as silicate or glass particles,
rial. Alginate is placed in the mouth in liquid state is added, forming paste or composite material.
528   Saunders Review of Dental Hygiene

20 (B)  Stresses from polymerization shrinkage can paste. Finishing abrasives are coarse, hard particles,
exceed the bond strength and cause a pulling away and polishing abrasives are fine particles.
of the material from the tooth structure, resulting in 31 (D)  Onlay is often desirable when full coverage of
marginal leakage. Two clinical problems that result tooth by crown is NOT necessary. Traditionally, only
from leakage are secondary caries and staining or dis- metals were used as restorative materials in produc-
coloration of restoration. tion of onlays because of high strength and durability.
21 (C) Weaker solution of acid etch is used on dentin As demand for esthetics has increased, dentistry has
than that used for enamel because of concern for begun to use materials such as ceramics in production
pulpal damage. of these restorations.
22 (D)  Tin-mercury compound that is formed when 32 (A)  Gold traditionally has been used in dentistry
amalgam is mixed results in gamma 2, weakest of because of superior properties, including ease with
the phases in the setting reaction process. Tin has a which it can be worked, resistance to tarnish and cor-
greater affinity for copper than mercury; thus higher rosion, ability to withstand the oral environment, and
copper alloys form a copper-tin compound instead of strength. Demand for MORE esthetic dentistry has
the tin-mercury compound. Copper-tin compound is resulted in restorations in which porcelain is fused to
sufficient to suppress gamma 2 formation, resulting metal (PFM) to give patient MORE natural-looking
in stronger restoration. replacement restoration.
23 (D)  Calcium hydroxide cement is used for BOTH di- 33 (B)  Porcelains (KHN 400-500) have a greater hard-
rect and indirect pulp capping and as protective bar- ness than enamel (KHN 350), which presents poten-
rier beneath composite restorations. One of unique tial problem when they are placed in opposition to
properties of this cement is that it stimulates forma- one another in the dental arches. Because porcelain
tion of reparative dentin. The pH of cement is basic. can abrade enamel, often is used ONLY on buccal
24 (B)  Any moisture on slab or from condensation re- and facial surfaces.
sults in accelerated setting times and adverse effects 34 (D)  Acrylic material used in the fabrication of den-
on properties of cement. tures can shrink if it is left in a dry environment. Re-
25 (A)  Zinc oxide–eugenol cements, because of neutral sulting dimensional change is corrected when water
pH, have sedative effect on the pulp. For this rea- is again absorbed by denture; however, dentures will
son, especially useful when deep cavity preparations feel tight when first replaced.
might lead to posttreatment sensitivity. 35 (D)  Major etiological factor of denture stomatitis
26 (D)  High powder-to-liquid ratio is employed to pro- is poor oral hygiene, which results in formation of
duce a mix having a thick, puttylike consistency that can mature dental biofilm (dental plaque). Contains mi-
be applied to the floor of cavity preparation with amal- croorganisms that cause tissue inflammation along
gam condenser. In addition, high powder-to-liquid ratio with pathogenic yeast microorganisms, primar-
results in quicker setting time and a stronger product. ily Candida albicans. Condition can be provoked
27 (D)  One advantage of the polycarboxylate cements is or worsened by constant wearing of denture. Can-
ability to bond chemically to tooth structure. Cement didiasis can be treated as localized infection with
adheres through an ionic interaction between nega- antifungal preparations and improved hygiene. May
tively charged molecules in the cement and positively be necessary to soak denture in antifungal solution
charged atoms, such as calcium in tooth structure. or bleach.
28 (D)  Finishing and polishing are indicated when 36 (D)  For BEST results in casting procedures, water
surface irregularities might serve as dental biofilm and investment powders of gypsum should be mea-
(dental plaque) traps or cause irritation to the tissue sured so that W/P ratio is CORRECT. Gypsum prod-
because of roughness of surface. In instances of frac- ucts are used MAINLY for positive reproductions
tured amalgams, open margins, or open contacts, new and replicas of oral structures, casts, dies, or models.
amalgam restoration SHOULD be placed. 37 (D)  Delayed expansion in amalgam restoration con-
29 (B)  MOST amalgams are ready for polishing 1 day taining zinc will occur because of contamination by
(24 hours) after placement. This time lag between moisture during manipulation. During manufacture,
placement and polishing allows the amalgam to fin- zinc reduces oxidation of the other metals in alloy.
ish hardening fully. Zinc reacts with water to produce hydrogen gas, caus-
30 (A)  Tin oxide (SnO2) is a pure white powder that is ing amalgam restoration to expand and seem to push
used extensively as final finishing and polishing agent out of the preparation (delayed expansion). Should
for teeth and metallic restorations. Intent of polishing NOT even be handled with fingers to reduce mois-
is to create smooth surface on restoration or appli- ture contamination (current standards do NOT allow
ance that does NOT contain scratches. Tin oxide is this anyway). Zinc-containing amalgams have longer
mixed with water, alcohol, or glycerin and used as clinical life.
Dental Biomaterials   529

38 (A)  Brinell and Knoop (KHN) both test hardness, re- oxide is mixed with eugenol, zinc oxide–eugenol
sistance of a solid to penetration. The higher the hard- (ZOE) cement is result.
ness number, the harder the material. 42 (D)  Magnesium is NOT found in an amalgam; is a
39 (C)  Orange solvent (love that tropical smell!) is metal alloy of tin, silver, copper, zinc. Dental amal-
BEST used to clean instrument that was used to mix gam is made by mixing approximately equal parts of
zinc ­oxide–eugenol (ZOE). Thus set ZOE materials powdered metal alloy with mercury.
can be dissolved with a variety of organic solvents, 43 (A)  Both the statement and the reason are correct
such as alcohol and orange solvent. Can also use al- and related. Alginates MUST be removed with quick
cohol to clean instrument. thrust (snap) because if NOT will cause irregularities
40 (C)  Dental alloys become amalgam when mixed in the impression.
with the metal mercury, ONLY pure metal that is 44 (B)  Overhanging margins on restorations result in
liquid at room temperature. When liquid mercury is retention of dental biofilm (dental plaque), which can
mixed with amalgam alloy, mercury both is absorbed result in recurrent caries and periodontal problems. Ma-
by particles and dissolves surface of the particles. trix band MUST be placed properly to prevent this.
Mercury toxicity is concern in dentistry because mer- 45 (A)  Use of rubber dam is contraindicated for patients
cury and its chemical compounds are toxic to kidneys with severe breathing and/or psychological problems,
and central nervous system. for patients who have behavior problems, and also
41 (C)  Preferred consistency of ZOE cement to be used when teeth are only partially erupted and clamp will
for a temporary restoration is puttylike. When zinc NOT fit securely onto them.
CHAPTER  16

Special Needs Patient Care


BASIC PRINCIPLES OF CARE FOR SPECIAL NEEDS b. Include loss of limbs, paralysis, arthritis, cere-
PATIENTS brovascular accident (CVA, stroke), myasthenia
Disability is a permanent or long-term condition, including ­gravis, Parkinson’s disease, multiple sclerosis.
physical, medical, psychological, and/or mental limita-
tions, that requires individual consideration in planning Levels of Function
treatment. Risk factors for NOT achieving oral health are Assessment of functional level involves evaluation
listed when involved. of ability to perform activities of daily living (ADLs)
• See CD- ROM for Chapter Terms and WebLinks. such as bathing, eating, dressing, speaking, walking. The
• See Chapters 6, General and Oral Pathology: common higher the functional level, the greater the ability to take
medical diseases and disorders; 10, Medical and Dental care of themselves. ADL assessments have different rat-
Emergencies: emergency protocol; 11, Clinical Treatment: ing scales and four levels. First level (I) refers to highest
modifications of dental treatment; 9, Pharmacology: drug level of function, and last refers to lowest level (IV). If
therapies and antibiotic premedication, substance abuse. guide dog is being used, do not pet or interfere with dog;
A. Types of disabilities: ask how to handle the dog.
1. Developmental disabilities: may be present at birth • See Chapter 18, Ethics and Jurisprudence: informed
(result of a genetic defect, brain damage, or nutri- consents and those who have lower functioning levels.
tional or other deficiency during prenatal develop- A. High function category (levels I and II):
ment) or may occur before adulthood; considered 1. Able to attend to MOST of ADL needs with some
permanent conditions (e.g., chromosomal abnor- supervision and encouragement.
malities, autism, cerebral palsy, fetal alcohol syn- 2. Require daily reminder for oral care and encour-
drome, postnatal infections, birth anoxia, epilepsy). agement to go slowly and thoroughly; may require
2. Acquired disabilities: obtained from external assistance with transportation.
forces during adulthood, related to illness or in- 3. Capable of giving informed consent.
jury (e.g., traumatic head injury, spinal cord injury, B. Moderate function category (level III):
multiple sclerosis, arthritis). 1. Need supervision and assistance with some ADLs.
B. Classifications: 2. May require use of gestures, demonstration, or
1. Medical disabilities: adaptive equipment for communication.
a. Medically compromised, associated with 3. NOT able to give informed consent; power of at-
­conditions that affect major organs of the body torney or guardianship documentation MUST be
(discussed in other chapters). obtained to determine with whom to discuss treat-
2. Communication disabilities: ment.
a.­ Related to neurological damage to parts of the C. Low function category (level IV):
brain responsible for language and speech de- 1. Little or NO ability to perform ADLs themselves.
velopment. 2. Require second or third party to provide daily care;
b. Include aphasia, apraxia, dysarthria. usually homebound.
3. Sensory disabilities: 3. NOT able to give informed consent; power of at-
a. Conditions associated with the senses. torney or guardianship documentation must
b. Include varying degrees of blindness and hear- be obtained to determine with whom to discuss
ing loss. ­treatment.
4. Cognitive disabilities:
a. Associated with reduced mental capabilities. Common Barriers to Healthcare
b. Include intellectual disability (mental retarda- Americans with Disabilities Act (ADA) helped improve
tion), Alzheimer’s disease, mental illness. access to healthcare. Includes laws that govern wheel-
5. Orthopedic disabilities: chair access to public buildings and restrooms, barrier-
a. Conditions associated with use of the legs and free public buildings, improved telecommunications
arms. for hearing- and vision-impaired individuals. Prohibits

530
Special Needs Patient Care   531

d­ iscrimination on basis of disability in employment, gov- C. Transport barriers:


ernment, public accommodations, education, commercial 1. MOST common; many prefer safety of the home
facilities, telecommunications, and transportation. Dental to problems associated with public or private
offices are viewed as places of public accommodation. ­transportation.
Special needs patients still face many barriers. 2. Can influence ability to reach important destina-
A. Communication barriers: tions such as the dental office.
1. Include attitudes of healthcare workers about D. Economic barriers:
treating and communicating with disabled individ­ 1. Greatest limitations to receiving necessary dental
uals, and patient and family attitudes toward ­dental care:
care. a. Have only Social Security and other governmen-
2. Forms that are readily understood by employees tal programs as means of economic ­support.
and patients; may include obligation to provide b. Those who are employed typically earn low wages.
translators at no cost. (1) Any money received is required for pri-
3. Involve hearing and visual losses and speaking mary needs such as shelter and food.
­difficulties: (2) Medical and dental care is many times
a. Always talk directly to patient, even when ­relegated to the bottom of list of needs.
­caregiver is present, unless patient is NOT able c. Those on Medicaid and Medicare CANNOT
to communicate. find providers who are willing to accept less
b. Patient informed consent is required (when than customary fees for services.
­patient is cognizant) before patient care can be 2. Paying for dental services difficult because most are
discussed with caregivers and others. paid out of pocket and are not covered by ­insurance.
B. Physical barriers: E. Motivational barriers:
1. Include stairs, narrow doorways, heavy doors, 1. MOST common for those who rely on others for
distant parking, area rugs or other floor coverings partial care.
that could cause tripping, lack of elevators, narrow 2. May be complicated by communication difficulties.
restroom stalls, restricted access to drinking foun- a. Although cognizant, may be NOT able to
tains, telephones, and restrooms. ­communicate needs to caregivers.
2. Proper design of office spaces to accommodate b. Some may also be forgetful; written ­instructions
needs of employees and patients in case of new in addition to verbal instructions should be
offices and some remodeling projects (Box 16-1). given to BOTH patient and caregiver.
a. Remove access barriers, where such removal is F. Employment barriers: responsible practices that
“readily achievable.” ­prohibit discrimination against those with physical or
b. Defines readily achievable to mean easily mental impairment coupled with reasonable accom-
­accomplishable without much difficulty or modations for employees.
­expense.
Patient with Communication Disorders
Patients with communication disorders are either NOT
Box 16-1  American Dental Association Standards able to make speech sounds because of structural disease
for Accessible Design
or damage, or NOT able to understand language or form
thoughts into words. Related in discussion to disabilities
After handicapped parking space, route of travel must be
in later sections.
free of obstruction and at LEAST 36 inches wide.
Minimum 32-inch door opening is needed with at least A. Types:
18 inches of clear wall space on pull side of door next 1. Aphasia:
to the handle. a. NOT able to put thoughts into words or to
Door handle must be NO higher than 48 inches above floor ­understand language.
and of lever type, able to be operated with closed fist.
b. Caused by neurological damage or organic
Greeting desk may be required to be NO more than
36 inches high at some point to accommodate someone brain disorder such as dementia.
in wheelchair. 2. Apraxia:
Hook for coats NO higher than 48 inches above the floor. a. NOT able to form speech sounds properly.
Circulation paths through public areas, all obstacles ­­ b. Caused by central nervous system (CNS) lesion
cane-detectable (located within 27 inches of floor or
or organic brain disorder such as dementia.
higher than 80 inches, or protruding less than 4 inches
from wall). 3. Dysarthria:
A 5-foot circle for turning wheelchair completely. a. NO clear speech pattern, slurring, as result
Cabinets must be between 28 and 34 inches in height. of damage to the CNS or peripheral nervous
­system (PNS).
532   Saunders Review of Dental Hygiene

b. Motor speech disorder associated with cerebro- C. Risk factors: choking when swallowing is difficult;
vascular accident (CVA, stroke), cerebral palsy, inadequate nutrition.
Parkinson disease. D. Barriers to care:
B. Oral signs: 1. Economic cost if the disability affects employment.
1. Depend on severity of condition and loss of muscle 2. Transportation if not able to drive.
control. 3. Communication when speech production or com-
2. Difficulty in clearing food and inability to clean prehension is difficult.
the teeth adequately may cause increased risk of E. Professional and homecare: maintenance of adequate
caries and periodontal disease. homecare, with assistance as needed.
3. Difficulty swallowing and NOT being able to F. Patient or caregiver education: discussion about oral
­perform or understand need for good oral hygiene health and need for excellent oral care; caregivers
may also complicate oral health. who provide oral care SHOULD hear ­instructions.

clinical study  

Age 17 YRS SCENARIO

Sex ⌧  Male   ■  Female During an intraoral examination


of a new patient, generalized
Chief Complaint Unknown ­moderate gingivitis is noted.
The patient has not had dental
Medical History Developmentally disabled
radiographs taken for 2 years. He
Wheelchair bound
Spastic movements is unable to keep his mouth open,
and communication with him is
Current Medications diazepam (Valium) 10 mg tid difficult.

Social History Lives with his family on weekends


Lives at special school during the week

1. What disability does the patient have? such as lengthened toothbrush handle and fixed tooth-
2. What barriers or risks does the patient face in ­receiving paste cap, in addition to powered toothbrushes and
adequate professional dental care? What radius for oral irrigators, may be helpful.
turning around a wheelchair does the dental office need 4. Communication difficulties may be present and may
to provide if it is being newly designed or ­remodeled? affect interaction between clinician and patient. Parent
3. Recommend homecare products and procedures for or caregiver can provide useful link when communi-
this patient. cations are ineffective. Clinicians should be prepared
4. What specific procedures should the dental office to safely handle a wheelchair transfer to dental chair.
­prepare to provide while treating him? Relaxed atmosphere and good rapport will help reduce
spastic movements associated with emotional distress.
1. The patient has cerebral palsy (CP), developmental Spastic movements can be risky for both patient and
disability present since birth, possibly caused by ex- clinician, so solid instrumentation fulcrums are re-
posure to virus prenatally or lack of adequate oxygen quired. Mouth props, body wraps, and head restraints
intake immediately after birth. may be needed. Extra cushioning and/or frequent
2. Physical barriers the patient may face include lack of repositioning can reduce discomfort. Because of his
wheelchair access. Communication barriers are also age, panoramic radiograph may be indicated to check
likely with spasticity because speech production is diffi- development of third molars and other oral structures.
cult. Risks associated with provision of professional oral Bitewing radiographs can be taken by having parent or
care include accidental injury from involuntary move- caregiver stabilize his head with one arm and hold film
ment. A 5-foot circle for turning a wheelchair completely and holder in his mouth with the other hand.
around is needed in a new or remodeled dental office.
3. Recommendations for homecare should include as Patient with Sensory Impairment
much emphasis on self-care as possible. If patient is Patient with sensory impairment has loss of sight or hear-
not able to adequately clean his mouth or if brushing ing that makes communication and other daily living is-
causes trauma because of spasticity, caregivers must sues difficult. Sensory impairments often occur as result of
perform toothbrushing procedures. Adaptive aids, ­infection, trauma, or disease, but some may be ­inherited.
Special Needs Patient Care   533

A. Types: b­ onnet or small shower cap can be used to cover


1. Hearing impairment: patient’s ears.
a. Severity can range from slight to total D. Barriers to care:
­deafness. 1. Physical obstacles such as doorways and stairs;
(1) Conductive: outer or middle ear involve- finding their way in new surroundings is difficult
ment of conduction pathways to inner ear. for blind.
(2) Sensorineural: damage to sensory hair cells 2. Lack of transportation; arranging transportation
of inner ear or its nerves. and/or relying on caregiver for scheduling and
(3) Mixed: combination of both. transportation is cumbersome.
(4) Central: damage to the CNS or pathways. 3. Lack of communication; MORE challenging for
b. Can occur as result of infection, trauma, dis- sensory-impaired individuals; can sometimes
ease, drugs, or heredity. cause healthcare providers to be apprehensive
(1) In adults: commonly noise induced or side about communicating with them.
effect of streptomycin as a child. 4. Economic issues; good employment opportunities
(2) In children: heredity, pregnancy, birth com- may be MORE limited for sensory-impaired ­person.
plications, meningitis. E. Professional and homecare:
c. May be indicated by inappropriate respon­ 1. For visually impaired:
ses to questions or lack of interest in verbal a. Positioning of caregivers and others to visual ad-
­communication. vantage of patient (directly in front of patient).
d. Hearing aids may help restore some hearing b. Verbally oriented approach; explain procedures
acuity (in-the-ear or canal); may involve a coch­ before performing them; always use “tell, touch,
lear implant. feel” approach; do NOT leave the room without
2. Visual loss or blindness: advising patient.
a. Many affected individuals are NOT able to read c. Use of large visual aids and materials and/or pro-
with correction. vision of tape-recorded homecare instructions.
b. FEW legally blind individuals are completely d. AVOID shining operatory lamp in patient’s
blind. eyes and yelling loudly.
c. Blind individuals may exhibit sensitivity to 2. For hearing impaired:
light. a. Elimination of loud or background noises when
B. Oral signs NOT directly associated with ­ visual or attempting to communicate, such as high- or
hearing impairment: low-speed suction (high-volume evacuator
1. Poor oral hygiene and accompanying oral disease [HVE]), radio, or piped-in music.
are common. b. If patient has some hearing, direct speech to the
2. May occur because of inadequate presentation of hearing ear; do NOT wear mask.
oral hygiene instruction. c. Positioning of caregivers and others so pa-
C. Factors that reduce effectiveness of oral care in the tient can see facial features (particularly lips and
dental office: tongue) of person speaking; do NOT wear mask.
1. For visually impaired, NOT able to see objects d. Use of sign language (manual communication
in path or to visualize instructions or procedures that includes finger spelling), message board,
­being done. or interpreter if patient NOT able to read lips
2. For hearing impaired: (speech); provide written homecare instructions;
a. NOT able to understand instructions; fear or MUST ask patient which means of communica-
shock in response to unexplained procedures. tion is preferred and how communication can
b. May feel discomfort when hearing aids are on be improved.
during noisy procedures. F. Patient or caregiver education:
c. SHOULD be asked to turn off or take off hearing 1. For visually impaired:
aids or unhook implants when noisy powered de- a. Extremely descriptive explanations are
vices such as high-speed handpiece or ultrasonic ­IMPORTANT because they make use of other
or sonic scaling instruments are ­being used. senses (particularly hearing), which are better
d. Conduction of vibrations from dental equip- developed.
ment via teeth and bone can also be disturbing b. Use appropriate changes in tone of voice when
and such equipment should be used ONLY if providing information, since facial expressions
necessary. may NOT be seen.
o9000 e. Moisture is a consideration for hearing aids c. Involves explanation and demonstration (on the
and cochlear implants; moisture-proof ­surgical hand) of each procedure; patient should handle
534   Saunders Review of Dental Hygiene

visual aids to improve understanding; caregiver b. May use mirror, interpreter, sign language, or
should be involved as needed. message board to explain things that are NOT
2. For hearing impaired: easily demonstrated.
a. Use of demonstration is particularly effective c. Includes take-home written instructions for
when explaining techniques of oral care. ­effective reinforcement.

clinical study  

Age 70 YRS SCENARIO

Sex ⌧  Male   ■  Female Patient has kept regular dental visits


and was seen 3 months ago for his
Chief Complaint “My gums are really bleeding on this tooth!” routine maintenance appointment.
On intraoral examination the newly
Medical History Hearing impaired, hearing aids in both ears
hired dental hygienist finds that
Streptomycin toxicity as child
he has a very red marginal gingiva
Current Medications None around #20, which is a full crown.

Social History Post office employee

1. What should the dental hygienist do about the patient’s Box 16-2  Intellectual Disability Defined
hearing device? Where should the hygienist stand by ­Intelligence Quotient (IQ) Scores*
when speaking to him?
2. Of what significance is the fact that he has bleeding Normal intellectual functioning, IQ = 100
gums? Mild mental retardation, IQ = 55-70 (educable)
3. What is the significance of the redness around #20? Moderate mental retardation, IQ = 40-55 (trainable)
4. To whom should the hygienist refer the patient to Severe mental retardation, IQ = 25-40 (some training
­possible)
for #20?
Profound mental retardation, IQ <25 (total care is required)

1. The dental hygienist should face the patient ­ without


*Because of variability in testing, these classifications are NOT always
wearing the mask when speaking to him and have him ­appropriate and an individualized approach is always indicated.
turn off the hearing device during noisy procedures.
2. Bleeding gingiva could be indicative of a change in
his oral hygiene care level, and the hygienist must find by infection, trauma, or disturbance during fetal
out what the patient is doing and how to help him with development; by trauma or infection during birth;
this problem. or by trauma or nutritional deficiencies during
3. Marginal redness around #20 could indicate that ­childhood.
the crown is invading the biological width of the a. Down syndrome (trisomy 21): form particularly
­tissues. IMPORTANT in dentistry because of associ-
4. The dental hygienist can only refer patients to the gen- ated intraoral anomalies (short, narrow palate;
eral dentist, although it is known that this patient may large, fissured tongue) and risk factors for peri-
need to see a periodontist. However, the supervising odontal disease (Figure 16-1). Usually walk
general dentist must make that referral. with a “waddle” and have dexterity problems,
so homecare can be a problem.
Cognitive Disabilities b. Fetal alcohol syndrome (FAS): disorder asso-
Common cognitive disabilities include intellectual dis- ciated with prenatal exposure to alcohol. In-
ability (mental retardation), cerebral palsy, autism, tellectual disability (mental retardation), poor
­attention deficit–hyperactivity disorder or attention defi- coordination, behavioral disorders, growth
cit disorder, learning disorder, Alzheimer’s disease. Men- disturbances, and abnormal facial features (epi-
tal illness is considered separately in the next section. canthal folds, low nasal bridge, small head, and
A. Intellectual disability (mental retardation): MOST small mouth [micrognathia]) (Figure 16-2).
common developmental disability (Box 16-2). 2. Systemic concerns tend to be MORE extensive
1. Below-average intellectual functioning (IQ below in more severely intellectually disabled (mentally
70 to 75) caused by chromosomal disturbances; ­retarded) individuals.
Special Needs Patient Care   535

3. Oral signs:
a. Delayed or irregular tooth eruption; small, cone-
shaped, fused, or missing teeth; malocclusion.
b. Repercussions of mouth breathing and tongue
thrusting, with drooling (sialorrhea) possible;
cracked lips.
c. Increased risk of periodontal disease.
4. Risk factors:
a. At risk for systemic and local infections ­because
of weakened immune system.
b. Increased risk of hepatitis B infection if has
been institutionalized.
c. Difficulty modifying behavior.
d. Poor coordination.
5. Barriers to care:
a. Dependence on caretaker to make and keep
dental appointments.
Figure 16-1  Child with Down syndrome. (From Bath-Balogh M, b. Cost of dental care.
Fehrenbach MJ: Illustrated dental ­embryology and anatomy, ed 2,
Philadelphia, 2006, Saunders/Elsevier.)
c. Mental limitations:
(1) Build trust; communicate at developmental
level; speak simply.
(2) Reward good behavior; restraints and seda-
tion to manage behavior are recommended
ONLY when necessary.
6. Professional and homecare:
a. Frequent oral prophylaxis to reduce risk of
­periodontal disease.
b. Lubrication of lips to reduce risk of cracking.
c. Awareness that gag reflex may be strong.
d. Use of anticholinergic drugs for reducing ­excess
salivation during intraoral procedures.
7. Patient or caregiver education:
a. Repetition of simple, demonstrable homecare
procedures with patient and caregiver.
b. Caregiver supervises and/or performs oral
­hygiene procedures depending on abilities.
c. Discussion of periodontal risk and need for
­excellent daily homecare, frequent progressive
oral prophylaxis and examination.
B. Cerebral palsy (CP): limitation ranges from mild to
Figure 16-2  Child with fetal alcohol syndrome. (From severe.
Bath-Balogh M, Fehrenbach MJ: Illustrated dental ­embryology and 1. Developmental, neuromuscular disorder that
­anatomy, ed 2, Philadelphia, 2006, Saunders/Elsevier.) ­results in NO ability to control muscular move-
ment (spasticity).
2. Caused by variety of injuries to the brain (e.g., in-
a. Affected individuals tend to have characteristic fection, trauma, poisoning, anoxia), before, ­during,
physical traits and body stature; head, face, eye- or not long after birth.
lids, ears, nose may be malformed, depending 3. Systemic concerns: chronic contraction of mus-
on syndrome and severity. cles, poor coordination and dexterity, intellectual
b. Other medical problems commonly associated disability (mental retardation) in LESS than half,
with intellectual disability (mental retardation): learning disabilities caused by sensory impair-
(1) Leukemia. ment (hearing and vision), possibly respiratory
(2) Congenital heart abnormalities. ­impairment.
(3) Infectious hepatitis and respiratory infections. a. Affected individuals who have seizure disor-
(4) Epilepsy and neuromuscular disorders. ders take appropriate drugs, antiepileptics.
536   Saunders Review of Dental Hygiene

b. Communication is often difficult because of should be cleared of items that may cause injury
motor difficulties (see earlier discussion). during convulsive movement).
4. Oral signs: h. Using muscle relaxants, sedation, and general
a. Lack of control of facial muscles; causes dif- anesthesia when they will be of benefit.
ficulties in speech (dysarthria), chewing, and i. Using sheet, blanket, bean-bag chair to make
swallowing (dysphagia); may have drooling patient feel secure.
(sialorrhea). 8. Patient or caregiver education:
b. Difficulty keeping mouth open during dental a. Adaptations of toothbrushes and floss handles
appointments. as needed.
c. Temporomandibular joint disorder (TMD), b. Evaluation of need for powered cleaning
tongue thrusting, mouth breathing, bruxism, ­devices (toothbrushes and oral irrigators).
­attrition. c. Explanation and demonstration of all homecare
d. Caries and periodontal disease related to in- procedures; great patience may be necessary,
ability to practice good oral hygiene measures but many patients are willing to learn.
because of limited coordination. d. Daily disclosing of dental biofilm and assistance
e. Gingival hyperplasia if taking phenytoin with removal if patient NOT able to thoroughly
­(Dilantin); degree of hyperplasia is related cleanse own mouth.
to level of oral care (i.e., as dental biofilm in- e. Use of mineralizing fluoride and calcium
creases, so does hyperplasia). ­products and chlorhexidine to control disease
5. Risk factors: inadequate fluoride intake, soft diet, as needed; used twice daily, chlorhexidine glu-
poor motor control, with NO ability to properly conate sprays effectively reduce dental biofilm.
maintain own oral care. f. Explanation of need for frequent oral prophy-
6. Barriers to care: laxis.
a. Communication difficulties between patient C. Autism: lifelong, behavioral developmental disability
and dental professional; low self-esteem may of unknown etiology.
also influence desire to communicate. 1. Affects many MORE males than females, becomes
b. Unfamiliarity of dental office; causes emo- evident during first 3 years of life.
tional distress, thereby increasing spastic 2. Identified by NO ability to communicate appropri-
movement. ately; does NOT interact or communicate at age-
c. Dependence on caregiver, lack of mobility. appropriate level, making behavior management
d. Lowered ability of dental professional to pro- quite difficult.
vide thorough treatment because of patient’s 3. MOST have some degree of intellectual disability
physical limitations. (mental retardation).
7. Professional and homecare: 4. Can be present alone, but many also have intellec-
a. Building trust, desensitizing patient to dental tual disability (mental retardation), seizures, and
routines, encouraging complete communica- other problems.
tion. 5. MOST prefer routine (e.g., same foods, same way
b. Realizing that communication barriers do NOT of doing things), and MOST perform repetitive
indicate incomprehension. motions (e.g., hand wringing, head banging).
c. Avoiding injury to patient or clinician from 6. Treated by behavior modification, speech and play
uncontrolled movements during instrumenta- therapy, drugs, psychotherapy.
tion (fulcrums are a must); do NOT move chair 7. Systemic concerns:
without notice; padding or restraints may be a. Injuries caused by repetitive motions such as
necessary to provide assistance in cooperating head banging and biting.
for the procedure; may need wheelchair transfer b. Metabolic disorders.
(discussed later). c. Nutritional deficiencies (because diet typically
d. Preventing aspiration of water or other materi- limited to foods that are familiar).
als placed in the oral cavity. 8. Oral signs:
e. Using anticholinergic drugs to reduce excess a. NO difference noted, unless has received insuf-
salivation during intraoral procedures. ficient care.
f. Involving caretaker and/or assistant dur- b. May have tendency for oral trauma because of
ing ­ treatment to prevent injury and expedite being aggressive or injuring themselves when
­treatment. brushing.
g. Assisting during seizures (during seizure c. May have increased risk of caries if has ­high-
­activity, patient should NOT be moved; area carbohydrate diet.
Special Needs Patient Care   537

9. Risk factors: 2. May experience difficulty in performing tasks that


a. Fear of procedures and NOT understanding require sustained attention; keep appointments
need to sit still. short.
b. Inadequate nutrition because of strong food 3. See at BEST time of day for patient when “meds
preferences. are peaking.”
10. Barriers to care: 4. Drugs can make person crave sugar, so increased
a. Stress of the dental visit. risk of caries.
b. Communication difficulties because of poor 5. SAME general considerations as other cognitive
­behavior control (caregivers may be embar- disabilities.
rassed about child’s behavior). E. Specific learning disability (SLD): disorder in one or
c. Managing behavior: more of the basic psychological or neurological pro-
(1) Desensitization over multiple appoint- cesses involved in understanding or in using spoken
ments, reinforcement of good behaviors. or written language.
(2) Using physical restraint when safety is 1. Disorders may be manifested in listening, thinking,
­concern (see later discussion). reading, writing,
(3) Sedation and/or general anesthesia (if other 2. Examples: dyslexia, dysgraphia, dysphasia, dyscal-
methods fail). culia, and other specific learning disabilities in the
d. Reliance on the caregiver to make and keep basic psychological and neurological processes.
­appointments. 3. Such disorders do NOT include learning problems
e. Likes structure; keep procedures same or resulting primarily from visual, hearing, or motor
­similar. disabilities, intellectual disability (mental retarda-
11. Professional and homecare: tion), psychological disabilities, or environmental
a. Consistency in care and among care ­providers; deprivation.
preference for routine dictates that SAME 4. SAME general considerations as other cognitive
­dental team members should see patient at each disabilities.
visit. F. Alzheimer’s disease: causes increasing loss of mental
b. Shorter, MORE frequent appointments in a function, affects >65 years of age.
quiet, calm environment are preferable to lon- 1. Cause is unknown, but genetics, virus, brain ­injury,
ger, infrequent visits; noises, movement, and metal toxicity are suspect.
other changes are disconcerting and SHOULD 2. Have difficulty with cognition, depression, bladder
be avoided/introduced slowly as needed. and bowel control, tiredness, emotional control,
c. Caregiver should be involved in preparing child weight loss, other medical conditions.
for dental visit. 3. May take antipsychotic (neuroleptic) drugs that
d. Procedures should be explained to caregiver so increase risk of bleeding, Candida infections, and
that some can be practiced at home in prepara- other infections.
tion for dental visit. 4. Oral signs: speech difficulties and poor oral hy-
e. Homecare instructions SHOULD be performed giene because of loss of cognitive abilities; may
consistently on daily schedule. include trauma from falls or elder abuse.
12. Patient or caregiver education: 5. Risk factors:
a. Short appointments. a. Emotional stress and generalized depression;
b. Use of BOTH verbal and nonverbal techniques easily confused and function better in quiet,
of communication to demonstrate simple oral ­familiar environment.
care instructions. b. Behavior may be uncooperative; dental provid-
c. Discussion with caregiver about need to eat less ers SHOULD speak in short, simple sentences
cariogenic foods. and move slowly.
d. Need for frequent preventive dental visits to c. Change in vasoconstrictors with local anes-
create routine and avoid need for extensive thesia may be needed because of prescribed
treatment. drugs.
D. Attention deficit–hyperactivity disorder (ADHD) or 6. Barriers to care:
attention deficit disorder (ADD): persistent pattern a. Difficult communication because of cognitive
of severe problems with attention, concentration, difficulties.
restlessness, distractibility, and/or hyperactivity and b. Lack of transportation; must rely on others
­impulsivity. when not able to transport self.
1. Interferes with a person’s ability to function at c. Caregiver is NOT present to offer homecare in-
school, at work, or in relationships. structions for patient.
538   Saunders Review of Dental Hygiene

7. Professional and homecare: understands the difference between good health and
a. Scheduling short, relaxed appointments during poor health.
MOST alert times.
b. Reintroducing staff and procedures to enhance Mentally Ill Patient
familiarity. Clinicians should be aware of the signs, symptoms, and p0110
c. Providing concrete homecare instructions; clinical treatment of common mental illnesses or mental
MUST be repeated because of memory and be- disabilities so CORRECT dental management can be fol-
havior difficulties. lowed. MANY patients may be reluctant to talk about prob-
d. Reminders to brush and/or assistance with oral lem. MOST patients whose emotional problems affect oral
care tasks as dementia increases. health can be helped by clinicians who are ­knowledgeable
8. Patient or caregiver education: and sensitive. Thorough discussion and medical consult
a. Gaining assistance of caregivers when not are recommended in cases where ­ complex dental treat-
able to maintain appropriate oral cleanliness. ment is ­required; MUST be compliant with drug therapy
b. Importance of frequent dental recalls. recommended by their health ­professionals.
c. Empathy with difficulties family caregiver faces MOST drugs taken will have xerostomia as a side ef-
in caring for loved one. fect. Illness will produce reduced rates of compliance for
preventive oral healthcare, as well as reduced ability to ob-
s0130 clinical study   tain or tolerate needed dental treatment. May have atypical
s9000 Scenario: A 9-year-old patient with Down syndrome, odontalgia (AO) in which chronic throbbing and burning
p9000 with the developmental level of 6-year-old, communi- pain in oral cavity or temporomandibular joint is pres-
cates by sign language but communicates orally with ent ­ without clear cause, sometimes with severe bruxism.
people with whom she is comfortable. This is her first ­History of substance abuse is quite common; may try to use
dental ­appointment. drugs or alcohol to self-medicate; tobacco use is also com-
mon. ­Substance abuse may induce or amplify ­symptoms.
1. Down syndrome is known by another name. What A. Anxiety disorders: even MORE likely to fear dental
is it? Identify several physical characteristics of this treatment; include phobia, generalized anxiety disorder,
­syndrome. social anxiety disorder, agoraphobia, posttraumatic stress
2. What significance does this syndrome have in den- disorder, panic disorder, obsessive-compulsive disorder.
tistry? 1. Panic disorder: chronic, debilitating condition that
3. Down syndrome and other forms of intellectual dis- can have a devastating impact.
ability (mental retardation) are associated with a num- a. Typically, first attack strikes without warning;
ber of systemic concerns. Identify several of these. affected person worries that another one may
4. Describe how to provide brushing instructions to the occur at any time.
patient. b. During attack: pounding heart, sweating palms,
overwhelming feeling of impending doom; may
1. Trisomy 21 is another name for Down syndrome. last only seconds or minutes; experience can be
­Individuals inherited extra copy (or portion) of profoundly disturbing.
­chromosome 21. Common physical characteristics c. In severe cases refuses to leave the house for
include small skull, flattened nose, large protruding fear of having attack; can lead to fear of being
tongue, extra skin folds under eyelids. in exposed places (agoraphobia).
2. Often, have delayed tooth eruption, have small and ir- d. May be taking selective serotonin reuptake
regularly shaped teeth, are mouth breathers and tongue ­inhibitors (SSRIs).
thrusters, have cracked lips, and are at increased risk 2. Obsessive-compulsive disorder (OCD):
for gingivitis and periodontal diseases. Most have a a. Occurs with equal frequency in men and
small oral cavity and enlarged fissured tongue. May be women; may have familial tendencies; has two
at risk for caries (if crave sugar). components as the name implies:
3. Systemic concerns and manifestations (signs) associ- (1) Obsessive component: recurrent thoughts,
ated with intellectual disability (mental retardation) ideas, or images that are disruptive force in
include unusual physical appearance, increased risk sufferer’s life; involuntary and result in the
of leukemia, congenital heart abnormalities, infec- compulsive response.
tious hepatitis B (in institutionalized individuals), (2) Compulsive component: response to obses-
­neuromuscular disorders, respiratory infections. sions in attempt to relieve subsequent mount-
4. The patient is able to comprehend simple, repetitive ing anxiety; may be aware of performing these
­brushing instructions. Demonstration of technique is rituals but are unable to stop themselves.
particularly important means of communication. She b. May be taking SSRIs.
Special Needs Patient Care   539

B. Mood disorders: includes clinical depression: B. Paralysis: loss of motor, sensory, and autonomic func-
1. Clinical depression: unusually intense and sus- tion of parts of body located below area of spinal cord
tained sadness, melancholia, or despair. injury (Christopher Reeve will always be Superman
a. Accompanied by preoccupation over past minor to us!).
failings, complaints of bodily aches and pains 1. May be caused by trauma, congenital defect, infec-
without physiological basis, social withdrawal. tion, hemorrhage, or infarction.
b. Increasing prevalence among elderly: MOST a. Spinal cord injuries typically result in partial or
common emotional disorder in those >65 years complete paralysis from trauma to spinal cord;
old. many occur in children and young adults.
c. Higher levels of attachment loss and alveolar (1) MOST common causes include auto acci-
bone loss (MOST likely related to stress and dents, violent crime, sporting accidents.
smoking factors). (2) Traumatic injury results from physical
d. May be taking tricyclic antidepressants, MAO trauma that causes compression, fracture,
inhibitors. or severing of the spinal column; example
o9010 2. Bipolar (affective mood) disorder (manic-depressive is compression neck injury suffered during
illness): may cycle between depression and ma- diving accident.
nia (abnormally elevated mood); similar drugs as (3) Amount of injury is related to location and
noted above for depression as well as drugs termed extent of injury; generally, ALL motor, sen-
“mood stabilizers” such as lithium and sodium val- sory, and autonomic function is lost below
proate. the point of injury.
C. Psychotic disorders: thoughts and sensations that (a) Quadriplegia (tetraplegia): injury is in
do NOT always accurately represent the world cervical (neck) region, with loss of sen-
around them, such as schizophrenia and delusional sation and movement in all four limbs
disorder. and the trunk; loss of sensation and
movement may not be complete, with
Orthopedic Disabled Patient some sensation and movement retained
Patients may have orthopedic disability, including loss of in parts of arms and legs.
limbs, paralysis, cerebrovascular accident (CVA, stroke), (b) Paraplegia: injury is in thoracic, lum-
arthritis, muscular dystrophy, multiple sclerosis, Parkin- bar, or sacral region; loss of sensation
son’s disease, myasthenia gravis, amyotrophic lateral and movement in the legs and in part or
sclerosis. Cerebral palsy was discussed earlier. all of the trunk, varies according to the
May be fitted for palatal augmentation prosthesis level of the injury; generally, the lower
(PAP), provided to assist with feeding and swallowing the injury, the less the loss of movement
functions with impaired tongue function by reshaping of and sensation.
the hard palate to improve tongue-palate contact. May also (c) Hemiplegia: total or partial paralysis of
have drooling (sialorrhea); anticholinergic drugs are used one side of the body that results from
to reduce excess salivation during intraoral procedures. To disease or injury to the motor centers of
protect breathing for some, rubber dam may work if can the brain.
breathe through nose; patient should NOT be positioned (4) Spina bifida (myelomeningocele): congeni-
supine but more upright (45°). MOST have transportation tal defect of the spinal column in which
difficulties to the dental office because of ambulatory prob- some of spinal cord is displaced through
lems; MOST need to use canes, walkers, or wheelchairs; opening in spinal column; mother might
eventually must rely on others; wheelchair transfer will have been deficient in folic acid.
need to be done in the dental office (see discussion next). (a) Addition of partial or full paralysis is
A. Amputation: removal of limb or appendage through MORE common.
accidental injury or surgical procedure; amount of (b) May have hydrocephalus, excessive ac-
functional disability is dependent on type and extent cumulation of spinal fluid in the brain
of amputation. that causes excessive skull growth and
1. May have diabetes mellitus and/or vascular dis- brain compression in infants; may have
ease; SHOULD have medical consult before ­dental shunt for drainage.
care, including review of drug therapy. (c) Seizure may occur.
2. With upper limb amputations may have difficulty b. Oral signs: related to ability to care for own oral
with removable prostheses and oral hygiene. health; may include dental caries and periodon-
3. May be taking antidepressants and have xero­ tal disease if oral hygiene is poor and diet or
stomia. motor function is restricted.
540   Saunders Review of Dental Hygiene

c. Risk factors: 6. Risk factors: continued high blood pressure; 4 to


(1) Respiratory difficulty from having NO 6 weeks delay of emergency and elective dental
­ability to cough, spasms. care after episode; MUST meet 4 METs to deter-
(2) Pressure (decubitus) sores, infection. mine FC; may be taking anticoagulants and non-
(3) NO ability to maintain body temperature. steroidal antiinflammatory agents (NSAIAs) (risk
(4) Autonomic dysreflexia, serious increase for bleeding; know patient’s international normal-
in blood pressure from bowel and bladder ized ratio [INR]) (see Chapter 9, Pharmacology).
­irritation; can lead to emergency; provide 7. Barriers to care: communication if speech and
basic life support (BLS) and activate EMS memory affected (see earlier discussion); eco-
­system. nomic problems if income is restricted because of
2. Barriers to care: disability or fixed income.
a. Lack of communication when learning disabili- 8. Professional and homecare:
ties (spina bifida), depression, and poor self- a. Medical consult for antibiotic premedication if
­image (paralysis) are present. valvular replacement, congenital cardiac dis-
b. Economic loss from reduction in or loss of em- ease, or heart transplantation involving valves;
ployment because of severe disability. careful documentation of drug needs.
3. Professional and homecare: b. See above discussion of delay of treatment after o9020
a. Antibiotic premedication because of shunt for episode.
draining excessive fluid from brain. c. Caution when administering local anesthetics
b. Providing thorough dental hygiene care in because of possible interactions with prescribed
a comfortable, relaxed environment. drugs; may need to lower vasoconstrictor
c. Awareness of the risk for respiratory difficulty amount .
from accidental airway blockage when ­ motor 9. Patient or caregiver education: emphasis on daily
dysfunction such as spasticity and tremor meticulous homecare, frequent dental recall, and
­occurs. need to take antibiotic premedication before appoint-
d. Maintenance of good oral hygiene and use of ment if needed and cardiac drugs as ­prescribed.
fluorides and calcium products to prevent ­dental D. Arthritis, COMMON disorder of musculoskeletal
caries. system that causes painful swelling of the body joints;
e. Encouragement and empathy; extremely IM- may have joint replacement.
PORTANT to develop rapport. 1. May be caused by infection, allergy, trauma, drug
4. Patient or caregiver education: reactions, or heredity; results in fatigue and loss of
a. Emphasis on self-care, although assistance of mobility and hand strength.
caregivers may be sought when efforts to main- 2. MORE common forms: rheumatoid arthritis (RA),
tain good oral hygiene are not feasible. juvenile rheumatoid arthritis, degenerative joint
b. Adaptive equipment and aids, including pow- diseases (osteoarthritis).
ered toothbrushes and oral irrigators. 3. RA with synovitis of peripheral joints: MOST
c. Need for mineralizing fluoride and calcium commonly occurs at 20 to 60 years of age; morn-
products. ing stiffness; symmetrical joint swelling and pain
C. Cerebrovascular accident (CVA, stroke): loss of brain with motion; clinical and radiographic findings in-
function. clude soft tissue swelling (nodules) and erosions,
1. Caused by loss of blood flow to the brain via clot, usually at proximal interphalangeal joints, ulnar
constriction, or rupture of blood vessel supplying deviation in phalanges, and joint space narrowing
the brain. (Figure 16-3).
2. Underlying diseases (e.g., hypertension, diabetes 4. Arthrogryposis: LESS common form caused by
mellitus, drug abuse, atherosclerosis) typically are decreased fetal movements in the womb; fetus
cause of constriction or tear. needs to move his or her limbs to develop muscle
3. Can have temporary or permanent loss of thought, and joints; if the joints do not move, extra connec-
memory, speech, sensation, motion. tive tissue develops around the joint and fixes it in
4. Side of brain affected is opposite to that of brain place (talonlike appendages).
injury; right hemiplegia (one sided) has to do with 5. Oral signs: increase in bleeding and oral infec-
verbal loss and left with physical loss. tion from antiinflammatory drugs (NSAIAs, cor-
5. Oral signs of CVD: NO greater, unless severely ticosteroids); TMD may be present (use of heat,
debilitated; may have difficulty with mouth clo- exercise, antiinflammatory drugs, analgesics,
sure and drooling (sialorrhea); can result in tongue ­surgery, prosthodontic rehabilitation); patients
being deviated to one side. with RA may also suffer from Sjögren’s syndrome,
Special Needs Patient Care   541

6. Barriers to care:
a. Lack of communication; speech difficulties oc-
cur as muscle weakness affects muscles of head
and neck.
b. Immobility and difficulty controlling movement.
c. Economic issues; caregivers are needed to pro-
vide full care as disorder progresses.
d. Dependence on caregiver to make and keep
dental appointments.
7. Professional and homecare:
a. Frequent oral prophylaxis to reduce risk of
­infection.
b. Short dental appointments.
Figure 16-3  Patient with rheumatoid arthritis. c. Use of bite block to keep mouth open after
­muscle loss prevents it (see later discussion).
8. Patient or caregiver education:
d­ ecreased saliva production that requires aggres- a. Supervision and/or performance of oral hygiene
sive preventive care. procedures by caregiver, if patient is not able.
6. Barriers to care: long appointments; keeping mouth b. Adaptive aids to accommodate muscle
opened can be uncomfortable; irritability is COM- ­weakness.
MON in those with chronic pain. c. Power-assisted devices to enable self-care.
7. Risk factors: side effects of drugs and difficulty d. Discussion with patient and/or caregiver about
with motor activities. risk of infection and need for excellent daily
8. Professional and homecare: assessing need for an- homecare and frequent professional oral pro-
tibiotic premedication in cases of joint replacement phylaxis and examination.
by medical consult, need for short appointments e. Use of lubricant on lips and oral tissues irritated
with frequent opportunities to close the mouth and by open mouth.
need to shift positions in the chair to relieve dis- F. Multiple sclerosis (MS): incurable, chronic, progres-
comfort. sive disease distinguished by demyelination of CNS
9. Patient or caregiver education: discussion of oral nerve sheaths; can go into remission for years, then
side effects of arthritis drugs; recommendation undergo exacerbation and relapse and resurface.
for powered toothbrushes and other adaptive aids 1. Sclerotic plaques replace myelin nerve sheaths;
(enlarged toothbrush handles, floss holders) as ­affects 20- to 40-year-olds.
needed; recommendation for frequent recall so 2. Cause is unknown, although autoimmune disease
that appointments are quicker and easier. and virus infection are suspected.
E. Muscular dystrophy (MD): inherited, progressive 3. Normal or near normal life expectancy can be
skeletal muscular disorder. ­expected; symptoms include general fatigue,
1. Duchenne (severe) MD: begins in early childhood and ­weakness, and numbness of body parts during
results in death by age 25; affects MAINLY males. ­periods of activity; heat sensitivity may occur.
2. Becker (mild) MD: begins in middle to late child- 4. Treated with drugs; may be taking corticosteroids,
hood and does NOT result in early, premature muscle relaxants, antidepressants, immunosup-
death; affects BOTH males and females. pressants.
3. Systemic concerns: muscle weakness caused by 5. Oral signs: facial pain, speech disorders (see ear-
replacement of skeletal (eventually heart and lung) lier discussion), facial paralysis in severe cases.
muscle; may have difficulty walking, standing, 6. Risk factors:
speaking, eating as disease progresses. a. Physical and emotional stresses, such as in-
4. Oral signs: related to loss of muscle control, fections, emotional stress, inadequate rest,
may include injury or infection; irritated gin- ­excessive exercise, depression.
giva may be caused by open mouth; poor oral b. Side effects of drugs; diet restrictions caused by
hygiene may occur because of reduced ability to muscle dysfunction.
provide self-care. 7. Barriers to care:
5. Risk factors: trauma from injury (e.g., falling, dif- a. Communication difficulties from dysfunction
ficulty swallowing, choking) because of loss of of speech muscles.
muscle control; oral infection may be caused by b. Economic issues when income is restricted
inadequate dental biofilm removal. ­because of reduction in or loss of employment.
542   Saunders Review of Dental Hygiene

8. Professional and homecare: 8. Professional and homecare:


a. Scheduling short, relaxed appointments, prefer- a. Scheduling appointments during MOST active
ably during times of remission. periods of the day, soon after drugs are taken
b. Keeping at comfortable temperature. and after good night’s sleep.
c. Sedation, general anesthesia, or hospitalization b. Availability of emergency equipment for respi-
may be required before treatment is provided. ratory distress during appointments.
d. Attention to proper cleansing when facial c. Homecare instructions related to preventing
­paralysis and weakness reduce ability to cleanse oral infections, which are risk factors for crisis
cheeks and tongue. development.
e. Homecare instructions related to frequent d. Powered toothbrushes and oral irrigators are
cleansing of the oral cavity; powered tooth- recommended for those who have difficulty
brushes, adaptive aids, and oral irrigators may manually cleaning the oral cavity.
be recommended for those who have difficulty e. Careful attention to proper nutrition, which be-
cleaning manually. comes MORE important when food selections
9. Patient or caregiver education: are limited by LESS ability to chew or swallow
a. Family assistance in maintaining good daily some foods.
oral hygiene when no longer able to do so. 9. Patient or caregiver education: keeping oral cavity
b. Daily fluoride supplementation. in excellent condition to reduce risk of crisis; fam-
c. Maintenance of adequate diet. ily members may be enlisted to assist in frequent
G. Myasthenia gravis (MG): autoimmune disease that or daily oral care procedures and transport patient
affects neuromusculature. to the dental clinic when no longer able.
1. Decrease in number of acetylcholine receptors re- H. Amyotrophic lateral sclerosis (ALS, Lou Gehrig’s dis-
sults in decreased transmission of nerve impulses. ease): progressive, fatal, neurodegenerative disease.
2. Causes generalized feeling of exhaustion from 1. Unknown etiology; degeneration of motor neu-
muscle fatigue and weakness. rons, nerve cells in CNS that control voluntary
3. Causes paralysis in its MOST severe form, muscle movement.
MAINLY affects younger women and older men. 2. Begins with muscle weakness and atrophy
4. Treated by anticholinesterase drug, surgical re- throughout the body, as BOTH upper and lower
moval of thymus, or steroidal hormones. motor neurons degenerate and die, ceasing to send
5. Oral signs: effects on facial and cervical muscula- messages to muscles.
ture; leads to loss of control of facial muscles and 3. Then NOT able to function, muscles gradually
results in difficulties with smiling, eating, swal- weaken, atrophy, develop fasciculations (twitches)
lowing, speaking, vision; when severe, can cause as result of denervation.
respiratory distress. 4. Treatment: palliative; patient may be taking mus-
6. Crisis can occur because of infection (oral in- cle relaxants and antiepileptics.
cluded), emotional stress, and surgery; two types 5. Barriers to care:
of crises: a. Communication difficulties if patient experi-
a. Myasthenic crisis: causes loss of swallowing ences facial weakness and paralysis.
and speaking ability and difficulty breathing b. Economic issues for those on fixed incomes.
and seeing; caused by undermedication, un- 6. Professional and homecare:
derlying illness, risk factors, or worsening of a. Short appointments.
disease. b. Powered toothbrushes and oral irrigators are
b. Cholinergic crisis: related to overmedication recommended for those who have difficulty
with anticholinesterase, occurs within hour of manually cleaning the oral cavity.
drug use, causes increased muscle weakness, c. Careful attention to proper nutrition, which be-
gastrointestinal upset, respiratory difficulties. comes MORE important when food selections
c. For BOTH crises, provide BLS and activate are limited by LESS ability to chew and swal-
EMS system. low some foods.
7. Barriers to care: 7. Patient or caregiver education: keeping oral cav-
a. Communication difficulties, if experiences ity in excellent condition; family members may
­facial weakness and paralysis (patient may hold be enlisted to assist in frequent or daily oral care
chin with hand to speak). ­procedures.
b. Generalized fatigue affects ability to get to the I. Parkinson’s disease: slow, progressive degeneration
dental office. of basal ganglion neurons; results in loss of control of
c. Economic issues for those on fixed incomes. voluntary musculature.
Special Needs Patient Care   543

1. Inability to produce adequate levels of dopamine,


brain chemical, related to degeneration, although POSITIONING AND STABILIZATION  
exact cause is unknown; possible causes are ex- Effective patient positioning and stabilization provide for
posure to environmental toxins, such as pesticides the well-being of patient and efficiency of clinician by
and mercury, or head injury. offering comfortable, safe environment. However, use of
2. Affects MORE men than women, tends to occur preventive measures SHOULD be considered for elective
during middle to later years of life (ages 50 to 65). dental care needs or even treatment deferral if patient’s
3. Pill-rolling motion of the fingers, masked facial behavior is uncontrollable and/or hysterical.
appearance, muscle rigidity, but understands clini- • See Chapter 16, Special Needs Patient Care: modifica-
cian, UNLIKE patients with Alzheimer’s. tions for medical disabilities.
4. Oral signs: drooling (sialorrhea), difficulty swal- A. Includes stabilization and positioning of patient chair
lowing, tremors, reduced ability or inability to care and patient’s head and body.
properly for mouth because of motor dysfunction, 1. Requires knowing when a patient is MORE com-
which may result in oral infection; speech may be fortable, either sitting upright or lying supine for
stammering and monotone (see earlier discussion); proper care.
xerostomia is COMMON side effect of drugs. a. Patients prone to decubitus ulcers, such as para-
5. Treatment: drugs such as levodopa (L-dopa, plegics, require careful readjustment of body
Madopar, Sinemet) transformed into dopamine or position and additional cushioning during long
surgery to provide relief from symptoms. appointments.
a. However, long-term use of L-dopa often leads b. See earlier discussion for each patient’s health
to motor complications that can be difficult to concern.
manage. 2. Support and restraint devices: MOST commonly
b. Dopamine agonists are used early, such as used with neuromuscular disorders or extreme be-
bromocriptine mesylate (Parlodel), pergolide havioral problems that put patient and/or clinician
mesylate (Permax), pramipexole (Mirapex), at physical risk from movement during instrumen-
­ropinirole hydrochloride (Requip). tation; should NOT be used routinely; sedation is
(1) Helps put off need for levodopa and im- another option but is used as a last resort.
prove motor function. a. Mouth props (blocks): rubber materials that
(2) Side effects include orthostatic hypoten- are used to hold the mouth open for patients
sion, irregular heartbeat, and chest pain just who are NOT able; some are placed in a closed
as with L-dopa. position and gradually opened as needed; oth-
6. Risk factors: difficulty with swallowing with re- ers place the jaws in an open position at all
stricted diet because of eating difficulties, difficulty times.
walking. b. Head stabilization: use of clinician’s arm to
7. Barriers to care: surround patient’s head, cup the chin, and hold
a. Communication difficulties because of embar- it still during procedures.
rassment about condition; slurred speech from c. Body wraps: sheets and blankets used to
motor dysfunction occurs later in course of wrap and secure the patient with straps; used
­disease. MAINLY for young children when full-body
b. Economic difficulties associated with loss of restraint is necessary.
employment and reliance on fixed income dur- d. Papoose boards: boards with padded straps
ing later years. that are used to secure patients of ANY age or
8. Professional and homecare: size during operatory procedures.
a. Short appointments. B. Wheelchair transfer to dental chair: depending on
b. Making patient comfortable during appoint- ­severity of disability:
ments, use of stable instrumentation to deal with 1. Know patient’s abilities; may be able to do it alone
tremors, and active listening and encourage- or with assistance.
ment, which are IMPORTANT to sense of well- a. Sliding board: board is made of hardwood
being. with smooth tapered ends.
9. Patient or caregiver education: frequent and proper b. Used when patient can transfer alone and ONLY
care of oral cavity, with emphasis on use of ­adaptive needs assistance, such as with paraplegics.
aids as needed; as disease progresses, family ­ 2. Use body and NOT back to move patient, pivoting
assistance should be requested to maintain oral health; from hips; lift patient, do NOT slide patient into
frequent dental recall is recommended; emphasize chair.
use of larger handled powered toothbrushes. 3. Single-provider wheelchair transfer (Figure 16-4):
544   Saunders Review of Dental Hygiene

b. Second clinician: place both hands under the


patient’s lower thighs; initiate and lead the lift
at a prearranged count (1-2-3-lift).
c. Both clinicians: using your leg and arm muscles
while bending your back as little as possible,
gently lift the patient’s torso and legs at the
same time.
C. Behavior management:
1. Involves familiarity with special characteristics
and needs of patient.
a. Patience with and empathy for the patient.
b. Dental visits are MORE successful if the
­patient is familiarized with the office by fre-
quent, short visits during which procedures are
­demonstrated.
2. Inclusion of caretakers in discussions of behavior
management.

Figure 16-4  Single-person wheelchair transfer of a patient Review Questions


to the dental chair.

1 Aphasia, apraxia, and dysarthria are examples of ________


a. Determine whether special padding should be disabilities.
transferred. A. communication
b. Determine whether urine-collecting device is B. developmental
present. C. cognitive
D. sensory
c. Determine whether there is need to secure limbs
E. nervous system
because of spastic movements. 2 Patients who can bathe, feed, and dress themselves but may
d. Clear the area, and position wheelchair at same need to be reminded to brush their teeth and hair are consid-
or lower height and in direction of and slight ered _________ functioning.
angle to dental chair. A. high
e. Secure transfer belt around waist and remove B. moderate
or release dental chair and wheelchair armrests; C. low
set the brake. D. not
f. Determine whether any special equipment must 3 When discussing homecare procedures with the mildly
be transferred or might be in the way and lift the mentally retarded individual,
footrests. A. speak directly to the caregiver.
B. use simple demonstrations.
g. While facing the patient, place your feet on ei-
C. send home printed materials.
ther side of the patient’s feet. D. discuss the risk of periodontitis.
h. With your knees holding patient’s legs stable, 4 The patient with cerebral palsy is likely to have difficulty
grasp patient under the arms and around the with all of the following, EXCEPT one. Which one is the
back to grasp the transfer belt. EXCEPTION?
i. Place the patient in a standing position. A. Controlling movement
j. Rotate patient into the dental chair in a smooth B. Communicating
movement. C. Caries risk
k. Position patient on the dental chair; place pad- D. Oral hygiene
ding as needed and straighten clothing. E. Comprehension
l. Thank patient for assistance. 5 When treating a patient with autism,
A. use nonverbal instructions.
4. Two-provider wheelchair transfer: for heavier and/
B. follow a consistent routine.
or more fragile patient. C. speak in a louder tone of voice.
a. First clinician: stand behind the patient; help D. schedule longer visits.
the patient cross his or her arms across chest;
place your arms under the patient’s upper arms
and grasp the patient’s wrists.
Special Needs Patient Care   545

6 The inability to properly form speech sounds is termed 15 All of the following disorders may result in facial paralysis,
A. apraxia. EXCEPT one. Which one is the EXCEPTION?
B. aphasia. A. Multiple sclerosis
C. dysarthria. B. Alzheimer’s disease
D. ataxia. C. Bell’s palsy
7 When providing oral healthcare instructions to a visually D. Myasthenia gravis
impaired individual, it is MOST important to 16 Which of the following conditions may indicate the need
A. talk more loudly. for a patient to be repositioned frequently during a dental
B. speak more slowly. appointment?
C. face the patient. A. Spina bifida
D. use visual aids. B. Multiple sclerosis
8 All of the following are oral manifestations associated with C. Down syndrome
arthritis, EXCEPT one. Which one is the EXCEPTION? D. Parkinson’s disease
A. Angular cheilitis 17 Which of the following is a device used specifically for the
B. Gingival bleeding full-body restraint of young children?
C. Oral infections A. Mouth prop
D. TMJ discomfort B. Head stabilization
9 Which one of the following adaptive aids would MOST C. Wraps
benefit the patient with arthritic joints of the hands? D. Papoose boards
A. Extended-handle toothbrush 18 When performing a wheelchair transfer, the dental profes-
B. Textured-handle toothbrush sional must do all of the following, EXCEPT one. Which
C. Enlarged-handle toothbrush one is the EXCEPTION?
D. Curved-handle toothbrush A. Clear the area of any obstacles
10 Which of the following is noted with a cerebrovascular B. Secure a transfer belt to the patient
­accident? C. Raise the chair above wheelchair height
A. Sudden loss of blood flow to the heart D. Stabilize the patient’s legs
B. Caused by restriction in or rupture of a blood vessel E. Grasp the patient under the arms and around back
C. Often preceded by rheumatic heart disease 19 The LEAST likely oral manifestation of Down syndrome is
D. Requires NO modification of dental procedures after A. early tooth eruption.
­incident B. fusion of teeth.
11 Patients who have suffered spinal cord injuries may have C. malocclusion.
difficulty performing oral care. When teaching oral health- D. tongue thrust.
care measures, the clinician should emphasize E. missing teeth.
A. self-care. 20 All of the following are moderate level of function (level
B. proper nutrition. III), EXCEPT one. Which one is the EXCEPTION?
C. good oral hygiene. A. The person may need supervision or assistance with
D. caregiver assistance. some of the activities of daily living.
12 The level of paralysis or dysfunction associated with a spi- B. The person may require use of gestures or demonstra-
nal cord injury is MOST closely related to the tion or use of adaptive equipment for communication.
A. type of traumatic injury. C. The person is able to give informed consent.
B. severity of traumatic injury. D. The person is unable to give informed consent.
C. location of the traumatic injury. 21 Education by the dental hygienist for a disabled patient
D. cause of traumatic injury. should emphasize discussion about oral health and need for
13 Which of the following statements regarding the patient excellent oral care, and the caregivers should hear the in-
with Alzheimer’s disease is CORRECT? structions at the same time as the patient.
A. Alzheimer’s disease is a form of dementia that tends to A. Both statements are true.
affect the oldest of the old. B. Both statements are false.
B. Patients are confused by change and prefer quiet C. The first statement is true, the second is false.
­surroundings. D. The first statement is false, the second is true.
C. A single long appointment is preferable to several short 22 The ultrasonic scaler should never be used on a hearing-
appointments. impaired person because the conduction of vibrations
D. Most patients can take care of their oral health needs from dental equipment via teeth and bone can be very
without the assistance of a caregiver. ­­disturbing.
14 The disease that affects the myelin sheath and results in fa- A. Both the statement and reason are correct and related.
tigue and numbness is B. Both the statement and reason are correct but NOT
A. muscular dystrophy. ­related.
B. multiple sclerosis. C. The statement is correct, but the reason is NOT.
C. Parkinson’s disease. D. The statement is NOT correct, but the reason is
D. Bell’s palsy. correct.
E. myasthenia gravis. E. NEITHER the statement NOR the reason is correct.
546   Saunders Review of Dental Hygiene

23 All of the following are cognitive disabilities, EXCEPT 2 ( A)  Assessment of functional levels is from high (re-
one. Which one is the EXCEPTION? quires little assistance with ADLs) to low (caretaker
A. Mental retardation provides MOST or all ADLs). High-functioning
B. Cerebral palsy ­patient can bathe, dress, eat, communicate, ambulate­
C. Autism
well enough to ­ function with ONLY minimal assis-
D. Parkinson’s disease
tance. Patient may require ­reminders to perform tasks.
24 Lack of folic acid in pregnancy can result in
A. spina bifida. 3 (B)  When discussing homecare procedures with
B. cerebral palsy. a mildly intellectually disabled (mentally retarded)
C. deafness. individual, make sure to use simple and demonstrable
D. short attention span. instructions. Include caregiver so that the caregiver
25 When an adult patient is a paraplegic, clinicians should can repeat the instructions as needed. Sending home
always assess what the patient can do himself or herself printed materials and discussing risks for periodon-
before helping the patient get into the dental chair. The pa- tal disease may be beneficial to the caregiver but are
tient will always need to be wrapped in sheets or blankets NOT appropriate for educating the patient.
during dental appointments. 4 (E)  Individuals with cerebral palsy typically are able
A. Both statements are true.
to comprehend instructions; LESS than half are men-
B. Both statements are false.
tally ­ retarded (intellectually disabled) and in need
C. The first statement is true, the second is false.
D. The first statement is false, the second is true. of simpler instructions. Have difficulty with motor
26 Levodopa (L-dopa, Madopar, Sinemet) is used to treat control, speaking, performing homecare procedures,
A. Parkinson’s disease. which often results in increased caries risk.
B. Alzheimer’s disease. 5 (B)  MOST patients with autism prefer routine and
C. specific learning disability. familiar procedures and surroundings. Instructions
D. depression. should be BOTH nonverbal and verbal and kept
27 Pill-rolling motion of the fingers has to do with which of the simple.­ Tone and loudness of the voice SHOULD be
following diseases? normal. Shorter dental visits are preferred.
A. Multiple sclerosis 6 (A)  Apraxia is inability to properly form speech
B. Rubeola
sounds. Aphasia is inability to understand language
C. Parkinson’s disease
or put thoughts into words, and dysarthria is slurring
D. Cerebral palsy
28 Aphasia is noted for being a(n) of speech patterns. All are considered communication
A. inability to put thoughts into words or to understand disorders. Ataxia is NOT a communication disorder
­language. but is a motor disorder that affects ambulation.
B. inability to properly form speech sounds. 7 ( C)  Always face a visually impaired patient when
C. motor speech disorder. ­providing oral healthcare instructions to individual.
D. loss of memory or thought. Speaking more loudly or more slowly is MORE
29 Athetosis, ataxia, and rigidity are associated with which of ­appropriate for dealing with hearing impairment.
the following illnesses or conditions? ­Visual aids may be used but MUST be enlarged to be
A. Parkinson’s disease of benefit.
B. Cerebral palsy
8 (A)  Gingival bleeding and oral infections are associ-
C. ADD
ated with drugs that are used to treat arthritis. TMJ
D. Paraplegia
30 Hydrocephalus discomfort occurs when the disease has affected the
A. is fluid around the heart and lungs. joint; may even be associated with TMD, disorder
B. is fluid in the spinal column. of the joint. Angular ­cheilitis is associated with den-
C. is fluid in the brain. ture wearing, malnutrition, and candidal infection but
D. has nothing to do with fluid, is just an enlarged head. NOT with arthritis.
9 (C)  Toothbrush with larger handle would be MORE
­beneficial for patient with arthritic joints of the hands.
Answer Key and Rationales Many arthritics have difficulty with grip strength.
Larger brush handle would allow greater control.
Extended-handle toothbrush is unnecessary unless
1 (A)  Communication disabilities are those related to the patient has difficulty raising the arms. Textured-
neurological brain damage that disturbs language handle toothbrush is helpful to prevent slipping but
and speech development. Aphasia is inability to put ONLY if on a larger handle that improves the gripping
thoughts into words or to understand language; apraxia ability of the arthritic. Curved-handle toothbrush is
is inability to form speech sounds; and dysarthria­ appropriate for reaching molar areas for patients with
­results in slurred speech patterns. ­limited arm movement.
Special Needs Patient Care   547

o6510 10 (B)  Restriction in or rupture of a blood vessel causes ­typically do NOT suffer from pressure ulcers unless
cerebrovascular accident (CVA, stroke). Involves bedridden or confined to wheelchair for long periods.
sudden loss of blood flow to the brain, NOT to the 17 (C)  Wraps are devices used specifically for full-body
heart. NOT associated with rheumatic heart disease, restraint of young children. Often sheets and blankets­
and often preceded by underlying diseases such as are wrapped around the child, from neck to toes.
hypertension, diabetes, drug abuse, or atherosclerosis. Mouth props and head stabilization secure ONLY the
Medical consult with patient’s primary care physician jaws and head. ­Papoose boards are another method of
is important because some modification of dental pro- full-body restraint and can be used to secure patients
cedures may be necessary, particularly within the 4 to of any age.
6 weeks after the incident, since there may need to be 18 (C)  When performing a wheelchair transfer, clinician
a delay of emergency and elective dental care. must clear the area of any obstacles, secure a transfer
11 (A)  Patients who have suffered spinal cord injury belt to the patient, and lower the chair at or below
should be taught oral self-care. Depression and low wheelchair height. While facing the patient, the clini-
self-esteem often accompany this disorder. Use of cian stabilizes the patient’s feet with clinician’s feet
adaptive aids, powered toothbrushes, and oral irriga- and legs on either side of patient’s knees, grasps the
tors gives patients some control over their own care. patient under the arms and around the back, lifts, and
Discussion of proper nutrition and good oral hygiene rotates the patient to the dental chair.
also is important. When necessary, caregiver can offer 19 (A)  Individuals with intellectual disability (mental
assistance to the disabled patient. retardation) often exhibit delayed (NOT early) tooth
12 (C)  Location of traumatic injury to the spine is eruption; fused, pegged, or missing teeth; malocclu-
MOST closely related to the degree of paralysis or sion; tongue thrusting; mouth breathing.
dysfunction associated with the spinal cord injury. 20 (C)  When the person is able to give consent, this is
Type, severity, cause of injury influence injury in lo- considered a high level of function (level I to II). An-
cation (vertebral level). swers A and B are moderate levels of function, and
13 (B)  Alzheimer’s disease, form of dementia, often answer D is both moderate and low.
affects individuals in their middle-to-later years. In- 21 (A)  Both statements are true and also related. Impor-
dividuals often are confused by change and prefer tant for the caregiver for a disabled patient giving oral
quiet surroundings. For this reason, shorter, MORE care to the patient to hear what is being advised by the
frequent appointments are ­ preferable to single, lon- dental hygienist and for the patient to know the type
ger appointments. Appointments should be relaxed of care that the caregiver will be giving.
and involve familiar routines. Oral healthcare instruc- 22 (D)  Statement is not correct, but reason is correct.
tions should be repetitious and simple. During the Conduction of vibrations from dental equipment such
early stages of disease, MOST patients can take care as ultrasonic scaler via teeth and bone can be very
of own oral health needs with just a reminder from disturbing to the hearing impaired and such scalers
caregiver. During later stages, caregiver will need to are used ONLY if necessary.
take over MORE of care. 23 (D)  All are cognitive disabilities EXCEPT Parkinson’s
14 (B)  Multiple sclerosis (MD) affects myelin sheath and disease. UNLIKE patient with Alzheimer’s, patient­
results in fatigue and numbness. Inherited disease in with Parkinson’s can understand the clinician.
which muscles atrophy and death may result. Myas- 24 (A)  Spina bifida (myelomeningocele) may be caused
thenia gravis (MG) and Parkinson’s disease are disor- by a lack (deficiency) of folic acid in pregnant wom-
ders of the nervous system. Bell’s palsy is a disease an’s diet. Cerebral palsy is usually caused by variety
involving facial paralysis and typically is of unknown of injuries to the brain (e.g., infection, trauma, poi-
cause, although any trauma to the facial nerve may soning, anoxia) before, during, or not long after birth.
result in this paralysis. Deafness can be side effect and result of the use of
15 (B)  Alzheimer’s disease affects cognition and is NOT antibiotic streptomycin (NO longer used) and/or
­associated with paralysis. Multiple sclerosis, Bell’s exposure­ of the pregnant woman to rubella (German
palsy, and myasthenia gravis are disorders that may measles). Short attention span has to do with attention
result in facial paralysis. deficit disorder (ADD).
16 (A)  Spina bifida, defect of neural tube development, 25 (C)  First statement is true and the second is false.
often results in paralysis below point of defect. Para- With paraplegic adult patients, SHOULD always as-
plegics, and others with paralysis, need to be repo- sess what they can do themselves before helping into
sitioned frequently during a dental appointment. the dental chair. This is because paraplegia is an injury
Additional cushioning also helps prevent decubi- in the thoracic, lumbar, or sacral region, with loss of
tus (pressure) ulcers. Patients with multiple sclero- sensation and movement in the legs and in part or all
sis (MS), Down syndrome, or Parkinson’s disease of the trunk, which varies according to the level of the
548   Saunders Review of Dental Hygiene

injury. Generally, the lower the injury, the less the loss 28 (A)  Aphasia is an inability to put thoughts into words
of movement and sensation. Wraps used MAINLY for or to understand language. Apraxia is the inability to
young children when full-body restraint is necessary; properly form speech sounds. Dysarthria is a motor
sheets and blankets are similarly used to wrap and se- speech disorder. Alzheimer’s disease has the loss of
cure the patient with straps. memory or thought.
26 (A)  Levodopa (L-dopa, Madopar, Sinemet) is used to 29 (B)  Cerebral palsy is associated with athetosis, ataxia,
treat Parkinson’s disease. Alzheimer’s is NOT treated and rigidity. Pill-rolling motion of the fingers is asso-
by specific medications; ADD is usually treated with ciated with Parkinson’s disease; short attention span
methylphenidate (Ritalin) or pemoline (Cylert); de- is associated with ADD; paralysis is associated with
pression is treated with ­antidepressants. paraplegia.
27 (C)  Pill-rolling motion of the fingers has to do with 30 (C)  Hydrocephalus is fluid in the brain, associated
Parkinson’s disease. Myelin sheath and Schwann’s frequently with spina bifida, although it can also lead
cells are related to multiple sclerosis; Koplik’s spots to an enlarged head.
are related to rubeola (measles); spastic movements
are related to cerebral palsy.
CHAPTER 17

Community Oral Health


UTILIZATION OF DENTAL SERVICES  p­ rocedures, emergency care, preventive services,
Use of dental services is defined as the proportion of the or combination.
population who receive dental care services within given 5. Poor children have ∼12 times MORE restricted-
period. Factors that affect frequency include age, gen- activity days because of dental-related illness than
der, economics, ethnicity, geographical location, general children from higher income families.
health status, acquisition of dental insurance. Factors in- D. Currently, individual who MOST regularly uses
fluence and affect each other, interrelationship makes it dental services can be described as a white, college-
difficult to determine exact influence of each component. ­educated woman with a higher than average income,
• See CD-ROM for Chapter Terms and WebLinks. lives in suburban area, has dental insurance, possesses
A. Needs: perceived vs. normative: the characteristics related to good to excellent general
1. Younger children with perceived needs (needs per- and oral health.
ceived by child or responsible adult) were MORE
likely to be episodic users of dental care than Payment for Dental Care
­children without perceived needs. Dental services are paid for by several means, includ-
2. Younger children with normative needs (defined by ing fee-for-service; third-party plans; usual, customary,
presence of untreated caries diagnosed by ­dentist) reasonable (UCR) fees; capitation plans; dental ser-
were less likely to be regular users. vice corporation plans (e.g., Delta Dental); indemnity
3. Older children with perceived or normative needs plans; direct reimbursement; managed care plans; public
were MORE likely to be episodic users, less likely financing.
to have had previous-year visit than children with A. Fee-for-service payment: two-party plan whereby the
no needs. individual who receives dental services pays the fee
B. National dental visits (need to know only approximate directly out of pocket to the provider (dentist).
values when given for dental health concepts): B. Third-party plans:
1. Only 76% of children ages 2 to 17 with dental visit 1. Include BOTH not-for-profit and for-profit plans.
in past year. a. Involve a contract between dental office (first
2. Only 64% of adults ages 18 to 64 with dental visit party), patient (second party), insurance com-
in past year. pany (third party).
3. Only 56% of adults ages ≥65 with dental visit in b. Involve collection of premiums from patient by
past year. the third party, which in turn pays dental pro-
C. Use of dental services is disproportionate among pop- vider for services rendered.
ulation group (factors noted above): c. Regard third party as the insurance company,
1. Dental visits have been found to be scarcer among carrier, insurer, underwriter, or administrative
children with lower family incomes. agent.
2. Access to a dental examination for Medicaid-­eligible d. Insurance holder may be patient’s employer,
children has been improved by federal and state pro- union group, government agency, or welfare
grams but remains worse than for those whose family fund.
incomes are above poverty (family income ≥201% C. UCR fees:
of Federal Poverty Level [FPL]) (discussed later). 1. Method of reimbursement used in prepayment
3. Fewer than 1 in 5 Medicaid-covered children re- plan.
ceived at least one preventive dental service in 2. Usual fee: fee most frequently charged by dentists
recent year; many states provide only emergency for particular dental service.
dental services to Medicaid-eligible adults. 3. Customary fee: maximum benefit payable under
4. BOTH groups, poor and near-poor, are consid- particular plan for a specific procedure; is deter-
ered to be at high risk for poor oral health and are mined by the administrator of the dental insurance
similar in terms of having had “any” dental visit— benefit plan and based on submitted fees; may or
visit for routine dental examinations, restorative may NOT correlate with patient’s submitted fee.

549
550   Saunders Review of Dental Hygiene

4. Reasonable fee: actual fee charged by ­dental pro- 4. Employer reimburses employee/patient according
vider for particular procedure; modified accord- to agreed on reimbursement schedule.
ing to each patient’s specific circumstances; may H. Managed care or health maintenance organization
or may NOT differ from dentist’s usual fee or (HMO):
­insurance company’s customary fee. 1. Was designed MAINLY to help lower cost of
D. Capitation plans: dentist contracts with an insurance healthcare, participants prepay “fixed” premium
program to provide most or all dental services covered for healthcare services.
by dental benefit program; dentist is given ­ payment 2. Curtails choice of providers; participants choose
on per-capita basis. providers from approved list.
1. Characteristics: 3. Currently operated by BOTH for- and not-for-profit
a. Involve fixed, monthly payments to providing organizations; NO longer has federal ­constraints or
dentist. funding.
b. Payments based on the number of patients I. Preferred provider organization (PPO): contracts
­assigned to dentist by benefit program. between practitioners and their insurers to provide
c. Patients are assigned to dentist to receive care. healthcare services for lower-than-average fees (fee-
d. Monthly payments are received by ­ dentist for-service plan).
whether or not patients assigned require 1. Patients may choose a participating provider to
­treatment. render a covered service.
e. Co-payments and yearly maximums are ­features 2. Service subsequently is paid for by the PPO.
of these plans. J. Public financing: federal, state, and tribal government
2. Insurance benefit program assumes that some provides healthcare for groups such as the military,
members will require a significant amount of care the Coast Guard, American Indians, Native Alaskans,
and that others will NOT seek care. federal penitentiary inmates.
E. Dental service corporations (include Delta Dental
Plans, MOST common example). GOVERNMENT’S ROLE IN ORAL
1. Provided by a dental service corporation, not- HEALTHCARE  
for-profit organization that negotiates and provides Philosophy of U.S. government gives responsibility for
dental care contracts. seeking and receiving oral healthcare services to each in-
2. Incorporated state by state and is sponsored by dividual. MOST health-related programs administered by
each state’s constituent dental society, governed federal government are conducted through Department of
by the insurance laws of each participating state. Health and Human Services (HHS). Federal government
3. Provides dental treatment for clients in private developed two programs for express purpose of improv-
practice facilities. ing nation’s capacity to provide oral health ­protection and
4. Assists dentists in providing group purchase of oral healthcare ­services.
dental care through traditional practice. A. Group I: programs to provide improved oral health
5. Quality assurance is provided by a committee of protection.
nonparticipating dentists who conduct posttreat- 1. Biological research.
ment evaluations to ensure acceptable quality. 2. Disease prevention and control.
6. Payments to dentists are made through the custom- 3. Planning and development of oral health pro-
ary fee structure. grams.
F. Indemnity plans (commercial): operate for profit. 4. Education and services research.
1. Traditional insurance plans that operate by means 5. Regulation and compliance functions such as qual-
of a submittal and reimbursement method. ity assurance and assessment.
2. Require NO filing of UCR fees by dentists. B. Group II: programs concerned with provision of oral
3. Based on insurance company-generated “fee pro- healthcare services.
file” that reflects the fees most often charged. 1. Oral healthcare services rendered to specific
G. Direct reimbursement: occurs when an employer groups by U.S. Public Health Service (PHS).
agrees to pay for a portion of an employee’s dental 2. The PHS are commissioned officers of ­ Dental
treatment. Corps who provide services to Indian Health
1. Allows employee to seek dental care with dentist ­Service (IHS), prisoners in federal penitentiaries,
of choice. personnel of the Coast Guard and U.S. Merchant
2. Involves direct payment of fees to dentist by Marine, and underserved populations.
­employee/patient. C. The PHS also administers research, prevention,
3. Employee/patient pays for dental service and ­resource planning, and development programs by
­submits record of treatment to employer. ­National Institutes of Health (NIH).
Community Oral Health   551

1. Largest single source of monetary support for b. Part B:


health research and development. (1) Voluntary supplementary plan for physi-
2. Dental component of the NIH is National Institute cian services and other healthcare services.
of Dental and Craniofacial Research (NIDCR), (2) Dental payments for routine dental care are
chief sponsor of oral health research. excluded.
3. Dental services are limited to those medically nec-
Federally Funded Services essary, such as oral/maxillofacial needs related to
Federally funded services include services for maternal medical condition.
and child health, Head Start programs, Medicaid, Medi- E. National Health Service Corps:
care, and the National Health Service Corps. 1. Federal health labor deployment program.
A. Maternal and Child Health Services (MCH): 2. Deploys commissioned officers and civil service
1. For women of childbearing age. employees of PHS to render health services in
2. For children less than 21 years of age. geographical areas that are underserved because of
3. For individuals with low incomes. healthcare shortages.
4. Through grants to improve healthcare for mothers F. State health agencies (agencies or departments headed
and children. by a state or territorial office):
5. Dental component of MCH: 1. Preventive services for children <18 years.
a. Maternity and infant care projects. 2. Restorative and emergency services, screening.
b. Children and youth projects. 3. Very small number of states provide specialty ser-
c. Dental health projects for children. vices such as orthodontics, prosthetics, or correc-
d. Crippled children services. tion and repair of cleft palate.
e. Women, Infant, and Children (WIC) program.
B. Head Start developed by Office of Economic Oppor- BIOSTATISTICS  
tunity: Biostatistics is the tool by which research data are ana-
1. Provides educational, health, and social services lyzed and results are defined.
to low-income preschool children; enables them to A. Sampling: provides representation of general
enter school on a level equal to that of their peers ­population, used in research to manage time and cost
from higher income families. ­involved in conducting research.
2. Required to provide dental services to all enrolled 1. Random sampling: every member in population
children: has individual and equal chance of being ­selected.
a. Oral health screening. a. Table of random numbers may be used to
b. Oral health education programs. ­select sample, or participants may be chosen by
c. Sealant programs. ­lottery.
C. Medicaid (Title 19 of Social Security Act): federal b. Prevents researcher bias.
program that distributes funds to states for provision 2. Stratified sampling: modification of random selec-
of healthcare services to indigent. tion.
1. States vary on provisions; for most, dental care is a. Divides population into subgroups before
­option but NOT required; dental benefits, if included, ­selection to ensure that all subgroups are
typically are limited; may be limited by age and SES. ­sampled.
2. Early periodic screening, diagnosis, treatment pro- b. Typically involves random selection within
gram (EPSDT): each subgroup (stratified random sampling).
a. Oral healthcare services for individuals who c. Prevents bias because eliminates possibility that
are <21 years. members of a subgroup will NOT be selected.
b. Periodic screening for health defects. 3. Convenience sampling: simplest method.
c. Any necessary diagnosis and treatment deter- a. Involves selecting convenient group (e.g., class-
mined by screening. room, church, club members).
D. Medicare (Title 18 of the Social Security Act): covers b. May encourage bias because of sampling of
individuals >65 years, some disabled individuals: ­select group; results may NOT be applicable to
1. Provides insurance protection against cost of general population.
healthcare. 4. Systematic sampling: similar to, but is NOT truly,
2. Consists of two main parts: random sampling because all members of a pop-
a. Part A: ulation do NOT have the same chance of being
(1) Basic plan for hospital and related care. ­selected.
(2) Dental payments for routine dental care are a. Typically begins by determining that every nth
excluded. member on a list will be selected.
552   Saunders Review of Dental Hygiene

b. Starting number is drawn and every nth ­member C. Graphs: visually represent distribution of scores along
that follows starting number is selected; another the Y- and X-axes.
method involves systematically choosing every 1. The Y-axis typically represents the frequency of
even-numbered individual and excluding all scores and typically is vertical.
odd-numbered individuals. 2. The X-axis typically represents the scale that mea-
B. Descriptive statistics: used to make inferences about sures a specific variable.
a population; involve frequency distribution, graphs, 3. Types (see CD-ROM for discussion):
central tendency, and variability. a. Bar graph: two-dimensional representation of
1. Frequency distribution: group of scores arranged discrete data.
from lowest to highest that contains frequency with b. Histograph: representation formed directly from
which each score occurs; scores can be grouped, a frequency distribution.
ungrouped, or cumulative. D. Central tendency: measure of the average score;
a. Relative frequency: expressed as the frequency summary or typical score of a distribution.
with which a specific score is earned (e.g., 1. Mean: arithmetic value that is computed by divid-
15 students scored an 80 on an exam); can be ing the total by its members (e.g., 450 students/3
expressed as a percentage. schools = 150; the mean is 150 students per school);
b. Relative percent: expressed as the percentage can be influenced by extreme scores.
of students who receive a particular score (e.g., 2. Median: point that divides a score distribution
three students scored 77, or 9.1% of the 33 stu- into two equal parts, with 50% of the scores fall-
dents who took the exam). ing below and 50% of the scores falling above that
c. Cumulative frequency: expressed as the fre- point (e.g., 22, 56, 57, 78, 79, 80, 83, 84, 85, 90,
quency of occurrence of scores, up to and in- 100; the median is 80 because 5 scores fall below
cluding any value in the data set; MAINLY it and 5 fall above it); is LEAST influenced by ex-
used for data grouped by class intervals (e.g., treme scores.
88 students scored in the range of 75% to 79% 3. Mode: value that occurs with the MOST frequency
or below). in a distribution (e.g., 1, 1, 3, 3, 3, 3, 3, 4, 5, 6, 7,
d. Cumulative percent: expressed as the per- 7, 7, 7, 8, 9; the mode is 3); MORE than one mode
cent frequency of occurrence of scores, up can occur; used MAINLY for quick computation.
to and including any value in a data set (e.g., E. Variability: BEST used for describing the spread,
96% of students scored 90 or below on the range, or distribution of scores.
exam). 1. Range: difference or distance between highest
2. MOST commonly expressed as a normal (bell) and lowest scores (e.g., range is 4 when scores
curve (DING DONG, I GET IT!). range from 8 to 12); is NOT stable with extreme
a. Exhibits a symmetrical grouping of scores scores because ONLY uses the highest and lowest
around the mean or center of the curve. scores.
b. Assumptions are made regarding the focal cen- 2. Variance: measure of average spread of scores
ter of the curve. around the mean; the greater the dispersion around
c. Has a total area of 1 (100%); its mean, median, the mean, the greater the variance; MORE useful
and mode are equal and are located in the center than the range because considers all scores.
of the distribution. 3. Standard deviation: positive square root of vari-
(1) The total area is divided into segments ance; the greater dispersion around the mean, the
called standard deviations. greater the standard deviation; MOST useful mea-
(2) The area between the mean and one stan- sure of variability in descriptive statistics; consid-
dard deviation to the right is 34.13%, and ers all scores in a distribution.
to the left is 34.13% (the total is approxi- F. Inferential statistics: used to make generalizations
mately 68.26% of the total distribution). from the statistical sample to the general population;
(3) The area between the first and sec- effective sampling techniques make the inferences
ond standard deviations on the right is MORE accurate.
13.59%, and on the left is 13.59% (the 1. Student’s t-test: procedure that is used MAINLY to
total is ­ approximately 27.18% of the total make comparisons between the means of two dif-
­distribution). ferent studies; determines the probability that the
(4) The area between the second and third stan- differences in the two means are real, and NOT
dard deviations on the right is 2.15%, and caused by chance.
on the left is 2.15% (total is approximately 2. ANOVA: used in place of the t-test whenever more
4.30% of the total distribution). than two means MUST be compared.
Community Oral Health   553

3. Chi-square test: compares the observed measure- 2. This study is an example of educational or behavioral
ment of a given characteristic with the expected research because it (a) assesses and evaluates the ap-
measurement for a sample; chi-square statistic is plication of an educational or behavioral technique in
a measure of the difference between the observed dentistry or dental hygiene to an individual or group;
and expected measurements; used MAINLY when (b) focuses on knowledge, attitudes, behaviors regard-
studying categorical information. ing oral health and disease; (c) was conducted during
4. Correlation analysis: involves the study of two a short period of time. Unlike this study, educational
variables and their effects on each other; MOST or behavioral studies typically involve smaller popula-
useful when the number of pairs of variables is tions than experimental studies.
large (>30); correlation coefficient is the number 3. Oral signs or manifestations of eating disorders in-
that summarizes the strength of the relationship clude enamel erosion (perimolysis), cheilitis, parotid
between two variables; is denoted as r = +1 or −1; gland enlargement, palatal bruising, thermal sensitiv-
the closer the correlation coefficient is to 1, the ity, enlarged interdental papillae.
stronger the relationship between two variables. 4. Fact that students scored much higher on posttest than
on pretest indicates that learning occurred as a result of
community health study   the information presented. However, increase in knowl-
Scenario: A junior high school counselor contacted local edge may not always lead to a change in behavior.
dental and medical professionals regarding a perceived
increase in the number of girls with eating disorders in EPIDEMIOLOGY AND PREVALENCE  
her school. Several local dental hygienists decided to Epidemiology is the study of disease prevalence. It is
develop a program for raising the awareness of the dan- conducted through systematic observation and is used in
gers of eating disorders for this population of students. medicine, social and computer science, biology, and sta-
To determine the current level of awareness, the dental tistics. Epidemiological methods can be applied in den-
hygienists developed a pretest that elicited information tistry to evaluate the specific disease patterns and needs of
regarding facts and fallacies related to eating disorders; a community. Comparisons can be made between groups
it also asked students whether they had an eating disor- of a defined population.
der and whether they knew of a classmate with an eating A. Incidence: rate at which a disease occurs.
disorder. 1. Expressed as the number of new cases during a
The pretest was voluntarily taken by 410 girls in grades specific time.
7 through 9. One week later, the participants gathered in 2. Incidence rate = The number of cases divided by
an auditorium, where they listened to information about individuals and time.
different eating disorders, their treatment, and the den- 3. Ir = Cases/Individuals and time
tal and medical effects of the disorders. The participants 4. Example: 50 individuals in a community of
subsequently watched a 20-minute video of personal 250,000 died of oral cancer during 2008; Ir = 50
accounts of battles against anorexia and bulimia. One deaths/250,000 individuals per year for the inci-
week after the presentation, the 410 participants were dence of oral cancer in that community.
given a posttest that covered the information on the pre- B. Prevalence: total number of cases of disease in exis-
test. Participants scored significantly higher on the facts tence in a given population at a given time.
and fallacies section of the posttest than they did on the 1. Prevalence = The number of cases divided by pop-
pretest. ulation multiplied by 100.
2. P = Cases/Population × 100
1. What type of sampling was used in this study? 3. Example: in 2008 there were 300 reported cases of
2. What type of research does this project involve? gingivitis among 1000 20-year-old students exam-
3. What are the oral signs of eating disorders? ined; 300 gingivitis cases/1000 students × 100%
4. The results of the study showed that students scored = 30% prevalence of gingivitis among the student
much higher on the posttest than on the pretest. What population.
does this indicate? C. Rate of disease: expressed as a ratio (fraction, numer-
ator/denominator).
1. This study used convenience sample (all female stu- 1. Numerator is occurrences of a disease; denomina-
dents in a school were chosen). Random study is one tor is possible occurrences of a disease.
in which every element in a population has an equal 2. Rate = Actual occurrences/Possible occurrences
chance of selection. Stratified sample involves se- 3. Example: in 2008 there were 25 cases of gingivi-
lecting members from the subpopulations of a group. tis among 250 teenagers examined; 25 actual/250
­Single-subject sampling involves one or a few sub- possible = 10% rate of gingivitis among the stu-
jects who exhibit a special condition. dent population (must be very good brushers!).
554   Saunders Review of Dental Hygiene

D. Morbidity (disease) can be expressed as a rate: mor- B. Trends in caries:


bidity rate is number of actual diseases divided by 1. NO change in prevalence of dental caries in pri-
number of possible diseases. mary teeth among children aged 2 to 11 years.
E. Mortality (death) can be expressed as a rate: mortal- 2. Reduction in prevalence of caries in permanent
ity rate is number of actual deaths divided by number teeth of up to 10% among persons aged 6 to 19
of possible deaths. years and up to 6% among dentate adults aged >20
F. Epidemic: significantly greater-than-normal inci- years.
dence of disease. 3. Increase of 13% in dental sealants among persons
1. Describes a disease that spreads rapidly through aged 6 to 19 years.
particular segment of the population. 4. A 6% reduction in total tooth loss (edentulism)
2. Describes a disease with MORE cases than ex- among persons >60 years.
pected. C. Statistic on caries in United States:
G. Endemic: disease with expected (typical) number of 1. Among children aged 2 to 11 years, 41% had den-
cases that continues over time; may be specific to par- tal caries in primary teeth.
ticular geographical area or population. 2. Among children and adolescents aged 6 to 19 years,
H. Pandemic: disease that occurs throughout population 42% had dental caries in permanent teeth, and ∼90%
of a country, people, or the world. of adults did.
I. Index (plural, indices): systematic way of collecting 3. Among children aged 6 to 19 years, 32% had re-
and arranging data gathered from observations so that ceived enamel sealants.
they can be quantified, analyzed, understood (specific 4. Adults aged ≥20 years retained a mean of 24/28
dental indices discussed later). natural teeth and 8% were edentulous.
1. Accomplished under specifically defined criteria 5. Among persons aged 6 to 39 years, 23% had
and conditions. enamel fluorosis (very mild or greater).
2. Example: record of data collected from periodon- 6. Disparities were noticed across ALL age groups;
tal probing. MAINLY among racial and ethnic groups, in per-
3. Characteristics of effective index: reliable; repro- sons with lower education and income, and by
ducible; valid; easily understood and explained. smoking status.

Epidemiology of Caries Epidemiology of Periodontal Disease


A 2005 report, “The Centers for Disease Control Surveil- The American Academy of Periodontology (AAP) Research,
lance for Dental Caries, Dental Sealants, Tooth Reten- Science and Therapy Committee (2005) reviewed evidence
tion, Edentulism, and Enamel Fluorosis—United States, concerning periodontal disease and formed conclusions.
1988-1994 and 1999-2002,” used the National Health National prevalence of moderate and severe periodontal
and Nutrition Examination Survey (NHANES), ongoing diseases among dentate adults aged 20 years and older was
survey of representative samples of population. estimated from the 1999-2004 National Health and Nutri-
A. Background concerning caries: tion Examination Survey conducted by NHANES.
1. Common chronic disease that causes pain and dis- • See Chapters 11, Clinical Treatment: charting and as-
ability across ALL age groups. sessment of periodontal disease; 13, Periodontology:
2. Although dental caries (tooth decay) is LARGELY risk factors for periodontal disease.
preventable, remains MOST common chronic A. Background concerning periodontal disease:
disease of children aged 5 to 17 years, four times 1. Interpreting results of epidemiological studies of
MORE common than asthma (42% versus 9.5%). gingivitis, and particularly those of periodontitis,
3. If left untreated, dental caries can lead to pain and is extremely difficult.
infection, tooth loss, edentulism (total tooth loss); 2. Prevalence of disease must be measured in terms
pain and suffering resulting from untreated tooth of specific definition of each disease and age group
decay can lead to problems in eating, speaking, affected.
and attending to learning. a. Example: although periodontitis may be defined
4. Enamel (dental) sealants are effective in prevent- in one study as an attachment loss of 2 mm, it
ing dental caries in occlusal (chewing) and other may be defined in another study as attachment
pitted and fissured surfaces of teeth. loss of 4 mm.
5. Enamel fluorosis is hypomineralization of enamel b. Therefore describing prevalence of periodontal
related to fluoride exposure during tooth formation disease in statistical terms can be difficult and
(first 6 years for MOST permanent teeth); expo- misleading.
sure to fluoride throughout life is effective in pre- c. Definition of disease must be determined before
venting dental caries. statistics are interpreted.
Community Oral Health   555

B. Trends concerning gingivitis: 4. Overall, moderate and severe periodontitis was


1. Dental biofilm (dental plaque) is closely correlated still prevalent; moderate disease, which is MOST
with gingivitis, relationship long considered one of open to ­ preventive measures, was MOST preva-
cause and effect. lent in non-Hispanic black males, the poorest, and
2. Gingivitis is found in early childhood, is MORE the least educated.
prevalent and severe in adolescence, then tends to
level off in older age groups. Dental Indices
3. Gingivitis, associated with widespread dental bio- Dental indices are used in studying BOTH caries and
film and calculus deposits, is standard in adults. periodontal disease and helping to create preventive pro-
4. Relatively FEW sites with gingivitis go on to grams or to understand individual needs for oral health.
­develop periodontitis; even so, prevention of gin- A. O’Leary plaque index: does NOT quantify dental bio-
givitis, in individual patients and in populations, is film; monitors oral hygiene performance, indicates
still FIRST step toward preventing periodontitis. location of dental biofilm.
C. Trends concerning periodontitis: 1. Assists in visualization of homecare progress, thus
1. Periodontitis results from a complex interplay assisting clinicians in emphasizing specific areas
­between bacterial infection and host response, of need and tailoring homecare with alternative
­often modified by behavioral factors. dental biofilm control aids.
2. Host response is KEY factor in clinical expression 2. Directions for use:
of periodontitis, with only some 20% of periodon- a. Cross out all missing teeth; divide the teeth into
tal diseases now attributed to bacterial variance. four parts (facial, lingual, distal, mesial) and
3. Some 50% of periodontal diseases have been at- the mouth into four quadrants; apply disclosing
tributed to genetic variance and >20% to tobacco solution and have patient rinse; evaluate pres-
use. ence of dental biofilm using air, mouth mirror,
4. While there is a clear causal relationship be- explorer.
tween poor oral hygiene and gingivitis, relation- b. Number of dental biofilm surfaces present/Total
ship of oral hygiene to periodontitis is LESS number of tooth surfaces examined × 100 = %
straightforward; in populations with poor oral dental biofilm.
hygiene, dental biofilm and supragingival calcu- B. Oral hygiene index, simplified (OHI-S): developed
lus accumulations correlate poorly with severe by Greene and Vermillion, used MAINLY for large
­periodontitis. populations.
5. Oral hygiene can favorably influence ecology of 1. Two components, debris index (dental biofilm, ma-
microbial flora in shallow-to-moderate pockets but teria alba, food) and calculus index, BOTH added
does NOT affect host response; oral hygiene alone together to obtain single score.
has little effect on subgingival microflora in deep 2. ONLY facial and lingual surfaces are scored; in-
pockets. cludes facial surfaces of teeth #3, #8, #14, and #24,
6. Some 5% to 20% of any population suffers from and lingual surfaces of teeth #19 and #30.
generalized severe periodontitis, although slight to 3. Scoring criteria:
moderate periodontitis affects majority of adults; a. 0 = No debris or stain present.
for those who are MOST susceptible, periodontitis b. 1 = Soft debris covering not more than one third
becomes MOST evident in teenage and early adult of tooth surface.
years rather than the later years. c. 2 = Soft debris covering more than one third but
D. Statistics for periodontal disease in United States: not more than two thirds of tooth surface.
1. Overall, prevalence of moderate and severe peri- d. 3 = Soft debris covering more than two thirds of
odontal disease ranged from 0.82% to 18.3% for tooth surface.
adults aged 20 to 34 years and 0.06% to 2.9% for e. Debris and calculus scores are combined, and
adults aged 75 years and older. total is divided by number of surfaces examined
2. Moderate disease was MOST prevalent in males, to obtain average OHI-S.
non-Hispanic blacks, group with lowest family C. Personal hygiene performance index–modified
poverty ratio (<100%), persons with less than high (PHP-M): developed by Podshadley and Haley,
school education. modified by Martens and Meskin; used MAINLY
3. Severe disease was MOST prevalent among males to provide patients with information about ­ dental
for all age groups (EXCEPT those aged 75 years biofilm that will assist in improvement of oral
and older), persons with a family poverty income health.
ratio between 100% to 200%, those with less than 1. Teeth that are selected during the initial visit are
high school education. used for comparison during subsequent visits.
556   Saunders Review of Dental Hygiene

2. Criteria for use: F. Gingival bleeding index (Ainamo and Bay): assesses
a. Select six teeth, divide each of them into five bleeding of gingival margin in response to probing.
areas, and label areas a, b, c, d, and e on BOTH 1. Used MAINLY as indicator of gingival health and
facial and lingual surfaces. disease.
b. Record presence of dental biofilm in each of let- 2. Scoring: positive score indicates percentage of all
tered areas and give one point for each area of gingival areas explored that bleed in response to
dental biofilm; scores range from 0 to 60 (best probing.
to worst). a. Divide the total number of areas that bleed by
D. Plaque index (PI): developed by Silness and Loe, used the number of gingival margins examined; then
MAINLY in conjunction with gingival index (GI) by multiply the result by 100 to arrive at a score
same authors; assesses thickness of dental biofilm on (percentage).
teeth at gingival margin. b. (+) = bleeding within 10 seconds after gentle
1. Specific teeth and entire dentition can be probing.
­assessed using the distal, mesial, facial, lingual c. (−) = absence of bleeding after 10 seconds fol-
surfaces. lowing probing.
2. Scoring criteria: visually examine dental biofilm G. Russell’s periodontal index (PI): assesses progressive
or probe to swipe along cervical third; disclosing stages of periodontal disease and amount of attach-
agent can be used. ment loss.
a. 0 = no dental biofilm adheres or visible at gingi- 1. Easy to use and understand; results are compara-
val third. tive, used MAINLY for major population groups.
b. 1 = dental biofilm adheres at gingival third; vis- 2. Criteria for use:
ible with explorer. a. Tissues are examined for gingival inflammation,
c. 2 = moderate amounts of dental biofilm visible pocket formation (with noncalibrated probe),
in sulcus or at gingival margin. and masticatory function and given a score.
d. 3 = heavy amounts of dental biofilm visible in 3. Scoring:
sulcus or at gingival margin. a. 0 = Negative: no overt inflammation in invest-
3. Criteria for use: ing tissues or loss of function from destruction
a. For individual teeth: total the score for each of of supporting tissues.
the four surfaces and divide by four; for groups b. 1 = Mild gingivitis: overt area of inflammation
of teeth, total individual scores for all teeth in is present in free gingiva but the area does not
the group and divide by number of teeth in the circumscribe the tooth.
group; used MAINLY for comparing areas of c. 2 = Gingivitis: inflammation completely cir-
the mouth. cumscribes tooth; no apparent break in epithe-
b. For individual dentition: total the individual lial attachment.
scores for all teeth and divide by the number of d. 6 = Gingivitis with pocket formation: epithelial
teeth; for groups of individuals, total individual attachment has been breached and pocketing
dentition scores and divide by number of mem- exists; no interference with normal masticatory
bers in the group. function; tooth is firm in its socket, no drifting
E. Gingival index (GI; Loe and Silness): index MOST is discernible.
frequently used to evaluate gingivitis; bleeding is e. 8 = Advanced destruction with loss of mastica-
MOST critical factor; also assesses tissue bleeding, tory function: tooth may be loose, drifting, or
color, contour, ulceration. dull on percussion or may be depressed in its
1. Scoring: use on all or six selected teeth; divide socket.
the teeth into four areas (facial, lingual, distal, and 4. Findings:
­mesial). a. For individuals: PI = Sum of individual scores/
a. Total scores from these four areas and divide by Number of teeth scored
number of surfaces examined. b. For groups: Group PI = Total of individual
b. 0 = Normal gingiva. scores/Number of individuals examined
c. 1 = Mild inflammation, slight change in color, 5. Clinical conditions and group PI score ranges:
slight edema, and no bleeding on probing. a. Indicates clinically normal tissues (0.0 to 0.2).
d. 2 = Moderate inflammation, redness, edema, b. Indicates simple gingivitis (0.3 to 0.9).
glazing, and bleeding on probing. c. Indicates incipient destructive periodontal dis-
e. 3 = Severe inflammation, marked redness, ease (0.7 to 1.9).
edema, ulceration, and a tendency toward spon- d. Indicates established destructive periodontal
taneous bleeding. disease (1.6 to 5.0).
Community Oral Health   557

e. Indicates terminal stages of periodontal disease 1. Evaluates six sextants; one score from each sextant
(3.8 to 8.0). is used; excludes third molars unless function as
H. Ramfjord’s periodontal disease index (PDI): ­evaluates second molars.
gingival health, probing depths, dental biofilm and 2. Probing is performed with the CPITN-E probe.
calculus deposits. a. First and second molars in posterior ­ sextants
1. Used for the Ramfjord teeth (#3, #9, #12, #19, #25, are examined and only worse of the two scores
and #28); MOST often are used in clinical studies is recorded; one maxillary tooth and one
as a representative sample of the entire dentition. ­mandibular anterior tooth are also scored for a
2. Gingiva is given a score between 0 and 3, depend- total of six scores.
ing on the severity of inflammation; pockets are b. Scoring uses a five-point Likert scale (0 = no
probed on the mesial and facial surfaces and are signs of periodontal disease and 4 = periodontal
given a score between 4 and 6. pockets of 6 mm or more).
3. Critical measurement is the distance from the CEJ 3. Scores are converted to a 4-point treatment needs
to the base of the sulcus. (TN) classification.
4. Scoring: a. TN 0 = No treatment is needed.
a. 0 = Absence of inflammatory signs. b. TN 1 = Requires improved oral hygiene.
b. 1 = Mild to moderate inflammatory gingival c. TN2 = Requires improved oral hygiene and
change that does not extend around the tooth. scaling.
c. 2 = Mild to moderately severe gingivitis that d. TN3 = Requires improved oral hygiene, scal-
extends around the tooth. ing, and complex treatment.
d. 3 = Severe gingivitis characterized by marked K. Specific caries indices:
redness, swelling, and the tendency to bleed 1. DMFT/dmft indices: provide broad picture of
and ulcerate. caries activity in specific population for BOTH
e. 4 = Gingival crevice extends apically past the ­permanent and primary dentitions.
CEJ but not more than 3 mm. a. Various components may be used to evaluate
f. 5 = Gingival crevice extends apically 3 to 6 mm specific information that is needed.
past the CEJ. b. Examples for permanent dentition: DMFT =
g. 6 = Gingival crevice extends apically more than decayed, missing, filled teeth and DMFS =
6 mm from the CEJ. decayed, missing, filled surfaces: DEFT = de-
h. Combined gingival and pocket scores reflect an cayed, extracted, filled teeth and DEFS = de-
individual’s periodontal status. cayed, extracted, filled surfaces; SIMILAR
I. Plaque Assessment Scoring System (PASS): evalu- methods used for primary dentition.
ates subgingival dental biofilm, UNLIKE O’Leary c. Also available are the df/def indices, which
plaque index, which only evaluates supragingival ­indicate decayed (d), indicated for ­extraction (e),
dental ­biofilm. and filled (f) in the primary ­dentition.
1. Selects five teeth for examination: four first molars d. Data collected from these indices can be used to
and maxillary incisor; if not available, teeth near determine community needs.
the lost molars are used, first distal, then mesial, 2. Unmet treatment needs (UTN) index of particular
then mandibular incisor is used. population:
2. Each tooth selected is divided into areas (mesial, a. UTN = Mean number of decayed teeth/Mean
distal, buccal, lingual) and a periodontal probe is number of decayed and filled teeth multiplied
swept around 1 mm into the sulcus; if dental bio- by 100.
film is visible on the probe, recorded as positive b. Can be used to compare treatment needs of one
score. population with those of another (e.g., UTN
3. There are 20 possible sites for dental biofilm of a community with fluoridated water may be
­surfaces to be evaluated per mouth; score is per- compared with UTN of a community that does
centage of surfaces positive for subgingival dental not have water fluoridation).
biofilm accumulation. 3. E CC and S-ECC (early childhood caries and
4. Ideal for the solo clinician and statistically reliable severe-early childhood caries) index: each
compared to the O’Leary PI. erupted primary tooth is examined and every
J. Community periodontal index of treatment needs surface.
(CPITN): evaluates pockets, bleeding, and dental a. ECC: 1 or more decayed (noncavitated or cavi-
biofilm retention factors; developed by Fédération tated lesions), missing (because of caries), or
­Dentaire Internationale and World Health Organiza- filled tooth surfaces in any primary tooth in a
tion to assess treatment needs of specific groups. child 71 months of age or younger.
558   Saunders Review of Dental Hygiene

b. S-ECC: younger than 3 years of age, any sign in which the study is of one or a few ­ subjects who
of smooth-surface caries; ages 3 through 5, 1 exhibit a special condition.
or more cavitated, missing (because of caries), 2. Best index to use for assessing the sealant needs of
or filled smooth surface in primary maxillary this population is the DMFS because it assesses the
anterior teeth, or a decayed, missing, or filled decayed, missing, and filled surfaces of teeth. This
score of >4 (age 3), >5 (age 4), or >6 (age 5) index would be helpful in identifying pit and fissure
surfaces. surfaces in need of sealants. DEFT and DEFS are in-
4. RCI (root caries) index: evaluates extent of root dices that identify decayed, extracted, and filled teeth
caries and risk for root caries disease. or surfaces of deciduous teeth; they would not be ap-
a. Includes only those root surfaces exposed to the propriate for studying decayed pits and fissures in
oral environment by gingival recession. the teeth of elementary school-aged children because
b. RCI = (R − D) + (R − F)/(R − D) + (R − F) + sealants are not routinely placed on deciduous teeth.
(R − N); where R = root surface, D = decayed 3. Study results indicate that there is a need for both seal-
root surface, F = filled root surface, N = intact, ant placement and school water fluoridation because
sound root surface. the DMFS is 4.1 (D = 0.9, M = 0, and F = 3.2). Wa-
L. Indices that measure fluorosis: ter fluoridation would decrease the number of smooth
1. Dean’s fluorosis index: MOST used in ­community surface caries, and sealants placed on remaining
studies even if less sensitive than other index; indi- ­caries-free surfaces would reduce the number of pit
vidual’s fluorosis score is based on the most severe and fissure caries.
form of fluorosis found on two or more teeth. 4. Fluoridation of the school water supply is an example
2. Tooth surface index of fluorosis (TSIF): MORE of primary preventive dentistry. It can prevent the on-
sensitive than Dean’s; each tooth is examined for set of disease (caries), reverse the initial stages of dis-
signs and assigned a numerical score. ease (demineralization), and arrest the process before
treatment is indicated. Secondary and tertiary preven-
COMMUNITY HEALTH STUDY  tion, respectively, are associated with restoring tissues
Scenario: Children of a Native American tribe between to near normal function and with replacing lost tissue
the ages of 12 and 14 were assessed for the need for seal- and providing rehabilitation.
ants. The children live in a rural nonfluoridated com- 5. Best way to promote the sealant project is to involve
munity. An alphabetized list of names of school-aged community leaders. Involving community leaders al-
children was used to select participants. Of the 980 chil- lows the project to take on value and priority. Other
dren on the list, every twentieth child was selected for the methods of promoting the project include informational
study. Among the 49 youths examined, 155 tooth surfaces meetings for parents, teacher-student discussions, ra-
had restorations, 42 surfaces had decay, and no teeth re- dio advertising, and informational flyers. Professional
quired extraction. fluoride treatment should be given immediately after
sealant placement such as with a fluoride varnish.
1. What method of sampling was used in this study?
2. Identify the best index to use for assessing the sealant Epidemiology of Oral Cancer
needs of this population. The “SEER Cancer Statistics Review,” a report by the
3. What do the results of the study indicate? National Cancer Institute, gives rates that are based on
4. Is fluoridation of the school water supply an example cases diagnosed in 2000 to 2004.
of primary, secondary, or tertiary prevention? • See Chapters 6, General and Oral Pathology: oral can-
5. Identify the best way to promote the sealant proj- cer and risk factors; 11, Clinical Treatment: examina-
ect. Should professional fluoride treatments be tion for oral cancer.
a part of the project? If so, when should they be A. Background on oral cancer:
given? 1. Death rate associated with this cancer is particu-
larly high but NOT because it is hard to discover
1. Systematic method of sampling was used in this study and/or diagnose, rather because it is routinely dis-
because every twentieth child was selected from an covered late in development.
alphabetized list. Other methods of sampling include 2. Often discovered only when has metastasized to
random sampling, in which every element in a popula- another location, MOST likely cervical lymph
tion has an equal chance of selection; convenience sam- nodes; prognosis at this stage of discovery is sig-
pling, in which a selection is made for convenience that nificantly worse than when caught while still in a
may not be representative of the general population; localized intraoral area; at these later stages, pri-
stratified sampling, in which members are randomly se- mary tumor has had time to invade deep into local
lected within subgroups; and single-subject sampling, structures.
Community Oral Health   559

3. Patients who survive first encounter with disease A. Community fluoride promotions:
have up to a 20 times higher risk of developing 1. Primary preventive oral care services to reduce or
second cancer; heightened risk factor can last for 5 prevent dental caries.
to 10 years after first occurrence. 2. Measured use of fluoride modalities is particularly
B. Trends on oral cancer: appropriate during time of anterior tooth enamel
1. Number of cases and associated levels of mortality development (i.e., age <6 years).
have NOT significantly improved in decades. 3. Community water fluoridation:
2. Death rate for oral cancer is higher than that of a. Artificial fluoridated water is the addition of
cancers widely discussed in the media (such as fluoride compounds to the community water sup-
ovarian). ply; amount is maintained at an optimum level to
3. Worldwide, problem is much greater, with decrease caries without causing fluorosis.
>400,000 new cases being found each year. b. CDC and PHS recommend OPTIMUM con-
C. Statistics on oral cancer in United States: centration of fluoride in community water sup-
1. Incidence: ply to be 0.7 to 1.2 ppm (depending on average
a. Median age at diagnosis for oral and pharyn- maximum daily air temperature):
geal cancer was 62 years of age. (1) If rate is 1 ppm, necessary to consume 1 L
b. Age-adjusted incidence rate was 10.5 per of water in order to take in 1 mg of fluoride;
100,000 men and women per year. highly improbable a person will receive
2. Prevalence: in 2004, ∼235,856 men and women more than the tolerable upper limit from
alive who had oral and pharyngeal cancer history, consuming optimally fluoridated water
150,946 men and 84,910 women. alone.
3. Mortality: (2) Individuals in hotter climates are likely
a. Median age at death for cancer of the oral cav- to drink MORE water and thus receive
ity and pharynx was 68 years of age. MORE fluoride, so amounts are adjusted
b. Age-adjusted death rate was 2.7 per 100,000 for warmer air temperatures.
men and women per year. (3) Example of addition of fluoride to a com-
munity water supply (where temperature is
PREVENTION OF DENTAL DISEASE   not a consideration): a town naturally has
There are different levels of prevention or health promo- 0.25 ppm concentration of fluoride, the
tion. Health promotion facilitates a voluntary adaptation community added 0.75 ppm to make com-
of health-seeking behaviors through any combination of munity water supply achieve an optimum
learning opportunities; involves behavior changes pro- concentration of fluoride at 1 ppm.
duced by advice and counseling. Primary prevention in- c. Environmental Protection Agency (EPA),
cludes techniques designed to prevent the onset of disease, which is responsible for the safety and qual-
reverse the initial stages of disease, or arrest a disease pro- ity of drinking water in the United States, sets
cess before treatment is needed. Secondary prevention maximum concentration of fluoride in commu-
includes techniques designed to terminate disease and nity water supply at 4 ppm (and secondary limit
restore tissues to near normal function. Tertiary preven- [i.e., nonenforceable] guideline at 2 ppm).
tion includes techniques designed to replace lost tissues (1) If the water supply has natural fluoridated
and rehabilitate to near normal function. ALL discussed water over 2 to 4 ppm, the water supply
can be considered primary preventive oral care services. undergoes defluoridation (partial) so the
water is at optimum concentration of fluo-
Prevention and Control of Caries ride to prevent serious side effects.
Caries can be controlled though use of fluorides, enamel (2) Community water supply becomes natu-
(pit and fissure) sealants, and diet counseling. The CDC rally fluoridated as ground water that is
has reported on these issues in “Preventing Dental Caries fluoridated by water flowing over rocks and
with Community Programs” (2006) and “Recommenda- soil containing fluoride.
tions for Using Fluoride to Prevent and Control Dental (3) Example of defluoridation (partial) to re-
Caries in the United States” (2001) (see CD-ROM). Dis- move fluoride: a rural town naturally has
cussion covers community promotions for use of fluoride, 4 ppm in their community water supply;
sealants, and diet counseling. the community removes 3 ppm to achieve
• See Chapters 6, General and Oral Pathology: fluorosis; an optimum concentration of ­ fluoride at
7, Nutrition: diet counseling; 11, Clinical Treatment: 1 ppm.
caries classification and charting, patient-use fluoride d. Serves as a model health promotion interven-
products and toxicity, enamel sealants, oral microbials. tion, since it is socially equitable.
560   Saunders Review of Dental Hygiene

(1) Effectively prevents dental caries in com- 5. Fluoride supplementation: prescribed by supervis-
munities with varying disease prevalence; ing dentist or physician because of low levels of
children in communities with water fluori- fluoride in community water supply (controversial
dation experienced 29% fewer cavities. method of prevention) (Table 17-1).
(2) At present, 67% of individuals on pub- a. For optimum benefits, use of dietary fluoride
lic water supply, MORE than 170 million supplements SHOULD begin when 6 months
people receiving community water fluori- old and be continued daily until 16 years old.
dation. b. Evidence for using to control dental caries is
(3) Water fluoridation is NOT expensive to mixed; taken by pregnant (or lactating) women,
administer; very inexpensive, with costs provides NO benefit for offspring.
ranging from $0.12 to $1.16 per individual c. In form of tablets and lozenges, liquids (includ­
per year; every dollar spent on community ing fluoride-vitamin preparations); MOST con-
water fluoridation saves from $7 to $42 in tain sodium fluoride (NaF) as active ­ingredient.
treatment costs depending on the size of the d. Manufactured with 1.0 mg, 0.5 mg, or 0.25 mg
community; savings are greatest in large fluoride; to maximize topical effect of fluoride,
­communities. tablets or lozenges intended to be chewed or
(4) At least 60% of U.S. population on public wa- sucked for 1 to 2 minutes before being swal-
ter supply has received fluoridated water since lowed; for infants, supplements are available as
1990, translating to savings in dental treatment a liquid (with dropper).
costs of over $25.7 billion in past decade. e. For infants and children aged <6 years, BOTH
(a) However, LACK of fluoridation in other benefit of dental caries prevention and risk for
30% of communities may disproportion- mild enamel fluorosis for developing, unerupted
ately affect poor and minority children. teeth are possible; although primary teeth of
e. Bottled water: some contain optimum concen- children aged 1 to 6 years would benefit from
trations (1.0 ppm); MOST contain <0.3 ppm, if fluoride’s posteruptive action, and some pre-
being used to replace tap water, may NOT be eruptive benefit for developing permanent teeth
receiving benefit of fluoride in community wa- could exist, fluoride supplements could also
ter supply; FDA regulations ONLY required to INCREASE the risk for enamel fluorosis at this
be listed if added during ­processing. age.
4. School water fluoridation: NOT as successful as f. MUST not have prescription larger than 120 mg
community water fluoridation, so phased out. of fluoride at one time to prevent accidental tox-
a. Used MAINLY in rural areas, where students icity; NO storage of large quantities in the home.
attend school together or in adjacent buildings g. However, lozenges for elderly or medically
with the same stand alone, separate from com- compromised, especially those who have limita­
munity water supply. tions with toothbrushing, may be ­effective for
b. Since children are at school only part of each topical use.
day, recommended level was 4.5 times the h. Example of fluoride supplementation (contro-
optimum concentration for community water versial): a 4-year-old child lives in a community
supply; however, could result in higher than that naturally has 0.4 ppm in community water
recommended concentrations, although NO supply, could receive daily fluoride supplement
lasting effects on children noted. of 0.25 mg until 16 years old.

Table 17-1  Fluoride supplementation dosage schedule*

Fluoride ion level in drinking water (ppm)†

Age <0.3 ppm 0.3-0.6 ppm >0.6 ppm

Birth–6 months None None None


6 months–3 years 0.25  mg/day‡ None None
3-6 years 0.50 mg/day 0.25 mg/day None
6-16 years 1.00 mg/day 0.50 mg/day None

*Approved by the American Dental Association, American Academy of Pediatrics, and American Academy of Pediatric Dentistry, 1994 (only if needed, controversial).
†1.0 ppm = 1 mg/liter
‡2.2 mg NaS = 1 mg fluoride ion.
Community Oral Health   561

6. School-based fluoride mouthrinses: NOT as b. Identifies cariogenic foods and makes recom-
­successful as community water fluoridation, so mendations for alternative food choices without
phased out. significantly changing the diet.
a. Rinsing (swishing) with diluted formulations of c. Allows patient and/or guardian to play role in
NaF, stannous (SnF2), or acidulated phosphate alternative food selections; also allows consid-
(APF) fluoride daily or weekly; rinsing with a 10- eration of likes and dislikes.
mL solution for 1 minute provides topical effect. d. Success is dependent on four elements:
b. Programs done during the 1980s had 3 million (1) Motivation and cooperation of the patient.
children enrolled; little success noted in caries (2) Establishment of a professional rapport.
reduction, especially when children were also (3) Consideration and fulfillment of the pa-
exposed to fluoridated water; involved teach- tient’s individual needs.
ers and other nondental personnel and consent (4) Completion of a follow-up assessment and
was difficult to achieve; risk of swallowing was evaluation.
­pronounced in younger groups.
7. Community fluoride varnishes: successful ­program COMMUNITY HEALTH STUDY 
but MUST involve trained healthcare professionals. Scenario: A dental hygienist is conducting a dental needs
a. Holds high concentration of fluoride on teeth assessment on 12-year-old children in a small community.
for many hours; MOST effective method of Most of these children have not had regular dental care for
fluoride application for primary caries. several years. Clinical findings indicate that most of the
b. Has ease of application, nonoffensive taste; children have moderate marginal inflammation and dental
smaller amounts required than gel or foam biofilm. In addition, decay is present on many ­occlusal sur-
­application. faces and along the gingival margin in some of the children.
c. BEST if incorporated for the very young such Survey results show that a large number of these children
as WIC programs, Head Start, or well-child frequently consume candy and sugared sodas and that their
checkups. homecare is poor (average child brushes with a fluoride
B. Community-based enamel sealant promotions: suc- toothpaste once daily and flosses sporadically). Commu-
cessful program but MUST involve trained dental nity water supply is fluoridated at 1 ppm in their moderate
professionals. climate.
1. School-based programs have 60% fewer new
­decayed pit and fissure surfaces in posteriors for 1. Identify two indices that would be most appropriate
up to 2 to 5 years after a single application; among for assessing the children’s oral condition and for use
children, 90% of caries is in pits and fissures. as motivational tools.
2. School-based sealant programs provide sealants 2. Should these children receive fluoride supplementa-
to children UNLIKELY to receive them otherwise tion in the form of tablets?
(e.g., children in low-income households); children 3. What is the optimum concentration level for fluoride
of racial and ethnic minority groups have twice as in a community water supply?
much untreated caries in permanent teeth, but re- 4. Describe two other dental and oral public healthcare
ceive only about half as many dental sealants. measures that might be recommended.
3. However, although 29 states reported dental seal-
ant programs serving 193,000 children, number 1. The DMFS (decayed, missing, filled surfaces) index
represents ONLY about 3% of poor children who would be appropriate for identifying specific car-
could receive sealants. ies activity and restorative needs and has greater
C. Diet counseling in community programs: sensitivity than the DMFT (decayed, missing, filled
1. Avenue for helping to control dental caries in pa- teeth) index. In addition, for both the children and
tients with high sugar consumption, high rates of their parents, may be used as visual tool to identify
caries, or salivary flow problems: patterns of caries and areas that need concentra-
a. All schoolchildren. tion. O’Leary plaque index would be appropriate
b. Children, adolescents, and adults with high for identifying the dental biofilm present and pro-
rates of caries. viding the children with opportunity to see exactly
c. Medically compromised patients. which surfaces have dental biofilm. This index also
d. Patients with decreased salivary flow or sali- gives the patient an effective visual display of ar-
vary gland hypofunction (xerostomia). eas that require attention (e.g., dental biofilm along
2. Characteristics of diet counseling: margins, which can be removed by brushing, and/or
a. Assesses cariogenic food intake by means of a in interproximal areas, which must be removed by
diet diary. flossing).
562   Saunders Review of Dental Hygiene

2. No, since they are receiving for the most part benefits educational principles necessary to develop and practice
from an optimum concentration level of fluoride in the the teaching and presentation of oral health education
community water supply. programs.
3. CDC and PHS recommend optimum fluoride con-
centration of 0.7 to 1.2 ppm, depending on average Assessment of Public Health Problem
maximum daily air temperature; individuals in hotter Public health problem is a condition or situation that is
climates are likely to drink more water and thus receive widespread or has potential to cause morbidity and mor-
more fluoride. EPA, which is responsible for the safety tality. Public health problem is one that is perceived by
and quality of drinking water in the United States, sets public, government, or public health authority as an ex-
maximum allowable limit for fluoride concentration isting problem. Population assessment is an organized,
in community water supply at 4 ppm (and secondary systematic approach to identifying a group with a health
limit [i.e., nonenforceable] guideline at 2 ppm). need, in this case an oral health need. An assessment de-
4. Two dental and oral public health measures to recom- fines the oral health problem, identifies the extent and
mend for these children are enamel sealants (occlusal severity of the problem, and assists in development of a
caries reduction) and some form of topical fluoride ap- community profile. Then the data are collected in several
plication (smooth surface caries reduction). Because ways such as survey questionnaire, clinical examination,
most of these children have several areas of occlu- or personal communication.
sal caries, sealants are recommended for the occlu-
sal surfaces that remain caries free. Topical fluoride Developing a Community (Population) Profile
(such as that furnished by varnish) is recommended to A community profile is developed by gathering compre-
strengthen the outer layer of enamel and to protect and hensive facts about a particular community regarding in-
decrease areas of decalcification along the margins. come, education, size, location, disease, health, and other
Use of topical fluoride in combination with enamel characteristics.
sealant placement will help to decrease both smooth A. Community profiles are developed from population
surface and pit and fissure caries activity. Varnish has characteristics that are determined by studying the
some advantages such as ease of application, nonof- following information:
fensive taste, and smaller amounts required than with 1. Number of individuals in a population; geographi-
gel or foam application. cal distribution of the population.
2. Growth rate of the population, density of the popu-
Prevention and Control of Periodontal Disease lation, urbanization vs. rural areas.
Prevention and control of periodontal disease have tra- 3. Ethnicity of the population, socioeconomic status of
ditionally focused on the mechanical removal of dental the population, nutritional status of the population.
biofilm and calculus deposits. Process is accomplished by 4. Types of housing available and standard of living
supragingival dental biofilm removal, daily toothbrush- (number of upper, middle, lower class).
ing and flossing, use of mouthrinses, regular supragin- 5. Public services and utility types, general health
gival and subgingival removal of deposits by a dental status.
hygienist and/or dentist, and use of chemotherapeutic B. Dental disease profiles indicate patterns and distribu-
agents. Future treatment may increase the patient’s host tions of disease, evaluated by the following:
immune response, identify susceptibility and risk for peri- 1. Clinical examination.
odontal disease, and focus on the elimination of specific 2. Review of patient dental records.
bacteria. 3. National health surveys.
C. Dental program profiles include assessment of:
Prevention and Control of Oral Cancer 1. Histories and types of currently existing programs.
Dental professional must conduct thorough head and a. Preventive.
neck examinations of patients and become familiar with b. Treatment oriented.
the prevalence of oral cancer, characteristics, and risk c. Educational.
factors. Dental professionals should identify, document, d. Research.
and refer patients with suspicious lesions. 2. Individual or organization responsible for each
program.
DEVELOPING COMMUNITY ORAL HEALTH 3. Success of the programs.
PROGRAMS   4. Community’s acceptance of the programs.
Developing community oral health programs involves D. Policy- and decision-making profiles include infor-
determining a problem, assessment, planning, imple- mation on:
mentation, and evaluation. In addition, developers must 1. Occupations of financial leaders.
possess the skills and knowledge of communication and 2. Community’s policy makers.
Community Oral Health   563

3. Community’s organizational structure. H. Interpretation of community profile data:


4. Oral health attitudes of community leaders. 1. After all of the information regarding community
E. Community resources profile includes assessment of: profile is gathered, current oral health needs and
1. Funding: priorities are established.
a. Availability of state and local funding for oral 2. Program goals and objectives subsequently are
care, third-party coverage. developed and are aimed at meeting the specific
b. Availability of federal funding, number of private needs identified.
funds through endowments and foundations.
2. Facilities: Planning the Program
a. Location of the closest major medical center; Program planning involves developing a lesson plan and
specialty services available at local medical defining goals and objectives.
center. A. Goals: general statements that describe major purpose
b. Number, type, and locations of dental facilities; of program, lecture, course, or unit of instruction.
use, accessibility, and provision of available B. Objectives: specific, precise, and immediate steps to
dental services. achieving a goal.
c. Whether OSHA and CDC guidelines are in C. Lesson plan:
place at each facility. 1. Instructional “set”:
d. Adequacy and efficiency of dental equipment, a. Introduces content procedures.
number of available operatories in each facility. b. Indicates the value and usefulness of informa-
e. Number of dental laboratories available. tion about to be presented.
3. Labor: c. Motivates and arouses student interest by mak-
a. Number of active, licensed dentists and dental ing it real.
hygienists, and number of dental assistants and d. Ascertains the knowledge base of the audience
laboratory technicians in the area. by presenting information that brings the entire
b. Availability and location of dental and dental audience to the same level; involves adminis-
hygiene schools in the immediate and surround- tering quiz and discussing material from the last
ing areas. presentation.
c. Number of active nurse aides in the commu- e. States objectives for the presentation.
nity. f. Establishes the mood and climate.
d. Number of public health nurses, public health 2. Lecture “body”:
dental hygienists, voluntary health agencies, a. Delivers the bulk of the information.
and nutritionists available. b. Presents the major points.
F. Fluoridated water profile includes: c. Relates information to personal experiences.
1. Type of drinking water available (well water ver- d. Solicits audience participation and uses appro-
sus central community water supplies). priate humor.
2. Fluoride content of water supply and whether or 3. The “closure”:
not it is at optimum levels. a. Summarizes material presented in the “body.”
3. History of water fluoridation in the community. b. Provides a sense of cohesiveness, purpose, and
4. Attitudes concerning water fluoridation held by accomplishment of stated objectives.
the community, dental professionals, decision c. Reviews and summarizes major principles and
makers. key points.
5. Existence of fluoridation laws and referendum d. Does NOT introduce new material.
availability. e. Involves taking questions from the audience
6. Fluoride status of the school water supply. and giving announcements.
G. Profile of professionally applied fluorides assesses: f. Provides the audience with a sense of achieve-
1. Fluoride supplement prescription recommenda- ment; involves thanking group for attention and
tions of community dentists and physicians. participation.
2. Fluoride supplement and rinse programs available
in community schools. Program Implementation
3. Status of fluoride administration at health centers Program implementation is process by which a program
and/or hospitals. is conducted and involves participants, roles, program
4. Toothbrushing program implementation in schools. specifics.
5. Use of fluoride dentifrice and frequency of brush- A. Integrates all external variables.
ing in these programs. B. Involves teamwork.
6. Success of all programs. C. Includes the operation of the plan.
564   Saunders Review of Dental Hygiene

D. Involves following a list of steps in the exact order in 3. Following items should be evaluated:
which they should occur. a. Clarity of speech, enthusiasm, body and facial
E. Involves assigning each individual to a specific expression, positive response to audience ques-
task. tions, maintenance of eye contact.
F. Defines who does what, when, where, and how. b. Smooth implementation of audiovisual ma-
G. Specifies the time allotment for each task and activity. terials, appropriate physical movement when
communicating, organization of presentation,
Program Evaluation comfortable and appropriate atmosphere.
Evaluation of a program involves defining the program c. Allotment of appropriate time for entire presen-
purpose and evaluating the plan, the delivery, and the test tation, minimal use of notes, provision of tran-
results. sitions and summaries.
A. Purpose of evaluation: D. Evaluating test results:
1. To assess and determine the health needs of an in- 1. Instructional objectives serve as a guide for the
dividual, group, or population; directly determines lecture; exams and quizzes are developed to cor-
the content of the health instruction and ensures relate with stated objectives.
that the information will be meaningful to the spe- 2. Exams and quizzes subsequently can be used
cific group identified. for evaluation; SHOULD accurately reflect the
2. To determine the strengths and weaknesses of the ­effectiveness of instruction; student feedback
teaching personnel, the teaching strategies, and the can provide information for the revision of future
organization of the instruction. ­lessons.
3. To assess the desired outcomes of oral health pro-
grams and the extent to which program objectives COMMUNITY HEALTH STUDY 
are met, and to determine a range of cognitive and Scenario: A dental hygienist prepared an oral health edu-
behavioral outcomes. cation program regarding smokeless tobacco for a sixth-
B. Evaluating the plan: grade class. One of the objectives of the program was as
1. Presenter should be thoroughly familiar with the follows: “After the lecture and videotape, ‘Big Dipper,’
subject matter; the material should be current and the student will list and describe three ingredients in a can
accurate. of chew.” The major points of the lecture were presented.
2. Information should be appropriate to the level and The dental hygienist related specific information in the
needs of the audience. content to personal experiences with patients.
3. Introduction (“set”) should be interesting and ap-
propriate, bulk (“body”) of the lecture should be 1. Describe the “condition element” of the instructional
organized and logical. objective.
4. Plan for explaining important points fully and 2. Identify how the dental hygienist can best evaluate the
thoroughly should be established. learning of the sixth-grade class.
5. Supplementary materials must be as current as pos- 3. During the lecture the dental hygienist presented the
sible, technical terms must be defined, transitions major points and related specific information to per-
from one piece of information to another should sonal experiences. Which part of the lecture was be-
be smooth; audiovisual materials should help illus- ing delivered?
trate and enhance the main points. 4. In which population group noted is oral cancer preva-
6. Points in the program for assessing the audience’s lent?
understanding of the information presented should
be identified. 1. “After the lecture and videotape” best describes con-
7. Ending (“closure”) should be effective and include dition element of the instructional objective. Gives
NO new material. the stipulation of the objective, “the student” refers
8. Lecture notes should be easy to read; information to the audience; “will describe and list” is a behavior
presented should allow students to reach the stated required of the student; and “three ingredients” de-
instructional objectives. scribes the degree element, or how many the student
C. Evaluating the delivery: must be able to list in order to pass.
1. Can be accomplished by reviewing a videotape of 2. Dental hygienist can best evaluate the learning of the
the presentation and gathering feedback from the sixth-grade class through testing.
audience and peers. 3. Presenting the major points of the lecture and re-
2. All parts of the presentation must be assessed, in- lating specific information to personal experiences
cluding content, organization, and style. occurs in the body of the lecture. Other components
Community Oral Health   565

of the lecture format include the set, during which c. Single-subject research: studies that outline
materials are introduced, and the closure, during only one or a few subjects who may have a
which information is summarized and NO new mate- ­special problem or disorder rarely described in
rial is presented. the literature.
4. Oral cancer is MOST prevalent among male and d. Important components:
­female tobacco users. (1) Reliability: measure of accuracy of the re-
search methods (e.g., whether the test, in-
RESEARCH METHODS IN COMMUNITY strument, inventory, or questionnaire gives
DENTISTRY  the same results each time so it is reproduc-
Research can be defined as a continual search for truth by ible).
means of the scientific method. Research in community (a) Inter-rater reliability: measure of re-
dentistry is categorized as biomedical research, which en- liability among two or more evaluators
compasses basic laboratory and clinical research, behav- over time; calibration conducted peri-
ioral science, research regarding educational techniques, odically during the study to check con-
and administrative and evaluative research of community currence among examiners.
dental programs. (b) Intra-rater reliability: measure of the
A. Biomedical clinical and experimental research typi- reliability of one evaluator over time;
cally involve clinical trials: calibration of individual examiners con-
1. Involve the use and application of epidemiology ducted throughout study to ensure that
and basic laboratory research. diagnostic technique does NOT change.
2. Used to evaluate specific dental AND dental hy- (2) Calibration: method of unifying examiners
giene techniques and therapeutic agents. in the diagnostic technique; accomplished
a. Investigational new drug (IND) number: by having examiners engage in trial runs on
(1) Must be obtained before assessing and several sample cases and comparing find-
evaluating a drug that has NOT been tested ings of and between examiners; process
on a human population. repeated until all examiners concur; should
(2) Obtained from Food and Drug Administra- be checked and recalibrated during study.
tion (FDA). (3) Validity: measure of accuracy of the re-
3. Requires review by Institutional Review Board search methods (e.g., does the test, instru-
(IRB). ment, inventory, or questionnaire measure
a. Committee that reviews proposed research pro- what it intended to measure?).
tocols involving human subjects for the protec- B. Educational and behavioral research:
tion, safety, and privacy of participants. 1. Assesses and evaluates application of an educa-
b. Committee must include the following: tional or behavioral technique in dentistry or den-
(1) Minimum of five members, male and fe- tal hygiene to individual or group.
male members. 2. Focuses on knowledge, attitudes, behaviors re-
(2) Professional representation by at least one garding oral health and disease.
nonscientific professional and at least one 3. Typically involves studies of short duration; typi-
consumer or lay individual NOT directly cally involves smaller populations than experi-
involved with the institution. mental studies.
4. Three categories of research: C. Administrative and evaluative research:
a. True experimental research: 1. Used for program evaluation in community dentistry:
(1) Involves random selection; all subjects in a. Assesses program’s operation, effectiveness,
the sample have an independent and equal and need for improvements.
chance of being selected; all elements b. Determines how a newest innovation has been
are selected at random (e.g., numbers are accepted and used in professional dental com-
drawn from a hat and replaced before each munity (e.g., placement of sealants over incipi-
consecutive drawing). ent decay).
(2) Involves random assignment; popula- 2. Often involves questionnaire surveys:
tion subjects are assigned to each group at a. Survey instrument: set of specifically designed
­random. questions that are carefully planned and tested
b. Quasi-experimental research: in terms of context, structure, appearance.
(1) Does NOT involve random selection; the (1) Used to gain knowledge from particular
subjects are selected as groups. population regarding information useful to
(2) Involves random assignment. researcher.
566   Saunders Review of Dental Hygiene

(2) Number of subjects depends on the com- was developed that requested information regarding age,
plexity of the study. the number of remaining natural teeth, whether dentures
(3) Goals of the study should be well defined; are worn, whether the senior citizen has difficulty eating,
ability to evaluate differences among swallowing, or speaking, perceived cause of any difficul-
groups should be tested. ties, and perceived needs. Eighty-one seniors were sur-
(4) Survey question types: veyed. Table 17-2 summarizes the results.
(a) Dichotomous response involves yes/no
or true/false questions. 1. What is the mean number of teeth for seniors 65 to
(b) Multiple choice involves list of possible 69 years of age? 70 to 74 years of age? 75 to 79 years
answers. of age?
(c) Quantifiable number response, answers 2. Based on the results of the survey, which one of these
involve specific number (e.g., number statements could be made: (a) These senior citizens
of days per week, month, or year). have fewer teeth than the general population; (b) Most
(d) Written response involves subjective of these senior citizens wear dentures; (c) These senior
answers to questions (difficult to inter- citizens have more bad breath than other groups; or
pret). (d) The senior citizens between the ages of 70 and 74
(e) Likert scale MOST often used to elicit are more likely to suffer a stroke.
feelings and NOT mere facts. 3. Based on the results of the survey, what should be the
3. Pretesting: emphasis of the 1-hour dental presentation for senior
a. Involves testing survey instrument on a small citizens?
group of individuals who are not part of, but have 4. What recommendations would help the senior citizens
characteristics similar to, the study sample. who are having difficulty swallowing?
b. Assists in gathering information and comments
regarding questionnaire itself. 1. Mean number of teeth remaining in seniors between
c. Allows errors, inconsistencies, misunderstand- ages of 65 and 69 is 16, between ages of 70 and 74 is
ings, length to be commented on and corrected 13, and between ages of 75 and 79 is 5.
before the study takes place. 2. According to results of the survey, most of these ­senior
citizens wear dentures. General population data are not
COMMUNITY HEALTH STUDY   available for comparison to determine whether these
Scenario: The activities coordinator of a senior citizens’ senior citizens have fewer teeth or more bad breath than
center in a rural community asked the dental hygiene pro- others. No information is provided that ­allows assump-
gram director to present a 1-hour dental program. Eighty- tions regarding likelihood of strokes in this group.
eight senior citizens attend the local senior center. The 3. Based on the results of survey, emphasis of the 1-hour
program director decided to survey the attendees to de- dental presentation should be on loose dentures. Two of
termine their dental concerns and possible needs before the three groups had perceived needs ­involving loose
developing the program. Twelve dental hygiene students dentures. Third group indicated that denture sores were
volunteered to gather the information. An eight-item survey problematic and may be related to loose dentures.

Table 17-2  Senior citizen survey results for community health study

Item Age 65-69 (46 persons) Age 70-74 (33 respondents) Age 75-79 (2 respondents)

Average number of 16 13 5
remaining teeth
Wear dentures 78% 87% 100%
Difficulty swallowing 52% 67% 50%
Difficulty speaking 23% 28% 50%
Difficulty eating 44% 33% 100%
Cause of difficulty Dry mouth, loose dentures Dry mouth, loose dentures, Dry mouth
stroke
Perceived needs Loose dentures, bad breath, Cleaning dentures, loose Preventing denture sores,
yellow teeth ­dentures, toothpaste bad breath
­selection, yellow teeth
Community Oral Health   567

4. Because many seniors felt that dry mouth contributes


to difficulty, they need the following recommenda- COMMUNITY HEALTH STUDY  
tions: sipping water, oral lubricants, air humidifica- Scenario: Omar University has just been awarded funds
tion, chewing sugar-free gum and candy. Referral to to conduct research in an elementary school setting that
a physician recommended if moistening therapies do targets fifth- and sixth-grade children. The dental hygiene
not help problem. professors at the university developed the research design
and submitted a plan regarding the research and educa-
Government-Conducted Community Dental tion they would provide. The target elementary school
Research was located in a nonfluoridated, rural community. After
Much of the money spent on community dental research examining the data collection, researchers found that the
is provided by NIDCR; research is funded according to incidence of caries, dental biofilm, and gingivitis was
its current priorities. high. The researchers planned intervention that includes
A. Intramural research: conducted within a facility by oral health education and sealant placement and not a
in-house researchers employed by that facility (e.g., school fluoridation program.
NIDCR).
B. Extramural research: 1. What type of extramural research and approval, fund-
1. Agreement between the federal government and an ing, and announcement process did Omar University
outside institution to conduct identified research. use to obtain funding?
2. Two types of extramural research: 2. What primary limitation is involved in the implemen-
a. Grants: tation of a school water fluoridation program?
(1) Research money awarded to individual or 3. After the oral health education program has been
institution to conduct research protocol. implemented, how can the presenters best evaluate
(2) Protocol has been developed and defined by whether the desired outcomes of the program were
individual or institution requesting money. ­accomplished?
b. Contracts: 4. Explain why many dental public health education
(1) Research money awarded to individual or programs are conducted in the elementary school
institution to conduct research protocol. setting.
(2) Differs from grant in that the protocol has
been defined by the federal government, 1. Extramural research and approval, funding, and
NOT by the researcher. ­announcement process that Omar University used to
(3) Researchers do NOT develop their own obtain funding involved a grant and request for appli-
protocols; they consent to conduct research cation (RFA). Grants are awarded to individuals and in-
that already has been defined by the federal stitutions who develop their own protocol for a specific
government. research area. Money is applied for through an RFA.
3. Announcement and application of grants and con- 2. Primary limitation of a school water fluoridation pro-
tracts; two types: gram is that the children are already 5 to 6 years of age
a. Requests for application (RFAs): when fluoridation begins, which decreases the benefi-
(1) Federal government announces a general cial effects on their teeth. However, fluoride level of
area of research priority. water is adjusted to 4.5 ppm for school fluoridation,
(2) Researchers who desire to apply submit a which may lead to higher than recommended concen-
grant with a protocol that addresses the pri- trations. Involving teachers and other nontrained per-
ority area. sonnel and getting permission are also disadvantages.
(3) Researchers may define and develop their At least the children would not swallow the rinse,
own goals and protocols for conducting the which younger children may do.
research. 3. Best way for the presenters to evaluate whether the
b. Requests for proposal (RFPs): desired outcomes have been accomplished is by eval-
(1) Federal government develops a detailed uating the extent to which the objectives were met.
definition of the research goals and One must compare the objectives to the actual content
­protocol. delivered.
(2) Researchers and institutions interested in 4. Many oral health education programs are conducted in
the defined protocol are invited to apply school settings because the information can be deliv-
through a contract. ered to large groups of students at once. Use of school
(3) Interested researchers must describe typically is the most time- and cost-effective approach,
methods for meeting the specific protocol and a school is often selected because of location and
terms. availability.
568   Saunders Review of Dental Hygiene

Review Questions   9 In 2008, 250 cases of squamous cell carcinoma were re-
ported among a group of 10,000 18-year-olds examined in
Fort Rock, Arizona. This is an example of
  1 Placing an amalgam restoration after removing caries from A. rate.
the tooth surface is an example of B. incidence.
A. health promotion. C. prevalence.
B. primary prevention. D. morbidity.
C. secondary prevention. E. mortality.
D. tertiary prevention. 10 Which of the following epidemiological indices would be
  2 The number of individuals in a given population who ­received BEST for evaluating caries in a group of adults between
dental care services during a specific time period refers to the ages of 70 and 85 who experience gingival recession,
A. demand. inflammation, and bleeding?
B. need. A. DMFT
C. use. B. DMFS
D. normative need. C. UTN
  3 Which of the following statements is NOT correct? D. RCI
A. More women than men use dental services. 11 Joe Black presents with generalized, moderate marginal
B. As an individual’s socioeconomic status (SES) rises, his dental biofilm, inflammation, and bleeding. He indicates
or her use of dental services increases. that he brushes once a day and flosses when food is caught
C. Caucasians use dental services more often than African between his teeth. Which of the following indices would
Americans or Hispanic Americans. be BEST for monitoring Joe’s oral hygiene performance,
D. Individuals with poor general health are more likely to indicating specific areas of need, and helping Joe with his
use dental services. oral care at home?
  4 Which factor below is interrelated with all of the factors that A. OHI-S
influence the use of dental care services? B. PHP
A. Age C. GI
B. Ethnicity D. O’Leary plaque index
C. Gender 12 Which of the following indices uses the selection of six
D. Socioeconomic status teeth to compare dental biofilm removal performance at
  5 Which of the following is a contract between the patient, the each subsequent visit and helps move a patient toward bet-
dental office, and the insurance company? ter oral health?
A. Fee for service A. OHI-S
B. Indemnity plan B. PHP
C. Third-party plan C. GI
D. Capitation plan D. Russell’s periodontal index (PI)
  6 Which statement below BEST describes the term “reason- 13 Which of the following indices contains BOTH a debris in-
able fee”? dex and a calculus index, which are summed for a single
A. The actual fee charged by the provider for a rendered score?
service A. OHI-S
B. The most frequently charged fee for a particular service B. PHP
C. A minimum fee set by the dental insurance company or C. GI
plan D. PI
D. A charged fee that exceeds the fee set by an insurance 14 The ideal epidemiological index for evaluating specific oral
company disease is one in which
  7 An insurance plan that gives a fixed, monthly payment to a A. elaborate mechanisms are used to evaluate all aspects of
dentist provider, whether or NOT an individual assigned to the disease.
that dentist receives care, is BEST described by which of B. simple indices, which are easily understood and ex-
the following terms? plained, are used.
A. Third-party plan C. the index is not reproducible.
B. Indemnity plan D. all possible factors are recorded.
C. Capitation plan 15 After the gingival health of Tom Cook, a 43-year-old busi-
D. Delta Dental Plan nessman, is evaluated, his teeth are divided into facial, lin-
  8 Which of the following statements correctly describes ­direct gual, distal, and mesial areas and the tissue is assessed for
reimbursement? color, contour, bleeding, and ulceration. Which of the fol-
A. An insurance company directly reimburses an individ- lowing indices is being used?
ual for a fee paid to a provider. A. O’Leary plaque index
B. An employer directly reimburses an employee for den- B. PHP
tal services paid for out of his or her pocket. C. PI
C. A fixed, monthly fee is paid to a providing dentist. D. Ramfjord’s periodontal disease index (PDI)
D. An actual fee exceeds the fee set by an insurance company. E. GI
Community Oral Health   569

16 Which of the following indices evaluates attachment loss 24 The University of Troy has proposed a study to evaluate
and the progressive stages of periodontal disease? the effects of a new antigingivitis drug, which has NOT
A. PI been tested in a human population. To conduct this study,
B. PDI researchers must FIRST obtain
C. CPITN A. approval from the IRB.
D. GI B. an IND number.
17 A public health dental hygienist has been asked to assess the C. FDA approval.
periodontal status and treatment needs of the older adult patients D. informed consent.
in an assisted living community. To collect this information, the 25 Which type of clinical trial research exhibits properties of
dental hygienist would use which of the following indices? both random assignment and random selection?
A. PI A. True experimental
B. PDI B. Quasi-experimental
C. CPITN C. Single-subject
D. GI D. Questionnaire survey
18 One of the following is an INCORRECT statement about 26 After the implementation of an early childhood caries
water fluoridation. Which one is INCORRECT? (ECC) program for low-income mothers, data were col-
A. Has been shown to be effective against decay lected to determine the program’s effectiveness. This study
B. Requires no individual effort is an example of
C. Is relatively expensive to administer A. clinical trial research.
D. Is a safe measure against caries B. administrative and evaluative research.
19 Enamel sealant placement is C. educational and behavioral research.
A. recommended for children and young adults living in D. experimental research.
fluoridated or nonfluoridated communities. 27 A dental hygienist wishes to assess the current skill level
B. recommended for all age groups in fluoridated and non- of a group of school nurses regarding the administration
fluoridated communities. of a fluoride mouthrinse program in the local elementary
C. recommended for all age groups in nonfluoridated com- schools. Which of the following is the suggested method for
munities only. collecting data regarding the skill level of these nurses?
D. limited in its use and is applied only by a dentist. A. Interview
E. limited in its use because of low rates of retention and B. Written questionnaire
efficacy. C. Interviews with the school principals
20 Roselyn, a 16-year-old, presents with several carious le- D. Direct observation
sions on occlusal surfaces. She lives in a community with 28 When conducting a questionnaire survey, it is important to
0.7 ppm of fluoride in the drinking water. Which of the fol- A. have respondents include their names on returned ques-
lowing would be the BEST oral health measure for prevent- tionnaires.
ing further decay? B. use questions that require a written response of feel-
o7300 A. Enamel sealants ings.
B. Fluoride mouthrinse C. identify specific populations who possess information
C. Diet counseling that is useful to the survey.
D. Fluoride supplements D. send the questionnaire to as many individuals as pos-
21 All of the following are necessary for successful diet coun- sible.
seling, EXCEPT one. Which one is the EXCEPTION? 29 Foster University recently was awarded money to conduct
A. Evaluation and counseling regarding food consumption research concerning the geriatric population. The profes-
are conducted in the same manner with all patients. sors at Foster developed the research design and submitted
B. Patients exhibit cooperation and motivation. a plan to address the oral health needs of the geriatric popu-
C. Rapport has been established between the oral health lation in their state. Which type of extramural research and
professional and the patient. approval, funding, and announcement process did Foster
D. Reevaluation and assessment have been completed. University undergo to receive this money?
22 When research is conducted in a university setting, a proj- A. Grant, RFA
ect must receive which of the following before research is B. Grant, RFP
­allowed to start? C. Contract, RFA
A. IND D. Contract, RFP
B. IRB approval 30 All of the following are characteristics of informed consent
C. Informed consent given during a research project, EXCEPT one. Which one
D. FDA approval is the EXCEPTION?
23 The IRB is responsible primarily for A. Each subject is guaranteed confidentiality.
A. approving research on human subjects. B. Subjects are penalized for withdrawal.
B. practically applying research. C. An explanation of the purpose of the research is given.
C. regulating administrative procedures. D. An explanation of procedures and treatments is given.
D. approving and improving the operation of a program.
570   Saunders Review of Dental Hygiene

31 Information regarding the growth rate, geographical distri- 39 Which teeth are known as the Ramfjord teeth?
bution, ethnicity, and socioeconomic status of a population A. #2, #6, #12, #19, #25, and #31
refers to which component of the community profile? B. #2, #8, #13, #18, #26, and #29
A. Community resources C. #3, #9, #12, #19, #25, and #28
B. Population characteristics D. #3, #10, #13, #17, #24, and #30
C. Dental disease 40 Which of the following was developed by the Office of
D. Policies and decision making Economic Opportunity and provides services to low-
32 In the community resources component of the community income preschool children?
profile, which of the following terms BEST describes the use, A. Medicaid
accessibility, and provision of available dental services? B. Medicare
A. Labor C. Head Start
B. Funds D. Maternal and Child Health Services
C. Facilities 41 Which of the following BEST describes dental services ren-
D. Fluorides dered through the Indian Health Service?
33 A student is providing a lecture on smokeless tobacco. She A. Are provided by dentists and auxiliaries who are em-
presents the major principles and key points of her lecture. ployed by the federal government
Which part of the lecture (program) is the student delivering? B. Are provided by dentists and auxiliaries and are financed
A. The set by the patient or through other means
B. The body C. Are provided by dentists and auxiliaries who are not
C. The closure government employees
D. The introduction D. Are provided by dentists and auxiliaries who are in a
34 A public health dental hygienist has designed an educational group practice with a health maintenance organization
program on early childhood caries for expectant parents. 42 Troy, Colorado, is a rural mountain town of approximately
Information regarding the audience’s current knowledge of 400 people. The town has inadequate dental services, and
the disease is gathered by means of a quiz administered at the closest town with comprehensive dental services is 180
the beginning of the presentation. The administration of the miles away. The federal government sends commissioned
quiz occurs during which part of the program? and civil service dental professionals to the area to meet the
A. The set dental needs of this community. This service is provided
B. The body by:
C. The closure A. Maternal and Child Health services
D. The introduction B. National Health Services Corps
35 Two students are reviewing a videotape of their presenta- C. State Health Agency services
tion. These students are evaluating the D. Medicaid services
A. plan. 43 Which government program includes an early periodic
B. delivery. screening, diagnosis, and treatment component?
C. learning. A. Medicaid
D. plan and the delivery. B. Head Start
E. plan, the delivery, and the learning. C. Medicare
36 It is MOST advantageous to conduct evaluation during D. Maternal and Child Health Services
which part of oral health education? 44 Which of the following was the primary goal of the health
A. The plan maintenance organizations (HMOs)?
B. The delivery A. To provide a range of healthcare services to partici-
C. The learning pants
D. The plan and the delivery B. To help lower the costs of healthcare
E. The plan, the delivery, and the learning C. To increase competition for patients
37 How can a program presenter BEST evaluate the desired D. To enable patients to have a wider choice of providers
outcome of his or her oral health education program? 45 A contract between several practitioners and an insurer to
A. By evaluating the extent to which the objectives were provide healthcare services for lower-than-average fees is
met referred to as a(n)
B. By evaluating the teaching strategies used A. health maintenance organization (HMO).
C. By evaluating the organization of the instruction B. capitation plan.
D. By evaluating the format and style of the presenter C. group practice.
38 One of the following is NOT a criterion for determining D. preferred provider organization (PPO).
whether a particular disease constitutes a public health 46 The rate of occurrence of a new disease in a population dur-
problem. Which one is the EXCEPTION? ing a given period is referred to as its
A. The knowledge of how to alleviate the problem is not A. incidence.
being applied. B. prevalence.
B. The disease is widespread throughout the community. C. patterns.
C. The disease cannot be prevented by any means. D. severity.
D. The disease can be cured easily.
Community Oral Health   571

47 All of the following statements are true, EXCEPT one. 4 (D)  Socioeconomic status (SES), measure of BOTH
Which one is the EXCEPTION? income and education, is interrelated with all of the
A. The prevalence of periodontal disease does not increase other factors that affect individual’s use of dental ser-
with age. vices. Other factors (age, ethnicity, and gender) are
B. Blacks consistently demonstrate a higher incidence of NOT interrelated with regard to effect on individual’s
periodontal disease than whites.
use of dental services.
C. Periodontal disease is inversely related to levels of edu-
5 (C)  Third-party payment plans are insurance contract
cation.
D. The prevalence of periodontal disease is higher in rural between dental office, patient, and insurance com-
than in urban areas. pany. Fee-for-service is a two-party plan whereby
48 Which of the following is a method of unifying examiners patient pays an out-of-pocket fee to provider. Indem-
in the diagnostic technique? nity plans operate in the same way as traditional in-
A. Calibration surance. Capitation plans refer to contracts between
B. Reliability dentists and insurance companies to provide services
C. Validity to patients.
D. Pretesting 6 (A)  Reasonable fees are actual fees charged by den-
49 The Plaque Assessment Scoring System (PASS) evaluates tal provider, modified according to each patient’s in-
A. supragingival dental biofilm.
dividual need. These fees may or may NOT exceed
B. fissure dental biofilm.
C. subgingival dental biofilm.
dentist’s usual fee and/or customary fee allowed by
D. both supragingival and subgingival dental biofilm. insurance company. MOST frequently charged fee
50 What is the MOST appropriate index to use to measure pro- is the usual fee. Customary fee is maximum benefit
gressive stages of periodontal disease and amount of clini- payable under particular plan.
cal attachment loss? 7 (C)  Capitation plan is defined as fixed monthly pay-
A. DFTS ment based on the number of patients assigned to the
B. PI dentist. Insurance company takes into account that
C. deft some patients will seek significant amount of care
D. dmfs and others will NOT seek any care. Third-party plan
E. PDI is a contract between dental office, patient, and insur-
ance company. Indemnity plans are traditional insur-
ance plans with submittal and reimbursement, and
Delta Dental provides dental care contracts through
Answer Key and Rationales a customary fee structure.
8 (B)  Direct reimbursement occurs when an employer
agrees to pay a portion of an employee’s dental ser-
1 (C)  Placing an amalgam is a secondary prevention vices. The employee seeks care and pays the dentist
technique, designed to terminate a disease (e.g., car- directly, and the employer reimburses the employee
ies) and restore tissues to near normal function (e.g., for those services. When insurance company directly
amalgam restoration). Primary prevention prevents, reimburses individual, this is through an indemnity
reverses, or arrests disease before treatment is needed plan. Capitation refers to fixed monthly fee that is
such as with the use of fluoride and enamel sealants paid to providing dentist. Actual fee charged by the
in regard to caries. Tertiary prevention replaces lost dentist or provider can exceed the fee set by the insur-
tissues and rehabilitates to near normal function such ance company and is referred to as customary; how-
as the delivery of a denture when teeth are lost so as to ever, patient typically pays difference.
still allow a person to eat and speak. Health promotion 9 (D)  Prevalence is term that describes 250 cases
refers to individual’s health-related behavior change. of squamous cell carcinoma in a group of 10,000
2 (C)  Use of dental services is defined as proportion of 18-year-olds in 2008. Prevalence is defined as the to-
a population who receive dental care services within tal number of cases of disease in a given population
a given period. Demand is a combination of perceiv- at a specific time. Rate refers to a ratio of actual to
ing need and seeking services. Need is identification possible cases. Incidence is number of new cases of a
of an oral disease, and normative need is amount of disease. Morbidity refers to disease, and mortality re-
oral care needed to keep a community healthy. fers to death; BOTH are typically described as rates.
3 (D)  The statement that “individuals with poor gen- 10 (D)  Root caries index evaluates BOTH the risk and
eral health are MORE likely to use dental services” the extent of root caries in areas exposed by gingival
is false. Those who do NOT seek routine, preventive recession and would be the BEST index for a group
care for general health are NOT likely to seek care for of adults between the ages of 70 and 85 who exhibit
oral health. Others are TRUE. recession, inflammation, and bleeding. DMFT and
572   Saunders Review of Dental Hygiene

DMFS indices evaluate decayed, missing, and filled This information is used to determine specific peri-
teeth and surfaces, respectively. The UTN index di- odontal treatment needs of a given population. Other
vides the mean number of decayed teeth by the mean indices are NOT designed to determine the specific
number of decayed and filled teeth to determine the needs of a population.
percentage of teeth that need treatment. The DMFT, 18 (C)  Water fluoridation is NOT expensive to adminis-
DMFS, and UTN indices do NOT address root ter; very inexpensive, ranging in cost from $0.12 to
­caries. $1.16 per individual per year. The other statements
11 (D)  O’Leary plaque index would be the BEST selec- are TRUE.
tion for monitoring Joe’s oral hygiene performance 19 (A)  Enamel sealants are recommended for children
by indicating the location of dental biofilm. It would and young adults in fluoridated and nonfluoridated
enable him to visualize areas that need concentration communities. They can be applied by dentists and
and areas of thorough dental biofilm removal. The dental hygienists or assistants (depending on state
OHI-S evaluates BOTH debris and calculus and is practice acts) and have excellent retention and effi-
used MAINLY on large populations. The GI evalu- cacy against decay.
ates gingivitis, and the PHP evaluates dental biofilm 20 (C)  Diet counseling is indicated when a patient has
on six selected teeth. a high rate of caries but is still taking the appropriate
12 (B)  The PHP index evaluates dental biofilm on six amount of fluoride. The age of the patient (16) indi-
selected teeth, gives specific information about a pa- cates the possible consumption of highly cariogenic
tient’s dental biofilm removal ability, and can also be foods. Enamal (pit and fissure) sealants are NOT in-
used during subsequent visits to demonstrate improve- dicated for carious lesions. She is receiving the COR-
ment over time. The OHI-S evaluates BOTH debris RECT amount of fluoride from the water supply;
and calculus for large populations. The GI evaluates therefore supplementation is NOT indicated.
gingivitis, and Russell’s PI assesses the progressive 21 (A)  Success requires that patients be evaluated and
stages of periodontal disease and attachment loss. counseled according to their specific needs and NOT
13 (A)  The OHI-S contains BOTH debris and calculus in the same manner as other patients. Recommenda-
components and examines the facial surfaces of teeth tions SHOULD be different for each patient. When
#3, #8, #14, and #24 and the lingual surfaces of teeth successful counseling takes place, patients exhibit
#19 and #30 (see rationale for question 12). cooperation and motivation, especially when rapport
14 (B)  Ideal index is reliable, reproducible, valid, easily has been established. Reevaluation and assessment
understood, and easily explained. Elaborate mecha- also must take place after the patient implements new
nisms that evaluate several factors are MORE diffi- recommendations.
cult to understand and reproduce. Indices should be 22 (B)  IRB approval is necessary before any research
able to be replicated easily, and only necessary fac- is conducted on human subjects. An IND number is
tors should be recorded for the study. obtained before evaluating new drugs never tested on
15 (E)  The GI is used MAINLY to evaluate gingivitis human populations. Informed consent occurs after
by assessing tissue for bleeding, color, contour, and IRB approval and refers to obtaining the consent of
ulceration. The O’Leary PI ONLY records the loca- the patient enrolled in a research study. FDA approval
tion of dental biofilm. The PHP assesses soft and hard is required to market a new drug after research has
debris and deposits. Russell’s PI assesses periodontal been conducted.
disease and attachment loss, and Ramfjord’s index 23 (A)  Primary responsibility of the IRB is to review
evaluates gingival dental biofilm, probing depths, and and approve research on human subjects in order to
calculus. protect their privacy and safety.
16 (A)  Russell’s PI is used MAINLY to evaluate the pro- 24 (B)  University of Troy must obtain an IND number
gressive stages of periodontal disease and attachment through the FDA if the drug being evaluated has NOT
loss in major populations. Scores range from 0 to 8 been tested in a human population. The IRB reviews
in severity. Ramfjord’s PDI evaluates gingival den- proposed research on human subjects to protect their
tal biofilm, probing depths, and calculus. The CPITN safety and privacy. Patients MUST give their in-
evaluates pockets, bleeding, and dental biofilm in six formed consent to participate in a study.
sextants and is also used MAINLY to identify the 25 (A)  True experimental research includes BOTH
treatment needs of specific groups. The GI evaluates random selection and random assignment; quasi-
gingivitis. ­experimental research does NOT involve random se-
17 (C)  The CPITN would be MOST appropriate for lection. Single-subject studies are conducted on one
older adult patients in the assisted living community. or a few subjects who exhibit a rare or specific condi-
It was developed by the World Health Organization to tion. Questionnaire surveys refer to research through
assess pocketing, bleeding, dental biofilm retention. written or oral response.
Community Oral Health   573

26 (B)  Collection of data to determine a program’s ef- the specialty services available, the number, type,
fectiveness BEST describes administrative and eval- and location of dental facilities, the number of op-
uative research, which evaluates the operation and eratories and laboratories available, and the adequacy
effectiveness of and the need for improvement in an of equipment. Labor describes the number of active
oral health education program. Clinical trials evaluate dental and other healthcare workers. Funds encom-
techniques and therapeutic interventions. Education pass all available monetary resources, and fluorides
and behavioral research assess behavioral education refer to BOTH water fluoridation and professionally
techniques and focus on knowledge, attitudes, and applied fluorides in the community.
behaviors. Experimental research involves clinical 33 (B)  The “body” portion of a presentation provides
trials and experiments. the major points and key principles. The “set” intro-
27 (D)  Direct observation would be the BEST method duces content and establishes mood. The “closure”
for learning about the school nurses’ skill level in flu- summarizes the body and does NOT provide new ma-
oride administration. It would enable the researcher terial. “Introductions” are similar to the set.
to identify any inconsistencies and/or needs for im- 34 (A)  Instructional set of a presentation introduces the
provement. Interviewing would provide ONLY self- content or procedure to be discussed, describes use-
reported information, whereas a written questionnaire fulness of the information, motivates the student and
would limit actual implementation techniques, and arouses the student’s interest, can be used to assess
interviewing the principals would NOT gather infor- the audience’s knowledge base by administering a
mation from the individuals directly involved. quiz or reviewing previous material, states the objec-
28 (C)  Questionnaire surveys SHOULD be sent to tives, and establishes a mood for the presentation.
subjects who possess specific information that the 35 (B)  Evaluating delivery of one’s presentation can
researcher wants to capture. Having the individuals be accomplished by reviewing a videotape and/or
write their names would violate confidentiality, re- gathering feedback from the audience and peers. Stu-
sponse of feelings is difficult to interpret, and sending dents’ learning can be evaluated through testing, and
the questionnaires to as many individuals as possible the plan is evaluated by comparison with the stated
would elicit unwanted information. objectives.
29 (A)  Grant is awarded to researchers to conduct re- 36 (E)  Evaluation should take place during all steps of
search protocol that has been defined by party receiv- developing an oral health education program.
ing the award. Grants are applied for through a request 37 (A)  The outcome of a program can be evaluated by
for application (RFA), in which a plan for meeting the comparing the objectives with the outcome and as-
priority area is submitted. Contract is awarded to in- sessing the degree to which each objective was met.
dividual or institution, and protocol already is defined Evaluating teaching strategies gives information on
by government agency awarding the money, applied the delivery and plan, the organization, and the pre-
for through an RFP (request for proposal). sentation format and style.
30 (B)  Subjects are NOT penalized for withdrawing 38 (C)  Public health problem must meet the following
from a study; rather, all subjects are guaranteed the criteria: (a) disease is widespread, (b) disease can be
ability to withdraw at any time without prejudice or prevented, alleviated, or cured; (c) knowledge of how
repercussion. Confidentiality is kept throughout the to alleviate problem is NOT being applied.
study. Purpose of the research and its procedures and 39 (C)  Ramfjord teeth are teeth #3, #9, #12, #19, #25,
treatments are fully explained to each participant. and #28, MOSTLY used in clinical trial research as
31 (B)  Population characteristics of a community pro- representative sample of the entire dentition.
file include the number of individuals in the popula- 40 (C)  Head Start was developed by Office of Economic
tion and their geographical distribution, growth rate, Opportunity as result of the Economic Opportunity
population density, urban vs. rural areas, ethnicity, Act. Primary purpose was to provide social services
SES, nutrition and diet, types of housing availabil- and health education to low-income preschool chil-
ity, standard of living, public services, utilities, and dren so that they would be able to enter school on an
general health status. Community resources include equal ground with higher income peers.
funds, facilities, and labor available for use in the 41 (A)  Dental services rendered by the Indian Health
community. Dental disease tracks the patterns and Service are provided by dentists and auxiliaries who
distribution of disease. Policy making and decision are employed by the PHS. These providers are com-
making outline the community’s organizational struc- missioned officers of the Dental Service Corps. Oth-
ture and the oral health attitudes of its leaders. ers do not fit this description.
32 (C)  Facilities describe the use, accessibility, and 42 (B)  National Health Services Corps is a federal
provision of dental services, including the location health labor deployment program. It provides com-
of the closest major medical center in a community, missioned officers and civil service individuals of the
574   Saunders Review of Dental Hygiene

PHS to areas where there are inadequate health ser- fee for healthcare and choose physicians within their
vices because of shortages. The federal government HMO. Capitation plan pays a fixed fee to a provider to
provides Maternal and Child Health Services for deliver care to participants who are assigned for care.
women of childbearing age with low incomes and for Group practice refers to a group of several healthcare
children under 21 years of age. State health agencies professionals who provide services together.
provide services locally in each state, and Medicaid 46 (A)  Incidence refers to the rate of occurrence of a
provides healthcare to those under 21 years of age new disease in a population during a given period of
who are in a “needy” group. time. Prevalence refers to the number of persons af-
43 (A)  Medicaid administers early periodic screening, fected by a disease at a specific point in time. Patterns
diagnosis, and treatment program. Medicaid services are formed when data are repeated over time. Sever-
are provided to specific needy people. Head Start is ity refers to the extent of a disease.
a federally funded program distributed by the Office 47 (A)  Although aging does NOT preordain periodon-
of Economic Opportunity to provide educational and tal disease, studies have shown that the prevalence
social services to low-income preschool children. of periodontal disease increases directly with age,
Medicare offers health insurance to individuals 65 MOST likely because of repeated exposures to bacte-
years of age and older. Maternal and Child Health rial infection over a lifetime.
Services provide health services for women of child- 48 (A)  Calibration is a method of unifying examiners in
bearing age and for children under 21 years of age. the diagnostic technique.
44 (B)  HMOs were developed MAINLY to help lower 49 (C)  PASS index evaluates subgingival dental bio-
the costs of healthcare. Range of services, competi- film, unlike the O’Leary plaque index, which only
tion, and a choice of providers are available without evaluates supragingival dental biofilm.
an HMO. 50 (B)  Russell’s periodontal index (PI) assesses pro-
45 (D)  A PPO is a contract between several practitioners gressive stages of periodontal disease and amount of
and an insurer to provide healthcare services for lower- attachment loss; used MAINLY for major population
than-average fees. Participants in HMOs pay a fixed groups.
CHAPTER  18

Ethics and Jurisprudence

ETHICS IN DENTISTRY AND RELATIVISM  B. Nonmaleficence: action is wrong if harm is inflicted


The study of ethics in dental hygiene involves the under- on others (e.g., staying current with advances in den-
standing of concepts of ethics, morals, mores. Relativism tal hygiene and dentistry and taking CE courses).
in ethics involves issues of behavior and belief. What is C. Beneficence: action is moral if it is good and helps a
judged right varies among individuals, situations, cultures. person or enhances the welfare of person.
Ethical theories form the basis of principles and rules. 1. Rules:
• See CD-ROM for Chapter Terms and WebLinks. a. Avoid inflicting harm (principle of nonmalefi-
A. Relativism: cence).
1. Cultural relativism: defines moral rightness and b. Prevent or remove harm.
wrongness based on cultural beliefs (common or c. Promote good and do well.
social morality). 2. Problems in dentistry related to beneficence:
2. Descriptive relativism: people from different cul- a. Balancing good against harm for the patient.
tures have different views regarding morals. b. Conflicts between patients and healthcare provid-
3. Normative relativism: ultimately, ALL ethical ers as to what is a good treatment record result.
judgments are arbitrary and not justifiable. c. Influence of a third party (e.g., caretaker, insur-
4. Personal relativism: individual determines what ance carrier) on what treatment can be done.
is right or wrong according to personal standards d. Conflicts that occur when patients refuse or dis-
of goodness and NOT according to cultural influ- agree with treatment.
ences. 3. Paternalism: acting as a parent would on behalf of
B. Ethical theories: a child, including overriding an autonomous deci-
1. Utilitarian theory: encompasses “the greatest good sion for the good of a patient.
for the greatest number” or universality; addresses D. Justice: treating individuals fairly; involves giving
consequences: patients their due or what is owed to them (“golden
  a. Also known as Mills’ “greatest happiness” theory. rule”). Example: production goals should not affect
       (1) Act: doing the right act will result in the the treatment we offer a patient.
best consequences. 1. Distributive justice applies to the proper and fair
      (2) Rule: actions are right or wrong based on allocation of many aspects of society (e.g., po-
their consequences. litical rights such as voting privileges, rights of
  b. Value: depends on whether something is deter- women and minorities).
mined to be good or harmful. 2. Formal principle of justice or equality means that
2. Deontological theory: focuses on the action, rule, or “equals must be treated equally, unequals treated
practice of an act rather than on the consequences; unequally.”
involves performing the right action regardless of 3. Theories of distribution:
the consequences (Kantian theory). a. Utilitarian theory: combines the principles of
3. Virtue theory: focuses on judging traits of charac- beneficence and nonmaleficence; aims to maxi-
ter as good or bad; promotes good choices. mize good and minimize bad.
b. Libertarian theory: offers healthcare based on
Ethical Principles respecting autonomy; need is NOT a factor;
Ethical principles are the laws or doctrines of ethics (see giving healthcare charitably based on patient’s
CD-ROM for ADHA Code of Ethics). wishes.
A. Autonomy: everyone has the right to hold and act on c. Egalitarian theory: emphasizes equality; based
personal values and beliefs such as the right to pri- on the idea that everything is for everyone.
vacy, freedom of choice, and accepting ­responsibility 4. Example of violation of justice: dentist uses a spe-
for one’s actions as long as harm is NOT inflicted on cific lab for HMO patients and another for private
others. pay patients to save overall costs.

575
576   Saunders Review of Dental Hygiene

E. Veracity: obligation to speak the truth and disclose DENTAL LIABILITY, JURISPRUDENCE,
information that is necessary for the patient to make AND TYPES OF LAW 
sound treatment decisions.
F. Fidelity: Understanding types of law and methods of avoiding law-
1. Moral obligation to keep promises and other suits or dental liability is vitally important in the practice
­commitments. of dental hygiene. Jurisprudence is the philosophy of
2. Duty on confidentiality: MUST keep confidential law. Civil law, which includes contract law and tort law,
the information provided by the patient. and criminal law are the two major types of law.
• See CD-ROM for Chapter Terms and WebLinks.
Ethical Dilemma and Decision Making A. Civil law:
An ethical dilemma occurs when two or more ethical 1. Involves crime against a person; concerned with
principles are morally justifiable but only one is acted actions that cause harm to an individual.
on. The outcome of such a situation varies according to 2. Individual files suit with a private attorney; re-
the principle that is chosen. Ethical decision making in- sponse to damages is typically measured in terms
volves several steps of analysis. Person responsible for of money.
the decision may need to make a moral evaluation of B. Contract law (form of civil law):
those involved, based on altruism. Patient is the autono- 1. Involves a breach of contract; involves the break-
mous person, and practitioner’s value system should NOT ing of a contract by either party (healthcare
come into play; patient’s values are what is important. ­provider or patient) or failing to keep one’s part of
With moral evaluation of publicity, the person pub- the ­contract.
licly states his or her evaluation and on what it is based, 2. Examples: provider does not provide care that was
while ultimacy is a judgment based on a moral evalua- discussed and agreed to, patient does not pay, or
tion that has no higher standard, based on societal trust. services take too long to complete.
Example of societal trust is maintaining patient confi- C. Tort law (form of civil law): involves a civil wrong or
dentiality and being concerned for the patient instead of injury to another person.
one’s self-interest. 1. Technical assault or battery is a wrongful act that
A. Identify the problem. is NOT consented to; includes performing a pro-
B. Relevant facts must be gathered; decision making be- cedure that patient has NOT been agreed to have
fore receiving information should be avoided. done. Examples: placing sealants or performing
C. Values must be identified, and for the dental professional fluoride treatment on a minor without consent of
these involve core values; we must put the patient first parent or guardian.
without discrimination, which will help to ensure opti- 2. Maligning a patient involves saying or writing
mal care for the patient and to maintain competency: something that may damage the patient’s reputa-
1. Acknowledging the rights of the patient. tion; includes slander (verbal) and libel (written).
2. Granting patient autonomy; allowing patient to 3. Negligence (often considered synonymous with
participate in decision making regarding ­treatment; malpractice) is carelessness, without the intent to
allowing patient right to consent to or refuse harm a patient; it occurs when the appropriate stan-
­treatment. dard of care is NOT met and some damage results.
3. Avoiding harm to others according to principle of Example: dentist knowingly permits a hazard to ex-
nonmaleficence. ist in the office, and consequently patient is injured.
4. Having respect for the dental profession. a. Liability for negligence requires the existence
5. Being true to oneself; following one’s conscience. of:
6. Following the law. (1) Healthcare provider undertaking care of
7. Generating options and reviewing available the patient; a duty to the patient results.
­options. (2) Healthcare provider breaching a duty owed
8. Choosing an option and justifying it. to the patient.
D. Things that SHOULD be taken into consideration: (3) Evidence of damage or harm to the patient.
1. Rights of the individuals involved. (4) Harm or damage to the patient being related
2. Duties of the professional involved. to the breach of duty.
3. Core values that apply. 4. Grounds for malpractice: MOST lawsuits involve
4. Benefits of the care provided or offered. failure to diagnose and treat periodontal disease.
5. Realistic alternatives. a. Failure to sterilize: sterilization practices should be
6. Patient knowledge. in accordance with recommendations of the Cen-
7. Financial or legal factors. ters for Disease Control and Prevention (CDC)
8. Need for outside consultation. and the American Dental Association (ADA).
Ethics and Jurisprudence   577

b. Failure to obtain or produce adequate radio- (admission against interest), a case could be made for
graphs; this may involve: negligence, which is grounds for malpractice.
(1) Failure to obtain radiograph when radio- 2. No one wishes to injure a patient; however, accidents
graph is ­indicated. do occur. The hygienist showed concern for her patient
(2) Unskillful use or improper processing of by taking immediate care of the wound. She handled
­radiograph, which jeopardizes diagnosis. the situation well. There is evidence that concern and
(3) Refusal by the patient; if a patient refuses care on part of clinician can greatly reduce incidences
to have a radiograph, a written release form of litigation by patients.
should be signed by the patient; a refusal 3. Patient’s record should include date and time of inci-
MUST be noted in the patient’s chart; radio- dent, detailed description of what occurred, hygienist’s
graphs are used for diagnosis and therefore actions, patient’s actions and reactions. The clinician
must remain a part of a patient’s record. should obtain the patient’s signature that patient was
c. Failure to refer: patient should be referred to informed and note on the record that patient agreed
a specialist when “average” dentist knows that that everything was all right.
procedure is complex.
d. Causing trauma: slipping of an instrument Prudent Dental Care
alone does NOT constitute negligence; how- Measures that are used to help avoid legal action include
ever, making a statement that may indicate that reasonably prudent or sensible practitioner, admissions
an error was made by the healthcare provider against interest, res ipsa loquitur, proper documentation.
(e.g., “I should have had a fulcrum”) is known Informed consent is discussed next.
as admission against interest: a statement, spo- A. Be aware that the reasonably prudent practitioner
ken or written, that may prove the opposite of measure is used by law to determine whether provider
what the person contends in court. has exercised reasonable care.
e. Failure to inform: patient must be informed B. Avoid admissions against interest, or making a state-
­before he or she can give consent. ment that serves to defeat your own interest; such ad-
D. Criminal law: pertains to actions that constitute a missions are legally admissible in court.
wrong against society. 1. Admission of negligence by employee occurs when
E. State officials respond to such actions, which on con- the healthcare provider admits to improper conduct.
viction are punishable by death, imprisonment, fine, 2. Res gestae means “part of the action” and refers to a
or removal from office. statement made during the time of the negligent act.
F. Examples: practicing profession of dental hygiene or C. Avoid res ipsa loquitur or “a matter that speaks for
dentistry without a license. itself,” referring to an injury that is directly related
to an instance of negligence (e.g., healthcare provider
PROFESSIONAL PRACTICE STUDY  fails to take a medical history or renders services
Scenario: The dental hygienist is completing an oral pro- while intoxicated).
phylaxis on a new patient. While she is retrieving a posterior D. Prove contributory negligence: that patient has con-
scaler from her tray, the instrument slips from her hand and tributed to harmful result.
lands on her patient’s head. The instrument tip embeds in E. Identify another proximate cause; reveal that harm to
the scalp tissues. The hygienist carefully removes the instru- patient may have resulted from another incident.
ment, cleanses the wound with antibacterial soap, applies F. Keep accurate and complete patient records, single
pressure to stop the bleeding, and apologizes for the injury. MOST important factor in the defense of MOST mal-
The patient remains calm and says, “Don’t worry about practice cases; record information in indelible ink; if an
it.” The hygienist documents the incident in the patient’s error is made in a patient’s chart, mark one line through
record. the statement and place your initials next to the area
(do NOT erase, black-out, or use correction fluid).
1. Is this an incidence of negligence? Why or why not? 1. Medical history should be current, include patient’s
2. Did the dental hygienist handle the situation appropri- signature, and be reviewed at each visit.
ately? 2. Diagnosis should be recorded (including dental
3. What types of information regarding the incident should hygiene diagnosis).
the dental hygienist include in the patient’s ­record? 3. Treatment plan(s) should include ALL options
given to patient. Note that all benefits, risks, and
1. Not incidence of negligence because there is no indica- alternatives (RB&A) were explained to patient
tion that accident was caused by particular action, such verbally and in writing, and have the ­patient sign
as not using a fulcrum. If the hygienist had ­indicated the chart stating that he or she understands the
to patient that there was a particular cause for incident treatment and risks.
578   Saunders Review of Dental Hygiene

4. Record of treatment should include date(s) of treat- from the last visit and finds no indication on the record
ment, procedure(s) performed, drugs used and/or of any medical problems, including IE or conditions that
administered; record should also indicate canceled involve its risk. The record of treatment indicates that the
and broken appointments. patient had generalized moderate chronic periodontitis,
5. Copies of all correspondence regarding patients should with moderate subgingival calculus, and that homecare
be kept, together with receipts of registered mail. instruction was given because of excess calculus, the
a. Radiographs should be: condition of the patient’s tissue, and excess bleeding. The
(1) Processed properly to avoid deterioration chart also indicates that the patient was scheduled to have
with age. tooth #31 extracted by an oral surgeon the day after the
(2) Carefully labeled with patient’s name, date oral prophylaxis appointment.
taken, and dentist’s name.
(3) Taken properly so that they are diagnosti- 1. What suit will the attorney most likely file against the
cally acceptable. dentist?
(4) Retained for minimum length of time re- 2. Is this suit processed under civil or criminal law?
quired by statute of limitations in a state. 3. Is the dentist and/or dental hygienist liable for this suit?
6. Letters to patients (correspondence) should be 4. What can the dentist do to help prove that the patient
carefully worded to ensure understanding and may not have contracted IE from the visit to the den-
minimize chance for libel. tist’s office?
G. Transfer of records:
1. For inactive patient, records are typically kept for 1. Negligence or malpractice; negligence implies care-
length of time required by the statute of limitations lessness without intent of harming a patient.
and/or state dental practice act. 2. Negligence is categorized under tort law, which is a
2. For active patients, records are kept by dentist un- branch of civil law (actions causing harm to individual).
less requested by another dentist. 3. There are four factors that must be present before den-
3. Steps for requesting transfer of records: tist and/or dental hygienist is liable for negligence:
a. Always keep on record the letter(s) requesting (1) healthcare provider must have undertaken the care
records. of patient; (2) healthcare provider must have breached
b. Send copies of records by way of registered a duty that was owed to patient; (3) harm or damage
mail; request a receipt. to patient must be proved; and (4) harm must be re-
c. Request that records be returned to the office lated to breach of duty. If any one of these four factors
after their use. is missing, ruling will be in favor of dentist or dental
H. Insurance claims include records only of performed hygienist.
procedures; falsifying information is fraudulent and 4. Two considerations may be used to spare the dentist
may result in a legal action. from judgment. First, dentist may try to prove con-
I. Statute of limitations is the legal time span in which a tributory negligence. Patient may have knowingly
lawsuit (civil) for a wrong must be filed. failed to inform dentist or note on the medical history
1. Malpractice suit, in most states, must be initiated that the patient had a heart condition (such as an ar-
within 2 years of time that the wrongful act was tificial valve replacement) or history of IE that poses
committed. risk for IE with dental treatment. The patient also may
2. Limitation for filing a breach of contract suit typi- have contributed to the negligence by not taking the
cally is 6 years; recommended that records be re- antibiotic premedication that the physician prescribed.
tained for minimum of 10 years. Second, the dentist may try to prove proximate cause.
J. Liability insurance should be carried by each licensed Cause of patient’s IE may have been the visit to oral
professional. surgeon because the tooth extraction may have neces-
1. Professional reliability: every professional has a sitated antibiotic premedication. For these reasons
responsibility to provide high-quality dental care. the dental team must make sure that patient’s medical
2. Respondeat superior: employer is often named in a ­history is updated regularly and any medical consulta-
suit in which an employee caused harm to a patient. tions followed through. Moreover, all records must be
complete.
PROFESSIONAL PRACTICE STUDY 
Scenario: A dentist after 33 years of the practice of den- Informed Consent
tistry receives a letter from an attorney stating that one Informed consent (in a contractual relationship) means
of his patients has contracted infectious endocarditis (IE) that a patient has given a healthcare provider permission
from a routine oral prophylaxis appointment 6 months to perform certain procedure(s) and that the patient un-
ago. The dentist reviews the patient’s medical history derstands any risks involved in the treatment.
Ethics and Jurisprudence   579

A. To give consent and avoid breach of contract, patient t­ransactions and national identifiers for providers,
must understand the following: health ­insurance plans, employers.
1. Nature of his or her condition. 1. The AS provisions also address security and pri-
2. Treatment proposed. vacy of health data.
3. Any risks involved or chances of failure. 2. Standards are meant to improve efficiency and ef-
4. Possible results of NOT treating the condition. fectiveness of U.S. healthcare system by encourag-
5. Any alternative procedures that may be necessary. ing widespread use of electronic data interchange
B. To validate consent, the following requirements must within U.S. healthcare system.
be met: B. Privacy Rule was added to establish regulations for
1. Consent must be informed. use and disclosure of Protected Health Information
2. Consent must be given for specific treatment(s). (PHI).
3. Individual giving consent must be legally competent. 1. General information:
4. Treatments consented to must be legal. a. PHI is any information about health status, provi-
5. Consent must NOT have been obtained through sion of healthcare, or payment for healthcare that
deceit or fraud. can be linked to an individual. Includes any part
C. Types of consent and refusal: of patient’s medical record or payment history.
1. Implied consent: implied by the actions of the b. Covered entities must disclose PHI to individual
patient (e.g., a patient comes to the office for an within 30 days upon request. They must also dis-
examination and consultation; therefore consent is close PHI when required to do so by law, such
established by the actions of provider). as reporting suspected abuse to state agencies.
a. Quasi-consent is a type of implied consent. c. Covered entity may disclose PHI to facilitate
b. Patient is in danger of injury or death and is unable treatment, payment, or healthcare operations
to give consent; emergency treatment is given. or if the covered entity has obtained authori-
2. Expressed consent: informed consent that is zation from the individual. However, when
given verbally or in writing (provides the MOST covered entity discloses any PHI, must make
­protection). a reasonable effort to disclose only minimum
3. Parental or guardian consent: required when necessary information required to achieve its
a minor or an individual who is NOT mentally purpose.
competent to give consent needs treatment; usu- 2. Dental setting implementation:
ally provided by a parent or other legal caretaker, a. Thus HIPAA requires the covered entity to des-
preferably in writing. ignate a privacy officer who understands how
4. Written and verbal mandatory consent MUST be the privacy rules affect the dental office and pa-
given for the following situations: tient care.
a. Administration of new drugs. (1) This individual is familiar with the require-
b. Use of a patient’s photograph (must be recog- ments and makes certain that policies and
nizable). procedures have been implemented and
c. Administration of general anesthesia. training has been conducted.
d. Treatment that will take longer than 1 year to (2) In many dental offices, office manager or front
complete. desk coordinator serves as privacy officer.
e. Treatment of children in a public program. b. PHI includes any information that is created or
o1440 5. Informed refusal against dental advice occurs received by dental office as it relates to patient’s
when patient declines treatment after treatment plan care.
is presented and has been advised of the risks asso- (1) Includes radiographs, images (both intra-
ciated with refusing a recommended procedure. oral and photographic), and traditional den-
tal record.
s0130 Federal Government Provisions (2) Also includes oral discussion of patient
Many federal government provisions involve the den- care that may be overhead by other parties.
tal practice. The Health Insurance Portability and Ac- c. Dental office should create a notice of privacy
countability Act (HIPAA) was enacted by U.S. Congress; policy that informs patients how their informa-
Title II of HIPAA, Administrative Simplification (AS) tion will be used and disclosed.
provisions. Note that use of Medical Safety Data Sheets (1) The privacy rules require covered entities
(MSDS) is a concern of the Occupational and Safety to have written, site-specific privacy pro-
Health Administration (OSHA), NOT HIPAA. cedures detailing how staff members may
A. HIPAA with AS provisions requires establish- have access to PHI and how PHI is used
ment of national standards for electronic healthcare and ­disclosed.
580   Saunders Review of Dental Hygiene

(2) In addition to the practice’s team members, b. Required specifications must be adopted and
business associates have access to PHI; administered as dictated by the rule.
business associates are defined as third c. Addressable specifications are more flexible.
parties, including consultants, billing and Individual covered entities can evaluate their
collection agencies, attorneys, software own situation and determine the best way to
trainers, and hardware technicians; privacy implement addressable specifications.
rules address how to deal with business as- D. HIPAA has also standardized the use of Health Care
sociates to ensure the privacy of PHI. Financing Administration’s Health Care Procedural
(3) Dental office needs to make a good faith ef- Coding System, which contains codes for dental ser-
fort to obtain each patient’s acknowledgment vices that were created by American Dental Associa-
of receipt of notice of privacy practices. tion and published as current dental terminology; first
(a) May be a separate form or incorporated digit, “0,” was replaced by a “D”; refer to “Current
within the office’s consent forms. NOT Dental Terminology, CDT 2007-2008” for the current
to be confused with the patient’s con- version of the Code on Dental Procedures and No-
sent to use or disclose PHI, which may menclature (“the Code”).
be required in some states. E. National Health Information Infrastructure Act, es- o9000
(b) It is highly recommended, when dealing tablished through the Department of Health and Hu-
with any media materials that contain man Services, mandates electronic medical records
PHI, to obtain a special authorization (EMRs); ensures that patient health records are ca-
that details how the PHI will be used pable of being sent electronically anywhere in the
and for how long. country with the patient’s agreement, as well as being
(4) According to federal law and most state safely stored electronically.
laws, patient has a legal right to the in-
formation contained in the dental record; PROFESSIONAL PRACTICE STUDY 
HIPAA gives patients the right to examine Scenario: A 47-year-old patient in a general dental prac-
and obtain a copy of their records, access tice has a thorough periodontal evaluation that reveals
needs to be made within 30 days of the generalized severe chronic periodontitis. The dental hy-
request, and offices may charge patients gienist explains treatment options to the patient and asks
for the cost of duplicating and sending the whether there are further questions. A series of four ap-
records. pointments is scheduled. The patient completes the se-
(5) Patients have the right to file a complaint ries of appointments and agrees to return in 6 weeks for a
if they feel their privacy rights have been reevaluation of treatment. The supervising dentist wants
violated; this complaint is filed directly to place images of the patient, before and after the treat-
with the provider or the U.S. Department ment, in his reception room for other patients to see.
of Health and Human Services Office for
Civil Rights; therefore information on how 1. Did the patient give consent for her treatment? If so,
to file a complaint should be included with what type of consent did she give?
the notice of privacy policy. 2. The dental hygienist asks the patient to allow the case
(6) Dental offices must provide training for to be used as a teaching tool in the practice, which
their team and maintain documentation of would involve both extraoral and intraoral photo-
the training; employees should be familiar graphs of both the face and mouth and full-mouth se-
with practice’s privacy policies and proce- ries of radiographs. What type of consent is required
dures and be aware of the disciplinary ac- before photographs are taken?
tions that will be taken if employee fails to 3. If, after thorough treatment of the periodontal condi-
follow these procedures. tion, further loss of alveolar bone is detected, what
C. Security Rule complements Privacy Rule, deals spe- should be done?
cifically with Electronic Protected Health Information 4. What does the supervising dentist need to consider
(EPHI). when using the patient’s images in his reception room?
1. Lays out three types of security safeguards re-
quired for compliance: administrative, physical, 1. The patient gave consent for treatment when it was
technical. agreed to schedule appointments and then agreed to
a. For each of these types, the Security Rule iden- return for reevaluation. This type of consent is called
tifies various security standards, and for each implied consent.
standard it names both required and addressable 2. Written, mandatory consent is required in situations in
implementation specifications. which new drugs are used, recognizable photographs of
Ethics and Jurisprudence   581

a patient are used, general anesthetics are administered, b. Withdrawal from a contract:
or treatment takes longer than 1 year to complete. (1) Written letter of intent to withdraw is sent
3. If the treatment performed is unsuccessful (as evi- to patient by certified or registered mail
denced by further progression of the disease), the (with return receipt).
general dentist should refer patient to an appropri- (2) Contains a statement of care that must be
ate specialist, which in this case would be a perio- completed.
dontist. (3) Suggests to patient how care can be obtained.
4. The images of the patient would be considered pro- (4) States that emergency care will be provided
tected health information (PHI) under the Health for 30 days and all treatment in progress
­Insurance Portability and Accountability Act (HIPAA). will be completed.
It is recommended that the dentist obtain special au- 6. Use standard materials, techniques, and drugs; any
thorization for use of these images from the patient. experimental procedures, techniques, or drugs
used for treatment require the patient’s informed
Legal Responsibilities and Duties ­consent.
Legal responsibilities in the dental practice setting in- 7. Charge reasonable fees; charge is reasonable when
clude the duties of the healthcare provider and the patient. customary and ordinary for service in the same or
Duties are the obligations that one person owes another. a similar community.
A. Duties of the healthcare provider: 8. Keep the patient informed of the progress of
1. Hold licensure; the healthcare provider must have ­treatment; give adequate and understandable
a valid professional license to practice. ­instructions to patient so they can be followed.
2. Protect and respect the personal and property rights 9. Achieve reasonable results; healthcare provider
of the patient; includes respecting patient’s right to has a legal duty to provide standard care but does
the ownership of dentures, even if patient has NOT NOT have to obtain perfect results.
paid for them. 10. Arrange for care during any temporary absence; if
a. Healthcare provider should AVOID asking pa- a healthcare provider is unavailable to treat patient,
tients personal questions. competent care must be available (e.g., a dentist
b. Patient has right to confidentiality; photo- who goes on vacation arranges to have someone
graphs of patients cannot be displayed and/ available for emergency care); failure to do so may
or published without written permission from result in abandonment.
patient. 11. Refer the patient to a specialist when patient’s
c. Any discussion in the office between the health- needs CANNOT be met; referral should be made
care provider and patient is considered privi- when a case is particularly difficult or beyond the
leged communication. dentist’s capabilities.
d. There are two exceptions to privileged commu- 12. Complete care within a reasonable amount of time;
nication: if the completion of treatment exceeds 1 year, pa-
(1) Abuse. tient must give written consent.
(2) Communicable diseases. B. Duties of the patient:
3. Provide only care that is necessary and to which 1. Pay reasonable fees; fees should be paid within a
the patient has agreed. reasonable period of time.
4. Exercise reasonable skill, care, and judgment. 2. Cooperate during treatment; involves ­ following
a. Reasonable skill involves not ONLY having ­instructions given by healthcare provider and
knowledge but the ability to use such knowledge. keeping appointments.
b. Reasonable care relates to how skill is used;
the healthcare provider should use the degree Government Regulations and Dental Practice Act
of care that a reasonably prudent practitioner Dental profession is regulated by government regulations
would use under similar circumstances. of state dental practice acts. Dental practice act defines
c. Reasonable judgment involves making the best the practice of the dental profession, including dental hy-
decision(s) possible regarding the procedures giene, and indicates how an individual may be granted a
performed. license to practice within a given state. Practice acts vary
5. Do NOT abandon the patient after individual be- from state to state. NO one should provide treatment that
comes a patient of record. their Dental Statute Act does not allow.
a. Abandonment is desertion; involves failure to A. Purpose of dental practice act:
provide services and needs to be agreed on by 1. Protects the public from incompetent practitioners
healthcare provider and patient; applies to prac- and ensures they will receive a minimum standard
titioner who is NOT available. of care.
582   Saunders Review of Dental Hygiene

2. Prohibits individuals from practicing in the profes- 2. State board of dentistry is a group of appointed or
sion before meeting specific qualifications. elected members (dentists, dental hygienists, and/or
3. Gives authority to state boards of dentistry to issue consumers) who administer the practice act, make rules
or deny licensure. on the administrative act, issue and revoke ­licenses, in-
4. Empowers state boards of dentistry to suspend or vestigate complaints, take disciplinary action, and mea-
revoke licenses. sure competence of graduates through state boards.
B. Violations of the dental practice act: 3. The hygiene graduate would violate the dental practice
1. Are crimes against society; involve any acts in vio- act, a crime against society, if he began practicing his pro-
lation of dental practice act of a particular state. fession without a license. If he were convicted of such a
2. Subject individual to professional discipline if violation, he would be subject to professional discipline.
convicted.
3. Include illegal acts that do NOT necessarily result
in physical harm (e.g., dental hygienist performs Review Questions
an oral prophylaxis before receiving licensure).
4. Examples of infractions:
a. Sex offenses against patients or employees.   1 What is the study of conduct, which is based on right and
b. Illegal prescription of narcotics. wrong issues, known as?
c. Delegation of tasks to one who is NOT qualified. A. Morals
d. Poor documentation or improper documentation. B. Mores
e. Insurance fraud. C. Ethics
D. Values
f. Tax evasion.
  2 The moral evaluation that because it is acceptable for one
C. State board of dentistry: individual to act a particular way, it is therefore acceptable
1. Members are appointed or elected and may include for others to act in the same way is referred to as
dentists, dental hygienists (in most states), and A. publicity.
consumers (in many states). B. ultimacy.
2. Duties: C. neutrality.
a. Administering the written word of state’s prac- D. universality.
tice act.   3 An individual who focuses on which actions are right is
b. Issuing and revoking licenses. known as a(n)
c. Investigating complaints and taking disciplin- A. action utilitarian.
ary action when necessary. B. value utilitarian.
C. consequential utilitarian.
d. Measuring the competence of graduates by way
D. deontologist.
of state board examinations. E. “golden rule” keeper.
  4 What is treating others fairly and living by the “golden rule”
PROFESSIONAL PRACTICE STUDY 
known as?
Scenario: A recent graduate from a dental hygiene pro- A. Justice
gram has completed all requirements for licensure. The B. Nonmaleficence
state board of dentistry will meet in 6 weeks to issue new C. Beneficence
licenses. A local dentist has just hired the new graduate D. Paternalism
and would like him to begin practicing immediately as   5 A healthcare provider is responsible for speaking the truth
a dental hygienist. and giving adequate information to allow sound decisions
regarding treatment. This is referred to as
A. fidelity.
1. What government regulation defines the requirements
B. reparation.
that an individual must meet to practice dental hygiene C. gratitude.
in a given state? D. veracity.
2. What is the state board of dentistry, and what are their   6 When an ethical decision must be made, all of the following
duties? steps are recommended, EXCEPT one. Which one is the
3. Can the dental hygiene graduate begin practicing be- EXCEPTION?
fore obtaining his professional license if he has met A. Granting patient autonomy
the requirements for licensure? B. Following an employer’s directive
C. Avoiding harm
1. Dental practice act is the government regulation that D. Respecting the profession
includes a definition for the practice of dental hygiene
and how an individual can be granted a license to
practice in the state.
Ethics and Jurisprudence   583

  7 When a dental hygienist does NOT perform a procedure in- 16 When a dentist is NOT available to a patient at any time
cluded in the contract with a patient, that is referred to as during treatment, it may be considered an act of
A. technical battery. A. withdrawal.
B. breach of contract. B. abandonment.
C. malpractice. C. neglect.
D. negligence. D. technical assault.
  8 Saying or writing something that may damage a patient’s 17 A patient should give written consent to continue dental
reputation is treatment if the treatment extends beyond
A. breach of contract. A. 3 months.
B. technical assault. B. 6 months.
C. maligning. C. 1 year.
D. negligence. D. 2 years.
  9 Malpractice is one type of 18 Who handles the violations of legal principles?
A. tort law. A. State board of dentistry
B. breach of contract. B. American Dental Association
C. criminal law. C. State practice act
D. technical battery. D. American Dental Hygienists’ Association
10 When an individual commits a wrongful act against society, 19 Treatment plan(s) should include all options given to the
it is considered an offense of patient. All risks are explained to the patient orally and in
A. tort law. writing, and the patient signs the chart stating that he or she
B. civil law. understands the treatment and risks.
C. contract law. A. Both statements are true.
D. criminal law. B. Both statements are false.
11 Failure on the part of the patient to follow instructions given C. The first statement is true, the second is false.
by the dentist during and after treatment is an example of D. The first statement is false, the second is true.
A. contributory negligence. 20 According to HIPAA, patients have the right to file a com-
B. respondeat superior. plaint if they feel their privacy rights have been violated;
C. res gestae. this complaint is filed directly with the provider or with the
D. admission against interest. A. state board of dentistry.
12 All of the following are CORRECT regarding patient re- B. American Dental Association and American Dental
cords, EXCEPT one. Which one is the EXCEPTION? ­Hygienists’ Association.
A. Are kept for 1 year C. U.S. Department of Health and Human Services Office
B. Include diagnoses for Civil Rights.
C. Are recorded in ink D. attorney general of the state.
D. Include treatment plans
13 What is the principle that makes the dentist responsible for
injuries caused by employees? Answer Key and Rationales
A. Admission against interest
B. Respondeat superior
C. Res gestae 1 (C)  Ethics deals with issues of right and wrong re-
D. Professional reliability garding conduct and character. Mores are customs of
14 A contract initiated by the actions of the parties concerned,
a group. Morals are standards of thought or specific
in absence of a legally binding contract, is a(n)
judgments that are typically, but not always, based
A. parental contract.
B. expressed contract. on strong religious beliefs. Values are beliefs and at-
C. written contract. titudes that are often established by individual’s up-
D. implied contract. bringing or religious affiliation.
15 Information acquired about a patient during the course of 2 (D)  Universality stresses that if it is good for indi-
treatment is considered privileged communication, even in vidual to act a certain way, it must be acceptable for
a court of law. EXCEPTIONS to this rule are made when pa- others to act in same way. With moral evaluation of
tients exhibit signs or symptoms of (1) alcoholism; (2) child publicity, one publicly states his or her evaluation and
abuse; (3) illegal drug use; (4) communicable ­disease. on what it is based. Neutralism (altruism) is based on
A. 1 and 2 how an evaluation is best for someone else, and ulti-
B. 2 and 3
macy is a judgment based on a moral evaluation that
C. 1 and 3
D. 2 and 4
has no higher standard.
E. 3 and 4 3 (A)  An action utilitarian deals with the actions of
what is right and is responsible for the consequences
of those actions. A value utilitarian focuses on what
counts as a good or harm. A consequential utilitarian
584   Saunders Review of Dental Hygiene

decides that an action is either right or wrong based 11 (A)  Contributory negligence describes a patient’s con-
on the consequences of the action. A deontologist fo- tribution to the harm. Example: a patient has periodontal
cuses on the action of an act without worrying about problems and is advised to follow a 3-month periodon-
its consequences. The “golden rule” refers to treating tal maintenance program. However, patient has failed
individuals fairly; involves giving patients their due to keep his appointments for the past 2 years. When he
or what is owed to them, that is, justice. finally does keep the appointment, his condition has
4 (A)  Treating others fairly and giving them their just become worse. Because the broken appointments were
due is known as justice. Nonmaleficence is identi- recorded in the patient’s chart, the dentist can claim
fying an action as wrong if harm is inflicted on an that the patient contributed to the ­negligence. Respon-
­individual. Beneficence defines an action as moral if deat superior ­indicates that the dentist is responsible
it is good and helps a person. Paternalism involves for the wrongful actions of his or her employees. Res
acting as a parent would, including overriding a deci- gestae is a statement that an ­individual makes during
sion for the good of an individual. a negligent act and that can be used against him or her
5 (D)  Veracity claims that a healthcare provider has in a lawsuit. Admission against interest is a statement
an obligation to speak the truth and keep the patient made at any time that goes against the legal interest of
abreast of all information that is needed to make the person making the statement.
sound decisions during treatment. Fidelity stresses 12 (A)  Patient records should be kept on file for the
the duty to maintain confidentiality. Reparation is length of the statute of limitations within a state. They
the act of making amends to an individual who was should always be neatly recorded in ink and SHOULD
harmed, and gratitude is the act of showing respect to include a complete record of the diagnosis, treatment
someone who has helped you. plans, updated medical history, correspondence,
6 (B)  When making an ethical decision, it is important radiographs, record of treatment.
to evaluate own values. Granting a patient’s autonomy 13 (B)  Respondeat superior is the principle that makes
is a necessary part of the process. Patient can be active the dentist responsible for a wrongful action by an
in determining his or her treatment and can participate employee that caused harm to a patient. Admission
in the decision making regarding the treatment plan. against interest is a statement that a person makes,
Avoiding harm to others is vital. Healthcare provider orally or in writing, that may prove to be the opposite
should have a conscience and a respect for the profes- of what that person is contending in court; statement
sion. Employer’s direction may NOT offer the appro- can be used in a lawsuit. Professional reliability states
priate guidelines for making a decision. that every professional has the responsibility to pro-
7 (B)  Breach of contract involves the breaking of a vide good dental care to ­patients.
contract (an agreement) by either party involved. 14 (D)  An implied contract is established by certain ac-
Technical battery is an act performed that was NOT tions on the part of the healthcare provider or patient.
agreed on by the patient. Malpractice, often con- An expressed contract is informed consent given
sidered synonymous with negligence, is a form of in writing or orally. Parental or guardian consent is
carelessness that brings harm to a patient. Act that given when the patient is a minor or is mentally in-
results in harm to the patient does NOT have to be competent to give consent. Written consent provides
­intentional. the MOST protection for the patient and is a form of
8 (C)  Maligning an individual involves saying or writ- expressed consent.
ing something that can damage that individual’s rep- 15 (D)  Privileged communication grants the patient auton-
utation. Slander involves saying something that can omy. What the patient confides in the office must remain
destroy an individual’s reputation, and libel involves confidential. However, there are two exceptions. When
writing something that yields the same result. suspected abuse or communicable disease has NOT
9 (A)  Malpractice is a civil wrong that is handled un- been reported, proper authorities must be ­contacted.
der tort law. It involves an injury to a patient that oc- 16 (B)  Abandonment is the result of desertion. Health-
curs without intent to harm the patient. Such injuries care provider fails to provide services that the two
typically are a result of carelessness. Criminal law in- parties had agreed on to be done. If the dentist is on
volves wrongs against society. In such cases the state vacation, provision must be made for the period of ab-
takes action against the healthcare provider. sence. Withdrawal is the act of terminating treatment
10 (D)  A criminal law pertains to a criminal action that before it is completed. Neglect is failure to meet the
is a wrong against society (e.g., individual practicing appropriate standard of care, and technical assault is a
dental hygiene without a license). Tort and contract wrongful act that has NOT been agreed on.
laws are examples of civil law; lawsuits are filed with 17 (C)  Treatment should be completed within 1 year of
a private attorney, and damages typically are ­measured its initiation. If it extends beyond that time, the patient
monetarily. should give written consent.
Ethics and Jurisprudence   585

18 (A)  State board of dentistry members are appointed or 20 (C)  According to HIPAA, patients have the right to
elected and are responsible for administering the written file a complaint if they feel their privacy rights have
law of a state, or its practice act. They also are respon- been violated; this complaint is filed directly with the
sible for issuing and revoking licenses and investigating provider or with the U.S. Department of Health and
any complaints submitted to them. The American Dental Human Services Office for Civil Rights; therefore
Association and the American Dental Hygienists’ Asso- ­information on how to file a complaint should be
ciation do not administer any legal actions. State prac- included with the notice of privacy policy.
tice act defines the practice of the dental profession and
provides guidelines regarding licensure within the state.
19 (A)  Both statements are true. Treatment plan(s)
should include all options given to patient; all risks
are ­explained to patient orally and in writing, and pa-
tient signs the chart stating that he or she ­understands
the treatment and risks.
This page intentionally left blank
Index

A Americans with Disabilities Act, 530–531


A bands, 59 Amides, 55
Abducens (sixth cranial) Amines, 54, 60
nerve, 40, 40f Amino acids, 56. See also under Nutrition
Abfraction, of teeth, 191 Anabolism, 57
ABO blood type, 44 Anal canal, 52
Abrasion, of teeth, 191 Anastomosis, 46
Abscess, 169, 194. See also under Periodontology, 59 Anatomy
Absorption, 69 cardiovascular system. See Cardiovascular system (CVS)
Accessory (eleventh cranial) nerve, 40f, 41 defined, 33
Acid etch, and enamel rods, 21 digestive system. See Digestive system
Acne, 169 endocrine system. See Endocrine system
Actin filaments, 59 general, 33–42
Adenoid cystic carcinoma, 179 integumentary system. See Integumentary system
Adenoma, 177 lymphatic system. See Lymphatic system
Adenosine monophosphate (AMP) muscular system. See Muscular system
defined, 57 nervous system. See Nervous system
and electron transport, 57 organs of, 33
energy stored in, 57 reproductive system. See Reproductive system
and glycolysis 57 respiratory system. See Respiratory system
goal of, 57 skeletal system. See Skeletal system
as nucleic acid, 56 urinary system. See Urinary system
and pyruvate dehydrogenase, 57 Anemia. See under Pathology, general and oral
and tricarboxylic acid cycle, 57 Anesthesia, local. See also under Pharmacology
Adrenal cortex gland, 44 action of, 474
Adrenal gland disorders, 171 aspiration while administering, 477
Adrenocorticotropic hormone (ACTH), 60 cartridges for, 480, 480f
Afferent (sensory) nerves, 37 chemical formula for, 473, 473f
Agranulocytosis, 175 clinical studies for, 471–472, 481–482, 482t, 487t, 489t, 494t­
ALARA concept, 140 CNS affected by, 474
Alcohols, 54 computer-controlled delivery of (CLAD), 480
Aldehydes, 54 correct administration of, 473
Alginate. See under Biomaterials, irreversible CVS affected by, 475
hydrocolloid (alginate) delivery of, 481–482
Allergy, 188 dissociation of, 473
Alveolar bone dosage calculations for, 476, 476b
illustrated, 20f, 26f hygienists licensed for, 473
makeup of, 23 injection procedures for
as part of periodontium, 23 infiltration technique, 483t
in periodontium, 24 mandibular AK blocks, 487
ramifications of, 24 mandibular buccal block, 487
Alveolar crest, 24, 24f, 26f mandibular G-G blocks, 487
Alveolar cystitis (dry socket), 194 mandibular guide, 486t
Alveolar mucosa, 81 mandibular IA block, 483
Alveolodental fiber groups, 25, 25t, 26f mandibular mental/incisive blocks, 487
Amalgam tattoo, 194 maxillary GP, NP, and AMSA blocks, 483
Amalgams. See under Biomaterials maxillary guide, 484–485t
Ameloblastoma, 178 maxillary, mandibular differences, 483
Ameloblasts, 13f, 14 maxillary MSA, ASA, and IO blocks, 483
Amelogenesis imperfecta, 199 maxillary PSA block, 483

Note: Page numbers followed by f indicate illustrations; t, tables; and b, boxed meterial.

587
588    Index

Anesthesia, local (Continued) Appendicular muscles (Continued)


local complications with leg, 36
hematoma, 490 shoulder, 35
needle breakage, 490 thigh, 36
pain, burning on administration, 490 Apposition stage, of tooth formation, 13f
paresthesia, 490 Arterial blood supply
transient facial nerve paralysis, 490 about, 93
trauma to tongue, lip, cheek, 490 common carotid arteries of, 93
trismus, 490 external carotid arteries of, 94
long-acting, 479t anterior branches of, 94
maximum recommended dose (MRD) of, 476 medial branches of, 94
medium-acting, 478t posterior branches of, 94
mode of action for, 473 terminal branches of, 94
needles for, 478 internal carotid arteries of, 93
and pH, 473 maxillary artery of, 94f
pharmacology of subclavian arteries of, 93
amide agents, 474 Arteries, of heart, 45–46, 48f
distribution, elimination of agents, 474 Arterioles, 46, 48f
ester agents, 474 ASA physical classifications, 322
understanding, 474 Asthma, 185
preparation for Atherosclerosis, 184, 215
preinjection procedure, 481 Atria, of heart, 44, 47f
setup, 481 Atrioventricular valve, 45
site preparation, 481 Attrition of teeth, 191
with standard precautions, 481 Authors, qualifications of, 1
respiratory system affected by, 475 Autism. See under Special needs patient care
short-acting, 478t Autonomic nervous system, 37, 39
syringes for, 477, 479f Axial muscles, 35
systemic complications with Axial skeleton, 34, 36f
about, 487
adverse drug reactions, 488 B
allergic reactions, 488 B cells, 44
causes of overdose, 488 Bacterial infections, 169
epinephrine overdose, 488 Basal cell carcinoma (BCC), 179
factors in overdose, 488 Basal metabolic rate (BMR), 222
mild to moderate overdose, 488 Basophil, 45t
moderate to high overdose, 488 Bell’s palsy, 40, 187
topical anesthetics Bell stage, of tooth formation, 13f
actions of, 477 Benign mucous membrane pemphigoid (BMMP), 190
used for, 477 Bicuspids. See under Permanent teeth; Primary teeth
used with local, 473 Bicuspid valve, 45
use of, in dental office, 473 Bifid (cleft) uvula, 11
vasoconstrictor action in Biochemistry. See also Biomolecules
adrenergic receptors of, 475 defined, 53–54
chemical structure of, 475, 475f hydrocarbons, 53–54
considerations for, 476 alcohols, 54
inclusion of, 475 aldehydes, 54
types of, 475 amides, 55
vasoconstrictor concentration in, 477b amines, 54
Aneurysmal bone cyst, 196 carboxyl groups, 54
Angioma (birthmark), 177 esters, 55
Angiotensin I and II, 62 ethers, 54
Angle’s classification, 122, 123t ketones, 54
Anodontia, 196 organohalogens, 54
Anorexia nervosa, 223 parent hydrocarbons, 54
Antibiotics. See under Pharmacology saturated and unsaturated, 55
Antibodies, 44, 46t metabolism, 57
Antidiuretic hormone (ADH), 60, 62 Biofilm. See under Nutrition; under Periodontology, dental biofilm
Antihypertensives. See under Pharmacology Biomaterials
Antimicrobials. See under Clinical treatment amalgam
Anxiety, controlling, 5 about, 513
Aphthous ulcers (canker sores), 193 bonding to tooth, 514
Apicoectomy, 522 burnishing of, 514
Appendicular muscles carving of, 514
ankle, foot, 36 components of, 513
forearm, 36 condensation of, 514
Index    589

Biomaterials (Continued) Biomaterials (Continued)


creep in, 514 of composites, 518
dimensional changes in, 514 diamond abrasives, 518
high-copper, 513 factors affecting, 517
low-copper, 513 procedures for, 517
mixing of, 514 pumice, 518
polishing of, 514 purpose of, 517
safety of, 515 rouge, 518
setting reactions of, 509 tin oxide, 518
shrinkage in, 514 impression materials
strength and stiffness of, 514 about, 506
used in children, 514 characteristics and properties of, 507
amorphous structures in, 505 clinical uses of, 508
attaching structures in composition and chemistry of, 506
adhesion, 505 compound, 506–508
adhesion failure, 505 hydrocolloid, 506
cohesion, 505 impression plaster, 506
contact angle, 505 inelastic, 506
mechanical bonding, 505 polyether, 506–508
surface energy, 505 polysulfide, 506–508
wetting, 505, 505f reversible hydrocolloid, 506–508, 507t
biological properties of, 506 silicone, 508
allergic response, 506 silicone rubber, 506–508
microleakage, 506 zinc oxide–eugenol (ZOE), 506–508
toxic effects, 506 indirect restoratives
cements about, 519
about, 515 ceramic crowns, 519
calcium hydroxide, 515t, 517 ceramic materials, 519
glass ionomer, 515t, 516 CEREC technique, 519
polycarboxylate, 515t, 516 composite materials, 519
zinc oxide–eugenol (ZOE), 515t, 516 dental casting alloy materials, 519
zinc phosphate, 515, 515t inlays, 519
clinical studies for, 503t, 512t, 521t jacket crowns, 519
composite restorations onlays, 519
about, 511 porcelain materials, 519
and acids, 512 porcelain-fused-to-metal crowns, 519
adhesives used with, 512 veneers, 519
composition of, 511 infection control in labs
compression, tensile strength of, 511 for dentures, 524
exposure to materials of, 512 for fixed and removable prostheses, 524
material loss and staining of, 512 materials and techniques for, 524
particle amount of, 511 personal protective equipment (PPE), 524
polymerization of, 511 standard precautions for, 523
polymerization shrinkage of, 511 irreversible hydrocolloid (alginate)
principles of bonding of, 512 characteristics, properties of, 507
radiopacity of, 511 clinical uses of, 508
thermal conductivity of, 511 composition of, 506, 506t
thermal expansion of, 511 as elastomeric material, 506
water absorption of, 511 filling and seating tray with, 507
dentures mixing, 507
about, 520 removing and pouring impression, 508
homecare for, 520 liners
liners and conditioners for, 520 mineral trioxide aggregate, 517
office care for, 520 liquids in, 505
parts and materials of, 520 margination
retention of, 520 consequences of, 523
direct restorative materials defined, 523
about, 510 manual techniques for, 523
permanent, 510 power-driven techniques for, 523
temporary, 510 matrices
esthetic restorations for amalgam restorations, 523
ceramics, 510 for composite restorations, 523
porcelains, 510 illustrated, 523f
finishing and polishing materials purpose of, 523
of amalgams, 518 mechanical properties of
carbide abrasives, 518 brittleness, 502
590    Index

Biomaterials (Continued) Biomolecules (Continued)


ductility, 502 catabolism, 57
elastic deformation, 502 enzymes for, 57
elastic limit, 503 nucleic acids
elongation, 502 adenosine monophosphate (AMP), 56
forces, 502 cytosine monophosphate (CMP), 56
hardness, 503, 503t deoxy AMP, 56
inelasticity, 503 deoxy CMP, 57
malleability, 502 deoxy GMP, 57
modulus of elasticity, 502 deoxy TMP, 56
plastic deformation, 502 deoxyribonucleic acid (DNA), 56
strain, 502 function of, 56
stress, 502, 502f gene expression in, 57
stress/strain curve, 503, 503f and gene therapy, 57
physical properties of, 504 and genetic engineering, 57
coefficient of thermal expansion, 504, 504t guanosine monophosphate (GMP), 56
color, 504 mutagenesis in, 57
corrosion, 504 ribonucleic acid (RNA), 57
galvanism, 504, 504f thymidine monophosphate (TMP), 56
thermal, 504 transcription to, 57
thermal conductivity, 504 translation from, 57
translucency, 504 uridine monophosphate (UMP), 56
replication materials peptides, 56
accelerators and retarders, 509 proteins, 56
cast trimming, 510 Birth defects. See Congenital malformations
dental stone, 509 Bladder, 53f, 55f, 56f
dispensing, 510 Bleaching, and enamel rods, 21
gypsum, 509, 509t Bleeding on probing (BoP), 432, 447
hemihydrate, 509 Blood. See under Cardiovascular system (CVS)
high-strength dental (die) stone, 509 Blood pressure
mixing, 510 abnormalities of
model plaster, 509 hypertension, 68
pouring, 510 hypotension, 68
uses of, 509 prehypertension, 68
rubber dams, 522–523 clinical study for, 68–70, 68t
solid structures in, 505 defined, 67
tooth whitening long-term regulation of, 67
effectiveness of, 522 medication actions on, 67
in-office procedures for, 522 short-term regulation of, 67
office-dispensed in-home procedures for, 522 Bone. See also Alveolar bone; Cranial bones; Facial bones; Skeletal
over-the-counter products for, 522 system; Skull; under Pathology, general and oral
varnish cortical plate of, 24
about, 515 cysts in, 196
uses of, 515, 515t illustrated, 35f, 36f
using, 517 interdental, 26f
Biomolecules trabecular, 24
about, 55 Bowman’s capsule, 66
amino acids, 56 Brain, 37, 39f
carbohydrates Brainstem, 37, 39f
disaccharides, 55 Branchial apparatus, formation of, 10
functions of, 55 Breast cancer, 179
monosaccharides, 55 Bronchial tree, 51f, 50
oligosaccharides, 55 Bronchitis, 185
polysaccharides, 55 Brush ends, 22
lipids Bud stage, of tooth formation, 13f
defined, 55 Bulimia nervosa, 111, 209, 223
fatty acids, 55 Bulimorexia, 224
high-density lipoproteins (HDL), 56 Burkitt’s lymphoma, 175
low-density lipoproteins (LDL), 56 Burns, 191
steroids, 56
structural, 55 C
triacylglycerols, 55 Calcitonin, 61
location of, 55 Calcium hydroxyapatite, 21
metabolism Cancer. See Neoplasia; under Community oral health; under Pathol-
adenosine monophosphate (AMP), 57 ogy, general and oral
anabolism, 57 Canines. See under Permanent teeth; Primary teeth
Index    591

Canker sores, 193 Cardiovascular system (CVS) (Continued)


Cap stage, of tooth formation, 13f blood cells in, 45t
Capillary beds, 46 blood vessels in
Carbohydrates. See also under Biomolecules; under Nutrition anastomosis in, 46
about, 211 arteries, 45–46
composition of, 211 arterioles, 46
dietary requirements for, 212 capillary beds, 46
disaccharides, 211 plexus in, 46
lactose, 211 pressure in, 46
maltose, 211 veins, 46
sucrose, 211 venous sinuses, 46
fibers, 211 venules, 46
cellulose, 211 diseases of. See under Pathology, general
hemicellulose, 212 and oral
pectin, 212 heart, 44
functions of, 212 about, 44
monosaccharides, 211 atria of, 44
fructose, 211 atrioventricular valve of, 45
galactose, 211 chambers of, 44
glucose, 211 illustrated, 47f
hexoses, 211 Purkinje fibers of, 45
pentoses, 211 semilunar valves of, 45
nutrient sources of, 212 tricuspid and bicuspid valves of, 45
polysaccharides, 211 valves of, 45
dextran, 211 ventricles of, 45
glycogen, 211 wall of, 44
insulin, 211 hepatic portal circulation in, 46
starch, 211 pulmonary circulation in, 46, 48f
sweeteners systemic circulation in, 46, 48f
brown sugar, 212 Caries. See also under Community
high-fructose corn syrup oral health
(HFCS), 212 charting, 355
maple syrup, 212 and clinical treatment, 358–359
sucrose, 212 epidemiology of, 554
turbinado syrup, 212 indices for, 557
sweeteners, alternative and nutritional management, 213, 225
mannitol, 212 pathogenesis, diagnosis of, 559–561
sorbitol, 212 prevention, control of, 559–561
sugar alcohols, 212 radiographic interpretation of, 155
xylitol, 212 risk factors for, 359b
sweeteners, artificial types, treatment of, 359–361
Acesulfame-K, 212 Carotene, 33
aspartame (NutraSweet, Equal), 212 Carotid artery, 45–46
saccharin, 212 Case-based scenarios, 1
sucralose (Splenda), 212 Catabolism, 57
Carboxyl groups, 54 CD-ROM, contents of, 2
Cardiac dysrhythmia, 184 Cecum, 52
Cardiovascular disease. See under Nutrition; under Pathology, general Cell physiology
and oral communication, 58
Cardiovascular system (CVS) defined, 57
about, 44 differentiation, specialization for, 58
blood in duplication for, 58
ABO and Rh types, 44 membrane potential, 59
antibodies in, 44 membrane transport, 58
B cells in, 44 physiological functions of, 58
cytokines, 44 specific functions of
erythrocytes in, 44 carrier-mediated transport, 58
leukocytes in, 44 diffusion mechanism transport, 58
lymphocytes in, 44 plasma membrane transport, 58
NK cells in, 44 synthesis of proteins, 58
plasma cells in, 44 Cells, cell structure. See under Histology
plasma in, 44 Cemental dysplasia (cementoma), 194
T cells in, 44 Cementoblasts, 24f
thrombocytes (platelets), 44 Cementocytes, 24f
and tissue cells, 45t Cementoenamel junction (CEJ), 21, 22, 24f
transfusion reactions, 44 Cements. See under Biomaterials
592    Index

Cementum Clinical studies (continued)


avascularity of, 22 for pathology, 173, 175, 180t, 181, 183t, 190, 197
cementoblasts in, 22 for periodontology, 427t, 429t, 441t, 443t, 445t
development of, 22 for pharmacology, 283b, 295b, 300b, 305b, 312b
illustrated, 20f, 26f for primary teeth, 119
makeup of, 22 for radiation and radiography, 141t, 147t, 151t, 160t
and PDL, 22 for respiratory infection, 65t
repair of, 22 for salivary glands, 99
shape of, 22 for sense organs, 42
Central nervous system. See under Nervous system for special needs patient care, 532, 534t
Cerebellum, 37, 39f for tonsillar tissue, 105
Cerebral cortex, 37 for trigeminal nerve, sensory root, 102
Cerebral palsy (CP). See under Special needs patient care for vitamins, 219t
Cerebrum, 37 Clinical treatment, 369
Chambers, of heart, 44 about, 358
Cheek, 80 abused dependent patient
Cherubism, 198 awareness of, 382
Chronic hyperplastic pulpitis (pulp polyp), 194 barriers to care for, 383
Chronic irreversible airway obstruction, 185 oral manifestations of abuse, 383
Chyme, 52 professional care and homecare, 383
Circumvallate papillae, 19t, 81f signs of emotional deprivation and abuse, 383
Cirrhosis, 171 signs of physical abuse, 382
Clavicle, 36f, 82f signs of physical neglect, 382
Cleft (bifid) uvula, 11 types of abuse, 382
Cleft lip, palate caries pathogenesis and diagnosis
barriers to care for, 11 etiology of, 358
causes of, 10, 11 pathogenesis of, 358
clinical study for, 11 caries risk factors
commissural lip pits in, 11 assessing, 359
described, 11 clinical manifestations of, 359
development of, 11 determining, 359
education for, 11 laser for, 359
oral manifestations of, 11 listed, 359b
professional care and homecare for, 11 quantitative light-induced fluorescence for, 359
risk factors for, 11 radiographs for, 359
surgical intervention for, 11 caries, types and treatment
types of, 11 arrested caries, 361
Cleidocranial dysostosis, 198 caries process, 356f
Clinical studies early childhood caries (ECC), 360, 360f
for anatomy, 93 progression of, 359
for biomaterials, 503t, 512t, 521t rampant caries, 359
for blood disorders, 175–179 recurrent caries, 361
for blood pressure, 68–70 root caries, 361
for bulimia, 111–118 charting dental caries
for cleft lip and palate, 11 by location, 355
for clinical treatment, 351, 356, 362t, 367t, 375t, 378t, 381t, 384, new, 355
387–388 charting other conditions
for dental indices, 558–559 abfraction, 356
for diabetes, 173–175 abrasion, 355
for endodontically treated tooth, 22 ankylosed teeth, 356
for general and oral pathology, 173–179, 180–190, 183t, 190–191, attrition, 355
197–200 dens-in-dente, 356
for histology, 23t dilaceration, 356
for infection control protocol, 266–268 erosion, 356
for infectious diseases, 243t fractured teeth, 355
on instrumentation, 399, 417, 417t fusion, 356
for local anesthesia, 482t, 487t gemination, 356
for lymphatic system, 50–53 hypocalcification, 356
for medical and dental emergencies, 325b, 327, 330b, 333b, 336t malposed teeth, 355
for microbiology and immunology, 243t, 248t, 250–253, 254t, 257t, open contact, 355
266–268 charting restorations
for minerals, 221t acrylic crowns, 352
for neoplasia, 180t, 181–183, 183t amalgam, 353
for nitrous oxide, 494t bridge, 354
for nutrition, 213–218, 219t, 221t gold foil, 354
for pain management, 471, 481, 482t, 487t, 489t, 494t metal casting, 353
Index    593

Clinical treatment (Continued) Clinical treatment (Continued)


overhang, 355 financial record, 342
porcelain crowns, 352 floss, 369
sealant, 353 fluoride
temporary, 352 about, 371
tooth-colored, 352 acidulated phosphate (APF), 371
using G.V. Black’s classification, 352, 355f application times, frequency, 372
clinical studies, 351t, 356t, 362t, 367t, 375t, 378t, 381t, 384, 387 excessive ingestion, treatment of, 372
communication lethal dose of, 372
importance of, 385 sodium (NaF), 371
individualized instruction planning, 387 stannous (SnF2), 371
learning ladder, 386f topical application of, 371
learning principles, 387 toxicology, 372
nonverbal, 385 varnish, 371
questioning techniques, 386 geriatric patients
verbal, 386 with adverse drug reactions, 379
dental record, 342 with arthritis, 379
dentifrices with CVD, 379
about, 370 defined as, 379
anticalculus, 370 with dementia, Alzheimer’s, 379
antigingivitis and antiplaque, 370 with diabetes mellitus, 379
basic ingredients of, 320 oral manifestations of, 379
desensitizing, 370 with osteoporosis, 379
fluoride-containing, 370 with sensory defects, 379
other ingredients of, 371 health history
whitening, 371 described, 343
for dentinal hypersensitivity health promotion, disease prevention
about, 363 as DHCP objective, 384
etiology of, 363 human needs, 385f
hydrodynamic theory of, 360f Maslow’s Hierarchy of Needs, 377f
management of, 363 motivation for, 384
pathogenesis of, 363 self-perception of need, 384–385
dentition assessment, 352 implant patients, 382
drug history interdental brushes, 369
components of, 345 interdental tips, 369
drug side effects intraoral examination
candidiasis, 346 buccal mucosa, 350
gingival hyperplasia, 346 floor of mouth, 350
xerostomia, 345 gingiva, 350
drugs affecting dental treatment hard and soft palate, 350
antianginals, 346 labial and alveolar mucosa, 350
anticoagulants and antiplatelets, 346 preparation for, 342
antidepressants, 347 tongue, 350
antihypertensives, 346 manual toothbrushes
cardiac drugs, 346 types of, 368
diuretics, 346 using, 368
thyroid drugs, 347 medical history
tranquilizers, 347 chief complaint, 343
vasoconstrictors, 347 described, 343
edentulous patient, 380 health status, 343
evaluating tooth stain questionnaire vs. interview, 343
differences in, 357 subjective vs. objective, 343
endogenous intrinsic, 357 medically compromised patients
exogenous extrinsic, 358 with allergy, 345
exogenous intrinsic, 358 with blood disorder, 344
and evidence-based decisions, 368 with cardiac dysrhythmia, 344
extraoral examination with congestive heart failure, 344
bones, muscles, lymph nodes, and glands, 349 with CVD, 343
described, 348 with diabetes mellitus, 344
ears, 349 with epilepsy, 345
eyes, 349 with hepatitis, 345
face, 348 with hypertension, 344
hair, 348 with joint replacement, 345
nose, 349 with renal disease, 344
overall physical, 348 with respiratory infection, 345
skin, 349 with sexually transmitted disease, 345
594    Index

Clinical treatment (Continued) Clinical treatment (Continued)


with valvular heart disorder, 344 professional care and homecare of, 381
with xerostomia, 345 removable appliances, 380
motivation, factors influencing removable prostheses, 380
pain, 385 pulp vitality testing
sincerity, concern, and support, 385 about, 364
unmet needs, 385 by dental history, 364
mouth protectors electric pulp test, 365
about, 375 by examination, 364
purpose of, 375 interpreting test results, 365
types of, 375 by percussion, 364
occlusal evaluation, 358 by radiograph, 364
oral antimicrobials by thermal test, 364
about, 373 sealants, enamel
basic ingredients of, 373 about, 373
chlorhexidine digluconate, 373 after application of, 375
oxygenating, 373 application of, 374
phenolic compound, 373 composition of, 374
povidone-iodine, 373 contraindications for, 374
quaternary ammonium compound, 373 indications for, 374
SnF2, 373 polymerization of, 374
oral irrigators, 370 purpose of, 373
patient assessment, 347 special patients, 376
patient examination tongue cleaners, 369
auscultation, 348 tooth identification
documentation, 348 anatomical charting for, 353f, 354f
olfaction, 348 dentures, 352
palpation, 347 implants, 352
visual observation, 347 missing teeth, 352
pediatric patients by number or symbol, 352
adolescents, 376 root canal therapy, 352
children, 376 supernumerary teeth, 352
importance of hygienist to, 376 unerupted permanent teeth, 352
infants, 376 Commissural lip pits, 11
periodontal evaluation Communication. See under Clinical treatment
about, 365 Communication disorders. See under Special needs patient care
additional tests, 367 Community health or research principles, 1
clinical attachment level, 366 Community health studies for community oral health, 553, 558–559,
dental biofilm, 366 561–562, 564–567
description of disease, 365 Community oral health
description of health, 365 biostatistics
documentation of, 367 about, 551
furcation involvement, 366–367 biostatistics sampling
mobility, 367 convenience, 531
mucogingival defects, 366 random, 551
patient noncompliance, 367 stratified, 551
probing, 366 systematic, 551
probing gingival pocket, 366 central tendency statistics
probing periodontal pocket, 366 mean, 552
radiographic examination, 366 median, 552
recession, 366 mode, 552
referral after, 367 community health studies for, 553, 558–559, 561–562, 564–567
power toothbrushes community program
types of, 369 developing, 562–563
of pregnant patients evaluating, 564
barriers to care for, 378 implementing, 563–564
dental care for, 378 planning, 563
general health of, 378 dental indices
medical manifestations of, 377 community health study for, 558–559
oral manifestations of, 377 community periodontal index, 557
trimesters of, 377 decayed, missing, filled teeth (DMFT), 557
tumors of, 363f for fluorosis, 558
for prostheses and appliances gingival bleeding index, 556
fixed appliances, 380 gingival index (GI), 556
fixed prostheses, 380 O’Leary plaque index, 555
problems with, 381 oral hygiene index, 555
Index    595

Community oral health (Continued) Community oral health (Continued)


personal hygiene performance index–modified (PHP-M), 555 in schools, 560, 561
Plaque Assessment Scoring System (PASS), 557 government’s role in, 550
plaque index (PI), 556 National Institutes of Health, 550
Ramfjord’s periodontal disease index (PDI), 557 Public Health Service, 550
root caries, 558 research and educational programs, 550
Russell’s periodontal index (PI), 556 inferential statistics
(severe) early childhood caries (ECC), 557 ANOVA, 552
unmet treatment needs (UTN), 557 chi-square test, 553
used for, 555 correlation analysis, 553
descriptive statistics student’s t-test procedure, 552
about, 552 oral cancer prevention and control, 562
bell curves in, 552 payment for dental care
frequency distribution, 552 fee-for-service, 549
graphs, 552 third-party plans, 549
developing community profile usual, customary, and reasonable (UCR) fees, 549
dental disease profiles for, 562 periodontal disease prevention and control, 562
fluoridated water profile for, 563 prevention of dental disease
gathering information for, 562 levels of health promotion, 559
policy- and decision-making profiles for, 562 research in
professionally applied fluoride profile, 563 about, 565
resources for, 563 administrative and evaluative, 565
developing community programs, 562 calibration of, 565
assessing public health problems, 562 educational and behavioral, 565
diet counseling in, 561 funded by government, 567
epidemiology illustrated, 566t
as disease prevalence, 553 quasi-experimental, 565
and endemics, 554 reliability of, 565
and epidemics, 554 single-subject, 565
and incidence, 553 true experimental, 565
index of, 554 validity of, 565
and morbidity, 554 sealant programs, 561
and mortality, 554 utilization of dental services
and pandemics, 554 national statistics on, 549
and prevalence, 553 perceived vs. normative needs, 549
and rate of disease, 553 population group differences in, 549
epidemiology of caries typical patient for, 549
about, 554 variability statistics
background of, 554 range, 552
statistics on, 554 standard deviation, 552
trends in, 554 variance, 552
epidemiology of oral cancer Component A of NBDHE, 1, 2
background on, 558 Component B of NBDHE, 1, 2
statistics on, 559 Composite restorations. See under Biomaterials
trends in, 559 Concepts and theory, importance of, 2
epidemiology of periodontal disease Concrescence, 197
about, 554 Condensing osteitis (focal sclerosing osteomyelitis), 194
background of, 554 Congenital heart disease, 184
gingivitis trends, 555 Congestive heart failure, 184
periodontitis trends, 555 Conjunctiva, 42f, 80
statistics on, 555 Connective tissues
federally funded services illustrated, 34f
Head Start, 551 in muscle, 59
Maternal and Child Health Services, 551 Cranial bones
Medicaid, 551 ethmoid, 84f, 85, 85f, 87f
Medicare, 551 frontal, 84, 85f, 87f
National Health Service Corps, 551 occipital, 84, 85–87f
state health agencies, 551 palatine, 86f
fluoridation parietal, 84f, 85, 85f, 87f
adding to water, 559 sphenoid (butterfly), 85, 85–87f, 92f
in bottled water, 560 temporal, 84–87f, 85, 92f
community varnish programs, 561 zygomatic, 85–86f
optimum concentration in water, 559 Cranial nerves
as prescribed supplement, 559, 560t defined, 39
promoted in community, 559 eighth (vestibulocochlear), 41
removing from water, 559 eleventh (accessory), 41
596    Index

Cranial nerves (Continued) Dental sac, 13f, 14


fifth (trigeminal), 39 Dentifrices. See under Clinical treatment
first (olfactory), 39 Dentigerous (follicular) cyst, 195
fourth (trochlear), 39 Dentin
frontal, 84f circumpulpal, 22
illustrated, 40f contour lines of Owen, 22
ninth (glossopharyngeal), 41 dentinogenesis of, 21
second (optic), 39 dysplasia in, 199
seventh (facial), 40 fluid in, 22
sixth (abducens), 40 illustrated, 13f
tenth (vagus), 41 imbrication lines of von Ebner, 21
third (oculomotor), 39 intertubular, 22
twelfth (hypoglossal), 41 makeup of, 21
Curets. See under Instrumentation odontoblastic process in, 22
Cuspids. See Permanent teeth, canines (cuspids); Primary teeth, peritubular, 22
canines (cuspids) predentin, 21
Cyclic neutropenia, 198 primary, 22
Cyst, defined, 194 and pulp, 23
Cystic fibrosis (CF), 186 sclerotic, 22
Cysts. See under Pathology, general and oral secondary, 22
Cytokines, 44 tertiary, 22
Cytosine monophosphate (CMP), 56 Tomes’ granular layer, 22
tubules in, 22
D types of, 22
Deciduous dentition. See Primary teeth Dentinal hypersensitivity, 22
Dens-in-dente, 196 Dentinoenamel junction (DEJ)
Dental anatomy. See also Permanent teeth; Primary teeth; Occlusion development of, 14
dentition periods and enamel formation, 21
mixed, 108 illustrated, 13f
permanent (adult), 108 tissues of, 20f
primary (deciduous), 108 Dentinogenesis imperfecta, 199
dentitions Denture-induced hyperplasia, 192
about, 106 Dentures. See under Biomaterials; under Clinical treatment
permanent (adult), 106, 107f Deoxy AMP, 56
primary (deciduous), 106, 107f Deoxy CMP, 57
eruption timetable Deoxy GMP, 57
favorable sequence for, 109f Deoxy TMP, 56
permanent teeth, 109t Deoxyribonucleic acid (DNA), 56–57
premature, 108 Dermis. See under Integumentary system
primary teeth, 108t Dermoid cyst, 196
variables influencing, 108 Diabetes. See also under Nutrition; under Pharmacology, for endocrine
tooth designation disorders
about, 108 about, 172
International Standards Organization Designation System (ISO), barriers to care for, 173
108 clinical study for, 173
Universal Tooth Designation System, 108 diagnostic, monitoring tests for, 172
tooth surfaces education for, 173
about, 107 gestational diabetes, 172
contact point, 107 glucose levels for, 172b
embrasure, 107 latent autoimmune, 172
height (crest) of curvature, 108 oral complications of, 172
interproximal space, 107 prediabetes, 172
tooth types professional care and homecare for, 173
about, 107 risk factors for, 173
canines, 107 signs and symptoms of, 172
incisors, 107 systemic complications of, 172
molars, 107 treatment for, 172
premolars, 107 type I diabetes, 172
Dental biofilm. See under Periodontology type II diabetes, 172
Dental healthcare personnel (DHCP), 262 Diabulimia, 224
Dental indices. See under Community oral health Diencephalon, 37, 39f
Dental lamina, 13f Digestion, 69
formation of, 14 Digestive system. See also Gastrointestinal tract physiology
successional, 14 components of, 51
and tooth germs, 14 esophagus, 51
Dental papilla, 13f, 14 gallbladder, 53
Index    597

Digestive system (Continued) Embryology (Continued)


illustrated, 52f placenta formation in, 7
large intestine, 52 second week activity of, 7
cecum in, 52 third week activity of, 7
functions of, 52 trilaminar disc differentiation, 7
rectum and anal canal in, 52 fetal period
liver length of, 8
blood in, 52 preimplantation period of
functions of, 52 blastocyst formation in, 7
illustrated, 52f chromosomal activity during, 7
location of, 52 fertilization, 7
pancreas mitosis, 7
composition of, 52 zygote formation in, 7
endocrine cells in, 52 stomodeum, oral cavity formation, 8
location of, 52 tongue development in, 12
pharynx, 51 tooth development in
small intestine, 52 apposition stage, 14
stomach, 52 bell stage, 14
chyme in, 52 bud stage, 13f, 14
functions of, 52 cap stage, 14
illustrated, 52f eruption process, 14
location of, 52 initiation stage, 12, 13f
mucosal lining of, 52 maturation, 15
rugae in, 52 Nasmyth’s membrane, 15
Dilaceration, 197 time span for, 12
Disabled patients. See under Special needs patient care Emphysema, 185
Disaccharides, 55. See also under Nutrition Enamel
Dosimeter, 140 amelogenesis in, 21
Down syndrome (trisomy 21), 8 bleaching of, 21
Drugs. See Pharmacology developmental disorders in. See under Pathology,
Dry socket, 194 general and oral
and enamel rods, 21
E and Hunter-Schreger bands, 21
Ear, 42f. See also Sense organs, auditory illustrated, 13f, 20f, 24f
Eating disorders. See under Nutrition lamellae, 21
Ecchymosis (bruise), 193 and lines of Retzius, 21
Ectodermal dysplasia, 199 makeup of, 21
Efferent (motor) nerves, 37 spindles, 21
Embryology and Tomes’ process, 21
congenital malformations in tufts, 21
amniocentesis for, 8 Enamel organ, 13f, 14
developmental disabilities, 8 Endocrine gland disorders, 169
Down syndrome (trisomy 21), 8 Endocrine system. See also Endocrinology
ectodermal dysplasia, 8 about, 43
ectopic pregnancy, 8 glands of
spina bifida, 8 about, 43
teratogens, 8 adrenal cortex, 44
defined, 7 anterior pituitary, 44
development of face and oral cavity intermediate pituitary, 44
branchial apparatus, 10 pancreas, 44
facial processes, 8 posterior pituitary, 44
frontonasal process and upper face, 9 testes, 44
mandibular arch and lower face, 9 thyroid, 44
maxillary process and midface, 9–10 hormones in, 43
overall growth of, 8 actions of, 44
palate, 10–11, 10f amines, 43
timing of, 8 and endocrine glands, 44
underlying bones, 8 proteins, polypeptides, 43
embryonic period of steroids, 43
bilaminar disc formation in, 7 negative feedback in, 61f
central nervous system formation, 7 Endocrinology. See also Endocrine system; under Pharmacology
connective tissue cells formation, 7 antidiuretic hormone (ADH), 60
embryo folds, 8 CNS hormones in
fourth week activity of, 7 melatonin, 60
mesenchyme formation, 7 defined, 60
mesoderm formation, 7 hormone categories in
598    Index

Endocrinology (Continued) Equilibrium, 42


amines, 60 Erosion, of teeth, 191
peptide, protein, 60 Eruption, 8, 14, 15
steroids, 60 Eruption cyst, 195
hormones affecting calcium metabolism, 61 Erythema multiforme (EM), 189
calcitonin, 61 Erythrocytes, 44
parathyroid hormone (PTH), 61 Esophagus
hormones of adrenal glands functions of, 51, 69
from cortex, 62 illustrated, 52f
glucocorticoids, 62 Esters, 55
mineralocorticoids, 62 Estrogen, 62
sex steroid hormones, 62 Ethers, 54
hormones of pancreas Ethics and jurisprudence
from exocrine glands, 61 civil law, 576
glucagon, 62 contract law, 576
insulin, 62 criminal law, 577
from isles of Langerhans, 62 dental practice acts
hormones released by hypothalamus purposes of, 581
adrenocorticotropic hormone (ACTH), 60 of state boards, 582
follicle-stimulating hormone (FSH), 60 violations of, 582
luteinizing hormone (LH), 60 ethical dilemmas and decision-making
thyroid-stimulating hormone (TSH), 60 about, 576
hormones released by pituitary and altruism, 576
growth hormone–releasing hormone (GH-RH), 60 identifying values in, 576
prolactin-releasing factor (PRF), 60 moral evaluation of publicity, 576
hormones secreted by cortex, 62 and patients’ rights, 576
hormones secreted by medulla, 62 and societal trust, 576
ovarian hormones and ultimacy, 576
ADH, 62 ethical principles
angiotensin I and II, 62 autonomy, 575
estrogen, 62 beneficence, 575
fluid volume and blood pressure, 62 fidelity, 576
and menstrual cycle changes, 62 justice, 575
progesterone, 62 nonmaleficence, 575
renin, 62 veracity, 576
satiety hormones, 63 grounds for malpractice
leptin, 63 causing trauma, 577
testicular hormones failure to diagnose, 576
testosterone, 62 failure to inform, 577
and thyroid hormones failure to radiograph, 577
disorders of, 61 failure to refer, 577
functions of, 61 failure to sterilize, 576
production and transport of, 61 Health Insurance Portability and Accountability Act (HIPAA)
types of, 61 Current Dental Terminology (CDT), 580
Endodontic therapy. See also Pulp; under Clinical treatment dental office implementation of, 579
abscesses in, 194 enactment of, 579
appropriateness of, 521 Health Care Procedural Coding System, 580
clinical study for, 22 privacy rule of, 579
described, 23 requirements of, 579
end result of, 521 security rule of, 580
restoration after, 522 informed consent
root end surgery (apicoectomy), 522 defined, 578
treatment sequence for, 521 expressed, 579
Endoplasmic reticulum (ER), 59 implied, 579
Enzymes, for metabolism, 57 informed refusal, 579
Eosinophil, 45t meaning to patients, 579
Epidermis parental or guardian, 579
illustrated, 34f legal responsibilities and duties
stratum basale, 33 of healthcare provider and patient, 580–582
stratum corneum, 33 professional practice studies for, 577–582
stratum granulosum, 33 prudent dental care
stratum lucidum, 33 avoiding admissions against interest, 577
stratum spinosum, 33 avoiding res ipsa loquitur, 577
Epidermoid cyst, 196 identifying a proximate cause, 577
Epilepsy. See under Pathology, general and oral keeping accurate records, 577
Epithelial rests of Malassez, 14 and liability insurance, 578
Index    599

Ethics and jurisprudence (Continued) Fluoride, fluoridation (Continued)


making insurance claims, 578 community profile for, 563
proving contributory negligence, 577 community varnish programs, 561
as reasonably prudent practitioner, 577 in kidney, 53
and statute of limitations, 578 optimum concentration of, 559
transferring records, 578 prescribed supplement, 559, 560t
and relativism, 575–576 promoted in community, 559
responsibilities and duties of healthcare provider removing from water, 559
holding of licensure, 581 in schools, 560–561
tort law Fluorosis, 197
negligence, 576 Foliate papillae, 19t
slander and libel, 576 Follicle-stimulating hormone (FSH), 60
technical assault and battery, 576 Food diary, 225
Eukaryotes, 233 Fordyce granules, 199
Euthyroid, 61 Frictional keratosis, 193
Evidence-based decision making, 368 Frontonasal process, 9, 9f
Exoenzymes, 233 Fungiform papillae, 19t, 81f
Explorers. See under Instrumentation Fusion, of teeth, 196
Eyelids, 80
Eyes, eyesight, 42f. See also Sense organs, visual; under Head and G
neck anatomy Gallbladder, 52f, 53
Gas diffusion, pressures, 64
F Gastrointestinal disease. See under Pathology, general and oral
Face, development of Gastrointestinal tract physiology. See also Digestive system;
bones in, 34 under Pathology, general and oral; under Pharmacology
branchial apparatus, 10 anatomical structures, functions of
frontonasal process and upper face, 9 large intestine, 70
mandibular arch and lower face, 9 oral cavity, 69
maxillary process and midface formation, 9–10 pancreas, 70
palate, 10–11 pharynx, 69
stomodeum and oral cavity formation, 8–11 salivary glands, 69
tongue, 12 small intestine, 70
Facial bones stomach, 69
about, 86 teeth, 69
lacrimal, 84–85f, 86, 87f tongue, 69
mandible, 82f, 84f, 87, 88f defined, 69
maxilla, 84–87f, 86 functions of, 69
palatine, 86, 86f Gastrointestinal disease. See under Pathology, general and oral
vomer, 84f, 86, 86f Gemination, 196
zygomatic, 86 Genetic engineering and therapy, 57
Facial nerve Geriatric patients. See under Clinical treatment
about, 103 Gingiva. See also under Periodontology
chorda tympani, 103 alveolar mucosa, 19, 20f
greater petrosal, 103 attached, 19, 20f
posterior auricular, stylohyoid, posterior digastric, 103 col, 19
Facial (seventh cranial) nerve dentinogingival junction tissues, 19
about, 40 fiber groups of, 20f
illustrated, 40f free gingival groove, 19, 20f
lesions of, 40 hyperplasia of, 192
path of, 40 interdental papilla, 19
and taste, 41 junctional epithelium, 20
Fatigue marginal (free), 19, 20f
central, 60 mucogingival junction of, 19
general, 60 probing depth for, 21
muscular, 60 sulcular epithelium, 19, 20f
neuromuscular junction, 60 sulcus in, 20f
Fatty acids, 55 tissues of, 19–21, 81
Fiber groups, 20f, 25, 25t, 26f Glandular tissue
Fibroma, 178, 193 about, 96–99
Fibromatosis, hereditary, 198 lacrimal, 96
Fibrous dysplasia, 177 parathyroid, 98, 98f
Filiform papillae, 19t salivary, 96
Floss, 369 clinical study for, 99
Fluoride, fluoridation. See also under Clinical treatment histology of, 97
adding to water, 559 illustrated, 97f
in bottled water, 560 minor glands, 97
600    Index

Glandular tissue (Continued) High-density lipoproteins (HDL), 56


parotid, 96 Histology. See also under Gingiva
parotid (Stensen’s) duct, 96, 97f cell structure in
sublingual, 97 about, 16
sublingual (Bartholin’s) duct, 96, 97f cytoskeletal structures in, 16
submandibular, 96, 97f endoplasmic reticulum (ER), 16
submandibular (Wharton’s) duct, 96, 97f extracellular fluid in, 16
thymus, 98 Golgi apparatus, 16
thyroid, 98, 98f illustrated, 15f
Glaucoma, 188 intracellular fluid in, 16
Globulomaxillary cyst, 195 lysosomes in, 16
Glossopharyngeal (ninth cranial) nerve mitochondria in, 16
about, 41 nucleus, 16
illustrated, 40f peroxisomes in, 16
path of, 41 plasma membrane, 15
Glucagon, 62 ribosomes in, 16
Glucocorticoids, 62 vesicles formed, 16
Glycolysis, 57 clinical study for, 23t
Goiter, 61 defined, 15
Granuloma, 194 dental tissues
Graves’ disease, 170 cementum. See Cementum
Gross anatomy, 33 dentin. See Dentin
Growth hormone–releasing hormone (GH-RH), 60 enamel. See Enamel
Guanosine monophosphate (GMP), 56 pulp. See Pulp
oral mucosa in
H illustrated, 17f
H zone, 59 lamina propria, 18
Hashimoto’s thyroiditis, 170 lining, 17
Head and neck anatomy masticatory, 18, 18t
arterial blood supply. See Arterial blood supply specialized, 19
clinical study for, 93 periodontium. See Periodontium
cranial bones. See Cranial bones tissues of
facial bones. See Facial bones connective, 16
facial nerve. See Facial nerve epithelial, 16
glandular tissue. See Glandular tissue lining mucosa, 17, 18t
lymph nodes. See Lymph nodes muscle, 17
muscles of, 88 nerve, 17
oral cavity. See Oral cavity Hodgkin’s lymphoma, 189
regions of, 80–81, 82f Homeostasis, 57
anterior, 81 Hormones. See under Endocrine system; under
frontal, 80 Endocrinology
hyoid bone, 82 Hydrocarbons. See under Biochemistry
infraorbital, 80 Hyoid bone, 82, 82f, 91f
mental, 80 Hyperadrenalism, 171
nasal, external nose, 80 Hyperpituitarism, 169
oral, 80 Hyperplasia, 192
orbital, 80 Hypersensitivity
parietal, occipital, 80 allergy, 188
posterior cervical triangle, 81 dentinal. See under Clinical treatment
temporal, 80 Hypertension, 68, 184
thyroid cartilage, 82 Hyperthyroidism, 170
skull. See Skull Hypoadrenalism, 171
sternocleidomastoid muscle (SCM), 81 Hypodermis, 34f
temporomandibular joint (TMJ), 92–93 Hypodontia, 196
tonsillar tissue. See Tonsillar tissue Hypoglossal (twelfth cranial) nerve, 40f, 41
trigeminal nerve. See Trigeminal nerve, sensory root Hypoglycemia, reactive, 213
venous drainage for. See Venous drainage Hypopituitarism, 170
Health history. See under Clinical treatment Hypotension, 68
Heart. See under Cardiovascular system (CVS) Hypothalamus, 37, 39f
Hemangioma, 177 Hypothyroidism, 61, 170
Hematoma, 193
Hemispheres, of brain, 37, 39f I
Hemophilia, 198 I bands, 59
Hepatic portal circulation, of CVS, 46 Ig antibodies, 46t
Hepatitis. See under Pathology, general and oral Immunoglobins (antibodies), 46t
Hertwig’s epithelial root sheath (HERS), 13f, 14, 22 Immunological disorders. See under Pathology, general and oral
Index    601

Immunology. See also Infection control agents; Infection control protocol Implants
about, 254 bone grafting, 458
allergic reaction types ratio for, 458
rubber, latex allergy, 256 contraindications for, 457
type I, 256 endosseous, 457
type IV (delayed), 256–257 failure and periimplantitis
types II, III, 256 characterized by, 459
allergies, 256 recognizing, 458
antibiotics role of smoking in, 459
about, 259 similarities to periodontitis, 459
ampicillin (polycillin), 259 fibrointegration, 458
beta-lactam, 259 illustrated, 158f
cell membranes of, 259 immediate-loading surgery, 457
cephalosporins, 259 maintenance of
DNA synthesis of, 259 by hygienist, 457
ideal, 259 instruments for, 458
judicial use of, 260 polishing for, 458
origins of resistance to, 260 prostheses, 458
penicillin (Pen-Vee K), 259 recare interval for, 458
sites of action of, 259 mini, 457
tetracyclines, 260 NK cells, 255
transcription action of, 260 osseointegration, 458
translation action of, 260 sinus lifting, 458
antigens, 254 subperiosteal, 457
antimetabolites tissue interface, 458
differences from antibiotics, 260 traditional surgery
isoniazid (INH), 260 about, 457
sulfonamides, 260 conditions for success, 458
autoimmunity first phase, 457
about, 257 second phase, 457
Hashimoto’s thyroiditis, 258 transosteal (staple), 457
multiple sclerosis (MS), 258 uses of, 457
myasthenia gravis (MG), 258 Incisors. See under Permanent teeth; Primary teeth
reasons for, 258 Indices, dental. See under Community oral health
rheumatoid arthritis (RA), 258 Infection control agents. See also under Biomaterials
Sjögren’s syndrome, 258 alcohols, 262
systemic lupus erythematosus (SLE), 258 alkylating agents, 262
type I diabetes mellitus, 258 autoclaving, 261
B cells, 255 cell death by, 261
B lymphocytes chemical agents, effects, 261
about, 255 detergents, 261
and humoral immunity, 255 dry heat, 261
illustrated, 256f effectiveness of, 261
producing antibodies, 255 filtration, 261
chemical, physical control of microorganisms, heavy metal compounds, 262
261 ionizing radiation (gamma rays), 261
chemotherapeutic drug resistance, 260 moist heat, 261
clinical study on, 243t, 248t, 250–253, 254t, 257t oxidizing agents, 262
development of immunity pasteurization, 261
primary and secondary, 255 phenolic compounds, 261
immunodeficiencies refrigeration, freezing, 261
about, 258 ultraviolet (UV) light, 261
opportunistic infection, 258 Infection control protocol. See also under Radiation, radiography
primary, 258 care, disposal of waste
reactivated infection, 258 biohazardous waste, 267
secondary, 258 EPA and ADA standards on, 235
severity of, 258 clinical studies for, 266–268
treatments for, 258 in dental labs, 523–524
lymphocytes, 254–255 in the dental office, 262
T cells, 255 high- and moderate-risk pathogens, 262
T lymphocytes low-risk pathogens, 262
about, 255 sharps, needlestick protocols
and cell-mediated immunity, 255 about, 267
illustrated, 256f needle recapping, 268
secreting cytokines, 255 occupational exposure incident, 268
Impetigo, 169 sharps handling, 267
602    Index

Infection control protocol (Continued) Instrumentation (Continued)


standard precautions, 263 illustrated, 400f
DHCP hand hygiene, 264 shank, 400
DHCP immunization, 263 working end, 400
disinfection, 265, 266t chisels (scalers)
gloves, 264, 264f described, 404
masks, 264 functions of, 404
patient medical history, 263 clinical studies for, 399, 417
personal protective equipment (PPE), 264 Insulin, 62
preprocedural antimicrobial mouthrinse, 266 contraindications for power-driven
procedures related to, 265 on children, 411
protective clothing, 265 on demineralized areas, 411
protective eyewear, face shields, 265 on gaggers and mouth breathers, 411
sharps, biohazard disposal, 266 with pacemakers, 411
sterilization, 265 respiratory, immunocompromised patients, 410
use of high-volume evacuation (HVE), 266 on restorations, 411
water line disinfection, 266 sensitivity, 411
transmission curets
avoidance of, 251 area-specific, 403, 403t
and chain of infection, 263 described, 402
conditions for, 263 functions of, 405
in dental office, 262 illustrated, 403f
through indirect contact, 263 Langer, 403
through inhalation, 263 O’Hehir, 403
through mucosa, 263 Turgeon, 403
Infectious diseases universal, 403
bacterial infections, 240 explorers
Actinomyces, Actinomyces israelii, 241, 241f #17-style, 401
group A Streptococcus, 240 described, 401
infective (bacterial) endocarditis (IE), 242 functions of, 401
legionnaire’s disease, 241 illustrated, 401f
meningitis, 241 ODU 11/12, 401
methicillin-resistant Staphylococcus aureus (MRSA), 242 pigtail and cowhorn, 401
osteomyelitis, 242 shepherd’s hook, 401
prosthetic joint infection, 242 files, periodontal
tuberculosis (TB), 240 described, 404
bacterial toxins, 243 functions of, 404
enterohemorrhagic Escherichia coli, 244 illustrated, 404f
staphylococcal enterotoxin, 244 fulcrum (finger rest), 405
toxic shock syndrome (TSS), 244 hoe scalers
whooping cough (pertussis), 244 described, 403
breaching barriers, 239 functions of, 404
clinical study on, 243t illustrated, 404f
nonspecific host defenses implant maintenance instruments, 404
biological, 240 manual instruments
chemical, 239 about, 400
physical, 239 mirrors
specific host defenses, 240 about, 401
transmission means types of, 401
airborne, 240 uses of, 401
by breached skin, 240 modified pen grasp
by contact, 240 advantages of using, 405
fecal-oral, 240 during assessment, 405
by living vector, 240 technique for, 404
perinatal, 240 during treatment, 405
by respiratory system, 240 palm thumb grasp
sexual mode, 240 technique, uses for, 405
by vehicle, 240 patient preparation for power-driven, 411
transmission routes for, 239 pen grasp, 404
Initiation stage, of tooth formation, 13f polishing methods
Injections. See under Anesthesia, local with air-powder polish, 413
Instrumentation with porte polisher, 413
adaptation, of instruments, 405 with rubber cup, brush, 413
basic instrument design positioning
about, 400 advantages of correct, 398
handle, 400 of clinician, 399
Index    603

Insulin (Continued) Insulin (Continued)


cumulative trauma disorders, 399 shape and presentation of stone, 407
ergonomics of, 399 stone maintenance, 407
importance of, 398 stone movement for, 408f
of patient, 398 wire edges, 408
power-driven scaling instruments sterilization
cavitation, 410 chemical sterilants, 416
and fluid lavage, 410 chemical vapor, 415
furcation tips, 410 dry heat, 415
indications for use, 408 ethylene oxide, 416
magnetorestrictive, 409 indications for, 415
periodontal, thin tips, 409 methods of, 415t
piezoelectric, 409 moist heat (autoclave), 415
sonic, 409 moist heat (quick turnover steam), 415
standard tips, 409 strokes, for instruments
technique for using, 409 differences in, 406
therapeutic effect of, 410 direction of, 406
ultrasonic, 409 exploratory, 406
power-driven vs. manual instruments length of, 406
advantages of power-driven, 410 parts of, 406
disadvantages of power-driven, 410 root planing, 406
probes walking, 406
described, 401 Insulin, 62
digital, 402 Integumentary system
functions of, 402 defined, 33
illustrated, 401f dermis, 33
Marquis, 402 epidermis, 33
Nabors’ (furcation), 402 hair, 33
other uses of, 402 hypodermis, 33
WHO, 402 illustrated, 34f
processing nails, 33
biological monitoring of, 416–417 sweat glands, 33
for handpieces, 416 Interdental brushes, 369
manual cleaning, 414 Interradicular septum, 26f
packaging, 415 Irreversible hydrocolloid. See under Biomaterials, irreversible
for prophy angles, contra angles, air-water syringes, 416 hydrocolloid (alginate)
safety needed for, 414 Irritation fibroma, 193
storing afterward, 416
ultrasonic cleaning, 414 J
ultrasonic tips, 416 Jaundice, 171
purpose of, 398–424 Junctional epithelium (JE), 20f
scalers Juxtaglomerular apparatus, in kidney, 53, 54f, 66
described, 402
functions of, 402 K
illustrated, 402f Kaposi’s sarcoma, 178
selective polishing Keratocysts. See under Pathology, general and oral
defined, 412 Keratosis, 193
purposes of, 412 Ketones, 54
selective polishing, contraindications for Kidneys. See under Urinary system
abrasiveness, 412 Kwashiorkor, 210–211
contamination, 412
frictional heat, 412 L
roughness, 412 Labial frenum, 81
tooth structure removal, 412 Lactose intolerance, 213
sharpening Lamina propria, 17f
angles for, 407f Langerhans’ cell histiocytosis X, 175
Arkansas stone, 407 Langerhans’ cells, 33
assessing for sharpness, 408 Large intestine, 52f, 70. See also under Digestive system
ceramic stone, 407 Larynx (voice box), 34, 50, 51f
defined, 407 Lateral canthus, 80
India stone, 407 Lateral periodontal cyst, 195
mechanically, 407 Leptin, 63
methods of, 407 Leukemia, 175
need, frequency of, 407 Leukoedema, 199
objectives of, 407 Leukopenia, 175
recontouring and replacing, 408 Lichen planus, 189
604    Index

Linea alba, 199 Maxillary process, 8, 9f, 10f


Lingual papilla(e), 19t Mean arterial pressure (MAP), 67
Lingual thyroid nodule, 199 Median palatine cyst, 195
Lingual varicosities, 200 Medical and dental emergencies
Lining mucosa, 18t allergic reactions
Lipids. See under Biomolecules; under Nutrition anaphylaxis, 326
Lipoma, 177–178 angioedema, 326
Liver, 171. See under Gastrointestinal tract physiology; causes of, 325
under Digestive system contact dermatitis, 326
Loop of Henle, 66 urticaria (hives), 326
Low-density lipoproteins (LDL), 56 ASA physical classifications
Ludwig’s angina, 194 ASA I: healthy, 322
Lungs, 50, 51f, 185. See also Respiratory physiology ASA II: mild systemic disease, 322
Luteinizing hormone (LT), 60 ASA III: severe systemic disease, 322
Lymph nodes ASA IV: severe systemic threat, 322
cervical ASA V: moribund, 322
accessory, 105, 106f ASA VI: dead, 322
anterior jugular, 104, 105f asthma, 327
external jugular, 104, 105f cardiac emergencies
inferior, 104, 106f angina pectoris, 328
jugulodigastric, 104, 106f basic life support, 328t
jugulo-omohyoid, 105, 106f myocardial infarction (MI), 329
submandibular, 104, 105f seriousness of, 327
submental, 103, 105f sudden cardiac arrest (SCA), 329
superior, 104, 106f cerebrovascular accident (CVA), 329–331
supraclavicular, 105, 106f common dental emergencies
of head avulsed tooth, 334
anterior auricular, 103, 104f broken instrument tip, 334
deep parotid, 103 chemical burns, 335
facial, 103, 104f displaced tooth, 334
illustrated, 104f foreign bodies in clinician’s eye, 335
occipital, 103, 104f foreign body injuries to patient, 334
retroauricular, 103, 104f heat burns, 335
retropharyngeal, 103 postsurgical hemorrhage, 334
superficial parotid, 103 reasons for, 334
Lymphatic ducts, 49f wounds to clinician, 335
Lymphatic system common office emergencies, 323
about, 46 hyperventilation, 324
clinical study for, 50–51 orthostatic hypotension, 324
illustrated, 49f syncope, 323
lymph nodes of, 46 Trendelenburg position, 323, 324f
lymphatic vessels of emergency kit
components of, 46–47 AED for, 323
ducts of, 47 contents of, 322
purpose of, 46 drugs for, 322
vessels of, 46–47 oxygen for, 323
organs of, 46f pulse oximeter for, 323
and spleen, 46 foreign body airway obstruction (FBAO), 324
and thymus, 46 Heimlich maneuver, 324–325
and tonsils, 46 health history, 320
vessels of, 46 hyperglycemia (high blood sugar), 332
Lymphocytes, 44, 45t, 254–255 hypoglycemia (low blood sugar), 332
office preparation for, 322
M prevention of, 320–341
M line, 59 seizures
Malignant melanoma, 179 causes of, 331
Mammary glands, 53 grand mal, 331
Mandible. See under Facial bones hysterical, 331
Mandibular arch, 9, 9f petit mal, 331
Mandibular processes, 9f status epilepticus, 331
Marasmus, 210 treatment for, 331
Mast cell, 45t clinical studies for, 325b, 327–333, 330b, 333b, 336t
Masticatory mucosa, 18t transient ischemic attack (TIA), 330
Matrices, 523 vital signs, 320
Maturation stage, of tooth formation, 13f blood pressure, 320
Maxilla. See under Facial bones pulse rate frequency, 321
Index    605

Medical and dental emergencies (Continued) Microbiology (Continued)


pupil size, 322 about, 247
respiration rate, 321 acute atrophic candidiasis, 247
temperature reading, 321 angular cheilitis, 247
Medical history. See under Clinical treatment candidiasis, 247
Melanin, 33 chronic hyperplastic candidiasis, 247
Melanocytes, 33 epidemiology of, 247
Melanocytic nevus, 179 histoplasmosis, 247
Melatonin systemic candidiasis, 247
in endocrine system, 60 thrush, 247
Menstrual cycle changes, 62 gram-negative wall macromolecules
Mental foramen, 85f, 88f lipopolysaccharides, 237
Metabolism. See under Biomolecules lipoproteins, 238
Microanatomy, 33 porins, 238
Microbiology. See also Infectious diseases gram-positive wall macromolecules
about, 233 lipoteichoic acids, 237
bacterial growth, nutrition, 238 teichoic acid, 237
autotrophs, 238 metabolism of
bacterial genetics, 239 about, 233
chemolithotrophs, 239 aerobes vs. anaerobes, 235
chemoorganotrophs, 239 anabolic reactions, 234
chemotrophs, 239 anaerobic fermentation, 235
generation time, 239 anaerobic respiration, 235
heterotrophs, 239 biosynthesis (anabolism), 235
hydrogen ion concentration (pH) for, 239 catabolic reactions, 234
media for, 238 energy production (catabolism), 234
microbial growth relationships, 239 exoenzymes, 233
microbial interactions, 239t fermentation, 234
nutritional growth requirements, 238 oxidation-reduction reactions, 234
oxygen requirements for, 239 respiration, 234
phases of growth, 238 transport of molecules for, 233
photolithotrophs, 238 microbial observation, 233
photoorganotrophs, 238 bright field microscopy, 235
phototrophs, 238 dark field microscopy, 235
saprophytes, 238 electron microscopy (EM), 235
bacterial structure and function, 236 fluorescence microscopy, 235
cellular shapes macroscopic, 235
bacillus, 236 microscopic, 235
coccus, 236 phase-contrast microscopy, 235
palisade arrangement, 236 molds and yeasts (fungi)
pleomorphic, 236 about, 247
spirillum, 236 molds (mycelium), 247
cellular ultrastructure yeasts, 247
capsule, 237 mycobacterial cell walls, 238
cell wall, 237 oral microorganisms, healthy, 244
chromosomal DNA, 236 parasites
cytoplasmic membrane, 236 about, 248
endospore, 237 ciliata (ciliates), 249
flagella, 237 helminths, 249
glycocalyx, 237 mastigophora (flagellates), 248
inclusion bodies, 236 protozoa, 248
pili and fimbriae, 237 sarcodina, 249
plasmids, 236 sporozoa, 249
ribosomes, 236 parasitic infections
slime layer, 237 Giardia lamblia, 249
chemical differences Toxoplasma gondii, 249
amino acid chains, 237 passive immunization, 256
gram-negative bacteria, 237 prokaryotes (bacteria), 233
gram-positive bacteria, 237 archaeobacteria, 233
peptidoglycan, 237 eubacteria, 233
classifications for, 233 mycoplasmas, 233
eukaryotes rickettsias, 233
algae, 233 sexual, fluid transmission
fungi, 233 about, 250
protozoa and helminths, 233 AIDS/HIV, 252, 252f
fungal infections cytomegalovirus (CMV), 253
606    Index

Microbiology (Continued) Muscles (Continued)


gonorrhea, 250 platysma, 89
hepatitis B (HBV), 251 risorius, 88
hepatitis C (HCV), 252 zygomaticus major, 89
herpes simplex virus (HSV), 251, 251f hyoid
human papillomaviruses (HPVs), 251 digastric, belly of, 91f
syphilis, 250, 251f infrahyoids, 91
specimen preparation, 236 location, action of, 90
acid-fast stain, 236 mylohyoid, 90, 91f
differential staining, 236 omohyoid, belly of, 91f
Gram stain, 236 sternohyoid, 91f
simple staining, 236 sternothyroid, 91f
clinical studies on, 243t, 248t, 250, 254t, 257t, 266–268 stylohyoid, 91f
vaccination, 255 suprahyoid, 90
vector-borne infections thyrohyoid, 91f
about, 249 of mastication
hepatitis A (HAV), 249 illustrated, 90f
malaria, 249 lateral pterygoid, 90
viral infections location and action of, 89
about, 245 masseter, 89, 90f
infectious mononucleosis, 246 medial pterygoid, 89
influenza (flu), 245 temporalis, 89, 90f
mumps, 246 and muscle fibers, 35, 37f
poliovirus, 246 tissues of, 17
rubella (German measles), 246 Muscular system. See also Muscles; Skeletal muscle physiology
rubeola (measles), 246 appendicular muscles, 35
varicella (chicken pox), herpes zoster (shingles), 247 attachment, actions of, 35
viruses, 233 origins and insertions of, 35
about, 244 axial muscles, 35
antiviral therapy, 245 functions of, 35
classification of, 245 illustrated, 37–38f
composition, morphology, 244 skeletal muscle
detection by culture of, 245 muscle cells, 35
pathogenesis of, 245 muscles and muscle fibers, 35
replication requirements for, 245 Myosin filaments, 59
Microdontia, 196
Microscopy, 235–236 N
Mineralocorticoids, 62 Nares, 80
Minerals. See under Nutrition Nasal (olfactory) pits, 9
Mirrors. See under Instrumentation Nasion, 80
Molars. See under Permanent teeth; Primary teeth Nasolabial cyst, 195
Monocyte/macrophage, 45t Nasolabial sulcus, 80
Monosaccharides, 55. See also under Nutrition Nasopalatine cyst, 195
Motility, 69 National Board Dental Hygiene Examination
Motor nerves, 36–37 about, 1
Mouth protectors, 375 attitude toward, 4
Mucocele, 192 becoming familiar with, 3
Mucoepidermoid carcinoma, 179 being rested for, 5
Multiple myeloma, 175 as career step, 4
Muscles. See also Muscular system; Skeletal muscle physiology and nutrition, 3–4
cervical organization of, 1
location of, 88 preparing for, 2–5
sternocleidomastoid muscle (SCM), 88 being rested for, 3
trapezius, 88 depth of study for, 2
of facial expression familiarization with, 3
buccinator, 88 group studying, 3
corrugator supercilii, 88 individual studying, 3
depressor labii inferioris, 89 nutrition for, 3–4
epicranius, 88 planning study time for, 3
formation of, 9 and procrastination, 3
illustrated, 89f scheduling study time for, 3
levator anguli, 89 self-confidence, attitude for, 4
levator labii, 89 study time for, 2–3
location, action of, 121 and time expediency, 4
mentalis, 89 question types on
orbicularis oculi, 88 case-based, 5
Index    607

National Board Dental Hygiene Examination (Continued) Neoplasia (Continued)


cause-and-effect, 4 types of, 180
completion, 4 vascular
critical thinking for, 4 hemangioma, 177
formats used for, 4 Kaposi’s sarcoma, 178
negative, 4 Nephrons, 53, 54f. See also Renal physiology
paired true-false, 4 Nervous system
testlet, 4 central nervous system (CNS), 37
review courses for, 2 brain, 37
simulated, 2 brainstem, 37
as state requirement, 1 cerebellum, 37
strategies for multiple choice, 5 cerebral cortex, 37
NBDHE. See National Board Dental Hygiene Examination cerebrum, 37
Neck anatomy. See Head and neck anatomy diencephalon, 37
Necrotizing sialometaplasia, 193 hemispheres, of brain, 37
Neoplasia hypothalamus, 37
barriers to oral care, 182 spinal cord, 37
benign, 177 thalamus, 37
adenoma, 177 components of, 36–37
angioma (birthmark), 177 motor nerves in, 36–37
lipoma, 177 peripheral nervous system (PNS), 37
biopsy procedures, 180 afferent, efferent divisions of, 37
biopsy, defined, 180 components of, 37
breast cancer, 179 parasympathetic division of, 39
cancer patient care, 181–183 somatic, autonomic systems of, 37, 39
about, 181 spinal nerves of, 37
radiation therapy for, 181 sympathetic division, 39
surgery, 181 sense organs. See Sense organs
care for oral and pharyngeal cancer sensory nerves in, 36–37
chemotherapy for, 181 tissues of, 17
care for systemic cancer Neurological disorders. See under Pathology, general and oral
chemotherapy for, 181–182 Neuroma, 193
complications, 182 Neurotransmitters, 58
clinical studies for, 180t, 181, 183t Neutropenia, 175
dental hygiene care Neutrophil/polymorphonuclear leukocyte, 45t
after treatment, 183 Nitrous oxide. See under Pain management
before treatment, 182 NK cells, 44
during treatment, 182 Nociceptors, 41
epithelial squamous cell Nonodontogenic cysts, 195
basal cell carcinoma (BCC), 179 Nose, 9, 50. See also under Head and neck anatomy
malignant melanoma, 179 Nucleic acids. See under Biomolecules
melanocytic nevus, 179 Nutrition. See also Carbohydrates; Nutritional counseling; Obesity;
papilloma, 178 under Periodontology
squamous cell carcinoma (SCC), 178, 178f Acceptable Macronutrient Distribution Ranges (AMDRs), 209, 210t
etiology of, 177 calorie and kilocalorie (kcal), 209
exfoliative cytology, 179 clinical studies for, 219t, 221t
indications for, 180 dental caries
malignant, 177 biofilm, 213
muscle and fermentable carbohydrate ingestion, 213
rhabdomyosarcoma, 178 food intake frequency, 213
neoplasm, 177 salivary flow rate, 213
odontogenic diabetes mellitus (DM), 213
ameloblastoma, 178, 178f insulin-dependent, 213
oral mucositis (OM), 182 non-insulin-dependent, 213
osseous Dietary Reference Intakes (DRIs), 209
osteoma, 178 disease management
osteosarcoma, 178 lactose intolerance, 213
osteoradionecrosis (ORN), 182 reactive hypoglycemia, 213
prostate cancer, 179 eating disorders
salivary gland about, 223
adenoid cystic carcinoma, 179 anorexia nervosa, 223
mucoepidermoid carcinoma, 179 bulimia nervosa, 223, 223f
pleomorphic adenoma, 179 bulimorexia, 224
soft tissue, 178 compulsive overeating, 224
fibroma, 178 diabulimia, 224
lipoma, 178 lipids (fats), 214–216
608    Index

Nutrition (Continued) Nutrition (Continued)


about, 214–216 complete proteins, 209
and atherosclerosis management, 215 dietary requirements for, 210
cholesterol (sterol), 215 functions of, 209
compound lipids, 214 incomplete proteins, 209
dietary requirements for, 215 nutrient sources of, 210
essential fatty acid, 214 role in oral health, 209
functions of, 215 role in periodontal health, 210
gums, 215 and testing, 3–4
high-density lipoproteins (HDL), 215 undernutrition, 209
hydrogenation, 215 vitamins
lipoproteins, 214 about, 216–219
low-density lipoproteins (LDL), 214 biotin, 218
monounsaturated fatty acid, 214 cobalamin (B12), 218
nutrient sources of, 215 fat-soluble, 216
and obesity management, 216 folate (folic acid), 218
Olestra, 215 niacin (B3), 217
phospholipids, 214 pantothenic acid (B5), 217
polyunsaturated fatty acid, 214 pyridoxine (B6), 217
saturated fatty acid, 214 repair process, 222
Simplesse, 215 riboflavin (B2), 217
true fats, 214 thiamin (B1), 217
very low-density lipoproteins, 214 vitamin A (retinol), 216
macronutrients, 209 vitamin C (ascorbic acid), 218, 218f
malnutrition, 209 vitamin D (calciferol), 216
minerals, 219–222 vitamin E (tocopherol), 216
about, 219–222 vitamin K, 216
calcium, 219 water-soluble vitamins, 217
chloride, 220 water, 222
chromium, 221 weight control, energy
cobalt, 221 about, 222–223
copper, 221 basal metabolic rate (BMR), 222
fluorine, 221 physical activity, 222
iodine, 221 specific dynamic activity (SDA), 223
iron, 221 Nutritional counseling
lead poisoning, 221 about, 225
macrominerals, 219 community health programs in, 550
magnesium, 220 diet counseling, 225
manganese, 221 diet management principles
phosphorus, 220 follow-up, 225
potassium, 220 food diary, 225
sodium, 220 interview, 225
sulfur, 221 nutritional management
trace elements, 221 after whitening, 226
zinc, 221 of chronic periodontitis, 226
nutrient effect orally, 222 of dental caries, 225
about, 222 of necrotizing periodontal disease, 226
immune mechanisms, 222
periodontal and dental health, 222 O
repair process, 222 Obesity
nutrients, 209 about, 224
overnutrition, 209 etiology of, 224
protein deficiencies and diseases prevention, weight control
adult PEM, 211 behavior modification for, 224
cachexia, 211 factors in, 224
kwashiorkor, 210–211 risk for CVD, 225
marasmus, 210 Obturator, for clefts, 11
overconsumption of protein, 211 Occlusion. See also under Periodontology
phenylketonuria (PKU), 210 about, 121–124
poor vegan diet, 211 centric occlusion (CO), 121
protein-energy malnutrition (PEM), 210 mandibular rest position, 121
starvation, 211 occlusal evaluation
proteins, 209–211 about, 121
about, 209–211 Angle’s classification, 122, 123t
amino acids, 209 anterior crossbite, 122
complementary proteins, 209 and attrition from bruxism, 123
Index    609

Occlusion (Continued)
centric occlusion, 123 P
centric prematurity, 124 Paget’s disease, 177
centric relation, 124 Pain management. See also Anesthesia, local
centric stops, 124 anxiety
common misalignments, 122f defined, 470–471
contact areas, 124 etiology of, 471
distal step, 123 and fear, 470–471
edge-to-edge bite, 122 and phobia, 470–471
labioversion, 122 treatment of, 471
lateral excursion, 124 clinical studies for, 471, 481, 482t, 487t, 489t, 494t
malocclusions, 121 with nitrous oxide
malposed dentition factors, 122 about, 490
for mixed dentition, 123 abuse of, 495
open bite, 122 administering, 491
overbite, 122 complications of using, 493
overjet, 122 contraindications for use of, 491
for permanent dentition, 123 excessive sweating with, 494
posterior crossbite, 122 nausea with, 493
for primary dentition, 122 pharmacology of, 491
primate spaces, 122 postprocedural steps for using, 492
protrusive excursion, 124 preprocedural steps for using, 492
terminal plane, 123 procedural steps for using, 492
occlusal plane signs and symptoms of use, 492, 493t
curve of Spee, 121 using ASA classifications with, 491
curve of Wilson, 121 using nitrous oxide–oxygen unit, 491
Oculomotor (third cranial) nerve, 39, 40f vomiting with, 493
Odontoblasts, 13f, 14, 22 pain
Odontogenic cysts, 195 control of, in dental office, 470
Odontogenic keratocyst, 195 defined, 470
Olfactory (first cranial) nerve, 39, 40f gate control theory of, 470
Oligosaccharides, 55 perception of, 470
Optic (second cranial) nerve, 39, 40f reaction to, 470
Oral cancer. See under Community oral health responses to, 470
Oral cavity. See also under Pathology, general and oral threshold of, 470
about, 80 types of, 470
development of, 8 and sensory innervation
developmental disorders of. See under Pathology, general and oral myelinated nerves, 471
floor of mouth, 81, 82f nerve bundle structure, 472
functions of, 69 nerve fiber structure, 471
illustrated, 10f, 52f nerve physiology, 472
labeling system for, 80 neuron structure, 472f
maxillary and mandibular jaws, 80 sensory, motor nerves of, 471
mucous membranes of, 80 Pain (nociceptors), 41
palate, 81 Palate
pharynx, 81 components of, 81
tongue, 81 development of, 10f, 10–11
variations in, 199–200 illustrated, 86f
Oral irrigators, 370 Pancreas, 44, 52f. See also under Gastrointestinal tract
Oral mucosa. See also under Histology physiology
layers of, 17f Pancreatitis, 173
lining, 18t Papilla(e), lingual, 19t
masticatory, 18t Papilloma, 178
regional differences in, 18t Papillon-Lefèvre syndrome, 198
types of, 17–19 Parathyroid gland disorders, 171
Oral mucositis (OM), 182 Parathyroid hormone, 61
Organohalogens, 54 Pathology, general and oral. See also Diabetes; Neoplasia;
Oropharyngeal membrane, 9f Temporomandibular joint (TMJ)
Osmosis, 58 blood disorders
Osteogenesis imperfecta, 199 about, 174
Osteoma, 178 agranulocytosis, 175
Osteoporosis, 176. See also under Pathology, general and oral anemias, 174
Osteoradionecrosis (ORN), 182 aplastic anemia, 174
Osteosarcoma, 178 Burkitt’s lymphoma, 175
Ovaries, 53 clinical study for, 175–180
Oviducts, 53 hemolytic anemia, 174
610    Index

Pathology, general and oral (Continued) Pathology, general and oral (Continued)
hypochromic anemia, 174 aura, 187
Langerhans’ cell histiocytosis X, 175 barriers to care for, 188
leukemia, 175, 176t education for, 188
leukopenia, 175 etiology of, 187
multiple myeloma, 175, 176f grand mal seizures in, 187
neutropenia, 175 oral manifestations of, 188
pernicious anemia, 174 petit mal seizures in, 187
polycythemia, 174 professional care and homecare, 188
secondary anemia, 174 risk factors for, 188
sickle-cell anemia, 174 status epilepticus in, 187–188
thrombocytopenia, 174 treatments for, 187
bone diseases gastrointestinal disease
about, 176 about, 186–187
fibrous dysplasia, 177 gastroesophageal reflux disease (GERD), 187
osteoporosis, 176 peptic ulcer disease (PUD), 187
Paget’s disease, 177 immunological disorders
clinical studies for, 173, 175, 180t, 181–183, 183t, about, 188
197–200 benign mucous membrane pemphigoid (BMMP), 190
defined, 168 erythema multiforme (EM), 187, 189
developmental disorders, oral cavity Hodgkin’s lymphoma, 189
about, 195 hypersensitivity (allergy), 188–189
aneurysmal bone cyst, 196 lichen planus, 189, 189f
anodontia, 196 pemphigus vulgaris, 190
concrescence, 197 scleroderma, 189
dens-in-dente, 196 Sjögren’s syndrome, 190
dental abnormalities, 196 systemic lupus erythematosus (SLE), 189, 189f
dental fluorosis, 196 inherited disorders
dentigerous (follicular) cyst, 195 about, 198
dilaceration, 197 amelogenesis imperfecta, 199
enamel hypocalcification, 197 cherubism, 198
enamel hypoplasia, 196 cleidocranial dysostosis, 198
enamel pearl, 196 cyclic neutropenia, 198
epidermoid and dermoid cysts, 196 dentin dysplasia, 199
eruption cyst, 194 dentinogenesis imperfecta, 199
fusion, 196 ectodermal dysplasia, 199
gemination, 196 hemophilia, 198
globulomaxillary cyst, 195, 195f hereditary fibromatosis, 198
hypodontia, 196 osseous, 197
lateral periodontal cyst, 195 osteogenesis imperfecta, 199
median palatine cyst, 195 Papillon-Lefèvre syndrome, 198
microdontia, 196 von Willebrand disease (VWD), 198
nasolabial cyst, 195 injury, inflammation, repair
nasopalatine cyst, 195 about, 168–173
nonodontogenic cysts, 195 abscess, 169
odontogenic cysts, 195 acne, 169
odontogenic keratocyst, 195 adrenal gland disorders, 171
primordial cyst, 195 bacterial infections, 169
simple bone cyst, 196 causes of, 168
static bone cyst, 196 cirrhosis, 171
supernumerary teeth, 196 clinical assessment for, 169
taurodontism, 197 dental hygiene diagnosis for, 169
diabetes mellitus. See Diabetes diagnostic process for, 168–173
diseases of cardiovascular system endocrine gland disorders, 169
about, 184 Graves’ disease, 170
barriers to care for, 184 Hashimoto’s thyroiditis, 170
cardiac dysrhythmia, 184 hepatitis A (HAV), 171–200
congenital heart disease, 184 hepatitis B (HB), 168
congestive heart failure, 184 hepatitis C, 168
coronary artery atherosclerosis, 184 hepatitis D, 168, 171
education for, 185 hepatitis E, 171
hypertension, 184 hyperadrenalism, 171
oral manifestations of, 184 hyperpituitarism, 169–170
professional care and homecare, 184 hypersecretion, 169
risk factors for, 184 hyperthyroidism, 170
epilepsy hypoadrenalism, 171
Index    611

Pathology, general and oral (Continued) Pathology, general and oral (Continued)
hypopituitarism, 170 professional care and homecare, 186
hyposecretion, 169 risk factors for, 186
hypothyroidism, 170 respiratory diseases
impetigo, 169 about, 185
and infectious diseases, 169 asthma, 185
and inflammatory diseases, 169 barriers to care for, 186
inflammatory response to, 168 bronchitis, 185
jaundice, 171 chronic irreversible airway obstruction, 185
lab assessment for, 169 cystic fibrosis (CF), 186
latent period of, 169 education for, 186
liver disorders, 171 emphysema, 185
metabolic disorders, 169 lung cancer, 185
palliative care for, 169 oral manifestations of, 186
pancreatic cancer, 173 professional care and homecare, 186
pancreatic disorders, 172 risk factors for, 186
pancreatitis, 173 traumatic oral lesions, 191–195
parathyroid gland disorders, 171 abfraction, 191
patient history for, 169 about, 191
pituitary disorders, 169 abrasion, 191
prognosis, treatment, care for, 169 amalgam tattoo, 194
response to, 168 attrition, 191
surgical assessment for, 169 blood-related lesions, 193
thyroid gland disorders, 170, 170f burns, 191
viral hepatitis, 171 from crack cocaine smoking, 192
viral infections, 169 ecchymosis (bruise), 193
neurological disorders erosion, 191
about, 187 hematoma, 193
Bell’s palsy, 187 mucocele, 192
glaucoma, 188 necrotizing sialometaplasia, 193
trigeminal neuralgia, 187 neuroma, 193
oral connective tissue hyperplasia petechiae, 193
denture-induced hyperplasia, 192 ranula, 192
frictional keratosis, 193 salivary gland duct obstruction, 192
gingival hyperplasia, 192 temporomandibular joint disorder (TMD). See Temporomandibular
irritation fibroma, 193, 193f joint disorder (TMD)
pyogenic granuloma, 192 tobacco-related lesions, 192
oral ulcerations Pemphigus vulgaris, 190
aphthous ulcers (canker sores), 193 Peptides, 56, 60
oral variations Periodontal disease See Periodontology; under Community oral health;
about, 199 under Nutrition; under Radiation
black hairy tongue, 200 Periodontal ligament (PDL)
fissured tongue, 199 alveolodental fibers of, 25, 25t
Fordyce granules, 199 components of, 25
geographic tongue, 199 epithelial rests of Malassez, 25
leukoedema, 199 fiber groups of, 25, 25t, 26f
linea alba, 199 fibroblasts of, 25
lingual thyroid nodule, 199 illustrated, 20f, 24f
lingual varicosities, 200 osteoclasts and odontoclasts in, 25
pigmentation, 199 purpose and origin of, 25
tori, 199 Sharpey’s fibers of, 25
periapical inflammation and pathology Periodontal probes. See under Instrumentation
about, 194 Periodontal probing, 21
alveolar cystitis (dry socket), 194 Periodontium. See also Periodontal ligament (PDL); Periodontology;
cemental dysplasia (cementoma), 194 under Pathology, general and oral
chronic hyperplastic pulpitis (pulp polyp), 194 alveolar bone of, 23–24
condensing osteitis (focal sclerosing osteomyelitis), 194 alveolar bone proper of, 24
endodontic abscess, 194 defined, 23
granuloma, 194 illustrated, 24f
Ludwig’s angina, 194 interdental septum in, 24
radicular cyst, 194 interradicular septum in, 24
renal disease lamina dura in, 24
about, 186 lamina propria in, 24
barriers to care for, 186 makeup of, 23
education for, 186 periodontal ligament (PDL) of, 25
oral manifestations of, 186 structures of, 23–26
612    Index

Periodontium (Continued) Periodontology (Continued)


Volkmann’s canals in, 24 dental hygiene plan of care
Periodontology. See also Implants; under Clinical treatment case type I (gingival disease), 450
abscesses of periodontium case type II (early periodontitis), 450
about, 446 case type III (moderate periodontitis), 450
gingival, 446 case type IV (advanced periodontitis), 450
pericoronal (pericoronitis), 446 case type V (severe periodontitis), 451
periodontal, 446 goals of plan, 450
adjunctive therapies in timing of, 450
antimicrobial rinses, 451 developmental, acquired deformities, 446
controlled-release gels, 452 endocrine disorder risk factors
desensitizing agents, 452 causes, treatment for, 426
locally delivered antibiotics, 452 diabetes mellitus, 426
subgingival irrigation, 452 hormone alterations, 426
systemic antibiotics, 452 hyperparathyroidism, 426
usefulness of, 451 hypothyroidism, 426
adverse drug reaction risk endodontic lesions, and periodontitis, 446
about, 428 etiology of periodontal disease
gingival hyperplasia, 428, 429f host response, 430
aggressive periodontitis microbiology and immunology, 433
about, 442 occlusal trauma, 435
characteristics of, 442 oral contributing factors, 434
generalized (GAP), 442 gingival diseases
localized (LAP), 442, 442f bacterial, viral, and fungal origin, 439, 439f
management of, 442 biofilm-induced, 438
severity of, 442 genetic origin, 439
therapy for, 443 hormone-induced, 438
bacteria-mediated tissue destruction leukemia-induced, 439
by bacterial enzymes, 434 medication-induced, 439
by lipopolysaccharides, 434 modified by medication, 440
by lymphotoxins, 434 non-biofilm-induced, 439
chronic periodontitis systemically induced, 439
about, 440 histological pathogenesis
characteristics of, 440 advanced lesion, 436
and clinical attachment level, 440, 441f early lesion, 436
clinical signs of, 440 established lesion, 436
localized vs. generalized, 441 initial lesion, 436
moderate, 441 HIV/AIDS status risk
severe, 441 AIDS-associated necrotizing ulcerative periodontitis, 429
slight, 441 as first sign, 429
classification of disease HIV-associated gingivitis, 429
about, 437–438, 438t HIV-associated periodontitis, 429, 429f
clinical assessment for immune response
about, 446–447 complement system, 432
clinical pathogenesis defined, 432
gingivitis, 437 nonspecific inflammatory response, 432
infrabony pocket, 437, 437f phagocytic system, 433
periodontitis, 437 specific host responses, 433
suprabony pocket, 437 inflammation, clinical signs of
clinical studies for, 427t, 429t, 441t, 443t, 445t bleeding on probing (BoP), 432
defense mechanisms, oral cavity gingival color changes, 432
gingival crevicular fluid, 431 gingival consistency changes, 432
intact epithelium, 430 gingival position changes, 432
saliva, 431 surface texture changes, 432
dental biofilm inflammatory response
about, 433 acute phase, 431
pathogens of, 433t cellular response, 431
subgingival, 434 chronic phase, 431
supragingival, 433 repair phase, 432
dental hygiene diagnosis for necrotizing periodontal diseases
categories for, 449 about, 444
defined, 449 necrotizing ulcerative gingivitis (NUG)
developing, 449 about, 444
establishing differential diagnosis, 449 characteristics of, 444
final, 449 management of, 444
result of, 449 necrotizing ulcerative periodontitis (NUP)
Index    613

Periodontology (Continued) Periodontology (Continued)


about, 444 effectiveness of, 456
associated with HIV/AIDS, 445 enhancing compliance to, 456
follow-up, prognosis for, 445 hygienist’s role in, 455
illustrated, 444f objectives of, 455
management of, 445 rationale for, 455
patient counseling for, 445 recare appointment for, 456
neutrophil abnormality risk recurrence of disease, 457
and biofilm, 430 success of, 455
chemotaxis defects, 430 periodontium
neutropenias, 430 defined, 425
PMN dysfunctions, 430 plan of action for disease
nonsurgical periodontal therapy (NSPT) determining, 450
defined, 451 goals for, 450
effectiveness of, 451 phase I initial therapy, 450
goals of, 451 phase II surgical therapy, 450
procedures, techniques of, 451 phase III restorative procedures, 450
rationales for, 451 phase IV maintenance therapy, 450
nutritional deficiency risk preliminary phase of, 450
antioxidants, 428 recare intervals for, 450
osteoporosis, 428 postsurgical procedures
populations at risk for, 427–428 periodontal dressing (pack), 455
protein deficiency, 428 postoperative instructions, 455
vitamin A, 428 sutures, 454
vitamin B12, 428 prognosis of disease
vitamin B2 (riboflavin), 428 defined, 449
vitamin B6, 428 for gingivitis, 450
vitamin C (ascorbic acid), 428 for individual teeth, 450
vitamin D, 428 for periodontitis, 450
and nutritional management, 226 radiographic assessment
occlusal trauma about, 448
clinical signs, symptoms of, 436 bone loss assessment, 448, 448f
disuse atrophy, 436 limitations of, 448
hyperfunction, 436 overhanging margin assessment, 448
hypofunction, 436 PDL assessment, 448
injury, 436 stress, psychosocial risk, 428
occlusal forces, 436 supplemental risk factor methods
primary, 436 biochemical assessments, 449
secondary, 436 immunological assessment tests, 449
oral contributing factors importance of, 449
dental stains, 435 microbiological tests, 449
faulty dentistry, 435 physical assessment tests, 449
food impaction, 435 software programs for, 449
local functional, 434–435 surgical periodontal therapy
local predisposing, 435 and bone loss, 453
materia alba, 435 considerations for, 452
pathogenesis of goals of, 452
and gingivitis, 436 hygienists’ understanding of, 452
periodontal assessment and patient health, 453
about, 447 and patient preference, 453
bleeding on probing (BoP), 447 presurgical drug considerations, 453
furcation involvement, 448 rationale for, 452
inflammation, 444 surgical procedures
malocclusion and occlusal pathology, 448 bone grafts, 454
McCall’s festoons, 447 crown lengthening, 453
periodontal probing, 447 effectiveness of, 454
presence of biofilm, 447 gingivectomy, 453
proximal contacts, 448 guided tissue regeneration (GTR), 454
restorations and appliances, 448 healing afterward, 454
Stillman’s clefts, 447 hygienist’s role in, 454
tissue destruction, 447 ostectomy, 453
tooth mobility, 448 osteoplasty, 453
periodontal diseases periodontal flap procedures, 453
described, 425 soft tissue grafts, 454
periodontal maintenance systemic disease, and periodontitis
compliance as key to, 456 about, 444
614    Index

Periodontology (Continued) Personal protective equipment (PPE) (Continued)


Chédiak-Higashi syndrome, 444 masks, 264
diseases affecting, 444 removal of, 264
hypophosphatasia, 444 Petechiae, 193
Papillon-Lefèvre syndrome, 444 Pharmacology
tobacco use risk factors administration types, 278
importance of, 425 adrenergic blockers, 281
smokeless tobacco users, 426 adrenergics, 280–281
smokers, 425 for alcoholic patients, 312–313
Peripheral nervous system (PNS). See under Nervous system for allergic patients
Permanent teeth. See also under Clinical treatment first-generation H1-receptor antagonists, 306
about, 12, 108 leukotriene-receptor antagonist, 307
anteriors rationale for, 306
about, 109 second-generation H1-receptor antagonists, 307
crown, 109 analgesics
lingual, 109 about, 284
proximal, 109 combinations and uses of, 285t
root, 109 drug interactions with, 284t
canines (cuspids) antianxiety drugs
about, 112 about, 282
illustrated, 112f barbiturates, 283
mandibular, 112 benzodiazepines (BZDs), 282
maxillary, 112 chloral hydrate, 283
eruption of, 14 meprobamate, 283
incisors zolpidem (Ambien), 283
about, 109 antibiotics
illustrated, 110f actions of, 291
mandibular, 111 aminoglycosides, 293
mandibular central, 111 cephalosporins, 292
mandibular lateral, 111 clindamycin, 293
maxillary, 109 macrolides, 293
maxillary central, 110 metronidazole, 293
maxillary lateral, 110 penicillins, 291
molars for prophylactic premedication, 294–313, 292b, 295b, 296t
about, 114–118 quinolones, 294
crowns, 114 sulfonamides, 293
features of, 117f tetracyclines, 292
mandibular, 117, 118f anticholinergics, 282
mandibular first, 117 antifungals
mandibular second, 117 fluconazole, 291
mandibular third, 117 ketoconazole, 290
maxillary, 115, 118f nystatin, 290
maxillary first, 116 antihypertensives
maxillary second, 116 adrenergic blockers, 297
maxillary third, 116 angiotensin II receptor blockers, 298
occlusal table, 114 angiotensin-converting enzyme (ACE) inhibitors, 297
roots, 114 antianginals, 299
posteriors anticoagulants, 300
crowns, 112 antihyperlipidemics, 299
occlusal table, 113, 116f antiplatelets, 300
proximal, 113 antiarrhythmics, 298
premolars (bicuspids) antithrombotics, 299–300
about, 113–114 calcium channel blockers, 298
illustrated, 113f digitalis glycoside, 298
mandibular, 114 diuretics, 296
mandibular first, 114 erectile dysfunction drugs, 300
mandibular second, 114 ezetimibe, 299
maxillary, 113 fibrates, 299
maxillary first, 114 general uses, effects, 296
maxillary second, 114 gingival effects of, 298t
occlusal table, 115f statins, 299
Personal protective equipment (PPE) vasodilators, 298
clothing, 265 antituberculins
facial, 265 actions of, 294
gloves, 264, 264f ethambutol, 294
in the lab, 524 isoniazid, 294
Index    615

Pharmacology (Continued) Pharmacology (Continued)


pyrazinamide (PZA), 294 opioid antagonists, 287
rifampin, 294 for osteoporotic patients, 306
rifapentine, 294 patients with CVD
antivirals rationale for, 296
actions of, 290 pharmacokinetics, 278
AIDS antivirals, 290 Physician’s Desk Reference (PDR), 278
amantadine, 290 for pregnant patients, 308, 309t
antiherpetic, 290 prescription writing
foscarnet, 290 effective, 279
interferons, 290 form and contents, 279, 279t, 280t
anxiety, pain management, 282–290 laws affecting, 280t
applied to patient, 278 reactions to drugs
autonomic nervous system effects agonist, 278
about, 280 allergic reactions, 279
parasympathetic division of, 280 antagonist, 278
sympathetic division of, 280 contraindications, 279
cholinergics, 281–282 excretion, 279
clinical studies for, 283b, 295b, 296b, 298b, 300b pharmacological effect, 278
dosage definitions potency, 278
effective, 278 side effect, 278
lethal, 278 synergism, 278
maximum recommended dose (MRD), 278 therapeutic effect, 278
therapeutic index (TI), 278 tolerance for, 278
for endocrine disorders toxicology, 279
adrenocorticosteroids, 303 for respiratory disease
antithyroid drugs, 305 anticholinergics, 308
for diabetes mellitus, 304 corticosteroid inhalers, 308
estrogens, 305 cromolyn, 308
female sex hormones, 305 methylxanthine, 308
for thyroid gland replacement, 304 rationale for, 308
for gastrointestinal tract disorders sympathomimetic agonists, 308
antidiarrheals, 307 for seizure disorders
antiemetics, 307 benzodiazepines (BZDs), 303
emetics, 307 carbamazepine, 303
H2-receptor antagonists, 307 phenobarbital, 303
laxatives, 307 phenytoin, 303
proton pump inhibitors, 307 rationale for, 278
rationale for, 307 valproic acid, 303
general anesthesia semisynthetic opioids, 287
benzodiazepines (BZDs), 289 for substance abuse
halogenated ether, 289 amphetamines, 311
halogenated hydrocarbon, 289 barriers to care, 310
Innovar, 290 CNS depressants, 311
ketamine, 289 cocaine, 311
nitrous oxide inhalation gas, 288 in dental offices, 310
opioids, 290 education for, 310
ultrashort-acting IV barbiturates, 289 nicotine, 311
indications of drug use, 278 oral manifestations, 310, 312
local anesthesia professional care and homecare, 310
about, 287 psychedelics, 281
injected, 287 risk factors, 310
lidocaine transoral, 287 vasoconstrictor anesthesia, 288
periodontal gel, 287 Pharynx, 51
for mentally disabled functions of, 69, 81
antipsychotics, 301 illustrated, 51–52f
first-generation antidepressants, 301 in respiratory system, 50
other antidepressants, 302 Phenylketonuria (PKU), 210
rationale for, 278 Physiology
second-generation antidepressants, 302 of blood pressure. See Blood pressure
nonopioid analgesics of cells. See Cell physiology
acetaminophen, 284t, 286 defined, 33, 57
efficacy comparison, 285t of endocrinology. See Endocrinology
NSAIAs (NSAIDs), 284t, 286 gastrointestinal. See Gastrointestinal tract physiology
salicylates, 284, 284t neurophysiology. See Neurophysiology
opioid analgesics, common, 286 renal. See Renal physiology
616    Index

Physiology (Continued) Pulp (Continued)


respiratory. See Respiratory physiology stones (denticles) in, 23
of skeletal muscle. See Skeletal muscle physiology vascular supply of, 23
Pigmentation, 199 Pyogenic granuloma, 192
Pituitary disorders, 169 Pyruvate dehydrogenase, 57
Pituitary glands, 44
Plasma, 44 Q
Plasma cell, 44, 45t Questions, on NBDHE
Platelets, 44 case-based type, 5
Pleomorphic adenoma, 179 cause-and-effect, 4
Plexus, 46 multiple choice, 1
Polycythemia, 174 negative-type, 4
Polysaccharides, 55. See also under Nutrition paired true-false, 4
PPE. See Personal Protective Equipment (PPE) question-type, 4
Prehypertension. See under Blood pressure review, 2
Premolars. See under Permanent teeth strategies for, 5
Prenatal development. See Embryology testlet-type, 4–5
Pressure, in blood vessels, 46
Primary teeth. See also under Clinical treatment R
about, 12, 118–119 Radial artery, 45–46
canines (cuspids) Radiation absorbed dose (RAD), 137
mandibular, 120 Radiation and radiography
maxillary, 120 about, 135
clinical study for, 119–121 anatomical landmarks for
crowns of, 119 about, 152–154
effects of premature loss, 14 anterior nasal spine, 152, 153f
features of, 119f illustrated, 153f
incisors incisive foramen, 152
mandibular central, 119 maxillary sinus, 152
maxillary central, 119 median palatal suture, 152, 153f
maxillary lateral, 119 mental foramen, 156f
molars nasal fossae, 152, 153f
crowns, 120 nasal septum, 152, 153f
mandibular first, 121 teeth, periodontium, 152
mandibular second, 121 anatomical structures, functions of
maxillary first, 120 coronoid process, 135, 154–155
maxillary second, 120 external oblique line (ridge), 154
occlusal table, 120 genial tubercles, 154
roots, 120 hamulus, 154, 155f
and odontoclast resorption, 14 illustrated, 155f
pulp cavity of, 119 internal oblique (mylohyoid) line (ridge), 154
roots of, 119 inverted Y-junction, 153f, 154
Primordial cyst, 195 lingual foramen, 154
Probes, periodontal. See under Instrumentation mandibular canal, 154
Probing, periodontal, 21 maxillary tuberosity, 154, 155f
Professional practice studies for ethics and jurisprudence, 577–582 mental foramen, 154
Progesterone, 62 mental ridge, 154
Prokaryotes (bacteria), 233 nutrient canals, 154
Proprioception, 41 submandibular fossa, 154
Prostate cancer, 179 zygomatic process, 153f, 154
Prosthetics. See Biomaterials, dentures atomic structure of
Proteins, 56, 60. See also under Nutrition illustrated, 136f
Provision of clinical dental hygiene services, 1 ionization, 135
Pterygomandibular fold, 81 nucleus, 135
Pulmonary arteries, 47f orbiting shells, 135
Pulmonary circulation, of CVS, 46, 47f clinical studies for, 141t, 147t, 151t, 160t
Pulmonary veins, 47–48f components of, 136f
Pulp. See also Endodontic therapy digital imaging
accessory canals in, 23 about, 140–141
anatomy of, 22 advantages of, 140
apical foramen of, 23 disadvantages of, 140
formation of, 22 principles of, 140
function of, 23 errors, intraoral technique
horns in, 23 about, 143–145
illustrated, 13f backward placement, 144, 145f
radicular, 23 bent film, 144, 144f
Index    617

Radiation and radiography (Continued) Radiation and radiography (Continued)


cone cut, 144, 144f equipment, supplies, 152
double exposure, 145, 145f exposure procedures, 152
elongation, 144, 144f interpretation
foreshortening, 143, 144f about, 154–161
movement, 144, 145f ameloblastoma, 160
overlap, 143, 143f avulsion, 159
examination by cementicle, 159
about, 142 cherubism, 160
bitewing (BW) view, 142 dental caries, 155
full-mouth (FMX) view, 143 enamel pearl, 159
intraoral views, 142–143 extraction, 159
occlusal view, 143 fibrous dysplasia, 160
periapical (PA) view, 142 gingivitis, 158
exposure and risk interproximal caries, 155, 157f
about, 139 malignancy, 159
of artificial radiation, 139 multiple myeloma, 160
of background radiation, 139 occlusal caries, 155
extraoral techniques occlusal forces, 158
about, 145–147 osteoradionecrosis, 160
focal trough, 145, 145f Paget’s disease, 160
lateral cephalometric projection, 146, 146f periapical lesions, 159, 160f
lateral jaw exposure, 146, 146f periodontal disease, 158, 158f, 159f
panoramic imaging, 145, 146f periodontitis, 158
posteroanterior projection, 147, 147f pulpal lesions, 159
Waters projection, 147, 147f pulp erasure, 159
factors determining injury pulp stones, 159, 160f
about, 138–139 radiolucency, 158
cell types, 138 radiopacity, 158, 158f
critical organs, 139 resorption, 159, 159f
exposure area, 138 restorations and materials, 155
Law of Bergonié and Tribondeau, 138 root caries, 157f
radiosensitivity, 138 root surface caries, 155
total dose, 138 tooth fractures, 159
generation of torus (tori), 158
about, 136–137 traumatic lesions, injuries, 158
image characteristics intraoral techniques
about, 141–142 about, 143
accurate image formation, 142 bisecting angle, 143, 143f
amperes, 141 buccal object (SLOB) rule, 143
beam intensity, 142 occlusal, 143
contrast, 142 paralleling, 143
density, 142 mechanisms of injury
distortion, 142 about, 138
kilovolt (kV), 142 direct theory, 138
kilovoltage peak (kVp), 142 dose-response curve, 138
magnification, 142 indirect theory, 138
milliamperes (mA), 141 processing problems, solutions
milliampere-seconds (mA-s), 141 air bubbles, 151
pneumatization, 142 chemical contamination, 150, 150f
quality, 142 exhausted solutions, 150
quantity, 141 fogged films, 151
sharpness, 142 overdeveloped film, 149, 149f
image production overlapped films, 150, 151f
about, 148–149 reticulation, 150
automatic processing, 148 scratched films, 151
darkroom lighting, 148 solution cutoff, 150
film duplication, 148 static electricity, 151, 151f
film mounting, 149 underdeveloped film, 149, 150f
film processing, 148 properties of, 135
manual processing, 148 protection from
quality assurance, 149 about, 139–140
rapid (hot) processing, 148 with ALARA concept, 140
infection control for for clinician, 140
about, 152–154 by collimation, 139
clinician preparation, 152 by fast film, 139
618    Index

Radiation and radiography (Continued) Renal physiology (Continued)


with film badges, 140 processes performed by, 66
by film-holding devices, 139 tubular components of, 66
by filtration, 139 vascular components of, 66
by lead apron, 139 Renin, 62
maximum permissible dose (MPD), 140 Reproductive system
by monitoring, 140 female, 53, 55f
for patient, 139 male, 53, 56f
by PID, 139 Research. See under Community oral health
by positioning, 140 Respiratory physiology. See also Respiratory system; under
with thermoluminescent dosimeter (TLD), 140 ­Pharmacology
special imaging techniques about, 63
about, 136–137, 148 acid-base balance regulation, 64
computed tomography (CT, CAT scan), 148 clinical study for, 65t
magnetic resonance imaging (MRI), 148 functions of system
sialography, 148 acid-base balance, 63
types of airflow regulation, 63
electromagnetic, 135 defense, 63
frequency, 135 drug delivery, 63
particulate, 135 gas exchange, 63
spectrum arrangement, 135 metabolism, elimination, 63
wavelength, 135 venous return, 63
units of measurement gas diffusion, 64
about, 137–138 lung mechanics, volume
dose, 137 compliance, 64
exposure, 137 resistance, 64
radiation absorbed dose (RAD), 137 O2 and CO2 handling, 64
roentgen (R), 137 and O2 use
x-ray, defined, 135 decrease of alveolar pO2, 64
x-ray machine decrease of blood flow, 65
anode, 136 decrease of hemoglobin, 64
autotransformer for, 137 enzyme poisoning, 65
cathode, 136 partial gas pressures , 64
circuit for, 136 ventilation, 63
components of, 135–137 decrease of, 63
control panel, 136 factors driving, 63
position-indicating device (PID), 136 increase of, 63
power supply, 136 tidal volume, 63
step-up transformer for, 137 Respiratory system. See also Respiratory physiology
transformer for, 136 about, 50
tubehead, 136 functions of, 50
x-ray tube, 136 illustrated, 51f
x-ray process lower respiratory tract
with anode, 137 bronchial tree, 50
with cathode, 137 lungs, 50
x-ray production trachea (windpipe), 50
about, 137 upper respiratory tract, 50
Bremsstrahlung radiation, 137 larynx (voice box), 50
characteristic radiation, 137 nose, 50
x-rays vs. matter pharynx, 50
about, 137 vocal folds (cords), 50
Compton scatter effect, 137 Review courses, for NBDHE, 2
photoelectric effect, 137 Rh blood type, 44
Thompson (coherent) scatter, 137 Rhabdomyosarcoma, 178
Radicular cyst, 194 Roentgen (R), 137
Ranula, 192 Rubber dams, 522–523
Rectum, 52, 52f, 55–56f
Reduced enamel epithelium (REE), 14 S
Relaxation techniques, 5 Salivary glands. See also under Gastrointestinal tract physiology;
Renal disease. See under Pathology, general and oral under Glandular tissue; under Neoplasia
Renal physiology. See also under Pathology, general and oral clinical study for, 99
defined, 65–66 duct obstruction in, 192
and excretion, 66 parotid, 52f
major kidney functions, 66 salivary amylase, 69
nephrons sublingual, 52f
defined, 66 submandibular, 52f
Index    619

Sarcolemma, 59 Skeletal system (Continued)


Sarcomere, 59 axial skeleton of
Saunders Review, using, 1–2 auditory ossicles, 34
Scalers, manual and power. See under Instrumentation face, 34
Scientific Basis for Dental Hygiene Practice skull, 34
Sclera, 80 vertebral column of, 34
Scleroderma, 189 defined, 33
Sealants, dental, 21, 561. See also under Clinical treatment illustrated, 35–36f
Secondary dentition. See Permanent teeth as locator, 82
Secretion, 69 long bones of, 34
Semilunar valves, 45 diaphysis, 34
Sense organs epiphysis, 34
auditory myosin filaments, 59
equilibrium, 42 osseous tissue of
receptors for, 41 compact bone, 33
regions of ear, 41 spongy bone, 34
and sound waves, 41–42 surface markings on, 34
defined, 41 Skin. See Integumentary system
general, 41 Skull
pain (nociceptors), 41 about, 82
proprioception, 41 anterior view, 84f
thermoreception, 41 illustrated, 36f
touch, pressure, 41 inferior view, external, 84, 86f
gustatory, olfactory, 41 lateral view, 84, 85f
and facial nerves, 41 major bones, sutures, 83
taste buds, 41 nasal cavity, 84
olfactory openings, nerves, and blood vessels, 83t
location of, 41 orbits, 83
visual, 41 superior view, internal, 84, 87f
clinical study for, 42–49 Small intestine. See under Digestive system; Gastrointestinal tract
photoreceptors, 43 physiology
protective features of eye, 41 Somatic nervous system, 37
refraction, 41 Sound waves, 41–42
structure of eyeball, 41 Special needs patient care
Sensory nerves, defined, 36–37 acquired disabilities, 530
Sex steroid hormones, 62 and American Dental Association standards, 531b
Sharpey’s fibers, 22, 24f, 25, 26f and Americans with Disabilities Act (ADA), 530–531
Sphenomandibular ligament, 92f attention deficit–hyperactivity disorder (ADHD), 537
Simple bone cyst, 196 autism
Sjögren’s syndrome, 190, 258 about, 536
Skeletal muscle physiology barriers to care for, 537
actin filaments, 59 medical manifestations of, 536
contraction factors oral manifestations of, 536
active tension, 60 patient, caregiver education, 537
and central fatigue, 60 professional care and homecare, 537
components of, 60 risk factors for, 537
and general fatigue, 60 barriers to care
and muscle fatigue, 60 communication, 531
and neuromuscular junction fatigue, 60 economic, 531
functions of, 59 employment, 531
junction and coupling of, 59 motivational, 531
structural components of physical, 531
connective tissue, 59 transport, 531
endoplasmic reticulum, 59 cerebral palsy (CP) patient
sarcolemma, 59 about, 535
sarcomere, 59 barriers to care for, 536
Skeletal system medical manifestations of, 535
appendicular skeleton oral manifestations of, 536
lower extremities, 34 patient and caregiver education, 536
pectoral girdle, 34 professional care and homecare for, 536
pelvic girdle, 34 risk factors for, 536
upper extremities, 34 clinical depression, 539
articulations (joints) clinical studies for, 532–538, 534t
amphiarthrosis, 34 cognitive disabilities, 530
diarthrosis (synovial joint), 34 cognitively disabled patients, 534–538
synarthrosis, 34 communication disabilities, 530
620    Index

Special needs patient care (Continued) Special needs patient care (Continued)
communication disorders visually impaired patients
aphasia, 531 about, 533
apraxia, 531 barriers to care for, 533
barriers to care for, 532 oral factors of, 533
dysarthria, 531 patient, caregiver education, 533
oral manifestations of, 532 professional care and homecare for, 533
professional care and homecare for, 532 Spina bifida, 8
risk factors for, 532 Spinal cord, 37, 39f
developmental disabilities, 530 Spleen, 46, 49f
disability, defined, 530 Squamous cell carcinoma, 178
hearing-impaired patients Static bone cyst, 196
about, 533 Statistics. See under Community oral health
barriers to care for, 533 Sternocleidomastoid muscle, 81, 82f, 88
oral factors of, 533 Sterilization. See under Instrumentation
patient, caregiver education, 534 Steroids, 56, 60
professional care and homecare for, 533 Stomach. See under Digestive system
intellectual disability (mental retardation) Stomodeum, 8–11, 9–10f
barriers to care for, 535 Structural lipids, 55
causes of, 534 Studies, clinical, 2
defined by IQ, 534b Study time, planning, 2–5
Down syndrome (trisomy 21), 534, 535f Sulcus terminalis, 81, 81f
fetal alcohol syndrome (FAS), 534, 535f Supernumerary teeth, 196
medical manifestations of, 534 Sweeteners. See under Carbohydrates
oral manifestations of, 535 Synapse, 36–37
patient, caregiver education, 535 Systemic circulation, of CVS, 46, 48f
professional care and homecare for, 535 Systemic lupus erythematosus, 189
risk factors for, 535
levels of function T
about, 530 T cells, 44
high (levels I, II), 530 T tubules, 59
low (level IV), 530 Taste buds, 41
moderate (level III), 530 Taurodontism, 197
medical disabilities Teeth; See also Permanent teeth; Primary teeth; under Clinical
defined, 530 ­treatment
mentally ill patients, 538–539 development of, 8–15
obsessive-compulsive disorder, 538 developmental disorders of. See under Pathology, general and oral
orthopedic disabilities formative stages of, 13f
about, 539 functions of, 69
amputation, 539 inherited disorders in. See under Pathology, general and oral
amyotrophic lateral sclerosis (ALS, Lou Gehrig’s disease), 542 mandibular, 88f
Becker muscular dystrophy, 541 maxillary
defined, 530 illustrated, 87f
Duchenne muscular dystrophy, 541 Temporomandibular joint disorder (TMD)
hemiplegia, 539 about, 191
multiple sclerosis (MS), 541 diagnostic categories of
myasthenia gravis (MG), 542 disorders of growth, 192
and palatal augmentation prostheses, 539 disorders of mobility, 192
paralysis and spinal cord injuries, 539 disorders of the TMJ, 192
paraplegia, 539 muscle, facial disorder, 192
Parkinson’s disease, 542 oral habits affecting, 192
quadriplegia, 539 signs, symptoms, 192
spina bifida, 539 etiology of, 192
panic disorder, 538 systems affected, 191
positioning, stabilization for Temporomandibular joint (TMJ)
about, 543 about, 92
behavior management, 544 disc of, 92, 92f
body wraps, 543 joint capsule of, 92, 92f
head stabilization, 543 ligaments of
mouth props and blocks, 543 sphenomandibular, 93
papoose boards, 543 stylomandibular, 92–93
wheelchair to dental chair transfer, 543, 544f TMJ, 92
psychotic disorders, 539 location of, 80
sensory disabilities postglenoid process, 92f
defined, 530 Test anxiety, controlling, 5
specific learning disability (SLD), 537 Testes, 44, 53, 56f
Index    621

Testosterone, 62 Trigeminal nerve, sensory root (Continued)


Thalamus, 37, 39f branches of, 100
Thermoreception, 41 buccal, 101
Thrombocytes (platelets), 44 illustrated, 101f
Thrombocytopenia, 174 incisive, 101
Thymidine monophosphate (TMP), 56 inferior alveolar (IA), 101f, 102
Thymus, 46, 49f lingual, 101, 101f
Thyroid disorders, 170f mental, 101
Thyroid gland, 44 mylohyoid, 101f, 102
Thyroid gland disorder, 170 maxillary division of
Thyroid-stimulating hormone, 60 about, 99
Tidal volume, 63 anterior superior alveolar (ASA), 100, 100f
Tissues. See also Glandular tissue; Tonsillar tissue; under Histology greater palatine (GP), 100, 100f
components and types, 16–17 illustrated, 100f
defined, 15 infraorbital (IO), 99–100, 100f
dental, 21–23 lesser palatine (LP), 100, 100f
gingival, 17–21 middle superior alveolar (MSA), 100, 100f
Tobacco-related lesions, 192 nasopalatine (NP), 100
Tomes’ granular layer, 24f posterior superior alveolar (PSA), 100, 100f
Tongue zygomatic, 99, 100f
anatomy of, 81 ophthalmic division of, 99
ankyloglossia in, 12 Trigeminal neuralgia, 39. See also under Pathology, general and oral
black hairy, 200 Trochlear (fourth cranial) nerve, 39, 40f
cleaning, 369
development of, 12, 34 U
examination of, 350 Ureters, 53, 56f
functions of, 69 Urethra, 53, 53f, 55–56f
geographic, 199 Uridine monophosphate (UMP), 56
illustrated, 10f, 52f, 81f Urinary system
and taste, 41 composition of, 53
Tonsillar tissue illustrated, 53f
about, 103–105 kidney
clinical study for, 105 blockages in, 53
illustrated, 81f corpuscle of, 53
lingual tonsil, 105 and fluoride, 53
palatine tonsils, 105 functions of, 53
as part of lymphatic system, 46 juxtaglomerular apparatus in, 53
pharyngeal tonsils (adenoids), 105 location of, 53
tubal tonsils, 105 nephron in, 53
Tooth germ, 13f, 14 ureters in, 53
Tooth whitening, 522, 524. See also under Biomaterials urethra in, 53
Toothbrushes, manual and power. See under Clinical treatment and urine, 53
Toothpaste. See under Clinical treatment Urine, 53
Topical anesthetics. See under Anesthesia, local Uterine (fallopian) tubes, 53, 55f
Tori, 199 Uterus, 53, 55f
Total peripheral resistance (TPR), 67
Trachea (windpipe), 50, 51f V
Triacylglycerols, 55 Vagina, 53, 55f
Tricarboxylic acid cycle, 57 Vagus (tenth cranial) nerve, 40f, 41
Tricuspid valve, 45, 47f Valves, of heart, 45
Trigeminal (fifth cranial) nerve Vegan diet, 211
about, 39 Veins, 46, 48f
formation of, 9 Venous drainage
ganglion of, 39 about, 94–95
illustrated, 40f facial, 95
mandibular division of, 40 illustrated, 95f
maxillary division of, 40 lingual, 95
motor root of, 40 submental, 95
ophthalmic division of, 40 superior labial, 95
sensory root of, 39 illustrated, 95f
Trigeminal nerve, sensory root pterygoid plexus of veins, 95f, 96
about, 99–102 retromandibular, 95
clinical study for, 102–103 illustrated, 95f
mandibular division of maxillary, 96
about, 100 superficial temporal, 96
anterior trunk of, 100 Venous sinuses, 46
622    Index

Ventilation. See under Respiratory physiology X


Ventricles, of heart, 45, 47f Xerostomia, 99, 190
Venules, 46, 48f X-rays. See Radiation, radiography
Vermilion zone, of lips, 80 Xylitol, 212
Vestibulocochlear (eighth cranial) nerve, 40f, 41–42
Viral infections, 169 Z
Viruses, 233 Z line, 59
Vitamins. See under Nutrition Zygomatic arch, 80, 90f
Vocal folds (cords), 50
Von Willebrand disease (VWD), 198

W
Water, 221–222
Whitening, 226, 522. See also under Biomaterials

Vous aimerez peut-être aussi